You are on page 1of 958

COMPLETE STUDY PACK FOR

ENGINEERING
ENTRANCES
OBJECTIVE
PHYSICS
Volume 1
COMPLETE STUDY PACK FOR

ENGINEERING
ENTRANCES
OBJECTIVE
PHYSICS
Volume 1

DC Pandey

ARIHANT PRAKASHAN (SERIES), MEERUT


Arihant Prakashan (Series), Meerut
All Rights Reserved

© AUTHOR

Administrative & Production Offices


Regd. Office
‘Ramchhaya’ 4577/15, Agarwal Road, Darya Ganj, New Delhi -110002
Tele: 011- 47630600, 43518550

Head Office
Kalindi, TP Nagar, Meerut (UP) - 250002
Tel: 0121-7156203, 7156204

Sales & Support Offices


Agra, Ahmedabad, Bengaluru, Bareilly, Chennai, Delhi, Guwahati,
Hyderabad, Jaipur, Jhansi, Kolkata, Lucknow, Nagpur & Pune.

ISBN 978-93-25299-08-5
PO No : TXT-XX-XXXXXXX-X-XX
Published by Arihant Publications (India) Ltd.
For further information about the books published by Arihant, log on to
www.arihantbooks.com or e-mail at info@arihantbooks.com
Follow us on
PREFACE
Engineering offers the most exciting and fulfilling of careers. As a Engineer you can find satisfaction by serving the
society through your knowledge of technology. Although the number of Engineering colleges imparting quality
education and training has significantly increased after independence in the country, but simultaneous increase in
the number of serious aspirants has made the competition difficult, it is no longer easy to get a seat in a
prestigious Engineering college today.
For success, you require an objective approach of the study. This does not mean you 'prepare' yourself for just
'objective questions'. Objective Approach means more than that. It could be defined as that approach through
which a student is able to master the concepts of the subject and also the skills required to tackle the questions
asked in different entrances such as JEE Main & Advanced, as well other regional Engineering entrances. These
two-volume books on Physics ‘Objective Physics (Vol.1 & 2)’ fill the needs of such books in the market in Physics
and are borne out of my experience of teaching Physics to Engineering aspirants.

The plan of the presentation of the subject matter in the books is as follows
— The whole chapter has been divided under logical topic heads to cover the syllabi of JEE Main & Advanced and various
Engineering entrances in India.
— The Text develops the concepts in an easy going manner, taking the help of the examples from the day-to-day life.
— Important points of the topics have been highlighted in the text. Under Notes, some extra points regarding the topics
have been given to enrich the students.
— The Solved Examples make the students learn the basic problem solving skills in Physics. In some of the example
problems How to Proceed has been provided to make the students skilled in systematically tackling the problems.
— An Extra Knowledge Points frequently follows the discussion of few topics which includes the important theorems, results
and formulae to increase the grasp of the subject matter.
— Additional Solved Examples given at the end of text part to make the students practice of the complete chapter as a
whole.
— Assignments at the end of the chapters have been divided into two parts Objective Questions and Entrance Corner.
— The answers / solutions to all the assignment questions have been provided.
— The Objective Questions have been divided into two levels. Level 1 contains elementary MCQs while Level 2 has MCQs
which are relatively tougher and take the students to a level required for various Engineering Entrances require in the
present scenario.
— Entrance Gallery and includes the previous years' questions asked in various Engineering entrances. At the end of the
book, JEE Main & Advanced & Other Regional Entrances Solved Papers have been given.
I am extremely thankful to Mrs. Sarita Pandey, Mr. Anoop Dhyani & Mr. Shubham Sharma of their endless effort
during the Project. I would open-heartedly welcome the suggestions for the further improvements of this book
(Vol.1) from the students and teachers.

DC Pandey
CONTENTS
1. UNITS, DIMENSIONS AND ERROR 5. LAWS OF MOTION 158-218
ANALYSIS 1-31 Ÿ Types of Forces
Ÿ Introduction Ÿ Free Body Diagram
Ÿ The International System of Units Ÿ Equilibrium
Ÿ Error Analysis Ÿ Newton’s Laws of Motion
Ÿ Significant Figures Ÿ Pulleys
Ÿ Dimensional Analysis Ÿ Constraint Equations
Ÿ Pseudo Force
2. VECTORS 32-60 Ÿ Friction
Ÿ Scalars and Vectors
Ÿ General Points Regarding Vectors 6. WORK, ENERGY AND POWER 219-261
Ÿ Addition and Subtraction of Two Vectors Ÿ RIntroduction to Work
Ÿ Components of a Vector Ÿ Work Done by a Constant Force
Ÿ Product of Two Vectors Ÿ Work Done by a Variable Force
Ÿ Conservative and Non-conservative
3. MOTION IN ONE DIMENSION 61-129 Force Field
Ÿ Introduction Ÿ Kinetic Energy
Ÿ Position, Position vector, Distance and Ÿ Work-Energy Theorem
Displacement
Ÿ Potential Energy
Ÿ Average speed, Average velocity,
Ÿ Law of Conservation of Mechanical Energy
Instantaneous speed, Instantaneous velocity
Ÿ Three Types of Equilibrium
Ÿ Uniform Motion
Ÿ Power
Ÿ Acceleration
Ÿ Uniformly accelerated motion
7. CIRCULAR MOTION 262-297
Ÿ Graphs Ÿ Kinematics of Circular Motion
Ÿ Relative Motion Ÿ Dynamics of Circular Motion
Ÿ Motion in a Vertical Circle
4. PROJECTILE MOTION 130-157
Ÿ Projectile Motion
8. COM, CONSERVATION OF
Ÿ Time of Flight (T) LINEAR MOMENTUM IMPULSE
Ÿ Horizontal Range (R) AND COLLISION 298-351
Ÿ Maximum Height (H) Ÿ Centre of Mass
Ÿ Law of Conservation of Linear Momentum
Ÿ Variable Mass 12. ELASTICITY 509-537
Ÿ Impulse Ÿ Introduction
Ÿ Collision Ÿ Elasticity
Ÿ Stress and Strain
9. ROTATION 352-407 Ÿ Hooke’s Law and the Modulus of Elasticity
Ÿ Moment of Inertia
Ÿ The Stress-Strain Curve
Ÿ Torque
Ÿ Potential Energy Stored in a Stretched Wire
Ÿ Rotation of a Rigid Body about a Fixed Axis
Ÿ Thermal Stresses and Strains
Ÿ Angular Momentum
Ÿ Conservation of Angular Momentum 13. FLUID MECHANICS 538-599
Ÿ Combined Translational and Rotational Ÿ Definition of a Fluid
Motion of a Rigid Body Ÿ Density of a Liquid
Ÿ Uniform Pure Rolling Ÿ Pressure in a Fluid
Ÿ Accelerated Pure Rolling Ÿ Archimedes’ Principle
Ÿ Angular Impulse Ÿ Flow of Fluids
Ÿ Bernoulli’s Equation
10. GRAVITATION 408-456
Ÿ Applications Based on Bernoulli’s Equation
Ÿ Introduction
Ÿ Viscosity
Ÿ Newton’s Law of Gravitation
Ÿ Stoke’s Law and Terminal Velocity
Ÿ Acceleration due to Gravity
Ÿ Surface Tension
Ÿ Gravitational Field
Ÿ Gravitational Potential 14. THERMOMETRY, THERMAL EXPANSION
Ÿ Gravitational Potential Energy AND KINETIC THEORY OF GASES 600-649
Ÿ Binding Energy Ÿ Thermometers and the Celsius
Ÿ Motion of Satellites Temperature Scale
Ÿ Kepler’s Laws Ÿ The Constant Volume Gas Thermometer and
the Absolute Temperature Scale
11. SIMPLE HARMONIC MOTION 457-508 Ÿ Quantity of Heat
Ÿ Introduction Ÿ Thermal Expansion
Ÿ The Causes of Oscillation Ÿ Concept of an Ideal Gas
Ÿ Kinematics of Simple Harmonic Motion Ÿ Gas Laws
Ÿ Force and Energy in Simple Harmonic Motion Ÿ Ideal Gas Equation
Ÿ Relation between Simple Harmonic Motion Ÿ Degree of Freedom
and Uniform Circular Motion Ÿ Internal Energy of an Ideal Gas
Ÿ Method of Finding Time Period of a Simple Ÿ Law of Equipartition of Energy
Harmonic Motion
Ÿ Molar Heat Capacity
Ÿ Vector Method of Combining Two or More
Ÿ Kinetic Theory of Gases
Simple Harmonic Motions in Same Direction
15. THE FIRST LAW OF Ÿ Plane Progressive Harmonic Waves
THERMODYNAMICS 650-694 Ÿ Speed of a Transverse Wave on a String
Ÿ The First Law of Thermodynamics Ÿ Energy in Wave Motion
Ÿ Further Explanation of the Three Terms Used Ÿ Principle of Superposition
in First Law
Ÿ Interference of Waves
Ÿ Different Thermodynamic Processes
Ÿ Reflection and Transmission of a Wave
Ÿ Efficiency of a Cycle
Ÿ Standing Waves
Ÿ Heat Engines
Ÿ Normal Modes of a String
Ÿ Refrigerator
Ÿ Sound Waves
Ÿ Second Law of Thermodynamics
Ÿ Displacement Wave, Pressure Wave and
Density Wave
16. CALORIMETRY 695-732
Ÿ Speed of a Longitudinal Wave
Ÿ Specific Heat
Ÿ Sound Waves in Gases
Ÿ Phase Changes and Latent Heat
Ÿ Sound Intensity
Ÿ Heat Transfer
Ÿ Interference of Sound Waves
17. WAVE MOTION 733-823 Ÿ Standing Longitudinal Waves in Organ Pipes
Ÿ Introduction Ÿ Beats
Ÿ Transverse and Longitudinal Waves Ÿ The Doppler Effect
Ÿ The General Equation of Wave Motion

JEE Advanced Solved Paper 2015 825-840


JEE Main & Advanced Solved Papers 2016 1-10
JEE Main & Advanced/ BITSAT/Kerala CEE/ KCET/AP & TS EAMCET/
VIT/MHT CET Solved Papers 2017 1-30
JEE Main & Advanced/ BITSAT/ KCET/AP & TS EAMCET/
VIT/MHT CET Solved Papers 2018 1-29
JEE Main & Advanced/ BITSAT/ AP & TS EAMCET/
MHT CET/WB JEE Solved Papers 2019-20 1-41

DEDICATION
This book is dedicated to my honourable grandfather
(LATE) SH. PITAMBER PANDEY
a Kumaoni poet; resident of village Dhaura (Almora) Uttarakhand
1
Units, Dimensions
and Error Analysis

1.1 Introduction
To measure a physical quantity, we need some standard units of that quantity. The Chapter Snapshot
measurement of the quantity is mentioned in two parts, the first part gives how many times ● Introduction
of the standard unit and the second part gives the name of the unit. Thus, suppose I say that ● The International System
the length of this wire is 5 m. The numeric part 5 says that it is 5 times of the unit of length of Units
and the second part metre says that unit chosen here is metre. ● Error Analysis
● Significant Figures
Fundamental and Derived Units ● Dimensional Analysis
There are a large number of physical quantities and every quantity needs a unit.
However, not all the quantities are independent. e.g. if a unit of length is defined, a unit of
volume is automatically obtained. Thus we can define a set of fundamental quantities and
all other quantities may be expressed in terms of the fundamental quantities. Fundamental
quantities are only seven in numbers. Units of all other quantities can be expressed in terms
of the units of these seven quantities by multiplication or division.
Many different choices can be made for the fundamental quantities. For example, if we
take length and time as the fundamental quantities then speed is a derived quantity and if
we take speed and time as fundamental quantities then length is a derived quantity.
Several systems of units are in use over the world. The units defined for the
fundamental quantities are called fundamental units and those obtained for derived
quantities are called the derived units.

Extra Edge
■ Fundamental units are also called the base units.
■ Both fundamental units (or base units) and derived units are jointly called the system of
units.
2 Objective Physics Vol. 1

(vi) Mole It is the amount of substance of a system


1.2 The International System which contains as many elementary particles
of Units (atoms, molecules, ions, etc.) as there are atoms in
12 g of carbon-12.
Three most commonly used systems of units used from (vii) Candela The candela is the luminous intensity, in
the earlier times are the CGS system, the FPS (or British) a given direction, of a source that emits mono-
system and the MKS system. The fundamental units for chromatic radiation of frequency 540 × 1012 hertz
length, mass and time in these systems are as follows: and that has a radiant intensity in that direction of
1/683 watt per steradian.
Table 1.1 System of units
System
Fundamental Fundamental Fundamental Extra Edge
unit of length unit of mass unit of time ■ Two Supplementary Units Besides the seven base
CGS centimetre gram second units, there are two supplementary units namely plane
FPS foot pound second angle and solid angle. Their units are radian (rad) and
MKS metre kilogram second steradian (sr).
(i) Plane Angle (dθ) Plane angle (dθ) is the ratio of arc
length ds to the radius r .
SI Units
r
In 1971, General Conference on Weights and Measures ds
held its meeting and decided a system of units which is known O dθ
as the International System of Units. It is abbreviated as SI
dθ = (ds/r) radian
from the French name Le Systeme International d’ unites. This
Fig. 1.1
system is widely used throughout the world. Table below
gives the seven fundamental quantities and their SI units. (ii) Solid Angle (d Ω) Solid angle d Ω is the ratio of the
intercepted area dA of the spherical surface
Table 1.2 Fundamental quantities and their SI units described about the apex O as the centre to the
square of its radius r .
S.No. Quantity SI unit Symbol r
1. Length metre m
dΩ
2. Mass kilogram kg O dA
3. Time second s
4. Electric current ampere A
dΩ = dA/r 2 steradian
5. Thermodynamic temperature kelvin K
Fig. 1.2
6. Amount of substance mole mol
7. Luminous intensity candela cd
Table 1.3
Definitions of Seven SI Units Power of 10 Prefix Symbol
(i) Metre 1 m = length of the path travelled by light in 6 mega M
1 3 kilo k
vacuum during a time interval of of a −2 centi c
299792458 −3 milli m
second. −6 micro µ
(ii) Second 1 s = 9192631770 time periods of a particular −9 nano n
−12 pico p
radiation from cesium-133 atom.
(iii) Kilogram 1 kg = mass of the international prototype of Table 1.4
the kilogram (a platinum-iridium alloy cylinder) kept at Size of object or distance Length (m)
International Bureau of Weights and Measures, at Distance of the boundary of observable universe 10 26
Sevres, near Paris, France. Size of our galaxy 10 21
(iv) Ampere It is the current which when flows through two Distance of the sun from the earth 1011
infinitely long straight conductors of negligible Distance of the moon from the earth 10 8
cross-section placed at a distance of one metre in Radius of the earth 10 7
vacuum produces a force of 2 × 10 −7 N/m between them. Wavelength of light 10 −7
Size of hydrogen atom 10 −10
(v) Kelvin 1 K = 1/273.16 part of the thermodynamic Size of atomic nucleus 10 −14
temperature of triple point of water. Size of a proton 10 −15
Units, Dimensions and Error Analysis 3

■ 1 parsec = 3.08 × 1016 m (Parsec is the distance at which (b) Sometimes errors arise on account of ignoring
average radius of earth’s orbit subtends an angle of certain facts. For example, in measuring time period
1 arc second)
of simple pendulum error may creep because no
■ 1 light year = 1 ly = 9.46 × 1015 m (Distance that light travels consideration is taken of air resistance. These errors
with velocity of 3 × 108 ms −1 in 1 year) can be reduced by applying proper corrections to
■ 1 astronomical unit = 1 AU (Average distance of the sun the formula used.
from the earth) = 1.496 × 1011 m
(c) Change in temperature, pressure, humidity, etc.,
■ 1 angstrom = 1 Å = 10−10 m may also sometimes cause errors in the result.
■ 1 fermi = 1 f = 10−15 m Relevant corrections can be made to minimise their
effects.
Table 1.5 Measurement of mass
Object Mass (kg) Random Error
30
Sun 10 The causes of random errors are not known. Hence, it
Earth 10 25 is not possible to remove them completely. These errors
Moon 10 23 may arise due to a variety of reasons.
Proton 10 −27 For example, the reading of a sensitive beam balance
Electron 10 −30 may change by the vibrations caused in the building due
to persons moving in the laboratory or vehicles running
Table 1.6 Measurement of time nearby. The random errors can be minimised by repeating
Event Time interval (s)
the observation a large number of times and taking the
arithmetic mean of all the observations. The mean value
Age of the universe 1017
would be very close to the most accurate reading.
Revolution period of the earth 10 7
a + a 2 +… + a n
Rotation period of the earth 10 5 Thus, a mean = 1
Travel time for light from the sun to the earth 10 2 n
Travel time for light from the moon to the earth 10 0
Time between successive human heart beats 10 0 Absolute Error
−8
Life time of an excited state of an atom 10 The difference between the true value and the
Period of light wave 10 −15 measured value of a quantity is called an absolute error.
Life span of most unstable particle 10 −24 Usually, the mean value a m is taken as the true value. So, if
a + a 2 +… + a n
am = 1 ,
1.3 Error Analysis n
then by definition, absolute errors in the measured
No measurement is perfect, as the errors involved in a
values of the quantity are,
measurement cannot be removed completely. Measured value
is always somewhat different from the true value. This ∆a1 = a m − a1
difference is called an error. ∆a 2 = a m − a 2
Errors can be classified in two ways. First classification is … … …
based on the cause of error. Systematic and random errors fall ∆a n = a m − a n
in this group. Second classification is based on the magnitude Absolute error may be positive or negative.
of error. Absolute error, mean absolute error and relative (or
fractional) error lie in this group. Mean Absolute Error
Now, let us discuss them separately. Arithmetic mean of the magnitudes of absolute errors
in all the measurements is called the mean absolute error.
Systematic Error Thus,
These are the errors whose causes are known to us. Such | ∆ a1 | + | ∆ a 2 | +… + | ∆a n |
errors can therefore be minimised. ∆a mean =
n
Following are few causes of these errors:
The final result of measurement can be written as
(a) Instrumental errors may be due to erroneous instruments.
These errors can be reduced by using more accurate a = a m ± ∆a mean
instruments and applying zero correction, when This implies that value of a is likely to lie between
required. a m + ∆a mean and a m − ∆a mean .
4 Objective Physics Vol. 1

Relative or Fractional Error Then, x + ∆x = ( a ± ∆a ) ± ( b ± ∆b)


= ( a ± b) ± ( ± ∆ a ± ∆ b)
The ratio of mean absolute error to the mean value of the
quantity measured is called relative or fractional error. Thus, = x ± ( ± ∆ a ± ∆ b)
∆a mean or ∆x = ± ∆a ± ∆b
Relative error =
am The four possible values of ∆x are
( ∆a − ∆b), ( ∆a + ∆b), ( −∆a − ∆b) and ( −∆a + ∆b).
Relative error expressed in percentage is called as the Therefore, the maximum absolute error in x is,
percentage error, i.e. ∆ x = ± ( ∆ a + ∆ b)
∆a mean
Percentage error = × 100 i.e. the maximum absolute error in sum and difference
am of two quantities is equal to sum of the absolute errors
in the individual quantities.
X Example 1.1 The diameters of a wire as measured
by screw gauge were found to be 2.620, 2.625, 2.630, X Example 1.2 The volumes of two bodies are
2.628 and 2.626 cm. Calculate: measured to be V1 = (10.2 ± 0.02) cm 3 and
(a) mean value of diameter. V2 = (6.4 ± 0.01) cm 3 . Calculate sum and difference in
(b) absolute error in each measurement. volumes with error limits.
(c) mean absolute error.
Sol. V1 = (10.2 ± 0.02 ) cm3
(d) fractional error.
(e) percentage error. and V2 = (6.4 ± 0.01) cm3
(f ) express the result in terms of percentage error. ∆V = ± (∆V1 + ∆V2 )
= ± (0.02 + 0.01) cm3 = ± 0.03 cm3
Sol. (a) Mean value of diameter
2.620 + 2.625 + 2.630 + 2.628 + 2.626 V1 + V2 = (10.2 + 6.4) cm3 = 16.6 cm3
am =
5 and . − 6.4) cm3 = 3.8 cm3
V1 − V2 = (102
= 2 .6258 cm Hence, sum of volumes = (16.6 ± 0.03) cm3
= 2 .626 cm (rounding off to three decimal places) and difference of volumes = (3.8 ± 0.03) cm3
(b) Taking am as the true value, the absolute errors in different
observations are, (ii) Errors in a product Let x = ab
∆a1 = 2 .626 − 2 .620 = + 0.006 cm
Then, ( x ± ∆x ) = ( a ± ∆a ) ( b ± ∆b)
∆a2 = 2 .626 − 2 .625 = + 0.001 cm
∆a3 = 2 .626 − 2 .630 = – 0.004 cm  ∆x   ∆a   ∆b 
or x 1 ±  = ab 1 ±  1 ± 
∆a4 = 2 .626 − 2 .628 = – 0.002 cm  x   a   b
∆a5 = 2 .626 − 2 .626 = 0.000 cm ∆x ∆b ∆a ∆a ∆b
(c) Mean absolute error, or 1± =1 ± ± ± ⋅ (as x = ab)
x b a a b
|∆a1| + |∆a2| + |∆a3| + |∆a4| + |∆a5|
∆amean = ∆x ∆a ∆b ∆a ∆b
5 or ± =± ± ± ⋅
0.006 + 0.001 + 0.004 + 0.002 + 0.000 x a b a b
=
5 ∆a ∆b
= 0.0026
Here, ⋅ is a small quantity, so can be neglected.
a b
= 0.003 (rounding off to three decimal places) ∆x ∆a ∆b
(d) Fractional error = ±
∆amean Hence, ± =± ±
am x a b
± 0.003 ∆x  ∆ a ∆b   ∆a ∆b 
= = ± 0.001 Possible values of are  + , − ,
2 .626 x  a b  a b
(e) Percentage error = ± 0.001 × 100 = ± 01
.%  ∆a ∆b   ∆a ∆b 
− +  and  − − .
(f) Diameter of wire can be written as d = 2 .626 ± 01
. %  a b  a b
Hence, maximum possible value of
Combination of Errors
∆x  ∆a ∆b 
(i) Errors in sum or difference Let x = a ± b. =± + 
x  a b
Further, let ∆a is the absolute error in the
measurement of a, ∆b is the absolute error in the Therefore, maximum fractional error in the product of
measurement of b and ∆x is the absolute error in the two (or more) quantities is equal to the sum of
measurement of x. fractional errors in the individual quantities.
Units, Dimensions and Error Analysis 5

a
(iii) Errors in division Let x= X Example 1.3 The mass and density of a solid
b sphere are measured to be (12.4 ± 0.1) kg and
a ± ∆a (4.6 ± 0.2) kg/m 3 . Calculate the volume of the sphere
Then, x ± ∆x =
b ± ∆b with error limits.
 ∆a 
a 1 ±  Sol. Here, m ± ∆m = (12 .4 ± 01 . ) kg
 ∆x   a  ρ ± ∆ρ = ( 4.6 ± 0.2 ) kg/m3
or x 1 ± = and
 x   ∆b  m 12 .4
b 1 ±  Volume, V = = = 2 .69 m3 = 2.7 m3
 b ρ 4.6
–1 (rounding off to one decimal place)
 ∆x   ∆a   ∆b   a ∆V  ∆m ∆ρ 
or 1 ±  = 1 ±  1 ±   as x =  Now, =±  + 
 x   a   b  b V  m ρ 
∆b  ∆m ∆ρ 
As < <1, so expanding binomially, we get or ∆V = ±  +  ×V
 m ρ 
b
01 0.2 
= ± 
.
 ∆x   ∆a   ∆b   12 .4
+  × 2 .7
1 ±  = 1 ±  1 m  4.6 
 x   a   b = ± 014 .
∆x ∆a ∆b ∆a ∆b ∴ V ± ∆V = (2 .7 ± 014 . ) m3
or 1± =1 ± + − ⋅
x a b a b
∆a ∆b X Example 1.4 Calculate percentage error in
Here, ⋅ is small quantity, so it can be neglected. determination of time period of a pendulum,
a b
Therefore, l
T = 2π
∆x ∆a ∆b g
± =± +
x a b where, l and g are measured with ±1% and ± 2% errors.
∆x  ∆a ∆b   ∆a ∆b 
Possible values of are  − ,  + , Sol. ∆T  1 ∆l
× 100 = ±  ×
1 ∆g
× 100 + ×

× 100
x  a b  a b T 2 l 2 g 
 ∆a ∆b   ∆a ∆b  1 1
– −  and  – + . = ±  × 1 + × 2  = ±1.5%
 a b  a b 2 2 

∆x
Therefore, the maximum value of X Example 1.5 The period of oscillation of a simple
x
pendulum is T = 2π L / g . Measured value of L is
 ∆a ∆b 
=± +  20.0 known to 1mm accuracy and time for 100
 a b
oscillations of the pendulum is found to be 90 s using
or the maximum value of fractional error in division of a wrist watch of 1s resolution. What is the accuracy in
two quantities is equal to the sum of fractional errors the determination of g?
in the individual quantities.
Sol. Time period of oscillation,
(iv) Errors in quantity raised to some power t ∆t
T= ⇒ ∆T =
an n n
Let x= m ∆T ∆t / n ∆t
b Therefore, = =
T t /n t
Then, ln ( x ) = n ln ( a ) − m ln ( b) L
Now, from the equation T = 2 π
Differentiating both sides, we get g
dx da db
= n⋅ −m g = (4 π 2 )  2 
L
We can see that
x a b T 
In terms of fractional error, we may write  ∆g 
∴ Percentage error in g = 100  
∆x ∆a ∆b  g 
± =±n +m ∆L ∆T 
x a b = 100   + 2 × 100  
 L   T 
Therefore, maximum value of 0.1 1
= × 100 + 2 × 100 ×  
∆x  ∆a ∆b  20.0  90 
= ± n +m 
x  a b = 2.72 %
6 Objective Physics Vol. 1

X Example 1.6 Calculate focal length of a spherical Rules for Counting Significant Figures
mirror from the following observations. For counting significant figures, we use the following rules:
Object distance u = (50.1 ± 0.5) cm and image Rule 1 All non-zero digits are significant. For example,
distancev = (20.1 ± 0.2) cm. x = 2567 has four significant figures.
Sol. Mirror formula is given by 1 = 1 + 1 Rule 2 The zeros appearing between two non-zero digits
f v u
are counted in significant figures, no matter where
uv (50.1) (20.1)
or f= = = 14.3 cm the decimal point is, if any.
u + v (50.1 + 20.1)
For example, 6.028 has 4 significant figures.
∆f  ∆u ∆v ∆u + ∆v 
Also, =±  + +
u + v 
Rule 3 If the number is less than 1, the zero(s) on the right
f u v
of decimal point but to the left of first non-zero
 0.5 0.2 0.5 + 0.2 
= ± + + digit are not significant.
 50.1 20.1 50.1 + 20.1
= ± [ 0.00998 + 0.00995 + 0.00997 ] For example, 0.0042 has two significant digits.
= ± (0.0299) Rule 4 The terminal or trailing zero(s) in a number without
∴ ∆f = 0.0299 × 14.3 = 0.428 = 0.4 cm a decimal point are not significant. Thus, 426 m
∴ f= (14.3 ± 0.4) cm = 42600 cm = 426000 mm has three significant
figures.
Extra Edge
■ Least Count The smallest value that can be measured
Rule 5 In a number with decimal, zeros to the right of last
by a measuring instrument is called its least count. For non-zero digit are significant.
example, least count of a metre scale is 0.1 cm of a For example, 4.600 or 0.002300 have four
vernier calliper is 0.01 cm and of a screw gauge (or a significant figures each.
spherometer) is 0.001 cm.
■ Least Count Error Least count error is associated with Rule 6 A choice of change of different units does not
the resolution of the error. Using instruments of higher change the number of significant digits or figures
precision, we can reduce the least count error. For in a measurement.
example, least count error by a vernier calliper is
For example, the length 7.03 cm has three
definitely less than the least count error by a metre
scale. significant figures. But in different units, the same
value can be written as 0.0703 m or 70.3 mm. All
these measurements have the same number of
significant figures (digits 7, 0 and 3) namely three.
1.4 Significant Figures
This shows that location of decimal point is of no
Significant figures in the measured value of a physical consequence in determining the number of
quantity tell the number of digits in which we have significant figures.
confidence. Larger the number of significant figures
obtained in a measurement, greater is the accuracy of the Point of Confusion and its Remedy
measurement. Suppose we change the units, then we will write
“All accurately known digits in a measurement plus the 2.30 m = 23.0 cm = 2300 mm = 000230
. km
first uncertain digit together form significant figures.”
When we are writing 2300 mm, then from Rule -4,
For example, when we measure the length of a straight
we would conclude erroneously that the number
line using a metre scale and it lies between 7.4 cm and
has two significant figures, while in fact it has three
7.5 cm, we may estimate it as l = 7.43 cm. This expression
significant figures and a mere change of units
has three significant figures out of these 7 and 4 are precisely
known but the last digit 3 is only approximately known. cannot change the number of significant figures.
To remove such ambiguities in determining the
Note Points number of significant figures, apply following rule:
/ As the significant figures indicate the precision of Rule 7 The power of 10 is irrelevant in determination of
measurement, it depends on the least count of the measuring significant figures.
instrument.
For example, in the measurements
/ In different units, the number of significant figures are same.
For example, an area 2.4 m2 has two significant figures. In . m = 2.30 × 10 2 cm
230
CGS system, this area can be written as 24000 cm2 or = 2.30 × 10 3 mm
2.4 × 104 cm2 . In this system of unit also, significant figures
are two (digits 2 and 4 ). = 2.30 × 10 −3 km
Units, Dimensions and Error Analysis 7

Table 1.7 Measurement of significant figures (ii) Multiplication or Division Suppose in the
Measured value Number of significant figures Rule number
measured values to be multiplied or divided, the
least number of significant digits be n, then in the
7285 4 1
product or quotient, the number of significant digits
7.0025 5 2 should also be n. e.g. 1.2 × 36.72 = 44.064 ≈ 44
0.00078 2 3 The least number of significant digits in the
78200 cm 3 4 measured values are two. Hence, the result when
7.20 3 5 rounded off to two significant digits become 44.
0.006300 4 5 Therefore, the answer is 44.
2.30 × 10 −3 km 3 7 1100
e.g. = 107.8431373 ≈ 110
10.2
Rounding Off a Digit As 1100 has minimum number of significant figures
Following are the rules for rounding off a measurement (i.e. 2), therefore the result should also contain only
two significant digits. Hence, the result when
Rule 1 If the number lying to the right of cut-off digit is
rounded off to two significant digits becomes 110.
less than 5, then the cut-off digit is retained as such.
However, if it is more than 5, then the cut-off digit 1100 m /s
e.g. = 107.8431373 ≈ 108
is increased by 1. 10.2 m /s
For example, x = 6.24 is rounded off to 6.2 to two / In this case, answer becomes 108. Think why?
significant digits and x = 5.328 is rounded off to
5.33 to three significant digits. Extra Edge
Rule 2 If the insignificant digit to be dropped is 5, then the
■ Exact numbers that appear in formulae, like 2 π in
l
rule is T =2π or S = 2 πr have a large (infinite) number of
g
(i) if the preceding digit is even, the insignificant
significant figures. The value of π = 3.1415926... can be
digit is simply dropped. written as 3.142 or 3.14 with limited number of
(ii) if the preceding digit is odd, the preceding digit is significant figure as required in specific cases.
raised by 1. d
■ Similarly in r = , the factor is an exact number, which
2
For example, x = 6.265 is rounded off to x = 6.26 to can be written as 2.0, 2.00 or 2.000 as required.
three significant digits and, x = 6.275 is rounded ■ Order of Magnitude In scientific notation, a measurement
off to x = 6.28 to three significant digits. can be written as (a × 10b ). Here a, which is called the
base number lies between 1 and 10. b is any positive or
Algebraic Operations with negative exponent (or power) of 10.
Significant Figures ■ For a ≤ 5, the order of magnitude is called 10b .
For 5 < a ≤ 10, the order of magnitude is called 10b + 1.
In addition, subtraction, multiplication or division the ■ For example; diameter of the earth (1.28 × 107 m) is of
final result should not have more significant figures than any the order of 107 m. Similarly, mass of an electron
of the original data from which it was obtained. To (9.11 × 10 − 31 kg) is of the order of 10 − 30 kg .
understand this, let us consider a chain, of which all links are
strong except the one. The chain will obviously break at the X Example 1.7 Write down the number of
weakest link. Thus, the strength of the chain cannot be more significant figures in the following:
than the strength of the weakest link in the chain. (a) 6428 (b) 62.00 m (c) 0.00628 cm
(i) Addition and Subtraction Suppose in the Sol. (a) 6428 has four significant figures.
measured values to be added or subtracted, the least (b) 62.00 m has four significant figures.
number of significant digits after the decimal is n. (c) 0.00628 cm has three significant figures.
Then, in the sum or difference also, the number of
significant digits after the decimal should be n. X Example 1.8 Round off to four significant figures
Example 1.2 + 3.45 + 6.789 = 11.439 ≈ 11.4 (a) 45.689 (b) 2.0082
Sol. (a) 45.69 (b) 2.008
Here, the least number of significant digits after the
decimal is one. Hence, the result will be 11.4 (when X Example 1.9 A cube has a side of length
rounded off to smallest number of decimal places). 1.2 × 10 −2 m. Calculate its volume with regard to
e.g. 12.63 − 10.2 = 2.43 ≈ 2.4 significant figures.
8 Objective Physics Vol. 1

Sol. V = l 3 = (1.2 × 10−2 m)3 = 1.728 × 10−6 m3 The thermodynamic temperature, the amount of
Q Length (l ) has two significant figures, the volume (V ) will also substance and the luminous intensity are denoted by the
have two significant figures. Therefore, the correct answer is symbols of their units K, mol and cd, respectively. The
V = 1.7 × 10−6 m3
physical quantity that is expressed in terms of the base
quantities is enclosed in square brackets.
X Example 1.10 Three masses, m1 = 226.2 g, Thus, Eq. (i) can be written as
m2 =127.36 g and m3 =10.214 g are kept together. [density] = [ML–3 ]
What is the total mass with regard to significant Such an expression for a physical quantity in terms of
figures? the fundamental quantities is called the dimensional
Sol. Total mass = m1 + m2 + m3 formula. Here, it is worthnoting that constants such as 5, π or
trigonometrical functions such as sin θ, cos θ, etc., have no
= (226.2 + 127.36 + 10.214) g = 363.774 g
units and dimensions.
But the least precise measurement (226.2 g) is correct to only
one decimal place. [sin θ ] = [cos θ ] = [tan θ ] = [log x ] = [ e x ] = [M 0 L0T 0 ]
∴ Total mass = 363.8 g
X Example 1.11 Find the dimensions of the
physical quantities
1.5 Dimensional Analysis (a) area (b) force (c) velocity (d) work and (e) power
Dimensions of a physical quantity are the powers to
Sol. (a) Area = length × breadth = length × length
which the fundamental quantities must be raised to represent
∴ [Area] = [L] × [L] = [L2 ]
the given physical quantity.
mass mass (b) Force = mass × acceleration = mass × (length) / (time)2
For example, density = = ∴ [Force] = [M] [L] / [T2 ] = [MLT−2 ]
volume (length) 3
(c) Velocity = displacement / time = length / time
or density = (mass) (length) –3 …(i)
∴ [Velocity] = [L] /[T] = [LT−1 ]
Thus, the dimensions of density are 1 in mass and − 3 in (d) Work = force × displacement = Force × length
length. The dimensions of all other fundamental quantities ∴ [Work] = [Force] × [Length]
are zero. For convenience, the fundamental quantities are = [MLT−2 ] [L] = [ML2 T−2 ]
represented by one letter symbols. Generally, mass is (e) Power = work / time
denoted by M, length by L, time by T and electric current ∴ [Power] = [Work] / [Time]
by A. = [ML2 T−2 ] / [T] = [ML2 T−3 ]

Table 1.8 Dimensional formulae and SI units of some physical quantities frequently used in physics
S.No. Physical quantity SI units Dimensional formula
1. Velocity = displacement/time m/s [M 0LT −1 ]
2. Acceleration = velocity/time m/s 2
[M 0LT −2 ]
3. Force = mass × acceleration kg-m/s 2 = newton or N [MLT −2 ]
4. Work = force × displacement 2 2
kg-m /s = N-m = joule or J [ML2T −2 ]
5. Energy J [ML2T −2 ]
6. Torque = force × perpendicular distance N-m [ML2T −2 ]
7. Power = work/time J/s or watt [ML2T −3 ]
8. Momentum = mass × velocity kg-m/s [MLT −1 ]
9. Impulse = force × time N-s [MLT −1 ]
10. Angle = arc/radius radian or rad [M 0L0T 0]
∆L ∆V
11. Strain = or no units [M 0L0T 0]
L V
12. Stress = force/area N/m 2 [ML−1T −2]
13. Pressure = force/area N/m 2 [ML−1T −2]
14. Modulus of elasticity = stress/strain N/m 2 [ML−1T −2]
Units, Dimensions and Error Analysis 9

S.No. Physical quantity SI units Dimensional formula


15. Frequency = 1/time period per sec or hertz (Hz) [M 0 L0T −1]
16. Angular velocity = angle/time rad/s [M 0 L0T −1]
17. Moment of inertia = (mass) × (distance) 2 kg-m 2 [ML2T 0]
18. Surface tension = force/length N/m [ML0T −2]
force × (distance)2
19. Gravitational constant = N-m 2/kg 2 [M −1 L3T −2]
(mass )2
20. Angular momentum kg-m 2/s [ML2T −1]
21. Coefficient of viscosity N-s/m 2 [ML−1T −1]
22. Planck’s constant J-s [ML2T −1]
23. Specific heat (s) J/kg-K [L 2 T −2 K −1 ]
24. Coefficient of thermal conductivity (K) watt/m-K [MLT −3 K −1 ]
25. Gas constant (R) J/mol-K [ML2 T −2 K −1 mol −1 ]
26. Boltzmann constant (k) J/K [ML2T −2 K −1 ]
27. Wien’s constant (b) m-K [ L0 K ]
28. Stefan’s constant (σ) watt/m -K2 4
[ML−3 K −4 ]
29. Electric charge C [AT]
30. Electric intensity N/C [MLT −3A −1 ]
31. Electric potential volt [ML2T −3 A −1 ]
32. Capacitance farad [M −1 L−2T 4 A 2 ]
33. Permittivity of free space C 2N −1m −2 [M −1 L−3T 4 A 2 ]
34. Electric dipole moment C-m [LTA]
35. Resistance ohm [ML2T −3A −2 ]
36. Magnetic field tesla (T) or weber/m 2 (Wb/m 2) [M T −2A −1 ]
37. Coefficient of self-induction henry [ML2T −2A −2 ]

Sol. The dimensional formula of G is [M–1 L3 T–2 ]


Uses of Dimensions Using Eq. (i), i.e.
Theory of dimensions have following main uses: n1[M1–1 L31 T1–2 ] = n2 [M–1 3 –2
2 L 2 T2 ]
(i) Conversion of units This is based on the fact that –1
M  L   T 
3 –2

the product of the numerical value ( n) and its n2 = n1  1   1   1 


 M2   L 2   T2 
corresponding unit ( u) is a constant, i.e.
Here, n1 = 6.67 × 10−11
n[ u] = constant M1 = 1 kg, M 2 = 1 g = 10–3 kg, L1 = 1 m ,
or n1 [ u1] = n2 [ u2 ] L2 = 1 cm = 10–2 m, T1 = T2 = 1 s
Suppose, the dimensions of a physical quantity are a Substituting these values in the above equation, we get
–1 3 –2
in mass, b in length and c in time. If the fundamental  1 kg   1 m  1 s 
n2 = 6.67 × 10−11  –3   10–2 m  1 s 
units in one system are M 1 , L1 and T1 and in the other  10 kg   
system are M 2 , L2 and T2 , respectively. Then, we –8
or n2 = 6.67 × 10
can write Thus, value of G in CGS system of units is
n1 [ M 1a Lb1 T1c ] = n2 [ M a2 Lb2 T2c ] …(i) 6.67 × 10−8 dyne-cm 2 /g 2 .

Here, n1 and n2 are the numerical values in two (ii) To check the dimensional correctness of a given
system of units, respectively. Using Eq. (i), we can physical equation Every physical equation
convert the numerical value of a physical quantity should be dimensionally balanced. This is called the
from one system of units into the other system. ‘Principle of Homogeneity’. The dimensions of each
term on both sides of an equation must be the same.
X Example 1.12 The value of gravitation constant On this basis, we can judge whether a given equation
is correct or not. But, a dimensionally correct
is G = 667
. × 10 –11 N-m 2 /kg 2 in SI units. Convert it
equation may or may not be physically correct.
into CGS system of units.
10 Objective Physics Vol. 1

or f = k [F ]a [l ]b [µ ]c …(i)
X Example 1.13 Show that the expression of the
Here, k is a dimensionless constant. Thus,
time period T of a simple pendulum of length l
[ f ] = [F ]a [l ]b [µ ]c
l
given by T = 2π is dimensionally correct. or [M0 L0 T–1 ] = [MLT– 2 ] a [L] b [ML–1 ] c
g
l or [M0 L0 T–1 ] = [Ma + c La + b− c
T−2 a ]
Sol. T = 2π
g For dimensional balance, the dimensions on both sides
[L] should be same.
Dimensionally, [ T ] = = [T] Thus, a+c=0 …(ii)
[LT−2 ]
a+ b−c = 0 …(iii)
As in the above equation, the dimensions of both sides are and − 2a = − 1 …(iv)
same. The given formula is dimensionally correct. Solving these three equations, we get
1 1
Principle of Homogeneity of Dimensions a = , c = − and b = − 1
2 2
This principle states that the dimensions of all the Substituting these values in Eq. (i), we get
terms in a physical expression should be same. f = k (F )1/ 2 (l )−1 (µ )− 1/ 2
1
For example, in the physical expression s = ut + at 2 , the k F
2 or f=
1 2 l µ
dimensions of s, ut and at all are same. 1
2 Experimentally, the value of k is found to be ⋅
2
/ The physical quantities separated by the symbols 1 F
+ , −, =, >, <, etc. have the same dimensions. Hence, f=
2l µ
X Example 1.14 The velocity v of a particle
depends upon the time t according to the equation Limitations of Dimensional Analysis
c The method of dimensions has the following limitations:
v = a + bt + ⋅ Write the dimensions of a, b, c
d +t (i) By this method, the value of dimensionless constant
and d. cannot be calculated.
Sol. From principle of homogeneity, (ii) By this method, the equation containing trigono-
–1 metrical, exponential and logarithmic terms cannot be
[a] = [v ] or [a] = [LT ]
[v ] [LT–1 ] analysed.
[bt ] = [v ] or [b ] = =
[t ] [T] (iii) If a physical quantity depends on more than three
–2 factors, then relation among them cannot be
or [b ] = [LT ]
Similarly, [d ] = [t ] = [ T ]
established because we can have only three equations
[c ] by equalising the powers of M, L and T.
Further, = [v ] or [c ] = [v ] [d + t ]
[d + t ]
or [c ] = [LT–1 ][ T] or [c ] = [L]
Extra Edge
■ Dimensional Constants The quantities like gravitational
constant, Planck’s constant, etc., possess dimensions and
(iii) To establish the relation among various physical also have a constant value. They are called dimensional
quantities, if we know the factors on which a given constants.
physical quantity may depend, we can find a ■ Non-dimensional Constants The constant quantities
formula relating the quantity with those factors. having no dimensions are called non-dimensional
constants. These include pure numbers (1, 2, 3...),
Let us take an example. π, e (= 2.718...) and all trigonometric functions.
■ In some cases, an equation seems to be dimensionally
X Example 1.15 The frequency ( f ) of a stretched incorrect although it is correct. For example, in the
string depends upon the tension F (dimensions of equation F = ma, if we substitute m = 1kg, then this
force), length l of the string and the mass per unit equation becomes,
F =a
length µ of string. Derive the formula for frequency.
Now, this equation seems dimensionally incorrect, because
Sol. Suppose that the frequency f depends on the tension F and a have different dimensions. But actually, this
equation is correct because a is multiplied with 1 kg mass,
raised to the power a, length raised to the power b and mass
whose dimension is [M]. So, we will have to multiply this
per unit length raised to the power c. Then, dimension with the dimensions of a to find the dimension
f ∝[F ]a [l ]b [µ ]c of F.
Units, Dimensions and Error Analysis 11

Example 3. The centripetal force F acting


Extra Knowledge Points on a particle moving uniformly in a circle
Least Count may depend upon mass (m), velocity (v) and
The minimum measurement that can be measured accurately by an radius (r) of the circle. Derive the formula
instrument is called the least count. for F using the method of dimensions.
The least count of a metre scale graduated in millimetre mark is 1 mm. Sol. Let F = k (m) x (v) y (r ) z …(i)
The least count of a watch having second’s hand is 1 second.
Here, k is a dimensionless constant of
Least count of vernier calliper = {Value of 1 part of main scale(s )} proportionality.
− {Value of one part of vernier scale (V )} Writing the dimensions of RHS and LHS in
or Least count of vernier calliper = 1 MSD – 1 VSD Eq. (i), we have
where, MSD = Main scale division [ MLT −2 ] = [M] x [LT −1 ] y [L] z
VSD = Vernier scale division = [M x Ly + z T − y ]
Value of 1 part of main scale (s ) Equating the powers of M, L and T of both sides,
Least count of vernier callipers =
Number of parts on vernier scale (n ) we have x = 1, y = 2 and y + z = 1
In the ordinary vernier callipers, value of 1 part of main scale division is 1 or z =1− y = −1
mm and number of parts on vernier scale are 10. Putting the values in Eq. (i), we get
1 mm
∴ LC = = 0.1 mm mv 2 mv 2
10 F = kmv 2 r −1 = k = (where, k = 1)
Pitch ( p ) r r
Least count of screw gauge =
Number of parts on circular scale (n ) Example 4. Write down the number of
In the ordinary screw gauge, pitch is 1mm and number of parts on significant figures in the following:
circular scale are 100.
(a) 6428 (b) 62.00 m
1 mm
∴ LC = = 0.01 mm (c) 0.00628 cm (d) 1200 N
100
Sol. (a) 6428 has four significant figures.
(b) 62.00 m has four significant figures.
(c) 0.00628 cm has three significant figures.

Additional Examples (d) 1200 N has four significant figures.

Example 5. Round off to four significant


figures :
1
Example 1. Check the correctness of the relation s = ut + 2
at 2 , (a) 45.689 (b) 2.0082
where u is initial velocity, a the acceleration, t the time and s the Sol. (a) 45.69 (b) 2.008
displacement. Example 6. Add 6.75 × 10 3 cm to
Sol. Writing the dimensions of either side of the given equation. 4.52 × 10 2 cm with regard to significant
LHS = s = displacement = [M 0 LT 0 ] figures.
RHS = ut = velocity × time Sol. a = 6.75 × 10 3 cm
= [M 0 LT −1 ] [T] = [M 0 LT 0 ] b = 4.52 × 10 2 cm = 0.452 × 10 3 cm
1 2
and at = (acceleration) × (time)2 = 0.45 × 10 3 cm (up to 2 places of decimal)
2
= [M 0 LT −2 ] [T] 2 = [M 0 LT 0 ] ∴ a + b = (6.75 × 10 3 + 0.45 × 10 3 ) cm
As LHS = RHS, formula is dimensionally correct. = 7.20 × 10 3 cm

Example 2. Write the dimensions of a and b in the relation, Example 7. A thin wire has a length of
b − x2 21.7 cm and radius 0.46 cm. Calculate the
P= volume of the wire to correct significant
at figures.
where, P is power, x the distance and t the time. Sol. Given, l = 21.7 cm, r = 0.46 mm = 0.046 cm
Sol. The given equation can be written as 22
Volume of wire, V = πr 2 l = (0.046) 2 (21.7)
Pat = b − x 2 7
Now, [ Pat ] = [ b ] = [ x 2 ] or [ b ] = [ x 2 ] = [M 0 L2 T 0 ] = 0.1443 cm3 = 0.14 cm3
[x 2 ] [L2 ] / The result is rounded off to least number of
and [a] = = = [M −1 L0 T 2 ] significant figures in the given measurements, i.e.
[ Pt ] [ML2 T −3 ] [T] 2 ( in 0.46 mm).
12 Objective Physics Vol. 1

Example 8. The refractive index (n) of glass is found Example 10. Calculate focal length of a spherical
to have the values 1.49, 1.50, 1.52, 1.54 and 1.48. mirror from the following observations. Object distance
Calculate u = (501. ± 05 . ) cm and image distance v = (201
. ± 0.2) cm.
(a) the mean value of refractive index, 1 1 1
(b) absolute error in each measurement, Sol. Since, = +
f v u
(c) mean absolute error,
uv (50.1) (20.1)
(d) fractional error, and or f = =
(e) percentage error. u + v (50.1 + 20.1)
Sol. (a) Mean value of refractive index, = 14.3 cm
1.49 + 1.50 + 1.52 + 1.54 + 1.48 ∆f  ∆ u ∆ v ∆ u + ∆ v
nm = Also, =± + + 
5 f u v u+v 
= 1.505 = 1.51  0.5 0.2 0.5 + 0.2 
=± + +
(rounded off to two decimal places) 50.1 20.1 50.1 + 20.1
(b) Taking n m as the true value, the absolute errors in = [0.00998 + 0.00995 + 0.00997]
different observations are,
= ± (0.0299)
∆n1 = 1.51 − 1.49 = + 0.02
∴ ∆f = 0.0299 × 14.3
∆n 2 = 1.51 − 1.50 = + 0.01
= 0.428 = 0.4 cm
∆n 3 = 1.51 − 1.52 = − 0.01
∆n 4 = 1.51 − 1.54 = − 0.03 ∴ f = (14.3 ± 0.4) cm
∆n5 = 1.51 − 1.48 = + 0.03 Example 11. Can a quantity have units but still be
(c) Mean absolute error, dimensionless. Give an example.
| ∆n1 | + | ∆n 2 | + | ∆n 3 | + | ∆n 4 | + | ∆n5 | Sol. Yes, the example is angle. Unit of angle is radian. But, it is a
∆nmean =
5 dimensionless quantity.
0.02 + 0.01 + 0.01 + 0.03 + 0.03
= Example 12. Can a quantity have neither units nor
5
dimensions.
= 0.02
Sol. Yes, strain has neither units nor dimensions.
± ∆n mean ± 0.02
(d) Fractional error = =
nm 1.51 Example 13. Write four pairs of physical quantities,
= ± 0.0132 which have the same dimensional formula.
Sol. 1. Work and energy
(e) Percentage error = ( ± 0.0132 × 100)
2. Pressure and stress
= ± 1.32%
3. Velocity gradient and frequency
Example 9. The radius of sphere is measured to be 4. Angular momentum and Planck's constant.
( 2.1 ± 0.5) cm . Calculate its surface area with error
Example 14. Give two examples which are not
limits.
constants but they are dimensionless.
Sol. Surface area, S = 4 πr 2
Sol. Strain and angular displacement are the two examples.
 22
= (4 )   (2.1) 2
7 Example 15. We measure the period of oscillation of
= 55.44 = 55.4 cm 2 a simple pendulum. In successive measurements, the
∆S ∆r readings turn out to be 2.63s, 2.56s, 2.42s, 2.71s and
Further, =2 2.80s. Calculate the absolute error, relative error or
S r
 ∆r  percentage error.
or ∆S = 2   (S ) Sol. Mean period of oscillation of the pendulum
 r 
2 × 0.5 × 55. 4 (2.63 + 2.56 + 2.42 + 2.71 + 2.80) s
= T=
2.1 5
= 26.38 = 2.624 s = 2.62 s
As the periods are measured to a resolution of 0.01 s,
= 26.4 cm 2
all times are to the second decimal; it is proper to put
∴ S = (55.4 ± 26.4) cm 2 this mean period also to the second decimal.
Units, Dimensions and Error Analysis 13

The absolute errors in the measurements are Example 16. The radius of sphere is measured to be
2.63 s − 2.62 = 0.01 s
(2.1 ± 0.5) cm. Calculate its surface area with error
2.56 s − 2.62 s = − 0.06 s
2.42 s − 2.62 s = − 0.20 s
limits.
2.71 s − 2.62 s = 0.09 s Sol. Surface area, S = 4 πr 2 = (4 )   (2.1) 2
22
2.80 s − 2.62 s = 0.18 s 7
∆T mean = [(0. 01 + 0. 06 + 0. 20 + 0. 09 + 0.18) s ]/5 = 0.11 s = 55. 44 = 55. 4 cm 2
That means, the period of oscillation of the simple ∆S ∆r
pendulum is (2.62 ± 0.11) s, i.e. it lies between (2.62 + 0.11) s Further, = 2⋅
S r
and (2.62 − 0.11 ) s or between 2.73 s and 2.51 s. A more
 ∆r  2 × 0.5 × 55. 4
correct way will be to write or ∆S = 2   (S ) =
T = 2.6 ± 0.1 s  r  21
.
For this reading, the percentage error is = 26.38 cm 2
∆a 0.1 = 26.4 cm 2
× 100 = × 100 = 4%
a 2.6 ∴ S = (55.4 ± 26.4) cm 2

NCERT Selected Questions


Q 1. The SI unit of energy is J = kg m 2s −2 ; that of speed (e) has two different dimensions of two different terms.
−1 −2 Hence, this option can also be ruled out.
v is ms and of acceleration a is ms . Which of the
formulae for kinetic energy ( K ) given below can Note that dimensional correctness on two sides cannot tell
which of the two, (b) or (d) is the correct formula. But we
you rule out on the basis of dimensional arguments know the correct formula is (b).
(m stands for the mass of the body)
(a) K = m2 v 3 (b) K = (1/ 2) mv 2 Q 2. A student measures the thickness of a human hair by
(c) K = ma (d) K = ( 3/ 16) mv 2 looking at it through a microscope of magnification
(e) K = (1/ 2) mv + ma
2
100. He makes 20 observations and finds that the
average width of the hair in the field of view of the
Sol. In a correct equation, both sides should have same
dimensions. Kinetic energy which has the SI unit kg-m 2s−2
microscope is 3.5 mm. What is the estimate on the
thickness of hair?
should have the dimensions,
[ K ] = [M] [ L2 ] [T −2 ] = [ML2 T −2 ] Sol. Width of hair after 100 times magnification is given 3.5 mm.
Right hand side of option 3.5 mm
∴ Thickness of hair = = 0.035 mm
(a) has the dimensions, 100
[ m2 v 3 ] = [M 2 ] [ LT −1 ] 3 = [M 2 L3 T −3 ] Q 3. Fill in the blanks by suitable conversion of units.
Since, these dimensions do not match with the (a) 1 kg m 2 s −2 = …… g cm 2 s −2
dimensions of kinetic energy. So, this option is ruled (b) 1 m = …… ly (light year)
out.
(c) 3.0 ms −2 = …… kmh −2
(b) has the dimensions,
(d) G = 6.67 × 1011 Nm 2 kg −2 = …… cm 3 s −2 g −1
 1 2
mv = [M] [LT −1 ] 2 = [ML2 T −2 ]
 2  Sol. (a) 1 kg m 2s−2 = 1 (103 g) (102 cm)2 s−2
These dimensions match with the dimensions of kinetic = 103 × 104 g cm 2s−2 = 107 g cm 2 s−2
energy.
(b) 1 light year (ly) = 9.46 × 1015 m
(c) has the dimensions,
1
[ ma ] = [M] [LT −2 ] = [MLT −2 ] ∴ 1 m= ly = 1.057 ×− 16 ly
9.46 × 1015
These dimensions do not match with the dimensions of −2
 1 
kinetic energy. So this option is ruled out. (c) 3.0 ms−2 = 3 × 10 −3 km ×  h −1
 60 × 60 
(d) has also the dimensions of option (b). So, these
dimensions match with the dimensions of kinetic = 3 × 10−3 × (3600)2 kmh −2
energy. = 3.888 × 104 kmh −2 = 3.9 × 104 kmh −2
14 Objective Physics Vol. 1

(d) G = 6.67 × 10−11 Nm 2 kg−2 Sol. n1u1 = n2u2


−11 −2
= 6.67 × 10 5 2
(10 dyne) (10 cm) (10 g) 2 3
u1 [ M a1 Lb1 T1c ]
−11 −6 2 −2
or n2 = n1 = n1
= 6.67 × 10 × 10 × 10 × 10 dyne cm g
5 4
u2 [ M a2 Lb2 T2c ]
−8
= 6.67 × 10 (g cm s−2 ) cm 2 g−2 a
M  L  T 
b c
= n1  1   1   1 
= 6.67 × 10 −8 cm 3 g−1 s−2  M 2   L 2   T2 
1 cal = 4.2 J = 4.2 kg m 2 s−2 ,
Q 4. Fill in the blanks
(a) The volume of a cube of side 1 cm is equal to … ∴ a = 1, b = 2 , c = − 2
1 −2
m 3.  1 kg   1 m  2  1 s 
∴ n2 = 4.2      
(b) The surface area of a solid cylinder of radius  a kg   b m   g s 
2.0 cm and height 10.0 cm is equal to…… mm 2 . n2 = 4.2 α −1 β− 2 γ 2
(c) A vehicle moving with a speed of 18 kmh −1
∴ 1 cal = 4.2 α −1 β− 2 γ 2 in new system.
covers…… m in 1s.
(d) The relative density of lead is 11.3. Its density is Q 6. A new unit of length is chosen such that the speed of
…… gcm −3 or …… kgm −3 . light in vacuum is unity. What is the distance
Sol. (a) The volume of a cube of side 1 cm is given by between the sun and the earth in terms of the new
V = (10−2 m)3 = 10−6 m 3
unit if light takes 8 min and 20 s to cover this
distance?
(b) The surface area of a solid cylinder of radius r and
height h is given by Sol. We are given that velocity of light in vacuum, c = 1new unit
A =Area of two circles + curved surface area of length s− 1.
= 2πr2 + 2πrh = 2πr (r + h) Time taken by light of sun of reach the earth, t = 8 min 20 s.
= 8 × 60 + 20 = 500 s
Here, r = 2 cm = 20 mm , h =10 cm = 100 mm
∴ Distance between the sun and the earth,
22
∴ A =2× × 20 (20 + 100) (mm)2 x = c × t = 1new unit of length s−1 × 500 s
7
= 500 new units of length
= 15099 mm 2 = 1.5099 × 104 mm 2
= 1.5 × 104 mm 2 Q 7. Which of the following is the most precise device
18 × 1000 m for measuring length?
(c) Here, v =18 km h −1 = = 5 ms−1
3600 s (a) A vernier callipers with 20 divisions on the
sliding scale.
t = 1s
(b) A screw gauge of pitch 1 mm and 100 divisions
∴ x = vt = 5 × 1 = 5 m
on the circular scale.
(d) Relative density of lead = 11.3
Density of water = 1 g cm −3
(c) An optical instrument that can measure length to
within a wavelength of light.
density of lead
Relative density of lead =
density of water Sol. The most precise device is that whose least count is
minimum.
∴ Density of lead
(a) Least count of vernier callipers = 1 MSD − 1 VSD
= relative density of lead × density of water
19 1
= 11.3 × 1 g cm −3 = 1 MSD − MSD = MSD
20 20
= 11.3 g cm −3 1 1
= mm = cm = 0.005 cm
In SI system, density of water = 103 kg m −3 20 200
(b) Least count of screw gauge
∴ Density of lead = 11.3 × 103 kg m −3
Pitch
=
= 1.13 × 104 kg m −3 Number of divisions of circular scale
1 1
Q 5. A calorie is a unit of heat or energy and it equals = mm = cm
100 1000
about 4.2 J, where 1 J = 1 kgm 2s −2 . Suppose we = 0.001 cm
employ a system of units in which the unit of mass (c) Wavelength of light, λ ≈ 10−5 cm = 0.00001 cm
equals α kg, the unit of length β m and the unit of
∴ Least count of optical instrument = 0.00001 cm
time is γ s. Show that a calorie has a magnitude
4.2 α −1 β −2 γ 2 in terms of new units.
Thus, clearly the optical instrument is the most precise.
Units, Dimensions and Error Analysis 15

Q 8. State the number of significant figures in the 1  ∆c   ∆d 


+  × 100 +  × 100 …(i)
following. 2 c   d 
(a) 0.007 m 2 (b) 2.64 × 1024 kg ∆a ∆c 
× 100 = 1%, × 100 = 4%

(c) 0.2370 g cm −3
a c
(d) 6.320 J  ...(ii)
∆b ∆d
(e) 6.032 Nm −2 (f) 0.0006032 m 2 × 100 = 3 %, × 100 = 2% 
b d 
Sol. The number of significant figures is given as below: ∴ From Eqs. (i) and (ii), we get
(a) 1 (b) 3 (c) 4 ∆P 1
× 100 = 3 × 1% + 2 × 3% + × 4% + 2%
(d) 4 (e) 4 (f ) 4 P 2
= 3 + 6 + 2 + 2 = 13%
Q 9. The length, breadth and thickness of a rectangular
sheet of metal are 4.234 m, 1.005 m and 2.01 cm The calculation of error clearly shows that the number of
significant figures is 2, so the result of P may be rounded off
respectively. Give the area and volume of the sheet
to two significant digits, i.e. P = 3.763 = 3.8.
to correct significant figures.
Sol. Here, length, l = 4.234 m, breadth, b = 1.005 m Q 12. A book with many printing errors contains four
different formulae for the displacement y of a particle
Thickness, h = 0.0201 m = 2.01 cm under going a certain periodic motion:
Area of the sheet = 2 ( lb + bh +hl ) 2πt
(a) y = a sin
= 2 (4.234 × 1005
. + 1005
. × 0.0201 + 0.0201 × 4.234 ) T
= 8.7209468 m 2 (b) y = a sin vt
a
As the least number of significant figures in thickness is 3, (c) y = sin ( t / a )
∴ Area = 8.72 m 2 T
Volume = l × b × h  a 2πt 2πt 
(d) y =   sin + cos 
= 4.234 × 1.005 × 0.0201 m 3  2  T T 
= 0.0855 m 3 (where, a = maximum displacement of the particle,
v = speed of the particle, T = time period of motion).
Q 10. The mass of a box measured by a grocer’s balance is
Rule out the wrong formulae on dimensional grounds.
2.3 kg. Two gold pieces of masses 20.15 g and
20.17 g are added to the box. What is (a) the total Sol. The argument of a trigonometrical function, i.e. angle is
mass of the box, (b) the difference in the mass of the dimensionless. Now, here in each case dimensions of LHS is
pieces to correct significant figures? [L] and dimensions of RHS in
Sol. (a) Total mass = (2.300 + 0.02015 + 0.02017) kg  2πt 
(a) = [L]  angle is dimensionless
 T 
= 2.34032 kg
As the least number of significant figures in the mass of (b) = [L] sin [LT −1 (T)] = [L] sin [L]
box is 2, so maximum number of significant figures in (angle is not dimensionless here)
the result can be 2. [L] [T]
(c) = sin = [LT ] sin [TL−1 ]
−1

∴ Total mass = 2.3 kg [T] [L]


(b) Difference in masses = 2017 . − 2015. = 0.02 g (angle is not dimensionless here)
 T T
Since, there are two significant figures, so the difference (d) = [L] sin + cos = [L ]
in masses to the correct significant figures is 0.02 g.  T T 
∴ Formulae (b) and (c) are wrong.
Q 11. A physical quantity P is related to four observables
a 3b2 Q 13. The unit of length convenient on the atomic scale is
a, b, c and d as follows P = . The percentage known as an angstrom and is denoted by Å.
°
cd 1A = 10 −10 m. The size of the hydrogen atom is about
errors of measurement in a, b, c and d are 1%, 3%, 0.5 Å. What is the total atomic volume in m 3 of a
4% and 2%, respectively. What is the percentage mole of hydrogen atoms?
error in the quantity P? If the value of P calculated
using the above relation turns out to be 3.763, to Sol. r = 0.5Å = 0.5 × 10 −10 m
what value should you round off the result? 4 3
V1 = volume of each hydrogen atom = πr
3
Sol. Percentage error in P is given by 4
∆P  ∆a   ∆b  = × 3.14 × (0.5 × 10−10 )3
× 100 = 3  × 100 + 2  × 100 3
P  a   b 
= 5.236 × 10 −31 m 3
16 Objective Physics Vol. 1

According to Avogadro’s hypothesis, one mole of hydrogen Sol. Time interval = 100 yr
contains
= 100 × 365 × 24 × 60 × 60s
N = 6.023 × 1023 atoms
= 3.155 × 109 s
∴ Atomic volume of 1 mole of hydrogen atoms,
Difference in time = 0.2 s
V = NV1 Difference in time(s)
or V = 6.023 × 1023 × 5.236 × 10−31 ∴ Fractional error =
Time interval(s)
= 3.154 × 10−7 m 3 0.2
= = 6.34 × 10−12
≅ 3 × 10−7 m 3 3.155 × 109
= 10 × 10−12 ≈ 10−11
Q 14. One mole of an ideal gas at standard temperature
and pressure occupies 22.4 L (molar volume). What ∴ In 1s, the difference is 10−11 to 6.34 × 1012.
is the ratio of molar volume to the atomic volume of Hence, degree of accuracy shown by the atomic clock in 1s
1 1
a mole of hydrogen? (Take the size of hydrogen is 1 part in −11 to
molecule to be about 1 Å). Why is this ratio so 10 6.34 × 10−12
large? or 1011 to 1012.

Sol. d = diameter of hydrogen molecule = 1 Å Q 17. Estimate the average mass density of sodium atom
assuming, its size to be about 2.5 Å (Use the known
Molar volume of one mole of hydrogen
values of Avogadro's number and the atomic mass
= 22.4 L of sodium). Compare it with the density of sodium
= 22.4 × 10−3 m 3 in its crystalline phase 970 kg m −3 . Are the two
r = radius of one molecule of hydrogen densities of the same order of magnitude? If so,
d why?
= = 0.5 Å = 0.5 × 10−10 m
2
Sol. Average radius of sodium atom,
Volume of one molecule of hydrogen
r = 2.5 Å = 2.5 × 10−10 m
4 4
= πr3 = π (0.5 × 10−10 )3 = 5.236 × 10−31 m 3 4
3 3 ∴ Volume of sodium atom = πr3
3
1 mole has 6.023 × 1023 atoms or molecules of H2. 4
= × 3.14 × (2.5 × 10−10 )3
∴ Atomic volume of one mole of hydrogen 3
= 6.023 × 1023 × 5.236 × 10−31 m 3 = 3154
. × 10−7 m 3 = 65.42 × 10−30 m 3
Mass of a mole of sodium = 23 g = 23 × 10−3 kg
Molar volume 22.4 × 10−3 m 3
∴ =
Atomic volume 3.154 × 10−7 m 3 One mole contains 6.023 × 1023 atoms, hence the mass of
sodium atom,
= 7.1 × 104 = 7 × 104
23 × 10−3
The large value of the ratio shows that the inter molecular M = kg
6.023 × 1023
separation in a gas is much larger than the size of a mulecule.
= 3.82 × 10−26 kg
Q 15. The nearest star to our solar system is 4.29 ly away. ∴ Average mass density of sodium atom,
How much is this distance in terms of parsecs? M 3.82 × 10−26
ρ= = kgm−3
Sol. Distance = 4.29 ly = 4.29 × 9.46 ×1015 m V 65.42 × 10−30
(Q 1 ly = 9.46 ×1015 m) = 0.64 × 103 kgm −3

4.29 × 9.46 × 1015 Density of sodium in crystalline phase = 970 kgm −3


= parsec
3.08 × 1016 = 0.970 × 103 kgm −3
(Q 1parsec = 3.08 × 10 16
m) Average mass density of sodium atom 0.64 × 103
∴ =
= 1.318 parsec = 1.32 parsec Density of sodium of crystalline phase 0.970 × 103
Q 16. It is claimed that the two cesium clocks if allowed of = 0.66
run for 100 yr free from any disturbance, may differ Both densities are of the same order, i.e. of the order of 103.
by only about 0.02 s. What does this imply for the
This is because in the solid phase atoms are tightly packed,
accuracy of the standard cesium clock in measuring so the atomic mass density is close to the mass density of the
a time interval of 1s? solid.
Units, Dimensions and Error Analysis 17

Q 18. A SONAR (sound navigation and ranging) uses Q 19. The farthest objects in our universe discovered by
ultrasonic waves to detect and locate objects under modern astronomers are so distant that light emitted
water. In a submarine equipped with a SONAR, the by them takes billions of years to reach the earth.
time delay between generation of a probe wave and These objects (known as quasars) have many
the reception of its echo after reflection from an puzzling features which have not yet been
enemy submarine is found to be 77.0 s. What is the
satisfactorily explained. What is the distance in km
distance of the enemy submarine? (speed of sound
of a quasar from which light takes 3.0 billion years
in water = 1450 ms −1 ).
to reach us?
Sol. Time taken by the wave to go from submarine to enemy
submarine is Sol. Time taken, t = 3 × 109 yr

t=
77
= 38.5 s = 3 × 109 × 365 × 24 × 60 × 60 s
2
Velocity of light, c = 3 × 108 ms−1
Speed of sound, v = 1450 ms−1
∴ Distance of quasar from earth = ct
Distance of enemy submarine,
= 3 × 108 × 3 × 109 × 365 × 24 × 3600 m
∴ s = vt = 1450 × 38.50
= 55825 m = 2.8 × 1025 m
= 55.825 km = 2.8 × 1022 km
Objective Problems
[ Level 1 ]
Measurement of Length and Time 11. Which of the following does not posses the same
1. Which one is not a unit of time? dimensions as that of pressure?
(a) Leap year (b) Year (c) Shake (d) Light year (a) Stress (b) Bulk modulus
(c) Thrust (d) Energy density
2. “Parsec” is the unit of
12. Which of the following is a dimensional constant?
(a) time (b) distance
(c) frequency (d) angular acceleration (a) Poission’s ratio (b) Refractive index
(c) Relative density (d) Gravitational constant
3. 1 light year distance is equal to
(a) 9.46 × 1010 km (b) 9.46 × 1012 km
13. Which one of the following is not the dimensionless
quantity?
(c) 9.46 × 109 km (d) 9.46 × 1015 km
(a) Planck’s constant (b) Dielectric constant
4. Parallactic second is equal to (c) Solid angle (d) Strain
(a) 9.4605 × 1015 m (b) 3.07 × 1016 m 14. Joule × second is the unit of
(c) 1.496 × 1011m (d) 3 × 108 m (a) energy (b) momentum
5. A new unit of length is chosen such that the speed of light (c) angular momentum (d) power
in vacuum is unity. What is the distance between the sun 15. Which of the following is not equal to watt?
and the earth in terms of the new unit, if light takes 8 min (a) joule/second (b) ampere × volt
and 20 s to cover this distance? (c) (ampere)2 × ohm (d) ampere/volt
(a) 300 (b) 400 (c) 500 (d) 600
6. A student measures the thickness of a human hair by 16. Which of the following is not the units of surface
looking at it through a microscope of magnification 100. tension?
He makes 20 observations and finds that the average (a) N / m (b) J / m 2
width of the hair is 3.5 mm. What is the estimate on the (c) kg/ s2 (d) None of these
thickness of the hair?
17. Wb/m 2 is equal to
(a) 0.0035 mm (b) 0.035 mm
(a) dyne (b) tesla (c) watt (d) henry
(c) 0.01 m (d) 0.7 mm
7. Which of the following is the most precise device for 18. Dimensional formula for electromotive force is same as
measuring length? that for
(a) potential (b) current
(a) A vernier calliper with 20 divisions on the sliding scale
(c) force (d) energy
(b) An optical instrument that can measure length to within a
wavelength of light 19. Which of the following has the dimensions of pressure?
(c) A screw gauge of pitch 1 mm and 100 divisions on the (a) [ML−2T −2 ] (b) [M −1L−1 ]
circular scale
(c) [MLT −2 ] (d) [ML−1T −2 ]
(d) All the above are equally precise
20. Dimensions of torque are
Units and Dimensions (a) [M 2L2T 2 ] (b) [ML2T −2 ]
8. The dimensions of impulse are equal to that of (c) [ML0T −1 ] (d) [ML2T −1 ]
(a) force (b) linear momentum
21. Dimensions of impulse are
(c) pressure (d) angular momentum
(a) [ML−2T −3 ] (b) [ML−2 ]
9. In the SI system, the unit of temperature is (c) [MLT −1 ] (d) [MLT −2 ]
(a) degree centigrade (b) kelvin
(c) degree celsius (d) degree fahrenheit 22. What is the dimensional formula of gravitational
constant?
10. Which one of the following have same dimensions?
(a) [ML2T −2 ]
(a) Torque and force
(b) [ML−1T −1 ]
(b) Potential energy and force
(c) Torque and potential energy (c) [M −1L3T −2 ]
(d) Planck’s constant and linear momentum (d) None of the above
Units, Dimensions and Error Analysis 19

23. Dimensions of surface tension are 35. Which of the following is dimensionally correct?
(a) [M 2L2T −2 ] (b) [M 2LT −2 ] (a) Pressure = energy per unit area
(c) [MT −2 ] (d) [MLT −2 ] (b) Pressure = energy per unit volume
(c) Pressure = force per unit volume
24. The dimensional formula for Young’s modulus is (d) Pressure = momentum per unit volume per unit time
(a) [ML−1T −2 ] (b) [M 0LT −2 ]
36. Assuming that the mass m of the largest stone that can be
(c) [MLT −2 ] (d) [ML2T −2 ]
moved by a flowing river depends upon the velocity v of
25. Which of the following is the dimensions of the the water, its density ρ and the acceleration due to gravity
coefficient of friction? g. Then, m is directly proportional to
(a) [M 2L2T] (b) [M 0L0T 0 ] (a) v 3 (b) v 4
(c) v 5 (d) v6
(c) [ML2T −2 ] (d) [M 2L2T −2 ]
37. If p represents radiation pressure, c represents speed of
26. The dimensional formula for the action will be
−2 2 −2 light and Q represents radiation energy striking a unit area
(a) [MLT ] (b) [M LT ]
per second, then non-zero integers x, y and z such that
(c) [ML2T −1 ] (d) [M 2L2T −2 ]
p x Q y c z is dimensionless are
27. [ML−1 T −1 ] stand for dimensions of (a) x = 1, y = 1, z = − 1
(a) work (b) torque (b) x = 1, y = − 1, z = 1
(c) linear momentum (d) coefficient of viscosity (c) x = − 1, y = 1, z = 1
(d) x = 1, y = 1, z = 1
28. Dimensions of relative density is
(a) [ML−2 ] (b) [ML−3 ] 38. The units of length, velocity and force are doubled.
Which of the following is the correct change in the other
(c) dimensionless (d) [M 2L−6 ]
units?
29. The dimensions of the ratio of angular to linear (a) Unit of time is doubled
momentum are (b) Unit of mass is doubled
(a) [M 0LT 0 ] (b) [MLT −1 ] (c) Unit of momentum is doubled
(d) Unit of energy is doubled
(c) [ML2T −1 ] (d) [M −1L−1T −1 ]
39. Which of the following pairs has the same units?
30. The dimensional formula for thermal resistance is
(a) Wavelength and Rydberg constant
(a) [ML2T −3K−1 ] (b) [ML2T −2A−1 ] (b) Relative velocity and relative density
(c) [ML2T −3K−2 ] (d) [M −1L−2T 3K] (c) Thermal capacity and Boltzmann constant
31. [ML2 T −3 A −1 ] is the dimensional formula for (d) Time period and acceleration gradient

(a) capacitance (b) resistance 40. The dimensional representation of specific resistance in
(c) resistivity (d) potential difference terms of charge Q is
32. Temperature can be expressed as a derived quantity in (a) [ML3T −1Q−2 ] (b) [ML2T −2Q2 ]
terms of any of the following. (c) [MLT −2Q−1 ] (d) [ML2T −2Q−1 ]
(a) length and mass (b) mass and time 41. Which of the following will have the dimensions of time?
(c) length, mass and time (d) None of these R L C
(a) LC (b) (c) (d)
t  L R L
33. Given that y = a cos  − qx , where t represents time. In
p  42. If C and R denote capacity and resistance, the dimensions
The following statements which is true? of CR are
(a) The unit of x is same as that of q (a) [M 0L0T]
(b) The unit of x is same as that of p (b) [ML0T]
(c) The unit of t is same as that of q
(c) [M 0L0T 2 ]
(d) The unit of t is same as that of p
(d) not expressible in terms of M, L and T
34. The dimensional formula [ML0 T −3 ] is more closely
43. The force F on a sphere of radius a moving in a medium
associated with
with velocity v is given by F = 6π ηa v. The dimensions
(a) power
(b) energy of ηare
(c) intensity (a) [ML−3 ] (b) [MLT −2 ]
(d) velocity gradient (c) [MT −1 ] (d) [ML−1T −1 ]
20 Objective Physics Vol. 1

x  55. Dimensions of electrical resistance are


44. The equation of a wave is given by y = a sin ω  − k
v  (a) [ML2T −3A−1 ] (b) [ML2T −3A−2 ]
where ω is angular velocity and v is the linear velocity. (c) [ML3T −3A−2 ] (d) [ML−1L3T 3A3 ]
The dimension of k will be
56. The magnetic moment has dimensions of
(a) [T −2 ] (b) [T1 ] (c) [T] (d) [LT]
(a) [LA] (b) [L2A] (c) [LT −1A] (d) [L2T −1A]
45. A force is given by F = at + bt , where t is the time. The
2

dimensions of a and b are 57. The dimensional representation of specific resistance in


−4
(a) [MLT ] and [MLT] −1
(b) [MLT ] and [MLT ] 0 terms of charge Q is
(c) [MLT −3 ] and [MLT −4 ] (d) [MLT −3 ] and [MLT 0 ] (a) [ML3T −1Q−2 ] (b) [ML2T −2Q2 ]
(c) [MLT −2Q−1 ] (d) [ML2T −2Q−1 ]
46. The dimensional formula for Planck’s constant and
angular momentum is Significant Figures
(a) [ML2T −2 ] and [MLT −1 ] (b) [ML2T −1 ] and [ML2T −1 ]
58. The significant figures of the number 6.0023 is
(c) [ML3T −1 ] and [ML2T −2 ] (d) [MLT −1 ] and [MLT −2 ]
(a) 2 (b) 5
1 (c) 4 (d) 1
47. The dimensions of ε 0 E 2 (ε 0 is the permittivity of the
2 59. What is the number of significant figures in
space and E is electric field), are 0.0310 × 103 ?
(a) [ML2T −1 ] (b) [ML−1T −2 ] (c) [ML2T −2 ] (d) [MLT −1 ] (a) 2 (b) 3
(c) 4 (d) 6
a a − t2
48. The dimensions of in the equation p = , where p
b bx 60. The number of significant figures in 11.118 × 10−6 V is
is pressure, x is distance and t is time, are (a) 3 (b) 4
(a) [M 2LT −3 ] (b) [MT −2 ] (c) 5 (d) 6
(c) [LT −3 ] (d) [ML3T −1 ] 61. In which of the following numerical values, all zeros are
49. Dimensions of velocity gradient are significant?
(a) [M 0L0T −1 ] (b) [ML−1T −1 ] (a) 0.2020 (b) 20.2
(c) 2020 (d) None of these
(c) [M 0LT −1 ] (d) [ML0T −1 ]
62. A student measured the diameter of a wire using a screw
50. The dimensional formula for emf e in MKS system will
gauge with least count 0.001 cm and listed the
be
measurements. The correct measurement is
(a) [ML2T −2Q−1 ] (b) [ML2T −1 ]
(a) 8.320 cm (b) 5.3 cm
(c) [ML−2Q−1 ] (d) [MLT −2Q−2 ] (c) 5.32 cm (d) 5.3200 cm
51. The velocity v of a particle at time t is given by 63. The length, breadth and thickness of rectangular sheet of
b
v = at + , where a, b and c are constants. The metal are 4.234 m, 1.005 m and 2.01 cm, respectively.
t+c The volume of the sheet to correct significant figures is
dimensions of a, b and c are, respectively (a) 0.0855 m 3 (b) 0.086 m 3
(a) [LT −2 ], [L] and [T] (b) [L2 ], [T] and [LT 2 ] (c) 0.08556 m 3 (d) 0.08 m 3
(c) [LT 2 ], [LT] and [L] (d) [L], [LT] and [T 2 ]
64. Three measurements are made as 18.425 cm, 7.21 cm and
1 5.0 cm. The addition should be written as
52. What is the units of k = ?
4 πε 0 (a) 30.635 cm (b) 30.64 cm (c) 30.63 cm (d) 30.6 cm
2 −1 −2 2 −2
(a) C N m (b) Nm C 65. Subtract 0.2 J from 7.26 J and express the result with
(c) Nm 2C2 (d) Unitless
correct number of significant figures.
53. Pressure gradient has the same dimensions as that of (a) 7.1 (b) 7.05
(a) velocity gradient (b) potential gradient (c) 7 (d) None of these
(c) energy gradient (d) None of these 66. Multiply 107.88 by 0.610 and express the result with
54. The unit of permittivity of free space, ε 0 is correct number of significant figures.
(a) 65.8068 (b) 64.807 (c) 65.81 (d) 65.8
(a) coulomb/newton-metre
(b) newton-metre 2/coulomb 2 67. When 97.52 is divided by 2.54, the correct result is
(c) coulomb 2/newton-metre 2 (a) 38.3937 (b) 38.394
(d) coulomb 2/(newton-metre) 2 (c) 65.81 (d) 38.4
Units, Dimensions and Error Analysis 21

68. The radius of a thin wire is 0.16 mm. The area of 77. A body travels uniformly a distance of (13.8 ± 0.2) m in a
cross-section of the wire in mm 2 with correct number of time (4.0 ± 0.3) s. The velocity of the body within error
significant figures is limit is
(a) 0.08 (b) 0.080
(a) (3.45 ± 0.2) ms−1
(c) 0.0804 (d) 0.080384
(b) (3.45 ± 0.3) ms−1
69. What is the number of significant figure in
(c) (3.45 ± 0.4) ms−1
(3.20 + 4.80) × 105 ?
(d) (3.45 ± 0.5) ms−1
(a) 5 (b) 4
(c) 3 (d) 2 78. If the error in the measurement of momentum of a
70. What is the value of [( 5.0 × 10 −6
) ( 5.0 × 10 −8
)] with due particle is ( + 100%), then the error in the measurement of
regards to significant digits? kinetic energy is
(a) 25 × 10−14 (b) 25.0 × 10−14 (a) 100% (b) 200%
(c) 300% (d) 400%
(c) 2.50 × 10−13 (d) 250 × 10−15
79. If the error in measuring diameter of a circle is 4%, the
71. The mass of a box is 2.3 kg. Two gold pieces of masses error in measuring radius of the circle would be
20.15 g and 20.17 g are added to the box. The total mass (a) 2% (b) 8%
of the box to correct significant figures is (c) 4% (d) 1%
(a) 2.3 kg (b) 2.34 kg
(c) 2.3432 kg (d) 2.31 kg 80. The values of two resistors are (5.0 ± 0.2) kΩ and
(10.0 ± 0.1) kΩ. What is the percentage error in the
72. Subtract 0.2 kg from 34 kg. The result in terms of proper
equivalent resistance when they are connected in
significant figure is
parallel?
(a) 33.8 kg (b) 33.80 kg
(a) 2% (b) 5%
(c) 34 kg (d) 34.0 kg
(c) 7% (d) 10%
73. The length, breadth and thickness of a block are given by 81. The heat generated in a wire depends on the resistance,
l = 12 cm, b = 6 cm and t = 2.45 cm. The volume of the current and time. If the error in measuring the above are
block according to the idea of significant figures should 1%, 2% and 1%, respectively. The maximum error in
be measuring the heat is
(a) 1 × 102 cm 3 (b) 2 × 102 cm 3 (a) 8% (b) 6%
(c) 1.763 × 102 cm 3 (d) None of these (c) 18% (d) 12%
82. A force F is applied on a square plate of side L. If the
Error Analysis percentage error in the determination of L is 2% and that
74. The length of a rod is (11.05 ± 0.2) cm. What is the length in F is 4%. What is the permissible error in pressure?
of the two rods? (a) 8% (b) 6%
(a) (22.1 ± 0.05) cm (c) 4% (d) 2%
(b) (22.1 ± 0.1) cm
83. A cuboid has volume V = l × 2l × 3l, where l is the length
(c) (22.10 ± 0.05) cm
(d) (22.10 ± 0.2) cm of one side. If the relative percentage error in the
measurement of l is 1%, then the relative percentage error
75. The radius of a ball is (5.2 ± 0.2) cm. The percentage in measurement of V is
error in the volume of the ball is approximately (a) 18% (b) 6%
(a) 11% (b) 4% (c) 3% (d) 1%
(c) 7% (d) 9%

76. A physical quantity Q is calculated according to the Miscellaneous Problems


expression 84. The ratio of the SI unit to the CGS unit of modulus of
A3 B3 rigidity is
Q=
C D (a) 102 (b) 10−2
(c) 10−1 (d) 10
If percentage errors in A , B, C , D are 2%, 1%, 3% and
4%, respectively. What is the percentage error in Q ? 85. Imagine a system of unit in which the unit of mass is
10 kg, length is 1 km and time is 1 min. Then, 1 J in this
(a) ± 8% (b) ± 10%
system is equal to
(c) ± 14% (d) ± 12%
(a) 360 (b) 3.6 (c) 36 × 105 (d) 36 × 10−5
22 Objective Physics Vol. 1

86. The dimensional formula for molar thermal capacity is 89. The square root of the product of inductance and
same as that of capacitance has the dimensions of
(a) gas constant (b) specific heat (a) length
(c) Boltzmann’s constant (d) Stefan’s constant (b) time
(c) mass
87. In measuring electric energy, 1kWh is equal to (d) no dimension
(a) 3.6 × 104 J (b) 3.6 × 106 J
(c) 7.3 × 106 J (d) None of these 90. With usual notation, the following equation, said to give
the distance covered in the nth second, i.e.
88. Out of the following four dimensional quantities, which ( 2n − 1)
one qualifies to be called a dimensional constant? sn = u + a is
2
(a) Acceleration due to gravity (a) numerically correct only
(b) Surface tension of water (b) dimensionally correct only
(c) Weight of a standard kilogram mass (c) both dimensionally and numerically only
(d) The velocity of light in vacuum (d) neither numerically nor dimensionally correct

[ Level 2 ]
Only One Correct Option dx  x − a
6. Given that ∫ = a n sin −1  
ε 0 lV 2ax − x 2  a 
1. A quantity is given by X = , whereV is the potential
t
where, a = constant. Using dimensional analysis, the
difference and l is the length. Then, X has dimensional
value of n is
formula same as that of
(a) 1 (b) zero
(a) resistance (b) charge (c) − 1 (d) None of these
(c) voltage (d) current

2. The length of a strip measured with a metre rod is 7. If E = energy, G = gravitational constant, I = impulse
GIM 2
10.0 cm. Its width measured with a vernier callipers is and M = mass, then dimensions of are same as
1.00 cm. The least count of the metre rod is 0.1 cm and E2
that of
that of vernier callipers 0.01 cm. What will be error in its
(a) time (b) mass
area? (c) length (d) force
(a) ± 13% (b) ± 7%
(c) ± 4% (d) ± 2% 8. The dimensional formula for magnetic flux is
(a) [ML2T −2A−1 ]
3. The length of cylinder is measured with a metre rod having
least count 0.1 cm. Its diameter is measured with vernier (b) [ML3T −2A−2 ]
calipers having least count 0.01 cm. Given that length is (c) [M 0L−2T −2A−2 ]
5.0 cm and radius is 2.0 cm. The percentage error in the (d) [ML2T −1A2 ]
calculated value of the volume will be
9. Using mass ( M ), length ( L ), time (T ) and current ( A ) as
(a) 1.5% (b) 2.5%
(c) 3.5% (d) 4% fundamental quantities, the dimensions of permeability
are
4. The random error in the arithmetic means of (a) [M −1LT −2A]
100 observations is x, then random error in the arithmetic
(b) [ML−2T −2A−1 ]
mean of 400 observations would be
1 (c) [MLT −2A−2 ]
(a) 4x (b)
4
x (d) [MLT −1A−1 ]
1
(c) 2x (d) x 10. Let g be the acceleration due to gravity at earth’s surface
2
and K the rotational kinetic energy of the earth. Suppose
5. Dimensions of ohm are same as the earth’s radius decreases by 2%. Keeping mass to be
h h2 constant, then
(a) (b)
e e (a) g increases by 2% and K increases by 2%
h h2 (b) g increases by 4% and K increases by 4%
(c) 2 (d) 2
e e (c) g increases by 4% and K increases by 2%
(where h is Planck’s constant and e is charge) (d) g increases by 2% and K increases by 4%
Units, Dimensions and Error Analysis 23

11. If the energy ( E ), velocity ( v ) and force ( F ) be taken as 20. Time for 20 oscillations of a pendulum is measured as
fundamental quantities, then the dimension of mass t 1 = 39.6 s, t 2 = 39.9 s and t 3 = 39.5 s.
will be What is the accuracy of the measurement?
(a) [Fv −2] (b) [Fv −1] (a) ± 0.1 s (b) ± 0.2 s (c) ± 0.01 s (d) ± 0.5 s
(c) [Ev −2] (d) [Ev 2]
More than One Correct Options
12. In a system of units, the units of mass, length and time
are 1 quintal, 1 km and 1 h, respectively. In this system, ab 2
1. Given, x = . If the percentage errors in a, b and c are
1 N force will be equal to c3
(a) 1 new unit (b) 129.6 new unit
± 1%, ± 3% and ± 2%, respectively.
(c) 427.6 new unit (d) 60 new unit
(a) The percentage error in x can be ±3%
13. If force F, length L and time T are taken as fundamental (b) The percentage error in x can be ±7%
units, the dimensional formula for mass will be (c) The percentage error in x can be ±18%
(a) [FL−1T 2 ] (b) [FLT − 2 ] (d) The percentage error in x can be ±19%
(c) [FL−1T −1 ] (d) [FL5T 2 ]
2. If P, Q, R are physical quantities, having different
 2π 
14. Given that y = A sin  ( ct − x ) , where y and x are dimensions, which of the following combinations can

 λ  never be a meaningful quantity?
measured in metre. Which of the following statements is (a) (P − Q )/ R (b) PQ − R (c) PQ / R (d) (R + Q )/ P
true?
3. If Planck’s constant (h) and speed of light in vacuum ( c )
(a) The unit of λ is same as that of x and A
(b) The unit of λ is same as that of x but not of A are taken as two fundamental quantities, which one of the
2π following can, in addition, be taken to express length,
(c) The unit of c is same as that of mass and time in terms of the three chosen fundamental
λ
2π quantities?
(d) The unit of (ct − x ) is same as that of
λ (a) Mass of electron (me )
(b) Universal gravitational constant (G )
15. The frequency of vibration of string is given by (c) Charge of electron (e)
1/ 2
p F  (d) Mass of proton (mp )
f =   . Here, p is number of segments in the
2l  m
Assertion and Reason
string and l is the length. The dimensional formula for m
Directions (Q. Nos. 1-17) These questions consists of two
will be
(a) [M 0LT −1 ] (b) [ML0T −1 ]
statements each printed as Assertion and Reason. While
(c) [ML−1T 0 ] (d) [M 0L0T 0 ] answering these questions, you are required to choose any one
of the following five responses.
16. You measure two quantities as A = 10 . m ± 0.2 m,
(a) If both Assertion and Reason are correct and Reason is
B = 2 .0 m ± 0.2 m. We should report correct value for the correct explanation of Assertion
AB as (b) If both Assertion and Reason are correct but Reason is
(a) 1.4 m ± 0.4 m (b) 1.41 m ± 0.15 m not the correct explanation of Assertion
(c) 1.4 m ± 0.3 m (d) 1.4 m ± 0.2 m
(c) If Assertion is true but Reason is false
17. Which of the following measurement is most precise? (d) If Assertion is false but Reason is true
(a) 5.00 mm (b) 5.00 cm (e) If both Assertion and Reason are false
(c) 5.00 m (d) 5.00 km
1. Assertion Pressure has the dimensions of energy
18. The mean length of an object is 5 cm. Which of the density.
following measurements is most accurate? energy [ML2 T −2 ]
(a) 4.9 cm (b) 4.805 cm Reason Energy density = =
(c) 5.25 cm (d) 5.4 cm volume [L3 ]

19. The vernier scale of a travelling microscope has = [ML−1 T −2 ] = pressure.


50 divisions which coincide with 49 main scale divisions.
If each main scale division is 0.5 mm, then what is the 2. Assertion Method of dimension cannot be used for
minimum inaccuracy in the measurement of distance? deriving formulae containing trigonometrical ratios.
(a) 0.02 mm (b) 0.05 mm Reason This is because trigonometrical ratios have no
(c) 0.01 mm (d) 0.1 mm dimensions.
24 Objective Physics Vol. 1

3. Assertion When percentage errors in the measurement 13. Assertion Systematic errors and random errors fall in
of mass and velocity are 1% and 2% respectively, the the same group of errors.
percentage error in KE is 5%.
Reason Both systematic and random errors are based on
1 ∆E ∆m 2∆v
Reason KE or E = mv 2 , = + the cause of error.
2 E m v
14. Assertion Absolute error may be negative or positive.
4. Assertion The error in the measurement of radius of the
sphere is 0.3%. The permissible error in its surface area is Reason Absolute error is the difference between the real
0.6%. value and the measured value of a physical quantity.
Reason The permissible error is calculated by the 15. Assertion The watches having hour hand, minute hand
formula and seconds hand have least count as 1 s.
∆A ∆r
=4 . Reason Least count is the maximum measurement that
A r
can be measured accurately by an instrument.
5. Assertion The light year and wavelength consist of 16. Assertion Pendulum bob is preferred to be spherical.
dimensions of length.
Reason Sphere has minimum surface area.
Reason Both light year and wavelength represent time.
17. Assertion A screw gauge having a smaller value of pitch
6. Assertion Number of significant figures in 0.005 is one has greater accuracy.
and that in 0.500 are three.
Reason The least count of screw gauge is directly
Reason This is because zeros before decimal are proportional to the number of divisions on circular scale.
non-significant.
Match the Columns
7. Assertion Out of two measurements l = 0.7 m and
l = 0.70 m, the second one is more accurate. 1. Match the following columns.
Reason In every measurement, more the last digit is not Column I Column II
accurately known. (A) R / L (p) Time
(B) C / R (q) Frequency
8. Assertion When we change the unit of measurement of a
(C) E / B (r) Speed
quantity, its numerical value changes.
(D) ε0 µ 0 (s) None of the above
Reason Smaller the unit of measurement smaller is its
numerical value. 2. Match the following columns.
9. Assertion L/ R and CR both have same dimensions. Column I Column II
Reason L/ R and CR both have dimensions of time. (A) Stress (p) Pressure
(B) Strain (q) Energy density
Magnetic dipole moment × moment induction (C) Modulus of elasticity (r) Angle
10. Assertion
Moment of inertia (D) Torque (s) Energy

Dimensional formula = [M 0 L0 T] 3. Suppose force ( F ), area ( A ) and time (T ) are the


Reason The given dimension is that of frequency. fundamental units, then the match the following columns.
Modulus of elasticity Column I Column II
11. Assertion has the unit ms −1 .
Density (A) Work (p) [A 1/ 2T −1 ]
(B) Moment of inertia (q) [FA 1/ 2 ]
1
Reason Acceleration has the dimensions of . (C) Velocity (r) [FA 1/ 2T 2 ]
( ε0 µ 0 ) t

an ∆x  ± ∆a   ± ∆b  4. Match the following columns.


12. Assertion If x = , then =n  − m 
bm x  a   b  Column I Column II

The change in a or b, i.e. ∆a or ∆b may be comparable to (A) Electrical resistance (p) [M−1L−2T 4 A 2 ]
a and b. (B) Capacitance (q) [ML2T −2A −2 ]

Reason The above relation is valid when ∆a << a and (C) Magnetic field (r) [ML2T −3A −2 ]

∆b << b. (D) Inductance (s) [MT −2A −1 ]


Units, Dimensions and Error Analysis 25

5. Match the following two columns. of 5 mA at 300 K, what will be the error in the value of
current in mA? [JEE Main]
Column I Column II
(a) 0.2 mA (b) 0.02 mA (c) 0.5 mA (d) 0.05 mA
(A) GMe M s (p) [M2L2T −3 ]
5. In a simple pendulum experiment, the maximum
(B) 3 RT (q) [ML3T −2 ] percentage error in the measurement of length is 2% and
M
that in the observation of the time period is 3%. Then, the
(C) F2 (r) [L2T −2 ] maximum percentage error in determination of the
q 2B 2 acceleration due to gravity g is [Kerala CEE]
(D) GMe (s) None of the above (a) 5% (b) 6% (c) 7% (d) 8%
Re (e) 10%
6. The pitch and the number of circular scale divisions in a
Entrance Gallery screw gauge with least count 0.02 mm are respectively.
[Kerala CEE]
2014 (a) 1 mm and 100 (b) 0.5 mm and 50
(c) 1 mm and 100 (d) 0.5 mm and 100
1. To find the distance d over which a signal can be seen
(e) 1 mm and 200
clearly in foggy conditions, a railway engineer uses
dimensional analysis and assumes that the distance 7. A physical quantity Q is found to depend on observables
depends on the mass density ρ of the fog, intensity x3 y2
x, y and z obeying relation Q = . The percentage
(power/area) S of the light from the signal and its z
frequency f. The engineer finds that d is proportional to error in the measurements of x, y and z are 1%, 2% and
S 1/ n . The value of n [JEE Advanced] 4% respectively. What is percentage error in the
(a) 2 (b) 3 quantity Q? [Karnataka CET]
(c) 1 (d) 4 (a) 4% (b) 3%
(c) 11% (d) 1%
2. During Searle’s experiment, zero of the vernier scale lies
between 3.20 × 10−2 m and 3.25 × 10−2 m of the main 2013
scale. The 20th division of the vernier scale exactly
8. Match List I with List II and select the correct answer
coincides with one of the main scale divisions. When an
using the codes given below the lists. [JEE Advanced]
additional load of 2 kg is applied to the wire, the zero of
the vernier scale still lies between 3.20 × 10−2 m and List I List II

3.25 × 10−2 m of the main scale but now the 45th P. Boltzmann constant 1. [ML2T −1 ]
division of vernier scale coincides with one of the main Q. Coefficient of viscosity 2. [ML−1T −1 ]
scale divisions. The length of the thin metallic wire is 2 m R. Planck's constant 3. [MLT −3 K −1 ]
and its cross-sectional area is 8 × 10−7 m 2 . The least S. Thermal conductivity 4. [ML2T −2 K −1 ]
count of the vernier scale is 1.0 × 10−5 m. The maximum
Codes
percentage error in the Young’s modulus of the wire is P Q R S P Q R S
[JEE Advanced] (a) 3 1 2 4 (b) 3 2 1 4
(a) 1 (b) 2 (c) 4 2 1 3 (d) 4 1 2 3
(c) 4 (d) 8
9. The diameter of a cylinder is measured using a vernier
3. A student measured the length of a rod and wrote it as callipers with no zero error. It is found that the zero of the
3.50 cm. Which instrument did he use to measure it? vernier scale lies between 5.10 cm and 5.15 cm of the
[JEE Main]
main scale. The vernier scale has 50 divisions equivalent
(a) A metre scale
(b) A vernier calliper where the 10 divisions in vernier scale to 2.45 cm. The 24th division of the vernier scale exactly
match with 9 divisions in main scale and main scale has coincides with one of the main scale divisions. The
10 divisions in 1 cm diameter of the cylinder is [JEE Advanced]
(c) A screw gauge having 100 divisions in the circular scale and (a) 5.112 cm (b) 5.124 cm
pitch as 1 mm (c) 5.136 cm (d) 5.148 cm
(d) A screw gauge having 50 divisions in the circular scale and
pitch 1 mm 10. Let [ ε 0 ] denotes the dimensional formula of the
4. The current voltage relation of diode is given by permittivity of vacuum. If M = mass, L = length, T = time
and A = electric current, then [JEE Main]
I = ( e1000 V / T − 1) mA, where the appliedV is in volts and
(a) [ε 0 ] = [M −1L−3 T 2A] (b) [ε 0 ] = [M −1L−3 T 4A2 ]
the temperature T is in degree kelvin. If a student makes −2 2 −1 −2
an error measuring ± 0.01V while measuring the current (c) [ε 0 ] = [M L T A ] (d) [ε 0 ] = [M −1L2T −1A2 ]
26 Objective Physics Vol. 1

11. The number of significant figures in the numbers 16. The dimensional formula of physical quantity is
4.8000 × 104 and 48000.50 are, respectively. [JEE Main] [M a Lb T c ]. Then, that physical quantity is
(a) 5 and 6 (b) 5 and 7 [Karnataka CET]
(c) 2 and 7 (d) 2 and 6 (a) spring constant, if a = 1, b = 1, and c = − 2
(b) surface tension, if a = 1, b = 1, and c = − 2
12. The dimensional formula for inductance is (c) force, if a = 1, b = 1, c = 2
[Karnataka CET] (d) angular frequency, if a = 0, b = 0, c = − 1
(a) [ML2 T −2A−2 ] (b) [ML2 TA−2 ]
(c) [ML2 T −1A−2 ] (d) [ML2 T −2A−1 ] 17. In a slide calliper, ( m + 1) number of vernier division is
equal to m number of smallest main scale divisions. If d
2012 unit is the magnitude of the smallest main scale division,
 4 MLg  then the magnitude of the vernier constant is [WB JEE]
13. In the determination of Young’s modulus Y =  (a) d /(m + 1) unit
 πld 2 
(b) d / m unit
by using Searle’s method, a wire of length L = 2 m and (c) md /(m + 1) unit
diameter d = 0.5 mm is used. For a load M = 2.5 kg, an (d) (m + 1)d / m unit
extension l = 0.25 mm in the length of the wire is
observed. Quantities d and l are measured using a screw 2011
gauge and a micrometer, respectively. They have the 18. The density of a solid ball is to be determined in an
same pitch of 0.5 mm. The number of divisions on their experiment. The diameter of the ball is measured with a
circular scale is 100. The contributions to the maximum screw gauge, whose pitch is 0.5 mm and there are
probable error of the Y measurement is [IIT JEE] 50 divisions on the circular scale. The reading on the
(a) due to the errors in the measurement of d and l are the same main scale is 2.5 mm and that on the circular scale is
(b) due to the error in the measurement of d is twice that due to 20 divisions. If the measured mass of the ball has a
the error in the measurement of l
relative error of 2%, the relative percentage error in the
(c) due to the error in the measurement of l is twice that due to
the error in the measurement of d density is [IIT JEE]
(d) due to the error in the measurement of d is four times that (a) 0.9% (b) 2.4%
due to the error in the measurement of l (c) 3.1% (d) 4.2%

14. Resistance of a given wire is obtained by measuring the 19. A screw gauge gives the following reading when used to
current flowing in it and the voltage difference applied measure the diameter of a wire.
across it. If the percentage errors in the measurement of Main scale reading : 0 mm
the current and the voltage difference are 3% each, then Circular scale reading : 52 divisions
error in the value of resistance of the wire is [AIEEE]
Given that, 1 mm on main scale corresponds to
(a) 6% (b) zero
100 divisions of the circular scale.
(c) 1% (d) 3%
The diameter of wire from the above data is [AIEEE]
15. A spectrometer gives the following reading when used to (a) 0.052 cm
measure the angle of a prism. (b) 0.026 cm
Main scale reading 58.5 degree. (c) 0.005 cm
(d) 0.52 cm
Vernier scale reading 9 divisions.
20. The mass and volume of a body are found to be
Given that, 1 division on main scale corresponds to 5.00 ± 0.05 kg and 1.00 ± 0.05 m 3 respectively. Then,
0.5 degree. Total division on the vernier scale is 30 and the maximum possible percentage error in its density is
match with 29 divisions of the main scale. The angle of [Kerala CEE]
the prism from the above data is [AIEEE] (a) 6% (b) 3%
(a) 58.59° (b) 59.77° (c) 10% .(d) 5%
(c) 58.65° (d) 59° (e) 7%
Answers
Level 1
Objective Problems
1. (d) 2. (b) 3. (b) 4. (b) 5. (c) 6. (b) 7. (b) 8. (b) 9. (b) 10. (c)
11. (c) 12. (d) 13. (a) 14. (c) 15. (d) 16. (d) 17. (b) 18. (a) 19. (d) 20. (b)
21. (c) 22. (c) 23. (c) 24. (a) 25. (b) 26. (a) 27. (d) 28. (c) 29. (a) 30. (d)
31. (d) 32. (d) 33. (d) 34. (c) 35. (b) 36. (d) 37. (b) 38. (c) 39. (c) 40. (a)
41. (c) 42. (a) 43. (d) 44. (c) 45. (c) 46. (b) 47. (b) 48. (b) 49. (a) 50. (a)
51. (a) 52. (b) 53. (d) 54. (c) 55. (b) 56. (b) 57. (a) 58. (b) 59. (b) 60. (c)
61. (b) 62. (a) 63. (a) 64. (d) 65. (a) 66. (d) 67. (d) 68. (b) 69. (c) 70. (a)
71. (a) 72. (c) 73. (b) 74. (d) 75. (a) 76. (d) 77. (b) 78. (c) 79. (c) 80. (c)
81. (b) 82. (a) 83. (c) 84. (d) 85. (d) 86. (c) 87. (b) 88. (d) 89. (b) 90. (c)

Level 2
Only One Correct Option
1. (d) 2. (d) 3. (b) 4. (b) 5. (c) 6. (b) 7. (a) 8. (a) 9. (c) 10. (b)
11. (c) 12. (b) 13. (a) 14. (a) 15. (c) 16. (d) 17. (a) 18. (a) 19. (c) 20. (b)

More than One Correct Option


1. (a,b) 2. (a,d) 3. (a,b,d)

Assertion and Reason


1. (a) 2. (a) 3. (a) 4. (c) 5. (a) 6. (c) 7. (b) 8. (c) 9. (a) 10. (d)
11. (b) 12. (d) 13. (a) 14. (a) 15. (d) 16. (a) 17. (c)

Match the Columns


1. A → q; B → p; C → r; D → s 2. A → r; B → p; C → s; D → q 3. A → q; B → r; C → p
4. A → s; B → p; C → r; D → q 5. A → q; B → r; C → r; D → s

Entrance Gallery
1. (b) 2. (c) 3. (b) 4. (a) 5. (d) 6. (c) 7. (c) 8. (c) 9. (b) 10. (b)
11. (b) 12. (a) 13. (a) 14. (a) 15. (c) 16. (d) 17. (a) 18. (c) 19. (a) 20. (a)
Solutions
Level 1 : Objective Problems Hence, [b ] =
[t 2 ]
1. Leap year, year and shake are the units of time. [px ]
3. 1 light year = ( 3 × 105 )( 365)( 24)( 3600) = 9.416 × 1012 km Dimensions of
a
= [px ] = [MT −2 ]
8. Impulse = change in linear momentum. b
13. Solid angle, strain and dielectric constant are 49. Velocity gradient is the change in velocity per unit length.
dimensionless quantities. 50. Unit of emf e is volt.
51. [a ] =   :[b ] = [vt ]:[c ] = [t ]
h v
14. Since, mvr = n ⋅ and E = hν
2π  t 
So, unit of h = joule-second = angular momentum 1 qq
54. F= × 122
17. Wb/m2 and tesla are the units of magnetic field. 4 πε0 r
21. Impulse = Force × time 1 qq (coulomb)2
⇒ ε0 = × 1 22 ⇒ ε0 =
24. Young’s modulus and pressure have the same dimensions. 4 π Fr newton - metre2
26. Action is a force. 55. From definition of time constant t = RC , where R is
Density of substance
28. Relative density = resistance and C is capacitance.
Density of water at 4° C temperature t [T ]
∴ R= = = [ML2 T −3 A −2 ]
= Dimensionless C [M−1 L−2 T 4 A 2 ]
36. m ∝ v ρ g . Writing the dimensions on both sides
a b c
56. M = NIA
−1 a
[M] = [LT ] [ML ] [LT ] −2 b −2 c ρl
57. Since, R = , where ρ is specific resistance
b a − 3b + c − a − 2c A
[M] = [M L T ]
[ρ] = 
RA  V W
∴ b =1 , R = ,V = = [ ML3Τ −1 Q −2 ]
 l  i Q
a − 3b + c = 0; − a − 2c = 0
68. R = 0.16 mm
Solving these, we get a =6 22
Hence, m ∝v 6 Hence, A = π R2 =× (016
. )2 = 0.080384
7
37. Since, pxQ yc z is dimensionless. Therefore, Since, radius has two significant figures so answer also will
have two significant figures.
[ML−1 T −2 ] x [MT −3 ]y [LT −1 ]z = [M0 L0 T 0 ]
∴ A = 0.080
Only option (b) satisfies this expression.
73. Minimum number of significant figure should be 1.
So, x =1, y = −1, z =1
75. Radius of ball = 5.2 cm
38. Since, units of length, velocity and force are doubled,
∆V ∆R 
= 3 
4
[force] [time] [length] V = πR 3 ⇒ 
Hence, [m] = , [time] = 3 V  R 
[velocity] [velocity]
 ∆V  × 100 = 3  0.2  × 100 =11%
Hence, unit of mass and time remains same.    
 V   5.2 
Momentum is doubled.
78. Since, error in measurement of momentum is + 100%
ρl
40. Since, R = , where ρ is specific resistance.
A ∴ p1 = p, p2 = 2p
 RA  V W p2 ( 2p)2
∴ [ρ] = , R = , V = , [ρ] = [ ML3 T −1Q −2 ] K1 = ,K2 =
 l  i Q 2m 2m
 K 2 − K1   4 − 1  × 100 = 300%
L
41. i = i0 {1 − e −t /( L / R ) } , where
is time constant and its % in K =   × 100 =  
R  K1   1 
dimension is same as for time. 81. H = i 2 R t
42. CR is time constant. ∴ % error in H = 2 (% error in i)
44. ωk is dimensionless. + (% error in R) + (% error in t)
45. [a ] =   and [b ] =  2 
F F F F −2
 t   t  82. p = = 2 = FL
A L
1
47. ε0 E 2 is energy density or energy per unit volume. % error in pressure = (% error in F) + 2 (error in L)
2 = ( 4%) + 2 ( 2%) = 8%
a − t2 1 1 T
48. p = , where p = pressure, t = time 89. f = or LC = =
bx 2π LC 2 πf 2 π
[pbx ] = [a ] = [t 2 ] Thus, LC has the dimensions of time.
Units, Dimensions and Error Analysis 29

Level 2 : Only One Correct Option 17. All given measurements are correct up to two decimal
places. As here 5.00 mm has the smallest unit and the error
3. Volume of cylinder, V = πr 2 L , r =  
D
in 5.00 mm is least (commonly taken as 0.01 mm if not
 2 specified), hence 5.00 mm is most precise.
∴  ∆V  × 100 = 2  ∆D  × 100 +  ∆L  × 100 Note In solving these types of questions, we should be careful about
     
 V   D   L  units although their magnitude is same.
18. Given length, l = 5 cm
= 2 
. 
001  01
. 
 × 100 +   × 100 = 2.5%
 40.   0.5  Now, checking the errors with each options one by one, we
get
4. Since, error is measured for 400 observations instead of
100 observations. So, error will reduce by 1/4 factor. ∆ l 1 = 5 − 4.9 = 0.1 cm
x ∆ l 2 = 5 − 4.805 = 0.195 cm
Hence, =
4 ∆ l 3 = 5. 25 − 5 = 0. 25 cm
h ∆ l 4 = 5. 4 − 5 = 0. 4 cm
5. Dimensions of ohm, R = 2 (e = charge = current × time)
e Error ∆l 1 is least.
= 2 = 2 = ( R ) as P =  
[Et ] P E Hence, 4.9 cm is most precise.
[it ] i t 19. By question, it is given that 50 VSD = 49 MSD
8. [φ] = [BS] = [MT −2 A −1 ] [L2 ] = [ML2 T −2 A −1 ] 1 VSD =
49
MSD
50
GM 1 L2
10. g = 2
: K = Iω2 = Minimum inaccuracy = 1 MSD − 1VSD
R 2 2I 49 1
= 1 MSD − MSD = MSD
Further, L will remain constant. 50 50
∴ K ∝
1
or K ∝
1 Given, 1 MSD = 0.5 mm
I 2 1
MR 2 Hence, minimum inaccuracy = × 0.5 mm
5 50
or K ∝ R −2 1
= = 0.01 mm
and g ∝ R −2 100
1 20. Given, t1 = 396 . s, t2 = 39.9 s and t3 = 39.5 s
11. Energy = mv 2
2 Least count of measuring instrument = 01 . s
[E ] (As measurements have only one decimal place)
[m] = 2 = [E v −2 ]
[v ] Precision in the measurement = Least count of the
measuring instrument = 01. s
12. [Froce] = [MLT 2 ]
Mean value of time for 20 oscillations is given by
1 N = 
1  1 
∴   (3600)
2
t +t +t . + 39.9 + 39.5
396
 100   1000  t= 1 2 3 = = 39.7 s
3 3
=1296
. units Absolute errors in the measurements
13. [FL T ] = [MLT ] [L−1 ] [T 2 ] = [M]
−1 2 −2
∆t1 = t − t1 = 39.7 − 396
. = 01
. s
π ∆t2 = t − t2 = 39.7 − 39.9 = − 0.2 s
14. Here, (ct − x ) is dimensionless.
λ ∆t3 = t − t3 = 39.7 − 39.5 = 0.2 s
ct | ∆t | + | ∆t 2 | + | ∆t 3 |
Hence, is also dimensionless and unit of ct is same as that Mean absolute error = 1
λ 3
of x. 0.1 + 0.2 + 0.2
=
Therefore, unit of λ is same as that of x. Also, unit of y is 3
same as that of A, which is also that unit of x. 0.5
= = 0.17 ≈ 0.2
15. m is mass per unit length. 3
16. Given, A = 10
. m ± 0. 2 m, B = 2.0 m ± 0. 2 m (rounding off up to one decimal place)
Let, Y = AB = (10 . ) = 1.414 m
. )( 20 ∴ Accuracy of measurement = ± 0.2 s

Rounding off to two significant digits Y =1.4 m More than One Correct Option
∆Y 1  ∆A ∆B  1  0.2 0.2  06
.
= + = + = 1. Maximum percentage error in
Y 2  A B  2 1.0 2.0  2 × 20
. x = (% error in a ) + 2 (% error inb ) + 3(% error in c )
06
.Y . × 1.4
06 = 1% + 6% + 6% =13%
⇒ ∆Y = =
2 × 20
. 2 × 20. Therefore, the correct options are (a) and (b).
2. In this question, it is given that P, Q and R are having
= 0. 212
different dimensions, hence they cannot be added or
Rounding off to one significant digit, ∆Y = 0. 2 m subtracted, so we can say that (a) and (d) are not
Thus, correct value for AB = Y + ∆Yr = 1.4 ± 0.2 m meaningful.
30 Objective Physics Vol. 1

3. We know that dimensions of h = [h] = [ML2 T −1 ] ∆Y ∆l 1.0 × 10−5 1


= = =
−1
[c ] = [LT ], [me ] = M Y l 2.5 × 10−4 25
∆Y 1
[G ] = [M−1 L3 T −2 ] × 100 = × 100 = 4%
Y 25
[e ] = [AT ], [mp ] = [ M]
3. As, 3.50 cm = 35 mm
2 −1 −1
 hc  = [ML T ][LT ] = [M2 ] A metre scale or foot scale gives measurement 35 mm than
 G  −1 3 −2
[M L T ] 36 mm. It does not give more precision.
hc For vernier scale with 1 MSD = 1 mm and 9 MSD = 10 VSD
M=
G Least count = 1 MSD − 1 VSD
9 1
h [ML2 T −1 ] = 1 mm − mm = mm = 0.1 mm
Similarly, = = [ML] 10 10
c [LT −1 ]
So, 35.0 mmcan be measured by vernier calliper.
h h G Gh
L= = = (c) Screw gauge will give 35.00 mm to 35.09 mm.
cM c hc c 3/ 2
(d) Screw gauge will give 35.00 or 35.02 or 34.98 mm.
As, c = LT −1
4. According to question, I = (e1000 V / T − 1) mA
[L ] Gh Gh
⇒ [T ] = = = I = 5 mA gives, 5 = e1000 V / T − 1
[c ] c 3/ 2 ⋅c c 5/ 2
⇒ e1000 V / T = 6 …(i)
Hence, (a), (b) or (d) any can be used to express L, M and T
Differentiating given equation,
in terms of three chosen fundamental quantities.
dI 1000 dI 6 × 1000
= e1000 V / T × ⇒ = = 20
Match the Columns dV T dV 300
dI = 20 dV = 20 (± 0.01) = ± 0.2 mA
3. [A] = [L2 ]
Here, voltage V is in volt, current I in mA.
∴ [L] = [A1/2 ] l
5. As, T = 2π
[T] = [T] g
[F] = [MLT −2 ] l l
⇒ T 2 = 4π2 ⇒ g = 4π2
∴ [M] = [FL−1 T 2 ] = [FA1/2 T 2 ] g T2
Now, [W] = [FL] = [FA1/2 ] ∆g ∆l ∆T
= +2
[I] = [ML ] = [FA
2 −1/2
T A] = [FA
2 1/2 2
T ] g l T
−1 −1 = 2% + 2 ( 3%) = 8%
[v] = [LT ] = [A 1/2
T ]
1 mm
6. (a) = 0.01 mm least count wrong.
Entrance Gallery 100
0.5 mm
1. According to analysis, d ∝ρx s y f z (b) = 0.01 mm least count wrong.
50
⇒ d = kρ s x y
f z
1 mm
(c) = 0.02 mm least count right.
Equating dimensions, 100
 ML2 T −3 
y 0.5 mm
(d) = 0.005 mm least count wrong.
[L] = [ML−3 ]x  2  [T ]
−1 z
100
 L  1 mm
(e) = 0.005 mm least count wrong.
[L] = [Mx + y L−3 x T −3 y − z ] 200
Equating powers on both sides, we get x 3y 2
7. Given, Q =
x + y = 0, 1 = − 3x, 0 = − 3y − z z
−1 1 ∆Q ∆x ∆y ∆z
x = , y = , z = −1 ⇒ =3 +2 +
3 3 Q x y z
⇒ d = k ρ−1/ 3 s 1/ 3 f −1
= 3 (1%) + 2 ( 2%) + ( 4%)
1 1 = ( 3 + 4 + 4) % = 11%
So, according to question, = ⇒ n= 3
n 3 8. As, pV = NkBT
F /A ∆Y ∆l
2. As, Y = ⇒ =
Boltzmann constant, [kB ] = 
as, F , A , L are exact quantities. pV 
l/ L Y l
 NT 
Here, ∆l = least count of vernier scale
= [ML2 T −2 K −1 ] ⇒ P → 4
= 1.0 × 10−5 m.
Stress
Coefficient of viscosity, η =
Measurement of l is along vernier scale only Velocity change per unit length
l = (45 −20) × 1.0 × 10−5
 MLT −2 / L2  −1 −1
= 2.5 × 10−4 m ⇒ [ η] =  −1  = [ML T ]
 T 
Here, one movement of vernier scale implies one least
count measurement. ⇒ Q→ 2
Units, Dimensions and Error Analysis 31

 ML2 T −2  Voltage difference V


E = hν ⇒ Planck’s constant, [h] =   = 
E 14. Resistance, R = =

 ν   T −1  Current I
∆R ∆V ∆I
= [ML2 T −1 ] %= %+ %
R V I
⇒ R →1 = 3% + 3% = 6%
∆Q KA ( T2 − T1 )  ML2 T −3 L  0.05
= ⇒ [K ] =   15. Least count = degree
∆t l 2
 L K  30
So, thermal conductivity = [K] = [ML T −3 K −1 ] Total reading = Main scale reading + (Vernier scale reading)
⇒ S→ 3 × (Least count)
 0.5 
= 58.5 degrees + (09)   degrees
9. Length of one Main Scale Division =1 MSD
 30 
= 5.15 − 5.10 = 0.05 cm
=  58.5 +  = 58.5 + 0.15
1.5
Length of one Vernier Scale Division = 1 VSD  10 
2.45 cm
= = 0.049 cm = 58.65 degree = 58.65°
50 −2
16. (a) [MLT ] = Force ⇒ Wrong
Least count = 1 MSD − 1 VSD = 0.05 cm − 0.049 cm
(b) [ML T −2 ] = Force ⇒ Wrong
= 0.001 cm
Vernier scale reading = ( 0.001 cm) (24) = 0.024 cm (c) [ML T 2 ] = Not force ⇒ Wrong
Main scale reading = 5.10 cm (d) [T −1 ] = Angular frequency (rad s−1 ) ⇒ Right
Total reading = 5.10 + 0.024 = 5.124 cm 17. 1 Main scale division = 1 MSD = d unit
Note The least count is also one division on main scale md
divided by number of division on vernier scale 1 Vernier scale division = 1 VSD = unit
m+1
 ( 5.15 − 5.10) cm = 0.05  = 0.001 cm
  Least count = Smallest measurement by instrument
 50 50 
m  m + 1 − m
10. As, force F =
1 q1 q2
⇒ ε0 =
q1 q2 = d− d = d 
4 πε0 r 2 4 π Fr 2 m+1  m+1 
d
 AT AT  = unit
[ ε0 ] = m+1
 MLT −2 L2 
0.5
= [M−1 L−3 T 4 A 2 ] 18. Least count of screw gauge =
50
11. 4.8000 × 104 has 4,8, 0, 0, 0 ⇒ 5 significant digits. = 001
. mm = ∆r
48000.50 has 4, 8, 0, 0, 0, 5, 0 ⇒ 7 significant digits. 0.5
Diameter, r = 2.5 mm + 20 ×
For number more than 1 digits, after decimal place are 50
significant.
= 2.70 mm
Power of 10 does not affect number of significant figures or
∆r 0.01 ∆r 1
digits. = or × 100 =
r 2.70 r 2.7
di
12. emf, ε = − L m m
dt Now, density d= =
V 4 r 3
⇒ εi = − L i
di π 
dt 3  2
⇒ [Power] = [L] [A 2 T −1 ] Here, r is the diameter.
∆d ∆m ∆r
× 100 =  + 3    × 100
 ML T  2 −3
⇒ ∴
[L] = [inductance] =  2 −1  d m  r 
 A T 
∆m ∆r
2 −2 −2
= [ML T A ] =× 100 + 3 ×   × 100
m  r 
4 MLg
13. Given, Y = , 1
πld 2 = 2% + 3 × = 3.11%
2.7
here M , L , g are exact (assumed)
∆Y ∆l 2 ∆d 19. Diameter of wire, d = MSR + CSR × LC
⇒ = +
Y l d 1
= 0 + 52 × = 0.52 mm
Contribution to maximum error, due to l measurement 100
∆l 0.05 /100 1 = 0.052 cm
= =
l 0.25 50 m ∆d ∆m ∆V
20. Density, d= ⇒ = +
Contribution to maximum error, due to d measurement V d m V
∆d
= 2 
0.05 /100  1 0.05 0.05
=2 = = × 100 + × 100 = 6
d  0.05  50 5 1
∆d
So, contribution due errors in measurement of d and l are ⇒ = 6%
same. d
2
Vectors

2.1 Scalars and Vectors


Any physical quantity is either a scalar or a vector. A scalar quantity can be described Chapter Snapshot
completely by its magnitude only. Addition, subtraction, division or multiplication of ● Scalars and Vectors
scalar quantities can be done according to the ordinary rules of algebra. Mass, volume, ● General Points
density, etc., are few examples of scalar quantities. If a physical quantity in addition to Regarding Vectors
magnitude has a specified direction as well as obeys the law of parallelogram of addition, ● Addition and Subtraction
then and then only it is said to be a vector quantity. Displacement, velocity, acceleration, of Two Vectors
etc., are few examples of vectors. ● Components of a Vector
● Product of Two Vectors
Extra Edge
■ Some Scalar Quantities Distance, speed, mass, volume, density, energy, power,
specific heat, time, frequency, work, pressure, surface tension, current, resistance, latent
heat, charge, etc.
■ Some Vector Quantities Displacement, velocity, acceleration, force, weight, momentum,
torque, electric field, magnetic field, etc.

2.2 General Points Regarding Vectors


Vector Notation
Usually a vector is represented by a bold capital letter, as A, B, C, etc.
The magnitude of a vector A is represented by A or | A | .

Graphical Representation of a Vector


Graphically, a vector is represented by an arrow drawn to a chosen scale, parallel to the
direction of the vector. The length and the direction of the arrow thus represent the
magnitude and the direction of the vector respectively.
Vectors 33

Thus, the arrow in Fig. 2.1 represents a vector A in Negative of a Vector


xy-plane making an angle θ with x-axis.
y It means a vector of same magnitude but opposite in
A direction.
A –A
Fig. 2.4
θ
Multiplication and Division of
x
Vectors by Scalars
Fig. 2.1 The product of a vector A and a scalar m is a vector mA
whose magnitude is m times the magnitude of A and which is
Equality of Vectors in the direction or opposite to A according as the scalar m is
positive or negative. Thus,
All vectors with the same magnitude and direction are
| mA | = mA
equal despite their different locations in space. Thus, if a
vector is displaced parallel to itself, it does not change. Further, if m and n are two scalars, then
( m + n) A = mA + nA and m ( nA ) = n ( mA ) = ( mn) A
A B
The division of vector A by a non-zero scalar m is defined
C 1
as the multiplication of A by ⋅
m

X Example 2.1 What is the angle between the two


vectors A and 2A?
Fig. 2.2
Sol. Angle between A and 2A is 0°, since2A is in the direction of A
In Fig. 2.2, A, B and C are all equal, since they have the and its magnitude is double.
same magnitude and direction even though they are
A
differently located in space.
2A

Angle between Two Vectors (θ ) Fig. 2.5

To find angle between two B A


vectors both the vectors are drawn 120°
2.3 Addition and Subtraction
from a point in such a manner that
arrows of both the vectors are
of Two Vectors
outwards from that point. Now, the
smaller angle is called the angle
(a) Addition
(i) The parallelogram law Let R be the resultant of two

between two vectors.


vectors A and B. According to parallelogram law of
For example in Fig. 2.3, angle B vector addition, the resultant R is the diagonal of the
between A and B is 60° not 120°.
parallelogram of which A and B are the adjacent sides
Because in Fig. (a) they are wrongly
as shown in figure. Magnitude of R is given by
drawn while in Fig. (b) they are
θ B R
drawn as we desire. A
(b)
Unit and Zero Vector Fig. 2.3 β
α
A vector of unit magnitude is called a unit vector and the A
$ read as ‘A hat or A θ
notation for it in the direction of A is A
Fig. 2.6
caret’.
Thus, A = AA$ R = A 2 + B 2 + 2 AB cos θ …(i)
A unit vector merely indicates a direction. Unit vectors Here, θ = angle between A and B. The direction of R
along x, y and z-directions are $i, $j and k.
$
can be found by angle α or β of R with A or B.
A vector of zero magnitude is called a zero or a null B sin θ
Here, tan α =
vector. Its direction is arbitrary. A + B cos θ
34 Objective Physics Vol. 1

A sin θ B sin (180° − θ ) B sin θ


and tan β = …(ii) tan α = = …(ii)
B + A cos θ A + B cos (180° − θ ) A − B cos θ
Special Cases A sin (180 − θ ) A sin θ
or tan β = = …(iii)
If θ = 0° , R = maximum = A + B B + A cos (180 − θ ) B − A cos θ
θ = 180°, R = minimum = A ∼ B
and if θ = 90° , R = A 2 + B 2
Extra Edge
■ A –B or B – A can also be found by making triangles
In all other cases, magnitude and direction of R can be as shown in Fig. 2.10 (a) and (b).
calculated by using Eqs. (i) and (ii). B B B–A
(ii) The triangle law According to this law, if the tail of
A–B or
one vector be placed at the head of the other, their sum
or resultant R is drawn from the tail end of the first to A
A
the head end of the other. As it is evident from the (a) (b)
figure that the resultant R is the same irrespective of Fig. 2.10
the order in which the vectors A and B are taken, thus,
R = A +B = B + A
A
X Example 2.2 Prove the results

R
R = A 2 + B 2 + 2 AB cos θ
B B B sin θ
R and tan α = in vector addition method.
A + B cos θ
A
Fig. 2.7 Sol. Let OP and OQ represent the two vectors A and B making an
angle θ. Then, using the parallelogram method of vector
Subtraction addition, OS represents the resultant vector R.
Q S
Negative of a vector say −A is a vector of the same
magnitude as vector A but pointing in a direction opposite to
that of A.
o

θ α 90
θ
A O
P N
–A
Fig. 2.11
Fig. 2.8
R=A+B
Thus, A − B can be written as A + ( −B ) or A − B is really OP = A, OQ = PS = B, OS = R
SN is normal to OP.
the vector addition of A and −B.
From the geometry of the figure,
Suppose angle between two vectors A and B is θ. Then, OS 2 = ON2 + SN2
angle between A and –B will be 180°– θ as shown in but ON = OP + PN = A + B cosθ
Fig. 2.9(b). SN = B sinθ
B 180° – θ OS 2 = ( A + Bcos θ )2 + (Bsinθ )2
A
α or R 2 = A 2 + B2 + 2 AB cosθ
β
⇒ R= A 2 + B2 + 2 AB cosθ
θ
SN B sin θ
A –B tan α = =
S=A–B OP + PN A + B cos θ
(a) (b)
Fig. 2.9
X Example 2.3 Find A + B and B

Magnitude of S = A – B will be thus given by A – B in the diagram shown in


S = | A – B | = A 2 + B 2 + 2 AB cos (180° − θ ) figure. Given A = 4 units and
θ = 60°
B = 3 units. A
or S = A 2 + B 2 − 2 AB cos θ …(i) Sol. Fig. 2.12
For direction of S, we will either calculate angle α or β, R= A 2 + B2 + 2 AB cos θ
where,
Vectors 35

= 16 + 9 + 2 × 4 × 3 cos 60° = 37 units


Here, R x , R y and R z are the components of R in x, y and
B R=A+B z-axes, respectively and $i , $j and k$ are unit vectors along
these directions. The magnitude of R is given by

α
R = R x2 + R y2 + R z2
A
θ R 
Fig. 2.13 This vector R makes an angle of α = cos −1  x  with
 R 
B sin θ 3 sin 60° Rx
tan α = = = 0.472 x-axis or cos α =
A + B cos θ 4 + 3 cos 60° R
∴ α = tan−1 ( 0.472 ) = 25.3°  Ry  Ry
β = cos −1   with y-axis or cos β =
Thus, resultant of A and B is 37 units at angle 25.3° from A  R  R
in the direction shown in figure.
R  R
Subtraction S= A 2 + B2 − 2 AB cos θ and γ = cos −1  z  with z-axis or cos γ = z
 R  R
= 16 + 9 − 2 × 4 × 3 cos 60°
= 13 units
/ cos α, cos β and cos γ are called direction cosines of R.
B sin θ Exercise Prove that
and tan α =
A − B cos θ
(a) cos 2 α + cos 2 β + cos 2 γ = 1
3 sin 60°
= = 1.04
4 − 3 cos 60° (b) sin 2 α + sin 2 β + sin 2 γ = 2
∴ α = tan−1 (1.04) = 46.1° Refer Fig. 2.16 (a)
Thus, A – B is 13 units at 46.1° from A in the direction We have resolved a two-dimensional vector R in
shown in figure below. mutually perpendicular directions x and y.
θ
A Component along x-axis = R x = R cos α or R sin β and
α component along y-axis = R y = R cos β or R sin α.
If $i and $j be the unit vectors along x and y -axis
–B S=A–B
respectively, we can write
Fig. 2.14 R = R x $i + R y $j
/ Components of a vector in mutually perpendicular directions
2.4 Components of a Vector are called the rectangular components.
y y
Two or more vectors which, when compounded in
accordance with the parallelogram law of vector R are said
Ry R Ry
to be components of vector R. The most important α α R
components with which we are concerned are mutually β β β
β
perpendicular or rectangular ones along the three α α P
O x O x
co-ordinate axes ox, oy and oz, respectively. Thus, a vector R Rx Rx
can be written as R = R x $i + R y $j + R z k.
$ (a) (b)
y Fig. 2.16
Refer Fig. 2.16 (b)
Vector R has been resolved in two axes x and y not
perpendicular to each other. Applying sine law in the
Ry

Ry triangle OPR shown above, we have


R

β R R Ry
α = x =
γ Rx x sin [180° − (α + β)] sin β sin α
R sin β
Rx =
z
R

or
Rz sin (α + β)
Rx
z R sin α
and Ry =
A vector R resolved into components sin (α + β)
along x,y and z-axes.
Fig. 2.15 If α + β = 90° , R x = R sin β and R y = R sin α
36 Objective Physics Vol. 1

Then,
Extra Edge OP is called position vector of point P with respect to the
■ Representation of a vector in terms of $i , $j and k$ origin O. This is also denoted by rP . From polygon law of
A force F is of 5 N as shown in figure. Let us represent vector addition, in the triangle OMP we can also see that,
this vector in terms of $i and $j. OP = OM + MP
y where, OM = x $i and MP = y$j
F ∴ OP = r = (x $i + y$j )
P
M
In the similar manner, we can show that if co-ordinates of
Fy point P are (x, y, z) then
o Position vector of P = OP = rP = (x $i + y$j + z k$ )
37
x
O Fx P ■ Displacement Vector In the figure shown, let :
A = (x A , y A ) and B = (xB , yB )
Fig. 2.17 Then, rA = position vector of A
= (x A $i + y A $j ) and
Fx = component of F along x-axis
rB = position vector of B
= F cos 37 ° = (5)   = 4 N
4
5 = (xB $i + yB $j )

Fy = component of F along y-axis If a particle is displaced from point A to point B, then AB


is called its displacement vector S.
= F sin 37 ° = (5)   = 3 N
3
5 y
B
Now, F = OP + PM (From polygon law of vector addition) S
A
or F = (4 $i + 3 $j ) N
rA rB
■ If F = (−6$i + 8 $j ) N, then its magnitude and direction can
be found as under
x
y O
F Fig. 2.20
^
M 8j In triangle OAB, from subtraction of vectors we can say
Fy 8N that,
AB or S = rB − rA
α
P x = (xB $i + yB $j ) − (x A $i + y A $j )
_ 6i^ O
= (xB − x A ) $i + (yB − y A ) $i
6N
In the similar manner, we can show that, if a particle is
Fig. 2.18 displaced from point A (x1, y1, z1) to point B (x 2 , y 2 , z 2 )
From parallelogram law, then, displacement of particle
S = rB − rA
F = OP + OM = (−6$i + 8 $j ) N
= (x $i + y $j + z k$ ) − (x $i + y $j + z k$ )
2 2 2 1 1 1
∴ |F| = F = (6)2 + (8)2 = 10 N
= (x 2 − x1) $i + (y 2 − y1) $j + (z 2 − z1) k$
8 4
tan α = =
6 3
X Example 2.4 Resolve a weight of 10 N in two
α = tan−1   = 53 °
4

 3 directions which are parallel and perpendicular to a
Hence, the given F has a magnitude of 10 N and it makes slope inclined at 30° to the horizontal.
an angle of 53° from negative x-axis towards positive
y-axis. Sol. Component perpendicular to the plane
■ Position Vector In the figure shown, suppose
co-ordinates of a point P are (x, y).
w| |
y
30° w⊥
P (x, y)
30° W = 10 N
rp
y Fig. 2.21

3
W⊥ = W cos 30° = (10) = 5 3N
x 2
O M
x and component parallel to the plane
1
Fig. 2.19 W|| = W sin 30° = (10)   = 5 N
2
Vectors 37

X Example 2.5 Resolve horizontally and vertically (vi) $i ⋅ $j = $j ⋅ k$ = $i ⋅ k$ = 1, 1 cos 90° = 0


a force F = 8 N which makes an angle of 45° with the (vii) ( a1 $i + b1 $j + c1 k$ ) ⋅ ( a 2 $i + b2 $j + c2 k$ ) = a1 a 2 + b1 b2
horizontal.
+ c1 c2
Sol. Horizontal component of F is FV
A ⋅B
1  (viii) cos θ = = (cosine of angle between A and B)
FH = F cos 45° = (8)   =4 2N AB
 2
and vertical component of F is F (ix) Two vectors are perpendicular if their dot product is
FV = F sin 45° = (8) 
1  zero. (θ = 90° )
 =4 2N
 2 A ⋅B
Two vectors in the form of $i, $j and
45° (x) Component of A along B = A cos θ =
FH B
k$ can be added, subtracted on
multiplied by a scalar directly as
Fig. 2.22
A ⋅B
done in the following example. Similarly, component of B along A = B cosθ =
A
A
X Example 2.6 Obtain the magnitude of 2A − 3B if
A = $i + $j − 2k
$ and B = 2$i − $j + k
$
θ B
Sol. 2 A − 3 B = 2 (i$ + $j − 2k$ ) − 3 (2 $i − $j + k$ )
A cos θ
= − 4$i + 5$j − 7k$
Fig. 2.24
∴ Magnitude of
2 A − 3 B = (−4)2 + (5)2 + (−7 )2 X Example 2.7 Work done by a force F on a body
= 16 + 25 + 49 = 90 is W = F ⋅ s, where s is the displacement of body. Given
= − 4$i + 5$j − 7k$ that under a force F = (2$i + 3$j + 4k$ ) N a body is
displaced from position vector r1 = (2$i + 3$j + k$ ) m to
2.5 Product of Two Vectors the position vector r2 = ( $i + $j + k$ ) m . Find the work
The product of two vectors is of two kinds. done by this force.
1. A scalar or dot product / Displacement vector is given by s = rf − ri
2. A vector or a cross product
Sol. The body is displaced from r1 to r2 . Therefore, displacement
of the body is
Scalar or Dot Product s = r2 − r1 = (i$ + $j + k$ ) − (2 $i + 3$j + k$ ) = (− $i − 2 $j ) m
The scalar or dot product of two B
Now, work done by the force is W= F ⋅ s
vectors A and B is denoted by A ⋅ B
= (2 i$ + 3$j + 4k$ )⋅(− i$ − 2 $j )
and is read as A dot B.
θ = (2 )(−1) + (3 )(−2 ) + (4) (01) = − 8 J
It is defined as the product of A
the magnitudes of the two vectors A Fig. 2.23 X Example 2.8 Find the angle between two vectors
and B and the cosine of their
A = 2$i + $j − k$ and B = $i − k.
$
included angle θ.
Thus, A ⋅ B = AB cos θ (a scalar quantity) Sol. A = |A| = (2 )2 + (1)2 (−1 )2 = 6

Important Points Regarding Dot Product B = |B|= (1)2 + (−1)2 = 2


The following points should be remembered regarding A ⋅B = (2i$ + $j − k$ )⋅(i$ − k$ )
the dot product : = (2 )(1) + (1) (0) + (−1)(−1) = 3
(i) A ⋅ B = B ⋅ A A⋅ B 3 3 3
Now, cos θ = = = =
(ii) A ⋅ (B + C ) = A ⋅ B + A ⋅ C AB 6⋅ 2 12 2

(iii) A ⋅ A = AA cos 0° = A 2 ∴ θ = 30°

(iv) A ⋅ B = A ( B cos θ ) = A (Component of B along A) X Example 2.9 Prove that the vectors
or A ⋅ B = B ( A cos θ ) = B (Component of A along B) A = 2$i − 3$j + k
$ and B = $i + $j + k
$ are mutually
(v) $i ⋅ $i = $j ⋅ $j = k$ ⋅ k$ = 1, 1 cos 0° = 1 perpendicular.
38 Objective Physics Vol. 1
∧ ∧
Sol. A ⋅ B = (2 $i − 3$j + k$ ) ⋅($i + $j + k$ ) triad of unit vectors i i
$i , $j and k$ (each
= (2 )(1) + (−3)(1) + (1)(1) = 0 = AB cos θ
∴ cos θ = 0 (as A ≠ 0, B ≠ 0) perpendicular to Plus Minus
∧ ∧ ∧ ∧
or θ = 90° each other) k j k j
or the vectors A and B are mutually perpendicular. $i × $j = − $j × $i = k$
Fig. 2.27
Vector or Cross Product $j × k$
The cross product of two B = − k$ × $j = $i and k$ × $i = − $i × k$ = $j
vectors A and B is denoted by (iv) A × (B + C ) = A × B + A × C
A × B and read as A cross B. It (v) A vector product can be expressed in terms of
is defined as a third vector C rectangular components of the two vectors and put in
whose magnitude is equal to θ
A the determinant form as may be seen from the
the product of the magnitudes Fig. 2.25 following:
of the two vectors A and B and
Let A = a $i + b $j + c k$ and B = a $i + b $j + c k$
1 1 1 2 2 2
the sine of the angle between A and B ,θ.
Then, A × B = ( a1 $i + b1 $j + c1 k$ ) × ( a 2 $i + b2 $j + c2 k$ )
Thus, if C = A × B, then C = AB sin θ.
The vector C is normal to the C=Ax B = a a ( $i × $i ) + a b ( $i × $j) + a c ( $i × k$ )
1 2 1 2 1 2
plane of A and B and points in the + b1 a 2 ( $j × $i ) + b1 b2 ( $j × $j)
direction in which a right handed
screw would advance when + b c ( $j × k$ ) + c a ( k$ × $i )
1 2 1 2
rotated about an axis + c1 b2 ( k$ × $j) + c1 c2 ( k$ × k$ )
perpendicular to the plane of the
two vectors in the direction from Since, $i × $i = $j × $j = k$ × k$ = a null vector and
A to B through the smaller angle $i × $j = k$ , etc., we have
B
θ between them or alternatively, A A × B = ( b c − c b ) $i + ( c a − a c ) $j
1 2 1 2 1 2 1 2
we might state the rule as :
If the fingers of the right Plane of A and B + ( a1 b2 − b1 a 2 ) k$
Fig. 2.26
hand be curled in the direction in or putting it in determinant form, we have
which vector A must be turned through the smaller included $i $j k$
angle θ to coincide with the direction of vector B, the thumb A × B = a1 b1 c1
points in the direction of C as shown in Fig. 2.26.
a2 b2 c2
Either of these rules is referred to as the right handed
screw rule. Thus, if n$ be the unit vector in the direction of C, It may be noted that the scalar components of the first
we have vector A occupy the middle row of the determinant.
C = A × B = AB sin θ n, $
X Example 2.10 Find one unit vector perpendicular
where 0 ≤θ ≤ π $ and B = $i − $j + k
to A = 2$i + 3$j + k $ both.
Important Points About Vector Product
Sol. As we have read, C = A × B is a vector perpendicular to both
(i) A × B = − B × A
A and B. Hence, a unit vector n$ perpendicular to A and B can be
(ii) The cross product of two parallel or anti-parallel written as
vectors is a null vector, as | A × B | = AB sin θ and C
n$ = =
A×B
sin θ = 0 for two parallel or anti-parallel vectors C | A × B|
( as θ = 0° or 180° ). $i $j k$

Thus, $i × $i = $j × $j = k$ × k$ = a null vector Here, A×B= 2 3 1


1 –1 1
(iii) If two vectors are perpendicular to each other, we have
θ = 90° and therefore, sin θ =1. So that A × B = AB n.
$ = i$ (3 + 1) + $j (1 − 2 ) + k$ (−2 − 3)
The vectors A, B and A × B thus form a right handed = 4$i − $j − 5k$
system of mutually perpendicular vectors. It follows at
Further, |A × B| = (4)2 + (−1)2 + (−5)2
once from the above that in case of the orthogonal
= 42
Vectors 39

∴ The desired unit vector is


A×B
n$ = Extra Knowledge Points
| A × B|
1
■ Pressure, surface tension and current are not vectors.
or n$ = (4i$ − $j − 5k$ )
42 ■ To qualify as a vector, a physical quantity must not only
possess magnitude and direction but must also satisfy
the parallelogram law of vector addition. For instance,
X Example 2.11 Show that the vector A = $i − $j + 2k$
the finite rotation of a rigid body about a given axis has
is parallel to a vector B = 3$i − 3$j + 6k
$. magnitude (the angle of rotation) and also direction (the
direction of the axis) but it is not a vector quantity. This is
Sol. A vector A is parallel to an another vector B if it can be written so for the simple reason that the two finite rotations of
as the body do not add up in accordance with the law of
A = mB vector addition. However, if the rotation be small or
infinitesimal, it may be regarded as a vector quantity.
Here, A = ($i − $j + 2k$ )
1
■ Area can behave either as a scalar or a vector and how
= (3 i$ − 3 $j + 6 k$ ) it behaves depends on circumstances.
3
1 ■ Moment of inertia is neither a vector nor a scalar as it
or A= B
3 has different values about different axes. It is tensor.
1 Although tensor is a generalized term which is
This implies that A is parallel to B and magnitude of A is
3 characterized by its rank. For example, scalars are
times the magnitude of B. tensor of rank zero. Vectors are tensor of rank one.
■ Area (vector), dipole moment and current density are
Note Points defined as vectors with specific direction.
/ Two vectors can be shown parallel (or anti-parallel) to one ■ Vectors associated with a linear or directional effect are
another if called polar vectors or usually, simply as vectors and
(i) the coefficients of $i , $j and k$ of both the vectors bear a positive those associated with rotation about an axis are referred
constant ratio. For example, a vector A = a1$i + b1$j + c1k$ is to as axial vectors. Thus, force, linear velocity and
parallel to an another vector B = a2 $i + b 2 $j + c2k$ if : acceleration are polar vectors and angular velocity,
a1 b 1 c1 angular acceleration are axial vectors.
= = = a positive constant. If the vectors are
a2 b 2 c2 ■ Students are often confused over the direction of cross
anti-parallel, then this constant ratio is negative. product. Let us discuss a simple method. To find
(ii) the magnitude of cross product of both the vectors is zero. For direction of A × B curl your fingers from A to B through
instance, A and B are parallel (or antiparallel) to each other if smaller angle. If it is clockwise, then A × B is
$i $j perpendicular to the plane of A and B and away from
k$
you and if it is anti-clockwise then, A × B is towards you
A × B = a1 b1 c1 = 0
perpendicular to the plane of A and B.
a2 b 2 c2
■ The area of triangle bounded by B
or A × B = 0$i + 0$j + 0k$ 1
vectors A and B is | A × B |.
2
X Example 2.12 Let a force F be acting on a body ■ Exercise : Prove the above
free to rotate about a point O and let r be the position result. A
R Q
vector of any point P on the line of action of the force.
Then, torque (τ ) of this force about point O is defined B
θ
as
τ=r×F O
A P

Given, F = (2$i + 3$j − k $) N ■ Suppose A and B represent the two adjacent sides of a
parallelogram OPQR, then,
and r = ( $i − $j + 6k
$)m
Diagonal OQ = | A + B | = A 2 + B 2 + 2AB cos θ
Find the torque of this force.
while diagonal RP = | A − B | = A 2 + B 2 − 2AB cos θ
$i $j k$
We can see that OQ = RP when θ = 90°.
Sol. τ = r × F = 1 –1 6 ■a ⋅ b = ab cos θ. Here, a and b are always positive as
2 3 –1 these are the magnitudes of a and b. Hence,
0° ≤ θ < 90° if a ⋅ b is positive
= i$ (1 − 18) + $j (12 + 1) + k$ (3 + 2 )
90° ≤ θ < 180° if a ⋅ b is negative.
or τ = (−17 $i + 13$j + 5k$ ) N-m and θ = 90° if a ⋅ b is zero.
Chapter Summary with Formulae
(i) Vector Addition (c) Cross product of two vectors of given magnitudes has
(a) Law of parallelogram of vector addition maximum value when they act at 90 °.
R = A+ B (d) $i × $j = k$ , $j × k $ = $i, k
$ × $i = $j, ^i
$i × k$ = − $j , k$ × $j = − $i
R =| R |
R=
A2 + B2 + 2AB cosθ $j × $i = − k$,
^ + ^j
$i × $i = $j × $j = k $ ×k $ = a null vector k
B sin θ
tanα =
A + B cos θ (vi) A Unit Vector in the Direction of A
Asin θ $ = A or A = A A $
and tan β = A
B + Acos θ A
(b) Vector addition of more than two vectors (vii) A Unit Vector Perpendicular to both A and B
Polygon law of vector addition can be applied for Let us call it C$ .
addition of two or more than two vectors. A×B
Then C$ = ±
|A × B|
(ii) Vector Subtraction
If S = A − B and S =| S| then A
S= A2 + B2 − 2AB cosθ
(iii) Dot Product of Two Vectors
A ⋅B = ABcosθ θ B
■ Important Points in Dot Product A cos θ
(a) A ⋅B = B ⋅A
(b) A ⋅ ( − B) = − A ⋅ B A⋅B
(viii) Component of A along B = Acosθ =
(c) A ⋅ ( B + C ) = A ⋅ B + A ⋅ C B
A⋅B
(d) If θ is acute, dot product is positive. If θ is obtuse dot Similarly, component of B along A = B cos θ =
A
product is negative and if θ is 90 ° dot product is zero.
(ix) Angle between Two Vectors
Hence dot product of two perpendicular vectors is
A ⋅ B
zero. θ = cos −1  
 AB 
(e) A ⋅ A = A2
$
(x) If Vectors are given in Terms of $i, $j and k
(f) Dot product is a scalar quantity.
(g) Work done W = F⋅ S = F ⋅ ( rf − ri ) Let A = a1 i + a2 j + a3 k and B = b1 i + b2 $j + b3 k
$ $ $ $ $,
$i ⋅ $i = $j ⋅ $j = k
$ ⋅k
$ = 1, $i ⋅ $j = $j ⋅ k
$ = $i ⋅ k
$ =0 then
(h)
(a)|A|= A = a12 + a22 + a32 and
(iv) Position Vector and y
Displacement Vector. S B |B|= B = b12 + b22 + b32
If coordinates of point A
rA rB (b) A + B = ( a1 + b1 ) $i + ( a2 + b2 ) $j + ( a3 + b3 ) k
$
A are ( x1, y1, z1 ) and (c) A − B = ( a1 − b1 ) i + ( a2 − b2 ) j + ( a3 − b3 ) k
$ $ $
coordinates of point B (d) A ⋅ B = a1b1 + a2 b2 + a3 b3
are ( x2 , y2 , z2 ). Then, x
O $i $j $
k
rA = position vector of A
(e)|A × B|= a1 a2 a3
= x1$i + y1$j + z1k
$
rB = x2 i + y2 j + z2 k
$ $ $ b1 b2 b3
S = rB − rA = ( x2 − x1 ) $i + ( y2 − y1 ) $j + ( z2 − z1 ) k
$ = ( a2 b3 − b2 a3 ) $i + ( b1a3 − b3 a1 ) $j + ( a1b2 − b1a2 ) k
$
= displacement vector from A to B (f) Component of A along B
A ⋅ B a1b1 + a2 b2 + a3 b3
(v) Cross Product of Two Vectors = Acosθ = =
Let C = A × B, then C = | C|= AB sin θ B b12 + b22 + b32

$ = A = a1 i + a2 j + a3 k
$ $ $
Direction of C is perpendicular to both A and B given by (g) Unit vector parallel to A = A
right hand screw law. We can also say that C is A a1 + a2 + a3
2 2 2

perpendicular to the plane formed by A and B. (h) Angle between A and B ,


Important points in cross product A ⋅ B
θ = cos −1 

(a) A × B = − B × A 
 AB 
(b) Cross product of two parallel or antiparallel vectors is  a1b1 + a2 b2 + a3 b3 
a null vector having zero magnitude and arbitrary ∴ θ = cos −1  
 a2 + a2 + a2 × b2 + b2 + b2 
direction.  1 2 3 1 2 3 
Additional Examples
Example 1. State, for each of the following physical Sol. Let a , b, c and d be the position vectors of A, B , C and D.
quantities, if it is a scalar or a vector : volume, mass, D( d )
speed, acceleration, density, number of moles, velocity, R
angular frequency, displacement, angular velocity. C( c )

Sol. Volume, mass, speed, density, number of moles and S


angular frequency are scalars. The rest are vectors. Q
Example 2. Explain why pressure and surface
tension are not vectors? P B( b )
A( a )
Sol. Pressure (force per unit area normal to it) or surface
Hence, position vectors of mid-points P , Q , R and S will
tension (force per unit length) are scalars. They have direction be
which is unique so need not to be specified. a + b b+c c+d d+a
, , and
Example 3. Discuss why an infinitesimal 2 2 2 2
We have, PQ = position vector of Q − position vector
displacement is regarded as a vector quantity. of P
b+c a + b c−a
Sol. Because two or more infinitesimal displacements are = − =
added by vector method. 2 2 2
and SR = position vector of R − position vector of S
Example 4. Is a quantity which has a magnitude and c+d d+a
= −
direction always a vector? Give examples. 2 2
c−a
Sol. No. A quantity which possesses both magnitude and =
2
direction may not be a vector. For example, electric current. ∴ PQ = SR
Similarly, we can show that
Example 5. If a and b are the vectors AB and BC QR = PS
determined by the adjacent sides of a regular hexagon. ∴ PQRS is a parallelogram.
What are the vectors determined by the other sides
Example 7. If a × b = b × c ≠ 0 with a ≠ − c, then
taken in order?
show that a + c = k b, where k is a scalar.
Sol. Given, AB = a and BC = b
From the method of vector addition (or subtraction) we Sol. a × b = b × c
can show that, a × b= −c× b
E D ∴ a × b +c× b= 0
(a + c ) × b = 0
∴ a × b ≠ 0, b × c ≠ 0, a , b, c , d are non-zero vectors.
–a
F C (a + c ) ≠ 0
Hence, a + c is parallel to b.
b ∴ a +c= kb (k = scalar)
A B
Example 8. If A = 2i$ − 3$j + 7k$ , B = i$ + 2$j and
CD = b − a C = $j − k$ . Find A ⋅(B × C).
Then, DE = − AB = − a
Sol. A ⋅ (B × C) = [ A BC],
EF = − BC = − b
2 −3 7
and FA = − CD = a − b
Volume of parallelopiped = 1 2 0
Example 6. If the mid-points of the consecutive sides 0 1 −1
of any quadrilateral are connected by straight line
= 2 (− 2 − 0) + 3 (− 1 − 0) + 7 (1 − 0)
segments, prove that the resulting quadrilateral is a = − 4 −3+7
parallelogram. =0
42 Objective Physics Vol. 1

Example 9. Find the resultant of three vectors Example 12. Let A ,B and C be unit vectors.
OA ,OB and OC shown in figure. Radius of circle is R. Suppose that A ⋅B = A ⋅ C = 0 and that the angle
π
C between B and C is , then prove that
6
B
° A = ± 2 (B × C )
45
45° Sol. Since, A ⋅ B = 0, A ⋅ C = 0
O A
Hence, (B + C) ⋅ A = 0
So, A is perpendicular to (B + C) and A is a unit vector
perpendicular to the plane of vectors B and C.
B× C
A=
Sol. OA = OC |B × C|
π 1 1
OA + OC is along OB, (bisector) and its magnitude is | B × C | = | B || C |sin = 1 × 1 × =
2R cos 45° = R 2 6 2 2
B× C
(OA + OC) + OB is along OB and its magnitude is ∴ A= = ± 2 (B × C)
|B × C |
R 2 + R = R (1 + 2 )
Example 13. A particle moves on a given line with
Example 10. Prove that a constant speed v. At a certain time, it is at a point P
| a × b | 2 = a 2 b 2 − (a ⋅ b) 2 on its straight line path. O is fixed point. Show that
(OP × v ) is independent of the position P.
Sol. Let | a | = a, | b | = b y
Sol. x P v
and θ be the angle between them. (x, y)
| a × b | 2 = (ab sin θ) 2
y
= a 2 b 2 sin 2 θ x
= a 2 b 2 (1 − cos 2 θ) O
= a 2 b 2 − (a ⋅ b cos θ) 2 Let v = v i$
= a 2 b 2 − (a ⋅ b) 2 OP = x$i + y$j
Take OP × v = (x$i + y$j ) × v$i
Example 11. Show that the vectors a = 3i$ − 2$j + k$ , = − y vk$ (∴ y is constant)
which is independent of position.
b = i$ − 3$j + 5k$ and c = 2i$ + $j − 4k$ form a right angled
triangle. Example 14. Prove that the mid-point of the
Sol. We have hypotenuse of right angled triangle is equidistant from
its vertices.
Sol. Here, ∠CAB = 90 °, let D be the mid-point of hypotenuse, we
b have
c
C

a D
b + c = (i$ − 3$j + 5 k$ ) + (2i$ + $j − 4 k$ )
= 3$i − 2$j + k$ = a
A B
Hence, a , b and c are coplanar.
Also, we observe that no two of these vectors are BD = DC
parallel, therefore the given vectors form a triangle. AB = AD + DB
Further, AC = AD + DC = AD + BD K(i)
a ⋅ c = (3$i − 2$j + k$ ) ⋅ (2i$ + $j − 4 k$ ) = 0 Since, ∠BAC = 90 °
∴ AB ⊥ AC
Dot product of two non-zero vectors is zero. Hence,
they are perpendicular so they form a right angled (AD + DB) ⋅ (AD + BD ) = 0
triangle. (AD − BD ) ⋅ (AD + BD ) = 0
| a | = 9 + 4 + 1 = 14 , AD 2 − BD 2 = 0
| b | = 1 + 9 + 25 = 35 ∴ AD = BD also, BD = DC
Q D is mid-point of BC.
and | c | = 4 + 1 + 16 = 21 Thus, | AD | = | BD | = | DC | . Hence proved.
NCERT Selected Questions
Q 1. State for each of the following physical quantities, if (e) No, as the two vectors of same dimensions can only be
it is a scalar or a vector : Volume, mass, speed, added, so addition of any two vectors is not a
acceleration, density, number of moles, velocity, meaningful algebraic operation.
angular frequency, displacement, angular velocity. (f) Yes, a component of a vector can be added to the same
vector as both of them are the vectors of same nature.
Sol. Scalar quantities Volume, mass, speed, density, number of
moles, angular frequency. Q 5. Read each statement below carefully and state with
Vector quantities Acceleration, velocity, displacement, reasons, if it is true or false.
angular velocity. (a) The magnitude of a vector is always a scalar.
Q 2. Pick out the two scalar quantities in the following (b) Each component of a vector is always a scalar.
list: Force, angular momentum, work, current, linear (c) The total path length is always equal to the
momentum, electric field, average velocity, magnitude of the displacement vector of a
magnetic moment, reaction as per Newton’s third particle.
law, relative velocity. (d) Three vectors not lying in a plane can never add
Sol. Work, current.
up to give a null vector.
Sol. (a) True,
Q 3. Pick out the only vector quantity in the following
(b) False. As each component of a given vector is always a
list: Temperature, pressure, impulse, time, power, vector.
total path, length, energy, gravitational potential, (c) False. It is true only if the particle moves along a straight
coefficient of friction, charge. line in the same direction.
Sol. Impulse. (d) True.

Q 4. State, with reasons, whether the following algebraic Q 6. Establish the following vector inequalities
operations with scalar and vector physical quantities geometrically or otherwise
are meaningful (a) | a + b | ≤ | a | + | b |
(a) adding any two scalars, (b) | a + b | ≥ | a | ~ | b |
(b) adding a scalar to a vector of the same (c) | a − b | ≤ | a | + | b |
dimensions, (d) | a − b | ≥ | a | ~ | b |
(c) multiplying any vector by any scalar, When does the equality sign above apply?
(d) multiplying any two scalars, Sol. Let the two vectors a and b be represented by the sides OP
(e) adding any two vectors, and PQ of the ∆OPQ taken in the same order, then their
(f) adding a component of a vector to the same resultant is represented by the side OQ of the triangle such
vector. that
Q
Sol. (a) No, adding any two scalars is not meaningful, because
only the scalars of same dimensions can be added.
(b) No, adding a scalar to a vector of the same dimensions is a+b
not meaningful because a scalar cannot be added to a b
vector.
(c) Yes, multiplying any vector by any scalar is meaningful O a P
algebraic operation. It is because when any vector is
multiplied by and scalar, then we get a vector having
magnitude equal to scalar number times the magnitude a –b
of the given vector, e.g. when acceleration a is
multiplied by mass m, we get force F = ma , which is a
R
meaningful operation.
(d) Yes, the product of two scalars gives a meaningful result OQ = a + b
e.g. when power P is multiplied by time t, then we get OP = a , PQ = b
work done (W ) i.e. W = Pt, which is a useful algebraic ∴ |OP | = | a |, | PQ | = | b|
operation. and |OQ | = | a + b|
44 Objective Physics Vol. 1

(a) | a + b| ≤ | a | + | b| or | a − b| > || a | − | b ||
Proof We know from the property of a triangle that its one The equality sign applies if a and b are collinear and act in
side is always less than the sum of the lengths of its two same direction.
other sides. Q 7. Given a + b + c + d = 0, which of the following
∴ OQ < OP + PQ or | a + b| < | a | + | b| statements are correct
The equality sign applies if a and b are collinear and act in (a) a, b, c and d each must be a null vector.
same direction.
(b) The magnitude of ( a + c) equals the magnitude
(b) | a + b| ≥ | a | − | b| of ( b + d).
Proof One side is more than the difference of two other (c) The magnitude of a can never be greater than the
sides
sum of the magnitudes of b, c and d.
∴ OQ > OP ~ PQ or | a + b | > | a | ~ | b |
The equality sign applies if a and b are collinear and act in Sol. (a) Statement is wrong because a + b + c + d can be zero in
opposite directions. many ways other then a , b, c and d must each be a null
vector.
(c) | a − b| ≤ | a | + | b|
(b) Statement is correct, as a + b + c + d = 0
Proof Again in ∆OPR
∴ a + c = − ( b + d)
∴ OR < OP + PR
or |a + c| = | b + d |
or | a − b| < | a | + |− b|
(c) Statement is correct, as a + b + c + d = 0
< | a | + | b|
∴ a = − ( b + c + d)
The equality sign applies if a and b are collinear and act in
opposite directions. or |a | = | b + c + d | …(i)
(d) | a − b| ≥ | OP − PR | From Eq. (i), we see that the magnitude of a is equal to
the magnitude of vector ( b + c + d). Since, the sum of
As one side of a triangle is more than the difference of its the magnitudes of b, c, d may be equal or greater than
two other sides, thus is ∆OPR the magnitude of a, hence the magnitude of a can
OR > |OP − OR | never be greater than the sum of the magnitudes of
∴ | a − b| > | | a | − | b || b, c and d.
Objective Problems
[ Level 1 ]
Scalar and Vector Quantities 12. If A + B = A − B, then magnitude of B is
1. Which is not a vector quantity? (a) | A | (b) 0
(a) Current (b) Displacement (c) 1 (d) None of these
(c) Velocity (d) Acceleration 13. If | A | = 2 and | B | = 4 and angle between them is 60°, then
2. Which one is a vector quantity? | A − B | is
(a) Temperature (b) Momentum (a) 13 (b) 3 3
(c) Work (d) Speed (c) 3 (d) 2 3
3. Which is a vector quantity?
14. Two vectors having magnitudes 8 and 10 can have
(a) Work (b) Power
maximum and minimum value of magnitude of their
(c) Torque (d) Gravitational constant
resultant as
4. Out of the following quantities, which is scalar? (a) 12, 6 (b) 10, 3
(a) Displacement (b) Momentum (c) 18, 2 (d) None of these
(c) Potential energy (d) Torque
15. The resultant of A and B makes an angle α with A and β
5. Which is a vector quantity?
with B, then
(a) Angular momentum (b) Work
(a) α < β (b) α > β if A < B
(c) Potential energy (d) Electric current
(c) α < β if A = B (d) α < β if A < B
6. Which of the following is a vector?
16. At what angle should the two forces 2P and 2P act so
(a) Pressure (b) Surface tension
(c) Moment of inertia (d) None of these
that the resultant force is P 10 ?
(a) 45° (b) 60°
7. Pressure is (c) 90° (d) 120°
(a) scalar (b) vector
17. Two vectors are such that | A + B | = | A − B |. The angle
(c) Both (a) and (b) (d) None of these
between the vector is
8. Surface area is (a) 0° (b) 30°
(a) scalar (b) vector (c) 60° (d) 90°
(c) Neither scalar nor vector (d) Both (a) and (b)
18. If two vectors are equal and their resultant is also equal to
9. Which of the following is not the vector quantity? one of them, then the angle between the two vectors is
(a) Torque (b) Displacement (a) 60° (b) 120°
(c) Dipole moment (d) Electric flux (c) 90° (d) 0°

19. If A = B + C and the magnitudes of A , B and C are 5, 4, 3


Position Vector and Displacement units, the angle between A and C is
Vector, Addition and Subtraction (a) cos−1 (3 / 5) (b) cos−1 (4 / 5)
of two Vectors, Equilibrium of (c)
π
(d) sin −1 (3 / 4 )
Vectors 2

10. If the resultant of two unequal vectors is equal to sum of 20. If A and B are two non-zero vectors having equal
magnitude, the angle between the vectors A and A − B is
their magnitudes, the angle between the vectors is
(a) 0°
(a) 90° (b) 180°
(b) 90°
(c) 0° (d) None of these
(c) 180°
11. Resultant of two vectors A and B is given by (d) dependent on the orientation of A and B
| R | = {| A | − | B |}, angle between A and B will be 21. For the resultant of two vectors to be maximum, what
(a) 90° must be the angle between them?
(b) 180° (a) 0° (b) 60° (c) 90° (d) 180°
(c) 0°
(d) None of the above
46 Objective Physics Vol. 1

22. Minimum number of forces of unequal magnitudes 30. Two vectors A and B inclined at angle θ have a resultant R
whose vector sum can equal to zero is which makes an angle φ with A. If the directions of A and
(a) two (b) three B are interchanged and resultant will have the same
(c) four (d) None of these (a) magnitude
(b) direction
23. For the figure, (c) magnitude as well as direction
(d) neither
31. What is correct?
(a) | a − b| = | a | − | b| (b) | a − b| ≤ | a | − | b |
C B (c) | a − b | ≥ | a | − | b | (d) | a − b | > | a | − | b|
32. A = 2$i + $j, B = 3$j − k$ and C = 6 i$ − 2 k$ .
Value of A − 2 B + 3C would be
A
(a) 20$i + 5$j + 4 k$ (b) 20$i − 5$j − 4 k$
(a) A + B = C (b) B + C = A
(c) C + A = B (d) C + A + B = 0 (c) 4 $i + 5$j + 20k$ (d) 5$i + 4 $j + 10k$

24. A force of 6 N and another of 8 N can be applied together 33. If A = 4i$ − 3$j and B = 6i$ + 8$j, then magnitude and
to produce the effect of a single force of direction of A + B with x-axis will be
(a) 1N (b) 11 N (a) 5, tan −1 (3 / 4 ) (b) 5 5 , tan −1 (1 / 2)
(c) 15 N (d) 20 N (c) 10, tan −1 (5) (d) 25, tan −1 (3 / 4 )
25. Position of a particle in a rectangular coordinate system is
34. The position vector of a moving particle at time t is
( 3, 2, 5). Then, its position vector will be
r = 3$i + 4t $j − t k$ . Its displacement during the time
(a) 3$i + 5$j + 2k$ (b) 3$i + 2$j + 5k$
interval t = 1s to t = 3 s is
(c) 5$i + 3$j + 2k$ (d) None of these (a) $j − k$ (b) 3$i + 4 $j − k$
26. If a particle moves from point P ( 2, 3, 5) to point (c) 9$i + 36 $j − 27 k$ (d) None of these
Q ( 3, 4, 5), then its displacement vector will be 35. Resultant of which of a following may be equal to zero?
(a) $i + $j + 10k$ (b) $i + $j + 5k$ (a) 10 N, 10 N, 10 N (b) 10 N, 10 N, 25 N
(c) 10 N, 10 N, 35 N (d) None of these
(c) $i + $j (d) 2$i + 4 $j + 6k$
36. A man first moves 3 m due east, then 6 m due north and
27. Five equal forces of 10 N each are applied at one point finally 7 m due west, then the magnitude of the resultant
and all are lying in one plane. If the angles between them displacement is (in metre)
are equal, the resultant force will be (a) 16 (b) 24 (c) 52 (d) 94
(a) zero (b) 10 N
(c) 20 N (d) 10 2 N 37. The resultant of two forces 3P and 2P is R. If the first
force is doubled, then the resultant is also doubled. The
28. Two vectors having equal magnitudes A make an angle θ angle between the two forces is
with each other. The magnitude and direction of the (a) 60° (b) 120° (c) 90° (d) 180°
resultant are respectively
θ
38. Two forces 8 N and 12 N act at 120°. The third force
(a) 2 A cos , along bisector required to keep the body in equilibrium is
2
θ (a) 4 N (b) 4 7 N
(b) A cos , at 45° from one vector
2 (c) 20 N (d) None of these
θ
(c) 2 A sin , along bisector 39. If a and bare two units vectors inclined at an angle of 60°
2
θ to each other, then
(d) A cos , along bisector (a) | a + b| > 1 (b) | a + b | < 1
2
(c) | a − b| > 1 (d) | a − b| < 1
29. Given that P + Q + R = 0. Which of the following
statement is true? 40. If 12 different coplanar forces (all of equal magnitudes)
(a) | P | + |Q | = | R | maintain a body in equilibrium, then the angle between
(b) |P + Q | = |R | any two adjacent forces is
(c) | P | − |Q | = | R | (a) 15° (b) 30°
(d) |P − Q | = |R | (c) 45° (d) 60°
Vectors 47

41. If P + Q = R and | P | = |Q | = 3 and | R | = 3, then the 50. The work done by a force F = ( i$ + 2$j + 3k$ ) N, to displace
angle between P and Q is a body from position A to position B is [The position
(a) π /4 (b) π /6 vector of A is r1 = ( i$ + 3$j + k$ ) m and the position vector
(c) π / 3 (d) π /2 of B is r = ( 2$i + 2$j + 3k$ ) m ]
2
42. Given that A + B + C = 0, out of three vectors two are (a) 5 J (b) 3 J (c) 2 J (d) 10 J
equal in magnitude and the magnitude of third vector is 51. The condition ( a ⋅ b) = a b is satisfied when
2 2 2
2 times that of either of two having equal magnitudes.
(a) a is parallel to b (b) a ≠ b
Then, angle between vectors are given by (c) a ⋅ b = 1 (d) a ⊥ b
(a) 30° , 60° , 90° (b) 45° , 45° , 90°
(c) 90° , 135° , 45° (d) 90° , 135° , 135° 52. If | A | = 2, | B | = 5 and | A × B | = 8. Angle between
43. Out of the following set of forces, the resultant of which A and B is acute, then A ⋅ B is
cannot be zero (a) 6 (b) 3 (c) 4 (d) 7
(a) 10, 10, 10 (b) 10, 10, 20 1
(c) 10, 20, 20 (d) 10, 20, 40 53. The modulus of the vector product of two vectors is
3
44. Two vectors A and B are such that A + B = C and times their scalar product. The angle between vectors is
π π
A 2 + B 2 = C2 . If θ is the angle between A and B, then the (a) (b)
6 2
correct statement is π π
2π (c) (d)
(a) θ = π (b) θ = 4 3
3
π 54. The area of the parallelogram determined by
(c) θ = 0 (d) θ =
2 A = 2$i + $j − 3k$ and B = 12$j − 2k$ is approximately
45. If A and B are two vectors such that | A + B | = 2 | A − B | , (a) 43 (b) 56
(c) 38 (d) 74
the angle between vectors A and B is
(a) 45° (b) 60° 55. Three vectors A , B and C satisfy the relation A ⋅ B = 0 and
(c) 30° (d) data insufficient
A ⋅ C = 0. Then, the vector A is parallel to
(a) B (b) C
Dot Product and Cross Product of (c) B ⋅ C (d) B × C
Vectors 56. What is the dot product of two vectors of magnitude
46. Given A = 3$i + 4$j and B = 6$i + 8$j, which of the following 3 and 5, if angle between them is 60°?
statement is correct? (a) 5.2 (b) 7.5
| A| 1 (c) 8.4 (d) 8.6
(a) A × B = 0 (b) =
|B | 2
(c) | A | = 15 (d) A ⋅ | B | = 48
Unit Vector, Angle between two
Vectors, Component of Vector,
47. In a clockwise system
(a) $j × k$ = i$ (b) k$ ⋅ i$ = 1 Conditions of two Vectors
(c) $i ⋅ $i = 0 (d) $j × $j = 1 57. Unit vector perpendicular to vector A = − 3$i − 2$j − 3k$
and B = 2$i + 4$j + 6k$ both is
48. A vector A points vertically upwards and B points
towards north. The vector product A × B is 3$j − 2k$ 3k$ − 2$j
(a) (b)
(a) zero 13 13
(b) along west − $j + 2k$ $i + 3$j − k$
(c) along east (c) (d)
13 13
(d) vertically downward
58. Vector P = 6 i$ + 4 2$j + 4 2 k$ makes angle from z-axis
49. | A × B | = 3 A ⋅ B, then the value of | A + B | is
1/ 2 equal to
 AB   2
(a)  A 2 + B 2 + 
 3 (a) cos−1   (b) cos−1 (2 2 )
 5
(b) A + B
(c) ( A 2 + B 2 + 3 AB )1/ 2  2 2
(c) cos−1   (d) None of these
(d) ( A + B + AB )
2 2 1/ 2  5 
48 Objective Physics Vol. 1

59. Given A = i$ + $j + k$ and B = − i$ − $j − k$ , then ( A − B ) 70. The angle between the two vectors − 2$i + 3$j + k$ and
will make angle with A $i + 2$j − 4k$ is
(a) 0° (b) 180°
(c) 90° (d) 60° (a) 45° (b) 90°
(c) 30° (d) 60°
60. Component of the vector A = 2$i + 3$j along the vector
71. The angle between the vector 2$i + 4k$ and the y-axis is
B = ( i$ + $j ) is
(a) 0° (b) 90°
5
(a) (b) 4 2 (c) 45° (d) 180°
2
2 72. A force of 5 N acts on a particle along a direction making
(c) (d) None of these an angle of 60° with vertical. Its vertical component
3
will be
61. Which of the following is a unit vector? (a) 10 N (b) 3 N
(a) $i + $j (b) cosθ i$ − sin θ $j (c) 4 N (d) 2.5 N
1 $ $
(c) sin θ i$ + 2 cosθ $j (d) (i + j) 73. If A = 3$i + 4$j and B = 7i$ + 24$j, the vector having the
3
same magnitude as B and parallel to A is
62. The component of a vector along any other direction is (a) 5$i + 20$j (b) 15$i + 10$j
(a) always less than its magnitude
(b) always greater than its magnitude (c) 20i$ + 15$j (d) 15i$ + 20$j
(c) always equal to its magnitude
(d) None of the above 74. The vector that must be added to the vector $i − 3$j + 2k$
and 3$i + 6$j − 7k$ so that the resultant vector is a unit
63. A vector P = 3i$ − 2$j + a k$ is perpendicular to the vector
vector along the y-axis is
Q = 2$i + $j − k$ . The value of a is
(a) 4 $i + 2$j + 5k$ (b) − 4 $i − 2$j + 5k$
(a) 2 (b) 1
(c) 4 (d) 3 (c) 3$i + 4 $j + 5k$ (d) null vector

64. If three vectors along coordinate axis represent the  1 $ 1 $


75. The expression  i+ j is a
adjacent sides of a cube of length b, then the unit vector  2 2 
along its diagonal passing through the origin will be (a) unit vector (b) null vector
$i + $j + k$ $i + $j + k$
(a) (b) (c) vector of magnitude 2 (d) scalar
2 36
$i + $j + 76. A vector is represented by 3$i + $j + 2k$ . Its length in
k$
(c) i$ + $j + k$ (d) X -Y plane is
3
(a) 2 (b) 14 (c) 10 (d) 5
65. Consider a vector F = 4$i − 3$j . Another vector
perpendicular of F is 77. What is the angle between $i + $j + k$ and $j ?
(a) 4 $i + 3$j (b) 6$i π π
(a) (b)
(c) 7k$ (d) 3$i − 4 $j 6 4
π
(c) (d) None of these
66. The angle between vectors ( A × B ) and ( B × A ) is 3
(a) zero (b) π 78. Let A = i$ A cos θ + $j A sin θ be any vector. Another vector
(c) π /4 (d) π / 2
B which is perpendicular to A can be expressed as
67. A vector perpendicular to both the vectors 2$i − $j + 5k$ and (a) $i B cosθ − $j B sin θ
x-axis is (b) $i B sin θ − $j B cosθ
(a) $j + 5$j (b) $j − 5 k$ (c) 5$j + k$ (d) $i + $j + k$
(c) $i B cosθ + $j B sin θ
68. The angle between A + B and A − B will be (d) $i B sin θ + $j B cosθ
(a) 90° (b) between 0° and 180°
79. Which of the following is the unit vector perpendicular to
(c) 180° only (d) None of these
A and B?
69. Unit vector parallel to the resultant of vector 8 $i + 8$j A$ ×B $ $×B
A $
will be (a) (b)
AB sin θ AB cosθ
(a) (24 $i + 5$j)/13 (b) (12 $i + 5$j)/13 A×B A×B
(c) (d)
(c) (6 $i + 5$j)/13 (d) None of these AB sinθ AB cosθ
Vectors 49

| a + b| 89. The angles which the vector A = 3$i + 6$j + 2 k$ makes with
80. If = 1, then angle between a and bis
| a − b| the coordinate axes are
3 6 2
(a) 0° (b) 45° (a) cos−1 , cos−1 and cos−1
(c) 90° (d) 60° 7 7 7
−1 4 −1 5 −1 3
81. A man walks 20 m at an angle 60° north of east. How far (b) cos , cos and cos
7 7 7
towards east has he travalled? −1 3 −1 4 −1 1
(c) cos , cos and cos
(a) 10 m (b) 20 m 7 7 7
(c) 20 3 m (d) 10 3 m (d) None of the above

82. If two vectors 2$i + 3$j + k$ and − 4$i − 6$j − λk$ are parallel Miscellaneous Problems
to each other, then value of λ is 90. If A = B, then which of the following is not correct?
(a) 0 (b) 2 $ =B
(a) A $ (b) | A | = | B |
(c) 3 (d) 4 $ $ $ +B
$
(c) AB = BA (d) A + B = A
83. What is the angle between ( P + Q ) and ( P × Q ) ?
91. The area of the parallelogram represented by the vectors
π
(a) Zero (b) A = 2$i + 3$j and B = $i + 4$j is
2
π (a) 14 units (b) 7.5 units (c) 10 units (d) 5 units
(c) (d) π
4 92. The forces, which meet at one point but their lines of
action do not lie on one plane, are called
84. If a unit vector is represent by 0.5i$ + 0.8$j + ck$ , then the (a) non-coplanar non-concurrent forces
value of c is (b) non-coplanar concurrent forces
(a) 1 (b) 011
. (c) coplanar concurrent forces
(c) 0.01 (d) 0.39 (d) coplanar non-concurrent forces
93. If the angle between two vectors A and B is 120°, its
85. If A = a1 i$ + b1 $j and B = a 2 i$ + b2 $j, the condition that resultant C will be
they are perpendicular to each other is (a) C = | A − B | (b) C < | A − B |
a1 b (c) C > | A − B | (d) C = | A + B |
(a) =− 2 (b) a1b1 = a2b2
b1 a2 94. The condition under which the vectors ( a + b) and ( a − b)
a b are parallel is?
(c) 1 = − 1 (d) None of these
a2 b2 (a) a ⊥ b (b) | a | = | b|
(c) a ≠ b (d) a is parallel to b
86. For what value of x, will the two vectors A = 2$i + 2$j − x k$
95. A cyclist is moving on a circular path with constant speed
and B = 2i$ − $j − 3k$ are perpendicular to each other? v. What is the change in its velocity after it has described
(a) x = − 2/ 3 an angle of 30°?
(b) x = 3/ 2 v
(a) v 2 (b)
(c) x = − 4 /3 2
(d) x = 2/ 3 (c) v 3 (d) None of these
87. A and B are two vectors given by A = 2$i + 3$j and 96. If A and B are two non-zero vectors having equal
B = i$ + $j. The magnitude of the component of A along magnitudes, the angle between the vectors A and the
B is resultant of A and B is
5 3 (a) 0°
(a) (b)
2 2 (b) 90°
7 5 (c) 180°
(c) (d)
2 13 (d) dependent on the orientation of A and B

88. The values of x and y for which vectors 97. Resultant of two vectors of equal magnitude A is
(a) 3A at 60° (b) 2A at 90°
A = ( 6i$ + x$j − 2k$ ) and B = ( 5i$ + 6$j − yk$ ) may be
(c) 2A at 120° (d) A at 180°
parallel are
2 36 5 98. If a$i + b$j is a unit vector and it is perpendicular to $i + $j,
(a) x = 0, y = (b) x = , y=
3 5 3 then value of a and b is
15 23 36 15 (a) 1, 0 (b) − 2, 0
(c) x = − , y= (d) x = − , y=
3 5 5 4 (c) 0.5, − 0.5 (d) None of these
50 Objective Physics Vol. 1

99. If a + b + c = 0 then a × b is forces F1 = F2 = F3 = F act on the same body as shown


(a) b × c (b) c × b
in figure, then the acceleration produced is
(c) a × c (d) None of these F2

100.Consider a force vector F = $i + $j + k$ . Another vector


perpendicular of F is
(a) 4 $i + 3$j (b) 6 $i 90°
135°
(c) 2$i − $j − k$ (d) 3$i − 4 $j m F1

101.A vector perpendicular to both the vector 2$i − 3$j and


F3
3$i − 2$j is
(a) ( 2 − 1) a (b) ( 2 + 1) a
(a) $j + 5k$ (b) $j − 5k$
(c) 2a (d) a
(c) 6k$ (d) i$ + $j + k$
105.The direction cosines of vector ( A − B ), if
102.A force F = ( 6i$ − 8$j + 10k$ ) N produces an acceleration of
A = 2$i + 3$j + k$ , B = 2$i + 2$j + 3k$ are
1 m/ s 2 in a body. The mass of body would be 1 −2
(a) 200 kg (b) 20 kg (a) 0, ,
5 5
(c) 10 2 kg (d) 6 2 kg 2 1
(b) 0, ,
5 5
103.If A = 4$i − 4$j − 3k$ and B = 6i$ + 8$j , then magnitude and 1
(c) 0, 0,
direction of A + B with x-axis will be 5
(a) 5, tan −1 (3/ 4 ) (d) None of the above
(b) 5 5 , tan −1 (1/ 2)
106.The vector sum of two forces is perpendicular to their
(c) 10, tan −1 (5) vector differences. In that case, the forces
(d) 25, tan −1 (3/ 4 ) (a) are not equal to each other in magnitude
(b) are parallel
104.When a force F acts on a body of mass m, then the (c) are perpendicular
acceleration produced in the body is a. If three equal (d) are equal to each other in magnitude

[ Level 2 ]
Only One Correct Option
1. If vectors A and B have an angle θ between them, then 3. The resultant of A and B is R1 . On reversing the vector B,
$ −B
value of | A $ | will be the resultant becomes R 2 . What is the value of R12 + R 22 ?
θ θ (a) A 2 + B 2 (b) A 2 − B 2
(a) 2 cos (b) 2 tan
2 2 (c) 2( A 2 + B 2 ) (d) 2 ( A 2 − B 2 )
θ
(c) 2 sin (d) None of these
2 4. If the sum of two unit vectors is a unit vector, then
2. A particle undergoes three successive displacements magnitude of difference in two unit vectors is
given by s 1 = 2 m north-east, s 2 = 2 m due south and (a) 2 (b) 3
(c) 1/ 2 (d) 5
s 3 = 4 m, 30° north of west, then magnitude of net
displacement is 5. The sum of two vectors A and B is at right angles to their
(a) 14 + 4 3 difference. Then,
(a) A=B
(b) 14 − 4 3
(b) A = 2B
(c) 4 (c) B = 2A
(d) None of the above (d) None of the above
Vectors 51

6. A vector having magnitude 30 unit makes equal angles 13. The resultant of two vectors P and Q is R. If Q is doubled,
with each of X , Y and Z axes. The components of vector the new resultant is perpendicular to P. Then, R equals to
along each of X , Y and Z axes are (a) P (b) (P + Q )
(a) 10 3 unit (b) 10 3 unit (c) Q (d) (P − Q )
(c) 15 3 unit (d) 10 unit
14. The angle between the vector A and B is θ. The value of
7. Figure shown ABCDEF as a regular hexagon. What is the triple product A ⋅ ( B × A ) is
the value of (a) A 2 B (b) zero
AB + AC + AD + AE + AF (c) A 2 B sin θ (d) A 2 B cos θ
E D
15. The sum of the magnitudes of two forces acting at a point
is 18 and the magnitude of their resultant is 12. If the
resultant is at 90° with the force of smaller magnitude,
F C what are the magnitudes of forces?
O (a) 12, 6 (b) 14 , 4
(c) 5, 13 (d) 10, 8

16. A vector a is turned without a change in its length


A B through a small angles dθ. The value of | ∆a| and ∆ a are
(a) AO (b) 2AO (c) 4AO (d) 6AO respectively
(a) 0, a dθ (b) a ⋅ d θ, 0
8. If A is a unit vector in a given direction, then the value of
(c) 0, 0 (d) None of these
$
A$ ⋅ dA is
dt 17. Find the resultant of three vectors OA , OB, and OC
1 shown in the figure. Radius of the circle is R.
(a) 0 (b) 1 (c) (d) 2
2 C
9. If F1 and F2 are two vectors of equal magnitudes F such B

that | F1 ⋅ F2 | = | F1 × F2 |, then | F1 + F2 | equals to


45°
(a) (2 + 2) F (b) 2F 45°
O A
(c) F 2 (d) None of these

10. Figure shows three vectors p, q and r, where C is the mid


point of AB. Then, which of the following relation is
correct?
A (a) 2R (b) R(1 + 2)
(c) R 2 (d) R( 2 − 1)

p C 18. In the diagram shown in figure


r
D C

O q Z
B Y

(a) p + q = 2r (b) p + q = r X B
W
(c) p − q = 2r (d) p − q = r

11. If a vector pis making angles α , β and γ respectively with A

the X , Y and Z axes, then sin 2 α + sin 2 β + sin 2 γ is (a) X = A + B − C + D (b) Y = B + C − A


equal to (c) Z = B + C (d) W = A + B
(a) 0 (b) 1 (c) 2 (d) 3
19. If a and b are two vectors, then the value of
12. What is the angle between P and the cross product of ( a + b) × ( a − b) is
( P + Q ) and ( P − Q )? (a) 2 (b × a)
(a) 90° (b) tan −1 (P / Q ) (b) − 2 (b × a)
(c) b× a
(c) tan −1 (Q / P ) (d) 0° (d) b× a
52 Objective Physics Vol. 1

20. Given that A + B = C and that C is perpendicular to A 1. Assertion A × B is perpendicular to both A + B as well
further if | A | = |C |, then what is the angle between as A − B.
A and B ? Reason A + B as well as A − B lie in the plane
π π 3π containing A and B while A × B lies perpendicular to the
(a) (b) (c) (d) π
4 2 4 plane containing A and B.
21. The resultant of two forces, one double and the other in 2. Assertion Angle between $i + $j and $i is 45°.
magnitude, is perpendicular to the smaller of the two
Reason i$ + $j is equally inclined to both $i and $j and the
forces. The angle between the two forces is
(a) 120° (b) 135° angle between $i and $j is 90°.
(c) 90° (d) 150°
3. Assertion Finite angular displacement is not a vector
22. If A + B = C, | A | = 2 | B | and B ⋅ C = 0, then quantity.
(a) | A + C | = | A + B | (b) | A + C | = B Reason If does not obey the vector laws.
(c) A ⋅ B < 0 (d) A ⋅ C may be zero
4. Assertion Vector product of two vectors may be greater
23. Two unit vectors when added give a unit vector. Then, than, equal to or less than the scalar product.
choose the correct statement. Reason At θ = 45°, two are equal.
(a) Magnitude of their difference is 3
(b) Magnitude of their difference is 1 5. Assertion Vector addition of two vectors is always
(c) Angle between the vectors is 90° greater than their vector subtraction.
(d) Angle between the sum and the difference of the two vectors Reason At θ = 90°, two are equal.
is 90°
6. Assertion Component of A along B is equal to
24. The velocity of a particle is v = 6i$ + 2$j − 2k$ . The
component of B along A.
component of the velocity parallel to vector a = $i + $j − k$
Reason Compound of A along B is A cos θ. Where, θ is
in vector form is
the angle between two vectors.
(a) 6$i + 2$j + 2k$ (b) 2$i + 2$j + 2k$
7. Assertion Small displacement is a vector quantity.
(c) $i + $j + k$ (d) 6$i + 2$j − 2k$
Reason Pressure and surface tension are also vector
25. The resultant R of vectors P and Q is perpendicular to | P | quantities.
and |Q | both, then angle between | P | and |Q | is 8. Assertion Component of A along B is equal to
(a) 45° (b) 135° component of B along A.
(c) 120° (d) All of these
Reason Value of component is always less than the
26. The value of $i × ( i$ × a ) + $j × ( $j × a ) + k$ × ( k$ × a ) is magnitude of vector.
(a) a (b) a × k$ 9. Assertion ( A + B ) ⋅ ( A − B ) is always positive.
(c) − 2 a (d) − a
Reason This is positive if | A | > | B |.
27. What is the angle between P and the resultant of ( P + Q )
10. Assertion We can find angle between two vectors by
and ( P − Q )? using the relation,
(a) Zero (b) tan −1 (P / Q )  | A × B |
−1
(d) tan −1 (P − Q )/(P + Q ) θ = sin −1  
(c) tan (Q / P )  AB 
|A × B |
Assertion and Reason Reason is always positive.
AB
Directions (Q. Nos. 1-13) These questions consists of two
11. Assertion ( A × B ) ⋅ ( B × A ) is − A 2 B 2 sin 2 θ. Here θ is
statements each printed as Assertion and Reason. While
the angle between A and B.
answering these questions you are required to choose any one
Reason ( A × B ) and ( B × A ) are two anti-parallel
of the following five responses.
vectors provided A and B are neither parallel nor
(a) If both Assertion and Reason are correct and Reason is
anti-parallel.
the correct explanation of Assertion
(b) If both Assertion and Reason are true but Reason is 12. Assertion If angle between a and b is 30°. Then, angle
not the correct explanation of Assertion b
(c) If Assertion is true but Reason is false between 2 a and − will be 150°.
2
(d) If Assertion is false but Reason is true
(e) If both Assertion and Reason are false Reason Sign of dot product of two vectors tells you
whether angle between two vectors is acute or obtuse.
Vectors 53

13. Assertion If | A | = | B | , then ( A + B ), ( A − B ) and Entrance Gallery


( A × B ) are three mutually perpendicular vectors.
2014
Reason Dot product of a null vector with any other
vector is always zero. 1. Two airplanes A and B are flying with constant velocity
in the same vertical plane at an angles 30° and 60° w.r.t.
Match the Columns the horizontal, respectively as shown in the figure given
below. The speed of A is 100 3 ms −1 . At time t = 0 s, an
1. For component of a vector A = ( 3i$ + 4$j − 5k$ ), match the
following columns. observer in A finds B at a distance of 500 m. This
observer sees B moving with a constant velocity
Column I Column II perpendicular to the line of motion of A. If at t = t 0 , A
(A) x-axis (p) 5 unit just escapes being hit by B , t 0 (in second) is
(B) Along another vector (q) 4 unit [JEE Advanced]
(2 i$ + j$ + 2 k$ )
(C) Along ( 6$i + 8$j − 10 k$ ) (r) Zero A

(D) Along another vector (s) None


B
( − 3i$ − 4j$ + 5k$ )

2. If θ is the angle between two vectors A and B, then match


the following two columns. 30° 60°

Column I Column II (a) 3 (b) 5


(c) 7 (d) 9
(A) A ⋅ B = | A × B| (p) θ = 90°
(B) A ⋅B = B 2
(q) θ = 0° or 180° 2. The coordinates of a particle moving in XY -plane at any
instant of time t are x = 4t 2, y = 3t 2 . The speed of the
(C) | A + B| = | A − B | (r) A =B
particle at that instant is [Kerala CEE]
(D) | A × B| = AB (s) None
(a) 10 t (b) 5 t
(c) 3 t (d) 2 t
3. Two vectors A and B have equal magnitude x . Angle (e) 13 t
between them is 60°. Then, match the following two
columns. 3. A particle has the position vector r = $i − 2$j + k$ and the
Column I . Column II linear momentum p = 2 i$ − $j + k$ . Its angular momentum
(A) | A + B| (p) 3 about the origin is [Kerala CEE]
x
2
(a) − i$ + $j − 3k$ (b) − i$ + $j + 3k$
(B) | A − B| (q) x
(c) $i − $j + 3k$ (d) $i − $j − 5 k$
(C) A ⋅ B (r) 3x
(D) | A × B| (s) None (e) $i − $j + 5k$

4. Vector A is pointing eastwards and vector B northwards. 4. Which of the following is not a vector quantity?
Then, match the following two columns. [Karnataka CET]
(a) Weight (b) Nuclear spin
Column I Column II
(c) Momentum (d) Potential energy
(A) (A + B ) (p) North-east
(B) (A − B ) (q) Vertically upwards 5. A force F = 5i$ + 2$j − 5k$ acts on a particle whose position
(C) (A × B ) (r) Vertically downwards vector is r = $i − 2$j + k$ . What is the torque about the
(D) (A × B ) × (A × B ) (s) None
origin? [Karnataka CET]
5. A vector has a magnitude x. If it is rotated by an angle θ, (a) 8 i$ + 10$j + 12 k$ (b) 8 i$ + 10$j − 12 k$
then magnitude of change in vector is nx. Match the (c) 8 $i − 10$j − 8 k$ (d) 10 $i − 10$j − k$
following two columns.
6. Consider three vectors A = i$ + $j − 2 k$ , B = i$ − $j + k$ and
Column I Column II
(A) θ = 60° (p) n = 3
C = 2 i$ − 3$j + 4k$ . A vector X of the form αA + βB (α and
(B) θ = 90° (q) n = 1 β are numbers) is perpendicular to C.
(C) θ = 120° (r) n = 2 The ratio of α and β is [WB JEE]
(D) θ = 180° (s) n = 2 (a) 1 : 1 (b) 2 : 1 (c) −1 : 1 (d) 3 : 1
54 Objective Physics Vol. 1

7. What is the torque of a force 3$i + 7$j + 4k$ about the 9. If A = B + C and have A , B , C scalar magnitudes of
origin, if the force acts on a particle whose position 5, 4, 3 units respectively, then the angle between A
vector is 2 $i + 2$j + 1k$ ? [J&K CET] and C is [WB JEE]
(a) cos−1 (3/ 5) (b) cos−1 (4 / 5)
(a) i$ − 5$j + 8k$
(c) π /2 (d) sin −1 (3/ 4 )
(b) 2 $i + 2$j + 2 k$
(c) $i + $j + k$ 2011
(d) 3$i + 2$j + 3 k$
10. Given, A = 2 $i + 3 $j and B = i$ + $j . The component of
2012 vector A along vector B is [WB JEE]
1 3
(a) (b)
8. If a ⋅ b = | a × b |, then θ will be [O JEE] 2 2
(a) 45° (b) 60° 5 7
(c) (d)
(c) 75° (d) 30° 2 2

Answers
Level 1
Objective Problems
1. (a) 2. (b) 3. (c) 4. (c) 5. (a) 6. (d) 7. (a) 8. (d) 9. (d) 10. (c)
11. (b) 12. (b) 13. (d) 14. (c) 15. (b) 16. (a) 17. (d) 18. (b) 19. (a) 20. (d)
21. (a) 22. (b) 23. (c) 24. (b) 25. (b) 26. (c) 27. (a) 28. (a) 29. (b) 30. (a)
31. (c) 32. (b) 33. (b) 34. (d) 35. (a) 36. (c) 37. (b) 38. (b) 39. (a) 40. (b)
41. (c) 42. (d) 43. (d) 44. (d) 45. (d) 46. (a,b) 47. (a) 48. (b) 49. (d) 50. (a)
51. (a) 52. (a) 53. (a) 54. (a) 55. (d) 56. (b) 57. (a) 58. (c) 59. (a) 60. (a)
61. (b) 62. (a) 63. (c) 64. (d) 65. (c) 66. (b) 67. (c) 68. (b) 69. (b) 70. (b)
71. (b) 72. (d) 73. (d) 74. (b) 75. (a) 76. (c) 77. (d) 78. (b) 79. (c) 80. (c)
81. (a) 82. (b) 83. (b) 84. (b) 85. (a) 86. (a) 87. (a) 88. (b) 89. (a) 90. (d)
91. (d) 92. (b) 93. (c) 94. (d) 95. (d) 96. (d) 97. (a,b) 98. (d) 99. (a) 100. (c)
101. (c) 102. (c) 103. (b) 104. (a) 105. (a) 106. (d)

Level 2
Only One Correct Option
1. (c) 2. (b) 3. (c) 4. (b) 5. (a) 6. (a) 7. (d) 8. (a) 9. (a) 10. (a)
11. (c) 12. (a) 13. (c) 14. (b) 15. (c) 16. (b) 17. (b) 18. (b,c) 19. (a) 20. (c)
21. (a) 22. (c) 23. (a,d) 24. (b) 25. (b) 26. (c) 27. (a)

Assertion and Reason


1. (a) 2. (b) 3. (a) 4. (b) 5. (d) 6. (d) 7. (c) 8. (e) 9. (d) 10. (d)
11. (a,b) 12. (b) 13. (b)

Match the Columns


1. (A → q, B → r, C → s, D → s) 2. (A → s, B → q,r, C → p, D → p) 3. (A → r, B → q , C → s, D → p)
4. (A → p,s, B → s, C → q, D → s) 5. (A → q, B → r, C → p, D → s)

Entrance Gallery
1. (b) 2. (a) 3. (b) 4. (d) 5. (a) 6. (a) 7. (a) 8. (a) 9. (a) 10. (c)
Solutions
Level 1 : Objective Problems 27. When drawn as per polygon law of vector addition, they will
form a closed regular polygon. Hence, resultant will be zero.
8. In φ = B ⋅ S, area is a vector. So area is sometimes as scalar
θ
and sometimes a vector. 28. Resultant will be 2Acos along bisector.
2
13. | A − B | = A 2 + B 2 − 2AB cosθ A AR
1
= 4 + 16 − 2 × 2 × 4 ×
2
= 12 = 2 3
θ/2
14. Maximum = A + B and minimum = A − B
θ/2
A
15. Resultant is inclined towards a vector having greater
magnitude. 31. a − b is nothing but addition of a and − b.
16. P 10 = 4P + 2P + 4 2 P cosθ
2 2 2 So, the magnitude of a − b will lie between | a | + | b | and
| a | − | b |.
∴ θ = 45°
32. A − 2B + 3C = (2$i + $j) − 2 (3$i − k$ ) + 3 (6i$ − 2k$ )
17. A 2 + B 2 + 2AB cosθ = 20$i − 5$j − 4k$
= A 2 + B 2 − 2AB cosθ 33. A + B = 10 $i + 5j$
∴ cosθ = 0 or θ = 90° ∴ | A + B | = 100 + 25
18. R = A = B =5 5
∴ R = R + R + 2RR cosθ
2 2 2
Angle of A + B with x-axis,
θ = tan −1   = tan −1 1
1 5
or cosθ = −  
2  10   2
∴ θ = 120° 34. S = r f − ri = r3 − r1
C 3
19. cosθ = = = ( 3i$ + 12j$ − 3k$ ) − ( 3i$ + 4j$ − k$ )
A 5
B A = 8j$ − 2k$
35. | A + B | max = A + B
and | A + B | min = A − B
36. S = 3i$ + 6j$ − 7i$ = − 4i$ + 6j$
∴ | S | = 16 + 36 = 52 m
θ
C 37. R = 9P + 4P 2 + 12P cosθ
2
…(i)
−1
∴ θ = cos ( 3/5) 2R = 36P + 4P + 24P cosθ
2 2
…(ii)
20. Suppose angle between two vectors A and B of equal 1
Solving them, we get cosθ = −
magnitude is θ. Then, angle between A and A − B will be 2
180° − θ θ ∴ θ = 120°
or 90° − . Hence, this angle will depend on the
2 2 38. Third force should be equal and opposite to the resultant of
angle between A and B or θ. the given vectors.
θ
B | a + b | = 2A cos
α A 2
= ( 2)(1)cos 30°
= 3
θ θ
A –B | a − b | = 2A cos
A –B 2
= ( 2)(1)cos 30°
23. From polygon law of vector addition, C + A = B . =1
24. Resultant should lie between F1 + F2 and F1 − F2 or it should 360°
40. For given condition, θ = = 30°
lie between 14 N and 2 N. n
26. S = r − r = ( 3 $i + 4 $j + 5 k$ ) − ( 2 $i + 3 $j + 5 k$ )
Q p 41. 3 = ( 3 )2 + ( 3 )2 + 2 ( 3 )( 3 )cosθ

= $i + j$ ⇒ θ = 60°
56 Objective Physics Vol. 1

42. Angle between A and B is 90° 59. A − B = 2i$ + 2j$ + 2k$ = 2 A


i.e. A − B and B are parallel.
A ⋅B 2 + 3 5
60. Component of A along B = A cosθ = = =
B 1+1 2
C
A
61. Magnitude of which is 1.
63. P ⋅ Q = 0

64. Diagonal vector A = b$i + bj$ + bk$


B or A = b 2 + b 2 + b 2 = 3b
Between B and C is135° and that between A and C is135°. $ $
∴ A$ = A = i + $j + k
43. Resultant of two vectors A and B lies between A + B and A 3
A − B. 65. F lies in xy-plane. Hence, 7 k$ which is perpendicular to
44. R = A 2 + B 2 + 2AB cosθ …(i) xy-plane is also perpendicular to F.
From Eq. (i), we get 67. Dot product of two perpendicular vectors should be zero.
R 2 = C 2 = A 2 + B 2 + 2AB cosθ 69. Resultant of two given vector is 12$i + 5$j . Magnitude of this
= A2 + B2 resultant will be 144 + 25 = 13. Hence, a unit vector parallel
⇒ cosθ = 0 12$i + 5j$
to this resultant would be .
π 13
⇒ θ=
2 70. Dot product of two given vectors is zero. Hence, they are
mutually perpendicular.
45. A + B + 2AB cosθ = 2 A + B − 2 AB cosθ
2 2 2 2
71. The given vector is in x z-plane. Hence, this is perpendicular
From this equation, we cannot find θ. to y-axis.
46. | A | = 9 + 16 = 5 and | B | = 36 + 64 = 10. B = 2 A or B is 72. FV = 5 cos 60° = 2.5 N
parallel to A. Ratio of their coefficients are equal, so they are
73. A vector parallel to A will be nA or ( 3ni$ + 4nj$ )
parallel or their cross product is zero.
49. AB sin θ = 3 AB cosθ or tanθ = 3 Now, | nA | = | B | is given.
Hence, n = 9 + 16
∴ θ = 60°
Now, | A + B | = A 2 + B 2 + 2AB cos60° = 49 + 576
or n= 5
= A 2 + B 2 + AB
∴ nA = 15i$ + 20j$
50. W = F ⋅ ( r2 − r1 )
74. A = ( $i − 3$j + 3k$ ) + ( 3$i + 6$j − 7k$ ) = $j (given)
51. (a ⋅ b )2 = a 2 b 2 cos2 θ = a 2 b 2 (given)
Hence, A = − 4$i − 2j$ + 5 k$
∴ θ = 0°
52. 8 = ( 5)( 2)sinθ 75. Magnitude of given vector is 1.
∴ sin θ = 4/5 76. In xy-plane vector is 3 $i + j$
or cosθ = 3/5 ∴ Length in xy-plane = 9 + 1 = 10
∴ A ⋅ B = AB cosθ = ( 5)( 2)( 3/5) = 6 A ⋅B  −1  1 
1 77. θ = cos−1   = cos  
53. AB sin θ = AB cosθ  AB   3
3
1 78. Dot product of two perpendicular vectors should be zero.
∴ tanθ = or θ = 30°
3 |a + b |
80. =1
54. Area of parallelogram= | A × B | |a − b |
55. A ⋅ B = 0 ⇒ ∴ A ⊥B i.e. |a + b | = |a − b |
A ⋅C = 0 or a + b + 2ab cosθ = a 2 + b 2 − 2ab cosθ
2 2

∴ A ⊥C ∴ cosθ = 0° or θ = 90°
i.e. A is perpendicular to both B and C. 81. 20 cos 60° = 10 m
56. A ⋅ B = ( 3)( 5)cos60° = 7.5
82. The coefficients of i$ , $j and k$ should bear a constant ratio.
A ×B
57. n$ = 2 3 1
| A × B| or = =
− 4 −6 − λ
58. γ = cos−1  z 
P
P
or λ=2
83. P × Q is perpendicular to the plane formed by P and Q.
Here, Pz = 4 2
P + Q lies in this plane. Hence, P + Q is perpendicular to
and P = (6)2 + ( 4 2)2 + ( 4 2)2 =10 P × Q.
Vectors 57

84. (0.5)2 + (08


. )2 + (c )2 = 1 F12 = 2 F (in opposite direction of F3 )
Now, resultant of F12 and F3 will be,
∴ c 2 = 0.11 or c = 0.11
Fnet = ( 2 − 1) F
85. Their dot product should be zero.
Fnet F
86. Dot product should be zero. ∴ a′ = = ( 2 − 1)
m m
A ⋅B = ( 2 − 1) a
87. Component of A along B =
B
105. A − B = j$ − 2k$ = C (say)
88. For vectors to be parallel, ratio of coefficients should be
same. C = 1+ 4 = 5
6 x −2 0 1
∴ = = cosα = = 0,cosβ =
5 −6 − y 5 5
89. A = 9 + 36 + 4 = 7 −2
and cosγ =
−1  3  etc. 5
∴ α = cos  
 7 106. ( A + B ) ⋅ ( A − B ) = 0
91. Area = | A × B | ∴ A ⋅ A + B ⋅ A − A ⋅B − B ⋅B = 0
Here, A × B = ( 2$i + 3j$ ) × ( i$ + 4j$ ) ∴ A + AB cosθ − AB cosθ − B 2 = 0
2

= 8 k$ − 3 k$ ∴ A=B
= 5 k$
Level 2 : Only One Correct Option
∴ Area = | A × B | = 5 Units
θ
1. | A − B | = 1 + 1 − 2 × 1 × 1 × cos θ = 2 sin
93. Resultant of two vectors lies between A + B and | A + B |. 2
94. Their cross product should be zero. Angle between A and B is also θ, but their magnitudes are 1.
(a + b ) × (a − b ) = 0 2. s 1 = ( 2 cos 45° ) i$ + ( 2 sin 45° ) $j = $i + j$
∴ 2 (a × b ) = 0 s 2 = 2 j$and s = ( − 4cos 30° ) i$ + ( 4 sin 30° ) $j
a =b = − 2 3 $i + 2$j
θ
97. AR = 2A cos Now s = s 1 + s 2 + s 3 = (1 − 2 3) i$ + j$
2
$ $ $
98. ( ai + bj ) ⋅ ( i + j$ ) = 0 or a + b = 0 …(i) ^
N( j )

Further, a2 + b2 =1
or a2 + b2 =1 …(ii)
^
99. a + b + c = 0 E( i )
∴ a + c=−b
or (a + c ) × b = − b × b = 0
or (a × b ) + (c × b ) = 0 ∴ s = 1 − 2 3 )2 + (1)2
or a × b = b ×c
= 1 + 12 + 1 − 4 3
100. F ⋅ F ′ = 0. Here, F ′ = unknown force.
= 14 − 4 3 m
101. Dot product should be zero.
F 3. R 21 = A + B + 2AB cosθ
2 2
102. a =
m R22 = A 2 + B 2 − 2AB cosθ
F
∴ m= ∴ R12 + R22 = 2 ( A 2 + B 2 )
a
103. A + B = 10$i + 5$j 4. Sum of two unit vectors is a unit vector, means angle
between those two unit vectors is120°.
∴ | A + B | = 100 + 25
∴ | S | = 1 + 1 − 2 × 1 × 1 × cos120° = 3
=5 5
5. R ⋅ S = ( A + B ) ⋅ ( A − B )
Angle of A + B with x-axis,
0 = A ⋅ A − A ⋅B + B ⋅ A − B ⋅B
θ = tan −1  
5
or 0 = A2 − B2 (as A ⋅ B = B ⋅ A )
 10 
∴ A =B
2 2
or A = B
= tan −1  
1
 2 6. Ax = A y = Az

104. a =
F Now, A = Ax2 + A 2y + Az2 = 3 Ax
m A 30
∴ Ax = = = 10 3
Resultant of F1 and F2 will be 3 3
58 Objective Physics Vol. 1

7. AB + AC + AD + AE + AF or Q 2 − P 2 = R 2 = 144 …(iii)
E D Solving the above equations, we get P = 5 and Q =13.

16. | ∆a | = a 2 + a 2 − 2 ⋅ a ⋅ a ⋅ cos( dθ) = 2a sin  
 2
F C dθ dθ
For small angles, sin ≈
O 2 2

∴ | ∆a | = 2a × = a ⋅ dθ
A B
2
17. Resultant of OA and OC is 2 R along OB.
= AB + ( AB + BC ) + ( AB + BC + CD ) + ( AF + FE ) + AF
= ( AB + CD ) + (BC ) + (BC ) + 2( AB + AF ) + (FE ) Hence, Rnet = R + 2R = (1 + 2) R
= AO + AO + AO + 2 AO + AO 18. Apply polygon law.
= 6AO 19. (a + b ) × (a − b ) = a × a − a × b + b × a − b × b
8. Since, A is a unit vector in a given direction. It should be a = 2( b × a )
constant unit vector.
20. B cosθ = C and B sinθ = A
dA
or =0
dt
9. F ⋅ F ⋅ cosθ = F ⋅ F ⋅ sin θ or tanθ =1 or θ = 45°
| F1 ⋅ F2 | = F 2 + F 2 + 2 ⋅ F ⋅ F ⋅ cos 45° B C
θ
=( 2+ 2)F
A
10. OB + BC = r or q + BC = r …(i)
But A =C
OA + AC = r
∴ B cosθ = B sin θ
or p + AC = r … (ii)
or θ = 45°
Adding these two equations, we get, p + q = 2r, as
∴ Angle between A and B is135°.
AC and BC are equal and opposite vectors.
2A sin θ
11. cos2 α + cos2 β + cos2 γ = 1 (a standard result) 21. tan 90° = =∞
A + 2A cosθ
or (1 − sin 2 α ) + (1 − sin 2 β ) + (1 − sin 2 γ ) = 1
R
∴ sin 2 α + sin 2 β + sin 2 γ = 2 2A

12. Cross product of (P + Q ) and (P − Q ) is perpendicular to the


plane formed by (P + Q ) and (P − Q ).
13. 2Q sinθ = P 90°
A
θ
R
∴ A + 2A cosθ = 0
1
2Q
or cosθ = − or θ = 120°
2
θ 22. B ⋅ C = 0
P
A C
Now, R = P 2 + Q 2 + 2PQ cos( 90° + θ)
= P 2 + Q 2 − 2PQ sin θ
=Q
θ
14. B × A is perpendicular to A. Hence A ⋅ (B × A ) will be zero. B
15. P + Q =18 …(i)
∴ B ⊥C
R =12 …(ii) B 1
A sinθ = B or sinθ = =
Q sinθ = P and Q cosθ = R A 2
R ∴ θ = 30°
Q
or A ⋅B < 0
23. From R = A 2 + B 2 + 2AB cosθ
θ We have,
P 1
1 = 1 + 1 + 2cosθ or cosθ = −
Squaring and adding, we get 2
P 2 + R 2 = Q2 ∴ θ = 120°
Vectors 59

| A × B | AB sin θ
S = A 2 + B 2 − 2AB cosθ 10. is = sin θ
AB AB
= 1 + 1 − 2 × 1 × 1 ×  − 
1 And the angle between two vectors.
 2 | A ×B|
0° ≤ 180°. Therefore, may be positive or zero.
= 3 AB
| A × B|
R Further, θ = sin −1  will give two values of θ. Those
B  AB 
are θ and180° − θ.
11. Angle between A × B and B × A is180°.
∴ ( A × B ) ⋅ (B × A ) = ( AB sin θ)( AB sin θ)(cos180° )
= − A 2 − B 2 sin 2 θ
60°
A
30° 12. Further, A ⋅ B = ABcosθ. Here, AB is always positive value of
cosθ determines the sign of A ⋅ B .
Q R

–B S

24. Magnitude of component of v along a


v ⋅a 6 + 2 − 2 θ
= =
a 3 P
=2 3 13. ( A × B ) ⋅ ( A − B ) = A 2 − B 2
i$ + j$ + k$ This dot product is zero only if A = B.
Now, a$ =
3 Therefore, they are perpendicular. Further, ( A × B ) is
∴Component in vector form perpendicular to both ( A + B ) and ( A − B ).
= 2 3 a$ = ( 2$i + 2j$ + 2 k$ )
25. Q sin θ = P
Match the Columns
1. ( 2$i + $j + 4k$ ) is perpendicular to A, because the dot product
Q cosθ = R = P
of these two vectors is zero.
R Further (6i$ + 4$j − 10k$ ) vectors is parallel to A. So, component
Q
of A along this vector is magnitude of A which is 5 2 unit.
The last vector i.e. ( − 3$i − 4$j + 5k$ ) is anti-parallel to A along
this vector is negative of magnitude of A or − 5 2 unit.
θ

P Entrance Gallery
1. At t = 0, airplane B is at location Q, airplane A is at location P.
Dividing these two equations, we get Relative velocity of B w.r.t. to A
tanθ =1 i.e. v BA = vB − vA = vB + ( − vA ) has direction from Q to P.
∴ θ = 45°
∴Angle between P and Q is135° R
26. Suppose a = a i$ + a j$ + a k$
1 2 3
P 30°

Now, ( $i × a ) = a2 k$ − a3 j$ vB

Now, i$ × ( $i × a ) = − a2 $j − a3 k$ A –vA
Q
B
A

j$ × ( j$ × a ) = − a1 i$ − a3 k$
v

Similarly, 120° 60°


30°
A′
and k$ × ( $j × a ) = − a1 $i − a2 $j B′

∴ $i ( $i × a ) + j$ ( j$ × a ) + k$ ( k$ × a ) = − 2 a Given, PQ = 500 m
From right angled triangle QPR
27. Resultant of (P + Q ) and (P − Q ) is P + Q + P − Q or 2P which is PQ 500 1
= tan 30° ⇒ =
parallel to P. PR PR 3
⇒ PR = 500 3 m
Assertion and Reason
4. | A × B | = AB sin θ and A ⋅ B = ABcosθ Given, speed of A = 100 3 ms−1
At t = t0 , airplane A reaches at location R
5. | A + B | = A 2 + B 2 + 2AB cosθ
PR 500 3
t0 = = = 5s
and | A − B | = A 2 + B 2 − 2AB cosθ Speed of plane A 100 3 ms−1
60 Objective Physics Vol. 1

2. As, position vector, r = x i$ + y $j i.e. τ = ( 2$i + 2$j + 1k$ ) × ( 3$i + 7$j + 4k$ )
r = 4t 2 $i + 3t 2 $j $i $j k$
dr
Velocity v= = 4( 2t ) i$ + 3 ( 2t ) $j = 2 2 1 = $i ( 2 × 4 − 1 × 7) − $j( 2 × 4 − 3 ×1)
dt
3 7 4
or v = 8t i$ + 6t $j
+ k$ ( 2 × 7 − 2 × 3) = $i − 5$j + 8k$
∴ Speed = | v | = 64t 2 + 36t 2 = 100 t 2 = 10t
8. As, a ⋅ b = | a × b |
3. Angular momentum, L = r × p
| a | | b | cos θ = | a | | b | sin θ
i$ $j k$
1 = tan θ
= 1 −2 1 ∴ tan 45° = tan θ
2 −1 1 θ = 45°
= $i ( − 2 + 1) − $j (1 − 2) + k$ ( − 1 + 4) = − i$ + $j + 3k$ 9. See the vector diagram given below OR = 5 units

4. Weight, nuclear spin, momentum are vector quantities, OP = 4 units


because they have magnitude and direction also. OQ = 3 units
Potential energy has magnitude only, it does not have
direction. So, it is not a vector. Q 4 R
5. Torque τ = r × F
$i $j C
k$ +C
= 1 − 2 1 = i$(10 − 2) − $j( − 5 − 5) + k$ ( 2 + 10) 3 =B 3
A 5
5 2 −5
θ
= 8$i + 10$j + 12k$ O P
4 B
6. The given vector, X = αA + βB = α ( $i + $j − 2k$ ) + β ( i$ − j$ + k$ )
= (α + β ) $i + (α − β ) $j + ( − 2 α + β ) k$ As, 5 = 42 + 32 + 2 × 4 × 3 × cosθ
and C = 2$i − 3j$ + 4k$ ⇒ 25 =16 + 9 + 24 cosθ
Now, αA + β B is perpendicular to C when ⇒ cosθ = 0
(αA + βB) ⋅ C = 0 π
⇒ (α + β )( 2) + (α − β )( − 3) + ( − 2α + β )( 4) = 0 ⇒ 90° =
2
⇒ 2α + 2β − 3α + 3β − 8α + 4β = 0
Angle between A and C = ∠QOR
⇒ − 9α + 9β = 0
α 4
⇒ − 9 α = − 9β ⇒ =1 sin ( ∠QOR ) =
β 5
∠QOR = sin −1  
4
∴ α : β =1 :1 ⇒
 5
7. Torque τ is perpendicular to the plane of r and F and is
given by τ = r × F 3
Also, cos( ∠QOR ) =
Y 5
F
∠QOR = cos−1  
3

 5
A⋅B
r 10. Component of A along B =
X | B|
O 2+ 3 5
Component of A along B = =
Z 2 2
3
Motion in One
Dimension

3.1 Introduction
Motion is change in position of an object with time. In the present chapter, we shall Chapter Snapshot
confine ourselves to the study of motion along a straight line, also called as rectilinear ● Introduction
motion. If size of the object is much smaller than the distance it moves in a reasonable
duration of time, then objects can be treated as point objects. ● Position, Position vector,
Distance and
Kinematics is the branch of physics which deals with the motion regardless of the Displacement
causes producing this motion. The study of causes of motion is called dynamics. ● Average speed, Average
In the chapter of kinematics, the physical quantities involved are : distance (or path velocity, Instantaneous
length), displacement, speed, velocity, acceleration and time. Of these, displacement, speed, Instantaneous
velocity and acceleration are vector quantities. So, they can be added or subtracted like velocity
vectors. But we can take liberty with vectors and they can be added/subtracted like scalars ● Uniform Motion
if all vectors are one dimensional (for example in rectilinear motion). In this method, one
direction is taken as the positive direction and the other as the negative direction. Now, all ● Acceleration
vector quantities are represented with sign. Then, they can be added/subtracted like ● Uniformly accelerated
scalars. motion
The origin and the positive direction of an axis are a matter of choice. You should first ● Graphs
specify this choice before you assign, signs to quantities like displacement, velocity and ● Relative Motion
acceleration.
For example, if rectilinear motion is taking place along horizontal direction, then sign
convention may be taken as shown in Fig. 3.1.
–ve +ve
Fig. 3.1

Similarly, if motion is taking place in vertical direction, then sign convention may be
taken as shown in Fig. 3.2
–ve

+ve
Fig. 3.2
62 Objective Physics Vol. 1

3.2 Position, Position Other Important Points


(i) Distance travelled by a particle is also called the path
Vector, Distance and length.
Displacement (ii) In rectilinear motion (or one-dimensional or 1-D
motion), we can choose an axis (say x-axis). Now,
position of the particle at any point can be represented
Position and Position Vector by its x-coordinate.
In cartesian coordinate system, position of any point
(iii) In this case, all vector quantities (displacement,
(say A) is represented by its coordinates ( x A , y A , z A ) with velocity and acceleration) can be represented simply
respect to an origin O. as scalars with proper signs (positive or negative).
y
(iv) Displacement in this case is equal to the final
A
rA x-coordinate minus the initial x-coordinate or
s = x f − xi
x
(v) Distance travelled is always positive but position of
O
the particle (or the x-coordinate) and displacement
Fig. 3.3
may be positive or negative.
Position vector of point A with respect to O will now be
X Example 3.1 A particle is moving along x-axis. At
r = OA = x $i + y $j + z k$
A A A A some given time x-coordinate of the particle is
Now, suppose coordinates of two points A and B are x = − 4 m. First it moves towards positive x-axis and
known to us and we want to find position vector of B with reaches upto x = + 6m. Then it moves along negative
respect to A, then x-axis and reaches upto x = −10 m. In the complete
AB = rB − r A journey, find
AB = ( x − x ) $i + ( y − y ) $j + ( z − z ) k$ (a) distance travelled by the particle
B A B A B A
y (b) displacement of the particle.

Sol. x = – 4m x = 6m
B
A x = –10 m x= 0 x axis
rA A B
rB
C B
A
O x
Fig. 3.6
Fig. 3.4
(a) Distance traversed = AB + BC
= 10 + 16 = 26 m
Distance and Displacement (b) Displacement = S = AC = X f − X i
Distance is the actual path Y = (−10 m) − (−4 m) = − 6m
length covered by a moving B / Displacement is a vector quantity. Negative sign with any
particle or body in a given time vector quantity represents its direction. For example,
S C
interval, while displacement is S = − 6 m, means 6m towards negative x-direction.
A
the change in position vector,
rA
i.e. a vector joining initial to
final positions. If a particle
rC
3.3 Average Speed, Average
X
moves from A to C (Fig. 3.5)
through a path ABC, then
Velocity, Instantaneous
Fig. 3.5
distance ( ∆s) travelled is the Speed, Instantaneous
actual path length ABC, while
the displacement is Velocity
s = rC − r A Average Speed and Velocity
If a particle moves in a straight line without change in The average speed of a particle in a given interval of
direction, the magnitude of displacement is equal to the time is defined as the ratio of distance travelled to the time
distance travelled, otherwise, it is always less than it. Thus, taken while average velocity is defined as the ratio of
|displacement | ≤ distance displacement to time taken.
Motion in One Dimension 63

Thus, if the distance travelled is ∆s and displacement of Sol. (a) Average speed = total distance = 26
total time 4
a particle is ∆ r in a given time interval ∆t, then
= 6.5 m /s
∆s total displacement 6
v av = Average speed = and (b) Average velocity = =−
∆t total time 4
∆r = −1.5 m/ s
v av = Average velocity =
∆t Note Points
/ Average velocity is a vector quantity. So, negative sign in it
Instantaneous Speed and Velocity merely represents its direction. For example, −1.5 m/ s means
Instantaneous speed and velocity are defined at a average velocity is 1.5 m/ s along negative x-direction.
particular instant and are given by / Since, distance ≥ | displacement |
∴ Average speed ≥ | Average velocity |
∆ s ds
v = lim =
∆ t → 0 ∆ t dt X Example 3.3 Instantaneous velocity (or we can
∆ r dr simply say velocity) of a particle moving in a three
and v = lim =
∆ t→ 0 ∆ t dt dimensional space is
v = (2$i − 3$j + 4k
$ ) m/ s
Other Important Points Find instantaneous speed of the particle at this instant.
(i) In rectilinear motion (or 1-D motion), when position
of any particle can be represented by its x-coordinate Sol. Instantaneous speed
only the above formulae can be represented as = magnitude of instantaneous velocity
= (2 )2 + (−3 )2 + (4 )2 = 29 m/s
Total distance
(a) Average speed =
Total time X Example 3.4 A particle is moving along x-axis. Its
Displacement s
(b) Average velocity = = x-coordinate varies with time as
Time t
x = 2t 2 − 4
x f − xi
= Here, x is in metre and t in second. Find
t (a) average velocity of the particle in a time interval from
dx t = 0 to t = 4 s.
(c) Instantaneous velocity = (b) instantaneous velocity of the particle at t =1 s.
dt
(d) Instantaneous speed = magnitude of instantaneous x − xi
Sol. (a) Average velocity = s = f
t t
velocity.
xf = x4 s = (2 ) (4 )2 − 4 = 32 m
(ii) Instantaneous speed is always equal to the magnitude
xi = x0 s = (2 ) (0 )2 − 4 = − 4 m
of instantaneous velocity.
(32 ) − (−4)
(iii) Average and instantaneous values can also be ∴Average velocity = = 9 m/s
4
represented as
(b) Expression of instantaneous velocity at a general
∆r ∆s s time t is given by
(a) Average velocity = = or
∆t ∆t t v=
dx d
= (2t 2 − 4)
s Xf − Xi
dt dt
∆s
= or = (in 1-D motion) = (4 t )
∆t t t ∴ At t = 1 s,
dr ds v = (4 ) (1) = 4 m/s
(b) Instantaneous velocity = or
dt dt
=
ds
or
dx
(in 1-D motion)
3.4 Uniform Motion
dt dt If velocity of a particle is constant, then it is called a
uniform motion. Velocity is a vector quantity. So, velocity is
X Example 3.2 In Example 3.1, if total time of constant means its direction and magnitude (or speed) both
journey is 4 s, then find are constant. Since direction is constant, hence the motion is
(a) average speed rectilinear (or 1-D) in the same direction. Displacement and
(b) average velocity distance both are equal in this case. Particle covers equal
64 Objective Physics Vol. 1

distances (or equal displacements) in equal intervals of time.


We can apply the formula, 3.5 Acceleration
distance or displacement Velocity is a vector quantity consisting of both
Speed or velocity = magnitude and direction. If velocity of particle changes
time
s (either in magnitude or in direction or both) it is said to be
or v= accelerated. If complete velocity vector is constant (uniform
t
motion), then acceleration of the particle is zero.
∴ s = vt
Average acceleration is defined as the ratio of change in
Extra Edge velocity, i.e. ∆ v to the time interval ∆ t in which this change
■ In case of uniform motion, instantaneous speed, occurs. Hence,
instantaneous velocity average speed (in any time ∆v
interval) and average velocity (again in any time interval) a av =
all are same. ∆t
v f − vi
=
X Example 3.5 A particle is moving along negative ∆t
x-direction with uniform velocity of 4 m/s. At time t = 0, The instantaneous acceleration is defined at a particular
particle is at x = 6m. Find x-coordinate of the particle instant and is given by
at t = 2 s. ∆ v dv
a = lim =
Sol. Velocity is a vector quantity and motion is one dimensional. ∆t→ 0 ∆t dt
So, we can write velocity vector (and other vector quantities also) / In rectilinear motion (say along x-axis), the formulae of
with sign. Hence, average and instantaneous acceleration can be written as
v = − 4 m/s (Given) ∆v vf − vi dv
aav = = and a =
In uniform motion, we can apply the equation ∆t ∆t dt
s = vt Here, acceleration and velocity are vector quantities and
but s = xf − xi motion is one-dimensional. So, they can be treated like
∴ xf − xi = vt scalars by assigning them proper signs.
∴ xf = xi + vt
or x2 s = x0 s + vt
X Example 3.7 At time t = 0 s, velocity of a particle
Substituting the values, we get is v 1 = (2$i − 3$j) m/s and at t =10 s velocity is
x2 s = 6 + (−4 ) (2 ) v 2 = ( 4$i − 2$j) m/ s. Find average acceleration in the
= −2 m
given time interval.
X Example 3.6 A particle travels half of the total
Sol. Average acceleration is given by
distance with constant velocity v1 and rest half distance ∆v v f − v i
with constant velocity v 2 . Find average velocity aav = = ...(i)
∆t ∆t
(or average speed) during the complete journey. Here, v f = v 2 s = (4$i − 2 $j ) m/s

Sol. In uniform motion, we can apply the equation and vi = v0 s = (2 $i − 3$j ) m/s

C and ∆t = 10 s.
s s
Substituting the values in Eq. (i) we have,
A v1 v2 B
(4$i − 2 $j ) − (2 $i − 3$j )
a av =
t1 t2 10
Fig. 3.7 = (0.2 $i + 0.1$j ) m/s 2
s
s = vt or t =
v X Example 3.8 A particle is moving along the
s s X-axis. The X-coordinate of the particle varies with
In the figure, t1 = and t 2 =
v1 v2
time as
total displacement
Now, average velocity = X = (3t 3 + 4t 2 − 6)
total time
s+ s 2s Here, X is in metre and t in second. Find
= =
t 1 + t 2 (s / v1 ) + (s / v 2 ) (a) instantaneous acceleration of the particle at t =1 s.
2 v1v 2 (b) average value of acceleration in a time interval from
=
v1 + v 2 t = 0 to t = 2 s.
Motion in One Dimension 65

Sol. (a) Expression of velocity as a function of time (called the Sign convention for (a) motion in vertical direction (b)
instantaneous velocity) is given by motion in horizontal direction is shown in Fig. 3.8.
dX d
v= = (3t 3 + 4t 2 − 6)
dt dt +ve –ve
or v = (9t 2 + 8t ) ...(i) +ve –ve
(a) (b)
By further differentiating Eq. (i) with respect to time
we will get expression of acceleration as a function Fig. 3.8
of time (called the instantaneous acceleration)
dv d
∴ a= = (9t 2 + 8t ) Note Points
dt dt
or a = 18t + 8 ...(ii) / Value of acceleration due to gravity g is 9.8 m/ s2 always in
Now, at t = 1 s, downward direction. So, according to our sign convention
a = 18 (1) + 8 = 26 m/s 2 (upward positive and downward negative)
(b) Average acceleration is given by g = − 9.8 m/ s2
∆v v f − v i For making the calculations simple we can also take
aav = = ...(iii)
∆t ∆t g = − 10 m/s 2 in some problems.
From Eq. (i), This value of g is independent of the direction of velocity
v f = v 2 s = (9) (2 )2 + (8) (2 ) = 52 m/s For example
A particle is projected upwards and it moves from B
v i = v 0 s = (9) (0)2 + (8) (0) = 0
A to B and then returns back from B to A.
Substituting the values in Eq. (iii) we have, Now, in upward journey from A to B velocity is
52 − 0
aav = = 26 m/s 2 upwards (means positive) and acceleration is
2 g (= −9.8 m/ s2 ). At highest point B, velocity is zero A
and acceleration is g (= −9.8 m / s2 ). Finally in Fig. 3.9
downward journey from B to A, velocity is
3.6 Uniformly Accelerated downwards (means negative) but acceleration is still
g (= −9.8 m/ s2 )
Motion / In rectilinear motion, if velocity and acceleration are in the
Equations of motion for uniformly accelerated motion same direction (both positive or both negative) then speed of
the particle increases. For example, in downward journey from
(a = constant) are as under B to A both velocity and acceleration are negative. Hence
1 speed is increasing. If velocity and acceleration are in
v = u + at, s = ut + at 2 , v ⋅ v = u ⋅ u + 2 a ⋅ s opposite directions (one positive and the other negative),
2
speed of the particle decreases. For example, in upward
Here, u = initial velocity of particle, v = velocity journey from A to B, velocity is upwards (positive) and
of particle at time t and s = displacement of particle in time t acceleration is downwards (negative). So, speed of the
/ If initial position vector of a particle is r0 , then position vector particle is decreasing.
at time t can be written as / For small heights acceleration due to gravity g is constant. So,
1 equations of constant acceleration (v = u + at etc. ) can be
r = r0 + s = r0 + ut + at 2
2 applied.
/ A particle is projected from the top of a tower AB in upward
One-dimensional Uniformly direction. It first moves from B to C, then from C to D and finally
hits the ground at time t. Then, there are two methods of findin t.
Accelerated Motion
C
If the motion of a particle is taking place in a straight
line, there is no need of using vector addition
B
(or subtraction) in equations of motion. We can directly use
the equations,
1
v = u + at, s = ut + at 2
2 A D
and v = u + 2as
2 2 Fig. 3.10

1 2
Just by taking one direction as the positive (and Method 1 We directly apply s = ut + at and substitute all
2
opposite to it as negative) and then substituting u, a, etc., quantities (s, u and a ) with sign. But in this equation,
with sign. Normally we take vertically upward direction displacement s is measured from the starting point. Here the
positive (and downward negative) and horizontally starting point is B.
rightwards positive (or leftwards negative). Method 2 Find tB → C and tC → D and then t = tB → C + tC → D
66 Objective Physics Vol. 1

X Example 3.9 A jet plane starts from rest with an or 5t 2 − 10t − 40 = 0


acceleration of 3 ms −2 and makes a run for 35 s before or t 2 − 2t − 8 = 0
taking off. What is the minimum length of the runway Solving this, we have
and what is the velocity of the jet at take off ? t = 4 s and −2 s.
Taking the positive value, answer is t = 4 s.
Sol. Here u = 0, a = 3 ms −2 , t = 35 s / The significance of t = − 2 s can be understood by following
Minimum length of the runway required is figure :
1 1
s = ut + at 2 = 0 + × 3 × (35)2 C C
2 2
t = 1s
= 1837.5 m tAB = tDE = 2s
Velocity of the jet at take off is tBC = tCD = 1s
v = u + at = 0 + 3 × 35 = 105 ms −1
t=0 B D
t = 2s
X Example 3.10 An electron travelling with a speed
of 5 × 10 3 ms −1 passes through an electric field with an
acceleration of 1012 ms −2 .
t = – 2s A
(a) How long will it take for this electron to double its E t = 4s
speed? Fig. 3.12
(b) What will be the distance covered by the electron in
Alternate Method
this time?
From B to C Using v = u + at
Sol. Here, u = 5 × 10 ms , 3 −1 or 0 = + 10 + (−10) (t )
3 −1
v = 2 × 5 × 10 ms , a = 10 ms 12 −2 or t = 1s = t BC
1
(a) v = u + at S = ut + at 2
2
v − u 2 × 5 × 103 − 5 × 103
∴ t = = 1
a 1012 = (+10) (1) + (−10) (1)2
2
5 × 103 −9
= = 5 × 10 s = 5 m = BC
1012
1 From C to A CA = CD + DA = 5 + 40 = 45 m
(b) s = ut + at 2
2 1
Using the equation, S = ut + at 2 we have
1 2
= 5 × 103 × 5 × 10−9 + × 1012 × (5 × 10−9 )2
2 1
(−45) = (0) t + (−10) t 2
= 3.75 × 10−5 m 2
X Example 3.11 A ball is thrown upwards from the Solving this equation, the positive value of t comes out to
be,
top of a tower 40 m high with a velocity of 10 m/s.
Find the time when it strikes the ground. Take t = 3s = t CA
g =10 m/ s 2 . ∴ Total time, t = t BC + t CA
= 1+ 3
Sol. In this problem, = 4s
u = +10 m/s
+ve
a = g = –10 m/s2
X Example 3.12 In rectilinear motion with constant
s=0
acceleration derive the expression of displacement in
t th second S t .

40 m Sol. St = displacement in t th second


= [Total displacement in t seconds]
− [Total displacement in (t − 1) seconds]
1 2  1
= ut + at − u (t − 1) + a (t − 1)2 

Fig. 3.11
 2   2 
u = + 10 m/s , a = − 10 m/s 2
Simplifying this equation, we get
and s = − 40 m (at the point where stone strikes the ground) 1
1 St = u + at − a
Substituting the above values in s = ut + at 2 , we have 2
2
/ In the above equation St is the displacement of t th second,
− 40 = 10 t − 5t 2
not the distance.
Motion in One Dimension 67

(a) slope of displacement-time graph gives velocity


3.7 Graphs  ds 
 as v =  .
Before studying this article, students are advised to go  dt 
through the theory of graphs. A brief summary is given (b) slope of velocity-time graph gives acceleration
below.  dv 
 as a =  .
(i) A linear equation between x and y represents a straight  dt 
line, e.g. y = 4x − 2, y = 5x + 3, 3x = y − 2 equations
(c) area under velocity-time graph gives displacement
represent straight line on x-y graph.
(as ds = v dt ).
(ii) x ∝ y or y = kx represents a straight line passing
(d) area under acceleration-time graph gives change
through origin.
in velocity (as dv = a dt ).
1
(iii) x ∝ represents a rectangular hyperbola on x-y graph.
y Table 3.1
Shape of rectangular hyperbola is as shown in Graph Area Slope
Fig. 3.13.
y S-t v
v-t S a

a-t vf − vi

(e) displacement-time graph in uniform motion is a


straight line passing through origin, if
x displacement is zero at time t = 0 (as s = vt).
(f) velocity-time graph is a straight line passing
Fig. 3.13
through origin in a uniform accelerated motion, if
(iv) A quadratic equation in x and y represents a parabola initial velocity u = 0 and a straight line not passing
on x-y graph, e.g. y = 3x 2 + 2, y 2 = 4x, x 2 = y − 2 through origin if initial velocity u ≠ 0 (as
v = u + at).
equations represent parabola on x-y graph.
(g) displacement-time graph in uniformly
(v) If z = yx or y ( dx ) or x ( dy), then value of z between x1
accelerated or retarded motion is a parabola
and x 2 or between y1 and y2 can be obtained by the
 1 2
area of graph between x1 and x 2 or y1 and y2 .  as s = ut ± at  .
 2 
dy
(vi) If z = , then value of z is given by the slope of x- y Now, we can plot v-t and s-t graphs of some
dx
standard results in tabular form as under. But note
graph.
that all the following graphs are drawn for
From the above six points, we may conclude that in one-dimensional motion with uniform velocity or
case of a one dimensional motion : with constant acceleration.

Table 3.2
S.No. Different cases v-t graph s-t graph Important points

1. Uniform motion v s (i) Slope of s-t graph = v


= constant
(ii) In s-t graph s = 0 at t = 0
v = constant

s = vt
t
t

Table 3.2 Contd.


68 Objective Physics Vol. 1

S.No. Different cases v-t graph s-t graph Important points

2. Uniformly accelerated v s (i) u = 0, i.e. v = 0 at t = 0


motion with u = 0 and (ii) a or slope of v-t graph is constant
s = 0 at t = 0 (iii) u = 0, i.e. slope of s-t graph at
t = 0, should be zero
1
v = at s = at 2
2
t
t

3. Uniformly accelerated v s (i)u ≠ 0, i.e. v or slope of s-t graph at


motion with u ≠ 0 but t = 0 is not zero
s = 0 at t = 0 (ii) v or slope of s-t graph gradually
goes on increasing
v = u + at 1
u s = ut + at 2
2

t
t

4. Uniformly accelerated v s (i) v = u at t = 0


motion with u ≠ 0 and (ii) s = s 0 at t = 0
s = s 0 at t = 0

s0 1 2
u s = s 0 + ut + at
v = u + at 2

t t

5. Uniformly retarded v s (i) Slope of s-t graph att = 0 gives u


motion till velocity (ii) Slope of s-t graph at t = t 0
becomes zero u becomes zero
v = u – at (iii) In this case u can’t be zero

t
t0
t
t0
6. Uniformly retarded v s (i) At time t = t 0, v = 0 or slope of s-t
and then accelerated graph is zero
in opposite direction (ii) In s-t graph slope or velocity first
u decreases then increases with
opposite sign.

t t
O t0 t0

Important Points in Graphs v s


■ Slopes of v-t or s-t graphs can never be infinite at any point,
because infinite slope of v-t graph means infinite acceleration. v1 s1
Similarly, infinite slope of s-t graph means infinite velocity.
Hence, the following graphs are not possible :
v2 s2
v s
t t
t0 t0

Fig. 3.15

t t ■ On time axis we always move forward.


■ Different values of displacements in s-t graph
Fig. 3.14 corresponding to given v-t graph can be calculated just by
■ At one time, two values of velocity or displacement are not calculating areas under v-t graph. There is no need of
possible. Hence, the following graphs are not acceptable: using equations like v = u + at , etc.
Motion in One Dimension 69

(b) Acceleration
X Example 3.13 Displacement-time graph of a DB DB
particle moving in a straight line is as shown in figure. = a= or DB = at
AD t
State whether the motion is accelerated or not. Distance travelled by the object in time t is
Describe the motion in detail. Given s0 = 20 m and s = Area of the trapezium OABE
t 0 = 4 s. = Area of rectangle OADE + Area of triangle ADB
s 1
= OA × OE + DB × AD
2
1 1
= ut + at × t or s = ut + at 2
t 2 2
t0
This proves the second equation of motion.
– s0 (c) Distance travelled by object in time t is
s = Area of trapezium OABE
1 1
Fig. 3.16 = (EB + OA) × OE = (EB + ED) × OE
2 2
Sol. Slope of s - t graph is constant. Hence, velocity of particle is Acceleration,
constant. Further at time t = 0, displacement of the particle from a = Slope of velocity-time graph AB
the mean position is − s0 or − 20 m. Velocity of particle, DB EB − ED EB − ED
or a= = or OE =
s 20
v = slope = 0 = = 5 m/s AD OE a
t0 4 1 (EB − ED)
s = (EB + ED) ×
Motion of the particle is as shown in figure. At t = 0, particle 2 a
is at − 20 m and has a constant velocity of 5 m/s. 1 2 2 1 2
= (EB − ED ) = (v − u 2 )
At t 0 = 4 s, particle will pass through s = 0 position. 2a 2a
v = 5 m/s or v 2 − u 2 = 2as
+ve This proves the third equation of motion.
s = – 20 m s=0
t=0 t = 4s X Example 3.15 Fig. 3.19. shows the distance-time
Fig. 3.17 graphs of two trains, which start moving simultaneously
in the same direction. From the graphs, find
X Example 3.14 Derive the following equations of (a) how much ahead of A and B when the motion starts ?
motion for uniformly accelerated motion from (b) what is the speed of B?
velocity-time graph : (c) when and where will A catch B?
1
(a) v = u + at (b) s = ut + at 2 (d) what is the difference between the speeds of A and B?
2
(c) v − u = 2as
2 2
A
200
B
Sol. Equations of motion by graphical method. Consider an 150
Q
Distance (km)

object moving along a straight line path with initial velocity u and
uniform acceleration a. Its velocity-time graph is straight line. 100 R
P
C B 50
v

0 S
0 1 2 3 4
Velocity

Time (h)
Fig. 3.19

u t D Sol. (a) B is ahead of A by the distance OP = 100 km, when the


A
E motion starts.
O Time t QR 150 − 100
(b) Speed of B = =
Fig. 3.18. Velocity-time graph PR 2−0
for uniform acceleration = 25 kmh−1
(a) We know that, (c) Since, the two graphs intersect at point Q, so A will
Acceleration = Slope of velocity-time graph AB catch B after 2 h and at a distance of 150 km from
DB v − u
or a= = the origin.
AD t QS 150 − 0
(d) Speed of A = = = 75 kmh−1
or v − u = at or v = u + at OS 2−0
This proves the first equation of motion. ∴ Difference in speeds = 75 − 25 = 50 kmh−1
70 Objective Physics Vol. 1

X Example 3.16 A car accelerates from rest at a Sol. Change in velocity = area under a-t graph
constant rate α for some time, after which it 1
Hence, vf − vi = (4) (4) = 8 m/s
decelerates at a constant rate β, to come to rest. If the 2
total time elapsed is t second evaluate (a) the maximum ∴ v f = v i + 8 = (2 + 8) m/s = 10 m/s
velocity reached and (b) the total distance travelled.
X Example 3.18 A rocket is fired vertically upwards
Sol. (a) Let the car accelerates for timet 1 and decelerates for time with a net acceleration of 4 m/s 2 and initial velocity
t 2 . Then, zero. After 5 s its fuel is finished and it decelerates with
t = t1 + t 2 …(i) g. At the highest point its velocity becomes zero. Then,
v it accelerates downwards with acceleration g and
v max A return back to ground. Plot velocity-time and
displacement-time graphs for the complete journey.
(Take g =10 m/s 2 )
O B t Sol. In the graphs,
t1 t2
v (m/s)
Fig. 3.20
and corresponding velocity-time graph will be as
shown in Fig. 3.20.
From the graph,
v
α = slope of line OA = max
t1 A
20
vmax
or t1 = …(ii) B C
α t (s)
v O 5 7 10.7
and β = − slope of line AB = max
t2
vmax
or t2 = …(iii) Fig. 3.22
β
From Eqs. (i), (ii) and (iii), we get v A = at OA = (4) (5) = 20 m/s
vmax v α + β v B = 0= v A − gt AB
+ max = t or vmax   =t
α β  αβ  v 20
∴ t AB = A = = 2s
αβt g 10
∴ vmax =
α+β ∴ t OAB = (5 + 2 ) s = 7 s
(b) Total distance = displacement = area under v -t graph Now, sOAB = area under v-t graph between 0 to 7 s
1 1 αβt 1
= × t × vmax = × t × = (7 ) (20) = 70 m
2 2 α+β 2
 αβt 2  s (m)
1
or Distance =  
2 α + β
B
70
/ This problem can also be solved by using equations of
motion (v = u + at etc.). Try it yourself. 50 A

X Example 3.17 Acceleration-time graph of a


particle moving in a straight line is shown in C
t (s)
Fig. 3.21. Velocity of particle at time t = 0 is 2 m/s. Find O 5 7 10.7
velocity at the end of fourth second. Fig. 3.23
a (m/s ) 2 1 2
Now, |sOAB| = |sBC| = gt BC
2
4 1 2
∴ 70 = (10) t BC
2
∴ t BC = 14 = 37 . s
t (s) ∴ t OABC = 7 + 37
. = 107 . s
O 2 4 Also sOA = area under v-t graph between OA
1
Fig. 3.21 = (5)(20) = 50 m
2
Motion in One Dimension 71

X Example 3.19 Velocity-time graph of a particle Similarly, relative acceleration of A with respect to B is
moving in a straight line is shown in Fig. 3.24. a AB = a A − a B
v (m/s) If it is a one dimensional motion we can treat the vectors
C
as scalars just by assigning the positive sign to one direction
20 and negative to the other. So, in case of a one dimensional
A B motion the above equations can be written as
10
D v AB = v A − v B and a AB = a A − a B
t (s)
O 2 4 6 8 Further, we can see that
Fig. 3.24 v AB = − v BA or a BA = − a AB
Plot the corresponding displacement-time graph of
the particle, if at time t = 0, displacement s = 0. Extra Edge
■ By the concept of relative motion we convert a two body
Sol. Displacement = Area under velocity-time graph motion into one body motion. For example, if A and B,
1 two bodies are in motion, then v AB (= v A − vB ) means B
Hence, sOA = × 2 × 10 = 10 m
2 is assumed to be at rest and A is moving with velocity
sAB = 2 × 10 = 20 m or sOAB = 10 + 20 = 30 m v AB . Similarly in vBA , body A is assumed to be at rest
1 and B moving with velocity vBA .
sBC = × 2(10 + 20) = 30 m
2
or sOABC = 30 + 30 = 60 m X Example 3.20 Seeta is moving due east with a
1
and sCD = × 2 × 20 = 20 m velocity of 1 m/s and Geeta is moving due west with a
2 velocity of 2 m/s. What is the velocity of Seeta with
or sOABCD = 60 + 20 = 80 m
respect to Geeta?
s (m)
Sol. It is a one dimensional motion. So, let us choose the east
80
direction as positive and the west as negative. Now, given that
60 vS = velocity of Seeta = 1 m/s
and vG = velocity of Geeta = −2 m/s
30 Thus, vSG = velocity of Seeta with respect to Geeta
= vS − vG = 1 − (−2 ) = 3 m/s
10 Hence, velocity of Seeta with respect to Geeta is 3 m/s due
t (s) east.
2 4 6 8
Fig. 3.25
Between 0 to 2 s and 4 to 6 s motion is accelerated, hence
X Example 3.21 Car A has an acceleration of
displacement-time graph is a parabola. Between 2 to 4 s, 2 m/ s 2 due east and car B, 4 m/s 2 due north. What is
motion is uniform, so displacement-time graph will be a the acceleration of car B with respect to car A?
straight line. Between 6 to 8 s, motion is decelerated hence
displacement-time graph is again a parabola but inverted in Sol. It is a two dimensional motion. Therefore,
shape. At the end of 8 s velocity is zero, therefore, slope of
N
displacement- time graph should be zero. The
corresponding graph is shown in Fig. 3.25.

W E
3.8 Relative Motion
The word ‘relative’ is a very general term, which can be
applied to physical, non-physical, scalar or vector S
quantities. For example, my height is five feet and six inches Fig. 3.26
while my wife’s height is five feet and four inches. If I ask a BA = acceleration of car B with respect to car A
you how high I am relative to my wife, your answer will be = aB − aA
two inches. What you did? You simply subtracted my wife’s Here a B = acceleration of car B
height from my height. The same concept is applied = 4 m/s 2 (due north)
everywhere, whether it is a relative velocity, relative and a A = acceleration of car A
acceleration or anything else. So, from the above discussion = 2 m/s 2 (due east)
we may now conclude that relative velocity of A with respect |a BA| = (4)2 + (2 )2 = 2 5 m/s 2
to B (written as vAB ) is 4
and α = tan−1   = tan−1 (2 )
v AB = v A − v B 2
72 Objective Physics Vol. 1

Thus, a BA is 2 5 m/s 2 at an angle of α = tan−1 (2) from


X Example 3.23 Car A and car B start moving
west towards north.
aBA simultaneously in the same direction along the line
aB = 4 m/s 2
joining them. Car A with a constant acceleration
a = 4 m/s 2 , while car B moves with a constant velocity
α v =1 m/s. At time t = 0, car A is 10 m behind car B. Find
the time when car A overtakes car B.
– aA = 2 m/s 2
Fig. 3.27 Sol. Given, u A = 0, u B = 1 m/s,
aA = 4 m/s 2 and aB = 0
X Example 3.22 Two particles A and B are moving
along x-axis. Their x-t graphs are shown in figure Assuming car B to be at rest, we have
below. Find velocity of A with respect to B. u AB = u A − u B = 0 − 1 = − 1 m/s
x (m) B aAB = aA − aB = 4 − 0 = 4 m/s 2
A Now, the problem can be assumed in simplified form as
follows

a = 4 m/s 2 v = 1 m/s
30°

60° A 10 m
t (s) B
O
Fig. 3.28 +ve

Sol. Velocity of A, Fig. 3.29


v A = Slope of x-t graph of A
= tan30° = 0.58 m /s Substituting the proper values in equation
1
Velocity of B, s = ut + at 2
2
v B = Slope of x-t graph of B 1
= tan 60° = 1.732 m/s We get 10 = − t + (4) (t 2)
2
Velocity of A with respect to B
or 2 t 2 − t − 10 = 0
v AB = v A − v B = 0.58 − 1.732
1± 1 + 80 1±
81 1 ± 9
= −1.152 m/s or t = = =
4 4 4
Negative sign indicates that A appears to be moving or t = 2.5 s and −2s
towards negative x-direction (to B) with velocity 1.152 m /s Ignoring the negative value, the desired time is 2.5 s.
Thus, a BA is 2 5 m / s 2 at an angle of α = tan −1 (2) / The above problem can also be solved without using the
concept of relative motion as under.
from west towards north.
At the time when A overtakes B,
The topic ‘relative motion’ is very useful in two and s A = sB + 10
three dimensional motion. Questions based on relative 1
motion are usually of following four types: ∴ × 4 × t 2 = 1 × t + 10
2
(a) Minimum distance between two bodies in motion or 2t 2 − t − 10 = 0
(b) River-boat problems Which on solving gives t = 2.5 s and –2 s, the same as we
(c) Aircraft-wind problems found above.
As per my opinion, this approach (by taking absolute values)
(d) Rain problems is more suitable in case of two body problem in one
dimensional motion. Let us see one more example in support
Minimum Distance between Two of it.

Bodies in Motion X Example 3.24 An open lift is moving upwards


When two bodies are in motion, the questions like, the with velocity 10 m/s. It has an upward acceleration of
minimum distance between them or the time when one body 2 m/s 2 . A ball is projected upwards with velocity 20 m/s
overtakes the other can be solved easily by the principle of relative to ground. Find
relative motion. In these type of problems, one body is
assumed to be at rest and the relative motion of the other (a) time when ball again meets the lift.
body is considered. By assuming so two body problem is (b) displacement of lift and ball at that instant.
converted into one body problem and the solution becomes (c) distance travelled by the ball upto that instant.
easy. Following example will illustrate the statement. (Take g =10 m/ s 2 )
Motion in One Dimension 73

Thus, the given problem can be simplified as


Sol. (a) At the time when ball again meets the lift,
vB = 20 km/h vBA = 20√2 km/h
sL = sB
1 1
∴ 10 t + × 2 × t 2 = 20 t − × 10 t 2
2 2
Solving this equation, we get 45°
2
2 m/s 10 m/s – vA = 20 km/h
20 m/s
+ve Fig. 3.32
Ball A is at rest and B is moving with v BA in the direction shown
in Fig. 3.33.
10 m/s2 A
L → Lift B → Ball C
Fig. 3.30
vBA
5 45°
t = 0 and t = s
3 B
5
∴ Ball will again meet the lift after s.
3
(b) At this instant
2
5 1 5
sL = sB = 10 × + × 2 ×  
Fig. 3.33
3 2  3 Therefore, the minimum distance between the two is
1 
=
175
m = 19.4 m smin = AC = AB sin 45° = 10   km
9  2
(c) For the ball u ↑ ↓ a. Therefore, we will first find t 0 , the = 5 2 km and the desired time is
time when its velocity becomes zero. BC 5 2
t = = (BC = AC = 5 2 km)
t 0 =u  = 20 = 2 s |v BA| 20 2
 a  10 1
5 = h = 15 min
As t  = s < t 0 , distance and displacement are 4
 3 
equal. River-Boat Problems
or d = 19.4 m
Concept of relative motion is more useful in two body
In river-boat problems, we come across the following
problem in two (or three) dimensional motion. This three terms :
can be understood by the following example. B

X Example 3.25 Two ships A and B are 10 km apart y


vbr w
on a line running south to north. Ship A farther north is
θ x
streaming west at 20 km/h and ship B is streaming
north at 20 km/h. What is their distance of closest A vr
approach and how long do they take to reach it?
Fig. 3.34
Sol. Ships A and B are moving with same speed 20 km/h in the
directions shown in figure. It is a two dimensional, two body
v r = absolute velocity of river,
problem with zero acceleration. Let us find v BA v br = velocity of boatman with respect to river
vA N or velocity of boatman in still water
A
E and v b = absolute velocity of boatman.
vB Here, it is important to note that v br is the velocity of
B boatman with which he steers and v b is the actual velocity of
AB = 10 km boatman relative to ground.
Further, v b = v br + v r
Fig. 3.31
Now, let us derive some standard results and their
special cases.
v BA = v B − v A A boatman starts from point A on one bank of a river
Here, |v BA| = (20)2 + (20)2 with velocity v br in the direction shown in Fig. 3.34. River
= 20 2 km/h is flowing along positive x-direction with velocity v r .Width
i.e. v BA is 20 2 km/h at an angle of 45° from east towards
of the river is w. Then,
north. v b = v r + v br
74 Objective Physics Vol. 1

Therefore, v bx = v rx + v brx = v r − v br sin θ vr


or sin θ =
and v by = v ry + v bry v br
= 0 + v br cos θ v 
= v br cos θ or θ = sin −1  r 
 v br 
Now, time taken by the boatman to cross the river is
Hence, to reach point B the boatman should row at an
w w
t= = v 
v by v br cos θ angle θ = sin −1  r  upstream from AB.
 v br 
w
or t= …(i) Further, since sin θ >| 1.
v br cos θ
Further, displacement along x-axis when he reaches on So, if v r ≥ v br , the boatman can never reach at point B.
the other bank (also called drift) is Because if v r = v br , sin θ = 1 or θ = 90° and it is just
w impossible to reach at B if θ = 90°. Moreover it can be
x = v bx t = ( v r − v br sin θ ) seen that v b = 0 if v r = v br and θ = 90°. Similarly, if
v br cos θ
w v r > v br , sin θ >1, i.e. no such angle exists. Practically
or x = ( v r − v br sin θ ) …(ii) it can be realized in this manner that it is not possible
v br cos θ
to reach at B, if river velocity ( v r ) is too high.
Two special cases are X Example 3.26 A man can row a boat with 4 km/h
(i) Condition when the boatman crosses the river in in still water. If he is crossing a river where the current
shortest interval of time is 2 km/h.
B
(a) In what direction will his boat be headed, if he wants
to reach a point on the other bank, directly opposite to
vbr
starting point?
(b) If width of the river is 4 km, how long will the man
take to cross the river, with the condition in part (a)?
A vr (c) In what direction should he head the boat, if he wants
Fig. 3.35 to cross the river in shortest time and what is this
minimum time?
From Eq. (i), we can see that time (t) will be minimum
(d) How long will it take him to row 2 km up the stream
when θ = 0° , i.e. the boatman should steer his boat
perpendicular to the river current. and then back to his starting point?
w Sol. (a) Given, that v br = 4 km/h and v r = 2 km/h
Also, t min = as cos θ =1
v br  v  2
∴ θ = sin−1  r  = sin−1  
 v br   4
(ii) Condition when the boatman wants to reach point
1
B, i.e. at a point just opposite from where he started = sin−1   = 30°
2
In this case, the drift (x) should be zero.
Hence, to reach the point directly opposite to starting
B point he should head the boat at an angle of 30° with
AB or 90° + 30° = 120° with the river flow.
(b) Time taken by the boatman to cross the river
vbr
w = width of river = 4 km
θ
v br = 4 km/h and θ = 30°
4 2
A vr ∴ t = = h
4 cos 30° 3
Fig. 3.36
(c) For shortest time θ = 0°
w 4
∴ x =0 and t min = = = 1h
v br cos 0° 4
w
or ( v r − v br sin θ ) =0
v br cos θ Hence, he should head his boat perpendicular to the
river current for crossing the river in shortest time
or v r = v br sin θ and this shortest time is 1 h.
Motion in One Dimension 75

(d) t = t CD + t DC sin 105°


or |v a| = × (400) km/h
sin 45°
 cos 15° 
v br – vr v br + vr =  (400) km/h
 sin 45° 
D C D C 0.9659 
=   (400) km/h
 0.707 
Fig. 3.37
= 546.47 km/h
CD DC ∴
or t = + The time of journey from A to B is
v br − v r v br + v r AB 1000
t = = h
2 2 1 4 |v a| 546.47
= + = 1+ = h
4−2 4+ 2 3 3 t = 1.83 h

Aircraft Wind Problems Rain Problems


This is similar to river boat problem. The only In these type of problems, we again come across three
difference is that v br is replaced by v aw (velocity of aircraft terms v r , v m and v rm , Here
with respect to wind or velocity of aircraft in still air), v r is v r = velocity of rain
replaced by v w (velocity of wind) and v r is replaced by v a v m = velocity of man (it may be velocity of cyclist or
(absolute velocity of aircraft). Further, v a = v aw + v w . The velocity of motorist also).
following example will illustrate the theory. and v rm = velocity of rain with respect to man.
Here, v rm is the velocity of rain which appears to the
X Example 3.27 An aircraft flies at 400 km/h in still man. Now, let us take one example of this.
air. A wind of 200 2 km/h is blowing from the south.
The pilot wishes to travel from A to a point B X Example 3.28 To a man walking at the rate of
north-east of A. Find the direction he must steer and 3 km/h the rain appears to fall vertically. When he
time of his journey if AB =1000 km. increases his speed to 6 km/h it appears to meet him at
an angle of 45° with vertical. Find the speed of rain.
Sol. Given that v w = 200 2 km/h.
Sol. Let $i and $j be the unit vectors in horizontal and vertical
N directions respectively.
B
Vertical ( ^j )
45° v = 200√2 km/h
va w

C
45° α
A vaw = 400 km/h ^
E Horizontal ( i )
Fig. 3.39
Fig. 3.38
Let velocity of rain
v aw = 400 km/h and v a should be along AB or in north-east v r = a $i + b$j …(i)
direction. Thus, the direction of v aw should be such as the
resultant of v w and v aw is along AB or in north-east Then speed of rain will be
direction. |v r| = a2 + b 2 …(ii)
Let v aw makes an angle α with AB as shown in Fig. 3.38. In the first case, v m = velocity of man = 3$i
Applying sine law in triangle ABC, we get
∴ v rm = v r − v = (a − 3)$i + b$j
AC BC
=
sin 45° sin α It seems to be in vertical direction. Hence,
 200 2  1 a − 3 = 0 or a = 3
1
sin α = 
BC 
or  sin 45° =   = In the second case v rm = 6$i
 AC   400  2 2
∴ v = (a − 6)$i + b$j = − 3i$ + b$j
∴ α = 30° rm

Therefore, the pilot should steer in a direction at an angle This seems to be at 45° with vertical.
of (45° + α ) or 75° from north towards east. Hence,|b| = 3
|v a| 400 Therefore, from Eq. (ii) speed of rain is
Further, =
sin (180° − 45° − 30° ) sin 45° |v r| = (3)2 + (3)2 = 3 2 km/h
76 Objective Physics Vol. 1

Extra Knowledge Points


■ If y (may be velocity, acceleration etc) is a function of Exercise A particle is thrown upwards with velocity
time or y = f (t ) and we want to find the average value of u ( > 20 m /s). Prove that distance travelled in last 2 s
y between a time interval of t1 and t 2 . Then, is 20 m.
< y >t1 to t 2 = average value of y between t1 and t 2 ■ Suppose we have given velocity-time (v-t) graph. We
t2 t2 want to plot corresponding displacement-time (s-t)
∫t f (t ) dt ∫t f (t ) dt graph then values of displacements at different times
= 1
or < y >t1 to t 2 = 1
can be found just by adding the corresponding areas
t 2 − t1 t 2 − t1
under v-t graph.
But if f (t ) is a linear function of t, then ■ The modulus of velocity is really the speed or | v | = v
y + yi
y av = f ■ Rate of change of velocity is acceleration, while rate of
2
change of speed is the tangential acceleration
Here, yf = final value of y and (component of acceleration along velocity). Thus,
yi = initial value of y dv dv d | v|
= a , while = = at
At the same time, we should not forget that dt dt dt
total displacement Angle between velocity vector and acceleration vector
v av = a decides whether the speed of particle is increasing,
total time
decreasing or constant.
change in velocity
and a av = a
total time
■ Example In one dimensional uniformly accelerated
motion, find average velocity from t = 0 to t = t .
Solution We can solve this problem by three methods.
Method 1. v = u + at θ
v
t
∫0 (u + at ) dt Speed increases if 0° ≤ θ < 90°
∴ < v >0 − t =
t −0 Speed decreases if 90° < θ ≤ 180°
1 Speed is constant if θ = 90°
= u + at
2 The angle θ between v and a can be obtained by the
Method 2. Since, v is a linear function of time, we can relation,
write,  v ⋅ a
θ = cos −1  
v + vi (u + at ) + u 1  va 
v av = f = = u + at
2 2 2 Exercise Prove that speed of a particle increases if dot
Total displacement product of v and a is positive, speed decreases if the
Method 3. v av =
Total time dot product is negative and speed remains constant, if
1 2 dot product is zero.
ut + at
2 1 ■ In the x-t graph of a particle moving along x-axis shown
= = u + at
t 2 in figure average velocity of the particle between time
A particle is thrown upwards with velocity u. Suppose it interval t1 and t 2 is :
takes time t to reach its highest point, then distance x
travelled in last second is independent of u.
u=0 X2 P2
last one second
P1
1s X1

⇒ O
t
t1 t2
u

P2

(x 2 – x 1 )
This is because this distance is equal to the distance
travelled in first second of a freely falling object. Thus,
θ
1 1 P1 M
s = g × (1)2 = × 10 × 1 = 5 m (t2–t1)
2 2
Motion in One Dimension 77

∆s ∆x x f − x i x 2 − x 1
v av = = = = = Slope of line Reaction time
∆t ∆t t 2 − t1 t 2 − t1 When a situation demands our immediate action, it
P1 P2 = tan θ takes some time before we really respond. Reaction
■ From displacement time graph time is the time a person takes to observe, think and act.
S For example, if a person is driving and suddenly a boy
we can determine the sign of
velocity its nature (whether appears on the road, then the time elapsed before he
speed of the particle is slams the brakes of the car is the reaction time.
increasing, decreasing or t2 > t1 ■ If a particle is speeding up, acceleration is in the
constant) and sign of direction of velocity; if its speed is decreasing,
acceleration. For example t acceleration is in the direction opposite to that of the
t1 t2
Slope of S-t graph = velocity. velocity. This statement is independent of the choice of
Slopes at t1 and t 2 both are positive. Hence, velocities at the origin and the axis.
t1 and t 2 are also positive. Further, ■ The sign of acceleration does not tell us whether the
Slope at t 2 > Slope at t1 particle's speed is increasing or decreasing. Ths sign of
acceleration depends on the choice of the positive
∴ Velocity at t 2 > velocity at t1 or,
direction of the axis. For example, if the vertically
speed of the particle is increasing, since speed is upward direction is chosen to be the positive direction,
increasing, so acceleration should have the same sign
the acceleration due to gravity is negative. If a particle is
as that of velocity. Or, acceleration is also positive.
falling under gravity, this acceleration, though negative,
Exercise What are the signs of accelerations in the results in increase in speed. For a particle thrown
three position-time graphs shown in figure. upward, the same negative acceleration (of gravity)
Solution. (a) positive (b) negative (c) zero results in decrease in speed.
x x x ■ The zero velocity of a particle at any instant does not
necessarily imply zero acceleration at that instant. A
particle may be momentarily at rest and yet have
non-zero acceleration. For example, a particle thrown
t t t up has zero velocity at its highest point but the
O O O
acceleration at that instant continues to be the
(a) (b) (c)
+ acceleration due to gravity.
Chapter Summary with Formulae
■ Basic Definition
(i) Displacement a (iv) Equations of integration
s = rf − ri
= ( x f − x i ) $i + ( yf − yi ) $j + ( z f − z i ) k$
∫ ds = ∫ vdt, ∫ dv = ∫ adt, ∫ vdv = ∫ ads
(ii) Distance = actual path length In first integration equation v should be either a constant
or function of t . In second equation 'a' should be either a
(iii) Average velocity
total displacement s constant or a function of t. Similarly, in third equation 'a'
= = should be either a constant or function of s.
total time t
(iv) Average speed
■ Two or Three Dimensional Motion with Uniform
total distance d Acceleration
= =
total time t (i) v = u + a t
1
(v) Average acceleration (ii) s = ut + a t2
change in velocity vf − vi 2
= =
time t (iii) v ⋅ v = u ⋅ u + 2a ⋅ s
(vi) Instantaneous velocity ■ Two or Three Dimensional Motion with Non-uniform
ds dr Acceleration
= or
dt dt ds dr
(i) v = or
(vii) Instantaneous acceleration dt dt
= rate of change of velocity (ii) a =
dv
dv d2 s d2 r dt
= = =
dt dt2 dt2 (iii) ∫ dv = ∫ a ⋅ dt
In One Dimensional Motion (iv) ∫ ds = ∫ v ⋅ dt

(viii) Instantaneous velocity


ds dx ■ Graphs
v= or
dt dt (i) Uniform motion v = constant, a = 0, s = vt
= slope of v-t graph
a
■ One Dimensional Motion with Uniform Acceleration
(i) v = u + at
1
(ii) s = ut + at2
2 t
1
(iii) s = s0 + ut + at2 v v
2
(iv) v2 = u2 + 2as
t
(v) st = displacement (not distance) in t th second
a t
= (u + at ) −
2 s s
● While using above equations, take a sign convention and
substitute all vector quantities ( v,u, a, s and st ) with sign.
1
● In equation s = ut + at2 , s is the displacement measured
2 t t
from the starting point.
s s
● st is the displacement between (t − 1) and t seconds.
■ One Dimensional Motion with Non-uniform
Acceleration t
(i) s-t → v-t → a-t → Differentiation t
(ii) a-t → v-t → s-t → Integration
(iii) Equations of differentiation
v=
ds
and =
dv
= v.
dv Since, a = 0, therefore slope of v-t graph = 0. Further,
dt dt ds v = constant, therefore, slope of s-t graph = constant.
s s
(ii) Uniformly accelerated or retarded motion
a = constant, v = u ± at,
1
s = ut ± at2
2 t t
a
Since a = constant, therefore slope of v-t
a
graph = constant.
Further, v is increasing or decreasing, therefore slope of
t t s-t graph should either increase or decrease.
(iii) Slope of s-t graph = velocity
Slope of v-t graph = acceleration
v v Area under v-t graph = displacement and
Area under a-t graph = change in velocity
■ Relative Motion
t t (i) vAB = velocity of A with respect to B = vA − vB
v s (ii) a AB = acceleration of A with respect to B = a A − a B .
or In one dimensional motion take a sign convention.
In this case,
(iii) vAB = vA − vB
t t (iv) aAB = aA − aB
Additional Examples
Example 1. A farmer has to go 500 m due north, Adding Eqs. (i) and (ii), we have
a + a2 
400 m due east and 200 m due south to reach his field. v 22 = u 2 + 2  1  (2 s ) K(iii)
If he takes 20 min to reach the field,  2 
(a) what distance he has to walk to reach the field ? In the second case,
a + a2 
(b) what is the displacement from his house to the v2 = u2 + 2  1  (2 s ) K(iv)
field ?  2 
(c) what is the average speed of farmer during the From Eqs. (iii) and (iv), we can see that
walk ? v2 = v
(d) what is the average velocity of farmer during the Example 3. In a car race, car A takes a time t less
walk ?
than car B at the finish and passes the finishing point
Sol. (a) Distance = AB + BC + CD with speed v more than that of the car B. Assuming
= (500 + 400 + 200 ) that both the cars starts from rest and travel with
= 1100 m constant acceleration a1 and a 2 respectively. Show that
(b) Displacement = AD = ( AB − CD ) 2 + BC 2 v = a1 a 2 t.
= (500 − 200 ) 2 + (400 ) 2 Sol. Let A takes t 1 second, then according to the given problem B
= 500 m will take (t 1 + t ) seconds. Further, let v1 be the velocity of B at
N finishing point, then velocity of A will be (v1 + v). Writing
B C equations of motion for A and B.
v1 + v = a1t 1 K(i)
v1 = a 2 (t 1 + t ) K(ii)
D W E From these two equations, we get
v = (a1 − a 2 ) t 1 − a 2 t K(iii)
Total distance travelled by both the cars is equal.
A or s A = sB
S 1 2 1 a2 t
Total distance or a1t 1 = a 2 (t 1 + t ) 2 or t 1 =
(c) Average speed = 2 2 a1 − a 2
Total time
Substituting this value of t 1 in Eq. (iii), we get the
1100
= = 55m/min desired result.
20 or v = ( a1a 2 ) t
AD
(d) Average velocity =
t Example 4. A particle is moving with a velocity of
500
= = 25m/min (along AD) v = (3 + 6t + 9t 2 ) cm/s. Find out
20
(a) the acceleration of the particle at t = 3 s.
Example 2. A particle starts with an initial velocity (b) the displacement of the particle in the interval t = 5 s
and passes successively over the two halves of a given to t = 8 s.
distance with accelerations a1 and a 2 respectively. Sol. (a) Acceleration of particle,
Show that the final velocity is the same as if the whole dv
distance is covered with a uniform acceleration a= = (6 + 18 t ) cm/s 2
dt
( a1 + a 2 ) At t = 3 s, a = (6 + 18 × 3) cm/s 2 = 60cm/s 2

2 (b) Given, v = (3 + 6t + 9t 2 )
Sol. In the first case, ds
or = (3 + 6t + 9t 2 )
u v1 v2 u a + a2 v dt
→ → → → 2s, 1 → or ds = (3 + 6t + 9t 2 ) dt
s, a1 s, a2 2
s 8
First case Second case ∴ ∫ 0 ds = ∫5 (3 + 6t + 9t 2 ) dt
v12 = u + 2a1 s
2
K(i)
∴ s = [3t + 3t 2 + 3t 3 ] 5
8
v 22 = v12 + 2a 2 s K(ii) or s = 1287 cm
Motion in One Dimension 81

Example 5. The motion of a particle along a straight Example 7. From an elevated point A, a stone is
line is described by the function x = (2t − 3) 2 , where x projected vertically upwards. When the stone reaches a
is in metre and t is in second. distance h below A, its velocity is double of what it was
(a) Find the position, velocity and acceleration at at a height h above A. Show that the greatest height
5
t = 2 s. attained by the stone is h.
(b) Find velocity of the particle at origin. 3
Sol. (a) Position, x = (2t − 3) 2 Sol. Let u be the velocity with which the stone is projected
vertically upwards.
dx
Velocity, v = = 4 (2t − 3)m/s Given that, v− h = 2 vh
dt
or (v − h ) 2 = 4 v h2
and acceleration,
dv ∴ u 2 − 2 g (− h ) = 4 (u 2 − 2 gh )
a= = 8m/s 2
dt 10 gh
∴ u2 =
At t = 2 s, 3
x = (2 × 2 − 3) 2 = 1.0 m u 2 5h
Now, h max = =
v = 4 (2 × 2 − 3) = 4 m/s 2g 3
and a = 8m/s 2 Example 8. A man crosses a river in a boat. If he
(b) At origin, x = 0 cross the river in minimum time he takes 10 min with a
or (2 t − 3) = 0 drift 120 m. If he crosses the river taking shortest path,
∴ v=4×0=0 he takes 12.5 min, find
(a) width of the river.
Example 6. An open elevator is ascending with zero
(b) velocity of the boat with respect to water.
acceleration. The speed v = 10 m/s. A ball is thrown
vertically up by a boy when he is at a height h = 10 m (c) speed of the current.
from the ground. The velocity of projection is v = 30 m/s Sol. Let v r = velocity of river,
with respect to elevator. Find v br = velocity of river in still water and
(a) the maximum height attained by the ball. w = width of river
(b) the time taken by the ball to meet the elevator w
Given, t min = 10 min or = 10 K(i)
again. v br
(c) time taken by the ball to reach the ground after Drift in this case will be,
crossing the elevator. A
Sol. (a) Absolute velocity of ball = 40 m/s (upwards)
∴ h max = hi + h f v br w
Here, hi = initial height = 10 m
and h f = further height attained by ball A
For minimum time
u 2 (40) 2
= = = 80 m x = v r t ⇒ ∴ 120 = 10 v r K(ii)
2 g 2 × 10 Shortest path is taken when v b is along AB. In this case,
∴ h max = (10 + 80) m = 90 m v b = v br
2
− v r2
(b) The ball will meet the elevator again when displacement w w
of lift = displacement of ball Now, 12.5 = = K(iii)
1 vb 2
v br − v r2
or 10 × t = 40 × t − × 10 × t 2
2 vr B
or t =6s
(c) Let t 0 be the total time taken by the ball to reach the vbr vb
ground. Then,
1
− 10 = 40 × t 0 − × 10 × t 02 A
Shortest path
2
Therefore, time taken by the ball to reach the ground Solving these three equations, we get
after crossing the elevator = (t 0 − t ) = 2.24 s v br = 20 m/min, v r =12 m/min and w = 200 m.
82 Objective Physics Vol. 1

Example 9. A man wants to reach point B on the Example 10. The acceleration versus time graph of a
opposite bank of a river flowing at a speed as shown in particle moving along a straight line is shown in the
figure. What minimum speed relative to water should figure. Draw the respective velocity-time graph.
the man have so that he can reach point B ? In which (Assuming at t = 0, v = 0.)
direction should he swim?

B
a
(m/s2)
u 2
45°
0 2 4 6 t (s)
–2
–4
A

Sol. Let v be the speed of boatman in still water. Sol. From t = 0 to t = 2 s, a = + 2 m/s 2
∴ v = at = 2 t
B
or v - t graph is a straight line passing through origin
vb with slope 2 m/s 2 .
v y At the end of 2 s,
θ 45° v = 2 × 2 = 4 m/s
x From t = 2 to 4 s, a = 0.
A u
Hence, v = 4 m/s will remain constant.
Resultant of v and u should be along AB. Components of From t = 4 to 6 s, a = − 4 m/s 2 .
v b (absolute velocity of boatman) along x and y
directions are, Hence, v = u − at = 4 − 4t (with t = 0 at 4 s)
v x = u − v sin θ and v y = v cos θ v = 0 at t = 1 s or at 5 s from origin.
vy v cos θ At the end of 6 s (or t = 2 s) v = − 4 m/s.
Further, tan 45° = or 1 = Corresponding v - t graph is as shown below.
vx u − v sin θ v (m/s)
u u
∴ v= =
sin θ + cos θ 2 sin (θ + 45°) 4
v is minimum at,
6
θ + 45° = 90 ° 0
2 4 5
t (s)

or θ = 45°
u –4
and v min =
2
NCERT Selected Questions
Q 1. The position-time (x- t ) graphs for two children A If t1 is time taken to reach office then
and B returning from their school O to their homes P x
t1 =
and Q respectively are shown in figure. Choose the v1
correct entries in the brackets below : 2.5 1
∴ t1 = = h
x 5 2
= 30 min
Q
P
A B
2.5

x (km)
A
B 2.0

O t 1.5
C

(a) ( A / B ) lives closer to the school than ( B / A ) 1


(b) ( A / B ) starts from school earlier than ( B / A ) 0.5 9.30 am 5.06 pm
(c) ( A / B ) walks faster than ( B / A ) O 9 10 11 12 1 2 3 4 5
(d) ( A / B ) overtakes ( B / A ) on the road (once/twice) am Time (t) in hour pm
Sol. (a) It is clear from the graph that OQ > OP, so A lives closer
to the school than B. When she stays at her office from 9.30 am to 5.00 pm, then
she is stationary.
(b) The position-time (x- t ) graph of A starts from the origin,
On return journey, speed of auto, v2 = 25 km/h.
so x = 0, t = 0 for A while the x - t graph of B starts from
C which shows that B starts later than A after a time ∴If t2 is time taken by her in return journey from office to her
interval OC. So, A starts from school (O ) earlier than B. home, then
x 2.5 1
x t2 = = = h = 6 min
v2 25 10
Q
Thus, she reaches back to her home at 5.06 pm.
P
Q 3. A drunkard walking in a narrow lane takes 5 steps
D
x forward and 3 steps backward, followed again by
A
B
5 steps forward and 3 steps backward and so on.
Each step is 1 m long and requires 1 s. Plot the
O C x- t graph of his motion. Determine graphically and
t
otherwise how long the drunkard takes to fall in a pit
(c) The speed is represented by the magnitude of slope of 13 m away from the start.
the x- t graph. More steeper the graph, more will be the
Sol. The x- t graph of the drunkard is shown in figure.
speed. As, the x- t graph of B is steeper than the x- t
graph of A, so we conclude that B walks faster than A.
(37, 13)
(d) As the x- t graphs for A and B intersect each other at
(29, 11)
point D only and B starts from the school later, so B
over takes A on the road only once. (21, 9)
(13, 7)
Q 2. A woman starts from her home at 9.00 am, walks (32, 8)
(5, 5)
with a speed of 5 kmh −1 on a straight road upto her (24, 6)
x (m)
office 2.5 km away, stays at the office upto 5.00 pm (16, 4)
and returns home by an auto with a speed of
(8, 2) 37
25 kmh −1 . Choose suitable scales and plot the 0
t(s)
x- t graph of her motion.
Sol. x- t graph of the motion of woman is shown in figure, Length of each step = 1 m, time taken for each step = 1s.
∴Time taken to move by 5 steps = 5 s.
v1 = speed of woman while walking at 5 kmh −1
5 steps forward and 3 steps backward means that the net
x = distance between home and office = 2.5 km distance covered by him in first 8 steps is 2 m.
84 Objective Physics Vol. 1

Distance covered by him in first 16 steps or To find t , let us use the relation,
16 s = 2 + 2 = 4 m. v = u + at
Distance covered by the drunkard in first 24 s, i.e. 24 steps Here, a = − 3.06 ms− 2 , v = 0, u = 35 m /s
= 2 + 2 + 2 = 6m.
v − u 0 − 35
and distance covered in 32 steps i.e. 32 s = 8 m. ∴ t= = = 11.44 s
a − 3.06
∴ Distance covered in first 37 steps = 8 + 5 = 13 m.
∴ t = 11.44 s
Distance of the pit from the start = 13 m.
∴Total time taken by the drunkard to fall in the pit = 37 s. Q 6. Two trains A and B of length 400 m each are moving
on two parallel tracks with a uniform speed of
Q 4. A jet airplane travelling at a speed of 500 kmh −1 72 kmh −1 in the same direction, with A ahead of B.
ejects its products of combustion at the speed of
The driver of B decides to overtake A and
1500 kmh −1 relative to the jet plane. What is the accelerates by 1 ms −2 . If after 50 s, the guard of B
speed of the combustion with respect to an observer
just brushes past the driver of A, what was the
on the ground?
original distance between them?
Sol. Let v j , vg and v0 be the velocities of jet, ejected gases and
Sol. Given that
observer on the ground respectively. 5
Let jet be moving towards right (+ ve direction). uA = uB = 72 kmh −1 = 72 × = 20 ms− 1
18
∴Ejected gases will move towards left (− ve direction). 1 2
Using the relations, s = ut + at , we get
∴According to the statement 2
v j − v0 = v j = 500 kmh −1 …(i) 1
sB = uB t + at 2
As observer is at rest 2
1
vg − v j = − 1500 kmh −1 (given) …(ii) = 20 × 50 + × 1 × (50)2
2
∴Adding Eqs. (i) and (ii), we get the speed of combustion = 1000 + 1250 = 2250 m
products w.r.t. observer on the ground
Also let sA be the distance covered by the train A, then
(v j − v0 ) (vg − v j ) = vg − v0 = 500 + (− 1500)
sA = uA × t or sA = 20 × 50 = 1000 m
or vg − v0 = − 1000 kmh −1 Original distance between the two trains = sB − sA
− ve sign shows that relative velocity of the ejected gases = 2250 − 1000
w.r.t. observer is towards left or in a direction opposite to the = 1250 m
motion of the jet plane.
Q 7. On a two lane road, car A is travelling with a speed
Q 5. A car moving along a straight highway with speed of 36 kmh −1 . Two cars B and C approach car A in
of 126 kmh −1 is brought to a stop within a distance opposite directions with a speed of 54 kmh −1 each.
of 200 m. What is the retardation of the car At a certain instant, when the distance AB is equal
(assumed uniform) and how long does it take for the to AC, both being 1km, B decides to overtake A
car to stop? before C does. What minimum acceleration of car B
Sol. Initial velocity of car, is required of avoid an accident?
5
u = 126 × ms−1 = 35 ms−1 …(i) 1 km 1 km
18
As the car finally comes to rest, v = 0
Distance covered, s = 200 m vA
A Two lane
Using the equation v − u = 2 as
2 2
road
vB vC
v 2 − u2 B C
∴ a= …(ii)
2s
Putting the values from Eq. (i) in Eq. (ii), we get
Sol. Speed of car A,
0 − (35)2
a= 5
2 × 200 vA = 36 × = 10 ms−1
18
35 × 35 − 49 −2
= = ms Let vB and vC be the speeds of cars B and C
400 15
∴ vB = vC = 54 kmh −1
= − 3.06 ms− 2 5
− ve sign shows that car is retarded. = 54 × = 15 ms−1 (given)
18
Motion in One Dimension 85

∴ Relative velocity of car B w.r.t. car A is given by Case II Relative speed of the bus coming from town B to A
vBA = vB − vA w.r.t. cyclist = (vb + 20) kmh −1.
= 15 − 10 = 5 ms−1 (Q Cyclist is moving from A to B ).
Since, the bus goes past the cyclist after every 6 min.
Also magnitude of relative velocity of car C w.r.t. car A is
∴Distance covered by the bus w.r.t. the cyclist
given by,
6
vCA = vC − (− vA ) d3 = (vb + 20) × km …(iv)
60
= 15 + 10 = 25 ms−1 Also distance covered by bus in T min is
AB = AC = 1 km (given) = 1000 m T
d4 = v b × km …(v)
Between cars A and C 60
s AC 1000 ∴Equating Eqs. (iv) and (v), we get
t= = = = 40 s
u vCA 25 6 T
(vb + 20) × = vb ×
60 60
Let a = acceleration of car B for t = 40 s
T
so it will cover 1000m in 40 s. or vb + 20 = vb × …(vi)
1 6
∴ Using the relation, s = ut + at 2 Dividing Eq. (vi) by Eq. (iii), we get
2
vb + 20
1 =3
We get, AB = vBAt + at 2 vb − 20
2
1 or vb + 20 = 3 vb − 60
1000 = 5 × 40 + a × (40)2 or 2vb = 80 or vb = 40 kmh −1
2
1600 Putting the value of vb in Eq. (iii), we get
= 200 + a × T
2 40 − 20 = 40 ×
800 a = 800 18
T
a = 1 ms−2 or 20 = 40 ×
18
18
Q 8. Two towns A and B are connected by a regular bus or T = 20 × = 9 min
40
service with a bus leaving in either direction every
∴ vb = 40 kmh −1,
T min. A man cycling with a speed of 20 kmh −1 in
the direction A to B notices that a bus goes past him T = 9 min
every 18 min in the direction of his motion and every Q 9. A player throws a ball upwards with an initial speed
6 min in opposite direction. What is the period T of
of 29.4 ms −1 .
the bus service and with what speed
(assumed constant) do the buses play on the road? (a) What is the direction of acceleration during the
upward motion of the ball?
Sol. Let the speed of each bus = vb kmh −1 (b) What are the velocity and acceleration of the ball
and speed of cyclist = vc = 20 kmh −1 at the highest point of its motion?
Case I Relative speed of the buses playing in the direction of (c) Choose the x = 0 m and t 0 = 0 s to be the location
motion of cyclist i.e. from A to and time of the ball at its highest point, vertically
downward direction to the positive direction of
B = vb − vc = (vb − 20) kmh −1
x-axis, and give the signs, velocity and
 18  acceleration of the ball during its upward and
Since, the bus goes past the cyclist every 18 min  = h ,
 60  downward motion.
∴ Distance covered by the bus w.r.t. the cyclist Sol. (a) Under gravity The direction of acceleration due to
18 gravity is always vertically downwards.
d1 = (vb − 20) × km …(i)
60 (b) At the highest point of its motion, its velocity becomes
Since, the bus leaves after every T min, so the distance zero and the acceleration is equal to the acceleration due
covered by the bus in T min is given by to gravity = 9.8 ms−2 in vertically downward direction.
T (c) When the highest point is chosen as the location for
d2 = vb × …(ii)
60 x = 0 and t = 0 and vertically downward direction to be
∴From Eqs. (i) and (ii), we get the positive direction of x-axis.
18 T During upward motion, sign of velocity is negative
(vb − 20) × = vb × and the sign of acceleration positive.
60 60
T During downward motion, sign of velocity is positive
or vb − 20 = vb × …(iii)
18 and the sign of acceleration is also positive.
86 Objective Physics Vol. 1

Q 10. Read each statement below carefully and state with (ii) 0 to 50 min interval
reasons and examples, if it is true or false. A particle Speed of return journey = 7.5 kmh −1
in one-dimensional motion Distance to be covered to reach home = 2.5 km
(a) with zero speed at an instant may have non-zero Distance
acceleration at that instant. ∴ Time taken in return journey =
Speed
(b) with zero speed may have non-zero velocity.
2.5 1
(c) with constant speed must have zero acceleration. = = h
7.5 3
(d) with positive value of acceleration must be = 20 min.
speeding up.
∴ Total time taken for the journey = 30 + 20 = 50 min
Sol. (a) True, example : if the ball is thrown vertically upward, 5
then it will have zero speed at the highest point and an Now net displacement of the man is zero = h
6
acceleration of 9.8 ms−2 in downward direction. 0
(b) False, because speed is the magnitude of velocity. ∴Average velocity = = 0
5
(c) True, if a particle is moving with constant velocity, the 6
speed is constant but acceleration is zero. Total distance covered during to whole journey
(d) True, if the positive direction of acceleration is along the = 2.5 + 2.5
direction of motion. = 5 km
5
Q 11. A man walks on a straight road from his home to a ∴Average speed = = 6 kmh −1
(5/ 6)
market 2.5 km away with a speed of 5 kmh −1 .
(iii) 0 to 40 min interval
Finding the market closed, he instantly turns and
walks back home with a speed of 7.5 kmh −1 . What is As the man takes 30 min; to go to market.
∴Time for which he performed return journey
the
= 40 − 30 = 10 min.
(a) magnitude of average velocity and
(b) average speed of the man over the interval of time Distance covered in 10 min
(i) 0 to 30 min = Velocity of return journery × time
(ii) 0 to 50 min 10
= 7.5 × = 1.25 km
(iii) 0 to 40 min 60
Total displacement ∴Net displacement of the man = 2.5 − 1.25 = 1.25 km
Sol. We know that average velocity = 1.25 3
Total time ∴Average velocity = = 1.25 ×
2 2
Total distance
and average speed = 3
Total time
3.75
(i) 0 to 30 min interval =
2
Distance covered in going to market = 2.5 km
= 1.875 kmh −1
Speed = 5 kmh −1
and average speed
∴Time taken to go to market = Distance /speed
Total distance
1 =
= 2.5/5 = h = 30 min Total time
2
2.5 + 1.25
Displacement =
∴ (a) Magnitude of average velocity = 2
Time
3
2.5 −1
= = 5 kmh 3
1 = 3.75 ×
h 2
2
11.25
2.5 =
(b) Average speed = = 5 kmh −1 2
1
2
h = 5.625 kmh −1
Motion in One Dimension 87

Q 12. Look at the graphs (a) to (d) carefully and state with Sol. Speed of thief’s car, vt = 192 kmh −1
reasons, which of these cannot possibly represent 5 160 −1
one-dimensional motion of a particle ? = 192 × ms−1 = ms
18 3
x v Speed of police van, vp = 30 kmh −1
5 25 −1
t t = 30 × = ms
18 3
Speed of the bullet with which it is fired = 150 ms−1
(a) (b) ∴ Actual speed of the bullet, vb = speed of police van + 150
Speed Total path length 25
= + 150
3
475 −1
t = ms
3
∴ Speed of the bullet with which it hits the thief’s car is
t = Relative speed of the bullet w.r.t. thief ’s car
(c) (d)
= vbt
Sol. (a) A line drawn for a given time parallel to position axis i.e. vbt = vb − vt
will cut the graph at two points which means that at a
 475 160 −1 315
given instant of time, the particle will have to positions. = −  ms = = 105 ms−1
 3 3  3
Which is not possible. Hence, graph (a) is not possible.
(b) At a given instant of time, the particle will have two Q 15. Suggest a suitable physical situation for each of the
values of velocity in positive, as well as in negative following graphs.
direction which is not possible in one dimensional
motion. v a B
(c) Speed can never be negative. Hence, this graph is not
possible.
(d) This does not represent one dimensional motion, as this t A C t
graph tells that the total path length decreases after
certain time but total path length of a moving particle
can never decrease with time. (a) (b)

Q 13. Figure below shows the x- t plot of one dimensional Sol. (a) The graph represents the case of a ball thrown up with
motion of a particle. Is it correct to say from the some initial velocity and rebounding from the floor with
graph that the particle moves in a straight line for reduced speed.
t < 0 and on a parabolic path for t > 0 ? If not, (b) It is the acceleration-time graph. The graph represents a
suggest a suitable physical context for this graph. uniformly moving cricket ball turned back after hitting
the bat for a very short time interval (represented by the
x portion ABC of the graph).

Q 16. A boy standing on a stationary lift (open from


above) throws a ball upwards with the maximum
initial speed he can, equal to 49 ms −1 . How much
O t time does the ball take to return to his hands? If the
lift starts moving up with a uniform speed of 5 ms −1
Sol. No, the x- t graph does not show the trajectory of the path of and the boy again throws the up with the maximum
speed he can, how long does the ball take to return
particle.
to his hands?
This may be x-t graph of one dimensional uniformly
accelerated motion. Sol. Case I When the lift is stationary
Let t be total time taken by ball in going vertically upward
Q 14. A police van moving on a highway with a speed of
and coming down to the hands of the boy.
30 kmh −1 fires a bullet at a thief’s car speeding
s = total displacement = 0
away in the same direction with a speed of 1
192 kmh −1 . If the muzzle speed of the bullet is ∴Using the relation, s = ut + at 2, we get
2
150 ms −1 , with what speed does the bullet hit the 1
thief’s car? 0 = 49t + (− 9.8) × t 2
2
88 Objective Physics Vol. 1

or 49t = 4.9t 2 Sol. (a) (i) Distance covered by an object in a given time interval
49 is equal to the area under speed-time graph.
or t= = 10 s.
4.9 ∴ Distance traversed by the particle between the interval
Case II When the lift starts moving with uniform speed t = 0 to 10 s = area of the triangle OAB
As the lift starts moving upwards with uniform speed of 1
= × 10 × 12
5 ms−1, there is no change in the relative velocity of the ball 2
w.r.t. the boy which remains 49 ms−1 due to the fact that = 60 m
there is no acceleration in the lift. i.e. initial velocity of the (ii) The average speed in a given time interval is given by
ball will remain 49 ms−1 only w.r.t. lift. Hence, the ball will Total distance covered in the given time interval
naturally return back to the boy’s hand after 10 s. vav =
Total time
Q 17. The speed-time graph of a particle moving along a ∴vav for the interval 0 s to 10 s is given by
fixed direction is shown below. Obtain the 60
vav = = 6 ms−1
distance traversed by the particle between 10
( a ) t = 0 s to 10 s (b) t = 2 s to 6 s. What is the (b) (i) The distance covered from 2 s to 6 s is calculated in
average speed of the particle over the intervals in the following way.
(a) and (b)? Area of speed-time graph in given interval
= distance travelled
1 1
= (3) (4.8 + 12) + (1) (12 + 9.6)
2 2
Speed (ms–1)

12 A
= 36 m
9.6
(ii) Average speed during this interval is given by
4.8 Total distance traversed
vav =
Total time
C B 36
O 2 5 6 10
t = = 9 ms−1
4
Objective Problems
[ Level 1 ]
Basic Definitions
1. A particle’s velocity changes from ( 2i$ + 3$j ) m/ s to 8. A wheel of radius 1 m rolls forward half a revolution on a
( 2i$ − 3$j ) m/ s in 2 s. The acceleration in m/ s 2 is horizontal ground. The magnitude of the displacement of
the point of the wheel initially in contact with the
(a) − ($i + 5$j) (b) ($i + 5$j)/ 2 ground is
(c) zero (d) (− 3j$ ) (a) 2π
(b) 2π
2. A boy is running over a circular track with uniform speed
of 10 m/ s. Find the average velocity for movement of boy (c) π2 + 4
from A to B (in m / s) (d) π
B
9. A particle is constrained to move on a straight line path. It
returns to the starting point after 10 s. The total distance
covered by the particle during this time is 30 m. Which of
20 m the following statements about the motion of the particle
is false?
(a) Displacement of the particle is zero
(b) Average speed of the particle is 3 m / s
A
(c) Displacement of the particle is 30 m
(d) Both (a) and (b)
10 40
(a) (b) (c) 10 (d) None of these
π π 10. A particle, moving with uniform speed v, changes its
direction by angle θ in time t. Magnitude of its average
3. A car has to cover the distance 60 km. If half of the total
acceleration during this time is
time it travels with speed 80 km/ h and in rest half time its 2v θ
speed becomes 40 km/ h, the average speed of car will be (a) zero (b) sin
t 2
(a) 60 km / h (b) 80 km / h (c) 120 km / h (d) 180 km / h v 2
(c) (d) None of these
4. A person moves towards east for 3 m, then towards north t
for 4 m and then moves vertically up by 5 m. What is his 11. A point traversed half of the distance with a velocity v 0 .
distance now from the starting point? The remaining part of the distance was covered with
(a) 5 2 m (b) 5 m velocity v1 for half the time and with velocity v 2 for the
(c) 10 m (d) 20 m other half of the time. The mean velocity of the point
5. A point traversed 3/ 4 th of the circle of radius R in time t. averaged over the whole time of motion is
v0 + v1 + v2
The magnitude of the average velocity of the particle in (a)
3
this time interval is 2v0 + v1 + v2
πR 3πR R 2 R (b)
(a) (b) (c) (d) 3
t 2t t 2t v0 + 2v1 + 2v2
(c)
3
6. A particle starts from the origin, goes along X -axis to the
2v0 (v1 + v2 )
point ( 20 m, 0) and then returns along the same line to the (d)
(2v0 + v1 + v2 )
point ( −20 m, 0). The distance and displacement of the
particle during the trip are 12. During the first 18 min of a 60 min trip, a car has an
(a) 40 m, 0 (b) 40 m , 20 m average speed of 11m/ s. What should be the average
(c) 40 m , − 20 m (d) 60 m , − 20 m
speed for remaining 42 min so that car is having an
7. An insect crawls a distance of 4 m along north in 10 s and average speed of 21 m/ s for the entire trip?
then a distance of 3 m along east in 5 s. The average (a) 25.3 m / s
velocity of the insect is (b) 29.2 m / s
7 1 1 4 (c) 31 m / s
(a) m/s (b) m/s (c) m/s (d) m/s
15 5 3 5 (d) 35.6 m / s
90 Objective Physics Vol. 1

Motion in 1-D with Constant 22. A helicopter, moving vertically upwards, releases a
packet when it is at a certain height above the ground.
Acceleration The packet initially moves upwards for a time t 1 and then
13. The velocity v of a particle as a function of its position ( x ) falls downwards for a t 2 until it reaches the ground.
is expressed as v = c1 − c 2 x , where c1 and c 2 are Then,
positive constants. The acceleration of the particle is (a) t1 < t2 (b) t1 = t2
c2 c1 + c2 (c) t1 > t2 (d) Data insufficient
(a) c2 (b) − (c) c1 − c2 (d)
2 2 23. A particle starts with a velocity of 2 ms −1 and moves in a
14. The displacement of a body in 8 s starting from rest with . ms −2 . The first time
straight line with a retardation of 01
an acceleration of 20 cm/ s 2 is at which the particle is 15 m from the starting point, is
(a) 64 m (b) 64 cm (a) 10 s (b) 20 s
(c) 640 cm (d) 0.064 m (c) 30 s (d) 40 s

15. A body falls from a height h = 200 m. The ratio of distance 24. If a stone is thrown up with a velocity of 9.8 m/ s, then
travelled in each 2 s, during t = 0to t = 6s of the journey is how much time will it take to come back?
(a) 1 : 4 : 9 (b) 1 : 2 : 4 (a) 1 s (b) 2 s
(c) 1 : 3 : 5 (d) 1 : 2 : 3 (c) 3 s (d) 4 s

16. A ball is released from height h and another from 2h. The 25. The motion of a particle is described by the equation
ratio of time taken by the two balls to reach the ground is v = at. The distance travelled by the particle in the first 4 s
(a) 1 : 2 (b) 2 : 1 (a) 4a (b) 12a
(c) 2 : 1 (d) 1 : 2 (c) 6a (d) 8a

17. A body falling from the rest has a velocity v after it falls 26. A particle starts from rest, accelerates at 2 m/ s 2 for 10 s
through a height h. The distance it has to fall down further and then goes for constant speed for 30 s and then
for its velocity to become double, will be decelerates at 4 m/ s 2 till it stops. What is the distance
(a) 8h (b) 6h travelled by it?
(c) 3h (d) 5h
(a) 750 m (b) 800 m
18. The velocity of a particle moving in the positive direction (c) 700 m (d) 850 m
of x-axis varies as v = 5 x. Assuming that at t = 0, 27. A body dropped from the top of a tower covers a distance
particle was at x = 0. What is the acceleration of the 7x in the last second of its journey, where x is the distance
particle? covered in first second. How much time does it take to
(a) 12.5 m / s2 (b) 7.5 m / s2 reach the ground?
(c) 5 m / s2 (d) 2.5 m / s2 (a) 3 s (b) 4 s
(c) 5 s (d) 6 s
19. A person throws balls into air after every second. The
28. A particle returns to the starting point after 10 s. If the rate
next ball is thrown when the velocity of the first ball is
of change of velocity during the motion is constant, then
zero. How high do the ball rise above his hand?
its location after 7 s will be same as that after
(a) 2 m (b) 5 m (c) 8 m (d) 10 m
(a) 1 s (b) 2 s
20. An object is moving with velocity 10 m/ s. A constant (c) 3 s (d) 4 s
force acts for 4 s on the object and gives it a speed of 29. A body is moving with uniform velocity of 8 ms −1 . When
2 m/ s in opposite direction. The acceleration produced is the body just crossed another body, the second one starts
(a) 3 m / s2 (b) − 3 m / s2 and moves with uniform acceleration of 4 m/ s 2 . The time
(c) 6 m / s2 (d) − 6 m / s2
after which two bodies meet, will be
(a) 2 s (b) 4 s
21. A ball is thrown vertically upward with a speed v from a
(c) 6 s (d) 8 s
height h metre above the ground. The time taken for the
ball to hit ground is 30. Two bodies A and B start from rest from the same point
v 2hg v 2hg with a uniform acceleration of 2 m/ s 2 . If B starts one
(a) 1− (b) 1+
g v2 g v2 second later, then the two bodies are separated, at the end
2hg v  2hg  of the next second, by
(c) 1+ (d) 1 + 1 + 2  (a) 1 m (b) 2 m
v2 g  v 
(c) 3 m (d) 4 m
Motion in One Dimension 91

31. Velocity of a body moving along a straight line with 39. A particle moves along x-axis as x = 4 ( t − 2) + a ( t − 2) 2 .
3 Which of the following statements is true?
uniform acceleration a reduces by of its initial velocity
4 (a) The initial velocity of particle is 4
in time t 0 . The total time of motion of the body till its (b) The acceleration of particle is 2a
velocity becomes zero is (c) The particle is at origin at t = 0
4 3 (d) None of the above
(a) t0 (b) t0
3 2 40. A stone thrown upward with a speed u from the top of the
5 8
(c) t0 (d) t0 tower reaches the ground with a speed 3 u. The height of
3 3
the tower is
32. A body travelling with uniform acceleration crosses two (a) 3 u2 / g (b) 4 u2 / g
points A and B with velocities 20 m/ s and 30 m/ s (c) 6 u2 / g (d) 9 u2 / g
respectively. The speed of the body at mid-point of A and
B is 41. A particle is dropped under gravity from rest from a
(a) 25 m / s (b) 25.5 m / s height h ( g = 9.8 m/ s 2 ) and it travels a distance 9h / 25 in
(c) 24 m / s (d) 10 6 m / s the last second, the height h is
(a) 100 m (b) 122.5 m
33. A particle starting from rest with constant acceleration (c) 145 m (d) 167.5 m
travels a distance x in first 2 s and a distance y in next 2 s,
42. An aeroplane is moving with a velocity u. It drops a
then
packet from a height h. The time t taken by the packet in
(a) y = x (b) y = 2x
(c) y = 3x (d) y = 4 x reaching the ground will be
 2g   2u
34. A man in a balloon rising vertically with an acceleration (a)   (b)  
 h  g
of 4.9 m/ s 2 releases a ball 2 s after the balloon is let go
 h  2h
from the ground. The greatest height above the ground (c)   (d)  
 2g   g
reached by the ball is ( g = 9.8 m/ s 2 )
(a) 14.7 m (b) 19.6 m 43. When a ball is thrown up vertically with velocity v 0 , it
(c) 9.8 m (d) 24.5 m
reaches a maximum height of h. If one wishes to triple the
35. A stone falls freely under gravity. The total distance maximum height, then the ball should be thrown with
covered by it in the last second of its journey equals the velocity
distance covered by it in first 3 s of its motion. The time (a) 3 v0 (b) 3 v0
for which the stone is in air is (c) 9 v0 (d) 3/ 2 v0
(a) 5 s (b) 12 s
(c) 15 s (d) 8 s 44. A body starts from rest with uniform acceleration a, its
velocity after n second is v. The displacement of the body
36. The displacement of a particle moving in a straight line is in last 3 s in (assume total time of journey from 0 to
described by the relation, s = 6 + 12 t − 2 t 2 . Here, s is in n second)
metre and t is in second. The distance covered by particle v (6n − 9) 2v (6n − 9)
(a) (b)
in first 5 s is 2n n
2v (2n + 1) 2v (n − 1)
(a) 20 m (b) 32 m (c) (d)
(c) 24 m (d) 26 m n n

37. The position of a particle along x-axis at time t is given by 45. A train accelerating uniformly from rest attains a
x = 2 + t − 3t 2 . The displacement and the distance maximum speed of 40 ms −1 in 20 s. It travels at this speed
travelled in the interval t = 0 to t = 1are respectively for 20 s and is brought to rest uniform retardation in
(a) 2, 2 (b) −2, 2.5 further 40 s. What is the average velocity during this
(c) 0, 2 (d) −2, 21
. period?
(a) 80/ 3 ms−1 (b) 40 ms−1
38. A stone is allowed to fall freely from rest. The ratio of the
(c) 25 ms−1 (d) 30 ms−1
times taken to fall through the first metre and the second
metre distance is 46. A particle is thrown vertically upwards. Its velocity at
(a) 2−1 half of the height is 10 m/ s. Then, the maximum height
(b) 2+1 attained by it is ( g = 10 m/ s 2 )
(c) 2 (a) 16 m (b) 10 m
(d) None of the above (c) 20 m (d) 40 m
92 Objective Physics Vol. 1

47. Which of the following represents uniformly accelerated 54. A particle starts from rest and traverses a distance l with
motion? uniform acceleration, then moves uniformly over a further
t+a t+a distance 2l and finally comes to rest after moving a further
(a) x = (b) x =
b b distance 3l under uniform retardation. Assuming entire
x+a motion to be rectilinear motion the ratio of average speed
(c) t = (d) x = t + a
b over the journey to the maximum speed on its ways is
(a) 1 / 5 (b) 2/ 5
48. Two particles A and B start from rest and move for equal (c) 3/ 5 (d) 4 / 5
time on a straight line. Particle A has an acceleration of
55. A point moves with uniform acceleration and v1 , v 2 and
2 m/ s 2 for the first half of the total time and 4 m/ s 2 for v 3 denote the average velocities in the three successive
the second half. The particle B has acceleration 4 m/ s 2 intervals of time t 1 , t 2 and t 3 . Which of the following
for the first half and 2 m/ s 2 for the second half. Which relation is correct?
particle has covered larger distance? (a) (v1 − v2 ) : (v2 − v3 ) = (t1 − t2 ) : (t2 + t3 )
(b) (v1 − v2 ) : (v2 − v3 ) = (t1 + t2 ) : (t2 + t3 )
(a) A
(c) (v1 − v2 ) : (v2 − v3 ) = (t1 − t2 ) : (t1 − t3 )
(b) B
(d) (v1 − v2 ) : (v2 − v3 ) = (t1 − t2 ) : (t2 − t3 )
(c) Both have covered the same distance
(d) Data insufficient 56. A body is thrown vertically upwards from the top A of
tower. It reaches the ground in t 1 sec. If it is thrown
49. If a ball is thrown vertically upwards with speed u, the
vertically downwards from A with the same speed it
distance covered during the last t second of its ascent is
reaches the ground in t 2 sec. If it is allowed to fall freely
(a) ut − (gt 2 / 2 ) (b) (u + gt ) t
from A, then the time it takes to reach the ground is given
(c) ut (d) gt 2 / 2 by
t1 + t2 t1 − t2
50. A body thrown vertically up from the ground passes the (a) t = (b) t =
2 2
height of 10.2 m twice in an interval of 10 s. What was its t1
initial velocity? (c) t = t1 t2 (d) t =
t2
(a) 52 m / s (b) 61 m / s
(c) 45 m / s (d) 26 m / s
Motion in 1-D with Variable
51. A body is projected with a velocity u. It passes through a
certain point above the ground after t 1 second. The time Acceleration
interval after which the body passes through the same 57. The position of a particle moving along the x-axis is
point during the return journey is expressed as x = at 3 + bt 2 + ct + d. The initial
u  u  acceleration of the particle is
(a)  − t12 (b) 2  − t1
g  g  (a) 6a (b) 2b
u   u2  (c) (a + b) (d) (a + c)
(c)  − t1 (d)  2 − t1
g  g  58. The acceleration a in m/s 2 , of a particle is given by
52. With what speed should a body be thrown upwards so a = 3 t 2 + 2 t + 2, where t is the time. If the particle starts
that the distances traversed in 5 th second and 6 th second out with a velocity v = 2 m/ s at t = 0, then the velocity at
are equal? the end of 2 s is
(a) 5.84 m / s (a) 12 m / s (b) 14 m / s (c) 16 m / s (d) 18 m / s
(b) 49 m / s 59. A particle initially at rest moves along the x-axis. Its
(c) 98 m / s acceleration varies with time as a = 4t. If it starts from the
(d) 98 m / s origin, the distance covered by it in 3 s is
53. A train accelerates from rest at a constant rate α for (a) 12 m (b) 18 m
(c) 24 m (d) 36 m
distance x1 and time t 1 . After that it retards to rest at
constant rate β for distance x 2 and time t 2 . Which of the 60. The acceleration a (in ms −2 ) of a body, starting from rest
following relations is correct? varies with time t (in second) according to the relation
x1 α t1 x1 β t1 a = 3t + 4. The velocity of the body starting from rest at
(a) = = (b) = =
x2 β t2 x2 α t2 time t = 2 s will be
x α t x β t (a) 10 ms−1 (b) 12 ms−1
(c) 1 = = 2 (d) 1 = = 2
x2 β t1 x2 α t1 (c) 14 ms−1 (d) 16 ms−1
Motion in One Dimension 93

61. The displacement ( x ) of a particle depends on time t as 68. The displacement of a particle moving in a straight line
x = αt 2 − βt 3 . Choose the incorrect statements from the depends on time as x = αt 3 + βt 2 + γt = δ.
following. The ratio of initial acceleration to its initial velocity
(a) The particle never returns to its starting point depends
2α (a) only on α and γ (b) only on β and γ
(b) The particle comes to rest after time
3β (c) only on α and β (d) only on α
(c) The initial velocity of the particle is zero 69. At time t = 0, a car moving along a straight line has a
(d) The initial acceleration of the particle is zero
velocity of 16 m/ s. It slows down with an acceleration of
62. The displacement of a particle is given by − 0.5t m/ s 2 , where t is in second. Mark the correct
y = a + bt + ct 2 − dt 4 . The initial velocity and statement (s).
acceleration are respectively (a) The direction of velocity changes at t = 8 s
(a) b, − 4 d (b) − b, − 2c (b) The distance travelled in 4 s is approximately 59 m
(c) b, 2c (d) 2c, − 4 d (c) The distance travelled by the particle in 10 s is 94 m
(d) The velocity at t = 10 s is 8 m / s
63. The displacement of a particle starting from rest (at t = 0 )
is given by s = 6t 2 − t 3 . The time in second at which the Motion in 2 or 3-D with Constant or
particle will attain zero velocity again, is
(a) 2
Variable Acceleration
(b) 4 70. A particle moves along the positive branch of the curve
(c) 6 x2 t2
(d) 8 y= where x = , x and y are measured in metres and
2 2
64. A particle moves along a straight line such that its t in second. At t = 2 s, the velocity of the particle is
displacement at any time t is given by (a) 2$i − 4 $j m / s (b) 4 $i + 2$j m / s
s = 3t 3 + 7t 2 + 14t + 5. The acceleration of the particle at (c) 2$i + 4 $j m / s (d) 4 $i − 2$j m / s
t = 1s is
(a) 18 m / s2 (b) 32 m / s2 71. The displacement of an object along the three axes are
(c) 29 m / s2 (d) 24 m / s2
given by x = 2t 2 , y = t 2 − 4t and z = 3t − 5. The initial
velocity of the particle is
65. A particle moves along a straight line. Its position at any (a) 10 unit (b) 12 unit
8t 3 (c) 5 unit (d) 2 unit
instant is given by x = 32t − , where x is in metre and t
4 72. A particle has an initial velocity of 3$i + 4$j and an
is in second. Find the acceleration of the particle at the
acceleration of 0.4i$ + 0.3$j. Its speed after 10s is
instant when particle is at rest.
(a) 10 units
(a) − 16 m / s2 (b) − 32 m / s2
(b) 7 units
(c) 32 m / s2 (d) 16 m / s2 (c) 7 2 units
(d) 8.5 units
66. A body starts from rest, with uniform acceleration a. The
acceleration of the body as function of time t is given by 73. A particle moves in the xy - plane according to the law
the equation a = pt , where p is a constant, then the x = k t , y = k t (1 − αt ), where k and α are positive
displacement of the particle in the time interval t = 0 to constants and t is time. The trajectory of the particle is
t = t 1 will be
αx 2
1 3 1 2 (a) y = kx (b) y = x −
(a) pt1 (b) pt1 k
2 3
ax 2
1 1 (c) y = − (d) y = αx
(c) pt12 (d) pt13 k
2 6

67. The acceleration of a particle is increasing linearly with 74. The position of a particle moving in the xy - plane at any
time t as bt. The particle starts from the origin with an time t is given by x = ( 3t 2 − 6t ) m, y = ( t 2 − 2t ) m. Select
initial velocity v 0 . The distance travelled by the particle the correct statement about the moving particle from the
in time t will be following.
(a) v0t +
1 3
bt (b) v0t +
1 3
bt (a) The acceleration of the particle is zero at t = 0 s
6 3 (b) The velocity of the particle is zero at t = 0 s
1
(c) v0t + bt 2
1
(d) v0t + bt 2 (c) The velocity of the particle is zero at t = 1s
3 2 (d) The velocity and acceleration of the particle are never zero
94 Objective Physics Vol. 1

75. Velocity and acceleration of a particle at some instant of 81. Which of the following graph correctly represents
time are v = ( 3i$ + 4$j ) m/ s and a = − ( 6$i + 8$j ) m/ s 2 velocity-time relationship for a particle released from rest
respectively. At the same instant particle is at origin. to fall freely under gravity?
Maximum x-coordinate of particle will be v v

(a) 15
. m (b) 0.75 m
(c) 2.25 m (d) 4.0 m (a) (b)
76. A particle’s velocity changes from ( 2i$ + 3$j ) m/ s into
t
( 3i$ − 2$j ) m/s in 2 s. If its mass is 1kg, the acceleration t
v v
2
( m/ s ) is
(a) − (i$ + 5$j) (b) (i$ + 5$j)/ 2
(c) (d)
(c) zero (d) ($i − 5$j)/ 2

77. A particle has an initial velocity of 3$i + 4$j and an t t

acceleration of 0.4$i + 0.3$j. Its speed after 10s is 82. The velocity-time graph of a body is shown in figure. It
(a) 10 units (b) 7 units implies that at point B
(c) 7 2 units (d) 8.5 units
v
78. The position vector of a particle is
r = a cos ωt i$ + sin ωt $j A
The velocity of the particle is C
B
(a) parallel to position vector
(b) perpendicular to position vector
t
(c) directed towards origin
(d) directed away from the origin (a) the force is zero
(b) there is a force towards motion
79. A body is projected from origin such that its position (c) there is a force which opposes motion
vector varies with time as r = [ 3t$i + ( 4t − 5t 2 ) $j] m and t (d) there is only gravitational force
is time in second. Then, 83. The velocity-time graph for a particle moving along
(a) x-coordinate of particle is 2.4 m when y-coordinate is zero x-axis is shown in the figure. The corresponding
(b) speed of projection is 5 m / s displacement-time graph is correctly shown by
(c) angle of projection with x-axis is tan −1 (4 / 3)
(d) time when particle is again at x-axis is 0.8 s
v
Graphical Problems
80. A particle projected vertically upwards returns to the
ground in time T. Which graph represents the correct
variation of velocity ( v ) against time ( t ) ? t

v v

x x
(a) (b)
(a) (b)
O T/2 T O T/2 T

t t
v v x x

(c) (d) (c) (d)

O T/2 T O T/2 T t t
Motion in One Dimension 95

84. Acceleration versus time graph of a body starting from The displacement and distance travelled by the body in
rest is shown in the figure. The velocity versus time graph 6 s are respectively
of the body is given by v (m/s)
5
a
4
3
2
1
3 t(s)
0
–1 1 2 4 5 6
t –2
O
–3
v
(a) 8 m , 16 m (b) 16 m , 32 m
v (c) 16 m , 16 m (d) 8 m , 18 m

(a) (b) 88. v 2 versus s -graph of a particle moving in a straight line is


as shown in figure. From the graph some conclusions are
O t t drawn. State which statement is wrong?
O
v2
v v

(c) (d)

t t s
O O

85. The acceleration of a train between two stations is shown (a) The given graph shows a uniformly accelerated motion
in the figure. The maximum speed of the train is (b) Initial velocity of particle is zero
(c) Corresponding s-t graph will be a parabola
+5 (d) None of the above
a
89. A graph between the v2 (m2/s2)
(m/s2)
4 8 12 16 t (s) square of the velocity of 25
–5 a particle and the
distance s moved by the
(a) 60 m / s (b) 30 m / s particle is shown in the
(c) 120 m / s (d) 90 m / s figure. The acceleration 9
of the particle is
86. Which graph represents uniform motion?
(a) − 8 m / s2
s (m)
(b) 4 m / s2 0 2
(c) − 16 m / s2
(a) (b) (d) None of the above
s s 90. If the velocity v of a particle
moving along a straight 20
t t
line decreases linearly with
v (in ms–1)

its displacement s from


20 ms −1 to a value
(c) (d) None of these approaching zero at
s = 30 m, then acceleration
s
of the particle at s = 15 mis 0
t s (in m) 30
2 2
(a) ms−2 (b) − ms−2
87. The velocity-time graph of a body moving in a straight 3 3
20 −2 20 −2
line is shown in the figure. (c) ms (d) − ms
3 3
96 Objective Physics Vol. 1

91. The distance-time graph of a particle at time t makes 95. The x- t equation is given as x = 2t + 1. The corresponding
angle 45° with the time axis. After one second, it makes v- t graph is
angle 60° with the time axis. What is the average (a) a straight line passing through origin
acceleration of the particle? (b) a straight line not passing through origin
(a) 3 −1 (b) 3+1 (c) 3 (d) 1 (c) a parabola
(d) None of the above
92. Acceleration-time graph for a particle moving in a
straight line is as shown in figure. Change in velocity of 96. The variation of velocity of a particle with time moving
the particle from t = 0 to t = 6s is along a straight line is illustrated in the adjoining figure.
a (m/s2) The distance travelled by the particle in 4 s is

30

Velocity (m/s)
4

20
6
t(s)
2 4
10

1 2 3 4
Time (s)
(a) 10 m / s (b) 4 m / s (c) 12 m / s (d) 8 m / s
(a) 60 m (b) 55 m
93. The velocity-time graph is shown in the figure for a (c) 25 m (d) 30 m
particle. The acceleration of the particle is
v (m/s)
97. The v- t graph of a moving object is given in figure. The
maximum acceleration is
15
80

60
Velocity (cm/s)

10
40

20
5
t (s)
1 2 3 4 10 20 30 40 50 60 70 80
Time (s)
(a) 22.5 m / s2 (b) 5 m / s2 (c) − 5 m / s2 (d) − 3 m / s2
(a) 1 cm / s2 (b) 2 cm / s2
94. Figure shows the position-time ( x- t ) graph of the motion (c) 3 cm / s2 (d) 6 cm / s2
of two boys A and B returning from school O to their
homes P and Q respectively. Which of the following 98. A lift is going up. The variation in the speed of the lift is
statements is true? as given in the graph. What is the height to which the lift
takes the passengers?

x
Q B's home
Velocity (m/s)

A's home B
3.6
P
A

2 Time (s) 10 12
O t
(a) A walks faster than B (a) 3.6 m
(b) Both A and B reach home at the same time (b) 28.8 m
(c) B starts for home earlier than A (c) 36.0 m
(d) B overtakes A on his way to home (d) Cannot be calculated from the above graph
Motion in One Dimension 97

99. A ball is dropped vertically from a height of above the 102. The v- t plot of a moving object is shown in the figure.
ground. It hits the ground and bounces up vertically to a The average velocity of the object during the first 10 s, is
height d / 2. Neglecting subsequent motion and air
5
resistance, its velocity v varies with the height h above the

Velocity (m/s)
ground is Time (s)
v v 0
5 10

d –5
(a) h (b) h
d
(a) zero (b) 2.5 ms−1
(c) 5 ms−1 (d) 2 ms−1

103. The acceleration-time ( a- t ) graph of a particle moving in


v v
a straight line is as shown in the figure. The velocity-time
graph of the particle would be
(c) d (d) a (m/s2)
d
h h
4

v
100.The given graph shows the O 2
v0 t (s)
variation of velocity with
displacement. Which one of the (a) a straight line (b) a parabola
graph given below correctly (c) a circle (d) an ellipse
represents the variation of
acceleration with displacement?
x0
x Relative Motion
a a 104. A particle ( A ) moves due north at 3 km/ h another particle
( B ) due west at 4 km/ h. The relative velocity of A with
(a) x (b) x respect to B is (tan 37° = 3 / 4 )
(a) 5 km / h, 37° north of east
(b) 5 km / h, 37° east of north
a a (c) 5 2 km / h , 53° east of north
(d) 5 2 km / h , 53° north of east
(c) x (d) x
105. A river is flowing from west to east at a speed of
8 m per min. A man on the south bank of the river,
capable of swimming at 20 m/ min in still water, wants to
swim across the river in the shortest time. He should
101.From the displacement-time graph find out the velocity swim in a direction
of a moving body (a) due north (b) 30° east of north
(c) 30° west of north (d) 60° east of north

106. A man standing on a road has to hold his umbrella at 30°


with the vertical to keep the rain away. He throws the
Time (s)

umbrella and starts running at 10 km/ h. He finds that


raindrops are hitting his head vertically. What is the
speed of rain with respect to ground?
30° (a) 10 3 km / h
O Displacement (m) (b) 20 km / h
20
1 (c) km / h
(a) m/s (b) 3 m / s 3
3
1 10
(c) 3 m/s (d) m/s (d) km / h
3 3
98 Objective Physics Vol. 1

107. What are the speeds of two objects if, when they move 115.Two stones are thrown up simultaneously with initial
uniformly towards each other, they get 4 m closer in each speeds of ‘u1 and u 2 ( u 2 > u1 ). They hit the ground after
second and when they move uniformly in the same 6 s and 10 s respectively. Which graph in figure correctly
direction with the original speeds, they get 4.0 m closer represents the time variation of ∆x = ( x 2 − x1 ), the
each 10s? relative position of the second stone with respect to the
(a) 2.8 m / s and 12. m/s first upto t = 10 s?
(b) 5.2 m / s and 4.6 m / s Assume that the stones do not rebound after hitting the
(c) 3.2 m / s and 21
. m/s ground.
(d) 2.2 m / s and 18
. m/s ∆x ∆x

108. Two trains are each 50 m long moving parallel towards


each other at speeds 10 m/ s and 15 m/ s respectively, at
what time will they pass each other?
(a) 8 s (b) 4 s t t
(c) 2 s (d) 6 s 0 2 4 6 8 10 0 2 4 6 8 10
(a) (b)
109. The rowing speed of a man relative to water is 5 km/ h
∆x ∆x
and the speed of water flow is 3 km/ h. At what angle to
the river flow should he head if he wants to reach a point
on the other bank, directly opposite to starting point?
(a) 127° (b) 143°
(c) 120° (d) 150° t t
0 2 4 6 8 10 0 2 4 6 8 10
110. A stationary man observes that the rain is falling
(c) (d)
vertically downward. When he starts running with a
velocity of 12 km/ h, he observes that the rains is falling 116. A boy is running on the plane road with velocity v with a
at an angle 60° with the vertical. The actual velocity of long hollow tube in his hand. The water is failing
rain is vertically downwards with velocity u. At what angle to
(a) 12 3 km / h (b) 6 3 km / h the vertical, he must incline the tube so that the water
(c) 4 3 km / h (d) 2 3 km / h drops enter it without touching its sides?
 v  v
111.Two bodies are held separated by 9.8 m vertically one (a) tan −1   (b) sin −1  
 u  u
above the other. They are released simultaneously to fall
 u  v
freely under gravity. After 2 s, the relative distance (c) tan −1   (d) cos−1  
 v  u
between them is
(a) 4.9 m (b) 19.6 m 117. Two points move in the same straight line starting at the
(c) 9.8 m (d) 39.2 m same moment from the same point in it. The first moves
112. A ball is dropped from the top of a building 100 m high. with constant velocity u and the second with constant
At the same instant, another ball is thrown upwards with acceleration f. During the time elapses before the second
catches, the first greatest distance between the particle is
a velocity of 40 ms −1 from the bottom of the building. The
u u2 f f
two balls will meet after (a) (b) (c) (d)
f 2f 2u2 u2
(a) 5 s (b) 2.5 s
(c) 2 s (d) 3 s 118.A person walks up a stalled escalator in 90 s. When just
113.A 100 m long train crosses a man travelling at 5 km/ h, in standing on the same moving escalator, he is carried in
opposite direction , in 7.2 s, then the velocity of train is 60 s. The time it would take him to walk up the moving
(a) 40 km / h escalator will be
(b) 25 km / h (a) 27 s (b) 50 s (c) 18 s (d) 36 s
(c) 20 km / h 119.Particle A is moving along x-axis. At time t = 0, it has
(d) 45 km / h
velocity of 10 m/ s and acceleration − 4 m/ s 2 . Particle B
114.A man is 25 m behind a bus, when bus starts accelerating has velocity of 20 m/ s and acceleration − 2 m/ s 2 .
at 2 m/ s 2 and man starts moving with constant velocity of Initially, both the particles are at origin. At time t = 2 s,
10 m/ s. Time taken by him to board the bus is distance between the two particles is
(a) 2 s (b) 3 s (a) 24 m (b) 36 m
(c) 4 s (d) 5 s (c) 20 m (d) 42 m
Motion in One Dimension 99

120.Driver of a train moving at a speed v1 sights a freight train 121.The speed of a boat is 5 km/ h in still water. It crosses a
at a distance d ahead of him on the same track moving in river of width 1km along the shortest possible path in
the same direction with a slow speed v 2 . He puts on the 15 min. Then, velocity of river will be
brakes and gives his train a constant deceleration α. Then, (a) 4.5 km / h (b) 4 km / h
there will be no collision, if (c) 15
. km / h (d) 3 km / h
 v − v2  122. A ship X moving due north with speed v observes that
(a) d >  1 
 2α  another ship Y is moving due west with same speed v. The
(v1 − v2 )2 actual velocity of Y is
(b) d <
2α (a) 2v towards south-west
(v − v2 ) 2 (b) 2v towards north-west
(c) d > 1
2α (c) 2v towards south-east
(d) None of the above (d) v towards north-east

[ Level 2 ]
Only One Correct Option 5. The displacement x of a particle in a straight line motion
is given by x = 1 − t − t 2 . The correct representation of
1. A ball is dropped onto the floor from a height of 10 m . It
the motion is
rebounds to a height of 5 m . If the ball was in contact
with the floor for 0.01s, what was its average acceleration
during contact? Take g = 10 m/ s 2 . x
x
(a) 2414 m / s2 (a) (b)

(b) 1735 m / s2 t
t
2
(c) 3120 m / s
(d) 4105 m / s2
(c) x (d) x
2. Let v and a denote the velocity and acceleration
respectively of a particle in the dimensional motion
t t
(a) The speed of the particle decreases when v ⋅ a < 0
(b) The speed of the particle increases whenv × a > 0
(c) The speed of the particle increases when v > a = 0
6. A swimmer crosses a river of width d flowing at velocity
(d) The speed of the particle decreases when | a | < | a | v. While swimming, he heads himself always at an angle
of 120° with the river flow and on reaching the other end
3. A stone is thrown vertically upward with an initial he finds a drift of d / 2in the direction of flow of river. The
15
. v0 speed of the swimmer with respect to the river is
velocity v 0 . The distance travelled in time is
g (a) (2 − 3) v
v2 3v02 (b) 2 (2 − 3) v
(a) 0 (b)
2g 8g (c) 4 (2 − 3) v
5v02 (d) (2 + 3) v
(c) (d) None of these
8g
7. Three particles start simultaneously from a point on a
4. The vertical height of point P above the ground is twice horizontal smooth plane. First particle moves with speed
that of Q. A particle is projected downward with a speed v1 towards east, second particle moves towards north
of 5 m/ s from P and at the same time another particle is with speed v 2 and third-one moves towards north-east.
projected upward with the same speed from Q. Both The velocity of the third particle, so that the three always
particles reach the ground simultaneously, then lie on a line, is
(a) PQ = 30 m v1 + v2
(a) (b) v1v2
(b) time of flight of stones = 3 s 2
(c) Both (a) and (b) are correct v1 v2 v1 v2
(c) (d) 2
(d) Both (a) and (b) are wrong v1 + v2 v1 + v2
100 Objective Physics Vol. 1

8. On a calm day, a boat can go across a lake and return in 15. If a particle takes t second less and acquires a velocity of
time T0 at a speed V. On a rough day, there is uniform v ms −1 more in falling through the same distance on two
current at speed v to help the onward journey and impede planets, where the accelerations due to gravity are 2 g and
the return journey. If the time taken to go across and 8 g respectively, then
return on the rough day be T, then T / T0 is (a) v = 4 gt (b) v = 5gt (c) v = 2gt (d) v = 16gt
1
(a) 1 − v 2 /V 2 (b) 16. Two particles start simultaneously from the same point
1 − v 2 /V 2
and move along two straight lines, one with uniform
1
(c) 1 + v 2 /V 2 (d) velocity v and other with a uniform acceleration a. If α is
1 + v 2 /V 2 the angle between the lines of motion of two particles,
9. A point moves in a straight line so that its displacement x then the least value of relative velocity will be at time
given by
at time t is given by x 2 = t 2 + 1. Its acceleration is
v v v v
(a) 1/ x (b) 1/x 3 (a) sinα (b) cosα (c) tanα (d) cotα
a a a a
(c) − 1/x 2 (d) − 1/ x 3
17. Speed-time graph of two cars A and B approaching
10. A car starts moving along a line, first with acceleration towards each other is shown in figure. Initial distance
a = 5 m/ s 2 starting from rest then uniformly and finally between them is 60 m . The two cars will cross each other
after time
decelerating at the same rate a comes to rest. The total
v (m/s)
time of motion is 25 s. The average speed during the time
is 20 m/ s 2 . How long does the particle move uniformly?
20
(a) 10 s (b) 12 s (c) 20 s (d) 15 s
A
11. A ball is thrown vertically upwards with a speed u. It
reaches a point B at a height h (lower than the maximum
height) after time t 1 . It returns to the ground after time t 2 B
from the instant it was at B during the upward journey.
Then, t 1 t 2 is equal to 2 4 t(s)
(a) 2h/ g (b) h/ g
(c) h/2g (d) h/4 g (a) 2 s (b) 3 s (c) 1.5 s (d) 2s

12. A target is made of two plates, one of wood and the other 18. The acceleration-time graph of a particle moving along a
of iron. The thickness of the wooden plate is 4 cm and that straight line is as shown in figure. At what time, the
of iron plate is 2cm . A bullet fired goes through the particle acquires its initial velocity?
wood first and then penetrates 1cm into iron. A similar a (m/s2)
bullet fired with the same velocity from opposite
direction goes through iron first and then penetrates 2cm 10
into wood. If a1 and a 2 be the retardations offered to the
bullet by wood and iron plates respectively, then
(a) a1 = 2a2 (b) a2 = 2a1
(c) a1 = a2 (d) Data insufficient

13. A body starts with an initial velocity of 10 ms −1 and is


moving along a straight line with constant acceleration. 4 t(s)
When the velocity of the particle is 50 ms −1 , the (a) 12 s (b) 5 s
acceleration is reversed in direction. The velocity of the (c) 8 s (d) 16 s
particle when it again reaches the starting point is
19. A street car moves rectilinearly from station A to the next
(a) 70 ms−1 (b) 60 ms−1 (c) 10 ms−1 (d) 30 ms−1
station B with an acceleration varying according to the
14. Two particles P and Q simultaneously start moving from law a = ( b − cx ), where b and c are constants and x is the
point A with velocities 15 m/ s and 20 m/ s respectively. distance from station A. The distance between the two
The two particles move with accelerations equal in stations and the maximum velocity are
magnitude but opposite in direction. When P overtakes Q b c
(a) x = 2b/ c, vmax = (b) x = , vmax = b / c
at B, then its velocity is 30 m/ s. The velocity of Q at point c 2b
B will be b c b
(c) x = , vmax = (d) x = b/ c, vmax =
(a) 30 m / s (b) 5 m / s (c) 20 m / s (d) 15 m / s 2c a c
Motion in One Dimension 101

20. A lift performs the first part of its ascent with uniform 26. A small electric car has a maximum constant acceleration
acceleration a and the remaining with uniform retardation of 1 m/ s 2 , a maximum constant deceleration of 2 m/ s 2
2a. If t is the time of ascent, find the depth of the shaft. and a maximum speed of 20 m/ s. The amount of time it
at 2 at 2 would take to drive this car 1 km starting from rest and
(a) (b) finishing at rest is
4 3
at 2 at 2 (a) 15 s (b) 50 s
(c) (d)
2 8 (c) 35 s (d) 65 s

21. A particle starts moving from rest in a straight line with 27. The diagram shows the variation of 1/ v (where, v is
constant acceleration. After time t 0 , acceleration velocity of the particle) with respect to time. At time
changes its sign (just opposite to the initial direction), t = 3 s using the details given in the graph, the
remaining the same in magnitude. Determine the time instantaneous acceleration will be equal to
1
from the beginning of motion in which the particle v
(m/s)
returns to the initial position.
(a) 2t0 (b) (2 + 2) t0
(c) 3t0 (d) (2 − 2) t0
1
22. An elevator car whose floor to ceiling distance is 2.7 m 3
starts ascending with a constant acceleration of 12 . m/ s 2 .
45°
After 2s of the start, a bolt falls from the ceiling of the
car. The free fall time of the bolt is ( g = 9.8 m / s 2 )
2.7 5.4 t(s)
(a) s (b) s 3s
9.8 9.8
5.4 5.4 (a) − 2 m / s2 (b) + 3 m / s2
(c) s (d) s
8.6 11 (c) + 5 m / s2 (d) − 6 m / s2

23. The displacement x of a particle varies with time t as 28. A particle starting from rest and moving with a uniform
x = ae −αt + beβt , where a, b, α and β are positive acceleration along a straight line covers distances a and b
constants. The velocity of the particle will in successive intervals of p and q second. The
(a) go on decreasing with time acceleration of the particle is
(b) be independent of α and β a+ b 2b
(a) (b)
(c) drop to zero when α = β 2 ( p + q) (q + 2 p) q
(d) go on increasing with time 2a a+ b
(c) (d)
p ( p + 2 p) ( p + q)2
24. Two boys are standing at the ends A and B on ground,
where AB = a. The boy at B starts running in a direction 29. A rocket is fired vertically up from the ground with a
perpendicular to AB with velocity v1 . The boy at A starts resultant acceleration of 10 m/ s 2 . The fuel is finished in
running simultaneously with constant velocity v and
catches the other boy in a time t, where t is 1 min and it continues to move up ( g = 10 m/ s 2 )
(a) the maximum height reached by rocket from ground is18 km
a a2
(a) (b) (b) the maximum height reached by rocket from ground is
v 2 + v12 v − v12
2
36 km
a a (c) the time from initial in which rocket is again at ground is
(c) (d)
(v − v1 ) (v + v1 ) (180 + 30 2 ) s
(d) the time from initial in which rocket is again at ground is
25. A man is, d distance behind a bus. The bus moves away (120 + 60 2 ) s
from the man with an acceleration a. At the same time,
man starts running towards bus with a constant velocity v. dv
30. Equation of motion of a body is = − 4v + 8, where v is
(a) The man catches the bus, if v ≥ 2 ad dt
(b) If man just catches the bus the time of catching bus will be the velocity in m/ s and t is the time in second. Initial
v velocity of the particle was zero. Then,
t=
a (a) the initial rate of change of acceleration of the particle is
(c) If man just catches the bus, the time of catching bus will be 8 m / s3
2v
t= (b) the terminal speed is 2 m / s
a
(c) Both (a) and (b) are correct
(d) The man will catch the bus, if v ≥ ad
(d) Both (a) and (b) are wrong
102 Objective Physics Vol. 1

31. Among the four graph shown in the figure there is only 2. At time t = 0, a car moving along a straight line has a
one graph for which average velocity over the time velocity of 16 ms −1 . It slows down with an acceleration
interval ( 0, T ) can vanish for a suitably chosen T. Which
of − 0.5 t ms −2 , where t is in second. Mark the correct
one is it?
statement (s).
x x (a) The direction of velocity changes at t = 8 s
(b) The distance travelled in 4 s is approximately 58.67 m
(a) t (b) (c) The distance travelled by the particle in 10 s is 94 m
(d) The speed of particle at t = 10 s is 9 ms−1
t
x
3. An object moves with constant acceleration a. Which of
x
the following expressions are also constant ?
(b)  
d | v| dv
(c) (d) (a)
dt  dt 
 v
t t d 
d (v 2 )  | v|
(c) (d)
32. A lift is coming from 8th floor and is just about to reach dt dt
4th floor. Taking ground floor as origin and positive
4. Ship A is located 4 km north and 3 km east of ship B. Ship
direction upwards for all quantities, which one of the
A has a velocity of 20 kmh −1 towards the south and ship
following is correct?
(a) x < 0, v < 0, a > 0 (b) x > 0, v < 0, a < 0
B is moving at 40 kmh −1 in a direction 37° north of east.
(c) x > 0, v < 0, a > 0 (d) x > 0, v > 0, a < 0 X and Y -axes are along east and north directions,
respectively.
33. A bird is tossing (flying to and fro) between two cars
(a) Velocity of A relative to B is − 32i$ − 44 $j
moving towards each other on a straight road. One car
has a speed of 18 km/h while the other has the speed of (b) Position of A relative to B as a function of time is given by
27 km/h. The bird starts moving from first car towards rA B = (3 − 32 t ) $i + (4 − 44 t ) $j
the other and is moving with the speed of 36 km/h and (c) Velocity of A relative to B is 32$i − 44 $j
when the two cars were separated by 36 km. What is the (d) Position of A relative to B as a function of time is given by
total distance covered by the bird? (32 t $i − 44 t $j)
(a) 28.8 km (b) 36.4 km
(c) 58.2 km (d) None of these 5. Starting from rest a particle is first accelerated for time t 1
34. A motor car moving at a speed of 72 km/h cannot come to with constant acceleration a1 and then stops in time t 2
a stop in less than 3.0 s while for a truck this time interval with constant retardation a 2 . Let v1 be the average
is 5.0 s. On a highway, the car is behind the truck both velocity in this case and s1 the total displacement. In the
moving at 72 km/h. The truck gives a signal that it is second case, it is accelerating for the same time t 1 with
going to stop at emergency. At what distance the car constant acceleration 2a1 and come to rest with constant
should be from the truck so that it does not bump onto retardation a 2 in time t 3 . If v 2 is the average velocity in
(collide with) the truck. Human response time is 0.5 s. this case and s2 the total displacement, then
(a) 6.75 m (b) 1.25 m (a) v2 = 2 v1
(c) 4.25 m (d) None of these (b) 2 v1 < v2 < 4 v1
(c) s2 = 2 s1
More than One Correct Options (d) 2 s1 < s2 < 4 s1

1. A particle having a velocity v = v 0 at t = 0 is decelerated 6. A particle is moving along a straight line. The
displacement of the particle becomes zero in a certain
at the rate | a | = α v, where α is a positive constant. time ( t > 0). The particle does not undergo any collision.
2 v0 (a) The acceleration of the particle may be zero always
(a) The particle comes to rest at t =
α (b) The acceleration of the particle may be uniform
(b) The particle will come to rest at infinity (c) The velocity of the particle must be zero at some instant
(c) The distance travelled by the particle before coming to rest (d) The acceleration of the particle must change its direction
2v 3/ 2
is 0 7. A particle is resting over a smooth horizontal floor. At
α
t = 0, a horizontal force starts acting on it. Magnitude of
(d) The distance travelled by the particle before coming to rest
2v 3/ 2
the force increases with time according to law F = αt,
is 0 where α is a positive constant.

Motion in One Dimension 103

From figure, which of the following statements are 11. A man who can swim at a velocity v relative to water
correct ? wants to cross a river of width b, flowing with a speed u.
y
b
(a) The minimum time in which he can cross the river is
2 v
1
(b) He can reach a point exactly opposite on the bank in time
b
t= if v > u
v − u2
2

(c) He cannot reach the point exactly opposite on the bank if


o x u>v

(a) Curve 1 can be the plot of acceleration against time (d) He cannot reach the point exactly opposite on the bank if
v>u
(b) Curve 2 can be the plot of velocity against time
v
(c) Curve 2 can be the plot of velocity against acceleration 12. The figure shows the
velocity ( v ) of a particle
(d) Curve 1 can be the plot of displacement against time T
plotted against time (t). O t
2T
8. A train starts from rest at S = 0 and is subjected to an (a) The particle changes its
direction of motion at some
acceleration as shown in figure.
point
a (ms–2) (b) The acceleration of the particle remains constant
(c) The displacement of the particle is zero
6
(d) The initial and final speeds of the particle are the same
13. The speed of a train increases at a constant rate α from
zero to v and then remains constant for an interval and
30
S (m) finally decreases to zero at a constant rate β. The total
distance travelled by the train is l. The time taken to
(a) Velocity at the end of 10 m displacement is 20 ms−1 complete the journey is t. Then,
(b) Velocity of the train at S = 10 m is 10 ms−1 l (α + β )
(a) t =
(c) The maximum velocity attained by train is 180 ms −1 αβ
(d) The maximum velocity attained by the train is 15 ms−1 l v  1 1
(b) t = +  + 
v 2  α β
9. For a moving particle, which of the following options 2lαβ
may be correct? (c) t is minimum when v =
(α − β )
(a) | vav | < vav
2lαβ
(b) | vav | > vav (d) t is minimum when v =
(α + β )
(c) vav = 0 but vav ≠ 0
(d) vav ≠ 0 but vav = 0 14. A particle moves in xy - plane and at time t is at the point
( t 2 , t 3 − 2 t ), then which of the following is/are correct?
10. Identify the correct graph representing the motion of a
particle along a straight line with constant acceleration (a) At t = 0, particle is moving parallel to y -axis
with zero initial velocity (b) At t = 0, direction of velocity and acceleration are
perpendicular
v v 2
(c) At t = , particle is moving parallel to x -axis
3
(d) At t = 0, particle is at rest
(a) (b) 15. A car is moving with uniform acceleration along a
t
t
straight line between two stops X and Y . If its speeds at X
0 0 and Y are 2 ms −1 and 14 ms −1 , then
x x (a) its speed at mid-point of XY is 10 ms−1
(b) its speed at a point A such that XA : AY = 1 : 3 is 5 ms−1
(c) the time to go from X to the mid-point of XY is double of that
(c) (d) to go from mid-point to Y
t (d) the distance travelled in first half of the total time is half of
t
0 0 the distance travelled in the second half of the time
104 Objective Physics Vol. 1

16. A graph of x versus t is shown in figure. Choose the 2. The maximum height reached by ball, as measured from
correct alternatives given below the ground would be
x (a) 73.65 m (b) 116.25 m
(c) 82.56 m (d) 63.25 m
B
A
C
3. Displacement of ball with respect to ground during its
flight would be
(a) 16.25 m (b) 8.76 m
(c) 20.24 m (d) 30.56 m
D t
4. The maximum separation between the floor of elevator
(a) The particle was released from rest at t = 0 and the ball during its flight would be
(b) At B, the acceleration a > 0 (a) 12 m (b) 15 m
(c) Average velocity for the motion between A and D is positive (c) 9.5 m (d) 7.5 m
(d) The speed at D exceeds that at E
Passage II (Q.5 to 7)
17. A man is standing on top of a building 100m high. He A situation is shown in which two objects A and B start
throws two balls vertically, one at t = 0 and after a time their motion from same point in same direction. The
interval (less than 2 s). The later ball is thrown at a graph of their velocities against time is drawn. u A and
velocity of half the first. The vertical gap between first u B are the initial velocities of A and B respectively. T is
and second ball is + 15 m at t = 2 s. The gap is found to the time at which their velocities become equal after start
remain constant. Select the correct alternatives. of motion. You cannot use the data of one question while
(a) The initial speeds of two balls (in m/s) are 10 and 20 solving another question of the same set. So, all the
(b) The initial speeds of two balls (in m/s) are 5 and 10 questions are independent of each other.
(c) The time interval between their throw is 0.5 s
(d) The time interval between their throw is 1 s
Velocity of A
uB
18. Following are four different relations about
displacement, velocity and acceleration for the motion of
a particle in general. Choose the incorrect one (s). uA
Velocity of B
1
(a) vav = [ v (t1 ) + v (t2 )]
2 t
r(t ) − r(t1 ) T
(b) vav = 2
t2 − t1
5. If the value of T is 4 s, then the time after which A will
1
(c) r = (v (t2 ) − v (t1 ) (t2 − t1 ) meet B is
2
v (t ) − v (t1 ) (a) 12 s
(d) a av = 2 (b) 6s
t2 − t1
(c) 8s
(d) data insufficient
Comprehension Based Questions
6. Let v A and v B be the velocities of the particles A and B
Passage I (Q.1 to 4)
An elevator without a ceiling is ascending up with an respectively at the moment A and B meet after start of the
motion. If u A = 5 ms −1 and u B = 15 ms −1 , then the
acceleration of 5 ms −2 . A boy on the elevator shoots a
ball in vertical upward direction from a height of 2 m magnitude of the difference of velocities v A and v B is
above the floor of elevator. At this instant the elevator is (a) 5 ms−1
10 ms−1
moving up with a velocity of 10 ms −1 and floor of the
(b)
(c) 15 ms−1
elevator is at a height of 50 m from the ground. The
(d) data insufficient
initial speed of the ball is 15 ms −1 with respect to the
elevator. Consider the duration for which the ball strikes, 7. After 10 s of the start of motion of both objects A and B,
the floor of elevator in answering following questions: find the value of velocity of A, if u A = 6 ms −1 ,
( g = 10 ms −2 ) u B = 12 ms −1 and at T velocity of A is 8 ms −1 and T = 4 s
1. The time in which the ball strikes the floor of elevator is (a) 12 ms−1
given by (b) 10 ms−1
(a) 2.13 s (b) 2.0 s (c) 15 ms−1
(c) 1.0 s (d) 3.12 s (d) None of the above
Motion in One Dimension 105

Assertion and Reason 7. Assertion Plotting the acceleration-time graph from a


Directions (Q. Nos. 1-20) These questions consists of two given position-time graph of a particle moving along a
straight line is possible.
statements each printed as Assertion and Reason. While
answering these question, you are required to choose any one
Reason From position-time graph, sign of acceleration
can be determined.
of the following five responses
(a) If both Assertion and Reason are correct and Reason is 8. Assertion The v-t graph perpendicular to time axis is not
the correct explanation of Assertion possible in practice.
(b) If both Assertion and Reason are correct but Reason is Reason Infinite acceleration cannot be realized in practice.
not the correct explanation of Assertion 9. Assertion Magnitude of average velocity is equal to
(c) If Assertion is true but Reason is false average speed, if velocity is constant.
(d) If Assertion is false but Reason is true Reason If velocity is constant, then there in no change in
(e) If both Assertion and Reason are false. the direction of motion.
1. Assertion A body is moving along a straight line such
10. Assertion The average velocity of a particle having
that its velocity varies with time as shown in figure.
initial and final velocity v1 and v2 is v1 + v2 / 2.
Magnitude of displacement of the body from t = 0 to
t = 12s is the same as the distance travelled by it in the Reason If r1 and r 2 be the initial and final displacement
given time duration. r − r2
in time t, then vav = 1 .
v t
11. Assertion If a particle is thrown upwards, then distance
travelled in last second of upward journey is independent
of the velocity of projection.
t (s) Reason In last second, distance travelled is 4.9 m.
2 4 6 8 10 12
(Take g = 9.8 m/ s 2 )
Reason For unidirectional motion of a body, 12. Assertion If acceleration of a particle moving in a
displacement = distance straight line varies as a ∝ t n , then
2. Assertion An object may have varying speed without s∝tn+2
having varying velocity.
Reason If a-t graph is a straight line,then s-t graph may
Reason If the velocity is zero at an instant, the be a parabola.
acceleration may not be zero at that instant.
13. Assertion Distance between two particles moving with
3. Assertion A body of mass 4 kg has an initial velocity constant velocities always remains constant.
5$i m / s. It is subjected to a force of 4 $j N. The Reason In the above case relative motion between them
displacement of the body from the origin after 4 s will be is uniform.
215. m.
14. Assertion Particle A is moving eastwards and particle B
Reason The equation v = u + a t can be applied to obtain northwards with same speed. Then, velocity of A with
v, if a is constant. respect to B is in south-east direction.
4. Assertion A body is momentarily at rest at the instant it Reason Relative velocity between them is zero as their
reverses the direction. speeds are same.
Reason A body cannot have acceleration if its velocity is 15. Assertion In the v-t diagram shown in figure, average
zero at a given instant of time. velocity between the interval t = 0 and t = t 0 is
5. Assertion On a curved path, average speed of a particle independent of t 0 .
can never be equal to average velocity. v
Reason Average speed is total distance travelled divided vm
by total time. Whereas average velocity is final velocity,
plus initial velocity divided by two.
6. Assertion Acceleration of a moving particle can change
its direction without any change in direction of velocity. t
t0
Reason If the direction of change in velocity vector
1
changes, the direction of acceleration vector also Reason Average velocity in the given interval is vm .
changes. 2
106 Objective Physics Vol. 1

16. Assertion In the equation, 3. For the velocity-time graph shown in the figure, in a time
1 interval from t = 0 to t = 6 s, match the
s = u + at − a
2 following columns.
v (m/s)
s is the distance travelled in t th second. 10
Reason The above equation is dimensionally incorrect.
s 2 4 6 t (s)
17. Assertion In the s-t graph shown
in figure, velocity of particle is
negative and acceleration is
positive.
Reason Slope of s-t graph is t
negative and increasing in Column I Column II
magnitude. (A) Change in velocity (p) − 5/3 SI unit
(B) Average acceleration (q) − 20 SI unit
18. Assertion A lift is ascending with decreasing speed
means acceleration of lift is downwards. (C) Total displacement (r) − 10 SI unit

Reason A body always moves in the direction of its (D) Acceleration at t = 3 s (s) − 5 SI unit
acceleration.
4. Let us cell a motion, A when velocity is positive and
s
19. Assertion In the s-t diagram increasing A −1 when velocity is negative and increasing.
shown in figure, the body starts R when velocity is positive and decreasing and R −1 when
in positive direction but not velocity is negative and decreasing. Now, match the
form s = 0. t0
t following two columns for the given s-t graph.
Reason At t = t 0 , velocity of s
body changes its direction of
motion.
N P
20. Assertion If velocity-time equation of a particle moving M Q
in a straight line is quadratic, then displacement-time t
equation can’t be linear.
Column I Column II
Reason If displacement-time is quadratic, then
velocity-time is linear. (A) M (p) A −1
(B) N (q) R −1
Match the Columns (C) P (r) A
(D) Q (s) R
1. Match the following columns.
Column I Column II 5. In the s-t equation ( s = 10 + 20 t − 5t 2 ), match the
(A) d v / dt (p) Acceleration following columns.
(B) d| v|/ dt (q) Magnitude of acceleration Column I Column II
(C) d r/dt (r) Velocity (A) Distance travelled in 3 s (p) −20 units
(D) d r (s) Magnitude of velocity (B) Displacement in 1 s (q) 15 units
dt (C) Initial acceleration (r) 25 units
(D) Velocity at 4 s (s) −10 units
2. Velocity of a particle is in negative direction with
constant acceleration in positive direction. Then, match 6. A particle is rotating in a circle of radius 1 m with
the following columns. constant speed 4 m/ s. In time 1s, match the following
Column I Column II (in SI units) columns.
(A) Velocity-time graph (p) Slope → negative Column I Column II
(A) Displacement (p) 8 sin 2
(B) Acceleration-time graph (q) Slope → positive
(B) Distance (q) 4
(C) Displacement-time graph (r) Slope → zero
(C) Average velocity (r) 2 sin 2
(s) |Slope | → increasing (D) Average acceleration (s) 4 sin 2
Motion in One Dimension 107

7. Match the following columns. Entrance Gallery


Column I Column II
2014
(A) Constant positive (p) Speed may increase
acceleration 1. A rocket is moving in a gravity free space with a constant
(B) Constant negative (q) Speed may decrease acceleration of 2 ms − 2 along + x-direction (see figure).
acceleration The length of a chamber inside the rocket is 4 m. A ball is
(C) Constant displacement (r) Speed is zero thrown from the left end of the chamber in + x -direction
(D) Constant slope of a-t graph (s) Speed must increase with a speed of 0.3 ms − 1 relative to the rocket. At the
same time, another ball is thrown in − x-direction with a
8. A balloon rises, up with constant net acceleration of speed of 0.2 ms − 1 from its right end relative to the
10 m/ s 2 . After 2s a particle drops from the balloon. After rocket. The time (in seconds) when the two balls hit each
further 2s match the following columns. (Take other is [JEE Advanced]
g = 10 m/ s 2 )
a = 2 ms–2
Column I Column II 0.3 ms–1 0.2 ms–1
x
(A) Height of particle from ground (p) Zero 4m
(B) Speed of particle (q) 10 SI unit
(a) 2 or 8 (b) 3
(C) Displacement of particle (r) 40 SI unit (c) 6 (d) 9
(D) Acceleration of particle (s) 20 SI unit
2. From a tower of height H , a particle is thrown vertically
9. Refer to the graph in figure. Match the following upwards with a speed u. The time taken by the particle, to
hit the ground, is n times that taken by it to reach the
Column I Column II highest point of its path. The relation between H , u and n is
(A) x (p) has v > 0 and a < 0 [JEE Main]
C
throughout (a) 2 gH = n2u2
(b) gH = (n − 2)2 u2
D
O t (c) 2 gH = nu2 (n − 2)
B
(d) gH = (n − 2) u2
A
3. The ratio of distance traversed in successive intervals of
time when a body falls freely under gravity from certain
(B) x (q) has x > 0 throughout height is [Kerala CEE]
and has a point with (a) 1 : 2 : 3 (b) 1 : 5 : 9
B1 v = 0 and a point with
C1 a=0
(c) 1 : 3 : 5 (d) 1 : 2 : 3
A1
D1 (e) 1 : 4 : 9
O t 4. A ball is dropped from the top of a tower of height 100 m
and at the same time another ball is projected vertically
upwards from ground with a velocity 25 ms −1 . Then, the
(C) x (r) has a point with zero distance from the top of the tower at which the two balls
displacement for t > 0
meet, is [Kerala CEE]
(a) 68.4 m (b) 48.4 m
(c) 18.4 m (d) 28.4 m
(e) 78.4 m
O t
5. A particle starting with certain initial velocity and
(D) x (s) has v < 0 and a > 0 uniform acceleration covers a distance of 12 m in first 3 s
and a distance of 30 m in next 3 s. The initial velocity of
the particle is [Kerala CEE]
(a) 3 ms−1 (b) 2.5 ms−1
(c) 2 ms−1 (d) 1.5 ms−1
O t (e) 1 ms−1
108 Objective Physics Vol. 1

6. A bullet when first into a target loses half of its velocity 2012
after penetrating 20 cm. Further distance of penetration
14. A person throws balls into air vertically upward in
before it comes to rest is [Kerala CEE]
regular intervals of time of 1 s. The next ball is thrown
(a) 6.66 cm (b) 3.33 cm
(c) 12.5 cm (d) 10 cm when the velocity of the ball thrown earlier becomes
(e) 5 cm zero. The height to which the balls rise is (assume,
g = 10 m /s 2 ) [Karnataka CET]
7. A car moves from A to B with a speed of 30 km/h and
(a) 20 m (b) 5 m
from B to A with a speed of 20 km/h. What is the average (c) 10 m (d) 7.5 m
speed of the car? [Karnataka CET]
(a) 25 km/h (b) 24 km/h 15. A ball is dropped from the roof of a tower of height h. The
(c) 50 km/h (d) 10 km/h total distance covered by it in the last second of its motion
is equal to the distance covered by it in first 3 s. What is
8. A body starts from rest and moves with constant the value of h ? [OJEE]
acceleration for t. It travels a distance x1 in first half of
(a) 125 m (b) 5 m
time and x 2 in next half of time, then [Karnataka CET] (c) 58 m (d) 250 m
(a) x2 = x1 (b) x2 = 2 x1
(c) x2 = 3 x1 (d) x2 = 4 x1 16. The velocity of a particle is given by v = 180 − 16x m /s.
9. A particle moves with constant acceleration along a Its acceleration will be [OJEE]
straight line starting from rest. The percentage increase in (a) − 8 m /s2
its displacement during the 4th second compared to that (b) − 6 m /s2
in the 3rd is [WB JEE] (c) 0.4 m/s2
(a) 33% (b) 40% (d) 5 m /s2
(c) 66% (d) 77%
10. A cricket ball thrown across a field is at heights h1 and h2 17. A body moves in a plane, so that the displacement along X
from the point of projection at times t 1 and t 2 and Y-axes is x = 3t 2 and y = 4t 2 , then velocity is [OJEE]
respectively after the throw. The ball is caught by a (a) 15 t 2
fielder at the same height as that of projection. The time (b) 20 t 2
of the ball in this journey is [WB JEE] (c) 5 t 2
h1t22 − h2t12 h1t12 + h2t22 (d) None of the above
(a) (b)
h1t2 − h2t1 h2t1 + h1t2 18. A particle is travelling along a straight line OX . The
h t 2 + h2t12 h t 2 − h2t22 distance x (in metre) of the particle from O at a time t is
(c) 1 2 (d) 1 1
h1t2 + h2t1 h1t1 − h2t2 given by x = 37 + 27t − t 3 , where t is time in second.
The distance of the particle from O when it comes to
2013 rest is [WB JEE]
11. A particle of mass 0.2 kg is moving in one dimension (a) 81 m
under a force that delivers a constant power 0.5 W to the (b) 91 m
particle. If the initial speed (in ms −1 ) of the particle is (c) 101 m
(d) 111 m
zero, then the speed (in ms − 1 ) after 5 s is [JEE Advanced]
(a) 4 m/s (b) 2 m/s 19. A bullet on penetrating 30 cm into its target loses its
(c) 1 m/s (d) 5 m/s velocity by 50%. What additional distance will it
12. Two bullets are fired simultaneously, horizontally and penetrate into the target before it comes to rest? [WB JEE]
with different speeds from the same place. Which bullet (a) 30 cm
will hit the ground first? [Karnataka CET, OJEE] (b) 20 cm
(c) 10 cm
(a) The faster one
(d) 5 cm
(b) Depends on their mass
(c) The slower one 20. From the top of a tower, 80 m high from the ground, a
(d) Both will reach simultaneously stone is thrown in the horizontal direction with a
13. The bus moving with a speed of 42 km/h is brought to a velocity of 8 ms − 1 . The stone reaches the ground after a
stop by brakes after 6 m. If the same bus is moving at a time t and falls at a distance of d from the foot of the
speed of 90 km/h, then the minimum stopping distance is tower are [WB JEE]
[J&K CET] (a) 6 s, 64 m (b) 6 s, 48 m
(a) 15.48 m (b) 18.64 m (c) 22.13 m (d) 27.55 m (c) 4 s, 32 m (d) 4 s, 16 m
Motion in One Dimension 109

2011 24. A particle crossing the origin of coordinates at time t = 0,


21. An object moving with a speed of 6.25 m/s is decelerated moves in the xy - plane with a constant acceleration a in
dv the y-direction. If its equation of motion is y = bx 2 (b is a
at a rate given by = −2.5 v , where v is the
dt constant), its velocity component in the x-direction is
instantaneous speed. The time taken by the object, to 2b a
(a) (b) [Kerala CEE]
come to rest, would be [AIEEE] a 2b
(a) 2 s (b) 4 s a b
(c) 8 s (d) 1 s (c) (d)
b a
22. A bus begins to move with an acceleration of 1 ms −2 . A (e) ab
man who is 48 m behind the bus starts running at 10 ms −1 25. A car moves a distance of 200 m. It covers first half of the
to catch the bus. The man will be able to catch the bus distance at speed 60 kmh −1 and the second half at speed
after [Kerala CEE] v. If the average speed is 40 kmh −1 , then value of v is
(a) 6 s (b) 5 s (c) 3 s (d) 7 s
(a) 30 kmh −1 (b) 13 kmh −1 [Karnataka CET]
(e) 8 s
(c) 60 kmh −1 (d) 40 kmh −1
23. A particle is moving with constant acceleration from A to
B in a straight line AB. If u and v are the velocities at A 26. The displacement-time graphs of two moving particles
and B respectively. Then, its velocity at the mid-point C make angles of 30º and 45º with the X-axis. The ratio of
will be [Kerala CEE] their velocities is [Karnataka CET]
2
 u2 + v 2 
(a)  
 2u 

Displacement
u+ v
(b)
2
v−u
(c)
2 45º
u +v2 2
30º
(d) O
2 Time

v −u2 2
(a) 1 : 3 (b) 1 : 2
(e)
2 (c) 1 : 1 (d) 3 : 2
Answers
Level 1
Objective Problems

1. (d) 2. (d) 3. (a) 4. (a) 5. (c) 6. (d) 7. (c) 8. (c) 9. (c) 10. (b)
11. (a) 12. (a) 13. (b) 14. (c) 15. (c) 16. (a) 17. (c) 18. (a) 19. (b) 20. (b)
21. (d) 22. (a) 23. (a) 24. (b) 25. (d) 26. (a) 27. (b) 28. (c) 29. (b) 30. (c)
31. (a) 32. (b) 33. (c) 34. (a) 35. (a) 36. (d) 37. (d) 38. (b) 39. (b) 40. (b)
41. (b) 42. (d) 43. (a) 44. (a) 45. (c) 46. (b) 47. (c) 48. (b) 49. (d) 50. (a)
51. (b) 52. (b) 53. (b) 54. (c) 55. (b) 56. (c) 57. (b) 58. (d) 59. (b) 60. (c)
61. (a,d) 62. (c) 63. (b) 64. (b) 65. (b) 66. (d) 67. (a) 68. (b) 69. (a,b,c) 70. (c)
71. (c) 72. (c) 73. (b) 74. (c) 75. (b) 76. (d) 77. (c) 78. (b) 79. (a,b,c,d) 80. (a)
81. (a) 82. (c) 83. (d) 84. (d) 85. (b) 86. (a) 87. (a) 88. (b) 89. (d) 90. (d)
91. (a) 92. (b) 93. (c) 94. (d) 95. (b) 96. (b) 97. (d) 98. (c) 99. (a) 100. (a)
101. (c) 102. (a) 103. (b) 104. (a) 105. (a) 106. (b) 107. (d) 108. (b) 109. (a) 110. (c)
111. (c) 112. (b) 113. (d) 114. (d) 115. (a) 116. (a) 117. (b) 118. (d) 119. (a) 120. (c)
121. (d) 122. (b)

Level 2
Only One Correct Option

1. (a) 2. (a,b) 3. (c) 4. (c) 5. (b) 6. (c) 7. (d) 8. (b) 9. (b) 10. (d)
11. (a) 12. (b) 13. (a) 14. (b) 15. (a) 16. (b) 17. (b) 18. (c) 19. (a) 20. (b)
21. (b) 22. (d) 23. (d) 24. (b) 25. (a,b) 26. (d) 27. (b) 28. (b) 29. (b,d) 30. (b)
31. (a) 32. (a) 33. (a) 34. (b)

More than One Correct Options


1. (a,d) 2. (all) 3. (b) 4. (a,b) 5. (a,d) 6. (b,c) 7. (a,b) 8. (b,c) 9. (a,c) 10. (a,d)
11. (a,b,c) 12. (all) 13. (b,d) 14. (a,b,c) 15. (a,c) 16. (a,c,d) 17. (a,d) 18. (a,c)

Comprehension Based Questions


1. (a) 2. (c) 3. (d) 4. (c) 5. (c) 6. (b) 7. (d)

Assertion and Reason


1. (a) 2. (d) 3. (b) 4. (c) 5. (c) 6. (a, b) 7. (d) 8. (a) 9. (a) 10. (d)
11. (a) 12. (b) 13. (d) 14. (c) 15. (a) 16. (e) 17. (d) 18. (c) 19. (c) 20. (b)

Match the Columns


1. (A → p, B → t, C → r, D → s) 2. (A → q, u, B → r, u, C → p, t) 3. (A → r, B → p, C → r, D → s)
4. (A → r, B → s, C → p, D → q) 5. (A → r, B → p, C → s, D → p) 6. (A → r, B → q, C → r, D → p)
7. (A → p, q, B → p, q, C → r, D → p, q) 8. (A → r, B → p, C → s, D → q) 9. (A → r, B → q, C → s, D → p)

Entrance Gallery
1. (a) 2. (c) 3. (c) 4. (e) 5. (e) 6. (a) 7. (b) 8. (c) 9. (b) 10. (a)
11. (a) 12. (d) 13. (d) 14. (b) 15. (a) 16. (a) 17. (d) 18. (b) 19. (c) 20. (a)
21. (a) 22. (e) 23. (d) 24. (b) 25. (a) 26. (a)
Solutions
Level 1 : Objective Problems Total time = t1 + 2t and total displacement = 2s
Total displacement
v f − vi ( 2i$ − 3j$ ) − ( 2i$ + 3j$ ) − 6j$ Mean velocity =
1. a = = = = − 3j$ m/s2 Total time
∆t 2 2 2s
Displacement 2R 2v 20 =
2. v av = = = = m/s ( s / v 0 ) + ( 2s / v1 + v 2 )
Time ( πR/v ) π π
2
Total displacement =
3. v av = (1 / v 0 ) + ( 2 / v1 + v 2 )
Total time 2v 0 (v1 + v 2 )
80t + 40 t =
= = 60 km/h ( 2v 0 + v1 + v 2 )
2t
11 × 18 + 42 × v
4. Distance from starting point = ( 3)2 + ( 4)2 + ( 5)2 12. 21 =
60
=5 2m ∴ v = 25.3 m/s
Displacement 2R c2
5. v av = = 13. v 2 = c1 − c2 x , comparing with v 2 = u2 + 2as , we have a = −
Time t 2
Total displacement 1 2 1
7. v av = 14. s = at = × (0.2)(64) = 6.4 m = 640 cm
Total time 2 2
1
( 4)2 + ( 3)2 1 15. h1 = × g × ( 2)2 = 2g
= = m/s 2
10 + 5 3
1
h 2 = × g × ( 4)2 − 2g = 6g
8. Horizontal distance covered by the wheel in half 2
revolution = πR 1
Final h 3 = × g × (6)2 − 8g = 10g
A' 2
∴ h1 : h 2 : h 3 = 1 : 3 : 5
2R 2h
16. t = or t ∝ h
g
A
πR 17. v = 2gh or v ∝ h
Initial
18. v 2 = 25x , comparing with v 2 = 2as , we have,
So, the displacement of the point which was initially in
a =12.5 m/s2
contact with ground = AA ′ ′ = ( πR )2 + ( 2R )2
19. Time taken to reach maximum height is 1 s.
=R π2 + 4 1
Height = free fall distance in1 s = g t 2 = 5 m
= π2 + 4 (as R =1 m) 2
9. Displacement of the particle will be zero, because if comes 20. v = u + at
back to its starting point. or − 2 = 10 + 4a
Average speed =
Total distance or a = − 3 m/s2
Total time 1 2
30 m 21. − h = vt − gt or gt 2 − 2vt − 2h = 0
= = 3 m/s 2
10 s
2v + 4v 2 + 8 gh
∴ t=
∆v | v f − v i |
10. aav = = 2g
∆t t
v 2hg 
= 1+ 1+ 2 
=
v 2 + v 2 − 2v.v.cosθ g  v 
t 22.
2v t1
= sin θ /2
t
11. v1t + v 2t = s t2
t1 = s/v0 v1 t v2t
t t
s
s
From figure, t1 < t2
s 1
∴ t= 23. 15 = 2t − × (01
. )t 2 or t =10s
v1 + v 2 2
112 Objective Physics Vol. 1

2u ds
24. t = 36. v = = 12 − 4t
g dt
1 Comparing with v = u + at , u = 12 m/s and a = − 4 m/s2
25. s = at 2
2 Velocity will become zero at 0 = 12 − 4t0 or t0 = 3 s.
26. s = s1 + s 2 + s 3 Since, the given time t = 5s is greater than t0 = 3 s
1 1 distance > displacement
= × 2 × (10)2 + ( 2)(10)( 30) + × 4 × ( 5)2
2 2 u2 1
Distance d = s 0 − t 0 + st 0 − t = + a (t − t0 )2
= 100 + 600 + 50 2 a 2
= 750 m (12)2 1
= + × 4 × ( 2)2 = 26 m
1 8 2
27. In first second distance travelled x = × g × t 2 = 5 m
2 dx
37. x = 2 + t − 3t , v =
2
= 1 − 6 t , velocity will become
(for g =10 m/s2 ) in last second 7x = 35 m dt
1
st th =  u + at − a  zero at time, 0 = 1 − 6 t0 or t0 = s,
1
Now,
 2  6
1
or
1
35 = 0 + 10 × t − × 10 Since, the given time t =1s is greater than t0 = s,
2 6
or t = 4s distance > displacement
28. 1 2 3 4 5 Displacement s = x f − x i
= ( 2 + 1 − 3) − ( 2 + 0 − 0) = − 2 m
Distance d = | s 0 − t 0 | + | st − t 0 |
5 u2 1
= + | a | (t − t0 )2
10 9 8 7 6 2| a | 2
29. Displacements of both should be equal. Or, Comparing v = 1 − 6t with v = u + ut,we have
1 u =1 m/s and a = − 6 m/s2
8 t = × 4 × t 2 or t = 4s
2 (1)2 1
2
+ × 6 × 1 − 
1
1 ∴ Distance =
30. s B = × 2 × (1) = 1 m
2
2×6 2  6
2
1 = 2.1 m
s A = × 2 × ( 2)2 = 4 m 1 2 2
2 38. 1 = g t1 or t1 =
∴ sA − sB = 3 m 2 g
u 1 2 4
31. = u − at0 2= gt or t =
4 2 g
3u
∴ a= but t2 = t − t1 =
4

2
4t0 g g
u 4
or = t0 t1 2/g
a 3 ∴ =
u 4 t2 4/g − 2/g
Now, 0 = u − at or t = = t0
a 3 2 2 (2 + 2)
= =
32. ( 30)2 = ( 20)2 + 2a ( 2s ) or 2as = 250 2− 2 2
Now, v 2 = ( 20)2 + 2as = 650 = ( 2 + 1)
∴ v = 25.5 m/s 39. x = 4 (t − 2) + a (t − 2)2
1 At t = 0, x = − 8 + 4a = 4a − 8
33. × ( a )( 2) = x or 2a = x
2
2 dx
1 1 v= = 4 + 2a (t − 2)
Now, ( a )( 4)2 − ( a )( 2)2 = 6a = 3x dt
2 2 At t = 0, v = 4 − 4a = 4 (1 − a )
34. After 2 s v = 4.9 × 2 = 9.8 d 2x
But acceleration, = 2a
1 dt 2
and h = × 4.9 × ( 2)2 = 9.8 m
2 40. v 2 = u2 + 2gh
v2
Greatest height, hmax = h + = 14.7 m ⇒ ( 3u)2 = ( − u)2 + 2gh
2g
4u2
1 ⇒ h= ↓ + ve
35. st = u + at − a g
2
1 41. Let h distance is covered in t second
or 0 + 10 t − 5 = × 10 × ( 3)2 1
2 ⇒ h = gt 2
or t = 5s 2
Motion in One Dimension 113

Distance covered in t th second =


1
g ( 2t − 1) x+a
47. t =
2 b
9h g ( x + a ) = bt 2
⇒ = ( 2t − 1) or
25 2
or x = − a + bt 2
From above two equations, h =122.5 m
Comparing this equation with general equation of
42. The initial velocity of airplane is horizontal, Hence, the 1
vertical component of velocity of packet will be zero. uniformly accelerated motion, s = s i + ut + at 2
2
1
∴ h = g t2 we see that s i = − a , u = 0 and acceleration = 2b.
2
48. Area under v-t graph gives displacement. We can see that
2h
So, t= area for B is greater than area of A. Hence, B has covered
g larger distance.
43. Hmax ∝ u2 v
B
∴ u ∝ Hmax
i.e. to triple the maximum height, ball should be thrown
with velocity 3 v 0 . A

44. v = an
t
a =  
v

 n
49. Let t second of upward journey = first t second of downward
an2 =   (n2 ) =
1 1 v vn
Now, Sn = journey (with zero initial velocity)
2 2  n 2
1
1 1 v  ∴Desired distance = gt 2
and Sn − 3 = a (n − 3) =   (n − 3)2
3
2
2 2  n
50. tABC =10s
∴ Displacement in last 3 s will be,
S = Sn − Sn − 3 ∴ tAB = 5s
v (n − 3)3  B B
= n − 
2 n 
5s 5s
6 n − 9 v
=  
 n 2
v A C
45. 40 = ( 20) a1,
∴ a1 = 2 m/s2 10.2 m
Further 40 = ( 40) a2 u
∴ a2 = 1 m/s2
Therefore, acceleration is 2 m/s2 and retardation1 m/s2 . At B, velocity becomes zero. Hence, at A velocity should be
1 1 50 m/s.
Now, s1 = a1t12 = × 2 × ( 20)2 = 400 m
2 2 Now, ( 50)2 = (u2 ) − 2 × 10 × 10.2
s 2 = v max t2 = 40 × 20 = 800 m ∴ u = 52 m/s
v2 ( 40)2
s 3 = max = = 800 m
2a2 2 ×1 51. Velocity of particle of this instant will be v = (u − gt1 )
Now, average velocity v
v t0 t0 t0 =
Total displacement
= g
Total time
400 + 800 + 800
= = 25 m/s
20 + 20 + 40
t1
(10)2
46. h = = 5m u
2g
h
2v
Now, the desired time interval will be or
g
10 m/s
2 (u − gt1 ) u 
= 2  − t1 
g g 
52. Total time taken in upward journey should be 5 s
∴ Total height = 2h = 10 m ∴ u = gt = 98
. × 5 = 49 m/s
114 Objective Physics Vol. 1

1 2 1
53. x1 = α t1 …(i) or h= gt1t2
2 2
1 2 1 2
x2 = β t2 …(ii) For free fall from rest h = gt
2 2
Further, v max = α t1 …(iii) ∴ t 2 = t1t2
and 0 = v max − β t2 …(iv) t = t1t2
From Eqs. (iii) and (iv), we have
dx
α t2 57. v = = 3at 2 + 2bt + C
= dt
β t1 dv
a= = 6at + 2b
Then, from Eqs. (i) and (ii), dt
x1 t1 β At t = 0, a = 2b
= =
x 2 t2 α
58. dv = a d t
54. Let vm be the maximum speed, v 2
∴ ∫ 2 dv = ∫ 0 ( 3t + 2t + 2) dt
2
vm = a1t1 and vm = 2a1 l

t2 =
2l
and vm = a2t3 = 2a2 ( 3l ) or v =18 m/s
vm 59. dv = a dt
l + 2l + 3l
Now, average speed v av = v t
t1 + t2 + t3
∴ ∫ 0 dv = ∫ 0 4t d t
6l v = 2t 2
v av = or
(vm /a1 ) + ( 2l /vm ) + (vm /a2 ) 3 3
6l Now, s = ∫ v d t = ∫ ( 2t 2 ) dt
= 0 0
 vm   2l   vm  2 2
 2 + + 2  = t = 18 m
 vm / 2l   vm   vm /6 l  3
6l 3v 60. dv = a d t
= = m
(10l / vm ) 5 2
or v = ∫ ( 3t + 4) dt = 14 m/s
v av 3 0
=
vm 5 61. x = 0 at t = 0 and t = α /β. So, the particle returns to starting
55. Average velocity in uniformly accelerated motion is given point at t = α /β
by, dx
1 v= = 2αt − 3βt 2
ut + at 2 dt
s 2 1
v av = = = u + at At t = 0,v = 0 i.e. initial velocity of particle is zero.
t t 2

Now, v1 = u + at1
1 v = 0 at t = 0 and t =
2 2β
1 Thus, the particle comes to rest after time
v 2 = (u + at1 ) + at2
2 t = 2α / 3β
1
and v 3 = u + a (t1 + t2 ) + at3 a=
dv
= 2α − 6 βt
2 dt
∴ (v1 − v 2 ): (v 2 − v 3 ) = (t1 + t2 ):(t2 + t3 )
At t = 0, a = 2a ≠ 0
56. Taking downward direction as the positive direction
u 62. y = a + bt + ct 2 − dt 4
dy
∴ v= = b + 2ct − 4dt 3
dt
dv
and a= = 2c − 12 dt 2
dt
Hence, at t = 0, v initial = b and ainitial = 2c
u h ds
63. v = = 12t − 3t 2
dt
Velocity is zero for t = 0
1 2
+ h = − ut1 + gt1 …(i) and t = 4s
2 ds
1 2 64. v = = 9t 2 + 14t + 14
+ h = ut2 + gt2 …(ii) dt
2 dv
Multiplying t2 by Eq. (i) and t2 by Eq. (ii) and adding, we get a= = 18t + 14
dt
1
h (t1 − t2 ) = gt1t2 (t1 + t2 ) At t =1s, a = 32 m/s2
2
Motion in One Dimension 115

65. v =
dx
= 32 − 8t 2 72. v = u + a t = ( 3$i + 4$j ) + (0.4$i + 0.3j$ )(10) = ( 7i$ + 7j$ )
dt
dv ∴ v = ( 7)2 + ( 7)2
v = 0 at t = 2s a= = −16 t
dt = 7 2 units
At 2s, a = − 32 m/s2 x
73. x = kt , t =
66. Integrate twice to convert a-t equation into s-t equation. k
y = k   1 − α ⋅ 
x x
67. a = bt Now,
 k  k
v t
∴ ∫v 0
dv = ∫ adt = ∫ bt ⋅ dt
a or y =x−
α x2
k
bt 2
∴ v = v0 + dx dv
2 74. v x = = 6t − 6, ax = x = 6 m/s2
dt dt
Further integrating we get, dv y
bt 3 v y = 2t − 2, a y = = 2 m/s2
s = v 0t + dt
6 At t =1s, v x and v y both are zero. Hence, net velocity is zero.
68. v = 3 αt 2 + 2 βt + γ 75. ux = 3 m/s, ax = − 6 m/s
vt = 0 = v i = γ
ux2 9
a = 6 αt + 2 β : at = 0 = ai = 2 β ∴ X max = = = 0.75 m
2 | ax | 12
vi γ
∴ = v f − vi
ai 2β 76. a =
t
dv
69. a = − 0.5 t or = − 0.5 t 77. v = u + a t = ( 3i$ + 4$j ) + (0.4i$ + 0.3j$ )(10) = ( 7i$ + 7j$ )
dt
v t ∴ v = ( 7)2 + ( 7)2
∴ ∫ 16 dv = ∫ − (0.5 t ) ⋅ dt
0
= 7 2 units
∴ v − 16 = − 0.25 t 2
78. r = ( a cos ωt ) i$ + ( a sin ωt ) j$
∴ v = 16 − 0.25 t 2 dr
∴v = = ( − aωsin ωt ) $i + ( aωcosωt ) $j
Velocity will change its direction at the instant when v = 0 dt
∴ 0 = 16 − 0.25 t 2 or t = 8s r⋅v =0
4 ∴ r⊥v
d 4 s = S4 s = ∫ v dt 79. Comparing with projectile motion,
0

r = (uxt ) i$ +  uyt − gt 2  j$
4 1
∫ 0 (16 − 0.25 t ) dt = 5866
. m ≈ 59 m
2
 2 
8 we have, ux = 3 m/s,uy = 4 m/s
Now, S8 s = ∫ (16 − 0.25 t 2 ) dt = 85.33 m
0 and g =10 m/s2
10
S10 s = ∫ (16 − 0.25 t 2 ) dt = 7667
. m 2u
T = y = 08 . s
0
g
0s 10 s 8s
(a) R = ux T = 2.4 m
A B C
(b) u = ux2 + u2y = 5 m/s
85.33 m
u   4
76.67 m (c) θ = tan −1  y  = tan −1  
 ux   3
d10s = AC + BC = AC + ( AC − BA )
= 2 AC − BA = 94 m (d) T = 0.8s
v10 s = 16 − (0.25)(10)2 = − 9 m/s 80. Velocity first decreases in upwards direction, then increases
dx 2t in downward direction.
70. v x = = =t
dt 2 81. Velocity will continuously increases, (starting from rest)
At 2s, v x = 2 m/s 82. In region B velocity is decreasing.
x 2 (t 2 / 2)2 t 4 83. Motion is first accelerated in positive direction, then
Further, y= = =
2 2 8 uniform in negative direction.
3
dy t 84. Acceleration in second case is more. Hence slope is more.
vy = =
dt 2 85. v max = Maximum positive area of a - t graph
At t = 2s, v y = 4 m/s = area between 0 and 8 s
71. v x = 4t ,v y = 2t − 4, v z = 3 = 30 m/s
At t = 0,v x = 0,v y = − 4 and v z = 3 86. Uniform motion means uniform velocity or constant slope
∴ v= v x2 + v 2y = 5 units of s -t graph.
116 Objective Physics Vol. 1

87. Displacement = positive area − negative area 100. Given line have positive intercept but negative slope. So, its
and distance = Σ(Areas) equation can be written as
88. For uniformly accelerate motion, v 2 = u2 + 2 as , i.e. v 2 versus  v0 
v = − mx + v 0  where,m = tanθ = x  …(i)
s graph is a straight line with interceptu2 and slope 2 a. Since,  0

intercept in non-zero, initial velocity is non-zero. By differentiating with respect to time, we get
16 dv dx
89. v 2 = u2 + 2 as , slope = 2a = − = − 8 m/s2 = −m = − mv
2 dt dt
or a = − 4 m/s2 Now, substituting the value of v from Eq. (i), we get
dv
90. a = v   = (10)  −  = −
dv 20 20 = − m [− mx + v 0 ]
m/s2
 ds   30  3 dt
91. v i = slope of s-t graph = tan 45° = 1 m/s = m2 x − mv 0
v f = slope of s-t graph = tan 60° = 3 m/s ∴ a = m2 x − mv 0
v f − vi
3 −1 i.e. the graph between a and x should have positive slope
Now aav = =
= ( 3 − 1) units
∆t 1 but negative intercept on a-axis. So, graph (a) is correct.
92. Change in velocity = net area under a -t graph 101. In first instant, you will apply v = tanθ and say
1
93.
15
a = slope of v-t graph = − = − 5 m/s2 v = tan 30° = m/s
3 3
94. Slope of B is more. Therefore, speed of B will be more. But it is wrong because formula v = tanθ is valid when angle
dx is measured with time axis.
95. v= = 2 m/s = constant
dt Here, angle is taken from displacement axis. So, angle from
v (m/s) time axis = 90° − 30° = 60°
Now, v = tan60° = 3

2 102. Since, total displacement is zero, hence average velocity is


also zero.
103. a-t equation from the given graph can be written as,
a = − 2t + 4
t (s)
or dv = a ⋅ dt = ( − 2t + 4) dt
Integrating we get, v = − t 2 + 4t
96. Distance = Area under v -t graph = A1 + A2 + A3 + A4
v-t equation is quadratic. Hence, v-t graph is a parabola.
104 . v AB = v A − v B
30
Velocity (m/s)

N
20

10 A1 A2
A3 A4 E

1 2 3 4
Time (second)
1 1
= × 1 × 20 + ( 20 × 1) + ( 20 + 10) × 1 + (10 × 1) vA vAB
2 2
= 10 + 20 + 15 + 10 = 55 m
97. Maximum acceleration means maximum change in
velocity in minimum time interval.
In time interval t = 30 to t = 40s,
∆v 80 − 20 60
a= = = = 6 cm/s2 α
∆t 40 − 30 10 –vB
1
98. Area of trapezium = × 36 . × (12 + 8) = 360
. m v AB + v A2 + v B2 = 5 km/h
2
99. For the given condition, initial height h = d and velocity of v 
α = tan −1  A 
the ball is zero. When the ball moves downward its velocity  vB 
increases and it will be maximum when the ball hits the
−1  3 
ground and just after the collision it becomes half and in = tan   = 37°
 4
opposite direction. As the ball moves upwards its velocity
again decreases and becomes zero at height d/2. This 105. For shortest time one should swim at right angles to river
explanation match with graph (a). current.
Motion in One Dimension 117

106. Velocity of rain is at 30° with vertical. So, its horizontal 1 2 u2


v and s2 = ft =
component is v R sin 30° = R . When man starts walking with 2 2f
2
v u2
10 km/h rain appears vertical. So, horizontal component R ∴ s max = s1 − s 2 =
2 2f
is balanced by his speed of10 km/h . Thus, s s
vR 118. v1 = ,v 2 =
= 10 or v R = 20 km/h 90 60
2 s s
40
. Now, t= =
107. v A + v B = 4 m/ s and v A − v B = = 0.4 m/s v1 + v 2 s
+
s
10 90 60
108. vr = 25 m/s 90 × 60
50 + 50 = = 36s
∴ t= = 4s 90 + 60
25
119. At 2 s
v 3
109. sinθ = r = 1
vbr 5 x A = 10 × 2 − × 4 × ( 2)2 =12 m
2
vr 1
and x B = 20 × 2 − × 2 × ( 2)2
2
vbr vb = 36 m
θ
∴ Distance between A and B at that instant is, 24 m.
120. Relative velocity of first train with respect to other
∴ θ = 37° = (v1 − v 2 )
The required angle is therefore 90° + θ = 127° v1 v2
v 12
110. tan60° = H or 3=
vV vV d

∴ vV = 4 3 km/h
(0) = (v1 − v 2 )2 − 2α s
111. Both the particles, will fall same distance in same time
where, α = relative retardation
interval.
So, the relative separation will remain unchanged. (v1 − v 2 )2
s=
112. Relative acceleration = 0, relative velocity is 40 m/s and 2α

relative separation is100 m. For avoiding collision,

∴ t=
100
= 2.5 s (v1 − v 2 )2
d>
40 2α
100
113. 7.2 = 121. vb =
1
= 4 km/h
(v + 5)( 5/18) 1/4
or v = 45 km/h
vr
114. Displacement of man = displacement of bus + 25
1
or 10t = × 2 × t 2 + 25 Net = vb
2 vbr
or 10t = t 2 + 25
Solving this equation, we get t = 5s.
115. Initially relative acceleration between them is zero, so vbr = 5 km/h
distance between them will increase linearly. Later when ∴ vr = 3 km/h
one stone strikes the ground relative acceleration is g so
122. v X = − v $j
distance will decrease parabolically.
v v N( ^j )
116. tanθ = H =
vV u
^
or θ = tan −1  
v W E( i )
 u
117. The greatest distance, when velocities of both are equal.
or ft = u
u S
∴ t=
f ∴ v Y = v YX + v X = − v$i + vj$
∴ |vY | = 2 v
u2
s1 = ut = Direction of v Y is north-west.
f
118 Objective Physics Vol. 1

7. Equation of line RS is y = − mx + C
Level 2 : Only One Correct Option
v f − vi v f − vi R
1. aav = = (as they are in opposite direction)
∆t ∆t
2ghf + 2ghi
=
∆t v2t P
2 × 10 × 5 + 2 × 10 × 10
=
0.01 45°
v1t
≈ 2414 m/s 2 O
S
dv a ⋅v
2. at = = rate of change of speed = a cosθ =
dt v v 
or y = −  2  x + v 2t
= component of acceleration along velocity responsible for  v1 
change in speed.
v or v1 y = − v 2 x + v1v 2t …(i)
3. Velocity of particle will become zero in time t0 = 0 Equation of line OP is
g
y =x …(ii)
1.5v 0 v
The given time t = is greater than the time t0 = 0 Point P is the point of intersection, we get
g g
v vt
Hence, distance > | displacement | xP = y P = 1 2
v1 + v 2
Distance = | S 0 − t 0 | + | St − t 0 |
v 02 1 ∴ OP = x P2 + y P2
= + g (t − t0 )2
2g 2 2 v1v 2t
=
5 v 02 v1 + v 2
=
8g OP 2 v1v 2
or = v3 =
4. − h = 5t − 5t 2 and − 2h = − 5t − 5t 2 t v1 + v 2
2S S S 2SV
O 5 m/s 8. T0 = and T = + =
V V + v V − v V 2 − v2
+ve
( 2SV /V 2 − v 2 )
∴ T / T0 =
5 m/s 2S/V
V2 1
2h O = =
V − v 2 1 − v 2 /V 2
2

dx
h 9. x 2 = t 2 + 1 or 2x ⋅ = 2t
dt
dx
or x⋅ =t
dt
From these two equations we get, t = 3s and h = distance dx t t
between P and Q = 30 m. ∴ = =
dx
dt x t2 + 1
5. = v = − 1 − 2t
dt t2
t2 + 1 −
Comparing with v = u + at , we have, u = −1 m/s and 2
d x t2 + 1
a = − 2 m/s2 =
(t 2 + 1)
dt 2
Att = 0, x =1 m. Then, u and a both are negative. Hence, 1 1
= 2 =
x-coordinate of particle will go on decreasing. (t + 1)3/ 2 x 3
d 2d
6. t = = Total displacement
vr cos30° 3 vr 10. Average velocity =
Total time
vr
25 – 2t
t t
30°
v = at
v = 5t
d  2d 
Now, drift = (v − vr sin 30° ) t = (v − vr /2)   1 1
× a × t 2 + ( at )( 25 − 2t ) + × a × t 2
2  3 vr  2 2
or 20 =
∴ 3 vr = 4v − 2vr 25
 4  Solving this equation with a = 5 m/s2 we get, t = 5 s
∴ vr =   v = 4 ( 2 − 3) v
 2+ 3 Thus, the particle moved uniformly for ( 25 − 2t ) or15 s.
Motion in One Dimension 119

2u v2 cx 2
11. Total time of journey is (t1 + t2 ). Therefore, = t1 + t2 …(i) Integrating we get, = bx −
g 2 2
1 2 ∴ v 2 = ( 2bx − cx 2 )
Further, h = ut1 − gt1 …(ii)
2 2b
(t + t2 ) t1 1 2 At other station v = 0 or x =
or h= g 1 − g t1 c
2 2
dv
or gt1t2 = 2h Further, v is maximum, where =0
dx
2h
∴ t1t2 = b
g or x= [From Eq. (i)]
c
12. 0 = u2 − 2a1 ( 4) − 2a2 (1) …(i) Substituting in Eq. (ii) we get,
0 = u − 2a2 ( 2) − 2a1 ( 2)
2
…(ii) b b2 b2
vm2 = 2b × − c × 2 =
From these two equations, we get, c c c
a2 = 2a1 or vm =
b
13. ( 50)2 = (10)2 + 2as …(i) c
a t
a
20. If t0 is the time during acceleration, then 0 will be the time
50 m/s 2
10 m/s during retardation.
s t
Now, t0 + 0 = t
Now, v = ( 50) + 2 ( − a )( − s ) = ( 50)2 + 2as
2 2
…(ii) 2
2t t t
Solving these two equations, we get ∴ t0 = and 0 =
v = 70 m/s 3 2 3
2 2
at 2
14. ForP , 30 = 15 + at s = a   + × 2a ×   =
1 2t 1 t
or at =15 m/s 2  3 2  3 3
For Q , v = 20 − at or v = 20 − 15 = 5 m/s 1 2
21. Taking, s = ut + at , from point B onwards we have,
2h 2
15. Time = and velocity = 2gh 1 1
g − at02 = ( at0 ) t − at 2
2 2
2h 2h
Now, t = − and v = 16gh − 4gh A B v = at +ve
2g 8g a 0

Dividing these two equations, we get


1 at 2
t 1 / 2g − 1 / 8g 2 0
=
v 8g − 2g
∴ t 2 − 2t0t − t02 = 0
2 −1 1
= = or v = 4gt 2t0 ± 4t02 + 4t02
8g − 4g 4g t= = (t0 + 2t0 )
2
16. vr is subtraction of vector. Hence,
∴ Total time = tAB + t = (2 + 2)t0
vr2 = x (say) = v 2 + ( at )2 − 2v ( at )cosα
22. Relative to lift
Now, vr will be minimum when x is minimum. Hence,
ur = 0,
dx
= 0 or 2a 2t − 2va cosα = 0
dt ar = ( 98
. + 1.2) = 11 m/s2
v cosα
∴ t= sr =
1
ar t 2
a 2
17. Slope of v-t graph gives acceleration/retardation
1
| a| = 5 m/s2 for both of them. A has acceleration while B has ∴ 2.7 = × 11 × t 2
2
retardation.
5.4
| s A | + | s B | = 60 or t= s
11
1 × 5 × t 2  + 20 t − 1 × 5 × t 2  = 60
or     23. Given, x = ae −αt + be βt
2   2 
dx
or t = 3s So, velocity v =
dt
18. Area of a-t graph gives change in velocity. When net area
will become zero, particle will acquire its initial velocity. = − aαe − αt + bβe βt
19. a = ( b − cx ) dv
a= = aα 2e − αt + bβ 2e βt
dv dt
or v⋅ = ( b − cx ) …(i) = − ve all the time.
dx
∴ v ⋅ dv = ( b − cx ) dx ∴ v will go on increasing.
120 Objective Physics Vol. 1

24. v 2t 2 = v12 t 2 + a 2 28. Let x be the acceleration. Then


1
a = xp2 …(i)
a2
∴ t= 2
v − v12
2
1
a + b = + x ( p + q )2 …(ii)
2
Solving these two equations, we get
vt 2b
x=
v1t ( q + 2p) q
29. At 60 s
A B
a 1
h1 = × 10 × (60)2 = 18000 m
25. Let us see the relative motion, distance moved by the man 2
before coming to rest
v1 = 10 × 60 = 600 m/s
v2
0 = v 2 − 2as or s = After that
2a
v12
Man Bus h2 = = 18000 m
2g
v Assume at
rest ∴ Hmax = h1 + h2 = 36000 m = 36 km
a
For time
d − 18000 = 600t − 5 t 2
Now, man can catch the bus if, or t 2 − 120 t − 3600 = 0
s ≥ d or v ≥ 2ad 120 + 14400 + 14400
or t=
If he just catches the bus, then 2
0 = v − at 120 + 120 2
= = 60 + 60 2
v 2
or t=
a ∴ Total time = 60 + (60 + 60 2 ) = (120 + 60 2 ) s
26. Car will acquire maximum speed i.e. 20 m/s in 20s with an dv
30. a = = − 4v + 8
acceleration of1 m/s2 .In retardation, it will take10s. Suppose dt
da dv
it moves with constant speed of 20 m/s of t second. Then, ∴ = − 4⋅ = − ( − 4v + 8)
v (m/s) dt dt
= 16 v − 32
 da  = − 32 m/s3 (as v i = 0)
 
 dt  t = 0
At terminal velocity a = 0
20 m/s ∴ v = 2 m/s
31. In graph (b) for one value of x
displacement there are two
t(s) different points of time. Hence, for
A B
20
one time, the average velocity is
20+t 30+t
positive and for other time is
Area of v-t graph = Total displacement equivalent negative.
1 As there are opposite velocities in
or [t + 30 + t ] × 20 = 1000 t
2 the interval 0 to T, hence average O T
velocity can vanish in (b). This can be seen in the figure
or t = 35 s
given
∴ Total time of journey = ( 30 + t ) = 65 s
Here, OA = BT (same displacement) for two different points
1 of time.
27. versus t equation from the given graph would be, x
v 32. As the lift is coming in downward
1  1  directions displacement will be 8th floor
= 3 +  −t ( y = − mx + c ) negative. We have to see whether the
v  3
motion is accelerating or retarding. x<0
1
or v= We know that due to downward
3 + 1  −t motion displacement will be negative.
 
 3 When the lift reaches 4th floor is about 4th floor
x<0 O
dv 1 to stop hence, motion is retarding in
∴ a= =
dt   2 nature hence, x < 0; a > 0. Ground floor
1  
 3 +  − t
3  
As displacement is in negative direction, velocity will also
 be negative i.e. v < 0.
Putting t = 3s we get, a = 3 m/s2 . This can be shown on the adjacent graph.
Motion in One Dimension 121

33. Given, speed of first car =18 km/h


More than One Correct Options
Speed of second car = 27 km/h
1. (a) a = − α v
∴ Relative speed of each car w.r.t. each other
dv
= 18 + 27 = 45 km/h ∴ = −α v
dt
Distance between the cars = 36 km t
1 0 −1/ 2
∴ Time of meeting the cars ( t ) =
Distance between the cars or ∫0 dt = − α v∫ v ⋅ dv
Relative speed of cars 0
36 4
= = h = 08
. h ∴ t=
2 v0
45 5 α
Speed of the bird (vb ) = 36 km/h (d) a = − α v
∴Distance covered by the bird = vb × t = 36 × 0.8 = 28.8 km dv
34. In this problem equations related to one dimensional ∴ v⋅ = −α v
ds
motion will be applied for acceleration positive sign will be s
1 0 1/ 2
used and for retardation negative sign will be used. ∴ ∫0 ds = − α v∫ v dv
Given, speed of car as well as truck = 72 km/h 0
5 2v 3/ 2
= 72 × m/s = 20 m/s ∴ s= 0
18 3α
Retarded motion for truck v = u + at t dv
2. a = − 0.5 t =
0 = 20 + at × 5 or a t = − 4 m/s 2 dt
v t
Retarded motion for the car v = u + ac t
20
∴ ∫ dv = ∫0 −0.5 t dt
0 = 20 + ac × 3 or a c = − m/s 2 16
3 ∴ v = 16 − 0.25t 2
Let car be at a distance x from truck, when truck gives the
0.25 t 3
signal and t be the time taken to cover this distance. s = ∫ vdt = 16t −
As human response time is 0.5 s, therefore, time of retarded 3
motion of car is (t − 0.5) s. v = 0 when 16 − 0.25 t 2 = 0
Velocity of car after time t, or t = 8s
v c = u − at = 20 −   ( t − 0.5)
20 So, direction of velocity changes at 8 s. upto 8 s
 3 (i.e. at 4 s),
Velocity of truck after time t, Distance = Displacement
vt = 20 − 4t ∴ At 4 s
To avoid the car bump onto the truck, 0.25 × ( 4)3
d = 16 × 4 − = 58.67 m
v c = vt 3
20
20 − ( t − 0.5) = 20 − 4t (0.25)(8)3
S8 s = 16 × 8 −
3 3
20 5 = 85.33 m
4t = ( t − 0.5) ⇒ t = (t − 0.5)
3 3 (0.25)(10)3
2.5 5 S10s = 16 × 10 −
3t = 5t − 2.5 ⇒ t = = s 3
2 4
= 76.67 cm
Distance travelled by the truck in time t,
1 85.33 m
st = ut t + at t 2 8s
2 0s
2
+ × ( −4) ×  
5 1 5
= 20 × 10 s
4 2  4
76.67 m
= 21875
. m
Distance travelled in 10s,
Distance travelled by car in time t = Distance travelled by
car in 0.5 s (without retardation) + Distance travelled by car d = (85.33) + (85.33 − 76.67)
in ( t − 0.5) s (with retardation) = 94 m
At 10 s
2
s c = ( 20 × 0.5) + 20 − 0.5 −    − 0.5 v = 16 − 0.25(10)2
5 1 20 5
4  2 3  4 
= − 9 m/s
= 23 .125 m ∴ Speed = 9 m/s
∴ s c − st = 23125
. − 21875
. = 1.250 m 3. If a = constant
Therefore, to avoid the bump onto the truck, the car must
dv
maintain a distance from the truck more than 1.250 m. Then | a | is also constant or = constant
dt
122 Objective Physics Vol. 1

4. rB = 0 at t = 0 6. In the complete journey,


rA = 3$i + 4$j
at t =0
v A = ( −20j$ )
Ù
N (j) S = 0 and a = constant = g (downwards)
F α
A 7. a = = t ...(i)
m m
4 km or a ∝t
B i.e. a -t graph is a straight line passing through origin.
E (^i )
3 km If u = 0, then integration of Eq. (i) gives,
αt 2
v= or v ∝ t 2
2m
v B = ( 40 cos 37° )$i + ( 40 sin 37° )$j
Hence, in this situation (when u = 0 ) v -t graph is a parabola
= ( 32i$ + 24$j) passing through origin.
v AB = ( −32i$ − 44$j) km/h 8. a - s equation corresponding to given graph is,
s
a =6−
→ 5
vB
v s
dv 
=  6 −  or ∫ vdv = ∫  6 −  ds
s s
∴ v⋅
ds  5 0 0
 5
37°
s2
or v = 125 −
At time t = 0, 5
rAB = rA − rB = ( 3i$ + 4$j) At s = 10 m, v = 10 m/s
Maximum values of v is obtained when
∴ At time t = t ,
dv
rAB = ( rAB at t = 0) + v ABt = 0 which gives s = 30 m
ds
= ( 3 − 32t )$i + ( 4 − 44 t )$j
( 30)2
5. a1 t1 = a2 t2 ∴ v max = 12 × 30 −
5
v max = a1 t1 = a2 t2 = 180 m/s
S1 = Area of v -t graph s d
1 9. v av = and v av =
= (t1 + t2 )( a1 t1 ) t t
2 Now, d ≥ |s |
v
∴ v av ≥ | v av |
1 2
10. v = at and x = at (u = 0) i.e. v -t graph is a straight line
vmax 2
a1 a2 passing through origin and x -t graph a parabola passing
through origin.
11. For minimum time
t1 t2 t b
∴ tmin =
S1 v
1
v1 = = a1 t1
t1 + t2 2
1 1 b v
S 2 = (t1 + t3 ) v max = (t1 + 2t2 )( 2a1 t1 )
2 2
S2 u
v2 = = a1 t1
(t1 + t3 ) For reaching a point exactly opposite
v
u
t3 = 2t2
vmax tmax = 2a1t1 b
v Net velocity
2a1 = 2a2t2
a2
t
t1 t3
From the four relations, we can see that Net velocity = v 2 − u2 (but v > u )
b
v 2 = 2v1 ∴ t=
and 2S1 < S2 < 4S1 net velocity
Motion in One Dimension 123

12. For t < T , v = −ve ∴ XA =


1
s and AY =
3
s
For t > T , v = + ve 4 4
v12 = ( 2)2 + 2a   = 4 +
At t = T , v = 0 s 192
= 52
 4 4
∴ Particle changes direction of velocity at t = T
S = Net area of v -t graph = 0 ∴ v1 = 52 ≠ 5 m / s
a = Slope of v -t graph = constant 10 = 2 + at1 (v = u + at )
v 8
13. v = αt1 ⇒ t1 = ∴ t1 = 14 = 10 + at2
α a
v 4
v = βt 2 ⇒ t 2 = ∴ t2 = or t1 = 2 t2
β a
v 1
S1 = ( 2t ) + a (t 2 ) = distance travelled in first half
2
v 1
S 2 = 2 ( 2t ) + a ( 2t )2
2
t1 t0 t2
t S3 = S2 − S1 = distance travelled in second half
We can see that,
 v v S
∴ t0 = t − t1 − t2 =  t − −  S1 ≠ 3
 α β 2
1 2 1 16. As per the diagram, at point A the graph is parallel to time
Now, l = αt1 + vt0 + βt 22
2 2 dx
2 2 axis hence, v = = 0. As the starting point is A hence, we
 v v 1 v 
= ( α )   + v  t − −  + ( β )  
1 v dt
2 α  α β 2 β can say that the particle is starting from rest.
At C, the graph changes slope, hence velocity also changes.
v2 v2
= vt − − As graph at C is almost parallel to time axis hence, we can
2α 2β
say that velocity vanishes.
l v 1 1
∴ t= +  +  As direction of acceleration changes hence, we can say that
v 2 α β it may be zero in between.
For t to be minimum, its first derivation with respect to From the graph it is clear that
velocity be zero or | slope at D | > | slope at E |
l α+β
0= − 2 + Hence, speed at D will be more than at E.
v 2αβ
17. Let the speeds of the two balls (1and 2) be v1 and v 2 , where
2lαβ
∴ v= if v1 = 2v ,v 2 = v
α+β
if y1 and y 2 and the distance covered by the balls 1 and 2 ,
14. x = t 2 respectively, before coming to rest, then
dx dv x v 2 4v 2 v2 v2
⇒ vx = = 2t ⇒ ax = =2 y1 = 1 = and y 2 = 2 =
dt dt 2g 2g 2g 2g
dy 4v 2 v 2
⇒ y = t 3 − 2t ⇒ v y = = 3t 2 − 2 Since, y1 − y 2 = 15 m, − = 15 m
dt 2g 2g
dv y
⇒ ay = = 6t 3v 2
dt or = 15 m
2g
At t = 0, v x = 0, v y = − 2, ax = 2 and a y = 0
or v 2 = 5m × ( 2 × 10) m/s2
∴ v = − 2$j and a = 2$i
or v =10 m/s
or v ⊥ a Clearly, v1 = 20 m/s and v 2 = 10 m/s
2 v2 ( 20m)2
At t = , ⇒ v y = 0, v x ≠ 0 . as y1 = 1 = = 20m ,
3 2g 2 × 10 m 15
Hence, the particle is moving parallel to x-axis.
y 2 = y1 − 15 m = 5 m
15. (14)2 = ( 2)2 + 2 as
If t2 is the time taken by the ball 2 toner a distance of 5m,
∴ 2 as = 192 units 1
then from y 2 = v 2t − gt22
v 2 = ( 2)2 + 2a  
s 2
At mid point,
 2 5 = 10 t2 − 5t22 or t22 − 2t2 + 1 = 0,
192
= 4+ =100 where t2 = 15
2 Since, t1 (time taken by ball 1 to cover distance of 20m ) is
∴ v =10 m/s 2s, time interval between the two throws
XA : AY =1 : 3 = t1 − t2 = 2s − 1s = 1s
124 Objective Physics Vol. 1

18. If an object undergoes a displacement ∆r in time ∆t, its ∴ uA t +


1 1
aA t 2 = uB t − aB t 2
average velocity is given by 2 2
∆ r r2 − r1 (uB − uA )
v= = , where r1 and r2 are position vectors ∴ t=2 = 2× 4=8s
∆t t2 − t1 ( aA + aB )
corresponding to time t1 and t2 . 6. SA = SB
It the velocity of an object changes from v1 to v 2 in time ∆t.
1 1
Average acceleration is given by 5t + aAt 2 = 15t − aBt 2
∆v v 2 − v1 2 2
aav = =
∆t t 2 − t 2 or 10 + aA t = 30 − aB t

But, when acceleration is non-uniform ∴ ( 5 + aA t ) − (15 − aB t ) = 10


v + v2 or v A − v B =10 m/s
v av ≠ 1
2 7. 8 = 6 + aAT = 6 + 4aA
∆r
We can write ∆v = ∴ aA = 0.5 m/s
∆t
At 10 s,
Hence, ∆r = r2 − r1 = (v 2 − v1 )(t2 − t1 )
v A = uA + aA t = (6) + (0.5)(10)
Comprehension Based Questions =11 m/s
1. Velocity of ball with respect to elevator is 15 m/s (up) and
elevator has a velocity of 10 m/s (up). Therefore, absolute
Assertion and Reason
velocity of ball is 25 m/s (upwards). Ball strikes the floor of 1. Velocity is always positive.
elevator if, 2. If speed varies, then velocity will definitely vary.
At highest point of a particle thrown upwards
25 m/s +
a ≠ 0 but v = 0.
F
3. a = = 1j$ N
2 10 m/s2 m
– 1 2
s = ut + a t = 20 $i + 8 $j
2m 2
1 ∴ | s | = 21.5 m
4. When a particle is released from rest,
50 m 10 m/s, 5 m/s2 v = 0 but a ≠ 0
5. On a curved path, distance > displacement
∴ Average speed > Average velocity
S1 = S2 + 2 Total displacement
Further, average velocity =
∴ 10t + 2.5t 2 = 25t − 5t 2 + 2 Total time
Solving this equation we get, v f − v i dv
6. a = = i.e. direction of acceleration is same as that
t = 2.13 s ∆t dt
of change in velocity vector, or in the direction of ∆v.
2. If the ball does not collides, then it will reach its maximum
dv
height in time, 8. a = = slope of v-t graph.
u 25 dt
t0 = = = 2.5 s
g 10 Perpendicular to t-axis, slope = ∞
Since, t < t0 , therefore as per the question ball is at its ∴ a=∞
maximum height at 2.13 s. 11. Last second of upward journey is the first second of
hmax = 50 + 2 + 25 × 2.13 − 5 × ( 2.13)2 = 82.56 m downward journey.
3. S = 25 × 2.13 − 5 × ( 2.13)2 = 30.56 m 1
∴ Distance travelled = g t 2
2
4. At maximum separation, their velocities are same
1
∴ 25 − 10t = 10 + 5t = × 9.8 × (1)2 = 4.9 m
2
or t =1 s
12. By differentiating a-t equation two times we will get s-t
Maximum separation = 2 + S2 − S1 equation. Further
= 2 + [25 × 1 − 5 × (1)2 ] − [10 × 1 + 2.5 (1)2 ]
a s
= 9.5 m
5. uA + aAT = uB − aBT

Putting T = 4s
we get 4( aA + aB ) = uB − uA t t
Now, SA = SB Straight line Parabola
Motion in One Dimension 125

13. v 1 and v 2 = constant 4. In motion M Slope of s-t graph is positive and increasing.
But distance is increasing. Therefore, velocity of the particle is positive and increasing.
v1 Hence, it is A type motion. Similarly, N , P and Q can be
observed from the slope.
1
5. Comparing the given equation with general equation of
2 1
displacement, s = s 0 + ut + at 2 , we get
2
v2
u = + 20 unit and a = −10 unit
v 4
14. N 6. ω = = = 4 rad /s
R 1
vB vA – vB v
B

vA E v
θ
R A

Displacement Area v - t graph


15. Average velocity = =
Time Time
1
vmt0 θ = ωt = 4 rad
1
=2 = vm
t0 2 θ
Displacement = AB = 2R sin
16. S is displacement not distance. 2
17. Slope in negative. Therefore, velocity is negative. = 2 sin 2 unit
Slope (therefore velocity) is increasing in magnitude. Distance = vt = 4 unit
Displacement
Therefore, acceleration is also negative. Average velocity = = 2 sin 2 unit
18. Ascending means velocity is upwards. Speed is decreasing. Time
It means acceleration is downwards. | ∆v |
And average acceleration =
Further, the body moves in the direction of velocity. ∆t
19. Slope of s-t graph = velocity = positive From subtraction of vectors, we can find that change in
θ
At t = 0, s ≠ 0 velocity from A and B is 2v sin = 8 sin 2 unit
2
Further at t = t0 ⇒ s = 0,v ≠ 0.
8sin 2
∴ Average acceleration =
Intergration 1
20. v-t → s -t
= 8 sin 2 unit
Differentiation
s-t → v-t 7. With constant positive acceleration, speed will increase
when velocity is positive, speed will decrease if velocity is
Match the Columns negative.
d | v|
1. is rate of change of speed of the particle. Similarly, with constant negative acceleration speed will
dt
increase, if velocity is also negative and speed will decrease, if
d |r |
is the rate by which distance of particle from the origin is velocity is positive.
dt
changing. 8. After 2s, velocity of balloon and hence the velocity of the
2. Corresponding v-t, a-t and s-t graphs are as shown below particle will be 20 m/s ( = at ) and its height from the ground
will be 20 m  = at 2  . Now, g will start acting on the
v a a 1
 2 
t t t
particle.
dx
9. We have to analyse slope of each curve i.e. . For peak
dt
dx
3. v i = +10 m/s and v f = 0 points will be zero as x is maximum at peak points.
dt
∴ ∆v = v f − v i = −10 m/s For graph (A), there is a point (B) for which displacement is
∆u − 10
aav = = zero. So, a matches with (r).
∆t 6
In graph (B), x is positive (> 0) throughout and at point
−5
= m/s2 dx
3 B1 , V = = 0.
dt
Total displacement = area under v-t graph (with sign)
since, at point of curvature changes a = 0, So b matches with
And acceleration = slope of v-t graph (q)
126 Objective Physics Vol. 1

dx u2 n2 u2
In graph (C), slope V = is negative hence, velocity will be ⇒ − H =n − ⇒ − 2gH = 2nu2 − n2u2
dt g 2 g
negative.
⇒ 2gH = n2u2 − 2nu2
so matches wth (s).
dx ⇒ 2gH = nu2 (n − 2)
In graph (D), as slope V = is positive hence, V > 0
dt 1 2
3. x = ut + at
Hence, d matches with (p). 2
For free fall starting from rest, u = 0, a = g
Entrance Gallery ⇒
1
x = gt 2
1. If balls are on floor or wall of rocket, then acceleration of 2
rocket do not affect time, when balls hit each other. Balls hit
t 0 T 2T 3T
after time t such that
0.3 t + 0.2 t = 4 g 2 g g
x
0 T ( 4T 2 ) ( 9T 2 )
4 2 2 2
⇒ t= = 8s …(i)
0.5 0 to T T to 2T 2T to 3T
∆t
If balls are not touching rocket wall or floor, then motion of
g 2 g g g
left ball is affected by acceleration of rocket. ∆x T ( 4 − 1) T 2 ( 9 − 4) T 2 = 5T 2
2 2 2 2
In small time 0.1s, left ball moves 0.3 × 0.1 = 0.03 m
1 Required ratio
Left wall moves 0 (0.1) + (2) (0.1)2 = 0.01 m
g 2 g g
2 T : 3T 2 : 5T 2 = 1 : 3 : 5
Initially, left ball is ahead of left wall at time t1 given by 2 2 2
1 4. 0 m/s
0 (t1 ) + ( 2)(t1 )2 = 0.3 t1 ⇒ t1 = 0.3 s
2
Left wall and left ball touch and velocity of wall is h
[0 + 2 (0.3)] = 0.6 ms− 1 , is more than velocity of ball.
Now, left wall takes the left ball along with it. 100 m
Left wall, left ball meet the right ball after time t2 such that
4 − (0.3) (0.3) − (0.2) (0.3) =  06
. t2 + (2) t22  + (0.2 t2 )
1 25 m/s
 2 
B
t22 + 0.8 t2 = 3.85
1 2
⇒ t22 + 0.8 t2 + 0.16 = 3.85 + 0.16 h= gt …(i)
2
⇒ (t2 + 0.4) = 4.01
2
1 2
gt  25t − gt 2  = 100 ⇒ t =
1 100
= 4s …(ii)
⇒ t2 = 2.0 − 0.4 = 1.6 s 2  2  25
Total time = t1 + t2 = 0.3 s + 1.6 s = 1.9 s ≈ 2 s …(ii) 1
From Eqs. (i) and (ii), we get h = (9.8) (4)2 = 78.4 m
2. 2
A
5. Given, s1 = 12 m, t1 = 3 s ⇒ s 2 = 30 + 12 = 42 m
u
1
O t2 = 3 + 3 = 6 s ⇒ s = ut + at 2
2
1 2 1
∴ s1 = ut1 + at1 ⇒ 12 = 3 × u + a × 32
H 2 2
9
12 = 3u + a …(i)
2
1
B Similarly, 42 = 6u + × a × 36 …(ii)
2
At A = highest point of path, velocity = 0 On solving Eqs. (i) and (ii), we get
0 = u − gt a = 2 m/s2 and u =1 m/s
u 6. Given, initial velocity, u = v
⇒ t = = time for 0→ A journey
g
Final velocity, v = v /2
Given time for 0→ A → B journey Distance, s = 20 cm
= nt …(i) Let the further distance of penetration before it comes to
0→ A → B journey rest be x.
1 2
− H = u (nt ) − g (nt )2 ⇒   = v 2 − 2a × 20
v
2 v 2 = u2 − 2as
 2
 u  1 2 u2
⇒ − H = un   − gn 2 v2 3v 2
g 2 g 40a = v 2 − ⇒ 40a = …(i)
4 4
Motion in One Dimension 127

and v 2 = u2 + 2as 10. For vertical motion, h = ut −


1 2
gt
⇒ v 2 = 0 + 2a × ( 20 + x ) 2
1
3 2 h1 = ut1 − gt12 …(i)
v2 = 2 × v ( 20 + x ) [From Eq. (i)] 2
160 1
3 80 h2 = ut2 − gt22 …(ii)
1 = ( 20 + x ) ⇒ = 20 + x 2
80 3 1 2u
80 80 − 60 20 0 = uT − gT 2
⇒ T= …(iii)
x= − 20 ⇒ x = = = 6.66 m 2 g
3 3 3
On multiplying Eq. (i) by t2 and Eq. (ii) by t1 and then
7. Let distance between A and B = d
subtracting Eq. (i) from Eq. (ii), we get
d 2 (h1t2 − h2t1 )
t1 = time for A → B, journey = 1
30 h1t2 − h2t1 = g t1t2 (t2 − t1 ) ⇒ g = …(iv)
2 t1t2 (t2 − t1 )
d
t2 = time for B → A, journey = On multiplying Eq. (i) by t22 and Eq. (ii) by t12 and then
20
Total distance travelled = d + d = 2d subtracting Eq. (i) from Eq. (ii), we get
Total time taken h t 2 − h2t12
h1t22 − h2t12 = ut1t2 (t2 − t1 ) ⇒ u = 1 2 …(v)
t1t2 (t2 − t1 )
=  +  =   =
d d d 1 1 d 5 d
= t1 + t2 = +
30 20 10  3 2  10  6  12 From Eqs. (iii), (iv) and (v), we get
2d h t 2 − h2t12
Average speed of car = = 24 km/h T= 12
d/12 h1t2 − h2t1
1
8. x = ut + at 2 11. v 2 − u2 = 2ax
2
For motion starting from rest, u = 0 mv 2 mu2
⇒ − = max
1 2 2
x = at 2
2 where, x = displacement
1 t
2
at 2 v = final velocity
x1 = a  = …(i)
2  2 8 u = initial velocity
1 at 2 and m = mass of particle.
x1 + x 2 = a (t )2 = Final kinetic energy – lnitial kinetic energy
2 2
= Force × displacement
at 2 at 2 3 2
x2 = − = at …(ii) ⇒ Power × Time = Work
2 8 8
⇒ (0.5) (5) = (0.2 kg) ( ax )
From Eqs. (i) and (ii), we get
⇒ ax = 12.5 …(i)
x 2 = 3x1
1 So, v 2 − (0)2 = 2 (12.5)
9. sn = u(n) + a (n) = distance travelled in n seconds
2
2 ⇒ v 2 = 25 ⇒ v = 5 ms− 1
1
sn − 1 = u (n − 1) + a (n − 1)2 12. Vertical motion for each bullet has same initial velocity
2 (zero), same displacement and same acceleration.
1 1
⇒ snth = sn − sn − 1 = u + a [n2 − (n − 1)2 ] From x = ut + at 2 …(i)
2 2
1
⇒ snth = u + a ( 2n − 1) t will be same for both bullets.
2 Eq. (i) do not involve mass.
= distance travelled in nth second 5
13. u = 42 × = 11.66 m/s and v = 0
1
snth = u + a ( 2n − 1) 18
2 So, S1 = 6 m
1 5
s 3 rd = 0 + a ( 2 × 3 − 1) = a (forn = 3 s) 5
when u = 90 × = 25 m/s and v = 0, then S2 = ?
2 2 18
1 7
s 4 th = 0 + a ( 2 × 4 − 1) = a (forn = 4 s) Case I v 2 = u2 + 2as
2 2
So, the percentage increase 0 = (11.66)2 + 2a × 5
− 11.66 × 11.66
7
a− a
5 a=
s 4 th − s 3 rd 12
× 100 = 2 2 × 100
s 3 rd 5
a a = −11.33 m/s2
2 Case II v 2 = u2 + 2as
2a
0 = ( 25)2 + 2 ( − 11.33) × S2
2 × 100 = 2 × 20 = 40%
5 625
a S2 = = 27.5 m
2 2 × 11.23
128 Objective Physics Vol. 1

14. 1 v=0 2 v=0 19. v 2 = u2 + 2ax


u
Given, v = for x = d1 = 30 cm
2
h
Let v = 0 for x = d1 + d 2
2
⇒  u  = u2 + 2ad
 
 2 1

1 2 3 3
⇒ − u2 = 2ad1 …(i)
t=0 t = 15 t = 25 4
02 = u2 + 2a ( d1 + d 2 )
In upward motion, the velocity of ball 1 in 1s becomes zero
0 = u − g (1) ⇒ − u2 = 2a ( d1 + d 2 ) …(ii)
⇒ u= g On dividing Eq. (ii) by Eq. (i), we get
1 4 d1 + d 2 d
⇒ h = ut − gt 2 = =1 + 2
2 3 d1 d1
1 d2 4 1
h = g (1) − g (1)2 ⇒ = −1 =
2 d1 3 3
g 10
= = = 5m d
d2 = 1 =
30 cm
2 2 3 3
15. If time for total journey = t = 10 cm
Distance covered in last 1s 1
20. Horizontal motion, h = 0 + gt 2
1 1 2
= gt 2 − g (t − 1)2
2 2 1
80 = × 10 × t 2
1 2
= g [t − (t 2 − 2 t + 1)]
2
2 ⇒ t 2 = 16
1
= g ( 2t − 1) …(i) ⇒ t = 4s
2
Distance covered = (8 ms−1 )( + 5) = 32 m
Distance covered in first 3 s
dv
1 9 21. = −2.5 v
= g ( 3)2 = g …(ii) dt
2 2
dv
1 9 ⇒ = −2.5 dt
Given, g ( 2t − 1) = g v
2 2
0 −1/ 2 t
⇒ 2t − 1 = 9 ⇒ ∫6.25 v dv = −2.5∫ dt
0
⇒ t = 5s
⇒ −2.5 [t ]t0 = [2v1/ 2 ] 06.25
1 1 1
Height h = gt 2 = g ( 5)2 = (10) (25) = 125 m
2 2 2 ⇒ t = 2s
16. The velocity of a particle is given by 22. Given, a = 1 m/s2 , s = 48 m, u = 10 m/s
dx 1
= v = 180 − 16x By equation of motion, s = ut + at 2
dt 2
dv 1 dx 1
a= = (180 − 16x )− 1/ 2 ( − 16) ⇒ 48 = 10t + × 1 × t 2
dt 2 dt 2
180 − 16x ⇒ t = 8s
a = −8× = − 8 m/s 2
180 − 16x 23. d d
A B
dx u C
17. x = 3t 2 ⇒ v x = = 3 ( 2t ) = 6t v
dt v1
dy v12 = u2 + 2ad
y = 4t 2 ⇒ v y = = 4 ( 2t ) = 8t …(i)
dt
v 2
= v12 + 2ad …(ii)
Velocity, v = v x $i + v y $j = 6t $i + 8t $j
Subtracting Eq. (i) and Eq. (ii), we get
| v| = (6t )2 + (8t )2 = 10t v12 − v 2 = u2 − v12
18. Distance, x = 37 + 27t − t 3 ⇒ 2v12 = u2 + v 2
dx
Velocity, v= = 0 + 27 (1) − 3t 2 u2 + v 2
dt ⇒ v1 =
2
Time, when v = 0 ⇒ 27 − 3t 2 = 0 ⇒ t 2 = 9 ⇒ t = 3 s
We have standard for this type of velocity
Distance at t = 3 s,
u2 + v 2
x = 37 + 27 ( 3) − ( 3)3 = 37 + 81 − 27 = 37 + 54 = 91 m v1 =
2
Motion in One Dimension 129

24. Given, y = bx 2 25. v1 v2


dy dx d d
= 2bx …(i)
dt dt d d  v + v2 
Total time = + =d 1 
dy  v1v 2 
= uy + at (Qa y = a ) (Qv y = uy + a y t) v1 v 2
dt
Total distance = 2d
dx
uy + at = 2bx Average velocity =
2d
=
2v 2v1
dt  v1 + v 2  v1 + v 2
⇒ t = 0, x = 0 imply uy = 0 d 
 v1v 2 
at dt = 2bx dx 2v1v 2
Take integration on both sides, we get Average velocity =
v1 + v 2
∫ at dt = ∫ 2bx dx
at 2 Given, v av = 40 km / h, v1 = 60 km/h and v 2 = ?
= bx 2 + c …(ii)
2 2 × 60 × v 2
∴ 40 =
At t = 0, x = 0, c = 0 60 + v 2
at 2 ⇒ 60 + v 2 = 3v 2
Then, = bx 2
2 v 2 = 30 km/h
at 2 a 26. Slope of displacement-time graph is velocity
⇒ x= =t v1 tan θ1 tan 30º 1
2b 2b = = =
dx a v 2 tan θ2 tan 45º 3
∴ vx = =
dt 2b ⇒ v1 : v 2 = 1 : 3
4
Projectile Motion

4.1 Projectile Motion


If a constant force (and hence constant acceleration) acts on a particle at an angle θ Chapter Snapshot
(≠ 0° or 180°) with the direction of its initial velocity (≠ zero), the path followed by the ● Projectile Motion
particle is a parabola and the motion of the particle is called projectile motion. Projectile ● Time of Flight (T)
motion is a two-dimensional motion, i.e. motion of the particle is constrained in a plane.
● Horizontal Range (R)
u
u ● Maximum Height (H)

(a) (b)

u u

(c) (d)

(e) (f)
Fig. 4.1

When a particle is thrown obliquely near the earth’s surface it moves in a parabolic
path, provided the particle remains close to the surface of earth and the air resistance is
negligible. This is an example of projectile motion. The different types of projectile motion
we come across are shown in Fig. 4.1.
In all the above cases, acceleration g of the particle is downwards.
Projectile Motion 131

Let us first make ourselves familiar with certain terms Time of Flight (T )
used in projectile motion.
y
Refer Fig. 4.2. Here, x and y-axes are in the directions
shown in figure. Axis x is along horizontal direction and axis
g y is vertically upwards. Thus,
ux = u cos α,
u y = u sin α, a x = 0
u A and ay = − g
At point B, s y = 0. So, applying
H
α 1
B x s y = u y t + a y t 2 , we have
O C 2
R 1
Fig. 4.2 0 = ( u sin α ) t − gt 2
2
Fig. 4.2 shows a particle projected from the point O with 2u sin α
∴ t = 0,
an initial velocity u at an angle α with the horizontal. It goes g
through the highest point A and falls at B on the horizontal 2u sin α
surface through O. The point O is called the point of Both t = 0 and t = correspond to the situation
g
projection, the angle α is called the angle of projection, the
where, s y = 0. The time t = 0 corresponds to point O and time
distance OB is called the horizontal range (R) or simply range
2u sin α
and the vertical height AC is called the maximum height (H). t= corresponds to point B. Thus, time of flight of
g
The total time taken by the particle in describing the path OAB
is called the time of flight (T). the projectile is given by
2u sin α
As we have already discussed, projectile motion is a T = tOAB or T =
two-dimensional motion with constant acceleration g
(normally g). Problems related to projectile motion of any
type can be solved by selecting two appropriate mutually Horizontal Range (R)
perpendicular directions (x and y) and substituting the Distance OB is the range R. This is also equal to the
proper values in equations, i.e. displacement of particle along x-axis in time t = T . Thus,
v x = ux + a x t, 1
applying sx = ux t + ax t 2 ,
1 2
sx = ux t + a x t 2 ,
2  2u sin α 
we get R = ( u cos α )   +0
v x = ux + 2a x sx ,
2 2
 g 
v y = u y + a y t, as a x = 0 and t = T
1 2u sin α
s y = u y t + a y t 2 and v y2 = u y2 + 2a y s y =
2 g
If any problem of projectile motion, we usually follow 2u sin α cos α
2
∴ R=
the three steps given below: g
Step 1 Select two mutually perpendicular directions x u sin 2 α
2

and y. =
g
Step 2 Write down the proper values of ux , a x , u y and u sin 2 α
2

a y with sign. or R=
g
Step 3 Apply those equations from the six listed above Here, two points are important regarding the range of a
which are required in the problem. projectile.
What should be the directions x and y or which (i) Range is maximum
equations are to be used, this you will learn after solving
where, sin 2α = 1 or α = 45° and this maximum range
some problems of projectile motion. Using the above
is
methodology, let us first prove the three standard results of
u2
time of flight (T), horizontal range (R) and the maximum R max = (at α = 45°)
height (H). g
132 Objective Physics Vol. 1

(ii) For given value of u range at α and range at (90° − α ) Extra Knowledge Points
are equal although times of flight and maximum ■ As we have seen in the above derivations that a x = 0,
heights may be different. Because, i.e., motion of the projectile in horizontal direction is
u 2 sin 2 (90° − α ) uniform. Hence, horizontal component of velocity
R 90° − α = u cos α does not change during its motion.
g ■ Motion in vertical direction is first retarded, then
u sin (180° − 2 α )
2 accelerated in opposite direction. Because, u y is
= upwards and a y is downwards. Hence, vertical
g component of its velocity first decreases from O to A
u 2 sin 2 α and then increases from A to B. This can be shown as
= in figure.
g y
= Rα
Y

A
u
uy
u α
x
u O ux B

60° ■ The coordinates and velocity components of the


30° projectile at time t are
X
x = sx = u x t = (u cos α ) t
Fig. 4.3 1
y = sy = u y t + a y t 2
So, R 30° = R 60° 2
1
= (u sin α ) t − gt 2
or R 20° = R 70° 2
This is shown in Fig. 4.3. v x = u x = u cos α and v y = u y + a y t = u sin α − gt

Maximum Height (H) Therefore, speed of projectile at time t is v = v x2 + v y2


At point A vertical component of velocity becomes zero, and the angle made by its velocity vector with positive
i.e., v y = 0. Substituting the proper values in : x-axis is
vy 
v y2 = u y2 + 2a y s y θ = tan−1  
vx 
we have, 0 = ( u sin α ) 2 + 2( −g )( H ) ■ Equation of trajectory of projectile
x
u sin α2 2 x = (u cos α ) t ⇒ ∴ t=
∴ H= u cos α
2g 1 2
Substituting this value of t in, y = (u sin α ) t − gt , we
2
get
X Example 4.1 Find the angle of projection of a
gx 2
projectile for which the horizontal range and maximum y = x tan α −
2 u 2 cos 2 α
height are equal.
gx 2
= x tan α − sec2α
Sol. Given, R=H 2 u2
2
u sin 2 α u 2 sin2 α gx 2
∴ = = x tan α − (1 + tan2 α )
g 2g 2 u2
sin2 α These are the standard equations of trajectory of a
or 2 sin α cos α = projectile. The equation is quadratic in x. This is why
2
the path of a projectile is a parabola. The above
sin α
or =4 equation can also be written in terms of range (R) of
cos α projectile as
or tan α = 4  x
y = x 1 –  tan α
∴ α = tan−1(4)  R
Projectile Motion 133

Now, let us take few examples based on the above theory.


X Example 4.5 A projectile is fired horizontally
X Example 4.2 Prove that the maximum horizontal with a velocity of 98 m/s from the top of a hill 490 m
range is four times the maximum height attained by the high. Find
projectile; when fired at an inclination so as to have u = 98 m/s
O
maximum horizontal range. x
Sol. For θ = 45°, the horizontal range is maximum and is given by
u2 y
Rmax =
g
Maximum height attained
u 2 sin2 45° A vx
Hmax = B
2g β
u2 R vy
= = max
4g 4 Fig. 4.4
or Rmax = 4 Hmax (a) the time taken by the projectile to reach the ground,
(b) the distance of the point, where the particle hits the
X Example 4.3 There are two angles of projection
ground from foot of the hill and
for which the horizontal range is the same. Show that
(c) the velocity with which the projectile hits the ground.
the sum of the maximum heights for these two angles is
independent of the angle of projection. (g = 9.8 m/s 2 )

Sol. There are two angles of projection α and (90°− α ) for which Sol. In this problem, we cannot apply the formulae of R, H and T
directly. We will have to follow the three steps discussed in the
the horizontal range R is same.
theory. Here, it will be more convenient to choose x and y
u 2 sin2 α directions as shown in the figure.
Now, H1 =
2g Here, u x = 98 m/s, a x = 0, u y = 0 and ay = g .
u 2 sin2 (90° − α ) (a) At A, sy = 490 m . So, applying
and H2 =
2g sy = u y t + 1 ay t 2
2
u 2 cos 2 α 1
= ∴ 490 = 0 + (9.8) t 2 ⇒ ∴ t = 10 s
2g 2

u2 u2 (b) BA = s x = u x t + 1 a x t 2
Therefore, H1 + H2 = (sin2 α + cos 2 α ) = 2
2g 2g
or BA = (98)(10) + 0 or BA = 980 m
Clearly, the sum of the heights for the two angles of (c) v x = u x = 98 m/s
projection is independent of the angles of projection. v y = u y + ay t = 0 + (9.8) (10) = 98 m/s

X Example 4.4 Show that there are two values of ∴ v= v 2x + v y2 = (98)2 + (98)2 = 98 2 m/s
time for which a projectile is at the same height. Also vy 98
and tan β = = =1
show mathematically that the sum of these two times is vx 98
equal to the time of flight. ∴ β = 45°
Thus, the projectile hits the ground with a velocity 98 2 m/s
Sol. For vertically upward motion of a projectile, at an angle of β = 45° with horizontal.
y = (u sin α ) t − 1 gt 2
2 X Example 4.6 A body is thrown horizontally from
or 1 gt 2 − (u sin α ) t + y = 0 the top of a tower and strikes the ground after three
2 seconds at an angle of 45° with the horizontal. Find the
This is a quadratic equation in t. Its roots are height of the tower and the speed with which the body
u sin α − u 2 sin2 α − 2 g y was projected. (Take g = 9.8 m / s 2)
t1 =
g

u sin α + u 2 sin2 α − 2 g y
Sol. As shown in the Fig. 4.4,
and t2 = uy = 0
g
and ay = g = 9.8 m/s 2
2 u sin α
∴ t1 + t 2 = =T (time of flight of the projectile)
g sy = u y t + 1 ay t 2
2
134 Objective Physics Vol. 1

= 0 × 3 + 1 × 9.8 × (3)2
2
= 44.1 m
Extra Knowledge Points
Further, v y = u y + ay t = 0 + (9.8) (3) ■ In projectile motion, the speed (and hence kinetic
= 29.4 m/ s energy) is minimum at highest point.
Speed = (cos θ ) times the speed of projection
As the resultant velocity v makes an angle of 45° with the
horizontal so,
and kinetic energy = (cos 2 θ ) times the initial kinetic
v energy
tan 45° = y Here, θ = angle of projection
vx
29.4
■ Path of a particle depends on the nature of
or 1= acceleration and the angle between initial velocity u
vx
and acceleration a. Following are few paths which are
v x = 29.4 m/s observed frequently.
Therefore, the speed with which the body was projected (a) If a = constant and θ is either 0° or180°, then path of
(horizontally) is 29.4 m/s. the particle is a straight line.
(b) If a = constant but θ is other then 0° or 180°, then
Note Points path of the particle is parabola (as in projectile
motion).
/ Projectile motion is a two-dimensional motion with constant
1 (c) If | a | = constant and a is always perpendicular to
acceleration g. So, we can use v = u + at,s = ut + at 2 , etc.,
2 velocity vector v, then path of the particle is a circle.
in projectile motion as well. ■ In projectile motion, it is sometimes better to write the
Here, u = u cos α $i + u sin αj$ and a = − g $j equations of H, R andT in terms of u x and u y as under.
Now, suppose we want to find velocity at time t. 2 uy u y2 2 ux uy
T = ⇒ H= and R =
y g 2g g
■ In projectile motion H = R , when u y = 4 u x or tan θ = 4.
u
v
Air drag

α
x Gravity Air drag Gravity
O g v

Fig. 4.5
If a particle is projected vertically upwards, then during
v = u + at = (u cos α $i + u sin α j$) − gt j$ upward journey gravity forces (weight) and air drag
or v = u cos α i$ + (u sin α − gt )j$ both are acting downwards. Hence, |retardation| > | g |.
During its downward journey, air drag is upwards while
Similarly, displacement at time t will be
1 2
gravity is downwards. Hence, acceleration < g.
s = ut + at Therefore, we may conclude that, time of ascent < time
2
of descent.
1
= (u cos α $i + u sin α j$)t − gt 2 j$ ■ Exercise In projectile motion, if air drag is taken into
2
consideration than state whether the H, R and T will
= ut cos α $i +  ut sin α − gt 2  j$
1
increase, decrease or remain same
 2 

Chapter Summary with Formulae


(i) Projectile Motion u2
(d) R max =
2u sinθ 2u y g
(a) T = =
g g θ = 45°
2
u2 sin2 θ u y (e) R θ = R90 °− θ for same value of u
(b) H = =
2g 2g (f) Equation of trajectory
u2 sin 2 θ g x2
(c) R = y y = x tan θ − 2
g 2u cos2 θ
ux = u cos θ
2u u gx2
= ux T = x y u
uy = u sin θ = x tan θ − (1 + tan2 θ) .
g 2u2
= x  1 −  tan θ
x
θ 
x R
Additional Examples
Example 1. Can there be motion in two dimensions Example 7. A boy is dropped freely from the window
with an acceleration only in one dimension ? of a train. Will the time of the free fall be equal, if the
Sol. Yes, in a projectile motion, the acceleration acts vertically
train is stationary, the train moves with a constant
downwards while the projectile follows a parabolic path.
velocity, the train moves with an acceleration ?

Example 2. A body projected horizontally from a Sol. The motion of the train does not affect the nature of the
tower moves with the same horizontal velocity, motion of the dropped body along the vertical. Therefore, in all
although it is under the action of force of gravity. the three cases, the time of free fall of the body will be equal.
Why?
Example 8. A particle is projected from horizontal
Sol. The force of gravity acts in the vertically downward making an angle 60° with initial velocity 40 m/s. Find
direction and has no effect on the horizontal component of the time taken to the particle to make angle 45° from
velocity and this makes the body to move with constant horizontal.
horizontal velocity.
Sol. At 45°, v x = v y or u x = u y − gt
Example 3. What is the angle between the direction u y − u x 40 (sin 60 ° − sin 30 °)
of velocity and acceleration at the highest point of a ∴ t= =
g 9.8
projectile path ?
= 1.5 s
Sol. 90°. At the highest point the vertical component of velocity
Example 9. A ball rolls off the edge of a horizontal
becomes zero, the projectile has only a horizontal velocity
while the acceleration due to gravity acts vertically downwards.
table top 4 m high. If it strikes the floor at a point 5 m
horizontally away from the edge of the table, what was
Example 4. A bullet is dropped from a certain its speed at the instant it left the table?
height and at the same time, another bullet is fired 1 2
horizontally from the same height. Which one will hit Sol. Using h = gt , we have
2
the ground earlier and why ? v
A
Sol. Since, the heights of both bullets from the ground is the
4m
same, so the time taken by both of them to reach the ground
will be the same, because initial velocity component in vertical
B C
direction is zero for both of them.
5m
Example 5. Is the maximum height attained by
projectile is largest when its horizontal range is 1 2 2h AB
h AB = gt AC or t AC =
maximum ? 2 g
Sol. No, horizontal range is maximum when θ = 45° and 2× 4
= = 0.9 s
maximum height attained by projectile is largest whenθ = 90 °. 9.8
Further, BC = vt AC
Example 6. A person sitting in a moving train BC 5.0
throws a ball vertically upwards. How does the ball or v= = = 5.55 m/s
t AC 0.9
appear to move to an observer (i) inside the train
(ii) outside the train?
Example 10. An aeroplane is flying in a horizontal
Sol. (i) To the observer inside the train, the ball will appear to direction with a velocity 600 km/h at a height of 1960
move straight vertically upwards and then downwards. m. When it is vertically above the point A on the
(ii) To the observer outside the train, the ball will appear ground, a body is dropped from it. The body strikes the
to move along the parabolic path. ground at point B. Calculate the distance AB.
136 Objective Physics Vol. 1
v
1 2 O
Sol. From h = gt , that the time interval of passing between the two walls
2
h
2h OA h is 2 .
We have t OB = g
g
A B
2 × 1960 Sol. Let ∆t be the time interval. Then,
=
9.8 2h
2 h = (u x ) (∆t ) or u x = K(i)
= 20 s ∆t
Horizontal distance, AB = vt OB y
 5 
= 600 × m /s (20 s) = 3333.33 m = 3.33 km
 18  v
uy
Example 11. A particle is projected with a velocity h h
x
of 20 m/s at an angle of 30° to an inclined plane of ux
2h
inclination 30° to the horizontal. The particle hits the
1 2
inclined plane at an angle of 30°, during its journey. Further, h = u y t − gt or gt 2 − 2u y t + 2 h = 0
Find the 2
(a) time of impact, 2 u y + 4u y2 − 8 g h
∴ t1 =
(b) the height of the point of impact from the horizontal 2g
plane passing through the point of projection.
2 u y − 4u y2 − 8 g h
and t2 =
Sol. The particle hits the plane at 30° (the angle of inclination of 2g
plane). It means particle hits the plane horizontally.
4 u y2 − 8 g h
∆t = t 1 − t 2 =
u g
g (∆t )
2 2

30°
or u y2 = + 2gh K(ii)
30°
4
u x2 + u y2 = u 2 = ( 2 g h ) 2
T u sin θ 20 sin (30 ° + 30 ° ) 4h 2 g 2 (∆t ) 2
(a) t = = = = 1.76 s ∴ + + 2gh = 4 gh
2 g 9.8 (∆t ) 2
4
u 2 sin 2 θ ( 20) 2 × sin 2 60 ° g2
(b) H = = = 15.3 m (∆t ) 4 − 2 g h (∆t ) 2 + 4 h 2 = 0
2g 2 × 9.8 4
2 g h ± 4g 2 h 2 − 4g 2 h 2 4 h
Example 12. A particle is projected with velocity (∆t ) 2 = =
g2 /2 g
2 gh so that it just clears two walls of equal height h,
h
or ∆t = 2
which are at a distance of 2h from each other. Show g

NCERT Selected Questions


Q 1. A bullet fired at an angle of 30° with the horizontal Maximum range is given by the formula
hits the ground 3 km away. By adjusting its angle of u2 3000 × 2
Rmax = =
projection can one hope to hit a target 5 km away. g 3
Assume, the muzzle speed to be fixed and neglect = 3464 m
air resistance.
But required Rmax = 5000 m
u2 sin 2 θ
Sol. Range, R =
g R max with initial velocity of projection is short of
u2
R 3000 3000 × 2 5000 m , so the bullet cannot hit the target 5 km
∴ = = =
g sin 2θ sin 60° 3 away.
Objective Problems
[ Level 1 ]
Direct Formula and Theory 9. An object is thrown along a direction inclined at an angle
of 45° with the horizontal direction. The horizontal range
1. A particle is projected at an angle of 45° with a velocity of
of the particle is equal to
9.8 m/s. The horizontal range will be [take g = 9.8 m/ s 2 ]
(a) vertical height
9.8 (b) twice the vertical height
(a) 9.8 m (b) 4.9 m (c) (d) 9.8 2
2 (c) thrice the vertical height
2. When a stone is projected which remains constant? (d) four times the vertical height
(a) Angular momentum 10. At the top of the trajectory of a particle, the acceleration is
(b) Linear momentum (a) maximum (b) minimum
(c) Vertical component of velocity (c) zero (d) g
(d) Horizontal component of velocity
3. A stone is projected with speed of 50 m/ s at an angle of 11. A football player throws a ball with a velocity of 50 m/s
60° with the horizontal. The speed of the stone at highest at an angle 30° from the horizontal. The ball remains in
point of trajectory is the air for ( g = 10 m/ s 2 ).
(a) 75 m / s (b) 25 m / s (a) 2.5 s (b) 1.25 s
(c) 50 m / s (d) cannot find (c) 5 s (d) 0.625 s

4. At the top of the trajectory of a projectile, the directions 12. For a projectile, the ratio of maximum height reached to
of its velocity and acceleration are the square of flight time is ( g = 10 m/ s 2 )
(a) parallel to each other (a) 5 : 4 (b) 5 : 2
(b) anti-parallel to each other (c) 5 : 1 (d) 10 : 1
(c) inclined to each other at an angle of 45°
(d) perpendicular to each other 13. An object is projected at an angle of 45° with the
5. A projectile, thrown with velocity v 0 at an angle α to the horizontal. The horizontal range and the maximum height
horizontal, has a range R. It will strike a vertical wall at a reached will be in the ratio
distance R/ 2from the point of projection with a speed of (a) 1 : 2 (b) 2 : 1
gR (c) 1 : 4 (d) 4 : 1
(a) v0 (b) v0 sinα (c) v0 cos α (d)
2 14. A bomb is dropped from an aeroplane moving
6. Two projectiles A and B are projected with same speed at horizontally at constant speed. If air resistance is taken
angles 30° and 60° to be horizontal, then which one is into consideration, then the bomb
wrong? (a) falls on earth exactly below the aeroplane
(b) falls on the earth behind the aeroplane
(a) RA = RB (b) H B = 3H A
(c) falls on the earth ahead of the aeroplane
(c) TB = 3 TA (d) None of these (d) flies with the aeroplane

7. A projectile fired with initial velocity u at some angle θ 15. In the motion of a projectile falling freely under gravity,
has a range R. If the initial velocity be doubled at the its (neglect air friction)
same angle of projection, then the range will be (a) total mechanical energy is conserved
(a) 2R (b) R/2 (b) momentum is conserved
(c) R (d) 4R (c) mechanical energy and momentum both are conserved
(d) None is conserved
8. If the initial velocity of a projection be doubled, keeping
the angle of projection same, the maximum height 16. The horizontal range of a projectile is 4 3 times of its
reached by it will maximum height. The angle of projection will be
(a) remain the same (b) be doubled (a) 60° (b) 37°
(c) become four times (d) be halved (c) 30° (d) 45°
138 Objective Physics Vol. 1

Formulae Based Problems with 25. A man can throw a stone such that it acquires maximum
horizontal range 80 m. The maximum height to which it
Slight Modification will rise for the same projectile in metre is
17. At what angle θ to the horizontal should an object is (a) 10 (b) 20
projected so that the maximum height reached is equal to (c) 40 (d) 50
the horizontal range? 26. The ratio of the speed of a projectile at the point of
(a) tan −1 (2) (b) tan −1 (4 ) projection to the speed at the top of its trajectory is x. The
(c) tan −1 (2/ 3) (d) tan −1 (3) angle of projection with the horizontal is
(a) sin −1 (x ) (b) cos−1 (x )
18. The maximum range of a gun on horizontal terrain is −1
(c) sin (1/ x ) (d) cos−1 (1/ x )
1.0 km. If g = 10 m/ s 2 , what must be the muzzle velocity
of the shell? 27. The velocity at the maximum height of a projectile is half
(a) 400 m / s (b) 200 m / s of its initial velocity of projection ( u ). Its range on
(c) 100 m / s (d) 50 m / s horizontal plane is
19. Two projectiles A and B are thrown from the same point 3u2 3 u2
(a) (b) ⋅
with velocities v and v/ 2, respectively. If B is thrown at g 2 g
an angle 45° with horizontal, what is the inclination of A u2 3 u2
(c) (d) ⋅
when their ranges are the same? 3g 2 g
 1 1  1
(a) sin −1   (b) sin −1  
 4 2  4 28. A projectile is thrown from a point in a horizontal plane
 1 1  1 such that the horizontal and vertical velocities are
(c) 2 sin −1   (d) sin −1  
 4 2  8 9.8 ms −1 and 19.6 ms −1 . It will strike the plane after
covering distance of
20. Two stones having different masses m1 and m2 are (a) 39.2 m (b) 19.6 m
projected at an angle α and ( 90° − α ) with same speed (c) 9.8 m (d) 4.9 m
from same point. The ratio of their maximum heights is
(a) 1 : 1 (b) 1: tanα 29. A stone is projected in air. Its time of flight is 3 s and
(c) tan α : 1 (d) tan 2 α : 1 range is 150 m. Maximum height reached by the stone is
( g = 10 ms −2 )
21. A body projected with velocity u at projection angle θ has
(a) 37.5 m (b) 22.5 m (c) 90 m (d) 11.25 m
horizontal range R. For the same velocity and projection
angle, its range on the moon surface will be 30. The greatest height to which a man can throw a stone is h.
( g moon = g earth / 6) The greatest distance to which he can throw it will be
R h
(a) 36R (b) (a) (b) h
36 2
R (c) 2h (d) 3h
(c) (d) 6R
16
31. The range of a projectile when launched at angle θ is
22. Three balls of same masses are projected with equal same as when launched at angle 2θ. What is the value of
speeds at angle 15° , 45° , 75° , and their ranges are θ?
respectively R1, R 2 and R 3 , then (a) 15° (b) 30° (c) 45° (d) 60°
(a) R1 > R2 > R3 (b) R1 < R2 < R3
(c) R1 = R2 = R3 (d) R1 = R3 < R2 32. A boy throws a ball with a velocity u at an angle θ with the
horizontal. At the same instant he starts running with
23. If 2 balls are projected at angles 45° and 60° and the uniform velocity to catch the ball before if hits the
maximum heights reached are same, what is the ratio of ground. To achieve this he should run with a velocity of
their initial velocities? (a) u cos θ (b) u sin θ
(a) 2 : 3 (b) 3 : 2 (c) u tan θ (d) u sec θ
(c) 3 : 2 (d) 2 : 3
33. The range of a particle when launched at an angle of 15°
24. A projectile is thrown at an angle θ with the horizontal with the horizontal is 1.5 km. What is the range of the
and its range is R1 . It is then thrown at an angle θ with projectile when launched at an angle of 45° to the
vertical and the range is R 2 , then horizontal?
(a) R1 = 4 R2 (b) R1 = 2R2 (a) 1.5 km (b) 3.0 km
(c) R1 = R2 (d) data insufficient (c) 6.0 km (d) 0.75 km
Projectile Motion 139

34. Galileo writes that for angle of projection of a projectile 42. A projectile is thrown upward with a velocity v 0 at an
at angle ( 45° + θ ) and ( 45° − θ ), the horizontal ranges angle α to the horizontal. The change in velocity of the
described by the projectile are in the ratio of ( if θ ≤ 45° ) projectile when it strikes the same horizontal plane is
(a) 2 : 1 (b) 1 : 2 (a) v0 sinα vertically downwards
(c) 1 : 1 (d) 2 : 3 (b) 2v0 sinα vertically downwards
(c) 2v0 sinα vertically upwards
35. If time of flight of a projectile is 10 s. Range is 500 m.
(d) zero
The maximum height attained by it will be
(a) 125 m (b) 50 m 43. Two paper screens A and B are separated by a distance of
(c) 100 m (d) 150 m 100 m . A bullet pierces A and then B. The hole in B is
36. Four bodies P , Q , R and S are projected with equal 10 cm below the hole in A. If the bullet is travelling
horizontally at the time of hitting A, then the velocity of
velocities having angles of projection 15° , 30° , 45° and the bullet at A is
60° with the horizontals respectively. The body having
(a) 100 m / s (b) 200 m / s
shortest range is (c) 600 m / s (d) 700 m / s
(a) P (b) Q (c) R (d) S
44. A piece of marble is projected from earth’s surface with
37. A stone is thrown at an angle θ to be the horizontal velocity of 19.6 2 m/ s at 45°. 2 s later its velocity makes
reaches a maximum height H. Then, the time of flight of an angle α with horizontal, where α is
stone will be (a) 45° (b) 30°
2H 2H (c) 60° (d) 0°
(a) (b) 2
g g
45. A body is projected at an angle of 30° with the horizontal
2 2H sinθ 2H sinθ
(c) (d) with momentum p. At its highest point, the magnitude of
g g the momentum is
38. Two balls are thrown simultaneously from ground with 3 2
(a) p (b) p
same velocity of 10 m/ s but different angles of projection 2 3
p
with horizontal. Both balls fall at same distance 5 3 m (c) p (d)
2
from point of projection. What is the time interval
between balls striking the ground? 46. A ball is projected upwards from the top of a tower with
(a) ( 3 − 1) s (b) ( 3 + 1) s velocity 50 ms −1 making an angle of 30° with the
(c) 3s (d) 1 s horizontal. If the height of the tower is 70 m, after what
time from the instant of throwing, will the ball reach the
39. For a given velocity, a projectile has the same range R for
ground? ( g = 10 ms −2 ).
two angles of projection. If t 1 and t 2 are the time of flight
in the two cases, then t 1 t 2 is equal to (a) 2 s (b) 5 s
(c) 7 s (d) 9 s
2R R
(a) (b)
g g g 2
47. The equation of projectile is y = 3x − x , the angle of
4R R 2
(c) (d)
g 2g its projection is
(a) 90° (b) zero
40. A projectile can have same range from two angles of (c) 60° (d) 30°
projection with same initial speed. If h1 and h2 be the
maximum heights, then 48. A body is projected horizontally with a velocity of 4 m/ s
(a) R = h1 h2 (b) R = 2 h1 h2 from the top of a high tower. The velocity of the body
(c) R = 2 h1 h2 (d) R = 4 h1 h2 after 0.7 s is nearly (take g = 10 m/ s 2 )
(a) 10 m / s (b) 8 m / s
Miscellaneous Problems (c) 19.2 m / s (d) 11 m / s

41. An arrow is shot into air. Its range is 200 m and its time of 49. A cricket ball is hit for a six the bat at an angle of 45° to
flight is 5s. If g = 10 m/ s 2 , then horizontal component of the horizontal with kinetic energy K. At the highest point,
velocity and the maximum height will be respectively the kinetic energy of the ball is
(a) 20 m / s, 62.50 m (b) 40 m / s, 31.25 m (a) zero (b) K
(c) 80 m / s, 62.5 m (d) None of these (c) K /2 (d) K / 2
140 Objective Physics Vol. 1

50. The height y and distance x along the horizontal for a 56. From the top of a tower of height 40 m, a ball is projected
body projected in the xy-plane are given by upwards with a speed of 20 m/ s at an angle of elevation
y = 8t − 5t 2 and x = 6t. The initial speed of projection is of 30°. The ratio of the total time taken by the ball to hit
(a) 8 m / s (b) 9 m / s the ground to its time of flight (time taken to come back
(c) 10 m / s (d) (10/ 3) m / s to the same elevation) is (take g = 10 m/ s 2 )
(a) 2 : 1 (b) 3 : 1
51. An aeroplane is flying in horizontal direction with a
(c) 3 : 2 (d) 1.5 :1
velocity of 600 km/ h and at a height of 1960 m. When it is
vertically above a point A on the ground a body is 57. The equation of motion of a projectile is
dropped from it, the body strikes the ground at point B. 3
Then, the distance AB will be y = 12x − x 2
4
(a) 3.33 km (b) 4.33 km What is the range of the projectile?
(c) 5.33 km (d) 6.33 km (a) 12 m (b) 16 m
52. A stone is projected with a velocity 20 2 m/ s at an angle (c) 20 m (d) 24 m

of 45° to the horizontal. The average velocity of stone 58. A ball is thrown at different angles with the same speed u
during its motion from starting point to its maximum and from the same point. It has the same range in both
height is (take g = 10 m/ s 2 ) cases. If y1 and y 2 be the heights attained in the two
(a) 20 m / s (b) 20 5 m /s cases, then y1 + y 2 equals to
(c) 5 5 m / s (d) 10 5 m / s u2 2u2
(a) (b)
g g
53. The maximum height attained by a projectile is increased u2 u2
by 10% by increasing its speed of projection, without (c) (d)
2g 4g
changing the angle of projection. The percentage
increases in the horizontal range will be 59. A projectile is fired from level ground at an angle θ
(a) 20% (b) 15% above the horizontal. The elevation angle φ of the highest
(c) 10% (d) 5% point as seen from the launch point is related to θ by the
relation.
54. A ball is thrown up with a certain velocity at an angle θ to
1
the horizontal. The kinetic energy KE of the ball varies (a) tan φ = tan θ (b) tan φ = tan θ
4
with horizontal displacement x as 1
(c) tan φ = tan θ (d) tan φ = 2 tan θ
2

60. Water is flowing from a horizontal pipe fixed at a height


(a) KE (b) KE
of 2 m from the ground. If it falls at a horizontal distance
of 3 m as shown in figure, the speed of water when it
O x O x leaves the pipe is (take g = 9.8 ms −2 )

(c) KE (d) KE

2m
O x O x

55. An object of mass m is projected with a momentum p at 3m


1 −1
such an angle that its maximum height is th of its (a) 2.4 ms (b) 4.7 ms−1
4
horizontal range. Its minimum kinetic energy in its path (c) 7.4 ms−1 (d) 6.2 ms−1
will be 61. A particle is thrown with a speed u at an angle θ with the
p2 p2 horizontal. When the particle makes an angle φ with the
(a) (b)
8m 4m horizontal, its speed changes to v, where
3 p2 p2 (a) v = u cos θ (b) v = u cos θ cos φ
(c) (d)
4m m (c) v = u cos θ sec φ (d) v = u sec θ cos φ
Projectile Motion 141

62. A ball is thrown up with a certain velocity at an angle θ to 67. A large number of bullets are fired in all directions with
the horizontal. The kinetic energy KE of the ball varies same speed v. What is the maximum area on the ground
with height h as on which these bullets will spread?
v2 v4 v4 v2
(a) π (b) π (c) π 2 (d) π 2
g g2 g2 g2
(a) KE (b) KE 68. A body of mass m is thrown upwards at an angle θ with
the horizontal with velocity v. While rising up the
O h O velocity of the mass after t seconds will be
h
(a) (v cosθ )2 + (v sin θ )2 (b) (v cosθ − v sin θ )2 − gt
(c) v 2 + g 2t 2 − (2v sin θ ) gt (d) v 2 + g 2t 2 − (2v cosθ ) gt
(c) KE (d) KE
69. Figure shows four paths for a kicked football. Ignoring
O O the effects of air on the flight, rank the paths according to
x x
initial horizontal velocity component highest first.
1 y
63. The equation of projectile is Y = 3X − gX 2 . The
2
velocity of projection is
(a) 1 m / s (b) 2 m / s
(c) 3 m / s (d) 1.2 m/s
1 2 3 4 x
0
64. A ball of mass m is projected from the ground with an
(a) 1, 2, 3, 4 (b) 2, 3, 4 , 1
initial velocity u making an angle of θ with the horizontal.
(c) 3, 4 , 1, 2 (d) 4 , 3, 2, 1
What is the change in velocity between the point of
projection and the highest point? 70. Which of the following is the graph between the height
(a) ucos θ downward (b) ucos θ upward ( h ) of a projectile and time ( t ), when it is projected from
(c) usin θ upward (d) usin θ downward the ground?
h h
65. A projectile is thrown at an angle θ such that it is just able
to cross a vertical wall at its highest point as shown in the
figure. (a) (b)
The angle θ at which the projectile is thrown is given by
O t O t

v0 h h

H
(c) (d)

θ O t O t

3H 71. Which of the following is the altitude-time graph for a


 1 projectile thrown horizontally from the top of the tower?
(a) tan −1   (b) tan −1 ( 3 )
 3 h h

 2  3
(c) tan −1   (d) tan −1  
 3  2 (a) (b)

66. A bomber plane moves horizontally with a speed of O t O t


500 m/ s and a bomb released from it strikes the ground in
h h
10 s. Angle at which it strikes the ground will be
( g = 10 m/ s 2 ).
 1 (c) (d)
(a) tan −1   (b) 60°
 5
t
(c) 45° (d) tan −1 (5) O O t
142 Objective Physics Vol. 1

72. The initial velocity of a particle of mass 2 kg is 80. Two stones are projected so as to reach the same distance
( 4i$ + 4$j ) m/ s. A constant force of − 20 $j N is applied on from the point of projection on a horizontal surface. The
maximum height reached by one exceeds the other by an
the particle. Initially, the particle was at ( 0, 0) . Find the
amount equal to half the sum of the height attained by
x-coordinate of the point where its y - coordinate is again
them. Then, angle of projection of the stone which attains
zero.
smaller height is
(a) 3.2 m (b) 6 m (c) 4.8 m (d) 1.2 m
(a) 45° (b) 60°
73. A particle is projected from the ground at an angle of θ (c) 30° (d) tan −1 (3/ 4 )
with the horizontal with an initial speed of u. Time after
which velocity vector of the projectile is perpendicular to 81. A projectile is thrown with an initial velocity of
the initial velocity. ( ai$ + bi$ ) ms −1 . If the range of the projectile is twice the
(a) u/ g sin θ (b) u/ g cos θ maximum height reached by it, then
(c) 2u/ g sin θ (d) 2u tan θ (a) a = 2b (b) b = a
(c) b = 2a (d) b = 4 a
74. A ball is projected with a velocity 20 3 m/ s at angle 60°
82. A projectile A is thrown v1
to the horizontal. The time interval after which the
at an angle 30° to the
velocity vector will make an angle 30° to the horizontal is v2
horizontal from point P.
(take g = 10 m/ s 2 )
At the same time another 30°
(a) 5 s (b) 2 s (c) 1 s (d) 3 s projectile B is thrown P Q

75. A projectile is thrown with a velocity of 10 m/ s at an with velocity v 2 upwards from the point Q vertically
below the highest point A would reach. For B to collide
angle of 60° with horizontal. The interval between the
v
moments when speed is 5g m/ s is ( g = 10 m/ s 2 ). with A, the ratio 2 should be
v1
(a) 1 s (b) 3 s (c) 2 s (d) 4 s
3
(a) (b) 2
76. A ball is projected from ground with a speed of 20 m/ s at 2
an angle of 45° with horizontal. There is a wall of 25 m 1 2
(c) (d)
height at a distance of 10 m from the projection point. The 2 3
ball will hit the wall at a height of
83. A particle is projected with a velocity of 30 m/ s, at an
(a) 5 m (b) 7.5 m
 3
(c) 10 m (d) 12.5 m angle of θ 0 = tan −1   . After 1 s, the particle is moving
 4
77. Two particles are simultaneously projected in opposite at an angle θ to the horizontal, where tan θ will be equal to
directions horizontally from a given point in space, where ( g = 10 m/ s 2 )
gravity g is uniform. If u1 and u 2 be their initial speeds,
(a) 1 (b) 2
then the time t after which their velocities are mutually 1 1
perpendicular is given by (c) (d)
2 3
u1 u2 u12 + u22
(a) (b) 84. The equations of motion of a projectile are given by
g g
u1 (u1 + u2 ) u2 (u1 + u2 ) x = 36 t m and 2 y = 96 t − 9.8 t 2 m. The angle of
(c) (d) projection is
g g
 4  3
(a) sin −1   (b) sin −1  
78. A ball is projected horizontally from the top of a tower  5  5
with a velocity v 0 . It will be moving at an angle of 60°  4  3
(c) sin −1   (d) sin −1  
with the horizontal after time.  3  4
v0 3 v0 v0 v0
(a) (b) (c) (d)
3g g g 2g 85. A ball is thrown from a point with a speed v 0 at an angle
of projection θ. From the same point and at the same
79. A ball rolls from the top of a stair way with a horizontal instant a person starts running with a constant speed v 0 / 2
velocity u m/ s. If the steps are h m high and b m wide, the to catch the ball. Will the person be able to catch the ball?
ball will hit the edge of the nth step, if If yes, what should be the angle of projection?
2hu 2hu2 2hu2 hu2 (a) Yes, 60° (b) Yes, 30°
(a) n = (b) n = (c) n = (d) n = 2
gb2 gb2 gb gb (c) No (d) Yes, 45°
Projectile Motion 143

[ Level 2 ]
T
Only One Correct Option 7. A ground to ground projectile is at point A at t = , is at
5T 3
1. A projectile has the maximum range 500 m. If the point B at t = and reaches the ground at t = T . The
6
projectile is thrown up a smooth inclined plane of 30° difference in heights between points A and B is
with the same (magnitude) velocity, the distance covered gT 2 gT 2 gT 2 gT 2
by it along the inclined plane till it stops will be (a) (b) (c) (d)
6 12 18 24
(a) 250 m (b) 500 m
(c) 750 m (d) 1000 m 8. A cart is moving horizontally along a straight line with a
constant speed of 30 m/ s . A projectile is to be fired from
2. A projectile is fired at an angle of 30° to the horizontal
the moving cart in such a way that it will return to the cart
such that the vertical component of its initial velocity is (at the same point on cart) after the cart has moved 80 m.
T
80 m/ s . Its time of flight is T. Its velocity at t = has a At what velocity (relative to the cart) must be projectile
4 be fired? (Take = 10 m/ s 2 )
magnitude of nearly 20 40 80
(a) 200 m / s (b) 300 m / s (a) 10 m/s (b) m/s (c) m/s (d) m/s
3 3 3
(c) 100 m / s (d) None of these
9. A body of mass 1kg is projected with velocity 50 m/ s at
3. A very broad elevator is going up vertically with a
an angle of 30° with the horizontal. At the highest point
constant acceleration of 2 m/ s 2 . At the instant when its
of its path a force 10 N starts acting on body for 5 s
velocity is 4 m/ s a ball is projected from the floor of the vertically upward besides gravitational force, what is
lift with a speed of 4 m/ s relative to the floor at an horizontal range of the body? ( g = 10 m/ s 2 )
elevation of 30°. The time taken by the ball to return the
(a) 125 3 m (b) 200 3 m
floor is ( g = 10 m/ s 2 )
(c) 500 m (d) 250 3 m
1 1
(a) s (b) s
2 3 10. A projectile is thrown with some initial velocity at an
1 angle α to the horizontal. Its velocity when it is at the
(c) s (d) 1 s
4 highest point is ( 2/ 5)1 / 2 times the velocity when it is at
4. A ball is dropped from a height of 49 m. The wind is height half of the maximum height. Find the angle of
blowing horizontally. Due to wind a constant horizontal projection α with the horizontal.
acceleration is provided to the ball. Choose the correct (a) 30° (b) 45° (c) 60° (d) 37°
statement (s ) . (Take g = 9.8 m/s 2 ) 11. Balls A and B are thrown from two points lying on the
(a) Path of the ball is a straight line same horizontal plane separated by a distance 120 m.
(b) Path of the ball is a curved one Which of the following statement (s ) is/are correct.
(c) The time taken by the ball to reach the ground is 3.16 s
(d) Actual distance travelled by the ball is more than 49 m
50 ms–1
5. A particle moves along a parabolic path y = − 9x 2 in such
30 ms–1
a way that the x component of velocity remains constant A 37° B
1
and has a value m/ s. The acceleration of the particle is 120 m
3
1 2 (a) The two balls can never meet
(a) m / s2 (b) 3 m / s2 (c) m / s2 (d) 2 m / s2 (b) The balls can meet, if the ball B is thrown 1 s later
3 3
(c) The two balls meet at a height of 45 m
6. Two particles A and B are projected simultaneously from (d) None of the above
a fixed point of the ground. Particle A is projected on a
12. Two second after projection, a projectile is travelling in a
smooth horizontal surface with speed v, while particle B
direction inclined at 30° to be horizontal. After 1 more
2v
is projected in air with speed . If particle B hits the second it is travelling horizontally. Then, ( g = 10 m/ s 2 )
3
(a) the velocity of projection is 20 3 m / s
particle A, the angle of projection of B with the vertical is (b) the angle of projection is 30° with horizontal
(a) 30° (b) 60° (c) Both (a) and (b) are correct
(c) 45° (d) Both (a) and (b) (d) Both (a) and (b) are wrong
144 Objective Physics Vol. 1
y
13. An object is projected with a velocity of 20 m/ s making 19. A man standing on a hill top v0 x
an angle of 45° with horizontal. The equation for projects a stone horizontally (0, 0)
trajectory is h = Ax − Bx 2 , where h is height, x is with speed v 0 as shown in
horizontal distance. A and B are constants. The ratio A : B figure. Taking the coordinates
is ( g = 10 m/ s 2 ) system as given in the figure.
(a) 1 : 5 (b) 5 : 1 The coordinates of the point,
θ
(c) 1 : 40 (d) 40 : 1 where the stone will hit the hill
14. A projectile is launched with a speed of 10 m/ s at an angle surface are
60° with the horizontal from a sloping surface of  2v 2 tan θ 2v 2 tan 2 θ   2v 2 2v 2 tan 2 θ 
(a)  0 ,− 0  (b)  0 , − 0 
inclination 30°. The range R is (Take g = 10 m/ s 2 )  g g   g g 
10 m/s  2v 2 tan θ 2v 2   2v 2 tan 2 θ 2v 2 tan θ 
(c)  0 , − 0 (d)  0 ,− 0 
60°  g g   g g 

R 20. The horizontal range and maximum height attained by a


projectile are R and H, respectively. If a constant
30° horizontal acceleration a = g / 4 is imparted to the
projectile due to wind, then its horizontal range and
(a) 4.9 m (b) 13.3 m (c) 9.1 m (d) 12.6 m maximum height will be
15. If the instantaneous velocity of a particle projected as H  H
(a) (R + H ), (b)  R +  , 2H
shown in figure is given by v = ai$ + ( b − ct )$j, where 2  2
a, b, and c are positive constants, the range on the (c) (R + 2H ), H (d) (R + H ), H
horizontal plane will be 21. A stone is projected from a point on the ground so as to
y
hit a bird on the top of a vertical pole of height h and then
attain a maximum height 2h above the ground. If at the
v instant of projection the bird flies away horizontally with
a uniform speed and if the stone hits the bird while
descending, then the ratio of the speed of the bird to the
x horizontal speed of the stone is
2 2 1 1 2
(a) 2ab/ c (b) ab/ c (a) (b) (c) + (d)
(c) ac/ b (d) a/2bc 2+1 2 −1 2 2 2+1

16. A particle is projected from horizontal making an angle 22. A projectile is thrown with velocity u making angle θ with
of 53° with initial velocity 100 m/ s. The time taken by the vertical. It just crosses the tops of two poles each of
particle to make angle 45° from horizontal is height h after 1 s and 3 s, respectively. The maximum
(a) 14 s (b) 2.0 s (c) 12 s (d) 4 s height of projectile is
(a) 9.8 m (b) 19.6 m (c) 39.2 m (d) 4.9 m
17. A ball is thrown from the ground to clear a wall 3 m high
at a distance of 6 m and falls 18 m away from the wall, the 23. A grasshopper can jump maximum distance 1.6 m. It
angle of projection of ball is negligible time of the ground. How far can it go in 10 s?
 3  2
(a) tan −1   (b) tan −1   (a) 5 2 m (b) 10 2 m (c) 20 2 m (d) 40 2 m
 2  3
−1  1 −1  3  More than One Correct Options
(c) tan   (d) tan  
 2  4
1. Two particles projected from the same point with same
18. Two particles are projected from the same point with speed u at angles of projection α and β strike the
same speed u at angles of projection α and β from horizontal ground at the same point. If h1 and h2 are the
horizontal. The maximum heights attained by them are h1 maximum heights attained by the projectile, R is the
and h2 respectively, R is the range for both and t 1 and t 2 range for both and t 1 and t 2 are their times of flights,
their times of flight respectively, then respectively , then
π π
(a) α + β = (b) R = 4 h1h2 (a) α + β = (b) R = 4 h1h2
2 2
t1 t1 h1
(c) = tanα (d) tanα = h1h2 (c) = tanα (d) tanα =
t2 t2 h2
Projectile Motion 145

2. A ball is dropped from a height of 49 m. The wind is 1. If α = 30° , β = 30°, the time of flight from P to Q is
blowing horizontally. Due to wind a constant horizontal u 3u 2u 2u
acceleration is provided to the ball. Choose the correct (a) (b) (c) (d)
g g g g
statement (s). [Take g = 9.8 ms −2 ]
2. If α = 30° , β = 30° and a = 4.9 m, the initial velocity of
(a) Path of the ball is a straight line
projection is
(b) Path of the ball is a curved one
(a) 9.8 ms−1 (b) 4.9 ms−1 (c) 4.9 2 ms−1 (d) 19.6 ms−1
(c) The time taken by the ball to reach the ground is 3.16 s
(d) Actual distance travelled by the ball is more then 49 m
Assertion and Reason
3. A particle is projected from a point P with a velocity v at Directions (Q. Nos. 1-13) These questions consists of two
an angle θ with horizontal. At a certain point Q it moves at statements each printed as Assertion and Reason. While
right angles to its initial direction. Then,
answering these questions you are required to choose any one
(a) velocity of particle at Q is v sinθ of the following five responses.
(b) velocity of particle at Q is v cotθ
(a) If both Assertion and Reason are correct and Reason
(c) time of flight from P to Q is (v /g ) cosec θ is the correct explanation of Assertion
(d) time of flight from P to Q is (v /g ) sec θ
(b) If both Assertion and Reason are true but Reason is
4. At a height of 15 m from ground velocity of a projectile is not the correct explanation of Assertion
v = (10 ^i + 10 ^j ). Here, ^j is vertically upwards and ^i is (c) If Assertion is true but Reason is false
(d) If Assertion is false but Reason is true
along horizontal direction then, ( g = 10 ms −2 )
(e) If both Assertion and Reason are false
(a) particle was projected at an angle of 45° with horizontal
(b) time of flight of projectile is 4 s 1. Assertion In case of projectile motion, the magnitude of
(c) horizontal range of projectile is 100 m rate of change of velocity is variable.
(d) maximum height of projectile from ground is 20 m
Reason In projectile motion, magnitude of velocity first
5. Which of the following quantities remain constant during decreases and then increases during the motion.
projectile motion?
2. Assertion A particle is projected with speed u at an
(a) Average velocity between two points
angle θ with the horizontal. At any time during motion,
(b) Average speed between two points
speed of particle is v at angle α with the vertical, then
dv
(c) v sin α is always constant throughout the motion.
dt
d2v Reason In case of projectile motion, magnitude of
(d) 2
dt radial acceleration at topmost point is maximum.

6. In the projectile motion 3. Assertion If in a projectile motion, we take air friction


shown is figure, given into consideration, then t ascent < t descent .
A B
t AB = 2 s, then Reason During ascent magnitude of retardation is
( g = 10 ms −2 ) 15 m greater than magnitude of acceleration during descent.
(a) particle is at point B at 4. Assertion In projectile motion, the angle between
3s O
20 m 40 m B
(b) maximum height of
instantaneous velocity vector and acceleration vector can
projectile is 20 m be anything between 0 to π (excluding the limiting case.)
(c) initial vertical component of velocity is 20 ms−1 Reason In projectile motion, acceleration vector is
(d) horizontal component of velocity is 20 ms−1 always pointing vertically downwards. (Neglect air
friction).
Comprehension Based Questions 5. Assertion In projectile motion if time of flight is made
Passage (Q. 1 to 2) two times, then maximum height will become four times.
u
Two inclined planes Reason T ∝ sin θ and H ∝ sin 2 θ
A B
OA and OB intersect in a
horizontal plane having
P a Q where, θ is angle of projection.
α β
their inclinations α and β 6. Assertion In projectile motion if time of flight is 4 s.
O
with the horizontal as Then, maximum height will be 20 m. ( g = 10 m/ s 2 )
shown in figure. A particle is projected from point P with
gT
velocity u along a direction perpendicular to plane OA. Reason Maximum height = .
The particle strikes plane OB perpendicularly at Q. 2
146 Objective Physics Vol. 1

7. Assertion For projection angle tan −1 ( 4 ), the horizontal Column I Column II

and maximum height of a projectile are equal. (A) Angle of projection (p) 20 m
(B) Angle of velocity with horizontal after 4 s (q) 80 m
Reason The maximum range of projectile is directly (C) Maximum height (r) 45°
proportional to square of velocity and inversely
(D) Horizontal range (s) tan−1  1 
proportional to acceleration due to gravity.  
2
8. Assertion A particle in xy-plane is governed by
x = a sin ωt and y = a cos ωt, where a as well as ω are 2. A particle is projected horizontally from a tower with
constants, then the particle will have parabolic mation. velocity 10 m/ s. Taking g = 10 m/ s 2 . Match the following
two columns at time t = 1s.
Reason A particle under the influence of mutually
perpendicular velocities has parabolic motion. Column I Column II

9. Assertion Particle-1 is dropped from a tower and (A) Horizontal component of velocity (p) 5 SI unit
particle-2 is projected horizontal from the same tower. (B) Vertical component of velocity (q) 10 SI unit
Then, both the particles reach the ground simultaneously. (C) Horizontal displacement (r) 15 SI unit
Reason Both the particles strike the ground with (D) Vertical displacement (s) 20 SI unit
different speeds.
3. A particle is projected from ground with velocity u at
10. Assertion If a particle is projected vertices upwards
angle θ from horizontal. Match the following two
with velocity u, the maximum height attained by the
columns.
particle is h1 . The same particle is projected at angle 30°
from horizontal with the same speed u. Now, the Column I Column II
maximum height is h2 . Thus, h1 = 4h2 . (A) Average velocity between initial (p) u sinθ
and final points
Reason In first case v = 0 at highest point and in second
(B) Change in velocity between initial (q) u cosθ
case v ≠ 0 at highest point. and final points

11. Assertion At highest point of a projectile dot product of (C) Change in velocity between initial (r) zero
and final points
velocity and acceleration is zero.
(D) Average velocity between initial (s) None of the above
Reason At highest point velocity and acceleration are and highest points
mutually perpendicular.
1 4. Given that u x = horizontal component of initial velocity
12. Assertion On the surface of moon, the value of g is th of a projectile, u y = vertical component of initial
6
the value on the surface of the earth. A particle is velocity, R = horizontal range, T = time of flight and
projected as projectile under similar condition on the H = maximum height of projectile. Now match the
surface of moon and on the surface of earth. Then, values following two columns.
of T , H and R on the surface of moon will become six Column I Column II
times.
(A) u x is doubled, u y is halved (p) H will remain unchanged
1
Reason T , H and R ∝ (B) u y is doubled u x is halved (q) R will remain unchanged
g
(C) u x and u y both are (r) R will become four times
13. Assertion At height 20 m from ground velocity of a doubled

projectile is v = ( 20$i + 10$j ) m/ s. Here, $i is horizontal and $j (D) Only u y is doubled (s) H will become four times

is vertical. Then, the particle is at the same height after 4 s.


5. Two particles are projected from a tower in opposite
Reason Maximum height of particle from ground is
directions horizontally with speed 10 m/ s each. At t = 1s
40 m. (Take g = 10 m/ s 2 ) match the following two columns.
Match the Columns Column I Column II

1. Trajectory of particle in a projectile motion is given as (A) Relative acceleration between two (p) Zero
x2 (B) Relative velocity between two (q) 5 SI unit
y = x − . Here, x and y are in metre. For this projectile (C) Horizontal distance between two (r) 10 SI unit
80
motion match the following with g = 10 m/ s 2 . (D) Vertical distance between two (s) 20 SI unit
Projectile Motion 147

Entrance Gallery 6. An aircraft is flying at a height of 3400 m above the


ground. If the angle subtended at a ground observation
2014 point by the aircraft positions 10 s apart is 30°, then the
speed of the aircraft is [Kerala CEE]
1. A particle of mass m is projected from the ground with an
initial speed u 0 at an angle with the horizontal. At the (a) 19.63 ms −1 (b) 1963 ms −1
(c) 108 ms −1 (d) 196.3 ms −1
highest point of its trajectory, it makes a completely
(e) 10.8 ms −1
inelastic collision with another identical particle, which
was thrown vertically upward from the ground with the 7. The height y and the distance x along the horizontal plane
same initial speed u 0 . The angle that the composite of a projectile on a certain planet (with no surrounding
system makes with the horizontal immediately after the atmosphere) are given by y = 8t − 5t 2 metre and
collision is [JEE Advanced] x = 6t metre, where t is in second. The velocity with
π π which the projectile is projected, is [Karnataka CET]
(a) (b) +α
4 4 (a) 14 ms −1 (b) 10 ms −1 (c) 8 ms −1 (d) 6 ms −1
π π
(c) − α (d)
4 2 2010
2013 8. A particle is moving with velocity v = k ( yi$ + x $j ), where
k is a constant. The general equation for its path is
2. A projectile is given an initial velocity of ( $i + 2$j ) m/s, [AIEEE]
where, $i is along the ground and $j is along the vertical. If (a) y = x 2 + constant (b) y2 = x + constant
g = 10 m/s 2 , then equation of its trajectory is [JEE Main] (c) xy = constant (d) y2 = x 2 + constant

(a) y = x − 5x 2 (b) y = 2x − 5x 2 9. A small particle of mass m is projected at an angle θ with


(c) 4 y = 2x − 5x 2
(d) 4 y = 2x − 25x 2 the x-axis with an initial velocity v 0 in the xy-plane as
v sin θ
3. A boy can throw a stone up to a maximum height of 10 m. shown in the figure. At a time t < 0 , the angular
g
The maximum horizontal distance that the boy can throw
the same stone up to will be [JEE Main]
momentum of the particle is [AIEEE]
y
(a) 20 2 m (b) 10 m
v0
(c) 10 2 m (d) 20 m

2012
4. A particle of mass m is projected with a velocity v making θ
x
an angle of 30° with the horizontal. The magnitude of
angular momentum of the projectile about the point of (a) − mgv0 t cosθ $j
2
(b) mgv0 t cosθ k$
projection when the particle is at its maximum height h is 1 1
(c) − mgv0 t 2 cosθ k$ (d) mgv0 t 2 cosθ $i
[AIEEE] 2 2
3 mv 2
(a) (b) zero 10. A boy throws a cricket ball from the boundary to the
2 g
wicket-keeper. If the frictional force due to air cannot be
mv 3 3 mv 3
(c) (d) ignored, the forces acting on the ball at the position X are
2g 16 g respected by [Karnataka CET]
X
2011 Boy

5. Two projectiles A and B thrown with speeds in the ratio


1: 2 acquired the same heights. If A is thrown at an angle
of 45° with the horizontal, the angle of projection of B
will be [Kerala CEE] (a) (b)
(a) 0° (b) 60°
(c) 30° (d) 45° (c) (d)
(e) 15°
Answers
Level 1
Objective Problems
1. (a) 2. (d) 3. (b) 4. (d) 5. (c) 6. (d) 7. (d) 8. (c) 9. (d) 10. (d)
11. (c) 12. (a) 13. (d) 14. (b) 15. (a) 16. (c) 17. (b) 18. (c) 19. (b) 20. (d)
21. (b) 22. (d) 23. (b) 24. (c) 25. (b) 26. (d) 27. (d) 28. (a) 29. (d) 30. (c)
31. (b) 32. (a) 33. (b) 34. (c) 35. (a) 36. (a) 37. (b) 38. (a) 39. (a) 40. (d)
41. (b) 42. (b) 43. (d) 44. (d) 45. (a) 46. (c) 47. (c) 48. (b) 49. (c) 50. (c)
51. (a) 52. (d) 53. (c) 54. (c) 55. (b) 56. (a) 57. (b) 58. (c) 59. (c) 60. (b)
61. (c) 62. (a) 63. (b) 64. (d) 65. (c) 66. (a) 67. (b) 68. (c) 69. (d) 70. (c)
71. (d) 72. (a) 73. (a) 74. (b) 75. (a) 76. (b) 77. (a) 78. (b) 79. (b) 80. (c)
81. (c) 82. (c) 83. (d) 84. (a) 85. (a)

Level 2
Only One Correct Option
1. (b) 2. (d) 3. (b) 4. (a,c,d) 5. (d) 6. (b) 7. (d) 8. (c) 9. (d) 10. (c)
11. (c) 12. (a) 13. (d) 14. (b) 15. (a) 16. (a.b) 17. (b) 18. (all) 19. (a) 20. (d)
21. (d) 22. (b) 23. (c)

More than One Correct Options


1. (all) 2. (a,c,d) 3. (b,c) 4. (b,d) 5. (c,d) 6. (all)

Comprehension Based Questions


1. (b) 2. (a)

Assertion and Reason


1. (d) 2. (b) 3. (a) 4. (a,b) 5. (a) 6. (c) 7. (b) 8. (e) 9. (b) 10. (b)
11. (a) 12. (a) 13. (b)

Match the Columns


1. (A → r, B → r, C → p, D → q) 2. (A → q, B → q, C → q, D → p) 3. (A → q, B → s, C → p, D → p)
4. (A → q, B → q, r, C → r, s, D → s) 5. (A → p, B → s, C → s, D → p)

Entrance Gallery
1. (a) 2. (b) 3. (d) 4. (d) 5. (c) 6. (d) 7. (b) 8. (d) 9. (c) 10. (c)
Solutions
Level 1 : Objective Problems 17. H = R or
u2 sin 2 θ u2 sin θ cosθ
= or tanθ = 4
u2 sin 2θ ( 9.8)2 sin 90° 2g g
1. R = = = 9.8 m
g 9.8 ∴ θ = tan −1 ( 4)
3. ax = 0 u2
18. Rmax = at θ = 45°
∴ ux = constant g
At highest point vertical component of velocity is zero. Only ∴ u = g Rmax = 100 m/s
horizontal component of velocity is present. 19. RA = RB
4. Velocity is horizontal and acceleration is vertical. v 2 sin 2θ (v /2)2 sin 90°
=
5. Projectile will strike at highest point of its path with velocity g g
v 0 cosα.
sin 2θ =
1 1
or θ = sin −1 1
T sin 30° 1  
6. T ∝sin θ, A = = or TB = 3 TA 2 2  4
TB sin 60° 3
1 −1  1 
HA sin 2 30° 1 ∴ θ = sin  
H ∝sin 2 θ, = = 2  4
HB sin 2 60° 3
20. H ∝sinα
or HB = 3 HA H1 sin 2 α
Rθ = R90 ° − θ ∴ = = tan 2 α
H2 sin ( 90° − α )
2

∴ RA = RB 1
21. R ∝
u2 sin 2 θ
7. R = g
g ∴ Rmoon = 6 Rearth
∴ R ∝ u2 22. At 45° range is maximum. At15° and 75° ranges are equal as
If initial velocity be doubled, then range will become four Rθ = R( 90 ° − θ )
times. u12 sin 2 45° u22 sin 2 60°
23. =
u sin θ
2 2
2g 2g
8. H =
2g u1 sin 60° 3 /2
∴ = = = 3: 2
∴ H ∝ u2 u2 sin 45° (1/ 2 )
If initial velocity be doubled, then maximum height 24. Rθ = R( 90 ° − θ )
reached by the projectile will become four times.
u2
9. R = 4H cotθ if θ = 45°, then R = 4H cot 45° = 4 H 25. θ = 45° , Rmax = = 80
g
10. Acceleration throughout the projectile motion remains
constant and equal to g. ∴ u2 = 800 m/s2
2usin θ 2 × 50 × sin 30° u2 sin 2 45° (800)(1/2)
11. Time of flight = = = 5s Now, H= =
g 10 2g 20
u2 sin 2 θ 2usinθ = 20 m
12. H = and T =
2g g
θ = cos−1  
u 1 1
26. = x or cosθ = ⇒ ∴
H u2 sin 2 θ/2 g g 5 ucosθ x x
So, = = =
T 4u2 sin 2 θ/2 g 2 8 4 u
27. ucosθ =
13. R = 4H cotθ 2
If θ = 45°, then 4H ⇒
R 4
= ∴ θ = 60°
H 1 u2 sin 2θ u2 sin120° 3 u2
Now, R = =
14. Air resistance is in opposite direction of velocity. g g 2g
15. An external force by gravity is present throughout the 2uxuy 2 × 9.8 × 19.6
motion. So, momentum will not be conserved. 28. R = = = 39.2 m
g 9.8
16. R = 4 3H
2uy
2u2 sin θcosθ  u2 sin 2 θ  29. T = =3
=4 3  g
g  2g 
1 ∴ uy =15 m/s
tanθ =
3 u2 (15)2
Now, H= y = = 11.25 m
∴ θ = 30° 2g 20
150 Objective Physics Vol. 1

u2 u2 sin 2 ( 90° − θ) u2 cos2 θ


30. =h (given) and h2 = =
2g 2g 2g
u2 sin 90° u2 2
 u2 sin 2θ  1 R 2
Now, Rmax = = = 2h h1h2 =   ⋅ =
g g  g  16 16
31. 2θ = 90° − θ ∴ R = 4 h1h2
or 3θ = 90° or θ = 30° 2uy
32. Velocity of boy should be equal to the horizontal 41. T =
g
component of velocity of ball.
gT
u2 sin ( 2 × 15° ) u2 ∴ uy = = 25 m/s
33. R15 ° = = = 1.5 km 2
g 2g
u2 ( 25)2
u2 sin ( 2 × 45° ) u2 Now, H= y =
R45 ° = = = 1.5 × 2 = 3 km 2g 20
g g
= 31.25 m
u2 sin( 90° − 2θ) u2 cos 2θ
34. For angle ( 45° − θ), R = = Further, R = ux T
g g
R
u2 sin( 90° + 2θ) u2 cos 2θ ∴ ux =
For angle ( 45° + θ), R = = T
g g
= 40 m/s
2usin θ
35. T = = 10 s 42. ∆a = a ∆t (as a = constant)
g
 2v sin α 
⇒ usin θ = 50 m/s = ( − g j$ )  0  = ( − 2v 0 α )$j
 g 
u2 sin 2 θ (usin θ)2
∴ H= =
2g 2g i.e. change in velocity is 2v 0 sin α, vertically downwards.
1
50 × 50 43. h = g t 2 (in vertical direction)
= = 125 m 2
2 × 10
2h 2 × 0.1
36. When the angle of projection is very far from 45°, then range ∴ t= = = 0.141s
will be minimum. g 10
u2 sin 2 θ 2usin θ 4u2 sin 2 θ Now, in horizontal direction,
37. H = and T = ⇒ T2 =
2g g g2 S 100
vx = x = ≈ 700 m/s
t 1.141
T2 8 8H 2H
∴ = ⇒ T= =2 44. ux =19.6 m/s and uy =19.6 m/s. After 2 s, vertical component
H g g g
of velocity v y will become zero. So, particle is at its highest
(10)2 sin 2θ 3
38. 5 3 = or sin 2θ = point.
g 2
3v
45. At highest point velocity will remain v cos30° or .
∴ 2θ = 60° or θ = 30° 2
Two different angles of projection are therefore, θ and 3p
Therefore, momentum will also remain .
( 90° − θ) or 30° and 60°. 2
2usin 30° 1
T1 = =1s 46. s y = uy t + a y t 2 or − 70 = 25t − 5t 2
g 2
usin 60° Solving this equation, we get t = 7s.
T2 = 2 = 3s
g gx 2
47. Compare with y = x tan θ = 2
∴ ∆t = T2 − T1 = ( 3 − 1)s 2u cos2 θ
u sin 2θ
2
tanθ = 3
39. R = at angles θ and ( 90° − θ)
g ∴ θ = 60°

Now, t1 =
2usinθ 48. v y = gt = 7 m/s,v x = 4 m/s
g
∴ v = v x2 + v 2y ≈ 8 m/s
2u sin( 90° − θ) 2u cosθ
and t2 = = 1
g g 49. At highest point speed will remain times ( = ucos 45° ).
2
2  u sin 2θ  2R
2
∴ t1t2 =  =
1
Therefore, kinetic energy will become times.
g g  g 2
u2 sinθ dy dx
40. R = at angles θ and ( 90° − θ) 50. v y = = 8 − 10t, v x = =6
g dt dt
At t = 0,v y = 8 m/s and v x = 6 m/s
u2 sin 2 θ
Now, h1 =
2g ∴ v = v x2 + v 2y = 10 m/s
Projectile Motion 151

2h 2 × 1960 2h 4
51. t = = = 20 s 60. t = = = 0.64 s
g 9.8 g (9.8)
20 3 3
∴Horizontal distance = 600 × km = 3.33 km Now, v=
= = 4.7 m/s
60 × 60 t 0.64
52. T = ( 24 sin θ/ g ) = 4 s , R = u2 sin 2 θ/ g = 80 m 61. Horizontal component of velocity remains unchanged.
Hence,
v cos φ = u cos θ
∴ v = u cos θ sec φ
s 20 62. K = K 0 − mgh
40 Here, K = kinetic energy at height h,
K 0 = initial kinetic energy. Variation of K with h is linear. At
u sin θ 2 2
and H= = 20 m highest point kinetic energy is not zero.
2g
gx 2
Displacement 63. Comparing with y = x tan θ − 2 we have,
Now, average velocity = 2u cos2 θ
Time
tanθ = 3 or θ = 60°
( 20)2 + ( 40)2
= and u2 cos2 θ = 1
2
=10 5 m/s or u = sec θ = sec 60° = 2 m/s
 T
64. ∆v = a∆t = a ⋅  
53. H and R both are proportional to u2 . Hence, percentage
 2
increases in horizontal range would also be 10%.
 usin θ 
54. At highest point kinetic energy will be minimum but not = ( − g j$ )   = ( − usin θ)$j
zero.  g 
R
55. H = Therefore, change in velocity isusinθin downward direction.
4
R/2 3H
u2 sin 2 θ 2u2 sin θ cosθ 65. = = 3
∴ = H H
2g 4g
(v 02 sin θcosθ)/g
or = 3
or tanθ =1 (v 02 sin 2 θ)/2g
∴ θ = 45°
2 cot θ = 3
p
At highest point momentum will remain . 2
2 or tanθ =
3
( p/ 2 )2 p2
∴ K = =
θ = tan −1 
2 
2m 4m or 
 3
1
56. From s y = uy t + a y t 2 we have 66. Horizontal component of velocity v x = 500 m/s and vertical
2
component of velocity while striking the ground.
− 40 = ( 20 sin 30° )t − 5t 2
u = 500 m/s
or t 2 − 2t − 8 = 0
Solving this, we get t = 4s
2u sin θ
T= 500 m/s
g
θ
2 × 20 × sin 30°
= = 2s
10 100 m/s
t
∴ =2 v y = 0 + 10 × 10 = 100 m/s
T
∴ Angle with which it strikes the ground
57. Put y = 0,
v 
3 θ = tan −1  y 
0 = 12x − x 2  yx 
4
= tan −1 
100  −1  1 
x =16 m  = tan  
 500   5
u2 sin 2 θ u2 sin 2 ( 90° − θ) u2
58. y1 + y 2 = + = 67. Area in which bullet will spread = πr 2
2g 2g 2g
v2
H 2H (u2 sin 2 θ/g ) For maximum area, r = Rmax = (when θ = 45° )
59. tan φ = = = 2 g
R/ 2 R u sin 2θ/g
 v 2  πv 4
sin 2 θ tan θ Maximum area = π Rmax
2
= π  = 2
= =
2sin 2cosθ 2  g  g
152 Objective Physics Vol. 1

68. Instantaneous velocity of rising mass after t s will be vV gt


78. tan60° = =
vH v0
vt = v x2 + v 2y
3 v0
where, v x = v cosθ = horizontal component of velocity ∴ t=
g
v y = v sinθ − gt = vertical component of velocity. 1 2
79. nh = gt and nb = ut
vt = (v cosθ)2 + (v sin θ − gt )2 2
2hu2
vt = v 2 + g 2 t 2 − ( 2v sin θ) gt From these two equation we get, n =
gb 2
u2 sin 2θ 2uxuy H1 + H2
69. R = = 80. H1 − H2 = or H1 = 3H2
g g 2
∴ Range ∝ horizontal initial velocity component (ux ). u2 sin 2 θ u2 sin 2 ( 90° − θ) 
∴ = 2 
In path 4 range is maximum of football has maximum 2g  2g 
horizontal velocity component in this path.
tan 2 θ = 3
72. Comparing with projectile motion we can see that it is like a
∴ tanθ = 3
projectile motion withux = 4 m/s,uy = 4 m/s and a y =10 m/s2 .
or θ = 60°
2uxuy
x-coordinate = range = Therefore, the other angle is ( 90° − θ) or 30°
g
2u u 2u2
2× 4× 4 81. R = 2H or x y = y
= = 3.2 m g 2g
10
or 2ux = y or 2a = b
73. v ⊥ u = 0 or v ⋅ u = 0
82. Vertical component of velocity of A should be equal to
or (u + a t)⋅ u = 0 vertical velocity of B.
or u ⋅ u + (a ⋅ u )t = 0 or v1 sin 30° = v 2
or u2 + gut cos( 90° + θ) = 0 v1
or = v2
(angle between u and u is 90° + θ) 2
v2 1
or u − g t sinθ = 0 ∴ =
u v1 2
or t=
g sin θ 4
83. ux = u cosθ0 = 20 × = 24 m/s
vy uy − gt 5
74. tan 30° = = 3
vx ux and uy = usin θ0 = 30 × = 18 m/s
5
( 20 3 sin 60° ) − 10 t After 1 s, ux will remain as it is uy will decreases by10 m/s or it
=
( 20 3 cos 60° ) will remain 8 m/s
v 8 1
or 10 = 30 − 10t ∴ tanθ = y = =
∴ t = 2s v x 24 3
75. v 2 = v 2y + v x2 or 5g = (uy − gt )2 + ux2 84. x = 36 t
50 = ( 5 3 − 10t ) + ( 5)
2 2 dx
or ∴ vx = = 36 m/s
dt
∴ ( 5 3 − 10t ) = ± 5
y = 48t − 4.9 t 2
5 3 + 15 5 3−5
t1 = and t2 = ∴ v y = 48 − 9.8 t
10 10
at t = 0 v x = 36 m/s
∴ t1 − t2 = 1 s
10 1 and v y =18 m/s
76. t = = s
20 cos 45° v 
2 So, angle of projection θ = tan −1  y 
1  vx 
Now, y = ( 20 sin 45° ) t − gt 2
= tan −1  
2 4
1 1 1 1  3
= 20 × × − × 10
θ = sin −1  
2 2 2 2 4
or
= 7.5 m  5
77. v 1 ⊥ v 2 85. Person will catch the ball, if its velocity will be equal to
∴ v1 ⋅ v 2 = 0 horizontal component of velocity of the ball.
v0
or (u1 $i − gt $j ) ⋅ ( − u2 i$ − gt $j ) = 0 = v 0 cosθ
2
∴ g 2t 2 = u1u2 1
⇒ cosθ =
u1u2 2
or t=
g ⇒ θ = 60°
Projectile Motion 153

2uy
Level 2 : Only One Correct Option 7. T =
g
u2
1. = Rmax = 500 ∴ uy =
gT
g 2
u2 u2 u2 hA = uy tA −
1 2
Now, S= = = = 500 m gtA
2a 2g sin 30° g 2
2
uy  2u  1  2u 
2. tan 30° = = uy  y  − g  y 
ux  3g  2  3g 
4 u2 y  4   gT 
1 80 2
∴ = gT 2
= =    =
3 ux 9 g  9g   2  9
80
or ux = m/s  5 2u  1  5 2u 
2

3 hB = uy  × y  − × g ×  × y 
9 g  2 6 g 
2u 2 × 80
T= y = = 16s 2 2
g 10 5 uy 5  gT  5
= =   = gT
2

T 18 g 18g  2  12
At t = = 4s
4 gT 2
80 ∴ hA − hB =
v x = ux = m/s 24
3 8. The time taken by cart to cover 80 m
s 80 8
= = s
v 30 2
uy
The projectile must be fired (relative to cart) vertically
upwards.
8/3 4
30° a = − g = − 10 ms−2 , v′ = 0 and t = = s
ux 2 3
4
and v y = uy + a y t ∴ v ′ = u + at or 0 = u − 10 ×
3
or v y = 80 + ( − 10)( 4) = 40 m/s 40 −1
or u= ms
2 40 3
∴ Speed = (80 3) + ( 40)2 = m/s
13 9. For 5 s weight of the body is balanced by the given force.
3. Let us see the motion relative to elevator, Hence, it will move in a straight line as shown.
ar = ab − ae = ( − 10) − ( + 2) = − 12 m/s2
2u 2 × usinθ
Now, T= y =
ar ar
2 × 4 × sin 30° 1
= = s
12 3
4. As initial velocity is zero. Particle will move in a straight line 5s
along anet .
u sin 2 θ
2
a R= + (ucosθ)( 5)
g
( 50)2 ⋅ sin 60°
= + ( 50 × cos 30° )( 5)
10
= 250 3 m
g anet 2
10. (ucos α ) = (ucosα 2 ) + {(usin α )2 − 2gh}
2h 2 × 49 5
Further, t= = = 10 = 3.16 s
g 9.8 H u2 sin 2 α
Here, h = =
5. Comparing with the trajectory of projectile in which 2 4g
particle is projected from certain height horizontally Solving this equation, we get α = 60º
(θ = 0° ). 11. Two balls will meet if,
gx 2 ( 50 cos 37° ) tA = 120 or tA = 3 s
y = x tan θ − 2
2u cos2 θ Vertical component of A is also 50 sin 37° or 30 m/s, so they
Putting θ = 0° and g = a = 18u = 2 m/s2
2
will meet if thrown simultaneously.
6. Their horizontal components should be same. hA = hB
2v 1
∴ ⋅ cosθ = v = 30 × 3 − × 10 × ( 3)2
3 2
or θ = 60° = 45 m
154 Objective Physics Vol. 1

12. T /2 = 2 + 1 = 3 s or T = 6s u2 sin 2θ
17. R = = 24 …(i)
2uy g
∴ =6
g
∴ uy = 30 m/s
v y uy − gt 30 − 20
Further, tan 30° = = =
vx ux ux
or ux =10 3 m/s 3m

or u = ux2 + u2y
6m 18 m
= 20 3 m/s
gx 2
u 30 In y = x tan θ −
tanθ = y = = 3 2u cos2 θ
2
ux 10 3
36g
or θ = 60° 3 = 6 tan θ − 2 …(ii)
2u cos2 θ
13. Standard equation of projectile motion
g sin 2θ sin θcosθ
gx 2 From Eq. (i), = =
y = x tan θ − 2 u2 24 12
2u cos2 θ
Substituting in Eq. (ii), we have
Comparing with given equation
3 9
g 3 = 6 tan θ − tan θ = tanθ
A = tanθ and B = 2 2
2u2 cos2 θ
−1  2 
A tan θ × 2u2 cos2 θ ∴ θ = tan  
So = = 40  3
B g
18. (a) Range becomes equal at complimentary angle.
( As θ = 45° , u = 20 m/s, g =10 m/s2 ) Hence, β = 90° − α
14. At B , s y = 0 u2 sin 2 α
(b) h1 =
1 2u − 2 (10) 4 2g
∴ uy t + a y t 2 = 0 or t = − y = = s
2 ay − 10 × 3/ 2 3 u2 cos2 α
h2 = (as β = 90 − α )
axt 2 = 10 ×    =13.33 m
1 1 1 16 2g
Now, AB = R =
2 2 2  3  2u2 sin α cosα
y
∴ 4 h1h2 =
g
u2 sin 2α
x 10 m/s = =R
A g
t1 ( 2usin α )/g
(c) = = tan α
B t2 ( 2ucosα/g )
30° h1
(d) = tanα
15. ux = a ,uy = b , g = c h2
1 2
2uxuy 2ab gt
R= = 19.
AB 2
= = tanθ
g c BC v 0t
16. Component 60 m/ s will remain unchanged. Velocity will
A v0
make 45° with horizontal when vertical component also
becomes ± 60 m/s.

θ C
B
80 m/s 100 m/s

θ
53°
2v 0 tanθ
60 m/s ∴ t=
g
Using, v = u + at (in vertical direction)
+ 60 = 80 + ( − 10)t1 2v 02 tanθ
Now, x-coordinate = v 0t =
∴ t1 = 2 s g
− 60 = 80 + ( − 10)t2 1 2 2v 2 tan 2 θ
and y-coordinate = − gt = − 0
∴ t2 = 14 s 2 g
Projectile Motion 155

20. H ′ = H (as vertical component of acceleration has not (v cot θ) ⋅ cos θ + v sin θ
=
changed) g
1 v cosec θ
R ′ = ux T +
ax T 2 =
2 g
1 g 4u2 sin 2 θ 4. ux = v x =10 m / s
=R+ × ×
2 4 g2
uy = v 2y + 2gh
u2 sin 2 θ
=R = (R + H) = (10)2 + ( 2)(10)(15)
2g
= 20 m/s
u2
23. Rmax = = 1.6 m (at θ = 45° ) Angle of projection,
g
u 
or u = 16 = 4 m/s θ = tan −1  y  = tan −1 ( 2)
 ux 
2uy
Now, T= 2uy ( 2)( 20)
g T= = = 4s
g 10
2( 4/ 2 ) 4 R = uxT = (10)( 4) = 40 m
= = s
10 5 2
u2y ( 20)2
10 H= = = 20 m
∴ Total distance travelled in 10 s = 1.6 × 2g 2 × 10
4/5 2
dv
= 20 2 m 5. = a = constant = g
dt
d 2v d a
More than One Correct Options = = 0 = constant
dt 2 dt
1. α + β = 90° or β = 90° − α 6. Horizontal component of velocity remains unchanged
u2 sin 2 α u2 cos2 α X
h1 = and h2 = XOA = 20 m = AB
2g 2g 2
tAB
2usinα 2u cosα ∴ tOA = =1 s
t1 = and t2 = 2
g g
For AB projectile
2u2 sin α cosα
R1 = R2 = =R 2u
g T = 2s= y
g
2. Since u = 0, motion of particle is a straight line in the
∴ uy =10 m/s
direction of anet .
u2 (10)2
aH H= y = = 5m
2g 2 × 10
∴ Maximum height of total projectile,
= 15 + 5 = 20 m
tOB = tOA + tAB = 1 + 2 = 3 s
For complete projectile
2uy
g T = 2 (tOA ) + tAB = 4 s =
a net g
∴ uy = 20 m / s
2h 2 × 49
t= = AB 40
g 9.8 ux = = = 20 m/s
tAB 2
= 3.16 s
3. Horizontal component of velocity remains unchanged Comprehension Based Questions
∴ v cos θ = v ′ cos( 90° − θ) 1. At Q, component parallel to OB becomes zero
or v ′ = v cot θ y
x
In vertical ( y ) direction,
v y = uy + a y t
u Q
v y − uy
∴ t=
ay 90° 30°
30° 30°
−v sin ( 90 − θ) − v sin θ
1
=
−g v x = ux + ax t
156 Objective Physics Vol. 1

0 = (u cos 30° ) + ( − g sin 30° ) t 11. Velocity is horizontal and acceleration is vertical.
u 3u 2uy 2 × 20
∴ t = cot 30° = 13. t = = = 4s
g g g 10
2. PQ = range = 2 (PM ) = 2a cos 30° u2
H = 20 + y
 3 2g
= ( 2)( 4.9)   = 4.9 3 m ( 20)2
 2  = 20 + = 40 m
2 × 10
u2 sin 2 (60° )
=
9.8
u2 ( 3/2) Match the Columns
∴ 4.9 3 =
9.8 1. Comparing with the standard equation of projectile,
∴ u = 9.8 m/s gx 2
y = x tan θ − 2
2u cosθ
u
We get θ = 45°
60° M Q 60° and u = 20 2 m/s
P 30° 30° Time period of this projectile is 4 s. Hence, after 4 s velocity
90° vector will again make 45° with horizontal.
a 120° v
30° 30°
O
Entrance Gallery
1. From momentum conservation equation, we have
/ Velocity at P is making an angle of 60° with horizontal and
p =p
i ⇒ m (u cos α )$i + m ( u2 − 2gH )$j = ( 2m) v
f 0 0 ...(i)
velocity at Q is making an angle of 60° with horizontal. That is
the reason PQ = range. Because under this condition, points j
√u20 – 2gH
P and Q lie on same horizontal line.
u0 cos α

Assertion and Reason m


i
dv
1. = | a | = 9.8 m/s2 = constant u2 sin 2 α
dt H= 0 ...(ii)
2g
2. Assertion v sin α = horizontal component of velocity u cos α $ u0 cos α $
From Eqs. (i) and (ii), we get v = 0 i+ j
= constant 2 2
Reason ar = g − at2 2
Since, both components of v are equal. Therefore, it is
making 45 ° with horizontal.
At highest point at = 0. Therefore, ar is maximum.
2. Initial velocity = ( $i + 2$j ) m / s
3. v = velocity
w = weight and Magnitude of initial velocity, u = (1)2 + ( 2)2 = 5 m/s
A = air resistance Equation of trajectory of projectile is
v v gx 2  tan θ = y = 2 = 2
y = x tan θ − 2 (1 + tan 2 θ)
2u  x 1 
10 ( x )2
∴ y = x ×2− [1 + ( 2)2 ]
2 ( 5 )2
10 ( x 2 )
w, A w, A = 2x − (1 + 4) = 2x − 5x 2
2× 5
Ascent Descent
3. Maximum speed with which the boy can throw stone is
u2 sin 2 θ 2usinθ u = 2gh = 2 ×10 ×10 =10 2 m / s
6. H = and T =
2g g
Range is maximum when projectile is thrown at an angle of
∴ usinθ =
gT 45 °. Thus,
2 u2 (10 2 )2
Rmax = = = 20 m
( g 2T 2 /4) gT 2 g 10
∴ H= =
2g 8 4. Angular momentum of the projectile
9. t1 = t2 =
2h
(h = height of tower ) L = mvh r⊥ = m(v cosθ)h
g (where, h is the maximum height)
v1 = 2gh  v 2 sin 2 θ 
⇒ = m (v cosθ)  
While v 2 = v12 + v 02 (v 0 = initial horizontal velocity)  2g 
2
u sin 30° u
2 2 2 mv 3 sin 2 θcosθ 3 mv 3
10. h1 =
u
, h2 = = L= =
2g 2g 8g 2g 16g
Projectile Motion 157

5. Given, condition h1 = h2 Given, y = 8t − 5t 2 …(v)


Height is given as On comparing both Eqs. (iv) and (v) , we get
u2 sin 2 θ ⇒ u sin θ =8 …(vi)
H=
2g From Eqs. (iii) and (vi), we get u =10 ms–1
u12 sin 2 45 ° = u22 sin 2 θ 8. Velocity is given as v = ky i$ + kx $j
u12 1 1 1 dx dy
sin 2 θ = sin 2 45 ° = ⋅ = = ky , = kx
u22 2 2 4 dt dt
1 dy dy dt kx
sinθ = ⇒ θ = 30 ° = × =
2 dx dt dx ky
L
6. tanθ = y dy = x dx
H
Aircraft y 2 = x 2 + constant
9. Angular momentum of a particle is given as L = m( r × v)
°

L = m v 0 cos θt $i + (v 0 sin θ t − gt 2 ) $j


30

1
H
 2 
× [v 0 cosθ $i + (v 0 sin θ − gt ) $j]
P
= mv 0 cosθt  − gt  k$ = − mgv 0t 2 cosθ k$
L 1 1
ground  2  2
10v 10. The forces acting on the ball will be (i) in the direction
tan 30 ° =
3400 opposite to its motion, i.e. frictional force and (ii) weight
340 mg.
⇒ v=
3
=196.3 m/s
7. Horizontal distance covered is
x = ucosθ(t ) …(i) f
Given, x =6t …(ii)
On comparing both Eqs. (i) and (ii), we get
⇒ ucosθ =6 …(iii) mg
1
Vertical distance covered is y = usin θ(t ) − gt 2 …(iv)
2 Hence, where f = frictional force
5
Laws of Motion

5.1 Types of Forces


There are basically three forces which are commonly encountered in mechanics. Chapter Snapshot
● Types of Forces
Field Forces ● Free Body Diagram
These are the forces in which contact between two objects is not necessary. ● Equilibrium
Gravitational force between two bodies and electrostatic force between two charges are
Newton’s Laws of Motion
two examples of field forces. Weight ( w = mg ) of a body comes in this category.

● Pulleys
Contact Forces ● Constraint Equations
Two bodies in contact exert equal and opposite forces on each other. If the contact is ● Pseudo Force
frictionless, then contact force is perpendicular to the common surface and known as ● Friction
normal reaction.
If, however the objects are in rough contact and move (or have a tendency to move)
relative to each other without losing contact, then frictional force arises which opposes
such motion. Again each object exerts a frictional force on the other and the two forces are
equal and opposite. This force is perpendicular to normal reaction. Thus, the contact force
( F ) between two objects is made up of two forces.
(i) Normal reaction ( N ) (ii) Force of friction ( f )
and since these two forces are mutually perpendicular.
F = N2 + f 2

Consider two wooden blocks A and B being rubbed against each other.
In Fig. 5.1, A is being moved to the right while B is being moved leftward. In order to
see more clearly which forces act on A and which on B, a second diagram is drawn showing
a space between the blocks but they are still supposed to be in contact.

A F1

F2 B

Fig. 5.1
Laws of Motion 159

In Fig. 5.2 the two normal A


reactions each of magnitude N are N F1
5.2 Free Body Diagram
perpendicular to the surface of f No system, natural or man made, consists of a single
contact between the blocks and the f body alone or is complete in itself. A single body or a part of
two frictional forces each of F2
N the system can, however, be isolated from the rest by
magnitude f act along that surface, B appropriately accounting for its effect on the remaining
each in a direction opposing the Fig. 5.2 system.
motion of the block upon which N
it acts.

Attachment to Another Body


Tension (T ) in a string and spring force ( F = kx ) come
Mass of book = m
in this group. Regarding the tension and string, the w = mg
following three points are important to remember. Fig. 5.5
1. If a string is inextensible, the magnitude of
acceleration of any number of masses connected A free body diagram (FBD) consists of a diagrammatic
through string is always same. representation of a single body or a sub-system of bodies
isolated from its surroundings showing all the forces acting
2. If a string is massless, the tension in it is same
everywhere. However, if a string has a mass, tension at on it.
different points will be different. Consider, for example, a book lying on a horizontal
surface.
a A free body diagram of the book alone would consist of
a
its weight ( w = mg ), acting through the centre of gravity and
a
M m F the reaction ( N ) exerted on the book by the surface.
m a
X Example 5.1 A cylinder of
M
weight w is resting on a V-groove
Fig. 5.3 as shown in figure. Draw its free
3. If there is a friction between string and pulley, tension body diagram.
Fig. 5.6(a)
is different on two sides of the pulley, but if there is no
friction between pulley and string, tension will be
same on both sides of the pulley. Sol. The free body diagram of the cylinder is as shown in
Fig. 5.6(b)
Last two points can be understood in diagram as follows:
T T T1 T1
m m
T T2
T T2 N1
w N2
M M
Fig. 5.6(b)
String is massless and String is massless and
there is no friction between there is a friction between Here, w = weight of cylinder and N1 and N2 are the normal
pulley and string string and pulley reactions between the cylinder and the two inclined walls.
T1 T2
m X Example 5.2 Three blocks A, B and C are placed
T3 one over the other as shown in figure. Draw free body
T4 diagrams of all the three blocks.
M A

String is not massless and there is B


a friction between pulley and string
C
Fig. 5.4
Spring force ( F = kx ) has been discussed in detail in the
chapter of work, energy and power. Fig. 5.7
160 Objective Physics Vol. 1

Sol. Free body diagrams of A, B and C are shown below In Fig. 5.11,
N1 N2
F1 = F cos θ = component of F along AC
F2 = F sin θ = component of F perpendicular to
wA wB wC AC or along AB
F B
N1 N2 N3 F2
FBD of A FBD of B FBD of C
Fig. 5.8
θ
F1 A C
Here, N1 = normal reaction between A and B.
N2 = normal reaction between B and C. Fig. 5.11
N3 = normal reaction between C and ground. Finding such components is referred to as resolving a
force in a pair of perpendicular directions. Note that the
X Example 5.3 A block of mass m is attached with component of a force in a direction perpendicular to itself is
two strings as shown in figure. Draw the free body zero. For example, if a force of 10 N is applied on an object
diagram of the block. in horizontal direction, then its component along vertical is
zero. Similarly, the component of a force in a direction
parallel to the force is equal to the magnitude of the force.
θ For example, component of the above force in the direction
of force (horizontal) will be 10 N.
X Example 5.4 Resolve a weight of 10 N in two
Fig. 5.9
directions which are parallel and perpendicular to a
Sol. The free body diagram of the block is as shown in slope inclined at 30° to the horizontal.
Fig. 5.10. Sol. Component perpendicular to the plane
T1

w| |

T2 θ 30° w⊥

30° w = 10
mg Fig. 5.12
Fig. 5.10 w⊥ = w cos 30°
3
= (10) = 5 3N
2
5.3 Equilibrium and component parallel to the plane
w|| = w sin 30°
Forces which have zero linear resultant and zero 1
turning effect will not cause any change in the motion of the = (10)   = 5 N
2
object to which they are applied. Such forces (and the
object) are said to be in equilibrium. For understanding the X Example 5.5 Resolve horizontally and vertically
equilibrium of an object under two or more concurrent or a force F = 8 N , which makes an angle of 45° with the
coplanar forces let us first discuss the resolution of force and horizontal.
moment of a force about some point.
Sol. Horizontal component of F is FV

Resolution of a Force FH = F cos 45°


1 
When a force is replaced by an equivalent set of = (8)  
 2 F
components, it is said to be resolved. One of the most useful
=4 2N
ways in which to resolve a force is to choose only two and vertical component of F is 45°
components (although a force may be resolved in three or FV = F sin 45° FH
more components also) which are at right angles also. The 1 
= (8) 
Fig. 5.13

magnitude of these components can be very easily found  2
using trigonometry. =4 2N
Laws of Motion 161

Components
X Example 5.6 A body is supported on a rough Force
plane inclined at 30° to the horizontal by a string Parallel to OX (horizontal) Parallel to OY (vertical)

attached to the body and held at an angle of 30° to f − f cos 30° − f sin 30°
the plane. Draw a diagram showing the forces acting N − N cos 60° N sin 60°
on the body and resolve each of these forces T T cos 60° T sin 60°
(a) horizontally and vertically, w 0 −w
(b) parallel and perpendicular to the plane. Resolving parallel and perpendicular to the plane in the
Sol. The forces are : N T senses OX ′ and OY ′ as shown, the components are
X'
The tension in the string T and Y'
30°
the normal reaction with the plane
N, f
The weight of the body w
w O
and the friction f.
30° Fig. 5.18
(a) Resolving horizontally and
vertically Fig. 5.14
Components
f cos 30° N Force Parallel to OX ′ Parallel to OY ′
N sin 60°
30° (parallel to plane) (perpendicular to plane)
f −f 0
N 0 N
60°
f f sin 30° T T cos 30° T sin 30°
N cos 60°
T sin 60° w − w sin 30° − w cos 30°
T

w Moment of a Force
60° The general name given to any turning effect is torque.
T cos 60°
The magnitude of torque, also known as the moment of a
Fig. 5.15
force F is calculated by multiplying together the magnitude
of the force and its perpendicular distance r⊥ from the axis
(b) Resolving parallel and perpendicular to the plane of rotation. This is denoted by C or τ (tau).
T i.e. C = Fr⊥
or τ = Fr⊥
N T sin 30° T cos 30°
f 30° Direction of Torque
The angular direction of a torque is the sense of the
rotation it would cause.
w sin 30° Consider a lamina that is
free to rotate in its own plane
30° about an axis perpendicular to F2 F1
w cos 30°
the lamina and passing through r2 r1
a point A on the lamina. In the A
w
Fig. 5.16 diagram the moment about the
axis of rotation of the force F1
Resolving horizontally and vertically in the senses OX and is F1 r1 anti-clockwise and the
OY as shown, the components are Fig. 5.19
moment of the force F2 is F2 r2
Y clockwise. A convenient way to differentiate between
clockwise and anti-clockwise torques is to allocate a
positive sign to one sense (usually, but not invariably, this is
anti-clockwise) and negative sign to the other.
With this convention, the moments of F1 and F2 are
X
O + F1 r1 and −F2 r2 (when using a sign convention in any
Fig. 5.17 problem it is advisable to specify the chosen positive sense).
162 Objective Physics Vol. 1

Magnitude of force 2F F 3F
Zero Moment
Perpendicular distance a 2a 4a sin 30° = 2 a
If the line of action of a force passes through the axis of from A
rotation, its perpendicular distance from the axis is zero. Anti-clockwise moment +2Fa –2Fa +6Fa
Therefore, its moment about that axis is also zero. about A
/ Later in the chapter of rotation, we will see that torque is a
vector quantity. Coplanar Forces in Equilibrium
X Example 5.7 ABCD is a square of side 2m and O When an object is in equilibrium under the action of a set
is its centre. Forces act along the sides as shown in the of two or more coplanar forces, each of three factors which
diagram. Calculate the moment of each force about comprise the possible movement of the object must be zero,
4N i.e. the object has
D C
(i) no linear movement along any two mutually
perpendicular directions OX and OY.
3N O 5N (ii) no rotation about any axis.
The set of forces must, therefore be, such that
A B (a) the algebraic sum of the components parallel to
2N
OX is zero or ΣFx = 0
Fig. 5.20
(a) an axis through A and perpendicular to the plane of (b) the algebraic sum of the components parallel to OY
square. is zero or ΣF y = 0
(b) an axis through O and perpendicular to the plane of (c) the resultant moment about any specified axis is
square. zero or Στ any axis = 0
Sol. Taking anti-clockwise moments as positive we have Thus, for the equilibrium of a set of two or more
Magnitude of coplanar forces
(a) 2N 5N 4N 3N
force ΣFx = 0
Perpendicular 0 2m 2m 0 ΣF y = 0
distance from A
Moment about A 0 −10 N-m +8 N-m 0
and Στ any axis = 0
Using the above three conditions, we get only
Magnitude of three sets of equations. So, in a problem number of
(b) 2N 5N 4N 3N
force
unknown should not be more than three.
Perpendicular 1m 1m 1m 1m
distance from O X Example 5.9 A rod AB Y
Moment about O +2 N-m –5 N-m +4 N-m –3 N-m rests with the end A on rough B
horizontal ground and the end B
X Example 5.8 Forces act as indicated on a rod against a smooth vertical wall.
AB which is pivoted at A. Find the anti-clockwise The rod is uniform and of
moment of each force about the pivot. weight w. If the rod is in
3F 30° A
equilibrium in the position O X
2F shown in figure. Find
Fig. 5.23
30° (a) frictional force at A,
A B (b) normal reaction at A,
F
a a 2a (c) normal reaction at B.
Fig. 5.21 Sol. Let length of the rod be 2l. Y
Using the three conditions of B NB
Sol. equilibrium. Anti-clockwise
moment is taken as positive.
°
30

(i) ΣFx = 0
sin

3F NA
2F ∴ NB − fA = 0
4a

w
30° or NB = fA …(i) 30° A
(ii) ΣFy = 0 X
A B O fA
F ∴ NA − w = 0
Fig. 5.24
Fig. 5.22 or NA = w …(ii)
Laws of Motion 163

(iii) Στ 0 = 0 The object is in equilibrium. Hence,


∴ NA (2 l cos 30° ) − NB (2 l sin 30° ) − w(l cos 30° ) = 0 (i) ΣFx = 0
3 ∴ 8 + 4 cos 60° − F2 cos 30° = 0
or 3NA − NB − w=0 …(iii)
2 3 20
or 8 + 2 − F2 = 0 or F2 = N
Solving these three equations, we get 2 3
3 (ii) ΣFy = 0 ∴ F1 + 4 sin 60° − F2 sin 30° = 0
(a) fA = w
2 4 3 F2
or F1 + − =0
(b) NA = w 2 2
3 F
F1 = 2 − 2 3
(c) NB = w or
2 2
10
= −2 3
Equilibrium of Concurrent Coplanar 3
4
or F1 = N
Forces 3
If an object is in equilibrium under two or more
concurrent coplanar forces, the algebraic sum of the Lami’s Theorem
components of forces in any two mutually perpendicular If an object O is in equilibrium under three concurrent
directions OX and OY should be zero, i.e. the set of forces forces F1 , F2 and F3 as shown in figure. Then,
must be such that F1 F F
= 2 = 3
(a) the algebraic sum of the components parallel to OX is sin α sin β sin γ
zero, i.e. ΣFx = 0.
(b) the algebraic sum of the components parallel to OY is This property of three concurrent forces in equilibrium
zero, i.e. ΣF y = 0. is known as Lami's theorem and is very useful method of
Thus, for the equilibrium of two or more concurrent solving problems related to three concurrent forces in
coplanar forces equilibrium.
ΣFx = 0 F2

ΣF y = 0
The third condition of zero moment about any
specified axis is automatically satisfied, if the moment
is taken about the point of intersection of the forces. α γ
So, here we get only two equations. Thus, number of F1
O
unknowns in any problem should not be more than F3 β
two. Fig. 5.27

X Example 5.10 An object is in equilibrium under X Example 5.11 One end of a string 0.5 m long is
four concurrent forces in the directions shown in fixed to a point A and the other end is fastened to a
figure. Find the magnitude of F1 and F2 . small object of weight 8 N . The object is pulled aside
F1 by a horizontal force F, until it is 0.3 m from the
4N
vertical through A. Find the magnitudes of the tension
T in the string and the force F.
30°
A
60°
8N
30° O T
F2
B F
Fig. 5.25 C

Sol. F1 Y
4N
8N
Fig. 5.28
30° X
O
60° Sol. AC = 0.5 m, BC = 0.3 m
8N ∴ AB = 0.4 m
30° O
F2 and if ∠ BAC = θ
AB 0.4 4
Then, cos θ = = =
Fig. 5.26 AC 0.5 5
164 Objective Physics Vol. 1

BC 0.3 3
and sin θ = = =
AC 0.5 5 5.4 Newton’s Laws of Motion
A
It is interesting to read Newton’s original version of the
θ T laws of motion.
θ Law I Every body continues in its state of rest or in
B
C
F uniform motion in a straight line unless it is compelled to
change that state by forces impressed upon it.
Law II The change of motion is proportional to the
8N magnitude of force impressed and is made in the direction of
Fig. 5.29 the straight line in which that force is impressed.
Law III To every action there is always an equal and
Here, the object is in equilibrium under three concurrent
forces. So, we can apply Lami’s theorem. opposite reaction or the mutual actions of two bodies upon
F 8 T each other are always directed to contrary parts.
or = =
sin (180° − θ) sin (90° + θ) sin 90° The modern versions of these laws are :
F 8
or = =T 1. A body continues in its initial state of rest or motion
sin θ cos θ
with uniform velocity unless acted on by an
8
∴ T= unbalanced external force.
cos θ
8 2. The acceleration of a body is inversely proportional to
= = 10 N its mass and directly proportional to the resultant
4/5
8 sin θ (8) (3/5) external force acting on it, i.e.
and F= = = 6N F
cos θ (4/5) ΣF = F net = ma or a = net
m
X Example 5.12 A block of mass m is at rest on a 3. Forces always occur in pairs. If body A exerts a force
rough wedge as shown in figure. What is the force on body B, an equal but opposite force is exerted by
exerted by the wedge on the block? body B on body A.

Working with Newton’s First and Second Laws


m Normally, any problem relating to Newton’s laws is
solved in following four steps:
θ 1. First of all we decide the system on which the laws of
motion are to be applied. The system may be a single
Fig. 5.30 particle, a block or a combination of two or more
blocks, two blocks connected by a string, etc. The only
Sol. Since, the block is permanently at rest, it is in equilibrium. Net restriction is that all parts of the system should have
force on it should be zero. In this case only two forces are acting the same acceleration.
on the block.
2. Once the system is decided, we make the list of all the
(1) Weight = mg (downwards)
(2) Contact force (resultant of normal reaction and friction
forces acting on the system. Any force applied by the
force) applied by the wedge on the block. system on other bodies is not included in the list of the
For the block to be in equilibrium these two forces forces.
should be equal and opposite. 3. Then, we make a free body diagram of the system and
Therefore, force exerted by the wedge on the block is indicate the magnitude and directions of all the forces
mg (upwards).
listed in step 2 in this diagram.
Note Points 4. In the last step, we choose any two mutually
perpendicular axes say x and y in the plane of the
/ From Newton’s third law of motion-force exerted by the block forces in case of coplanar forces. Choose the x-axis
on the wedge is also mg but downwards.
along the direction in which the system is known to
/ The result can also be obtained in a different manner. The have or is likely to have the acceleration. A direction
normal force on the block is N = mg cos θ and the friction
force on the block is f = mg sin θ (not µ mg cos θ)
perpendicular to it may be chosen as the y-axis. If the
These two forces are mutually perpendicular. system is in equilibrium, any mutually perpendicular
∴ Net contact force would be N 2 + f 2
directions may be chosen. Write the components of all
the forces along the x-axis and equate their sum to the
or (mg cos θ)2 + (mg sin θ)2 which is equal to mg. product of the mass of the system and its acceleration,
Laws of Motion 165

i.e. ΣFx = ma …(i)


This gives us one equation. Now, we write the Extra Knowledge Points
components of the forces along the y-axis and equate ■ If a pulley is massless, net force on it is zero even if it is
the sum to zero. This gives us another equation, i.e. accelerated. For example in the following figure
ΣF y = 0 …(ii) T1

Note Points
T1
/ If the system is in equilibrium, we will write the two equations
as T1
ΣFx = 0 and ΣFy = 0
T2 T2
/ If the forces are collinear, the second equation, i.e. ΣFy = 0 is P
not needed. FBD of pulley P
T2
T2
X Example 5.13 Two blocks of mass 4 kg and 2 kg
are placed side by side on a smooth horizontal surface
as shown in the figure. A horizontal force of 20 N is
applied on 4 kg block. Find
20 N T1 = 2T2 whether the pulley is accelerated on not
4 kg 2 kg
provided the pulley is massless. This is because wF
mass x acceleration and wF will be zero, if pulley is
Fig. 5.31 massless.
(a) the acceleration of each block, I have found students often confused over the
(b) the normal reaction between two blocks. resolution of forces. If the body is in equilibrium, no
matter in which direction you resolve the forces. Net
Sol. (a) Since, both the blocks will move with same acceleration force should be zero in any direction. If the body is
(say a) in horizontal direction. accelerated, resolve in the direction of acceleration
Let us take both the blocks as a system. Net external and its perpendicular. In perpendicular direction, net
force on the system is 20 N in horizontal direction. force is zero and in the direction of acceleration it is
a y mwF .
20 N
4 kg 2 kg
x X Example 5.14 Three blocks of mass 3 kg, 2 kg
Fig. 5.32 and 1 kg are placed side by side on a smooth surface
Using ΣFx = ma x , as shown in figure. A horizontal force of 12 N is
10 applied on 3 kg block. Find the net force on 2 kg
20 = (4 + 2 ) a = 6 a or m/s 2 a=
3 block.
(b) The free body diagram of both the blocks are as shown 12N
3kg 2kg 1kg
in Fig. 5.33.
y
20 N Fig. 5.34
4 kg 2 kg
N Sol. Since, all the blocks will move with same acceleration (say a)
x in horizontal direction. Let us take all the blocks as a system.
a a
Fig. 5.33 Net external force on the system is 12 N in horizontal
direction.
Using ΣFx = ma x , y
10
For 4 kg block, 20 − N = 4a = 4 × 12N
3 3kg 2kg 1kg
x
40 20
N = 20 − = N
3 3
a
This can also be solved as under.
10 20 Fig. 5.35
For 2 kg block, N = 2 a = 2 × = N
3 3 Using ΣFx = ma x , we get
12
Here, N is the normal reaction between the two blocks. 12 = (3 + 2 + 1) a = 6 a or a= = 2 m/s 2
/ In free body diagram of the blocks, we have not shown the 6
forces acting on the blocks in vertical direction, because Now, let F be the net force on 2 kg block in x-direction,
normal reaction between the blocks and acceleration of the then using ΣFx = ma x for 2 kg block, we get
system can be obtained without using ΣFy = 0. F = (2 )(2 ) = 4 N
166 Objective Physics Vol. 1

/ Here, net force F on 2 kg block is the resultant of N1 and F = 120 N


N 2 (N1 > N 2 )
where, N1 = normal reaction between 3 kg and 2 kg block
and N 2 = normal reaction between 2 kg and 1 kg block 4 kg
Thus, F = N1 − N 2

X Example 5.15 In the arrangement shown in


2 kg
figure. The strings are light and inextensible. The
surface over which blocks are placed is smooth. Find Fig. 5.40

4kg 2kg
F = 14N Sol. (a) Let a be the acceleration of the blocks and T the tension in
1kg
the string as shown in figure.
Fig. 5.36 F = 120N
(a) the acceleration of each block, y
(b) the tension in each string. 4 kg
Sol. (a) Let a be the acceleration of each block and T1 and T2 be
T x
the tensions, in the two strings as shown in figure.
y
2 kg
T2 T1 F = 14 N
4 kg 2 kg 1kg
x Fig. 5.41
Fig. 5.37 Taking the two blocks and the string as the system.
Using ΣFy = may , we get
Taking the three blocks and the two strings as the
system. F – 4g − 2 g = (4 + 2 ) a or 120 − 40 − 20 = 6 a
a or 60 = 6 a ⇒ ∴ a = 10 m/s 2
(b) Free body diagram of 2 kg block is as shown in Fig. 5.42.
F = 120N
4 kg F = 14 N
2 kg 1kg
T
a
Fig. 5.38 4g

Using ΣFx = ma x 2 kg a
or 14 = (4 + 2 + 1) a
14
or a= = 2 m/s 2
7 2g
(b) Free body diagram (showing the forces in x-direction only) of 2g
4 kg block and 1 kg block are shown in figure.
y Fig. 5.42
a = 2 m/s2 a = 2 m/s2
Using ΣFy = may , we get
T2 T1 F = 14 N T − 2 g = 2 a or T − 20 = (2 )(10)
4 kg 1kg x ∴ T = 40 N
/ If the string is having some mass, tension in it is different at
Fig. 5.39 different points. Under such condition tension on the string at
some point is calculated as under:
Using ΣFx = ma x , F = 100 N
For 1 kg block, F − T1 = (1)(a)
or 14 − T1 = (1)(2 ) = (2 ) a
∴ T1 = 14 − 2 = 12 N 2 kg
For 4 kg block, T2 = (4)(a) A
∴ T2 = (4)(2 ) = 8 N C
B
X Example 5.16 Two blocks of mass 4 kg and 2 kg 4 kg
are attached by an inextensible light string as shown
Fig. 5.43
in figure. Both the blocks are pulled vertically
upwards by a force F =120 N . Find In the above figure, the length of the string connecting the two
blocks is 2 m and mass is 2 kg. Tension at A, B and C (centre
(a) the acceleration of the blocks,
point) can be calculated by considering the motion of system
(b) the tension in the string. (Take g =10 m/ s 2 ) below A, B and C. For example
Laws of Motion 167

F − weight of 2 kg – weight of 4 kg – weight of string Net pulling force


a=
mass of 2 kg + mass of 4 kg + mass of string a=
Total mass to be pulled
100 − 20 − 40 − 20
= (g = 10 m/ s2 ) mg g
2+ 4+ 2 = =
20 3m 3
= = 2.5 m/ s2
8 Let us now, take few examples based on pulling force
Refer Fig. 5.44 (a) method.
A TA
C TC X Example 5.17 In the below figure, mass of A, B
B TB and C are 1 kg, 3 kg and 2 kg, respectively. Find
B a B a a
4 kg 4 kg 4 kg

B
(a) (b) (c) C
Fig. 5.44

A
60°
TA − mAB g − 40 = (mAB + 4)a 30°
or TA − 20 − 40 = (2 + 4)(2.5) Fig. 5.46
or TA = 75 N
(a) the acceleration of the system,
Refer Fig. 5.44 (b)
(b) tensions in the string.
TC − mBC g − 40 = (mBC + 4)a
Neglect friction. ( g =10 m / s 2 )
or TC − 10 − 40 = (1 + 4)(2.5)
or TC = 62.5 N Sol. (a) In this case, net pulling force
Refer Fig. 5.44 (c) TB − 40 = 4a = mA g sin 60° + mB g sin 60° − mC g sin 30°
or TB = 40 + 4 × 2.5 3  3  1
= (1)(10) + (3)(10)   − (2)(10)  
or TB = 50 N 2  2  2
= 21.17 N
Total mass being pulled = 1 + 3 + 2 = 6 kg
5.5 Pulleys ∴ Acceleration of the system, a =
21.17
= 3.53 m / s 2
6
As an author, I personally feel that problems based on
(b) For the tension in the string between A T1
pulleys become very simple using pulling force method. and B,
Now, let us see what is this pulling force method with the FBD of A
a
help of an example. A
mA g sin 60° − T1 = (mA )(a)
Suppose, two unequal masses m and 2m are attached to ∴ T1 = mA g sin 60° − mA a
the ends of a light inextensible string which passes over a = mA (g sin 60° − a)
smooth massless pulley. We have to find the acceleration of  3  mA g sin 60°
∴ T1 = (1)  10 × − 4.1
the system. We can assume that the mass 2m is pulled  2  Fig. 5.47
downwards by a force equal to its weight, i.e. 2 mg. = 5.13 N
Similarly, the mass m is being pulled by a force of mg For the tension in the string between B and C,
downwards. Therefore, net pulling force on the system is FBD of C
2 mg − mg = mg and total mass being pulled is 2 m + m = 3m. T2 − mC g sin 30° = mC a
∴ T2 = mC (a + g sin 30° )
 1 
∴ T2 = 2 3.53 + 10  
  2  
= 17.01 N
a a
5.6 Constraint Equations
m 2m
These equations basically establish the relation between
accelerations (or velocities) of different masses attached by
mg 2mg string(s). Usually, it is observed that the number of constraint
Fig. 5.45
equations are as many as the number of strings in the system
under consideration. From the following few examples, we
∴ Acceleration of the system is can better understand the method.
168 Objective Physics Vol. 1

On double, differentiating with respect to time, we get


X Example 5.18 Using constraint method find the a1 + a4 = 0 …(iii)
relation between accelerations of 1 and 2. and a2 + a3 − 2 a4 = 0 …(iv)
But since, a4 = − a1 [From Eq. (iii)]
We have a2 + a3 + 2 a1 = 0
This is the required constraint relation between a1, a2 and
x1 a3 .
x2
1
1
X Example 5.20 Using constraint equations find
2
the relation between a1 and a 2 .
2
Fig. 5.49 Fig.

Sol. At any instant of time, let x1 and x2 be the displacements of


1 and 2 from a fixed line (shown dotted).
Then, x1 + x2 = constant.
or x1 + x2 = l (length of string)
T2
a
C
2
mC g sin 30°
Fig. 5.48
Fig. 5.53
Differentiating with respect to time, we have
Sol. Points 1, 2, 3 and 4 are movable. Let their displacements
v1 + v 2 = 0 from a fixed line be x1, x2 , x3 and x4 .
or v1 = − v 2
x1 + x3 = l1
Again differentiating with respect to time, we get
( x1 − x3 ) + ( x4 − x3 ) = l2
a1 + a2 = 0 or a1 = − a2
( x1 − x4 ) + ( x2 − x4 ) = l3
This is the required relation between a1 and a2 , i.e.
accelerations of 1 and 2 are equal but in opposite On double differentiating with respect to time, we will get
directions. following three constraint relations,
a1 + a3 = 0 …(i)
X Example 5.19 Find the constraint relation
between a1 , a 2 and a 3 .
x3
3 x4
x2
x1 x1
x4 4
1
1
x2
4 x3 2

Fig. 5.54
2
2 3 3 a1 + a4 − 2 a3 = 0 …(ii)
a1 + a2 − 2 a4 = 0 …(iii)
Fig. 5.51 Fig. 5.52
Solving Eqs. (i), (ii) and (iii), we get
Sol. Points 1, 2, 3 and 4 are movable. Let their displacements a2 = − 7 a1
from a fixed line be x1, x2 , x3 and x4 . which is the desired relation between a1 and a2 .
We have
x1 + x4 = l1 (length of first string)…(i)
X Example 5.21 At certain moment of time,
and ( x2 − x4 ) + ( x3 − x4 ) = l2 (length of second string) velocities of 1 and 2 both are 1 m/s upwards. Find the
or x2 + x3 − 2 x4 = l2 …(ii) velocity of 3 at that moment.
Laws of Motion 169

Sol. In example 5.19, we have found ( −ma 0 ) while using F = ma (second law) in non-inertial
a2 + a3 + 2 a1 = 0 frame. Suppose a block A of mass m is placed on a lift
Similarly, we can find ascending with an acceleration a 0 . Let N be the normal
v 2 + v 3 + 2 v1 = 0 1 reaction between the block and the floor of the lift. Free
Taking, upward direction as positive we are body diagram of A in ground frame of reference (inertial) is
given
v1 = v 2 = 1 m/s
shown in Fig. 5.57.
N′
∴ v 3 = − 3 m/s ∴ N − mg = ma 0
2
i.e. velocity of block 3 is 3 m/s (downwards). 3 or N = m ( g + a 0 ) …(i)
Fig. 5.55 But if we draw the free body diagram of A A (At rest)
5.7 Pseudo Force with respect to the elevator (a non-inertial
Before studying pseudo force let us first discuss frame frame of reference) without applying the
of reference. A system of coordinate axes which defines the pseudo force, as shown in Fig. 5.58, we get mg
position of a particle or an event in two or three dimensional N ′ − mg = 0 Fig. 5.58
space is called a frame of reference. The simplest frame of or N ′ = mg …(ii)
reference is, of course, the familiar cartesian system of Since, N ′ ≠ N , either of the equations N′
coordinates, in which the position of the particle is specified is wrong. But if we apply a pseudo force in
by its three coordinates x, y and z. Frame of references are of non-inertial frame of reference, N ′ becomes
two types A (At rest)
equal to N as shown in Fig. 5.59.
Inertial Frame of Reference Acceleration of block with respect to
elevator is zero
A non-accelerating frame of reference is called an mg + Fp
∴ N ′ − mg − ma 0 = 0
inertial frame of reference. A frame of reference moving Here FP = ma0
with a constant velocity is an inertial frame of reference. or N ′ = m (g + a 0 ) …(iii)
Fig. 5.59
∴ N′= N
Non-inertial Frame of Reference Pseudo force is given by FP = −ma 0 . Here, a 0 is the
An accelerating frame of reference is called a acceleration of the non-inertial frame of reference and m the
non-inertial frame of reference. mass of the body under consideration. In the whole chapter,
/ A rotating frame of reference is a non-inertial frame of we will show the pseudo force by FP .
reference, because it is also an accelerating one.
Thus, we may conclude that pseudo force is not a real
Now, let us come to the pseudo force. When we draw the free body diagram of a mass with
force. Newton’s first two laws hold respect to an inertial frame of reference, we apply only the
good in an inertial frame only. a0
real forces (forces which are actually acting on the mass),
However, we people spend most of our but when the free body diagram is drawn from a non-inertial
A
time on the earth which is an frame of reference a pseudo force (in addition to all real
(approximate) inertial frame. We are Fig. 5.56 forces) has to be applied to make the equation F = ma, valid
so familiar with the Newton’s laws that
in this frame also.
we will still like to use ‘total force equals mass times
/ In case of rotating frame of reference, this pseudo force is
acceleration’ even when we use a non-inertial frame. called the centrifugal force when applied for centripetal
This can be done, if we agree to call ( −ma 0 ) N acceleration. Let us take few examples of pseudo forces.
a force acting on the particle. Then, while
preparing the list of the forces acting on a X Example 5.22 In the below figure, the coefficient
particle P, we include all the (real) forces acting A a0
of friction between wedge (of mass M) and block (of
on P by all other objects and also include an mass m) is µ.
imaginary force ( −ma 0 ). Here, a 0 is the Find the minimum horizontal force F required to keep
acceleration of the non-inertial frame under mg the block stationary with respect to wedge.
consideration.
Fig. 5.57
After applying this additional imaginary
force (called pseudo force) ( −ma 0 ), we can now use ‘total M m
force equals mass time acceleration’ even in non-inertial F
frames also. Now, with the help of a simple example let us
see what problem arises, if we don’t apply the pseudo force Fig. 5.60
170 Objective Physics Vol. 1

Sol. Such problems can be solved with or without using the concept
of pseudo force. Let us solve the problem by both the methods.
X Example 5.23 All surfaces are smooth in
a = acceleration of (wedge + block) in horizontal direction following figure. Find F such that block remains
F stationary with respect to wedge.
= .
M+ m

Inertial Frame of Reference (Ground) m


F
FBD of block with respect to ground (only real forces M
have to be applied) with respect to ground block is moving θ
with an acceleration a. Therefore, Fig. 5.63
F = µN
y Sol. Acceleration of (block + wedge)
F
a=
N x (M + m)
Let us solve the problem by both the methods.

mg
From Inertial Frame of Reference
a (Ground)
Fig. 5.61
FBD of block w.r.t. N cos θ
ΣF y = 0 and ΣFx = ma ground (apply real forces) y
mg = µN and N = ma with respect to ground, block
g N sin θ
∴ a= is moving with an acceleration x
µ a
g
∴ F = ( M + m) a = ( M + m) ∴ ΣF y = 0
µ mg
⇒ N cos θ = mg …(i)
a
and ΣFx = ma
Non-inertial Frame of Reference (Wedge) Fig. 5.64
⇒ N sin θ = ma …(ii)
FBD of m with respect to wedge (real + one pseudo
From Eqs. (i) and (ii), we get
force) with respect to wedge block is stationary.
a = g tan θ
F = µN
∴ F = ( M + m) a
= ( M + m) g tan θ
FP = ma N From Non-inertial Frame of Reference
(Wedge)
FBD of block w.r.t. wedge (real forces + pseudo force)
mg
N cos θ
Fig. 5.62

∴ ΣF x = 0 = ΣF y
∴ mg = µN FP = ma N sin θ

and N = ma
g
∴ a= mg
µ
Fig. 5.65
and F = ( M + m) a
w.r.t. wedge, block is stationary
g
= ( M + m) ∴ ΣF y = 0 ⇒ N cos θ = mg …(iii)
µ
ΣFx = 0 ⇒ N sin θ = ma …(iv)
From the above discussion, we can see that from both From Eqs. (iii) and (iv), we will get the same result
the methods results are same. i.e. F = ( M + m) g tan θ.
Laws of Motion 171

X Example 5.24 A bob of mass m is suspended ∴ ΣF y = 0


from the ceiling of a train moving with an ⇒ T cos θ = mg …(iv)
acceleration a as shown in figure. Find the angle θ in From Eqs. (iii) and (iv), we get the same result, i.e.
equilibrium position. a
θ = tan −1  
g
θ
a
X Example 5.25 In the below figure, a wedge is
fixed to an elevator moving upwards with an
acceleration a. A block of mass m is placed over the
wedge. Find the acceleration of the block with respect
Fig. 5.66 to wedge. Neglect friction.

Sol. This problem can also be solved by both the methods. a

Inertial Frame of Reference (Ground)


m
FBD of bob w.r.t. ground (only real forces)
T cos θ

T θ
θ a
Fig. 5.69
T sin θ
Sol. Since, acceleration of block w.r.t. wedge (an accelerating or
y
non-inertial frame of reference) is to be find out.
x
FBD of block w.r.t. wedge is shown in Fig. 5.70.
mg mg
N
Fig. 5.67

with respect to ground, bob is also moving with an


acceleration a
∴ ΣFx = 0
⇒ T sin θ = ma …(i) θ
and ΣF y = 0
⇒ T cos θ = mg …(ii) mg + FP = mg + ma
From Eqs. (i) and (ii), we get
a
tan θ =
g θ
) sin
a +a
or θ = tan −1   t=
(g
g a ne
θ

Non-inertial Frame of Reference (Train) Fig. 5.70

FBD of bob w.r.t. train (real forces + pseudo force) The acceleration would had been g sin θ (down the plane),
T cos θ if the lift were stationary or when only weight (i.e. mg) acts
downwards.
T
θ Here, downward force is m(g + a)
So, acceleration of the block (of course w.r.t. wedge) will be
ma (g + a) sin θ down the plane.
FP = ma T sin θ

mg mg 5.8 Friction
Fig. 5.68 As we have discussed in Article 5.1 friction is the
with respect to train, bob is in equilibrium parallel component of contact force between two bodies in
∴ ΣFx = 0 contact. These forces are basically electromagnetic in
⇒ T sin θ = ma …(iii) nature. Friction can operate between a given pair of solids
172 Objective Physics Vol. 1

between a solid and a fluid or between a pair of fluids. For example, N1 = normal reaction between A and B
Frictional force exerted by fluids is called viscous force. = mAg
When two bodies slip over each other the force of friction is N 2 = normal reaction between B and C
called kinetic friction, but when they do not slip but have a = (mA + mB )g and so on.
tendency to do so the force of friction is called static friction. A
Regarding friction it is worth noting that B
1. If a body is at rest and no pulling force is acting on it,
C
force of friction on it is zero.
D
2. If a force is applied to pull the body and it does not
move, the friction acts which is equal in magnitude Fig. 5.71
and opposite in direction to the applied force, i.e.
friction is self adjusting force. Further, as the body is The theory of static and kinetic friction can be better
at rest the friction is called static friction. understood by the following simple example.
3. If the applied force is increased, the force of static
friction also increases. If the applied force exceeds a
X Example 5.26 Suppose a block of mass 1 kg is
placed over a rough surface and a horizontal force F
certain (maximum) value, the body starts moving.
is applied on the block as shown in figure. Now,
This maximum force of static friction up to which
let us see what are the values of force of friction f
body does not move is called limiting friction. Thus,
and acceleration of the block a, if the force F is
static friction is a self adjusting force with an upper
gradually increased. Given, that µ s = 0.5, µ k = 0.4 and
limit called limiting friction.
g =10 m/s 2 .
4. This limiting force of friction ( f L ) is found
experimentally to depend on normal reaction ( N ).
Hence, F

fL ∝ N
or fL =µs N Fig. 5.72
Here, µ s is a dimensionless constant and called
coefficient of static friction, which depends on nature Sol. Free body diagram of block is
of surfaces in contact. ΣFy = 0

5. If the applied force is further increased, the friction ∴ N − mg = 0


opposing the motion is called kinetic or sliding or N = mg = (1)(10) = 10 N
friction. Experimentally, it is well established that fL = µ s N = (0.5)(10) = 5 N
kinetic friction is lesser than limiting friction and is and fk = µ k N = (0.4)(10) = 4 N
given by a
fk =µk N
where, µ k is coefficient of kinetic friction and less mg F
than µ s .
N
f
Note Points
/ In problems, if µ s and µ k are separately not given but only µ y
is given. Then, use
fL = fk = µN
/ If more than two blocks are placed one over the other on a
horizontal ground, then normal reaction between two blocks
x
will be equal to the weight of the blocks over the common
surface. Fig. 5.73
Laws of Motion 173

Below is explained in tabular form, how the force of friction f depends on the applied force F .

Fnet
F f Fnet = F − f Acceleration of block a = Diagram
m

0 0 0 0

F=2N
2N 2N 0 0 f=2N

F=4N
4N
4N 0 0 f=4N

F=5N
5N 5N 0 0 f=5N

a = 2 m/s2

6N 4N F=6N
2N 2 m/s 2
fk = 4 N

a = 4 m/s2

8N 4N 4N 4 m/s 2 F=6N
fk = 4 N

Graphically this can be understood as under µN


tan λ = =µ
f (N) N
or λ = tan −1 (µ ) …(i)
fL = 5 N
A
This angle λ is called the angle of friction.
fk = 4 N

Angle of Repose (α )
45°
F (N) Suppose a block of mass m is placed on an inclined
O 5
plane whose inclination θ can be increased or decreased.
Fig. 5.74 Let, µ be the coefficient of friction between the block and the
plane. At a general angle θ,
Note that f = F till F ≤ f L . Therefore, slope of line OA
will be 1 ( y = mx ) or angle of line OA with F-axis is 45°.
/ Hence, we will take coefficient of friction as µ unless and until m
specially mentioned in the question µ s and µ k , separately.

Angle of Friction (λ) θ

At a point of rough contact, where N F Fig. 5.76


slipping is about to occur the two forces
acting on each object are the normal Normal reaction, N = mg cos θ
reaction N and frictional force µN . λ
Limiting friction
The resultant of these two forces is µN
f L = µN = µ mg cos θ
F and it makes an angle λ with the Fig. 5.75
and the driving force (or pulling force)
normal, where F = mg sin θ (Down the plane)
174 Objective Physics Vol. 1

From these three equations, we see that when θ is


increased from 0° to 90°, normal reaction N and hence, the Extra Knowledge Points
limiting friction f L is decreased while the driving force F is ■ Friction always opposes the relative motion between
increased. There is a critical angle called angle of repose two bodies in contact. They form an action and
reaction pair, i.e. two equal and opposite forces act on
(α ) at which these two forces are equal. Now, if θ is further
two different bodies.
increased, then the driving force F becomes more than the
limiting friction f L and the block starts sliding. A F
Thus, fL = F
at θ =α B
or µ mg cos α = mg sin α
or tan α = µ
■ The direction of friction force on each of them is such
or α = tan −1 (µ ) …(ii) as it either stops the relative motion or attempts to do
so. For example, if a force F is applied on block A of a
From Eqs. (i) and (ii), we see that angle of friction ( λ ) is two block system, the direction of frictional forces at
numerically equal to the angle of repose. different contacts on different bodies will be as shown.
or λ =α
From the above discussion, we can conclude that A F
f1
if θ < α, F < f L the block is stationary f1
if θ = α, F = f L the block is on the verge of sliding
B
and if θ > α, F > f L the block slides down with
acceleration. f2 f2

N, fL, F
Here, f1 = force of friction between A and B
mg and f2 = force of friction between B and ground
F ■ Force of friction f = 0, if no driving force is applied.
f ≤ fL ( = µs N )
µmg N If driving force is applied but no relative motion is there
and f = µk N
fL
If relative motion is there.
■ A common mistake which the students are in hurry is
θ that they always write fL = µmg (in case of horizontal
O θ=α 90° ground) or fL = µmg cos θ (in inclined surface). The
Fig. 5.77 actual formula is fL = µN. Here, N is equal to mg or
mg cos θ up to when no force is acting at some angle
F − fL ( ≠ 0° ) with the plane.
a= F
m F

= g (sin θ − µ cos θ )
F or
How, N , f L and F varies with θ, this can be shown
graphically as shown in Fig. 5.77 N = mg N ≠ mg
N = mg cos θ fL = µmg fL ≠ µmg
or N ∝ cos θ F
f L = µmg cos θ F
or f L ∝ cos θ
F = mg sin θ
θ θ
or F ∝ sin θ
N = mg cos θ N ≠ mg cos θ
Normally, µ <1 fL = µmg cos θ fL ≠ µmg cos θ
So, fL < N.
Laws of Motion 175

■ A car (or any vehicle) accelerates and decelerates by X Example 5.27 A particle of mass 1 kg rests on
friction. So, maximum acceleration or deceleration of a rough contact with a plane inclined at 30° to the
car on horizontal ground can be µg, unless some horizontal and is just about to slip. Find the
external force is applied.
coefficient of friction between the plane and the
■ Exercise Think about maximum acceleration or particle.
retardation on an inclined road.
■ The common problem which I feel, students often face Sol. The given angle 30° is really the angle of repose α. Hence,
is the resolution of forces. Following two rules can be µ = tan 30°
made in this regard. 1
=
Rule 1 If the body is in equilibrium, you can resolve the 3
forces in any direction. Net force should be zero in all
directions. A body moving with constant velocity is also X Example 5.28 A block of weight w rests on a
in equilibrium. horizontal plane with which the angle of friction is λ.
Rule 2 If the body is accelerated, resolve the forces A force P inclined at an angle θ to the plane is
along acceleration and perpendicular to it. Net force
along acceleration = m a and net force perpendicular
applied to the plane until it is on the point of moving.
to acceleration is zero. Find the value of θ for which the value of P will be
■ To find net force on a body find the acceleration of the least.
body. Net force = mass × acceleration. P

18N θ
1 kg 2 kg 3 kg

Smooth Fig. 5.78


a = 3 m/s2
Sol. In the limiting case, contact force F is inclined at λ to the
For example, in the figure shown,
normal. Only three forces act on the block. Applying Lami’s
18
a= = 3 m /s 2 theorem, we get
1+ 2 + 3
Therefore, net force on 1 kg block = 1 × 3 = 3 N. F (90°– θ) P
λ
This 3 N is the resultant of the applied force 18 N and
normal reaction between 1 kg block and 2 kg block.
Similarly, net force on 2 kg block = 2 × 3 = 6 N.
This is the resultant of normal reactions (1 kg and 2 kg)
and (2 kg and 3 kg) blocks.
w
■ Force of friction does not oppose the motion of a body
Fig. 5.79
but it opposes the relative motion between two bodies
in contact. P w
=
sin (180° − λ ) sin (90° − θ + λ )
1 kg
F
Rough =
sin (90° + θ)
2 kg F
P w
or =
sin λ cos (θ − λ )
Smooth w sin λ
or P=
As far as motion of individual body is concerned it is cos (θ − λ )
sometimes friction which is responsible for its motion. P will be least when cos (θ − λ ) is greatest, because w and
For example, in the figure shown the 1 kg block moves λ are constant.
with 2 kg block only due to friction. i.e. when cos (θ − λ ) = 1
and θ − λ = 0° or θ = λ
Additional Examples
Example 1. A stone when thrown on a glass window Sol. No. Force diagram on both sides is same. Tension (T) in
smashes the windowpane to pieces, but a bullet from upward direction and weight (mg) in downward direction. If
the gun passes through making a clean hole. Why? T > mg, both will move up with same velocity and same
acceleration.
Sol. Due to its small speed, the stone remains in contact with the
windowpane for a longer duration. It transfers its motion to the Example 7. Determine the tensions T1 and T2 in the
pane and breaks it into pieces. But the parcticles of
windowpane near the hole are unable to share the fast motion strings as shown in figure.
of the bullet and so remain undisturbed.
60°
Example 2. If a ball is thrown up in a moving train,
T1
it comes back to the person's hands. Why?
Sol. Both during its upward and downward motion, the ball T2
continues to move (inertia of motion) with the same horizontal 4 kg-wt
velocity as the train. In this period, the ball covers the same
horizontal distance as the train and so it comes back to the
thrower's hands. Sol. Resolving the tension T1 along horizontal and vertical
directions. As the body is in equilibrium,
Example 3. Why we are hurt less when we jump on
60°
a muddy floor in comparision to a hard floor? T1
Sol. When we jump on a muddy floor, the floor is carried in the 60°
direction of the jump and the time interval ∆t for which force T2
acts is increased. This decreases rate of change of momentum
and hence the force of reaction. Therefore, we are hurt less.

Example 4. Automobile tyres are generally provided w = 4 × 9.8 N


with irregular projections over their surfaces. Why? T1 sin 60 ° = 4 × 9.8 N K(i)
T1 cos 60 ° = T 2 K(ii)
Sol. Irregular projections increase the friction between the 4 × 9.8
rubber tyres and the road. This provides a firm grip between T1 =
sin 60 °
the tyres and the road and prevents slipping. 4 × 9.8 × 2
= = 45.26 N
Example 5. Why frictional force gets increased 3
when a surface is polished beyond a certain limit? T 2 = T1 cos 60 °
Sol. When surfaces are highly polished, the area of contact = 45.26 × 0.5 = 22.63 N
between them increases. As a result of this, a large number of
atoms and molecules lying on both the surfaces start exerting Example 8. A ball of mass 1 kg hangs in
strong attractive forces on each other and therefore frictional equilibrium from two strings OA and OB as shown in
force increases. figure. What are the tensions in strings OA and OB?
Example 6. A rope is hanging from a tree as shown (Take g = 10 ms −2 )
in figure. Equal mass of bananas are fastened to the A B
higher end of the rope, and a monkey is hanging along 30° 60°
the rope near its lower end. The monkey climbs along
the rope, will it be able to eat the bananas? T1 T2
90°
O 150°
120°

w = 10 N

Sol. Various forces acting on the ball are as shown in figure. The
three concurrent forces are in equilibrium. Using Lami’s
theorem,
T1 T2 10
= =
sin 150 ° sin 120 ° sin 90 °
Laws of Motion 177

T1 T2 10 Sol. In mode (a), the man applies a force equal to 25 kg weight in


or = =
sin 30 ° sin 60 ° 1 upward direction. According to Newton’s third law of motion,
∴ T1 = 10 sin 30 ° there will be a downward force of reaction on the floor.
= 10 × 0.5 = 5 N
and T 2 = 10 sin 60 °
3
= 10 ×
2
=5 3N

Example 9. A 4 m long ladder weighing 25 kg (a) (b)


rests with its upper end against a smooth wall and
lower end on rough ground. What should be the ∴ Total action on the floor by the man
minimum coefficient of friction between the ground and = 50 kg-wt + 25 kg-wt
the ladder for it to be inclined at 60° with the = 75 kg-wt
= 75 × 9.8 N
horizontal without slipping? (Take g = 10 ms −2 )
= 735 N
Sol. In figure, AB is a ladder of weight w which acts at its centre In mode (b), the man applies a downward force equal
of gravity G. to 25 kg-wt. According to Newton’s third law, the
N1 reaction will be in the upward direction.
∴ Total action on the floor by the man
= 50 kg-wt – 25 kg-wt
30° = 25 kg-wt
= 25 × 9.8 N = 245 N
As the floor yields to a downward force of 700 N, so the
G
man should adopt mode (b).
N2 w
Example 11. A block of mass 200 kg is set into
60° C motion on a frictionless horizontal surface with the
B f help of frictionless pulley and a rope system as shown
∠ ABC = 60 ° in figure (a). What horizontal force F should be
∴ ∠BAC = 30 ° applied to produce in the block an acceleration of
Let N 1 be the reaction of the wall, and N 2 the reaction 1 m/ s 2 ?
of the ground.
Force of friction f between the ladder and the ground a = 1 m/s2
acts along BC. F
For horizontal equilibrium,
f = N1 K(i)
For vertical equilibrium,
N2 = w K(ii) (a)
Taking moments about B, we get for equilibrium, a = 1 m/s2
N 1 (4 cos 30 ° ) − w (2 cos 60 ° ) = 0 K(iii)
Here, w = 250 N F T
T
Solving these three equations, we get
f = 72.17 N and N 2 = 250 N F
f 72.17
∴ µ= = = 0.288 (b)
N2 250
Sol. As shown in Fig. (b), when force F is applied at the end of the
Example 10. A block of mass 25 kg is raised by a
string, the tension in the lower part of the string is also F. If T is
50 kg man in two different ways as shown in figure. the tension in string connecting the pulley and the block, then
What is the action on the floor by the man in the two T = 2F
cases? If the floor yields to a normal force of 700 N, But, T = ma = (200 )(1) = 200 N
which mode should the man adopt to lift the block ∴ 2 F = 200 N
without the floor yielding? (Take g = 9.8 m /s 2 ) or F = 100 N
178 Objective Physics Vol. 1

Fnet − f L
Example 12. A block of mass 1 kg is pushed a=
against a rough vertical wall with a force of 20 N, m
1
coefficient of static friction being ⋅ Another horizontal 10 2 − 5
= = 9.14 m/s 2
4 1
force of 10 N is applied on the block in a direction
parallel to the wall. Will the block move? If yes, in Example 13. Figure shows a man standing
which direction? If no, find the frictional force exerted stationary with respect to a horizontal conveyor belt
by the wall on the block. ( g = 10 m/ s 2) that is accelerating with 1 ms −2 . What is the net force
on the man? If the coefficient of static friction between
Sol. Normal reaction on the block from the wall will be
the man’s shoes and the belt is 0.2, up to what
N = F = 20 N
acceleration of the belt can the man continue to be
stationary relative to the belt? Mass of the man = 65 kg.
(g = 9.8 m/s 2 )
F = 20 N
A

Therefore, limiting friction


 1
f L = µN =   (20 ) = 5 N
 4
Weight of the block is
w = mg = (1)(10 ) = 10 N Sol. As the man is standing stationary w.r.t. the belt,
10 N ∴ Acceleration of the man = Acceleration of the belt
= a = 1 ms −2
45° Mass of the man,
m = 65 kg
Net force on the man = ma = 65 × 1 = 65 N
Given, coefficient of friction, µ = 0.2
∴ Limiting friction, f L = µmg
10 N If the man remains stationary with respect to the
Fnet = 10√2 N
maximum acceleration a 0 of the belt, then
A horizontal force of 10 N is applied to the block. The ma 0 = f L = µmg
resultant of these two forces will be 10 2 N in the ∴ a 0 = µg
direction shown in figure. Since, this resultant is
greater than the limiting friction. The block will move = 0.2 × 9.8
in the direction of Fnet with acceleration. = 1.96 ms−2
NCERT Selected Questions
Q 1. Give the magnitude and direction of the net force (b) Just after it is dropped from the window of a
acting on train running at a constant velocity of 36 km / h
(a) a drop of rain falling down with a constant speed. (c) Just after it is dropped from the window of a
(b) a cork of mass 10 g floating on water. train accelerating with 1 ms −2
(c) a kite skillfully held stationary in the sky. Sol. (a) ∴ Net force on the stone = mg
(d) a car moving with a constant velocity of
= 0.1 × 9.8 = 0.98 N
30 km / h on a rough road.
and it acts vertically downwards.
Sol. (a) Since, the drop of rain is falling with a constant speed, (b) When the stone is dropped from the window of this
so the acceleration of the rain drop is zero. Hence, net train, then it is again falling freely, so
force on the drop is zero. net force on the stone = its own weight
(b) As the cork is floating in water, so the weight of the cork = mg
is balanced by upthrust. Therefore, net force on the cork = 0.98 N
is zero.
and it acts vertically downwards
(c) Since, the kite is held stationary, so its acceleration is
(c) Net force on stone is again mg or 0.98 N
zero. Hence, the net force acting on the kite is zero.
(d) Since, the car is moving with a constant velocity, so its Q 4. A constant retarding force of 50 N is applied to a
acceleration is zero. Hence, the net force acting on the body of mass 20 kg moving initially with a speed of
car is zero.
15 ms −1 . How long does the body take to stop?
Q 2. A pebble of mass 0.05 kg is thrown vertically
Sol. Using the relation F = ma,
upwards. Give the direction and magnitude of the
net force on the pebble F
We get a=
(a) during its upward motion. m
50
(b) during its downward motion. or a=−
20
(c) at the highest point where it is momentarily at
rest. = − 2.5 ms−2 (retardation)
Using the relation v = u + at,
Do your answers change, if the pebble was thrown We get 0 = 15 + (− 2.5) t
at an angle of 45° with the horizontal direction? or 2.5 t = 15
Ignore air resistance. 15
∴ t= =6s
Sol. (a) Net force on the pebble = mg (Q a = g ) 2.5
= 0.05 × 9.8 = 0.49 N Q 5. A constant force acting on a body of mass 3.0 kg
(Acts vertically downwards) . ms −1 to 35
changes its speed from 20 . ms −1 in 25 s.
(b) Net force on the pebble = mg (Q a = g ) The direction of motion of the body remains
= 0.05 × 9.8 = 0.49 N unchanged. What is the magnitude and direction of
(Acts vertically downwards) the force?
(c) When the stone is at the highest point, then also the net
force ( = mg ) acts in vertically downward direction. Sol. Using the relation v = u + at,
∴ Net force on the pebble = mg v − u 3.5 − 2
We get a= =
= 0.05 × 9.8 = 0.49 N t 25
1.5 15
If the pebble was thrown at an angle of 45° with the = = ms−2
horizontal direction, then it will not affect the force on 25 250
the pebble. Hence, our answers will not change in ∴Using the relation F = ma,
any case. We get the magnitude of the force as
Q 3. Give the magnitude and direction of the net force 15
F = 3×
acting on a stone of mass 0.1 kg. 250
(a) Just after it is dropped from the window of a = 0.18 N
stationary train. The direction of the force is along the direction of motion.
180 Objective Physics Vol. 1

Q 6. A body of mass 5 kg is acted upon by two Q 8. A rocket with a lift-off mass 20000 kg is blasted
perpendicular forces 8 N and 6 N. Give the upwards with an initial acceleration of 5.0 ms −2 .
magnitude and direction of the acceleration of the Calculate the initial thrust (force) of the blast.
body.
Sol. Let T be the initial thrust acting upward.
Sol. To find a let us first find net force on the body. ∴ Equation of motion becomes,
Let F be the net force acting on the body. T − mg = ma or T = mg + ma
B C = m (g + a) = 2 × 104 (9.8 + 5)
= 2.96 × 105 N ≈ 3.0 × 105 N
F2 = 6 N

F
Q 9. A bob of mass 0.1 kg hung from the ceiling of a room
90º α
by a string 2 m long is set into oscillation. The speed of
O F1 = 8 N A the bob at its mean position is 1 ms −1 . What is the
Magnitude of F The magnitude of F is given by
trajectory of the bob, if the string is cut when the bob is
(a) at one of its extreme positions.
F = F12 + F22 (b) at its mean position.
= 82 + 6 2 Sol. (a) We know that at each extreme position, the
= 10 N instantaneous velocity of the bob is zero. If the string is
Direction of F Let α be the angle made by F with F1 cut at the extreme position, it is under the action of g
only, hence the bob will fall extreme downwards.
F 6
Then, tanα = 2 = = 0.75 (b) At mean position, the bob is having a velocity of 1 ms−1
F1 8
along the tangent of the arc which is in the horizontal
or α = 37° with the direction of 8 N.
direction. If the string is cut at the mean position, the
This is the direction of resultant force and hence the bob will behave as a horizontal projectile. Hence, it will
direction of acceleration of the body. follow a parabolic path.
F 10
a= = = 2 ms−2
m 5 Q 10. A man of mass 70 kg stands on a weighing scale in a
∴ Magnitude of acceleration = 2 ms−2 and acts in the lift which is moving
direction of resultant force. (a) upwards with uniform speed of 10 ms −1 .
Q 7. The driver of a three wheeler moving with a speed of (b) downwards with a uniform acceleration of 5 ms −2 .
36 km/ h sees a child standing in the middle of the road (c) upwards with uniform acceleration of 5 ms −2 .
and brings his vehicle to rest in 4.0 s just in time to What would be the readings on the scale in each
save the child. What is the average retarding force on case?
the vehicle? The mass of the three wheeler is 400 kg (d) What would be the reading if the lift mechanism
and the mass of the driver is 65 kg. failed and it falls down freely under gravity?
Sol. Initial speed of the three wheeler,
Sol. (a) When the speed of the lift is uniform, its acceleration
5 a = 0. So, the weighing machine will show his weight,
u = 36 km −1 = 36 × ms−1
18 i.e.
= 10 ms−1 N = w = mg = 70 × 9.8 = 686 N
Final speed of the three wheeler, ∴ Reading of the scale = 686 N
v=0 = 70 kg
Time, t=4s (b) When the lift moves downward with an acceleration
Total mass, m = mass of three wheeler + mass of driver a = 5 ms−2
= 400 + 65 = 465 kg ∴ Resultant force acts vertically downward
Using the relation v = u + at, i.e. F = mg − N
v − u 0 − 10 ma = mg − N
We get a= = or
t 4
or N = mg − ma = m(g − a)
= − 2.5 ms−2
= 70 × 4.8 = 336 N
− ve sign shows that it is retardation.
∴Reading of the scale = 336 N
∴ |F| = m |a | 336
= 465 × (2.5) = 1162.5 N = kg
9.8
= 1.16 × 103 N = 1.2 × 103 N = 34.29 kg
Laws of Motion 181

(c) In this case, the left is moving upwards with an Q 12. Two masses 8 kg and 12 kg are connected at the two
acceleration,
ends of a light inextensible string that goes over a
a = 5 ms−2
frictionless pulley. Find the acceleration of the masses
∴ Resultant force acting is upward and the tension in the string when the masses are
F = N − mg released.
or N = ma + mg Sol. The equations of motion of m1 and m2 are given by
= m (g + a)
= 70 (9.8 + 5) = 14.8 × 70
= 1036 N
∴ Reading of the scale = 1036 N
T T a
1036
= kg
9.8 a
= 105 .7 kg
m1
(d) In this case, the lift falls freely under gravity, i.e. with an
acceleration m2
a = g = 9.8 ms−2 m1g
∴ N = m (g − a) = m (g − g ) = 0 m2g
Thus, the reading of the scale is zero. This is the state of
T − m1g = m1a …(i)
weightlessness.
and m2g − T = m2a …(ii)
Q 11. Two bodies of masses 10 kg and 20 kg respectively Putting m1 = 8 kg and m2 = 12 kg and T a

kept on a smooth, horizontal surface are tied to the g = 10 ms−2, we get T


ends of a light string. A horizontal force F = 600 N is a = 2 m / s2 m2 m1
applied to (i) A, (ii) B along the direction of string. a
and T = 96 N
What is the tension in the string in each case? m2g m1g

Sol. F = 600 N Q 13. Figure below shows a man standing stationary with
m1 = 10 kg respect to a horizontal conveyor belt that is
m2 = 20 kg accelerating with 1 ms −2 . What is the net force on the
B man? If the coefficient of static friction between the
A
T T m2 man’s shoes and the belt is 0.2, up to what acceleration
m1 F or a
of the belt can the man continue to be stationary
relative to the belt? (Mass of the man = 65 kg)
Let T be the tension in the string and a be the
acceleration of the system, in the direction of force
applied.
(a) When force is applied on the heavier mass.
Then, equations of motion of A and B are
m1 a = T …(i)
m2a = F − T …(ii)
Solving the equations, we get
F 600
T = = = 200 N Sol. As the man is standing stationary w.r.t. the belt, so
3 3
acceleration of man = acceleration of belt = a = 1 ms−2.
(b) When the force is applied on lighter mass, then
B Mass of man, m = 65 kg
A
m1 T' m2 ∴ Net force on the man, F = ma = 65 × 1 = 65 N.
F
The direction of this force is in the direction of motion of the
Let T be the tension in the string in this case. belt. Coefficient of static friction between man’s shoes and
belt.
Then, equations of motion of A and B are
µ s = 0.2
F − T ′ = m1a …(iii)
Let a′ be the acceleration of the belt up to which the man can
T ′ = m2a …(iv) continue to be stationary relative to the belt.
Again solving the equations for given values, we have ∴ ma′ = maximum value of static friction
2 2 or ma = µ s N = µ s mg
or T = F = × 600 = 400 N or a = µ s g = 0.2 × 9.8 = 1.96 ms−2
3 3
182 Objective Physics Vol. 1

Q 14. A helicopter of mass 1000 kg rises with a vertical Q 16. Ten one-rupee coins are put on top of each other on a
acceleration of 15 ms −2 . The crew and the passengers table. Each coin has mass m. Give the magnitude and
weigh 300 kg. Give the magnitude and direction of the direction of
(a) force on floor by the crew and passengers. (a) the force on the 7th coin (counted from the bottom)
due to all the coins on its top.
(b) action of the rotor of the helicopter on the
surrounding air. (b) the force on the 7th coin by the eighth coin.
(c) force on the helicopter due to the surrounding air, (c) the reaction of the 6th coin on the 7th coin.
(Take g = 10 ms −2 ). Sol. (a) If F1 be the force on 7th coin (counted from the bottom)
Sol. Mass of the helicopter, N1
F1 = weight of three coins above it = 3 mg (downward)
m = 1000 kg (b) F8 = Force on 7th cion by 8th coin = weight of 8th coin ÷
Mass of the crew and the weight of two coins above the 8th coin
passengers, m = 300 kg = mg + 2mg = 3mg (N) and it acts downwards.
Vertical acceleration of the (c) The sixth coin experiences force equal to weight of the
mg
helicopter, a = 15 ms−2 four coins above it. Hence, reaction due to 6th coin on
7th coin = 4 mg (N) and it acts vertically upwards.
Acceleration due to gravity, g = 10 ms−2
(a) Let N 1 be the force by the floor on the crew and the Q 17. A monkey of mass 40 kg
passengers climbs on a rope which can
N 1 − mg = ma stand a maximum tension of
or N 1 = m (g + a) = 300 (10 + 15) 600 N. In which of the
= 300 × 25 = 7500 N following cases will the rope
break : the monkey
and it acts upwards.
By 3rd law, force on the floor by the crew and passengers (a) climbs up with an acceleration of 6 ms −2
= 7500N (b) falls down with an acceleration of 4 ms −2
(b) Let N 2 = force by air on the rotor (action) upwards
(c) climbs up with a uniform speed of 5 ms −1
∴ N 2 − (M + m) g = (M + m) a
(d) falls down the rope nearly freely under gravity
N 2 = (M + m) (g + a)
= (1000 + 300) (10 + 15) (Ignore the mass of the rope) and take g = 10 ms −2 .
= 1300 × 25 = 32500 N Sol. (a) When the monkey climbs up with
∴By third law, force (action) of the rotor of helicopter a = 6 ms−2, the T
on the surrounding air = 32500 N downwards.
∴ Equation of motion is m a
(c) According to Newton’s third law of motion, action and
T − mg = ma
reaction are equal and opposite. mg
∴ T = mg + ma = m (g + a)
∴Force on the helicopter due to surrounding air
= 32500 Nand it acts upwards. = 40 (10 + 6) = 40 × 16
= 640 N
Q 15. A stream of water flowing horizontally with a speed of Thus, T > Tmax hence the rope will break in this case.
15 ms −1 gushes out of a tube of cross-sectional area
10−2 m 2 , and hits a vertical wall nearby. What is the (b) When the monkey falls down with
force exerted on the wall by the impact or water, a = 4 ms−2, then T
m a
assuming it does not rebound? Equation of motion is
mg − T = ma or T = mg − ma
Sol. Area of cross-section of the tube, a = 10−2 m 2 mg
= m (g − a) = 40 (10 − 4 )
Speed of stream of water = 15 ms−1 = 40 × 6 = 240 N
V = volume of water coming out per second from the tube As T < Tmax , so the rope will not break in this case.
= a × V = 15 × 10−2 m 3s−1 (c) When the monkey climbs up with a uniform speed of
Also we know that density of water, ρ = 103 kg m −3 5 ms−1, then there is no acceleration, i.e. a = 0, hence
∴ If m be the mass of water striking the wall per second, then T = mg = 40 × 10 = 400 N
−2 −1 −1
m = ρV = 10 × 15 × 10 kg s = 150 kg s
3
which is < Tmax , so the string will not break.
As on hitting the wall, water does not rebound, so (d) When the monkey falls freely, then a = g
F = change in momentum per second ∴ T = m(g − a) = m (g − g ) = 0
= mass of water flowing out per second × velocity i.e. the monkey is in the state of weightlessness and
= 150 × 15 = 2250 N T = 0, hence the rope will not break.
Objective Problems
[ Level 1 ]
Equilibrium of Forces 6. A block of mass 10 kg is suspended by three strings as
1. A rope of length L and mass M is hanging from a rigid shown in the figure. The tension T2 is
support. The tension in the rope at a distance x from the
60° 30°
rigid support is
 L − x T2
(a) Mg (b)   Mg T3
 L 
 L  x T1
(c)   Mg (d) Mg 10 kg
 L − x L
100
2. The pulleys and strings shown in the figure are smooth (a) 100 N (b) N
3
and of negligible mass. For the system to remain in (c) 3 × 100 N (d) 50 3 N
equilibrium, the angle θ should be
7. A man of mass 50 kg stands on a frame of mass 30 kg. He
α pulls on a light rope which passes over a pulley. The
other end of the rope is attached to the frame. For the
2m system to be in equilibrium what force man must exert on
the rope?
m m

(a) 0° (b) 30°


(c) 45° (d) 60°

3. A weightless rod is acted upon by upward parallel forces


of 4 N and 2 N magnitudes at ends A and B, respectively.
The total length of the rod AB = 3 m. To keep the rod in
equilibrium, a force of 6 N should act in the following
manner (a) 40 g (b) 80 g
(a) downward at any point between A and B (c) 30 g (d) 50 g
(b) downward at the mid-point of AB 8. Three blocks are placed at rest on a smooth inclined plane
(c) downward at a point C such that AC = 1 m
with force acting on m1 parallel to the inclined plane.
(d) downward at a point C such that BC = 1m
Find the contact force between m2 and m3 .
4. Ten coins are placed on top of each other on a horizontal
table. If the mass of each coin is 10 g and acceleration due m3
to gravity is 10 ms −2 , what is the magnitude and direction m2
of the force on the 7th coin (counted from the bottom) m1
due to all the coins above it?
F θ
(a) 0.3 N downwards
(b) 0.3 N upwards (m1 + m2 + m3 ) F m3 F
(c) 0.7 N downwards (a) (b)
m3 m1 + m2 + m3
(d) 0.7 N upwards
(c) F − (m1 + m2 ) g (d) None of these
5. An object is resting at the bottom of two strings which are
9. Four forces act on a point object. The object will be in
inclined at an angle of 120° with each other. Each string
equilibrium, if
can withstand a tension of 20 N. The maximum weight of
(a) all of them are in the same plane
the object that can be sustained without breaking the (b) they are opposite to each other in pairs
strings is (c) the sum of x , y and z-components of forces is zero separately
(a) 10 N (b) 20 N (d) they form a closed figure of 4 sides when added as per
(c) 20 2 N (d) 40 N polygon law
184 Objective Physics Vol. 1

10. A light string going over a clamped pulley of mass m 15. A body of mass 2 kg is hung on a spring balance mounted
supports a block of mass M as shown in the figure. The vertically in a lift. If the lift moves up with an
force on the pulley by the clamp is given by acceleration equal to the acceleration due to gravity, the
m
reading on the spring balance will be
(a) 2 kg (b) 2 × g kg
(c) 4 × g kg (d) 4 kg

16. A monkey is descending from the branch of a tree with a


M
constant acceleration. If the breaking strength of the
(a) 2 Mg (b) 2 mg branch is 75% of the weight of the monkey, the minimum
(c) g (M + m) + m
2 2
(d) g (M + m)2 + M 2 acceleration with which the monkey can slide down
without breaking the branch is
11. A non-uniform rod AB of weight w P
(a) g (b) 3g / 4
30º Q
is supported horizontally in a (c) g /2 (d) g /4
60º
vertical plane by two light strings G 17. The pendulum hanging from the ceiling of a railway
PA and QB as shown in the figure. A B carriage makes an angle 30° with the vertical when it is
G is the centre of gravity of the rod.
w accelerating. The acceleration of the carriage is
If PA and QB make angles 30° and
AG 3 2
60° respectively with the vertical, the ratio is (a) g (b) g
GB 2 3
g
1 1 1 (c) g 3 (d)
(a) (b) 3 (c) (d) 3
2 3 3
18. Two bodies of masses 5 kg and 3 kg respectively are
12. A 40 N block supported by two ropes. One rope is
connected to two ends of a light string passing over
horizontal and the other makes an angle of 30° with the
horizontal frictionless pulley. The tension in the string is
ceiling. The tension in the rope attached to the ceiling
( Take g = 9.8 m/s 2 )
is approximately
(a) 80 N (b) 40 N (a) 60 N (b) 36.75 N
40 (c) 73.50 N (d) 18 N
(c) 40 3 N (d) N
3
19. Three blocks of masses m1 , m2 and m3 are connected by
13. A ball of mass 1kg hangs in equilibrium from a two massless strings as shown on a frictionless table. They are
strings OA and OB as shown in figure. What are the pulled with a force T3 = 40 N. If m1 = 10 kg, m2 = 6 kg
tensions in strings OA and OB? (Take g = 10 m/ s 2 ) and m3 = 4 kg, the tension T2 will be
T1 T2 T3
A B m1 m2 m3
30º 60º
(a) 20 N (b) 40 N
(c) 10 N (d) 32 N
T1 90º T
2
20. Three equal weights A , B and C of mass 2 kg each are
120º O 150º
hanging on a string passing over a fixed pulley which is
w = 10 N frictionless as shown in figure. The tension in the string
connecting weight B and C is
(a) 5 N, 5 N (b) 5 3 N, 5 3 N
(c) 5 N, 5 3 N (d) 5 3 N, 5 N

Acceleration
14. A mass is hanging on a spring balance which is kept in a
lift. The lift ascends with increasing speed. The spring
balance will show in its reading A
B
(a) increase
(b) decrease
(c) no change C
(d) change will depend upon velocity (a) zero (b) 13 N (c) 3.3 N (d) 19.6 N
Laws of Motion 185

21. Consider an elevator moving downwards with an 27. The acceleration of the 2 kg block if the free end of string
acceleration a, the force exerted by a passenger of mass m is pulled with a force of 20 N as shown is
on the floor of the elevator is
(a) ma (b) ma − mg
(c) mg − ma (d) mg + ma

22. Two masses M 1 and M 2 are attached to the ends of a F = 20 N


string which passes over a pulley attached to the top of an
inclined plane. The angle of inclination of the plane is 30°
and M 1 = 10 kg, M 2 = 5 kg. What is the acceleration of 2 kg
mass M 2 ?
(a) zero (b) 10 m / s2
2
(c) 5 m / s upward (d) 5 m / s2 downward

28. If the elevator in the shown figure is moving upwards


with constant acceleration 1 m/ s 2 , the tension in the string
M1
connected to block A of mass 6 kg would be
( Take g = 10 m/ s 2 )
M2
θ
1 m/s2
(a) 10 m / s2 (b) 5 m / s2
(c) Zero (d) Data insufficient A

23. A force of 50 N is required to push a car on a level road


with constant speed of 10 ms −1 . The mass of the car is (a) 60 N (b) 66 N
500 kg. What forces should be applied to make the car (c) 54 N (d) 42 N
accelerate at 1 ms −2 ?
29. Two weights w1 and w 2 are suspended from the ends of a
(a) 450 N (b) 500 N
(c) 550 N (d) 2500 N light string passing over a smooth fixed pulley. If the
pulley is pulled up at an acceleration g. The tension in the
24. A 50 kg boy stands on a platform spring scale in a lift that string will be
is going down with a constant speed 3 m/ s. If the lift is 4 w1 w2 2w1 w2
(a) (b)
brought to rest by a constant deceleration in a distance of w1 + w2 w1 + w2
9 m, what does the scale read during this period? w − w2 w1 w2
(c) 1 (d)
( Take g = 9.8 m/ s 2 ) w1 + w2 2 (w1 + w2 )
(a) 500 N (b) 465 N 30. Two blocks, each having a mass M, rest on frictionless
(c) 515 N (d) Zero
surfaces as shown in the figure. If the pulleys are light
25. An elevator and its load have a total mass of 800 kg. If the and frictionless and M on the incline is allowed to move
elevator, originally moving downward at 10 m/ s, is down, then the tension in the string will be
brought to rest with constant deceleration in a distance of
25 m, the tension in the supporting cable will be
( Take g = 10 m/ s 2 )
M
(a) 8000 N
(b) 6400 N
(c) 11200 N
(d) 9600 N
θ M
26. A block is placed on the top of a smooth inclined plane of
inclination θ kept on the floor of a lift. When the lift is (a)
2
Mg sinθ
descending with a retardation a, the block is released. The 3
acceleration of the block relative to the incline is 3
(b) Mg sinθ
(a) g sinθ 2
(b) asinθ Mg sin θ
(c)
(c) (g − a) sin θ 2
(d) (g + a) sin θ (d) 2Mg sinθ
186 Objective Physics Vol. 1

31. When a force F acts on a body of mass m, the acceleration 35. A block of mass 5 kg resting on a horizontal surface is
produced in the body is a. If three equal forces connected by a cord, passing over a light frictionless
F1 = F2 = F3 = F act on the same body as shown in pulley to a hanging block of mass 5 kg. The coefficient of
figure. The acceleration produced is kinetic friction between the block and the surface is 0.5.
Tension in the cord is ( Take g = 9.8 m/s 2 )
F 2
A
5 kg

135º 90º
m F
1 5 kg B

(a) 49 N (b) zero


F
3
(c) 36.75 N (d) 12.75
(a) ( 2 − 1) a (b) ( 2 + 1) a 36. A wooden box of mass 8 kg slides down an inclined plane
(c) 2 a (d) a of inclination 30° to the horizontal with a constant
acceleration of 0.4 m /s 2 . What is the force of friction
32. A balloon of weight w is falling vertically downward with
a constant acceleration a (< g ). The magnitude of the air between the box and inclined plane? ( Take g = 10 m/s 2 )
resistance is (a) 36.8 N (b) 76.8 N
 a (c) 65.6 N (d) None of these
(a) w (b) w 1 + 
 g 37. A 30 kg block rests on a rough horizontal surface. A force
 a a
(c) w 1 −  (d) w of 200 N is applied on the block. The block acquires a
 g g speed of 4 m/s starting from rest in 2 s. What is the value
of coefficient of friction?
33. In the arrangement shown in the figure, the pulley has a
(a) 10/ 3 (b) 3 /10
mass 3m. Neglecting friction on the contact surface, the
force exerted by the supporting rope AB on the ceiling is (c) 0.47 (d) 0.184

A 38. Starting from rest, a body slides down a 45° inclined


plane in twice the time it takes to slide down the same
B distance in the absence of friction. The coefficient of
friction between the body and the inclined plane is
(a) 0.2 (b) 0.25
m (c) 0.75 (d) 0.5
2m
39. A car having a mass of 1000 kg is moving at a speed of
(a) 6 mg (b) 3 mg
(c) 4 mg (d) None of these
30 m/s. Brakes are applied to bring the car to rest. If the
frictional force between the tyres and the road surface is
Friction 5000 N, the car will come to rest in
(a) 5 s (b) 10 s (c) 12 s (d) 6 s
34. Two blocks are connected over a massless pulley as
40. A 100 N force acts horizontally on a block of mass 10 kg
shown in figure. The mass of block A is 10 kg and the
placed on a horizontal rough table of coefficient of
coefficient of kinetic friction is 0.2. Block A sliders down
friction µ = 0.5. If g at the place is 10 ms −2 , the
the incline at constant speed. The mass of block B in
acceleration of the block is
kg is
(a) zero (b) 10 m / s2
(c) 5 m / s2 (d) 5.2 m/s2

41. A block of mass 2 kg is placed on the floor. The


coefficient of static friction is 0.4. If a force of 2.8 N is
B applied on the block parallel to the floor, the force of
30°
friction between the block and floor is (Take
(a) 5.4 (b) 3.3 g = 10 ms −2 )
(c) 4.2 (d) 6.8 (a) 2.8 N (b) 8 N (c) 2 N (d) zero
Laws of Motion 187

42. A mass placed on an inclined place is just in equilibrium. 48. A body is projected up a 45° rough incline. If the
If µ is coefficient of friction of the surface, then coefficient of friction is 0.5, then the retardation of the
maximum inclination of the plane with the horizontal is block is
(a) tan −1 µ (b) tan −1 (µ / 2) (a)
g
(b)
g
−1 −1
(c) sin µ (d) cos µ 2 2 2
3g g
(c) (d)
43. A block of mass 0.1 kg is held against a wall applying a 2 2 2
horizontal force of 5 N on the block. If the coefficient of
friction between the block and the wall is 0.5, the 49. The coefficient of friction between the tyres and road is
magnitude of the frictional force acting on the block is 0.4. The minimum distance covered before attaining a
(a) 2.5 N (b) 0.98 N speed of 8 ms −1 starting from rest is nearly ( g =10 m /s 2 )
(c) 4.9 N (d) 0.49 N (a) 8.0 m (b) 4.0 m
(c) 10.0 m (d) 16.0 m
44. A block of mass 4 kg is placed on a rough horizontal
50. A block has been placed on an inclined plane. The slope
plane. A time dependent force F = kt 2 acts on the block,
angle θ of the place is such that the block slides down the
where k = 2 N/ s 2 . Coefficient of friction µ = 0.8. Force of plane at a constant speed. The coefficient of kinetic
friction between block and the plane at t = 2s is friction is equal to
(a) 8 N (b) 4 N (a) sinθ (b) cos θ
(c) 2 N (d) 32 N (c) g (d) tan θ
45. A body is projected along a rough horizontal surface with 51. A mass m rests on a horizontal surface. The coefficient of
a velocity 6 m/ s. If the body comes to rest after travelling friction between the mass and the surface is µ. If the mass
9 m, then coefficient of sliding friction, is ( g = 10 m/ s 2 ) is pulled by a force F as shown in figure. The limiting
(a) 0.5 (b) 0.4 friction between mass and the surface will be
(c) 0.6 (d) 0.2
F
46. A block of mass m is given an initial downward velocity
v 0 and left on an inclined place (coefficient of friction 60º
= 0.6 ). The block will

x
v0
  3 
(a) µmg (b) µ mg −   F 
  2 
30º   F    F 
(c) µ mg −    (d) µ mg +   
  2    2 
(a) continue of move down the plane with constant velocity v0
(b) accelerate downward 52. A block of mass 5 kg is kept on a horizontal floor having
(c) decelerate and come to rest
coefficient of friction 0.09. Two mutually perpendicular
(d) first accelerate downward then decelerate
horizontal forces of 3 N and 4 N act on this block. The
47. A block of weight 5 N is pushed against a vertical wall by acceleration of the block is ( g = 10 m/ s 2 )
a force 12 N. The coefficient of friction between the wall (a) zero
and block is 0.6. The magnitude of the force exerted by (b) 0.1 m/s2
the wall on the block is (c) 0.2 m / s2
(d) 0.3 m / s2

53. A block of mass 3 kg is at rest on a rough inclined plane


as shown in the figure. The magnitude of net force
12 N exerted by the surface on the block will be
3 kg

30º

(a) 12 N (b) 5 N (a) 15 3 N (b) 15 N


(c) 7.2 N (d) 13 N (c) 10 N (d) 30 N
188 Objective Physics Vol. 1

54. A block of mass m is at rest on an inclined plane which is 59. The upper half of an inclined plane of inclination θ is
making angle θ with the horizontal. The coefficient of perfectly smooth while the lower half rough. A block
friction between the block and plane is µ. Then, frictional starting from rest at the top of the plane will again come
force acting between the surfaces is to rest at the bottom if the coefficient of friction between
the block and the lower half of the plane is given by
m (a) µ = 2tan θ (b) µ = tan θ
2 1
(c) µ = (d) µ =
tan θ tan θ

θ 60. A block is gently placed on a conveyor belt moving


horizontally with constant speed. After t = 4 s, the
(a) µ mg velocity of the block becomes equal to the velocity of the
(b) µ mg sin θ belt. If the coefficient of friction between the block and
(c) µ (mg sin θ − mg cosθ)
the belt is µ = 0.2, then the velocity of the conveyor belt is
(d) mg sinθ
(a) 3 ms−1 (b) 4 ms−1
55. A block of mass 2 kg rests on a rough inclined plane (c) 6 ms−1 (d) 8 ms−1
making an angle of 30° with the horizontal. The
coefficient of static friction between the block and the 61. The breaking strength of the cable used to pull a body is
plane is 0.7. The frictional force on the block is 40 N. A body of mass 8 kg is resting on a table of
(a) 9.8 N coefficient of friction µ = 0.20. The maximum
(b) 0.7 × 9.8 × 3 N acceleration which can be produced by the cable is
(c) 9.8 × 3 N (Take g = 10 ms −2 )
(d) 0.7 × 9.8 N
(a) 6 ms−2 (b) 3 ms−2
56. A minimum force F is applied to a block of mass 102 kg (c) 8 ms−1 (d) 8 ms−2
to prevent it from sliding on a plane with an inclination
angle 30° with the horizontal. If the coefficients of static 62. A 13 m ladder is placed against a smooth vertical wall
and kinetic friction between the block and the plane are with its lower end 5m from the wall. What should be the
0.4 and 0.3 respectively, then the force F is minimum coefficient of friction between ladder and floor
(a) 157 N so that it remains in equilibrium?
(b) 224 N (a) 0.36 (b) 0.72
(c) 315 N (c) 0.21 (d) 0.52
(d) zero
63. A box of mass 8 kg is placed on a rough inclined plane of
57. A uniform rope of length l lies on a table. If the inclination θ. Its downward motion can be prevented by
coefficient of friction is µ, then the maximum length l1 of applying an upward pull F. And it can be made to slide
the hanging part of the rope which can overhang from the upwards by applying a force 2F. The coefficient of
edge of the table without sliding down is friction between the box and the inclined plane is
(a) l/µ 1
(b) l/(µ + 1) (a) tanθ (b) 3tanθ
3
(c) µl/ (µ + 1) 1
(d) µl/ (µ − 1) (c) tanθ (d) 2tanθ
2
58. An insect crawls up a hemispherical surface very slowly
64. A block of mass m, lying on a rough horizontal plane, is
(see the figure). The coefficient of friction between the
1 acted upon by a horizontal force P and another force Q,
insect and the surface is . If the line joining the centre of inclined at an angle θ to the vertical upwards. The block
3
will remain in equilibrium, if minimum coefficient of
the hemispherical surface to the insect makes an angle α
friction between it and the surface is
with the vertical, the maximum possible value of α is
(P + Q sin θ )
given by (a)
(mg − Q cosθ )
(P cosθ + Q )
α (b)
(mg − Q sin θ )
(P − Q cosθ )
(c)
(mg + Q sin θ )
(P sin θ − Q )
(a) cotα = 3 (b) tanα = 3 (d)
(mg − Q cosθ )
(c) sec α = 3 (d) cosec α = 3
Laws of Motion 189

65. In the figure shown, if coefficient of friction is µ, then m2 69. A ball of mass 0.5 kg moving with a velocity of 2 m/s
will start moving upwards if strikes a wall normally and bounces back with the same
speed. If the time of contact between the ball and the wall
is one millisecond, the average force exerted by the wall
on the ball is
(a) 2000 N (b) 1000 N
(c) 5000 N (d) 125 N
m2
70. If a body loses half of its velocity on penetrating 3 cm in a
m1 wooden block, then how much will it penetrate more
θ before coming to rest?
m1 m1 (a) 1cm (b) 2 cm
(a) > sin θ − µ cosθ (b) > sin θ + µ cosθ (c) 3 cm (d) 4 cm
m2 m2
m m 71. A ball is dropped on to the floor from a height of 10 m. It
(c) 1 > µ sin θ − cosθ (d) 1 > µ sin θ + cosθ
m2 m2 rebounds to a height of 2.5 m. If the ball is in contact with
floor for 0.01 s. What is the average acceleration during
66. A block of mass M rests on a rough horizontal surface as contact?
shown. Coefficient of friction between the block and the (a) 700 m/s 2 (b) 1400 m/s 2
surface is µ. A force F = Mg acting at angle θ with the (c) 2100 m/s 2 (d) 2800 m/s 2
vertical side of the block pulls it. In which of the
following cases, the block can be pulled along the 72. Pushing force making an angle θ to the horizontal is
surface? applied on a block of weight w placed on a horizontal
F table. If the angle of friction is φ, the magnitude of force
required to move the body is equal to
θ
wcosθ wsin θ
M (a) (b)
µ cos (θ − φ ) cos (θ + φ )
wtan φ wsin φ
θ  (c) (d)
(a) tanθ ≥ µ (b) tan  ≥ µ sin (θ − φ ) tan (θ − φ )
2 
 θ 73. A block of mass mis placed on a smooth plane inclined at
(c) cos θ ≥ µ (d) cot   ≥ µ
 2
an angle θ with the horizontal. The force exerted by the
67. A man of mass 60 kg is standing on a horizontal conveyor plane on the block has a magnitude
belt. When the belt is given an acceleration of 1 ms −2 , the (a) mg (b) mg sec θ
man remains stationary with respect to the moving belt. If (c) mg cos θ (d) mg sin θ
g = 10 ms −2 , the net force acting on the man is 74. Two blocks are in contact on a frictionless table. One has
a = 1 ms–2 mass m and the other 2 m. A force F is applied on 2 m as
shown in the figure. Now, the same force F is applied
from the right on m. In the two cases, the ratio of force of
contact between the two blocks will be
2m
m
(a) zero (b) 120 N F
(c) 60 N (d) 600 N

Miscellaneous Problems (a) same (b) 1: 2


(c) 2 : 1 (d) 1 : 3
68. Three masses of 6 kg, 4 kg and 2 kg are
attached to a rigid support as shown in 6 kg
75. A dynamometer D is attached to two bodies of masses
figure. If the string attached to the support M = 6 kg and m = 4 kg. Forces F = 20 N and f = 10 N are
breaks and the system falls freely, then the applied to the masses as shown. The dynamometer reads
tension in the string connecting 4 kg and
4 kg D
2 kg mass is F M m f
(a) zero (b) 8 kg-wt
(c) 12 kg-wt (d) 6 kg-wt (a) 10N (b) 20 N
2 kg (c) 6 N (d) 14 N
190 Objective Physics Vol. 1

76. A smooth inclined plane of length L, having an 81. There is no slipping between the two blocks. What is the
inclination θ with horizontal is inside a lift which is force of friction between two blocks?
moving down with retardation a. The time taken by a 2 kg
block to slide down the inclined plane from rest will be
4 kg F = 18 N
2L 2L
(a) (b) Smooth
asinθ g sinθ
(a) Zero (b) 9 N (c) 12 N (d) 6 N
2L 2L
(c) (d)
(g − a)sin θ (g + a)sin θ 82. Two masses are connected by a string which passes
over a pulley accelerating upwards at a rate A as
77. Consider the shown y
shown. If a1 and a 2 be the accelerations of bodies 1
arrangement. Assume and 2 respectively, then
all surfaces to be
x
smooth. If N represents m A
magnitudes of normal M
reaction between block θ
and wedge, then
acceleration of M along horizontal is equal to
N sin θ
(a) (along + ve x-axis) a1
M a2
N cosθ
(b) (along − ve x-axis) 1
M
N sin θ
(c) (along − ve x-axis) 2
M (a) A = a1 − a2 (b) A = a1 + a2
N sinθ
(d) (along − ve x-axis) a − a2 a + a2
m+ M (c) A = 1 (d) A = 1
2 2
78. In the above problem, normal reaction between ground 83. In the shown arrangement, mass of A = 1kg, mass of
and wedge will have magnitude equal to
B = 2 kg. Coefficient of friction between A and B = 0.2.
(a) N cosθ + Mg (b) N cosθ + Mg + mg
(c) N cosθ − Mg (d) N sinθ + Mg + mg A
B F = 10 N
79. For the arrangement shown in the figure, the tension in
 3
the string is given by sin 37° = 
 5 There is no friction between B and ground. The frictional
force exerted by A on B equals
(a) 2 N (b) 3 N
(c) 4 N (d) 5 N

10 kg 84. Find the force exerted by 5 kg block on floor of lift, as


shown in figure. (Take g = 10 m/ s 2 )
4 kg
µ = 0.7
37º
2
(a) 30 N (b) 40 N (c) 60 N (d) 30 N 5 m/s

80. A body of mass 10 kg is placed on rough surface, pulled


by a force F making an angle of 30° above the horizontal. 2 kg
If the angle of friction is also 30°, then the minimum 5 kg
magnitude of force F required to move the body is equal
to (Take g = 10 m/ s 2 ) (a) 100 N (b) 115 N
(a) 100 N (b) 50 2 N (c) 100 2 N (d) 50 N (c) 105 N (d) 135 N
Laws of Motion 191

[ Level 2 ]
Only One Correct Option If a horizontal force of magnitude 2mg is applied to the
plank B, then acceleration of A relative to the plank and
1. A 4 kg block A is placed on the top of 8 kg block B which
relative to the ground respectively, are
rests on a smooth table.
A
A
B 2mg
B F
g 2g
(a) 0, (b) 0,
A just slips on B when a force of 12 N is applied on A. 2 3
3g g 2g g
Then, the maximum horizontal force F applied on B to (c) , (d) ,
make both A and B move together, is 5 5 5 5
(a) 12 N (b) 24 N 6. A pivoted beam of negligible mass has a mass suspended
(c) 36 N (d) 48 N
from one end and an at wood’s machine suspended from
2. Find the value of friction forces between the blocks A and the other. The frictionless pulley has negligible mass and
B and between B and ground. (Take g = 10 m/ s 2 ) dimension.
µ = 0.1 l1 l2
A 5 kg
F = 80 N
B 15 kg µ = 0.6 M1
Ground
(a) 90 N, 5 N (b) 5 N, 90 N
(c) 5 N, 75 N (d) 0 N, 80 N

3. A block of mass m is placed on another block of mass M M2 M3


which itself is lying on a horizontal surface. The
coefficient of friction between two blocks is µ 1 and that Gravity is directed downwards and M 2 = 3M 3, l2 = 3l1 .
between the block of mass M and horizontal surface is M
µ 2 . What maximum horizontal force can be applied to the Find the ratio 1 which will ensure that the beam has no
M2
lower block so that the two blocks move without
tendency to rotate just after the masses are released.
separation? M1 M1
m
(a) =2 (b) =3
M2 M2
M F M1
(c) =4 (d) None of these
M2
(a) (M + m)(µ 2 − µ 1 )g (b) (M − m)(µ 2 + µ 1 )g
(c) (M − m)(µ 2 + µ 1 )g (d) (M + m)(µ 2 + µ 1 )g 7. If coefficient of friction between all surfaces is 0.4, then
find the minimum force F to have equilibrium of the
4. A block of mass m is placed on the top of another block of system. (Take g = 10 m/ s 2 )
mass M as shown in the figure . The coefficient of friction
between them is µ.

a
M

25 kg 15 kg F
Wall
The maximum acceleration with which the block M may
move so that m also moves along with it is
M
(a) µg (b) µ g (a) 62.5 N (b) 150 N
m (c) 135 N (d) 50 N
m g
(c) µ g (d)
M µ 8. In the arrangement shown in figure, there is a friction
force between the blocks of masses m and 2m. The mass
5. Block A of mass m rests on the plank B of mass 3m which of the suspended block is m. The block of mass m is
is free to slide on a frictionless horizontal surface. The stationary with respect to block of mass 2m. The
coefficient of friction between the block and plank is 0.2.
192 Objective Physics Vol. 1

minimum value of coefficient of friction between m and 14. A pendulum of mass m hangs from a support fixed to a
2m is trolley. The direction of the string (i.e. angle θ) when
the trolley rolls up a plane of inclination α with
acceleration a is

a
m
m
θ
2m
Smooth
1 1 1 1
(a) (b) (c) (d)
2 2 4 3
α
9. A block of mass 5 kg is kept on a horizontal floor having
coefficient of friction 0.09. Two mutually perpendicular (a) 0 (b) tan −1 α
horizontal forces of 3 N and 4 N act on this block. The  a + g sin α   a
(c) tan −1   (d) tan −1  
acceleration of the block is (g = 10 m/ s 2 )  g cosα   g
(a) zero (b) 0.1 m / s2 (c) 0.2 m / s2 (d) 0.3 m / s2
15. Two masses m and M are attached with strings as shown.
10. Block B moves to the right with a constant velocity v 0 . For the system to be in equilibrium, we have
The velocity of body A relative to B is
v0 M θ

A B
45º 45º
v v
(a) 0 , towards left (b) 0 , towards right
2 2
3v0 3v0
(c) , towards left (d) , towards right
2 2 m

11. A mass of 3 kg descending vertically downwards 2M 2m


(a) tanθ = 1 + (b) tanθ = 1 +
supports a mass of 2 kg by means of a light string passing m M
M m
over a pulley. At the end of 5 s the string breaks . How (c) tanθ = 1 + (d) tanθ = 1 +
2m 2M
much high from now the 2 kg mass will go? (g = 10 m/ s 2 )
(a) 4.9 m (b) 9.8 m 16. Two blocks of mass m = 5 kg and M = 10 kg are
(c) 19.6 m (d) 2.45 m connected by a string passing over a pulley B as shown.
12. The rear side of a truck is open and a box of mass 20 kg is Another string connects the centre of pulley B to the floor
placed on the truck 4 m away from the open end. µ = 015 . and passes over another pulley A as shown. An upward
and g = 10 m/ s . The truck starts from rest with an
2 force F is applied at the centre of pulley A. Both the
pulleys are massless. The accelerations of blocks m and
acceleration of 2 m/ s 2 on a straight road. The box will fall
M, if F is 300 N are
off the truck when it is at a distance from the starting
(Take g = 10 m/ s 2 )
point equal to
F
(a) 4 m (b) 8 m
(c) 16 m (d) 32 m
A
13. In the figure, pulleys are smooth and
strings are massless, m1 = 1 kg and
1
m2 = kg. To keep m3 at rest, mass m3
3 B
should be
(a) 1 kg
2 m3
(b) kg m M
3
1
(c) kg (a) 5 m / s2, zero (b) zero, 5 m / s2
4 m1
(d) 2 kg m2 (c) zero, zero (d) 5 m / s2, 5 m / s2
Laws of Motion 193

17. A body weighs 8 g when placed in one pan and 18 g when 21. Two blocks A and B each of mass m are placed on a
placed on the other pan of a false balance. If the beam is smooth horizontal surface. Two horizontal force F and
horizontal when both the pans are empty, the true weight 2F are applied on the blocks A and B respectively as
of the body is shown in figure. The block A does not slide on block B.
(a) 13 g (b) 12 g A B
(c) 15.5 g (d) 15 g

18. A man of mass m has fallen into a ditch of width d. Two F m m 2F


of his friends are slowly pulling him out using a light rope 30º
and two fixed pulleys as shown in figure. Both the friends
exert force of equal magnitudes F. Then, the normal reaction acting between the two blocks is
F F
(a) F (b) (c) (d) 3 F
2 3

22. Two beads A and B move along a C


semicircular wire frame as shown B
in figure . The beads are connected
by an inelastic string which always O(Centre)
d remains tight. At an instant the
speed of A is u, ∠BAC = 45° and A
u
∠BOC = 75°, where O is the centre
When the man is at a depth h, the value of F is D
of the semicircular arc. The speed
mg
(a) d 2 + 4 h2 (b) hmg of bead B at that instant is
4h
mg 2 u 2
(c) dmg (d) h + d2 (a) 2u (b) u (c) (d) u
2h 2 2 3

19. A man of mass m stands on a platform of equal mass m 23. In the figure shown, a person wants to raise a block lying
and pulls himself by two ropes passing over pulleys as on the ground to a height h. In both the cases, if time
shown in figure. If he pulls each rope with a force equal required is the same then in which case he has to exert
to half his weight, his upward acceleration would be more force. Assume pulleys and strings are light

(i) (ii)

(a) (i) (b) (ii)


(c) same in both (d) Cannot be determined

(a)
g
(b)
g 24. Two blocks of masses m and 2m are placed one over the
2 4 other as shown in figure.The coefficient of friction
µ
(c) g (d) zero between m and 2m is µ and between 2m and ground is .
3
20. If the coefficient of friction between A and B is µ, the If a horizontal force F is applied on upper block and T is
maximum acceleration of the wedge A for which B will tension developed in string, then choose the incorrect
remain at rest with respect to the wedge is alternative.

B
A
m

2m
a
45º
µ
(a) If F = mg , T = 0 (b) If F = µmg , T = 0
 1 + µ 1 − µ g 3
(a) µg (b) g   (c) g   (d) µmg
1 − µ  1 + µ µ (c) If F = 2µmg , T = (d) If F = 3µmg , T = 0
3
194 Objective Physics Vol. 1

25. Two blocks of masses 2 m and m are in 30. A sphere of mass m is held between two smooth inclined
equilibrium as shown in the figure . 3
walls. For sin 37° = , the normal reaction of the wall (2)
Now, the string between the blocks is 5
suddenly broken. The accelerations of is equal to
the blocks A and B respectively at that 2m A
instant are m B (2)
g g g g
(a) g and g (b) g and (c) and g (d) and
2 2 2 2 (1)

26. A block of mass m is placed on a wedge of mass 2m 37º


which rests on a rough horizontal surface. There is no 37º
friction between the block and the wedge. The minimum
coefficient of friction between the wedge and the ground 16 mg 25 mg 39 mg Mg sinθ
(a) (b) (c) (d)
so that the wedge does not move is 25 21 21 2
31. A block of mass m is kept on an inclined plane of a lift
moving down with acceleration of 2 m/s 2 . What should
m be the coefficient of friction to let the block move down
2m
with constant velocity relative to lift.

45º
m
(a) 0.1 (b) 0.20 (c) 0.3 (d) 0.4 2 m/s2

27. Two blocks of mass 5 kg and 3 kg are attached to the ends


of a string passing over a smooth pulley fixed to the 30º
ceiling of an elevator. A man inside the elevator
accelerated upwards, finds the acceleration of the blocks 1 3
(a) µ = (b) µ = 0.4 (c) µ = 0.8 (d) µ =
9 3 2
to be g. The acceleration of the elevator is
32
g g g g More than One Correct Options
(a) (b) (c) (d) 1. Two blocks each of mass 1 kg
3 4 8 6
are placed as shown. They are F m2
28. The line of action of the resultant of two like parallel connected by a string which m1
forces shifts by one-fourth of the distance between the passes over a smooth
forces when the two forces are interchanged. The ratio of (massless) pulley. There is no
the two forces is friction between m1 and the ground. The coefficient of
(a) 1 : 2 (b) 2 : 3 (c) 3 : 4 (d) 3 : 5 friction between m1 and m2 is 0.2. A force F is applied to
29. Two masses A and B of 10 kg and 5 kg respectively are m2 . Which of the following statement is/are correct?
connected with a string passing over a frictionless pulley (a) The system will be in equilibrium if F ≤ 4 N
fixed at the corner of a table as shown in figure. The (b) If F > 4 N, tension in the string will be 4 N
coefficient of friction of A with the table is 0.2. The (c) If F > 4 N, the frictional force between the blocks will be
minimum mass of C that may be placed on A to prevent it 2N
from moving is equal to (d) If F = 6 N, tension in the string will be 3 N
O
C
2. Two particles A and B, each
of mass m are kept stationary
A by applying a horizontal force α
F = mg on particle B as T1

B
shown in figure. Then,
A
(a) tan β = 2 tan α
(b) 2T1 = 5T2 β T2
B
(c) 2 T1 = 5T2
(a) 15 kg (b) 10 kg (c) 5 kg (d) 20 kg F = mg
(d) None of the above
Laws of Motion 195

(a) the value of F to keepC moving with constant speed is 80 N


3. The velocity-time graph of the figure shows the motion
(b) the value of F to keepC moving with constant speed is 120 N
of a wooden block of mass 1 kg which is given an initial
(c) if F is 200 N, then acceleration of B is 10 ms−2
push at t = 0 along a horizontal table.
(d) to slide C towards left, F should be at least 50 N (Take
g = 10 ms−2)
4
7. A man pulls a block of mass equal to himself with a light
v (ms– 1)

string. The coefficient of friction between the man and


the floor is greater than that between the block and the
floor
(a) if the block does not move, then the man also does not move
4 t (s)
(b) the block can move even when the man is stationary
(a) The coefficient of friction between the block and the table is (c) if both move, then the acceleration of the block is greater
0.1 than the acceleration of man
(b) The coefficient of friction between the block and the table is (d) if both move, then the acceleration of man is greater than the
0.2 acceleration of block
(c) If the table was half of its present roughness, the time taken 8. A block of mass 1 kg is at rest relative to a smooth wedge
by the block to complete the journey is 4 s
(d) If the table was half of its present roughness, the time taken
moving leftwards with constant acceleration a = 5 ms −2 .
by the block to complete the journey is 8 s Let N be the normal reaction between the block and the
wedge. Then, (g = 10 ms −2 )
4. As shown in the figure, A is a man of mass 60 kg standing
on a block of mass 40 kg kept on ground. The coefficient
1 kg
of friction between the feet of the man and the block is
0.3 and that between B and the ground is 0.2. If the
person pulls the string with 125 N force, then
a
A
θ

(a) N = 5 5 N (b) N =15 N


B 1
(c) tanθ = (d) tanθ = 2
2

(a) B will slide on ground 9. For the given situation shown in figure, choose the
(b) A and B will move with acceleration 0.5 ms−2 correct options (g = 10 ms −2 )
(c) the force of friction acting between A and B will be 40 N
(d) the force of friction acting between A and B will be 180 N µs = 0.4 2 kg
µk = 0.2
5. In the figure shown, A and B are free to move. All the 4kg F = 2t
surfaces are smooth. Mass of A is m. Then,
µs = 0.6
A
µk = 0.4
B

θ (a) At t = 1s, force of friction between 2 kg and 4 kg is 2 N


(b) At t = 1s, force of friction between 2 kg and 4 kg is zero
(a) the acceleration of A will be more than g sin θ (c) At t = 4 s, force of friction between 4 kg and ground is 8 N
(b) the acceleration of A will be less than g sinθ (d) At t = 15 s, acceleration of 2 kg is 1 ms−2
(c) normal reaction on A due to B will be more than mg cos θ 10. In the figure shown, all the strings
(d) normal reaction on A due to B will be less than mg cos θ
are massless and friction is absent
6. M A = 3 kg, M B = 4 kg, and A everywhere. Choose the correct
options.
M C = 8 kg. µ between any two B T2
F (a) T1 > T3 1 kg
surfaces is 0.25. Pulley is C (b) T3 > T1 2 kg
frictionless and string is
(c) T2 > T1 T3 T1
massless. A is connected to wall through a massless rigid
(d) T2 > T3 2 kg 1 kg
rod.
196 Objective Physics Vol. 1

11. Force acting on a block versus time graph is as shown in 15. Mass m1 moves on a slope making an angle θ with the
figure. Choose the correct options. ( Take g = 10 ms −2 ) horizontal and is attached to mass m2 by a string passing
F (N) over a frictionless pulley as shown in figure. The
coefficient of friction between m1 and the sloping surface
10 is µ. Which of the following statements are true?
2 kg F

µ = 0.3
m1
10 t(s) m2
(a) At t = 2 s, force of friction is 2 N B
(b) At t = 8 s, force of friction is 6 N
θ
(c) At t = 10 s, acceleration of block is 2 ms−2
(d) At t = 12 s, velocity of block is 8 ms−1 (a) If m2 > m1 sin θ ,the body will move up the plane
(b) If m2 > m1 (sin θ + µ cosθ), the body will move up the plane
12. For the situation shown in figure, mark the correct (c) If m2 < m1 (sin θ + µ cosθ), the body will move up the plane
options. (d) If m2 < m1 (sin θ − µ cosθ), the body will move down the
µ = 0.4 plane
2 kg 16. In figure, a body A of mass m slides on plane inclined at
4 kg F = 2t angle θ1 to the horizontal and µ is the coefficient of
friction between A and the plane. A is connected by a
Smooth light string passing over a frictionless pulley to another
(a) At t = 3 s, pseudo force on 4 kg when applied from 2 kg is body B, also of mass m, sliding on a frictionless plane
4 N in forward direction inclined at an angle θ 2 to the horizontal. Which of the
(b) At t = 3 s, pseudo force on 2 kg when applied from 4 kg is following statements are true?
2 N in backward direction
(c) Pseudo force does not make an equal and opposite pairs
A
(d) Pseudo force also makes a pair of equal and opposite forces
B
13. For the situation shown in figure, mark the correct m m
options. θ1 θ2
F
θ (a) A will never move up the plane
M (b) A will just start moving up the plane when
sin θ 2 − sin θ 1
µ µ=
cosθ 1
(a) Angle of friction is tan −1 (µ ) (c) For A to move up the plane, θ 2 must always be greater than
(b) Angle of repose is tan −1 (µ ) θ1
(c) Atθ = tan −1 (µ ), minimum force will be required to move the (d) B will always slide down with constant speed
block
µMg Comprehension Based Questions
(d) Minimum force required to move the block is
1 + µ2 Passage I (Q.1 to 5)
A man wants to slide down a block of mass m which is
14. In figure, the coefficient of friction between the floor and kept on a fixed inclined plane of inclination 30° as shown
the body B is 0.1. The coefficient of friction between the in the figure. Initially, the block is not sliding.
bodies B and A is 0.2. A force F is applied as shown on B.
m
The mass of A is m/ 2 and of B is m. Which of the
following statements are true?
A
30°
B F
To just start sliding, the man pushes the block down the
(a) The bodies will move together if F = 0.25 mg incline with a force F. Now, the block starts accelerating.
(b) The body A will slip with respect to B if F = 0.5 mg To move it downwards with constant speed, the man
(c) The bodies will be at rest if F = 0.1mg starts pulling the block with same force. Surfaces are
(d) The maximum value of F for which the two bodies will such that ratio of maximum static friction to kinetic
move together is 0.45 mg friction is 2. Now, answer the following questions.
Laws of Motion 197

1. What is the value of F? Assertion and Reason


mg mg mg 3 mg Directions (Q. Nos. 1-20) These questions consists of two
(a) (b) (c) (d)
4 6 4 2 3 statements each printed as Assertion and Reason. While
2. What is the value of µ s , the coefficient of static friction? answering these questions you are required to choose any one
4 2 3 1 of the following five responses.
(a) (b) (c) (d) (a) If both Assertion and Reason are correct and Reason is
3 3 3 3 3 3 2 3
the correct explanation of Assertion
3. If the man continues pushing the block by force F, its (b) If both Assertion and Reason are true but Reason is
acceleration would be not the correct explanation of Assertion
g g g g (c) If Assertion is true but Reason is false
(a) (b) (c) (d) (d) If Assertion is false but Reason is true
6 4 2 3
(e) If both Assertion and Reason are false
4. If the man wants to move the block up the incline, what
minimum force is required to start the motion? 1. Assertion A block of weight 10 N is pushed against a
2 mg 7mg 5mg vertical wall by a force of 15 N. The coefficient of
(a) mg (b) (c) (d)
3 2 6 6 friction between the wall and the block is 0.6. Then, the
magnitude of maximum frictional force is 9 N.
5. What minimum force is required to move it up the incline
Reason For given system block will remain stationary.
with constant speed?
2 mg 7mg 5mg 2. Assertion The weighing machine measures the weight
(a) mg (b) (c) (d)
3 2 6 6 of a body.
Passage II (Q.6 to 7) Reason Weightlessness means the absence of weight.
A lift with a mass 1200 kg is raised from rest by a cable
3. Assertion When a person walks on a rough surface, the
with a tension 1350 g-N. After some time, the tension
net force exerted by surface on the person in the direction
drops to 1000 g-N and the lift comes to rest at a height of of his motion.
25 m above its initial point. (1 g-N = 9.8 N)
Reason It is the force exerted by the road on the person
6. What is the height at which the tension changes? that causes the motion.
(a) 10.8 m (b) 12.5 m (c) 14.3 m (d) 16 m
4. Assertion A body of mass 10 kg is placed on a rough
7. What is greatest speed of lift? inclined surface ( µ = 0.7). The surface is inclined to
(a) 9.8 ms−1 (b) 7.5 ms−1 horizontal at angle 30°. Acceleration of the body down
(c) 5.92 ms−1 (d) None of the above the plane will be zero.
Passage III (Q.8 to 9) Reason Work done by friction is always negative.
Blocks A and B shown in the figure are connected with a 5. Assertion In the system of µ2 m F
bar of negligible weight. A and B each has mass 170 kg, two blocks of equal masses
the coefficient of friction between A and the plane is 0.2 as shown, the coefficient of m µ1
and that between B and the plane is 0.4 (g = 10 ms −2 ) friction between the blocks
B (µ 2 ) is less than coefficient
of friction (µ 1 ) between lower block and ground.
For all values of force F applied on upper block lower
A
block remains at rest.
8m
Reason Frictional force on lower block due to upper
block is not sufficient to overcome the frictional force on
θ
lower block due to ground.
15 m
6. Assertion During free fall of a person, one feels
8. What is the total force of friction between the blocks and weightlessness because his weight becomes zero.
the plane?
Reason He falls with an acceleration of g.
(a) 900 N (b) 700 N (c) 600 N (d) 300 N
7. Assertion Earth is an inertial frame.
9. What is the force acting on the connecting bar?
(a) 140 N (b) 100 N Reason A frame in motion is sometimes an inertial
(c) 75 N (d) 125 N frame and sometimes a non-inertial.
198 Objective Physics Vol. 1

8. Assertion Friction opposes motion of a body. 16. Assertion Two forces are acting on a rope lying on a
smooth table as shown in figure. In moving from A to B,
Reason Static friction is self adjusting while kinetic
tension on string decreases from 2F to F.
friction is constant.
B A
9. Assertion Friction always opposes the relative motion F 2F
of two bodies.
Reason Without friction also, one can move on a smooth Reason Situation will become in determinant, if we take
surface. it a massless string.

10. Assertion Static friction acting on a body is always 17. Assertion A massless rod AB is suspended with the help
greater than the kinetic friction acting on this body. of two strings as shown in figure . Tension on these two
strings are T1 and T2 . A force F is applied at distance x
Reason Coefficient of static friction is more than the from end B . If x is decreased, then T1 will decrease and T2
coefficient of kinetic friction. will increase.
11. Assertion A particle is thrown vertically upwards. If air
resistance is taken into consideration, then retardation in T1 T2
upward journey is more than the acceleration in
downward journey. A B
F
Reason Same mechanical energy is lost in the form of x
heat due to air friction.
Reason When x = 0, T1 = 0 and T2 = F .
12. Assertion A string has a mass m. If it is accelerated
tension is non-uniform and if it is not accelerated tension 18. Assertion In the diagram shown in figure, string is
is uniform. massless and pulley is smooth, Net force on 1 kg block is
10
Reason Tension force is an electromagnetic force. N.
3
13. Assertion If two equal and opposite forces act on a
body, then it remains in equilibrium.
Reason Linear acceleration of body should be zero
under the above condition.
14. Assertion In the figure shown,
block of mass m is stationary with
2 kg
respect to lift . Force of friction
acting on the block is greater than m
a
mg sin θ. 1 kg

Reason If lift moves with Reason Net force on both the blocks will be same.
θ
constant velocity, then force of
friction is equal to mg sin θ. 19. Assertion Block A is resting on one corner of a box as
shown in figure. Acceleration of box is ( 2i$ + 2$j ) m/s 2 .
15. Assertion In the shown figure tension T connected to the
Let N 1 is the normal reaction on block from vertical wall
ceiling is 20 N < T < 40 N N 1
and N 2 from ground of box. Then 1 = .
N2 0
T
(Neglect friction)

2 kg
y
A
x
1 kg

Reason System is not stationary. Reason N 1 = 0, if lift is stationary.


Laws of Motion 199

20. Assertion If vector sum of two or more than two forces 4. A block of mass m is thrown upwards with some initial
is zero. Then, net moment of all the forces about any velocity as shown in figure. On the block
point is constant.
Reason Two equal and opposite forces make a couple.
Moment of this couple = (force × distance between the m
forces).

Match the Columns


θ
1. In the diagram shown in figure, match the following
columns (g = 10 m/ s 2 ).
20√2 N Column I Column II

(A) Net force in horizontal (p) Zero


45º direction

(B) Net force in vertical (q) m( g sinθ + µg cos θ)


4 kg
direction

(C) Net force along the plane (r) m( g sinθ cos θ + µg cos 2 θ)
µs = 0.8, µk = 0.6
(D) Net force perpendicular to (s) m( g sin2 θ + µg sinθ cos θ)
plane
Column I Column II

(A) Normal reaction (p) 12 SI unit 5. In the diagram shown in figure, match the following
(B) Force of friction (q) 20 SI unit columns. (g = 10 m/s 2 )
(C) Acceleration of block (r) zero F2 = 18 N
1 kg
(s) 2 SI unit
2 kg
2. In the diagram shown in figure, all pulleys are smooth 3 kg Smooth
and massless and strings are light. Match the following
columns.
F = 80 N F1 = 60 N θ = 30º

Column I Column II

(A) Acceleration of 2 kg block (p) 8 SI unit


3kg
1 kg
(B) Net force on 3 kg block (q) 25 SI unit
2 kg 4 kg
(C) Normal reaction between (r) 2 SI unit
2 kg and 1 kg
Column I Column II
(D) Normal reaction between (s) 45 N
(A) 1 kg block (p) will remain stationary
3 kg and 2 kg
(B) 2 kg block (q) will move down
(C) 3 kg block (r) will move up 6. Velocity of three particles A , B and C varies with time t
(D) 4 kg block (s) 5 m/s 2 as v A = ( 2t i$ + 6$j ) m/ s , vB = ( 3i$ + 4$j ) m/ s and
v = ( 6$i − 4t $j ). Regarding the pseudo force match the
C
3. Match the following columns.
following columns.
Column I Column II Column I Column II
(A) Force of friction (p) Opposes motion
(A) On A as observed by B (p) Along positive x-direction
(B) Normal reaction on a (q) Opposes relative motion
block kept on horizontal (B) On B as observed by C (q) Along negative x-direction
ground
(C) On A as observed by C (r) Along positive y-direction
(r) Is always mg
(s) May be equal to mg (D) On C as observed by A (s) Along negative y-direction
200 Objective Physics Vol. 1

7. In the diagram shown in figure, match the following 4. To determine the coefficient of friction between a rough
columns. surface and a block, the surface is kept inclined at 45° and
µ = 0.4 the block is released from rest. The block takes a time t in
10 m/s
1 kg moving a distance d. The rough surface is then replaced
by a smooth surface and the same experiment is repeated.
20 m/s The block now takes a time t / 2in moving down the same
2 kg
distance d. The coefficient of friction is [WB JEE]
(a) 3/4 (b) 5/4 (c) 1/2 (d) 1/ 3
µ = 0.6
5. A constant retarding force of 80 N is applied to a body of
Column I Column II
mass 50 kg which is moving initially with a speed of
(A) Absolute acceleration of (p) 11 m/s 2 20 m/s. What would be the time required by the body to
1 kg block
come to rest? [J&K CET]
(B) Absolute acceleration of (q) 6 m/s 2 (a) 15 s (b) 14 s (c) 12.5 s (d) 18 s
2 kg block
(C) Relative acceleration (r) 17 m/s 2 6. An object is gently placed on a long converges belt
between the two moving with 11ms −1 .If the coefficient of friction is 0.4,
(s) None of the above then the block will slide in the belt up to a distance of
[J&K CET]
(a) 10.21 m (b) 15.125 m (c) 20.3m (d) 25.6 m
Entrance Gallery
2014 2012
7. A small block of mass of 0.1 Q
1. A particle of mass m is at rest at the origin at time t = 0. It
kg lies on a fixed inclined 1N
is subjected to a force, F ( t ) = F0 e − bt in the x-direction. plane PQ which makes an
Its speed v( t ) is depicted by which of the following angle θ with the horizontal. A
curves? [JEE Main] horizontal force of 1 N acts on θ
F0 F0 the block through its centre of O P
mb mb mass as shown in the figure. The block remains stationary
if (Take, g = 10m/s 2 ) [IIT JEE]
(a) (b) (a) θ = 45°
v(t) v(t) (b) θ > 45 ° and a frictional force acts on the block towards P
(c) θ > 45 ° and a frictional force acts on the block towards Q
t t
(d) θ < 45 ° and a frictional force acts on the block towards Q
8. A body of mass m is travelling with a velocity u. When a
F0 F0
constant retarding force F is applied, it comes to rest after
mb mb
travelling a distance s1 . If the initial velocity is 2u with the
same force F, the distance travelled before it comes to
(c) (d)
v(t) v(t) rest is s2 . Then, [Karnataka CET]
s1
(a) s2 = 4 s1 (b) s2 = 2s1 (c) s2 = (d) s2 = s1
2
t t
9. A block kept on a rough surface starts sliding when the
2. A block of mass m is placed on a surface with a vertical inclination of the surface is θ with respect to the
cross-section given by y = x 3 / 6. If the coefficient of horizontal. The coefficient of static friction between the
friction is 0.5, the maximum height above the ground at block and the surface is [Karnataka CET]
which the block can be placed without slipping is (a) sec θ (b) sin θ (c) tan θ (d) cos θ
[JEE Main]
1 2 1 1 10. A box of mass 2 kg is placed on the roof of a car. The box
(a) m (b) m (c) m (d) m would remain stationary until the car attains a maximum
6 3 3 2
acceleration. Coefficient of static friction between the
3. A box is lying on an inclined plane what is the coefficient box and the roof of the car is 0.2 and g = 10 ms −2 .The
of static friction if the box starts sliding when an angle of maximum acceleration of the car, for the box to remain
inclination is 60 ° . [Karnataka CET] stationary, is [WB JEE]
(a) 1.173 (b) 1.732 (c) 2.732 (d) 1.677 (a) 8 m s−2 (b) 6 m s−2 (c) 4 ms−2 (d) 2 m s−2
Laws of Motion 201

11. Three blocks of masses 4 kg, 2 kg and 1 kg, respectively 0.5. Coefficient of friction between A and B is 0.4. A
are in contact on a frictionless table as shown in the horizontal force of 10 N is applied on block B.
figure. If a force of 14 N is applied on the 4 kg block, the A
contact force between the 4 kg and the 2 kg block will be 2 kg
[WB JEE] B
8 kg 10 N
14 N
4 kg 2 kg 1 kg

(a) 2 N (b) 6 N The force of friction between A and B is [Karnataka CET]


(c) 8 N (d) 14 N (a) zero (b) 50 N (c) 40 N (d) 100 N

2011 2010
12. Two masses m1 = 1 kg and m2 = 2 kg are connected by a 16. A block of mass m is on an inclined plane of angle θ.
light inextensible string and suspended by means of a The coefficient of friction between the block and the
weightless pulley as shown in the figure. plane is µ and tan θ > µ . The block is held stationary by
applying a force P parallel to the plane. The direction of
force pointing up the plane is taken to be positive. As P
is varied from P1 = mg (sin θ − µ cos θ ) to
P2 = mg (sin θ + µ cos θ ), the frictional force f versus P
graph will look like [IIT JEE]

1kg m1
P
m2 2 kg θ

f f
Assuming that both the masses start from rest,
P2
the distance travelled by the centre of mass in 2 s is (a) (b)
P1 P P1 P2 P
(take, g = 10m/s 2 ) [Kerala CEE]
20 40
(a) m (b) m
9 9 f f
2 1
(c) m (d) m P1 P2
3 3 P1
(c) (d)
(e) 4 m P2 P P

13. A block at rest slides down a smooth inclined plane


which makes an angle 60 ° with the vertical and it reaches
17. Two fixed frictionless inclined plane making an angle
the ground in t 1 second. Another block is dropped
30° and 60° with the vertical are shown in the figure.
vertically from the same point and reaches the ground in
Two blocks A and B are placed on the two planes.
t 2 second. Then, the ratio of t 1 : t 2 is [Kerala CEE]
What is the relative vertical acceleration of A with
(a) 1 : 2 (b) 2 : 1
respect to B? [AIEEE]
(c) 1 : 3 (d) 1 : 2
(e) 3 : 1 A
14. The resultant of two forces acting at an angle of 120° is
10 kg-wt and is perpendicular to one of the forces. That B
force is [Karnataka CET]
10
(a) kg-wt (b) 10 kg-wt
3 60° 30°
(c) 20 3 kg-wt (d) 10 3 kg-wt –2
(a) 4.9 ms in horizontal direction
15. Block A of mass 2 kg is placed over block B of mass 8 kg. (b) 9.8 ms–2 in vertical direction
The combination is placed over a rough horizontal (c) zero
surface. Coefficient of friction between B and the floor is (d) 4.9 ms–2 in vertical direction
Answers
Level 1
Objective Problems
1. (b) 2. (c) 3. (c) 4. (a) 5. (b) 6. (d) 7. (a) 8. (b) 9. (b,c,d) 10. (d)
11. (c) 12. (a) 13. (c) 14. (a) 15. (d) 16. (d) 17. (d) 18. (b) 19. (d) 20. (b)
21. (c) 22. (c) 23. (c) 24. (c) 25. (d) 26. (d) 27. (b) 28. (b) 29. (a) 30. (c)
31. (a) 32. (c) 33. (d) 34. (b) 35. (c) 36. (a) 37. (c) 38. (c) 39. (d) 40. (c)
41. (a) 42. (a) 43. (b) 44. (a) 45. (d) 46. (c) 47. (d) 48. (c) 49. (a) 50. (d)
51. (b) 52. (b) 53. (d) 54. (a) 55. (a) 56. (a) 57. (c) 58. (a) 59. (a) 60. (d)
61. (b) 62. (c) 63. (a) 64. (a) 65. (b) 66. (d) 67. (c) 68. (a) 69. (a) 70. (a)
71. (c) 72. (b) 73. (c) 74. (b) 75. (d) 76. (d) 77. (c) 78. (a) 79. (b) 80. (d)
81. (d) 82. (c) 83. (a) 84. (c)

Level 2
Only One Correct Option
1. (b) 2. (d) 3. (d) 4. (a) 5. (d) 6. (b) 7. (a) 8. (c) 9. (b) 10. (b)
11. (a) 12. (c) 13. (a) 14. (c) 15. (a) 16. (a) 17. (b) 18. (a) 19. (d) 20. (b)
21. (d) 22. (a) 23. (a) 24. (c) 25. (b) 26. (b) 27. (c) 28. (d) 29. (a) 30. (d)
31. (a)

More than One Correct Options


1. (a,c,d) 2. (a,c) 3. (a,d) 4. (a,b) 5. (a,d) 6. (a,c) 7. (a,b,c) 8. (a,c) 9. (b,c) 10. (b,c,d)
11. (all) 12. (b,c) 13. (all) 14. (all) 15. (b,d) 16. (b,c)

Comprehension Based Questions


1. (b) 2. (a) 3. (d) 4. (c) 5. (d) 6. (c) 7. (c) 8. (a) 9. (a)

Assertion and Reason


1. (c) 2. (d) 3. (a,b) 4. (c) 5. (a) 6. (d) 7. (d) 8. (d) 9. (d) 10. (d)
11. (b) 12. (d) 13. (d) 14. (b) 15. (b) 16. (b) 17. (b) 18. (c) 19. (b) 20. (b)

Match the Columns


1. (A → q, B → p, C → s) 2. (A → rt, B → p, C → q, D → qs) 3. (A → t, B → s)
4. (A → r, B → s, C → q, D → p) 5. (A → r, B → t, C → q, D → t) 6. (A → t, B → r, C → r, D → q)
7. (A → s, B → p, C → s)

Entrance Gallery
1. (b) 2. (a) 3. (b) 4. (a) 5. (c) 6. (b) 7. (c) 8. (a) 9. (c) 10. (d)
11. (d) 12. (a) 13. (b) 14. (a) 15. (b) 16. (c) 17. (d)
Solutions
Level 1 : Objective Problems  F − (m1 + m2 + m3 )g sin θ 
or N = m3 g sin θ + m3  
M
1. Tx = (mass of rope of length L - x) × g = ( L − x )g  (m1 + m2 + m3 ) 
L m3 F
=
2. Equilibrium ofm: T = mg … (i) m1 + m2 + m3
Equilibrium of 2 m: 2T cosθ = 2 mg … (ii) 10. T = mg
Solving these two equations, we get θ = 45° T
3. Net moment about point C should be zero. Or
4x = 2( 3 − x )
mg
∴ x =1 m
4. Three coins are above the 7th coin. Therefore, force from T
3 coins above it will be
3mg = 3(10 × 10−3 )(10) = 0.3 N (in downward direction) Force on the pulley (other than from clamp)

5. 2T cos 60° = w Fnet = (T + mg )2 + T 2

= g ( M + m)2 + M 2
T T
60º Since, pulley is in equilibrium, clamp will exert the same
60º
amount of force in opposite direction. Or pulley will also
exert this much force on clamp.
T 3T2
11. 1 =
w 2 2
3T1/2 T2/2
or w =T
G
or w max = Tmax = 20N T1/2 3T2/2
A B
6. T3 sin 60° = T2 sin 30° w
T T1 = 3T2
∴ T3 = 2 or …(i)
3
3T1 T
T2 × AG = 2 × BG
2 2
AG T
30º
T3 ∴ = 2
60º BG 3T1
T2 1
But = from Eq. (i)
T1 3
AG 1
Hence, =
T1 = 100 N BG 3
Now, T2 cos 30° + T3 cos 60° = T1 = 100 12. T sin 30° = w = 40
3T2 T2 1
or + × = 100 30º
12 3 2 T
8T2 30º
or = 100 or T2 = 50 3 N
4 3
7. Force exerted by man on rope transfers to it in the form of
T
tension. ∴ = 40
2
Net upwards force on the system is 2T or 2F.
or T = 80 N
Net downward force is ( 50 + 30) g = 8 g.
13. Apply Lami’s theorem at O
For equilibrium of system, 2F = 80 g or F = 40g T1 T2 10 10
Net pushing force = = = = 10
8. Acceleration of system a = sin150° sin120° sin 90° 1
Total mass 1
∴ T1 = 10 sin150° = 10 × = 5N
F − (m1 + m2 + m3 )g sin θ 2
or a=
(m1 + m2 + m3 ) T2 = 10 sin 120°
Equation of motion for m3 3
= 10 × = 5 3N
N − m3 g sinθ = m3 a 2
204 Objective Physics Vol. 1

14. If lift is accelerating, reading will be more, if it is 20. Acceleration of system,


decelerating reading will be less and if it is moving with Net pulling force
a=
constant velocity reading will be same. Total mass
15. Suppose F is reading of spring balance. Then, 4g − 2g g
= =
F − 2g = 2g or F = 4g 6 3
So, it is 4 kg or 4g-N. Equation of motion of block C is
Note That 1 kg = g-N mC g = TBC = mC a
16. 3 ∴ TBC = mC ( g − a )
Tmax = mg T
= 2  9.8 −
4 9.8 
 = 13 N
 3 
21. Mg − N = ma or N = m ( g − a )
a a 22. Since, M1 g sin 30° = M 2 g
∴ Net pulling force = 0
or acceleration of system = 0
Friction coefficient is not required in this question.
mg – T 23. Maximum friction is 50 N.
mg mg, a =
m Now, F − f max = ma
This is the minimum value of a, because if a is increased or F = f max + ma
3 = 50 + 500 × 1
from this value T will be less than mg. Which is less than
4 = 550N
Tmax . Or we can say for minimum value of a ,T should be 24. 0 = ( 3)2 − 2( a )( 9)
maximum. 1
3 ∴ a = = 0.5 m/s2 (upwards)
mg − mg = ma 2
4
g N = m( g + a)
or a=
4 = 50 ( 9.8 + 0.5) = 515 N
17. T cosθ = mg and T sinθ = ma 25. 0 = (10)2 − 2( a )( 25)
∴ a = 2 m/s2 (upwards)
θ Now, T − 800g = 800a
θ T ∴ T = 800(10 + 2) = 9600 N
26. Lift is descending with retardation.Therefore, acceleration
a is upwards or pseudo force ma is upwards. Relative to lift
a
force in downward direction is m ( g + a ) in place of mg.
Therefore in smooth plane acceleration will be ( g + a )sin θ
mg not g sinθ.
From these two equations, we get 27. Upward force on 2 kg block in upward direction will be 40 N
g ( = 2F ) in the form of tension.
a = g tan θ = g tan 30° =
3 40 N
Net pulling force 5g − 3g g
18. a = = =
Total force 8 4
Equation of motion of 5 kg block gives 2 kg a
5g
5g − T = 5a =
4
15g
∴ T= 20 N
4
= 36.75 N 40 − 20
∴ a= = 10 m/s2
19. Acceleration of system, 2
T3 T − mA g = mA a
a= 28.
m1 + m2 + m3
∴ T = mA ( g + a )
40
= = 6 (10 + 1) = 66 N
10 + 6 + 4
29. Writing equation of motion for two weights,
= 2 m/s2 w
w1 − T = 1 ( ar − a ) …(i)
Equation of motion of m3 is g
T3 − T2 = m3 a w2
T − w2 = ( ar + a ) …(ii)
∴ T2 = T3 − m3 a = 40 − 4 × 2 = 32 N g
Laws of Motion 205

Solving Eqs. (i) and (ii), we get Net pulling force 5g − 0.5 × 5 × g g
35. a = = =
a=g Total mass 10 4
Equation of motion for the hanging mass will be
5g
5g − T = 5a =
4
15 15 × 98
.
∴ T= g= = 36.75 N
4 4
ar w ar + a 36. mg sinθ − f = ma (f = force of friction)
1 ar – a
or f = m = ( g sin θ − a )
= 8 10 × − 0.4 = 36.8 N
w2 ar 1
 2 
Acceleration relative to pulley Acceleration relative to v 4
ground 37. a = = = 2 m/s2
t 2
T=
4w1w 2
with a = g F − f = ma
w1 + w 2 or 200 − µ × 30 × 10 = 30 × 2
Net pulling force ∴ µ = 0.47
30. Acceleration of system, a =
Total mass 1 2 2s 1
38. s = at or t = or t ∝
Mg sinθ 2 a a
=
2M t1 a2 g sin θ 1
1 = or 2 = =
a = g sinθ t2 a1 g sin θ − µg cosθ 1−µ
2
as sin 45° = cos 45°
Now, the block on ground is moving due to tension
1
Mg sinθ ∴ 1−µ =
Hence, T = Ma = 4
2
∴ µ = 0.75
31. F = ma F
39. Retardation, a = = 5 m/s2
Resultant of three forces F1 , F2 and F3 will be ( 2 − 1)F. m
Therefore, acceleration of body is also ( 2 − 1)a. 0 = 30 − 5t
32. Suppose air resistance is F (upwards), then from equation ∴ t =6s
of motion of balloon, we have F − µmg 100 − 0.5 × 10 × 10
40. a = = = 5 m/s2
w m 10
w − F = mass × acceleration = ⋅ a
g 41. f max = µmg = 0.4 × 2 × 10 = 8N
 a Since, the applied force is less than f max , force of friction
∴ F = w 1 −  will be equal to the applied force or 2.8 N.
 g
42. Maximum inclination of the plane with horizontal = angle
33. Acceleration of system, TAB
of repose = tan −1 (µ ).
Net pulling force
a= 43. N = F = 5N
Total mass
2mg − mg g ∴ f max = µN = 2.5N
= = F
3m 3 w = mg = 0.1 × 9.8 = 0.98 N
Now, from equation of motion of m T T Since, w < f max , force of friction will be 0.98 N.
mg 44. f max = µmg = 0.8 × 4 × 10 = 32 N
T − mg = ma =
3 At t = 2 s, F = kt 2 ( 2)( 2)2 = 8 N
w
4mg
∴ T= Since, applied force f < f max , force of friction will be 8 N.
3
µmg
For equilibrium of pulley 45. Retardation, a = = µg = 10 µ
m
TAB = 2T + weight of pulley
Now, 0 = u2 − 2as
8mg 17mg
= + 3mg =
3 3 or 0 = (62 ) − 2(10µ )( 9)
34. Net pulling force on the system should be zero as velocity is ∴ µ = 0.2
constant. Hence, 3
46. f max = µmg cosθ = 06
. × mg × = 0.52mg
mA g sin 30° = µmA g cos 30° + mB g 2
 µmA 3  mg
mB =  A  − 
m mg sin θ = = 0.5 mg
∴ 
 2   2  2
Since, f max > mg sin θ
1 3
= 10  − 0.2 ×  = 3.3 kg Block will decelerate and come to rest.
 2 2 
206 Objective Physics Vol. 1

47. N = applied force = 12 N 59. v 2 = 2a1 s (a1 = g cosθ)


∴ f max = µN = 7.2 N 0 = v 2 − 2a2 s (a2 = µg cosθ − g sin θ)
Since, weight w < f max From these two equations, we see that a1 = a2
Force of friction f = 5 N or g sin θ = µg cosθ − g sin θ or 2tanθ = µ
∴ Net contact force = N 2 + f 2
= (12)2 + ( 5)2 = 13 N 60. Due to friction ( a = µg ), velocity of block will become equal
48. Acceleration, a = ( g sin θ + µg cosθ) to velocity of belt. Relative motion between two will stop.
g 1 3g ∴ v = at = µgt = 0.2 × 10 × 4 = 8 m/s
= + 0.5 × g × =
2 2 2 2 Tmax − µmg 40 − 0.2 × 8 × 10
61. amax = = = 3 m/s2
µmg m 8
49. Retardation, a = = µg = 4 m/s2
m 62. N 2 = w
0 = (8)2 − 2( 4)( s )
P N1
∴ s = 8.0m
N2 13
50. Angle of plane is just equal to the angle of repose. 14
or µ = tan θ
θ θ
3F
51. N = mg − F sin 60° = mg − µN2 5
2 w
 3F  Net moments of all the forces about point P should be zero.
∴ Limiting friction = µN = µ  mg − 
 2 
w  cosθ + µN 2 (l sin θ) = N 2 (l cosθ)
l

2 
52. Net external force, F = ( 4)2 + ( 3)2 = 5 N
l 5 12 5
Maximum friction fmax = µmg = (0.09)( 5)(10) = 4.5 N or w × × + µwl × = w × l ×
2 13 13 13
Since, F > f max block will move with an acceleration,
or 5 + 24 µ = 10
F − f max 5 − 4.5
a= = = 0.1m/s2 ∴ µ = 0.21
m 5
63. F = mg sin θ − µmg cosθ … (i)
53. Since, the block is at rest under two forces
(i) weight of block. 2F = mg sin θ + µmg cosθ … (ii)
(ii) contact force from the plane (resultant of force of or 2 mg sin θ − 2 µmg cosθ = mg sin θ + µmg cosθ
1
friction and normal reaction). or µ = tan θ
Contact force should be equal to weight (or 30 N) in 3
upward direction. Because under two forces a body remains 64. N = mg − Q cosθ θ
Q
in equilibrium when both the forces are equal to magnitude P + Q sin θ = µN = µ (mg − θcosθ) P
but opposite in direction. P + Q sin θ m
∴ µ=
54. Block is at rest. Hence, mg − Q cos θ
f = mg sinθ ≠ µmg cosθ 65. m1 g > m 2 g sin θ + µm 2 g cosθ
55. Angle of repose, θr = tan −1 (µ s ) = tan −1 (0.7)
m1
or tan θr = 0.7 or > sin θ + µ cosθ
m2
Angle of plane is θ = 30° ,tan θ = tan 30° = 0.577
θ
Since, tan θ < tan θr ,θ < θr 66. N = Mg − F cosθ = Mg − Mg cosθ = 2g sin 2
2
Block will not slide or f = mg sin θ ≠ µmg cosθ
Further block can be pulled if
or f = ( 2)( 9.8)sin 30° = 9.8 N
F sin θ ≥ µ cosθ
56. mg sin θ = (102)(10)sin 30° = 510 N θ θ θ θ
or 2Mg sin ⋅ cos ≥ 2 µMg sin 2 or cot ≥ µ
µ s mg cos θ = (0.4)(102)(10)cos 30° = 353N 2 2 2 2
F = 510 − 353 = 157N 67. Fnet = mass × acceleration
57. In critical case, weight of hanging part = force of friction on 68. In free fall, T = 0
the part of rope lying on table. ∆v 0.5[2 − ( − 2)]
m m 69. Fav = maav = m = = 2000 + N
∴ ⋅ l1 g = µ (l − l1 )g ∆t 10−3
l l
70. Assuming the resistance force or retardation to be constant.
 µ  2
Solving this we get, l1 =  l  v  = v 2 − 2as
1 + µ    … (i)
 2 1

58. α = angle of repose 2


0 =   − 2as 2
v
α … (ii)
or tanα = µ =
1  2
3 α s1
Solving these two equations, we get s 2 = = 1 cm
∴ cotα = 3 3
Laws of Motion 207

∆v v f + vt should be equal to N cosθ + Mg .


71. aav = =
∆t ∆t
2ghf + 2ghi N'
=
∆t
2 × 98
. × 2.5 + 2 × 98
. × 10
=
001
. Ncos θ + Mg
= 2100 m/s2
72. N = w + F sinθ 79. Net pulling force on the system F = 10g sin 37° − 4g = 20 N
∴ f max = µN = (tan φ)(w + F sin θ) Maximum force of friction
4
F f max = µmg cos 37° = 0.7 × 10 × 10 × = 56 N
5
θ
Since, F < f max system will not move. Equilibrium of 4 kg
gives T = 40N.
1
80. µ = tan 30° = (30° = angle of friction)
3
To move the body, N = mg − F sin 30° F
F cosθ = f max = (tan φ)(w + F sin θ)
= 100 − 
F 30º
Solving this equation, we get  2 10 kg
w sin θ F cos30° = µN
f =
cos(θ + φ) 3F 1  F 3F F
or = 100 −  or = 100 −
73. Force of friction is zero. Only contact force is normal 2 3 2 2 2
reaction which is mg cosθ. or F = 50N
74. Acceleration in both the cases will be same. 18
81. a = = 3 m/s2
N1 = ma but N 2 = ( 2m)a 6
N1 1
∴ = 2 kg block moves by friction. Hence,
N2 2 f = ma = 2 × 3 = 6N
75. Acceleration of system, 82. x P − x1 + x P − x 2 = length of string = constant
F−f 20 − 10
a= = Differentiating twice with respect to time, we get
M + m 6+ 4 a + a2
aP = 1
=1 m/s2 (towards left) 2
Let F0 be the reading of dynamometer, then the equation of Here, aP = A , a1 is positive and a2 is negative. Hence,
motion of mass m would be a − a2
A= 1
F0 − f = ma or F0 = f + ma = 10 + ( 4)(1) 2
=14N A
76. Moving down with retardation a means, lift is accelerated
upwards.With respect to lift pseudo force on the block will
be ma in downward direction, where mis the mass of block .
So, downward force mg on the block will be replaced by a1
m ( g + a ). Therefore, acceleration of block relative to plane
will be 1
xP
ar = ( g + a )sinθ (down the plane) a2
1
L = ar t 2  s = 1 at 2 
From  
2  2  x1
2
2
t= x2
ar
2L
= 83. Block A moves due to friction . Maximum acceleration of A
( g + a )sinθ
f µmg
can be max or or µg = 0.2 × 10 = 2 m/s2 . If both the
77. Wedge moves due to horizontal component of normal m m
reaction. blocks move together, then combined acceleration of A and
N N sinθ
Thus, a= H = (along − ve x-axis) B can be
10
of 3.33 m/s 2 . Since, this is more than the
M M 3
78. Net force on M in vertical direction should be zero. maximum acceleration of A. Slipping between them will
In vertically downwards, two forces N cosθ and Mg are take place and force of friction will be maximum or
acting. Therefore, N ′ the normal reaction from ground µmA g = 2 N.
208 Objective Physics Vol. 1

84. N Net force 2mg g


a= = =
2 kg
Total mass 4m 2
g
5 kg 5 m/s
2 Since, a = is greater than maximum acceleration of A
2
which can be given to it by friction.Therefore, slipping will
take place.
70 N
g
aA = 0.2g =
N − 70 = 7 × 5 … (i) 5
∴ N =105 N 2mg − 0.2mg
aB =
F = 300 N 3m
3g
= 06
. g=
Level 2 : Only One Correct Option 5
2g
1. When force is applied on A aAB = aA − aB = −
FA µmA g 5
= … (i) 3g
mA + mB mB or in backward direction.
5
When force is applied on B 6. l1 l2
FB µmA g
= … (ii)
mA + mB mA 2T2
T1
Dividing these two equations, we get T2
FB mB
= M1 M2 T3
FA mA
m 8
∴ FB = B ⋅ FA = × 112 = 24N M3
mA 4
T1l1 = 2T2l 2 … (i)
2. Maximum force of friction between ground and B is T1 = M1 g … (ii)
(15 + 5)(06
. )(10) = 120 N  M g − M2 ⋅ g 
 2 
f=0 M 2 g − T2 = M 2  3 
A M
 M2 + 2 
 3 
f=0
B F = 80 N M2 g
or T2 = … (iii)
F = 80 N 2
Solving these three equations, we get
By applying 80 N force on B, blocks will not move. Hence, M1
=3
force diagram is as shown in above figure. M2
3. Maximum acceleration of m by friction = their maximum 7. 2T = 250
common acceleration
µ 1m1 g F − µ 2 ( M + m)g 0.4F 0.4F
∴ = 2T T
m2 ( M + m)
m will move by friction.
∴ F = (µ 1 + µ 2 )( M + m)g F F
F F
f max = µmg
µmg
∴ ( am )max = = µg 0.4F
m 250 N 150 N
4. m will move by friction,
f max = µmg T =125 N
µmg T + 0.4F = 150
∴ ( am )max = = µg
m
∴ F = 62.5 N
5. Block A moves due to friction. Maximum value of friction
can be µmA g. Therefore, maximum acceleration of A can be 8. Maximum acceleration due to friction of mass m over mass
µmA g g 2m can be µg. Now, for the whole system
or µg = 0.2g = . When force 2 mg is applied on lower
mA 5 Net pulling force
a=
block, common acceleration (if both move together) will be, Total mass
mg
m 0.2 mg ∴ µg =
0.2 mg 4m
2 mg 1
3m or µ=
4
Laws of Motion 209

9. Net external force, F = ( 4)2 + ( 3)2 = 5 N m1 g − T = m1 a …(ii)


Maximum friction, f max = µmg T − m2 g = m2 a …(iii)
= (009
. )( 5)(10) Solving Eqs. (ii) and (iii), we get
= 4.5 N m3 = 1 kg
Since, F > f max , block will not move with an acceleration, 14. T sin θ − mg sin α = ma
F − f max T cos θ = mg cos α
a=
m
T
5 − 4.5
= θ a
5
= 0.1m/s2
10. TB = 3T and TA = 2A mg sin α
mg cos α
T 3 α
∴ vA = B ⋅ vB = v0 (towards right)
TA 2
3v v From these two equations, we get
∴ v AB = 0 − v 0 = 0 , (towards right) a + g sin α
2 2 tan θ =
g cos α
In such cases, velocity and acceleration are in increase ratio
of tensions.  a + g sin α 
θ = tan −1  
11. Acceleration of system before breaking the string was,  g cos α 
Net pulling force 15. 2T1 cos 45° mg
a=
Total mass mg
∴ T1 = … (i)
3g − 2g g
= = 2
5 5 T2 sinθ
g T2
After 5 s, velocity of system v = at = × 5 = g m/s
5 θ
v2 g2 g
Now, h= = = = 4.9 m T1 M
T1 T2 cosθ
2g 2g 2
45º 45º T √2
12. Maximum acceleration of the box can be µg or 1.5 m/s , 2 1

while acceleration of truck is 2 m/s 2 . Therefore, relative


acceleration of the box will be a r = 0.5 m/s 2 (backward). It T
Mg + 1
will fall off the truck in a time. √2
m
t=
2l  s = 1 at 2 
  T1 mg
ar  2  T2 cosθ = =
2 2
2× 4 mg +
mg
= = 4s
0.5 tanθ = 2 = 1 + 2M
mg m
Displacement of truck up to this instant is 2
1 1 F F
sr = a r t 2 = × 2 × ( 4)2 = 16 m 16. = 75 N or < 100 N
2 2 4 4
F = 300 N
13. m3 is at rest. Therefore,
2T = m3 g …(i)

F/2
F
= 75 N
F 4
2
2T 2T

F F
m3 P 4 4
T T

m1
m2 50 N 100 N
Therefore, aM = 0
Further if m3 is at rest, then pulley P is also at rest. Writing 75 − 50
am = = 5m/s2
equations of motion. 5
210 Objective Physics Vol. 1

17. In false balance, l1 ≠ l 2 22. To remain the string tight,


Moments about O should be zero. component of velocities along B 75º
∴ w1l1 = wl 2 ⇒ wl1 = w 2l 2 the line joining A and B should
be same. 30º
Dividing two equations, we get 60º
l1 O l2 45º
∴ ucos 45° = ucos60°
or v = 2u A
w1 w2 v 45º
u
l1 O l2
23. In first case,
T1 = F1
w1 w2

w1 w
= or w = w1w 2
w w2
= ( 8 × 18 )g = 12g
18. 2F cosθ = mg
In second case,
F d/2 F T2 = 2F2

F θ θ F

mg ∴ In first case, person will have to apply more force.


d 2 24. f1 = maximum value of friction between m and 2m = µmg
mg h2 +
mg 4 f 2 = maximum value of friction between 2m and ground
∴ F= = µ
2cosθ 2h = ( 3m)g = µ mg
mg 3
= d 2 + 4h2
4h Now, let us free body diagram of m and 2m in all four cases.
µ
19. Total upward force = 2 
mg  µ
 = mg (a) 3 mg m F= mg
 2  3
Total downward force is also mg µ
T=0 mg
∴ Fnet = 0 = anet 3
2m
20. N sin θ + µN cosθ = ma µ
N mg
3

µmg m F = µmg
(b)
a
T=0
2m µmg
µmg
f = µN
mg
µmg m F = 2µmg
N cosθ = µN sin θ = mg (c)
Putting θ = 45° and solving these two equations, we get T=0
1 + µ  2m µmg
a = g 
1 − µ  µmg
2F − F F
21. a = = (towards left)
2m 2m µmg m F = 3µmg
Writing equation of right hand side block (d)
F T=0
2F − N sin 30° = ma =
2 2m µmg
N
= 2F − =
F 3F µmg
2 2 2
∴ N = 3F / Tension will be zero in all four cases.
Laws of Motion 211

25. T = mg
T a

ar
m T = mg

3 kg ar
mg
3g + 3a
When the string breaks, T will suddenly disappear. 5 kg
T
∴ am = (downwards)
m
5g + 5a
mg
= =g
m 28. F1r1 = F2r2 (equating the moments about line of action or
T mg g
a2m = = = (upwards) force)
2m 2m 2 r1 F2
g ∴ =
26. a = g sinθ = r2 F1
2
Let L be the distance between the forces. Then,
N /√2
 F2   F1 
r1 =  L and r2 =  L
 F1 + F2   F1 + F2 
an L
N /√2 m Now, given that r1 = r2 +
45º 4
a = g sin θ  F2   F1  L
=
g av or  L =  L +
√2  F1 + F2   F1 + F2  4
mg
Solving this equations, we get
g 1 g
aH = aV = × = F1 3
2 2 2 =
F2 5
N mg
= … (i)
2 2 29. µ(mA + mC )g = mB g
N mg mB 5
mg − = … (ii) ∴ mC = − mA = − 10 =15 kg
2 2 µ 0.2
F
N′ = + 2mg … (iii) 30. N1 sin 37° = N 2 sin 74°
2
N or N1 = 2N 2 cos 37°
µN ′ = … (iv)
2 N1
N' N2
37º 74º

µN' N
2m 2
mg

Now, N1 cos 37° + N 2 cos 74° = mg


N + 2mg
2 or 2N 2 cos2 37° + N 2 ( 2cos2 37° − 1) = mg

2N 2   + N 2 
16 32 
Solving above four equations, we get or − 1 = mg
µ = 0.2  25   25 
27. Let acceleration of lift is a upwards. Then, with respect to 39
or N 2 = mg
lift 25
25mg
Net pulling force ∴ N2 =
ar = 39
Total mass
( 5g + 5a ) − ( 3g + 3a ) 31. m ( g − a )sin θ = µm ( g − a )cosθ N
= 1
8 or µ = tan θ = tan 30° =
9 ( g + a) 3
or g=
32 4 θ
g
∴ a= mg – ma
8
212 Objective Physics Vol. 1

T1 sin α = T2 sin β = 2 mg 
1 
More than One Correct Options  2

1. Maximum value of friction between two blocks
∴ T1 sin α = mg ...(v)
f max = 0.2 × 1 × 10 = 2 N
From Eqs. (iv) and (v), we get
F T 1
m2 tan α = and T1 = 5 mg
2
2N tan β = tan 45° = 1 and T2 = 2 mg
2N m1
T
∴ tan β = 2 tan α and 2 T1 = 5 T2
3. a = slope of v - t graph = −1 m / s2
In critical case,
T = 2N µ mg
∴ Retardation = 1 m/s2 = = µg
F =T + 2= 4N m
1 1
∴ System is in equilibrium if f ≤ 4 N or µ = = = 0.1
g 10
For F > 4 N
If µ is half, then retardation a is also half. So, using
F − T + 2 = m2 a = (1)( a ) ...(i)
v = u − at or 0 = u − at
T − 2 = m1 a = (1)( a ) ...(ii)
u 1
a or t= or t ∝
a a
m2 we can see that t will be two times.
T
F 2N 4. Maximum force of friction between A and B
2N ( f1 )max = 0.3 × 60 × 10 = 180 N
m1 T Maximum force of friction between B and ground
( f 2 )max = 0.3 × (60 + 40) g = 300 N
a a
On solving these two equations, we get A T = 125 N
F f1
T= f1
2 B T = 125 N
When F = 6 N, T = 3 N (f1)max a
2. Resultant of mg and mg is 2 mg. 125 − f1 = 60 a ...(i)
125 + f1 − 300 = 40 a ...(ii)
T2 On solving these two equations, we get
b
a = 0.5 m/s2
mg
B and f1 = 95 N
f1 = 95 N is less than its maximum value of 180 N.
mg sin θ
5. ax = = g sin θ
mg m

Therefore, T2 should be equal and opposite of this. Or N x y


T2 = 2 mg ...(i)
Further,
T2 cosβ = mg ...(ii) q
and T2 sin β = mg ...(iii) sin mg cos q
mg
or sin β = cos β ⇒ β = 45°
It is also moving in y -direction.
T1 ∴ mg cosθ> N
a mg cosθ − N
ay =
mg m
B Now, a = ax2 + a 2y > g sin θ
b

mg
ax
T1 cos α = mg + T2 cos β a

= mg + 2 mg 
1  ay

 2
or T1 cosα = 2 mg ...(iv)
Laws of Motion 213

6. Maximum value of friction between A and B is At t = 1 s, F = 2 N < 36 N, therefore system remains stationary
( f1 )max = 0.25 × 3 × 10 = 7.5 N and force of friction between 2 kg and 4 kg is zero.
Maximum value of friction between B and C At t = 4 s, F = 8 N < 36 N. Therefore, system is again
( f 2 )max = 0.25 × 7 × 10 = 17.5 N stationary and force of friction on 4 kg from ground is 8 N.
and maximum value of friction between C and ground, At t = 15 s, F = 30 N < 36 N and system is stationary.
( f 3 )max = 0.25 × 15 × 10 = 37.5 N 10. Net pulling force = 0
F0 = force on A from rod ⇒ a =0
T1 = 1 × g =10 N
A F0
7.5 N T3 = 2 × g = 20 N
7.5 N T2 = 20 + T1 = 30 N
B T
17.5 N 11. f max = 0.3 × 2 × 10 = 6 N
(f2)max = 17.5 N At t = 2 s, F = 2 N < f max
F C T ∴ f =F =2N
(f3)max = 37.5 N
At t = 8 s, F = 8 N > f max
If C is moving with constant velocity, then B will also move ∴ f =6 N
with constant velocity. At t = 10 s, F = 10 N > f max
For B, T = 17.5 + 7.5 = 25 N ∴ f =6N
F = 17.5 + 25 + 37.5 = 80 N F − f 10 − 6
For C, a= =
m 2
For F = 200 N,
= 2 m/s2
Acceleration of B towards right
F = f max = 6 N at 6 s
= acceleration of C towards left
For 6 s ≤ t ≤10s,
= a ( say )
T − 7.5 − 17.5 = 4 a F − f t −6
Then ...(i) a= = = 0.5t − 3
200 − 17.5 − 37.5 − T = 8 a ...(ii) m 2
v 10
On solving these two equations, we get
∫ dv = ∫ adt = ∫ (0.5t − 3) dt
a =10 m/s2 0 6

7. Since, µ 1 > µ 2 v = 4 m/s


After 10 s,
∴ ( f1 )max > ( f 2 )max
F − f 10 − 6
m m a= = = 2 m/s2
T T
m 2
Man Block = constant
(f1)max = m1mg (f2)max = m2mg ∴ v ′ = v + at
Further if both move, = 4 + 2 (10 − 10) = 8 m/s
T − µ mg 12. Maximum force of friction between 2 kg and 4 kg
a=
m = 0.4 × 2 × 10 = 8 N
µ of block is less. Therefore, its acceleration is more. 2 kg moves due to friction. Therefore, its maximum
8. N cosθ = mg =10 ...(i) acceleration may be
8
N sin θ = ma = 5 ...(ii) amax = = 4 m/s2
2
On solving these two equations, we get
Slip will start when their combined acceleration becomes
N 4 m/s2 .
q F 2t
∴ a= or 4 = or t =12s
a m 6
At t = 3s
F 2t
a2 = a4 = =
m 6
mg
2× 3
= = 1 m/s2
9. f1 → force of friction between 2 kg and 4 kg 6
Both a2 and a4 are towards right. Therefore, pseudo forces F1
f 2 → force of friction between 4 kg and ground
(on 2 kg from 4 kg) and F2 (on 4 kg from 2 kg) are towards left.
( f S 1 )max = 0.4 × 2 × 10 = 8 N
F1 = ( 2)(1) = 2 N
FK 1 = 0.2 × 2 × 10 = 4 N F2 = ( 4)(1) = 4 N
( f S2 )max = 0.6 × 6 × 10 = 36 N From here, we can see that F1 and F2 do not make a pair of
FK 2 = 0.4 × 6 × 10 = 24 N equal and opposite forces.
214 Objective Physics Vol. 1

14. Consider the below diagram. Frictional force on B ( f1 ) and Let the body moves down the plane, in this case f acts up
frictional force on A ( f 2 ) will be as shown. the plane.
Hence, m1 g sin θ − f > m2 g
Let A and B are moving together
F − f1 F − f1 2( F − f1 ) ⇒ m1 g sin θ − µ m1 g cos θ > m2 g
a common = = =
mA + mB (m / 2) + m 3m ⇒ m1 (sin θ − µ cos θ) > m2
⇒ m2 < m1 (sin θ − µ cos θ)
Pseudo force on A = (mA ) × a common
2( F − f1 ) m 2 ( F − f1 ) Hence, option (d) is correct.
= mA × = × 16. Let A moves up the plane frictional force on A will be
3m 2 3m
downward as shown.
( F − f1 )
= N1
3 A
B N2
A θ1
f2 f2
θ c os
s θ2
1
s in mg
B F co θ
mg

mg
f1 f mg 2
θ1 mg mg θ2

sin
θ2
The force ( F ) will be maximum when
Pseudo force on A = Frictional force on A When A just starts moving up
Fmax − f1
⇒ = µ mA g mg sin θ1 + f = mg sin θ2
3
⇒ mg sin θ1 + µmg cos θ1 = mg sin θ2
m
= 0. 2 × × g = 01 . mg sin θ2 − sin θ1
2 ⇒ µ=
cosθ1
⇒ Fmax = 0.3 mg + f1
3 When A moves upwards
= 0.3 mg + (0.1) mg = 0.45 mg
2 f = mg sin θ2 − mg sin θ1 > 0
Hence, maximum force up to which bodies will move ⇒ sin θ2 > sin θ1
together is Fmax = 0.45 mg ⇒ θ2 > θ1
(a) Hence, for F = 0. 25 mg < Fmax bodies will move together.
(b) For F = 0.5 mg > Fmax , body A will slip with respect to B. Comprehension Based Questions
(c) For F = 0.5 mg > Fmax , bodies slip. 1. Let µ K = µ , then µ S = 2µ
3
( f1 ) max = µ mB g = (01. ) × m × g = 015 . mg According to first condition,
2
F + mg sin θ = µ S mg cos θ = 2 µ mg cos θ ...(i)
( f 2 ) max = µ mA g = (0. 2)   ( g ) = 01
m
. mg According to second condition,
 2
mg sin θ = F + µ K mg cos θ
Hence, minimum force required for movement of the
system ( A + B ) = F + µ mg cos θ ...(ii)
Fmin = ( f1 ) max + ( f 2 ) max Putting θ = 30°, we get
= 0.15 mg + 0.1 mg = 0.25 mg  3
F + mg /2 = 2µ mg  
(d) Maximum force for combined movement Fmax = 0.45 mg.  2 
15. Let m1 , moves up the plane. Different forces involved are or 3 µ mg = F + 0.5 mg ...(iii)
shown in the diagram.
mg  3
= F + µ mg  
N
T 2  2 
T
or 0.5 3 µ mg = 0.5 mg − F ...(iv)

si nθ m2 Dividing Eqs. (iii) and (iv), we get


mg θ mg
f mg cos θ m 2g F=
m 1g 6
2. Substituting value of F in Eq. (iii), we have
N = Normal reaction 2
f = Frictional force µ= =µK
3 3
T = Tension in the string 4
∴ µ S = 2µ =
f = µ N = µ m1 g cos θ 3 3
For the system (m1 + m2 ) to move up F + mg sin θ − µ K mg cos θ
3. a =
m2 g − (m1 g sin θ + f ) > 0 m
⇒ m2 g − (m1 g sin θ + µ m1 g cos θ ) > 0  3
(mg /6) + (mg /2) − 
2 
 mg  
⇒ m2 > m1 (sin θ + µ cos θ)  3 3  2  g
= =
Hence, option (b) is correct. m 3
Laws of Motion 215

4. F ′ = mg sin θ + µ S mg cos θ
Assertion and Reason
mg 4  3  7mg
= + mg   = 1. Normal reaction = 15 N
2 3 3  2  6
5. F ′ ′ = mg sin θ + µ K mg cos θ
 3  5mg
= (mg /2) + 
2 
 mg  = 15 N
 3 3  2  6
1350 × 9.8 − 1200 × 9.8
6. Acceleration, a1 = =1.225 m/s2
1200 f max = µN = 9 N
vf = 0 Weight = 10 N
h2
a2 Since, weight > f max
v
∴ 9 N friction will act and block will move downwards.
1
h1 a1 4. mg sinθ = 10 × 10 × = 50 N
2
u=0 3
µmg cosθ = 0.7 × 10 × 10 × = 60.62 N
1200 g − 1000 g 2
Retardation, a2 = =1.63 m/s2
1200 Since µmg cosθ is more, block will remain stationary.
h1 + h2 = 25 ...(i) 5. ( f 2 )max = µ 2mg
v = 2a1 h1 ( f1 )max = µ 1 ( 2m)g
or 2a1 h1 = 2a2 h2 Since, ( f1 )max > ( f 2 )max
h1 a2 1.63 Lower block will not move at all.
∴ = = = 1.33 ...(ii)
h2 a1 1.225 7. Due to rotation of earth it is non-inertial. A frame moving
Solving these equations, we get with constant velocity is inertial.
h1 = 14.3 m 8. Friction opposes the relative motion of the bodies in
7. v = 2a1h1 = 2 × 1.225 × 14.3 contact not the motion.
9. Friction opposes the relative motion of the bodies in
= 5.92 m/s Ans.
contact. By throwing something backwards you can move
∴ θ = tan −1   = 28°
8 8 forwards.
8. tanθ =
15  15  10. Static friction varies between zero to a maximum limiting
( f A )max = 0.2 × 170 × 10 × cos 28° value.
= 300.2 N ≈ 300 N w+F
11. Retardation in upward journey, a1 =
( f B )max = 0.4 × 170 × 10 × cos 28° m
= 600.4 N ≈ 600 N F
Now, v
(mA + mB ) g sin θ = ( 340)(10)sin 28° =1596 N
Since, this is greater than ( f A )max + ( f B )max , therefore blocks
slides downward and maximum force of friction will act on
both surfaces w F
w
v
∴ f total = ( f A )max + ( f B )max = 900 N
Upward journey Downward journey
(mA + mB ) g sin θ − f total
9. a = w−F
mA + mB Acceleration in downward journey, a2 =
m
1596 − 900 ∴ a1 > a2
= = 2.1 m/s2
340 12. Tension is non-uniform even if string is not accelerated.
a
F

A ) max
(f A
q
in
gs
mA
At rest
F = force on connecting bar
mA g sin θ − F − ( f A )max = mA a 13. Fnet = 0
∴ F = mA g sin θ − ( f A )max − mA a ∴ a net = 0
= 170 × 10 × sin 28° − 300 − 170 × 2.1 But, torque may not be zero. Therefore, it can’t be in
=141 N rotational equilibrium.
216 Objective Physics Vol. 1

14. f − mg sin θ = ma sin θ Now, we can draw free body diagrams of all the four blocks.
a sinθ 20 N 20 N
f

1 kg a2 = 0 2 kg
mg sinθ
2
a1 = 10 m/s
10 N 20 N
∴ f = m ( g + a )sinθ >mg sinθ
15. Let T0 is the tension in the string connecting the two blocks.
20 N 20 N
Then,
T0 − 10 = 1 × a
∴ T0 = 10 N a3 =
10 m/s2 3 kg 4 kg a4 = 20 = 5 m/s2
3 4
20 − T0 = 2 × a
∴ T0 < 20 N
16. In case of massless string, 30 N 40 N
2F − F 3. Force of friction opposes relative motion between two
a= =∞
0 bodies in contact. It does not simply opposes the relative
motion. Further, N = mg , only when no inclined force is
17. Consider moment of forces about the point of application
acting on the block as shown in figure.
of force.
F F
18. Fnet = ma
19. N1 = max − 2m
N 2 = m ( g + a y ) = 12m
N1 1
∴ = N < mg N > mg
N2 6
4. Force of friction and mg sinθ both are downwards.
Match the Columns ∴ Acceleration of the block is
1. N = mg − 20 2 sin 45° = 20N a = ( g sin θ + µg cosθ) down the plane
µ s N =16 N Now, net force in any direction is equal to
and µ k N =12 N F = m (component of acceleration that direction)
5. Acceleration of system,
Since, 20 2 cos 45°> µ s N block will move and kinetic friction
60 − 18 − (m1 + m2 + m3 )g sin 30°
will act, a= = 2 m/s 2
(m1 + m2 + m3 )
20 2 cos 45° − µ k N
a=
m Net force on 3 kg block = m3 a = 6 N
20 − 12
= = 2 m/s 2 From free body diagram of 3 kg block, we have
4
N12 − m1 g sin 30° −18 = m1 a
2. Since, the pulleys are smooth, net force on each pulley
should be zero. With this concept, tensions on all strings are ∴ N12 = 25 N
shown below. Now, we can draw free body diagrams of all From free body diagram of 3 kg block, we have
the four blocks.
60 − m3 g sin 30° − N 32 = m3 a
F = 80 N ∴ N 32 = 39 N
dv A
6. a A = = ( 2j ) m /s
$ 2
dt
dv B
aB = =0
dt
d vC
aC = = ( −4j$ ) m/s 2
40 N 40 N dt
Now, pseudo force in opposite direction of acceleration of
frame from where object is observed.
20 N 7. Force diagram on both the blocks is as shown in figure.
20 N 20 N
1 kg
1kg 3kg 4N
20 N
4N
2kg 2 kg
4kg 18 N
Laws of Motion 217

The coefficient of friction, µ = 1 − 2  tan θ


1
Entrance Gallery  n 
1. As the force is exponentially decreasing, so its acceleration
µ = 1 − 2  tan 45°
1
i. e. rate of increase of velocity will decrease with time. Thus, ⇒
 2 
the graph of velocity will be an increasing curve with
decreasing slope with time. 4 −1
µ=
F F dv 4
a = = 0 e −bt =
m m dt 3
⇒ µ=
v t F −bt 4
⇒ ∫0 dv = ∫0 m e dt
0
F 80
5. Here, a = = = 1.6 m/s2
F0  1  −bt
t m 50
⇒ v=  e u = 20 m / s and v = 0
m  −b  0
0 From equation of motion, v = u + at
F
= 0 e −bt 0 = 20 − 1. 6 × t
mb t 20
F0 0 F t= = 12.5 s
v= (e − e −bt ) = 0 (1 − e −bt ) 1.6
mb mb 6. u =11 m/s
F
with v max = 0 Q a = µg
mb
Given a = 0.4 × 10 = 4 m/s2
2. A block of mass m is placed on a surface with a vertical
cross-section, then According to question, v = 0
Y So from, v 2 = u2 + 2as
m 0 = (11)2 + 2( −4) × s
11 × 11
y s= = 15.125 m
8
θ 7. Weight, w = mg = 0.1 × 10 = 1 N
X
Q F2
x 3 
d 
dy  6  x2
tanθ = =
dx dx 2 θ
At limiting equilibrium, we get 1N
µ = tan θ F1
x2
0.5 = ⇒ x 2 =1 ⇒ x = ±1 θ
2 P
x 2 w=1N
Now, putting the value of x in y = , we get
6 F1 = component of weight = 1 ⋅ sin θ = sin θ
When, x =1 When, x = −1 F2 = component of applied force = 1 ⋅ cosθ = cosθ
(1)3 1 ( −1)3 −1 Now, at θ = 45 ° , F1 = F2 and block remains stationary
y= = y= =
6 6 6 6 without the help of friction.
So, the maximum height above the ground at which the For θ> 45° , F1 > F2 , so friction will act towards Q.
1
block can be placed without slipping is m. For θ< 45°, F2 > F1 and friction will act towards P.
6
8. From equation of motion,
3. Angle of inclination, θ = 60°
v 2 = u2 + 2as …(i)
The formula of the coefficient of static friction µ is
µ = tan θ = tan60° = 3 where, v = final velocity
u = initial velocity
Thus, µ =1.732
4. If the same wedge is made rough, then time taken by it to a = acceleration
come down becomes n times more. and s = distance travelled.
Now,
Block Block
Case I (0)2 + ( 2u)2 − 2a ( s1 ) [using Eq. (i)]

d d u = 2as1
2
…(ii)
Case II (0)2 = ( 2u)2 − 2as 2 4u2 = 2as 2 …(iii)
Rough surface Smooth surface
[using Eq. (i)]
45º 45º Hence, from Eqs. (ii) and (iii), we get
s 2 = 4s1
218 Objective Physics Vol. 1

9. As, the block is at rest, 13.


Balancing horizontal forces component on the block,
f = µmg cosθ = µN

°
60
N
θ

θ
sin

mg cosθ 1
l= g cos60° t12
g

mg …(i)
m

2
θ 1
l cosθ = g t22 …(ii)
i.e. mg cosθ(µ s ) = mg sin θ 2
⇒ tanθ = µ s t12 1 4
∴ = =
Coefficient of static friction, µ s = tan θ t22 cos2 60 ° 1
10. Given, m = 2kg ,µ = 0.2 and g = 10 ms−2 t1 : t2 = 2 :1
1 x
Here, ma = µmg 14. tan 30° = =
3 10
a = µg
10
a = 0.2 × 10 ⇒ x=
3
⇒ a = 2 ms−2 x
11. We know that, F = ma

F2 30°
N 14 N
F1

15. Net frictional force between block and surface is


4 kg
F = µR = 0.5 × 10 × 10 = 50 N
F 14
a= = = 2 ms−2 Applied force is 10 N and it is less than 50 N.
m 17
Hence, from the figure ∴System is at rest and no friction between
A and B.
14 − N = 4a
16. When, P = mg (sin θ − µ cosθ)
14 − N = 8
⇒ N = 6N Then, f = µmg cosθ (upwards)
12. Given, m1 = 1 kg, m2 = 2 kg and g = 10 m/s2 When P = mg sinθ,
Then, f =0
 m − m1 
a= 2 g and when P = mg (sin θ + µ cosθ)
 m1 + m2 
Then, f = µ mg cosθ (downwards)
 2 −1  10 17. mg sinθ = ma
=  10 =
1 + 2  3
∴ a = g sinθ
1 2 1
Q s = ut + at ⇒ s = at 2 [Qu = 0] where, a is along the inclined plane.
2 2 ∴ Vertical component of acceleration is
1 10 20
= × × 4= g sin 2 θ.
2 3 3
20 20 ∴ Relative vertical acceleration of A with respect to B is
2× −1 × g
m= 3 3 = 20 g (sin 2 60 ° − sin 2 30 ° ) = = 4.9 m s−2 (in vertical direction)
3 9 2
6
Work, Energy
and Power

6.1 Introduction to Work


In our daily life ‘work’ has many different meanings. For example, Ram is working in Chapter Snapshot
a factory. The machine is in working order. Let us work out a plan for the next year, etc. In ● Introduction to Work
physics however, the term ‘work’ has a special meaning. In physics, work is always ● Work Done By a
associated with a force and a displacement. We note that for work to be done, the force Constant Force
must act through a distance. Consider a person holding a weight a distance h off the floor as
● Work Done By a Variable
shown in figure. In everyday usage, we might say that the man is doing a work, but in our
Force
scientific definition, no work is done by a force acting on a stationary object. We could
eliminate the effort of holding the weight by merely tying the string to some object and the ● Conservative and
weight could be supported with no help from us. Non-conservative Force
Field
● Kinetic Energy
● Work-Energy Theorem
T
● Potential Energy
● Law of Conservation of
Mechanical Energy
h h ● Three Types of
T
Equilibrium
No work is done by the man holding the weight at a fixed position. ● Power
The same task could be accomplished by tying the rope to a fixed point.
Fig. 6.1

Let us now see what does ‘work’ mean in the language of physics.

6.2 Work Done by a Constant Force


Let us first consider the simple case of a constant force F acting on a body. Further, let
us also assume that the body moves in a straight line in the direction of force. In this case,
we define the work done by the force on the body as the product of the magnitude of the
force F and the distance S through which the body moves.
220 Objective Physics Vol. 1

e.g. the work W is given by Sol. Weight = mg = (2 ) (10) = 20 N


Work done by the applied force WF = F h cos 0°
W = F⋅S
As the angle between force and displacement is 0°
or WF = (40) (2 ) ( 1) = 80 J
F
Similarly, work done by its weight
Wmg = (mg ) (h) cos 180° or Wmg = (20) (2 ) (−1) = − 40 J
S
X Example 6.2 Two unequal masses of
Fig. 6.2
1 kg and 2 kg are attached at the two ends
On the other hand, in a situation when the constant force of a light inextensible string passing over
does not act along the same direction as the displacement of a smooth pulley as shown in figure. If the
the body, the component of force F along the displacement S system is released from rest, find the work 1 kg
is effective in doing work. done by string on both the blocks in 1 s.
F (Take g =10 m/s 2 ) 2 kg
θ
Sol. Net pulling force on the system is
Fig. 6.5
Fnet = 2 g − 1g = 20 − 10 = 10 N
S Total mass being pulled
Fig. 6.3 m = (1 + 2 ) = 3 kg
Thus, in this case, work done by a constant force F is
given by
W = (component of force along the displacement)
× (displacement)
or W = ( F cos θ ) ( S ) a
1 kg T
or W = F ⋅ S (from the definition of dot product) a
So, work done is a scalar or dot product of F and S. 1g 2 kg 2 kg a
Regarding work done it is worth noting that
1. Work can be positive, negative or even zero also, 2g 20 N
depending on the angle (θ ) between the force vector F Fig. 6.6 (a) Fig. 6.6 (b)
and displacement vector S. Work done by a force is zero
when θ = 90°, it is positive when θ < 90° and negative Therefore, acceleration of the system will be
F 10
when θ > 90°. For example, when a person lifts a body, a = net = m/s 2
m 3
the work done by the lifting force is positive (as θ = 0° ) Displacement of both the blocks in 1 s is
but work done by the force of gravity is negative (as 1 1 10 5
S = at 2 =   (1)2 = m
θ = 180° ). Similarly, work done by centripetal force is 2 2  3 3
always zero (as θ = 90° ). Free body diagram of 2 kg block is shown in Fig. 6.6 (b).
2. Work depends on frame of reference. With change of Using Σ F = ma, we get
10
frame of reference inertial force does not change while 20 − T = 2 a = 2  
displacement may change. So, the work done by a  3
force will be different in different frames. For 20 40
or T = 20 − = N
example, if a person is pushing a box inside a moving 3 3
train, then work done as seen from the frame of ∴ Work done by string (tension) on 1 kg block in 1 s is
reference of train is F ⋅ S while as seen from the ground W1 = (T ) (S ) cos 0°
it is F ⋅ ( S + S 0 ) . Here, S 0 is the displacement of train 40 5
=     (1)
relative to ground.  3   3
200
X Example 6.1 A block of =J
F 9
mass m = 2 kg is pulled by a Similarly, work done by string on 2 kg block in 1 s will be
force F = 40 N upwards W2 = (T )(S ) (cos 180° )
through a height h = 2 m. Find 40 5
=     (−1)
m
the work done on the block by  3   3
the applied force F and its 200
=− J
weight mg. ( g =10 m/ s 2 ) Fig. 6.4
9
Work, Energy and Power 221

direction of this force F is always towards its mean position


6.3 Work Done by a ( x = 0) and the magnitude is directly proportional to x
Variable Force F ∝x (Hooke’s law)
So far we have considered the work done by a force ∴ F = − kx …(i)
which is constant both in magnitude and direction. Let us Here, k is a constant called force constant of spring and
now consider a force which acts always in one direction but depends on the nature of spring. From Eq. (i), we see that F
whose magnitude may keep on varying. We can choose the is a variable force and F - x graph is a straight line passing
direction of the force as x-axis. Further, let us assume that through origin with slope = − k. Negative sign in Eq. (i)
the magnitude of the force is also a function of x or say F ( x )
implies that the spring force F is directed in a direction
is known to us. Now, we are interested in finding the work
opposite to the displacement x of the block.
done by this force in moving a body from x1 to x 2 .
F
F F

x=x
x
W = Area

x1 x2 x x
dx
Fig. 6.7 Fig. 6.9

Work done in a small displacement from x to x + dx Let us now find the work done by this force F when the
will be block is displaced from x = 0 to x = x. This can be obtained
dW = F ⋅ dx either by integration or the area under F - x graph.
x
Now, the total work can be obtained by integration of Thus, W = ∫ dW = ∫ F dx
0
the above elemental work from x1 to x 2 or
x 1 2
x2
W = ∫ dW = ∫
x2
F ⋅ dx = ∫ − kx dx = − kx
x1 x1
0 2
x2
Here, work done is negative, because force is in
It is important to note that ∫ F dx is also the area under opposite direction of displacement.
x1
F - x graph between x = x1 to x = x 2 . Similarly, if the block moves from x = x1 to x = x 2 . The
limits of integration are x1 and x 2 and the work done is
Spring Force x2 1
W = ∫ − kx dx = k ( x12 − x 22 )
An important example of the above idea is a spring that
x1 2
obeys Hooke’s law. Consider the situation shown in figure. X Example 6.3 A force F = ( 2 + x ) acts on a particle
in x-direction, where F is in newton and x in metre.
Find the work done by this force during a displacement
from x =1.0 m to x = 2.0 m.
Sol. As the force is variable, we shall find the work done in a small
x=0
F
displacement from x to x + dx and then integrate it to find the
total work. The work done in this small displacement is
dW = F dx = (2 + x) dx
2. 0 2. 0
Thus, W = ∫1. 0 dW = ∫1. 0 (2 + x) dx
x 2. 0
 x2 
Fig. 6.8 = 2 x +  = 3.5 J
 2 1. 0
One end of a spring is attached to a fixed vertical
k
support and the other end to a block which can move on a X Example 6.4 A force F = − ( x ≠ 0) acts on a
horizontal table. Let x = 0 denote the position of the block x2
when the spring is in its natural length. When the block is particle in x-direction. Find the work done by this force
displaced by an amount x (either compressed or elongated) a in displacing the particle from. x = + a to x = + 2 a.
restoring force (F) is applied by the spring on the block. The Here, k is a positive constant.
222 Objective Physics Vol. 1

Sol. W = ∫ F dx X Example 6.6 An object is displaced from point


−k 
+2 a  k k
+2 a A (2m, 3m, 4m) to a point B (1m, 2m, 3m) under a
= ∫+ a  2  dx =  
x   x + a
= −
2a constant force F = (2$i + 3$j + 4k$ ) N . Find the work done
by this force in this process.
/ It is important to note that work comes out to be negative
rf
which is quite obvious as the force acting on the particle is in Sol. W = ∫ F⋅ d r
negative x-direction  F = − 2  while displacement is along
k ri

 x  ( 1 m , 2m , 3m )
∫( 2m, 3m, 4m ) (2 i
= $ + 3$j + 4k$ )⋅(dx $i + dy$j + dzk$ )
positive x-direction. (from x = a to x = 2a)
= [2 x + 3 y + 4 z]( 1m, 2m, 3m ) = − 9 J
( 2m, 3m, 4m )
X Example 6.5 A force F acting on a particle varies
with the position x as shown in figure. Find the work Alternate Solution
done by this force in displacing the particle from Since, F = constant, we can also use
W = F ⋅S
F(N)
Here, S = rf − ri
10 = ($i + 2 $j + 3k$ ) − (2 $i + 3$j + 4k$ )
= (− $i − $j − k$ )
–2
x (m) ∴ W = (2 $i + 3$j + 4k$ ) ⋅ (− $i − $j − k$ )
2
= − 2 − 3 − 4 = − 9J

–10 X Example 6.7 An object is displaced from position


vector r1 = (2$i + 3$j) m to r2 = ( 4$i + 6$j)m under a force
Fig. 6.10
F = (3x 2 $i + 2 y$j) N . Find the work done by this force.
(a) x = − 2 m to x = 0 (b) x = 0 to x = 2 m. r2

Sol. (a) From x = − 2m to x = 0, displacement of the particle is


Sol. W = ∫r
1
F ⋅dr
r2
along positive x-direction while force acting on the particle is = ∫r (3 x2 $i + 2 y $j )⋅(dx $i + dy$j + dzk$ )
along negative x-direction. Therefore, the work done is negative 1
r2
and given by the area under F-x graph. = ∫r (3 x2 dx + 2 ydy)
1 1
∴ W = − (2 ) (10) = − 10 J
2 = [ x3 + y2 ](( 42,, 63 )) = 83 J
(b) From x = 0 to x = 2 m, displacement of particle and force
acting on the particle both are along positive x-direction.
In the above two examples, we saw that while
Therefore, work done is positive and given by the area calculating the work done we did not mention the path
under F - x graph, or through which the object was displaced. Only initial and
1 final coordinates were required. It shows that in both the
W = (2 ) (10) = 10 J
2 examples, the work done is path independent or work done
will be equal on whichever path we follow. Such forces in
6.4 Conservative and which work is path independent are known as conservative
forces.
Non-conservative Force 1 B

Field 2
In Article 6.3, we considered the forces which were
although variable but always directed in one direction. A
However, the most general expression for work done is 3
Fig. 6.11
dW = F ⋅ dr
rf rf Thus, if a particle or an object is displaced from position
and W =∫ dW = ∫ F ⋅ dr A to position B through three different paths under a
ri ri
conservative force field. Then
Here, dr = dx$i + dy$j + dzk$
W1 = W2 = W3
ri = initial position vector
Further, it can be shown that work done in a closed path
and r f = final position vector
is zero under a conservative force field. (WAB = − WBA or
Conservative and non-conservative forces can be better WAB + WBA = 0). Gravitational force, Coulomb’s force are
understood after going through the following two examples. few examples of conservative forces. On the other hand, if
Work, Energy and Power 223

the work is path dependent or W1 ≠ W2 ≠ W3 , the force is ∴ Wnet = ∆KE = K f − K i


called a non-conservative. Frictional forces, viscous forces Let, F1 , F2 , ... be the individual forces acting on a
are non-conservative in nature. Work done in a closed path particle. The resultant force is F = F1 + F2 + ... and the work
is not zero in a non-conservative force field. done by the resultant force is
W = ∫ F ⋅ dr = ∫ ( F1 + F2 + ... ) ⋅ dr
6.5 Kinetic Energy
= ∫ F1 ⋅ dr + ∫ F2 ⋅ dr + ...
Kinetic energy (KE) is the capacity of a body to do work
by virtue of its motion. If a body of mass m has a velocity v where, ∫ F1 ⋅ dr is the work done on the particle by F and
its kinetic energy is equivalent to the work which an external so on. Thus, work energy theorem can also be written as :
force would have to do to bring the body from rest upto its work done by the resultant force is equal to the sum of the
velocity v. The numerical value of the kinetic energy can be work done by the individual forces.
calculated from the formula.
Regarding the work-energy theorem it is worth noting
1
KE = mv 2 that
2 1. If Wnet is positive, then K f − K i = positive, i.e.
This can be derived as follows : K f > K i or kinetic energy will increase and vice-versa.
Consider a constant force F which acting on a mass m 2. This theorem can be applied to non-inertial frames
initially at rest, gives the mass a velocity v. If in reaching also. In a non-inertial frame it can be written as : work
this velocity, the particle has been moving with an done by all the forces (including the pseudo forces)
acceleration a and has been given a displacement s, then = change in kinetic energy in non-inertial frame.
F = ma (Newton’s law) Let us take an example.
v = 2as
2
m
Work done by the constant force = Fs
 v2  1 a
m
or W = ( ma )   = mv 2 f = ma
 2a  2
(a) (b)
But the kinetic energy of the body is equivalent to the
work done in giving the body this velocity.
1 m
Hence, KE = mv 2
2 fp = ma f = ma

Regarding the kinetic energy the following two points (c)


are important to note. Fig. 6.12
1. Since, both m and v 2 are always positive. KE is always
positive and does not depend on the direction of Refer Fig. (a)
motion of the body. A block of mass m is kept on a rough plank moving with
2. Kinetic energy depends on the frame of reference. For an acceleration a. There is no relative motion between block
example, the kinetic energy of a person of mass m and plank. Hence, force of friction on block is f = ma in
sitting in a train moving with speed v is zero in the forward direction.
1 Refer Fig. (b)
frame of train but mv 2 in the frame of the earth.
2 Horizontal forces on the block has been shown from
ground (inertial) frame of reference.
6.6 Work-Energy Theorem If the plank moves a distance s on the ground the block
This theorem is a very important tool that relates the will also move the same distance s as there is no slipping
works to kinetic energy. According to this theorem, between the two.
Work done by all the forces (conservative or Hence, work done by friction on the block (w.r.t.
non-conservative, external or internal) acting on a particle ground) is
or an object is equal to the change in kinetic energy of it. W f = fs = mas
224 Objective Physics Vol. 1

From work-energy principle if v is the speed of block Further, WT = 0, as tension is always perpendicular to
displacement.
(w.r.t. ground).
Wmg = − mgh
KE = W f or Wmg = − mgl (1 − cos θ)
1 Substituting these values in Eq. (i), we get
or mv 2 = mas
2 WF = mgl (1 − cos θ)
or v = 2as / Here, the applied force F is variable. So if we do not apply
the work energy theorem we will first find the magnitude of F
Thus, velocity of block relative to ground is 2as. at different locations and then integrate dW ( = F⋅d r) with
proper limits.
Refer Fig. (c)
Free body diagram of the block has been shown from
accelerating frame (plank). 6.7 Potential Energy
Here, f p = pseudo force = ma The energy possessed by a body or system by virtue of
Work done by all the forces, its position or configuration is known as the potential
energy. For example, a block attached to a compressed or
W = W f + W fp = mas − mas = 0
elongated spring possesses some energy called elastic
From work-energy theorem,
potential energy. This block has a capacity to do work.
1
mv r2 = W = 0 Similarly, a stone when released from a certain height also
2 has energy in the form of gravitational potential energy.
or vr = 0 Two charged particles kept at certain distance has electric
Thus, velocity of block relative to plank is zero. potential energy.
Regarding the potential energy it is important to note
X Example 6.8 An object of mass m is tied to a
string of length l and a variable force F is applied on it that it is defined for a conservative force field only. For
which brings the string gradually at angle θ with the non-conservative forces it has no meaning. The change in
vertical. Find the work done by the force F . potential energy ( dU ) of a system corresponding to a
conservative internal force is given by
 dU 
dU = − F ⋅ dr = − dW F = − 
θ l  dr 
f rf

F
or ∫i dU = − ∫
ri
F ⋅ dr
m rf
or U f −U i = − ∫ F ⋅ dr
Fig. 6.13 ri

We generally choose the reference point at infinity and


Sol. In this case three forces are acting on the object
assume potential energy to be zero there, i.e. if we take
ri = ∞ (infinite) and U i = 0, then we can write
h = l (1 – cos θ)
r
θ l U = − ∫ F ⋅ dr = − W
T ∞

or potential energy of a body or system is the negative of


F
h work done by the conservative forces in bringing it from
mg infinity to the present position.
Fig. 6.14 Regarding the potential energy it is worth noting that
1. Tension (T ), 1. Potential energy can be defined only for conservative
2. Weight (mg ) and forces and it should be considered to be a property of
3. Applied force (F ) the entire system rather than assigning it to any
Using work-energy theorem specific particle.
Wnet = ∆ KE 2. Potential energy depends on frame of reference.
or WT + Wmg + WF = 0 …(i)
as ∆KE = 0 Now, let us discuss three types of potential energies
because Ki = Kf = 0 which we usually come across.
Work, Energy and Power 225

Elastic Potential Energy Extra Knowledge Points


In Article 6.3, we have discussed the spring forces. We ■ Change in potential
have seen there that the work done by the spring force energy is equal to the
1
(ofcourse conservative for an ideal spring) is − 2 k x 2 , when negative of work done by
the spring is stretched or compressed by an amount x from its the conservative force
O
unstretched position. Thus, ( ∆U = − ∆W ). If work Ground
done by the
 1  conservative force is negative change in potential
U = − W = −  − kx 2 
 2  energy will be positive or potential energy of the
system will increase and vice-versa.
1 2 This can be understood by a simple example.
or U = kx (k = spring constant)
2 Suppose a ball is taken from the ground to some
height, work done by gravity is negative, i.e. change in
Note that elastic potential energy is always positive. potential energy should increase or potential energy of
the ball will increase. Which happens so
Gravitational Potential Energy ∆Wgravity = − ve
∴ ∆U = + ve ( ∆U = − ∆W )
The gravitational potential energy of two particles of or Uf − Ui = + ve
masses m1 and m2 separated by a distance r is given by dU
■ F = − , i.e. conservative forces always act in a
mm dr
U = −G 1 2 direction, where
r
potential energy of the
Here, G = universal gravitation constant system is decreased.
F
This can also be shown
N-m 2
= 6.67 × 10 −11 as in figure.
kg 2
If a ball is dropped from a
If a body of mass m is raised to a height h from the certain height. The force on it (its weight) acts in a
direction in which its potential energy decreases.
surface of the earth, the change in potential energy of the
■ Suppose a particle is released from point A with u = 0.
system (earth + body) comes out to be
u=0
mgh A
∆U = (R = radius of the earth)
 h
1 +  h
 R v
B
or ∆U ≈ mgh
if h << R Friction is absent everywhere. Then velocity at B will be
Thus, the potential energy of a body at height h, i.e. mgh v = 2gh
is really the change in potential energy of the system for (irrespective of the track it follows from A to B)
h << R . So be careful while usingU = mgh, that h should not Here, h = hA − hB
be too large. This we will discuss in detail in the chapter of ■ Suppose a car is
moving with constant A
gravitation. v C
speed in the track as
v
shown in figure.
Electric Potential Energy Then, the equations v
of motion are as
The electric potential energy of two point charges q1 under. B
and q 2 separated by a distance r in vacuum is given by mv 2
mv 2
mg − NA = or NA = mg −
1 q q RA RA
U = . 1 2 mv 2 mv 2
4π ε 0 r NB − mg = or NB = mg +
RB RB
1 N -m 2 NC = mg
Here, = 9 . 0 × 10 9 and
4πε 0 C2 Thus, we can say NB > NC > NA
= constant
226 Objective Physics Vol. 1

Solution Friction is absent. Therefore, mechanical


6.8 Law of Conservation of

energy of the system will remain conserved. From


Mechanical Energy constraint relations we see that speed of both the
blocks will be same. Suppose it is v. Here, gravitational
Suppose, only conservative forces operate on a system potential energy of 2 kg block is decreasing while
of particles and potential energy U is defined corresponding gravitational potential energy of 1 kg block is
increasing. Similarly, kinetic energy of both the blocks
to these forces. There are either no other forces or the work is also increasing. So we can write
done by them is zero. We have
Decrease in gravitational potential energy of 2 kg block
Uf −U i = − W = increase in gravitational potential energy of 1 kg
and W = K f − K i (from work energy theorem) block + increase in kinetic energy of 1 kg block +
increase in kinetic energy of 2 kg block.
then U f − U i = − (K f − K i )
1 1
or U f + K f =U i + Ki …(i) ∴ mB gh = mA gh + mAv 2 + mB v 2
2 2
The sum of the potential energy and the kinetic energy is 1 1
or ( 2)(10)(1) = (1)(10)(1) + (1) v + ( 2) v 2
2
called the total mechanical energy. We see from Eq. (i), that 2 2
the total mechanical energy of a system remains constant or 20 = 10 + 0.5 v 2 + v 2
if only conservative forces are acting on a system of or 1.5 v 2 = 10
particles and the work done by all other forces is zero. ∴ v 2 = 6.67 m 2 / s 2
This is called the conservation of mechanical energy. or v = 2.58 m/s
■ If some non-conservative forces such as friction are
The total mechanical energy is not constant, if
also acting on some parts of the system and work done
non-conservative forces such as friction is acting between
by any other forces (excluding the conservative forces)
the parts of a system. However, the work energy theorem is is zero. Then, we can apply
still valid. Thus, we can apply Wnc = E f − E i
Wc + Wnc + Wext = K f − K i or Wnc = (Uf − Ui ) + (Kf − Ki )
Here, Wc = − (U f − U i ) = ∆U + ∆K
i.e. work done by non-conservative forces is equal to
So, we get Wnc + Wext = ( K f + U f ) − ( K i + U i ) the change in mechanical (potential + kinetic) energy.
But note that here all quantities are to be substituted
or Wnc + Wext = E f − E i
with sign. Let us see an example of this.
Here, E = K + U is the total mechanical energy. In the arrangement shown in figure, mA = 1 kg,
mB = 4 kg. String is light and inextensible while pulley is
smooth. Coefficient of friction between block A and the
Extra Knowledge Points table is µ = 0.2 .Find the speed of both the blocks when
block B has descended a height h = 1 m. (Take
■ If only conservative forces are acting on a system of g = 10 m/s 2 )
particles and work done by any other external force is
zero, then mechanical energy of the system will remain A
conserved. In this case some fraction of the
mechanical energy will be decreasing while the other
will be increasing. Problems can be solved by
equating the magnitudes of the decrease and the B
increase. Let us see an example of this.

■ Solution From constraint relation, we see that


v A = vB = v (say)
Force of friction between block A and table will be
f = µmAg = (0.2) (1) (10) = 2 N
A ∴ Wnc = ∆U + ∆K
B 1
∴ −fs = − mB gh + (mA + mB ) v 2
2
1
In the arrangement shown in figure string is light and or ( −2) (1) = − ( 4) (10) (1) + ( 4 + 1) v 2
2
inextensible and friction is absent everywhere. Find the
speed of both the blocks after the block A has −2 = − 40 + 2.5 v 2 or 2.5 v 2 = 38
ascended a height of 1 m. Given that mA = 1 kg and ∴ v 2 =15.2 m 2 /s 2
mB = 2 kg (g = 10 m/s 2 ). or v = 3.9 m / s
Work, Energy and Power 227

dU dU
X Example 6.9 In the So, − = 0, or =0
arrangement shown in dr dr
A
figure m A = 4.0 kg and i.e. at equilibrium position slope of U-r graph is zero or
mB =1.0 kg. The system is the potential energy is optimum (maximum or minimum or
released from rest and constant). Equilibrium are of three types, i.e. the situation
block B is found to have a dU
where, F = 0 and = 0 can be obtained under three
speed 0.3 m/s after it has B dr
descended through a conditions. These are stable equilibrium, unstable
distance of 1 m. Find the Fig. 6.16 equilibrium and neutral equilibrium. These three types of
coefficient of friction between the block and the table. equilibrium can be better understood from the given three
Neglect friction elsewhere. (Take g =10 m/s 2 ) figures

Sol. From constraint relations, we can see that


vA = 2 vB
Therefore, v A = 2(0.3) = 0.6 m/ s
as v B = 0.3 m/s (given)
Applying Wnc = ∆U + ∆K (a) (b)
1 1
we get − µ mA gS A = − mB gS B + mA v A2 + mB v B2
2 2
Here, S A = 2S B = 2 m as S B = 1m (given)
1 1 (c)
∴ −µ(4.0)(10) (2) = −(1)(10)(1) + (4)(0.6) + (1) (0.3)2
2
2 2 Fig. 6.16
or −80 µ = −10 + 0.72 + 0.045
or 80 µ = 9.235 or µ = 0.115 Three identical balls are placed in equilibrium in
positions as shown in Fig. (a), (b) and (c) respectively.
6.9 Three Types of Equilibrium In Fig. (a), ball is placed inside a smooth spherical shell.
This ball is in stable equilibrium position. In Fig. (b), the ball
A body is said to be in translatory equilibrium, if net
is placed over a smooth sphere. This is in unstable
force acting on the body is zero, i.e.
equilibrium position. In Fig. (c), the ball is placed on a
Fnet = 0 smooth horizontal ground. This ball is in neutral equilibrium
If the forces are conservative position.
dU
F =− The table given below explains what is the difference
dr and what are the similarities between these three equilibrium
and for equilibrium F = 0. positions in the language of physics.

Table 6.1
S. No. Stable Equilibrium Unstable Equilibrium Neutral Equilibrium
1. Net force is zero. Net force is zero. Net force is zero.
2. dU dU dU
= 0 or slope of U-r graph is zero. = 0 or slope of U-r graph is zero. = 0 or slope of U-r graph is zero.
dr dr dr

3. When displaced from its equilibrium When displaced from its equilibrium position, a net When displaced from its equilibrium position the
position a net restoring force starts force starts acting on the body which moves the body has neither the tendency to come back nor
acting on the body which has a body in the direction of displacement or away from to move away from the original position.
tendency to bring the body back to its the equilibrium position.
equilibrium position.
4. Potential energy in equilibrium Potential energy in equilibrium position is Potential energy remains contant even, if the
position is minimum as compared to maximum as compared to its neighbouring body is displaced from its equilibrium position
its neighbouring points points d 2U
or =0
d 2U d 2U dr 2
or = positive or = negative
dr 2 dr 2
5. When displaced from equilibrium When displaced from equilibrium position the When displaced from equilibrium position the
position the centre of gravity of the centre of gravity of the body comes down. centre of gravity of the body remains at the same
body goes up. level.
228 Objective Physics Vol. 1

X Example 6.10 The potential energy of a


Extra Knowledge Points conservative system is given by
■ If we plot graphs between F and r or U and r, F will be U = ax 2 − bx
zero at equilibrium while U will be maximum, minimum
or constant depending on the type of equilibrium. This
(where, a and b are positive constants. Find the
all is shown in figure. equilibrium position and discuss whether the
equilibrium is stable, unstable or neutral.
F
Sol. In a conservative field
dU
F=−
dx
r
d
A B C D ∴ F=− (ax2 − bx)
dx
= b − 2 ax
For equilibrium F = 0
U or b − 2 ax = 0
b
∴ x=
2a
C d 2U
B D
r From the given equation we can see that = 2a
A dx2
(positive), i.e. U is minimum.
b
Therefore, x = is the stable equilibrium position.
2a
dU
At point A, F = 0, = 0, but U is constant. Hence, A is
dr
neutral equilibrium position. At points B and D,
6.10 Power
dU Power is the rate at which a force does work. If a force
F = 0, = 0 but U is maximum. Thus, these are the
dr does 20 J of work in 10 s, the average rate at which it is
points of unstable equilibrium. working is 2 J/s or the average power is 2 W.
dU
At point C, F = 0, = 0, but U is minimum. Hence, The work done by a force F in a small displacement dr
dr
point C is in stable equilibrium position. is dW = F ⋅ dr . Thus, the instantaneous power delivered by
the force is
dW
P=
dt
dr
= F⋅ = F⋅ v
dt
= Fv cos θ
Thus, power is equal to the scalar product of force with
(a)
velocity. It is zero, if force is perpendicular to velocity. For
example, power of a centripetal force in a circular motion is
zero.

X Example 6.11 A train has a constant speed of 40


m/s on a level road against resistive force of magnitude
3 × 10 4 N. Find the power of the engine.
Sol. At constant speed, there is no acceleration, so the
(b) forces acting on the train are in equilibrium.
Therefore, F=R
Oscillations of a body take place about stable
equilibrium position. For example, bob of a pendulum ∴ F = 3 × 104 N
oscillates about its lowest point which is also the stable or P = Fv
equilibrium position of bob. Similarly, in Fig. (b), the We have, power = 3 × 104 × 40
ball will oscillate about its stable equilibrium position. = 1.2 × 106 W
Work, Energy and Power 229

1
X Example 6.12 A train of mass 2.0 ×10 5 kg has a F = R + (2 . 0 × 105 ) g ×
50
constant speed of 20 m/s up a hill inclined at 4. 0 × 104 = R + 39200 or R = 800 N
1
θ = sin −1   to the horizontal when the engine is Therefore, the resistance is 800 N.
 50  X Example 6.13 A block of mass m is pulled by a
working at 8 . 0 × 10 5 W. Find the resistance to motion constant power P placed on a rough horizontal plane.
of the train. (g = 9.8 m /s 2 ). The friction coefficient between the block and surface
is µ. Find the maximum velocity of the block.
Sol. Since, P = Fv
P Sol. Power P = F ⋅ v = constant
F= P 1
v F= or F ∝
v v
8. 0 × 105
= as v increases, F decreases.
20 when F = µmg , net force on block becomes zero, i.e. it has
= 4.0 × 104 N maximum or terminal velocity
P
At constant speed, the forces acting on the train are in ∴ P = ( µmg ) vmax or vmax =
µmg
equilibrium. Resolving the forces parallel to the hill

Extra Knowledge Points


■ Work is a scalar quantity. It can be positive, negative or A ball of mass m is released from a height h as shown in
zero. The angle between F and s decides whether the figure. The velocity of particle at the instant when it
work done is positive, negative or zero. strikes the ground can be found using energy
conservation principle by following two methods.
If 0° ≤ θ < 90° ,W = positive
u=0
If θ = 90° , W = 0 and
If 90° < θ ≤ 180° , W = negative h
■ The CGS unit of work is erg.
1 J = 107 erg h=0
Power is also measured in horse power (HP)
v
■ If only conservative forces are acting on a system, the
mechanical energy of the system remains constant. Method 1 Decrease in gravitational PE = increase in KE
1
Mechanical energy comprises of kinetic and potential. In or mgh = mv 2
potential usually gravitational and elastic comes in the 2
question (as far as problems of work, power and energy or v = 2 gh
are concerned). Now it may happen that some part of the Method 2 (PE + KE)i = (PE + KE)f
energy might be decreasing while other part might be For gravitational PE we take ground as the reference
increasing. Energy conservation equation now can be point.
1
written in two ways. ∴ mgh + 0 = 0 + mv 2
First method Magnitude of decrease of energy 2
= magnitude of increase of energy. or v = 2 gh
■ If the system consists of frictional forces as well. Then,
Second method E i = E f (i → initial and f → final)
some mechanical energy will be lost in doing work
i.e. write down total initial mechanical energy on one side against friction
and total final mechanical energy on the other side.
or Ef < Ei
While writing gravitational potential energy we choose
some reference point (where h = 0), but throughout the Now, suppose work done by friction is asked in the
question this reference point should not change. Let us question, then find E f − E i and if work done against
friction is asked then write down E i − E f .
take a simple example.
Chapter Summary with Formulae
■ Work Done
(i) By a constant force
W = F ⋅ s = F ⋅ ( rf − ri ) = Fs cos θ
= Force ×displacement in the direction of force.
(ii) By a variable force
xf
W= ∫ F ⋅ dx,
xi
where F = f ( x )

(iii) By area under F-x graph If force is a function of x, we can find work done by area under F-x graph with projection along x-axis. In
this method magnitude of work done can be obtained by area under F-x graph, but sign of work done should be decided by you. If
force and displacement both are positive or negative, work done will be positive. If one is positive and other is negative, then work
done will be negative.
■ Power of a Force
(i) Average power
Total work done WTotal
Pav = =
Total time taken t
(ii) Instantaneous power
Pins = rate of doing work done
dW
= = F ⋅ v = Fv cos θ
dt
■ Conservative and Non-conservative Forces
In case of conservative forces
(i) Work done is path independent.
(ii) In a closed path net work done is zero.
(iii) Potential energy is defined only for conservative forces.
(iv) If only conservative forces are acting on a system, its mechanical energy should remain constant.
Forces which are not conservative are non-conservative.
■ Potential Energy
(i) Potential energy is defined only for conservative forces.
(ii) In a conservative force field, difference in potential energy between two points is negative of work done by conservative forces in
displacing the body (or system) from some initial position to final position. Hence,
∆U = − W or U B − U A = − WA→ B
(iii) Absolute potential energy at a point can be defined with respect to a reference point where potential energy is assumed to be zero.
Reference point corresponding to gravitational potential energy is assumed at infinity. Reference point corresponding to spring
potential energy is taken at natural length points of spring.
Now, negative of work done in displacement of body from reference point (say O) to the point under consideration (say P) is called
absolute potential energy at P. Thus,
U P = − WO→ P
■ Spring Force
(i) F = − kx or F ∝−x
1
(ii) U x = kx2
2
(iii) F

x
Additional Examples
Loss in KE
Example 1. How much work is done by a coolie ∴ Distance travelled =
walking on a horizontal platform with a load on his Retarding force
head ? As both the kinetic energy and retarding force are same, so
both, lorry and the car would come to rest in the same
Sol. Zero. In order to balance the load on his head, the coolie distance.
applies a force on it in the upward direction equal to its weight.
His displacement is along the horizontal direction. Thus, the Example 7. A small body of mass m is located on a
angle between force F and displacement s is 90°. Therefore horizontal plane at the point O. The body acquires a
work done, W = Fs cos θ = Fs cos 90 ° = 0.
horizontal velocity v 0 due to friction. Find, the mean
Example 2. Can a body have energy without power developed by the friction force during the
momentum ? motion of the body, if the frictional coefficient µ = 0.27,
m = 1.0 kg and v 0 = 1.5 m/s.
Sol. Yes, there may be an internal energy in the body due to the
thermal agitation of the particles of the body. Sol. The body gains velocity due to friction. The acceleration
due to friction.
Example 3. Can a body have momentum without force of friction µmg
a= = = µg
energy ? mass m
Further, v 0 = at
Sol. No, if a body has momentum, it must be in motion and
v v
consequently possess kinetic energy. Therefore, t = 0 = 0 K(i)
a µg
Example 4. A light body and a heavy body have From work energy theorem,
the same momentum. Which one will have greater work done by force of friction = change in kinetic energy
kinetic energy ? 1
or W = mv 02 K(ii)
P2 2
Sol. Kinetic energy, K =
W
2m Mean power =
1 t
For constant P, K ∝ 1
m From Eqs. (i) and (ii), we get Pmean = µmgv 0
Thus, the lighter body has more kinetic energy than the 2
heavier body. Substituting the values, we have
1
Example 5. A light body and a heavy body have the Pmean = × 0.27 × 1.0 × 9.8 × 1.5 ≈ 2.0 W
2
same kinetic energy. Which one will have the greater
momentum? Example 8. An object of mass 5 kg falls from rest
through a vertical distance of 20 m and attaches a
Sol. Momentum,
velocity of 10 m/s. How much work is done by the
P = 2mK resistance of the air on the object?
i.e. P∝ m (g = 10 m/s 2 )
Thus, the heavier body has a greater momentum than the Sol. Applying work-energy theorem,
lighter one. work done by all the forces = change in kinetic energy
1
Example 6. A lorry and a car with the same kinetic or W mg + W air = mv 2
2
energy are brought to rest by the application of the
1
brakes which provide equal retarding force. Which of ∴ W air = mv 2 − W mg
2
them will come to rest in a shorter distance? 1
= mv 2 − mgh
Sol. From work-energy theorem, 2
Loss in KE of the vehicle 1
= × 5 × (10 ) 2 − (5) × (10 ) × (20 )
= Work done against retarding force 2
= Retarding force × distance travelled = − 750 J
232 Objective Physics Vol. 1

Example 9. A rod of length 1.0 m and mass 0.5 kg Example 11. A smooth narrow tube in the form of
fixed at one end is initially hanging, vertical. The other an arc AB of a circle of centre O and radius r is fixed
end is now raised until it makes an angle 60° with the so that A is vertically above O and OB is horizontal.
vertical. How much work is required? Particles P of mass m and Q of mass 2 m with a light
Sol. For increase in gravitational potential energy of a rod we  π r
see the centre of the rod. inextensible string of length   connecting them are
 2 
60°
placed inside the tube with P at A and Q at B and
⇒ C
C released from rest. Assuming the string remains taut
during motion, find the speed of particles when P
reaches B.
W = change in potential energy
l A
= mg (1 − cos θ) P
2
Substituting the values, we have r
 1.0 
W = (0.5) (9.8)   (1 − cos 60 ° )
 2
= 1.225 J O B
Q
Example 10. A small mass m starts from rest and
slides down the smooth spherical surface of R. Assume Sol. All surfaces are smooth. Therefore, mechanical energy of
zero potential energy at the top. Find the system will remain conserved.
(a) the change in potential energy ∴ Decrease in PE of both the blocks
(b) the kinetic energy = increase in KE of both the blocks
(c) the speed of the mass as a function of the angle θ  πr  1
∴ (mgr ) + (2mg)   = (m + 2m)v 2
made by the radius through the mass with the  2 2
vertical. 2
or v= (1 + π )gr
Sol. In the figure, h = R (1 − cos θ) 3

h Example 12. An automobile of mass m accelerates,


θ starting from rest. The engine supplies constant power
P, show that the velocity is given as a function by time
1/ 2
 2Pt 
by v =  
 m
(a) As the mass comes down, potential energy will Sol. Power P = constant
decrease. Hence,
∆U = − mgh = − mgR (1 − cos θ) Work done upto time t is W = Pt
(b) Magnitude of decrease in potential energy = increase From work energy theorem,
in kinetic energy W = ∆KE
∴ Kinetic energy = mgh = mgR (1 − cos θ) or
1
Pt = mv 2
1 2
(c) mv 2 = mgR (1 − cos θ) 1/ 2
2  2Pt 
∴ v= 
∴ v = 2gR (1 − cos θ) m
NCERT Selected Questions
Q 1. The sign of work done by a force on a body is ∴ Net acceleration with which the body moves,
important to understand. State carefully if the F’
a=
following quantities are positive or negative m
5.04
(a) work done by a man in lifting a bucket out of a = = 2.52 ms−2
well by means of a rope tied to the bucket. 2
(b) work done by gravitational force in the above If S be the distance covered by the body in 10 s, then
case. 1
S = at 2 (as u = 0)
(c) work done by friction on a body sliding down an 2
inclined plane. 1
= × 2.52 × (10)2
(d) work done by an applied force on a body moving 2
on a rough horizontal plane with uniform ∴ S = 126 m
velocity. (a) Work done by the applied force,
(e) work done by the resistive force of air on a W 1 = FS = 7 × 126 = 882 J
vibrating pendulum in bringing it to rest. (b) Work done by the force of friction
W2 = − F S
Sol. Work done by a force is given by W = F ⋅ S = FS cosθ,
= − 196
. × 126
where, θ = angle between F (force) and displacement S.
= − 246.96 J
(a) To lift the bucket, force equal to or more than the weight
of the bucket has to be applied vertically upwards and = − 247 J
the bucket moves along the same direction, thus θ = 0, so (c) Work done by the net force W 3 = FS
W = FS is positive. = 5.04 × 126 = 635.04 J
(b) Here, bucket moves in a direction opposite to the (d) According to work energy theorem, change in kinetic
gravitational force which acts vertically downwards. energy = work done or change in KE = 635.04 J
∴ θ = 180°
So W = FS cos180° Q 3. Underline the correct alternative.
= FS (− 1) = − FS (a) When a conservative force does positive work
or W = negative. on a body, the potential energy of the body
(c) Work done by friction on a body sliding down an increases/decreases/ remain unaltered.
inclined plane is negative as friction is in opposite
direction of motion, thus θ = 180° (b) Work done by a body against friction always
∴ W = − FS . results in a loss of its kinetic / potential energy.
(d) As the body moves in the same direction in which the Sol. (a) Potential energy of the body decreases.Because
force is applied, so θ = 0, thus W = FS i.e. it is positive. Work done by conservative forces
(e) Work done is negative as the direction of the resistive W = (– change in PE) = − ∆U .
force of air on the vibrating pendulum is opposite to the
If W = + ve , then ∆U = − ve
direction of displacement (i.e. motion) of the bob.
(b) Work done by a body against friction results in a loss of
Q 2. A body of mass 2 kg initially at rest moves under the its kinetic energy.
action of an applied horizontal force of 7 N on a
Q 4. State if each of the following statements are true or
table with coefficient of kinetic friction = 0.1. false. Give reasons for your answer.
Compute the
(a) Work done in the motion of a body over a closed
(a) work done by the applied force is 10 s. loop is zero for every force in nature.
(b) work done by friction in 10 s.
(b) In an inelastic collision, the final kinetic energy
(c) work done by the net force on the body in 10 s. is always less than the initial kinetic energy of
(d) change in kinetic energy of the body in 10 s and the system.
interpret your results.
Sol. (a) False. Work done in the motion of a body over a closed
Sol. Force of friction, f = uk mg loop is zero only when the body moves under the action
or conservative force.
= 0.1 × 2 × 9.8 = 1.96 N
(b) True. Usually but not always, because in an inelastic
∴ Net force F ′ = F − f
collision, some kinetic energy usually changes into
= (7 − 1.96) N = 5.04 N some other forms of energy.
234 Objective Physics Vol. 1

Q 5. A body is moving unidirectionally under the Q 7. A rain drop of radius 2 mm falls from a height of
influence of a source of constant power. Its 500 m above the ground . It falls with decreasing
displacement in time t is proportional to acceleration (due to viscous resistance of the air)
(a) t 1\ 2 (b) t until at half its original height, it attains its
3/ 2 maximum (terminal) speed, and moves with
(c) t (d) t 2 uniform speed thereafter. What is the work done by
Sol. Let, the constant power P acts on the body of mass m for a the gravitational force on the drop in the first and
time t to give it a velocity v. second half of its journey? What is the work done by
the resistive force in the entire journey, if its speed
∴ Its KE = work done = power × time
on reaching the ground is 10 ms −1 ?
1 2
or mv = Pt
2 Sol. Distance travelled in the half journey,
H 500
or v=
2Pt
=
2P 1/ 2
t …(i) S = = = 250m
m m 2 2
4
Also we know that ∴ Volume of the drop, V = πr3
3
dx
v= or
4
V = × π × (2 × 10−3 )3
dt 3
or dx = vdt …(ii) = 3.35 × 10−8 kg
If x be the displacement of the body, then ∴ If m be the mass of the rain drop, then
x = ∫ dx = ∫ v dt m = Vρ = 3.35 × 10−8 × 103
2P 1/ 2 = 3.35 × 10−5 kg
m∫
= t dt
∴ Gravitational force on the drop,
 1 + 1 F = mg = 3.35 × 10−5 × 9.8 N
2P  t 2 
=   ∴Work done by the gravitational force on the drop in the
m  1 + 1
2  first half of its journey is given by
W = FS = 3.35 × 10−5 × 9.8 × 250 J
2P t 3/ 2
= . = 0.082 J
m  3
 
 2 Since, the distance travelled in the second half is also same,
therefore, work done in the second half is also same i.e.
2 2P 3/ 2
= t 0.082 J.
3 m Let us assume that the rain drop is initially at rest
Here, P = constant and m is also constant for a body, i.e. u=0
so x = constant × t 3/ 2 or x ∝ t 3/ 2. Final velocity on hitting the ground
v = 10 ms−1
Q 6. A body constrained to move along z-axis of a
coordinate system is subjected to a constant force F ∴ Change in KE of the rain drop,
1 1
given by ∆Ek = mv 2 − mu2
2 2
F = − $i + 2$j + 3k$ N
1 2
where $i , $j , and k$ are the unit vector along the x, y = mv − 0
2
and z-axis of the system respectively. What is the 1
work done by this force in moving the body a = × 3.35 × 10−5 × (10)2
2
distance of 4 m along the z-axis? = 1675
. × 10−3 J = 0.00167 J
Sol. Since, the body is displaced 4 m along z-axis only Work done by gravitational force
∴ S = 4 k$ = mgh
Also F = − i$ + 2$j + 3k$ = 3.35 × 10−5 × 9.8 × 500
= 016415
. J
∴Work done is given by
Work done by the resistive force in the entire journey
W = F ⋅S
= change in KE – work done by gravitational force
= (− $i + 2$j + 3k$ ).(0i$ + 0$j + 4 k$ ) = 0.001675 − 016415
. = − 0163
. J
= (− 1) × 0 + 2 × 0 + 3 × 4 = 12 J i.e. the work done by the resistive force is negative.
Work, Energy and Power 235

Q 8. A pump on the ground floor of a building can pump 19 × 9.8 × 1.5


3
v=
up water to fill a tank of value 30 m in 15 min. If the 10
tank is 40 m above the ground and the efficiency of = 27.93
the pump is 30%, how much electric power is = 5.285 ms−1
consumed by the pump?
or v = 5.3 ms−1
Sol. Mass of water to be pumped up,
m = Volume × density of water Q 10. A person trying to loose weight (dieter) lifts a 10 kg
= 30 × 10 = 3 × 10 kg
3 4 mass 0.5 m 1000 times. Assume that the potential
energy lost each time she lowers the mass is
Height of the tank, h = 40 m dissipated.
∴Work done by the pump to fill the tank, (a) How much work does she do against the
W = mgh = 3 × 104 × 9.8 × 40 J gravitational force?
= 1176
. × 107 J (b) Fat supplies 3.8 ×107 J of energy per kg, which
Time, t = 15 min = 15 × 60 s = 900 s is converted to mechanical energy with a 20%
efficiency rate. How much fat will the dieter use
∴Required average power,
up?
W 1.176 × 107
P= = = 13.07 kW Sol. (a) Work done against gravitational force
t 900
Percentage efficiency of pump = 30% = mngh
Output power = 10 × 1000 × 9.8 × 0.5
Now η% = × 100
Input power = 49000 J
or Input power = power consumed by the pump (b) Energy supplied by fat per kg = 3.8 × 107 J
Output power i.e. 3.8 × 107 J energy is equivalent to 1 kg of fat
= × 100
η% ∴ 49 × 103 J energy will be equivalent to
13.07
= × 100 = 43.55 kW 49 × 103
30 = kg fat
3.8 × 107
= 43.6 kW
49
Fat produced = × 10−4 kg
Q 9. The bob of a pendulum is released from a horizontal 3.8
position A as shown in the figure. If the length of the η% = 20%
pendulum is 1.5m, what is the speed with which the Fat produced
bob arrives at the lower most point B, given that it ∴ η% = × 100
Fats used up by dieter
dissipated 5% of its initial energy against air
resistance? 10−4 100
or 20 = 49 × ×
3.8 Fats used up by dieter
Sol. At point A, the energy of the pendulum is entirely PE. At
point B, the energy of the pendulum is entirely KE. It means or fats used up by the dieter
that as the bob of the pendulum lowers from A to B, PE is 49 × 10−4 100
= ×
converted into KE. Thus, at B, KE = PE. But 5% of the PE is 3.8 20
dissipated against air resistance. 245 −4
A = × 10
O 1.5 m 3.8
245
= × 10−3 kg
38
1.5 m = 6.447 × 10−3 kg
= 6.45 × 10−3 kg

Q 11. A large family uses 8 kW of power.


B
(a) Direct solar energy is incident on the horizontal
KE at B = 95% of PE at A …(i) surface at an average rate of 200 W/ m 2 . If 20%
1 95
∴ From Eq.(i), mv 2 = mgh of this energy can be converted to useful
2 100
95 electrical energy, how large an area is needed to
or v2 = 2 × gh supply 8 kW?
100
95 (b) Compare this area to that of the roof of a typical
=2× × 9.8 × 15
.
100 house.
236 Objective Physics Vol. 1

Sol. Total power used by the family = 8k W = 8000 W Q 12. A bolt of mass 0.3 kg falls from the ceiling of an
(a) Average rate of incidence of the direct solar energy on elevator moving down with a uniform speed of
the horizontal surface = 200 W/m 2 7 ms −1 . It hits the floor of the elevator (length of the
Energy converted into useful electrical energy elevator = 3 m) and does not rebound.
20
= 20% of 200W/m 2 = × 200 = 40W/m 2
100 What is the heat produced by the impact? Would
Total power required by family = 8000 W your answer be different, if the elevator were
∴ Area required to produce the same amount of stationary?
8000
electrical energy = = 200 m 2 Sol. Potential energy of the bolt at the ceiling
40
(b) In order to compare this area to that of the roof of a = mgh
typical house, let a be the side of the roof = 0.3 × 9.8 × 3
∴ Area of roof = a × a = a2 = 8.82 J
Thus, a2 = 200 m 2 The bolt does not rebound, so the whole of the potential
or a = 200 m 2 = 14.14 m energy is converted into heat according to the law of
conservation of energy.
∴ Area of roof = 14.14 × 14.14 m 2
Since, the value of acceleration due to gravity is the same in
Thus, 200 m 2 is comparable to the roof of a large house all inertial systems, therefore the answer will not change
of dimensions 14.14 m × 14.14 m = 14 m × 14 m even, if the elevator is stationary or moving.
Objective Problems
[ Level 1 ]
Work Done by Different Forces 9. A ball is released from the top of a tower. The ratio of
work done by force of gravity in first, second and third
1. A particle moves under a force f = cx from x = 0 to
second of the motion of ball is
x = x1 . The work done is (a) 1: 2: 3
cx 2 (b) 1: 4 :16
(a) cx12 (b) 1
2 (c) 1: 3: 5
(c) zero (d) cx13 (d) 1: 9: 25

2. A particle moves along the x-axis from x = 0 to x = 5 m 10. An object of mass 5 kg is acted upon by a force that varies
under the influence of a force given by F = 7 − 2x + 3x 2 . with position of the object as shown. If the object starts
out from rest at a point x = 0. What is its speed at x = 50m?
Work done in the process is
F(N)
(a) 70 (b) 270
(c) 35 (d) 135

3. A body constrained to move in the y -direction, is


10
subjected to a force F = ( − 2$i + 15$j + 6k$ ) N. What is the
work done by this force in moving the body through a
x(m)
distance of 10 m along the y -axis? 25 50
(a) 20 J (b) 150 J (a) 12.2 ms−1 (b) 18.2 ms−1
(c) 160 J (d) 190 J
(c) 16.4 ms−1 (d) 20.4 ms−1
4. Work done by a force F = ( i$ + 2$j + 3k$ ) N acting on a
11. The force F acting on a particle is moving in a straight
particle in displacing it from the point r = i$ + $j + k$ to
1 line as shown in figure. What is the work done by the
the point r 2 = i$ − $j + 2k$ is force on the particle in the 1 m of the trajectory?
(a) 3J (b) −1 J F(N)
(c) zero (d) 2J
5
5. A mass M is lowered with the help of a string by a
g
distance x at a constant acceleration . The magnitude of
2
work done by the string will be
1 x(m)
(a) Mgx (b) Mgx 2 1 3 4
2
1 (a) 5 J (b) 10 J
(c) Mgx (d) Mgx 2
2 (c) 15 J (d) 2.5 J

6. The work done by pseudo forces is 12. A position dependent force, F = 7 − 2x + 3x 2 N acts on a
(a) positive (b) negative small body of mass 2 kg and displaces it from x = 0 to
(c) zero (d) All of these x = 5 m. The work done in joule is
7. The work done by a spring force (a) 35 (b) 70
(a) is always negative (c) 135 (d) 270
(b) is always positive
13. Mark out the correct statement (s).
(c) is always zero
(d) may be positive and negative (a) Total work done by internal forces on a system is always
zero
8. A force ( 3$i + 4$j ) N acts on a body and displaces it by (b) Total work done by internal forces on a system may
sometimes be zero
( 3i$ + 4$j ) m. The work done by the force is
(c) Total work done by friction may sometimes be zero
(a) 10 J (b) 12 J (c) 16 J (d) 25 J (d) Total work done by friction is always zero
238 Objective Physics Vol. 1

14. A body of mass 1 kg moves from points A (2m, 3m, 4m,) 22. A long spring is stretched by 2 cm. Its potential energy is
to B (3m, 2m, 5m,). During motion of body, a force U. If the spring is stretched by 10 cm, its potential energy
F = ( 2N )$i − ( 4N )$j acts on it. The work done by the force would be
U U
on the particle during displacement is (a) (b) (c) 5U (d) 25U
25 5
(a) 6 J (b) 2 J
(c) – 2 J (d) – 6 J 23. An engine develops 10 kW of power. How much time
will it take to lift a mass of 200 kg to a height of 40 m?
15. A force F = Ay 2 + By + C acts on a body in the ( g = 10ms −2 ).
y -direction. The work done by this force during a (a) 4 s (b) 5 s (c) 8 s (d) 10 s
displacement from y = − a to y = a is
2 Aa3 2 Aa3 24. Velocity-time graph of a particle of mass 2 kg moving in
(a) (b) + 2Ca a straight line is as shown in figure. Work done by all the
3 3
2 Aa3 Ba2 forces on the particle is
(c) + + Ca (d) None of these v (m/s)
3 2
20
16. Force acting on a particle is ( 2$i + 3$j ) N . Work done by
this force is zero, when a particle is moved on the line
3 y + kx = 5. Here, value of k is
(a) 2 (b) 4 t(s)
2
(c) 6 (d) 8
(a) 400 J (b) – 400 J
17. A block of mass 10 kg is moving in x-direction with a (c) – 200 J (d) 200 J
constant speed of 10 m/ s. It is subjected to a retarding
25. A vehicle needs an engine of 7500 W to keep it moving
force F = ( − 0.1 x ) N during its travel from x = 20 m to
with a constant velocity of 20 m/s on a horizontal surface.
x = 30 m. Its final kinetic energy will be The force resisting the motion is
(a) 475 J (b) 450 J
(a) 375 dyne (b) 375 N
(c) 275 J (d) 250 J (c) 150000 dyne (d) 150000 N

Potential Energy, Kinetic Energy, 26. A mass of 1 kg is acted upon by a single force
Power, Work Energy Theorem and F = ( 4i$ + 4$j ) N. Under this force it is displaced from
(0, 0) to (1m, 1m). If initially the speed of the particle was
Types of Equilibrium 2m/s, its final speed should be
18. Under the action of a force, a 2 kg body moves such that (a) 6 m/s (b) 4.5 m/s
t3 (c) 8 m/s (d) 4 m/s
its position x as a function of time t is given by x = ,
3 27. A particle moves with a velocity ( 5$i − 3$j + 6k$ ) m/ s
where, x is in metre and t in second. The work done by the under, the influence of a constant force
force in the first two seconds is F = (10i$ + 10$j + 20k$ ) N. The instantaneous power
(a) 1600 J (b) 160 J applied to the particle is
(c) 16 J (d) 1.6 J
(a) 200 J/s (b) 40 J/s
19. If the speed of a vehicle increase by 2m/ s, its kinetic (c) 140 J/s (d) 170 J/s
energy is doubled, then original speed of the vehicle is 28. Power applied to a particle varies with time as
(a) ( 2 + 1) m / s (b) 2( 2 − 1) m / s P = ( 3t 2 − 2t + 1) W, there t is in second. Find the change
(c) 2( 2 + 1) m / s (d) 2 ( 2 + 1) m / s in its kinetic energy between time t = 2 s and t = 4 s.
(a) 32 J (b) 46 J (c) 61 J (d) 102 J
20. Unit of power is
(a) kilowatt hour (b) kilowatt per hour 29. A particle is moving in a conservative force field from
(c) kilowatt (d) erg point A to point B.U A and U B are the potential energies
of the particle at point A and B and WC is the work done
21. When work done by force of gravity is negative
by conservative forces in the process of taking the
(a) PE increase
particle from A and B.
(b) KE decreases
(c) PE remains constant (a) WC = U B − U A (b) WC = U A − U B
(d) PE decreases (c) U A > U B (d) U B > U A
Work, Energy and Power 239

30. A box is moved along a straight line by a machine 37. A particle moves in a straight line with retardation
delivering constant power. The distance moved by the proportional to its displacement. Its loss of kinetic energy
body in time t is proportional to for any displacement x is proportional to
(a) t 1/ 2 (b) t 3/ 4 (c) t 3/ 2 (d) t 2 (a) x (b) x 2
(c) ln x (d) ex
31. In which of the following cases, the potential energy is
defined? 38. A body of mass 5 kg is raised vertically to a height of
(a) Both conservative and non-conservative forces 10 m by a force 170 N. The velocity of the body at this
(b) Conservative force only height will be
(c) Non-conservative force only (a) 9.8 m/s (b) 15 m/s
(d) Neither conservative nor-conservative forces (c) 22 m/s (d) 37 m/s

32. A particle of mass 0.01 kg travels with velocity given by 39. A body of mass m thrown vertically upwards attains a
4$i + 16k$ ms −1 . After sometime, its velocity becomes maximum height h. At what height will its kinetic energy
be 75% of its initial value?
8$i + 20$j ms −1 . The work done on particle during this h h
(a) (b)
interval of time is 6 5
(a) 0.32 J (b) 6.9 J (c) 9.6 J (d) 0.96 J h h
(c) (d)
4 3
33. A running man has half the KE that a boy of half his
mass. The man speeds up by 1 ms −1 and then has the 40. A particle of mass m accelerating uniformly has velocity
same KE as that of boy. The original speeds of man and v at time t 1 . What is work done in time t?
boy in ms −1 are
2
1 mv 2 2 1  m 2
(a) t (b)   t
(a) ( 2 + 1),( 2 − 1) (b) ( 2 + 1), 2( 2 + 1) 2 t12 2  t1 
(c) 2, 2 (d) ( 2 + 1), 2( 2 − 1) mv 2 2 2mv 2 2
(c) t (d) t
t12 t12
34. A motor drives a body along a straight line with a
constant force. The power P developed by the motor must 41. A mass of 1 kg is acted upon by a single force
vary with time t as F = ( 4i$ − 4$j ) N. Under this force it is displaced from
(0, 0) to (1m, 1m). If initially, the speed of the particle
was 2 m/s, its final speed is approximately
(a) P (b) P (a) 6.4 m/s (b) 4.5 m/s
(c) 8.6 m/s (d) 7.2 m/s

42. An engine exerts a force F = ( 20i$ − 3$j + 5k$ ) Nand moves


t t
with velocity v = ( 6i$ + 20$j − 3k$ ) m/s. The power of the
engine (in watt) is
(a) 45 (b) 75
(c) P (d) P
(c) 20 (d) 10

43. An object of mass m, initially at rest under the action of a


t t
constant force F attains a velocity v in time t. Then, the
average power supplied to mass is
35. A 50 kg girl is swinging on a swing from rest. Then, the
power delivered when moving with a velocity of 2 m/s mv 2
(a)
upwards in a direction making an angle 60° with the 2t
Fv
verticle is (b)
2
(a) 980 W (b) 490 W (c) 490 3 W (d) 245 W
(c) Both are correct
36. Power supplied to a particle of mass 2 kg varies with time (d) Both are wrong
3t 2 44. A force F acting on a body depends on its displacement S
as P = W. Here, t is in second. If velocity of particle
2 as F ∝ S −1/ 3 . The power delivered by F will depend on
at t = 0 is v = 0. The velocity of particle at time t = 2 s displacement as
will be (a) S 2/ 3 (b) S −5/ 3
(a) 1 m/s (b) 4 m/s (c) 2 m/s (d) 2 2 m/s (c) S 1/ 2 (d) S 0
240 Objective Physics Vol. 1

45. The force acting on a body moving along x-axis varies 50. A toy gun uses a spring of very large value of force
with the position of the particle as shown in the figure. constant k. When charged before being triggered in the
The body is in stable equilibrium at upward direction, the spring is compressed by a small
F distance x. If mass of shot is m, on being triggered it will
go upto a height of
kx 2 x2
(a) (b)
mg kmg
x1 x2 x
kx 2 (kx )2
(c) (d)
2mg mg
(a) x = x1 (b) x = x2
(c) Both x1 and x2 (d) Neither x1 nor x2 51. A body is falling under gravity. When it loses a
gravitational potential energy by U, its speed is v. The
46. The given plot shows the variation of U, the potential mass of the body shall be
energy of interaction between two particles with the 2U U
distance separating them r. (a) (b)
v 2v
U 2U U
(c) 2 (d)
v 2v 2

E 52. A bead can slide on a smooth circular wire frame of


radius r which is fixed in a vertical plane. The bead is
B F
displaced slightly from the highest point of the wire
D
C frame. The speed of the bead subsequently as a function
of the angle θ made by the bead with the vertical line is
1. B and D are equilibrium points.
(a) 2gr (b) 2gr(1 − sin θ )
2. C is a point of stable equilibrium.
3. The force of interaction between the two particles is (c) 2gr(1 − cosθ ) (d) 2 gr
attractive between points C and D and repulsive
between D and E. 53. A body is moved along a straight line with constant
4. The force of interaction between particles is repulsive power. The distance moved by the body in time t is
between points E and F. proportional to
Which of the above statements are correct? (a) t 3/ 2 (b) t 3/ 2
(a) 1 and 2 (b) 1 and 4 (c) t 1/ 4 (d) t 3
(c) 2 and 4 (d) 2 and 3 54. A particle is released from a height H. At certain height
its kinetic energy is two times its potential energy. Height
Smooth Surfaces where Mechanical and speed of particle at that instant are
Energy remains Conserved (a)
H 2gH
, (b)
H
,2
gH
47. Three particles A, B and C are projected from the top of a 3 3 3 3
tower with the same speed. A is thrown straight upwards 2H 2gH H
(c) , (d) , 2gH
B straight down and C horizontally. They hit the ground 3 3 3
with speeds v A , v B and vC , then which of the following 55. A uniform chain has a mass M and length L. It is placed
is correct? on a frictionless table with length l0 hanging over the
(a) vA = vB > vC (b) vA = vB = vC edge. The chain begins to slide down . Then, the speed v
(c) vA > vB = vC (d) vB > vC > vA with which the end slides down from the edge is given by
48. A body of mass 0.1 kg moving with a velocity of 10 m/s g g
(a) v = (L + l0 ) (b) v = (L − l0 )
hits a spring (fixed at the other end) of force constant L L
1000 N/ m and comes to rest after compressing the spring. g 2
(c) v = (L − l02 ) (d) v = 2g (L − l0 )
The compression of the spring is L
(a) 0.01 m (b) 0.1 m
(c) 0.2 m (d) 0.5 m 56. If a body of mass 200 g falls from a height 200 m and its
total potential energy is converted into kinetic energy, at
49. A mass of 2 kg falls from a height of 40 cm on a spring the point of contact of the body with the surface, then
with a force constant of 1960 N/m. The spring is decrease in potential energy of the body at the contact is
compressed by (take g = 9.8 m/ s 2 ) ( g = 10 m/ s 2 )
(a) 10 cm (b) 1.0 cm (a) 900 J (b) 600 J
(c) 20 cm (d) 5 cm (c) 400 J (d) 200 J
Work, Energy and Power 241

57. A stone of mass 2 kg is projected upwards with KE of 62. The net work done by kinetic friction
98 J. The height at which the KE of the body becomes (a) is always negative
half its original value, is given by (Take g = 9.8 m/ s 2 ) (b) is always zero
(c) may be negative and positive
(a) 5 m (b) 2.5 m
(d) is always positive
(c) 1.5 m (d) 0.5 m
63. A block of mass m is pulled along a horizontal surface by
58. The system shown in the figure is released from rest. At
applying a force at an angle θ with the horizontal. If the
the instant when mass M has fallen through a distance h,
block travels with a uniform velocity and has a
the velocity of m will be
displacement d and the coefficient of friction is µ, then
the work done by the applied force is
F
M>m
θ

µmgd
(a)
m M
cosθ + µ sin θ
µmgd cosθ
(b)
(a) 2gh (b)
2ghM cosθ + µ sin θ
m µmgd sin θ
(c)
2gh (M − m) 2gh (M + m) cosθ + µ sin θ
(c) (d)
m+ M m−M µmgd cosθ
(d)
cosθ − µ sin θ
59. If v be the instantaneous velocity of the body dropped
from the top of a tower, when it is located at height h, 64. A horizontal force F pulls a 20 kg box at a constant
then which of the following remains constant? speed along a rough horizontal floor. The coefficient of
v2 v2 friction between the box and the floor is 0.25. The work
(a) gh + v 2 (b) gh + (c) gh − (d) gh − v 2
2 2 done by force F on the block in displacing it by 2 m is
(a) 49 J (b) 98 J
60. A 2.0 kg block is dropped from a height of 40 cm onto a (c) 147 J (d) 196 J
spring of spring constant k = 1960N/ m. Find the
maximum distance the spring is compressed. 65. The coefficient of friction between the block and plank is
µ and its value is such that block becomes stationary with
2.0 kg respect to plank before it reaches the other end. Then,
40cm
m v0

M
k = 1960 N/m

(a) the work done by friction on the block is negative


(b) the work done by friction on the plank is positive
(a) 0.080 m (b) 0.20 m (c) 0.40 m (d) 0.10 m (c) the net work done by friction is negative
(d) the net work done by the friction is zero
Rough Surfaces where Mechanical
66. A plank of mass 10 kg and a block of mass 2 kg are
Energy does not remain Constant placed on a horizontal plane as shown in the figure. There
61. A pendulum of length 2 m left at A. When it reaches B, it is no friction between plane and plank. The coefficient of
loses 10% of its total energy due to air resistance. The friction between block and plank is 0.5. A force of 60 N is
velocity at B is applied on plank horizontally. In first 2 s the work done
A by friction on the block is

2 kg

10 kg 60 N
B

(a) 6 m/s (b) 1 m/s (a) – 100 J (b) 100 J


(c) 2 m/s (d) 8 m/s (c) zero (d) 200 J
242 Objective Physics Vol. 1

67. A block of mass 5 kg slides down a rough inclined Then,


surface. The angle of inclination is 45°. The coefficient y
of sliding friction is 0.20 . When the block slides 10 cm, P (a, a)
the work done on the block by force of friction is
1
(a) − J (b) 1J
2
(c) − 2 J (d) −1J
45°
68. A block of mass 1 kg slides down a rough inclined plane x
O Q
of inclination 60° starting from its top. If coefficient of
(a) W 1 = W 2 (b) W 1 = 2W 2
kinetic friction is 0.5 and length of the plane d = 1m, then
(c) W 2 = 2W 1 (d) W 2 = 4W 1
work done against friction is
(a) 2.45 J (b) 4.9 J 74. Two masses of 1 g and 4 g are moving with equal kinetic
(c) 9.8 J (d) 19.6 J energies. The ratio of the magnitudes of their momenta is
(a) 4 : 1 (b) 2 : 1
69. A particle moves on a rough horizontal ground with some
th (c) 1 : 2 (d) 1 : 16
 3
initial velocity say v 0 . If   of its kinetic energy is lost
 4 75. A uniform chain of length L and mass M is lying on a
due to friction in time t 0 , then the coefficient of friction smooth table and one third of its length is hanging
between the particle and the ground is vertically down over the edge of the table . If g is
v0 v0 acceleration due to gravity, the work required to pull the
(a) (b)
2gt0 4 gt0 hanging part on the table is
3v0 v MgL
(c) (d) 0 (a) MgL (b)
4 gt0 gt0 3
MgL MgL
(c) (d)
9 18
Miscellaneous Problems
70. If the linear momentum is increased by 50%, then kinetic 76. The net work done by the tension in the figure, when the
energy will be increased by bigger block of mass M touches the ground is
(a) 50 % (b) 100 %
(c) 125 % (d) 25 %
71. A rod of mass mand length l is lying on a horizontal table.
Work done in making it stand on one end will be
mgl
(a) mgl (b) T
2 m
mgl
(c) (d) 2mgl
4
M
72. A body of mass mwas slowly pulled up the hill by a force d
F which at each point was directed along the tangent of
the trajectory. All surfaces are smooth. Find the work
performed by this force. (a) + Mgd (b) − (M + m)gd
(c) − mgd (d) zero

77. An object of mass m is tied to a string of length L and a


variable horizontal force is applied on it which starts at
h zero and gradually increases until the string makes an
F
m angle θ with the vertical. Work done by the force F is
l
(a) mgl (b) – mgl θ L
(c) mgh (d) zero
73. A particle is moved from (0, 0) to ( a, a ) under a force F
m
F = ( 3$i + 4$j ) from two paths. Path 1 is OP and path 2 is
(a) mgL (1 − sin θ ) (b) mgL
OQP. Let W1 and W2 be the work done by this force in
(c) mgL (1 − cosθ ) (d) mgL (1 + cosθ )
these two paths.
Work, Energy and Power 243

78. A particle at rest on a frictionless table is acted upon by a 83. A body is attached to the lower end of a vertical spiral
horizontal force which is constant in magnitude and spring and it is gradually lowered to its equilibrium
direction. A graph is plotted of the work done on the position. This stretches the spring by a length d. If the
particle W , against the speed of the particle v. If there are no same body attached to the same spring is allowed to fall
frictional forces acting on the particle, the graph will look suddenly, what would be the maximum stretching in this
like case?
(a) d (b) 2d
1
(c) 3d (d) d
W 2
(a) W (b)
84. The pointer reading versus load graph for a spring
balance is as shown
v v
Extension (cm)

(c) W (d) W
10

v v
Load (kgf)
79. The energy required to accelerate a car from rest to 10 ms 1.0
is W. The energy required to accelerate the car from
The spring constant is
10 ms −1 to 20 ms −1 is 15 kgf 5 kgf 0.1 kgf 10 kgf
(a) W (b) 2 W (a) (b) (c) (d)
cm cm cm cm
(c) 3 W (d) 4 W
85. v -t graph of an object 20
80. Given that the displacement of the body in metre is a
of mass 1 kg is shown
function of time as follows
in figure. Select the
x = 2t 4 + 5 wrong statement. 10

The mass of the body is 2 kg. What is the increase in its (a) Work done on the
object in 30 s is zero v (m/s)
kinetic energy one second after the start of motion?
(b) The average
(a) 8 J (b) 16 J acceleration of the
(c) 32 J (d) 64 J 10 20 30
object is zero t (s)
81. The momentum of a body is P and its kinetic energy is E. (c) The average velocity
of the object is zero
Its momentum becomes 2P. Its kinetic energy will be
(d) The average force on the object is zero
E
(a) (b) 3E
2 86. If v, p and E denote velocity, linear momentum and KE of
(c) 2E (d) 4E the particle, then
82. A body moves from rest with a constant acceleration. dE dE dv dE dE
(a) p = (b) p = (c) p = (d) p = ×
Which one of the following graphs represents, the dv dt dt dv dt
variation of its kinetic energy K with the distance 87. The force required to stretch a spring varies with the
travelled x ? distance as shown in the figure . If the experiment is
K K
performed with the above spring of half the length, the
line OA will
(a) (b) F

x x A
O O
K K

(c) (d) x
O
(a) shift towards F-axis (b) shift towards x-axis
x x
O O (c) remain as it is (d) become double in length
244 Objective Physics Vol. 1

88. Kinetic energy of a particle moving in a straight line 90. A ball is thrown vertically upwards with a velocity of
varies with time t as K = 4t 2 . The force acting on the 10 m/s. It returns to the ground with a velocity of 9 m/s. If
g = 9.8 m/s 2 , then the maximum height attained by the
particle
(a) is constant ball is nearly (assume air resistance to be uniform)
(b) is increasing (a) 5.1 m (b) 4.1 m (c) 4.61 m (d) 5.0 m
(c) is decreasing
(d) first increases and then decreases 91. An open knife edge of mass m is dropped from a height h
on a wooden floor. If the blade penetrates upto the depth
89. A car moving with a speed of 40 km/h can be stopped by d into the wood, the average resistance offered by the
applying brakes atleast 2m. If the car is moving with a wood to the knife edge is
speed of 80 km/h, the minimum stopping distance is 2
 h  h
(a) 8m (a) mg 1 +  (b) mg 1 + 
(b) 2m  d  d
(c) 4m  h  d
(c) mg 1 −  (d) mg 1 + 
(d) 6m  d  h

[ Level 2 ]
Only One Correct Option
1. A small body of mass m slides without friction from the 4. A uniform flexible chain of mass m and length l hangs in
top of a hemisphere of radius r. At what height will the equilibrium over a smooth horizontal pin of negligible
body be detached from the centre of the hemisphere? diameter. One end of the chain is given a small vertical
displacement so that the chain slips over the pin. The
h speed of chain when it leaves pin is
gl
(a) (b) gl
r r 2
(a) h = (b) h =
2 3 (c) 2gl (d) 3gl
2r r
(c) h = (d) h =
3 4 5. The potential energy of a particle of mass 1 kg is,
U = 10 + ( x − 2) 2 . Here,U is in joule and x in metre on the
2. A particle of mass 1 g executes an oscillatory motion on
the concave surface of a spherical dish of radius 2 m positive x-axis. Particle travels upto x = + 6m. Choose the
placed on a horizontal plane. If the motion of the particle correct statement.
begins from a point on the dish at a height of 1 cm from (a) On negative x-axis particle travels upto x = − 2 m
the horizontal plane and the coefficient of friction is 0.01, (b) The maximum kinetic energy of the particle is 16 J
the total distance covered by the particle before it comes (c) Both (a) and (b) are correct
(d) Both (a) and (b) are wrong
to rest, is approximately
(a) 2.0 m (b) 10.0 m 6. A body is moving down an inclined plane of slope 37°.
(c) 1.0 m (d) 20.0 m
The coefficient of friction between the body and plane
3. A mass-spring system oscillates such that the mass moves varies as µ = 0.3x, where x is the distance travelled down
on a rough surface having coefficient of friction µ. It is the plane by the body. The body will have maximum
compressed by a distance a from its normal length and on  3
speed. sin 37° =  ( g = 10 m/ s 2 )
being released, it moves to a distance b from its  5
equilibrium position. The decrease in amplitude for one
(a) At x = 116
. m
half-cycle ( −a to b ) is (b) At x = 2m
µmg 2 µmg µg K (c) At bottom of plane
(a) (b) (c) (d)
K K K µmg (d) At x = 2.5 m
Work, Energy and Power 245

7. A force of F = 0.5 N is applied on lower block as shown 11. An ideal massless spring S can be compressed 1 m by a
in figure . The work done by lower block on upper block force of 100 N in equilibrium. The same spring is placed
for a displacement of 3 m of the upper block with respect at the bottom of a frictionless inclined plane inclined at
to ground is (Take g = 10 m/ s 2 ) 30° to the horizontal. A 10 kg block M is released from
rest at the top of the incline and is brought to rest
µ = 0.1
momentarily after compressing the spring by 2 m. If
1 kg g = 10 m/ s 2 , what is the speed of mass just before it
touches the spring?
2 kg F
M
Smooth
S
(a) − 0.5 J (b) 0.5 J
(c) 2 J (d) – 2 J 30°

8. The potential energy between the atoms in a molecule is (a) 20 m/s (b) 30 m/s
given by (c) 10 m/s (d) 40 m/s

a b 12. A pendulum of mass 1 kg and length l = 1 m is released


U (x ) = −
x x 12 6 from rest at angle θ = 60°. The power delivered by all the
where, a and b positive constants and x is the distance forces acting on the bob at angle θ = 30° will be
between the atoms . The atom is in equilibrium when ( g = 10 m/ s 2 ).
1 (a) 13.4 W (b) 20.4 W
 a6
(a) x = 0 (b) x =   (c) 24.6 W (d) zero
 2b
1 1 13. A small block of mass m is kept on a rough inclined
 2a 6  11a 6 surface of inclination θ fixed in an elevator. The elevator
(c) x =   (d) x =  
 b  5b 
goes up with a uniform velocity v and the block does not
1 slide on the wedge. The work done by the force of
9. A bead of mass kg starts from rest from A to move in a friction on the block in a time t will be
2
vertical plane along a smooth fixed quarter ring of radius (a) zero (b) mgvt cos2 θ
1
5 m, under the action of a constant horizontal force (c) mgvt sin 2 θ (d) mgvt sin 2 θ
F = 5N as shown. The speed of bead as it reaches the 2
point B is ( Take g = 10 m/ s 2 ) 14. In position A kinetic energy of a particle is 60 J and
A potential energy is – 20 J. In position B, kinetic energy is
100 J and potential energy is 40 J. Then, in moving the
particle from A to B
(a) work done by conservative forces is – 60 J
R=5m (b) work done by external forces is 40 J
(c) net work done by all the forces is 40 J
B (d) net work done by all the forces is 100 J
(a) 14.14 m/s (b) 7.07 m/s (c) 5 m/s (d) 25 m/s 15. A block A of mass M rests on a wedge B of mass 2M and
10. A car of mass m is accelerating on a level smooth road inclination θ. There is sufficient friction between A and B
so that A does not slip on B. If there is no friction between
under the action of a single force F. The power delivered
B and ground, the compression in spring is
to the car is constant and equal to P. If the velocity of the
B
car at an instant is v, then after travelling how much
distance it becomes double? M
K
A
F 2M
m

7 mv 3 4 mv 3 Mg cosθ Mg cosθ sin θ


(a) (b) (a) (b)
3P 3P K K
mv 3 18 mv 3 Mg sinθ
(c) (d) (c) (d) zero
P 7P K
246 Objective Physics Vol. 1

16. A smooth chain AB of mass m rests against a surface in 3. One end of a light spring of spring constant k is fixed to a
the form of a quarter of a circle of radius R. If it is wall and the other end is tied to a block placed on a
released from rest, the velocity of the chain after it comes smooth horizontal surface. In a displacement, the work
over the horizontal part of the surface is  1
done by the spring is +   kx 2 . The possible cases are
R A  2
(a) the spring was initially compressed by a distance x and was
finally in its natural length
(b) it was initially stretched by a distance x and finally was in its
natural length
B (c) it was initially in its natural length and finally in a
compressed position
(a) 2gR (b) gR
(d) it was initially in its natural length and finally in a stretched
 2 position
(c) 2gR 1 −  (d) 2gR(2 − π )
 π 4. Identify the correct statement about work energy theorem.
17. Two inclined frictionless tracks, one gradual and the (a) Work done by all the conservative forces is equal to the
decrease in potential energy
other steep meet at A from where two stones are allowed (b) Work done by all the forces except the conservative forces is
to slide down from rest, one on each track as shown in equal to the change in mechanical energy
figure. (c) Work done by all the forces is equal to the change in kinetic
A
energy
(d) Work done by all the forces is equal to the change in
potential energy
I II
h 5. A disc of mass 3m and a disc of mass m are connected by
θ 1
θ 2
a massless spring of stiffness k. The heavier disc is placed
B on the ground with the spring vertical and lighter disc on
Which of the following statement is correct? top. From its equilibrium position the upper disc is
(a) Both the stones reach the bottom at the same time but not pushed down by a distance δ and released. Then,
3 mg
with the same speed (a) if δ > , the lower disc will bounce up
(b) Both the stones reach the bottom with the same speed and k
2 mg
stone I reaches the bottom earlier than stone II (b) if δ = , maximum normal reaction from ground on
(c) Both the stones reach the bottom with the same speed and k
stone II reaches the bottom earlier than stone I lower disc = 6 mg
2 mg
(d) Both the stones reach the bottom at different times and with (c) if δ = , maximum normal reaction from ground on
different speeds k
lower disc = 4 mg
More than One Correct Options 4 mg
(d) if δ > , the lower disc will bounce up
k
1. The potential energy of a particle of mass 5 kg moving in
x - y plane is given as U = ( 7x + 24 y ) J, x and y being in 6. In the adjoining figure block A is of mass mand block B is
metre. Initially at t = 0, the particle is at the origin (0, 0) of mass 2 m. The spring has force constant k. All the
moving with a velocity of (8.6 $i + 23.2 $j ) ms −1 . Then, surfaces are smooth and the system is released from rest
with spring unstretched.
(a) the velocity of the particle at t = 4 s, is 5 ms−1
(b) the acceleration of the particle is 5 ms−2 A
(c) the direction of motion of the particle initially (at t = 0) is at
right angles to the direction of acceleration B
(d) the path of the particle is circle 4mg
(a) The maximum extension of the spring is
2. The potential energy of a particle is given by formula k
U = 100 − 5x + 100 x 2 , U and x are in SI units. If mass of (b) The speed of block A, when the extension in spring is
2mg 2m
the particle is 0.1 kg, then the magnitude of its , is 2g
k 3k
acceleration
(c) Net acceleration of block B, when the extension in the spring
(a) at 0.05 m from the origin is 50 ms−2 2
(b) at 0.05 m from the mean position is 100 ms−2 is maximum, is g
3
(c) at 0.05 m from the origin is 150 ms−2 2mg
(d) Tension in the thread for extension of in spring is mg
(d) at 0.05 m from the mean position is 200 ms−2 k
Work, Energy and Power 247

7. If kinetic energy of a body is increasing, then 2


(a) work done by conservative forces must be positive
(b) work done by conservative forces may be positive
(c) work done by conservative forces may be zero 1 A
(d) work done by non-conservative forces may be zero

8. At two positions kinetic energy and potential energy of a


particle are (a) work done by gravity on block A in motion-2 is less than in
K 1 = 10 J : motion-1
(b) work done by normal reaction on block A in both the motions
U 1 = − 20 J, will be positive
K 2 = 20 J, (c) work done by force of friction in motion-1 may be positive
(d) work done by force of friction in motion-1 may be negative
U 2 = − 10 J
In moving from 1 to 2 10. A man of mass m, standing at the bottom of the stair case,
(a) work done by conservative forces is positive
of height L climbs it and stands at its top.
(a) Work done by all forces on man is equal to the rise in
(b) work done by conservative forces is negative
potential energy mgL
(c) work done by all the forces is positive
(b) Work done by all forces on man is zero
(d) work done by all the forces is negative (c) Work done by the gravitational force on man is mgL
9. Block A has no relative motion with respect to wedge (d) The reaction force from a step does not do work, because the
point of application of the force does not move while the
fixed to the lift as shown in figure during motion-1 or
force exists
motion-2

Comprehension Based Questions


Passage (Q. 1 to 2) Assertion and Reason
The figure shows the variation of potential energy of a Directions (Q. Nos. 1-16) These questions consists of two
particle as a function of x, the x-coordinate of the region. statements each printed as Assertion and Reason. While
It has been assumed that potential energy depends only answering these questions you are required to choose any one
on x. For all other values of x, U is zero, i.e. for x < − 10 of the following five responses.
and x > 15, U = 0. (a) If both Assertion and Reason are correct and Reason is
U (x ) the correct explanation of Assertion
50J
(b) If both Assertion and Reason are true but Reason is
25
not the correct explanation of Assertion
x(m) (c) If Assertion is true but Reason is false
– 10 –5 6 10 15 (d) If Assertion is false but Reason is true
– 35 (e) If both Assertion and Reason are false

1. Assertion Consider a person of mass 80 kg who is


Based on above information answer the following climbing a ladder. In climbing up a vertical distance of
questions. 5 m, the contact force exerted by ladder on person’s feet
1. If total mechanical energy of the particle is 25 J, then it does 4000 J of work. (Consider g = 10 m/ s 2 )
can be found in the region Reason Work done by a force F is defined as the dot
(a) − 10 < x < − 5 and 6 < x < 15 product of force with the displacement of point of
(b) − 10 < x < 0 and 6 < x < 10 application of force.
(c) −5 < x < 6
(d) −10 < x < 10 2. Assertion The work done in bringing a body down from
the top to the base along a frictionless inclined plane is
2. If total mechanical energy of the particle is − 40 J, then it
the same as the work done in bringing it down from the
can be found in region vertical side.
(a) x < − 10 and x > 15
(b) − 10 < x < − 5 and 6 < x < 15
Reason The gravitational force on the body along
(c) 10 < x < 15 the inclined plane is the same as that along the
(d) It is not possible vertical side.
248 Objective Physics Vol. 1

3. Assertion An object A is A 9. Assertion A body is moving along x-axis. Force


dropped from the top of an B F = − 2x 2 is acting on it work done by this force in
incline at t = 0, as shown in moving the body from x = − 2 to x = + 2 is zero.
figure. It will fall under gravity as
indicated by the arrow. At the Reason From x = − 2 to x = 0, work done is negative and
same time, i.e. t = 0, another from x = 0 to x = + 2 work done is negative.
object B begins to slide down the
θ 10. Assertion Spring force is a conservative force.
frictionless incline.
Reason Potential energy is defined only for conservative
The two objects during their motion to the ground level forces.
will be travelling at equal speeds.
Reason Net force on both the objects during their 11. Assertion If work done by conservative force is
motion is same. negative, then potential energy associated with that force
should increase.
4. Assertion In projectile motion, the rate of change in Reason This is from the reaction
magnitude of potential energy of a particle first decreases
∆U = − W
and then increases during motion.
Here, ∆U is change in potential energy and W is work
Reason In projectile motion, the rate of change in linear
done by conservative force.
momentum of a particle remains constant during motion.
12. Assertion Work done by constant force is equal to
5. Assertion At any instant the magnitude of rate of
magnitude of force multiplied by displacement.
change of potential energy of the projectile of mass 1 kg
is numerically equal to magnitude of a ⋅ v (where, a is Reason Work done is scalar quantity. It may be positive,
acceleration due to gravity and v is velocity at that negative or zero.
instant). 13. Assertion Velocity of a block changes from
Reason The graph representing power delivered by the ( 2$i + 3$j ) m/ s to ( −4i$ − 6$j ) m/ s. Then, work done by all
gravitational force acting on the projectile with time will the forces during this interval of time is positive.
be straight line with negative slope.
Reason Speed of block is increasing
6. Assertion A block of mass m starts moving on a rough
14. Assertion If a force is applied on a rigid body. Body is in
horizontal surface with a velocity v. It stops due to
motion but point of application of force is stationary.
friction between the block and the surface after moving
Then, work done by the force is zero.
through a certain distance. The surface is now tilted to an
angle of 30° with the horizontal and the same block is Reason If body is moving, then point of application of
made to go up on the surface with the same initial force should also move.
velocity v. The decrease in the mechanical energy in the 15. Assertion Force applied on a block moving in one
second situation is smaller than that in the first situation. dimension is producing a constant power, then the
Reason The coefficient of friction between the block motion should be uniformly accelerated.
and the surface decreases with the increase in the angle of Reason This constant power multiplied with time is
inclination. equal to the change in kinetic energy.
7. Assertion Surface between the blocks A and B is rough 16. Assertion Total work done by spring for may be
work done by friction on block B is always negative. positive, negative or zero.
Reason Direction of spring force is always towards
A
mean position.
B F
Match the Columns
Smooth
1. Match the following columns.
Reason Total work done by friction in both the blocks is
Column I Column II
always zero.
(A) Work done by all forces (p) Change in potential energy
8. Assertion At stable equilibrium position of a body (B) Work done by conservative (q) Change in kinetic energy
kinetic energy can’t be zero. Because it is maximum. forces
(C) Work done by external (r) Change in mechanical
Reason During oscillations of a body potential energy is forces energy
maximum at stable equilibrium position. (s) None
Work, Energy and Power 249

2. A force F = kx (where, k is a positive constant) is acting a (m/s2 ) U(J)


on a particle. Work done 2 120
x (m) 8 x (m)
Column I Column II
4 8 4
(A) In displacement from (p) Negative
x = 2 to x = 4 –120
(B) In displacing the body from (q) Positive
x = − 4 to x = − 2
Column I Column II
(C) In displacing the body from (r) zero
x = − 2 to x = + 2 (A) Kinetic energy (p) 120 J
(B) Work done by conservative forces (q) 240 J
3. F-x and correspondingU-x graph are as shown in figures.
(C) Total work done (r) 112 J
Three points A , B and C in F-x graph may be
(D) Work done by external (s) None
corresponding to P , Q and R in the U-x graph. Match the forces
following.
6. A body is moved along a straight line by a machine
F
delivering a power proportional to time ( P ∝ t ). Then,
B x
A C
match the following.
Column I Column II

(A) Velocity is proportional to (p) t


U
(B) Displacement is proportional to (q) t 2
Q R x
P (C) Work done is proportional to (r) t 3

7. Match the following.


Column I Column II
Column I Column II
(A) Electrostatic potential (p) Positive
(A) A (p) P
energy
(B) B (q) Q
(B) Gravitational potential (q) Negative
(C) C (r) R energy
(s) None (C) Elastic potential energy (r) Zero
(D) Magnetic potential energy (s) Not defined
4. A block of mass m is stationary with respect to a rough
wedge as shown in figure. Starting from rest in time t ,
( m = 1kg , θ = 30° , a = 2 m/ s 2 , t = 4 s ) work done on block Entrance Gallery
2014
1. Consider an elliptically shaped rail PQ in the vertical
plane with OP = 3 m and OQ = 4 m. A block of mass 1 kg
a
m is pulled along the rail from P to Q with a force of 18 N,
which is always parallel to line PQ (see figure).
θ Assuming no frictional losses, the kinetic energy of the
block when it reaches Q is ( n × 10) J. The value of n is
Column I Column II
(A) By gravity (p) 144 J
(take, acceleration due to gravity = 10 ms −2 )
(B) By normal reaction (q) 32 J [JEE Advanced]
Q
(C) By friction (r) 56 J
(D) By all the forces (s) 48 J
(t) None 4m

90°
5. Acceleration versus x and potential energy versus x graph
O P
of a particle moving along x-axis is as shown in figure . 3m
Mass of the particle is 1 kg and velocity at x = 0 is 4 m/s. (a) 50 J (b) 100 J
Match the following at x = 8m (c) 5 × 102 J (d) 200 J
250 Objective Physics Vol. 1

2. When a rubber band is stretched by a distance x, it exerts 9. A body of mass 5 kg is thrown vertically up with a kinetic
a restoring force of magnitude F = ax + bx 2 , where a and energy of 490 J. The height at which the kinetic energy of
b are constants. The work done in stretching the the body becomes half of the original value is
unstretched rubber band by L is [JEE Main] [Karnataka CET]
1 (a) 12.5 m (b) 10 m
(a) aL2 + bL3 (b) (aL2 + bL3 ) (c) 2.5 m (d) 5 m
2
aL2 bL3 1  aL2 bL3  10. A particle is projected from the ground with a kinetic
(c) + (d)  + 
2 3 2  2 3  energy E at an angle of 60° with the horizontal. Its kinetic
energy at the highest point of its motion will be [WB JEE]
3. A block of mass 0.18 kg is attached to a spring of force
(a) E/ 2 (b) E/2
constant 2 N/m. The coefficient of friction between the
(c) E/4 (d) E/8
block and the floor is 0.1. Initially the block is at rest and
the spring is unstretched. An impulse is given to the 11. A cubical vessel of height 1 m is full of water. What is the
block. The block sides a distance of 0.66 m and comes to amount of work done in pumping water out of the vessel?
rest for the first time the initial velocity of the blook in (take, g = 10 ms −2 ) [WB JEE]
m/s is v = N /10, then N is [J & K CET] (a) 1250 J (b) 5000 J
(a) 2 (b) 3 (c) 1000 J (d) 2500 J
(c) 4 (d) 6
12. A body of mass 6 kg is acted upon by a force which
2012 causes a displacement in it given by, x = ( t 2 / 4 ) m, where
4. A spring stores 1 J of energy for a compression of 1 mm. t is the time in second. The work done by the force in 2 s
The additional work to be done to compress it further by is [WB JEE]
1 mm is [Kerala CEE] (a) 12 J (b) 9 J
(c) 6 J (d) 3 J
(a) 1 J (b) 2 J
(c) 3 J (d) 4 J 13. A box is moved along a straight line by a machine
(e) 0.5 J delivering constant power. The distance moved by the
5. Two bodies of masses m1 and m2 are acted upon by body in time t is proportional to [WB JEE]
constant force F for a time t. They start from rest and (a) t 1/ 2 (b) t 3/ 4 (c) t 3/ 2 (d) t 2
acquire kinetic energies E1 and E 2 , respectively. Then, 2010
E1
is [Karnataka CET]
E2 14. A light inextensible string that goes over a smooth fixed
pulley as shown in the figure connects two blocks of
m1m2 m1
(a) (b) masses 0.36 kg and 0.72 kg. Taking, g = 10 ms −2 , find
m1 + m2 m2
m2 the work done (in joule) by string on the block of mass
(c) (d) 1 0.36 kg during the first second after the system is released
m1
from rest. [IIT JEE]
6. Force of 50 N acting on a body at an angle θ with
horizontal. If 150 J work is done by displacing it 3 m,
then θ is [OJEE]
(a) 60° (b) 30°
(c) 0° (d) 45° 0.36
kg
2011 0.72
kg
7. At time t = 0 s particle starts moving along the x-axis. If (a) 8 J (b) 9 J (c) 7 J (d) 0.48 J
its kinetic energy increases uniformly with time t , the net 15. The potential energy function for the force between two
force acting on it must be proportional to [AIEEE] atoms in a diatomic molecule is approximately given by,
1 a b
(a) t (b) constant (c) t (d) U ( x ) = 12 − 6 , where a and b are constants and x is the
t
x x
8. A ball dropped from a height of 2 m rebounds to a height distance between the atoms. If the dissociation energy of
of 1.5 m after hitting the ground. Then, the percentage of the molecule is D = [U ( x = ∞ ) − U at equilibrium ], D is [AIEEE]
energy lost is [Karnataka CET] b2 b2 b2 b2
(a) (b) (c) (d)
(a) 25 (b) 30 (c) 50 (d) 100 2a 12a 4a 6a
Work, Energy and Power 251

16. A particle acted upon by constant forces 4$i + $j − 3k$ and


3$i + $j − k$ is displaced from the point $i + 2$j + 3k$ to the 20

point 5$i + 4$j + k$ . The total work done by the forces in 15


SI unit is [Kerala CEE] F(N) 10
(a) 20
(b) 40 5
(c) 50
(d) 30
0 5 10 15 20 25 30
(e) 35
s (m)
17. The work done by a force acting on a body is as shown in
the graph. The total work done in covering an initial (a) 225 J (b) 200 J
distance of 20 m is [Karnataka CET] (c) 400 J (d) 175 J

Answers
Level 1
Objective Problems
1. (b) 2. (d) 3. (b) 4. (b) 5. (c) 6. (d) 7. (d) 8. (d) 9. (c) 10. (a)
11. (d) 12. (c) 13. (b,c) 14. (a) 15. (b) 16. (a) 17. (a) 18. (c) 19. (c) 20. (c)
21. (a) 22. (d) 23. (c) 24. (b) 25. (b) 26. (b) 27. (c) 28. (b) 29. (b) 30. (c)
31. (b) 32. (d) 33. (b) 34. (a) 35. (c) 36. (c) 37. (b) 38. (c) 39. (b) 40. (a)
41. (b) 42. (a) 43. (c) 44. (d) 45. (b) 46. (c) 47. (b) 48. (b) 49. (a) 50. (c)
51. (c) 52. (c) 53. (c) 54. (b) 55. (c) 56. (c) 57. (b) 58. (c) 59. (b) 60. (d)
61. (a) 62. (c) 63. (b) 64. (b) 65. (a,b,c) 66. (b) 67. (a) 68. (a) 69. (a) 70. (c)
71. (b) 72. (c) 73. (a) 74. (c) 75. (d) 76. (d) 77. (c) 78. (d) 79. (c) 80. (d)
81. (d) 82. (c) 83. (b) 84. (c) 85. (c) 86. (a) 87. (a) 88. (a) 89. (a) 90. (c)
91. (a)

Level 2
Only One Correct Option
1. (c) 2. (c) 3. (b) 4. (a) 5. (c) 6. (d) 7. (b) 8. (c) 9. (a) 10. (a)
11. (a) 12. (a) 13. (c) 14. (a,c) 15. (d) 16. (c) 17. (c)

More than One Correct Options


1. (a,b) 2. (a,b,c) 3. (a,b) 4. (b,c) 5. (b,d) 6. (a) 7. (b,c,d) 8. (b,c) 9. (all) 10. (b,d)

Comprehension Based Questions


1. (a) 2. (d)

Assertion and Reason


1. (d) 2. (b) 3. (c) 4. (b) 5. (c) 6. (c) 7. (c) 8. (d) 9. (e) 10. (a)
11. (a) 12. (d) 13. (a,b) 14. (c) 15. (d) 16. (b)

Match the Columns


1. A → q, B → s, C → r 2. A → q, B → p, C → r 3. A → r, B → s, C → p
4. A → t, B → p, C → s, D → q 5. A → r, B → q, C → p, D → t 6. A → p, B → q, D → q

7. A → pqr, B → qr, C → pr, D → pqr

Entrance Gallery
1. (a) 2. (c) 3. (c) 4. (c) 5. (c) 6. (c) 7. (d) 8. (a) 9. (d) 10. (c)
11. (b) 12. (d) 13. (c) 14. (a) 15. (c) 16. (b) 17. (b)
Solutions
Level 1 : Objective Problems 5 5
12. W = ∫ F ⋅ dx = ∫ ( 7 − 2x + 3x 2 ) dx = 135J
0 0
x1 cx12
1. W = ∫ c x ⋅ dx = 14. r = rB − rA = ( $i − j$ + k$ )m
0 2
5 5
2. W = ∫ Fdx = ∫ ( 7 − 2x + 3x 2 )dx W = F ⋅ r = 2 + 4 = 6J
0 0 +a
+a  Ay 3 By 2  2Aa 3
= [7x − x + 2
x 3 ]50 = 135 units 15. W = ∫ Fdy =  + + Cy  = + 2Ca
−a
 3 2 −a 3
3. W = ( y - component of force) × (displacement along y - axis)
= 15 × 10 = 150 J 16. Line and force should be perpendicular, as the work done is
4. W = F ⋅ S = F ⋅( r2 − r1 ) given to be zero.
= ( $i + 2$j + 3k$ ) ⋅ [( $i − $j + 2k$ ) − ( $i + $j + k$ )] F 3
m1 = y = = slope of force
= ( $i + 2j$ + 3k$ ) ⋅ ( − 2$j + k$ ) Fx 2
k
= − 4 + 3 = −1 J m2 = − = slope of line
Mg Mg 3
5. Mg − T = or T = m1m2 = − 1
2 2
∴ k=2
T
17. K f − K i = W
∴ K f = Ki + W
1 30
= × 10 × (10)2 + ∫ ( − 0.1 x )dx
2 20
g/2
= 475 J
dx
18. v = = t2
dt
At t = 0,v = 0,
At t = 2 s,v = 4 m/s
Mg
From work energy theorem,
Mgx W = change in kinetic energy
or magnitude of work done = T × x =
2 = K f − Ki
7. Work done by spring force from A to B is negative. But work 1
= m(v 2f − v i2 )
done by this force from B to A will be positive. 2
A B 1
= × 2 × (16 − 0)
2
=16J
F F = Spring force 19. K f = 2K i

m (v + 2)2 =  mv 2  ( 2)
Mean position 1 1
or
2 2 
8. W = F ⋅ S = ( 3$i + 4$j ) ⋅ ( 3$i + 4$j ) = 9 + 16 = 25J or v + 2 = 2v
9. Ratio of displacement is 2
v= = 2( 2 + 1) m/s
1 1 1 1 1 2 −1
g (1)2 ( g )( 2)2 − g (1)2 : g ( 3)2 − g ( 2)2
2 2 2 2 2 21. When a body is moved upwards, work done by gravity is
or 1 : 3 : 5. negative but potential energy increases.
Therefore, ratio of work done will also be1 : 3 : 5. 1
22. U = Kx 2 or U ∝ x 2
As W = mgs 2
or W ∝s Stretch is increased to 5 times. Therefore, stored potential
10. Change in kinetic energy = work done energy will be increased by 25 times.
= Area under F -x graph W
1 1 23. P =
∴ × 5 × v = 10 × 25 + × 25 × 10
2
t
2 2 W mgh
∴ t= =
= 375 P P
∴ v =12.2 m/s 200 × 10 × 40
= = 8s
11. W = area under F -x graph. 10 × 103
Work, Energy and Power 253

24. Work done by all forces = change in kinetic energy 3


= − 50 × 9.8 × 2 × = − 490 3 W
1 2
= m (v 2f − v i2 )
2 ∴Power delivered is 490 3 W
1 2  3t 2 
= × 2(0 − 400) = − 400J 1
36. W = ∫ Pdt or mv 2 = ∫ Pdt = ∫ 
2  dt = 4J
2 0  2 
25. P = F ⋅ v
2× 4
P 7500 ∴ v= = 2m/s
∴ F= = = 375N m
v 20
37. Compare with spring force,
26. Work done = change in kinetic energy.
(1,1 ) 1 F = –kx
∫( 0, 0 )( 4i + 4j$ ) ⋅ ( dx i + dy j$ ) = 2 × 1 × (v f − 2 )
∴ $ $ 2 2 A
B
1 x
or 8 = (v 2f − 4)
2 Mean
or v f = 4.5m/s
From A to B,
27. P = F ⋅ v = (10i$ + 10j$ + 20 k$ ) ⋅ ( 5i$ − 3j$ + 6k$ ) Loss in KE = gain in elastic PE
= 50 − 30 + 120 1
= kx 2 ∝ x 2
=140J /s 2
4 4
28. ∆KE = W = ∫ Pdt = ∫ ( 3t 2 − 2t + 1) dt F − W 170 − 50
38. a = = = 24 m/s 2
2 2 m 5
= [t 3 − t 2 + t ]42 v = 2as = 2 × 24 × 10
= 46J = 21.9 m/s
29. Work done by conservative force = − ∆U = 22 m/s
30. W =P ×t (P = constant) 39. mgh = K
1 3
or mv 2 = Pt (From work-energy theorem) Now, K + mgh′ = K (at height h′)
2 4
∴ v ∝ t1/ 2 3
∴ mgh + mgh′ = mgh
4
Integrating, we get s ∝ t 3/ 2 .
h
1 or h′ =
32. W = ∆KE = m (v 2f − v i2 ) 4
2
1 40. v = at1
= × 0.01[(64 + 400) − (16 + 256)] v
2 ∴ a=
= 0.96 J t1
1 1
33. mv12 =  × × v 22 
1 1 1 m W = mv 2 = m ( at )2
2 2  2 2  2 2
2
v1 = speed of man and v 2 = speed of boy 1 v 
= m  × t
1 1 m 2  t1 
Now, m (v1 + 1)2 = × × v 22
2 2 2
1 v 2t 2
= m 2
Solving these two equations, we get 2 t1
v1 = ( 2 + 1) m/s and v 2 = 2( 2 + 1) m/s 1
41. m (v 2f − v i2 ) = W = F ⋅( r f − ri )
34. F = constant 2
∴a = constant or v = at 42. P = F ⋅ v = ( 20$i − 3$j + 5k$ ) ⋅ (6$i + 20$j − 3k$ )
Now, P = F ⋅ v = F ⋅ at or P ∝ t (as F and a both are constant) = 120 − 60 − 15 = 45W
Hence, P -t graph is a straight line passing through origin. 43. P = Impulse = Ft
35. Two forces are acting on bob, tension and weight. Power of P 2 F 2t 2
tension will be zero and that of weight is, W = KE = =
2m 2m
 F  t ⋅ Ft
θ
 
 
= m
T 2
v = 2 m/s ( at )Ft Fvt
θ = 60°
= =
2 2
θ W F ⋅v
∴ Average power = =
mg t 2
KE mv 2
P = mgv cos( 90° + θ) Average power = =
= − mgv sin60° t 2t
254 Objective Physics Vol. 1

44. F ∝ S −1/ 3 or a ∝ S −1/ 3 53. P = constant


ds  a = v ⋅
−1/ 3 dv  1
or vdv ∝ S  ∴ W = mv 2 = P × t
 ds  2
Integrating we have, ⇒ v ∝ t 3/ 2 (integration)
v 2 ∝ S 2/ 3 54. (mgH − mgh) = 2mgh
or v ∝ S1/ 3 H
−1/ 3 ∴ h=
Now, power P ∝ F ⋅ v ∝ S ⋅ S1/ 3 or P ∝ S 0 3
45. When displaced from x 2 in negative direction, force is v = 2g( H − h)
positive. So this force is of restoring nature, or bringing the
= 2g 
2H  gH
body back. Hence, at x 2 , body is in stable equilibrium  =2
 3  3
position.
46. In stable equilibrium, PE is minimum 55. Decrease in gravitational PE = increase in kinetic energy
dU
F=− = − slope of v-r graph and negative force means
dr v
attraction and positive force means repulsion. C1
47. Change in potential energy for all three particles is same. C2
Hence, change in kinetic energy will also be same.
Or v A = v B = vC
48. Decrease in kinetic energy = increase in elastic potential energy. − M ⋅ l g l 0  −  MgL  = 1 Mv 2
or  0   
1 1  L 2  2  2
∴ mv 2 = Kx 2
2 2 g 2 2
or v= ( L − l0 )
m l
or x= ⋅v
K 56. Decrease in PE = mgh
0.1
= × 10 57. At this height half is PE
1000 98
= 0.1 m ∴ mgh =
2
49. Decrease in gravitational potential energy or 2 × 9.8 × h = 49
= increase in elastic potential energy or h = 2.5 m
1
or mg(h + x ) = x 2 58.
1
( M − m)gh = ( M + m)v 2
2 2
1
or 2 × 9.8( 40
. + x ) = × 1960 × x 2 2gh( M − m)
2 ∴ v=
M+m
Solving this equation, we get
1
x = 0.1 m or 10 cm. 59. mgh + mv 2 = constant = mgh (H = initial height)
2
50. Increase in gravitational potential energy = decrease in
v2
elastic potential energy or gh + = constant
2
1 2
∴ mgh = kx 60. Decrease in gravitational potential energy = increase in
2
elastic potential energy
kx 2
∴ h= 1
2mg ∴ mg(h + x ) = Kx 2
2
1 2U x = 0.1 m
51. mv 2 = U or m = 2 Solving we get
2 v 1
61. mv 2 = (0.9)(mgh)
52. h = r(1 − cosθ) 2
h ∴ v = 18 . gh = 18. × 10 × 2
= 6m/s
θ r 63. N = mg − F sinθ
Block moves with uniform velocity. Hence, net force = 0
or, F cosθ = µN = µ (mg − F sin θ)
µmg
∴ F=
cosθ + µ sin θ
Now, decrease in PE = increase in KE
1 µ mgd cosθ
or mgr(1 − cosθ) = mv 2 W = Fs cosθ =
2 cosθ + µ sin θ
or v = 2gr(1 − cosθ) 64. F = µmg and W = F ⋅ S
Work, Energy and Power 255

65.
f
L/6
m Sm M
SM
f M
Sm ≠ SM 3
66. Maximum acceleration of 2 kg block due to friction can be
µg or 5 m/s2 .
Combined acceleration, if both move together with same
acceleration would be,
60 76. Work done by tension on M is negative (force and
a= = 5 m/s2
12 displacement are in opposite directions). But work done by
Since, both accelerations are equal, upper block will move tension on m is positive. Net work done will be zero.
with acceleration 5 m/s2 due to friction. 77. W = change in potential energy = mgh = mgL(1 − cosθ).
1 1
In first two seconds, S = at 2 = × 5 × 4 = 10m 78. From work-energy theorem,
2 2 W = change in kinetic energy
and force of friction f = ma =10N 1
or W = mv 2
∴ W f = fs cos00 = 100 J 2
67. W f = fs cos180° = (µmg cosθ)( − 1)( S) ∴ W - v graph is a parabola.
1
1 79. W = × m × (10)2 = 50m
= − 0.2 × 5 × 10 × × 0.1 2
2
1
1 W ′ = m × ( 400 − 100) = 150m = 3W
=− J 2
2
dx
68. Work done against friction = (µmg cosθ)d 80. v = = 8t 3
dt
3 1
69. th is lost. Hence, left is th, or, v 0 s = 0,v1 s = 8 m/s
4 4
1
v 02 ∴ ∆KE = × 2 × (64 − 0) J
v =
2
2
u
v0 = 64 J
∴ v= = v 0 − at0
2 P2
81. K = or K ∝ P 2
= v 0 − ugt0 2m
v When P is doubled, kinetic energy will become four times.
or u= 0
2gt0 82. From work-energy theorem,
P2 (1.5P 2 ) Work done = change in kinetic energy
70. K = ,K ′ =
2m 2m ∴ Fx = K (as F = constant, because a = constant)
P2 Therefore, K -x graph is a straight line passing through
= 2.25 = 2.25K
2m origin.
∴Increase in kinetic energy is 125%. mg
83. In equilibrium kd = mg or d =
l k
71. Centre of mass of the rod moves a height h = .
2 In allowed to fall suddenly, if does not stop in its equilibrium
72. W = change in potential energy = mgh. position. In that case,
decrease in gravitational PE = increase in elastic PE
73. Given force is a constant force and work done by a constant
1 2mg
force is always path independent. or mgd ′ = k d ′2 or d ′ = = 2d
2 k
74. P = 2Km or P ∝ m
84. F = Kx or K = slope of F -x graph (F along y-axis)
P1 1 1
∴ = = Here, F is along x-axis.
P2 4 2 10
. kgf
So, K = = 0.1
M L 10 cm
75. Mass has its centre of mass below the table surface.
3 6 Total displacement
85. Average velocity =
∴ W = mgh Time
=   ( g ) 
M L Area under v - t graph
=
 3   6 Time
MgL Since, area ≠ 0
=
18 ∴Average velocity ≠ 0
256 Objective Physics Vol. 1

1 2µmg
86. E = mv 2 or ( a − b) =
2 K
dE 2µmg
∴ = mv ∴ Decrease in amplitude =
dv K
dE
or P= (as mv = P) 4. Decrease in gravitational potential energy = increase in
dv kinetic energy.
F l
87. K = = slope of F - x graph. Initially centre of mass of chain was at distance below the
x 4
1 l
K ∝ pin and in final position it is at distance below the pin.
l 2
Length is reduced to half. Therefore, K will become two l
Hence, centre of mass has descended .
times. Slope will increase. 4
88. K = 4t 2 or v 2 ∝ t 2
∴ v ∝t
v varies linearly with time when acceleration or force is ⇒
constant.
89. Speed is doubled. Therefore, kinetic energy will become
four times. Hence, minimum stopping distance will also
become four times.
l 1
1 ∴ mg = mv 2
90. Total loss in friction = K i − K f = m (100 − 81) 4 2
2
gl
19
= m (m = mass) or v=
2 2
Half of the loss will be in upward journey. 5. At x = 6 m, U = 26J (extreme position)

Hence, mgh +  m = K i = m (100) = 50m


1 19 1 On the other side
2 2  2 U = 26 = 10 + ( x − 2)2 or x − 2 = ± 4
∴ h = 4.61 m ∴ x = 6 m and x = − 2m
91. F . d = mg(h + d ) Thus, x = + 6 m and x = − 2m are the extreme positions. At
x = 2 m,potential energy is minimum, which is 10 J. Hence, at
∴ F = mg 1 + 
h
 d this position kinetic energy will be maximum, which is equal
Work done against resistance to total mechanical energy( = 26J ) minus minimum potential
= decrease in mechanical energy energy ( =10J ). Thus maximum kinetic energy is 16 J.
6. Body will have maximum speed where,
Level 2 : Only One Correct Option Mg sin θ = µ mg cosθ
or sin 37° = (0.3x ).cos 37°
1. When released from top with zero velocity block leaves
2 or x = 2.5 m
contact at cosθ =
3 7. Maximum acceleration of 1 kg block may be
amax = µg = 1 m/s2
θ r h Common acceleration, without relative motion between two
blocks may be,
0.5
∴ h = r cosθ =
2r a= m/s2
3 3
Since, a < amax
2. Since, the particle is released from a small height, θ (angle of
radius with vertical) will be very small. Force of friction There will be no relative motion and blocks will move with
0.5
throughout the journey can be assumed to be µmg. Particle acceleration m/s2 .
3
will finally come to rest when whole of its energy ( = mgh) is
Force of friction by lower block on upper block,
lost in the work done against friction. Let particle stops after
f = ma = (1)  = N (towards right)
0.5 1
travelling a distance d. Then,
 3 6
µmgd = mgh
h 10−2 ∴ W = f × s = 0.5J
or d= = = 10
. m 8. At equilibrium,
µ 001.
dU
3. From − a to b (∆KE = 0 as mass will stop for a moment at F=− =0
dx
extreme position) decrease in elastic potential energy = or ( − 12)ax −13 + (6 bx −7 ) = 0
work done against friction. 1/ 6
x =  
1 1 2a
∴ K a 2 − Kb 2 = µmg( a + b ) ∴
2 2  b
Work, Energy and Power 257

9. From work-energy theorem 14. Work done by conservative forces = Ui − U f


1
W F + Wmg + W N = mv 2 Work done by external forces = E f − Ei
2
1 and net work done by all the forces = K f − K i
F ⋅ R + mgR + 0 = mv 2
2 15. On M horizontal components of N and f are balanced
1 1 1 (as Mg is vertical). Hence on 2M also they will be balanced.
5 × 5 + × 10 × 5 = × × v 2
2 2 2 ∴Horizontal Kx force on 2M should be zero.
∴ v = 200 = 14.14 m/s 16. h = R(1 − cosθ)

10. P = F .v = m  v ⋅
dv 
 ⋅v θ

 ds 
2v P s
∴ ∫v v 2 ⋅ dv =
m ∫0
⋅ ds
h
2v
v 3  PS 7mv 3 h=0
  = or S=
3
 v m 3P
11. F = kx  m 
dm =   ⋅ dθ

F 100
k= = N/m = 100 N/m  π/2 
x 1 2mdθ
=
Now from energy conservation, between natural length of π
spring and its maximum compression state. dv i = ( dm)gh
1 1 2
mv 2 + mgh = kx max 2mgR(1 − cosθ)dθ
=
2 2 π
90 °
Ui = ∫
2
∴ v=
kx max
− 2gh ∴ dUi
0
m
= mgR 1 − 
2
(100)( 2)2
− ( 2)(10)   π
2
=
10  2 Uf = 0
= 20 m/s Decrease in PE = increase in KE.

12. Power of tension = 0 17. As the given tracks are frictionless, hence, mechanical
energy will be conserved. As both the tracks having
60º common height, h .
30º From conservation of mechanical energy,
1
mv 2 = mgh (for both tracks I and II)
2
T
v = 2gh
Hence, speed is same for both stones.
For stone I, a1 = acceleration along inclined plane = g sin θ1
v 60° Similarly, for stone II a2 = g sin θ2 as θ2 > θ1 hence, a2 > a1 .
And both length for track II is also less hence, stone II
mg reaches earlier than stone I.

Power of mg = (mg )(v )cos60° More than One Correct Options


Here, v = 2gh and h = l (cos 30° − cos60° )
 ∂U $ ∂U $ 
1. F = −  i+ j = ( −7i$ − 24$j) N
13. Block does not slide. Hence, the force of friction,
 ∂X ∂y 
F  7 $ 24 $ 
a= = − i − j m/s
S m  5 5 
θ 2 2
| a | =   +   = 5 m/s2
° – 7 24
90  5  5
f

θ Since, a = constant, we can apply,


v = u + at
f = mg sinθ
= (8.6 $i + 23.2 $j) +  − i$ −
7 24 $ 
j ( 4)
In time t, displacement S = vt  5 5 
∴ W f = f ⋅ s ⋅ cos( 90° − θ)
= ( 3i$ + 4$j) m/s
= (mg sin θ)(vt )(sin θ)
= mgvt sin 2 θ | v | = ( 3)2 + ( 4)2 = 5 m/s
258 Objective Physics Vol. 1

2. F = −
dU
= 5 − 200x 6. (a) Decrease in potential energy of B = increase in spring
dX potential energy
Mean position is at F = 0 1
∴ 2mg Xm = k Xm 2
5 2
or at, x= = 0.025 m
200 4mg
∴ Xm =
F 5 − 200 x k
a= = = ( 50 − 2000 x ) ...(i)
m 0.1 (b) Ei = E f
At 0.05 m from the origin, 2
0 = (m + 2m) v 2 + × k × 
1 1 2mg   2mg 
x = + 0.05 m  −( 2mg )  
2 2  k   k 
or x = − 0.05 m
m
Substituting in Eq. (i), We have ∴ v = 2g
3k
| a | =150 m/s2
kXm − 2mg
(c) a = (upwards)
or = 50 m/s2 2m
At 0.05 mfrom the mean position means,  4mg 
k  − 2mg
x = 0.075  k 
= =g
or x = − 0.025 m 2m
Substituting in Eq. (i) we have, (d) T − 2mg = ma ...(i)
| a | =100 m/s2 2mg − T = 2ma ...(ii)
3. Spring force is always towards mean position. If Solving these two equations, we get
displacement is also towards mean position, F and S will be a = 0 and T = 2mg
of same sign and work done will be positive. a
4. Work done by conservative force = − ∆U m T
kx = 2 mg
Work done by all the forces = ∆K
Work done by forces other than conservative forces = ∆E T
5. (a) At equilibrium a
2m
m Equilibrium position
2 mg

7. No solution is required.
8. Work done by conservative forces
3m = Ui − U f = − 20 + 10 = − 10 J
Work done by all the forces,
kδ 0 = mg = K f − K i = 20 − 10 = 10 J
mg 9. (a) Work done by gravity in motion 1 is zero (θ = 90° ) and in
or δ0 =
k motion 2 is negative (θ = 180° ).
where, δ 0 = compression (b) In both cases angle between N and S is acute.
3mg (c) and (d) Depending on the value of acceleration in motion
(b) δ Total = δ + δ 0 =
k 1, friction may act up the plane or down the plane.
Therefore, the angle between friction and displacement
Fmax = kδ max = k 
3mg 
 = 3mg (downward) may be obtuse or acute. So, work done by friction may be
 k 
negative or positive.
Nmax 10. When a man of mass m climbs up the staircase of height L,
work done by the gravitational force on the man is-mgl
work done by internal muscular forces will be mgL as the
3m change in kinetic energy is almost zero.
3 mg Fmax Hence, total work done = − mgL + mgL = 0
As the point of application of the contact forces does not
∴ N max = 3mg + Fmax = 6mg
move, hence work done by reaction forces will be zero.
4mg
(d) If δ > , then upper block will move a distance
k
4mg 3mg
Comprehension Based Questions
X> − δ 0 or X > from natural length. 1. U = E − K = 25 − K
k k
Hence in this case, extension Since, K ≥0
3mg ∴ U ≤ 25 J
X>
k 2. U = E − K = − 40 − K
or F = kx > 3mg (upwards on lower block) Since, K ≥0
So lower block will bounce up. ∴ U ≤ − 40 J
Work, Energy and Power 259

Assertion and Reason 12. Work done by a constant force


1. W = 0. That feet remains stationary, which is in contact with = Force × displacement in the direction of force.
the ladder. 13. According to work-energy theorem, work done by all the
2. Change in potential energy is same. forces = change in kinetic energy.
1
3. Assertion Change in PE is same. 15. W = Pt = mv 2
2
Reason Acceleration a is different. Hence, the net force is
v ∝ t1/ 2
different.
Differentiating we get,
= (mgh) = mg  uy t − gt 2 
dU d d 1
4. Assertion
dt dt dt  2  a ∝ t −1/ 2
= mg(uy − gt ) = mgv y 16. WAC = + ve
dU x x
∴ = mg| v y |
dt
| v y | first decreases then increase with time.
dP C
Reason = F = mg = constant x=0
dt
A B
= (mgh) = mg  uy t − gt 2 
dU d d 1
5. Assertion
dt dt dt  2 
WCB = − ve and WACB = 0
= mg(uy − gt ) = mgv y

dv
= mg | v y | Match the Columns
dt 1. Work done by conservative forces in negative of change in
For m =1 kg potential energy.
dU
= g(uy − gt ) …(i) 2. From x = 2 to x = 4, force is positive and displacement is also
dt
positive. Hence, the work done is positive. Similar logic can
a ⋅ v = ( − g j$ ) ⋅ [u i$ + (u − gt )j$ ]
x y
be applied to other parts also.
= g(uy − gt ) …(ii) 3. A is the point of stable equilibrium, so potential energy is
From Eqs. (i) and (ii), we can see that two magnitudes are minimum. Similarly, point C is the unstable equilibrium
equal. position, where potential energy should be maximum.
Reason P = F ⋅ v = ( − mg $i ) ⋅ [v $i + (u − gt )j$ ]
x y 4. In t = 4s,
= − mguy + mg t 2
v = at = 8m/s and s =
1 2
at = 16 m
2
So, P versus t graph has the positive slope.
1
6. Assertion Decrease in mechanical energy in case 1 will be KE = mv 2 = 32 J
1 2
∆U1 = mv 2 From work-energy theorem,
2
But decrease in mechanical energy in case 2 will be Work done by all the forces = ∆KE = 32J
1 Work done by gravity = − mgh = − (1)(10)(16)
∆U2 = mv 2 − mgh
2 = −160J
∴ ∆U2 < ∆U1 Writing equation of motion, we have,Σ Fy = ma
or assertion is correct. f
y
N
Coefficient of friction is the mutual property of two surfaces. θ a
It does not depend on angle of inclination. x
7. Work done by friction on A is positive and on B is negative. If θ
there is no slipping between the two blocks, then SA = SB .
θ
Therefore, the net work done will be zero, otherwise not.
mg = 10 N
SB SA

N cos 30° + f sin 30° − 10 = ma = 2


B A
f f or 3 N + f = 24 …(i)
Σ Fx = 0
8. If oscillations are not taking place, then kinetic energy may
∴ N sin 30° = f cos 30°
be zero at stable equilibrium position.
or N = 3f …(ii)
9. Force is continuously negative. But displacement is positive
(from x = − 2 to x = + 2) Solving Eqs. (i) and (ii), we have
∴Work done is negative. f = 6N
260 Objective Physics Vol. 1

and N=
18
=6 3N 3. Here, m = 0.18 kg,
3 K = 2 N/m µ = 0.1, x = 0.06 m
Now, W N = ( N cosθ)( s ) According to conservation of mechanical energy principle,
 3 we know
= (6 3 )   (16) =144J Decrease in mechanical energy = work done against friction
 2 
1 1
mv 2 − kx 2 = µmgh
W F = ( f sin θ)( s ) 2 2
1 2 µmgx + kx 2
= (6) (16) = 48J v=
2 m
5. v f − v i = area of a-x graph =12 m/s Substituting the values of m,µ, g, x, we get
∴ v f = 12 + 4 = 16 m/s 2 × 0.1 × 0.28 × 9.8 × 0.06 + 2 × 0.06 × 0.06
v=
1 0.18
∆ KE = m(v 2f − v i2 ) =120J
2  4
v =   m/s
 10 
Work done by all the forces = ∆ KE = 120J
So, N = 4
1
K f = mv 2f = 128 J 4. Given, W1 = 1 J, x = 1 × 10−3 m
2
Work done to compress it 1 mm,
Work done by conservative forces = Ui − U f = 240J
1 1
W1 = kx12 ⇒ 1 = × k × (1 × 10−3 )2
Work done by external force 2 2
= total work done – work done by conservative forces ⇒ k = 2 × 106
= – 112 J Work done to compress it further by 1 mm,
1 1
6. P ∝ t W2 = kx 22 = × 2 × 106 × (1 × 10−3 + 1 × 10−3 )2
2 2
W = ∫ Pdt = ∫ αt dt or W ∝ t 2
W2 = 4 × 10−6 × 106 = 4 J
Since, work done is equal to change is KE The additional work done = W2 − W1 = 4 − 1 = 3 J
Hence, v 2 ∝ t 2 or v ∝ t 5. Momentum acquired by the bodies,
ds p1 = p2 = Ft
Further, v=
dt Now, their kinetic energies,
ds p2 p2
∴ ∝ t or ds ∝ t dt E1 = and E2 =
dt 2m1 2m 2
or s ∝t2 (by integration) E1 m2
∴ =
Entrance Gallery E 2 m1
1. From work-energy theorem, 6. Given, F = 50 N, W =150J, s = 3 m
Work done by all forces = Change in kinetic energy Workdone, W = Fs cosθ
or W F + Wmg = K f − K i 150 = 50 × 3 × cosθ
18 × 5 + (1 × 10)( −4) = K f cosθ =
150
=1
90 − 40 = K f 150
⇒ θ = 0°
⇒ K f = 50 J
dK
2. Thinking Process We know that change in potential 7. Given, = constant
dt
energy of a system corresponding to a conservative internal
K ∝t ∴K = 1 mv 2 
force as,  
f
 2 
U f − Ui = − W = − ∫ F ⋅ d r 1
i mv 2 ∝ t
Given, that F = ax + bx 2 2
⇒ v∝ t
We know that work done in stretching the rubber band by L
dK
is given by ∴ P = Fv = = constant
L dt
| dW | = | Fdx | ⇒ | W | = ∫ ( ax + bx 2 ) dx 1
0 F∝
L 3 L v
 ax 2   bx 1
=  +  ⇒ F∝
 2 0  3 0 t
 aL2 a × (0)   b × L3 b × (0)3 
2 mg ( 2 − 1. 5)
= − 8. Percentage of energy loss = × 100
+  −  mgh
 2 2   3 3 
aL2 bL3 mg (0. 5)
=|W| = + = × 100 = 25%
2 3 mg × 2
Work, Energy and Power 261

9. According to law of conservation of energy, 1 2 1 g 2 g


s=at =   (1) =
1 1
mu2 = mv 2 + mgh 2 2  3 6
2 2  g
T − 0.36g = 0.36a = 0.36  
490 = 245 + 5 × 9.8 × h  3
245
h= = 5m ∴ T = 0.48 g
49
1
10. At the ground, E = mu2
3
1
At highest point, E ′ = m (ucos60° )2
2 a T
E
E′ =
4 a
0.36 kg
11. The total volume of water in T
vessel,
1 0.36 g
m 0.72 kg
V = l 3 = 1 m3 2 l
m = 1 × 1000 = 1000 kg Centre of mass
1 0.72 g
W = mgh = 1000 × 10 × = 5000J
2 l
Now, WT = Ts cos0° (on 0.36 kg mass)
t2 l
12. Given, m = 6 kg, x = = (0.48g)   (1) = 0.08(g 2 )
g
4  6
dx t
=v = = 0.08(10)2 = 8 J
dt 2
t = 0s a b
Velocity at, 15. Given, potential energy function U( x ) = −
⇒ v(0) = 0 x12 x 6
∴ The initial kinetic energy,
1
K i = m (0)2 U( x = ∞ ) = 0
= −  13 − 7 
2 dU 12a 6b
As, F=−
Velocity at
2
t = 2s, v ( 2) = = 1 dx  x x 
2 At equilibrium, F =0
and the final kinetic energy, 2a
1 1 ∴ x6 =
K f = m(1)2 = × 6 × 1 = 3 b
2 2 a b −b2
According to work-energy theorem ∴ Uequilibrium = 12
− 8
=
 2a   2a  4a
Work, W = K f − K i = 3 − 0 = 3J    
 b  b
dv
13. We know that, P = Fv = m ⋅ ⋅v b2
dt ∴ D = [U( x − ∞ ) − Uequilibrium ] =
vdv P 4a
=
dt m 16. Work done, W = F ⋅ ds = (F1 + F2 ) ⋅ (s 2 − s 1 )
Integrating on both side, we get = {( 4i$ + $j − 3k$ ) + ( 3i$ + $j − k$ )}
Pdt v 2 Pt {( 5$i + 4$j + k$ ) − ( $i + 2$j + 3k$ )} = ( 7$i + 2$j − 4k$ ) ⋅ ( 4$i + 2j$ − 2k$ )
∫ v dv = ∫ m , ⇒ 2 = m
= 28 + 4 + 8 = 40 J
2P 1/ 2 dx 2P 1/ 2
v= t , = t 17. Work done, W = Area ABCEFDA
m dt m
2P 1/ 2 E
∫ dx = m ∫ t dt , 20

2P t 3/ 2 2 2P 3/ 2 15
x= = t
m 3/2 3 m F(in N) B C
10
⇒ x ∝t 3/ 2
5
14. Let acceleration, A D F
given, m1 = 0.36 kg , m2 = 0.72 kg 0 5 10 15 20 25 30
s(in m)
and g = 10 ms−1
Net pulling force = Area ABCD + Area CEFD
a=
Total mass 1 1
= × (15 + 10) × 10 + × (10 + 20) × 5
0.72g − 0.36 g g 2 2
= =
0.72 + 0.36 3 = 125 + 75 = 200 J
7
Circular Motion
7.1 Kinematics of Circular Motion
Angular Variables Chapter Snapshot
Circular motion is a two dimensional motion or motion in a plane. ● Kinematics of Circular
Suppose a particle P is moving in a circle of radius r and centre O. Motion
Dynamics of Circular
The position of the particle P at a given instant may be described by the angle θ between

Motion
OP and OX. This angle θ is called the angular position of the particle. As the particle
moves on the circle its angular position θ changes. Suppose, the point rotates an angle ∆θ in ● Motion in a Vertical Circle
time ∆t. The rate of change of angular position is known as the angular velocity (ω ). Thus,
Y

P′
P
∆θ

θ
O r X

Fig. 7.1

∆θ dθ
ω = lim =
∆t → 0 ∆t dt
The rate of change of angular velocity is called the angular acceleration (α). Thus,
dω d 2 θ
α= =
dt dt 2
If angular acceleration is constant, we have
1
θ = ω 0 t + αt 2
2
ω = ω 0 + αt
and ω 2 = ω 20 + 2αθ
Here, ω 0 and ω are the angular velocities at time t = 0 and t and θ the angular position at
time t. The linear distance PP ′ travelled by the particle in time ∆t is
∆s = r∆θ
∆s ∆θ
or lim = r lim
∆t → 0 ∆t ∆t → 0 ∆t
Circular Motion 263

ds dθ Acceleration
or =r
dt dt Acceleration of the particle in circular motion has two
or v = rω components.
Here, v is the linear velocity of the particle. (i) Tangential component ( a t ) This is the component
Following points are worthnoting regarding circular of a in the direction of velocity, which is responsible
motion of any particle : for change in speed of particle. It is also equal to rate
of change of speed. Hence,
Velocity a t = component of a along v
A particle P in circular motion has two types of dv d | v |
velocities and corresponding two speeds. = =
dt dt
(i) Linear velocity (v) and linear speed ( v ) Linear
This component is tangential.
velocity of particle is displacement of particle per unit
time. (ii) Radial component ( a r ) This is component of a
Y towards centre. This is responsible for change in
v v2
P
direction of velocity. This is equal to or rω 2 . Thus,
r
θ
X v2
ar = = rω 2
r
Three Possible Types of Circular Motion
Fig. 7.2 In circular motion direction of velocity definitely
changes. Hence, a r can never be zero. But speed may remain
ds
or v= constant ( a t = 0), may be increasing (a t = positive) or may
dt be decreasing (a t = negative).
Magnitude of linear velocity is called linear speed, v Accordingly we can classify circular motion in
which is distance travelled per unit time. following three types.
ds v
or v = | v| = (i) Uniform circular motion In this type
dt of circular motion, speed of particle θ
(ii) Angular velocity (ω ) and angular speed (ω 0 ) Small remains constant. Hence, a t = 0 and
v2 a = ar
angular displacements dθ is a vector quantity. Rate of ar = or rω 2 . Thus, net acceleration
change of angular displacement is called angular r
velocity (ω ). Thus, is also equal to a r . Angle between v
Fig. 7.3
and a is 90°.

ω= (unit is rad/s) v2
dt θ = 90° ⇒ a = a r = or rω 2
r
Magnitude of ω is called angular velocity. Thus,
⇒ v = constant
dθ dθ
ω0 = |ω| = = (ii) Circular motion of increasing v at
dt dt speed In this type of circular motion θ
Note that ω is an axial vector, direction of which is given speed of particle increases. Hence, a t a
is positive or in the direction of
by right hand screw law. For example, in the given figure ω ar
velocity. Thus, net acceleration in this
is perpendicular to paper outwards. Relation between linear
speed v and angular speed ω 0 is case will be a = a t2 + a r2 . Angle
Fig. 7.4
v = rω 0 between a and v will be acute.
But relation between linear velocity v and angular θ = acute ⇒ a = a t2 + a r2
velocity ω is given by dv d| v|
where at = or
v =ω × r dt dt
2
Here, r is the radius but r is position vector of particle v
and ar = or rω 2
with respect to centre of circle. r
264 Objective Physics Vol. 1

(iii) Circular motion of decreasing speed In this type of ∴ Net acceleration of particle at t = 2s is
circular motion speed of particle decreases. Hence, a t a = (a r )2 + (a t )2 = (8)2 + (4)2 or a = 80 m/s 2
is negative or in opposite direction of velocity. Thus, / On any curved path (not necessarily a circular one) the
net acceleration in this case is also a = a t2 + a r2 but acceleration of the particle has two components a t and a r in
two mutually perpendicular directions. Component of a along
angle between v and a is obtuse. v is a t and perpendicular to v is a r . Thus,
v
| a | = a 2t + a 2r
θ
ar
at
7.2 Dynamics of Circular
a Motion
Fig. 7.5 If a particle moves with constant speed in a circle,
motion is called uniform circular motion. In uniform circular
θ = obtuse motion a resultant non-zero force acts on the particle. This is
a = a t2 + a r2 because a particle moving in a circle is accelerated even, if
speed of the particle is constant. This acceleration is due to
dv d| v|
where, Ta t = or the change in direction of the velocity vector. As we have
dt dt seen in article 7.1 that in uniform circular motion tangential
v2 acceleration ( a t ) is zero. The acceleration 2of the particle is
and ar = or rω 2 v
r towards the centre and its magnitude is . Here, v is the
/ That radial acceleration (a r ) is also sometimes called normal speed of the particle and r the radius ofr the circle. The
acceleration (an ). direction of the resultant force F is, towards centre and its
X Example 7.1 A particle moves in a circle of radius magnitude is
mv 2
0.5 m with a linear speed of 2 m/s. Find its angular F = ma or F = or F = mrω 2 (as v = rω)
speed. r
Sol. The angular speed is Here, ω is the angular speed of the particle. This force F
v 2 is called the centripetal force. Thus, a centripetal force of
ω= = = 4 rad/s
r 0.5 mv 2
magnitude is needed to keep the particle moving in a
r
X Example 7.2 A particle moves in a circle of
circle with constant speed. This force is provided by some
radius 0.5 m at a speed that uniformly increases. Find
external source such as friction, magnetic force, Coulomb
the angular acceleration of particle, if its speed
force, gravitation, tension, etc.
changes from 2.0 m/s to 4.0 m/s in 4.0 s.
/ I have found students often confused over the O
Sol. The tangential acceleration of the particle is centripetal force. They think that this force acts on a
dv 4.0 − 2.0
at = = = 0.5 m/s 2 particle moving in a circle. This force does not act but
dt 4.0 required for moving in a circle which is being
The angular acceleration is provided by the other forces acting on the particle. T
a 0.5
α= t = = 1 rad/s 2 Let, us take an example. Suppose a particle of mass v
r 0.5 m is moving in a vertical circle with the help of a string
of length l fixed at point O. Let v be the speed of the mg
X Example 7.3 The speed of a particle moving in a
particle at lowest position. When I ask the students
circle of radius r = 2 m varies with time t as v = t 2 what forces are acting on the particle in this
Fig. 7.6
where, t is in second and v in m/s. Find the radial, position ? They immediately say, three forces are acting on the
tangential and net acceleration at t = 2 s. particle (1) tension T (2) weight mg and (3) centripetal force
Sol. Linear speed of particle at t = 2 s is mv 2
(r = l ).
v = (2 )2 = 4 m/s l
However, they are wrong. Only first two forces T and mg are
∴ Radial acceleration acting
v 2 (4)2 mv 2
ar = = = 8 m/s 2 on the particle. Third force is required for circular motion
r 2 l
The tangential acceleration is which is being provided by T and mg. Thus, the resultant of
dv mv 2
at = = 2t these two forces is towards O. Or we can write
dt l
∴ Tangential acceleration at t = 2 s is mv 2
T − mg =
a t = (2 ) (2 ) = 4 m/s 2 l
Circular Motion 265

Circular Turning of Roads part of the road is some what lifted compared to the inner
part.
When vehicles go through turnings, they travel along a
N
nearly circular arc. There must be some force which will
θ
produce the required centripetal acceleration. If the vehicles
mv 2 G
travel in a horizontal circular path, this resultant force is also r
horizontal. The necessary centripetal force is being provided
to the vehicles by following three ways.
1. By friction only. θ
2. By banking of roads only. mg
3. By friction and banking of roads both. Fig. 7.7
In real life the necessary centripetal force is provided by
Applying Newton’s second law along the radius and the
friction and banking of roads both. Now let us write
first law in the vertical direction, then
equations of motion in each of the three cases separately and
see what are the constraints in each case. mv 2
N sin θ =
r
By Friction Only and N cos θ = mg
Suppose a car of mass m is moving at a speed v in a From these two equations, we get
horizontal circular arc of radius r. In this case, the necessary v2
centripetal force to the car will be provided by force of tan θ = …(i)
rg
friction f acting towards centre.
or v = rg tan θ …(ii)
mv 2
Thus, f = / This is the speed at which car does not slide down even if
r
track is smooth. If track is smooth and speed is less than
Further, limiting value of f is µN rg tan θ , vehicle will move down so that r gets decreased
or f L = µN = µmg ( N = mg ) and if speed is more than this vehicle will move up.
Therefore, for a safe turn without sliding
mv 2 Conical Pendulum
≤ fL If a small particle of mass m tied to a string is whirled in
r
a horizontal circle, as shown in Fig. 7.8. The arrangement is
mv 2
or ≤ µmg called the conical pendulum. In case of conical pendulum
r
the vertical component of tension balances the weight while
v2 its horizontal component provides the necessary centripetal
or µ≥
rg force. Thus,
or v ≤ µrg mv 2
T sin θ = …(i)
r
Here, two situations may arise. If µ and r are known to
us, the speed of the vehicle should not exceed µrg and if v
and r are known to us, the coefficient of friction should be θ
v2 L
greater than .
rg
T
/ You might have seen that if the speed of the car is too high, θ
r m
car starts skidding outwards. With this radius of the circle
increases or the necessary centripetal force is reduced
r = L sin θ
(centripetal force ∝ 1 ) . mg
r
Fig. 7.8

By Banking of Roads Only and T cos θ = mg …(ii)


Friction is not always reliable at circular turns, if high From these two equations, we can find
speeds and sharp turns are involved. To avoid dependence
on friction, the roads are banked at the turn so that the outer v = rg tan θ
266 Objective Physics Vol. 1

v g tan θ This resultant force imparts a centripetal acceleration to


∴ Angular speed ω = = the cyclist.
r r
F
So the time period of pendulum is
2π r N mv 2
T= = 2π G G
ω g tan θ θ r
θ
L cos θ f
= 2π
g mg mg
Fig. 7.10
L cos θ
or T = 2π Resultant of N and f , i.e. F should pass through G, the
g centre of gravity of cyclist (for complete equilibrium,
/ This is similar to the case, when necessary centripetal force rotational as well as translational). Hence,
to vehicles is provided by banking. The only difference is that f mv 2
the normal reaction is being replaced by the tension. tan θ = , where f =
N r
2
v
‘Death Well’ or Rotor and N = mg ⇒ ∴ tan θ =
rg
In case of ‘death well’ a person drives a bicycle on a
vertical surface of a large wooden well while in case of a Centrifugal Force
rotor at a certain angular speed of rotor a person hangs
Newton’s laws are valid only in inertial frames. In
resting against the wall without any support from the
non-inertial frames a pseudo force −ma has to be applied on
bottom. In death well walls are at rest and person revolves
a particle of mass m (a = acceleration of frame of reference).
while in case of rotor person is at rest and the walls rotate.
After applying the pseudo force one can apply Newton’s
r laws in their usual form. Now suppose a frame of reference
f
is rotating with constant angular velocity ω in a circle of
N
f radius r. Then it will become a non-inertial frame of
N r
mg
acceleration rω 2 towards the centre.
mg Now, if we see an object of mass m from this frame then
obviously a pseudo force of magnitude mrω 2 will have to be
applied to this object in a direction away from the centre.
(A) (B)
This pseudo force is called the centrifugal force. After
applying this force we can now apply Newton’s laws in their
Death well Rotor
usual form. Following example will illustrate the concept
Fig. 7.9
more clearly.
In both cases friction balances the weight of person X Example 7.4 A particle of mass m is placed over a
while reaction provides the centripetal force for circular horizontal circular table rotating with an angular
motion, i.e. velocity ω about a vertical axis passing through its
f = mg centre. The distance of the object from the axis is r. Find
mv 2 the force of friction f between the particle and the table.
and N = = mrω 2 ( v = rω )
r Sol. Let us solve this problem from both frames. The one is a frame
fixed on ground and the other is a frame fixed on table itself.
A Cyclist on the Bend of a Road ω N
In figure,
N = normal reaction
F = N2 + f 2 r f
mg = weight

When the cyclist is inclined to the centre of the rounding f = force of friction

of its path, the resultant of N , f and mg is directed mg


horizontally to the centre of the circular path of the cycle. Fig. 7.11
Circular Motion 267

From Frame of Reference Fixed on Ground (Inertial) towards O. This force will be provided by the resultant
of T and mg cos α. Thus,
Here, N will balance its weight and the force of friction mv 2
or T − mg cos α =
f will provide the necessary centripetal force. L
Thus, f = mrω 2  v2 
T = m  g cos α + 
 L 
From Frame of Reference Fixed on Table Itself 2
(Non-inertial)  mv 2 
(ii) |Fnet | = (mg sin α )2 +  
In the free body diagram of particle with respect to  L 
table, in addition to above three forces (N, mg and f ) a v4
= m g 2 sin2 α +
pseudo force of magnitude mrω 2 will have to be applied in a L2
direction away from the centre. But one thing should be
clear that in this frame the particle is in equilibrium, i.e. N X Example 7.6 A hemispherical bowl of radius R is
will balance its weight in vertical direction while f will rotating about its axis of symmetry which is kept
balance the pseudo force in horizontal direction. vertical. A small ball kept in the bowl rotates with the
N bowl without slipping on its surface. If the surface of
the bowl is smooth and the angle made by the radius
through the ball with the vertical is α. Find the angular
f Pseudo force = mrω2 speed at which the bowl is rotating.
Sol. Let ω be the angular speed of rotation of the bowl.Two forces
are acting on the ball.
mg 1. Normal reaction N
Fig. 7.12 2. Weight mg
or f = mrω 2 ω

Thus, we see that f comes out to be mrω 2 from both the R α


frames. N
A
Now, let us take few more examples of circular motion. r

mg
X Example 7.5 A simple pendulum is constructed
Fig.7.14
by attaching a bob of mass m to a string of length L
fixed at its upper end. The bob oscillates in a vertical The ball is rotating in a circle of radius r (= R sin α ) with
circle. It is found that the speed of the bob is v when the centre at A at an angular speed ω. Thus,
string makes an angle α with the vertical. Find the N sin α = mrω2
tension in the string and the magnitude of net force on = mRω2 sinα …(i)
the bob at that instant.
and N cos α = mg …(ii)
Sol. (i) The forces acting on the bob are Dividing Eqs. (i) by (ii), we get
1 ω2 R
=
cos α g
α
g
∴ ω=
T R cos α

mg sin α
α 7.3 Motion in a Vertical Circle
mg cos α Suppose a particle of mass m is attached to an
mg
Fig. 7.13
inextensible light string of length R. The particle is moving
in a vertical circle of radius R about a fixed point O. It is
(a) the tension T imparted a velocity u in horizontal direction at lowest point
(b) the weight mg
A. Let v be its velocity at point B of the circle as shown in
As the bob moves in a circle of radius L with centre
mv 2 figure. Here
at O. A centripetal force of magnitude is required
L h = R (1 − cos θ ) …(i)
268 Objective Physics Vol. 1

From conservation of mechanical energy Condition of Leaving the Circle


( 2gR < u < 5gR )
O v
T
If u < 5gR , the tension in the string will become zero
θ B before reaching the highest point. From Eq. (iii), tension in
R mg cos θ
h
mg sin θ
the string becomes zero (T = 0)
u
A −v 2 2gh − u 2
Fig. 7.15 where, cos θ = or cos θ =
Rg Rg
1
m ( u 2 − v 2 ) = mgh or v 2 = u 2 − 2gh …(ii) Substituting, this value of cos θ in Eq. (i), we get
2
2gh − u 2 h
The necessary centripetal force is provided by the =1 −
resultant of tension T and mg cos θ Rg R
mv 2 u 2 + Rg
∴ T − mg cos θ = …(iii) or h= = h1 (say) …(iv)
R 3g
Now, following three conditions arise depending on the
or we can say that at height h1 tension in the string
value of u.
becomes zero. Further, if u < 5gR , velocity of the particle
Condition of Looping the Loop becomes zero, when
0 = u 2 − 2gh
(u ≥ 5 gR )
u2
or h= = h2 (say) …(v)
The particle will complete the circle, if the string does 2g
not slack even at the highest point (θ = π ). Thus, tension in
the string should be greater than or equal to zero (T ≥ 0) at i.e. at height h2 velocity of particle becomes zero.
θ = π. In critical case substituting T = 0 and θ = π in Eq. (iii), Now, the particle will leave the circle, if tension in the
we get string becomes zero but velocity is not zero. or T = 0 but
2
mv min v ≠ 0. This is possible only when
mg = 2
or v min = gR or v min = gR
R u 2 + Rg u 2
h1 < h2 or <
(at highest point) 3g 2g
Substituting θ = π in Eq. (i), h = 2R or 2u 2 + 2Rg < 3u 2 or u 2 > 2Rg
Therefore, from Eq. (ii)
or u > 2Rg
2
u min = v min
2
+ 2gh
Therefore, if 2gR < u < 5gR , the particle leaves the
or 2
u min = gR + 2g (2R ) = 5 gR or u min = 5 gR
circle.
Thus, if u ≥ 5gR , the particle will complete the circle. v
T=0
P v≠0
At u = 5gR , velocity at highest point is v = gR and
tension in the string is zero. h>R
P T=0 O θ
vmin = gR
R

O u
A
Fig. 7.17
R
umin = 5gR
u From Eq. (iv), we can see that h > R , if u 2 > 2gR . Thus,
T = 6 mg
A the particle, will leave the circle when h > R or
Fig. 7.16 90° < θ < 180° . This situation is shown in the figure.
Substituting θ = 0° and v = 5gR in Eq. (iii), we get 2gR < u < 5gR
T = 6 mg or in the critical condition tension in the string at
lowest position is 6 mg. This is shown in figure. or 90° < θ < 180°
/ That after leaving the circle, the particle will follow a parabolic
If u < 5gR , following two cases are possible. path.
Circular Motion 269

Condition of Oscillation (0 < u ≤ 2gR ) Condition of Leaving the Circle


The particle will oscillate, if velocity of the particle 2gR < u < 5gR
becomes zero but tension in the string is not zero. or v = 0,
but T ≠ 0. v
N=0
v≠0
This is possible when h2 < h1
θ
u 2 u 2 + Rg h>R
or <
2g 3g
u
or 3u 2 < 2u 2 + 2Rg
Fig. 7.20
or u 2 < 2Rg
or u < 2Rg
Condition of Oscillation is 0 < u ≤ 2 gR
Moreover, if h1 = h2 , u = 2Rg and tension and velocity
both becomes zero simultaneously.

O v=0
N≠0
u h≤R
θ v=0
R T≠0
P h≤R Fig. 7.21
u
A
Fig. 7.18
Motion of a Ball over a Smooth Solid
Sphere
Further, from Eq. (iv), we can see that h ≤ R , if Suppose a small ball of mass m is given a velocity v over
u ≤ 2Rg . Thus, for 0 < u ≤ 2gR , particle oscillates in the top of a smooth sphere of radius R. The equation of
lower half of the circle (0° < θ ≤ 90° ). This situation is motion for the ball at the topmost point will be
shown in the figure. u=0
v
0 < u ≤ 2gR h
N=0
or 0° < θ ≤ 90° θ
θ
/ The above three conditions have been derived for a particle
mg v
moving in a vertical circle attached to a string. The same R
conditions apply, if a particle moves inside a smooth
spherical shell of radius R. The only difference is that the
tension is replaced by the normal reaction N.
(a) (b)
Condition of Looping the Loop is Fig. 7.22

u ≥ 5gR mv 2
mg − N =
v = √gR, N = 0 R
mv 2
v or N = mg −
R
From this equation we see that value of N decreases as v
increases. Minimum value of N can be zero. Hence,
2
u mv max
0 = mg −
R
u = √5gR, N = 6 mg
Fig. 7.19
or v max = Rg
270 Objective Physics Vol. 1

From here we can conclude that ball will lose contact Here, u 2 = gl …(ii)

with the sphere right from the beginning, if velocity of ball and v = speed of particle in position B
at topmost point v > Rg . If v < Rg it will definitely lose ∴ v 2 = u 2 − 2 gh …(iii)
2
contact but after moving certain distance over the sphere. Further, T − mg cos θ =
mv
Now let us find the angle θ where the ball loses contact with l
the sphere if velocity at topmost point is just zero. mv 2
or mg − mg cos θ = (T = mg )
Fig. 7.22 (b) l
h = R (1 − cos θ ) …(i) or v 2 = gl(1 − cos θ) …(iv)
2 2
v = 2gh
2
…(ii) Substituting values of v , u and h from Eqs. (iv), (ii) and (i)
in Eq. (iii), we get
mv 2
mg cos θ = (as N = 0) …(iii) gl(1 − cos θ) = gl − 2 gl(1 − cos θ)
R 2
or cos θ =
Solving Eqs. (i), (ii) and (iii), we get 3
2
 2 θ = cos −1  
θ = cos −1   = 48.2°
or
 3
 3
2
Substituting cos θ = in Eq. (iv), we get
Thus, the ball can move on the sphere maximum upto 3
 2 gl
θ = cos −1   . v=
 3 3

Exercise : Find angle θ where the ball will lose


contact with the sphere if velocity at topmost point is Extra Knowledge Points
v gR ■ In circular motion linear velocity is always tangential to
u = max = . the circle (either clockwise or anti-clockwise).
2 2
v v
 3
θ = cos −1   = 41.4°
 4 or

Hint : Only Eq. (ii) will change as,


v 2 = u 2 + 2gh ( u ≠ 0)
(a) (b)
X Example 7.7 A heavy particle hanging from a ■ In uniform circular motion acceleration (also called the
fixed point by a light inextensible string of length l is centripetal acceleration) is always towards the centre
projected horizontally with speed gl. Find the speed  v2 
of the particle and the inclination of the string to the = or rω 2  .
 r 
vertical at the instant of the motion when the tension in
■ If a particle of mass m is connected to a light rod and
the string is equal to the weight of the particle.
whirled in a vertical circle of radius R, then to complete
Sol. Let T = mg at angle θ as shown in figure. the circle, the minimum velocity of the particle at the
h = l ( 1 − cos θ) …(i) bottommost point is not 5 gR . Because in this case,
Applying conservation of mechanical energy between velocity of the particle at the topmost point can be zero
also. Using conservation of mechanical energy
points A and B, we get
between points A and B as shown in figure, we get
v=0 B

d
T B h = 2R
θ O
mg cos θ R
R
h mg sin θ
u
A u = √ gl A
u ≥ 2 √gR u ≥ 2 √gR
Fig. 7.23
d<<R
1
m(u 2 − v 2 ) = mgh 1
m(u 2 − v 2 ) = mgh
2 2
Circular Motion 271

1
or mu 2 = mg ( 2 R ) (as v = 0)
2
∴ u = 2 gR
Therefore, the minimum value of u in this case is 2 gR .Same is the case when a particle is compelled to move inside a
smooth vertical tube as shown in figure.
■ In uniform circular motion although the speed of the particle remains constant yet the particle is accelerated due to change
in direction of velocity. Therefore the forces acting on the particle in uniform circular motion can be resolved in two directions
one along the radius (parallel to acceleration) and another perpendicular to radius (perpendicular to acceleration). Along
mv 2
the radius net force should be equal to and perpendicular to it net force should be zero.
R
■ Oscillation of a pendulum is part of a circular motion. At point A and C since velocity is zero, net centripetal force will be zero.
Only tangential force is present. From A and B or C to B speed of the bob increases. Therefore, tangential force is parallel to
velocity. From B to A or B to C speed of the bob decreases. Hence, tangential force is antiparallel to velocity.

C A
B

Chapter Summary with Formulae


(i) Circular Motion A particle in circular motion may have two types of speeds.
(a) Linear speed v and
(b) Angular speed ω
These two are related by the equation
v = Rω (R = radius of circular path)
(ii) Centripetal acceleration in uniform circular motion :
v2
a= = Rω2
R
Direction of this acceleration is always towards centreand lying in the plane of circle.
(iii) Linear velocity in circular motion is always tangential to the circle and lying in the plane of circle.
(iv) Angular velocity is perpendicular to plane of circle given by screw law.
(v) v = ω × r
(vi) In uniform circular motion body is accelerated.
(vii) In uniform circular motion linear speed and angular velocity are constants. Linear velocity and linear acceleration keep on changing,
due to change in their directions.
Additional Examples
Example 1. For uniform circular motion, does the  θ v2  θ
a n = g cos   ⇒ ∴ = g cos   … (ii)
direction of centripetal acceleration depends upon the  2 R  2
sense of rotation ? Substituting value of v from Eq. (i) in Eq. (ii), we have
Sol. The direction of centripetal acceleration does not depend radius of curvature, v
on the clockwise or anti-clockwise sense of rotation of the 2
 u cos θ  θ/2
body. It always acts along the radius towards the centre of the  
circle.  cos  θ 
  2  u 2 cos 2 θ θ/2
Example 2. A stone tied to the end of a string is R=  =
 θ  θ
whirled in a circle. If the string breaks, the stone files g cos   g cos 3   g
 2  2
away tangentially. Why ?
Sol. The instantaneous velocity of the stone going around the Example 5. Two particles A Y
circular path is always along tangent to the circle. When the
string breaks, the centripetal force ceases. Due to inertia, the
and B start at the origin O and
travel in opposite directions m
stone continues its motion along tangent to the circular path. 5.0
along the circular path at
Example 3. A particle moves in a circle of radius constant speeds v A = 0.7m/s and B
2.0 cm at a speed given by v = 4 t , where v is in cm/s v B = 1.5 m/ s, respectively.
A
X
and t in seconds. Determine the time when they vB = 1.5 m/s O v = 0.7 m/s
A
(a) Find the tangential acceleration at t =1 s. collide and the magnitude of the
(b) Find total acceleration at t =1 s. acceleration of B just before this happens.
Sol. (a) Tangential acceleration Sol. 1.5 t + 0.7 t = 2πR = 10 π
dv d
at = or at = (4t ) = 4 cm/s 2 10π v 2
dt dt ∴ t= = 14.3 s ⇒ a = B = 0.45 m/s 2
i.e. at is constant or tangential acceleration at 2.2 R
t = 1 s is 4 cm/s 2 .
Example 6. A boy whirls a stone in a horizontal
(b) Normal acceleration circle of radius 1.5 m and at height 2.0 m above level
v 2 (4t ) 2 16t 2 ground. The string breaks, and the stone flies off
an = = or a n = = 8.0 t 2
R R 2.0 horizontally and strikes the ground after travelling a
At t = 1 s, a n = 8.0 cm/s 2 horizontal distance of 10 m. What is the magnitude of
∴ Total acceleration a = at2 + a n2 the centripetal acceleration of the stone while in
circular motion?
or a = (4 ) 2 + (8) 2
2h 2× 2
= 80 = 4 5 cm/s 2 Sol. t = = = 0.64 s
g 98
.
10
Example 4. A particle is projected with a speed u v= = 15.63 m/s
at an angle θ with the horizontal. What is the radius of t
curvature of the parabola traced out by the projectile v2
∴ a= = 163 m/s 2
at a point, where the particle velocity makes an angle R
θ
with the horizontal? Example 7. A car is travelling along a circular
2 curve that has a radius of 50 m. If its speed is 16 m/s
Sol. Let v be the velocity at the desired point. Horizontal and is increasing uniformly at 8 m/s 2 , determine the
component of velocity remains unchanged. Hence, magnitude of its acceleration at this instant.
v cos θ/ 2 = u cos θ 2
u cos θ  dv
2
 v2 
∴ v= … (i) Sol. a = at + a n
2 2
=   + 
cos θ/ 2  dt   R
Radial acceleration is the component of acceleration 2
 256 
perpendicular to velocity or = (8) 2 +   = 9.5 m/s
2
 50 
NCERT Selected Questions
Q 1. A stone tied to the end of a string 80 cm long is 5 15 −1
Sol. Speed of cyclist v = 27 kmh −1 = 27 × ms−1 = ms
whirled in a horizontal circle with a constant speed. 18 2
If the stone makes 14 revolutions in 25 s, what is the Radius of circular turn, r = 80 m.
magnitude and direction of acceleration of the Here, the cyclist will be acted upon by two accelerations ac
stone? and at .
Centripetal acceleration (ac ) is given by
Sol. Radius of the horizontal circle, r = 80 cm = 0.80 m
2
n = 14  15
 
v2
 2
t = 25 s ac = = = 0.703 ms−2
r 80
Angular speed of revolution of the stone,
v
θ 2πn  n
ω= = = 2π  
t t  t
22  14 
=2× ×  O ac
7  25 P
88 θ
= rads−1 a
25
at
∴Magnitude of acceleration produced in the stone will be
equal to the magnitude of centripetal acceleration
= rω 2 Let P be the point at which the cyclist applies brakes, then
2 the tangential acceleration at (which will be negative) will
 88
= 0.80 ×   act opposite to velocity of cyclist and is given by
 25
Change in velocity
= 9.91 ms−2 at =
Time
The direction of the acceleration is towards the centre. = 0.5 ms−2
Q 2. An aircraft executes a horizontal loop of radius 1 km As both ac and at act along the radius of circle and tangent to
with a speed of 900 kmh −1 . Compare its centripetal the circle respectively, so ac and at are mutually
acceleration with the acceleration due to gravity. perpendicular, thus the magnitude of the net acceleration a,
is given by
Sol. Here, r = 1km = 1000 m
a = ac2 + at2
1000
v = 900 kmh −1 = 900 × ms−1 = (0.703)2 + (0.5)2
3600
= 250 ms−1 = 0.863 ms−2
The centripetal acceleration of the aircraft is given by Direction of a Let θ be the angle made by the net
2 2 acceleration with the velocity of cyclist, then
v (250) 62500
a= = = a 0.703
r 1000 1000 tan θ = c =
at 0.5
= 62.5 ms−2
= 1.406
Acceleration due to gravity,
−2 ∴ θ = 54.44 °
g = 9.8 ms
Centripetal acceleration a 62.5 Q 4. Read each statement below carefully and state with
∴ = =
Acceleration due to gravity g 9.8 reasons, if it is true or false.
a (a) The net acceleration of a particle in the circular
or = 6.4 or a = 6.4 g motion is always along the radius of the circle
g
towards the centre.
Q 3. A cyclist is ridding with a speed of 27 kmh −1 . As he (b) The velocity vector of a particle at a point is
approaches a circular turn on the road of radius always along the tangent to the path of the
80 m, he applies brakes and reduces his speed at the particle at that point.
constant rate of 0.5 ms −1 every second. What is the (c) The acceleration vector of a particle in uniform
magnitude and direction of the net acceleration of circular motion averaged over one cycle is a null
the cyclist on the circular turn? vector.
274 Objective Physics Vol. 1

Sol. (a) The statement is false since, the centripetal acceleration Q 7. A stone of mass m tied to the end of a string revolves
is towards the centre only in the case of uniform circular in a vertical circle of radius R. The net forces at the
motion (constant speed) for which it is true. lowest and highest points of the circle directed
(b) True, the velocity of a particle is always along the vertically downwards are (choose the correct
tangent to the path of the particle at that point either in alternative).
rectilinear or circular or curvilinear motion.
Lowest point Highest point
(c) True, because in one cycle change in velocity = 0
(a) mg – T1 mg + T2
Q 5. One end of a string of length l is connected to a (b) mg + T1 mg − T2
particle of mass m and the other to a small peg on (c) mg + T1 − (mv12 )/ R mg − T2 + (mv12)/R
smooth horizontal table. If the particle moves in a (d) mg – T1 − (mv12)/R mg + T2 + (mv12)/R
circle with speed v, the net force on the particle
(directed towards the centre) is Here, T1 , T2 and ( v1 , v 2 ) denote the tension in the
string (and the speed of the stone) at the lowest and
mv 2
(i) T (ii) T − the highest point respectively.
l
2 Sol. In the figure shown, here L and H show the lowest and
mv
(iii) T + (iv) 0 highest points respectively.
l H

Sol. (i) The net force on the particle is T, because tension T mg


mv 2 T2
provides the necessary centripetal force, i.e. T =
l
C
Q 6. A stone of mass 0.25 kg tied to the end of a string is T1
whirled round in a circle of radius 1.5 m with speed
40 rev/min in a horizontal plane. What is the tension
in the string? What is the maximum speed with L
which the stone can be whirled around, if the string
can withstand a maximum tension of 200 N? mg

Sol. Frequency of revolution of stone, At point L T1 acts towards the centre of the circle and mg
40 acts vertically downward.
f = 40rev/min = rev/s
60 ∴ Net force on the stone at the lowest point in the downward
Angular speed of the stone, ω = 2πf direction = mg − T1
40 4 π At point H Both T2 and mg act vertically downward
= 2π × = rad s−1 towards the centre of the circle.
60 3
The centripetal force is provided by the tension (T ) in the ∴ Net force on the stone at the highest point in the downward
string direction = T2 + mg
mv 2 So, option (a) is the correct alternative.
i.e. T = = mrω 2
r
2 Q 8. An aircraft executes a horizontal loop at a speed of
 4π 
= 0.25 × 1.5 ×   N 720 km/h with its wings banked at 15°. What is the
 3
radius of the loop?
= 6.58 N ≈ 6.6 N
As the string can withstand a maximum tension of 200 N Sol. Speed of aircraft, v = 720 kmh −1
2 5
∴ Tmax =
mvmax = 720 × = 200 ms−1
r 18
rTmax v2
or vmax = For a banked curve, tan θ =
m rg
1.5 × 200 v2
= or r=
0.25 g tan θ
= 34.64 ms−1 =
(200)2
≈ 35.0 ms−1 9.8 × tan 15°

∴ T = 6.6 N, = 15.24 × 103 m


vmax = 35.0 ms−1 = 15.24 km
Circular Motion 275

Q 9. A train rounds a circular bend of radius 30 m at a g = 9.8 ms−2


speed of 54 kmh −1 . The mass of the train is 10 6 kg. ∴ From Eq. (ii),vmin = 9.8 × 25
What is the angle of banking required to prevent = 15.65 ms−1
wearing out of the rail?
≈ 16 ms−1
Sol. Radius of circular bend, r = 30 m
Q 11. A 70 kg man stands in contact against the inner wall
Speed of train = v = 54 kmh −1
of a hollow cylindrical drum of radius 3 m rotating
5 about its vertical axis with 200 rev/min. The
= 54 ×
18 coefficient of friction between the wall and his
= 15 ms−1 clothing is 0.15. What is the minimum rotational
The angle of banking is given by speed of the cylinder to enable the man to remain
stuck to the wall (without falling) when the floor is
v2
tan θ = suddenly removed?
rg
Sol. The cylinder being vertical, the normal reaction of the wall
= (15)2 / 30 × 9.8 on the man acts horizontally and provides the necessary
= 0.7653 centripetal force
∴ θ = 37°25′ = 37.42° ∴ N = mrω 2 …(i)

Q 10. You may have seen in a circus, a motorcyclist The frictional force f , acting upwards balances his weight
driving in vertical loop inside a ‘death well’ i.e. f = mg …(ii)
(a hollow spherical chamber with holes, so that The man will remain stuck to the wall after the floor is
spectators can watch from outside). What is the removed, i.e. he will continue to rotate with the cylinder
minimum speed required at the uppermost position without slipping
to perform a vertical loop, if the radius of the if f ≤ µN
chamber is 25 m? or mg ≤ µmrω 2
Sol. When the motorcyclist is at the highest point of the ‘death g
well’, the normal reaction N on him by the ceiling of the or ω2 ≥
µr
chamber acts downwards. His weight mg also acts
downwards. g
or ω≥ …(iii)
mv 2 µr
∴ N + mg = …(i)
r ∴ The minimum angular speed of rotation of the cylindrical
where, v = speed of the motorcyclist, drum is given by
m = mass of the motorcyclist (mass of motorcycle + driver) g
ω min = …(iv)
The minimum speed required to perform a vertical loop is µr
given by Eq. (i), when N = 0 Given µ = 0.15, r = 3 m , g = 9.8 ms−2 …(v)
2
mg =
mvmin ∴ From Eqs. (iv) and (v), we get
r 9.8
ω min =
∴ vmin = gr …(ii) 0.15 × 3
Here, r = radius of the loop = 4.67 rad / s ≈ 5rad/s
Objective Problems
[ Level 1 ]
Kinematics of Circular Motion 8. The ratio of angular speeds of minute hand and hour hand
of a watch is
1. A particle is moving on a circular path of 10 m radius. At
any instant of time its speed is 5 m/s and the speed is (a) 1 : 12 (b) 6 : 1
(c) 12 : 1 (d) 1 : 6
increasing at a rate of 2 m/s 2 . At this instant the
magnitude of the net acceleration will be 9. The angular speed of a flywheel making 120 rev/min is
(a) 3.2 m/s 2 (b) 2 m/s 2 (a) 2π rad / s (b) 4 π 2 rad /s
(c) 2.5 m/s 2 (d) 4.3 m/s 2
(c) π rad / s (d) 4 π rad / s
2. In a typical projectile motion tangential acceleration at 10. A particle moves with constant speed v along a circular
the topmost point P of the trajectory is path of radius r and completes the circle in time T. The
y (Vertical)
acceleration of the particle is
2πv 2πr
(a) (b)
T T
v0 2πr2 2πv 2
P
(c) (d)
T T

θ 11. A body is moving in a circular path with acceleration a. If


x (Horizontal) its speed gets doubled, find the ratio of acceleration after
(a) g (b) g cos θ and before the speed is changed
(c) 0 (d) None of these (a) 1 : 4 (b) 1 : 2
(c) 2 : 1 (d) 4 : 1
3. A point on the rim of a flywheel has a peripheral speed of
10 m/s at an instant when it is decreasing at the rate of 12. The circular orbit of two satellites have radii r1 and r2
60 m/s 2 . If the magnitude of the total acceleration of the respectively ( r1 < r2 ). If angular velocities of satellites are
point at this instant is 100 m/s 2 , the radius of the flywheel same, then their centripetal accelerations are related as
is (a) a1 > a2 (b) a1 = a2
(a) 1.25 m (b) 12.5 m (c) a1 < a2 (d) Data insufficient
(c) 25 m (d) 2.5 m
13. The wheel of a toy car rotates about a fixed axis. It slows
4. A particle is moving on a circular track of radius 30 cm down from 400 rps to 200 rps in 2 s. Then, its angular
with a constant speed of 6 m/s. Its acceleration is retardation in rad/s 2 is (rps = revolutions per second)
(a) zero (b) 120 m/s 2 (a) 200 π (b) 100
(c) 1.2 m/s 2 (d) 36 m/s 2 (c) 400π (d) None of these
5. In case of a uniform circular motion, velocity and 14. A car is circulating on a circular path of radius r. At some
acceleration are instant its velocity is v and rate of increase of speed is a.
(a) perpendicular (b) in same direction The resultant acceleration of the car will be
(c) in opposite direction (d) not related to each other v2 v2
(a) + r2 (b) +a
6. Let a r and a t represent radial and tangential a2 r
accelerations. The motion of a particle may be circular, if v4  v2 
(c) + a2 (d)  + a
(a) ar = 0, at = 0 (b) ar = 0, at ≠ 0 r2  r 
(c) ar ≠ 0, at = 0 (d) None of these
15. A point starts from rest and moves along a circular path
7. The speed of a particle moving in a circle is increasing. with a constant tangential acceleration. After one
The dot product of its acceleration and velocity is rotation, the ratio of its radial acceleration to its
(a) negative tangential acceleration will be equal to
(b) zero (a) 1 (b) 2π
(c) positive 1
(c) π (d) 4π
(d) may be positive or negative 2
Circular Motion 277

16. A particle moves in a circular path of radius R with an 23. A motorcyclist wants to drive on the vertical surface of
angular velocity ω = a − bt, where a and b are positive wooden ‘well’ of radius 5 m, with a minimum speed of
constants and t is time. The magnitude of the acceleration 5 5 m/s. The minimum value of coefficient of friction
2a between the tyres and the wall of the well must be
of the particle after time is
b (Take g = 10 m/s 2 )
a
(a) (b) a2R (a) 0.10 (b) 0.20
R (c) 0.30 (d) 0.40
(c) R (a2 + b) (d) R a4 + b2
24. A particle of mass 2 kg is moving along a circular path of
17. A car wheel is rotated to uniform angular acceleration radius 1 m. If its angular speed is 2π rad s −1 , the
about its axis. Initially its angular velocity is zero. It centripetal force on it is
rotates through an angle θ1 in the first 2 s. In the next 2 s, (a) 4π N (b) 8π N
θ (c) 4 π 4 N (d) 8π 2 N
it rotates through an additional angle θ 2 , the ratio of 2 is
θ1
25. Two particles of equal masses are revolving in circular
(a) 1 (b) 2
paths of radii r1 and r2 respectively with the same speed.
(c) 3 (d) 4
The ratio of their centripetal forces is
Dynamics of Circular Motion (a)
r2
(b)
r2
r1 r1
18. A coin, placed on a rotating turn-table slips, when it is 2 2
placed at a distance of 9 cm from the centre. If the angular r  r 
(c)  1  (d)  2 
velocity of the turn-table is trippled, it will just slip, if its  r2   r1 
distance from the centre is
(a) 27 cm (b) 9 cm 26. A mass of 100 g is tied to one end of a string 2 m long.
(c) 3 cm (d) 1 cm The body is revolving in a horizontal circle making a
maximum of 200 rev/min. The other end of the string is
19. If mass, speed and radius of the circle of a particle fixed at the centre of the circle of revolution. The
moving uniformly in a circular path are all increased by maximum tension that the string can bear is
50%, the necessary force required to maintain the body (approximately)
moving in the circular path will have to be increased by
(a) 8.76 N (b) 8.94 N
(a) 225% (b) 125%
(c) 89.42 N (d) 87.64 N
(c) 150% (d) 100%
27. A particle of mass mis executing uniform circular motion
20. A ball is placed on a smooth inclined plane of inclination
on a path of radius r. If p is the magnitude of its linear
θ = 30° to the horizontal, which is rotating at frequency
momentum. The radial force acting on the particle is
0.5 Hz about a vertical axis passing through its lower
rm
end. At what distance from the lower end does the ball (a) pmr (b)
p
remain at rest?
(a) 0.87 m (b) 0.33 m (c) 0.5 m (d) 0.67 m mp2 p2
(c) (d)
r rm
21. A person wants to drive on the vertical surface of a large
cylindrical wooden ‘well’ commonly known as ‘death 28. A mass of 2 kg is whirled in a horizontal circle by means
well’ in a circus. The radius of the well is R and the of a string at an initial speed of 5 rev/min. Keeping the
coefficient of friction between the tyres of the motorcycle radius constant the tension in the string is doubled. The
and the wall of the well is µ s . The minimum speed, the new speed is nearly
motorcycle must have in order to prevent slipping, should 5
(a) rpm (b) 10 rpm
be 2
Rg µs µ sg R (c) 10 2 rpm (d) 5 2 rpm
(a) (b) (c) (d)
µs Rg R µ sg
29. A stone of mass of 16 kg is attached to a string 144 m
22. The maximum tension that an inextensible ring of radius long and is whirled in a horizontal circle. The maximum
1m and mass density 0.1 kg m −1 can bear is 40 N. The tension the string can withstand is 16 N. The maximum
maximum angular velocity with which it can be rotated in velocity of revolution that can be given to the stone
a circular path is without breaking it, will be
(a) 20 rad/s (b) 18 rad/s (a) 20 ms−1 (b) 16 ms−1
(c) 16 rad/s (d) 15 rad/s (c) 14 ms−1 (d) 12 ms−1
278 Objective Physics Vol. 1

30. Three identical particles are joined together by a thread as 37. A particle is moving in a vertical circle. The tensions in
shown in figure. All the three particles are moving in a the string when passing through two positions at angles
horizontal plane. If the velocity of the outermost particle 30° and 60° from vertical (lowest position) are T1 and T2
is v 0 , then the ratio of tensions in the three sections of the respectively, then
string is (a) T1 = T2 (b) T2 > T1
O A B C (c) T1 > T2 (d) Data insufficient
l l l 38. A body of mass 1 kg is moving in a vertical circular path
(a) 3 : 5 : 7 (b) 3 : 4 : 5 of radius 1 m. The difference between the kinetic
(c) 7 : 11 : 6 (d) 3 : 5 : 6 energies at its highest and lowest positions is
(a) 20 J (b) 10 J
31. Toy cart tied to the end of an unstretched string of length
(c) 4 5 J (d) 10( 5 − 1) J
a, when revolved moves in a horizontal circle of radius 2a
with a time period T. Now, the toy cart is speeded up until 39. A particle of mass m is being circulated on a vertical
it moves in a horizontal circle of radius 3a with a period circle of radius r. If the speed of particle at the highest
T. If Hooke’s law ( F = kx ) holds, then point be v, then
3  3 mv 2 mv 2
(a) T ′ = T (b) T ′ =   T (a) mg = (b) mg >
2  2 r r
 3 mv 2 mv 2
(c) T ′ =   T (d) T ′ = T (c) mg ≤ (d) mg ≥
 2 r r

32. A national roadway bridge over a canal is in the form of 40. The string of a pendulum is horizontal. The mass of bob
an arc of a circle of radius 49 m. What is the maximum attached to it is m. Now, the string is released. The tension
speed with which a car can move without leaving the in the string in the lowest positions, is
ground at the highest point? (Take g = 9.8 m/s 2 ) (a) mg (b) 2 mg
(c) 3 mg (d) 4 mg
(a) 19.6 m/s (b) 40 m/s
(c) 22 m/s (d) None of these 41. A bucket full of water is rotated in a vertical circle of
radius R. If the water does not split out, the speed of the
Vertical Circular Motion bucket at topmost point will be
33. A car when passes through a convex bridge exerts a force (a) Rg (b) 5gR
on it which is equal to  R
Mv 2 Mv 2
(c) 2Rg (d)  
(a) Mg + (b)  g
r r
Mv 2 42. A pendulum bob has a speed of 3 m/s at its lowest
(c) Mg − (d) None of these
r position. The pendulum is 0.5 m long. The speed of the
bob, when string makes an angle of 60° to the vertical is
34. A block of mass m at the end of a string is whirled round (g = 10 m /s 2 )
in a vertical circle of radius R. The critical speed of the 1
block at top of its swing below which the string would (a) 2 m/s (b) m/s
2
slacken before the block reaches the bottom is? (c) 1 m/s (d) 2.5 m/s
(a) 5 Rg (b) 3 Rg
(c) 2 Rg (d) Rg 43. A simple pendulum of length l has a maximum angular
displacement θ. The maximum kinetic energy of the bob
35. A sphere is suspended by a thread of length l. What of mass m will be
minimum horizontal velocity has to be imparted to the (a) mgl (1 − cos θ ) (b) mgl cos θ
ball for it to reach the height of the suspension? (c) mgl sin θ (d) None of these
(a) 5 gl (b) 2 gl 44. A small ball is pushed from a height h along a smooth
(c) gl (d) 2gl hemispherical bowl of radius R. With what speed should
the ball be pushed so that it just reaches the top of the
36. A pendulum bob on a 2m string is displaced 60° from the opposite end of the bowl?
vertical and then released. What is the speed of the bob as (a) 2gh
it passes through the lowest point in its path? (b) 2g (R + h)
(a) 2 m/s (b) 9.8 m/s
1 (c) 2g (R − h)
(c) 4.43 m/s (d) m/s
2 (d) None of the above
Circular Motion 279

45. A child is swinging a swing. Minimum and the maximum 50. A block of mass m slides from the rim of a hemispherical
heights of swing from the earth’s surface are 0.75 m and bowl of radius R. The velocity of the block at the bottom
2 m respectively. The maximum velocity of this swing is will be
( g = 10 m/s 2 ) (a) 3Rg (b) 2gR
(a) 5 m/s (b) 10 m/s (c) Rg (d) 4gR
(c) 15 m/s (d) 20 m/s
51. A body is moving in a vertical circle of radius r such that
46. A small body of mass m slides without friction from the the string is just taut at its highest point. The speed of the
top of a hemisphere of radius r. At what height will the particle when the string is horizontal is
body be detached from the centre of the hemisphere?
(a) gr (b) 2gR (c) 3gr (d) 4gR

52. A stone is attached to one end of a string and rotated in a


h
vertical circle. If string breaks at the position of
maximum tension, it will break at
r r C
(a) h = (b) h =
2 3
2r r
(c) h = (d) h =
3 4
D B
47. A frictionless track ABCDE ends in a circular loop of
radius R. A body slides down the track from point A
which is at height h = 5 cm. Maximum value of R for a
body to complete the loop successfully is A
D
E (a) A (b) B (c) C (d) D
A R C
5 cm
Miscellaneous Problems
B 53. A particle moves in a uniform circular motion. Choose
10 the wrong statement.
(a) 2 cm (b) cm
3 (a) The particle moves with constant speed
15 18 (b) The acceleration is always normal to the velocity
(c) cm (d) cm
4 3 (c) The particle moves with uniform acceleration
(d) The particle moves with variable velocity
48. A small sphere of mass m is suspended by a thread of
length l. It is raised upto the height of suspension with 54. A circular disc of radius R is rotating about its axis O with
thread fully stretched and released. Then, the maximum a uniform angular velocity ω rad/s as shown in the figure.
tension in thread will be The magnitude of the relative velocity of point A relative
to point B on the disc is
B
ω
l

O θ

(a) mg (b) 2 mg  θ
(a) zero (b) Rω sin  
(c) 3 mg (d) 6 mg  2
49. A stone of mass 1 kg is tied to the end of a string 1 m  θ  θ
(c) 2Rω sin   (d) 3 Rω sin  
long. It is whirled in a vertical circle. The velocity of the  2  2
stone at the bottom of the circle is just sufficient to take it
55. A circular curve of a highway is designed for traffic
to the top of circle without slackening of the string. What moving at 72 km/h. If the radius of the curved path is
is the tension in the string at the top of the circle? 100 m, the correct angle of banking of the road should be
(Take g = 10 ms −2 )  2  3  1  1
(a) tan −1   (b) tan −1   (c) tan −1   (d) tan −1  
(a) Zero (b) 1 N (c) 10 N (d) 10 N  5  5  5  4
280 Objective Physics Vol. 1

56. A stone is tied with a string and is rotated in a circle 64. A particle is moving in a circle with uniform speed v. In
horizontally. When the string suddenly breaks, the stone moving from a point to another diametrically opposite
will move point
(a) tangential to the motion (a) the momentum changes by mv
(b) away from the centre (b) the momentum changes by 2 mv
(c) towards the centre  1
(c) the kinetic energy changes by   mv 2
(d) None of the above  2
(d) the kinetic energy changes by mv 2
57. When the angular velocity of a uniformly rotating body
has increased thrice, the resultant of forces applied to it 65. An object is moving in a circle of radius 100 m with a
increases by 60 N. Find the accelerations of the body in constant speed of 31.4 m/s. What is its average speed for
the two cases. The mass of the body, m = 3 kg one complete revolution?
(a) 2.5 ms−2 , 7.5 ms−2 (a) Zero
(b) 7.5 ms−2 , 22.5 ms−2 (b) 31.4 m/s
(c) 3.14 m/s
(c) 5 ms−2 , 45 ms−2
(d) 2 × 31.4 m/s
(d) 2.5 ms−2 , 22.5 ms−2
66. In 1.0 s, a particle goes from point A to point B, moving
58. A particle is moving along a circular path of radius 5 m
in a semicircle of radius 1.0 m (see figure). The
with a uniform speed 5 ms −1 . What will be the average
magnitude of the average velocity is
acceleration when the particle completes half revolution?
A
(a) Zero (b) 10 ms−2
10 −2
(c) 10π ms−2 (d) ms
π 1.0m
 3
59. If the banking angle of curved road is given by tan −1  
 5
and the radius of curvature of the road is 6m, then the safe B
driving speed should not exceed ( g = 10 m/s −2 ) (a) 3.14 m/s (b) 2.0 m/s
(a) 86.4 km/h (b) 43.2 km/h (c) 1.0 m/s (d) zero
(c) 21.6 km/h (d) 30.4 km/h
67. A cyclist starts from the centre O of a circular park of
60. A particle moving along a circular path due to a radius 1 km, reaches the edge P of the park, then cycles
centripetal force having constant magnitude is an along the circumference and returns to the centre along
example of motion with QO as shown in the figure. If the round trip takes 10 min.,
(a) constant speed and velocity the net displacement and average speed of the cyclist
(b) variable speed and variable velocity (in m and km per hour) are
(c) variable speed and constant velocity
Q
(d) constant speed and variable velocity

61. A particle moves with constant angular velocity in a


circle. During the motion its P
(a) energy is conserved O
(b) momentum is conserved
(c) energy and momentum both are conserved
(d) None of the above
π+4
(a) 0, 1 (b) ,4
62. A car is moving on a circular path and takes a turn. If R1 2
and R 2 be the reactions on the inner and outer wheels π+4
(c) 21.4, (d) 0, 21.4
respectively, then 2
(a) R1 = R2 (b) R1 < R2
68. A particle of mass m is circulating on a circle of radius r
(c) R1 > R2 (d) R1 ≥ R2
having angular momentum L about centre. Then, the
63. An unbanked curve has a radius of 60 m. The maximum centripetal force will be
speed at which a car can make a turn, if the coefficient of L2 L2
(a) (b)
static friction is 0.75 is mr mr2
(a) 2.1 m/s (b) 14 m/s L2 L
(c) (d)
(c) 21 m/s (d) 7 m/s mr3 mr2
Circular Motion 281

69. Three particles A , B and C move in a circle of radius 71. A train has to negotiate a curve of radius 400 m. By how
1 much height should the outer rail be raised with respect to
r = m, in anti-clockwise direction with speed 1 m/s,
π inner rail for a speed of 48 km/h? The distance between
2.5 m/s and 2 m/s respectively. The initial positions of the rails is 1 m
A , B and C are as shown in figure. (a) 4.4 cm
B (b) 9 cm
(c) 2.2 cm
(d) 3.3 cm
A C 72. A string of length l fixed at one end carries a mass mat the
O
2
other end. The strings makes rev/s around the axis
π
through the fixed end as shown in the figure, the tension
The ratio of distance travelled by B and C by the instant
in the string is
A , B and C meet for the first time is
(a) 3 : 2 (b) 5 : 4
(c) 3 : 5 (d) 3 : 7 θ
l
70. A motorcyclist moving with a velocity of 72 km/h on a T
flat road takes a turn on the road at a point, where the
radius of curvature of the road is 20 m. The acceleration
m
due to gravity is 10 m/s 2 . In order to avoid sliding, he r
must bend with respect to the vertical plane by an angle
(a) θ = tan −1 (4 ) (b) θ = 45° (a) 16 ml (b) 4 ml
−1 −1 (c) 8 ml (d) 2 ml
(c) θ = tan (2) (d) θ = tan (6)

[ Level 2 ]
Only One Correct Option string of length l is executing periodic motion with
amplitude θ = 60° while other body is at rest on the
1. A heavy particle is tied to the end A of a string of length
surface.The minimum coefficient of friction between the
1.6 m. Its other end O is fixed. It revolves as a conical
mass 4m and the horizontal surface must be
pendulum with the string making 60° with the horizontal.
4m
Then,
(a) its period of revolution is 4 π / 7 s
2
(b) the tension in the string is times the weight of the
3 θ
particle
(c) the speed of the particle is 2.8 3 m /s
2
(d) the centripetal acceleration of the particle is 9.8/ 3 m/ s m
1 1
2. A bullet of mass m moving with a horizontal velocity u (a) (b)
4 3
strikes a stationary wooden block of mass M suspended 1 2
by a string of length L = 50 cm. The bullet emerges out of (c)
2
(d)
3
u
the block with speed . If M = 6m, the minimum value of 4. A mass is attached to the end of a string of length l which
4
u so that the block can complete the vertical circle is tied to a fixed point O. The mass is released from the
(Take g = 10 m /s 2 ) initial horizontal position of the string. Below the point O
at what minimum distance a peg P should be fixed so that
(a) 10 m/s
the mass turns about P and can describe a complete circle
(b) 20 m/s
(c) 30 m/s in the vertical plane?
(d) 40 m/s  3  2
(a)   l (b)   l
 5  5
3. Two bodies of mass m and 4m are attached to a spring as l 2l
shown in the figure. The body of mass m hanging from a (c) (d)
3 3
282 Objective Physics Vol. 1

5. The bob of a 0.2 m pendulum describes an arc of circle 12. A particle of mass 1 g executes an oscillatory motion on
in a vertical plane. If the tension in the cord is 3 times, the concave surface of a spherical dish of radius 2 m
the weight of the bob when the cord makes an angle 30° placed on a horizontal plane. If the motion of the particle
with the vertical, the acceleration of the bob in that begins from a point on the dish at a height of 1 cm from
position is the horizontal plane and the coefficient of friction is 0.01,
g 3g g the total distance covered by the particle before it comes
(a) g (b) (c) (d) to rest, is approximately
2 2 4
(a) 2.0 m (b) 10.0 m
6. An automobile enters a turn of radius R. If the road is (c) 1.0 m (d) 20.0 m
banked at an angle of 45° and the coefficient of friction is 13. A particle suspended by a light inextensible thread of
1, the minimum speed with which the automobile can
length l is projected horizontally from its lowest position
negotiate the turn without skidding is
with velocity 7gl/ 2. The string will slack after swinging
rg rg
(a) (b) through an angle equal to
2 2
(a) 30° (b) 90°
(c) rg (d) zero (c) 120° (d) 150°
7. A jeep runs around a curve of radius 0.3 km at a constant 14. A particle moves from rest at A on the surface of a
speed of 60 ms −1 . The jeep covers a curve of 60° arc smooth circular cylinder of radius r as shown in figure. At
(a) resultant change in velocity of jeep is 60 ms−1 B it leaves the cylinder. The equation relating α and β is
(b) instantaneous acceleration of jeep is 12 ms−2 A
(c) average acceleration of jeep is approximately 11.5 ms−2
(d) instantaneous and average acceleration are same in this case B
r α
8. A stone is tied to a string of length l and is whirled in a β
vertical circle with the other end of the string as the
centre. At a certain instant of time, the stone is at its (a) 3 sin α = 2 cos β (b) 2 sin α = 3 cos β
lowest position and has a speed u. The magnitude of the (c) 3 sin β = 2 cos α (d) 2 sin β = 3 cos α
change in velocity as it reaches a position, where the
string is horizontal ( g being acceleration due to gravity) 15. A ball suspended by a thread swings in a vertical plane so
is that its acceleration in the extreme position and lowest
position are equal. The angle θ of thread deflection in the
(a) 2(u2 − gl ) (b) u2 − gl
extreme position will be
(c) u − u2 − 2gl (d) 2gl (a) tan −1 (2)
9. A stone of mass 1 kg tied to a light inextensible string of (b) tan −1 ( 2 )

length L =
10  1
m, whirling in a circular path in a vertical (c) tan −1  
3  2
plane. The ratio of maximum tension to the minimum  1
(d) 2 tan −1  
tension in the string is 4. If g is taken to be 10 m/s 2 , the  2
speed of the stone at the highest point of the circle is
16. A body of mass m hangs at one end of a string of length l,
(a) 10 m/s (b) 5 2 m/s
the other end of which is fixed. It is given a horizontal
(c) 10 3 m/s (d) 20 m/s
velocity so that the string would just reach, where it
10. A wet open umbrella is held vertical and whirled about makes an angle of 60° with the vertical. The tension in
the handle at a uniform rate of 21 rev in 44 s. If the rim of the string at bottommost point position is
the umbrella is a circle of 1 m in diameter and the height (a) 2 mg (b) mg
of the rim above the floor is 4.9 m. The locus of the drop (c) 3 mg (d) 3 mg
on floor is a circle of radius
(a) 2.5 m (b) 1 m (c) 3 m (d) 1.5 m 17. A simple pendulum oscillates in a vertical plane. When it
passes through the bottommost point, the tension in the
11. A 50 kg girl is swinging on a swing from rest. Then, the string is 3 times the weight of the pendulum bob. What is
power delivered when moving with a velocity of 2 m/s the maximum displacement of the pendulum of the string
upwards in a direction making an angle 60° with the with respect to the vertical?
vertical is (a) 30° (b) 45°
(a) 980 W (b) 490 W (c) 490 3 W(d) 245 W (c) 60° (d) 90°
Circular Motion 283

 1  1
18. A boy whirls a stone in a horizontal circle of radius 1.5 m (a) 0°, cos−1   , 90° (b) 90°, cos−1   , 0°
and at height 2.0 m above level ground. The string breaks  3  3
 1 
and the stone flies off tangentially and strikes the ground (c) 0°, cos−1 3 , 90° (d) cos−1   , 90° , 0°
after travelling a horizontal distance of 10 m. What is the  3
magnitude of the centripetal acceleration of the stone
while in circular motion? More than One Correct Options
(a) 163 m/s 2 (b) 64 m/s 2 1. A ball tied to the end of the string swings in a vertical
(c) 15.63 m/s 2 (d) 125 m/s 2 circle under the influence of gravity.
19. A stone is rotated in a vertical circle. Speed at (a) When the string makes an angle 90° with the vertical, the
tangential acceleration is zero and radial acceleration is
bottommost point is 8gR, where R is the radius of circle. somewhere between minimum and maximum
The ratio of tension at the top and the bottom is (b) When the string makes an angle 90° with the vertical, the
tangential acceleration is maximum and radial acceleration
(a) 1 : 2 (b) 1 : 3
is somewhere between maximum and minimum
(c) 2 : 3 (d) 1 : 4
(c) At no place in circular motion, tangential acceleration is
20. The kinetic energy K of a particle moving along a circle equal to radial acceleration
of radius R depends on the distance covered s as K = as 2 . (d) When radial acceleration has its maximum value, the
tangential acceleration is zero
The force acting on the particle is

1/ 2 2. A small spherical ball is suspended through a string of
2as2 s2 
(a) (b) 2as 1 + 2  length l. The whole arrangement is placed in a vehicle
R  R 
1/ 2
which is moving with velocity v. Now, suddenly the
 s2  vehicle stops and ball starts moving along a circular path.
(c) as 1 + 2  (d) None of these
 R  If tension in the string at the highest point is twice the
weight of the ball, then (Assume that the ball completes
21. A conical pendulum of length L makes an angle θ with the the vertical circle)
vertical. The time period will be (a) v = 5 gl
(b) v = 7 gl
θ
(c) velocity of the ball at highest point is gl
(d) the velocity of the ball at the highest point is 3gl

3. A particle is describing circular motion in a horizontal


m plane in contact with the smooth surface of a fixed right
R
circular cone with its axis vertical and vertex down. The
L cos θ L height of the plane of motion above the vertex is h and
(a) 2π (b) 2π the semi-vertical angle of the cone is α. The period of
g g cos θ
revolution of the particle
L tan θ L
(c) 2π (d) 2π
γ g tan θ

22. A particle starts travelling on a circle with constant


tangential acceleration. The angle between velocity
vector and acceleration vector, at the moment when h
particle complete half the circular track, is α
(a) tan −1 (2π ) (b) tan −1 (π )
(a) increases as h increases
(c) tan −1 (3π ) (d) zero
(b) decreases as h decreases
23. A simple pendulum is vibrating with an (c) increases as α increases
α (d) decreases as α increases
angular amplitude of 90° as shown in the
figure. For what value of α, is the 4. In circular motion of a particle,
acceleration directed? (a) particle cannot have uniform motion
(i) Vertically upwards (b) particle cannot have uniformly accelerated motion
(ii) Horizontally (c) particle cannot have net force equal to zero
(iii) Vertically downwards (d) particle cannot have any force in tangential direction
284 Objective Physics Vol. 1

5. A smooth cone is rotated with an angular velocity ω as 3. Minimum velocity of the particle is
shown. A block A is placed at height h. A block has no gL gL gL gL
(a) 4 (b) 2 (c) (d) 3
motion relative to cone. Choose the correct options, when 3 3 3 3
ω is increased.
ω
4. The kinetic energy of particle at the lower most position
is
A
4 mgL 8 mgL 2 mgL
(a) (b) 2mgL (c) (d)
3 3 3

h 5. Velocity of particle when it is moving vertically


downward is
10 gL gL
(a) (b) 2
(a) Net force acting on block will increase 3 3
(b) Normal reaction acting on block will increase 8 gL 13 gL
(c) (d)
(c) h will increase 3 3
(d) Normal reaction will remain unchanged
Assertion and Reason
Comprehension Based Questions Directions (Q. Nos. 1-16) These questions consists of two
Passage I (Q. 1 to 2) statements each printed as Assertion and Reason. While
A ball with mass m is attached to the answering these questions you are required to choose any one
end of a rod of mass M and length l. of the following five responses.
The other end of the rod is pivoted so l
O (a) In both Assertion and Reason are correct and Reason
that the ball can move in a vertical is the correct explanation of Assertion.
circle. The rod is held in the (b) It both Assertion and Reason are correct but Reason is
horizontal position as shown in the not the correct explanation of Assertion.
figure and then given just enough a (c) If Assertion is true but Reason is false.
downward push so that the ball swings down and just (d) If Assertion is false but Reason is true.
reaches the vertical upward position having zero speed (e) If both Assertion and Reason are false.
there. Now answer the following questions
1. Assertion One end of a massless rod of length l is hinged
1. The change in potential energy of the system (ball + rod) so that it is free to rotate in a vertical plane about a
is horizontal axis. If a particle is attached to the other end of
the rod, then the minimum speed at lower most position
(a) mgl (b) (M + m) gl
of the particle is 5gl to complete the circular motion.
M  (M + m)
(c)  + m gl (d) gl Reason Work done by centripetal force on the particle is
 2  2
always zero.
2. The initial speed given to the ball is
2. Assertion When water in a bucket is whirled fast
Mgl + 2mgl overhead, the water does not fall out at the top of the
(a) (b) 2gl
m circular path.
2Mgl + mgl
(c) (d) None of these Reason The centripetal force in this position on water is
m
more than the weight of water.
/ Attempt the above question after studying chapter of
rotational motion. 3. Assertion A ball tied by thread is undergoing circular
motion (of radius R) in a vertical plane. (Thread always
Passage II (Q. 3 to 5)
remains in vertical plane). The difference of maximum
A small particle of mass m attached and minimum tension in thread is independent of speed
with a light inextensible thread of ( u ) of ball at the lowest position ( u > 5gR ).
length L is moving in a vertical
circle. In the given case particle is Reason For a ball of mass m tied by thread undergoing
moving in complete vertical circle vertical circular motion (of radius R), difference in
L
and ratio of its maximum to maximum and minimum magnitude of centripetal
minimum velocity is 2 : 1. acceleration of the ball is independent of speed ( u ) of ball
m
at the lowest position ( u > 5gR ).
Circular Motion 285

4. Assertion A car moves along a road with uniform speed. 12. Assertion If a particle is rotating in a circle, then
The path of car lies in vertical plane and is shown in centrifugal force is acting on the particle in radially
figure. The radius of curvature ( R ) of the path is same outward direction.
everywhere. If the car does not loose contact with road at
Reason Centrifugal force is equal and opposite to the
the highest point, it can travel the shown path without
centripetal force.
loosing contact with road anywhere else.
Car 13. Assertion Angle (θ ) between a and v in circular motion
is
h 0° < θ < 180°
Reason Angle between any two vectors lies in the above
range.
Reason For car to loose contact with road, the normal
reaction between car and road should be zero. 14. Assertion A small block of mass m
is rotating in a circle inside a
5. Assertion Uniform circular motion is uniformly smooth cone as shown in figure. In
accelerated motion. this case the normal reaction,
Reason Acceleration in uniform circular motion is N ≠ mg cos θ
always towards centre. θ
Reason In this case acceleration of
6. Assertion In vertical circular motion speed of a body
the block is not along the surface of cone. It is horizontal.
cannot remain constant.
Reason In moving upwards work done by gravity is 15. Assertion When a car takes a circular turn on a
negative. horizontal road, then normal reaction on inner wheels is
7. Assertion In circular motion acceleration of particle is always less than the normal reaction on outer wheels.
not always towards centre. Reason This is for rotational equilibrium of car.
Reason If speed of particle is not constant acceleration
16. Assertion On every satellite of the earth we feel
is not towards centre.
weightlessness.
8. Assertion In circular motion average speed and average
velocity are never equal. Reason The gravitational force acting on it by the earth
is completely utilized in providing the necessary
Reason In any curvilinear path these two are never
equal. centripetal force.

9. Assertion In circular motion dot product of v and ω is Match the Columns


always zero. 1. Three balls each of mass 1 kg are attached with three
Reason ω is always perpendicular to the plane of the strings each of length 1 m as shown in figure. They are
circular motion. rotated in a horizontal circle with angular velocity
mv 2 ω = 4 rad/s about point O. Match the following columns.
10. Assertion Centripetal force acts on a particle ω
R O
rotating in a circle. T1 T2 T3
Reason Summation of net forces acting on the particle is
mv 2 Column I Column II
equal to in the above case.
R (A) T1 (p) Maximum

11. Assertion A particle is rotating in a circle of radius 1 m. (B) T2 (q) Minimum


At some given instant its speed is 2 m/s. Then (C) T3 (r) 80 N
acceleration of particle at the given instant is 4 m/s 2 . (s) 48 N
Reason Centripetal acceleration at this instant is 4 m/s 2 (t) 90 N
towards centre of circle.
286 Objective Physics Vol. 1

2. A particle is suspended from a string of length R. It is Entrance Gallery


given a velocity u = 3 gR at the bottom.
Match the following columns. 2014
C 1. A bob of mass m attached to an inextensible string of
length l is suspended from a vertical support. The bob
rotates in a horizontal circle with an angular speed ω
D B rad/s about the vertical support. About the point of
suspension, [JEE Main]
u (a) angular momentum is conserved
A (b) angular momentum changes in magnitude but not in
direction
Column I Column II
(c) angular momentum changes in direction but not in
(A) Velocity at B (p) 7 mg magnitude
(B) Velocity at C (q) 5gR (d) angular momentum changes both in direction and
(C) Tension in string at B (r) 7 gR magnitudes
(D) Tension in string at C (s) 5 mg
2. A particle is moving uniformly in a circular path of radius
(t) None
r. When it moves through an angular displacement θ, then
3. In the system shown in m the magnitude of the corresponding linear displacement
A will be [WB JEE]
figure, mass m is released R
from rest from position A.  θ  θ
2m (a) 2 r cos   (b) 2 r cot  
Suppose potential energy of B  2  2
m at point A with respect to  θ  θ
(c) 2 r tan   (d) 2 r sin  
point B is E. Dimensions of  2  2
m are negligible and all surfaces are smooth. When mass m
reaches at point B. 2013
3. The work done on a particle of mass m by a force,
Column I Column II
(A) Kinetic energy of m (p) E/3
 x $i + y $j
K 2 
 ( x + y ) ( x + y 2 ) 3/ 2 
2E 2 3/ 2 2
(B) Kinetic energy of 2 m (q)
3
(C) Momentum of m (r) 4 (K being a constant of appropriate dimension), when the
mE
3 particle is taken from the point ( a, 0) to the point ( 0, a )
(D) Momentum of 2 m (s) 2 along a circular path of radius a about the origin in the x- y
mE
3 plane is [JEE Main]
(t) None 2Kπ Kπ
(a) (b)
a a
4. A pendulum is released from point A as shown in figure. Kπ
(c) (d) zero
At some instant net force on the bob is making an angle θ 2a
with the string. Then, match the following columns.
4. A hoop of radius r and mass m rotating with an angular
velocity ω 0 is placed on a rough horizontal surface. The
initial velocity of the centre of the hoop is zero. What will
be the velocity of the centre of the hoop when it ceases to
slip? [JEE Main]
C A rω 0 rω 0 rω 0
(a) (b) (c) (d) rω 0
4 3 2
B

Column I Column II 2012


(A) For θ = 30° (p) Particle may be moving 5. Two cars of masses m1 and m2 are moving in circles of
along BA
radii r1 and r2 respectively. Their speeds are such that
(B) For θ = 120° (q) Particle may be moving
along CB they make complete circles in the same time t. The ratio
(C) For θ = 90° (r) Particle is at A of their centripetal accelerations is [AIEEE]
(D) For θ = 0° (s) Particle is at B (a) m1r1 : m2r2 (b) m1 : m2
(t) None (c) r1 : r2 (d) 1 : 1
Circular Motion 287

2011 metre and t is in second. The radius of the path is 20 m.


The acceleration of P, when t = 2 s is nearly [AIEEE]
6. A ball of mass 0.5 kg is attached to the end of a string
having length 0.5 m as shown in figure. The ball is
rotated on a horizontal circular path about vertical axis. B
The maximum tension that the string can bear is 324 N.
P(x, y)
The maximum possible value of angular velocity of ball
(in rad/s) is [IIT JEE] m
20

O A
−2
L (a) 13 ms (b) 12 ms −2
(c) 7.2 ms −2 (d) 14 ms −2

11. For a particle in uniform circular motion, the acceleration


m a at a point P ( R , θ ) on the circle of radius R is (here θ, is
measured from the x-axis) [AIEEE]
(a) 9 (b) 18 (c) 27 (d) 36 y

7. A bridge is in the form of a semicircle of radius 40 m. The


greatest speed with which a motor cycle can cross the P (R, θ)
bridge without leaving the ground at the highest point is θ
(take, g = 10 ms −2 ) and (frictional force is negligibly O x
small) [Kerala CEE]
(a) 40 ms−1 (b) 20 ms−1 (c) 30 ms−1 (d) 15 ms −1
(e) 25 ms −1
v2 v2 v2 v2
8. A particle of mass m is moving in a horizontal circle of (a) − cos θ $i + sin θ $j (b) − sin θ $i + cosθ $j
k R R R R
radius r, under a centripetal force F = 2 , where k is a v2 v2 v2 v2 $
r (c) − cosθ $i − sin θ $j (d) $i + j
R R R R
constant. [Kerala CEE]
(a) The potential energy of the particle is zero 12. If α is angular acceleration, ω is angular velocity and a is
k the centripetal acceleration, then which of the following
(b) The potential energy of the particle is
r is true? [MHT CET]
k
(c) The total energy of the particle is − ωa v
2r (a) α = (b) α =
v ωa
k
(d) The kinetic energy of the particle is − va a
r (c) α = (d) α =
ω ωv
k
(e) The potential energy of the particle is −
2r 13. If KE of the particle of mass m performing UCM in a
9. A particle is moving with a constant speed v in a circle. circle of radius r is E. Find the acceleration of the particle
What is the magnitude of average after half rotation? [MHT CET]
2
[WB JEE] 2E  2E 
v (a) (b)  
(a) 2v (b) 2 mr  mr 
π 4E
v v (c) 2Emr (d)
(c) (d) mr
2 2π
14. A particle of mass m is circulating on a circle of radius r
2010 having angular momentum L, then the centripetal force
10. A point P moves in counter-clockwise direction on a will be [MHT CET]
circular path as shown in the figure. The movement of P (a) L2 / mr (b) L2m / r
is such that it sweeps out a length s = t 3 + 5, where s is in (c) L2 / mr3 (d) L2 / mr2
Answers
Level 1
Objective Problems
1. (a) 2. (c) 3. (a) 4. (b) 5. (a) 6. (c) 7. (c) 8. (c) 9. (d) 10. (a)
11. (d) 12. (c) 13. (a) 14. (c) 15. (d) 16. (d) 17. (c) 18. (d) 19. (b) 20. (d)
21. (a) 22. (a) 23. (d) 24. (d) 25. (a) 26. (d) 27. (d) 28. (d) 29. (d) 30. (d)
31. (b) 32. (c) 33. (c) 34. (d) 35. (d) 36. (c) 37. (c) 38. (a) 39. (d) 40. (c)
41. (a) 42. (a) 43. (a) 44. (c) 45. (a) 46. (c) 47. (a) 48. (c) 49. (a) 50. (b)
51. (c) 52. (a) 53. (c) 54. (c) 55. (a) 56. (a) 57. (d) 58. (d) 59. (c) 60. (b)
61. (a) 62. (b) 63. (c) 64. (b) 65. (b) 66. (b) 67. (d) 68. (c) 69. (b) 70. (c)
71. (a) 72. (a)

Level 2
Only One Correct Option
1. (b,d) 2. (d) 3. (c) 4. (a) 5. (a) 6. (d) 7. (a,b,c) 8. (a) 9. (a) 10. (a)
11. (c) 12. (c) 13. (c) 14. (c) 15. (d) 16. (a) 17. (d) 18. (a) 19. (b) 20. (b)
21. (b) 22. (a) 23. (a)

More than One Correct Options


1. (b,d) 2. (b,d) 3. (a,d) 4. (a,b,d) 5. (a,b)

Comprehension Based Questions


1. (c) 2. (d) 3. (b) 4. (c) 5. (a)

Assertion and Reason


1. (d) 2. (a) 3. (a) 4. (d) 5. (d) 6. (d) 7. (a) 8. (a) 9. (a) 10. (e)
11. (d) 12. (d) 13. (c) 14. (a) 15. (a) 16. (d)

Match the Columns


1. (A → p, B → r, C → qs) 2. (A → r, B → q, C → p, D → t) 3. (A → q, B → p, C → r, D → r)
4. (A → qp, B → t, C → r, D → s)

Entrance Gallery
1. (c) 2. (d) 3. (d) 4. (c) 5. (c) 6. (d) 7. (b) 8. (a) 9. (b) 10. (d)
11. (c) 12. (a) 13. (a) 14. (c)
Solutions
Level 1 : Objective Problems At t =
2a
,ω = − a
b
1. a = at2 + an2
an = Rω2 = Ra 2 …(ii)
at = rate of change of speed = 2 m/s 2
Now, a= at2 + an2
v 2 ( 5)2
an = = = 2.5 m/s 2
R 10 = R a + b2 4

∴ a = at2 + an2 = ( 2)2 + ( 2.5)2 = 3.2 m/s2 17. α = constant


1
2. At highest point of a projectile, tangential component of ∴ θ= ∝t2
acceleration is zero as acceleration is vertically downwards 2
( g ) and velocity is horizontal or the angle between ∴ θ ∝t 2
acceleration and velocity is 90°. 2
θ2 + θ1  2 + 2 
or =  =4
3. an = a − 2
at2 = (100) − (60) = 80 m/s
2 2 2
θ1  2 
v2 v 2 (10)2 θ2
Now an = or R = = = 1.25 m or =3
R an 80 θ1
v 2 (6)2 18. Necessary centripetal force to the coin is provided by
4. at = 0, a = an = = = 120 m/s2
R 0.3 friction. Thus,
µg
5. Velocity is tangential and acceleration is radial. mr ω2max = µmg or r = 2
ωmax
6. In circular motion, ar can never be zero.
ωmax is made three times. Therefore, distance from centre r
7. Speed of particle is increasing, means tangential 1
will remain times.
component of acceleration is positive. 9
2π 2π mv 2 (1.5m) (1.5v )2 mv 2
8. ωmin = rad/min and ωhr = rad/min 19. F = , F′ = = 2.25 = 2.25 F
60 12 × 60 r (1.5r ) r
ωmin 2 π / 60
∴ = Therefore, F has to be increased by 125%.
ωhr 2 π /12 × 60 20. ω = 2 πf = 2 π (0.5) = π rad/s

9. 120 rev/min = 120 × rad/s = 4π rad/s ω
60 N
θ
v2 2π
10. Acceleration = ω2r = = ωv = v
r T
mg
v2 d
11. a = or a ∝v 2 θ
R R
12. a = r ω2 or a ∝r
N sin θ = mR ω2
∆ω
13. α = but ω = 2 πf N cosθ = mg
∆t
Rω2
2 π∆f 2 π × 200 or tan θ =
∴ α= = g
∆t 2
= ( 200π ) rad/s 2 g tan θ
∴ R=
ω2
2
v2  v2 10 ⋅ tan 30° 1
14. a = an2 + at2 =   + a 2 = 2 + a 2 = ≈ m
 r  r ( π2 ) 3

an v 2 / R R 1/ 3
15. = (let ar = a ) Now, d= =
at a cos θ cos θ

Here, vb 2 = 2al = 2a ( 2 πR ) = 4 πR. 1/ 3 2


= = = 0.67 m
4π 3/ 2 3
Therefore, the ratio is .
l mv 2 µ mv 2
21. N = , mg = µ s N = s
dω R R
16. α = =−b
dt Rg
∴ v=
at = Rα = − Rb …(i) µs
290 Objective Physics Vol. 1

22. To find tension in the ring, let us take an arc which subtends T2 − T3 = m ω2 2l
angle 2( d θ) at centre. Tangential components of T cancel ⇒ T2 = mω2 ( 5l )
out each other, while inward components provide the
necessary centripetal force. Thus, T1 − T2 = mω2l
dθ dθ ⇒ T1 = mω2 (6l )
T T T3 : T2 : T1 = 3 : 5 : 6
⇒ mv 2
ω dθ dθ 31. Kx =
r
Kxr
v=
m
2T sin ( d θ) = ( dm) R ω2 2 πr mr
T= = 2π
= ( λ 2R ⋅ d θ)( Rω2 ) v Kx
Here, λ → linear mass of density. ∴ T∝
r
For small angles, sin d θ ≈ d θ x
∴ 2Td θ = 2λR 2ω2 dθ r′x 3a × a 3
T′=T =T = T
or T = λR 2ω2 rx ′ 2a × 2a 2

Tmax 1 32. v max = gR = 9.8 × 49 = 22 m/s


ωmax = ⋅
λ R
v
40 1
= ×
0.1 1
= 20 rad/s
mv 2 µmv 2
23. N = , mg = µN = Mv 2
N
R R 33. Mg − N = v
Rg 5 × 10 r
∴ µ= 2 = = 0.4
v ( 5 5 )2 Mv 2
or N = Mg −
r Mg
24. Centripetal force = mRω2 = ( 2)(1)( 2 π )2 = 8 π 2 N 2
mv min
mv 2 1 34. mg = or v min = Rg (at topmost point)
25. F = . If m and v are constants, then F ∝ R
r r
35. Kinetic energy given to a sphere at lowest point= potential
F1  r2  energy at the height of suspension
∴ = 
F2  r1 
PE = mgl
26. Maximum tension = m ω2 r = m × 4 π 2 × n2 × r
By substituting the values, we get Tmax = 87.64 N
l
2
mv 2 m  p  p2
27. Radial force = =   =
r 
r m  mr
28. Tension in the string T = mω2r = 4 π 2n2mr 1
KE = mv2
2
∴ T ∝ n2
n2 T 1
⇒ = 2 ⇒ mv 2 = mgl
n1 T1 2
∴ v = 2gl
2T
⇒ n2 = 5 = 5 2 rpm
l
T 36. h = l (1 − cos 60° ) = =1 m
2 2
mv
29. Maximum tension = = 16 N l
r
16 × v 2
60

⇒ = 16
°

144
⇒ v =12 m/s
30. Let ω is the angular speed of revolution.
T1 T2 T3
O h
A B C
v
l l l
v = 2gh = 2 × 9.8 × 1 = 4.43 m/s
T3 = m ω3 ( 3l )
Circular Motion 291

mv 2 46. When released from top with zero velocity block leaves
37. Tension, T = + mg cosθ contact at
r
For, θ = 30° ,
mv12 θ r h
T1 = + mg cos 30°
r
mv 22
θ = 60°, T2 = + mg cos 60° cosθ =
2
r 3
∴ T1 > T2 as v1 > v 2 2r
∴ h = r cosθ =
and cos 30° > cos 60° 3
38. Difference in kinetic energy 47. 2gh = 5gR
= 2mgr 2h 2 × 5
∴ R= = = 2 cm
= 2 × 1 × 10 × 1 5 5
= 20J mv 2
48. T − mg = or T = 3 mg
39. Otherwise the particle will fall down. l
40. v 2 = 2gh = 2gR
h=l
v 2 = 2gh
h

h=R T
v

v mg
2 Tension will be maximum at bottommost point.
mv
T − mg =
R 49. In critical case tension at topmost point is zero.
Substituting value of v 2 , we get 50. v = 2gh = 2gR
T = 3 mg
41. Minimum velocity at topmost point is Rg .
h=R
R
42. h = R (1 − cos 60° ) =
2 v

60° R v 51. v = u − 2gh = 5gr − 2gr = 3gr


2

h
u

h=r
v = u2 − 2gh
u
43. h = l (1 − cos θ)
vm2 = 2gh 52. Maximum tension will be at bottommost point.
= 2gl (1 − cos θ) 54. | v AB | = | v A − v B | = ( Rω)2 + ( Rω)2 − 2( Rω)( Rω)cos θ

1
K m = mvm2 θ
= 2R ω sin
2 2
= mgl (1 − cos θ) 5
55. v = 72 km/h = 72 × m/s = 20 m/s
44. h′ = R − h 18
∴ v min = 2gh′  v2 
θ = tan −1  
 Rg 
v h'  400   2
= tan −1   = tan
−1
 
 100 × 10   5
h
65. As the speed is constant throughout the circular motion
therefore, its average speed is equal to instantaneous
45. v = 2gh = 2gh (hmax − hmin ) speed.
= 2 × 10 × 1.25 Total displacement 2 m
66. Average velocity = = = 2 ms−1
= 5 m/s Total time 1s
292 Objective Physics Vol. 1

( 2R + πR / 2) km v2
67. Average speed = ∴ =
T
=
g
=
9.8
m/s 2
(1 / 6) h R 2m 3 3
= (12R + 3 πR ) km/h 9.8
km v=
× 0.8 m/s
= (12 + 3π ) (as R =1 km) 3
h R
= 21.4 km/h Time period = 2π
v
68. L = mvr
2. From momentum conservation,
L
∴ v= u
mu = m + (6 m) 5 × 10 × 0.5
mr 4
mv 2 L2 Solving we get, u = 40 m/s
F= =
r mr 3 l
3. h = l (1 − cos 60° ) = , v 2 = 2gh = gl
d B v Bt v B 2.5 5 2
69. = = = =
dC vC t vC 2 4 mv 2
Now, Tmax − mg = (at bottommost point)
5 l
70. v = 72 km/h = 72 × m/s = 20 m /s
18 ∴ Tmax = 2mg = µ s ( 4 mg )
v2 ∴ µ s = 0.5
tan θ =
Rg 4. v 2 = 2gl = 5gR
5
71. v = 48 × m /s = 13.33 O
18 x

R
h
v
θ ∴ R = 0.4 l or x = l − R = 0.6 l
1m
2
T − mg cos 30° 
v2 5. a = an2 + at2 =   + ( g sin 30° )
2
tan θ = = 0.044  m 
Rg
2
h  3 1
0.044 = = g 3 −  +   =g
1  2   4
∴ h = 0.044 m = 4.4 cm
N µN mv min
2

72. T sin θ = mr ω 2 6. − =
2 2 R
or T sin θ = m (l sin θ) ω2 N
µN
∴ T = mlω = ml ( 2 πf )
2 2

2
= ml  2 π ×  =16ml
2
 π amin

Level 2 : Only One Correct Option 45°


1. R = 1.6 cos 60° = 0.8 m mg

O ⇒ v min = 0, as µ =1
Distance travelled
7. ∆t =
Speed
1.6m
( 2 πR / 6) 3.14 × 300
= = = 5.23 s
v 60 × 3
60°
A (a) | ∆v | = | v f | − | v i |
= v 2 + v 2 − 2v ⋅ v cos 60°
T sin 60° = mg = 2v sin 30° = 60 m/s
2mg v2
or T= (b) ai = = 12 m/s2
3 R
mv 2 | ∆v | 60
T cos60° = (c) | a av | = = = 11.5 m/s2
R ∆t 5.23
Circular Motion 293

8. h = l 12. Since, the particle is released from a small height, θ ( angle


v of radius with vertical) will be very small.
Force of friction throughout the journey can be assumed to
be µmg. Particle will finally come to rest when whole of its
energy ( = mgh) is lost in the work done against friction. Let
particle stops after travelling a distance d. Then,
l
µmgd = mgh
h 10−2
or d= = = 1.0 m
u µ 0.01
v = u − 2gh = u2 − 2gl
2 13. h = l + l sin θ = l (1 + sin θ) v

| ∆v | = | v f − v i | v 2 = u2 − 2gh
θ
= subtraction of vectors at 90°
= u2 − 2gl (1 + sin θ)
h
= v + u − 2vu ⋅ cos 90°
2 2
String will slack where, component of
= v 2 + u2 = (u2 − 2gl ) + u2 weight towards centre is just equal to
centripetal force or
u
= 2 (u2 − gl ) mv 2 m 2
mg sin θ = = [u − 2gl (1 + sin θ)]
9. Minimum tension is at topmost point (speed = v) and l l
maximum tension at bottommost point (speed = u). 7gl
Substituting u =
2
2
mu2
mg + 1
Tmax
= L =4 we get, sin θ = or θ = 30°
Tmin mv 2 2
− mg + ∴The desired angle is 90° + 30° or 120°.
L
u = v + 2g ( 2L )
2 2 14. hAB = (r cos α − r sin β )

Solving we get, v =10 m/s Velocity of particle at B


d 21 × 2 π × 0.5 v = 2ghAB = 2g (r cos α − r sin β )
10. v = = = 1.5 m/s
t1 44 Particle will leave contact at B, if component of weight is just
2h 2 × 4.9 equal to centripetal force (towards centre).
t2 = = =1s
g 9.8 mv 2
or mg sin β =
r
or sin β = 2 cos α − 2 sin β
∴ 3 sin β = 2 cos α
0.5 m r v 2 2gh 2gR (1 − cos θ)
15. g sin θ = = =
R R R
1.5 m or sin θ = 2 (1 − cos θ)
θ θ θ
Horizontal distance travelled by drop = vt2 = 1.5 m 2 sin cos = 2  2 sin 2 
2 2  2
∴ r = (1.5)2 + (0.5)2 = 2.5 m
θ 1
11. Two forces are acting on bob, tension and weight. Power of ∴ tan =
2 2
tension will be zero and that of weight is,
θ
= tan −1  
1
2  2
θ
θ = 2 tan −1  
T 1
or
v = 2 m/s  2
θ = 60°
/ In extreme position of pendulum only tangential component
θ of acceleration (at = g sin θ) is present. In lowest position, only
normal acceleration (an = v 2 /R ) is present.
mg 16. When body is released from the position p (inclined at angle
P = mgv cos( 90° + θ) θ from vertical) then velocity at mean position
= − mgv sin 60° v = 2gl (1 − cos θ)
3 mv 2
= − 50 × 9.8 × 2 × ∴Tension at the lowest point = mg +
2 I
= − 490 3 W m
= mg + [2gl (1 − cos 60° )]
l
∴ Power delivered is 490 3 W. = mg + mg = 2 mg
294 Objective Physics Vol. 1

17. Tension at mean position, an


tan θ = = 2π
mv 2 at
mg + = 3 mg
r ∴ θ = tan −1 ( 2 π )
v = 2gl …(i) an
anet
and if the body displace by angle θ with the vertical, then
v = 2gl (1 − cos θ) …(ii)
Comparing Eqs. (i) and (ii),
cosθ = 0
⇒ θ = 90°
2l 2 × 2.0 θ at
18. t = = = 0.64 s
g 9.8 23. When a is horizontal
v
α
at = g
v2
an =
2.0 m R

At α = 90° acceleration
is downwards At α = 0° acceleration
10m is upwards
10 m
v= = 15.625
t s α
v2 an
a= = 163 m /s2
R
19. Ttop = 3mg and Tbottom = 9 mg
1 α
20. mv 2 = as 2
2 Horizontal
mv 2 2as 2 at
or Fn = = …(i)
R R an v 2 /l 2gh / l
2a tan α = = =
Further, v= ⋅s at g sin α g sin α
m 2g l cos α / l
=
or at =
dv
=
2a ds
⋅ =
2a
⋅v g sin α
dt m dt m = 2cot α
2a 2a 2as 1
= ⋅ ⋅s = tan α = 2 or cosα =
m m m 3
α = cos−1 
∴ Ft = mat = 2as 1 
…(ii) ∴ 
 3
∴ Fnet = Fn2 + Ft 2

s2 More than One Correct Options


= 2as 1 +
R2 1. Radial acceleration is given by,
C
21. T cosθ = mg , T sin θ = mRω 2

Rω2 R ( 2 π / T )2
or tan θ = =
g g
D B
R
∴ T = 2π θ
g tan θ
P
But R = L sin θ
L A
∴ T = 2π v2
g cos θ ar =
R
22. v = 2at s = 2 at ( πR )
At A, speed is maximum.
v2 Therefore, ar is maximum. At C, speed is minimum.
∴ an = = 2π at
R Therefore, ar is minimum. Tangential acceleration is g sin θ.
an At point B,θ = 90°.
or = 2π
at Therefore, tangential acceleration is maximum ( = g ).
Circular Motion 295

mu2 On solving these two equations, we get


2. T + mg =
l
u g
ω= tan θ
R
mg T h
But R=
h = 2l tan θ
L
g
∴ ω= tan θ
h
v
1
mu2 or ω∝ (θ = constant)
∴ 2 mg + mg = or u = 3gl h
l
v 2 = u2 + 2gh = 3 gl + 2g ( 2l ) Comprehension Based Questions
= 7 gl 1. ∆U = ∆Urod + ∆Uball
∴ v = 7gl
= Mg   + mgl
l
3. R = h cot α  2
N cosα = mg ...(i)
=  + m gl
M
mv 2  2 
N sin α = ...(ii)
R v
2. ω =
Solving these two equations, we get l
v = Rg tan α Now decrease in rotational kinetic energy = increase in
= (h cot α )( g tan α ) potential energy

I ω2 =  + m g l
= gh 1 M

2  2 
2 πR 2 π h cot α
Now, T= =
1  Ml 2  v 2
+ ml 2    =  + m g l
v gh M
or
2  3 l   2 
N
α  M + m gl
 
R  2 
∴ v=
α  M + m
 
 6 2
h mg
3. Maximum velocity is at bottommost point and minimum
α velocity is at topmost point.
2
umin + 2g ( 2L ) 2
h =
= 2π cot α umin 1
g
On solving, we get
∴ T ∝ h and T ∝cot α
gL
4. No solution is required. umin = 2
3
5. N cosθ = mg ...(i)
gL
N sin θ = mRω2 ...(ii) 4. umax = 2 umin = 4
3
ω 1
∴ K max =
2
N
8 mgL
θ
2
mumax =
3
R
θ 5. v = u max
2
− 2g ( L )

h mg

θ
h=L
Net force is the resultant of N and mg.
v
From Eq. (ii), we can see that N will increase with increase umax
in the value of ω.
16 gL 10 gL
If N will increase then net force will also increase. = − 2gL =
3 3
296 Objective Physics Vol. 1

Again, vC2 = v A2 − 2ghAC


Assertion and Reason
1. In case of massless rod, v min = 4gl . = ( 9gR ) − 2 ( 2R ) = 5gR
∴ vC = 5gR
3. Let the minimum and maximum tensions be Tmax and Tmin
mvC2
and the minimum and maximum speed be u and v. Further, TC + mg =
v R
∴ TC = 4 mg
3. Apply conservation of linear momentum in horizontal
Tmin direction and conservation of mechanical energy.
Tmax 4. Angle between net force and the string can never be obtuse.
It is 90° at A, 0° at B and acute in between.

u
Entrance Gallery
1. Angular momentum of the pendulum about the suspension
mu2
∴ Tmax = + mg point O is
R O O O
mv 2
Tmin = − mg
R
 u2 v 2 
∴ ∆T = m  −  + 2mg
R R l

u2 v 2
From conservation of energy, − = 4g L L
R R
m vrad Lrad
If is independent of u and ∆T = 6 mg.
∴Reason is correct explanation of assertion. vtan v Ltan
4. The normal reaction is not least at topmost point, hence
assertion is false. Then, v can be resolved into two components, radial
component v rad and tangential component v tan . Due to v rad ,
5. Direction of acceleration continuously changes. L will be tangential and due to v tan , L will be radially
9. v is always perpendicular to ω. outwards as shown above.
mv 2 So, net angular momentum will be as shown in figure,
10. does not act. But this much force is required. whose magnitude will be constant (| L | = mvl ). But its
R
direction will change as shown in the figure.
v 2 ( 2)2 L = m( r × v)
11. ac = = = 4 m/s 2
R 1
where, r = position of particle from point O.
But no information is given for tangential acceleration at . 2. Suppose a particle is moving in a circular path of radius r as
12. Centripetal force acts when the particle is observed from a shown in the figure.
rotating non-inertial frame. 2r sin (θ/2)
13. Angle between any two vectors lies in the following range, A B C
0° ≤ θ ≤ 180°.
14. In vertical direction, r θ/2
N cosθ = mg θ
O
In horizontal direction
mv 2
N sin θ =
R

Match the Columns θ


In ∆AOB ,sin   =
AB
(QAO = r )
1. T3 = (1)( 3)( 4) = 48 N ( F = mRω )
2 2  2  AO
θ θ
T2 − T3 = (1)( 2)( 4)2 AB = AO sin   ⇒ AB = r sin  
 2  2
∴ T2 = 80 N
θ θ
T1 − T2 = (1)(1)( 4)2 AC = AB + BC = r sin   + r sin   (QAB = BC )
 2  2
∴ T1 = 96 N θ
AC = 2r sin  
2. v B2 = uA2 − 2ghAB = ( 9gR ) − ( 2gR ) = 7gR  2
∴ v B = 7gR So, the magnitude of the corresponding linear
θ
mv B2 displacement will be 2 r sin   .
Further, TB + = 7 mg  2
R
Circular Motion 297

3. A circle with radius r as shown below, such that the position 7. Given, r = 40 m and g =10 m/s 2
of point P from centre O, i.e. OP = r is given as,
We have, v = gr
P
= 10 × 40 = 400
r
y = 20 m/s
O x 8. For horizontal planes potential energy remains constant
equal to zero, if we assumes surface to be the zero level.
2πr
9. Time, T = v
v
r = OP = x$i + y$j
T πr
F= 2
k k
( xi$ + y$j) = 3 ( r ) and t0 = =
2 v t = t0 t=0
( x + y 2 )3/ 2 r
2r 2v
Since, F is along r or in radial direction. Therefore, work ∴ v av = =
π r /v π
done is zero. v
4. 10. Given, s = t3 + 5
ds
∴ Speed, v = = 3t 2
v dt
dv
ω0 ω and rate of change of speed, at = = 6t
dt
ω = v/r
∴ Tangential acceleration at t = 2s
From conservation of angular momentum about bottommost at = 6 × 2 = 12 ms−2
point.
v ωr and at t = 2 s, v = 3( 2)2 = 12 ms−1
mr 2ω0 = mvr + mr 2 × ⇒ v= 0
r 2 v 2 144
∴ Centripetal acceleration, ac = = ms−2
v2 R 20
5. As, centripetal acceleration is given as, ac =
r ∴ Net acceleration = at2 + ac ≈ 14 ms−2
v12 11. For a particle in uniform y er
For first body of mass m1 , ac1 = et
r1 circular motion,
v 22 where, e$ r is radial unit vector θ
For second body of mass m2 , ac2 =
r2 and e$ t is tangential unit
Also time to complete one revolution by both body is same. vector. As, e$ r = $i cosθ + $jsin θ
O x
Hence, v2
a= towards centre of
2 πr1 2 πr2 v r R
= ⇒ 1 = 1 …(i)
v1 v2 v 2 r2 circle, i.e. ( −e$ r )
v12 r2 v2
i.e. ac1 : ac2 = × [From Eq. (i)] a= ( − cosθ $i − sin θ $j)
r1 v 22 R
v2 v2
r2 r r or a = − cosθ $i − sin θ $j
= 12 × 2 = 1 = r1 : r2 R R
r2 r1 r2
v
12. Centripetal acceleration, a =
6. T cosθ component will cancel mg. t
T sinθ component will provide necessary centripetal force ω ωv ωa
Angular acceleration, α = = =
to the ball towards centre C. t vt v
1 1 v2
13. Kinetic energy, E = mv 2
or mr =E
2 2 r
θ 1
or mra = E
T cosθ l 2
2E
or a=
θ T mr
T sinθ 14. Angular momentum L = r × p = r × m × v
C
L
mg r v= …(i)
mr
∴ T sinθ = mrω2 = m (l sin θ) ω2 mv 2
Now, as centripetal force, Fc = …(ii)
T r
or T = mlω 2
⇒ ω= rad/s
ml Substituting the value of v from Eq. (i) in Eq. (ii), we get
2
Tmax 324 m L  L2
or ωmax = = = 36 rad/s Fc = =
ml 0.5 × 0.5  
r  mr  mr 3
8
COM,Conservation
of Linear Momentum,
Impulse and Collision
8.1 Centre of Mass
When we consider the motion of a system of particles, there is one point in it which Chapter Snapshot
behaves as though the entire mass of the system (i.e. the sum of the masses of all the ● Centre of Mass
individual particles) is concentrated there and its motion is the same as would ensue if the ● Law of Conservation of
resultant of all the forces acting on all the particles were applied directly to it. This point is Linear Momentum
called the centre of mass (CM) of the system. The concept of CM is very useful in solving ● Variable Mass
many problems, in particular, those concerned with collision of particles. ● Impulse
Collision
Position of Centre of Mass ●

First of all, we find the position of CM of a system of particles. Just to make the subject
easy we classify a system of particles in three groups:
1. System of two particles,
2. System of a large number of particles,
3. Continuous bodies.
Now, let us take them separately.

Position of CM of Two Particles


Centre of mass of two particles of mass m1 and m2 separated by a distance of d lies in
between the two particles. The distance of centre of mass from any of the particle ( r ) is
inversely proportional to the mass of the particle ( m).
CM

m1 m2
r1 r2
Fig. 8.1

1 r1 m2
i.e. r∝ or =
m r2 m1
Centre of Mass, Conservation of Linear Momentum, Impulse and Collision 299

or m1 r1 = m2 r2 So, the cartesian coordinates of the CM will be


n
 m2  Σ mi x i
r1 =  d m1 x1 + m2 x 2 + K + mn x n i =1
 m2 + m1  xCM = =
m1 + m2 + K + mn Σ mi
 m1  n
and r2 =  d Σ mi x i
 m1 + m2  or xCM =
i =1
M
Here, r1 = distance of CM from m1 n n
and r2 = distance of CM from m2 Σ mi yi Σ mi z i
i =1 i =1
Similarly, yCM = and zCM =
From the above discussion, we see that M M
d
r1 = r2 = if m1 = m2 , i.e. CM lies midway between the X Example 8.2 The position vector of three particles
2
two particles of equal masses. of mass m1 = 1 kg, m2 = 2 kg and m3 = 3 kg are
Similarly, r1 > r2 if m1 < m2 and r1 < r2 if m1 > m2 , i.e. r1 = ( $i + 4$j + k$ ) m, r2 = ( $i + $j + k$ ) m and
CM is nearer to the particle having larger mass. r3 = (2$i − $j − 2 k$ ) m respectively. Find the position
vector of their centre of mass.
X Example 8.1 Two particles of mass 1 kg and 2 kg
are located at x = 0 and x = 3 m. Find the position of Sol. The position vector of CM of the three particles will be given
their centre of mass. by
m1r1 + m2 r2 + m3 r3
Sol. Since, both the particles lie on x-axis, the CM will also lie on rCM =
m1 + m2 + m3
x-axis. Let the CM is located at x = x, then
Substituting the values, we get
r1 = distance of CM from the particle of mass 1 kg = x
and r2 = distance of CM from the particle of mass 2 kg 1($i + 4$j + k$ ) + 2 ($i + $j + k$ ) + 3(2 $i − $j − 2 k$ )
rCM =
= ( 3 − x) 1+ 2 + 3
9$i + 3$j − 3k$
m1 = 1 kg CM m1 = 2 kg =
6
x=0 x=x x=3 1 $ $ $
rCM = (3 i + j − k) m
r1 = x r2 = (3–x) 2
Fig. 8.2 X Example 8.3 Four particles of mass 1 kg, 2 kg,
r1 m2 x 2
Using, = or = or x = 2 m 3 kg and 4 kg are placed at the four vertices A, B, C
r2 m1 3− x 1
and D of a square of side 1 m. Find the position of
Thus, the CM of the two particles is located at x = 2 m. centre of mass of the particles.

Position of CM of a Large Number of Sol. Assuming D as the origin, DC as x-axis and DA as y-axis, we
have
Particles y

If we have a system consisting of n particles, of mass A B


m1 , m2 , K, mn with r1 r2 , ..., rn as their position vectors at a
given instant of time. The position vector rCM of the CM of
x
the system at that instant is given by D C
n

m r + m2 r2 + K + mn rn i Σ= 1 mi ri
Fig. 8.3

rCM = 1 1 = n m1 = 1 kg, ( x1, y1) = (0, 1 m)


m1 + m2 + K + mn m2 = 2 kg, ( x2 , y2 ) = (1 m, 1 m)
Σ mi
i =1 m3 = 3 kg, ( x3 , y3 ) = (1 m, 0)
n
Σ mi ri and m4 = 4 kg, ( x4 , y4 ) = (0, 0)
i =1
or rCM = Coordinates of their CM are
M m x + m2 x2 + m3 x3 + m4 x4
xCM = 1 1
m1 + m2 + m3 + m4
Here, M = m1 + m2 + K + mn and Σ mi ri is called the
1(0) + 2(1) + 3(1) + 4(0)
first moment of the mass. =
1+ 2+ 3+ 4
Further, ri = x i $i + yi $j + z i k$ 5 1
= = m = 0.5 m
and rCM = xCM $i + yCM $j + zCM k$ 10 2
300 Objective Physics Vol. 1

Similarly, L M 
yCM =
m1 y1 + m2 y2 + m3 y3 + m4 y4 ∫0(x ) 
 L
dx 

m1 + m2 + m3 + m4 =
1(1) + 2(1) + 3(0) + 4(0) M
=
1+ 2+ 3+ 4 1 L L
3 = ∫ x dx =
= m = 0.3 m L 0 2
10
∴ ( xCM , yCM ) = ( 0.5m, 0.3 m) The y-coordinate of CM is
Thus, position of CM of the four particles is as shown in
figure. yCM =
∫ y dm = 0 (as, y = 0)
A B ∫ dm
Similarly, zCM = 0
CM
L 
0.5 m 0.3 m i.e. the coordinates of CM of the rod are  , 0, 0 or it
D C 2 
Fig. 8.4 lies at the centre of the rod.
Proceeding in the similar manner, we can find the CM of
Position of CM of Continuous Bodies certain rigid bodies. Centre of mass of some well known
If we consider the body to have continuous distribution rigid bodies are given below:
of matter, then summation in the formula of CM is replaced 1. Centre of mass of a uniform rectangular, square or
by integration. Suppose x, y and z are the coordinates of a circular plate lies at its centre.
small element of mass dm, we write the coordinates of CM
as

xCM =
∫ x dm = ∫ x dm CM CM
∫ dm M

yCM =
∫ y dm = ∫ y dm
∫ dm M
CM

and zCM =
∫ z dm = ∫ z dm
∫ dm M
Fig. 8.6
Let us take an example.
2. Centre of mass of a uniform semicircular ring lies at
Centre of Mass of a Uniform Rod 2R
a distance of h = from its centre, on the axis of
Suppose a rod of mass M and length L is lying along π
the x-axis with its one end at x = 0 and the other at x = L. symmetry where R is the radius of the ring.
P Q
x CM
R 2R
x=0 x=L
dx π
Fig. 8.5 O
M Fig. 8.7
Mass per unit length of the rod =
L 3. Centre of mass of a uniform semicircular disc of
4R
Hence, the mass of the element PQ of length dx situated radius R lies at a distance of h = from the centre on
M 3π
at x = x is dm = dx the axis of symmetry as shown in Fig. 8.8.
L
The coordinates of the element PQ are ( x, 0, 0).
R CM
Therefore, x-coordinate of CM of the rod will be 4R
L 3π

xCM =
∫0 x dm O
Fig. 8.8
∫ dm
Centre of Mass, Conservation of Linear Momentum, Impulse and Collision 301

4. Centre of mass of a hemispherical shell of radius R


R Extra Knowledge Points
lies at a distance of h = from its centre on the axis of
2 ■ For a laminar type (2-dimensional) body the formulae
symmetry as shown in figure. for finding the position of centre of mass are as follows:
A r + A2 r2 + K + An rn
(i) rCM = 1 1
A1 + A2 + .. + An
R CM
R A1x1 + A2 x 2 + K + An xn
2 (ii) x CM =
O A1 + A2 + K + An
Fig. 8.9 A1y1 + A2 y 2 + K + An yn
y CM =
A1 + A2 + K + An
5. Centre of mass of a solid hemisphere of radius R lies A1z1 + A2 z 2 + K + An zn
3R and z CM =
at a distance of h = from its centre on the axis of A1 + A2 + K + An
8
Here, A stands for the area.
symmetry.
■If some mass or area is removed from a rigid body,
then the position of centre of mass of the remaining
R CM 3R portion is obtained from the following formulae:
8 m r − m2 r2 A r − A2 r2
O (i) rCM = 1 1 or rCM = 1 1
m1 − m2 A1 − A2
Fig. 8.10
m1x1 − m2 x 2 A x − A2 x 2
(ii) x CM = or x CM = 1 1
m1 − m2 A1 − A2
X Example 8.4 Find the position of centre of mass of m1y1 − m2 y 2 A1y1 − A2 y 2
the uniform lamina as shown in figure. y CM = or y CM =
m1 − m2 A1 − A2
Y
m1z1 − m2 z 2 A1z1 − A2 z 2
and z CM = or z CM =
m1 − m2 A1 − A2
Here, m1, A1,r1 x1, y1 and z1 are the values for the whole
a mass while m2 , A2 , r2 , x 2 , y 2 and z 2 are the values for
O X the mass which has been removed. Let us see two
examples in support of the above theory.
Students are often confused over the problems of centre
of mass. They cannot answer even the basic problems of
Fig. 8.11
CM. For example, let us take a simple problem : two
Sol. Here, A1 = area of complete circle = πa2 particles one of mass 1 kg and the other of 2 kg are
projected simultaneously with the same speed from the
A2 = area of small circle roof of a tower, the one of mass 1 kg vertically upwards
2
πa2
= π   =
a and the other vertically downwards. What is the
2 4 acceleration of centre of mass of these two particles?
( x1, y1 ) = coordinates of centre of mass of small circle = (0, 0) When I ask this question in my first class of centre of
mass, three answers normally come among the students
( x2 , y2 ) = coordinates of centre of mass of small circle g
g , and zero. The correct answer is g. Because
=  , 0
a
3
2 
A x − A2 x2
m1 a 1 + m2 a 2
Using, xCM = 1 1 a CM =
A1 − A2 m1 + m2

π a2  a  1 a1 = a 2 = g
−  
Here, (downwards)
−  
We get, xCM = 4 2 =  8
a=−
a (1)(g ) + ( 2)(g )
 3 ∴ a CM = = g (downwards)
π a2 6 1+ 2
π a2 −  
4  4
The idea behind this is that apply the basic equations
and yCM = 0 as y1 and y2 both are zero.Therefore, when asked anything about centre of mass. Just as a
coordinates of CM of the lamina as shown in above figure revision I am writing below all the basic equations of CM
are  − , 0 .
a at one place.
 6 
302 Objective Physics Vol. 1

m1r1 + m2r2 + K + mn rn drCM dr dr dr


rCM = M = m1 1 + m2 2 + K + mn n
m1 + m2 + K + mn dt dt dt dt
m1x1 + m2 x 2 + K + mn xn dr
x CM = Since, = velocity
m1 + m2 + K + mn dt
m1y1 + m2 y 2 + K + mn yn Therefore, M v CM = m1 v 1 + m2 v 2 + K + mn v n …(i)
y CM =
m1 + m2 + K + mn
or velocity of the CM is
m1z1 + m2 z 2 + K + mn zn m v + m2 v 2 + .. + mn v n
z CM =
m1 + m2 + K + mn v CM = 1 1
M
m v + m2 v 2 + K + mn vn
v CM = 1 1 n
m1 + m2 + ... + mn Σ mi v i
i =1
p CM = p1 + p 2 + ... + pn or v CM =
M
m1a 1 + m2a 2 + K + mn a n
a CM = Further, mv = momentum of a particle p. Therefore, Eq. (i)
m1 + m2 + K + mn
can be written as
and FCM = F1 + F2 + ... + Fn n
pCM = p1 + p 2 + ... + p n or PCM = Σ Pi
■ If a projectile explodes in air in different parts, the path i =1
of the centre of mass remains unchanged. This is Differentiating Eq. (i) with respect to time t, we get
because during explosion no external force (except dv dv dv 2 dv n
gravity) acts on the centre of mass. The situation is as M CM = m1 1 + m2 + K + mn
shown in figure. dt dt dt dt
Explosion or Ma CM = m1 a 1 + m2 a 2 + K + mn a n …(ii)
B m a + m2 a 2 + K + mn a n
u or a CM = 1 1
M
Path of CM n
Σ mi a i
i =1
or a CM =
M
A C
Further, in accordance with Newton’s second law of
Path of CM is ABC, even though the different parts motion F = ma. Hence, Eq. (ii) can be written as
travel in different directions after explosion.Suppose a n
system consists of more than one particle (or bodies). FCM = F1 + F2 + ... + Fn or FCM = Σ Fi
Net external force on the system in a particular i =1
direction is zero. Initially the centre of mass of the Thus, as pointed out earlier also, the centre of mass of a
system is at rest, then obviously the centre of mass will
system of particles moves as though it were a particle of
not move along that particular direction even though
some particles (or bodies) of the system may move mass equal to that of the whole system with all the external
along that direction. The following example will forces acting directly on it.
illustrate the above theory.
X Example 8.5 Two particles A B
and B of mass 1 kg and 2 kg
uB
Motion of the Centre of Mass respectively are projected in the
90 m
directions shown in figure with uA
Let us consider the motion of a system of n particles of
speeds u A = 200 m/s and uB = 50 m/s.
individual masses m1 , m2 , ..., mn and total mass M. It is
Initially they were 90 m apart. Find A
assumed that no mass enters or leaves the system during its
the maximum height attained by the
motion, so that M remains constant. Then, as we have seen,
we have the relation centre of mass of the particles. Fig. 8.12
Assume acceleration due to gravity to
m r + m2 r 2 + K mn r n
rCM = 1 1 be constant. ( g = 10 m/ s 2 ).
m1 + m2 + K + mn
Sol. Using, mA rA = mB rB
m1 r1 + m2 r 2 + K + mn r n
= or 1(rA ) = 2 (rB )
M
or rA = 2 rB …(i)
or M rCM = m1 r1 + m2 r 2 + K + mn r n and rA + rB = 90 m …(ii)
Differentiating this expression with respect to time t, we Solving these two equations, we get
have rA = 60 m and rB = 30 m
Centre of Mass, Conservation of Linear Momentum, Impulse and Collision 303

i.e. CM is at height 60 m from the ground at time t = 0. mA a A + mB a B


Now, a CM =
m a + mB a B mA + mB
Further, a CM = A A
mA + mB 2 (a) − 1(a) a g
= = = (downwards)
= g = 10 m/s 2 (downwards) 1+ 2 3 9
Alternate Method
as aA = aB = g (downwards)
Free body diagram of block A is shown in figure.
mA u A + mB u B
u CM = T
mA + mB
1(200) − 2 (50)
=
1+ 2
100 A a
= m/s (upwards)
3
Let h be the height attained by CM beyond 60 m. Using
2 2 mAg = 2g
vCM = uCM + 2 a CM h
Fig. 8.15
2
100 
or 0 =   − (2 ) (10) h 2g − T = mA (a) or T = 2 g − mA a
 3 
4g
= 2 g − (2 )   =
g
(100)2  3 3
or h=
180 Free body diagrams of A and B both are as shown in
Fig. 8.16.
= 55.55 m
T T
Therefore, maximum height attained by the centre of mass
is
H = 60 + 55.55
= 115.55 m A B

X Example 8.6 In the arrangement shown in figure,


m A = 2 kg and mB =1 kg. String is light and mAg mB g
inextensible. Find the acceleration of centre of mass of Fig. 8.16
both the blocks. Neglect friction everywhere. Net force on both the blocks
a CM =
mA + mB
(mA + mB )g − 2T
=
2+1
8g
3g −
= 3 = g (downwards)
A
3 9
B

Fig. 8.13 8.2 Law of Conservation of


Sol. Net pulling force on the system is (mA − mB ) g
Linear Momentum
The product of mass and the velocity of a particle is
or (2 − 1) g = g
defined as its linear momentum ( p). So,
Total mass being pulled is mA + mB or 3 kg
p = mv
The magnitude of linear momentum may be written as
p = mv
or p2 = m2 v 2
1 
a A = 2m  mv 2 
2 
B a
= 2mK
Fig. 8.14 Thus, p = 2Km
Net pulling force g p2
∴ a= = or K=
Total mass 3 2m
304 Objective Physics Vol. 1

Here, K is the kinetic energy of the particle. In X Example 8.7 A wooden plank of mass 20 kg is
accordance with Newton’s second law, resting on a smooth horizontal floor. A man of mass
dv d ( mv ) dp 60 kg starts moving from one end of the plank to the
F = ma = m = =
dt dt dt other end. The length of the plank is 10 m. Find the
dp displacement of the plank over the floor when the man
Thus, F=
dt reaches the other end of the plank.
In this case, the external force applied to a particle (or a
body) be zero, we have
dp
F= = 0 or p = constant
dt
Showing that in the absence of an external force, the
linear momentum of a particle (or the body) remains 10 m
constant. This is called the law of conservation of linear Fig. 8.17
momentum. This law may be extended to a system of
particles or to the centre of mass of a system of particles. For Sol. Here the system is man + plank. Net force on this system in
example, for a system of particles it takes the form. horizontal direction is zero and initially the centre of mass of the
system is at rest. Therefore, the centre of mass does not move in
If net force (or the vector sum of all the forces) on a horizontal direction.
system of particles is zero, the vector sum of linear
momentum of all the particles remain conserved, or
If F = F1 + F2 + F3 + ... + Fn = 0
Then, p1 + p 2 + p 3 + ... + p n = constant
The same is the case for the centre of mass of a system of x
particles, i.e. if x=0 10 m Initial position
FCM =0, pCM = constant.
Thus, the law of conservation of linear momentum can
be applied to a single particle, to a system of particles or
even to the centre of mass of the particles.
The law of conservation of linear momentum enables us
x 10 – x Final position
to solve a number of problems which cannot be solved by a
straight application of the relation F = ma. Fig. 8.18

For example, suppose a particle of mass m initially at Let x be the displacement of the plank. Assuming the
rest, suddenly explodes into two fragments of masses m1 and origin, i.e. x = 0 at the position shown in figure.
m2 which fly apart with velocities v 1 and v 2 , respectively. As we said earlier also, the centre of mass will not move in
Obviously, the forces resulting in the explosion of the horizontal direction (x-axis). Therefore, for centre of mass to
remain stationary,
particle must be internal forces, since no external force has
been applied. In the absence of the external forces, therefore, xi = xf
10 10
60(0) + 20   60(10 − x) + 20  − x
the momentum must remain conserved and we should have
mv = m1v 1 + m2 v 2 2 2 
=
60 + 20 60 + 20
Since, the particle was initially at rest, v =0 and
10
therefore, 6(10 − x) + 2  − x
m1 v 1 + m2 v 2 = 0 5 2 
or =
m | v 1 | m2 4 8
v 1 = − 2 v 2 or = 60 − 6 x + 10 − 2 x
m1 | v 2 | m1 =
8
Showing at once that the velocities of the two fragments or 5 = 30 − 3 x + 5 − x
must be inversely proportional to their masses and in
or 4 x = 30
opposite directions along the same line. This result could not
30
possibly be arrived at from the relation F = ma, since we or x= m
4
know nothing about the forces that were acting during the
explosion. Nor, could we derive it from the law of or x = 7.5 m
conservation of energy. / The centre of mass of the plank lies at its centre.
Centre of Mass, Conservation of Linear Momentum, Impulse and Collision 305

X Example 8.8 A man of mass m1 is standing on a In such problems, you have nothing to do but apply a
platform of mass m2 kept on a smooth horizontal thrust force ( Ft ) to the main mass in addition to the all other
surface. The man starts moving on the platform with a forces acting on it. This thrust force is given by,
velocity v r relative to the platform. Find the recoil  dm 
Ft = v rel  
velocity of platform.  dt 
Sol. Absolute velocity of man = v r − v, where v = recoil velocity Here, v rel is the velocity of the mass gained or mass
of platform. Taking the platform and the man as a system, net
external force on the system in horizontal direction is zero. The
ejected relative to the main mass. In case of rocket, this is
linear momentum of the system remains constant. Initially both dm
sometimes called the exhaust velocity of the gases. is the
the man and the platform were at rest. dt
rate at which mass is increasing or decreasing.

vr – v vr
v
v v+dv
dm
m m – dm
Fig. 8.19 system
Fig. 8.21
m1v r
Hence, 0 = m1(v r − v ) − m2 v ⇒ ∴ v=
m1 + m2
The expression for the thrust force can be derived from
X Example 8.9 A gun (mass = M) fires a bullet the conservation of linear momentum in the absence of any
(mass = m ) with speed v r relative to barrel of the gun external forces on a system as follows :
which is inclined at an angle of 60° with horizontal. Suppose at some moment t = t mass of a body is m and
The gun is placed over a smooth horizontal surface. its velocity is v. After some time at t = t + dt its mass
Find the recoil speed of gun. becomes ( m − dm) and velocity becomes v + d v . The mass
Sol. Let the recoil speed of gun is v. Taking gun + bullet as the dm is ejected with relative velocity v r . Absolute velocity of
system. Net external force on the system in horizontal direction mass dm is therefore ( v r + v + dv ). If no external forces are
is zero. Initially the system was at rest. Therefore, applying the acting on the system, then linear momentum of the system
principle of conservation of linear momentum in horizontal
will remain conserved, or
direction, we get
vr Hence, pi = p f
or mv = ( m − dm) ( v + dv ) + dm ( v r + v + dv )
60° vr sin 60°
or mv = mv + mdv − dmv − ( dm) ( dv )
vr cos 60°– v + dmv + v r dm + ( dm) ( dv )
v
M  dv   dm 
Components of velocity ∴ mdv = − v r dm or m   = v r  
of bullet relative to ground  dt   dt 
 dv 
Fig. 8.20 Here, m   = thrust force (Ft )
 dt 
Mv − m( v r cos 60° − v ) = 0 dm
mv r cos 60° mv r and − = rate at which mass is ejecting
∴ v= or v = dt
M +m 2( M + m)
Problems Related to Variable Mass can be
8.3 Variable Mass Solved in Following Three Steps
1. Make a list of all the forces acting on the main mass
In our discussion of the conservation of linear and apply them on it.
momentum, we have so far dealt with systems whose mass
remains constant. We now consider those systems whose 2. Apply an additional thrust force Ft on the mass, the
  dm  
mass is variable, i.e. those in which mass enters or leaves the magnitude of which is v r  ±   and direction is
system. A typical case is that of the rocket from which hot   dt  
gases keep on escaping, thereby continuously decreasing its given by the direction of v r in case the mass is
mass.
306 Objective Physics Vol. 1

increasing and otherwise the direction of − v r if it is Note Points


decreasing.
Ft = vr  −
dm  dm
3. Find net force on the mass and apply /  is upwards, as vr is downwards and is
 dt  dt
dv
Fnet = m (m = mass at that particular instant) negative.
dt / If gravity is ignored and initial velocity of the rocket u = 0,
m 
Eq. (i) reduces to v = vr ln  0  .
Rocket Propulsion  m 

Let m0 be the mass of the rocket at time t = 0. m its mass X Example 8.10 (a) A rocket set for vertical firing
at any time t and v its velocity at that moment. Initially let us weighs 50 kg and contains 450 kg of fuel. It can have a
suppose that the velocity of the rocket is u. maximum exhaust velocity of 2 km/s. What should be its
u v minimum rate of fuel consumption
(i) to just lift it off the launching pad.
At t=0 (ii) to give it an acceleration of 20 m/s 2 .
At t=t
v=u m=m (b) What will be the speed of the rocket when the rate of
m = m0 v=v consumption of fuel is 10 kg/s after whole of the fuel is
consumed? (Take g = 9.8 m / s 2 )
Exhaust velocity = vr Sol. (a) (i) To just lift it off the launching pad
Fig. 8.22 weight = thrust force
− dm 
 −dm  or mg = v r  
Further, let   dt 
 be the mass of the gas ejected per
 dt   − dm  = mg
or  
unit time and v r the exhaust velocity of the gases. Usually  dt  vr
 −dm  Substituting the values, we get
  and v r are kept constant throughout the journey of  − dm  = (450 + 50)(9.8) = 2.45 kg/s
 dt   
 dt  2 × 103
the rocket. Now, let us write few equations which can be
(ii) Net acceleration a = 20 m/s 2
used in the problems of rocket propulsion. At time t = t,
∴ ma = Ft − mg
1. Thrust force on the rocket F
 dm  or a= t −g
Ft = v r  –  (upwards) m
 dt  a=
v r  − dm 
or   −g
m  dt 
2. Weight of the rocket
W = mg (downwards)  − dm  = m (g + a)
This gives  
 dt  vr
3. Net force on the rocket
Substituting the values, we get
Fnet = Ft – W (upwards)
 − dm  = (450 + 50)(9.8 + 20)
 −dm  
 dt 

or F net = v r   − mg 2 × 103
 dt 
= 7.45 kg/s
F (b) The rate of fuel consumption is 10 kg/s. So, the time for
4. Net acceleration of the rocket a =
m the consumption of entire fuel is
450
dv v r  −dm  t = = 45 s
or =   −g 10
dt m  dt  m 
Using Eq. (i), i.e. v = u − gt + v r ln  0 
 m
 −dm 
or dv = v r   − g dt
 m  Here, u = 0, v r = 2 × 103 m/s
v m −dm t m0 = 500 kg
or ∫u dv = v r ∫m0 m
− g ∫ dt
0 and m = 50 kg
 m0  Substituting the values, we get,
or v − u = v r ln   − gt 500 
 m v = 0 − (9.8)(45) + (2 × 103 ) ln  
 50 
m  v = −441 + 4605.17
Thus, v = u – gt + v r ln  0  …(i) or
 m or v = 4164.17 m/s or v = 4.164 km/s
Centre of Mass, Conservation of Linear Momentum, Impulse and Collision 307

/ The velocity component in the direction of $j is unchanged.


8.4 Impulse This is because there is no impulse component in this
direction.
Consider a constant force F which acts for a time t on a
body of mass m thus. changing its velocity from u to v. X Example 8.13 A particle of mass 2 kg is initially
Because the force is constant, the body will travel with at rest. A force starts acting on it in one direction
constant acceleration a. Force is where whose magnitude changes with time. The force-time
F = ma and at = v − u graph is as shown in figure.
F
Hence, t = v − u or Ft = mv − mu F(N)
m
The product of constant force F and the time t for which
it acts is called the impulse ( J) of the force and this is equal 20
to the change in linear momentum which it produces.
Thus, impulse ( J) = Ft = ∆p = p f − p i 10

Instantaneous Impulse There are many occasions t(s)


O 2 4 6 10
when a force acts for such a short time that the effect is
instantaneous, e.g. a bat striking a ball. In such cases, Fig. 8.24
although the magnitude of the force and the time for which it
acts may each be unknown but the value of their product (i.e. Find the velocity of the particle at the end of 10 s.
impulse) can be known by measuring the initial and final Sol. Using impulse = change in linear momentum
momenta. Thus, we can write (or area under F-t graph)
m(v f − v i ) = Area
J = ∫ Fdt = ∆p = p f − p i
We have
1
or 2 (v f − 0) = × 2 × 10 + 2 × 10
2
Regarding the impulse, it is important to note that 1 1
impulse applied to an object in a given time interval can also + × 2 × (10 + 20) + × 4 × 20
2 2
be calculated from the area under force-time (F-t) graph in = 10 + 20 + 30 + 40
the same time interval. or 2 v f = 100
X Example 8.11 A truck of mass 2 × 10 3 kg ∴ v f = 50 m/s
travelling at 4 m/s is brought to rest in 2 s when it
strikes a wall. What force (assume constant) is exerted
by the wall?
8.5 Collision
Contrary to the meaning of the term collision in our
Sol. Using impulse = change in linear momentum.
everyday life, in physics it does not necessarily mean one
particle ‘striking’ against other. Indeed two particles may not
F u +ve even touch each other and may still be said to collide. All that
is implied is that as the particles approach each other,
(i) an impulse (a large force for a relatively short
Fig. 8.23
time) acts on each colliding particles.
We have F ⋅ t = mv f − mv i = m(v f − v i ) (ii) the total momentum of the particles remain
or F(2 ) = 2 × 103 [0 − (−4)] conserved.
or 2 F = 8 × 103
The collision is in fact a redistribution of total
or F = 4 × 103 N momentum of the particles. Thus, law of conservation of
linear momentum is indispensible in dealing with the
X Example 8.12 A ball of mass m, travelling with phenomenon of collision between particles. Consider a
velocity 2i + 3 j receives an impulse −3m$i. What is the situation shown in figure.
velocity of the ball immediately afterwards? Two blocks of masses m1 and m2 are moving with
Sol. Using, J = m(v f − v i ) velocities v1 and v 2 ( < v1 ) along the same straight line in a
−3m $i = m [v f − (2 $i + 3$j )] smooth horizontal surface. A spring is attached to the block
or v f = − 3$i + (2 $i + 3$j ) of mass m2 . Now, let us see what happens during the
or v = − $i + 3$j
collision between two particles.
f
308 Objective Physics Vol. 1

v1 v2 F F / In the above situation, we have assumed spring to be


perfectly elastic, i.e. it regains its original shape and size
m1 m2 m1 m2 after the two blocks are separated. In actual practice, there is
no such spring between the two blocks. During collision both
v1 > v2 the blocks (or bodies) are a little bit deformed. This situation is
(a) (b) similar to the compression of the spring. Due to deformation two
equal and opposite forces act on both the blocks. These two
v v F F forces redistribute their linear momentum in such a manner that
both the blocks are separated from one another. The collision is
m1 m2 m1 m2 said to be elastic if both the blocks regain their original shape
and size completely after they are separated. On the other
Maximum compression =xm hand if the blocks do not return to their original form the
(c) (d) collision is said to be inelastic. If the deformation is
v1' v2' permanent and the blocks move together with same velocity
after the collision, the collision is said to be perfectly inelastic.
m1 m2
X Example 8.14 Two blocks A and B of equal mass
v1' < v2' m =1.0 kg are lying on a smooth horizontal surface as
(e) shown in figure. A spring of force constant k = 200 N/m
Fig. 8.25 is fixed at one end of block A. Block B collides with
block A with velocity v 0 = 2.0 m/s. Find the maximum
Fig. (a) Block of mass m1 is behind m2 . Since, v1 > v 2 , compression of the spring.
the blocks will collide after some time. 2.0 m/s
Fig. (b) The spring is compressed. The spring force
F ( = kx ) acts on the two blocks in the directions shown in A
B
figure. This force decreases the velocity of m1 and increases
the velocity of m2 .
Fig. 8.26
Fig. (c) The spring will compress till velocity of both
the blocks become equal. So, at maximum compression (say Sol. At maximum compression ( xm ) velocity of both the blocks is
x m ) velocities of both the blocks are equal (say v). same, say it is v. Applying conservation of linear momentum, we
Fig. (d) Spring force is still in the directions shown in have,
figure, i.e. velocity of block m1 is further decreased and that (mA + mB )v = mB v 0
of m2 is increased. The spring now starts relaxing. or (1.0 + 1.0)v = (1.0) v 0
v 2.0
Fig. (e) The two blocks are separated from one or v = 0 = = 1.0 m/s
2 2
another. Velocity of block m2 becomes more than the
velocity of block m1 , i.e. v 2′ > v1′ . Using conservation of mechanical energy, we have
1 1 1 2
mB v 02 = (mA + mB ) v 2 + kxm
Equations which can be Used in the Above 2 2 2
Substituting the values, we get
Situation 1 1 1
× (1) × (2.0)2 = × (1.0 + 1.0) × (1.0)2+ × (200) × xm
2
Assuming spring to be perfectly elastic following, two 2 2 2
2
equations can be applied in the above situation. or 2 = 1.0 + 100 xm
(i) In the absence of any external force on the system the or xm = 0.1 m = 10.0 cm
linear momentum of the system will remain conserved
before, during and after collision, i.e. Types of Collision
m1 v1 + m2 v 2 = ( m1 + m2 ) v Collision between two bodies may be classified in two
= m1 v1′ + m2 v 2′ …(i) ways :
(ii) In the absence of any dissipative forces, the 1. Elastic collision and inelastic collision.
mechanical energy of the system will also remain 2. Head on collision or oblique collision.
conserved, i.e. As discussed earlier also collision between two bodies
1 1 1 1 is said to be elastic if both the bodies come to their original
m1 v12 + m2 v 22 = (m1 + m2 ) v 2 + kx m2 shape and size after the collision, i.e. no fraction of
2 2 2 2
1 1 mechanical energy remains stored as deformation potential
= m1 v1′ 2 + m2 v 2′ 2 …(ii) energy in the bodies. Thus, in addition to the linear
2 2
Centre of Mass, Conservation of Linear Momentum, Impulse and Collision 309

momentum, kinetic energy also remains conserved before 4 m/s 3 m/s


m m
and after collision. On the other hand, in an inelastic
collision, the colliding bodies do not return to their original Before Collision
shape and size completely after collision and some part of
the mechanical energy of the system goes to the deformation
3 m/s 4 m/s
potential energy. Thus, only linear momentum remains m m
conserved in case of an inelastic collision.
After Collision
Further, a collision is said to be head on (or direct) if the
directions of the velocity of colliding objects are along the v=0
2 m/s
line of action of the impulses, acting at the instant of m m
collision. If just before collision, at least one of the colliding
objects was moving in a direction different from the line of Before Collision
action of the impulses, the collision is called oblique or v=0
2 m/s
indirect. m m
Problems related to oblique collision are usually not
asked in any medical entrance test. Hence, only head on After Collision
collision is discussed below. Fig. 8.28

Head on Elastic Collision 2. If m1 > > m2 and v1 = 0.


v1 = 0
Let the two balls of mass m1 and m2 collide each other
v2 m1
elastically with velocities v1 and v 2 in the directions shown m2
in Fig. 8.27. Their velocities become v1′ and v 2′ after the
collision along the same line. Applying conservation of Before Collision
linear momentum, we get v1′ ≈ 0
m2 m1 v2′ ≈ – v2
v2 v1 v2
m2 m1

Before Collision After Collision


m2 m1 Fig. 8.29
v2' v1'
m2
After Collision Then, ≈0
Fig. 8.27
m1
 m 
m1 v1 + m2 v 2 = m1 v1 ′ + m2 v 2 ′ …(i) With these two substitutions  v1 = 0 and 2 = 0
In an elastic collision kinetic energy before and after  m1 
collision is also conserved. Hence, We get the following two results
1 1 1 1 v1 ′ ≈ 0 and v 2 ′ ≈ − v 2
m1 v12 + m2 v 22 = m1 v1′ 2 + m2 v 2′ 2 …(ii)
2 2 2 2 i.e. the particle of mass m1 remains at rest while the
Solving Eqs. (i) and (ii) for v1 ′ and v 2 ′, we get particle of mass m2 bounces back with same speed
 m − m2   2m2  v2 .
v1′ =  1  v1 +   v2 …(iii) 3. If m2 > > m1
 m1 + m2   m1 + m2 
and v1 = 0
 m − m1   2m1 
and v 2′ =  2  v2 +   v1 …(iv) v1 = 0
 m1 + m2   m1 + m2  v2
m2 m1

Special Cases Before Collision


1. If m1 = m2 , then from Eqs. (iii) and (iv), we can see that
v1′ ≈ 0
v2′ ≈ – v2
v1′ = v 2 and v 2′ = v1 v2
m2 m1
i.e. when two particles of equal mass collide
elastically and the collision is head on, they exchange After Collision
their velocities., e.g. Fig. 8.30
310 Objective Physics Vol. 1

m1 Since, the collision is elastic. Using Eqs. (iii) and (iv)


With the substitution ≈ 0 and v1 = 0, we get the discussed in the theory the velocities after collision are
m2
 m − 2 m  4m 
results v1 ′ =   (− 2 gH ) +   2 gH
v1 ′ ≈ 2v 2 and v 2 ′ ≈ v 2  m + 2 m  m + 2 m

i.e. the mass m1 moves with velocity 2v 2 while the 2 gH 4 2 gH


= +
velocity of mass m2 remains unchanged. 3 3
It is important to note that Eqs. (iii) and (iv) and their three 5
/
= 2 gH
special cases can be used only in case of a head on elastic 3
collision between two particles. I have found that many  2 m − m  2m 
students apply these two equations even if the collision is and v2 ′ =   ( 2 gH ) +   (− 2 gH )
 m + 2 m  m + 2 m
inelastic and do not apply these relations where clearly a
head on elastic collision is given in the problem. 2 gH 2 2 gH
= −
3 3
X Example 8.15 Two particles of mass m and 2m 2 gH
moving in opposite directions collide elastically with =−
3
velocities v and 2v. Find their velocities after collision. i.e. the velocities of the balls after the collision are as
Sol. Here, v1 = − v, v 2 = 2 v, m1 = m and m2 = 2 m. shown in figure.
2v
2m v
m +ve

Fig. 8.31

Substituting these values in Eqs. (iii) and (iv), we get


 m − 2m  4m 
v1′ =   (− v ) +   (2 v )
 m + 2 m   m + 2 m √2gH v1' = 5 √2gH
v2' = 3
v 8v 3
or v1′ = + = 3v
3 3 Fig. 8.34
 2m − m  2m 
and v1′ =   (2 v ) +   (− v ) Therefore, the heights to which the balls rise after the
 m + 2 m   m + 2 m collision are
2 2 (v ′ )2
or v1′ = v − v = 0 h1 = 1 (using, v 2 = u 2 − 2 gh)
3 3 2g
2
2m 3v  5 2 gH 
m  
  25
or h1 = 3 or h1 = H
2g 9
Fig. 8.32
2
 2 gH 
i.e. the second particle (of mass2m) comes to a rest while 2
 
(v 2 ′ )  3 
the first (of mass m) moves with velocity 3v in the and h2 = or h2 =
2g 2g
direction shown in Fig. 8.32. H
or h2 =
9
X Example 8.16 Two pendulum bobs of mass m and
/ Since, the collision is elastic, mechanical energy of both the
2m collide elastically at the lowest point in their balls will remain conserved, or
motion. If both the balls are released from a height H Ei = Ef
above the lowest point, to what heights do they rise for ⇒ (m + 2m) gH = mgh1 + 2mgh2
the first time after collision? ⇒ 3mgH = (mg) 
25 
H  + (2mg) H 
 
 9   9
Sol. Given, m1 = m, m2 = 2 m,
⇒ 3mgH = 3mgH

Head on Inelastic Collision


As we have discussed earlier also, in an inelastic
2 1
+ve collision, the particles do not regain their shape and size
completely after collision. Some fraction of mechanical
v2 v1 energy is retained by the colliding particles in the form of
Fig. 8.33 deformation potential energy. Thus, the kinetic energy of
v1 = − 2 gH and v 2 = 2 gH the particles no longer remains conserved. However, in the
Centre of Mass, Conservation of Linear Momentum, Impulse and Collision 311

absence of external forces, law of conservation of linear or separation speed = approach speed
momentum still holds good. or e =1
v2 v1 Let us now find the velocities of two particles after
m2 m1
collision if they collide directly and the coefficient of
Before Collision restitution between them is given as e.
m2 m1
v2
v2' v1' v1 = 0
m2 m1
+ve
Before Collision
After Collision
v2' v1'
Fig. 8.35

Suppose the velocities of two particles of mass m1 and


m2 before collision be v1 and v 2 in the directions shown in After Collision
figure. Let v1 ′ and v 2 ′ be their velocities after collision. The Fig. 8.37
law of conservation of linear momentum gives Applying conservation of linear momentum
v'
m1 v1 + m2 v 2 = m1 v1 ′ + m2 v 2 ′ …(viii)
Further, separation speed = e (approach speed)
or v1′ − v 2′ = e ( v 2 − v1 ) …(ix)
Fig. 8.36 Solving Eqs. (viii) and (ix), we get
 m − em2   m2 + em2 
m1 v1 + m2 v 2 = m1 v1′ + m2 v 2′ …(v) v1′ =  1  v1 +   v2 …(x)
 m1 + m2   m1 + m2 
Collision is said to be perfectly inelastic if both the
particles stick together after collision and move with same  m − em1   m1 + em1 
and v 2′ =  2  v2 +   v1 …(xi)
velocity, say v′ as shown in figure. In this case, Eq. (v) can be  m1 + m2   m1 + m2 
written as
m1 v1 + m2 v 2 = ( m1 + m2 ) v ′ Special Cases
m v + m2 v 2 1. If collision is elastic, i.e. e =1, then
or v′ = 1 1 …(vi)
m1 + m2  m − m2   2m2 
v1′ =  1  v1 +   v2
 m1 + m2   m1 + m2 
Newton’s Law of Restitution  m − m1   2m1 
When two objects are in direct (head on) impact, the and v 2′ =  2  v2 +   v1
 m1 + m2   m1 + m2 
speed with which they separate after impact is usually less
than or equal to their speed of approach before impact. which are same as Eqs. (iii) and (iv).
Experimental evidence suggests that the ratio of these 2. If collision is perfectly inelastic, i.e. e = 0, then
m v + m2 v 2
relative speeds is constant for two given set of objects. This v1′ = v 2′ = 1 1 = v ′ (say )
property formulated by Newton, is known as the law of m1 + m2
restitution and can be written in the form which is same as Eq. (vi).
Separation speed
=e …(vii) 3. If m1 = m2 and v1 = 0, then
Approach speed m2 m1
v
The ratio e is called the coefficient of restitution and is v1 = 0
constant for two particular objects.
In general, 0 ≤ e ≤1 Before Collision

e = 0, for completely inelastic collision, as both the v2' v1'

objects stick together. So, their separation speed is zero or


e = 0 from Eq. (vii).
After Collision
e =1, for an elastic collision, as we can show from Fig. 8.38
Eqs. (iii) and (iv), that
1 + e 1 − e
v1′ − v 2′ = v 2 − v1 v1′ =   v 2 and v 2′ =  v
 2   2  2
312 Objective Physics Vol. 1

Note Points Sol. The speed of wall will not change after the collision. So, let v
be the velocity of the ball after collision in the direction shown in
/ If mass of one body is very-very greater than that of the other, figure. Since, collision is elastic (e = 1),
then after collision velocity of heavy body does not change
appreciably. (Whether the collision is elastic or inelastic).
u=0 1 m/s
2 m/s 1 m/s v

Before Collision After Collision


Fig. 8.42
v0 = √2gh
Fig. 8.39 separation speed = approach speed
or v − 1= 2 + 1
/ In the situation shown in figure, if e is the coefficient of
or v = 4 m/s
restitution between the ball and the ground, than after nth
collision with the floor the speed of ball will remain e n v 0 and it
will go upto a height e 2 n h or,
X Example 8.19 After perfectly inelastic collision
vn = e v0 = e
n n
2gh and hn = e h 2n between two identical particles moving with same
speed in different directions, the speed of the particles
X Example 8.17 A ball of mass m moving at a speed becomes half the initial speed. Find the angle between
v makes a head on collision with an identical ball at the two before collision.
rest. The kinetic energy of the balls after the collision is Sol. Let θ be the desired angle. Linear momentum of the system
3/4 th of the original. Find the coefficient of restitution. will remain conserved. Hence
m
v
m p2 = p12 + p22 + 2 p1 p2 cos θ
v
2m
m v
Before Collision θ 2
m
v2' v1'
v
Fig 8.43
After Collision 2
2 m  v   = (mv )2 + (mv )2 + 2(mv )(mv ) cos θ
Fig. 8.40 or   
 2
Sol. As, we have seen in the above discussion, that under the or 1 = 1 + 1 + 2 cos θ or cos θ = − 1
2
given conditions
1+ e ∴ θ = 120°
v1 ′ =  1 − e v
 v and v 2 ′ =  
 2   2 
3 1 1 3 1
Given that K f = K i or mv1 ′2 + mv 2 ′2 =  mv 2  Extra Knowledge Points
4 2 2 4 2 
Substituting the value, we get ■ During collision if mass of one body is very much
2 2
1 + e + 1 − e = 3 greater than the mass of the other body then the
    velocity of heavy body remains almost unchanged
 2   2  4
after collision, whether the collision is elastic or
or (1 + e ) + (1 − e ) = 3 or 2 + 2e 2 = 3
2 2
inelastic.
1 1
or e2 = or e = ■ Net force on a system is zero, it does not mean that
2 2
centre of mass is at rest. It might be moving with
constant velocity.
X Example 8.18 A ball is ■ Centre of mass of a rigid body is not necessarily the
moving with velocity 2 m/s 2 m/s 1 m/s geometric centre of the rigid body.
towards a heavy wall moving ■ If two many particles are in air, then relative
towards the ball with speed acceleration between any two is zero but acceleration
1m/s as shown in figure. of their centre of mass is g downwards.
Assuming collision to be Fig. 8.41 ■ Coefficient of restitution is the mutual intrinsic
elastic, find the velocity of properties of two bodies. Its value varies from
ball immediately after the collision. 0 to 1.
Chapter Summary with Formulae
■ Position of Centre of Mass
(i) Two point masses
m1 CM m2

r1 r2

d
1 r1 m2
r∝ or =
m r2 m1
∴ m1r1 = m2 r2
m2
r1 = .d
m1 + m2
m1
r2 = .d
m1 + m2
(ii) More than two point masses
m + m2 2
(a) CM = 1 1
m1 + m2
m1x1 + m2 x2 m y + m2 y2 m1z1 + m2 z2
(b) XCM = ⇒ YCM = 1 1 ⇒ ZCM =
m1 + m2 m1 + m2 m1 + m2
(iii) More than two rigid bodies
(a) Centre of mass of symmetrical rigid body (like sphere, disc, cube etc.) lies at its geometric centre.
(b) For two or more than two rigid bodies we can use,
m r + m2 r2 m x + m2 x2
rCM = 1 1 ⇒ XCM = 1 1
m1 + m2 m1 + m2
m1 y1 + m2 y2 m1z1 + m2 z2
YCM = and ZCM =
m1 + m2 m1 + m2
(c) If three dimensional rigid body has uniform density, then mass in above formulae can be replaced by volume (V )
V r + V2 r2
rCM = 1 1
V1 + V2
(d) In case of two dimensional body, mass can be replaced by area ( A).
A r + A2 r2
rCM = 1 1
A1 + A2
(e) If some portion is removed from the body. Then,
A1 r1 − A2 r2
rCM = (in case of two dimensional body)
A1 − A2
Here, A1 = Area of whole body (without removing)
r1 = Position vector of its centre of mass
A1 = Area of removed portion
r2 = Position vector of centre of mass of removed portion
■ Other Formulae of Centre of Mass
(i) FCM = F1 + F2
(ii) pCM = p1 + p2
m v + m2 v2
(iii) vCM = 1 1
m1 + m2
m1 a1 + m2 a2
(iv) aCM =
m1 + m2
m1 s1 + m2 s2
(v) sCM =
m1 + m2
Additional Examples
Example 1. Define centre of mass. Example 6. The friction coefficient between the
Sol. The point in a system, where the whole mass of the system horizontal surface and each of the block shown in the
can be supposed to be concentrated is called centre of mass of figure is 0.2. The collision between the blocks is
the system. perfectly elastic. Find the separation between them
when they come to rest.
Example 2 Should the centre of mass of a body (Take g = 10 m/s 2 )
necessarily lie inside the body? 1.0 m/s
Sol. Not necessarily. For example, the centre of mass of a ring
lies at the centre of the ring, i.e. at a point, where actually there 2 kg 4 kg
is no mass.
16 cm
Example 3. What is the difference between centre of
gravity and centre of mass? Sol. Velocity of first block before collision,
Sol. The centre of gravity of a body is a point, where the whole v12 = 1 2 − 2(2) × 0.16
weight of the body may be supposed to act. Further, the total = 1 − 0.64
gravitational torque on the body about its centre of gravity is v1 = 0.6 m/s
always zero. The centre of mass of a body is a point, where, the
whole mass of the body can be supposed to be concentrated. By conservation of momentum, 2 × 0.6 = 2v1′ + 4 v 2′
The motion of the body under the action of external forces can also v 2′ − v1′ = v1 for elastic collision
be studied by studying the motion of centre of mass, when all It gives v 2′ = 0.4 m/s
the external forces are applied directly on it. The centre of v1′ = − 0.2 m/s
mass and centre of gravity are two different concepts. Now, distance moved after collision
However, the centre of gravity of the body coincides with the (0.4 ) 2 (0.2) 2
s1 = and s 2 =
centre of mass in uniform gravity or gravity-free space. 2× 2 2×2
Example 4. If an external force can only change the ∴ s = s 1 + s 2 = 0.05 m
= 5 cm
state of motion of CM of a body, how does the internal
force of the brakes bring a car to rest? Example 7. A pendulum bob of mass 10 −2 kg is
Sol. Actually, it is not the external force which brings the car to raised to a height 5 × 10 −2 m and then released. At the
rest. The internal force of the brakes on the wheel locks the bottom of its swing, it picks up a mass 10 −3 kg. To what
wheel. Now a large frictional force comes into play between the height will the combined mass rise?
wheels and the ground. This force is external to the system and
brings the car to rest. Sol. Velocity of pendulum bob in mean position
v1 = 2gh = 2 × 10 × 5 × 10 −2
Example 5. Two men stand facing each other on two
boats floating on still water at a distance apart. A rope = 1 m/s
is held at its ends by both. The two boats are found to When the bob picks up a mass 10 −3 kg at the bottom,
meet always at the same point, whether each man pulls then by conservation of linear momentum the velocity
of coalesced mass is given by
separately or both pull together, why? Will the time m1 v1 + m 2 v 2 = (m1 + m 2 ) v
taken be different in the two cases? Neglect friction. 10 −2 + 10 −3 × 0 = (10 −2 + 10 −3 ) v
Sol. Two men on the boats floating on water constitute a single
10 −2 10
system. So, the forces applied by the two men are internal or v= −2
= m/s
forces. Whether each man pulls separately or both pull 1.1 × 10 11
together, the centre of mass of the system of boats remains v 2 (10 /11)2
fixed due to the absence of any external force. Consequently, Now h= =
2g 2 × 10
the two boats meet at a fixed point, which is the centre of mass
of the system. = 4.1 × 10 −2 m
Centre of Mass, Conservation of Linear Momentum, Impulse and Collision 315

Example 8. Three identical balls, ball I, ball II and (The stiffness constant of spring is 100 N/m)
ball III are placed on a smooth floor on a straight line 1 kg
at the separation of 10 m between balls as shown in
figure. Initially balls are stationary. 5 kg
Ball I is given velocity of 10 m/s towards ball II, collision 4m
between ball I and II is inelastic with coefficient of
Sol. Let the velocity of the block and the plank, when the block
restitution 0.5 but collision between ball II and III is
leaves the spring be u and v respectively.
perfectly elastic. What is the time interval between two 1 1 1
consecutive collisions between balls I and II ? By conservation of energy kx 2 = mu 2 + Mv 2
2 2 2
(M = mass of the plank, m = mass of the block)
I II III ⇒ 100 = u 2 + 5v 2 K(i)
By conservation of momentum
mu + Mv = 0
10 m 10 m
u = − 5v K(ii)
Sol. Let velocity of I ball and II ball after collision be v1 and v 2 Solving Eqs. (i) and (ii), we get
10
v 2 − v1 = 0.5 × 10 K(i) 30 v 2 = 100 ⇒ v = m/s
3
mv 2 + mv1 = m × 10 K(ii)
From this moment until block falls, both plank and
⇒ v 2 + v1 = 10 block keep their velocity constant.
Solving Eqs. (i) and (ii) 10
Thus, when block falls, velocity of plank = m/s.
v1 = 2.5 m/s, 3
v 2 = 7.5 m/s
Ball II after moving 10 m collides with ball III Example 10. A particle of mass 2 kg moving with a
elastically and stops. But ball I moves towards ball II. velocity 5 i m/s collides head-on with another particle
Time taken between two consecutive collisions of mass 3 kg moving with a velocity −2i m/s. After the
10 − 10 ×
2.5 collision the first particle has speed of 1.6 m/s in
10 7.5 negative x direction. Find
=
7.5 2.5 (a) velocity of the centre of mass after the collision.
=4s (b) velocity of the second particle after the collision.
(c) coefficient of restitution.
Example 9. A plank of mass 5 kg is placed on a m1 u 1 + m 2 u 2
frictionless horizontal plane. Further a block of mass Sol. (a) v c = = 0.8 $i m/s
m1 + m 2
1 kg is placed over the plank. A massless spring of
(b) v 1 = − 1.6 $i m/s
natural length 2 m is fixed to the plank by its one end.
The other end of spring is compressed by the block by From CM, m1 u 1 + m 2 u 2 = m1 v 1 + m 2 v 2
half of spring’s natural length. They system is now ⇒ v 2 = 2.4 $i m/s
released from the rest. What is the velocity of the plank v − v1 4
(c) e = 2 =
when block leaves the plank ? u1 − u 2 7
u
NCERT Selected Questions
Q 1. A nucleus is at rest in the laboratory frame of Sol. Here, mass of gun, M = 100 kg
reference. Show that if it disintegrates into two Mass of shell, m = 0.02 kg
smaller nuclei the products must be emitted in Speed of shell, v = 80 ms−1
opposite directions. and let recoil speed of gun = V
Sol. Let m = mass of the nucleus at rest. As, initially both the gun and the shell are at rest before
firing, so the initial momentum (p i ) of the system = 0
u = its initial velocity = 0
Final momentum (p f ) of the system after firing
Also let m1 , m2 be the masses of the two smaller product
nuclei and v 1 , v 2 be their respective velocities. = MV + mv = 100 V + 0.02 × 80
If p i and p f be the initial and final momentum of the nucleus ∴ According to the law of conservation of linear momentum,
and the two nuclei respectively, then p i = p f or 0 = 100 V + 1.6
1.6
p i = mu = 0 ...(i) or V =− = 0.016 ms−1
100
and p f = m1v 1 + m2v 2 ...(ii)
∴ Recoil speed of gun = V = 0.016 ms−1 = 1.6 cms−1
Now, according to the law of conservation of linear
momentum, we know that Q 4. (a) The rate of change of total momentum of a
pi = p f many particle system is proportional to the
external force/sum of the internal forces on the
or 0 = m1v 1 + m2v 2
system.
or m2v 2 = − m1v 1 (b) In an inelastic collision of two bodies, the
mv quantities which do not change after the
or v2 = − 1 1 ...(iii)
m2 collision are the total kinetic energy/total linear
The negative sign in Eq. (iii) shows that v 1 and v 2 are in
momentum/total energy of the system of two
opposite directions, i.e. the two smaller nuclei are emitted in
bodies.
opposite directions. Sol. (a) Internal forces cannot change the total or net momentum
of a system. Hence, the rate of change of total
Q 2. Two billiard balls each of mass 0.05 kg moving in momentum of many particle system is proportional to
opposite directions with speed 6 ms −1 collide and the external force on the system.
rebound with the same speed. What is the impulse (b) In an inelastic collision of two bodies, the quantities
imparted to each ball due to the other? which do not change after the collision are total linear
momentum and total energy of the system of two bodies
Sol. ∴Initial momentum of each ball (if the system is isolated). The total KE of the system, is
p i = mv = (0.05) (6) not conserved as it may change to an equivalent amount
= 0.30 kg ms−1 of energy in some other form.

As after collision, the direction of velocity of each ball is Q 5. State if each of the following statements are true or
reversed on rebounding. false. Give reasons for your answer.
∴ Final momentum of each ball = m(− v ) (a) In an elastic collision of two bodies, the
p f = 0.05 (−6) momentum and energy of each body is
= −0.30 kg ms −1 conserved.
(b) In an inelastic collision, the final kinetic energy
∴ Impulse imparted of each ball is always less than the initial kinetic energy of
= p f − pi the system.
= change in momentum of each ball
Sol. (a) False. The total momentum and energy of the system are
= −0.30 − (0.30) conserved and not of each body.
= −0.60 kg ms−1 (b) True. Usually but not always because in an inelastic
or magnitude of impulse imparted by one ball due to collision, some kinetic energy usually changes into
collision with the other = 0.6 kg ms−1. The two impulses are some other forms of energy.
opposite in direction. Q 6. Answer carefully, with reasons.
(a) In an elastic collision of two billiard balls, is the
Q 3. A shell of mass 0.020 kg is fired by a gun of mass total kinetic energy conserved during the short
100 kg. If the muzzle speed of the shell is time of collision of the balls (i.e. when they are
8.0 ms −1 ,then what is the recoil speed of the gun? in contact)?
Centre of Mass, Conservation of Linear Momentum, Impulse and Collision 317

(b) Is the total linear momentum conserved during 1


Sol. Before collision, total KE of the system = mv 2
the short time of an elastic collision of two 2
balls? ...(i)
(c) What are the answers to (a) and (b) for an After collision, KE of the system
inelastic collision? 1  v
2
1
In case (i) KE = 2m   = mv 2 ...(ii)
Sol. (a) No, the total kinetic energy is not conserved during the 2  2 4
given elastic collision because a part of the kinetic 1
energy is used in deforming the balls in that short In case (ii) KE = mv 2 ...(iii)
2
interval for which they are in contact during collision 2
1  v 1
and gets converted into potential energy. In an elastic In case (iii) KE = (3m)   = mv 2 ...(iv)
collision, the KE before and after collision is same. 2  3 6
(b) Yes, the total linear momentum is conserved during the Thus, from the above equations we observe that the KE is
short time of an elastic collision of two balls. conserved only in case (ii), hence case (ii) is the only
(c) In an inelastic collision, total KE is not conserved during possible result after collision.
collision and even after collision. The total linear
momentum is however conserved during as well as after
Q 9. The bob A of a pendulum released from 30° to the
collision.
vertical hits another bob B of the same mass at rest
on a table as shown in figure given below. How high
Q 7. A molecule in a gas container hits a horizontal wall does the bob A rise after the collision? Neglect the
with speed 200 ms −1 and angle 30° with the normal size of the bobs and assume the collision to be
and rebounds with the same speed. Is momentum elastic.
conserved in the collision? Is the collision elastic or
inelastic?
30°
Sol. Yes, linear momentum is always conserved whether the A
collision is elastic or inelastic. Thus, in the given situation, m
momentum of system is conserved.
m
Wall
Normal B

30° Sol. In elastic head on collision, when two equal masses collide
30° each other, then they exchange their velocities.
In the present case, bob A is moving with certain speed and
bob B is at rest. Hence, after collision, bob A comes to rest
and the bob B starts moving with the speed of the bob A. The
As, the speed of the molecule remains the same before and bob A transfers whole of its momentum to ball B and hence
after the collision, so kinetic energy of the molecule is also bob A, will not rise at all after the collision.
conserved.
Hence, the collision is elastic. Q 10. A trolley of mass 300 kg carrying a sand bag of
25 kg is moving uniformly with a speed of 27 km/h
Q 8. Two identical ball bearings in contact with each on a frictionless track. After a while, sand starts
other and resting on a frictionless table are hit leaking out of a hole on the floor of trolley at the
head-on by another ball bearing of the same mass rate of 0.05 kg s −1 . What is the speed of the trolley
moving initially with a speed v. If the collision is after the entire sand bag is empty?
elastic, which of the following is a possible result
after collision? Sol. The trolley carrying a sand bag is moving with a speed of
1 2 3 1 2 3 27 kmh −1. It means that the system of trolley and sand bag is
not acted upon by any external force. If the sand leaks out of
(i) ⇒
v v=0 v/2 a hole on the floor of trolley, it does not give rise to any
external force on the trolley.
1 2 3
Therefore, speed of the trolley will not change when the
(ii) ⇒ sand is leaking out or even after the sand bag becomes
v=0 v
1 2 3 empty. This is because the weight of sand is leaks by taking
away the same momentum.
(iii) ⇒
v/3 ∴ Speed = 27 km/h
318 Objective Physics Vol. 1

Q 11. A bullet of mass 0.012 kg and horizontal speed As, no external force is applied on the system
−1
70 ms strikes a block of wood of mass 0.4 kg and ∴ pf = pi
instantly comes to rest with respect to the block. The or 200v′ + 20v′ − 80 = 2200
block is suspended from the ceiling by means of a or 220v′ = 2200 + 80 = 2280
thin wire. Calculate the height to which the block 2280
or v′ = = 10.36 ms−1
rises. Also, estimate the amount of heat produced in 220
the block. Time taken by the child to run a distance of 10 m over the
Sol. Let final speed of bullet and block after collision which trolley,
move together = v t=
10 m
= 2.5 s
According to the law of conservation of momentum, 4 ms−1
initial momentum of (bullet + block) before collision = final If x be the distance moved by the trolley in this time, then
momentum of bullet and block after collision x = v′ × t = 10.36 × 2.5
or 0.012 × 70 + 0.4 × 0 = (0.012 + 0.4) v = 25.9 m
0.012 × 70
or 0.012 × 70 = 0.412 v or v = Q 13. Give the location of the centre of mass of a (i) sphere
0.412
−1 (ii) cylinder (iii) ring and (iv) cube each of uniform
or v = 2.04 ms
mass density. Does the centre of mass of a body
Let h = height through which the block rises after collision. necessarily lie inside the body?
v 2 (2.04)2 Sol. (i) Centre of sphere
Then, h= =
2g 2 × 9.8 (ii) Mid- point of axis of symmetry of the cylinder, i.e. its
= 0.212 m = 21.2 cm geometrical centre.
(iii) Centre of ring.
Now, heat produced in the block = loss of KE
(iv) Point of intersection of diagonals, i.e. at its geometrical
= KE of bullet − KE of block and bullet
centre.
1 1
= × 0.012 × (70)2 − (0.412) × (2.04 2 ) No, in some cases, centre of mass may lie outside.
2 2
= 28.543 J Q 14. In the HCl molecule, the separation between the
nuclei of the two atoms is about
Q 12. A trolley of mass 200 kg moves with a uniform
speed of 36 km/h on frictionless track. A child of mass 1.27 Å (1 Å = 10 −10 m). Find the approximate
20 kg runs on the trolley from one end to the other location of the CM of the molecule, given that a
(10 m away) with a speed of 4 ms −1 relative to the chlorine atom is about 35.5 times as massive as a
trolley in a direction opposite to the trolley’s motion, hydrogen atom and nearly all the mass of an atom is
and jumps out of the trolley. What is the final speed of concentrated in its nucleus.
the trolley? How much has the trolley moved from the Y
time the child begins to run?
H CM Cl
Sol. Mass of the trolley, m1 = 200 kg X

5
Speed of the trolley, v = 36 kmh −1 = 36 × = 10 ms−1 x
18
Mass of the child, m2 = 20 kg
1.27Å
If pi be the initial momentum of the system before the child
starts running. Sol. Let CM be at a distance of x from H-atom.
Then, pi = (m1 + m2 ) v If x1 and x2 be the position vectors of the H and Cl atoms
= (200 + 20) × 10 w.r.t. the CM as origin, then
= 220 × 10 m x + m2x2
xCM = 0 = 1 1 ...(i)
= 2200 kgms−1 ...(i) m1 + m2
When the child starts running, with a velocity of 4 ms−1 in a or m1x1 + m2x2 = 0
direction opposite to trolley, then let v′ be the final speed of or m(− x ) + 35.5 m (1.27 − x ) = 0
the trolley (w.r.t. earth) or mx = 35.5 m ( 1.27 − x )
∴ Speed of the child relative to earth = v′ − 4 or x + 35.5x = 35.5 × 1.27
∴ Momentum of the system when the child is running, 35.5 × 1.27
∴ x= =1.235 Å ≈ 1.24 Å
pf = m1v′ + 20(v′ − 4 ) 36.5
= 200v′ + 20(v′ − 4 ) ...(ii) From H-atom and on the line joining H and Cl atoms.
Centre of Mass, Conservation of Linear Momentum, Impulse and Collision 319

Q 15. A child sits stationary at one end of a long trolley If A and a be their respective areas, then
2
moving uniformly with a speed v on a smooth  R πR 2
A = πR 2 and a=π   =
horizontal floor. If the child gets up and runs about  2 4
on the trolley in any manner, what is the speed of the Here O is the centre of the original disc.
CM of the (trolley + child) system? and O1 is the centre of the removed circular hole.
Also let O2 = centre of the remaining portion.
Sol. The speed of the centre of mass of the system and child
remains unchanged. It is because the state of system can ρ = mass per unit area of the disc.
change only under the effect of an external force and in this If m and m2 be the masses of the original disc and the
case no external force is acting. The forces involved in removed portion respectively, then
running on the trolley are from within the system, i.e. m = ρA = πR 2ρ
internal forces.
πR 2
Q 16. From a uniform disc of radius R, a circular section and m2 = ρa = ρ
4
of radius R /2 is cut out. The centre of the hole is at If m1 be the mass of remaining portion, then
R /2 from the centre of the original disc. Locate the πR 2ρ
centre of gravity of the resulting flat body. m1 = m − m2 = πR 2ρ −
4
3
= πR 2ρ
4
Let O be the origin.
R m x + m2x2
O
A Then, xCM = 1 1 ...(i)
O2 O1 m1 + m2
R/2
x 3 2   πR 2   R 
 πR ρ (− x ) +  ρ  
4   4   2
0=
3 2 πR 2 
 πR ρ + ρ
Sol. Here, radius of original disc = R 4 4 
R
Radius of removed circular section =
R ∴ x=
2 6
Objective Problems
[ Level 1 ]
Position of Centre of Mass
1. A uniform square plate ABCD has a mass of 10 kg. If two 5. Four particles of mass m1 = 2m, m2 = 4m, m3 = m and m4
points masses of 5 kg each are placed at the corners C and are placed at four corners of a square. What should be the
D as shown in the adjoining figure, then the centre of value of m4 so that the centre of mass of all the four
mass shifts to the mid-point of particles are exactly at the centre of the square?
A F B m4 m3

E O G

D C m1 m2
H
(a) OH (b) DH (c) OG (d) OF
(a) 2m (b) 8m
2. In carbon monoxide molecules, the carbon and the oxygen (c) 6m (d) None of these
atoms are separated by distance 1.2 × 10−10 m. The 6. Three rods of the same mass are placed as shown in the
distance of the centre of mass, from the carbon atom is figure. What will be the coordinate of centre of mass of
(a) 0.48 × 10−10 m (b) 0.51 × 10−10 m the system?
(c) 0.56 × 10−10 m (d) 0.69 × 10−10 m y

3. A square plate of side 20 cm has uniform thickness and (0, a)


density. A circular part of diameter 8 cm is cut out
symmetrically and shown in figure. The position of
centre of mass of the remaining portion is
O (a, 0) x
a a  a a
(a)  ,  (b)  , 
2 2  2 2
O O1 2a 2a  a a
(c) , (d)  , 
3 3  3 3

7. Centre of mass of three particles of masses 1 kg, 2 kg and


(a) at O1 3 kg lies at the point (1, 2, 3) and centre of mass of
(b) at O another system of particles of total mass 3 kg lies at the
(c) 0.54 cm from O on the left hand side point ( −1, 3, − 2). Where should we put a particle of mass
(d) None of the above
5 kg so that the centre of mass of the entire system lies at
4. A circular plate of diameter d is kept in contact with a the centre of mass of Ist system?
square plate of edge d as shown in figure. The density of (a) (0, 0, 0)
the material and the thickness are same everywhere. The (b) (1, 3, 2)
centre of mass of the composite system will be (c) (−1, 2, 3)
(d) None of the above

8. A uniform metal disc of radius R is taken and out of it a


disc of diameter R is cut off from the end. The centre of
mass of the remaining part will be
R R
a a (a) from the centre (b) from the centre
4 3
(a) inside the circular plate (b) inside the square plate R R
(c) from the centre (d) from the centre
(c) at the point of contact (d) outside the system 5 6
Centre of Mass, Conservation of Linear Momentum, Impulse and Collision 321

9. A uniform circular disc of radius a is taken. A circular 14. A circular ring of mass 6 kg and radius a is palced such
portion of radius b has been removed from it as shown in that its centre lies at the origin. Two particles of masses
the figure. If the centre of hole is at a distance c from the 2 kg each are placed at the intersecting points of the circle
centre of the disc, the distance x 2 of the centre of mass of with positive x-axis and positive y-axis. Then, the angle
the remaining part from the initial centre of mass O is made by the position vector of centre of mass of entire
given by system with x-axis is
(a) 45° (b) 60°
 4
(c) tan −1   (d) 30°
a  5
b
O2 O O1 x-axis 15. From a circular disc of radius R, a square is cut out with a
radius as its diagonal. The centre of mass of remainder is
x2 c at a distance (from the centre)
R R
(a) (b)
(4 π − 2) 2π
πb2 cb2 R R
(a) (b) (c) (d)
(a2 − c2 ) (a2 − b2 ) (π − 2) (2π − 2)
πc2 ca2
(c) (d)
(a − b2 )
2
(c − b2 )
2
Motion of Centre of Mass
10. Four rods AB, BC, CD and DA D C 16. Two particles A and B initially at rest, move towards each
have mass m, 2m, 3m and 4m other under a mutual force of attraction. At the instant
respectively. The centre of 1 2 when the speed of A is v and the speed of B is 2v, the
mass of all the four rods speed of centre of mass of the system is
(a) lie in region O (a) zero (b) v
(b) lie in region 2 3 (c) 1.5 v (d) 3v
4
(c) lie in region 3 17. Consider a system of two identical particles. One of the
(d) lie at O A B particles is at rest and the other has an acceleration a . The
11. All the particles of a body are situated at a distance R centre of mass has an acceleration
from the origin. The distance of centre of mass of the 1
(a) zero (b) a
body from the origin is 2
(a) = R (b) ≤ R (c) a (d) 2 a
(c) > R (d) ≥ R
18. Two blocks of masses 10 kg and 4 kg are connected by a
12. A uniform metal rod of length 1 m is bent at 90° so as to spring of negligible mass and placed on a frictionless
form two arms of equal length. The centre of mass of this horizontal surface. An impulse gives a velocity of 14 m/s
bent rod is to the heavier block in the direction of the lighter block.
 1 The velocity of the centre of mass is
(a) on the bisector of the angle,   m from vertex
 2 (a) 30 m/s (b) 20 m/s
(c) 10 m/s (d) 5 m/s
 1 
(b) on the bisector of angle,   m from vertex
 2 2 19. A body falling vertically downwards under gravity
 1 breaks in two parts of unequal masses. The centre of mass
(c) on the bisector of the angle,   m from vertex
 2 of the two parts taken together shifts horizontally towards
 1  (a) heavier piece
(d) on the bisector of the angle,   m from vertex
 4 2 (b) lighter piece
(c) does not shift horizontally
13. Three identical spheres, each of mass 1 kg are placed (d) depends on the vertical velocity at the time of breaking
touching each other with their centres on a straight line. 20. A ball kept in a closed box moves in the box making
Their centre are marked P, Q and R, respectively. The collisions with the walls. The box is kept on a smooth
distance of centre of mass of the system from P is surface. The velocity of the centre of mass
PQ + PR + QR PQ + PR (a) of the box remains constant
(a) (b)
3 3 (b) of the box plus the ball system remains constant
PQ + QR (c) of the ball remains constant
(c) (d) None of these
3 (d) of the ball relative to the box remains constant
322 Objective Physics Vol. 1

21. Two balls are thrown simultaneously in air. The 28. In a free space, a rifle of mass M shoots a bullet of mass m
acceleration of the centre of mass of the two balls while at a stationary block of mass M distance D away from it.
in air When the bullet has moved through a distance d towards
(a) depends on the direction of the motion of the balls the block, the centre of mass of the bullet-block system is
(b) depends on the masses of the two balls at a distance of
(c) depends on the speeds of two balls
(D − d ) m
(d) is equal to g (a) from the bullet
M +m
22. A man of mass M stands at one end of a plank of length L md + MD
(b) from the block
which lies at rest on a frictionless surface. The man walks M +m
to the other end of the plank. If the mass of the plank is 2md + MD
(c) from the block
M M +m
, the distance that the man moves relative to the
3 (D − d ) M
(d) from the bullet
ground is M +m
3L L 4L L
(a) (b) (c) (d)
4 4 5 3 29. Two particles P and Q initially at rest, move towards each
other under a mutual force of attraction. At the instant
23. In a gravity free space, a man of mass M standing at a when the speed of P is v and the speed of Q is 2v, the
height h above the floor, throws a ball of mass m straight speed of the centre of mass of the system is
down with a speed u. When the ball reaches the floor, the (a) 3v (b) 1.5 v
distance of the man above the floor will be (c) zero (d) data insufficient
 m  M
(a) h 1 +  (b) 1 +  h
 M  m 30. Blocks A and B are resting on a smooth horizontal surface
m given equal speeds of 2 m/s in opposite sense as shown in
(c) h (d) h the figure.
M

24. Two particles of equal mass have coordinates 2 m/s 2 m/s


( 2m, 4m, 6m ) and ( 6m, 2m, 8m ). Of these one particle has 4 kg 4 kg
a velocity v1 = ( 2 i$ ) m/s and another particle has velocity
Origin (4.5 m, 0)
v = ( 2 $j ) m/s at time t = 0. The coordinate of their centre
2
of mass at time t = 1s will be At t = 0, the position of blocks are shown, then the
(a) (4 m, 4 m, 7 m) (b) (5 m, 4 m, 7 m) coordinates of centre of mass t = 3 s will be
(c) (2 m, 4 m, 6 m) (d) (4 m, 5 m, 4 m) (a) (1, 0) (b) (3, 0)
(c) (5, 0) (d) (2.25, 0)
25. A ball falls freely from a height of 45 m. When the ball is
at a height of 25 m it explodes into two equal pieces. One 31. Two balls of equal mass are projected from a tower
of them moves horizontally with a speed of 10 ms −1 . The simultaneously with equal speeds. One at angle θ above
the horizontal and the other at the same angle θ below the
distance between the two pieces on the ground is
horizontal. The path of the centre of mass of the two
(a) 20 m (b) 30 m
(c) 40 m (d) 60 m balls is
(a) a vertical straight line
26. A metre stick is placed vertically at the origin on a (b) a horizontal straight line
frictionless surface. A gentle push in positive x direction (c) a straight line at an angle α (< θ ) with horizontal
is given to the top most point of the rod, when it has fallen (d) a parabola
completely x-coordinate of centre of rod is at 32. A cracker is thrown into air with a velocity of 10 m/s at an
(a) origin (b) − 0.5 m angle of 45° with the vertical. When it is at a height of
(c) −1m (d) +0.5 m (1/2) m from the ground, it explodes into a number of
27. Two blocks of masses 10 kg and 30 kg are placed along a pieces which follow different parabolic paths. What is the
vertical line. The first block is raised through a height of velocity of centre of mass, when it is at a height of 1 m
7 cm. By what distance should the second mass be moved from the ground? (g = 10 m/s 2 )
to raise the centre of mass by 1 cm? (a) 4 5 ms−1
(a) 2 cm upward (b) 2 5 ms−1
(b) 1 cm upward
(c) 5 4 ms−1
(c) 2 cm downward
(d) 1 cm downward (d) 5 ms−1
Centre of Mass, Conservation of Linear Momentum, Impulse and Collision 323

33. Two bodies having masses m1 and m2 and velocities u 1 39. A stationary bomb explodes into two parts of masses 3 kg
and u 2 collide and form a composite system. If and 1 kg. The total KE of the two parts after explosion is
m1 u 1 + m2 u 2 = 0 ( m1 ≠ m2 ), then velocity of composite 2400 J. The KE of the smaller part is
system will be (a) 600 J (b) 1800 J
(a) u 1 − u 2 (b) u 1 + u 2 (c) 1200 J (d) 2160 J
u + u2
(c) 1 (d) zero 40. When a ball of mass 5 kg hits a bat with a velocity 3 m/s,
2 in positive direction and it moves back with a velocity
34. An isolated particle of mass m is moving in horizontal 4 m/s, find the impulse in SI units.
plane (x-y), along the x-axis, at a certain height above the (a) 5 (b) 15
ground, it suddenly explodes into two fragments of (c) 25 (d) 35
m 3m
masses and . An instant later, the smaller fragment 41. One projectile moving with velocity v in space, gets burst
4 4 into 2 parts of masses in the ratio 1 : 3. The smaller part
is at y = + 15 cm. The larger fragment at this instant is at becomes stationary. What is the velocity of the other
(a) y = − 5 cm (b) y = + 20 cm
part?
(c) y = + 5 cm (d) y = − 20 cm 4v 3v
(a) 4v b) v (c) (d)
35. A man of mass m is standing on a plank of equal mass m 3 4
resting on a smooth horizontal surface. The man starts
42. A bomb of mass 9 kg explodes into two pieces of mass
moving on the plank with speed u relative to the plank.
3 kg and 6 kg. The velocity of 3 kg mass is 16 m/s. The
m
velocity of 6 kg mass is
(a) 4 m/s (b) 8 m/s (c) 16 m/s (d) 32 m/s

43. A body at rest breaks into two pieces of equal masses.


The parts will move
The speed of the man relative to the ground is (a) in same direction
u (b) along different lines
(a) 2u (b) (c) in opposite directions with equal speeds
2
u (d) in opposite directions with unequal speeds
(c) zero (d)
4 44. The magnitude of the impulse developed by a mass of
36. A body of mass a moving with a velocity b strikes a body 0.2 kg which changes its velocity from 5$i − 3$j + 7k$ m/s
of mass c and gets embedded into it. The velocity of the to 2$i + 3$j + k$ m/s is
system after collision is (a) 2.7 N-s (b) 1.8 N-s (c) 0.9 N-s (d) 3.6 N-s
a+ c ab a a
(a) (b) (c) (d) 45. Gravels are dropped on a conveyor belt at the rate of
ab a+ c b+ c a+ b
0.5 kg/s. The extra force required in newtons to keep the
37. A man of mass m moves with a constant speed on a plank belt moving at 2 m/s is
of mass M and length l kept initially at rest on a (a) 1 (b) 2 (c) 4 (d) 0.5
frictionless horizontal surface, from one end to the other 46. A particle of mass 15 kg has an initial velocity
in time t. The speed of the plank relative to ground while vi = i$ − 2$j m/s. It collides with another body and the
man is moving, is impact time is 0.1 s, resulting in a velocity
l M l m  v f = 6i$ + 4$j + 5k$ m/s after impact. The average
(a)   (b)   force of
t  m t m + M
impact on the particle is
l M  (a) 15 | 5$i + 6$j + 5k$ | (b) 15 | 5$i + 6$j − 5k$ |
(c)   (d) None of these
t  M + m
(c) 150 | 5$i − 6$j + 5k$ | (d) 150 | 5$i + 6$j + 5k$ |

Conservation Laws, Impulse and 47. Two trains A and B are running in the same direction on
Variable Mass System parallel rails such that A is faster than B. Packets of equal
weight are transferred between them. What will happen
38. A bullet of mass 5 g is fired at a velocity of 900 ms −1 due to this
from a rifle of mass 2.5 kg. What is the recoil velocity of (a) A will be accelerated but B will be retarded
the rifle? (b) B will be accelerated but A will be retarded
(a) 0.9 ms−1 (b) 180 ms−1 (c) there will be no change in A but B will be accelerated
(c) 900 ms−1 (d) 1.8 ms−1 (d) there will be no change in B but A will be accelerated
324 Objective Physics Vol. 1

48. A shell of mass m is moving horizontally with velocity v 0 54. A mass of 100 g strikes the wall
and collides with the wedge of mass M just above point A, with speed 5 m/s at an angle as
60°
as shown in the figure. As a consequence, wedge starts to shown in figure and it rebounds
move towards left and the shell returns with a velocity in with the same speed. If the
x-y plane. The principle of conservation of momentum can contact time is 2 × 10−3 s, what 60°

be applied for is the force applied on the mass


C by the wall
100 g
(a) 250 3 N to right
(b) 250 N to right
M (c) 250 3 N to left
v0 m (d) 250 N to left
θ
B A 55. A cannon ball is fired with a velocity 200 m/s at an angle
(a) system (m + M ) along any direction of 60° with the horizontal. At the highest point of its
(b) system (m + M ) along vertical flight, it explodes into 3 equal fragments, one going
(c) system (m + M ) horizontally vertically upwards with a velocity 100 m/s, the second
(d) None of the above one falling vertically downwards with a velocity 100 m/s.
49. A shell is fired from cannon with a velocity v at an angle θ The third fragment will be moving with a velocity
(a) 100 m/s in the horizontal direction
with the horizontal direction. At the highest point in its
(b) 300 m/s in the horizontal direction
path it explodes into two pieces of equal mass. One of the
(c) 300 m/s in a direction making an angle of 60° with the
pieces retraces its path to the cannon and the speed horizontal
(in ms −1 ) of the other piece immediately after the (d) 200 m/s in a direction making an angle of 60° with the
explosion is horizontal
3 3 56. A particle of mass m moving with velocity u makes an
(a) 3v cos θ (b) 2v cos θ (c) v cos θ (d) v cos θ
2 2 elastic one dimensional collision with a stationary particle
of mass m. They are in contact for a brief time T. Their force
50. In a two block system an initial velocity v 0 with respect to T
ground is given to block A of interaction increases from zero to F0 linearly in time
2
A v0 Rough T
and decreases linearly to zero in further time . The
B 2
Smooth magnitude of F0 is
mu 2mu
(a) the momentum of block A is not conserved (a) (b)
(b) the momentum of system of blocks A and B is conserved T T
mu
(c) the increase in momentum of B is equal to the decrease in (c) (d) None of these
momentum of block A 2T
(d) All of the above 57. A ball of mass m moving with velocity v 0 collides a wall
51. If the net external forces acting on the system of particles as shown in figure. After impact it rebounds with a
is zero, then which of the following may vary? 3
velocity v 0 . The impulse acting on ball during impact is
(a) Momentum of the system (b) Velocity of centre of mass 4
(c) Position of centre of mass (d) None of these
m
52. For a rocket moving in free space, the fraction of mass to v0
be disposed off to attain a speed equal to two times the 37° X
exhaust speed is given by (given e 2 = 7.4 ) 53°
(a) 0.63 (b) 0.37
(c) 0.50 (d) 0.86 Y
3v
53. An object of mass 3 m splits into three equal fragments. m 4 0
Two fragments have velocities v $j and v$i. The velocity of
m $ 3
the third fragment is (a) − v0 j (b) − mv0 $i
2 4
v(i$ + $j) 5
(a) v($j − $i ) (b) v($i − $j) (c) − v($i + $j) (d) (c) − mv0 $i (d) None of these
2 4
Centre of Mass, Conservation of Linear Momentum, Impulse and Collision 325

58. A unidirectional force F varying with time t as shown in 64. A rocket of mass 6000 kg is fired from earth with exhaust
the figure acts on a body initially at rest for a short speed of gases 1 km/s. What should be the rate of ejection
duration 2T. Then, the velocity acquired by the body is of gas, so that it moves with initial acceleration of
F 20 m/s 2 ? (take g = 10 m/s 2 )
F0
(a) 60 kg/s (b) 180 kg/s
(c) 200 kg/s (d) 120 kg/s
T 2T t
65. A rocket with a lift-off mass 3.5 × 104 kg is blasted
– F0 upwards with an initial net acceleration of 10 m/s 2 . Then,
the initial thrust of the blast is
πF0T πF0T (a) 3.5 × 105 N (b) 7.0 × 105 N
(a) (b)
4m 2m (c) 14.0 × 105 N (d) 17.5 × 105 N
F0T
(c) (d) zero
4m
Collision
59. In the figure given the position-time graph of a particle of
66. In an elastic collision
mass 0.1 kg is shown in the given figure. The impulse at
(a) both momentum and KE are conserved
t = 2 s is (b) only momentum is conserved
(c) only KE is conserved
6 (d) Neither KE nor momentum is conserved
4
x(m)
2
67. A bullet of mass m and velocity v is fired into a block of
t (s) mass M and sticks to it. The final velocity of the system
2 4 6 equals
M m
(a) 0.2 kg-ms−1 (b) −0.2 kg-ms−1 (a) ⋅v (b) ⋅v
m+ M m+ M
(c) 0.1 kg-ms−1 (d) −0.4 ms−1 m+ M
(c) ⋅v (d) None of these
m
60. A player takes 0.1 s in catching a ball of mass 150 g
moving with velocity of 20 m/s. The force imparted by 68. Two perfectly elastic particles A and B of equal mass
the ball on the hands of the player is travelling along the line joining them with velocities
(a) 0.3 N (b) 3 N (c) 30 N (d) 300 N
15 m/s and 10 m/s. After collision, their velocities will be
61. If two balls, each of mass 0.06 kg, moving in opposite (a) 10 m/s, 10 m/s
directions with speed 4 m/s collide and rebound with the (b) 15 m/s, 15 m/s
same speed, then the impulse imparted to each ball due to (c) 10 m/s, 15 m/s
the other is (d) 15 m/s, 10 m/s
(a) 0.92 kg-m/s (b) 0.24 kg-m/s 69. A body of mass m moving with velocity v collides head
(c) 0.48 kg-m/s (d) 0.52 kg-m/s
on with another body of mass 2m which is initially at rest.
62. A batsman hits a 150 g ball moving horizontally at 20 m/s The ratio of KE of colliding body before and after
back to bowler at 12 m/s. If the contact of bat and ball collision will be
lasted for 0.04 s, then the average force exerted by the bat A B
on the ball is (i) 0 25
(a) 30 N (b) 120 N (ii) 5 20
(iii) 10 15
(c) 150 N (d) 120 × 103 N
(iv) 20 5
63. An open railroad car of mass M is moving with initial (a) 1 : 1 (b) 2 : 1
(c) 4 : 1 (d) 9 : 1
velocity v 0 on a straight horizontal frictionless track. It
suddenly starts raining at time t = 0. The raindrops fall 70. A block of mass m moving at a velocity v collides with
vertically with velocity v and add a mass of µ kg/s of another block of mass 2m at rest. The lighter block comes
water. Determine the velocity v of car after t seconds to rest after collision. Find the coefficient of restitution.
Mv0 µ t v0 1
(a) (b) (a) (b) 1
M − µt M + µt 2
Mv0 1 1
(c) (d) None of these (c) (d)
M + µt 3 4
326 Objective Physics Vol. 1

71. A steel ball strikes a steel plate at an angle θ with the 78. A sphere of mass m moving with a constant velocity u
vertical. If the coefficient of restitution is e, the angle at hits another stationary sphere of same mass. If e is the
which the rebound will take place is coefficient of restitution, then ratio of velocities of the
 tan θ  v
(a) θ (b) tan −1 two spheres 1 after collision will be
 e  v2
 e  u
(c) e tan θ (d) tan −1  tan θ  2 1
 
1− e 1+ e e+1 e−1
72. Two ice skaters A and B approach each other at right (a) (b) (c) (d)
1+ e 1− e e−1 e+1
angles. Skater A has a mass 30 kg and velocity 1 m/s
skater B has a mass 20 kg and velocity 2 m/s. They meet 79. A smooth sphere of mass M moving with velocity u
and cling together. Their final velocity of the couple is directly collides elastically with another sphere of mass m
(a) 2 m/s (b) 1.5 m/s (c) 1 m/s (d) 2.5 m/s at rest. After collision their final velocities are V and v
73. The collision of two balls of equal mass takes place at the respectively. The value of v is
origin of coordinates. Before collision, the components 2uM 2um 2u 2u
(a) (b) (c) (d)
of velocities are (v x = 50 cm/s, v y = 0) and m M m M
1+ 1+
(v x = − 40cm/s and v y = 30cm/s). The first ball comes to M m
rest after collision. The velocity components (v x and v y 80. A metal ball falls from a height of 32 m on a steel plate. If
respectively) of the second ball are the coefficient of restitution is 0.5 to what height will the
(a) 10 and 30 cm/s (b) 30 and 10 cm/s ball rise after second bounce?
(c) 5 and 15 cm/s (d) 15 and 5 cm/s (a) 2 m (b) 4 m
74. If a body of mass m collides head on, elastically with (c) 8 m (d) 16 m
velocity u with another identical body at rest. After 81. A body of mass m1 moving with a velocity 3 ms −1 collides
collision velocity of the second body will be
with another body at rest of mass m2 . After collision the
(a) zero (b) u
velocities of the two bodies are 2 ms −1 and 5 ms −1
(c) 2u (d) data insufficient
respectively along the direction of motion of m2 . The ratio
75. Two balls of equal mass have a head on collision with m1
speed 4 m/s each travelling in opposite directions. If the is
m2
coefficient of restitution is 1/2, the speed of each ball after
5 1 12
impact will be (a) (b) 5 (c) (d)
12 5 5
(a) 1 m/s (b) 2 m/s
(c) 3 m/s (d) data insufficient 82. The bob A of a simple pendulum is released when the
76. A ball hits the floor and rebounds after an inelastic string makes an angle of 45° with the vertical. It hits
collisions. In this case another bob B of the same material and same mass kept at
(a) the momentum of the ball just after the collision is the same rest on a table. If the collision is elastic, then
as that just before the collision O
(b) the mechanical energy of the ball remains the same in the
collision 45°
(c) the total momentum of the ball and the earth is conserved
(d) the total energy of the ball and the earth is conserved A
B
77. Two vehicles of equal masses are moving with same
speed v on two roads inclined at an angle θ. They collide (a) both A and B rise to the same height
inelastically at the junction and, then move together. The (b) both A and B come to rest at B
speed of the combination is (c) both A and B move with the same velocity of A
(d) A comes to rest and B moves with the velocity of A
m
83. A body of mass M 1 collides elastically with another body
θ v
of mass M 2 at rest. There is maximum transfer of energy
θ v
when
m (a) M1 > M2
(b) M1 < M2
3 v θ (c) M1 = M2
(a) v cosθ (b) 2v cos θ (c) v cos θ (d) cos
2 2 2 (d) same for all values of M 1 and M 2
Centre of Mass, Conservation of Linear Momentum, Impulse and Collision 327

84. A mass m moves with a velocity v and collides Miscellaneous Problems


inelastically with another identical mass. After collision,
v 88. A mass m with velocity u strikes a wall normally and
the 1st mass moves with velocity in a direction returns with the same speed. What is magnitude of the
3
change in momentum of the body when it returns
perpendicular to the initial direction of motion. Find the (a) 4 mu (b) mu (c) 2 mu (d) zero
speed of the 2nd mass after collision
89. Consider the following two statements.
A. Linear momentum of the system remains constant.
v
3 B. Centre of mass of the system remains at rest.
(a) A implies B and B implies A
(b) A does not imply B and B does not imply A
At rest
(c) A implies B but B does not imply A
m m (d) B implies A but A does not imply B
Before collision After collision
90. Consider the following two statements.
2 v A. Linear momentum of a system of particles is zero.
(a) v (b)
3 3 B. Kinetic energy of a system of particles is zero.
(c) v (d) 3v (a) A implies B and B implies A
(b) A does not imply B and B does not imply A
85. Two identical balls marked 2 and 3, in contact with each (c) A implies B but B does not imply A
other and at rest on a horizontal frictionless table are hit (d) B implies A but A does not imply B
head on by another identical ball marked 1 moving
91. A particle of mass m has momentum p. Its kinetic energy
initially with a speed v as shown in figure. Assume the
collision is elastic, then mark out the correct statement. will be
p2 p2
v (a) mp (b) p2m (c) (d)
1 2 3 m 2m
v 92. A machine gun fires a steady stream of bullets at the rate
(a) Ball 2 comes to rest and ball 2 and 3 move with speed
2 of n per minute into a stationary target in which the
each bullets get embedded. If each bullet has a mass m and
(b) Ball 1 and 2 come to rest and ball 3 moves with speed v arrives at the target with a velocity v, the average force on
v
(c) Each ball moves with speed the target is
3 60 v mn v mv
(d) None of the above (a) 60 mnv (b) (c) (d)
mn 60 60n
86. Two balls of equal masses have a head on collision with
93. 10000 small balls, each weighing 1 g, strikes one square
speed 6 m/s each. If the coefficient of restitution is 1/3, cm of area per second with a velocity 100 m/s in a normal
the speed of each ball after impact will be direction and rebound with the same velocity. The value
(a) 18 m/s of pressure on the surface will be
(b) 2 m/s
(a) 2 × 103 N/m 2 (b) 2 × 105 N/m 2
(c) 6 m/s
(d) data insufficient (c) 107 N/m 2 (d) 2 × 107 N/m 2

87. The two diagrams show the situations before and after a 94. A particle is projected from a point at an angle with the
collision between two spheres A and B of equal radii horizontal. At any instant t, if p is the linear momentum
moving along the same straight line on a smooth and E the kinetic energy, then which of the following
graph is/are correct?
horizontal surface. The coefficient of restitution e is E

Before collision After collision


A B (a) (b) E
A B

O p O p

8 m/s 2 m/s 2 m/s 5 m/s

1 1 (c) E (d) E
(a) (b)
3 2
2 3 O p2
O p2
(c) (d)
3 4
328 Objective Physics Vol. 1

95. A 2 kg block of wood rests on a long table top. A 5 g with a speed v along the line joining A and B collides with
bullet moving horizontally with a speed of 150 m/s is shot A, the maximum compression in the spring is
into the block and sticks to it. The block then slides 2.7 m m v
(a) v (b) m
along the table top and comes to a stop. The force of 2k 2k
friction between the block and the table is mv mv
(c) (d)
(a) 0.052 N (b) 3.63 N (c) 2.50 N (d) 1.04 N k 2k
96. A projectile of mass m is fired with velocity v from a 102.A bullet moving with a speed of 100 ms −1 can just
point P, as shown in the figure. Neglecting air resistance, penetrate two planks of equal thickness. Then, the
the magnitude of the change in momentum between the number of such planks penetrated by the same bullet
points P and arriving at Q is when the speed is doubled will be
v
(a) 4 (b) 8
(c) 6 (d) 10

103.A ball is projected vertically down with an initial velocity


45° from a height of 20 m onto a horizontal floor. During the
P Q impact it loses 50% of its energy and rebounds to the
mv same height. The initial velocity of its projection is
(a) zero (b) (c) mv 2 (d) 2mv
2 (a) 20 ms−1 (b) 15 ms−1 (c) 10 ms−1 (d) 5 ms−1

97. If the KE of a body is increased by 300%, its momentum 104.A block C of mass m is moving with velocity v 0
will increase by and collides elastically with block A of mass m and
(a) 100% (b) 150% (c) 300% (d) 175% connected to another block B of mass 2 m through spring
of spring constant k. What is k if x 0 is compression of
98. If the linear momentum is increased by 50%, the kinetic spring when velocity of A and B is same?
energy will increase by
v0
(a) 50% (b) 100% (c) 125% (d) 25% C A B

99. If the kinetic energy of a body increases by 0.1%, the mv02 mv02 3mv02 2mv02
percent increase of its momentum will be (a) (b) (c) (d)
x02 2x02 2x02 3x02
(a) 0.05% (b) 0.1% (c) 1.0% (d) 10%

100.A particle of mass m at rest is acted upon by a constant 105.A bullet of mass m is fired into a block of wood of mass
force F for a time t. Its kinetic energy after an interval t is M which hangs on the end of pendulum and gets
F 2t 2 F 2t 2 F 2t 2 Ft embedded into it. When the bullet strikes the wooden
(a) (b) (c) (d) block, the pendulum starts to swing with maximum rise
m 2m 3m 2m
R. Then, the velocity of the bullet is given by
101.Two identical blocks A and B, each of mass m resting on M M +m
(a) 2gR (b) 2gR
smooth floor are connected by a light spring of natural m+ M m
length L and spring constant K with the spring at its M
(c) 2gR (d) None of these
natural length. A third identical block C (mass m) moving m

[ Level 2 ]
Only One Correct Options When the compression in spring is maximum the velocity
of block C with respect to block A is (Neglect the friction
1. A block A of mass M moving with speed u collides
elastically with block B of mass m which is connected to everywhere)
 M 
block C of mass m with a spring. (a) zero (b)   u
u
 M + m
A B C
 m   m
M m m (c)   u (d)   u
 M + m M
Centre of Mass, Conservation of Linear Momentum, Impulse and Collision 329

2. A particle A of mass m initially at rest slides down a 7. n elastic balls are placed at rest on a smooth horizontal
height of 1.25 m on a frictionless ramp, collides with and plane which is circular at the ends with radius r as shown
sticks to an identical particle B of mass m at rest as shown m
in the figure. The masses of the balls are m, ,
in the figure. 2
m m m
A , K , n − 1 respectively. What is the minimum
22 2
1.25 m velocity which should be imparted to the first ball of mass
B C
m such that this nth ball will complete the vertical circle?
m 2m

Then, particles A and B together collide elastically with


r
particle C of mass 2 m at rest. The speed of particle C
after the collision with combined body ( A + B ) would be m
(g = 10 m/s 2 ) n− 1 n− 1
 3  4
(a)   5gr (b)   5gr
(a) 2.0 m/s (b) 1.25 m/s  4  3
(c) 2.5 m/s (d) 5 m/s n− 1 n− 1
 3  2
(c)   5gr (d)   5gr
3. A bullet is fired from a gun. The force on the bullet is  2  3
given by F = 600 − 2 × 105 t
8. A particle of mass m moving with speed v hits elastically
where, F is in newton and t in second. The force on the another stationary particle of mass 2 m inside a smooth
bullet becomes zero as soon as it leaves the barrel. What horizontal circular tube of radius r. The time after which
is the average impulse imparted to the bullet? the second collision will take place is
(a) 8 N-s (b) Zero (c) 0.9 N-s (d) 1.8 N-s 2πr 4πr
(a) (b)
4. At high altitude, a body explodes at rest into two equal v v
fragments with one fragment receiving horizontal 3πr πr
(c) (d)
velocity of 10 m/s. Time taken by the velocity vectors of 2v v
the fragments to make 90° is (g = 10 m/s 2 ) 9. Two blocks of mass m and 2m are kept on a smooth
(a) 0.5 s (b) 4 s (c) 2 s (d) 1 s horizontal surface. They are connected by an ideal spring
of force constant k. Initially the spring is unstretched. A
5. A train of mass M is moving on a circular track of radius
constant force is applied to the heavier block in the
R with constant speed v. The length of the train is half of
direction shown in figure. Suppose at time t displacement
the perimeter of the track. The linear momentum of the
of smaller block is x, then displacement of the heavier
train will be
block at this moment would be
2Mv
(a) πMv (b) F
π m 2m
πMv
(c) (d) Mv
2

6. A pendulum consists of a wooden bob of mass m and x Ft 2 x


(a) (b) +
length l. A bullet of mass m1 is fired towards the 2 6m 3
pendulum with a speed v1 and it emerges from the bob x Ft 2 x
(c) (d) −
v 3 4m 2
with speed 1 . The bob just completes motion along a
3 10. A bullet of mass 20 g and moving with 600 m/s collides
vertical circle. Then, v1 is with a block of mass 4 kg hanging with the string of
length 0.4 m. What is velocity of bullet when it comes out
of block, if block rises to height 0.2 m after collision?
O (a) 200 m/s (b) 150 m/s
(c) 400 m/s (d) 300 m/s

m1 v1 m m1 v1/3 11. A particle of mass 1 kg is thrown vertically upward with


speed 100 m/s. After 5 s, it explodes into two parts. One
m 3m part of mass 400 g comes back with speed 25 m/s, what is
(a) 5gl (b) 5gl
m1 2m1 the velocity of other part just after explosion?
2  m m  (a) 100 m/s upward (b) 600 m/s upward
(c)   5gl (d)  1  gl
3  m1   m (c) 100 m/s downward (d) 300 m/s upward
330 Objective Physics Vol. 1

12. A ball falling freely from a height of 4.9 m/s, hits a 19. A particle of mass m moving with a velocity ( 3i$ + 2$j ) m/s
3
horizontal surface. If e = , then the ball will hit the collides with a stationary body of mass M and finally
4 m 1
moves with a velocity ( −2i$ + $j ) m/s. If = , then
surface, second time after M 13
(a) 1.0 s (b) 1.5 s (a) the impulse is ± m (5i$ + $j)
(c) 2.0 s (d) 3.0 s
1 $ $
(b) the velocity of the M is (5i + j)
13. A disc of mass 10 g is kept floating horizontally by 13
throwing 10 marbles per second against it from below. If (c) Both (a) and (b) are wrong
the mass of each marble is 5 g. What will be velocity with (d) Both (a) and (b) are correct
which the marble are striking the disc? Assume that the 20. A small ball rolls off the top landing of the staircase. It
marble strikes the disc normally and rebound downwards strikes the mid-point of the first step and then the
with the same speed. mid-point of the second step. The steps are smooth and
(a) 2.98 m/s (b) 0.98 m/s identical in height and width. The coefficient of
(c) 0.49 m/s (d) 1.96 m/s
restitution between the ball and the first step is
14. If a man of mass M jumps to the ground from a height h 3 1 1
(a) 1 (b) (c) (d)
and it moves a small distance x inside the ground, the 4 2 4
average force acting on him from ground is 21. You are supplied with three identical rods of same
Mgh Mgx
(a) (b) length and mass. If the length of each rod is 2π. Two of
x h
2 them are converted into rings and then placed over the
 h
(c) Mg   (d) None of these third rod as shown in figure. If point A is considered as
 x
origin of the coordinate system, the coordinate of the
15. A machine gun fires a bullet of mass 40 g with a velocity centre of mass will be (you may assume AB as x-axis of
the coordinate system)
1200 ms −1 . The man holding it, can exert a maximum
force of 144 N on the gun. How many bullets can be fired
per second at the most?
(a) One (b) Four
(c) Two (d) Three A B
π 1 π 2  1  2
(a)  ,  (b)  ,  (c)  π ,  (d)  π , 
16. A straight rod of length L has one of its ends at the origin 2 3 2 3  3  3
and the other at x = L. If the mass per unit length of the
rod is given by Ax, where A is constant, where is its mass 22. A mass of 10 g moving horizontally with a velocity of
centre? 100 cm/s strikes a pendulum bob of mass 10 g. Length of
L L 2L 3L string is 50 cm. The two masses stick together. The
(a) (b) (c) (d)
3 2 3 4 maximum height reached by the system now is
17. In a one dimensional collision between two identical (g = 10 m/s 2 )
particle A and B. B is stationary and A has momentum p
before impact. During impact B gives an impulse J to A.
Then, coefficient of restitution between the two is
2J 2J
(a) −1 (b) +1
p p
J J (a) 7.5 cm (b) 5 cm
(c) +1 (d) −1
p p (c) 2.5 cm (d) 1.25 cm

18. Three identical blocks A, B and C are placed on 23. Both the blocks as shown in the given arrangement are
horizontal frictionless surface. The blocks B and C are at given together a horizontal velocity towards right. If aCM
rest. But A is approaching towards B with a speed 10 m/s. be the subsequent acceleration of the centre of mass of
A B C the system of blocks, then aCM equals
1kg µ = 0.1
2kg µ = 0.2
The coefficient of restitution for all collision is 0.5. The
5 7
speed of the block C just after collision is approximately (a) zero (b) m/s2 (c) m/s2 (d) 2 m/s2
(a) 5.6 m/s (b) 6.4 m/s (c) 3.2 m/s (d) 4.6 m/s 3 3
Centre of Mass, Conservation of Linear Momentum, Impulse and Collision 331

y
24. An object comprises of a uniform 3. A particle of mass m strikes a horizontal smooth floor
ring of radius R and its uniform B with a velocity u making an angle θ with the floor and
chord AB (not necessarily made of rebound with velocity v making an angle φ with the floor.
the same material) as shown in The coefficient of restitution between the particle and the
x floor is e. Then,
figure. Which of the following can A
not be the centre of mass of the (a) the impulse delivered by the floor to the body is
mu (1 + e) sin θ
object?
(b) tan φ = e tan θ
R R  R R
(a)  ,  (b)  ,  (c) v = u 1 − (1 − e2 ) sin 2 θ
3 3  2 2
 R R (d) the ratio of the final kinetic energy to the initial kinetic
(c)  ,  (d) None of these energy is cos2 θ + e2 sin 2 θ
 4 4
4. A particle of mass m moving with a velocity
25. A girl throws a ball with initial velocity v at an inclination
( 3i$ + 2$j ) ms −1 collides with another body of mass M and
of 45°. The ball strikes the smooth vertical wall at a
horizontal distance d from the girl and after rebounding finally moves with velocity ( −2 ^i + ^j ) ms −1 . Then, during
returns to her hand. What is the coefficient of restitution the collision
between wall and the ball? (a) impulse received by m is m (5i$ + $j)
gd
(a) v − gd
2
(b) 2 (b) impulse received by m is m (−5i$ − $j)
v − gd
(c) impulse received M is m (−5i$ − $j)
gd v2
(c) (d) (d) impulse received by M is m (5i$ + $j)
v2 gd
5. All surfaces shown in figure are smooth. System is
More than One Correct Option released from rest. x and y components of acceleration of
1. A particle of mass m, moving with velocity v collides a COM are
stationary particle of mass 2m. As a result of collision, the m1 x
particle of mass m deviates by 45° and has final speed of
v
. For this situation, mark out the correct statement (s).
2 y
(a) The angle of divergence between particles after
π m2
collision is
2
(b) The angle of divergence between particles after m1m2 g
π (a) (acm )x = (b) (acm )x =
m1m2 g
collision is less than m1 + m2
2 (m1 + m2 )2
2
(c) Collision is elastic  m2   m2 
(d) Collision is inelastic (c) (acm ) y =   g (d) (acm ) y =   g
 m1 + m2   m1 + m2 
2. A pendulum bob of mass m m l
connected to the end of an ideal 6. A block of mass m is placed at rest on a smooth wedge of
string of length l is released from mass M placed at rest on a smooth horizontal surface.As
rest from horizontal position as the system is released
shown in the figure. At the lowest
m
point, the bob makes an elastic 5m
collision with a stationary block of
mass 5m, which is kept on a frictionless surface. Mark out
the correct statement(s) for the instant just after the M
impact.
17
(a) Tension in the string is mg
9
(b) Tension in the string is 3 mg (a) the COM of the system remains stationary
2gl (b) the COM of the system has an acceleration g vertically
(c) The velocity of the block is downward
3
(d) The maximum height attained by the pendulum bob after (c) momentum of the system is conserved along the horizontal
4l direction
impact is (measured from the lowest position) (d) acceleration of COM is vertically downward (a < g )
9
332 Objective Physics Vol. 1

7. In the figure shown, coefficient of restitution between They meet at a point P on the rod and the spider eats the
1 v
A and B is e = , then moth. After this the spider moves with a velocity
2 2
relative to the rod towards the end A. The spider takes
A B
v
negligible time in eating on the other insect. Also, let
m m
L
v = , where T is a constant having value 4 s.
Smooth T
v 3. Displacement of the rod by the time the insect meet the
(a) velocity of B after collision is
2 moth is
3 L 3L
(b) impulse between two during collision is mv (a) (b) L (c) (d) zero
4 2 4
3
(c) loss of kinetic energy during the collision is mv 2 4. The point P is at
16
1 (a) the centre of the rod
(d) loss of kinetic energy during the collision is mv 2 (b) the edge of the table supporting the end B
4
(c) the edge of the table supporting end A
8. In case of rocket propulsion, choose the correct options. (d) None of the above
(a) Momentum of system always remains constant
5. The speed of the rod after the spider eats up the moth and
(b) Newton’s third law is applied
(c) If exhaust velocity and rate of burning of mass is kept moves towards A is
constant, then acceleration of rocket will go on increasing v
(a) (b) v
(d) Newton’s second law can be applied 2
v
(c) (d) 2v
Comprehension Based Questions 6
Passage I (Q. 1 to 2) 6. After starting from end B of the rod the spider reaches the
A block of mass 2 kg is attached with a spring of spring end A at a time
constant 4000 Nm−1 and the system is kept on smooth (a) 40 s (b) 30 s (c) 80 s (d) 10 s
horizontal table. The other end of the spring is attached
7. By what distance the centre of mass of the rod shifts
with a wall. Initially spring is stretched by 5 cm from its
during this time?
natural position and the block is at rest. Now, suddenly 8L 4L
an impulse of 4 kg-ms −1 is given to the block towards the (a)
3
(b)
3
wall. L
(c) L (d)
1. Find the velocity of the block when spring acquires its 3
natural length.
(a) 5 ms−1 (b) 3 ms−1 Assertion and Reason
(c) 6 ms−1 (d) None of these Directions (Q. Nos. 1-23) These questions consist of two
2. Approximate distance travelled by the block when it statements each printed as Assertion and Reason. While
comes to rest for a second time (not including the initial answering these questions you are required to choose any one
one) will be (Take 45 = 6.70) of the following five responses.
(a) 30 cm (b) 25 cm (a) If both Assertion and Reason are correct and Reason is
(c) 40 cm (d) 20 cm the correct explanation of Assertion
(b) If both Assertion and Reason are true but Reason is
Passage II (Q. 3 to 7) not the correct explanation of Assertion
A uniform bar of length A B
(c) If Assertion is true but Reason is false
12L and mass 48 m is (d) If Assertion is false but Reason is true
supported horizontally on (e) If both Assertion and Reason are false
two smooth tables as 3L 4L 1. Assertion In head inelastic collision, the final
shown in figure. A small
moth (an insect) of mass 8m is sitting on end A of the rod momentum is less than the initial momentum.
and a spider (an insect) of mass 16mis sitting on the other Reason For inelastic collision, 0 ≤ e < 1. Hence, the
end B. Both the insects moving towards each other along magnitude of relative velocity of separation after
the rod with moth moving at speed 2v and the spider at collision is less than relative velocity of approach before
half this speed (absolute). collision.
Centre of Mass, Conservation of Linear Momentum, Impulse and Collision 333

2. Assertion A projectile gets exploded at its highest 11. Assertion In an elastic collision between two bodies,
point. All the pieces get only horizontal velocities. The the relative speed of the bodies after collision is equal to
centre of mass will always fall at a point which is farther the relative speed before the collision.
than the point where the projectile would have fallen in Reason In an elastic collision, the linear momentum of
unexploded condition. the system is conserved.
Reason The weight of the projectile is the external force 12. Assertion A moving ball having an inelastic collision
for projectile. with a moving wall can have larger kinetic energy after
3. Assertion If a projectile explodes in mid air, linear collision.
momentum of centre of mass of different fragments Reason During a collision between two bodies, transfer
remains constant. of energy may takes place from one body to another.
Reason When net force on a system of particles is zero, 13. Assertion A rocket launched vertically upward
linear momentum of the system remains constant. explodes at the highest point it reaches. The explosion
produces three fragments with non-zero initial velocity.
4. Assertion A rocket moves forward by pushing the Then, the initial velocity vectors of all the three
surrounding air backwards. fragments are in one plane.
Reason It derives the necessary thrust to move forward Reason For sum of momentum of three particles, all the
according to Newton’s third law of motion. three momentum vectors must be coplanar.
5. Assertion A body is thrown with a velocity u inclined to 14. Assertion If net force on a system is zero, then
the horizontal at some angle. It moves along a parabolic momentum of every individual body remains constant.
path and falls to the ground. Linear momentum of the Reason If momentum of a system is constant, then
body, during its motion, will remain conserved. kinetic energy of the system may change.
Reason Throughout the motion of the body, a constant
15. Assertion Two blocks of masses m A and mB (> m A ) are
force acts on it.
thrown towards each other with same speed over a rough
6. Assertion In an elastic collision between two bodies, ground. The coefficient of friction of both the blocks with
the magnitude of relative velocity of the bodies after ground is same.
collision is equal to the magnitude of relative velocity Initial velocity v v
A B
before collision. of CM is
Reason In an elastic collision, the linear momentum of towards left. Rough
the system is conserved. Reason Initial
acceleration of centre of mass is towards right.
7. Assertion When a body dropped from a height explodes
in mid air, its centre of mass keeps moving in vertically 16. Assertion In inelastic collision linear momentum of
downward direction. system does not remain constant during collision. But
before collision and after collision it is constant.
Reason Explosion occurs under internal forces only.
External force is zero. Reason In elastic collision, momentum remains
constant during collision also.
8. Assertion The centre of mass of an electron and proton,
when released from rest moves faster towards proton. 17. Assertion If a projectile explodes in mid air, then no
external force acts on the projectile during explosion.
Reason Proton is heavier than electron.
Reason Centre of mass in this case follows the same
9. Assertion The relative velocity of the two particles in path.
head-on elastic collision is unchanged both in magnitude
18. Assertion Two
and direction. identical spherical
Reason The relative velocity is unchanged in magnitude spheres are half filled
but gets reversed in direction. with two liquids of
densities ρ1 and
10. Assertion A given force applied in turn to a number of ρ 2 (> ρ1 ). The centre of (i) (ii)
different masses by cause the same rate of change in mass of both the spheres lie at same level.
momentum in each but not the same acceleration to all.
dp F Reason The centre of mass will lie at centre of the
Reason F = and a = sphere.
dt m
334 Objective Physics Vol. 1

19. Assertion Two spherical bodies of mass ratio 1 : 2 2. In the arrangement shown in figure match the following
travel towards each other (starting from rest) under the 2 m/s
k = 4 N/m
action of their mutual gravitational attraction. Then, the 2 kg 2 kg
ratio of their kinetic energies at any instant is 2 : 1.
All smooth
Reason At any instant their momenta are same.
Column I Column II
20. Assertion Two bodies moving in opposite directions
(A) Velocities of centre of mass (p) 2 SI unit
with same magnitude of linear momentum collide each
(B) Velocity of combined mass when (q) 1 SI unit
other. Then, after collision both the bodies will come to compression in the spring is
rest. maximum
(C) Maximum compression in the spring (r) 4 SI unit
Reason Linear momentum of the system of bodies is
(D) Maximum potential energy stored in (s) 0.5 SI unit
zero. the spring

21. Assertion Two blocks A and B are connected at the two


ends of an ideal spring as shown in figure. Initially spring 3. A particle of mass m, kinetic energy K and momentum p
was relaxed. Now, block B is pressed. Linear momentum collides head on elastically with another particle of mass
of the system will not remain constant till the spring 2 m at rest. Match the following columns (after collision).
reaches its initial natural length. Column I Column II
(A) Momentum of first particle (p) 3/ 4 p
(B) Momentum of second particle (q) −K / 9
A B
(C) Kinetic energy of first particle (r) − p/ 3
(D) Kinetic energy of second particle (s) 8K / 9
Reason An external force will act from the wall on
block A. 4. Match the following columns.
22. Assertion In the figure shown, linear momentum of ( p = momentum of particle, K = kinetic energy of
system (of blocks A and B) moves towards right during particle)
motion of block A over the block B. Column I Column II
(A) p is increased by 200%, (p) 800%
A corresponding change in K
(B) K is increased by 300%, (q) 200%
B corresponding change in p
(C) p is increased by 1%, (r) 0.5%
corresponding change in K
(D) K is increased by 1%, (s) 2%
Rough corresponding change in p

Reason Force of friction will act on B towards right 5. Four point masses are placed at four corners of a square
(from ground). of side 4 m as shown in the figure. Match the following
columns.
23. Assertion Centre of mass and centre of gravity of a y
body will coincide on moon.
4 kg 3 kg
Reason There is no gravity on moon.

Match the Columns


x
1. If net force on a system of particles is zero, then match O
1 kg 2 kg
the following columns.
Column I Column II
Column I Column II (A) x-coordinate of centre of mass of (p) (7/2) m
4 kg and 2 kg
(A) Acceleration of centre of mass (p) Constant
(B) x-coordinate of centre of mass of (q) 4/3 m
(B) Momentum of centre of mass (q) Zero 4 kg, 2 kg and 3 kg
(C) Velocity of centre of mass (r) May be zero (C) y-coordinate of centre of mass of (r) 3m
1 kg, 4 kg and 3 kg
(D) Velocity of an individual (s) May be constant (D) y-coordinate of centre of mass of (s) (20/9) m
particle of the system 1 kg and 3 kg
Centre of Mass, Conservation of Linear Momentum, Impulse and Collision 335

6. In a two block system shown in figure match the Entrance Gallery


following.
Rough
10 m/s
2014
1 kg
5 m/s 1. In elastic collision, [Kerala CEE]
2 kg (a) both momentum and kinetic energy are conserved
Smooth (b) neither momentum nor kinetic energy is conserved
(c) only momentum is conserved
Column I Column II (d) only kinetic energy is conserved
(A) Velocity of centre of (p) Keep on changing all the time (e) forces involved in the interaction and non-conservative
mass
2. A smooth massless string passes over a smooth fixed
(B) Momentum of centre of (q) First decreases then become
mass zero pulley. Two masses m1 and m2 , ( m1 > m2 ) are tied at the
two ends of the string. The masses are allowed to move
(C) Momentum of 1 kg block (r) Zero
under gravity starting from rest. The total external force
(D) Kinetic energy of 2 kg (s) Constant
block
acting on the two masses is [WB JEE]
(m1 − m2 )2
(a) (m1 + m2 ) g (b) g
7. A particle of mass 1 kg is projected upwards with velocity m1 + m2
60 m/s. Another particle of mass 2 kg is just dropped from a (m1 + m2 )2
certain height. After 2 s, match the following (c) (m1 − m2 ) g (d) g
m1 − m2
[Take g = 10 m/ s 2 ]
Column I Column II
2013
(A) Acceleration of CM (p) Zero 3. A particle of mass m is attached to one end of a massless
(B) Velocity of CM (q) 10 SI unit
spring of force constant k, lying on a frictionless
horizontal plane. The other end of the spring is fixed. The
(C) Displacement of CM (r) 20 SI unit
particle starts moving horizontally from its equilibrium
(s) None
position at times, t = 0 with an initial velocity u 0 . When
8. In the diagram shown in figure mass of both the balls is the speed of the particle is 0.5 u 0 , it collides elastically
same. Match the following columns. with a rigid wall. After this collision, [JEE Advanced]
v v' (a) the speed of the particle when it returns to its equilibrium
position is u0
1 2 ⇒ 2 (b) the time at which the particle passes through the equilibrium
m
Before After position for the first time is t = π
collision collision k
(c) the time at which the maximum compression of the spring
Column I Column II 4π m
occurs is t =
(A) For v ′ = v (p) e = 0 3 k
(B) For v ′ = v / 2 (q) e = 1 (d) the time at which the particle passes through the equilibrium
5π m
(C) For v ′ = 3 / 4 v (r) e = 1/ 2 position for the second time is t =
3 k
(s) Data is insufficient
4. A bob of mass m, suspended by a string of length l1 , is
9. A particle of mass 1 kg has velocity v1 = ( 2t ) $i and
given a minimum velocity required to complete a full
another particle of mass 2 kg has velocity v = ( t 2 ) $j. 2 circle in the vertical plane. At the highest point, it
Match the following columns. collides elastically with another bob of mass m
Column I Column II suspended by a string of length l2 , which is initially at
(A) Net force on centre of mass at 2 s (p) 20 unit rest. Both the strings are massless and inextensible. If
9 the second bob, after collision acquires the minimum
(B) Velocity of centre of mass at 2 s (q) 68 unit speed required to complete a full circle in the vertical
80 / 3 unit
l
(C) Displacement of centre of mass (r) plane, the ratio 1 is [JEE Advanced]
in 2 s l2
(s) None (a) 1 (b) 3 (c) 4 (d) 5
336 Objective Physics Vol. 1

5. This question has Statement I and Statement II, of the Case (A) the disc with its face vertical and parallel to x-z
four choices given after the statements, choose the one plane,
that best describes the two statements. [JEE Main] Case (B) the disc with its face making an angle of 45° with
Statement I A point particle of mass m moving with x- y plane and its horizontal diameter parallel to x-axis. In
speed v collides with stationary point particle of mass M. both the cases, the disc is welded at point P and the systems
If the maximum energy loss possible is given as, are rotated with constant angular speed ω about the z-axis.
1   m 
f  mv 2  , then f =  . 6. Which of the following statements regarding the angular
2   M + m speed about the instantaneous axis (passing through the
centre of mass) is correct? [IIT JEE]
Statement II Maximum energy loss occurs when the
(a) It is 2ω for both the cases
particles get stuck together as a result of the collision.
(b) It is ω for case (A) and ω/ 2 for case (B)
(a) Statement I is correct, Statement II is correct, and
Statement II is the correct explanation of Statement I (c) It is ω for case (A) and 2ω for case (B)
(b) Statement I is correct, Statement II is correct, but Statement II (d) It is ω for both the cases
is incorrect explanation of Statement I
7. Which of the following statements about the instantaneous
(c) Statement I is correct, Statement II is incorrect
(d) Statement I is incorrect, Statement II is correct
axis (passing through the centre of mass) is correct?
(a) It is vertical for both the cases A and B [IIT JEE]
2012 (b) It is vertical for case A and is at 45° to the x-z plane and lies in
the plane of the disc for case B
Paragraph (c) It is horizontal for case A and is at 45° to the x-z plane and is
The general motion of a rigid body can be considered to normal to the plane of the disc for case B
be a combination of (i) a motion of its centre of mass (d) It is vertical for case A and is at 45° to the x-z plane and is
about an axis and (ii) its motion about an instantaneous normal to the plane of the disc for case B
axis passing through the centre of mass. 8. Two solid cylinders P and Q of same mass and same
z
ω
radius start rolling down a fixed inclined plane from the
same height at the same time. Cylinder P has most of its
mass concentrated near its surface, while Q has most of its
P P Q
Q mass concentrated near the axis. Which statements (s) is
y
(are) correct? [IIT JEE]
x
(a) Both cylinders P and Q reach the ground at the same time
(b) Cylinder P has larger linear acceleration than cylinder Q
These axes need not be stationary. Consider, for example,
(c) Both cylinders reach the ground with same translational
a thin uniform disc welded (rigidly fixed) horizontally at
kinetic energy
its rim to a massless stick, as shown in the figure.
(d) Cylinder Q reaches the ground with larger angular speed
When the disc-stick system is rotated about the origin on
a horizontal frictionless plane with angular speed ω, the 2011
motion at any instant can be taken as a combination of
9. A thin ring of mass 2 kg and radius 0.5 m is rolling
(i) a rotation of the centre of mass of the disc about the
without slipping on a horizontal plane with velocity
z-axis, and (ii) a rotation of the disc through an
1 m/s. A small ball of mass 0.1 kg, moving with velocity
instantaneous vertical axis passing through its centre of
20 m/s in the opposite direction, hits the ring at a height
mass (as is seen from the changed orientation of points P
of 0.75 m and goes vertically up with velocity 10 m/s.
and Q).
Immediately after the collision, [IIT JEE]
Both these motions have the same angular speed ω this case.
10 m/s
Now, consider two similar systems as shown in the 20 m/s
figure.
0.75 m 1 m/s
z z ω
ω
Q Q
(a) the ring has pure rotation about its stationary CM
45°
(b) the ring comes to a complete stop
P y P y (c) friction between the ring the ground is to the left
x Case (A) x Case (B)
(d) there is no friction between the ring and the ground
Centre of Mass, Conservation of Linear Momentum, Impulse and Collision 337

10. A ball of mass 0.2 kg rests on v m/s 14. Statement I Two particles moving in the same
a vertical post of height 5 m. direction do not lose all their energy in a completely
A bullet of mass 0.01 kg, inelastic collision.
travelling with a velocity Statement II Principle of conservation of momentum
v m/s in a horizontal holds true for all kinds of collisions. [AIEEE]
direction, hits the centre of (a) Statement I is correct, Statement II is correct, Statement II is
0 20 100
the ball. After the collision, the correct explanation of Statement I
the ball and bullet travel independently. The ball hits the (b) Statement I is correct Statement II is correct, Statement II
ground at a distance of 20 m and the bullet at a distance of incorrect explanation of Statement I
100m from the foot of the post. The initial velocity v of (c) Statement I is incorrect, Statement II is correct
the bullet is [IIT JEE] (d) Statement I is correct, Statement II is incorrect
(a) 250m/s (b) 250 2 m/s 15. A T shaped object with dimensions shown in the figure, is
(c) 400m/s (d) 500m/s lying on a smooth floor. A force F is applied at the point P
parallel to AB, such that the object has only the
11. A stationary bomb explodes into three pieces. One piece translational motion without rotation. Find the location of
of 2 kg mass moves with a velocity of 8 ms −1 at right P with respect to C. [AIEEE]
angles to the other piece of mass 1 kg moving with a l
velocity of 12 m s −1 . If the mass of the third piece is A B
0.5 kg, then its velocity is [Kerala CEE]
(a) 10 ms−1 (b) 20 ms−1 P
(c) 30 ms−1 (d) 40 m s−1 F 2l
(e) 50 m s−1

12. A cricket ball of mass 0.25 kg with speed 10 m/s collides


with a bat and returns with same speed with in 0.01s. The 2 3 4
(a) l (b) l (c) l (d) l
force acted on bat is [WB JEE] 3 2 3
(a) 25N (b) 50N (c) 250N (d) 500N
16. The figure shows the position-time (x- t) graph of
2010 one-dimensional motion of a body of mass 0.4 kg. The
magnitude of each impulse is [AIEEE]
13. A point mass of 1 kg collides elastically with a stationary
point mass of 5 kg. After their collision, the 1 kg mass 2
reverses its direction and moves with a speed of 2 ms −1 . x (m)
Which of the following statement(s) is/are correct for the
system of these two masses? [IIT JEE]
(a) Total momentum of the system is 3 kg-ms−1 0 2 4 6 8 10 12 14 16
(b) Momentum of 5 kg mass after collision is 4 kg-ms−1 t (s)
(c) Kinetic energy of the centre of mass is 0.75 J (a) 0.4 N-s (b) 0.8 N-s
(d) Total kinetic energy of the system is 4 J (c) 1.6 N-s (d) 0.2 N-s
Answers
Level 1
Objective Problems
1. (a) 2. (d) 3. (d) 4. (b) 5. (d) 6. (d) 7. (d) 8. (d) 9. (b) 10. (a)
11. (b) 12. (d) 13. (b) 14. (a) 15. (a) 16. (a) 17. (b) 18. (c) 19. (c) 20. (b)
21. (d) 22. (b) 23. (a) 24. (b) 25. (a) 26. (a) 27. (d) 28. (d) 29. (c) 30. (d)
31. (d) 32. (a) 33. (d) 34. (a) 35. (b) 36. (b) 37. (b) 38. (d) 39. (b) 40. (d)
41. (c) 42. (b) 43. (c) 44. (b) 45. (a) 46. (d) 47. (b) 48. (c) 49. (a) 50. (d)
51. (d) 52. (d) 53. (c) 54. (d) 55. (b) 56. (b) 57. (c) 58. (d) 59. (b) 60. (c)
61. (c) 62. (b) 63. (c) 64. (b) 65. (b) 66. (a) 67. (b) 68. (c) 69. (d) 70. (a)
71. (b) 72. (c) 73. (c) 74. (b) 75. (b) 76. (c) 77. (a) 78. (b) 79. (c) 80. (a)
81. (b) 82. (d) 83. (c) 84. (a) 85. (b) 86. (b) 87. (b) 88. (c) 89. (d) 90. (d)
91. (d) 92. (c) 93. (d) 94. (d) 95. (a) 96. (c) 97. (a) 98. (c) 99. (a) 100. (b)
101. (a) 102. (b) 103. (a) 104. (d) 105. (b)

Level 2
Only One Correct Option
1. (c) 2. (c) 3. (c) 4. (d) 5. (b) 6. (b) 7. (a) 8. (a) 9. (d) 10. (a)
11. (a) 12. (b) 13. (b) 14. (a) 15. (d) 16. (c) 17. (a) 18. (a) 19. (d) 20. (b)
21. (d) 22. (d) 23. (d) 24. (b) 25. (b)

More than One Correct Options


1. (b,d) 2. (a,c,d) 3. (all 4. (b,d) 5. (b,c) 6. (c,d) 7. (b,c) 8. (b,c,d)

Comprehension Based Questions


1. (b) 2. (b) 3. (d) 4. (b) 5. (c) 6. (c) 7. (a)

Assertion and Reason


1. (d) 2. (d) 3. (d) 4. (d) 5. (d) 6. (b) 7. (c) 8. (d) 9. (d) 10. (a)
11. (d) 12. (a,b) 13. (a) 14. (d) 15. (b) 16. (d) 17. (d) 18. (e) 19. (a,b) 20. (d)
21. (a) 22. (a) 23. (e)

Match the Columns


1. (A → q, B → r, C → r, D → s) 2. (A → q, B → q, C → q , D → p) 3. (A → r, B → t, C → t, D → s)
4. (A → p, B → t, C → s, D → r) 5. (A → q, B → s, C → p, D → r) 6. (A → r,s, B → rs, C → q, D → q)
7. (A → q, B → p, C → r ) 8. (A → q, B → p, C → r) 9. (A → q, B → r, C → p)

Entrance Gallery
1. (a) 2. (b) 3. (d) 4. (d) 5. (a) 6. (d) 7. (b) 8. (a) 9. (a,c) 10. (d)
11. (d) 12. (d) 13. (b) 14. (b) 15. (c) 16. (b)
Solutions
Level 1 : Objective Problems 7. Centre of mass of Ist system already lies at (1, 2, 3). Therefore,
1. Centre of mass of square plate is at O. Centre of mass of two centre of mass of 3 kg and 5 kg should lie at (1, 2, 3).
masses of 5 kg each is at H. Hence, centre of mass of the 3( − $i + 3$j − 2 k$ ) + 5 r5
∴ = ( $i + 2$j + 3k$ )
whole system is at mid-point of OH. ( 3 + 5)
2. Distance distributes in inverse ratio of masses. Hence, 11 $ 7 $
Solving this we get, r5 = i + j + 6 k$
 m0  −10  16  5 5
rc = d   = 1.2 × 10  
 m0 + mc   16 + 12  i.e. 5 kg mass should be kept at (11/5, 7/5, 6)
−10
= 0.69 × 10 m 8. Centre of mass of complete disc should lie at point O. C1 is
A2 the position of centre of mass of remaining portion and C 2
3. A1 x1 = A2 x 2 or x1 = ⋅ x2
A1 the position of centre of mass of the removed disc.
 π  (8)2 R
∴ x (Area of remaining portion) = (Area of removed disc)
 
  2
= 4 2 × 6 = 0.12 cm from O.
( 20)

O O1
x2
x1

π 2  πR 2  R  πR 2 
4. Area of circle, a = A1 , area of square = a 2 = A2 . ∴ x  πR 2 − =  
4  4  2 4 
Since, A2 > A1 , centre of mass will lie inside the square plate. R
∴ x=
5. x CM = 0 6
y 9. Centre of mass of whole system was at point O. Hence,
m4 m x 2 (area of remaining portion) = c (area of removed disc)
∴ x 2 ( πa 2 − πb 2 ) = c( πb 2 )
a
cb 2
x ∴ x2 =
a2 − b2
a
10. C1 → Position of centre of mass of rods AB and CD (nearer to
2m a a 4m CD, as it is heavy)

m1 x1 + m2 x 2 + m3 x 3 + m4 x 4 D C
or =0
m1 + m2 + m3 + m4
C1
or ( 2m)( − a ) + 4m ( a ) + m ( a ) + m4 ( − a ) = 0 C
or m4 = 3m
Similarly, y CM = 0 C2
or ( 2m)( − a ) + 4m ( − a ) + m( a ) + m4 ( a ) = 0
or m4 = 5 m
Since, value of m4 are different to satisfy both x CM = 0 and A B
y CM = 0.
Hence, it is not possible. C 2 → Position of centre of mass of rods BC and DA.
m x + m2x 2 + m3x 3 C → Overall centre of mass of all four rods.
6. X CM = 1 1
m1 + m 2 + m 3 11. For a single particle distance of centre of mass from origin is
R. For more than one particles distance ≤ R.
m(0) + (m)   + m  
a a
m R R m
 2  2 a
= = x=0
m+ m+ m 3
For example for two particles of equal mass, kept as shown
a
Similarly, y CM = in figure, distance = 0.
3
340 Objective Physics Vol. 1

1 16. Pi = 0
12. OC1 = m
4 ∴ Pf = 0 or centre of mass should be at rest at all instants.
(m)(0) + (m)(a ) 1
17. a CM = = a
m+ m 2
C2 10(14) + 4(0)
18. v CM = = 10 m/s
C 10 + 14
19. Centre of mass does not change its path during explosion.
45°
O Therefore, it will keep on falling vertically.
C1
20. Net external force is zero. Hence, velocity of CM of the box
∴ OC = OC1 cos 45° and ball system will remain constant.
1
= m 21. Both the balls in air have acceleration g downwards. Hence,
4 2
the acceleration of their centre of mass will also be g
m × O + m × PQ + m × PR
13. X CM (from P ) = (m =1 kg ) downwards.
m+ m+ m
22. Let plank moves x distance in opposite direction. Then,
displacement of man relative to ground will be, ( L − x ).
P Q R Applying
M
mR x R = mL x L or M ( L − x ) = x
3
PQ + PR 3L
or X CM = Solving this equation we get, x =
3 4
3L L
14. P is the position of centre of mass of particle at 2 and 3. ∴ Displacement of man relative to ground = L − =
Q is position of centre of mass of all three particles. 4 4
y 23. Centre of mass will remain at height h.
m × 0 + MH
3 ∴ hCM = =h
m+ M

H = h 1 +
m
P

Q
θ ∴ 
x  M
1 2
24. After 1 s, coordinates of first particle will become
( 4 m, 4 m, 6 m) and co-ordinates of second particle will
become (6 m, 4 m, 8 m).
4+ 6
∴ X CM = = 5m
y CM m1 y1 + m2 y 2 + m3 y 3 2
tan θ = =
x CM m1 x1 + m2 x 2 + m3 x 3 4+ 4
YCM = = 4m
6×0+ 2×0+ 2× a 2
= 6+ 8
6×0+ 2× a + 2×0 and Z CM = = 7m
2
=1
/ In this problem, answer is independent of the fact whether
or θ = 45°
the first particle has velocity v1 or v 2 . Think why?
15. A1 (CC1 ) = A2 (CC 2 )
25. Remaining time for the pieces to reach the ground will be,
10 m/s 10 m/s

x x

2 × 45 2 × 20
t= − =1s
10 10
x = 10 × 1 = 10 m
R
Side of square will be ∴ Distance between two pieces will be 20 m.
2
26. In the absence of friction, centre of mass will not move in
A1
∴ CC 2 = (CC1 ) x-direction.
A2 m y + m2y2
( R / 2 )2 27. y CM = 1 1
=  R m1 + m2
 
π R 2 − ( R / 2 )2  2 
(10)( 7) + ( 30) y 2
 R  or +1 =
=  10 + 30
 4π − 2 ∴ y 2 = − 1 cm
Centre of Mass, Conservation of Linear Momentum, Impulse and Collision 341

28. Distance between bullet and block at this instant is ( D − d ) 38. m1v1 = m2v 2 or v 2 =
m1v1
M m2
Distance from bullet = (D − d )
M+m 5 × 10− 3 × 900
= =1.8 m/s
m 2.5
And distance from block = (D − d )
M+m 39. K 1 + K 2 = 2400 ...(i)
29. Initially centre of mass was at rest. Hence, at any instant p1 = p2
centre of mass will be at rest as net external force is zero.
∴ 2K 1m1 = 2K 2m 2
30. At t = 0, centre of mass is at mid- point or at (2.25 m, 0) K 1 m2 3
or = = ...(ii)
Velocity of centre of mass is zero. Hence, centre of mass will K 2 m1 1
remain at this position all the time.
Solving these two equations, we get
31. Vertical component of velocity of CM is zero. Horizontal
component of velocity of CM is non-zero. Acceleration of K 1 = Kinetic energy of smaller part =1800 J
CM is g downwards. Hence, path is a parabola as show in 40. Impulse = change in linear momentum
figure. = p f − pi
CM
= − 5× 4− 5× 3
= − 35 N-s
∴ Magnitude of impulse in SI Units = 35
41. mv =  m v ′
3
4 
4v
∴ v′ =
32. During explosion of a projectile path of centre of mass does 3
not change. At height 1 m, 42. p3 = p6 or 3 × 16 = 6 × v or v = 8 m/s. As pi = 0 or both the
v = u − 2gh (from conservation of mechanical energy)
2
pieces should have equal and opposite momentum.
= 4 5 m/s 43. pi = 0
m u + m2 u 2 ∴p f = 0, i.e. both the pieces should have equal and opposite
33. u CM = 1 1 =0 momentum.
m1 + m2
44. Impulse J = ∆ p = m( v f − v i )
as m1u1 + m2u 2 = 0 is given. Hence, velocity of composite
system will be zero. 0.2[( 2$i + 3$j + k$ ) − ( 5$i − 3$j + 7k$ )]
34. Centre of mass will not move along y-axis.
= ( − 0.6$i + 1.2$j − 1.2k$ )
or YCM = 0 (always)
35. m (u − v ) = mv ∴ J = (0.6)2 + (1.2)2 + (1.2)2
u = 1.8 N-s
∴ v=
2
45. F = thrust force = vr 
dm 

 dt 
= 2 × 0.5
u–v
=1N
v ∆p m( v f − v i )
46. F = =
m (u – v) = mv
∆t 4t
15 [(6$i + 4$j + 5k$ ) − ( $i − 2$j )]
=
∴ Velocity of man relative to ground 0.1
u = 150 ( 5i + 6j + 5k$ )
$ $
=u−v =
2
47. Packet from A falls with greater momentum on B.
36. m1v1 + m 2v 2 = (m1 + m 2 ) v
Therefore, B is slightly accelerated.
∴ a ⋅ b + c ⋅ 0 = (a + c)v 48. In horizontal direction, net force on the system is zero.
ab
⇒ v= 49. From conservation of linear momentum,
a+c
l m m/2 m/2
37. vr = . Now, m (vr − v ) = Mv (v = speed of plank) v'
t v cos θ
mvr v cos θ
∴ v=
M+m m m
m(v cos θ) = v ′ − v cos θ
l m  2 2
=  
t  M + m ∴ v = 3v cosθ
342 Objective Physics Vol. 1

50. Force of friction on A is backwards and force of friction on B F


is forwards. Net external force on the system is zero. Hence,
momentum of system will remain conserved. As the F0
momentum of system is conserved, increase in momentum
of B is equal to decrease in momentum of A.
51. The centre of mass under the given condition may be at rest T
or may be moving with constant velocity. T/2 T
52. v = vr ln  0 
m 1 2mu
 m ∴ F0T = mu or F0 =
2 T
2vr = vr ln  0 
m $ 4 $ 3 $
or 57. v i = v 0 cos 37° i + v 0 sin 37° j = v 0 i + v 0 j
$
 m 5 5
2 = ln  0 
m 3 $ 3
∴ v f = − v 0 cos 53° i + v 0 sin 53° j $
 m 4 4
m0 9 $ 3 $
∴ = e 2 = 7.4 =− v0 i + v0 j
m 20 5
m 5
or = 0.14 J = m( v f − v i ) = − mv 0 $i
m0 4
This is fraction of mass remaining. Hence, fraction of mass 58. Net impulse = area under F-t graph = 0
disposed off = 0.86 ∴ p or v = 0
53. Initial momentum of 3m mass = 0 ...(i) 59. Just before 2 s, v i = slope of x-t graph = 2 m/s
Due to explosion, this mass splits into three fragments of Just after 2 s, v f = slope of x-t graph = 0
equal masses. ∴ Impulse at 2 s
v = ∆p = p f − pi = m(v f − v i ) = − 0.2 kg-m/s
At rest m ∆p
60. F =
3m ∆t
Before collision v 0.150 × ( 20 − 0)
m ∴ F= = 30N
0.1
m 61. Impulse = ∆p = p1 − p2 = mv1 − mv 2
v After collision
= m(v1 − v 2 )
Final momentum of system = 0.06[4 − ( −4)]
= mv + mv$i + mv$j ...(ii) = 0.06 × 8 = 0.48 kg-m/s
m ∆v
From law of conservation of linear momentum 62. F =
∆t
mv = mvi$ + mvj$ = 0 0.150 [20 − ( −12)] 0.150 × 32
= = = 120N
⇒ v = − v ( i$ + j$ ) 0.04 0.04
54. Force = Rate of change of momentum 63. Vr = v (Opposite to the motion of railroad car)
Initial momentum p = mv sin θ$i + mv cos θ$j dv
1 ∴ Ft = − µv = ( M + µt ) ⋅
dt
Final momentum p 2 = − mv sin θ$i + mv cos θ$j

dt  dv 
= −µ  
∆p M + µt  v 
∴ F=
∆t t dty 1 v dv
=
−2mv sin θ ∴ ∫0 M + µt
=−
µ ∫v 0 v
2 × 10−3
M + µt  1
ln  v 
1
Substituting m = 0.1 kg, v = 5 m/s, θ = 60° ∴  = ln  0 
µ  M  µ v 
Force on the ball F = − 250 3 N Mv 0
Solving we get, v =
Negative sign indicates direction of the force. M + µt
55. p i = p f (at highest point) 64. F − mg = ma (F = thrust force)
( 3m)(100$i ) = m(100$j ) − m(100$j ) + m( v ) or F = m( g + a)
Vr  −
dm 
∴ v = 300$i (of third part)  = m( g + a)
 dt 
56. In one dimensional elastic collision between two equal
∴  − dm  = m ( g + a )
masses velocities are interchanged. Therefore, change in  
 dt  Vr
linear momentum of any of the particle will be mu. Now,
impulse or area under F-t graph gives the change in linear 6000 × 30
= = 180 kg/s
momentum. 1000
Centre of Mass, Conservation of Linear Momentum, Impulse and Collision 343

65. F − mg = ma (F = thrust) 72. Mass of combined body = 50 kg

pB = 40 kg-m/s p = 50 kg-m/s

mg
pA = 30 kg-m/s
F = m( g + a )
= (3.5 × 104 ) (10 + 10) ∴ Velocity of combined body =
50 kg-m/s
= 1 m/s
= 7.0 × 10 N 5 5 kg
67. mv = (m + M ) v ′ m × 50 + m( −40)
73. v x = = 5 cm/s
m+ m
 m 
∴ v′ =  v m(0) + m( 30)
m+ M  vy = = 15 cm/s
m+ m
68. In perfectly elastic collision between two bodies of equal
74. Between elastic collision of two identical masses, velocities
masses, velocities are exchanged.
are interchanged.
m 2m 1
75. Relative speed of approach is 8 m/s, e = . Therefore,
2 1 2
Rest relative speed of separation will be 4 m/s.

 m − m1   2m1  4 m/s 4 m/s 2 m/s 2 m/s


69. v 2′ =  2 v +  v ⇒
m + m  2 m + m  1
 2 1  1 2
76. Net force on ball and earth system is zero.
v1 = 0 77. 2mv cos θ = ( 2m) v ′
v2  m + m1 
∴ = 2  ∴ v ′ = v cosθ
 
v 2′  m 2 − m1  78. From conservation of linear momentum,
 m + 2m  u ⇒
=  =−3 2 1 2 1 v1
 m − 2m 
2
mu = mv1 + mv 2
K2 v  or u = v1 + v 2 ...(i)
∴ = 2 =9
K 2′  v 2′ 
From definition of e,
v1 − v 2 = eu ...(ii)
70. From conservation of linear momentum, we can see that Solving these two equations, we get
v
1+ e
v1 = 
velocity of 2m will become after collision (as mass is
2 u
 2 
doubled)
1−e
and v 2 =  u
m
v
2m m 2m  2 
⇒ v/2
v1  1 + e 
Rest ∴ = 
v2  1 − e 
Relative velocity of separation  m − m1 
Now, e= 2m1u1
Relative velocity of approach 79. v 2 =  2 u +
m + m  2 m + m
v/2 1  1 2 1 2
= = 2Mu
v 2 = m M m
M+m M
v sin θ tan θ
71. tan θ′ = = u1 = u u2 = 0 v1 = v v2 = v
ev cos θ e =
2u
m Before collision After collision
ev cosθ 1+
v' M
θ 4
80. hn = he 2n = 32   =
v sinθ 1 32
⇒ ⇒ = 2m (here, n = 2,e =1 / 2)
 2 16
θ
θ 81. If target is at rest, then final velocity of bodies are
v
v cosθ
v sinθ u1 v2
m1 m2 m1 m2
tan θ 
or θ = tan −1  
u2 = 0
 e  After collision
Before collision
344 Objective Physics Vol. 1

 m − m2  89. If CM is at rest, it definitely means momentum of the


v1 =  1 u ...(i)
m + m  1 system is constant. But momentum of the system is
 1 2
constant, it does not mean centre of mass is at rest.
2m 1u 1
and v2 = ...(ii) 90. Kinetic energy of the system is zero, it definitely means
m1 + m 2 momentum is zero. But momentum of the system is zero, it
v 1 m 1 − m 2 2 m1 does not mean kinetic energy is zero.
From Eqs. (i) and (ii), = = ⇒ =5 Example O→v
v2 2m 1 5 m2
v ←O
82. Due to the same mass of A and B as well as due to elastic p =O
collision velocities of spheres get interchanged after the
K = 2  mv 2  = mv 2 .
collision. 1
but
2 
84. Let mass A moves with velocity v and collides inelastically
with mass B, which is at rest. p2
91. K = and p = 2 Km are two standard results.
v/ 3 2m
A m 92. F = rate of change of linear momentum
A v B
n
m m In 1 s, bullets are embedded. Momentum of each bullet is
θ 60
m v mv.
B
F =   mv
n

According to problem, mass A moves in a perpendicular  60 
direction and let the mass B moves at angle θ with the 93. In1cm2 area,104 balls are striking per second. Therefore, in
horizontal with velocity v.
1 m2 area,108 balls will strike per unit time.
Initially in horizontal momentum of system
Change in momentum of each ball per second will be 2mu.
(before collision) = mv ...(i)
F ( ∆p / ∆t )
Final horizontal momentum of system ∴ Pressure = =
A A
(after collision) = mv cosθ ...(ii)
= 108 × 2 mu
From the conservation of linear momentum
= 108 × 2 × 10−3 × 100
mv = mV cosθ ⇒ v = v cosθ ...(iii)
= 2 × 107 N/m2
Initial perpendicular direction momentum of system
(before collision) is zero. 94. p2 = 2Em or p2 ∝ E
Final momentum of system in this direction is 2
i.e. p versus E graph is a straight line passing through
mv
− mV sin θ origin.
3
95. Velocity of block just after collision will be
From conservation of momentum
5 × 10−3 × 150
mv
− mV sin θ = 0 v=
3 ( 2 + 5 × 10−3 )
v (From conservation of linear momentum)
⇒ = V sin θ ...(iv)
3 = 0.374 m/s
By solving Eqs. (iii) and (iv), we have Let, F be the force of friction. Then, work done against
v2 friction = initial kinetic energy
v2 + = V 2 (sin 2 θ + cos2 θ) 1
3 or F × 2.7 = × 2.005 × (0.374)2
4v 2 2 2
⇒ =V 2 ⇒ V = v or F = 0.052 N
3 3
96. ∆p = p f − p i = m( v f − v i )
85. In elastic collision, both linear momentum and kinetic energy
should remain constant. This is possible in option (b).  v $ v $  v $ v $ 
= m   i− j −  i+ j
86. Relative speed of approach is 12 m/s.  2 2   2 2  
1
e= = − 2 mv $j
3
2 m/s ∴ | ∆p | = 2 mv
6 m/s 6 m/s 2 m/s
⇒ 97. Let initial kinetic energy, E1 = E
Therefore, relative speed of separation will be 4 m/s. Final kinetic energy, E2 = E + 300% of E = 4E
Relative velocity of separation 5 − 2 1 As p ∝ E
87. e = = =
Relative velocity of approach 8 − 2 2 p2 E2 4E
⇒ = = =2
88. ∆p = p f − pi = − mu − mu = − 2mu but | ∆p | = 2mu p1 E1 E
pi = mu pj = mu ⇒ p2 = 2p1 ⇒ p2 = p1 + 100% of p1
i.e. Momentum will increase by 100%.
Centre of Mass, Conservation of Linear Momentum, Impulse and Collision 345

p1 3p1 s ∝ u2
98. Let p1 = p, p2 = p1 + 50% of p1 = p1 + = or (If retardation is constant)
2 2 If the speed of the bullet is double, then bullet will cover
E ∝ p2 four times distance before coming to rest
2 i.e. s 2 = 4( s1 ) = 4( 2s ) ⇒ s 2 = 8s
E2  p2  So, number of planks required = 8
⇒ = 
E1  p1  103. Let ball is projected vertically downward with velocity v
2 from height h.
 3p / 2  9
= 1  = v v=0
 p1  4
⇒ E2 = 2.25 E2 = E1 + 1.25E1
h
∴ E2 = E1 + 125% of E1
i.e. Kinetic energy will increase by 125%.
99. p = 2mE
∴ p∝ E 1
Total energy at point A = mv 2 + mgh
1 2
Percentage increase in p ≈ (percentage increase in E)
2 During collision loss of energy is 50% and the ball rise up to
1 same height. It means it posses only potential energy at
= (0.1%) = 0.05% same level.
2
50%  mv 2 + mgh = mgh
1
p2 2 
100. Kinetic energy E =
2m 1 1 
 mv + mgh = mgh
2
( Ft )2 F 2t 2 22 
= = (As p = Ft )
2m 2m v = 2gh = 2 × 10 × 20
101. After striking with A, the block C comes to rest and block A
∴ v = 20 m/s
moves with velocity v, when compression in spring is
104. Their common velocity would be
maximum both A and B will be moving with common mv 0 v
velocity v. v= = 0
m + 2m 3
C A B Now, applying conservation of mechanical energy
2
mv 02 = kx 02 + ( 3m)  0 
m m m 1 1 1 v
2 2 2  3 
2 mv 02
k=
From law of conservation of linear momentum, 3 x 02

mv = (m + m) V ⇒ V =
v 105. v ′ = 2gR
2 From conservation of linear momentum,
From law of conservation of energy, 2Ω 3Ω
KE of block C = KE of system + PE of system i
1 1 1
mv 2 = ( 2m) v 2 + kx 2
2 2 2
2 2Ω 3Ω

1 1  v
mv = ( 2m)   + kx 2
2 1 10 V
2 2  2 2
1
⇒ kx = mv
2 2
2 mv = ( M + m) v ′
m M+m
⇒ x =v ∴ v= ⋅v′
2k m
M + m
102. Let the thickness of each of plank is s. If the initial speed of =   2gR
bullet is 100 m/s, then it stops by covering a distance 2s.  m 

u = 100 m/s
Level 2 : Only One Correct Option
v=0
 M − m
1. v A =  u
 M + m

2s Velocity of C at maximum compression


1  2M   M 
vC =   u=  u
By applying v 2 = u2 − 2as ⇒ 0 = u2 − 2as 2  M + m  M + m
u2  m 
s= ∴ vCA = vC − v A =  u
2a  M + m
346 Objective Physics Vol. 1

2. Velocity of A just before collision 3 F  2


∴   t = x + 2xm
= 2gh = 2 × 10 × 1.25 2  3m 
= 5 m/s Ft 2 x
5 ∴ X 2m = −
Velocity of ( A + B ) just after collision = = 2.5 m/s 4m 2
2
10. Velocity of block just after collision = 2gh
(From conservation of linear momentum)
In elastic collision between two bodies of equal masses = 2 × 10 × 0.2
velocities are interchanged. = 2 m/s
Hence, velocity of C will become 2.5 m/s.
Now, applying conservation of linear momentum just
600
3. F = 0 at t = s = 0.003 s before and just after collision.
2 × 105
0.02 × 600 = 4 × 2 = 0.02 × v
0.003
Impulse = ∫ F dt = 0.9 N-s ∴ v = 200 m/s
0
11. Velocity of particle after 5 s
4. At 1 s
10 m/s 10 m/s v = u − gt
= 100 − 10 × 5
= 100 − 50
10 m/s 10 m/s = 50 m/s (upwards)
45° 45° Conservation of linear momentum gives
Mv = m1v1 + m2v 2 ...(i)
v1 Taking upward direction positive,
v2 10 m/s 10 m/s
v1 = − 25 m/s, v = 50 m/s
5. pnet = ∫ dp sin θ = ∫
π  M ⋅ dθ v ⋅ sin θ = 2Mv M =1 kg, m1 = 400 g = 0.4 kg
 
0  π  π
m2 = M − m1 = 1 − 0.4
= 0.6 kg
dp
From Eq. (i),
90 ° –
θ 1 × 50 = 0.4 × ( −25) + 0.6 v 2
dp or v 2 = 100 m/s (upwards)
12. Velocity on hitting the surface
θ = 2 × 9.8 × 49 = 9.8 m/s
Velocity after first bounce
3
6. From conservation of linear momentum, v= × 9.8
v 4
m1v1 = m 5gl + m1 1
3 2v
Time taken from first bounce to the second bounce =
3 m g
or v1 = 5gl
2 m1 3 1
= 2× × 9.8 × =1.5 s
7. In head-on elastic collision 4 9.8
 m/ 2 − m   2(m)  ∆p ∆ (mn v )
v ′1 =   (0) +
4 13. F = =
 m / 2 + m  (v ) = 3 v ∆t ∆t
 m / 2 + m   v
m m/2 m/2 v m/4
v 1
2 1 Then
n −1
v ′n =  
4
Finally, ⋅ v = 5gr
 3 v
n −1
v =   |∆v| = 2v
3
∴ 5gr
 4 Here, m = mass of one marble = 5 g
8. Relative speed of separation = relative speed of approach = v n
= number of molecules striking per second =10
∆t
2πr
∴ Time of next collision =
Mg = m   | ∆v |
n
v ∴
 ∆t 
mXm + 2mX 2m
9. X CM = Here, M = mass of disc
2m
(10 × 10−3 ) (9.80) = (5 × 10−3 ) (10) 2v
1 x + 2x 2m
∴ aCMt 2 = or v = 0.98 m/s
2 3
Centre of Mass, Conservation of Linear Momentum, Impulse and Collision 347

ev2
14. Mgh = Fx v1
Mgh C
∴ F= x
v1 ⇒ v1
x D
∆p tAB = x/v1 A
15. F = = n(mv )
∆t tCD = x/2v1 v2
B
Here, n = number of bullets fired per second.
F
∴ n=
mv v2
C to D v 2 = 2gx = gt ⇒ ∴ t=
144 g
= =3
0.04 × 1200 From A to B, time will become two times.

16. X CM = ∫ X dm Applying s = ut +
1 2
at in vertical direction, we have
2
∫ dm
1
− x = (ev 2 )( 2t ) − × g × ( 2t )2
L 2
=
∫0 ( Ax ) ⋅ x ⋅ dx = 2L 2ev 22 2v 22
L
3 −x = −
∫0 ( Ax ) dx g g
RSOS − x = 2e( 2x ) − 2( 2x )
17. e =
3
RSOA ∴ e=
J + ( J − p) 2 J 4
= = −1
p p 21. 2 πR = 2 π ⇒ ∴ R =1
m × 0 + m ×1 + m ×1 2
J J y CM = = ,
J–p m+ m+ m 3
v
⇒ J m( π ) + m(0) + m( 2 π )
A B A B x CM = =π
m+ m+ m
18. Between A and B 22. From conservation of linear momentum, velocity of
m × 10 = mv A + mv B ...(i) combined mass just after collision will be 50cm/s as mass
1 v − vA has doubled.
e= = B
2 10 u2 (0.5)2
Now, H= = m
or vB − vA = 5 ...(ii) 2g 20
10 m/s vA vB = 1.25 cm
A B ⇒ A B External force
23. aCM =
Total mass
Solving Eqs. (i) and (ii), we get Force of friction from ground
v B = 7.5 m/s =
Total mass
Hence, A has given 75% of its speed to B will also transfer its 0.2 × (2 + 1) (10)
= = 2 m /s2
75% speed to C. 1+ 2
75
∴ vC = × 7.5 24. The centre of mass of the object must lie on the line
100 segment joining (0, 0) and ( R / 2, R / 2). Here (0, 0) is the
= 5.625 m/s centre of mass of the ring and ( R / 2, R / 2) is the centre of
19. m ( 3$i + 2$j ) = m( −2$i + $j ) + M v mass of the chord.
(Put M =13 m)
= 1 + 
d d 1 2d
25. T = +
( 5$i + $j ) v / 2 ev / 2  e v
∴ v=
13 2v / 2  1  2d
or = 1 + 
J m = P f − Pi of m g  e v
= [( −2i$ + j$ ) − ( 3$i + 2$j )] −m( 5i$ + $j ) gd
or e= 2
v − gd
∴ JM = + m( 5i$ + j$ )
20. More than One Correct Options
x 1. Momentum remains conserved in any type of equation
x/2 x/2 v/2
m v
x 45°
m
x/2 x/2 Pi = mv

pi = p f
348 Objective Physics Vol. 1

4. Impulse = ∆p = m ( v f − v i )
(mv $i ) = 
mv $ mv $ 
∴ i+ j + ( 2m) v
2 2 2 2  ∴ Impulse recieved by m
v = velocity of mass 2m = m[( −2i$ + $j) − ( 3i$ + 2$j)] = m( −5i$ − $j)
= 0.32 i$ − 0.35$j
Impulse recieved by M = − (impulse received by m )
= (0.32v ) i$ − (0.18v ) $j = m( 5i$ + $j)
v = 0.37v Net pulling force m2 g
0.18 v 5. a = =
tan θ = = 0.5625 Total mass m1 + m2
0.32 v a
∴ θ = 29.35°
m1
0.32v
Q
m2 a
0.18v
v
Since, (θ + 45° ) < 90°. Therefore, the angle of divergence Now, ( acm )x =
m1 a
=
m1m2 g
between particles after collision is less than 90°. m1 + m2 (m1 + m2 )2
1
Further, K i = mv 2 and m2 a  m2 
2
2 ( acm ) y = =  g
2 m1 + m2  m1 + m2 
1 v  1
Kf = m   + ( 2m)(0.37v )2
2  2 2 6. Out of two blocks, one block of mass m is moving in vertical
K f < Ki . direction also (downwards). Therefore, COM is moving
vertically system is not conserved in vertical direction.
Therefore, collision is inelastic.
7. | Impulse| = | ∆p1 or ∆p2 |
2. Just after collision,
 m − 5m  2
vm =   2gl = − 2gl ( 1+2e ) v
 m + 5m  3 m m
2 V 1

 2×m  2gl 3v
v5 m =   2gl = =
 m + 5m  3 ( 1–2e ) v = v
4
4

mvm2 m  8gl 
= m  v  =
T − mg = =  3 3mv

l l  9  4  4
17 Loss of kinetic energy = K i − K f
∴ T = mg
9
1 2
v  3
2
mv 2 −  m   + m    = mv 2
1 3v 1
vm2 4l =
hm = = 2  4 2  4   16
2g 9 2
3. u cos θ = v cos φ ...(i) 8. External force gravity acts on system. Therefore,
momentum of system is not conserved.
v sin φ = eu sin θ Mass keep on decreasing. Therefore, acceleration will keep
or eu sin θ = v sin φ ...(ii) on increasing.
u cosθ v sinφ
θ v
Comprehension Based Questions
u φ 1. Impulse = mu
u sinθ v cosθ
Impulse 4
From Eqs. (i) and (ii), we can see that, ∴ u= = = 2 m/s
m 2
tan φ = e tan θ
Now, Ei = E f
Momentum or velocity changes only in vertical direction.
1 1
∴ | Impulse| = | ∆P | ∴ × 2 × ( 2) + × 4000 × (0.05)2
2
2 2
= m (usin θ + eu sin θ)
1
= m (1 + e ) u sin θ = × 2 × v2
2
v = (v cos φ)2 + (v sin φ)2
Solving we get,
= (u cos θ)2 + (eu sin θ)2 v = 3 m/s
= u (cos θ + e sin θ)
2 2 2 2 2. Again using,
Ei = E f
= u 1 − (1 − e 2 )sin 2 θ
1 1
1 ∴ × 2 × ( 2) + × 4000 × (0.05)2
2
mv 2 2 2
Kf v2
=2 = 2 = cos2 θ + e 2 sin 2 θ 1
K i 1 mu2 u = × 4000 × x 2
2
2
Centre of Mass, Conservation of Linear Momentum, Impulse and Collision 349

∴ x = 0.067 m = 6.7 m = compression 14. Two bodies are released from rest in space. Net force on the
∴ d = 5 cm + 6.7 cm + 6.7 cm + 6.7 cm system is zero. Momentum of system is constant. But
momentum of individual body is not constant. Further,
≈ 25 cm
kinetic energy of system is also increasing.
3. pi = p f
F F
1 2
∴ 0 = (8m)( 2v ) − (16m)(v ) + ( 48m) v ′
Here, v′ = absolute speed of rod mA ( + v ) + mB ( −v )
=0 15. v CM = = − ve
mA + mB
∴ Displacement of rod = 0
Force of friction, f B > f A
4. X A + X B =12 L
f B is acting towards right and f A towards left.
v
XA XB 16. Momentum is always constant during any type of collision.
17. Gravity forces act during explosion.
∴ 2vt + vt = 12 L 18. Mass of second liquid is more. Therefore, centre of mass of
∴ vt = 4L ⇒ X B = vt = 4L second liquid is at lower heights.
5. ( 24 m)  − u = ( 48 m) u p2
v
19. K =
2  2m
v –u As momentum is equal and opposite.
2 1
u ∴ K∝
24 m 48 m m
v 20. Only in case of perfectly inelastic collision they will come to
∴ u=
6 rest.
6. t = t1 + t2 21. Till then spring will remain compressed. Therefore, a force
will act on block A from the wall.
 12L − 4L  20L
=   + 
4L
= 22. Force of friction will act on B towards right (from ground).
 v   v/2  v
23. Atmosphere is absent on the surface of moon. Gravity is
L present.
But =T = 4s
v
∴ t = 80 s Match the Columns
m1v1 + m2v 2
7. Till t1 , rod is stationary. For time t2 , rod is moving with 2. v CM = = 1 m/s
absoulte speed u ( = v /6) m1 + m2

∴ Displacement of rod =   t2
v During maximum compression also, velocity of combined
 6 mass is 1 m/s
Now, Umax = K i − K f
=   
v 16L  8L
= 1 1
 6  v  3 = × 2( 2)2 − × 4 × (1)2 = 2 J
2 2
Assertion and Reason From
1
KX max = 2 J
2
2
2. It will fall at the same point.
We have, X max =1 m
3. Path of centre of mass remains unchanged.
3. p1 + p2 = p ...(i)
4. Rocket does not push the surrounding air, but Newton’s
p,K p1 p2
third law acts between rocket and the exhaust gases. m m ⇒ m 2m
7. Weight is the external force.
8. Centre of mass remains stationary. Further, K1 + K 2 = K
| RVOS | p12 p2 p2
9. e = or + 2 =
| RVOA | 2m 4m 2m
In elastic collision, e =1 or 2p12 + p22 = 2p2 ...(ii)
∴ | RVOS | = | RVOA | Solving these two equations, we get
11. In elastic collision magnitude of relative velocity of 4 p K
p2 = p and p1 = − , K 1 =
separation = magnitude of relative velocity of approach. 3 3 9
Relative speed after collision = Relative speed before 8K
and K2 =
collision 9
5 m/s 2 m/s p2
4. (a) K =
2m
For example in the shown figure, ( 3p)2  p2 
K′= = 9  = 9K
| RVOA | = 7 m/s 2m  2m 
But, relative speed = 3 m/s ∴ % increase in K = 800%
350 Objective Physics Vol. 1

(b) p = 2km
Entrance Gallery
p′ = 2( 4K ) m = 2 2km = 2p
1. In elastic collision, both momentum and kinetic energy are
∴ % increase in p =100% conserved.
(c) p = 2km = ( 2km)1/ 2 2. We know that,
2
1 m −m2 
% increase in p = (% increase in K ) = 0.5% Acceleration, aCM =  1  ×g (Qm1 > m2 )
2  m1+ m 2 
m x + K + mn xn
5. Apply X CM = 1 1 So, resultant external force,
m1 + K + mn 2
 m − m2 
m y + K + mn yn F = (m 1 + m 2 ) a CM = (m 1 + m 2 ) ×  1  ×g
and YCM = 1 1 m + m 
m1 + K + mn  1 2

m1v1 + m2v 2 (1)(10) + ( 2)( −5) (m1 − m 2 )2


6. v CM = = =0 F= ×g
m1 + m2 3 (m1 + m 2 )
Similarly, pCM = 0 3. (a) At equilibrium (t = 0), particle has maximum velocity
Net force on the system is zero, hence, V cm and Pcm will umax . Therefore, velocity at time t can be written as,
remain constant. 0.5u0
u0
Velocity of 1 kg and 2 kg blocks keep on changing initially
and finally both of them stop as V cmwas zero.
m a + m2 a2 C A B
7. From aCM = 1 1 u0
20 m/s
m1 + m2
u = umax cosωt + u0 cosωt ⇒ u = 0.5 u0 = u0 cos ωt
(1)( −10) + 2( −10) π 2π π
= = −10 m /s2 ∴ ωt = ⇒ t= ⇒ t=
T
3 CM 3 T 3 6
m u + m2u2 (1)(60) + ( 2)(0)
uCM = 1 1 = T T T 2π m
m1 + m2 3 (b) t = tAB + tBA = + = =
6 6 3 3 k
= + 20 m/s 10 m/s 2 T T T 7 7π m
(c) t = tAB + tBA + tAC = + + = T=
8. When e =1, collision is elastic and equal 6 6 4 12 6 k
masses exchange their velocities. (d) t = tAB + tBA + tAC + tCA
For e = 0, collision is perfectly inelastic. Hence, velocity of T T T T 5 5π m
= + + + = T=
each will remain half. 6 6 4 4 6 3 k
3 4. Velocity of first bob at highest point,
In the last case, when v 2 = v
4 v1 = gR = gl1 (to just complete the vertical circle)
v
Then, v1 = (from conservation of momentum) Velocity of second bob just after elastic collision
4
= Velocity of second bob at the bottom most point
3v v
− l1
1 = 5gl 2 ⇒ =5
∴ e= 4 4 = l2
v 2
9. F = F + F = m a + m a = ( 2$i + 8$j ) 5. Maximum energy loss
CM 1 2 1 1 2 2
p2 p2  p2 
∴ | FCM | = 4 + 64 = − QKE = 
2m 2(m + M )  2m 
CM = 68 unit
m v + m2 v 2 Before collision, the mass is m and after collision mass is
v CM = 1 1 m + M.
m1 + m2
$ + ( 2)( 4j$ ) 4$i + 8j$ p2  M  1  M  1
(1)( 4i) = = mv 2   = mv f
2
=
3
=
3 2m  (m + M )  2 m + M  2
∴ | v CM | =
1
16 + 64 =
80
unit  M 
∴ f = 
3 3  m+ M 
2
s 1 = ∫ v 1 dt = ( 4$i )
0 Sol.6 and 7
s 2 = ∫ v 2 dt =  $j 
2 8 (i) Every particle of the disc is rotating in a horizontal circle.
0 3  (ii) Actual velocity of any particle is horizontal.
m s + m2s 2
Now, s CM = 1 1 (iii) Magnitude of velocity of any particle is given by
m1 + m2
v = rω
(1)( 4$i ) + 2  $j 
8
where, r is the perpendicular distance of that from actual
 3   4 16 $ 
= = + j axis of rotation (z-axis).
3  3$i 9 
(iv) When it is broken into two parts, then actual velocity of any
16 256 20 particle is resultant of two velocities,
∴ | s CM | = + =
9 81 9 v1 = r1ω1 and v 2 = r2 ω2
Centre of Mass, Conservation of Linear Momentum, Impulse and Collision 351

Here, We have, 20 = v1 × 1 or v1 = 20 m/s


r1 = perpendicular distance of centre of mass from z-axis 100 = v 2 × 1 or v 2 = 100 m/s
ω1 = angular speed of rotation of centre of mass from Now, from conservation of linear momentum before and
z-axis after collision, we have
r2 = distance of particle from centre of mass 0.01v = (0.2 × 20) + (0.01 × 100)
ω2 = angular speed of rotation of the disc about the axis On solving, we get
passing through centre of mass v = 500 m/s
(v) Net v will be horizontal, if v1 and v 2 both are horizontal. 11. Momentum of third piece,
Further, v1 is already horizontal, because centre of mass is px = 2×8 = 16
rotating about a vertical z-axis. To make v 2 also be p = px2 + p2y = (16)2 + (12)2
vertical. p = 20 kg-m/s
8. Along y -axis, momentum remains zero. Here, p 20
v= = = 40 m/s
v1 m 0.5
py = 1×12 = 12
30° 12. Change in momentum
60°
Force acted on the bat
∆p = 2mv = 2 × 0.25 × 10 = 5 kg-m/s
v2
∆p 5
F= = = 500 N
4v1 sin 30° = v 2 sin 60° ∆ t 0.01
v1 3
= 13. Momentum of 5kg mass after the collision is 4 kg m/s.
v2 4 m1 m2
9. The data is incomplete. Let us assume that friction from v1 v2
ground on ring is not impulsive during impact.
14. If it is a completely inelastic collision then,
From linear momentum, conservation is in the horizontal
direction, we have m1v1 + m 2v 2 = m1v + m 2v
–ve +ve m v + m 2v 2
v= 1 1
m1 + m 2
( −2 × 1) + (0.1 × 20) = (0.1 × 0) + (2 × v )
p12 p2
Here, v is the velocity of centre of mass of ring after impact. KE = + 2
2m1 2m 2
Solving the above equation, we have
v =0 As, p1 and p 2 both simultaneously cannot be zero therefore,
total kinetic energy cannot be lost.
Thus, centre of mass becomes stationary.
15. For translatory motion the force should be applied on the
Linear impulse during impact
centre of mass of the body so, we have to calculate the
(i) In horizontal direction, J1 = ∆p = 0.1 × 20 = 2 N-s location of centre of mass of T shaped object.
(ii) In vertical direction, J 2 = ∆p = 0.1 × 10 = 1 N-s Let mass of rod AB is m so the mass of rod CD will be 2m.
Let y1 is the centre of mass of rod AB and y 2 is the centre of
1 N-s
mass of rod CD. We can consider that whole mass of the rod
is placed at their respective centre of mass i.e. mass m is
2 N-s placed at y1 and mass 2m is placed at y 2 .
ω Taking point C at the origin position vector of points
+ve _ve y1 and y 2 an be written as
r1 = 2l j$ , r2 = l j$
Writing the equation (about CM),
and m1 = m and m2 = 2m
angular impulse = change in angular momentum
Position vector of centre of mass of the system,
We have,
m r + m2 r2 m2l j$ + 2ml j$ 4ml $j 4l $j
 3 1 rCM = 1 1 = = =
×  − 2 × 0.5 × = 2 × (0.5)2 ω −
1 1 
1×  m1 + m2 m + 2m 3m 3
 2 2  2 
 0.5 
16. From the graph, it is a straight line so, it represents uniform
Solving this equation, ω comes out to be positive or motion. Because of impulse direction of velocity changes as
ω anti-clockwise. So just after collision rightwards slipping can be seen from the slope of the graph.
is taking place. Hence, friction is leftwards. Therefore, 2
Initial velocity, v 1 = = 1 ms−1
option (c) is also correct. 2
10. Time taken by the bullet and ball to strike the ground is −2
Final velocity, v 2 = = − 1 ms−1
2h 2× 5 2
t= = =1 s
g 10 p i = mv 1 = 0.4 N-s
Let v1 and v 2 are the velocities of ball and bullet after p f = m v 2 = − 0.4 N-s
collision. Then, applying J = p f − p i = − 0.4 − 0.4 = − 0.8 N-s
x = vt ∴ | J | = 0.8 N-s
9
Rotation

9.1 Moment of Inertia


Like the centre of mass, the moment of inertia is a property of an object that is related Chapter Snapshot
to its mass distribution. The moment of inertia (denoted by I) is an important quantity in ● Moment of Inertia
the study of system of particles which are rotating. The role of the moment of inertia in the
● Torque
study of rotational motion is analogous to that of mass in the study of linear motion.
Moment of inertia gives a measurement of the resistance of a body to a change in its ● Rotation of a Rigid Body
rotational motion. If a body is at rest, the larger the moment of inertia of a body, the more about a Fixed Axis
difficult it is to put that body into rotational motion. Similarly, the larger the moment of ● Angular Momentum
inertia of a body, the more difficult it is to stop its rotational motion. The moment of inertia ● Conservation of Angular
is calculated about some axis (usually the rotational axis) and it depends on the mass as Momentum
well as its distribution about that axis. ● Combined Translational
and Rotational Motion of
Moment of Inertia of a Single Particle a Rigid Body
For a very simple case, the moment of inertia of a single particle about ● Uniform Pure Rolling
an axis is given by ● Accelerated Pure Rolling
I = mr 2 …(i) r
m ● Angular Impulse
Here, m is the mass of the particle and r its distance from the axis
under consideration.
Fig. 9.1
Moment of Inertia of a System of Particles
The moment of inertia of a system of particles about an r1
m1
axis is given by
I = Σ mi ri2 …(ii) r2
i m2
r3
where, ri is the perpendicular distance from the axis to the ith m3
particle, which has a mass mi .

Moment of Inertia of Rigid Bodies Fig. 9.2

For a continuous mass distribution such as found in a rigid body, we replace the
summation of Eq. (ii) by an integral. If the system is divided into infinitesimal elements of
mass dm and if r is the distance from a mass element to the axis of rotation, the moment of
inertia is
Rotation 353

I = ∫ r 2 dm dI = r 2 dm = r 2 (ρdV )
Here, ρ = density of cylinder
r and dV = volume of shell = 2πrl dr
dm ∴ dI = 2πρl r 3 dr
The cylinder’s moment of inertia is found by integrating
this expression between 0 and R
R πρl 4
Fig. 9.3 So, I = 2πρl ∫ r 3 dr = R …(iii)
0 2
where, integral is taken over the system.
The density ρ of the cylinder is the mass divided by the
Moment of Inertia of a Uniform Cylinder volume.
M
Let us find the ∴ ρ= …(iv)
moment of inertia of a πR 2 l
uniform cylinder about R From Eqs. (iii) and (iv), we have
dr 1
an axis through its centre l r I = MR 2
of mass and 2
perpendicular to its base. Proceeding in the similar manner we can find the
Mass of the cylinder is M moment of inertia of certain rigid bodies about some given
and radius is R. (a) (b) axis. Moments of inertia of several rigid bodies with
We first divide the Fig. 9.4 symmetry are listed in Table. 9.1.
cylinder into annular shells of width dr and length l as shown In all cases except ( f ), the rotational axis AA′ passes
in figure. The moment of inertia of one of these shells is through the centre of mass.

Table 9.1
(a) Very thin circular hoop A′ (b) Uniform circular hoop A′
R1
I = MR 2 R R12 + R 22
I=M
2
R2
A A

(c) Uniform solid cylinder A (d) Uniform solid sphere A′

1 2
I= MR 2 R I= MR 2
2 5
R

A′
(e) Uniform thin rod (f) Uniform thin rod A
A
1 1 2
I= Ml 2 I= Ml
12 3
l l

A′
A′
354 Objective Physics Vol. 1

(g) Very thin spherical shell A′ (h) Thin circular sheet A

2 1
I= MR 2 I= MR 2 R
3 4
R

A
A′
(i) Thin rectangular sheet (j) Uniform right cone
A′

 a2 + b 2  3
I=M   I= MR 2
 12  a 10
b R
A A′

Theorem on Moment of Inertia the lamina is equal to the sum of the moments of inertia of
the lamina about two axes perpendicular to each other, in its
There are two important theorems on moment of inertia, own plane and intersecting each other at the point, where the
which in some cases, enable the moment of inertia of a body perpendicular axis passes through it. Let x and y axes be
to be determined about an axis, if its moment of inertia about chosen in the plane of the body and z-axis perpendicular to
some other axis is known. Let us now discuss both of them. this plane, three axes being mutually perpendicular, then the
theorem states that
Theorem of Parallel Axis z y
A very useful theorem, called the
parallel axis theorem relates the moment
of inertia of a rigid body about two
parallel axes, one of which passes CM xi
through the centre of mass. P
ri
Two such axes are shown in figure yi
r o x
for a body of mass M. If r is the distance
between the axes and I CM and I are the
Fig. 9.5
respective moments of inertia about
Fig. 9.7
them, these moments are related by
I = I CM + Mr 2 I z = Ix + I y
/ From the above theorem we can see that among too many
parallel axes moment of inertia is least about an axis which
Radius of Gyration
passes through centre of mass. e.g.I 2 is least among I1, I 2 and I 3 . Radius of gyration ( K ) of a body about an axis is the
Similarly, I 5 is least among I 4 , I 5 and I 6 . effective distance from this axis, where the whole mass can
1 be assumed to be concentrated so that the moment of inertia
2 remains the same. Thus,
3 4
5
CM CM
6

K M
Fig. 9.6 ⇒
M

Theorem of Perpendicular Axis


This theorem is applicable only to the plane bodies (two
dimensional). The theorem states that the moment of inertia Fig. 9.8
of a plane lamina about an axis perpendicular to the plane of
Rotation 355

I
I = MK 2 or K=
M Extra Knowledge Points
e.g. Radius of gyration of a disc about an axis ■ Theorem of parallel axis is applicable for any type of
perpendicular to its plane and passing through its centre of rigid body whether it is a two dimensional or three
mass is dimensional, while the theorem of perpendicular axis is
applicable for laminar type or two dimensional bodies
1
MR 2 only.
2 R
K= = ■ In theorem of perpendicular axis, the point of
M 2 intersection of the three axes (x, y and z) may be any
point on the plane of body (it may even lie outside the
X Example 9.1 Three rods each of mass m and body). This point may or may not be the centre of mass
length l are joined together to form an equilateral of the body.
triangle as shown in figure. Find the moment of
■ Moment of inertia of a part of a rigid body
(symmetrically cut from the whole mass) is the same as
inertia of the system about an axis passing through its that of the whole body. e.g. in Fig. (a), moment of
centre of mass and perpendicular to the plane of the inertia of the section shown (a part of a circular disc)
triangle. about an axis perpendicular to its plane and passing
A 1
through point O is MR 2 as the moment of inertia of
2
1
the complete disc is also MR 2 . This can be shown as
2
in figure.
CM 1
Suppose, the given section is th part of the disc, then
n
mass of the disc will be nM.
B C
Fig. 9.9

Sol. Moment of inertia of rod BC about an axis perpendicular to R


plane of triangle ABC and passing through the mid-point of rod O M
BC (i.e. D) is
A

(a) (b)

1
CM I disc = (nM )R 2
2
r 1 1
30° ∴ I section = I disc = MR 2
B C n 2
D
■ If a rigid body is rotating about a fixed axis with angular
Fig. 9.10
speed ω, all the particles in rigid body rotate same
ml 2 angle in same interval of time, i.e. their angular speed
I1 = is same (ω ). They rotate in different circles of different
12
From theorem of parallel axis, moment of inertia of this rod radii. The planes of these circles are perpendicular to
about the asked axis is the rotational axis. Linear speeds of different particles
I2 = I1 + mr 2 are different. Linear speed of a particle situated at a
2
distance r from the rotational axis is
ml 2
+ m 
l 
=  ω
12 2 3
ml 2
=
6 r
∴ Moment of inertia of all the three rods is P
 ml 2 
I = 3I2 = 3  
 6 
ml 2
=
2
356 Objective Physics Vol. 1

X Example 9.2 Find the moment of inertia of a Torque of a Force about a Line
solid sphere of mass M and radius R about an axis xx' Consider a rigid body rotating about a fixed axis AB. Let
shown in figure. F be a force acting on the body at point P. Take the origin O
x somewhere on the axis of rotation. The torque of Fabout O is

P
O r
x' F
Fig. 9.11

Sol. From theorem of parallel axis, x


B
Ixx' = ICM + Mr 2
CM Fig. 9.15
2
= MR 2 + MR 2
5
7 x τ = r ×F
= MR 2 r=R
5 Its component along AB is called the torque of F about
Fig. 9.12 AB.
X Example 9.3 Consider a uniform rod of mass m
and length 2l with two particles of mass m each at its
X Example 9.4 A small ball of mass 1.0 kg is
attached to one end of a 1.0 m long massless string
ends. Let AB be a line perpendicular to the length of
and the other end of the string is hung from a point.
the rod and passing through its centre. Find the
When the resulting pendulum is 30° from the vertical,
moment of inertia of the system about AB.
what is the magnitude of torque about the point of
Sol. MI of the system about AB A
suspension? (Take g =10 m / s 2 )
IAB = Irod + Iboth particles
Sol. Two forces are acting on the ball
m(2 l )2 l l
(i) Tension ( T ) (ii) Weight (mg )
= + 2 (ml 2 )
12 m m Torque of tension about point O is zero, as it passes
7
= ml 2 through O.
3 O r⊥
P
v = rω B
or v ∝r Fig. 9.13 30°
1.0 m

9.2 Torque T 30°


Suppose a force F is acting on a particle P and let r be
the position vector of this particle about some reference
point O. The torque of this force F, about O is defined as mg

P Fig. 9.16
F
τ mg = F × r⊥
Here, r⊥ = OP = 1.0 sin 30°
r

= 0.5 m
∴ τ mg = (mg )(0.5)
= (1)(10)(0.5)
O
Fig. 9.14 = 5 N -m

τ =r ×F X Example 9.5 Find the torque of a force


F = ( $i + 2$j − 3k$ ) N about a point O. The position
This is a vector quantity having its direction
perpendicular to both r and F, according to the rule of cross vector of point of application of force about O is
product. r = (2$i + 3$j − k$ ) m .
Rotation 357

Sol. Torque, τ = r × F
$i $j k$
Angular Acceleration
= 2 3 −1 = $i (−9 + 2 ) + $j (−1 + 6) + k$ (4 − 3) The angular acceleration α measures the time rate of
1 2 −3 change of the angular velocity. Hence, the magnitude of this
vector may be written as,
or τ = (−7 $i + 5$j + k)N-m
$

α= …(ii)
9.3 Rotation of a Rigid Body dt
It is also possible to express α as,
about a Fixed Axis
d 2θ
When a body is rotating about a fixed axis, any point P α=
located in the body travels along a circular path. Before, dt 2
analysing the circular motion of point P, we will first study
The line of action of α is the same as that for ω, however
the angular motion properties of a rigid body.
its sense of direction depends on whether ω is increasing or
Angular Motion decreasing with time. In particular if ω is decreasing, α is
dθ called an angular deceleration and therefore, has a sense of
Since, a point is without
dimension, it has no angular direction which is opposite to ω.
α
motion. Only lines or bodies
undergo angular motion. Let us ω
Torque and Angular Acceleration for a
consider the angular motion of a Rigid Body
radial line r located with the The angular acceleration of a rigid body is directly
shaded plane. proportional to the sum of the torque components along the
axis of rotation. The proportionality constant is the inverse
Angular Position of the moment of inertia about that axis or
The angular position of r is
defined by the angle θ, measured
O Στ
α=
between a fixed reference line OA I
A
and r. dθ Thus, for a rigid body we have the rotational analog of
Newton’s second law
Angular Displacement
Στ = Iα …(iii)
The change in the angular Fig. 9.17
position, often measured as a Following two points are important regarding the
differential d θ is called the angular displacement. (Finite above equation
angular displacements are not vector quantities, although (i) The above equation is valid only for rigid bodies. If
differential rotations d θ are vectors). This vector has a
the body is not rigid like a rotating tank of water, the
magnitude dθ and the direction of d θ is along the axis.
angular acceleration α is different for different
Specifically, the direction of d θ is determined by right particles.
hand rule; that is, the fingers of the right hand are curled with
the sense of rotation, so that in this case the thumb or d θ (ii) The sum Στ in the above equation includes only the
points upward. torques of the external forces, because all the internal
torques add to zero.
Angular Velocity α
The time rate of change in
ω Rotation with Constant Angular
the angular position is called the Acceleration
angular velocity ω. Thus, O
If the angular acceleration of the body is constant, then
dθ r
ω= …(i) A Eqs. (i) and (ii) when integrated yield a set of formulae
dt P which relate the body’s angular velocity, angular position

It is expressed here in scalar θ and time. These equations are similar to equations used for
form, since its direction is rectilinear motion. Table given below compares the linear
Fig. 9.18
always along the axis of
rotation, i.e. in the same direction as d θ. and angular motion with constant acceleration.
358 Objective Physics Vol. 1

Table 9.2 1 1
K =Σ mi v i2 = Σ mi (ωri ) 2
Straight line motion with Fixed axis rotation with constant 2 i i 2
constant linear acceleration angular acceleration 1 2 1
a = constant α = constant = ω Σ mi ri2 = Iω 2 (as Σ mi ri2 = I )
2 i 2 i
v = u + at ω = ω 0 + αt
1 2 1 1
s = s 0 + ut + at θ = θ0 + ω 0 t + αt 2 Thus, KE = Iω 2
2 2 2
2 2
v = u + 2 a (s − s0 ) ω 2 = ω 20 + 2α (θ − θ 0 ) Sometimes it is called the rotational kinetic energy.
Here, θ 0 and ω 0 are the initial values of the body’s X Example 9.6 The angular position of a point on
angular position and angular velocity respectively. the rim of a rotating wheel is given by
θ = 4t − 3t 2 + t 3 , where θ is in radians and t is in
Motion of Point P seconds. What are the angular velocities at
As the rigid body as shown in figure rotates, point P travels (a) t = 2.0 s and
along a circular path of radius r and centre at point O. (b) t = 4.0 s
(c) What is the average angular acceleration for the time
interval that begins at t = 2.0 s and ends at t = 4.0 s?
(d) What are the instantaneous angular acceleration at
ω the beginning and the end of this time interval?
Sol. Angular velocity
O
O dθ d
r v r v ω= = (4t − 3 t 2 + t 3 )
φ P dt dt
Q
rP or ω = 4 − 6t + 3t 2
P
(a) At t = 2.0 s,
ω = 4 − 6 × 2 + 3(2 )2
Fig. 9.19 or ω = 4 rad/s
(b) At t = 4.0 s,
Position ω = 4 − 6 × 4 + 3(4)2
or ω = 28 rad/s
The position of P is defined by the position vector r,
(c) Average angular acceleration
which extends from O to P. ω − ωi 28 − 4
αav = f =
tf − ti 4−2
Velocity
or αav = 12 rad/s 2
The velocity of P has a magnitude given by,
(d) Instantaneous angular acceleration is
v = ωr …(iv) dω d
α= = (4 − 6 t + 3 t 2 )
As shown in figures the direction of velocity v is dt dt
tangential to the circular path. or α = − 6 + 6t
At t = 2.0 s,
Kinetic Energy of a Rigid Body Rotating α = − 6 + 6 × 2 = 6 rad/s 2

About a Fixed Axis At t = 4.0 s


α = − 6 + 6 × 4 = 18 rad/s 2
Suppose a rigid body is rotating about a fixed axis with
angular speed ω. Then, kinetic energy of the rigid body X Example 9.7 A solid fly wheel of 20 kg mass
will be and 120 mm radius revolves at 600 rev/min. With what
ω force must a brake lining be pressed against it for the
flywheel to stop in 3 s, if the coefficient of friction
ri is 0.1?
mi
Sol. n0 = 600 rev / min (revolutions per minute)
600
= rev / min
60
= 10 rev /s (revolutions per second)
So, initial angular velocity,
Fig. 9.20 ω0 = 2 πn0 = (2 π )(10) = 20π rad/s
Rotation 359

Let α be the constant angular retardation, then applying


ω = ω0 − αt 9.4 Angular Momentum
or 0 = (20 π ) − 3(α ) A mass moving in a straight line has linear momentum
20 ( p). When a mass rotates about some point/axis, there is
or α= π rad/s 2
3 momentum associated with rotational motion called the
τ angular momentum (L ). Just as net external force is required
Further, α=
I to change the linear momentum of an object a net external
Here, τ = µNR (R = radius) torque is required to change the angular momentum of an
or τ = µFR (F = applied force) object. The angular momentum is classified in following
1 two types
as N=F and I= mR 2
2 A θ
From the above equations,
20 µFR 2 µF p = mv
π= =
3 1 2 mR r r⊥ = r sin θ
mR
2
10 πmR θ
or F=
3µ O
Substituting the values, we have Fig. 9.21
10 × 22 × 20 × 0.12
F=
3 × 7 × 0.1
or F = 251.43 N
Angular Momentum of a Particle
about Some Point
X Example 9.8 A wheel rotates around a stationary Suppose a particle A of mass m is moving with linear
axis so that the rotation angle θ varies with time as momentum p = m v. Its angular momentum L about point O
θ = at 2, where α = 0.2 rad/s 2 . Find the magnitude of is defined as
net acceleration of the point A at the rim at the L = r × p = r × ( m v ) = m( r × v )
moment t = 2.5 s, if the linear velocity of the point A at Here, r is the radius vector of particle A about O at that
this moment is v = 0.65 m/s. instant of time. The magnitude of L is
Sol. Instantaneous angular velocity at time t is L = mvr sin θ
dθ d = mvr⊥
ω= = (at 2 )
dt dt
or ω = 2at = 0.4 t (as a = 0.2 rad/s 2 )
Here, r⊥ = r sin θ is the perpendicular distance of line of
action of velocity v from point O. The direction of L is same
Further, instantaneous angular acceleration is
dω d as that of r × v.
α= = (0.4 t )
dt dt / The angular momentum of a particle about a line (say AB) is
or α = 0.4 rad/s 2 the component along AB of the angular momentum of the
particle about any point (say O) on the line AB. This
Angular velocity at t = 2.5 s is component is independent of the choice of point O, so far as
ω = 0.4 × 2.5 = 1.0 rad/s it is chosen on the line AB.
Further, radius of the wheel
v X Example 9.9 A particle of mass m is moving
R=
ω along the line y = b, z = 0 with constant speed v. State
0.65
or R= = 0.65 m whether the angular momentum of particle about
1.0
Now, magnitude of total acceleration is
origin is increasing, decreasing or constant.
a= an2 + a2t Sol. | L | = mvr sin θ = mvr⊥ = mvb y
P
2 v
Here, an = Rω ∴ |L | = constant as m, v and b
= (0.65)(1.0)2 all are constants. r r⊥ = b
Direction of r × v also remains
= 0.65 m /s 2
the same. Therefore, angular θ
and at = Rα momentum of particle about O
x
= (0.65)(0.4) origin remains constant with due
course of time. Fig. 9.22
= 0.26 m/s 2
/ In this problem, | r | is increasing, θ is decreasing but r sin θ,
∴ a = (0.65)2 + (0.26)2 i.e. b remains constant. Hence, the angular momentum
or a = 0.7 m/s 2 remains constant.
360 Objective Physics Vol. 1

y
X Example 9.10 A particle
of mass m is projected from
P
v
9.5 Conservation of Angular
origin O with speed u at an r r⊥ = b Momentum
angle θ with positive x-axis. As we have seen in Article 9.4, the angular momentum
Positive y-axis is in vertically θ
O
x of a particle about some reference point O is defined as,
upward direction. Find the L=r×p …(i)
Fig. 9.23
angular momentum of particle
at any time t about O before the particle strikes the Here, p is the linear momentum of the particle and r its
ground again. position vector with respect to the reference point O.
Differentiating Eq. (i) with respect to time, we get
Sol. L = m(r × v ) dL dp dr
=r× + ×p …(ii)
Here, r (t ) = x$i + y$j dt dt dt
1 2 $
= (u cos θ)t $i + (ut sin θ − gt ) j dp dr
2 Here, = F and = v (velocity of particle)
dt dt
and v(t ) = v x $i + v y $j
Hence, Eq. (ii) can be rewritten as,
= (u cos θ)$i + (u sin θ − gt )$j
∴ L = m (r × v )
dL
= r ×F+ v × p
$i $j k$ dt
1 Now, v × p = 0, because v and p are parallel to each
= m d (u cos θ)t (u sin θ)t − gt 2 0
2 other and the cross product of two parallel vectors is zero.
u cos θ u sin θ − gt 0
Thus,
2 2
= m[(u sin θ cos θ)t − (u cos θ) gt dL dL
1 = r × F = τ or τ = …(iii)
− (u 2 sin θ cos θ)t + (u cos θ) gt 2  k$ dt dt
2 
1
Which states that the time rate of change of angular
=− m(u cos θ) gt 2 k$ momentum of a particle about some reference point in an
2
inertial frame of reference is equal to the net torques acting
Angular Momentum of a Rigid Body dp
on it. This result is rotational analog of the equation F = ,
dt
Rotating about a Fixed Axis which states that the time rate of change of the linear
Suppose a particle P of mass m is A
ω momentum of a particle is equal to the force acting on it.
going in a circle of radius r and at some Eq. (iii) like all vector equations, is equivalent to three scalar
instant the speed of the particle is v. r
O P
equations, namely
For finding the angular momentum of
 dL   dL   dL 
the particle about the axis of rotation, τ x =   , τ y =   and τ z =  
the origin may be chosen anywhere on  dt  x  dt  y  dt  z
the axis. We choose it at the centre of
the circle. In this case, r and p are The same equation can be generalized for a system of
perpendicular to each other and r × p is dL
B particles as, τ ext = . According to which the time rate of
along the axis. Thus, component of dt
Fig. 9.24
r × p along the axis is mvr itself. The change of the total angular momentum of a system of
angular momentum of the whole rigid body about AB is the particles about some reference point of an inertial frame of
sum of components of all particles, i.e. reference is equal to the sum of all external torques (of
L = Σ mi ri v i course the vector sum) acting on the system about the same
i reference point.
Here, v i = ri ω
dL
∴ L = Σ mi ri2ω Now, suppose that τ ext = 0, then = 0, so that
i dt
or L = ω Σ mi ri2 L = constant.
i
or L = Iω (as Σ mi ri2 = I ) “When the resultant external torque acting on a system
i is zero, the total vector angular momentum of the system
Here, I is the moment of inertia of the rigid body about remains constant. This is the principle of the conservation of
AB. angular momentum.
Rotation 361

For a rigid body rotating about an axis (the z-axis, say) Thus, the velocity of
P v
that is fixed in an inertial reference frame, we have point P is the vector sum of
L z = Iω vp


v CM and v p , CM as shown in 90°

It is possible for the moment of inertia I of a rotating figure.
C v
body to change by rearrangement of its parts. If no net Kinetic energy of rigid
external torque acts, then L z must remain constant and if I ω
body in combined
does change, there must be a compensating change in ω. The translational and rotational
principle of conservation of angular momentum in this case is motion.
expressed as
Iω = constant …(iv) Here, two energies are Fig. 9.26
associated with the rigid body.
X Example 9.11 A wheel of moment of inertia I  1 2 
One is translational  = m vCM  and another is rotational
and radius R is rotating about its axis at an angular  2 
speed ω 0 . It picks up a stationary particle of mass m
 1 2
at its edge. Find the new angular speed of the wheel.  = I CM ω  . Thus, total kinetic energy of the rigid body is
 2 
Sol. Net external torque on the system is zero. Therefore, angular 1 1
momentum will remain conserved. Thus, K = mvCM 2
+ I CM ω 2
Iω 2 2
I1ω1 = I2ω2 or ω2 = 1 1
I2
Here, I1 = I, ω1 = ω0 , I2 = I + mR 2 9.7 Uniform Pure Rolling
Iω0
∴ ω2 = Pure rolling means no relative motion (or no slipping) at
I + mR 2
point of contact between two bodies.

9.6 Combined Translational


CM
v

and Rotational Motion of ω P
Rω v
a Rigid Body Q
Up until now we have considered only bodies rotating (a) (b)
about some fixed axis. Let us consider combined Fig. 9.27
translational and rotational motion of a rigid body. For example, consider a disc of radius R moving with
In such problems if two things are known, linear velocity v and angular velocity ω on a horizontal
(i) velocity of centre of mass ( vCM ) ground. The disc is said to be moving without slipping, if
(ii) angular velocity of the rigid body (ω ). velocities of points P and Q (shown in Fig. b) are equal,
The motion of whole rigid body can be described. vP = vQ
or v − Rω = 0 or v = Rω
If v P > v Q or v > Rω, the motion is said to be forward
slipping and if v P < v Q or v < Rω, the motion is said the
CM
vCM CM vCM backward slipping (or sometimes called forward slipping).
ω
ω
v
O ⇒
(a) (b) ω
P
v0 Rω v0 v
Fig. 9.25
Q
For example, let the velocity of centre of mass of a rigid Fig. 9.28
body shown in figure is v and angular velocity of the rigid
body is ω.Then, velocity of any point P on the rigid body can Thus, v = Rω is the condition of pure rolling on a
be obtained as, stationary ground. Sometimes it is simply said rolling.
v p = v CM + v p ,CM Suppose, the base over which the disc in rolling, is also
Here, | v CM | = v moving with some velocity (say v 0 ), then in that case
and v p, CM = rω in a direction perpendicular to line CP. condition of pure rolling is different.
362 Objective Physics Vol. 1

For example, in the above figure, sometimes act in forward F

v P = v Q or v − Rω = v 0 direction, sometimes in backward


Thus, in this case v − Rω ≠ 0, but v − Rω = v 0 . By direction or under certain
uniform pure rolling we mean that v and ω are constant. They conditions it may be zero. Here,
are neither increasing nor decreasing. we should not forget the basic
nature of friction, which is a self
X Example 9.12 A disc of radius R start at time adjusting force (upto a certain Fig. 9.31
t = 0 moving along the positive x-axis with linear maximum limit) and which has a
speed v and angular speed ω. Find the x and y tendency to stop the relative motion between two bodies in
coordinates of the bottommost point at any time t. contact. Let us take an example illustrating the above theory.
y α
Suppose a force F is applied F
at the topmost point of a rigid
C
body of radius R, mass M and a
v C
ω moment of inertia I about an axis
x passing through the centre of
O f
mass. Now, the applied force F
Fig. 9.29 can produce by itself Fig. 9.32
Sol. At time t the bottommost point will rotate an angle θ = ωt with (i) a linear acceleration a and
respect to the centre of the disc C. The centre C will travel a (ii) an angular acceleration α.
distance s = vt .
y If a = Rα, then there is no need of friction and force of
friction f = 0.If a < Rα, then to support the linear motion the
force of friction f will act in forward direction. Similarly, if
C
C a > Rα, then to support the angular motion the force of
R
θ friction will act in backward direction. So, in this case force
P Q
x
of friction will be either backward, forward or even zero
O M also. It all depends on M, I and R. For calculation you can
s = vt
choose any direction of friction. Let we assume it in forward
Fig. 9.30 direction,
In the figure, PQ = R sin θ = R sin ωt Let, a = linear acceleration, α = angular acceleration
and CQ = R cos θ = R cos ωt then,
Coordinates of point P at time t are, F F+f
x = OM − PQ = vt − R sin ωt a = net = …(i)
and y = CM − CQ
M M
= R − R cos ωt τ (F − f )R
α= c = …(ii)
∴ ( x, y) ≡ (vt − R sin ωt , R − R cos ωt ) I I
For pure rolling to take place,
9.8 Accelerated Pure Rolling a = Rα …(iii)
So far we were discussing the uniform pure rolling in Solving Eqs. (i), (ii) and (iii), we get
which v and ω were constants. Now, suppose an external ( MR 2 − I )
force is applied to the rigid body, the motion will no longer f = .F …(iv)
( MR 2 + I )
remain uniform. The condition of pure rolling on a
stationary ground is From Eq. (iv) following conclusions can be drawn
v = Rω (i) If I = MR 2 (e.g. in case of a ring)
Differentiating this equation with respect to time we f =0
have
dv dω if a force F is applied on the top of a ring, the force of
=R or a = Rα friction will be zero and the ring will roll without
dt dt
slipping.
Thus, in addition to v = Rω at every instant of time, (ii) If I < MR 2 , (e.g. in case of a solid sphere or a hollow
linear acceleration = R × angular acceleration or a = Rα for
sphere), f is positive force of friction will be forward.
pure rolling to take place. Here, friction plays an important
role in maintaining the pure rolling. The friction may (iii) If I > MR 2 , f is negative force of friction will be
Rotation 363

backwards. Although under no condition I > MR 2 . Solving Eqs. (i), (ii) and (iii), we get
(Think why?). So, force of friction is either in forward Mg sin θ
f = …(iv)
direction or zero. MR 2
1+
Here, it should be noted that the force of friction f I
obtained in Eq. (iv) should be less than the limiting g sin θ
friction (µ Mg ), for pure rolling to take place. Further, and a= …(v)
I
we saw that, if I < MR 2 , force of friction acts in 1+
MR 2
forward direction. This is because α is more, if I is
From Eq. (v), we can see that if a solid sphere and a
 τ
small  α =  i.e. to support the linear motion force of hollow sphere of same mass and radius are released from a
 I rough inclined plane, the solid sphere reaches the bottom first
friction is in forward direction. because
/ It is often said that rolling friction is less than the sliding
friction. This is because the force of friction calculated by
I solid < I hollow
equation number (iv) normally comes less than the sliding or a solid > a hollow
friction (µ k N ) and even sometimes it is in forward direction,
i.e. it supports the motion.
∴ t solid < t hollow
There are certain situations in which the direction of Further, the force of friction calculated in Eq. (iv) for
friction is fixed. For example, in the following situations the pure rolling to take place should be less than or equal to the
force of friction is backward. maximum friction µ Mg cos θ.
Mg sin θ
or ≤ µ Mg cos θ
MR 2
Rough
1+
I
tan θ
or µ≥
θ MR 2
1+
I

X Example 9.13 In the arrangement shown in


F figure the mass of the uniform solid cylinder of radius
F R is equal to m and the masses of two bodies are
equal to m1 and m 2 . The thread slipping and the
friction in the axle of the cylinder are supposed to be
Fig. 9.33 absent. Find the angular acceleration of the cylinder
T
and the ratio of tensions 1 of the vertical sections of
Rolling on Rough Inclined Plane T2
As we said earlier also, force of friction in this case will the thread in the process of motion.
be backward. Equations of motion are
M,R,I
a α f
in θ 1
M gs
2
θ m1

Fig. 9.34
m2
Mg sin θ − f
a= …(i) Fig. 9.35
M
fR
α= …(ii) Sol. Let α = angular acceleration of the cylinder
I
and a = linear acceleration of two bodies
For pure rolling to take place, Equations of motion are,
a = Rα …(iii) For mass m1, T1 − m1g = m1a …(i)
364 Objective Physics Vol. 1

For mass m 2 , m 2 g − T2 = m 2 a …(ii) For cylinder, T + f = Ma1 …(ii)


(T − T1 ) R (T − f ) R
For cylinder, α= 2 …(iii) α= …(iii)
1 1 MR 2
mR 2 2
2
α The string attaches the mass m to the highest point of the
cylinder, hence
T1 T2
v m = vCM + Rω
Differentiating the above equation, we get
m1 a m2 a
a2 = a1 + R α …(iv)
We also have (for rolling without slipping)
T1 T2 m 1g m 2g
a1 = R α …(v)
Fig. 9.36 Solving these equations, we get
8 mg
For no slipping condition a2 =
3M + 8 m
a = Rα …(iv)
Solving these equations, we get
α=
2 (m 2 − m1) g Alternate Solution (Energy Method)
( 2 m1 + 2 m 2 + m ) R Since, there is no slipping at all contacts mechanical
T1 m1(m + 4 m 2 ) energy of the system will remain conserved.
and =
T2 m 2 (m + 4 m1) ∴ Decrease in gravitational potential energy of block
m in time t = increase in translational kinetic energy of block
X Example 9.14 Consider the arrangement shown + increase in rotational as well as translational kinetic
in figure. The string is wrapped around a uniform energy of cylinder.
cylinder which rolls without slipping. The other end of 1 1 1
the string is passed over a massless, frictionless pulley ∴ mgh = mv 22 + Iω 2 + Mv12
2 2 2
to a falling weight. Determine the acceleration of the
1  1
falling mass m in terms of only the mass of the or mg  a 2 t 2  = m ( a 2 t ) 2
cylinder M, the mass m and g. 2  2
1 1 2 1
+  MR  (αt ) + M ( a 1 t ) …(vi)
2 2
M 
2 2  2
Solving Eqs. (iv), (v) and (vi), we get the same result.

m 9.9 Angular Impulse


Fig. 9.37 The angular impulse of a torque in a given time interval
t2
is defined as ∫ τ dt
Sol. Let T be the tension in the string and f the force of (static) t1
friction, between the cylinder and the surface
Here, τ is the resultant torque acting on the body.
T
Further, since
α T dL
a1
a2 τ=
dt
f
mg
∴ τ dt = d L
t2
Fig. 9.38 or ∫t 1
τ dt = angular impulse
a1 = acceleration of centre of mass of cylinder towards right,
= L 2 − L1
a2 = downward acceleration of block m,
α = angular acceleration of cylinder (clockwise) Thus, the angular impulse of the resultant torque is
Equations of motion are equal to the change in angular momentum. Let us take an
For block, mg − T = ma2 …(i) example based on the angular impulse.
Chapter Summary with Formulae
■ Moment of Inertia (vi) Solid sphere
(i) Thin rod 2 1
3 2
5 4
θ θ 1

x
6
2 2 2
I1 = 0, I2 =
ml
, I3 =
ml
, I1 = mR2
12 3 5
7
ml2 ml2 I2 = I1 + mR2 = mR2
I4 = sin2 θ, I5 = sin2 θ, 5
12 3
I6 = mx2 (vii) Hollow sphere

(ii) Circular disc 2 1


4 2
mR2
I1 = I2 =
4
mR2
I3 = I1 + I2 = 1
2 x x
3
5 5
I4 = I2 + mR2 = mR2
4
3
I5 = I3 + mR2 = mR2 2
2 I1 = mR2
3
(iii) Circular ring I2 = I1 + mR2
4 2
mR2
I1 = I2 = 5
= mR2
2 3
I3 = I1 + I2 = mR2
1 ■ Two Theorems
3 x x
I4 = I2 + mR2 = mR2 (i) Theorem of parallel axes
2 5 3
I5 = I3 + mR2 = 2mR2 2
1

(iv) Rectangular slab


mb2 CM
I1 = 2
12
r
ma2
I2 = 1 3
12 x
b
m
I3 = I1 + I2 = ( a2 + b2 ) I = I CM + mr2 or I2 = I1 + mr2
12 a (ii) Theorem of perpendicular axes This theorem is
(v) Square slab applicable only for a two dimensional body of negligible
2 thickness. If x and y are two perpendicular axes lying in
3
the plane of body. z is the axis perpendicular to plane of
body and passing through point of intersection of x and
y, then
Iz = Ix + Iy
a θ
x 1 ■ Radius of gyration (K) It is an imaginary distance from
4 the axis at which whole mass of the rigid body, if kept as a
point mass, moment of inertia remains unchanged.
I
Thus, I = mK2 or K =
a m
ma2 ■ Three type of motions of a rigid body A rigid body is
I1 = I2 = I3 =
12 made up of many point masses (or particles). If distance
ma2 between any two particles remains constant, body is said to
I4 = I1 + I3 = be rigid body. A rigid body may have either of the following
6 three type of motions
(i) only translational motion momentum of rigid body about an axis passing from a
(ii) only rotational motion general point O and perpendicular to plane of paper.
(iii) both rotational and translational motion.
Only in case of translational motion velocity and
acceleration of all particles are same. In rotational or
rotational plus translational motion different particles have C
v
different displacements, different velocities and different ω
accelerations.
r⊥
■ Angular Momentum ( L )
r
Angular momentum can be defined in following three ways : .
(i) Angular momentum of a particle (A) in motion, about O
a fixed point (O)
This will contain two terms
(a) I cω
(b) mvcr⊥ = mvr⊥
A θ p =mv From right hand screw law, we can see that I cω and
r⊥ θ mvr⊥ both terms are perpendicular to the paper in
inward direction. Hence, they are added.
r
r⊥ = r sin θ or LTotal = I cω + mvr⊥
In a problem, if these two terms are in opposite
directions, then they will be subtracted.
O ■ Equations of Motion of Pure Rotational Motion of a
Suppose a particle A has a linear momentum p = mv as Rigid Body about a Fixed Axis
shown. (i) In pure rotational motion all particles (except those lying
Its position vector about a fixed point O at this instant is on the axis) of the rigid body rotate in circles. The planes
r. Then angular momentum of particle A about point O of these circles are mutually parallel. The centres of
will be, circles lie on the axis. Radii of different circles are
different. Angular velocity corresponding to all particles
L = r × p = r × ( mv ) = m ( r × v )
in circular motion is same. This is also called angular
Magnitude of L is L = mvr sinθ = mvr⊥ , where θ is the velocity of rigid body. Plane of every circle is
angle between r and p. Further, r⊥ = r sinθ is the perpendicular to the axis of rotation.
perpendicular distance on line of action of p (or v) from (ii) Velocity of any particle P is, v = r ω, tangential to its own
point O. Direction of L will be given by right hand screw circle. Since, ω is same for all particles we can say that,
law. In the shown figure direction of L is perpendicular v ∝ r.
to paper inwards. ω
If the particle passes through point O, r⊥ = 0. Therefore,
angular momentum is zero.
(ii) Angular momentum of a rigid body in pure rotation r P
about axis of rotation
If a rigid body is in pure rotation about a fixed axis, then
angular momentum of rigid body about this axis will be
given by L = Iω
ω ω (iii) Acceleration of particle P will have two components an
and at (as it is rotating in a circle). In the figure,
v2
an = rω2 or (towards centre)
r

at = rα = r ⋅ (tangential to the circle)
dt
∴ a= a2n + at2
dω τ 
(a) (b) (iv) If α  = =  is constant, then
 dt I
This is actually component of total angular momentum
about axis of rotation. Direction of this component is ω = ω0 + αt,
again given by right hand screw law. In figure (a), this is 1
θ = ω0 t + αt2
along the axis in upward direction. In figure (b) this is 2
along the axis in downward direction.
and ω2 = ω20 + 2αθ
(iii) Angular momentum of a rigid body in rotation plus
translation motion about a general axis Here, ω is angular velocity at time t and ω0 , initial
angular velocity.
Suppose a rigid body is in rotation and translational
motion. Velocity of its centre of mass is v and angular If α is not constant, then we will have to go for
velocity of rigid body is ω. We want to find angular differentiation or integration.
Rotation 367

The basic equations of differentiation or integration are, ■ Pure Rolling


dθ dω dω
ω= ,α = = ω⋅ Pure rolling (also called rolling without slipping) may be of
dt dt dθ two types
(equations of differentiation)

∫ dθ = ∫ ωdt, ∫ dω = ∫ αdt, ∫ ω dω = ∫ α ⋅ dθ v
R
(equations of integration) ω
(v) Number of rotations made by rigid body,
Angle rotated θ P
N = = Rω v
2π 2π P
■ Rotational Plus Translational Motion of a Rigid Body
(i) Uniform pure rolling In which v and ω remain
A complex motion of rotation plus translation can be constant.
simplified by considering,
Condition of pure rolling is v = Rω. In this case,
(i) the translational motion of centre of mass of the rigid bottommost point of the spherical body is at rest. It has
body and no slipping with its contact point on ground. Because
(ii) rotation about centre of mass. ground point is also at rest.
As discussed earlier also, in this type of motion velocities If v > Rω, then net velocity of point P is in the direction of
of different points of the rigid body are different. v (in the direction of motion of body). This is called
forward slipping.
P 90° - θ If v < Rω, then net velocity of point P is in opposite
v
rω direction of v. This is called backward slipping.
r ω
θ
r = PC
C v
ω R
v
P
v0
■ To find velocity of a general point (say P) we will be required
following two things Rω P v
(i) velocity of centre of mass of the rigid body v,
(ii) angular velocity of rigid body ω. v0
Now, velocity of point P is the vector sum of two terms v
and rω. Here, v is common for all points, while rω is
If a spherical body is rolling over a plank, condition for no
different for different points, as r is different.
slipping between spherical body and plank is,
In the figure shown, v − Rω = v0
vP = v2 + ( rω)2 + 2 ( v )( rω)cos(90 °− θ)
(ii) Accelerated pure rolling If v and ω are not constant
= v2 + r2ω2 + 2vrωsin θ then,a = Rα is an additional condition for pure rolling on
horizontal ground, which takes place in the presence of
■ To find acceleration of point P we will be required following
some external forces.
three things
(i) acceleration of centre of mass of the rigid body a
(ii) angular velocity of rigid body ω and
dω 
(iii) angular acceleration  α =
v
 of the rigid body.
 dt  ω
■ Now, acceleration of point P is the vector sum of three terms
(i) a Here, friction plays very important role. Magnitude and
(ii) an = rω2 (acting towards centreO) direction of friction is so adjusted that equation a = Rα is
satisfied. If friction is insufficient for satisfying the equation
(iii) at = rα (acting tangentially)
a = Rα, slipping (either forward or backward) will occur
and kinetic friction will act.
P a ■ Motion of a Spherical Body on Rough Inclined Surface
an
at
a
C v
ω, α

Here, a is common for all points, while an and at are


different. θ
(i) Minimum value of coefficient of friction required for pure ■ Angular Impulse
rolling,
Linear impulse when multiplied by perpendicular distance
tan θ
µ min = gives angular impulse. Angular impulse is equal to change
mR2
1+ in angular momentum.
I
(ii) If µ = 0, body will slip downwards (only translational J
motion) with an acceleration, r⊥
P
a1 = g sinθ
v
(iii) If µ > µ min , body will roll down without slipping with an C ω
acceleration,
(rotation + translation both)
g sinθ
a2 = A rigid body is kept over a smooth table. It is hit at point P by
I
1+ a linear impulse J at a perpendicular distance r⊥ from C as
mR2 shown. Since, it is hit at some perpendicular distance from C
(iv) In the above case (when µ > µ min ) force of friction will its motion is rotational plus translational. Velocity of centre
act upwards. Magnitude of this force is, of mass will be given by,
mg sinθ
f = J
mR2 v= (as J = mv)
1+ m
I
Angular velocity of rigid body is,
(v) If µ < µ min , body will roll downwards with forward
J × r⊥
slipping. Maximum friction will act in this case. The ω=
acceleration of body is I
(as J × r⊥ = angular impulse = change in angular
a3 = g sin θ − µg cos θ
momentum = Iω).
Additional Examples
Example 1. Define radius of gyration. Example 6. About which axis would a uniform cube
Sol. It is defined as the distance from the axis of rotation at have a minimum rotational inertia ?
which, if whole mass of the body were supposed to be Sol. About a diagonal, because the mass is more concentrated
concentrated, the moment of inertia would be same as with the about a diagonal.
actual distribution of the mass of body in the form of the
constituting particles. Example 7. A disc is recast into a thin walled
Example 2. There is a stick half of which is wooden cylinder of same radius. Which will have larger
and half is of steel. It is pivoted at the wooden end and moment of inertia?
a force is applied at the steel end at right angles to its Sol. Hollow cylinder will have larger moment of inertia, because
length. Next, it is pivoted at the steel end and the same most of its mass is located at comparatively larger distance
force is applied at the wooden end. In which case is the from the axis of rotation.
angular acceleration more and why ?
Example 8. Why there are two propellers in a
Sol. We know, τ = I α helicopter ?
or α = τ /I
Sol. If there were only one propeller in the helicopter, then due
In the second case, half of the stick made of steel
(heavier material) lies at the lesser distance from the to conservation of angular momentum, the helicopter itself
axis of rotation. Therefore, MI of the stick will be lesser would have turned in the opposite direction.
than that in the first case. Hence, the angular
acceleration in the second case will be more than that Example 9. A thin wheel can stay up right on its rim
in the former case. for a considerable length of time when rolled with a
considerable velocity, while it falls from its upright
Example 3. How will you distinguish between a position at the slightest disturbance when stationary.
hard boiled egg and a raw egg by spinning each on a Give reason.
table top ?
Sol. When the wheel is rolling upright, it has angular
Sol. The egg which spins faster will be the hard boiled egg. It is momentum in the horizontal direction, i.e. along the axis of
because, a hard boiled egg will spin (rotate) more or less as a the wheel. If slight disturbing force is applied on the wheel then
rigid body, whereas a raw egg will not do so. In case of a raw its torque just changes the direction of angular momentum.
egg, its matter in the liquid state moves away from the axis of The wheel, topples (or falls) due to this torque when
rotation, thereby increasing its moment of inertia. As MI of the wheel is at rest.
raw egg is more, the angular acceleration produced will be
lesser, when the same torque is applied in both the cases to set Example 10. If angular momentum is conserved in a
them spinning. system whose moment of inertia is decreased, will its
Example 4. The angular velocity of revolution of the rotational kinetic energy be conserved ?
earth around the sun increases, when it comes closer to Sol. Here, L = Iω = constant
the sun. Why? Rotational KE is given by
1 1 I 2 ω 2 1 L2
Sol. The earth revolves around the sun in elliptical orbit with the K = Iω2 = = ⋅
sun at one of the two focii of the elliptical orbit. Therefore, 2 2 I 2 I
moment of inertia of the earth about an axis through the sun 1
For constant L, K ∝
keeps on changing due to change in its distance from the sun. I
Since, no external torque acts on the earth, its angular So, when the moment of inertia decreases, the
momentum (Iω ) must remain conserved. Thus, angular rotational KE increases. Hence, rotational KE is not
velocity of the earth increases, when moment of inertia of the conserved.
earth decreases and vice-versa.
Example 11. If no external torque acts on a body,
Example 5. Why in hand driven grinding machine, will its angular velocity remain constant? Give reason.
handle is put near the circumference of the stone or Sol. When no external torque acts on a body, its angular
wheel ? momentum remains constant. But
Sol. For a given force, torque can be increased, if the L = Iω
perpendicular distance of the point of application of the force Clearly, the angular velocity ω will remain constant
from the axis of rotation is increased. Hence, the handle put only if the moment of inertia I of the body also
near the circumference produces maximum torque. remains constant.
370 Objective Physics Vol. 1

Example 12. A disc starts rotating with constant Example 15. A solid cylinder of mass m and radius
angular acceleration of π rad/s 2 about a fixed axis r starts rolling down an inclined plane of inclination θ.
perpendicular to its plane and through its centre. Find Friction is enough to prevent slipping. Find the speed
(a) the angular velocity of the disc after 4 s, of its centre of mass when its centre of mass has fallen
(b) the angular displacement of the disc after 4 s. a height h.
Sol. Here, α = π rad/s 2 Sol. Considering the two shown positions of the cylinder. As it does
not slip hence, total mechanical energy will be conserved.
ω 0 = 0, t = 4 s
(a) ω ( 4 s ) = 0 + (π rad/s 2 ) × 4 s = 4 π rad/s
1
1 h
(b) θ ( 4 s ) = 0 + (π rad/s 2 ) × (16 s 2 ) = 8π rad 2
2
θ
Example 13. A wheel rotates with an angular
acceleration given by α = 4at 3 − 3bt 2 , where t is the
time and a and b are constants. If the wheel has initial Energy at position 1 is
angular speed ω 0 , write the equations for the E1 = mgh
Energy at position 2 is
(i) angular speed, (ii) angle displacement. 1 2 1
dω E2 = mv CM + I CM ω 2
Sol. (i) α= 2 2
dt v CM
Q =ω
⇒ dω = α dt r
ω t t mr 2
⇒ ∫ω 0
dω = ∫ α dt = ∫ (4at 3 − 3bt 2 ) dt
0 0
and I CM =
2
⇒ ω = ω 0 + at 4 − bt 3 ⇒
3
E 2 = mv CM 2

dθ 4
(ii) Further, ω= From condemnation of energy, E1 = E 2
dt
⇒ dθ = ω dt ⇒ v CM =
4
gh
θ t t 3
⇒ ∫ 0 dθ = ∫ 0ω dt = ∫ 0(ω 0 + at − bt 3 ) dt
4

at 5 bt 4 Example 16. If the radius of the earth contracts to


⇒ θ = ω0 t + − half of its present value without change in its mass,
5 4
what will be the new duration of the day?
Example 14. A particle of mass m is projected with Sol. Present angular momentum of the earth
velocity v at an angle θ with the horizontal. Find its 2
L 1 = Iω = MR 2 ω
angular momentum about the point of projection when 5
it is at the highest point of its trajectory. New angular momentum, because of change in radius
Sol. y 2  R
2
L2 = M   ω′
5  2
If external torque is zero, then angular momentum
must be conserved
H L1 = L 2
x 2 1 2
O MR 2 ω = × MR 2 ω ′
5 4 5
At the highest point it has only horizontal velocity i.e. ω′ = 4 ω
v x = v cos θ. 1 1
Length of the perpendicular to the horizontal velocity T ′ = T = × 24
4 4
from O is the maximum height, where
=6 h
v 2 sin 2 θ
H max = Example 17. A solid ball of radius 0.2m and mass
2g
1 kg is given an instantaneous impulse of 50 N-s at
mv sin θ cos θ
3 2
⇒ Angular momentum L = point P as shown in figure. Find the number of
2g rotations made by the ball about its diameter before
Rotation 371

hitting the ground. The ball is kept on smooth surface Example 19. A horizontal force F acts on the sphere
initially. at its centre as shown in figure. Coefficient of friction
50 N-s between ground and sphere is µ. What is maximum
value of F, for which there is no slipping?
P
30° F

0.2 m

Sol. F − f = Ma
2 2 a 2
Sol. Impulse gives translational velocity f ⋅ R = MR ⇒ f = Ma
5 R 5
Impulse 2 2
u= ⇒ f = F ⇒ F ≤ µmg
Mass 7 7
= 50 m/s ⇒
7
F ≤ µmg
T = time of flight of projectile 2
2u sin θ 2 × 50 × sin 60 °
= = Example 20. A tangential force F acts at the top of
g 10
a thin spherical shell of mass m and radius R. Find the
= 5 3s acceleration of the shell, if it rolls without slipping.
Impulse give angular impulse also F
Impulse × R
ω=
I
Impulse × R R
or ω=
2
mR 2
5
Number of rotations,
ωT
n= Sol. Let f be the force of friction between the shell and the
2π horizontal surface.
3125 3
= F

Example 18. A sphere of mass m attached to a


spring on incline as shown in figure is held in
unstretched position of spring. Suddenly sphere is left f
free what is the maximum extension of spring, if For translational motion
friction allows only rolling of sphere about horizontal F + f = ma K (i)
diametre? For rotational motion,
a
FR − fR = Iα = I
R
(Qa = Rα for pure rolling)
a
⇒ F− f =I 2 K (ii)
θ R
Adding Eqs. (i) and (ii)
2 F = m + 2  a = m + m a = ma
Sol. At the point of maximum extension sphere is at rest. No I 2 5
work is done by frictional force in rolling.  R   3  3
⇒ Loss in gravitational PE = Gain in PE of spring Q I mR 2 
5 2
or F = ma shell =

1 2
kx = mgx sin θ 6 
 3 
2 6F
2 mg sin θ ⇒ a=
⇒ x= 5m
k
NCERT Selected Questions
Q 1. (a) Find the moment of inertia of a sphere about a Q 2. Torques of equal magnitude are applied to a hollow
tangent to the sphere, given that the MI of the cylinder and a solid sphere both having the same
sphere about any of its diameter to be 2 MR 2 / 5, mass and radius. The cylinder is free to rotate about
where, M is the mass of the sphere and R is the its standard axis of symmetry and the sphere is free
radius of the sphere. to rotate about an axis passing through its centre.
Which of the two will acquire a greater angular
(b) Given that the MI of a disc of mass M and radius
R about any of its diameters to the MR 2 / 4, find speed after a given time?
its MI about an axis normal to the disc and Sol. Let I 1 and I 2 be the moments of inertia of the hollow
passing through a point on its edge. cylinder about its axis of symmetry and solid sphere about
2 its axis through its centre respectively.
Sol. (a) I AB = MR 2 ...(i)
5 Then, I 1 = MR 2 ...(i)
Let,CD be a tangent of the sphere D 2
B and I 2 = MR 2 ...(ii)
parallel to the diameter AB of the 5
sphere.
Let τ = magnitude of the torque applied on each of them.
∴ Distance between the two parallel R
If α 1 and α 2 be the angular accelerations produced in the
O
axis is R. If ICD be its MI about CD
cylinder and sphere respectively, then
axis, then according to the theorem of
parallel axis, A τ = I1α 1
C and τ = I2α 2
ICD = I AB + MR 2
∴ I 1α 1 = I 2 α 2
2 α 1 I2
= MR 2 + MR 2 or =
5 α 2 I1
7 2
= MR 2 MR 2
5 5 2
= =
(b) Let EF be an axis perpendicular to the plane of disc and MR 2 5
passing through its CM. Clearly, DG is the axis normal 5
or α2 = α1
to the disc and passing through a point D on its edge. 2
B = 2.5 α 1 ...(iii)
If ω 1 and ω 2 be the angular speed of the cylinder and sphere
after a time t, then
D OF
C ω 1 = ω 0 + α 1t ...(iv)
and ω 2 = ω 0 + α 2t
G = ω 0 + 2.5 α 1t ...(v)
E
A where, ω 0 = initial angular speed
Clearly, the axis DG is parallel to the axis EF. ∴ From Eqs. (iv) and (v), it is clear that
∴ If I EF be the MI of the disc about EF axis. ω2 > ω1
Then, according to theorem of perpendicular axes, ∴ The sphere will acquire more angular speed as compared to
that of the cylinder after a given time.
I EF = I AB + ICD
Q 3. A solid cylinder of mass 20 kg rotates about its axis
MR 2 MR 2
= + with angular speed 100 rad s −1 . The radius of the
4 4 cylinder is 0.25 m . What is the kinetic energy
Here, perpendicular distance between EF and DG axis = R associated with the rotation of the cylinder? What is
∴ If I DG be the MI of the disc about the required axis, then the magnitude of angular momentum of the cylinder
according to theorem of parallel axes about its axis?
I DG = I EF + MR 2 Sol. If I be the MI of the cylinder about its axis,
1
= MR 2 + MR 2 then I =
1 1
MR 2 = × 20 × (0.25)2
2 2 2
3
= MR 2 = 0.625 kg -m 2
2
Rotation 373

∴ KE associated with the rotating cylinder is given by, 2  5


2
1  5 1 2
= ×   ×  I 1ω 12 =  I 1ω 1
1 1 5  2  2  2 2 
KE = Iω 2 = × 0.625 × (100)2
2 2 5 1 2
= 3125 J  Iω
Final KE of rotation 2  2 1 1 5
∴ = =
Also using the relation, Initial KE of rotation 1 2 2
 I 1ω 1
or L = 2I KE 2 
Clearly, new KE of rotation (when child folds his hands
We have, 5
back) is times greater than the initial KE of rotation.
L = 2 × 0.625 × 3125 or L = 62.5 J-s 2
The increase in KE of rotation on folding back his hands
Q 4. (a) A child stands at the centre of a turn table with appears due to the use of child’s own internal energy i.e.
his two arms outstretched. The turn table is set muscular energy (used in folding the arms) to increase
rotating with an angular speed of 40 rpm. How the rotational kinetic energy.
much is the angular speed of the child, if he
folds his hands back and thereby reduces his Q 5. A rope of negligible mass is wound round a hollow
moments of inertia to 2/5 times the initial cylinder of mass 3 kg and radius 40cm. What is the
value? Assume that the turn table rotates angular acceleration of the cylinder, if the rope is
without friction. pulled with a force of 30 N? What is the linear
(b) Show that the child’s new kinetic energy of acceleration of the rope? Assuming that there is no
rotation is more than the initial kinetic energy of slipping.
rotation. How do you account for this increase in Sol. If I be the MI of the hollow cylinder about its axis, then
kinetic energy? I = MR 2 = 3(0.4)2 = 0.48 kg-m 2
2
Sol. (a) I2 = I 1 (Given) If τ = torque acting on the cylinder, then
5
τ = FR = 30 × 0.4 = 12 N-m
40
f1 = 40 rpm = rps τ 12
60 ∴ α= = = 25 rad s−2
2π × 40 I 0.48
∴ ω 1 = 2πf1 = rad s−1
60 ∴ a = Rα = 0.4 × 25 = 10 m/s 2
4
= π rad s−1
3 Q 6. To maintain a rotor at a uniform angular speed of
Let, ω 1 and ω 2 be the angular speed of the child with 200 rad s −1 , an engine needs to transmit a torque of
outstretched and folding arms respectively. 180 N-m. What is the power required by the engine?
∴ According to the law of conservation of angular
Sol. Here, ω = 200 rad s−1
momentum, we get
τ = 180 N-m
I 1ω 1 = I 2 ω 2
I I 4π Using the relation, P = τω, we get
or ω 2 = 1 ω 1 or ω 2 = 1 × P = 180 × 200
I2 2 3
I2
5 = 36000 W = 36 kW
5 4 π 10π
= × = rad s−1 Q 7. A solid sphere rolls down two different inclined
2 3 3 planes of the same heights but different angles of
∴ Frequency of revolution f2 is given by inclination.
ω 10π
f2 = 2 = rps (a) Will it reach the bottom with the same speed in
2π 3 × 2π each case?
5 (b) Will it take longer to roll down one plane than
= × 60 rpm = 100 rpm
3 the other?
∴ f2 = 100 rpm
Sol. (a) As work done by friction in case of pure rolling = 0
1
(b) Initial KE of rotation = I 1 ω 21 Therefore, in both cases mechanical energy will remain
2
constant.
1
Final KE of rotation = I 2 ω 22 Kinetic energy in reaching bottom in both cases is mgh.
2
2 (b) Yes, it will take longer time down one plane than the
1  2   5  5 
=  I 1   ω 1
2
Q ω 2 = ω 1 other. It will be longer for the plane having smaller angle
2  5   2  2  of inclination.
374 Objective Physics Vol. 1

Q 8. A hoop of radius 2 m weight 100 kg. It rolls along a v 2 − u2 0 − 52


s= =
horizontal floor so that its centre of mass has a speed 2a  9.8
2 − 
of 20 cm −1 . How much work has to be done to stop  3
25
it? = × 3 = 3.83 m
2 × 9.8
Sol. If ω be the angular velocity of the CM of the hoop, then (b) Let T = time taken by the cylinder to return to the
ω= =
v 0.20
= 0.10 rad s−1 bottom T = 2t , where t = time of ascending or
r 2 descending
I = mr2 = 100 × (2)2 = 400 kg -m 2 g sin θ 9.8 −2
∴ a= = ms
I 3
∴ Total KE of ring = Rotational KE + Translational KE 1+
1 1 mR 2
or E = Iω 2 + mv 2 s = 3.83 m
2 2
1 1 Here, initial velocity = 0
or E = × 400 × (0.10)2 + × 100 × (0.20)2 1
2 2 ∴Using the relation, s = ut + at 2 , we get
=4J 2
2s 2 × 3.83
Work done to stop it = Total KE of the hoop = 4 J t= = = 1.53 s
a  9.8
 
Q 9. The oxygen molecule has a mass of 5.30 × 10 −26 kg  3
and a moment of inertia of 1.94 × 10 −46 kg-m 2 ∴ T = 2 × 1.53 = 3.06 s ≈ 3 s
about an axis through its centre perpendicular to the
Q 11. A disc rotating about its axis with angular speed ω 0
lines joining the two atoms. Suppose, the mean
is placed lightly [without any translational push] on
speed of such a molecule in a gas is 500 ms −1 and
2 a perfectly frictionless table. Will the disc roll on
KE of rotation is of its KE of translation. Find the the surface?
3
average angular velocity of the molecule. Sol. The disc cannot roll on smooth surface, if it is not placed on
rolling condition.
Sol. According to the given condition,
2
KE of rotation = × KE of translation
Q 12. Read each statement below carefully and state with
3 reasons, if it is true or false.
1 2 2 1 2 mv 2 (a) The instantaneous speed of the point of contact
or Iω = mv 2 or ω = during rolling is zero.
2 3 2 3 I
(b) For perfect rolling motion, work done against
2 5.30 × 10−26 × (500)2
= × friction is zero, if it is taking place on stationary
3 1.94 × 10−46 ground.
= 6.75 × 1012 rad s−1 (c) A wheel moving down a perfectly frictionless
inclined plane will undergo slipping (not rolling)
Q 10. A cylinder rolls up an inclined plane of angle of motion.
inclination 30°. At the bottom of the inclined plane,
the centre of mass of the cylinder has a speed of Sol. (a) It is true. A rolling body can be imagined to be rotating
5 ms −1 . about an axis passing through the point of contact of the
body with the ground and hence its instantaneous speed
(a) How far will the cylinder go up the plane? is zero, if it is taking place on stationary ground.
(b) How long will it take to return to the bottom? (b) It is true. If rolling is taking place on stationary ground.
g sin θ In a perfect rolling motion, the work done against
Sol. (a) Retardation, a = −
I friction is zero,because for perfect rolling, the point of
1+ contact should be momentarily at rest. So, no work is
mR 2
done against friction at this point.
Let the cylinder be solid, then
1 (c) True. On an inclined plane a body rolls due to the force
I = mR 2 of friction acting on it. If the wheel is moving down a
2
perfectly frictionless inclined plane, it will be under the
g sin θ 2 1
∴ a=− = − × 9.8 × effect of its weight only. Since, the weight of the wheel
1 3 2
1+ acts along the vertical through its CM, the wheel will not
2 rotate.
9.8 −2
=− ms It will keep on slipping, i.e. in the absence of friction no
3 rolling will take place and the body slips due to
Using the relation, v 2 − u2 = 2as, we get translational force.
Objective Problems
[ Level 1 ]
Moment of Inertia 8. Three thin rods each of length L and mass M are placed
1. Moment of inertia of a body depends upon along x, y and z-axis such that one end of each rod is at
(a) axis of rotation origin. The moment of inertia of this system about z-axis
(b) torque is
(c) angular momentum 2 4 ML 2
(a) ML 2 (b)
(d) angular velocity 3 3
5ML 2 ML 2
2. Analogue of mass in rotational motion is (c) (d)
3 3
(a) moment of inertia
(b) angular momentum 9. The ratio of the radii of gyration of a circular disc and a
(c) gyration circular ring of the same radii about a tangential axis
(d) None of the above
perpendicular to plane of disc or ring is
3. One circular ring and one circular disc both having the (a) 1 : 2 (b) 5: 6
same mass and radius. The ratio of their moments of 3
inertia about the axes passing through their centres and (c) 2 : 3 (d)
2
perpendicular to planes will be
(a) 1 : 1 (b) 2 : 1 10. Figure represents the MI of the solid sphere about an axis
(c) 1 : 2 (d) 4 : 1 parallel to the diameter of the solid sphere and at a
distance x from it. Which one of the following represents
4. The radius of gyration of a solid sphere of radius R about
the variations of I with x?
a is tangential
l l
7 2
(a) R (b) R
5 5
5 (a) (b)
(c) R (d) R
7 x x

5. The ratio of the radii of gyration of a hollow sphere and a


l l
solid sphere of the same radii about a tangential axis is
7 5
(a) (b)
3 21 (c) (d)
21 25 x x
(c) (d)
5 9 O O

6. What is the moment of inertia of a solid sphere of density 11. If I 1 is the moment of inertia of a thin rod about an axis
ρ and radius R about its diameter? perpendicular to its length and passing through its centre
105 5 176 5
(a) Rρ (b) Rρ of mass and I 2 is the moment of inertia of the ring about
176 105
an axis perpendicular to plane of ring and passing
105 2 176 2
(c) Rρ (d) Rρ through its centre formed by bending the rod, then
176 105
I1 3 I1 2 I1 π 2 I π2
(a) = 2 (b) = 2 (c) = (d) 1 =
7. Let I A and I B be moments of inertia of a body about two I2 π I2 π I2 2 I2 3
axes A and B respectively. The axis A passes through the
centre of mass of the body but B does not 12. A rod is placed along the line y = 2x with its centre at
(a) IA < IB origin. The moment of inertia of the rod is maximum
(b) I A < I B , whether the axes are parallel or not parallel about
(c) if the axes are parallel I A < I B (a) x-axis (b) y-axis
(d) if the axes are not parallel I A ≥ I B (c) z-axis (d) Data insufficient
376 Objective Physics Vol. 1

13. The moment of inertia of a thin I4 18. A particle of mass 1 kg is kept at (1 m, 1 m, 1 m). The
I1
rectangular plate ABCD of D C moment of inertia of this particle about z-axis would be
uniform thickness about an axis I2 (a) 1 kg-m 2 (b) 2 kg-m 2
passing through the centre O O (c) 3 kg-m 2 (d) None of these
A B
and perpendicular to the plane
of the plate is I3 19. Two uniform, thin identical rods each of mass M and
length l are joined together to form a cross. What will be
(a) I 1 + I 2 (b) I 2 + I 4 the moment of inertia of the cross about an axis passing
(c) I 1 + I 3 (d) I 1 + I 2 + I 3 + I 4 through the point at which the two rods are joined and
14. A wheel comprises of a ring of radius R and mass M and perpendicular to the plane of the cross
three spokes of mass m each. The moment of inertia of Ml 2 Ml 2
(a) (b)
the wheel about its axis is 12 6
Ml 2 Ml 2
(c) (d)
4 3

R
20. A square lamina is as shown in figure. The moment of
inertia of the frame about the three axes shown in figure
are I 1 , I 2 and I 3 respectively. Select the correct
alternative.
I3
 m I2
(a)  M +  R 2 (b) (M + m) R 2
 4
 M + m 2
(c) (M + 3m) R 2 (d)   R
 2 
I1
15. Moment of inertia of a rod of mass m and length l about
its one end is I. If one-fourth of its length is cut away,
then moment of inertia of the remaining rod about its one
end will be (a) I 2 = I 3 > I 1 (b) I 1 > I 2 > I 3
3 9 27 I (c) I 2 = I 3 < I 1 (d) I 1 < I 2 < I 3
(a) I (b) I (c) I (d)
4 16 64 16
21. Moment of a force of magnitude 10 N acting along
16. For the uniform T shaped structure, with mass 3M , positive y-direction at point (2m, 0, 0) about the point
moment of inertia about an axis normal to the plane and (0, 1m, 0) in N-m is
passing through O would be (a) 10 (b) 20 (c) 10 2 (d) 30
2l O
22. One circular ring and one circular disc both having the
same mass and radius. The ratio of their moments of
inertia about the axes passing through their centres and
l perpendicular to planes will be
(a) 1 : 1 (b) 2 : 1
(c) 1 : 2 (d) 4 : 1

2 23. The radius of gyration of a uniform rod of length L about


(a) Ml 2 (b) Ml 2 an axis passing through its centre of mass is
3
Ml 2 L L2
(c) (d) None of these (a) (b)
3 2 3 12
L L
17. The moment of inertia of a solid cylinder of mass M, (c) (d)
3 2
length 2R and radius R about an axis passing through the
centre of mass and perpendicular to the axis of the 24. Five particles of masses 2 kg each are attached to the rim
cylinder is I 1 and about an axis passing through one end of a circular disc of radius 0.1 m and negligible mass.
of the cylinder and perpendicular to the axis of cylinder is Moment of inertia of the system about the axis passing
I2 through the centre of the disc and perpendicular to its
(a) I 2 − I 1 = MR 2 (b) I 2 = I 1 plane is
I 19 (a) 1 kg-m 2 (b) 0.1 kg-m 2
(c) 2 = (d) I 1 − I 2 = MR 2
I 1 12 (c) 2 kg-m 2 (d) 0.2 kg-m 2
Rotation 377

25. ABC is a right angled triangular A C


30. A semicircular 2-dimensional plate
plate of uniform thickness. The of mass m has radius r and centre
sides are such that AB > BC as I1 I3 C. Its centre of mass is at a distance r
shown in figure. I 1 , I 2 and I 3 are x from C. Its moment of inertia
moments of inertia about AB, BC about an axis through its centre of
and AC respectively. Then, which of B C mass and perpendicular to its plane is
I2
the following relation is correct? 1 2 1
(a) I 1 = I 2 = I 3 (b) I 2 > I 1 > I 3 (a) mr (b) mr2
2 4
(c) I 3 < I 2 < I 1 (d) I 3 > I 1 > I 2 1 1
(c) mr2 + mx 2 (d) mr2 − mx 2
26. The moment of inertia of a cube of mass m and side a 2 2
about one of its edges is equal to
2 4 8
31. Two point masses m1 and m2 and attached from two ends
(a) ma2 (b) ma2 (c) 3ma2 (d) ma2
3 3 3 of a rod of negligible mass. The system is rotated about
an axis perpendicular to the length of the rod. The
27. The moment of inertia of a semicircular ring of mass M
minimum moment of inertia of the system is
and radius R about an axis which is passing through its
centre and at an angle θ with the line joining its ends as (a) about centre of mass
shown in figure is m1m2l 2
(b)
m1 + m2
(c) Both (a) and (b)
(d) None of the above

θ 32. The ratio of the radii of gyration of a circular disc and a


circular ring of the same radius about a tangential axis in
MR 2 MR 2 the plane is
(a) at θ = 0° (b) if θ = 0°
4 2 (a) 3: 4 (b) 5: 6
MR 2 MR 2 (c) 6: 5 (d) 4: 3
(c) if θ = 45° (d) if θ = 90°
2 2
28. A square is made by joining four rods each of mass M Torque, Angular Momentum,
and length L. Its moment of inertia about an axis PQ, in Conservation of Angular
its plane and passing through one of its corner is
Momentum and Angular Velocity
P
33. If a person standing on a rotating disc stretches out his
45°
hands, the angular speed will
(a) increase (b) decrease
(c) remain same (d) None of these
L
34. The angular momentum of a system of particles is
Q
conserved
4 8 10 (a) when no external force acts upon the system
2
(a) 6ML (b) ML 2 (c) ML 2 (d) ML 2 (b) when no external torque acts upon the system
3 3 3
(c) when no external impulse acts upon the system
29. From a uniform square plate of side a and mass m, a square (d) when axis of rotation remains same
a
portion DEFG of side is removed. Then, the moment of 35. Angular momentum is
2
(a) moment of momentum
inertia of remaining portion about the axis AB is
a
(b) product of mass and angular velocity
A B (c) product of moment of inertia and velocity
(d) moment in angular motion
1
E F 36. If the radius of the earth contracts of its present day
a/2 n
value, the length of the day will be approximately
D G C 24 24
(a) h (b) h
7ma2 3ma2 3ma2 9ma2 n n2
(a) (b) (c) (d)
16 16 4 16 (c) 24n h (d) 24 n2 h
378 Objective Physics Vol. 1

37. A thin circular ring of mass M and radius R is rotating 44. The torque of a force F = − 6i$ acting at a point r = 4$j
about its axis with a constant angular velocity ω. Two about origin will be
objects each of mass m are attached gently to the ring. (a) − 24 k$ (b) 24 k$
The wheel now rotates with an angular velocity
(c) 24 $j (d) 24 i$
ωM ω (M − 2m)
(a) (b)
(m + M ) (M + 2m) 45. A particle of mass 2 kg located at the position ( $i + $j ) m
ωM ω (M + 2m)
(c) (d) has a velocity 2 ( + i$ − $j + k$ ) m/s. Its angular momentum
(M + 2m) M
about z-axis in kg-m 2 /s is
38. A particle of mass 5 g is moving with a uniform speed (a) +4 (b) +8
of 3 2 cm/s in the x-y plane along the line y = 2 5 cm. (c) −4 (d) −8
The magnitude of its angular momentum about the 46. A wheel of radius 20 cm forces applied to it as shown in
origin in g- cm 2 /s is the figure. The torque produced by the forces 4 N at A,
(a) zero (b) 30 8 N at B, 6 N at C and 9 N at D at angles indicated is
(c) 30 2 (d) 30 10 4N
A
39. A particle of mass m = 5 units is moving with a uniform 90°
speed v = 3 2 units in the x-y plane along the line
y = x + 4. The magnitude of the angular momentum 30° 20 cm
D
about origin is 8N
90°
B
(a) zero (b) 60 unit
(c) 7.5 unit (d) 40 2 unit
C 9N
40. A uniform disc of radius a and mass m, is rotating freely
with angular speed ω in a horizontal plane about a 6N
smooth fixed vertical axis through its centre. A particle, (a) 5.4 N-m anti-clockwise (b) 1.80 N-m clockwise
also of mass m, is suddenly attached to the rim of the disc (c) 2.0 N-m clockwise (d) 3.6 N-m clockwise
and rotates with it. The new angular speed is
ω ω 47. The torque of force F = − 3 i$ + $j + 5k$ acting on a point
(a) (b)
6 3 r = 7$i + 3$j + k$ about origin will be
ω ω
(c) (d) (a) 14 i$ − 38$j + 16k$ (b) 4 i$ − 4 $j + 6k$
2 5
(c) −14 $i − 38$j + 16k$ (d) −21$i − 3$j + 5k$
41. When a body is projected at an angle with the horizontal
in the uniform gravitational field of the earth, the angular 48. O is the centre of an equilateral triangle ABC. F1 , F2 and
momentum of the body about the point of projection, as it F3 are three forces acting along the sides AB, BC and AC
proceeds along its path as shown in figure.
(a) remains constant A
(b) increases
(c) decreases
(d) initially decreases and after its highest point increases.
O
42. A force F = ai$ + 3$j + 6k$ is acting at B F3 F2
a point r = 2i$ − 6$j − 12k$ . The value of a for which F1 C

angular momentum about origin is conserved is What should be the magnitude of F3 so that the total
(a) zero (b) 1 (c) −1 (d) 2 torque about O is zero?
43. A thin uniform circular disc of mass M and radius R is (F1 + F2 )
(a) (b) (F1 − F2 )
rotating in a horizontal plane about an axis passing 2
through its centre and perpendicular to its plane with an (c) (F1 + F2 ) (d) 2(F1 + F2 )
angular velocity ω. Another disc of same dimensions but 49. A diver in a swimming pool bends his head before
1
of mass M is placed gently on the first disc coaxially. diving, because it
4 (a) decreases his moment of inertia
The angular velocity of the system is (b) decreases his angular velocity
2 4 3 1
(a) ω (b) ω (c) ω (d) ω (c) increases his moment of inertia
3 5 4 3 (d) decreases his linear velocity
Rotation 379

50. A particle of mass m is projected with a velocity v 56. A wheel is rotating at 900 rpm about its axis. When
making an angle of 45° with the horizontal. The power is cut-off it comes to rest in 1 min. The angular
magnitude of angular momentum of projectile about the retardation in rad/s 2 is
point of projection when the particle is at its maximum π π
(a) (b)
height h is 2 4
mvh π π
(a) zero (b) (c) (d)
2 6 8
(c) mvh (d) 2 mvh
57. If the equation for the displacement of a particle moving
51. A sphere rolls without slipping on a rough horizontal on a circular path is given by θ = 2t 3 + 0.5, where θ is in
surface with centre of mass speed v 0 . If mass of the radian and t is in second, then the angular velocity of the
sphere is M and its radius is R, then what is the angular particle after 2 s is
momentum of the sphere about the point of contact? (a) 8 rad/s (b) 12 rad/s
5 7 (c) 24 rad/s (d) 36 rad/s
(a) Mv0R (b) Mv0R
2 5
3 1 58. A constant torque of 1000 N-m turns a wheel of moment
(c) Mv0R
5
(d) Mv0R
2 of inertia 200 kg-m 2 about an axis through its centre. Its
angular velocity after 3 s is
Pure Rotational Motion (a) 1 rad/s (b) 5 rad/s
(c) 10 rad/s (d) 15 rad/s
52. The figure shows the angular velocity versus time graph
of a flywheel. The angle, in radians through which the 59. A table fan, rotating at a speed of 2400 rpm is switched
flywheel turns during 25 s is off and the resulting variation of the rpm with time is
shown in the figure. The total number of revolutions of
the fan before it comes to rest is
30

ω 2400

(rad/s) rpm
0 Time (s) 25 600
5 20
(a) 120 (b) 480 t (s)
8
(c) 600 (d) 750
(a) 420 (b) 190
53. A body is in pure rotation. The linear speed v of a (c) 280 (d) 380
particle, the distance r of the particle from the axis and
v 60. A wheel is at rest. Its angular velocity increases
the angular velocity ω of the body are related as ω = . uniformly and becomes 80 rad/s after 5 s. The total
r
angular displacement is
Thus,
1 (a) 800 rad (b) 400 rad
(a) ω ∝ (b) ω ∝ r (c) 200 rad (d) 100 rad
r
(c) ω = 0 (d) ω is independent of r 61. A rigid body rotates about a fixed axis with variable
54. The motor of an engine is rotating about its axis with an angular velocity equal to α − βt , at the time t, where α , β
angular velocity of 100 rpm. It comes to rest is 15 s, after are constants. The angle through which it rotates before
being switched off. Assuming constant angular its stops
deceleration. What are the numbers of revolutions made α2 α 2 − β2
(a) (b)
by it before coming to rest? 2β 2α
(a) 12.5 (b) 40 (c) 32.6 (d) 15.6 α 2 − β2 (α − β ) α
(c) (d)
55. A wheel is subjected to uniform angular acceleration 2β 2
about its axis. Initially its angular velocity is zero. In the 62. A flywheel having a radius of gyration of 2 m and mass
first 2 s, it rotates through an angle θ1 , in the next 2 s, it
θ 10 kg rotates at an angular speed of 5 rad/s about an axis
rotates through an angle θ 2 . The ratio of 2 , is perpendicular to it through its centre. The kinetic energy
θ1 of rotation is
(a) 1 (b) 2 (a) 500 J (b) 2000 J
(c) 3 (d) 5 (c) 1000 J (d) 250 J
380 Objective Physics Vol. 1

63. A wheel is rotating at the rate of 33 rpm. If it comes to 71. A wheel of radius R rolls on the ground with a uniform
stop in 20 s. Then, the angular retardation will be velocity v. The velocity of topmost point relative to the
(a) π rad /s (b) 11π rad /s2 bottommost point is
π 11π (a) v (b) 2v
(c) rad /s2 (d) rad /s2 v
200 200 (c) (d) zero
2
Rotational and Translational Motion 72. A sphere can roll on a surface inclined at an angle θ, if the
64. A sphere of moment of inertia I rolls down an inclined friction coefficient µ > ( 2 / 7) g sin θ. Now, suppose the
plane without slipping. The ratio of the rotational kinetic friction coefficient is (1/ 7) g sin θ and the sphere is
energy to the translational kinetic energy is nearly released from rest on the incline
2 7 (a) it will stay at rest
(a) (b) (b) it will make pure translational motion
5 5
7 5 (c) it will translate and rotate about the centre
(c) (d) (d) the angular momentum of the sphere about its centre will
2 2
remain constant.
65. Total KE of sphere of mass M rolling with velocity v is
7 5 73. A solid homogeneous sphere is moving on a rough
(a) Mv 2 (b) Mv 2 horizontal surface, partly rolling and partly sliding.
10 6
7 10 During this kind of motion of the sphere
(c) Mv 2 (d) Mv 2 (a) total KE is conserved
5 7
(b) angular momentum of the sphere about the point of contact
66. A sphere cannot roll without applying external force on with the plane is conserved
(a) a smooth inclined surface (c) only the rotational KE about centre of mass is conserved
(b) a smooth horizontal surface (d) angular momentum about centre of mass is conserved.
(c) a rough inclined surface 74. A spherical ball rolls on a table without slipping. The
(d) a rough horizontal surface
fraction of its total energy associated with rotation is
67. The centre of a wheel rolling on a plane surface moves 3 2 2 3
(a) (b) (c) (d)
with a speed v 0 . A particle on the rim of the wheel at the 5 7 5 7
same level as the centre will be moving at speed 75. The least coefficient of friction for an inclined plane
(a) zero (b) v0 inclined at an angle α with horizontal, in order that a solid
(c) 2v0 (d) 2v0 cylinder will roll down without slipping is
2 2
68. A solid sphere, a hollow sphere and a disc, all having (a) tan α (b) tan α
same mass and radius, are placed at the top of a smooth 3 7
1 4
incline and released. Least time will be taken in reaching (c) tan α (d) tan α
the bottom by 3 3
(a) the solid sphere (b) the hollow sphere 76. A string of negligible thickness
(c) the disc (d) all will take same time is wrapped several times around
a cylinder kept on a rough
69. A solid sphere, a hollow sphere and a disc, all having
horizontal surface. A man
same mass and radius, are placed at the top of an inclined
standing at a distance l from the cylinder holds one end of
plane and released. The friction coefficients between the the string and pulls the cylinder towards him. There is no
objects and the incline are same and not sufficient to slipping anywhere. The length of the string passed
allow pure rolling. Least time will be taken in reaching through the hand of the man while the cylinder reaches
the bottom by his hands is
(a) the solid sphere (a) l (b) 2l
(b) the hollow sphere (c) 3l (d) 4l
(c) the disc
(d) all will take same time 77. Two uniform solid spheres having unequal masses and
unequal radii are released from rest from the same height
70. In the previous question, the smallest kinetic energy at
on a rough incline. If the spheres roll without slipping
the bottom of the incline will be achieved by
(a) the heavier sphere reaches the bottom first
(a) the solid sphere
(b) the bigger sphere reaches the bottom first
(b) the hollow sphere
(c) the two spheres reach the bottom together
(c) the disc
(d) the information given is not sufficient to tell which sphere
(d) all will achieve same kinetic energy will reach the bottom first
Rotation 381

78. The ratio of the time taken by a solid sphere and that 85. The speed of a uniform spherical shell after rolling down
taken by a disc of the same mass and radius to roll down a an inclined plane of vertical height h from rest, is
rough inclined plane from rest from the same height is 10gh 6gh
(a) (b)
(a) 15 : 14 (b) 15 : 14 7 5
(c) 14 : 15 (d) 14 : 15 4 gh
(c) (d) 2gh
5
79. An inclined plane makes an angle of 30° with the
horizontal. A solid sphere rolling down this inclined 86. A disc of radius R rolls on a rough horizontal surface.
plane from rest without slipping has a linear acceleration The distance covered by the point A in one revolution is
equal to
g 2g R
(a) (b)
3 3 C
5g 5g A
(c) (d)
7 14
(a) 2πR (b) 2R
80. A solid cylinder and a solid sphere both having the same (c) 8R (d) πR
mass and radius are released from a rough inclined plane. 87. A wheel of bicycle is rolling without slipping on a level
Both roll without slipping. Then, road. The velocity of the centre of mass is vCM then true
(a) the force of friction that acts on the two is the same statement is
(b) the force of friction is greater in case of a sphere than for a
cylinder A
(c) the force of friction is greater in case of a cylinder than for a
sphere
CM vCM
(d) the force of friction will depend on the nature of the surface
of the body that is moving and that of the inclined surface
B
and is independent of the shape and size of the moving body
(a) the velocity of point A is 2vCM and velocity of point B is zero
81. A solid sphere of mass M rolls without slipping on an (b) the velocity of point A is zero and velocity of point B is 2vCM
inclined plane of inclination θ. What should be the (c) the velocity of point A is 2vCM and velocity of point B is −vCM
minimum coefficient of friction, so that the sphere rolls (d) the velocities of both A and B are vCM
down without slipping?
2 2 88. A disc is rolling without slipping on a horizontal surface
(a) tan θ (b) tan θ
5 7 with C, as its centre and Q and P the two points
5 equidistant from C. Let v P , v Q and vC be the magnitudes
(c) tan θ (d) tan θ
7 of velocities of points P, Q and C respectively, then

82. A solid sphere of mass m rolls without slipping on an Q


inclined plane of inclination θ. The linear acceleration of C
the sphere is P
7 2
(a) g sin θ (b) g sin θ (a) vQ > vC > vP (b) vQ < vC < vP
5 7
3 5 1
(c) g sin θ (d) g sin θ (c) vQ = vP , vC = vP (d) vQ < vC > vP
7 7 2

83. A ring is kept on a rough inclined surface. But the Miscellaneous Problems
coefficient of friction is less than the minimum value
required for pure rolling. At any instant of time let K T 89. A rigid body rotates with an angular momentum L. If its
and K R be the translational and rotational kinetic rotational kinetic energy is made 4 times, its angular
energies of the ring, then momentum will become
(a) K R = KT (b) K R > KT (a) 4L (b) 16L
(c) KT > K R (d) K R = 0 (c) 2 L (d) 2L

84. A sphere is rolling down a plane of inclination θ to the 90. A constant torque acting on a uniform circular wheel
horizontal. The acceleration of its centre down the plane is changes its angular momentum from A 0 to 4 A 0 in 4 s.
(a) g sin θ The magnitude of this torque is
(b) less than g sin θ (a)
3 A0
(b) 4 A0
(c) greater than g sin θ 4
(d) zero (c) A0 (d) 12 A0
382 Objective Physics Vol. 1

91. A uniform rod of mass 2 kg and length 1 m lies on a 97. A particle is moving in a circular orbit with constant
smooth horizontal plane. A particle of mass 1 kg moving speed. Select wrong alternate.
at a speed of 2 m /s perpendicular to the length of the rod (a) Its linear momentum is conserved
1 (b) Its angular momentum is conserved
strikes it at a distance m from the centre and stops.
4 (c) It is moving with variable velocity
What is the angular velocity of the rod about its centre (d) It is moving with variable acceleration
just after the collision?
98. A fly wheel is in the form of a uniform circular disc of
(a) 3 rad /s (b) 4 rad /s
(c) 1 rad /s (d) 2 rad /s radius 1 m and mass 2 kg. The work which must be done
on it to increase its frequency of rotation from 5 rev s −1
92. A ring and a disc of different masses are rotating with the to 10 rev s −1 is approximately
same kinetic energy. If we apply a retarding torque τ on
the ring, it stops after making n revolutions. After how (a) 1.5 × 102 J (b) 3.5 × 102 J
many revolutions will the disc stop, if the retarding (c) 1.5 × 103 J (d) 3.0 × 103 J
torque on it is also τ ?
n 99. If F be a force acting on a particle having the position
(a)
2
(b) n vector r and τ be the torque of this force about the origin,
(c) 2n (d) Data insufficient then
(a) r × τ = 0 and F × τ = 0 (b) r × τ = 0 and F × τ ≠ 0
93. A thin bar of mass m and length l is free to rotate about a (c) r × τ ≠ 0 and F × τ ≠ 0 (d) r × τ ≠ 0 and F × τ = 0
fixed horizontal axis through a point at its end. The bar is
brought to a horizontal position (θ = 90° ) and then 100. A body is under the action of two equal and oppositely
released. The angular velocity when it reaches the lowest directed forces and the body is rotating with constant
point is non-zero angular acceleration. Which of the following
(a) directly proportional to its length and inversely proportional cannot be the separation between the lines of action of
to its mass the forces?
(b) independent of mass and inversely proportional to the square (a) 1 m (b) 0.4 m (c) 0.25 m (d) Zero
root of its length
(c) dependent only upon the acceleration due to gravity and the 101. A particle P is moving in a circle of radius a with a
mass of the bar uniform speed u. C is the centre of the circle and AB is a
(d) directly proportional to its length and inversely proportional diameter. The angular velocities of P about A and C are
to the acceleration due to gravity in the ratio
(a) 1 : 1 (b) 1 : 2
94. The rotational KE of a body is E and its moment of (c) 2 : 1 (d) 4 : 1
inertia is I. The angular momentum is
(a) EI (b) 2 (EI ) 102. A particle performs uniform circular motion with an
E angular momentum L. If the frequency of the particle
(c) (2EI ) (d)
I motion is doubled, the angular momentum becomes
L L
95. A disc is free to rotate about a (a) 2L (b) 4L (c) (d)
2 4
smooth horizontal axis passing
through its centre of mass. A particle 103. A rod of uniform mass and of length L P
is fixed at the top of the disc. A slight P'
can freely rotate in a vertical plane
push is given to the disc and it starts about an axis passing through O. The
rotating. During the process angular velocity of the rod when it α
(a) only mechanical energy is conserved falls from position P to P ′ through an
(b) only angular momentum (about the
axis of rotation) is conserved
angle α is
(c) both mechanical energy and angular momentum are 6g
conserved (a) sin α O
5L
(d) neither the mechanical energy nor the angular momentum
are conserved 6g α
(b) sin
L 2
96. Work done by friction in case of pure rolling
6g α
(a) is always zero (c) cos
L 2
(b) is always positive
6g
(c) is always negative (d) sin α
(d) may be positive, negative or zero L
[ Level 2 ]
Only One Correct Option 6. A solid sphere rolls without
1. A ball rolls without slipping. The radius of gyration of slipping and presses a spring of
the ball about an axis passing through its centre of mass spring constant k as shown in
M v
is K. If radius of the ball be R, then the fraction of total figure. Then, the compression in k
energy associated with its rotational energy will be the spring will be
K2 R2
(a) (b) 2M 2M
K2 + R 2 K2 + R 2 (a) v
3k
(b) v
5k
K2 + R 2 K2 5k 7M
(c) (d) (c) v (d) v
R2 R2 7M 5k
2. A particle of mass m is projected with velocity u at an 7. A portion of a ring of radius R has been removed as
angle of θ with the horizontal. The initial angular shown in figure. Mass of the remaining portion is m.
momentum of the particle about the highest point of its Centre of the ring is at origin O. Let I A and I O be the
trajectory is equal to moment of inertias passing through points
mu3 sin 2 θ cos θ y
(a)
3g
3 mu3 sin 2 θ cos θ
(b)
2g
A x
mu3 sin 2 θ cos θ O
(c)
2g
(d) None of the above

3. A uniform rod of mass 2 kg and length 1 m lies on a


smooth horizontal plane. A particle of mass 1 kg moving
at a speed of 2 m/s perpendicular to the length of the rod A and O are perpendicular to the plane of the ring. Then,
1
strikes it at a distance m from the centre and stops. (a) I O = mR 2 (b) I O = I A
4 (c) I O > I A (d) I A > I O
What is the angular velocity of the rod about its centre
just after the collision? 8. Consider three solid spheres, sphere (i) has radius r and
(a) 3 rad/s (b) 4 rad/s mass m, sphere (ii) has radius r and mass 3m, sphere
(c) 1 rad/s (d) 2 rad/s (iii) has radius 3r and mass m. All can be placed at the
same point on the same inclined plane, where they will
4. A cord is wound around the circumference of wheel of roll without slipping to the bottom. If allowed to roll
radius r. The axis of the wheel is horizontal and MI is I. A down the incline, then at the bottom of the incline
weight mg is attached to the end of the cord and falls (a) sphere (i) will have the largest speed
from rest. After falling through a distance h, the angular (b) sphere (ii) will have the largest speed
velocity of the wheel will be (c) sphere (ii) will have the largest kinetic energy
1/ 2
2gh  2mgh  (d) all the spheres will have equal speeds
(a) (b)  
I + mr2  I + mr2  m
9. A circular disc of mass m
1/ 2
 2mgh  and radius R rests flat on a R
(c)   (d) 2gh
 I + 2mr2  horizontal frictionless m v
surface. A bullet, also of
5. The speed of a homogeneous solid sphere after rolling mass m and moving with a velocity v, strikes the disc and
down an inclined plane of vertical height h, from rest gets embedded in it.
without sliding is The angular velocity with which the system rotates after
 g the bullet strikes the hoop is
(a) gh (b)   gh
 5 v v
(a) (b)
2R 3R
 4  10
(c)   gh (d)   gh 2v 3v
 3  7 (c) (d)
3R 4R
384 Objective Physics Vol. 1

10. A small pulley of radius 20 cm and 15. A uniform rod AB of length 7m is undergoing combined
moment of inertia 0.32 kg -m 2 is used to rotational and translational motion such that at some
hang a 2 kg mass with the help of massless instant of time, velocities of its end points A and centre C
string. If the block is released, for no are both perpendicular to the rod and opposite in
slipping condition acceleration of the direction, having magnitudes 11 m/s and 3 m/s
2 kg respectively as shown in the figure.
block will be
11 m/s A
(a) 2 m/s2 (b) 4 m/s2
(c) 1 m/s2 (d) 3 m/s2

11. A disc of mass m0 rotates freely about a fixed horizontal C 3 m/s


axis through its centre. A thin cotton pad is fixed to its
rim, which can absorb water?
B

Velocity of centre C and angular velocity of the rod


remain constant
(a) acceleration of point A is 56 m/s2
(b) acceleration of point B is 56 m/s2
(c) at the instant shown in the figure acceleration of point B is
more than that of point A
The mass of water dripping onto the pad is µ per second. (d) angular velocity of the rod is 4 rad/s
After what time will the angular velocity of the disc get 16. A rod of length L whose lower end is fixed along the
reduced to half of its initial value? horizontal plane starts to topple from the vertical
2m0 3m0 m0 m0
(a) (b) (c) (d) position. The velocity of the upper end of the rod when it
µ µ µ 2µ
hits the ground is
12. A solid sphere and a solid cylinder of same mass are (a) 3gL (b) 2gL (c) gL (d) 5gL
rolled down on two inclined planes of heights h1 and h2
17. A disc of mass m and radius R is rolling on horizontal
respectively. If at the bottom of the plane the two objects
ground with linear velocity v. What is the angular
have same linear velocities, then the ratio of h1 : h2 is
momentum of the disc about an axis passing through
(a) 2 : 3 (b) 7 : 5
(c) 14 : 15 (d) 15 : 14 bottommost point and perpendicular to the plane of
motion?
13. If I 1 is the moment of inertia of a thin rod about an axis 3 1 4
(a) mvR (b) mvR (c) mvR (d) mvR
perpendicular to its length and passing through its centre 2 2 3
of mass and I 2 is the moment of inertia of the ring about
an axis perpendicular to plane of ring and passing 18. A force F is applied on the top of a F
cube as shown in figure. The
through its centre formed by bending the rod, then
I1 3 I1 2 coefficient of friction between the
(a) = (b) = cube and the ground is µ. If F is
I2 π 2 I2 π 2
gradually increased, the cube will topple before sliding,
I1 π 2 I1 π 2
(c) = (d) = if
I2 2 I2 3 1 1
(a) µ > 1 (b) µ < (c) µ > (d) µ < 1
2 2
14. A horizontal disc rotates freely about a vertical axis
through its centre. A ring, having the same mass and 19. Two uniform rods of equal length but different masses
radius as the disc is now gently placed on the disc. After are rigidly joined to form an L-shaped body, which is
some time, the two rotate with a common angular then pivoted as shown in figure. If in equilibrium the
velocity body is in the shown configuration, ratio M / m will be
(a) some friction exists between the disc and the ring
(b) the angular momentum of the disc plus ring is conserved
m M
2
(c) the final common angular velocity is rd of the initial
3 90°
angular velocity of the disc 30°
2
(d) rd of the initial kinetic energy is converted into heat
3 (a) 2 (b) 3 (c) 2 (d) 3
Rotation 385

20. The figure shows a uniform rod Y 26. Two men each of mass m stand on the rim of a
lying along the x-axis. The locus horizontal circular disc, diametrically opposite to each
of all the points lying on the x-y other. The disc has a mass M and is free to rotate about a
plane, about which the moment vertical axis passing through its centre of mass. Each
of inertia of the rod is same as X mass start simultaneously along the rim clockwise and
that about O is O reaches their original starting points on the disc. The
(a) an ellipse (b) a circle angle turned through by the disc with respect to the
(c) a parabola (d) a straight line ground (in radian) is
8m π 2m π
21. Two discs have same mass and thickness. Their materials (a) (b)
4m + M 4m + M
are of densities d1 and d 2 . The ratio of their moments of
mπ 4m π
inertia about an axis passing through the centre and (c) (d)
M +m 2M + m
perpendicular to the plane is
2 2
d  d  27. A ring of mass m and radius R has three particles attached
(a) d1 : d2 (b) d2 : d1 (c)  1  (d)  2 
 d2   d1  to the ring as shown in the figure. The centre of the ring
has a speed v 0 . The kinetic energy of the system in case
22. A solid sphere of mass 2 kg rolls up a 30° incline with an of no slipping is
initial speed of 10 m/s. The maximum height reached by
the sphere is ( g = 10 m/s 2 ) m
2m m
(a) 3.5 m (b) 7.0 m
(c) 10.5 m (d) 14.0 m

23. A hole of radius R/ 2 is cut from a thin circular plate of


(a) 6m v02
radius R and mass M. The moment of inertia of the plate
(b) 12m v02
about an axis through O perpendicular to the x-y plane
(i.e. about the z-axis) is (c) 4 mv02
y (d) 8mv02

R/2 More than One Correct Options


R
x 1. A mass m of radius r is rolling horizontally without any
O
slip with a linear speed v. It then rolls upto a height given
3 v2
by
5 7 13 13
4 g
(a) MR 2 (b) MR 2 (c) MR 2 (d) MR 2 (a) the body is identified to be a disc or a solid cylinder
7 12 32 24
(b) the body is a solid sphere
24. If a disc of mass m and radius r is reshaped into a ring or (c) moment of inertia of the body about instantaneous axis of
radius 2r, the mass remaining the same, the radius of 3
rotation is mr2
gyration about centroidal axis perpendicular to plane 2
goes up by a factor of (d) moment of inertia of the body about instantaneous axis of
7
(a) 2 (b) 2 rotation is mr2
5
(c) 2 2 (d) 4

25. A spool is pulled horizontally by two equal and opposite 2. Four identical rods each of mass m and length l are joined
forces as shown in the figure. Which of the following to form a rigid square frame. The frame lies in the x - y
statements is correct? plane, with its centre at the origin and the sides parallel to
the x and y-axes. Its moment of inertia about
F 2 2
(a) the x-axis is ml
3
F 4
(b) the z-axis is ml 2
Rough 3
(c) an axis parallel to the z-axis and passing through a corner is
(a) The centre of mass moves towards left 10 2
(b) The centre of mass moves towards right ml
3
(c) The centre of mass remains stationary 5
(d) The net torque about the centre of mass of the spool is zero. (d) one side is ml 2
3
386 Objective Physics Vol. 1

3. A uniform circular ring rolls without B 7. A uniform rod of length l and mass 2 m rests on a smooth
slipping on a horizontal surface. At horizontal table. A point mass m moving horizontally at
any instant, its position is as shown A
v right angles to the rod with velocity v collides with one
C
O end of the rod and sticks it. Then
in the figure. Then
(a) section ABC has greater kinetic 2v
(a) angular velocity of the system after collision is
energy than section ADC D 5l
(b) section BC has greater kinetic energy than section CD v
(b) angular velocity of the system after collision is
(c) the section BC has the same kinetic energy as section DA 2l
(d) the sections CD and DA have the same kinetic energy (c) the loss in kinetic energy of the system as a whole as a result
3
of the collision is mv 2
4. A cylinder of radius R is to roll without slipping between 10
two planks as shown in the figure. Then (d) the loss in kinetic energy of the system as a whole as a result
7mv 2
v of the collision is
24
8. A non-uniform ball of radius R and radius of gyration
about geometric centre = R/ 2, is kept on a frictionless
surface. The geometric centre coincides with the centre
of mass. The ball is struck horizontally with a sharp
3v impulse = J. The point of application of the impulse is at
a height h above the surface. Then
v (a) the ball with slip on surface for all cases
(a) angular velocity of the cylinder is counter clockwise
R (b) the ball will roll purely, if h = 5R / 4
2v (c) the ball will roll purely, if h = 3R / 2
(b) angular velocity of the cylinder is clockwise
R (d) there will be no rotation, if h = R
(c) velocity of centre of mass of the cylinder is v towards left
9. A hollow spherical ball is given an initial push up an
(d) velocity of centre of mass of the cylinder is 2v towards right
incline of inclination angle α. The ball rolls purely.
5. A uniform rod of mass m = 2 kg and length l = 0.5 m is Coefficient of static friction between ball and incline = µ.
sliding along two mutually perpendicular smooth walls During its upwards journey
with the two ends P and Q having velocities v P = 4 m/s (a) friction acts up along the incline
and v Q = 3 m/s as shown in figure. Then, (b) µ min = (2 tan α)/ 5
(c) friction acts down along the incline
(d) µ min = (2 tan α )/ 7
Q
10. A uniform disc of mass m and radius R rotates about a
fixed vertical axis passing through its centre with angular
vQ = 3 m/s velocity ω. A particle of same mass m and having velocity
of 2ωR towards centre of the disc collides with the disc
P moving horizontally and sticks to its rim.
vP = 4 m/s (a) The angular velocity of the disc will become ω/3
(b) The angular velocity of the disc will become 5 ω/ 3
(a) The angular velocity of rod,ω = 10 rad /s, counter clockwise 37
(b) The angular velocity of rod, ω = 5.0 rad /s, counter (c) The impulse on the particle due to disc is mωR
3
clockwise
(d) The impulse on the particle due to disc is 2mωR
(c) The velocity of centre of mass of rod, vcm = 2.5 m/s
25 11. The end B of the rod AB which y
(d) The total kinetic energy of rod, K = J A
3 makes angle θ with the floor is
6. A wheel is rolling without C being pulled with a constant
slipping on a horizontal plane velocity v 0 as shown in figure.
The length of the rod is l. θ v0
with velocity v and acceleration O x
B
a of centre of mass as shown in
B (a) Atθ = 37° velocity of end A is 4 v
0
3
figure. Acceleration at
A downwards
(a) A is vertically upwards 5v0
(b) B may be vertically downwards (b) At θ = 37° angular velocity of rod is
3l
(c) C cannot be horizontal
(c) Angular velocity of rod is constant
(d) a point on the rim may be horizontal leftwards
(d) Velocity of end A is constant
Rotation 387

Comprehension Based Questions Assertion and Reason


Passage I (Q. 1 to 3) Directions (Q. Nos. 1-20) These questions consists of two
Consider a uniform disc of statements each printed as Assertion and Reason. While
mass m, radius r, rolling F answering these questions you are required to choose any one
without slipping on a rough of the following five responses.
α
surface with linear (a) If both Assertion and Reason are correct and Reason is
acceleration a and angular the correct explanation of Assertion
acceleration α due to an
(b) If both Assertion and Reason are true but Reason is
external force F as shown in the figure. Coefficient of
not the correct explanation of Assertion
friction is µ
(c) If Assertion is true but Reason is false
1. The work done by the frictional force at the instant of (d) If Assertion is false but Reason is true
pure rolling is (e) If both Assertion and Reason are false
µmgat 2
(a) (b) µmgat 2 1. Assertion Moment of inertia about an axis passing
2
at 2 through the centre of mass is always minimum.
(c) µmg (d) zero
α Reason Theorem of parallel axis can be applied for 2-D
as well as 3-D bodies.
2. The magnitude of frictional force acting on the disc is
ma 2. Assertion A sphere is placed in pure rolling condition
(a) ma (b) µmg (c) (d) zero
2 over a rough inclined surface. Then, force of friction will
3. Angular momentum of the disc will be conserved about act in downward direction.
(a) centre of mass
(b) point of contact
(c) a point at a distance 3R/ 2 vertically above the point of
contact
(d) a point at a distance 4 R/ 3 vertically above the point of
contact

Passage II (Q. 4 to 6) θ
A tennis ball, starting from rest, rolls down the hill in the
drawing. At the end of the hill the ball becomes airborne, Reason Angular acceleration (actually retardation) due
leaving at an angle of 37° with respect to the ground. to friction is anti-clockwise.
Treat the ball as a thin-walled spherical shell. 3. Assertion A ring and a disc of same mass and radius
begin to roll without slipping from the top of an inclined
v surface at t = 0. The ring reaches the bottom of incline in
h
time t 1 while the disc reaches the bottom in time t 2 , then
37° H t1 < t 2 .
Reason Disc will roll down the plane with more
x
acceleration, because of its lesser value of moment of
inertia.
4. The velocity of projection v is
10 4. Assertion A body is moving along a circle with a
(a) 2gh (b) gh
7 constant speed. Its angular momentum about the centre
5 6 of the circle remains constant.
(c) gh (d) gh
7 5 Reason In this situation, a constant non-zero torque acts
on the body.
5. Maximum height reached by ball H above ground is
9h 18h 18h 27h 5. Assertion A solid and a hollow sphere both of equal
(a) (b) (c) (d)
35 35 25 125 masses and radii are put on a rough surface after rotating
with some angular velocity say ω 0 . Coefficient of
6. Range x of the ball is
144 48
friction for both the spheres and ground is same. Solid
(a) h (b) h sphere will start pure rolling first.
125 25
48 24 Reason Radius of gyration of hollow sphere about an
(c) h (d) h
35 7 axis passing through its centre of mass is more.
388 Objective Physics Vol. 1

6. Assertion Speed of any point or rigid body 14. Assertion If a particle moves with a constant velocity,
executing rolling motion can be calculated by then angular momentum of this particle about any point
expression v = r ω, where r is distance of point from remains constant.
instantaneous centre of rotation.
Reason Angular momentum has the units of Planck’s
Reason Rolling motion of rigid body can be considered
constant.
as a pure rotation about instantaneous centre of rotation.
15. Assertion Moment of inertia about an axis passing
7. Assertion The condition of equilibrium for a rigid body is
through centre of mass is maximum.
Translational equilibrium Σ F = 0 and
Rotational equilibrium Στ = 0 Reason Theorem of parallel axis can be applied only for
two dimensional body of negligible thickness.
Reason A rigid body must be in equilibrium under the
action of two equal and opposite forces. 16. Assertion If we draw a circle around the centre of mass
of a rigid body, then moment of inertia about all parallel
8. Assertion The angular velocity of a rigid body in
motion is defined for the whole body. axes passing through this circle has a constant value.
Reason All points on a rigid body performing pure Reason Dimensions of radius of gyration are [ M 0 LT 0 ].
rotational motion are having same angular velocity.
17. Assertion In rotational plus translational motion of a
9. Assertion A uniform disc of rigid body different particles of the rigid body may have
radius R is performing impure v0
different velocities but they will have same accelerations.
ω0
rolling motion on a rough
horizontal plane as shown in Reason Translational motion of a particle is equivalent
figure. After some time the disc to the translational motion of a rigid body.
ω R
comes to rest. It is possible only when v 0 = 0 .
2 18. Assertion Two identical solid spheres are rotated from
Reason For a body performing pure rolling motion, the rest to same angular velocity ω about two different axes
angular momentum is conserved about any point in as shown in figure. More work will have to be done to
space.
rotate the sphere in case-2.
10. Assertion A solid sphere and a ring of same mass and 1 2
radius are released simultaneously from the top of an
inclined surface. The two objects roll down the plane
without slipping. They reach the bottom of the incline
with equal linear speeds.
Reason Decrease in potential energy for both is the
same.
11. Assertion If a particle is rotating in a circle, then angular
momentum about any point is mvR. Reason Moment of inertia in case-2 is more.
Reason In circular motion, angular momentum about 19. Assertion Angular momentum of sun and planet system
centre is always constant. about any point remains constant.
12. Assertion A solid sphere cannot roll without slipping on Reason Two equal and opposite forces will act on them.
smooth horizontal surface. Net torque of those two set of forces about any point is
Reason If the sphere is left free on smooth inclined zero.
surface, it cannot roll without slipping.
20. Assertion Two identical spherical balls are released
13. Assertion Two axes AB and CD are as shown in figure. from two inclined plane. First is sufficiently rough and
Given figure is of a semi circular ring. As the axis moves
second is smooth. Both the balls will have same kinetic
from AB towards CD, moment of inertia first decreases
then increases. energy on reaching the bottom.
A B

h h

C D

Reason Centre of mass lies somewhere between AB and


CD. Reason Linear velocity of second ball will be more.
Rotation 389

Match the Columns 4. A semicircular ring has mass m and radius R as shown in
figure. Let I 1 , I 2 , I 3 and I 4 be the moments of inertias of
1. Four rods of equal length l and mass m each from a the four axes as shown in figure. Axis 1 passes through
square as shown in figure. Moment of inertia about three centre and is perpendicular to plane of ring. Then, match
axes 1, 2 and 3 are say I 1 , I 2 and I 3 . Then, match the the following columns
following columns
3 4

3
1
1
2
2

Column I Column II
Column I Column II (A) I1 (p) mR 2
2
(A) I1 (p) 4 2
ml (B) I2 (q) 3 mR 2
3
2
2
(B) I2 (q) ml 2 (C) I3 (r) mR 2
3
(D) I4 (s) Data is insufficient
(C) I3 (r) 1 2
ml
2 5. A solid sphere is rotating about an axis as shown in
(s) None of the above figure. An insect follows the dotted path on the
circumference of sphere as shown. Then, match the
2. A disc rolls on ground without slipping. Velocity of following columns
centre of mass is v. There is a point P on circumference of
disc at angle θ. Suppose, v P is the speed of this point.
Then, match the following columns Insect

C v

θ
P Column I Column II
(A) Moment of inertia (p) Will remain constant
(B) Angular velocity (q) Will first increase, then decrease
Column I Column II (C) Angular momentum (r) Will first decrease, then increase
(D) Rotational kinetic energy (s) Will continuously decrease
(A) If θ = 60° (p) vP = 2 v
(B) If θ = 90° (q) vP = v 6. A uniform cube of mass m and side a is placed on a
(C) If θ = 120° (r) vP = 2v frictionless horizontal surface. A vertical force F is
applied to the edge as shown in figure. Match the
(D) If θ = 180° (s) vP = 3v
following (most appropriate choice)
F
3. If radius of the earth is reduced to half without changing
its mass, then match the following columns
Column I Column II A

(A) Angular momentum of the earth (p) Will become two times
Column I Column II
(B) Time period of rotation of the (q) Will become four times (a) mg/4 < F < mg /2 (p) Cube will move up.
earth
(b) F > mg /2 (q) Cube will not exhibit motion.
(C) Rotational kinetic energy of the (r) Will remain constant (c) F > mg (r) Cube will begin to rotate and slip at A.
earth
(d) F = mg/4 (s) Normal reaction effectively at a/3
(s) None of the above from A, no motion.
390 Objective Physics Vol. 1

7. A uniform sphere of mass m and radius R is placed on a 5. A sphere rolls down an inclined plane of inclination θ.
rough horizontal surface (figure). The sphere is struck What is the acceleration as the sphere reaches bottom?.
horizontally at a height h from the floor. Match the [O JEE]
following 5 3
(a) g sin θ (b) g sin θ
7 5
2 2
(c) g sin θ (d) g sin θ
h 7 5

6. A particle is moving in a circle with uniform speed v. In


moving from a point to another diametrically opposite
Column I Column II
point [O JEE]
(a) h = R / 2 (p) Sphere rolls without slipping with a constant (a) the momentum changes by mv
velocity and no loss of energy. (b) the momentum changes by 2mv
(b) h = R (q) Sphere spins clockwise, loses energy by (c) the kinetic energy changes by (1/ 2) mv 2
friction. (d) the kinetic energy changed by mv 2
(c) h = 3R / 2 (r) Sphere spins anti-clockwise, loses energy by
friction. 2012
(d) h = 7 R / 5 (s) Sphere has only a translational motion, loses
energy by friction. 7. A lamina is made by removing a small disc of diameter
2R from a bigger disc of uniform mass density and radius
Entrance Gallery 2R, as shown in the figure. The moment of inertia of this
lamina about axes passing through O and P is I O and I P ,
2014 respectively. Both these axes are perpendicular to the
1. A force F = 5i$ + 2$j − 5k$ acts on a particle whose position I
plane of the lamina. The ratio P to the nearest integer is
vector is r = i$ − 2$j + k$ . What is the torque about the IO [IIT JEE]
origin ? [Karnataka CET]
(a) 8$i + 10$j + 12 k$ (b) 8$i + 10$j − 12 k$ 2R
(c) 8i$ − 10$j − 8k$ (d) 10i$ − 10$j − k$
2R P
O
2. A body having a moment of inertia about its axis of
rotation equal to 3 kg-m 2 is rotating with angular
velocity of 3 rad s − 1 . Kinetic energy of this rotating body
is same as that of a body of mass 27 kg moving with a
velocity v. The value of v is [Karnataka CET] (a) 3 (b) 4 (c) 5 (d) 6
(a) 1 m s− 1 (b) 0.5 m s− 1 (c) 2 m s− 1 (d) 1.5 m s− 1 8. A small mass m is attached to a massless z
3. A solid uniform sphere resting on a rough horizontal string whose other end is fixed at P as
shown in the figure. The mass is P
plane is given a horizontal impulse directed through its
centre so that it starts sliding with an initial velocity v 0 . undergoing circular motion in the x- y
When it finally starts rolling without sliping the speed of plane with centre at O and constant
its centre is [WB JEE] angular speed ω . If the angular O m
momentum of the system, calculated ω
2 3 5 6
(a) v0 (b) v0 (c) v0 (d) v0 about O and P are denoted by LO and
7 7 7 7
L P respectively, then [IIT JEE]
2013 (a) L O and L P do not vary with time
4. A uniform circular disc of mass 50 kg and radius 0.4 m is (b) L O varies with time while L P remains constant
rotating with an angular velocity of 10 rad/s about its (c) L O remains constant while L P varies with time
(d) L O anad L P both vary with time.
own axis, which is vertical. Two uniform circular rings,
each of mass 6.25 kg and radius 0.2 m, are gently placed 9. A thin uniform rod, pivoted at O, is z
symmetrically on the disc in such a manner that they are rotating in the horizontal plane with ω
touching each other along the axis of the disc and are constant angular speed ω, as shown in v
horizontal. Assume that the friction is large enough such the figure. At time t = 0, a small insect O
that the rings are at rest relative to the disc and the system starts from O and moves with constant speed v w.r.t. the
rotates about the original axis. The new angular velocity rod towards the other end. It reaches the end of the rod at
(in rad /s −1 ) of the system is [JEE Advanced]
t = T and stops. The angular speed of the system remains
(a) 8 (b) 7 (c) 9 (d) 11
Rotation 391

ω throughout. The magnitude of the torque | τ | on the 2011


system about O, as a function of time is best represented
13. A thin horizontal circular rotating about a vertical axis
by which plot ? [IIT JEE]
passing through its centre. An insect is at rest at a point
near the rim of the disc. The insect now moves along a
diameter of the disc to reach its other end. During the
(a) |τ | (b) |τ |
journey of the insect, the angular speed of the disc
(a) continuously decreases [IIT JEE]
O t O T t
(b) continuously increases
T
(c) first increases and then decreases
(d) remains unchanged

14. A body is rotating with angular velocity 30 rad s −1 . If its


(c) |τ | (d) |τ |
kinetic energy is 360 J, then its moment of inertia is
(a) 0.8 kg-m 2 [IIT JEE]
O T t O T t (b) 0.4 kg-m 2
(c) 1 kg-m 2
10. Two identical discs of same radius R are rotating about (d) 1.2 kg-m 2
their axes in opposite directions with the same constant
15. A pulley of radius 2 m is rotating about its axis by a force
angular speed ω. The discs are in the same horizontal
F = ( 20t − 5t 2 ) N (where, t is measured in sec) applied
plane. At time t = 0, the points P and Q are facing each
other as shown in the figure.The relative speed between tangentially. If the moment of inertia of the pulley about
the two points P and Q is v r . In one time period (T ) of its axis of rotation is 10 kg - m 2 , the number of rotations
rotation of the discs, v r as a function of time is best made by the pulley before its direction of motion, if
represented by [IIT JEE] reserved, is [AIEEE]
(a) more than 3 but less than 6
ω ω (b) more than 6 but less than 9
(c) more than 9
P Q
(d) less than 6
R R
16. A mass m hangs with the help of a string wrapped around
a pulley on a frictionless bearing. The pulley has mass m
vr vr
and radius R. Assuming pulley to be a perfect uniform
(a) (b) circular disc, the acceleration of the mass m, if the string
does not slip on the pulley, is [AIEEE]
2
0 0 (a) g (b) g
T t T t 3
g 3
vr vr (c) (d) g
3 2
(c) (d)
17. A ring starts to roll down the inclined plane of height h
without slipping. The velocity with which it reaches the
0
T t 0
T t ground is [Kerala CEE]
10gh 4 gh
(a) (b)
11. A solid cylinder rolls from an inclined plane of height h. 7 7
The velocity of body at bottom is [O JEE] 4 gh
(c) (d) 2gh
2gh 4 gh 3
(a) (b)
3 3 (e) gh
3gh
(c) (d) gh 18. The angular momentum of a particle describing uniform
2
circular motion is L. If its kinetic energy is halved and
12. A wire is wound on a hollow cylinder of radius 40 cm. angular velocity doubled, its new angular momentum is
Mass of the cylinder is 3 kg. Force of 30 N is applied on [Kerala CEE]
L L
wire. The angular acceleration is [O JEE] (a) 4L (b) (c) (d) 2L
4 2
(a) 25 rad /s2 (b) 30 rad /s2 L
(e)
(c) 35 rad /s2 (d) 40 rad /s2 8
392 Objective Physics Vol. 1

19. A solid sphere of mass m rolls down an inclined plane 24. The moment of inertia of a thin uniform rod of length L
without slipping, starting from rest at the top of an and mass M about an axis passing through a point at a
inclined plane. The linear speed of the sphere at the distance of 1/3 from one of its ends and perpendicular to
bottom of the inclined plane is v. The kinetic energy of the rod is [MHT CET]
the sphere at the bottom is [Karnataka CET] ML2 ML2
(a) (b)
7 2 12 9
(a) mv 2 (b) mv 2
10 5 7ML2 ML2
(c) (d)
5 1 48 48
(c) mv 2 (d) mv 2
3 2
25. Moment of inertia of a rod of mass M and length L about
2010 an axis passing through a point midway between centre
and end is [MHT CET]
20. A binary star consists of two stars A (mass 2.2M S ) and B ML2 ML2
(mass 11M S ), where M S is the mass of the sun. They are (a) (b)
6 12
separated by distance d and are rotating about their centre 7ML2 7ML2
(c) (d)
of mass, which is stationary. The ratio of the total angular 24 48
momentum of the binary star to the angular momentum
of star B about the centre of mass is [IIT JEE] 26. From a disc of radius R, a concentric circular portion of
(a) 7 (b) 6 (c) 9 (d) 10 radius r is cut out so as to leave an annular disc of mass
M. The moment of inertia of this annular disc about the
21. From a circular ring of mass M and radius R, an arc axis perpendicular to its plane and passing through its
corresponding to a 90° sector is removed. The moment of centre of gravity is [MHT CET]
inertia of the remaining part of the ring about an axis 1
passing through the centre of the ring and perpendicular (a) M (R 2 + r2 )
2
to the plane of the ring is k times MR 2 . Then, the value of 1
k is [Kerala CEE] (b) M (R 2 − r2 )
2
3 7 1 1
(a) (b) (c) (d) 1 (c) M (R 4 + r4 )
4 8 4 2
1 1
(e) (d) M ′ (R 4 − r4 )
8 2
22. A wheel of moment of inertia 2.5 kg-m 2 has an initial 27. Which relation is not correct of the following? [MHT CET]
angular velocity of 40 rad s −1 . A constant torque of
(a) Torque = moment of inertia × angular acceleration
10 N-m acts on the wheel. The time during which the
(b) Torque = dipole moment × magnetic induction
wheel is accelerated to 60 rad s −1 is [Kerala CEE]
(c) Moment of inertia = torque/angular acceleration
(a) 4 s (b) 6 s (c) 5 s (d) 2.5 s
(e) 4.5 s (d) Linear momentum = moment of inertia × angular
velocity
23. When a disc is rotating with angular velocity ω , a particle
28. Moment of inertia of a disc about a diameter is I . Find
situated at a distance of 4 cm just begins to slip. If the
the moment of inertia of disc about an axis perpendicular
angular velocity is doubled, at what distance will the
to its plane and passing through its rim?
particle start to slip? [MHT CET]
[MHT CET, J&K CET]
(a) 1 cm (b) 2 cm (a) 6 I (b) 4 I
(c) 3 cm (d) 4 cm (c) 2 I (d) 8 I
Answers
Level 1
Objective Problems
1. (d) 2. (b) 3. (b) 4. (a) 5. (b) 6. (b) 7. (c) 8. (a) 9. (d) 10. (d)
11. (d) 12. (c) 13. (b) 14. (b) 15. (c) 16. (b) 17. (a) 18. (b) 19. (b) 20. (c)
21. (b) 22. (b) 23. (a) 24. (b) 25. (b) 26. (a) 27. (b,c,d) 28. (c) 29. (b) 30. (d)
31. (c) 32. (b) 33. (b) 34. (b) 35. (a) 36. (b) 37. (c) 38. (d) 39. (b) 40. (b)
41. (b) 42. (c) 43. (b) 44. (a) 45. (d) 46. (b) 47. (a) 48. (c) 49. (a) 50. (b)
51. (b) 52. (c) 53. (d) 54. (a) 55. (c) 56. (a) 57. (c) 58. (d) 59. (c) 60. (c)
61. (a) 62. (a) 63. (d) 64. (a) 65. (a) 66. (a) 67. (c) 68. (d) 69. (d) 70. (b)
71. (b) 72. (c) 73. (b) 74. (b) 75. (c) 76. (b) 77. (c) 78. (d) 79. (d) 80. (c)
81. (b) 82. (d) 83. (c) 84. (b) 85. (b) 86. (c) 87. (a) 88. (a) 89. (d) 90. (a)
91. (a) 92. (b) 93. (b) 94. (c) 95. (a) 96. (d) 97. (a) 98. (c) 99. (a) 100. (d)
101. (b) 102. (a) 103. (b)

Level 2
Only One Correct Option
1. (a) 2. (c) 3. (a) 4. (b) 5. (d) 6. (d) 7. (a,d) 8. (c,d) 9. (a) 10. (a)
11. (d) 12. (c) 13. (c) 14. (c) 15. (d) 16. (a,b,d) 17. (a,b,d) 18. (a) 19. (a) 20. (c)
21. (d) 22. (b) 23. (b) 24. (b) 25. (c) 26. (c) 27. (b)

More than One Correct Options


1. (a,c) 2. (all) 3. (a,b,d) 4. (a,d) 5. (a,c,d) 6. (all) 7. (a,c) 8. (b,d) 9. (a,b) 10. (a,c)
11. (a,b)

Comprehension Based Questions


1. (d) 2. (c) 3. (c) 4. (d) 5. (d) 6. (a)

Assertion and Reason


1. (d) 2. (d) 3. (d) 4. (c) 5. (a) 6. (a) 7. (c) 8. (b) 9. (c) 10. (d)
11. (e) 12. (d) 13. (a) 14. (b) 15. (b) 16. (b) 17. (d) 18. (a) 19. (a) 20. (b)

Match the Columns


1. (A → q, B → s, C → p) 2. (A → q, B → p, C → s, D → r) 3. (A → r, B → s, C → q) 4. (A → r, B → p, C → p, D → q)
5. (A → q, B → r, C → p, D → r) 6. (A → q, B → r, C → p, D → s) 7. (A → r, B → s, C → p, D → p)

Entrance Gallery
1. (a) 2. (a) 3. (c) 4. (a) 5. (a) 6. (b) 7. (a) 8. (c) 9. (a) 10. (a)
11. (b) 12. (a) 13. (c) 14. (a) 15. (a) 16. (b) 17. (e) 18. (b) 19. (a) 20. (b)
21. (a) 22. (c) 23. (a) 24. (b) 25. (d) 26. (b) 27. (d) 28. (b)
Solutions
Level 1 : Objective Problems 14. I = Iring + 3I spoke
1
3. IRing = mR , I disc = mR 2
2  mR 2 
= MR 2 + 3   = ( M + m)R 2
2  3 
4. K = radius of gyration
ml 2 ( 3m / 4)( 3l / 4)2
7 15. I = , I′=
mR 2 3 3
I 5 7
= = = R 27  ml 2  27
m m 5 =  = I
67  3  64
2
mR 2 + mR 2  Ml 2 
I 3 5
5. I H = = = R 16. I = 3   = Ml
2
m m 3  3 
2
mR 2 + mR 2 17. From theorem of parallel axes,
I 5 7
IS = = = R I 2 = I1 + MR 2
m m 5
18. Perpendicular distance from z-axis would be
IH 25 5
∴ = =
IS 21 21 (1)2 + (1)2 = 2 m
∴ I = mr 2 = (1)( 2 )2
mR 2 =  πR 3ρ R 2 =
2 2 4 176 5
6. I = ρR
5 5  3  105 = 2 kg -m2
7. I CM is less than I about any other axis not passing through  M (l / 2)2  Ml 2
19. I = 4  =
centre of mass but only when two axes are parallel. 2 3  6
8. Moment of inertia of the rod lying along z-axis will be zero.
20. For square lamina I 2 = I 3 . This value will be less than I1 ,
ML 2
Of the rods along x and y-axis will be each. Hence, total because mass is nearer to axis in this case.
3
2 21. Moment = Force × r⊥ = 10 × 2 = 20 N-m
moment of inertia is ML 2 .
3 y
I
9. Radius of gyration, K = r⊥ 10 N
m 1m
1
mR 2 + mR 2 x
2 3
K disc = = R 2m
m 2
mR 2 + mR 2 mR 2
K ring = = 2R 22. The desired ratio is = 2 :1
1
m mR 2
2
K disc 3/ 2 3
∴ = =
I (mL 2 /12) L
K ring 2 2 23. K = = =
m m 2 3
10. I = I CM + mx 2
24. I = 5mR 2 = 5 × 2 × (0.1)2 = 0.1 kg -m2
i.e. I-x graph is a parabola not passing through origin.
25. For a given body mass is same, so it will depend only on the
11. l = 2πR distribution of mass about the axis.
l The mass is farthest from axis BC, so I 2 is maximum. Mass is
∴ R=
2π nearest to axis AC, so I 3 is minimum.
ml 2 Hence, the correct sequence will be
I1 =
12 I 2 > I1 > I 3 .
ml 2 26. From theorem of perpendicular axes, we have
I 2 = mR 2 =
4π2 2
I = IC + m 
a 

I1 π 2  2
∴ =
I2 3  ma 2 ma 2  ma 2
= + +
12. Moment of inertia depends on the distribution of mass  12 12  2
about the axis.
2
13. Apply theorem of perpendicular axis. = ma 2
3
Rotation 395

27. If we complete the ring, its mass becomes 2 m. I2


K2 =
1 m2
∴ I Whole ring = ( 2M )( R 2 ) = MR 2
2
1 ( 3 / 2) m 2 R 2 3
∴ IHalf ring = ( MR 2 ) = = R
2 m2 2
This value is independent of angle θ. K1 5
28. ∴ =
1 2 K2 6
1
33. Iω = constant ω ∝
I
r
On stretching hands, I will increase, therefore ω will
decrease.
dL
34. τ = . If τ = 0,L = constant
dt
I1 = I 2 + ( 4M ) r 2 35. L = r × p

 ML2 
2 36. Iω = constant
 2 L
= 4 sin 2 45°  + 4M   R2  as I ∝ R 2 and ω ∝ 1 
 3   2  or = constant  
T  T
8
= ML2
3 ∴ T ∝R2
 ma 2 /4 m a 2   ma 2 /4 ma 2  1
As R ′ = R
29. I =  + +  +  n
 12 × 4 4 × 16   12 × 4 16 × 4  T 24
∴ T′= = h
 ma 2 /4 m  3a  2  n2 n2
+ +   
 12 × 4 4 4  37. ω1 I1 = ω2 I 2
3ma 2 ω1 I1
= ∴ ω2 =
16 I2

30. { IC }circular =
1
( 2m) r 2 = mr 2  MR 2 
or ω2 = ω  2
2 2
+
 MR 2mR 
 M 
=ω 

C  M + 2m 
C 38. L = mvr1 = ( 5)( 3 2 )( 2 5 ) = 30 10 g-cm2 / s
y
1
∴ { IC }semicircle = mr 2 m v
2
= I CM + mx 2 r⊥= y = 2 5 cm
1
∴ I CM = mr 2 − mx 2 x
2
31. Minimum moment of inertia is about centre of mass. 39. r⊥ = 4 sin 45° = 2 2 unit
I min = m1r12 + m 2r22
y
2 2 v = 3 2 unit
 m2   m1 
= m1  l  + m2  l 45°
m + m  m + m 
 1 2   1 2  4
r⊥
 m1m 2  2 45° x
= l O
m + m 
 1 2

I1 ( 5 / 4) m1 R 2 5
32. K 1 = = = R
m1 m1 4
L = mvr⊥ = 5( 3 2 )( 2 2 ) = 60 unit
40. I1ω1 = I 2ω2
I1
∴ ω2 = ⋅ ω1
I2
(1/2 ma 2 ) ω
= ⋅ω =
(1/2 ma 2 ) + ma 2 3
396 Objective Physics Vol. 1

41. τ 0 = mg × r⊥
MR 2  0 
2 v
L = MRv 0 +
y 5 R
7
=MRv 0
5
52. Angle rotated θ = area under ω-t graph.
x 53. If r, the distance of point from axis will increase, then
O r⊥ mg corresponding v of the point will also increase accordingly.
Hence, ω is constant.
As r⊥ is continuously increasing or torque is continuously
54. 0 = ω0 − αt
increasing on the particle. Hence, angular momentum is
ω (100 × 2 π )/60
continuously increasing. ∴ α= 0 =
t 15
42. τ = r × F should be zero. Or r should be parallel to F.
= 0.7 rad/s2
I1
43. I1ω1 = I 2ω2 or ω2 = ω1 ω20
I2 Now, angle rotated before coming to rest θ =

(1 / 2 MR 2 )
= ω (100 × 2 π / 60)2
1 or θ=
(1 / 2 MR 2 ) + (1/4 MR 2 ) 2 × 0.7
2
4 = 78.33 rad
= ω θ
5 or number of rotations, n = = 12.5

44. τ = r × F
1
55. θ1 = (α )( 2)2 = 2α ,
45. About origin, angular momentum will be 2
 $i j$ k$  1 1
θ2 = α ( 4)2 − α ( 2)2 = 6α
  2 2
m( r × v ) = 2 1 1 0
  θ2 3
 2 −2 2  ∴ =
  θ1 1
= 4$i − 4$j − 8k$ 900 × 2 π
56. ω0 = rad/s = 30 π rad/s
Therefore, component of angular moment about z-axis 60
would be −8 kg-m2 /s. Now, 0 = ω0 − αt
ω 30 π
46. Force 8 N can also be resolved in tangential and radial or α= 0 = rad/s 2
t 60
directions.
π
τ net = ( 9 + 8 sin 30° − 4) (0.2) N-m clockwise = rad/s2
2
=1.8 N-m clockwise

47. τ = r × F 57. ω = = 6t , At t = 2s
2
dt
48. For total torque about O to be zero, ω = 6( 2)2 = 24 rad/s
A
τ 1000
58. Angular acceleration, α = = = 5 rad/s2
I 200
r r
ω = αt = ( 5)( 3) = 15 rad/s
O
r 59. Area under n-t graph will give total number of revolutions
B F3 F2
before coming to stop.
F1
Area = Total number of revolutions
= × 8 × 
1800 
+ × 16 × 
F1r + F2r − F3r = 0 1 600 1 600 
 + 8× 
⇒ F3 = F1 + F2 2  60  60 2  60 

50. L = mvr⊥ = 120 + 80 + 80


= 280
60. ω = αt
v/√2
ω
∴ α=
r⊥ t
h
80
= = 16 rad/s2
5
51. About bottommost point, 1
θ = αt 2
2
1
v0 = (16)( 5)2
2
= 200 rad
Rotation 397

61. ω = α − βt. Comparing with ω = ω0 − αt KR 2


74. For a sphere = in case of pure rolling.
Initial angular velocity = α KT 5
Angular retardation = β Hence,
KR 2
=
α2 K 7
∴ Angle rotated before it stops is
2β tan α tan α 1  as I = 1 mR 2 
75. µ min = = = tan α  
mR 2 1 + 2 3  2 
from 0 = α − 2βθ
2
1+
I
62. I = mK = 10( 2) = 40 kg-m2
2 2
76. In case of pure rolling, velocity of topmost point is 2 times
1 1
K R = Iω2 = ( 40)( 5)2 = 500 J the velocity of centre of mass.
2 2
77. In case of pure rolling,
ω0
63. α = (Q0 = ω0 − αt ) g sin θ I 2
t a= ; =
I mR 2
5
2 π × 33 / 60 11 π 1+
= = rad /s2 mR 2
20 200
5
K I 2 ∴ a = g sin θ
64. R = = 7
K T mR 2 5
Since, a is independent of m and R, both spheres reach the
KR 2
65. = bottom together.
KT 5 g sin θ
78. In case of pure rolling, a =
K T =  Mv 2  = Mv 2
7 7 1 7 I
∴ K = 1+
5 52  10 mR 2
66. On smooth horizontal surface, if v = Rω, I 2
= for solid sphere
but external force = 0. Then, it can roll without slipping. mR 2 5
I 1
67. v P = r ω = ( 2 R ) ω = 2 v 0 and = for a disc.
mR 2 2
ω 5
∴ For a solid sphere, a1 = g sin θ
7
v0 ⇒ P
2
r and for a hollow sphere, a2 = g sin θ
3
v0 = Rω ω 1 2 2s
From s = at , t =
2 a
68. Acceleration of all bodies on smooth surface = g sin θ t1 a2
or =
(equal). t2 a1
∴ All bodies will take equal time.
2/ 3 14
69. For all bodies a = g sin θ − µg cos θ = =
5/ 7 15
Hence, all will take equal time. g sin θ I 2
79. a = , = for solid sphere
70. Translational kinetic energy of all bodies is same. As a is I mR 2
5
1+
same, but moment of inertia of hollow sphere is maximum. mR 2
Hence, rotational kinetic energy for hollow sphere will be g sin 30° 5g
∴ a= =
least, as its angular acceleration will be least. Because force 2 14
1+
of friction for all bodies is same (maximum). Hence, total 5
kinetic energy of hollow sphere will be minimum. 80. In case of pure rolling,
71. v P = r ω = ( 2R ) ω = 2v mg sin θ
Force of friction, f =
 mR 2 
P 1+  
 I 

v ⇒ r Since, moment of inertia of disc is more, force of friction on


cylinder will be more.
ω
tan θ tan θ 2
81. µ min = = = tan θ
v0 = R ω ω mR 2 1 + 5 7
1+
I 2
72. µ < µ min but µ ≠ 0. Therefore, sphere will roll with forward g sin θ
slipping. 82. a =
I
1+
73. Since, the sphere is sliding, maximum force of friction will mR 2
act passing through the bottommost point. Torque of this g sin θ 5
= = g sin θ
force about point of contact is zero. Hence, angular 2 7
1+
momentum will remain conserved. 5
398 Objective Physics Vol. 1

1 2
83. Friction in this case is backwards. If it is insufficient, it is a 94. E = Iω
case of forward slipping or K T > K R . 2
g sin θ ω=
2E
84. a = or a < g sin θ
I I
1+
mR 2 Now, L = Iω = 2EI
85. In pure rolling mechanical energy remains conserved. 95. Angular momentum will not remain conserved due to a
Therefore, at bottommost point, total kinetic energy will be torque of weight of particle about axis of rotation.
mgh. 96. It depends whether pure rolling is taking place on ground or
2 over a platform.
Ratio of rotational to translational kinetic energy will be .
3 97. Direction of linear velocity always keeps on changing.
3 Hence, linear momentum is varying.
∴ K T = (mgh)
5 1
98. W = change in rotational kinetic energy = I (ω2f − ωi2 )
1 2
= mv 2
2 1 1 2
=  mR  [( 2 πf f ) − ( 2 πf i )2 ]
2

6gh 22 
∴ v=
5 1
= × 2 × (1)2 × 4 × π 2 (100 − 25)
86. Path of A is cycloid and distance travelled in one rotation is 4
8R. = 1.48 × 103 J ≈ 1.5 × 103 J

88. v = rω or v ∝ r 99. τ = r × F, i.e. τ is perpendicular to both r and F


∴ τ ⋅ r = 0 and τ ⋅ F = 0
100. Otherwise net torque or net angular acceleration will
become zero.
101. From the property of a circle, if an arc subtends an angle θ at
any point A on circumference, then it will subtend an angle
ω
2θ at centre C. So in the same time interval a particle rotating
rQ > rC > rP in a circle turn double the angle with respect to centre point
∴ vQ > vC > v P C compared to point A.
89. L = 2KI (Just like p = 2km) ∴ ωC = 2ωA
102. L = Iω = I ( 2 πf )
K is made 4 times. Hence, L will become 2 times as L ∝ K .
Frequency f is doubled. Hence, angular momentum will
90. Angular impulse = Change in angular momentum become 2L.
∴ τ × t = L f − Li 103. Decrease in gravitational PE = Increase in rotational KE
or 4τ = 4A0 − A0
3A
∴ τ= 0
4
91. Li − l f (about centre of mass of the rod) α
2 L /2 cosα
ML
mvr = ⋅ω α
12
12mvr O
∴ ω=
ML2 1 2
∴ mgh =Iω
(12)(1)( 2)  
1 2
 4
= = 3 rad/s 1  mL2  2
mg  − cosα  = 
L L
( 2)(1)2 ω
 2 2  2  3 
92. Work done by retarding torque = change in kinetic energy
3g
Since, kinetic energy of both are equal, both will rotate ∴ ω= (1 − cos α )
L
same revolutions.
3g  2 α 6g α
93. Decrease in gravitational potential energy = Increase in =  2 sin = sin
L  2 L 2
rotational kinetic energy about an axis passing through end
point. Level 2 : Only One Correct Option
l 1  ml 2  2 K2
∴ mg =  ω 1.
KR
=
I
, if v = Rω in case of pure rolling = 2
2 2 3  K T mR 2
R
3g g  K2  KR K 2
⇒ ω= or ω µ ∴ KR =  2  K Total or =
K + R  K Total K 2 + R 2
l l 2
Rotation 399

2. LP = (mu sin θ)   (mu cos θ)( H )


R 7. Whole mass has equal distance from the centre O. Hence,
 2 I 0 = mR 2 . Further centre of mass of the remaining portion
will be to the left of point O. More the distance of axis from
centre of mass, more is the moment of inertia.
P Hence, I A > I 0 .
usinθ g sin θ
8. a =
I
1+
mr 2
I 2
For a sphere, =
ucosθ mr 2 5
 u sin θ   u2 sin 2 θ  5
Hence, a = g sin θ = constant.
= (mu sin θ)   (u cos θ) = (mu cos θ)  
 g   2g  7
Hence, speed of all spheres is same at the bottom. Sphere
mu3 sin 2 θ cos θ
= (ii) has the largest mass. Hence, it will have the maximum
2g kinetic energy.
3. From Li = L f about centre of rod. 9. Conserved angular momentum about common centre of
mass.
m ⋅ vr⊥ = I CM ⋅ ω
 2
R 
2
mv = I Total ω = m   + mR 2 + m    ω
R R 1
(1)( 2)   = × ( 2)(1)2 ⋅ ω
1 1
⇒ 2  2 2  2
 4  12  
∴ ω = 3 rad/s v
∴ ω=
2R
4. v = r ω
a Ia
10. TR = Iα,TR = I or T = 2
Decrease in gravitational potential energy = Increase in R R
kinetic energy

R
T
ω

T
2 kg M
h
Mg − T = Ma
Mg = Ma + T =  M + 2  a
I
or
v  R 
Mg 2 × 10
or a= = = 2 m/s2
1 2 1  M + I   2 + 0.32 
∴ mgh = Iω + mv 2    
2 2  R2  0.04 
1 2 1 1
= Iω + m(r ω)2 11. I1ω1 = I 2ω2 , ω2 = ω1
2 2 2

∴ ω=
2mgh ∴ I 2 = 2I1
I + mr 2
= 2  m0 R 2 
1 1
m0 R 2 + (µt ) R 2
5. At bottommost point, total kinetic energy will be mgh. Ratio 2  2 
2 m0
of rotational to translational kinetic energy will be . ∴ t=
5 2µ
5 1
∴ K T = mgh = mv 2
12. ( K T )sphere = ( K T )cylinder (m and v are same)
7 2
10 5 2
∴ v= gh ∴ (mgh1 ) = (mgh 2 )
7 7 3
h1 14
6. In case of pure rolling, ratio of rotational to translational ∴ =
2 7 h 2 15
kinetic energy is . Therefore, total kinetic energy is times
5 5 l
13. l = 2πR ⇒ ∴ R=
the translational kinetic energy. At maximum compression 2π
whole of energy is elastic potential. Hence,
ml 2
7 1 2 1 I1 =
 Mv  = k x max
2
12
52  2
ml 2 I π2
7M I 2 = mR =
2
⇒ ∴ 1 =
∴ x max = v 4π 2
I2 3
5k
400 Objective Physics Vol. 1

14. I1ω1 = I 2ω2 19. Net torque about O should be zero


O
(mR 2 ) ω0 =  mR 2 + mR 2  ω2
1 1
∴ 60° 30°
2 2 
ω0
ω2 =
3
K i = I1ω12 =  mR 2  ω20
1 1 1
2 22 
mg Mg
1
= mR ω02 2
4 l l
∴ Mg sin 30° = mg sin 60°
 ω0 
2 2 2
K f =  mR 2 + mR 2 
1 1
  M sin 60°
22   3 = = 3
m sin 30°
1 K
= mR 2ω20 = i
12 3 20. From theorem of parallel axes, moment of inertia about an
l
2K i axis passing through a distance from centre in all
∴ is converted into heat. 2
3
ml 2
15. Relative velocity of A with respect to C perpendicular. directions will have moment of inertia . All such points
3
to AC is 14 m/s. at same distance from centre will lie in a circle.
14 14
∴ ω= = = 4 rad /s 21. ( πR 2td ) = m
AC 7 / 2
m
A and α are zero. Hence, acceleration of any point is ∴ R2 =
πtd
a n = r ω2
1 m2 1
Now, I = mR 2 = or I ∝
∴ |a A | =|a B | 2 2 πtd d
l 2 I1 d 2
= r ω2 =
⋅ ω = 56 m/s2 ∴ =
2 I 2 d1
16. Decrease in gravitational potential energy = Increase in
KR 2
rotational kinetic energy about point O. 22. In case of pure rolling, =
KT 5
7
⇒ ∴ Total kinetic energy = times the translational kinetic
5
O O ω energy. At highest point whole of the kinetic energy will be
converted into potential energy.
L 1  mL  2
2
3g
or mg =  ω or ω = ∴ mgh =
7 1 2
2 2 3  L  mv 
52 
Now, v =r ω (in pure rotation)
7v 2 7(10)2
3g or h= = = 7m
∴ v=L = 3gL 10g 10 × 10
L
1 3 23. Moment of inertia of remaining portion
17. L = mvR + IC ω = mvR + mvR = mvR
2 2 1 M 2
 R   = 13 MR
2
−     R +  M 
1
18. Cube will slide, if F > µmg ...(i) I= MR 2
      
2
2    2  4  2
2 4  32
N
1
mr 2
I 2 r
24. K 1 = 1 = =
m m 2
I2 m( 2r )2
K2 = = = 2r
f m m
O K2
mg
∴ =2 2
K1
Cube will topple about O, if 25. About bottommost point net torque is clockwise. Hence,
τ F > τmg centre of mass moves with acceleration a = Rα towards
F ⋅ a > mg  
a right.

 2
F
1
or F > mg ...(ii)
2 F
1
If µ > , Eq. (ii) will be satisfied earlier. α
2
Rotation 401

26. Let disc rotates anti-clockwise by an angle θ. Then, angular 3. O → Instantaneous axis of rotation
displacement of each men is ( 2π − θ) in clockwise direction.
In the absence of external torque. P
Applying : ΣI A θA = ΣIC θC (A = anti-clockwise, C = clockwise) r v
1
MR 2 ⋅ θ = 2(mR 2 )( 2 π − θ)
2 O ω
8mπ
or θ=
4m + M v =rω
or v ∝r
27. In case of a ring, its translational and rotational kinetic
energies are equal. Hence, total kinetic energy of ring will More the value of r from O more is the speed of point P.
be mv 02 . 4. v 0 + Rω0 = v ...(i)

Kinetic energy of system v 0 − Rω0 = 3v ...(ii)


1 1
= mv 02 + ( 2m)( 2 v 0 )2 + × m × ( 2v 0 )2
2 2 v0
1
+ × m × ( 2 v 0 )2 ω0
2
= 6mv 02 Solving these two equations,
we get
More than One Correct Options v
v 0 = 2v and ω0 = −
K I R
1. R =
K T mr 2 5. x 2 + y 2 = l 2 = (0.25) m2
 mr 2  yω = 4 ...(i)
∴ KT =   K Total
 I + mr 
2
xω = 3 ...(ii)
 I + mr 2   1 mv 2  O
or K Total =     3 m/s ω
 mr
2
 2 
l
 3 v2  y
= mgh = mg   C
4 g 
1
Solving, we get I = mr 2 x 4 m/s
2
So, it is either solid cylinder or disc. Squaring and adding, we get
 ml 2 l 
2
( x 2 + y 2 ) ω2 = 25 or (0.25) ω2 = 25
2. (a) I x = I y = 2  + m   
 2 ∴ ω =10 rad/ s
 12 
l
2 OC = = 0.25 m
= ml 2
2
3
v CM = (OC )ω = 2.5 m/s
y 1 1
K Total = mv CM 2
+ I CM ω2
2 2
1 2 × 0.25 
= × 2 × (2.5)2 + 
1 2
 (10)
z 2 2  12 
x
25
r = J
3
6. Net acceleration of any point on the rim is vector sum of
a , Rω2 and Rα with a = R α

4
(b) I z = I x + I y = ml 2 Rω2
3
(c) I = I z + ( 4m) r 2

2
a
Rα A
= ml 2 + ( 4m) 
4 l 
 a + Rα
3  2
10 2 B
= ml 2 Rω
a
3 Rω2
ml 2
(d) I = 2 + ml 2 Rα
3
5 aA = Rω2 → vertically upwards
= ml 2
3 If a = Rω2 , aB is vertically downwards and so on.
402 Objective Physics Vol. 1

7. C1 is centre of mass of rod, tan α


µ min =
mR 2
C 2 is centre of mass of both 1+
I
tan α 2
⇒ = = tan α
3 5
l C1 1+
3 C2 2
v0
l ω f
v
ω
6
m v α
Pi = Pf a
∴ mv = 3mv 0
α
v
⇒ v0 = ...(i)
3 10. I1 ω1 = I 2 ω2
Li = L f about C 2 we have, I1
l ∴ ω2 = ⋅ ω1
mv = IC 2 ω I2
6
(1/2 mR 2 )
  2ml 2  2
l 
2 = ⋅ω
 + 2m   + m   
l
=  (1/2 mR 2 + mR 2 )
 3  6
  12   ω
=
2v 3
∴ ω=
5l pi = mv i = m ( 2ωR ) = 2mωR
1 ω
p f = mv f = m  R  = mωR
K i = mv 2 1
2 3  3
2
( 3m)   + IC 2 ω2
1 v 1
Kf =
2  3 2
2 ω
= mv 2 +  ml 2  
1 1 5 2 v ω
 3
6 2  12  5 l  pf
1 p
= mv 2 i
5
∴ Loss of kinetic energy = K i − K f Impulse, J = |p f − pi |
3
= mv 2 = pi2 + p2f (as θ = 90°)
10
8. I = mK 2  where, K = R = radius of gyration  37
  = mω R
 2 
3
2
= m   11. Velocity component along AB = 0
R
 2
y
1
= mR 2 A
4 v
J
(h – R) θ v0
v h x
ω B

∴ v 0 cos θ = v sin θ
Ball will roll purely, if
or v = v 0 cot θ = f1 (θ)
v = Rω
4
 J  J (h − R )  At θ = 37° ,v = v 0
∴   =R 2
3
 m  (1/4) mR  Relative velocity ⊥ to AB
ω=
Solving this equation, we get l
5R v sin θ + v cos θ
h= = 0 = f 2 (θ)
4 l
Further, if ball is struck at centre of mass, there will be no (v )( 3/5) + ( 4/3 v 0 )( 4/5)
= 0
rotation only translation. l
9. Friction always acts upwards. If the ball moves upwards, it 5v 0
=
becomes a case of retardation with pure rolling. 3l
Rotation 403

5. Pure rolling will start when


Comprehension Based Questions ω0 ω
1. Bottommost point where friction actually acts is at rest.
F⋅R 2F
2. α = = ⇒ v
3
mR 2 3mR
2
µmg

µmgR 
F v = Rω, µgt = R ω0 − t
 I 
Rω0
∴ t= ;I < I hollow
µmgR solid
α µg +
I
2F
a = Rα = ∴ tsolid < thollow
3m
7. Under the action of two equal and opposite forces body is in
2F
F − f = ma = translational equilibrium, not in rotational equilibrium.
3
9. Angular momentum about bottommost point will be
F 1
∴ f = = ma 1
3 2 L = mv 0 R − mR 2ω0
2
ω0 R
If L = 0 or v 0 = , disc will come to rest after some time.
F a 2
1 1
10. mv 2 + Iω2 = mgh (for both)
2 2
f
I is different for two bodies.
3. About point P net torque of F and f is zero. So, angular
Hence, v and ω will be different, as v = Rω.
momentum is conserved.
11. Angular momentum about centre is mvR.
P 12. Sphere can roll without slipping on surface, if v = r ω
ω
F R/2

F
f= 3 v
4. K Total after falling a height h is mgh
KR 2
= 14. L = mvr sin θ or mvr⊥
KT 3
In case of constant velocity m : v and r⊥ all are constant.
mv 2 =   (mgh)
1 3
∴ KT = Therefore, angular momentum is constant.
2  5
h
Further, L = n (in Bohr’s theory).
6gh
∴ v= 2π
5 Therefore L and h have same units.
v 2 sin 2 sin 37° (6gh/5)( 3/5)2 27h
5. H = = = 15. When compared with many parallel axes, I CM is least. But, it
2g 2g 125
is wrong to say that I CM is least.
2u2 sin 37° cos 37° 16. I = IC + mr 2
6. R = x =
g
Radius of gyration is a distance.
( 2)(6gh/5)( 3/5)( 4/5) 144
= = h 20. In first case, work done by friction will be zero. That’s why
g 125
mechanical energy will remain conserved. Further, in
second case, linear velocity will be more because this ball
Assertion and Reason has only translational kinetic energy.
1. When comparing between parallel axes moment of inertia
about an axis passing through centre of mass is minimum. Match the Columns
2. Linear acceleration due to mg sin θ is downwards. Hence,  ml 2  l 
2
2
for pure rolling to continue force of friction should act 1. I1 = 2   + 2(m)   = ml 2
 2   2 3
upwards so that angular acceleration is anti-clockwise.
 ml 2   5
g sin θ I2 = 0 + 2   + ml 2 =   ml 2
3. a =  3
I  3 
1+
mR 2  ml 2  2
I3 = 4  sin 2 45°  = ml 2 = I1
4. L about centre = mvR = constant  3  3
τ centre = 0 Note I1 = I 3 (think why?)
404 Objective Physics Vol. 1

θ
2. In general as v P = 2v sin   . 7. Consider the diagram where a sphere of m and radius R,
 2 struck horizontally at height h above the floor
3. In the absence of external torque, angular momentum
remains constant. F
(h–R)
L = Iω = constant
I ∝R 2 h R
1
∴ I will remain th or ω will become 4 times.
4
1
Therefore, time period (T ) will remain th
4 v
L2 The sphere will roll without slipping when ω = , where v is
Further, K = r
2I linear velocity and ω is angular velocity of the sphere.
Since, angular momentum is constant and I has become Now, angular momentum of sphere, about centre of mass
1 [We are applying conservation of angular momentum just
th.
4 before and after struck]
mv(h − R ) = I ω =  mR 2   
Therefore, kinetic energy will become 4 times. 2 v
5   R
4. First find I for complete ring, by doubling the given mass.
2
Then, the desired value is half the calculated value. ⇒ mv(h − R ) = mvR
5. τ = 0 5
2 7
L2 h− R= R ⇒ h= R
∴ L = constant, K = and Iω = constant, 5 5
2I Therefore, the sphere will roll without slipping with a
Insect first moves away from the axis, the towards it. Hence, constant velocity and hence, no loss of energy, so D→ p
I will first increase and then decrease. Torque due to applied force, F about centre of mass
6. Consider the below diagram τ = F (h − R ) (clockwise)
F For τ = 0, h = R, sphere will have only translational motion. It
would lose energy by friction.
Hence, B→ s
The sphere will spin clockwise when τ > 0 ⇒ h > R
A Therefore, C→ q
The sphere will spin anti-clockwise when
mg τ < 0 ⇒ h < R, A → r

Moment of the force F about point A, τ1 = F × a Entrance Gallery


(anti-clockwise)
1. Given, F = 5$i + 2 $j − 5k$ and r = i$ − 2$j + k$
Moment of weight mg of the cube about point A,
a Z
τ 2 = mg × (clockwise)
2
Cube will not exhibit motion, if τ1 = τ 2 τ

(Q In this case, both the torque will cancel the effect of each
other)
O X
a
∴ F × a = mg × r F
2
mg Y θ
⇒ F=
2
Cube will rotate only when, τ1 > τ 2
a mg We know that, τ = r × F
⇒ F × a > mg × ⇒ F> So, torque about the origin will be given by
2 2
$i $j k$
a
Let normal reaction is acting at from point A, then
3 = 1 −2 1
a
mg × = F × a or F =
mg
(For no motion) 5 + 2 −5
3 3
mg mg = i$ (10 − 2) − j$ ( − 5 − 5) + k$ ( 2 + 10) = 8 i$ + 10j$ + 12 k$
When F = which is less than ,. there will be no
4 3 1 1
2. We know that kinetic energy, K = mv 2 = Iω2
 F < mg  2 2
motion.  
 3  where, m = 27 kg (mass of the body)
∴ A→ q B→ r C→ p D→ s ω = 3 rad /s (angular velocity)
Rotation 405

6. Initial velocity, v1 = v vk
I = 3 kg-m2 (moment of inertia)
We have, mv 2 = Iω2 Final velocity, v 2 = − v
Initial momentum, p1 = mv
Iω2 3 × 32 Final momentum, p2 = m ( − v ) = − mv,
v2 = ⇒ v2 =
m 27 v
Change in momentum, ∆p = p1 − p2
27
v =
2
= 1 ⇒ v = 1 = 1 ms−1 = mv − ( − mv ) = 2mv
27
7. Here, T = total portion, R = remaining portion and C = cavity
3. Let the final velocity be v.
and let σ = mass per unit area.

v0
R r
r = √5 R
2R

So, angular momentum will remain conserved along point of


contact.
By conservation of angular momentum, Then, mT = π( 2R )2 σ = 4 πR 2 σ
Angular momentum will remain conserved along point of
mc = π( R )2 σ = πR 2 σ
contact,
mT ( 2R )2 −  mC R 2 + mC r 2 
3 1
Iω = constant For I P , I R = I T − IC =
2  2 
2
mv 0r = mvr + mr 2 × ω Qω = v 
 
= ( 4 πR 2 σ )( 4R 2 ) −  ( πR 2 σ )R 2 + ( πR 2 σ )( 5R 2 )
5  r 3 1
2  2 
mr 2  
2 v
mv 0r = mvr +
5 r  = (18.5 πR 4 σ )
2 1 3
⇒ v0 = v + v For IO , I R = I T − IC = mT ( 2R )2 − mC R 2
5 2 2
1 3
7
vv0 = = ( 4 πR 2 σ )( 4R 2 ) − ( πR 2 σ )( R 2 )
5 2 2
5 = 6.5 πR 4 σ
⇒ v = v0
7 I P 18.5 πR 4 σ
∴ =
4. We know that angular momentum is given by L = Iω. IO 6.5 πR 4 σ
I1ω1 = I 2ω2 = 2.846
ω1 ω2 Therefore, the nearest integer is 3.
8. Angular momentum of a particle about a point is given by

L = r × p = m (r × v )
For L O ,
LO
[Disc → M, R Ring → m, r] v
Then, 90°
r
 1
MR 2  O R
 I1   2 
∴ ω2 =   ω1 =   ω1
 I2  1
 MR + 2(mr )  | L O | = (mvr sin θ) = m ( Rω)( R )sin 90° = mR 2ω = constant.
2 2
2 
Direction of L O is always upwards. Therefore, complete L O
 50(0.4) 2  is constant, both in magnitude as well as direction.
= 2
(10)
 50(0.4) + 8 × (6.25) × (0.2) 
2
For L P ,

=   × 10 = 8 rad/s
8 LP
10  P
5. Acceleration of the sphere,
g sin θ l
a= v
v2
1+ 2
r R r
g sin θ
= | L P | = (mvr sin θ) = (m)( Rω)(l )sin 90°
2
1+
5 = (mRlω)
5 Magnitude of L P will remain constant but direction of L O
a = g sin θ
7 keeps on changing.
406 Objective Physics Vol. 1

9. Angular moment, | L | or L = Iω (about axis of rod) 15. For, F = 20t − 5t 2


Here, m = mass of insect FR dθ
⇒ ω= = 4t − t 2 ⇒ = 4t − t 2
∴ L = ( I rod + mv 2t 2 ) ω I dt
ω t
⇒ ∫0d θ = ∫ 0 ( 4t − t
2
dL ) dt
Now, | τ| = = ( 2mv 2tω) or | τ | ∝t
dt
t3
i.e. the graph is straight line passing through the origin. ⇒ ω = 2t 2 −
After time T , 3
dL When direction is reversed,
L = constant | τ | or =0
dt ω = 0, i. e. t = 0, 6 s
10. Language of question is wrong, because relative speed is Now, dθ = ω dt
not the correct word. Relative speed between two is always θ 6  2 t3 
zero. The correct word is magnitude of relative velocity. ∫ 0 dθ = ∫0  2t − 3  dt
6
 2t 3 t 4 
⇒ θ=  − 
 3 12  0
v v
t = T/4 ⇒ θ = 144 − 108
v v
|vr| = 2v = 2 Rω = 36 rad
|vr| = 0
=0 ∴ Number of rotations,
θ 36
n= = <6
2π 2π
16. For the motion of the block,
t = T/2 mg − T = ma …(i)
|vr| = 0

v = Rω
Corresponding to above values the correct graph is (a). R
11. The velocity of solid cylinder, m
2gh T
v=
k2 T
1+
R m
k2
For solid cylinder, = 1/2 For the rotation of the pulley,
R
τ = TR = Iα
4gh
⇒ v= 1
3 ⇒ T = mRα …(ii)
2
2
mv
12. Centripetal force, F = As string does not slip on the pulley
r
then, a = Rα …(iii)
F = ma ⇒ 30 = 3 × a ⇒ a =10 rad/ s2 On solving Eqs. (i), (ii) and (iii), we get
The angular acceleration given that 2g
a=
a 3
α=
r 17. For a ring, K 2 = r 2 , then
10
= 2gh 2gh
40 × 10− 2 v2 = ⇒ v2 = = gh ⇒ v = gh
K2 2
1+ 2
= 25 rad /s2 r
1 18. We know that,
13. Moment of inertia = MR 2 + mx 2
2 L = Iω …(i)
where, m = mass of insect L2 = 2KI
x = distance of insect from centre From Eq. (i), we get
2  
Clearly, as the insect moves along the diameter of the disc, K
mass of inertia first decreases, then increases. L 2K  2 L
L = 2K
2
⇒ L= ⇒ L′ = =
By conservation of angular momentum, angular speed first ω ω 2ω 4
increases, then decreases. 1  K2
1 19. Total KE at bottom = 1 +
mv 2 
14. Rotational kinetic energy, K R = Iω2 2  R2 
2
2K 2 × 360 1 2  2 7
∴ Its moment of inertia = 2R = = 0.8 kg-m2 = mv 1 + = mv 2
2  5  10
ω ( 30)2
Rotation 407

Ltotal ( I A + I B ) ω ML2
20. = (as ω will be same in both cases) 24. I CM = (about middle-point)
LB IB ⋅ω 12
IA m r2 r ∴ I = I CM + Mx 2
= + 1 = A A2 + 1 = A + 1 (as, mArA = mBrB )
IB mBrB rB 2
ML2
+ M  
L
=
11  as r ∝ 1  12  6
= +1=6  
2.2  m ML2
I=
21. The moment of inertia of ring = MR 2 9
1
The moment of inertia of removed sector = MR 2 25. From theorem of parallel axes,
4 2
ML2
+M 
L
The moment of inertia of remaining part I = I CM + Mx 2 =
12  4 
1 3
= MR 2 − MR 2 = MR 2 …(i)
4 4 ML2 ML2 7ML2
= + =
According to question, the moment of inertia of the 12 16 48
remaining part = kMR 2 …(ii) 26. Mass for unit area =
M
On comparing Eq. (i) and Eq. (ii), we get π (R2 − r 2 )

k=
3 M ⋅ πrR 2 MR 2
∴ Mass of whole disc = = 2
4 π (R − r ) (R − r 2 )
2 2

22. Given, moment of inertia = 2.5 kg m −2 1 Mπr 2 1 Mr 4


−1 −1
MI of disc of radius, r = ⋅ r 2
=
ω = 40 rad s , ω = 60 rad s , τ =10 N-m 2 π (R2 − r 2 ) 2 (R2 − r 2 )
As, τ = Iα 1 Mr 2 ⋅ r 2 Mr 4
MI of whole disc = =
⇒ 10 = 2. 5 × α 2 (R − r ) 2 (R2 − r 2 )
2 2

⇒ α = 4 rad s−2 M  R4 − r 4 
∴ MI of annular disc =  
Now, ω = ω0 + αt 2  R2 − r 2 
⇒ 60 = 40 + 4 × t
M (R2 − r 2 )
⇒ 20 = 4t =
2
⇒ t = 5s
27. The correct relations all as below:
23. Angular velocity = ω
Torque = Moment of inertia × angular acceleration
1
Centripetal force, F = mrω2 or r ∝ Torque = dipole moment × magnetic field
ω2
Moment of inertia = torque ÷ angular acceleration
r1 ω22
∴ =
r2 ω12 Given, momentum = mass × velocity
28. Moment of inertia of a disc about a diameter is given by
4 4ω 2
or = 1
r2 ω2 MR 2 = I
4
or r2 = 1 cm ⇒ MR 2 = 4I (given)
10
Gravitation

10.1 Introduction
Why are planets, moon and the sun all nearly spherical? Why do some Earth satellites Chapter Snapshot
circle the Earth in 90 min, while the moon takes 27 days for the trip? And why don’t ● Introduction
satellites fall back to the Earth? The study of gravitation provides the answers for these and
● Newton’s Law of
many related questions.
Gravitation
Gravitation is one of the four classes of interactions found in nature. These are given below: ● Acceleration due to
(i) The gravitational force Gravity
(ii) The electromagnetic force ● Gravitational Field
(iii) The strong nuclear force (also called the hadronic forces) ● Gravitational Potential
(iv) The weak nuclear forces ● Gravitational Potential
Although, of negligible importance in the interactions of elementary particles, gravity Energy
is of primary importance in the interactions of large objects. It is gravity that holds the ● Binding Energy
universe together. ● Motion of Satellites
In this chapter, we will learn the basic laws that govern gravitational interactions. ● Kepler’s Laws

10.2 Newton’s Law of Gravitation


Along with his three laws of motion, Newton published the law of gravitation in 1687.
According to him, every particle of matter in the universe attracts every other particle with
a force that is directly proportional to the product of the masses of the particles and
inversely proportional to the square of the distance between them.
Thus, the magnitude of the gravitational force F between two particles m1 and m2
placed at a distance r is
mm mm
F ∝ 1 2 2 or F = G 1 2 2
r r
Here, G is a universal constant called gravitational constant whose magnitude is
G = 6.67 × 10 –11 N-m 2 /kg 2
= 6.67 × 10 −8 dyne/cm 2-g 2
[G ] = [M −1 L3T −2 ]
The direction of the force F is along the line joining the two particles.
Gravitation 409

Following three points are important regarding the According to Newton’s second law, the acceleration
gravitational force: due to gravity
(i) Unlike the electrostatic force, it is independent of the F GM
medium between the particles. g= = 2
m R
(ii) It is conservative in nature.
This expression is free from m. If two bodies of
(iii) It expresses the force between two point masses
different masses are allowed to fall freely, they will have
(of negligible volume). However, for external points
the same acceleration, i.e. if they are allowed to fall from
of spherical bodies the whole mass can be assumed to
the same height, they will reach the earth simultaneously.
be concentrated at its centre of mass.

Gravity Variation in the Value of g


In Newton’s law of gravitation, gravitation is the force The value of g varies from place to place on the surface
of attraction between any two bodies. If one of the bodies is of earth. It also varies as we go above or below the surface
Earth, then the gravitation is called gravity. Hence, gravity of Earth. Thus, value of g depends on the following factors:
is the force by which Earth attracts a body towards its centre.
It is a special case of gravitation. Shape of the Earth
The earth is not a perfect sphere. It is somewhat flat at
X Example 10.1 Spheres of the same material and the two poles. The equatorial radius is approximately 21 km
same radius r are touching each other. Show that more than the polar radius. And since,
gravitational force between them is directly GM 1
proportional to r 4 . g = 2 or g ∝ 2
R R
Sol. The value of g is minimum at the equator and maximum
at the poles.

Height above the Surface of Earth


The force of gravity on an object of mass m at a height h
2r
above the surface of the earth is
GMm
Fig. 10.1 F=
( R + h) 2
m1 = m2 = (volume) × (density)
4
=  πr 3  ρ ∴ Acceleration due to gravity at this height will be
3  F GM
Gm1m2 g′ = = m
∴ F= m ( R + h) 2 h
r2
4 4
G  πr 3   πr 3  ρ2 This can also be written as
3  3  GM
= g′ = R
r2 2
F ∝ r4  h M
or R 2 1 + 
 R Fig. 10.2
g GM
10.3 Acceleration due to or g′ = 2
as
R2
=g
 h
Gravity 1 + 
 R
When a body is dropped from a certain height above the Thus, g′ < g
ground, it begins to fall towards the earth under gravity. The
acceleration produced in the body due to gravity is called the i.e. the value of acceleration due to gravity g goes on
acceleration due to gravity. It is denoted by g. Its value close decreasing as we go above the surface of the earth. Further,
−2
to the earth’s surface is 9.8 m/s 2 .  h
g ′ = g 1 + 
Suppose that the mass of the earth is M, its radius is R,  R
then the force of attraction acting on a body of mass m close  2h 
to the surface of the earth is or g ′ ≈ g 1 −  ,
 R
GMm
F= if h<<R
R2
410 Objective Physics Vol. 1

Depth below the Surface of Earth Extra Knowledge Points


Let an object of mass m is situated at a depth h below the
earth’s surface. Its distance from the centre of the earth is ■ Acceleration due to Moon’s gravity on Moon’s surface
g
( R − h). This mass is situated at the surface of the inner solid is e , because
6
sphere and lies inside the outer spherical shell. The Mm 1 Me  GM 
gravitational force of attraction on a mass inside a spherical ≈ g = 2 
Rm2 6 Re2  R 
shell is always zero. Therefore, the object experiences
gravitational attraction only due to inner solid sphere. While acceleration due to Earth’s gravity on Moon’s surface
The mass of this sphere is g g
is approximately e 2 or e . This is because distance
( 60) 3600
h of Moon from the Earth’s centre is approximately equal to
m 1
60 times the radius of the Earth and g ∝ 2 . This can be
r
O understood from the figure.
Earth Moon
R

60Re g1 g2
Fig. 10.3

 M  4
M′ =   π ( R − h)
3 Re
 4 / 3 πR  3
3

( R − h) 3 ge ge
or M′ = M g1 = while g 2 =
R3 ( 60)2 6
GM ′ m GMm ( R − h)
F= =
( R − h) 2 R3
Axial Rotation of Earth
F
and g′ = Let us consider a particle P at rest on the surface of the
m Earth, in latitude φ. Then, the pseudo force acting on the
Substituting the values, we get particle is mrω 2 in outward direction. The true acceleration g
 h is acting towards the centre O of the Earth. Thus, the
g ′ = g 1 −  i.e. g ′ < g
 R effective acceleration g′ is the resultant of g and rω 2 . After
/ We can see from this equation that g′ = 0 at h = R , derivation we can find the following relation
i.e. acceleration due to gravity is zero at the centre of the earth. y
Thus, the variation in the value of g with r (the distance
from the centre of Earth) is as follows: r P rω2
g
g′ g′
φ
O x
R
1
g′ ∝ r g′ ∝
r2
Fig. 10.5

r=R
r
g ′ = g − Rω 2 cos 2 φ
Fig. 10.4
Following conclusions can be drawn from the above
For r ≤ R , discussion
 h gr
g ′ = g 1 −  = (i) The effective value of g is not truely vertical.
 R R (ii) The effect of centrifugal force due to rotation of Earth
as R − h = r or g ′ ∝ r is to reduce the effective value of g.
g gR2 (iii) At equators φ = 0°.
For r > R , g′ = =
 h
2
r2 Therefore, g ′ = g − Rω 2 and at poles φ = 90° ,
1 + 
 R Therefore, g ′ = g
1 Thus, at equator g′ is minimum while at poles g′ is
or g′ ∝ 2
r maximum.
Gravitation 411

X Example 10.2 Assuming Earth to be a sphere E

of uniform mass density, how much would a body


weigh half way down the centre of the Earth, if it
1
weighed 100 N on the surface? E∝r E∝
r2
Sol. Given, mg = 100 N
r
g ′ = g  1 − 
h
r=R
 R
Fig. 10.6
h 1
=
R 2

1
g ′ = g  1 −  =
g Field due to a Point Mass
 2 2 Suppose, a point mass M is M r P
mg 100
∴ mg ′ = = = 50 N placed at point O. We want to find O
2 2 Fig. 10.7
the intensity of gravitational field E
at a point P, a distance r from O. Magnitude of force F acting
X Example 10.3 Suppose the Earth increases its
on a particle of mass m placed at P is
speed of rotation. At what new time period will the
GMm
weight of a body on the equator becomes zero? F= 2
(Take g =10 m/ s 2 and radius of Earth R = 6400 km ) r
F GM GM
Sol. The weight will become zero when
∴ E = = 2 or E = 2
m r r
g′ = 0
The direction of the force F and hence of E is from P to
or g − Rω2 = 0 (on the equator g ′ = g − Rω2 )
O as shown in Fig. 10.7
g
or ω=
R
Gravitational Field due to a Uniform
2π g R
∴ = or T = 2 π
T R g Solid Sphere
Substituting the values,
Field at an External Point
6400 × 103
2π A uniform sphere may be treated as a single particle of
T= 10 h same mass placed at its centre for calculating the
3600
gravitational field at an external point. Thus,
or T = 1.4 h
GM
Thus, the new time period should be 1.4 h instead of 24 h E (r ) = 2
for the weight of a body to be zero on the equator. r
1
For r ≥ R or E ( r ) ∝ 2
10.4 Gravitational Field r
Here, r is the distance of the point from the centre of the
The space around a body in which any other body sphere and R the radius of sphere.
experiences a force of attraction is called the gravitational
field of the first body. Field at an Internal Point
The force experienced (both in magnitude and The gravitational field E
direction) by a unit mass placed at a point in a gravitational due to a uniform sphere at an
field is called the gravitational field strength or intensity of internal point is proportional GM
gravitational field at that point. Usually, it is denoted by E. to the distance of the point R2 1
E∝r E∝
Thus, from the centre of the sphere. r2
F At the centre itself, it is zero
E= GM
r
m R
and at surface it is 2 , where
R Fig. 10.8
In Article 10.3, we have seen that acceleration due to
F R is the radius of the sphere.
gravity g is also .Hence, for the Earth’s gravitational field Thus,
m
GM
g and Eare same. Unless the effect of rotation of Earth about E (r ) = r,
its own axis is to be considered. The E versus r (the distance R3
from the centre of Earth) graph are same as that of g′ versus r for r ≤ R or E ( r ) ∝ r
graph. Hence, E versus r graph is as shown in Fig. 10.8.
412 Objective Physics Vol. 1

Field due to a Uniform Spherical Shell 10.5 Gravitational Potential


At an External Point If a body is moved in a gravitational field from one place
For an external point, the shell may be treated as a single to the other either work is done against the gravitational
particle of same mass placed at its centre. Thus, at an attraction or it is obtained.
external point the gravitational field is given by
The work done in bringing a unit mass from infinity to a
E
point in the gravitational field is called the ‘gravitational
potential’ at that point.
GM
R2 E∝
1 This work is obtained (not done) by the agent in
r2 bringing the mass. The gravitational potential is denoted by
r V. So, let W joule of work is obtained in bringing a test mass
R
m from infinity to some point, then gravitational potential at
Fig. 10.9
that point will be
GM W
E (r ) = , for r ≥ R V=
r2 m
At r=R (the surface of shell )
Since, work is obtained, it is negative. Hence,
GM
E= 2 gravitational potential is always negative.
R
and
1
otherwise E ∝ 2 Potential due to a Point Mass
r Suppose a point mass M is situated at a point O. We
want to find the gravitational potential due to this mass at a
At an Internal Point point P a distance r from O. For this let us find work done in
The field inside a uniform spherical shell is zero. taking the unit mass from P to infinity. This will be
Thus, E versus r graph is as shown in Fig. 10.9.
M r P
O
Field due to a Uniform Circular Ring Fig. 10.12
at a Point on its Axis ∞
W = ∫ F dr
Field strength at a point P on the r
axis of a circular ring of radius R and ∞ GM GM
mass M is given by
r
P =∫ 2
dr =
R E r r r
GMr Hence, the work done in bringing unit mass from
E (r ) = Fig. 10.10
( R + r 2 ) 3/ 2
2 GM
infinity to P will be − . Thus, the gravitational potential
This is directed towards the centre of the ring. It is zero r
at P will be
at the centre of the ring and maximum at r = R (can be
2 GM
V =−
dE r
obtained by putting = 0). Thus, E-r graph is as shown in
dr
Fig. 10.11. Potential due to a Uniform Solid Sphere
E
Potential at an External Point
2GM The gravitational potential due to a uniform sphere at an
3 √3R 2 external point is same as that due to a single particle of same
mass placed at its centre. Thus,
r GM
R
V (r ) = − , where r ≥ R
2 r
Fig. 10.11
At the surface, r=R
2GM GM
The maximum value is E max = . and V =−
3 3R 2 R
Gravitation 413

Potential at an Internal Point Potential due to a Uniform Ring at a


At some internal point, potential at a distance r from the Point on its Axis
centre is given by The gravitational potential at a distance r from the centre
GM on the axis of a ring of mass M and radius R is given by
–1.5
R
GM

R
r O P
R r

V
Fig. 10.13 Fig. 10.15
GM
V (r ) = − (1.5R 2 − 0.5r 2 ) r ≤ R V (r ) = −
GM
, 0≤r≤∞
R3
R 2 + r2
GM
At r = R , V = − GM
R At r = 0, V = − , i.e. at the centre of the ring
1.5 GM R
while at r = 0, V = − GM
R gravitational potential is − .
R
i.e. at the centre of the sphere, the potential is 1.5 times
GM
the potential at surface. The variation of V versus r graph is as –
R
shown in Fig. 10.13.
r
Potential due to a Uniform Thin
Spherical Shell V
Fig. 10.16
Potential at an External Point
To calculate the potential at an external point, a uniform The V-r graph is as shown in Fig. 10.16.
spherical shell may be treated as a point mass of same
magnitude at its centre. Thus, potential at a distance r is X Example 10.4 Two concentric spherical shells
given by have masses m1 and m2 and radii r1 and r2 ( r2 > r1 ).
GM What is the force exerted by this system on a particle
V (r ) = − , where r ≥ R of mass m3 , if it is placed at a distance r ( r1 < r < r2 )
r
GM from the centre?
At r = R , V = −
R Sol. The outer shell will have no contribution in the gravitational
field at point P.
Gm1
Potential at an Internal Point ∴ EP =
r2
The potential due to a uniform spherical shell is constant
m2
throughout at any point inside the shell and this is equal to
GM
− . Thus, V-r graph for a spherical shell is as shown in m1
R m3
O P
Fig. 10.14. r
r1
– GM
R
r2

r Fig. 10.17
R
Thus, force on mass m3 placed at P is
Gm1m3
V F = (m 3 EP ) or F =
r2
Fig. 10.14
The field EP and the force F both are towards centre O.
414 Objective Physics Vol. 1

X Example 10.5 A particle of mass 1 kg is kept on Gravitational Potential Energy for a


the surface of a uniform sphere of mass 20 kg and
radius 1.0 m. Find the work to be done against the
System of Particles
gravitational force between them to take the particle The gravitational potential energy for a system of
away from the sphere. particles (say m1 , m2 , m3 and m4 ) is given by
m m m m m m m m m m m m 
Sol. Potential at the surface of sphere, U =− G  4 3 + 4 2 + 4 1 + 3 2 + 3 1 + 2 1 
GM (6.67 × 10−11 )(20)  r43 r42 r41 r32 r31 r21 
V=− =− J/kg
R 1 n ( n −1)
= − 1.334 × 10−9 J/kg Thus, for a n particle system there are pairs
2
i.e. 1.334 × 10−9 J work is obtained to bring a mass of 1 kg and the potential energy is calculated for each pair and
from infinity to the surface of sphere. Hence, the same
amount of work will have to be done to take the particle
added to get the total potential energy of the system.
away from the surface of sphere. Thus,
W = 1.334 × 10−9 J Gravitational Potential Energy of a
Body on Earth’s Surface
10.6 Gravitational Potential The gravitational potential energy of mass m in the
gravitational field of mass M at a distance r from it is
Energy GMm
U =−
The concept of potential energy has already been r
discussed in the chapter of work, energy and power. The The Earth behaves for all external points as, m
word potential energy is defined only for a conservative
if its mass M were concentrated at its centre.
force field. There we have discussed that the change in R
Therefore, a mass m near Earth’s surface may M
potential energy ( dU ) of a system corresponding to a
be considered at a distance R (the radius of
conservative internal force is given by
Earth) from M. Thus, the potential energy of Fig. 10.19
dU = − F ⋅ dr
f rf m due to Earth will be
or ∫i dU = − ∫
ri
F ⋅ dr
U =−
GMm
rf R
or U f −U i = − ∫ F ⋅ dr
ri

We generally choose the reference point at infinity and 10.7 Binding Energy
assume potential energy to be zero there, i.e. if we take m
Total mechanical energy (potential +
ri = ∞ (infinite) and U i = 0, then we can write
r kinetic) of a closed system is negative. The
U = − ∫ F ⋅ dr = − W modulus of this total mechanical energy is R
M

known as the binding energy of the system. This
or potential energy of a body or system is negative of
work done by the conservative forces in bringing it from is the energy due to which system is closed or Fig. 10.20

infinity to the present position. different parts of the system are bound to each
other.
Gravitational Potential Energy of a two Suppose, the mass m is placed on the surface of Earth.
Particle System The radius of the Earth is R and its mass is M. Then, the
The gravitational potential energy of two particles of kinetic energy of the particle K = 0 and potential energy of
GMm
masses m1 and m2 separated by a distance r is given by the particle is U = − .
Gm1 m2 R
U =−
r Therefore, the total mechanical energy of the particle is
r GMm
m1 m2 E = K +U = 0 −
R
Fig. 10.18 GMm
or E =−
This is actually the negative of work done in bringing R
those masses from infinity to a distance r by the GMm
gravitational forces between them. ∴ Binding energy = | E | =
R
Gravitation 415

It is due to this energy, the particle is attached with the Thus, what we read the mgh is actually, the difference
Earth. If minimum this much energy is supplied to the in potential energy (not the absolute potential energy),
particle in any form (normally kinetic) the particle no longer that too for h < < R .
■ Maximum height attained by a particle. Suppose a
remains bound to the Earth. It goes out of the gravitational
field of Earth. particle of mass m is projected vertically upwards with
a speed v and we want to find the maximum height h
attained by the particle. Then, we can use
Escape Velocity conservation of mechanical energy, i.e.
As we discussed the binding energy of a particle on the Decrease in kinetic energy = increase in gravitational
GMm potential energy of particle.
surface of Earth kept at rest is . If this much energy in 1
R ∴ mv 2 = ∆U
the form of kinetic energy is supplied to the particle, it 2
leaves the gravitational field of the Earth. So, if v e is the 1 mgh
or mv 2 =
escape velocity of the particle, then 2 h
1+
1 GMm R
mv e2 = Solving this, we get
2 R v2
h=
2GM v2
or ve = or v e = 2gR 2g −
R R
GM From this we can see that
as g= 2 (i) if v = ve or v 2 = ve2 = 2 gR , h = ∞ and if
R
v2
(ii) v is small, h =
Substituting the value of g (9.8 m/s 2 ) and 2g
R (6.4 × 10 6 m), we get Both the results are quite obvious.

v e ≈11.2 km /s
Thus, the minimum velocity needed to take a particle X Example 10.6 Three masses of 1 kg, 2 kg and
infinitely away from the earth is called the escape velocity. 3 kg are placed at the vertices of an equilateral
On the surface of the earth, its value is 11.2 km/s. triangle of side 1 m. Find the gravitational potential
energy of this system.
(Take, G = 6.67 × 10 −11 N-m 2 /kg 2 )
Extra Knowledge Points
 m3 m2 m3 m1 m2 m1 
Let us find the difference in potential A Sol. U = − G  + + 
r21 

energy of a mass m in two positions shown  r32 r31


h
in figure. The potential energy of the mass B Here, r32 = r31 = r21 = 1.0 m
on the surface of Earth (at B) is
m1 = 1 kg
GMm m2 = 2 kg
UB = − R
M
R and m3 = 3 kg
and potential energy of mass m at height h Substituting in above, we get
3 × 2 3 × 1 2 × 1
above the surface of Earth (at A) is U = − (6.67 × 10−11 )  + + 
GMm  1 1 1 
UA = − (UA > UB )
R +h or U = − 7.337 × 10−10 J
GMm  GMm 
∴ ∆U = UA − UB = − − −  X Example 10.7 Calculate the escape velocity from
R +h  R 
the surface of Moon. The mass of the Moon is
1 1  GMmh
= GMm  − = 7.4 × 10 22 kg and radius = 1.74 × 10 6 m .
 R R + h  R (R + h )
GMmh  GM  Sol. Escape velocity from the surface of Moon is
=  2 = g
2  h R  2GM m
R 1 +  ve =
 R Rm
mgh Substituting the values, we have
∴ ∆U =
h 2 × 6.67 × 10–11 × 7.4 × 1022
1+ ve =
R 1.74 × 106
For h < < R , ∆U ≈ mgh 3
= 2.4 × 10 m/s or 2.4 km/s
416 Objective Physics Vol. 1

X Example 10.8 A particle is projected from the GM v


vo = = gR = e
surface of Earth with an initial speed of 4.0 km/s. R 2
Find the maximum height attained by the particle. Substituting the values of G, M and R. Then,
Radius of Earth = 6400 km, and g = 9.8 m/s 2.
v e =11.2 km/s
Sol. The maximum height attained by the particle is v o = 7.9 km/s
v2
h= Period of Revolution
v2
2g − The period of revolution (T ) is given by
R
Substituting the values, we have 2πr 2πr
T= or T =
(4.0 × 103 )2 vo GM
h=
(4.0 × 103 )2
2 × 9.8. − r
6.4 × 106
= 9.35 × 10 m5 r3 r3
or T = 2π or T = 2π (as GM = gR 2 )
or h ≈ 935 km GM gR 2

10.8 Motion of Satellites From this expression of T, we can make the following
conclusions.
Just as the planets revolve around the sun, in the same (i) T ∝ r 3/ 2
way few celestial bodies revolve around these planets.
These bodies are called satellites. For example, the Moon, is or T 2 ∝ r3
a satellite of the Earth. Artificial satellites are launched from (which is also the Kepler’s third law)
the Earth. Such satellites are used for telecommunication, (ii) Time period of a satellite very close to the Earth’s
weather forecast and other applications. The path of these surface ( r ≈ R ) is
satellites are elliptical with the centre of the Earth at a focus. R
However, the difference in major and minor axes is so small T = 2π
g
that they can be treated as nearly circular for not too
sophisticated calculations. Let us derive certain Substituting the values, we get
characteristics of the motion of satellites by assuming the T ≈ 84.6 min
orbit to be perfectly circular. (iii) Suppose, the height of a satellite is such that the time
period of the satellite is 24 h and it moves in the same
Orbital Speed sense as the earth. The satellite will always be overhead
The necessary centripetal force to the satellite is being a particular place on the equator. As seen from the
provided by the gravitational force exerted by the Earth on earth, this satellite will appear to be stationary. Such a
the satellite. Thus, satellite is called a geostationary satellite. Putting
T = 24 h in the expression of T, the radius of
geostationary satellite comes out to be r = 4.2 × 10 4
vo km. The height above the surface of the earth is about
M
r m 3.6 ×10 4 km.
R Satellite
Earth
Energy of Satellite
The potential energy of the system is
Fig. 10.22
GMm
U =−
mv o2 GMm r
= 2 The kinetic energy of the satellite is
r r
1
GM 1 K = mv o2
∴ vo = or v o ∝ 2
r r 1  GM 
= m 
Hence, the orbital speed ( v o ) of the satellite decreases as 2  r 
the orbital radius ( r ) of the satellite increases. Further, the
1 GMm
orbital speed of a satellite close to the Earth’s surface ( r ≈ R ) or K=
is 2 r
Gravitation 417

The total energy is X Example 10.9 A spaceship is launched into a


GMm circular orbit close to the Earth’s surface. What
E = K +U = −
2r additional velocity has now to be imparted to the
GMm spaceship in the orbit to overcome the gravitational
or E =− pull? Radius of Earth = 6400 km, g = 9.8 m/s 2 .
2r
This energy is constant and negative, i.e. the system is Sol. The speed of the spaceship in a circular orbit close to the
Earth’s surface is given by
closed. The farther the satellite from the Earth the greater its
total energy. vo = gR
and escape velocity is given by
ve = 2 gR
K ∴ Additional velocity required to escape
ve − vo = 2 gR − gR
r
= ( 2 − 1) gR
E
E=K+U Substituting the values of 2 g and R, we get
U
ve − vo = 3.278 × 103 m/s

Fig. 10.23
10.9 Kepler’s Laws
Kepler discovered three empirical laws that accurately
Extra Knowledge Points described the motions of the planets. The three laws may be
■ Total energy of a closed system is always negative. For stated as
example, energy of planet-sun, satellite-Earth or (i) Each planet moves in an elliptical orbit, with the sun at
electron-nucleus system are always negative. one focus of the ellipse. This law is also known as the
■ If the law of force obeys the inverse square law . law of elliptical orbits and obviously gives the shape
 1 − dU  of the orbits of the planets round the sun.
F ∝ 2 , F = 
 r dr 
(ii) The radius vector, drawn from the sun to a planet,
|U |
K = =|E | sweeps out equal areas in equal time, i.e. its areal
2 velocity (or the area swept out by it per unit time) is
It is also same for electron-nucleus system, because constant.
1
there also, the electrostatic force Fe ∝ 2 ⋅ This is referred to as the law of areas and gives the
r
■ The areal velocity of a planet is constant (Kepler’s
relationship between the orbital speed of the planet
second law) and is given by and its distance from the sun.
dA L (iii) The square of the planet’s time period is proportional
=
dt 2m to the cube of the semi-major axis of its orbit. This is
Here, L is the angular momentum of the planet about
known as the harmonic law and gives the relationship
the sun. between the size of the orbit of a planet and its time of
revolution.
Chapter Summary with Formulae
1. Gravitational Force between Two Point Masses is
mm
F = G 12 2 (iv) Due to a spherical shell
r E
2. Acceleration Due to Gravity
GM
(i) On the surface of Earth g = = 9.81 ms −2 GM
R2 R
2

(ii) At height h from the surface of the Earth,


≈ g  1 −
g 2h 
g′ = 
1 +  h
2  R r
 
 R Inside points, E i = 0
if h < < R GM
Outside points, E o = 2
(iii) At depth d from the surface of the Earth, r
g ′ = g  1 − 
d GM
Just outside the surface, E = 2
 R R
g′ = 0 if d = R , i.e. at centre of the Earth On the surface, E - r graph is discontinuous.
3. Field Strength 4. Gravitational Potential
(i) Gravitational field strength at a point in gravitational (i) Gravitational potential at a point in a gravitational
field is defined as field is defined as the negative of work done by
F
E= = gravitational force per unit mass. gravitational force in moving a unit mass from
m infinity to that point. Thus,
(ii) Due to a point mass W
GM VP = − ∞→ P
E= 2 (towards the mass) m
r
E (ii) Due to a point mass
V

1 Gm
or E= V =−
r2 r
(iii) Due to a solid sphere V→−∞
E as r → 0 and V → 0
as r→∞
GM (iii) Due to a solid sphere
GM
R2 Inside points, Vi = − 3 (1.5 R2 − 0.5 r2 )
R
GM
At r = R, V = − , i.e. on surface
r R
GM
GM At r = 0, V = − 1.5 , i.e. at centre
Inside points, E i = r R
R3 V
At r = 0, E = 0 (at centre),
GM
At r = R, E = 2 , i.e. on surface
R R
r
GM 1
Outside points, E o = 2 or E o ∝ 2 GM
r r –
R
GM
At r = R, E = 2 , i.e. on surface –
1.5GM
R R
As, R→ ∞,E→0 V - r graph is parabolic for inside points and potential
On the surface, E - r graph is continuous. at centre is 1.5 times the potential at surface.
GM mgh
Outside points, Vo = − ∆U = U h − U0 or ∆U =
r h
1+
GM R
At r = R, V = − , i.e. on surface
R If h << R, ∆ U ≈ mgh
As r → ∞,V → 0 6. Escape Velocity
(iv) Due to a spherical shell (i) From the surface of Earth,
V ve = 2gR
=
2GM  as g = GM 
 
R  R2 
R ≈ 11.2 kms −1
r
(ii) Escape velocity does not depend upon the angle at
GM
– which particle is projected from the surface.
R
7. Motion of Satellites
GM
(i) Orbital speed, vo =
GM r
Inside points, Vi = − = constant
R 2 π 3 /2
(ii) Time period, T = r
GM
Outside points, Vo = − GM
r GMm
5. Gravitational Potential Energy (iii) Kinetic energy, K =
2r
(i) This is negative of work done by gravitational forces GMm
in making, the system from infinite separation to the (iv) Potential energy, U = −
r
present position.
(ii) Gravitational potential energy of two point masses is (v) Total mechanical energy,
Gm1m2 GMm
U=− E=−
r 2r
(iii) For finding gravitational potential energy is more Near the surface of the Earth, r ≈ R and
than two point masses we have to make pairs of GM
masses. Neither of the pair should be repeated. For vo = = gR = 7.9 kms −1.
R
example, in case of four point masses,
This is the maximum speed of Earth’s satellite.
m m m m m m mm
U = −G  4 3 + 4 2 + 4 1 + 3 2 Time period of such a satellite would be
 43
r r42 r41 r32
2 π 3 /2 R
T= R = 2π
m3 m1 m2 m1 
+ + GM g
r31 r21  = 84.6 min
For n point masses, total number of pairs will be This is the minimum time period of any Earth’s
n( n − 1) satellite.
.
2 8. Kepler’s Laws
(iv) If a point mass m is placed on the surface of Earth, the Kepler’s three empirical laws describe the motion of the
potential energy here is planets.
Uh
First law Each planet moves in an elliptical orbit with
h the sun at one focus of the ellipse.
m
U0 Second law The radius vector drawn from the sun to a
planet, sweeps out equal areas in equal time interval,
i.e. areal velocity is constant. This law is derived from
R law of conservation of angular momentum.
M dA L
= = constant.
dt 2m
GMm Here, L is angular momentum and m is mass of planet.
U0 = −
R Third law T2 ∝ r3 , where r is semi-major axis of
and potential energy at height h is elliptical path.
GMm
Uh = − Circle is a special case of an ellipse. Therefore, second
( R + h) and third laws can also be applied for circular path. In
The difference in potential energy would be third law, r is radius of circular path.
Additional Examples
Example 1. Moon has got no atmosphere on it. Example 7. What is the fractional decrease in the
Why? value of free-fall acceleration g for a particle when it is
Sol. The escape velocity in case of Moon is 2.38 km/s and is lifted from the surface to an elevation h ? (h < < R)
comparable to the rms velocity of all the constituents of GM
Sol. Q g =
atmosphere like O 2 , N 2 , CO 2 and water vapours. R2
Example 2. Moon is continuously revolving around dg − 2GM
the Earth without falling towards it. Why? =
dR R3
Sol. It is on account of the fact that the gravitational force of dg − 2GM 1 dg  h
⇒ = . ⇒ = − 2 
attraction between Moon and Earth is used up in providing, h R 2
R g  R
the necessary centripetal force for keeping the Moon in an
orbit around the Earth. Example 8. Two concentric shells of mass M 1 and
Example 3. A person sitting in a satellite of Earth M 2 are concentric as shown. Calculate the
feels weightlessness but a person standing on Moon gravitational force on m due to M 1 and M 2 at points
does not feel weightlessness though Moon is also a P ,Q and R.
satellite of Earth. R
c
Sol. When a person is sitting in an artificial satellite of Earth, M2
the gravitational attraction on him due to Earth provides the
necessary centripetal force. But when he is standing on Moon, M1
Q
the gravitational attraction acting on him due to Moon, b a
accounts for his weight on Moon, because mass of Moon is P
large and it applies considerable force on person.

Example 4. Is it possible to shield a body from


gravitational effect? Sol. At P , F=0
GM1m
Sol. No, it is not possible to shield a body from gravitational At Q, F=
effects. b2
G (M1 + M 2 ) m
At R, F=
Example 5. What is the difference between inertial c2
mass and gravitational mass of a body?
Example 9. Assuming the radius of the Earth to be
Sol. The inertial mass of a body is a measure of its inertia and is
6.4 × 10 6 m.
given by the ratio of the external force applied on it to the
acceleration produced in it on the other hand. The (a) Calculate the time period T of a satellite for
gravitational mass of a body is a measure of the gravitational equatorial orbit at 1.4 × 10 3 km above the surface of
pull acting on it due to the Earth. This is the ratio of the Earth.
gravitational force acting on it and the acceleration produced
(b) What is the speed of the satellite in this orbit?
by it.
Sol. (a) r = R earth + h
Example 6. If suddenly the gravitational force of
attraction between the Earth and a satellite revolving = (6.4 × 10 6 + 1.4 × 10 6 ) m ⇒ r = 7.8 × 10 6 m
around it becomes zero, what will happen to the  4π 2r 3 
T= 
satellite? GM earth 
Sol. If the gravitational force of the Earth suddenly becomes = 6831 s
GM earth
zero, the satellite will stop revolving around the Earth and it (b) Speed of satellite, v =
will move in a direction tangential to its original orbit with a r
speed with which it was revolving around the Earth. = 7174 m/s
Gravitation 421

Example 10. Three particles each of mass m are Example 12. Three concentric shells of masses
located at the vertices of an equilateral triangle of side M 1 , M 2 and M 3 having radii a , b and c respectively
a. At what speed must they move, if they all revolve are situated as shown in figure. Find the force on a
under the influence of their gravitational force of particle of mass m.
attraction in a circular orbit, circumscribing the (a) When the particle is located at Q.
triangle while still preserving the equilateral triangle? (b) When the particle is located at P.
M3

M2

M1
x P

Q
y

Sol. F A = F AB + F AC
GM 2 
= 2  2  cos 30 °
 a 
GM 2  Sol. Attraction at an external point due to spherical shell of
=  2 ⋅ 3
 a   GM 
mass M is  2  while at an internal point is zero.
a r 
r=
3 (a) Point is external to shell M1 , M 2 and M 3 ,
mv 2 So, force at Q will be
Now, =F GM1m GM 2 m GM 3 m
r FQ = + +
mv 2 3 GM 2 y2 y2 y2
or = 2 3 Gm
a a = 2 (M1 + M 2 + M 3 )
GM y
∴ v= (b) Force at P will be
a
GM1m GM 2 m
FP = + +0
Example 11. A planet of mass m revolves in an x2 x2
elliptical orbit around the sun so that its maximum and Gm
= 2 (M1 + M 2 )
minimum distances from the sun are equal to ra and r p x
respectively. Find the angular momentum of this planet
relative to the sun. Example 13. If the radius of the Earth contracts to
Sol. Using conservation of angular momentum, half of its present value without change in its mass,
mv p rp = mv a ra what will be the new duration of the day?
As, velocities are perpendicular to the radius vectors at Sol. Present angular momentum of the Earth
apogee and perigee. 2
L 1 = Iω = MR 2 ω
⇒ v p rp = v a ra 5
Using conservation of energy, New angular momentum, because of change in radius
GMm 1 − GMm 1 2  R
2
− + mv p2 = + mv a2 L 2 = M   ω′
rp 2 ra 2 5  2
By solving, the above equations, If external torque is zero, then angular momentum must
2GMra be conserved
vp = L1 = L 2
rp (rp + ra )
2 1 2
MR ω = × MR 2 ω ′
2
L = mv p rp 5 4 5
2 GMrp ra i.e. ω′ = 4 ω
=m 1 1
(rp + ra ) T ′ = T = × 24 = 6 h
4 4
NCERT Selected Questions
Q 1. Answer the following: Sol. From the relation, T 2 ∝ R 3 we have
2/ 3
(a) You can shield a charge from electrical forces by  T ′  r′ 
  = 
putting it inside a hollow conductor. Can you T   r
shield a body from gravitational influence of 2/ 3
 1
nearby matter by putting it inside a hollow or r′ =   r
 2
sphere or by some other means? 1/ 3
 1
(b) An astronaut inside a small spaceship orbiting or r′ =   r
 4
around the earth cannot detect gravity. If the
1 r r
space station orbiting around the earth has a = 1/ 3
r= 1/ 3
= = 0.63r
large size, can he hope to detect gravity? (4 ) (4 ) 159
.
Therefore, orbital size of planet = 0.63 times the orbital size
Sol. (a) No, induction effects take place in electrostatics not in of the earth.
gravitation.
(b) Yes, he can hope to detect gravity. If the size of the Q 4. If one of the satellites of the jupiter has an orbital
spaceship is extremely large, then the magnitude of the period of 1.769 days and the radius of the orbit is
gravity will become appreciable and hence the 4.22 ×10 8 m. Show that mass of the jupiter is about
gravitational effect of the spaceship may become one thousandth that of the sun.
measurable.
Sol. Using the relation,
Q 2. Choose the correct alternatives. GM
= ω2,
(a) Acceleration due to gravity increases/decreases r3
2
with increasing altitude. GM J  2π 
= 
(b) Acceleration due to gravity increases/decreases r3 T 
with increasing depth (assume the earth to be a 4 π 2r3
sphere of uniform density). or MJ =
T 2G
(c) Acceleration due to gravity is independent of the 4 × 9.87 × (4.22 × 108 )3
mass of the earth/mass of the body. =
(15.2841 × 104 )2 × 6.67 × 10−11
 1 1
(d) The formula – GMm  −  is more or less = 1.9 × 1027 kg
 r2 r1 

~ 2 × 1027 kg
accurate than the formula mg ( r2 − r1 ) for the
M J 2 × 1027 1
difference of potential energy between two ∴ = ≈
points r2 and r1 distance away from the centre of M S 2 × 1030
1000
the earth. 1
or MJ ≈ MS
1000
Sol. (a) Acceleration due to gravity decreases with increasing
i.e. mass of the jupiter is about one thousandth of the mass of
altitude.
the sun.
(b) Acceleration due to gravity decreases with increasing
depth. Q 5. Let us assume that our galaxy consists of 2.5 × 1011
(c) Acceleration due to gravity is independent of the mass
stars each of one solar mass. How long will a star at
a distance of 50000 light years from the galactic
of the body.
centre take to complete one revolution? Take the
 1 1 diameter of the Milky way to be 10 5 light years.
(d) The formula – GMm  −  is more accurate than the
 r2 r1 
Sol. One solar mass = 2 × 1030 kg
formula mg (r2 − r1 ) for the difference of potential
1 light year = 9.46 × 1015 m
energy between two points r1 and r2 distance away from
Let M = Mass of stars in the galaxy
the centre of the earth.
= 2.5 × 1011 × 2 × 1030 kg
Q 3. Suppose there existed a planet that went around the = 5 × 1041 kg
sun twice as fast as the earth. What would be its r = Radius of orbit of a star
orbital size as compared to that of the earth? = Distance of a star from galactic centre
Gravitation 423

= 50000 light years (e) Potential energy depends upon the distance between the
= 50000 × 9.46 × 1015 m sun and the comet, hence PE changes in the elliptical
orbit as the distance between sun and the comet
Diameter of Milky way = 105 light years
changes continuously.
Using the relation, (f) As total energy is the sum of KE and PE, the total
2
GM  2π  energy always remains constant.
= ω 2 =   , we get
r3 T 
Q 9. A rocket is fired from the earth towards the sun. At
4 π 2r3 4 × 9.87 × (5 × 9.46 × 1019 )3 what distance from the earth’s centre is the
T =
2
=
GM 6.67 × 10−11 × 5 × 1041 gravitational force on the rocket zero? Mass of the
= 12527.5 × 1028 s2 sun = 2 × 10 30 kg, mass of the earth = 6.0 × 10 24 kg.
∴ T = 111.93 × 1014 s
Neglect the effect of the other planets, etc. (Orbital
radius = 1.5 × 1011 m)
111.93 × 1014
= yr = 3.55 × 108 yr
365 × 24 × 3600 Sol. Let P be a point at a distance r from the earth’s centre where
gravitational force due to sun and the earth are equal and
Q 6. Choose the correct alternative. opposite and hence gravitational force on the rocket is zero.
(a) If the zero of potential energy is at infinity, the s
total energy of an orbiting satellite is negative of
the kinetic/potential energy. x–r
(b) The energy required to rocket an orbiting x
satellite out of the earth’s gravitational influence P
is more/less than the energy required to project a r
stationary object at the same height (as the e
satellite) out of the Earth’s influence.
GM s m GM em
Sol. (a) Kinetic energy (b) Less. ∴ =
(x − r)2 r2
Q 7. Does the escape velocity of a body from the earth GM s GM
or = 2e
depend on (a) the mass of the body, (b) the height (x − r)2 r
from where it is projected, (c) the direction of
2 × 1030 6 × 1024
projection? or =
(x − r)2 r2
Sol. (a) No, we know that the escape velocity of the body is
(x − r)2 2 × 1030 1
given by or = = × 106
2GM r2 6 × 1024 3
ve = , where M and R are mass and radius of the
R x − r 103
or =
earth. Thus clearly, it does not depend on the mass of r 3
the body.  3 
or r= x
(b) Yes.  1735
(c) No, it does not depend on the direction of projection. 3
= × 1.5 × 1011
Q 8. A comet orbits the sun in highly elliptical orbit. 1735
Does the comet has a constant (a) linear speed, 3 × 15 × 100 × 108
=
(b) angular speed, (c) angular momentum, 1735
(d) kinetic energy, (e) potential energy and (f) total or r = 2.594 × 108 m
energy throughout its orbit? Neglect any mass loss
= 2.6 × 108 m from the earth.
of the comet when it comes very close to the sun.
Sol. (a) The comet moves faster when it is close to the sun and Q 10. How will you weigh the sun, i.e. estimate its mass?
moves slower when it is farther away from the sun. You will need to know the period of one of its
Hence, the linear speed of the comet does not remain planets and the radius of the planetary orbit. The
constant. mean orbital radius of the Earth around the Sun is
(b) Angular speed of the comet is not constant. 1.5 ×10 8 km. Estimate the mass of the Sun.
(c) The angular momentum of the comet remains constant.
Sol. We know that the earth revolves around the sun in an orbit of
(d) Since, the linear speed of the comet around the sun
changes continuously, so its kinetic energy also radius 1.5 × 1011m and completes one revolution around the
changes continuously. sun in 365 days.
424 Objective Physics Vol. 1

∴ R = Radius of orbit of the earth= 1.5 × 1011m Q 13. Assuming the earth to be a sphere of uniform mass
T = Time period of earth around the sun density, how much would a body weigh half way
= 365 days = 365 × 24 × 60 × 60s down to the centre of the earth, if it weighed 250 N
G = 6.67 × 10−11 Nm 2 kg−2 on the surface?
 d
We know that Sol. gd = g 1 − 
R GM s  R
=
T 2 4 π 2R 2 Here, d=
R
4π R
2 3 2
or Ms =  R/2  1
T 2G gd = g 1 −  = g 1 − 
 R  2
4 × 9.87 × (1.5 × 1011 )3
= kg =
g
(365 × 24 × 60 × 60)2 × 6.67 × 10−11 2
or M s = 2.0 × 1030 kg ∴ wd = mgd = m ×
g
2
Q 11. A saturn year is 29.5 times the earth year. How far is 1 1
= mg = w
the saturn from the sun, if the earth is 1.50 ×10 8 km 2 2
away from the sun? 1
= × 250
2
Sol. According to the Kepler’s third law,
= 125 N
T 2 ∝ R3
∴Weight of the body half way down to the centre of earth
Ts2 Rs3  Ts 
2
R 
3
= 125 N.
∴ = or   =  s
Te2 Re3  Te   Re  Q 14. A rocket is fired vertically with a speed of 5 km s −1
Now, Ts = 29.5 Te (given) or
Ts
= 29.5
from the earth’s surface. How far from the earth
Te does the rocket go before returning to the earth?
and Re = Distance of earth from sun = 1.5 × 108 km Mass of the earth = 6.0 × 10 24 kg, mean radius of
3
earth = 6.4 × 10 6 m, G = 6.67 × 10 −11 Nm 2 kg −2
 Rs 
(29.5)2 =   GM
 1.5 × 108  Sol. g = = 9.8 m /s2
R2
or Rs3 = (29.5)2 × (1.5 × 108 )3
Maximum height is given by
or Rs3 = 2.937 × 1027 km 3 Rv 2
∴ h=
∴ Rs = (2.937 × 1027 km)1/ 3 2gR − v 2

= 1.43 × 109 km = 1.43 × 1012 m Here, v = 5 km s−1 = 5000 ms−1 (given)


R = 6.4 × 10 m
6
Q 12. A body weighs 63 N on the surface of the earth.
Putting these values, we get
What is the gravitational force on it due to the earth
at a height equal to half of the radius of the earth? 6.4 × 106 × (5 × 103 )2
h=
−2
2 × 9.8 × 6.4 × 106 − (5 × 103 )2
 h
Sol. gh = g 1 +  = 1.6 × 106 m = 1600 km
 R
∴ Distance from centre of the earth
R
Here, h= = R + h = 6.4 × 106 + 1.6 × 106
2
 R / 2
−2 = 8.0 × 106 m
∴ gh = g 1 + 
 R  Q 15. The escape speed of a projectile on the earth’s
or gh = g
4 surface is 11.2 km s −1 . A body is projected out with
9 thrice this speed. What is the speed of the body far
w = mg = 63N …(given) away from the earth? Ignore the presence of the sun
4 4 and other planets.
and wh = mgh = m × g = mg
9 9 Sol. According to the principle of conservation of the energy
4 Initial KE + Initial PE = Final KE + Final PE
= × 63 = 28 N
9 1 2 GMm 1
or mv − = mv′ 2 + 0
wh = 28 N 2 R 2
Gravitation 425

1 1 GMm  1 1 
or mv′ 2 = mv 2 − …(i) = 6.67 × 10−11 × 2 × 1030  − 
2 2 R  2 × 107 1012 
Also let, ve = escape velocity = 13.34 × 109 (5 × 10−8 − 10−12 )
1 2 GMm
mve = …(ii) = 13.34 × 1019 × 4.9999 × 10−8
2 R
∴ From Eqs. (i) and (ii), we get or v = 6.6699 × 1012 or v = 2.583 × 106 ms−1
1 1 1
mv′ 2 = mv 2 − mve2 …(iii) = 2.6 × 106 ms−1
2 2 2
or v′ 2 = v 2 − ve2
Q 17. Two heavy spheres each of mass 100 kg and radius
0.10 m are placed 1.0 m apart on a horizontal table.
ve = 11.2 kms−1  What is the gravitational force and potential at the
Now,  …(iv) (given)
v = 3ve  mid-point of the line joining the centres of the
∴ From Eqs. (iii) and (iv), we get
spheres? Is an object placed at that point in
equilibrium? If so, is the equilibrium stable or
v′ 2 = (3ve )2 − ve2 unstable?
= 9ve2 − ve2 = 8ve2
Sol.
= 8 × (11.2)2 A O B
or v′ = 8 × (11.2)2 = 8 × 11.2
Net force on any mass placed at the centre of two spheres is
= 2 × 1.414 × 11.2
zero.
= 31.68 kms−1
If V be the total gravitational potential at point O, i.e.
∴ Speed of the body far away from the earth, mid-point of the line joining the centres of two spheres, then
v′ = 31.68 kms−1 = 31.7 kms−1 V = VA + VB
G × 100  G × 100
Q 16. Two stars each of 1 solar mass (2 × 10 30 kg) are =− + − 
(0.5)  (0.5) 
approaching towards each other for a head on
2 × G × 100
collision. When they are at a distance 10 9 km, their =−
speeds are negligible. What is the speed with which 0.5
they collide? The radius of each star is 10 4 km. − 2 × 6.67 × 10−11 × 100
=
Assume the stars to remain undistorted untill they 0.5
collide. (Use the known value of G) = −2.7 × 10−8 J kg−1
Sol. Here, mass of each star, M = 2 × 1030 kg = one solar mass. = − 27 × 10−9 J kg−1
Initial distance between two stars, r = 109 km = 1012 m An object placed at the mid-point is in an unstable
equilibrium.
Size of each star = radius of each star, R = 104 km = 107 m
Let v = speed with which they just collide with each other. Q 18. As you have learnt in the text, a geostationary
satellite orbits the earth at a height of nearly
36000 km from the surface of the earth. What is the
potential due to the earth’s gravity at the site of this
R R satellite? (Take the potential energy at infinite to be
zero.) Mass of earth = 6.0 × 10 24 kg, radius
r' = 6400 km.
Sol. Mass of the earth, M = 6.0 × 1024 kg
∴ Total final KE of stars at the time of collision
1 1 Radius of the earth, R = 6400 km
= Mv 2 + Mv 2 = Mv 2. Distance of the satellite from earth’s surface, h = 36000 km
2 2
∴ According to the law of conservation of energy. ∴The gravitational potential due to earth’s gravity at the site
Total initial energy = Final KE + Final PE − GM 6.67 × 10−11 × 6 × 1024
of the satellite = =−
R+h (6400 + 3600) × 103
GM 2 GM 2
or − = Mv 2 −
40.02 × 1013
r 2R =− Jkg−1
 1 1 425 × 105
or v = GM 
2
− 
 2R r = −9.43 × 106 Jkg−1
Objective Problems
[ Level 1 ]
Gravitational Force, Acceleration 9. The weight of a body at the centre of the earth is
due to Gravity (a) zero
(b) infinite
1. The mass of a planet is twice the mass of the earth and
(c) same as on the surface of the earth
diameter of the planet is thrice the diameter of the earth,
(d) None of the above
then the acceleration due to gravity on the planet’s
surface 10. If the change in the value of g at a height h above the
(a) g /2 (b) 2g (c) 2g / 9 (d) 3g / 2 surface of earth is the same as at a depth d below it (both
h and d are much smaller than the radius of the earth),
2. If radius of the earth is R, then the height h at which the
then
value of g becomes one-fourth, is
(a) d = h (b) d = 2h
(a) 2R (b) 3R (c) R (d) 4R (c) d = h/2 (d) d = h2
3. Weight of a body is maximum at
11. The rotation of the earth having radius R about its axis
(a) poles (b) equator
speeds up to a value such that a man at latitude angle 60°
(c) centre of the earth (d) at latitude 45°
feels weightlessness. The duration of the day in such a
4. The depth d, at which the value of acceleration due to case is
1 R π R
gravity becomes times the value at the surface, is (a) π (b)
n g 2 g
(R = radius of the earth) π R g
(c) (d) π
R  n − 1 3 g R
(a) (b) R  
n  n 
12. Gravitational force between a point mass m and M
R  n 
(c) (d) R   separated by a distance is F. Now, if a point mass 2m is
n2  n + 1 placed next to m in contact with it, the force on M due to
5. If a planet consists of a satellite whose mass and radius m and the total force on M are
were both half that of the earth, the acceleration due to (a) 2F, F (b) F, 2F
gravity at its surface would be (c) F, 3F (d) F, F
(a) 4.9 m/s 2 (b) 9.8 m/s 2 13. The weights of an object in a coal mine, at sea level and at
(c) 19.6 m/s 2 (d) 29.4 m/s 2 top of mountains are w1 , w 2 and w 3 respectively, then
6. Two point masses each equal to 1 kg attract one another (a) w1 < w2 > w3 (b) w1 = w2 = w3
with a force of 10−9 kg-wt. The distance between the two (c) w1 < w2 < w3 (d) w1 > w2 > w3
point masses is approximately 14. When a body is taken from the equator to the poles, its
(G = 6.6 × 10−11 MKS units) weight
(a) remains constant
(a) 8 cm (b) 0.8 cm
(c) 80 cm (d) 0.08 cm (b) increases
(c) decreases
7. A body has a weight 72 N. When it is taken to a height (d) increases at N-pole and decreases at S-pole
h = R = radius of the earth, it would weight
15. If the earth suddenly shrinks (without changing mass) to
(a) 72 N (b) 36 N
(c) 18 N (d) zero half of its present radius, the acceleration due to gravity
will be
8. The angular speed of the earth in rad/s so that the object on (a) g/ 2 (b) 4g
equator may appear weightlessness, is (radius of earth (c) g/4 (d) 2g
= 6400 km)
16. The diameters of two plantes are in the ratio 4:1 and their
(a) 1.23 × 10−3
mean densities in the ratio 1:2. The acceleration due to
(b) 6.20 × 10−3 gravity on the planets will be in ratio
(c) 1.56 (a) 1 : 2 (b) 2 : 3
(d) 1.23 × 10−5 (c) 2 : 1 (d) 4 : 1
Gravitation 427

17. If density of the earth increased 4 times and its radius 24. The angular velocity of rotation of a star (of mass M and
become half of what it is, our weight will? radius R) at which the matter will start escaping from its
(a) Be four times its present value equator is
(b) Be doubled 2GR GM
(c) Remain same (a) (b)
M R3
(d) Be halved
GR 2GM 2
18. The acceleration due to gravity near the surface of a (c) (d)
M R
planet of radius R and density d is proportional to
d d 25. At what depth below the surface of the earth acceleration
(a) (b) dR 2 (c) dR (d)
R2 R due to gravity will be half its value 1600 km above the
19. If earth is supposed to be sphere of radius R, if g 30° is surface of the earth?
value of acceleration due to gravity at latitude of 30° and (a) 4.3 × 106 m (b) 2.4 × 106 m
g at the equator, the value of g − g 30° is (c) 3.2 × 106 m (d) 1.6 × 106 m
1 3
(a) ω 2R (b) ω 2R
4 4
1
Gravitational Field and Potential
(c) ω 2R (d) ω 2R 26. Unit of gravitational potential is
2
(a) Joule (b) Joule/kilogram
20. If M is the mass of the earth and R its radius, the ratio of (c) Joule kilogram (d) None of these
the gravitational acceleration and the gravitational
27. A mass m is placed inside a hollow sphere of mass M as
constant is
shown in figure. The gravitation force on mass m is
R2 M
(a) (b)
M R2
M
(c) MR 2 (d) m
R r

21. A mass M is split into two parts, m and (M − m), which


are then separated by a certain distance. What ratio of R
m/M maximizes the gravitational force between the two
parts?
(a) 1/3 (b) 1/2 GMm GMm
(c) 1/4 (d) 1/5 (a) (b)
R2 r2
22. Assuming, the earth to have a constant density, point out GMm
(c) (d) zero
which of the following curves show the variation of (R − r)2
acceleration due to gravity from the centre of earth to the
28. For a uniform ring of mass M and radius R at its centre,
points far away from the surface of earth
(a) field and potential both are zero
g g GM
(b) field is zero but potential is
R
(c) field is zero but potential is − GM / R
(a) (b) GM GM
(d) magnitude of field is 2 and potential −
R R
R r R r 29. Two bodies of masses m and M are placed a distance d
g apart. The gravitational potential at the position, where
the gravitational field due to them is zero, is V. Then,
G Gm
(c) (d) None of these (a) V = − (m + M ) (b) V = −
d d
GM G
(c) V = − (d) V = − ( m + M )2
R r d d

23. The height above the surface of the earth, where 30. A thin rod of length L is bent to form a semicircle. The
1 mass of rod is M. What will be the gravitational potential
acceleration due to gravity is of its value at surface of
64 at the centre of the circle?
the earth is approximately GM GM
(a) − (b) −
(a) 45 × 10 m6
(b) 54 × 10 m
6 L 2πL
πGM πGM
(c) 102 × 10 m 6
(d) 72 × 106 m (c) − (d) −
2L L
428 Objective Physics Vol. 1

31. A uniform solid sphere of mass m and radius r is 37. By which curve will be variation of gravitational
surrounded symmetrically by a uniform thin spherical potential of a hollow sphere of radius R with distance be
shell of radius 2r and mass m. depicted?
(a) The gravitational field at a distance of 15
. r from the centre is V V
2 Gm
9 r2
(b) The gravitational field at a distance of 2.5r from centre is R
8 Gm (a) O (b) O
R r r
25 r2
(c) The gravitational field at a distance of 1.5r from centre is
zero
(d) The gravitational field between the sphere and spherical
shell is uniform V V

32. Inside a uniform shell,


(a) potential energy is zero (b) potential is zero R R
(c) potential is constant (d) All of these (c) O (d) O
r r
33. At what height the gravitational field reduces by 75%, the
gravitational field at the surface of earth?
(a) R (b) 2R (c) 3R (d) 4R
(where, R is the radius of the earth) 38. Which one of the following graphs represents correctly
represent the variation of the gravitational field (E ) with
34. If V is the gravitational potential on the surface of the
earth, then what is its value at the centre of the earth? the distance (r ) from the centre of a spherical shell of
mass M radius R?
(a) 2V (b) 3V
E E
3 2
(c) V (d) V
2 3
35. 3 particles each of mass m are kept at vertices of an (a) (b)
equilateral triangle of side L. The gravitational field at
centre due to these particles is
(a) zero r=R r r=R r
3GM
(b)
L2 E E
9GM
(c)
L2 (c) (d)
12GM
(d)
3L2
36. The diagram showing the variation of gravitational r=R r r=R r
potential of the earth with distance from the centre of the
earth is Gravitational Potential Energy,
V V Energy Conservation and Escape
Velocity
(a) O R (b) O R 39. In a gravitational field, if a body is bound with the earth,
r r then total mechanical energy has
(a) positive
(b) zero
(c) negative
V V (d) may be positive, negative or zero
40. The velocity with which a projectile must be fired to
escape from the earth does not depend upon
(c) O R (d) O R
(a) mass of the earth
r r
(b) mass of the projectile
(c) radius of the earth
(d) None of the above
Gravitation 429

41. When a body is lifted from surface of the earth height 51. At what angle with the horizontal should a projectile be
equal to radius of the earth, then the change in its PE is fired with the escape velocity to enable it escape from
1 gravitational pull of the earth?
(a) mg R (b) 2mg R (c) mg R (d) 4mg R
2 (a) Less than 45° (b) 45°
(c) More than 45° (d) Any angle
42. Escape velocity is given by
(a) 2gR (b) gR (c) 2 gR (d) 2 gR 52. Energy required in moving a body of mass m from a
distance 2R to 3R from centre of the earth of mass M is
43. Escape velocity on the earth is 11.2 km/s. What would be GMm GMm
(a) (b)
the escape velocity on a planet whose mass is 1000 times 12R 2 3R 2
and radius is 10 times that of the earth? GMm GMm
(c) (d)
(a) 112 km/s (b) 11.2 km/s 8R 6R
(c) 1.12 km/s (d) 3.7 km/s
53. The kinetic energy needed to project a body of mass m
44. The escape velocity from the earth is v e . A body is from the earth surface (radius R) to infinity is
projected with velocity 2v e . With what constant velocity mgR
(a) (b) 2mgR
will it move in the inter planetary space? 2
(a) ve (b) 2ve mgR
(c) mgR (d)
(c) 3ve (d) 5ve 2
54. Gas escapes from the surface of a planet, because it
45. The escape velocity for a body of mass 1 kg from the acquires an escape velocity. The escape velocity will
earth’s surface is 11.2 kms −1 . The escape velocity for a depend on which of the following factors:
body of mass 100 kg would be
I. Mass of the planet
. × 102 kms −1
(a) 112 (b) 112 kms −1
II. Mass of the particle escaping
(c) 11.2 kms −1 . × 10−2 kms −1
(d) 112
III. Temperature of the planet
46. When escape velocity is given to a particle on surface of IV. Radius of the planet
the earth, its total energy is Select the correct answer from the codes given below
(a) zero (b) greater than zero (a) I and II
GMm
(c) less than zero (d) − (b) II and IV
2R (c) I and IV
where, M = mass of the earth, m = mass of particle and (d) I, III and IV
R = radius of the earth. 55. What is the energy required to launch a m kg satellite
47. The escape velocity of a body projected vertically from the earth’s surface in a circular orbit at an altitude of
upward from the earth’s surface is 11.2 km/s. If the body 2R (R = radius of the earth)
is projected in a direction making 30° angle to the 2
(a) mgR (b) mgR
vertical, its escape velocity in this case will be 3
112
. 5 1
(a) 11.2 km/s (b) km/s (c) mgR (d) mgR
2 6 3
3 112
.
. ×
(c) 112 km/s (d) km/s Motion of Satellites in Circular
2 3
48. The escape velocity of a particle of mass m varies as Orbits and Planets in Elliptical
(a) m2 (b) m (c) m0 (d) m−1 Orbits
49. The ratio of the radius of the earth to that of the moon is 56. A satellite of the earth is revolving in a circular orbit with
10. The ratio of the acceleration due to gravity on the a uniform speed v. If gravitational force suddenly
earth to that on the moon is 6. The ratio of the escape disappears, the satellite will
velocity from the earth’s surface to that from the moon is (a) continue to move with speed v along the original orbit
(a) 4 (b) 6 (b) move with the velocity v tangentially to the original orbit
(c) 12 (d) None of these (c) fall downward with increasing velocity
50. With what velocity should a particle be projected so that (d) ultimately come to rest somewhere on the original orbit
its height becomes equal to radius of the earth? 57. Kepler’s law states that square of the time period of any
1/ 2 1/ 2
 GM   8GM  planet about the sun is directly proportional to
(a)   (b)  
 R   R  1
(a) R (b)
1/ 2 1/ 2 R
 2GM   4GM 
(c)   (d)   1
 R   R  (c) R 3 (d) 3
R
430 Objective Physics Vol. 1

58. A body is projected from earth’s surface to become its 67. The period of revolution of planet A round the sun is 8
satellite, its time period of revolution will not depend times that of B. The distance of A from the sun is how
upon many times greater than that of B from the sun?
(a) mass of the earth (a) 5 (b) 4 (c) 3 (d) 2
(b) its own mass 68. Kepler’s second law is based on
(c) gravitational constant
(a) newton’s first law
(d) radius of orbit
(b) newton’s second law
59. Satellite is revolving around the earth. If its radius of (c) special theory of relativity
orbit is increased to 4 times of the radius of geostationary (d) conservation of angular momentum
satellite, what will become its time period? 69. An artificial satellite moving in a circular orbit around the
(a) 8 days (b) 4 days earth has total mechanical energy E 0 .
(c) 2 days (d) 16 days Its potential energy is
60. The ratio of mean distances of three planets from the sun (a) −2E0 (b) 15
. E0 (c) 2E0 (d) E0
are 0.5 : 1 : 1 : 5, then the square of time periods are in the 70. An artificial satellite moving in a circular orbit around the
ratio of earth has a total (kinetic + potential) energy E 0 . Its
(a) 1 : 4 : 9 (b) 1 : 9 : 4 potential energy and kinetic energy respectively are
(c) 1 : 8 : 27 (d) 2 : 1 : 3
(a) 2E0 and – 2E0 (b) – 2E0 and E0
61. A planet moves around the sun. It is closest to sun at a (c) 2E0 and − E0 (d) – 2E0 and – E0
distance d1 and have velocity v1 .At farthest distance d 2
71. If mean radius of earth is R, its angular velocity is ω, and
its speed will be
the acceleration due to gravity at the surface of the earth
d12v1 d2v1
(a) (b) is g, then the cube of the radius of the orbit of
d22 d1
geostationary satellite will be
d1v1 d22v1 R 2g R 2ω 2
(c) (d) (a) (b)
d2 d12 ω2 g
62. Two satellites A and B, ratio of masses 3 : 1 are in circular R 2g Rg
(c) (d) 2
orbits of radii r and 4r. Then, the ratio of total mechanical ω ω
energy of A and B is 72. An earth’s satellite is moved from one stable circular
(a) 1: 3 (b) 3:1 (c) 3: 4 (d) 12:1
orbit to another higher stable circular orbit. Which one of
63. The ratio of distance of two satellites from the centre of the following quantities increase for the satellite as a
earth is 1: 4 . The ratio of their time periods of rotation result of this change.
will be (a) Angular momentum (b) Kinetic energy
(a) 1: 4 (b) 4 :1 (c) 1: 8 (d) 8:1 (c) Angular velocity (d) Linear orbital speed
64. For a satellite in elliptical orbit which of the following 73. When a planet moves around sun, its
quantities does not remain constant? (a) areal velocity is constant
(a) Angular momentum (b) linear velocity is constant
(b) Momentum (c) angular velocity is constant
(c) Areal velocity (d) All of the above
(d) Total energy 74. The orbital velocity of a body close to the earth’s surface
65. The orbital velocity of an artificial satellite in a circular is
orbit just above the earth’s surface is v. For a satellite (a) 8 km/s (b) 11.2 km/s
orbiting at an altitude of half the earth’s radius, the orbital (c) 3 × 108 m/s (d) 2.2 × 103 km/s
velocity is 75. For a satellite orbiting very close to earth’s surface, total
3 3 energy is
(a) v (b) v
2 2 GMm
(a) zero (b)
2 2 R
(c) v (d) v GMm GMm
3 3 (c) − (d) −
R 2R
66. In case of an orbiting satellite, if the radius of orbit is
decreased 76. Two identical satellites are orbiting at distances R and 7R
(a) its kinetic energy decreases from the surface of the earth, R being the radius of the
(b) its potential energy increases earth. The ratio of their
(c) Both (a) and (b) are correct (a) kinetic energies is 4 (b) potential energies is 4
(d) Both (a) and (b) are wrong (c) total energies is 4 (d) All of these
Gravitation 431

77. The distance of two planets from the sun are 1013 and Miscellaneous Problems
1012 m, respectively. The ratio of the periods of the
85. The acceleration due to gravity g and mean density of
planet is
1 earth ρ are related by which of the following relations?
(a) 100 (b)
10 (G = gravitational constant and R = radius of the earth)
(c) 10 (d) 10 10 4 πgR 2 4 πgR 3
(a) ρ = (b) ρ =
3G 3G
78. The period of a satellite in a circular orbit around a planet 3g 3g
is independent of (c) ρ = (d) ρ =
4 πGR 4 πGR 3
(a) the mass of the planet
(b) the radius of the planet 86. What is not conserved in case of celestial bodies
(c) the mass of the satellite revolving around sun all the time?
(d) All the three parameters (a), (b) and (c) (a) Kinetic energy (b) Mass
(c) Angular momentum (d) None of these
79. Two satellites A and B go round a planet P in circular
orbits having radii 4R and R, respectively. If the speed of 87. The distance of the centres of moon and the earth is D.
the satellite A is 3v, the speed of the satellite B will be The mass of the earth is 81 times the mass of the moon.
4 3 At what distance from the centre of the earth, the
(a) 12v (b) 6v (c) v (d) v gravitational force on a particle will be zero?
3 2
D 2D 4D 9D
80. A satellite is revolving in circular orbit of radius r around (a) (b) (c) (d)
2 3 3 10
the earth of mass M. Time of revolution of satellite is
r5 88. The rotation of the earth about its axis speeds up such that
(a) T ∝ a man on the equator becomes weightless. In such a
GM
situation, what would be the duration of one day?
r3
(b) T ∝ R 1 R
GM (a) 2π (b)
g 2π g
r
(c) T ∝ 1
GM 2 (c) 2π Rg (d) Rg

3
r3
89. A body of mass m is kept at a small height h above the
(d) T ∝ ground. If the radius of the earth is R and its mass is M,
GM
the potential energy of the body and the earth system
4
(with h = ∞ being the reference position) is
81. Two satellites A and B, ratio of masses 3 : 1 are in circular GMm − GMm
(a) + mgh (b) + mgh
orbits of radii r and 4r. Then, the ratio of total mechanical R R
energy of A to B is GMm GMm
(c) − mgh (d) − − mgh
(a) 1: 3 (b) 3:1 (c) 3: 4 (d) 12:1 R R
82. Which of the following quantities does not depend upon 90. A simple pendulum has a time period T1 when on the
the orbital radius of a satellite? earth’s surface and T2 when taken to a height R above the
T T2 earth’s surface, where R is the radius of the earth. The
(a) (b) T
R R value of 2 is
T1 (b) 2
T2 T2 (a) 1 (c) 4 (d) 2
(c) 2 (d) 3
R R 91. A particle is projected vertically upwards from the
(R = radius of orbit) surface of earth (radius R) with a kinetic energy equal to
83. A satellite moves round the earth in a circular orbit of half of the minimum value needed for it to escape. The
radius R making one revolution per day. A second height to which it rises above the surface of earth is
satellite moving in a circular orbit, moves round the earth (a) R (b) 2 R
once in 8 days. The radius of the orbit of the second (c) 3 R (d) 4 R
satellite is 92. Suppose, the gravitational attraction varies inversely as
(a) 8 R (b) 4 R the distance from the earth. The orbital velocity of a
(c) 2 R (d) R satellite in such a case varies as nth power of distance,
84. Kepler discovered where n is equal to
(a) laws of motion (a) −1
(b) laws of rotational motion (b) zero
(c) laws of planetary motion (c) +1
(d) laws of curvilinear motion (d) +2
432 Objective Physics Vol. 1

93. Consider the two identical particles shown in the given 99. If orbital velocity of planet is given by v = G a M b R c ,
figure. They are released from rest and may move then
towards each other under influence of mutual gravitation 1 1 1
(a) a = ,b = ,c = −
force 3 3 3
d 1 1 1
(b) a = , b = , c = −
m m 2 2 2
1 1 1
(i) The velocity of the centre of mass of the two particle (c) a = , b = − , c =
2 2 2
system 1 1 1
(d) a = , b = − , c = −
(a) is zero 2 2 2
(b) is constant (≠ 0)
(c) increases as the separation decreases 100.Suppose a smooth tunnel is dug along a straight line
(d) None of the above joining two points on the surface of the earth and a
(ii) Speed of each particle, when the separation reduces to particle is dropped from rest at its one end. Assume that
mass of the earth is uniformly distributed over its volume.
half its initial value
Then,
Gm 2Gm
(a) (b) (a) the particle will emerge from the other end with velocity
d d
GM e
Gm , where M e and Re are earth’s mass and radius
(c) (d) None of these 2Re
2d
respectively
94. If G is universal gravitational constant and g is (b) the overlapping will come to rest at centre of the tunnel
acceleration due to gravity, then the unit of the quantity (c) potential energy of the particle will be equal to zero at centre
G of tunnel, if it is along a diameter
is (d) acceleration of the particle will be proportional to its
g distance from mid-point of the tunnel
(a) kg-m 2 (b) kg/m
(c) kg/m 2 (d) m 2/kg
101. A planet is revolving round the sun in an elliptical orbit,
If v is the velocity of the planet when its position vector
95. If gravitational attraction between two points masses be from the sun is r, then areal velocity of the planet is
mm
given by F = G 1 n 2 . Then, the period of a satellite in a (a) | v × r |
r (b) 2| r × v |
circular orbit will be proportional to 1
n− 1 n+ 1 (c) (r × v)
2 2
2
(a) r (b) r
n (d) None of the above
(c) r 2 (d) independent of n
102. The magnitude of gravitational potential energy of a body
96. The centripetal force on a satellite orbiting round the at a distance r from the centre of earth is u. Its weight at a
earth and the gravitational force of the earth acting on the distance 2r from the centre of earth is
satellite both equal F. The net force on the satellite is u u u 4r
(a) (b) (c) (d)
(a) zero (b) F r 4r 2r u
(c) F 2 (d) 2 F
103. Two satellites of same mass are launched in the same
97. Reason of weightlessness in a satellite is orbit of radius r around the earth so as to rotate opposite
(a) zero gravity to each other. If they collide inelastically and stick
(b) no atmosphere together as wreckage, the total energy of the system just
(c) zero reaction force by satellite surface after collision is
(d) None of the above
2GMm
(a) −
98. Assume that the acceleration due to gravity on the surface r
of the moon is 0.2 times the acceleration due to gravity on GMm
(b) −
the surface of the earth. If R e is the maximum range of a r
projectile on the earth’s surface, what is the maximum GMm
(c)
range on the surface of the moon for the same velocity of 2r
projection? (d) zero
(a) 0.2 Re 104. Earth orbiting satellite will escape, if
(b) 2 Re (a) its speed is increased by 41%
(c) 0.5 Re (b) its KE is doubled
(d) 5 Re (c) Both (a) and (b) are correct
/ Speed of projectile is not very high. (d) Both (a) and (b) are wrong
Gravitation 433

105. A planet of mass m is in an elliptical orbit about the sun 107.What impulse need to be given to a body of mass m,
with an orbital period T. If A be the area of orbit, then its released from the surface of earth along a straight tunnel
angular momentum would be passsing through centre of earth, at the centre of earth, to
2mA mA
(a) (b) mAT (c) (d) 2mAT bring it to rest? (Mass of earth M, radius of earth R).
T 2T
GM
106. What additional velocity must be given to a satellite (a) m
R
orbiting around earth with radius R to become free from
GMm
the earth’s gravitational field? Mass of earth is M. (b)
R
GM GM
(a) ( 2 − 1) (b) ( 2 − 1) GM
R 2R (c) m
2R
GM GM
(c) ( 3 − 1) (d) ( 2 + 1) (d) Zero
R R

[ Level 2 ]
Only One Correct Option 5. A body is projected vertically upwards from the surface
1. A rocket is launched vertical from the surface of the earth of earth with a velocity equal to half the escape velocity.
of radius R with an initial speed v. If atmospheric If R be the radius of earth, maximum height attained by
resistance is neglected, the maximum height attained by the body from the surface of the earth is
the rocket is R
(a)
R R 6
(a) h = (b) h = R
 2gR   2gR  (b)
 2 − 1  2 + 1 3
 v   v 
2R
R2 R2 (c)
(c) h = (d) h = 3
 2gR   2gR  (d) R
 2 − 1  2 + 1
 v   v 
6. Pertaining to two planets, the ratio of escape velocities
2. Suppose, the gravitational force varies inversely as the from respective surfaces is 1: 2, the ratio of the time
n th power of distance. Then, the time period of a planet period of the same simple pendulum at their respective
in circular orbit of radius r around the sun will be surfaces is 2:1 (in same order). Then, the ratio of their
proportional to average densities is
1 1 (a) 1:1 (b) 1: 2
(n + 1) (n − 1)
(a) r 2 (b) r 2 (c) 1: 4 (d) 8:1
1
(n − 2 )
(c) rn (d) r 2 7. Four equal masses (each of mass M) are placed at the
corners of a square of side a. The escape velocity of a
3. A solid sphere of mass M and radius R has a spherical body from the centre O of the square is
R
cavity of radius such that the centre of cavity is at a 2GM 8 2GM
2 (a) 4 (b)
R a a
distance from the centre of the sphere. A point mass m
2 R 4GM 4 2GM
is placed inside the cavity at a distance from the centre (c) (d)
4 a a
of sphere. The gravitational force on mass m is 8. A point P ( R 3 ,0,0) lies on the axis of a ring of mass M
11GMm 14GMm
(a) (b) and radius R. The ring is located in y-z plane with its
R2 R2
GMm GMm centre at origin O. A small particle of mass m starts from
(c) (d)
2R 2 R2 P and reaches O under gravitational attraction only. Its
4. The required kinetic energy of an object of mass m so that speed at O will be
it may escape, will be GM Gm
(a) (b)
1 1 R R
(a) mgR (b) mgR GM Gm
4 2 (c) (d)
(c) mgR (d) 2mgR 2R 2R
434 Objective Physics Vol. 1

9. Two spherical bodies of masses m and 5m and radii R and 15. Two particles of mass m and M are initially at rest at
2R respectively are released in free space with initial infinite distance. Find their relative velocity of approach
separation between their centres equal to 12 R. If they due to gravitational attraction, when d is their separation
attract each other due to gravitational force only, then the at any instant
distance covered by smaller sphere just before collision 2G (M + m) G (M + m)
(a) (b)
will be d d
G (M + m) G (M + m)
(c) (d)
R 2R 2d 4d
12 R 16. An earth satellite of mass m revolves in a circular orbit at
m
5m
a height h from the surface of the earth. R is the radius of
the earth and g is acceleration due to gravity at the surface
(a) 5 R (b) 7.5 R of the earth. The velocity of the satellite in the orbit is
(c) 2.5 R (d) 6 R
given by
10. Energy of a satellite in circular orbit is E 0 . The energy gR 2
(a) (b) gR
required to move the satellite to a circular orbit of 3 times R+h
the radius of the initial orbit is gR gR 2
2 E 3 (c) (d)
(a) E0 (b) 2E0 (c) 0 (d) E0 R+h R+h
3 3 2
11. Two identical thin rings each of radius R are coaxially 17. The orbital angular momentum of a satellite revolving at
placed at a distance R. If the rings have a uniform mass a distance r form the centre is L. If the distance is
distribution and each has mass m1 and m2 respectively, increased to 16r, then the new angular momentum will be
then the work done in moving a mass m from centre of (a) 16 L (b) 64 L
one ring to that of the other is L
(c) (d) 4 L
Gm(m1 ± m2 )( 2 − 1) 4
(a) zero (b)
2R 18. The ratio of energy required to raise a satellite to a height
Gm 2 (m1 ± m2 ) Gmm1 ( 2 ± 1) h above the earth surface to that required to put it into the
(c) (d) orbit is
R m2R
(a) h : 2R (b) 2h : R
12. A person brings a mass of 1 kg infinity to a point A. (c) R : h (d) h : R
Initially, the mass was at rest but it moves at a speed of
19. A body which is initially at rest at a height R above the
2 m/s as it reached A. The work done by the person on the
surface of the earth of radius R, falls freely towards the
mass is −3 J . The potential at A is
earth, then its velocity on reaching the surface of the
(a) – 3 J/kg (b) – 12 J/kg
(c) – 5 J/kg (d) None of these
earth is
(a) (2gR ) (b) (gR )
13. A satellite is moving in a circular orbit round the earth
3
with a diameter of orbit 2R . At a certain point a rocket (c) gR (d) (4gR )
fixed to the satellite is fired such that it increases the 2
velocity of the satellite tangentially. The resulting orbit of 20. What is the energy required to launch a m kg satellite
the satellite would be from the earth’s surface in a circular orbit at an altitude of
(a) same as before 2R ? (R = radius of the earth )
(b) circular orbit with diameter greater than 2 R 2
(c) elliptical orbit with minimum distance from the earth equal (a) mgR (b) mgR
3
to R 5 1
(d) elliptical orbit with maximum distance from the earth equal (c) mgR (d) mgR
6 3
to R
21. A body of supercondense material with mass twice the
14. A particle would take a time t to move down a straight
mass of earth but size very small compared to the size of
tube from the surface of the earth (supposed to be a
earth starts from rest from h << R above the earth’s
homogeneous sphere) to its centre. If gravity were to
t surface. It reaches earth in time
remain constant, then the time would be t. The ratio of h 2h
t′ (a) t = (b) t =
will be g g
π π π (c) t =
2h
(d) t =
4h
(a) (b) (c) 2π (d)
2 2 3 2 3g 3g
Gravitation 435

22. A solid sphere of uniform density 28. A satellite is revolving round the earth with orbital speed
and radius R applies a gravitational R/2 A v o . If it stops suddenly, the speed with which it will strike
force of attraction equal to F1 on a R the surface of earth would be (v e = escape velocity of a
particle placed at a distance 2R particle on earth’s surface.)
from the centre of the sphere. A 2R ve2
R (a) (b) 2vo (c) ve2 − vo2 (d) ve2 − 2vo2
spherical cavity of radius is now vo
2
29. Two particles of equal mass m go round a circle of radius
made in the sphere as shown in the figure. The sphere
R under the action of their mutual gravitational attraction.
with cavity now applies a gravitational force F2 on the
F The speed of each particle is
same particle. The ratio 2 is Gm Gm
F1 (a) v = (b) v =
R 2R
5 7 3 7 1 Gm 4Gm
(a) (b) (c) (d) (c) v = (d) v =
9 8 4 9 2 R R
23. The magnitudes of the gravitational force at distances r1 30. Four particles, each of mass M, move along a circle of
and r2 from the centre of a uniform sphere of radius R radius R under the action of their mutual gravitational
and mass M are F1 and F2 , respectively. Then, (more than attraction. The speed of each particle is
one are correct) GM GM
F1 r1 (a) (b) 2 2
(a) = , if r1 < R and r2 < R R R
F2 r2
F r2 GM GM  2 2 + 1
(b) 1 = 12 , if r1 > R and r2 > R (c) (2 2 + 1) (d)  
F2 r2 R R  4 
F1 r2
(c) = , if r1 < R and r2 < R 31. Suppose a vertical tunnel is along the
F2 r1 y m
diameter of earth, assumed to be a sphere
F1 r22 of uniform mass density ρ. If a body of
(d) = , if r1 > R and r2 > R
F2 r12 mass m is thrown in this tunnel, its
acceleration at a distance y from the
24. The escape velocity from earth is v e . A body is projected
centre is given by
with velocity 2v e . With what constant velocity will it 4π 3
move in the inter planetary space? (a) Gρym (b) πρy
3 4
(a) ve (b) 2ve (c) 3ve (d) 5ve 4 4
(c) πρy (d) πGρy
25. Suppose, the gravitational attraction varies inversely as 3 3
the distance from the earth. The orbital velocity of a 32. Gravitational field at the centre of a semicircle formed by
satellite in such a case varies as nth power of distance, a thin wire AB of mass M and length l is
where n is equal to y
(a) – 1 (b) zero (c) + 1 (d) + 2
26. Let E be the energy required to raise a satellite to height h
above the earth’s surface and E′ be the energy required to
E A O B x
put the same satellite into orbit at that height. Then, is GM GM
E′ (a) 2 along x -axis (b) along y-axis
equal to l πl 2
GM 2πGM
(a)
2h
(b)
2h
(c)
R
(d)
2R (c) 2π 2 along x -axis (d) along y-axis
(R + 2h) (2R + 3h) R+h 2h + R l l2
33. A mass m is at a distance a from one end of a uniform rod
27. Two spheres of masses m and 2 m are separated by of length l and M. The gravitational force on the mass due
m to the rod is
distance d. A particle of mass is projected straight from
5 M a
2m towards m with a velocity v 0 . Which of the following
statements is correct? l
(a) Velocity of the particle decreases constantly GMm GmM
(b) Velocity of the particle increases constantly (a) (b)
(a + l ) a(l + a)
(c) Acceleration of the particle may become momentarily zero GMm GmM
(c) (d)
(d) The particle may retrace its path depending on value of v0 a 2(l + a)
436 Objective Physics Vol. 1

34. A uniform ring of mass m is lying at a distance 3 a from 40. The minimum energy required to launch a m kg satellite
the centre of a sphere of mass M just over the sphere from the earth’s surface in a circular orbit at an altitude of
(where, a is the radius of the ring as well as that of the 2R where R is the radius of earth, will be
sphere). Then, magnitude of gravitational force between 1 5 2 1
(a) mgR (b) mgR (c) mgR (d) mgR
them is 6 6 3 5
a
m
41. Three point masses each of mass m rotate in a circle of
radius r with constant angular velocity ω due to their
mutual gravitational attraction. If at any instant, the
√3a
masses are on the vertex of an equilateral triangle of side
a, then the value of ω is
M Gm 3Gm
a (a) (b)
a3 a3
Gm
(c) (d) None of these
GMm GMm 3a3
(a) (b)
8a2 3a2 42. A sphere of mass M and radius R 2 has a concentric cavity
GMm GMm
(c) 3 2 (d) 3 of radius R1 as shown in figure. The force F exerted by
a 8a2
the sphere on a particle of mass m located a distance r
35. A ring of mass m1 and radius R is fixed in space at some from the centre of sphere varies as (0 ≤ r ≤ ∞)
location. An external agent brings a point mass m2 from
infinity to centre of the ring .Work done by the external
agent will be R1
GM 1m2 GM 1m2
(a) − (b) R2
R R
G m12 + m22 GM 1m2
(c) (d)
R R (m12 + m22 )
F F
36. Energy required in moving a body of mass m from a
distance 2R to 3R from centre of earth of mass M is
(a) (b)
GMm GMm
(a) (b)
12R 2 3R 2
GMm GMm
(c) (d) r r
8R 6R
37. A planet of mass m moves around the sun of mass M in an F F
elliptical orbit. The maximum and minimum distance of
the planet from the sun are r1 and r2 , respectively. The (c) (d)
time period of the planet is proportional to
(a) r13/ 2 (b) (r1 + r2 )3/ 2
(c) (r1 − r2 )3/ 2 (d) r13/ 2 r r

38. A body attains a height equal to the radius of the earth. 43. If an artificial satellite is moving in a circular orbit
The velocity of the body with which it was projected is around the earth with a speed equal to half the magnitude
GM 2GM of the escape velocity from the earth, the height of the
(a) (b)
R R satellite above the surface of the earth is
1 GM GM R R
(c) (d) (a) 2 R (b) (c) R (d)
4 R 2R 2 4
M
39. If the mass of moon is , where M is the mass of earth, More than One Correct Options
81
find the distance of the point, where gravitational field due 1. Three planets of same density have radii R1 , R 2 and R 3
to earth and moon cancel each other, from the centre of such that R1 = 2R 2 = 3R 3 . The gravitational field at their
moon. Given that distance between centres of earth and respective surfaces are g 1 , g 2 and g 3 and escape
moon is 60 R, where R is the radius of earth velocities from their surfaces are v1 , v 2 and v 3 , then
(a) 4 R (b) 8 R (a) g1 / g2 = 2 (b) g1 / g3 = 3
(c) 12 R (d) 6 R (c) v1 / v2 = 1/ 4 (d) v1 / v3 = 3
Gravitation 437

2. For a geostationary satellite orbiting around the earth 6. Four point masses are placed at four corners of a square
identify the necessary condition. as shown. When positions of m and 2m are interchanged,
(a) It must lie in the equatorial plane of the earth 4m 3m
(b) Its height from the surface of the earth must be 36000 km
R
(c) Its period of revolution must be 2π , where R is the
g
radius of the earth
(d) Its period of revolution must be 24 h m 2m
3. A ball of mass m is dropped from a height h equal to the (a) gravitational field strength at centre will increase
radius of the earth above the tunnel dug through the earth (b) gravitational field strength at centre will decrease
as shown in the figure. Choose the correct options. (c) gravitational potential at centre will remain unchanged
(d) gravitational potential at centre will decrease
m
7. Two identical particles 1 and 2 are projected from surface
h of the earth with same velocities in the directions shown
in figure.
R
1
C 2

(a) Particle will oscillate through the earth to a height h on both (a) Both the particles will stop momentarily (before striking
sides with ground) at different times
(b) Particle will execute simple harmonic motion (b) Particle 2 will rise up to lesser height compared to particle 2
(c) Motion of the particle is periodic (c) Minimum speed of particle 2 is more than that of particle 1
(d) None of the above (d) Particle-1 will strike the ground earlier
4. Two point masses m and 2m are kept at points A and B as 8. A planet is moving round the sun in an elliptical orbit as
shown. E represents magnitude of gravitational field shown. As, the planet moves from A to B
strength and V the gravitational potential. As, we move B
from A to B
A
m 2m Sun
Planet
A B
(a) its kinetic energy will decrease
(a) E will first decrease then increases (b) its potential energy will remain unchanged
(b) E will first increase then decreases (c) its angular momentum about centre of sun will remain
(c) V will first decrease then increases unchanged
(d) V will first increase then decreases (d) its speed is minimum at A
5. Two spherical shells have mass m and 2m as shown. 9. A satellite of mass m is just placed over the surface of the
Choose the correct options. earth. In this position, mechanical energy of satellite is
2m E1 . Now, it starts orbiting round the earth in a circular
path at height h = radius of the earth. In this position,
m kinetic energy, potential energy and total mechanical
C energy of satellite are K 2 , U 2 and E 2 , respectively. Then,
A B E1 E1 U2
(a) U 2 = (b) E2 = (c) K 2 = − E2 (d) K 2 = −
2 4 2
10. A satellite is revolving round the earth in circular orbit
(a) if mass of the earth is made four times, keeping other
(a) Between A and B gravitational field strength is zero factors constant, orbital speed of satellite will become two
(b) Between A and B gravitational potential is constant times
(c) There will be two points one lying between B and C and (b) corresponding to change in part (a), times period of satellite
other lying between C and infinity, where gravitational field will remain half
strength are same (c) when value of G is made two times orbital speed increases
(d) There will be a point between B and C , where gravitational and time period decreases
potential will be zero (d) G has no effect on orbital speed and time period
438 Objective Physics Vol. 1

11. If the mass of the sun were ten times smaller and 4. Assertion Four point masses each of mass m are placed
gravitational constant G were ten times larger in at points 1, 2, 3 and 6 of a regular hexagon of side a.
magnitude. Then, Then, the gravitational field at the centre of hexagon is
(a) walking on ground would become more difficult Gm
(b) the acceleration due to gravity on the earth will not change .
(c) raindrops will fall much faster a2
5 4
(d) airplanes will have to travel much faster
12. If the sun and the planets carried huge amounts of
opposite charges,
6 3
(a) all three of Kepler’s laws would still be valid
(b) only the third law will be valid
(c) the second law will not change
(d) the first law will still be valid 1 2

13. There have been suggestions that the value of the Reason The field strength due to masses at 3 and 6 are
gravitational constant G becomes smaller when cancelled out.
considered over very large time period (in billions of
years) in the future. If that happens, for our earth, 5. Assertion The field strength at the centre of a ring is
(a) nothing will change zero.
(b) we will become hotter after billions of years Reason At the centre of the ring, slope of v-r graph is
(c) we will be going around but not strictly in closed orbits zero.
(d) after sufficiently long time we will leave the solar system
6. Assertion Angular momentum of a planet is constant
Assertion and Reason about any point.
Directions (Q. Nos. 1-20) These questions consist of two Reason Force acting on the planet is a central force.
statements each printed as assertion and reason. While 7. Assertion The binding energy of a satellite does not
answering these questions, you are required to choose anyone depend upon the mass of the satellite.
of the following five responses.
Reason Binding energy is the negative value of total
(a) If both Assertion and Reason are true and Reason is
energy of satellite.
the correct explanation of Assertion.
(b) If both Assertion and Reason are true but Reason is 8. Assertion If the product of surface area and density is
not the correct explanation of Assertion. same for two planets, escape velocities will be same for
(c) If Assertion is true but Reason is false. both.
(d) If Assertion is false but Reason is true.
(e) If both Assertion and Reason are false.
Reason Product of surface area and density is
proportional to the mass of the planet per unit radius of
1. Assertion If gravitational potential at some point is zero, the planet.
then gravitational field strength at that point will also be
zero. 9. Assertion Kepler’s laws for planetary motion are
consequence of Newton’s laws.
Reason Except at infinity gravitational potential due to a
system of point masses at some finite distance can’t be Reason Kepler’s laws can be derived by using
zero. Newton’s laws.
2. Assertion Gravitational force between two masses in air 10. Assertion The centres of two cubes of masses m1 and
is F. If they are immersed in water, force will remain F.
m2 are separated by a distance r. The gravitational force
Reason Gravitational force does not depend on the Gm1 m2
medium between the masses. between these two cubes will be .
r2
3. Assertion The centre of semicircular ring of mass mand
Gm Reason According to Newton’s law of gravitation,
radius R is the origin O. The potential at origin is − .
R gravitational force between two point masses m1 and m2
y Gm1 m2
separated by a distance r is .
r2
11. Assertion Mass of the rod AB is m1 and of particle P is
x
O m2 . Distance between centre of rod and particle is r.
Reason The gravitational field strength is towards Then, the gravitational force between the rod and the
y-axis. particle is
Gravitation 439

Gm1 m2 19. Assertion Gravitational potential and gravitational


F=
r 2 potential energy both are related to the work done by
A B P gravitational force in the gravitational field.
Reason Gravitational field strength is related to the
gravitational force in gravitational field.
r
20. Assertion If a particle is projected from the surface of
the earth with velocity equal to escape velocity, then total
Gm1 m2 mechanical energy is zero.
Reason The relation F = can be applied directly
r2 Reason Total mechanical energy of any closed system is
only two find force between two particles. always negative.
m2
12. Assertion Two spherical shells Match the Columns
have masses m1 and m2 . Their radii m1 1. On the surface of the earth, acceleration due gravity is g
are r1 and r2 . Let r be the distance of
and gravitational potential is V. Match the following
a point from centre. Then, r1
columns.
gravitational field strength and
Column I Column II
gravitational potential both are r2
equal to zero for 0 < r < r1 (A) At height h = R, value of g (p) Decreases by a factor 1
4
Reason In the region r1 < r < r2 , gravitational field (B) At depth h = R , value of g (q) Decreases by a factor 1
strength due to m2 is zero. But gravitational potential due 2 2
to m2 is constant (but non-zero). (C) At height h = R, value of V (r) Increases by a factor 11
8
13. Assertion If radius of the earth suddenly shrinks to half R
(D) At depth h = , value of V (s) Increases by a factor 2
its present without changing its mass value, then the 2
period of an earth’s satellite will not change. (t) None of the above
Reason Time period of a satellite does not depond upon
2. A particle is projected from the surface of the earth with
the mass of the earth. speed v. Suppose it travels a distance x when its speed
14. Assertion On two sides of a point mass, gravitational v v
becomes v to and y when speed changes from to 0.
field strength is same at same distance. 2 2
Reason As we move away from a point mass value of Similarly, the corresponding times are suppose t 1 and t 2 .
gravitational potential decreases. Then,
Column I Column II
15. Assertion Areal velocity of a planet around sun will x
(A) (p) =1
become two times if mass of planet is halved. y
L
Reason Areal velocity = , where L is angular (B) t1 (q) >1
2m t2
momentum of planet about centre of sun. (r) <1
16. Assertion Let W1 is the work done in taking away a 3. Density of a planet is two times the density of the earth.
satellite from the surface of the earth to its orbit and then Radius of this planet is half. Match the following
W2 the work done in rotating the satellite in circular orbit (as compared to the earth) columns.
there. Then, Column I Column II
W1 = W2 (A) Acceleration due to gravity on this (p) Half
GMm planet’s surface
Reason W1 = W2 = (B) Gravitational potential on the surface (q) Same
2r
(C) Gravitational potential at centre (r) Two times
17. Assertion Plane of a satellite always passes through the
(D) Gravitational field strength at centre (s) Four times
centre of the earth.
Reason Gravitational force on satellite is always 4. In elliptical orbit of a planet, as the planet moves from
towards centre of the earth. apogee position to perigee position, match the following
18. Assertion On earth’s satellite, we feel weightlessness. columns.
Moon is also satellite of the earth. But on the surface of Column I Column II
moon we do not feel weightlessness. (A) Speed of planet (p) Remains same
Reason Gravitational force by the earth on us on the (B) Distance of planet from centre of sun (q) Decreases
surface of moon is zero. But, gravitational force by moon (C) Potential energy (r) Increases
on us on its surface is non-zero. (D) Angular momentum about centre of sun (s) Cannot say
440 Objective Physics Vol. 1

5. Match the following columns. 9. Match the following columns.


Column I Column II
Column I Column II
(A) Time period of an earth (p) Independent of mass of
(A) Kepler’s first law (p) T 2 ∝ r 3 satellite in circular orbit satellite
(B) Kepler’s second law (q) Areal velocity is constant (B) Orbital velocity of satellite (q) Independent of radius of
orbit
(C) Kepler’s third law (r) Orbit of planet is elliptical (C) Mechanical energy of (r) Independent of mass of the
satellite earth
(s) None of the above
6. Let V and E denote the gravitational potential and
gravitational field at a point. Then, match the following
10. If the earth decreases its rotational speed. Match the
columns.
following columns.
Column I Column II
Column I Column II
(A) E = 0, V = 0 (p) At centre of spherical shell (A) Value of g at pole (p) Will remain same
(B) E ≠ 0, V = 0 (q) At centre of solid sphere (B) Value of g at equator (q) Will increase
(C) V ≠ 0, E = 0 (r) At centre of circular ring (C) Distance of geostationary (r) Will decrease
satellite
(D) V ≠ 0, E ≠ 0 (s) At centre of two point (D) Energy of geostationary satellite (s) Cannot say
masses of equal magnitude

11. Match the following columns. (for a satellite in circular


7. Two concentric spherical shells are as shown in figure. orbit)
Match the following columns.
Column I Column II
(A) Kinetic energy (p) GMm
B −
2r
A
(B) Potential energy (q) GM
D
r
(C) Total energy (r) GMm

C r
(D) Orbital energy (s) GMm
2r

Entrance Gallery
Column I Column II 2014
1
(A) Potential at A (p) greater than B 1. A planet of radius R = × (radius of the earth) has the
(B) Gravitational field at A (q) less than B 10
same mass density as the earth. Scientist dig a well of
(C) As one moves from C to D (r) Potential remains constant R
depth on it and lower and wire of the same length and
(D) As one moves from D to A (s) Gravitational field decreases 5
(t) None of the above of linear mass density 10−3 kgm −1 into it. If the wire is
not touching anywhere, the force applied at the top of the
8. Match the following columns. wire by a person holding it in place is (take the radius of
earth = 6 × 106 m and the acceleration due to gravity on
the earth is 10 ms −2 ).
Column I Column II
[JEE Advanced]
(A) Kinetic energy of a particle (p) Work done by gravitational (a) 96 N (b) 108 N (c) 120 N (d) 150 N
in gravitational field is force should be positive
increasing 2. Four particles, each of mass M and equidistant from each
(B) Potential energy of a particle (q) Work done by external force other, move along a circle of radius R under the action of
in gravitational field is should be non-zero their mutual gravitational attraction. The speed of each
increasing particle is [JEE Main]
(C) Mechanical energy of a (r) Work done by gravitational GM GM
particle in gravitational field force should be negative
(a) (b) 2 2
R R
is increasing
GM 1 GM
(c) (1 + 2 2 ) (d) (1 + 2 2 )
(s) Cannot say anything R 2 R
Gravitation 441

3. If a body of mass mhas to be taken from the surface to the 2012


earth to a height h = R , then the amount of energy 10. Two spherical planets P and Q have the same uniform
required is (R = radius of the earth) [Kerala CEE] density ρ, masses M P and M Q and surface areas A and
mgR mgR mgR 4 A, respectively. A spherical planet R also has uniform
(a) mgR (b) (c) (d)
3 2 12 density ρ and its mass is ( M P + M Q ). The escape
mgR velocities from the planets P , Q and R, are v P , v Q and v R ,
(e)
9 respectively. Then, [JEE Main]
4. The total energy of an artificial satellite of mass m (a) vQ > vR > vP (b) vR > vQ > vP
revolving in a circular orbit around the earth with a speed 1
(c) vR / vP = 3 (d) vP / vQ =
v is [Kerala CEE] 2
1 1 1
(a) mv 2 (b) mv 2 (c) − mv 2 (d) − mv 2 2011
2 4 4
1 11. A satellite is moving with a constant speed v in a circular
(e) − mv 2
2 orbit about the earth. An object of mass m is ejected from
5. What is a period of revolution of the earth satellite? the satellite such that it just escapes from the gravitational
Ignore the height of satellite above the surface of the pull of the earth. At the time of its ejection, the kinetic
earth. energy of the object is [IIT JEE]
Given, the value of gravitational acceleration 1 3
(a) mv 2 (b) mv 2 (c) mv 2 (d) 2mv 2
g = 10 ms −2 , radius of the earth R e = 6400 km . 2 2

(Take, π = 3.14) [Karnataka CET] 12. Two particles of equal mass mgo around a circle of radius
(a) 85 min (b) 156 min (c) 83.73 min (d) 90 min R under the action of their mutual gravitational attraction.
The speed of each particle with respect to their centre of
6. What happens to the acceleration due to gravity with the
mass is [AIEEE]
increase in altitude from the surface of the earth?
Gm Gm
(a) Increases [J&K CET] (a) (b)
R 4R
(b) Decreases
GM Gm
(c) First decreases and then increases (c) (d)
(d) Remains same 3R 2R

7. A body weighs 45 N on the surface of the earth. What is 13. Two bodies of masses m and 4m are placed at a distance r.
the gravitational force on it due to the earth at a height The gravitational potential at a point on the line joining
equal to half of the radius of the earth? [J&K CET] them where the gravitational field is zero, is [AIEEE]
4Gm 6Gm
(a) 20 N (b) 45 N (a) − (b) −
(c) 40 N (d) 90 N r r
9Gm
(c) − (d) zero
2013 r
8. Two bodies each of mass M are kept fixed with a 14. A body is projected with a velocity of 2 × 11.2 km/s from
separation 2L. A particle of mass m is projected from the the surface of earth. The velocity of the body when it
mid-point of the line joining their centres perpendicular escapes the gravitational pull of the earth is [Kerala CEE]
to the line. The gravitational constant is G. The correct (a) 3 × 11.2 km/s (b) 11.2 km/s
statement(s) is (are) [JEE Advanced]
(c) 2 × 11.2 km/s (d) 0.5 × 11.2 km/s
(a) The minimum initial velocity of the mass m to escape the
GM (e) 2 × 11.2 km/s
gravitational field of the two bodies is 4
L 15. A satellite is launched into a circular orbit of radius R
(b) The minimum initial velocity of the mass m to escape the around of the earth. A second satellite is launched into an
GM orbit of radius 4R. The ratio of their respective periods is
gravitational field of the two bodies is 2
L [Kerala CEE]
(c) The minimum initial velocity of the mass m to escape the (a) 4 : 1 (b) 1 : 8 (c) 8 : 1 (d) 1 : 4
2GM (e) 1 : 2
gravitational field of the two bodies is 1
L 16. If the earth were to suddenly contract to th of its present
(d) The energy of the mass m remains constant n
radius without any change in its mass, the duration of the
9. What is the minimum energy required to launch a satellite new day will be nearly [WB JEE]
of mass mfrom the surface of a planet of mass M and R in 24
(a) h (b) 24n h
a circular orbit an altitude of 2R? [JEE Main] n
5 GmM 2 GmM GmM GmM 24
(a) (b) (c) (d) (c) 2 h (d) 24 n2 h
6R 3R 2R 3R n
442 Objective Physics Vol. 1

17. If g is the acceleration due to gravity on the surface of the 2


21. The ratio of radii of the earth to another planet is and
earth, the gain in potential energy of an object of mass m 3
raised from the earth’s surface to a height equal to the 4
the ratio of their mean densities is . If an astronaut can
radius R of the earth is [WB JEE] 5
mgR mgR jump to a maximum height of 1.5 m on the earth, with the
(a) (b)
4 2 same effort, the maximum height he can jump on the
(c) mgR (d) 2mgR planet is [Kerala CEE]
(a) 1 m (b) 0.8 m (c) 0.5 m (d) 1.25 m
2010 (e) 2 m
18. A thin uniform angular disc (see figure) of mass M has 22. At what depth below the surface of the earth, the value of
outer radius 4R and inner radius 3R. The work required to g is the same as that at a height of 5 km? [Kerala CEE]
take a unit mass from point P on its axis to infinity is (a) 1.25 km (b) 2.5 km (c) 5 km (d) 7.5 km
(e) 10 km
P 23. A body is at rest on the surface of the earth. Which of the
4R
following statement is correct? [OJEE]
(a) No force is acting on the body
3R (b) Only weight of the body acts on it
2R (c) Net downward force is equal to the net upward force
(d) None of the above statement is correct
[IIT JEE] 24. The density of the earth in terms of acceleration due to
2GM gravity ( g ) , radius of the earth ( R ) and universal
(a) (4 2 − 5)
7R gravitational constant (G ) is [OJEE]
2GM 4 πRG 3πRG
(b) − (4 2 − 5) (a) (b)
7R 3g 4g
GM 4g 3g
(c) (c) (d)
4R 3πRG 4 πRG
2GM
(d) ( 2 − 1) 25. A body is taken to a height of nR from the surface of the
5R
earth. The ratio of the acceleration due to gravity on the
19. Gravitational acceleration on the surface of a planet is surface to that at the altitude is [MHT CET]
−2 −1
6
g, where g is the gravitational acceleration on the (a) (n + 1) 2
(b) (n + 1) (c) (n + 1) (d) (n + 1)
11
26. If the density of the earth is doubled keeping radius
surface of the earth. The average mass density of the
2 constant, find the new acceleration due to gravity?
planet is times that of the earth. If the escape speed on (Take, g = 9.8 m /s 2 ) [MHT CET]
3
−1 (a) 9.8 m/s2 (b) 19.6 m/s2
the surface of the earth is taken on be 11 kms , the
(c) 4.9 m/s2 (d) 39.2 m/s2
escape speed on the surface of the planet in kms −1 will be
27. If ρ is the density of the planet, the time period of near by
[IIT JEE] satellite is given by [MHT CET]
(a) 5 (b) 7 4π 4π
(c) 3 (d) 11 (a) (b)
3Gρ Gρ
20. The height at which the acceleration due to gravity 3π π
g (c) (d)
becomes (where, g = the acceleration due to gravity on Gρ Gρ
9
the surface of the earth) in terms of R, the radius of the 28. In a satellite, if the time of revolution is T, then KE is
earth, is [AIEEE] proportional to [MHT CET]
R 1 1
(a) 2R (b) (a) (b)
3 T T2
1
(c)
R
(d) 2R (c) 3 (d) T −2/ 3
2 T
Answers
Level 1
Objective Problems

1. (c) 2. (c) 3. (a) 4. (b) 5. (c) 6. (a) 7. (c) 8. (a) 9. (a) 10. (b)
11. (a) 12. (c) 13. (a) 14. (b) 15. (b) 16. (c) 17. (b) 18. (c) 19. (b) 20. (b)
21. (b) 22. (c) 23. (a) 24. (b) 25. (a) 26. (b) 27. (d) 28. (c) 29. (d) 30. (d)
31. (b) 32. (c) 33. (a) 34. (c) 35. (a) 36. (c) 37. (c) 38. (d) 39. (c) 40. (b)
41. (c) 42. (a) 43. (a) 44. (c) 45. (c) 46. (a) 47. (a) 48. (c) 49. (d) 50. (a)
51. (d) 52. (d) 53. (c) 54. (c) 55. (c) 56. (b) 57. (c) 58. (b) 59. (a) 60. (c)
61. (c) 62. (d) 63. (c) 64. (b) 65. (c) 66. (d) 67. (b) 68. (d) 69. (c) 70. (c)
71. (a) 72. (a) 73. (a) 74. (a) 75. (d) 76. (d) 77. (d) 78. (c) 79. (b) 80. (b)
81. (d) 82. (d) 83. (b) 84. (c) 85. (c) 86. (a) 87. (d) 88. (a) 89. (b) 90. (d)
91. (a) 92. (b) 93. (a,a) 94. (d) 95. (b) 96. (b) 97. (c) 98. (d) 99. (b) 100. (d)
101. (c) 102. (b) 103. (a) 104. (c) 105. (a) 106. (a) 107. (a)

Level 2
Only One Correct Option
1. (a) 2. (a) 3. (c) 4. (c) 5. (b) 6. (c) 7. (b) 8. (a) 9. (b) 10. (a)
11. (b) 12. (c) 13. (c) 14. (a) 15. (a) 16. (d) 17. (d) 18. (b) 19. (b) 20. (c)
21. (c) 22. (d) 23. (a,d) 24. (c) 25. (b) 26. (a) 27. (c,d) 28. (d) 29. (c) 30. (d)
31. (d) 32. (d) 33. (b) 34. (d) 35. (a) 36. (d) 37. (b) 38. (a) 39. (d) 40. (b)
41. (b) 42. (b) 43. (c)

More than One Correct Options


1. (a,b,d) 2. (a,b,d) 3. (a,c,d) 4. (a,d) 5. (a,b,c) 6. (a,c) 7. (b,c,d) 8. (c,d) 9. (all) 10. (a,b)
11. (a,c,d) 12. (a,c,d) 13. (c,d)

Assertion and Reason


1. (d) 2. (a) 3. (b) 4. (d) 5. (a,b) 6. (d) 7. (d) 8. (a) 9. (d) 10. (d)
11. (e) 12. (d) 13. (b) 14. (e) 15. (a) 16. (e) 17. (a) 18. (c) 19. (b) 20. (b)

Match the Columns


1. (A → q, B → q, C → s) 2. (A → r, B → r) 3. (A → q, B → p, C → p, D → q) 4. (A → r, B → q, C → q, D → p)
5. (A → r, B → q, C → p) 6. (A → t, B → t, C → p,q,r,s, D → t) 7. (A → q, B → t, C → r, D → s) 8. (A → s, B → r, C → q)
9. (A → p, B → p, C → s) 10. (A → p, B → q, C → q, D → q) 11. (A → s, B → r, C → p, D → q)

Entrance Gallery
1. (b) 2. (d) 3. (c) 4. (e) 5. (c) 6. (b) 7. (a) 8. (b) 9. (a) 10. (b)
11. (b) 12. (b) 13. (c) 14. (a) 15. (b) 16. (c) 17. (b) 18. (a) 19. (c) 20. (a)
21. (c) 22. (e) 23. (e) 24. (d) 25. (a) 26. (b) 27. (c) 28. (d)
Solutions
Level 1 : Objective Problems 12. F =
Gm1m2
or F ∝ m1m2 .
GM M r2
1. g = or g∝
R2 R2 On M due to masses m force is F and due to mass 2m force
2g is 2F. Therefore, the net force is 3F.
Mass is 2 times and diameter is 3 times. Hence, g ′ =
9 13. In a coal mine and at the top of a mountain value of g is less,
g hence apparent weight is less.
2. With height, g ′ = 2
… (i)
1 + h  14. Because acceleration due to gravity increases.
 
 R GM
15. g = 2 . If radius shrinks to half of its present value, then g
g R
Given g ′ = , substituting in Eq. (i) we get, h = R.
4 will become four times.
4
3. At poles value of g is maximum. There is no effect of 16. g = GπRρ
rotation of the earth. 3
g1 ρ1 R1 1 4 2
4. At depth, g ′ = g 1 −  or g 1 − 
h d ⇒ = = × =
 R  R g 2 ρ2 R2 2 1 1
g′ 1  d  n −1 17. g ∝ρR
= = 1 −  or d = R  
g n  R  n  4
18. g = πρGR
GM M 3
5. g = 2 or g ∝ 2
R R ⇒ g ∝ dR ( ρ = d given in the problem)
Gm1m2 19. Acceleration due to gravity at latitude φ is given by
6. F=
r2 g ′ = g − Rω2 cos2 φ
Gm1m2 . × 10−11 × 1 × 1
667 3 2
∴ r= = At 30° , g 30 ° = g − Rω2 cos2 30° = g − Rω
F . × 10−9
98 4
= 0082
. m = 8.2 cm ∴
3
g − g 30 ° = ω2 R
g 4
7. g ′ = 2
(at h = R )
1 + h  GM
20. Acceleration due to gravity, g =
  R2
 R
g g M
= ∴ =
4 G R2
72 Gm ( M − m)
∴ W′= = 18N 21. F =
4 r2
8. At equator, g ′ = g − Rω2 For maximum force,
dF
=0
0 = g − Rω2 dm
g 9.8 d  GmM Gm2 
∴ ω= = ⇒  − 2  =0
R 6400 × 103 dm  r 2 r 
= 1.23 × 10−3 rad /s ⇒ M − 2m = 0 ⇒
m 1
=
M 2
9. At centre of earth value of g is zero. Hence, weight is zero.
1
22. g ∝ r (if r < R) and g ∝ (if r < R)
10. g 1 −  = 1 − 
2h d r2
(at h << R )
 R  R g
23. g ′ =
2h d 2
or 1− =1 − 1 + h 
R R  
 R
∴ d = 2h g
Given, g′ =
11. 0 = g − Rω2 cos2 60° 64
4g h h
or ω2 = ∴ 1+ =8 or =7
R r R
g ∴ h = 7R = 44.8 × 106 m
or ω2 = 2
R
24. At equator g ′ = g − Rω2 . Matter will start escaping from
2π g
=2 equator when g ′ = 0
T R
g GM
R ∴ ω= =
∴ T=π R R3
g
Gravitation 445

25. g 1 −  =
d 1 g 3 GM Gm
34. At centre, V c = − , on surface V s = −
 R 2  h
2 2 R R
1 +  3
 R ∴ Vc = Vs
d 1 16 2
or 1− = 2
= 35. Due to three particles, net intensity at the centre
21 + 
R 1600 50
 I = I A + I B + IC = 0. Because, these three intensities are
 6400  equal in magnitude and the angle between each other is
d 34 120°.
=
R 50 A
d = (6.4 × 106 m) 
34

 50 
= 4.352 × 106 m IA

26. Potential is work done per unit mass. Hence, unit is J/kg. IB IC
27. Inside a shell, field strength is zero. Therefore, force on a 120º
particle is zero. B C
29. Enet = 0
−Gm 
2
r −GM − GM
Gm GM 36. V in =  3 −    ,V surface = ,V out =
∴ = , x is distance from m 2R   R R r
x 2 ( d − x )2 
x m 37. For hollow sphere,
∴ = −GM − GM − GM
d−x M V in = , V surface = ,V out =
R R r
m
x= ⋅d i.e. potential remains constant inside the sphere and it is
m+ M
equal to potential at the surface and increase when the
M point moves away from the surface of sphere.
and d−x= ⋅d
m+ M 38. Intensity will be zero inside the spherical shell.
V =−
Gm GM
− E = 0 up to r = R
x d−x and
1
E ∝ 2 , when r > R
Gm( m + m ) GM m + M r
=− − 39. Total mechanical energy of any closed system is always
m⋅ d M ⋅d
G negative.
=− ( m+ M) 2
40. v e = 2gR ,v e is independent of mass of projectile.
d
πR = L mgh
30. 41. ∆U = , h = R (given)
L 1 + h 
∴ R=  
 R
π
mgR
GM πGM ∴ ∆U =
V =− =− 2
R L
5 GM M
31. At distance 2.5 r or r 43. v e = 2gR = 2 2 ⋅ R or v e ∝
2 R R
m, 2r Mass is 1000 times and radius is 10 times. Therefore, escape
velocity will become 10 times.
m, r
44. Ui + K i = U f + K f
GMm 1 1
− + m( 2v e )2 = 0 + mv 2
R 2 2
GM 1 2
or − + 2v e = v
2
R 2
2GM 8GM
or − + = v2
G(m + m) 8Gm R R
E= =
= 3 
2
25r 2 6GM 2GM 
 5r  or v= 
  R  R 
2 
33. Reduces by 75%, means 25% is left. = 3( 2gR ) = 3v e
GM 1  GM  45. Escape velocity is independent of mass of projectile which
=  2 
r2 4 R  is projected.
or r = 2R 46. At v < v e , total energy is negative.
∴ h=r − R v = v e , total energy is zero and
v = v e , total energy is positive
= 2R − R = R
446 Objective Physics Vol. 1

47. Escape velocity is independent of angle at which particle is 62. Energy of satellite is given by
projected. GMm
48. Escape velocity is independent of mass of particle which is E=−
2r
projected.
m
49. v = 2gR or E∝
r
ve ge R 63. T ∝ r 3/ 2
∴ = × e
vm gm Rm
64. In elliptical orbit velocity, both in magnitude and direction
6 10 keep on changing.
= × = 60
1 1 GM
65. v =
50. Decrease in kinetic energy = increase in potential energy r
1 mgh 1
∴ mv 2 = , (given, h = R) or v∝
2 h
1+ r
R
v2 r
2gR GM = 1
∴ v =
2
= gR = v1 r2
2 R
GM r1
or v= ∴ v2 = ⋅v
R r2
51. Escape velocity is independent of the angle at which it is R 2
projected. = ⋅v = v
R 3
52. Change in potential energy in displacing a body from r1 to r2 R+
2
is given by GMm GMm GMm
1 1  66. K = ,U = − ,E = −
∆U = GMm  −  2r r 2r
 r1 r2  If r is decreased, K will increase but U and E will decrease.
= GMm   1

1  67. T ∝ r 3/ 2

 2R 3R  dA L
68. =
GMm dt 2m
=
6R 69. Potential energy is two times the mechanical energy.
1 1 U = 2E and K = − E
53. mv e2 = m × 2gR = mgR 70.
2 2
∴ U = 2E0 and K = − E0
2GM
54. v e = 2 πr 3/ 2 2π 3
R 71. T = or GM = ⋅r
GM T
i.e. escape velocity depends upon the mass and radius of
the planet. 2π

55. E = Energy of satellite – energy of satellite on surface of earth T
GMm  GMm  ∴ ωr 3/ 2 = GM
=− − −
2( 3R )  R  GM gR 2
or r3 = = 2
5 GMm ω2 ω
=
6 R 72. L = mvr or L ∝ vr
5
= mgR  as GM = gR 
  v∝
1
6  R 
r
56. In circular path, if necessary centripetal force disappears,
∴ L ∝ r 1/ 2
body moves in tangential path.
i.e. with increase in r , L will increase.
2 π(r )3/ 2
58. T = ,T is independent of m, the mass of satellite. GMm
GM 75. Energy of a satellite is given by E = −
2r
59. T ∝ r 3/ 2 For a satellite very close to the earth, r = R
3/ 2
T2  r2  ∴ E=−
GMm
∴ = 
T1  r1  2R
or T2 = T1 ( 4)3/ 2 = 8T1 = 8 days 76. r1 = R + R = 2R and r2 = 7R + R = 8R
GMm
1
60. T 2 ∝ r 3 ,r1 : r2 : r3 =
: 1:
3 Now, K =
2 2 2r
1 27 K 1 r2
∴ T1 : T2 : T3 = : 1 :
2 2 2
= 1 : 8 : 27 = =4
8 8 K 2 r1
61. d1v1 = d 2v 2 (from conservation of angular momentum at U=−
GMm U1 r2
, = =4
these two points) r U2 r1
dv GMm U r
∴ v2 = 1 1 and E=− or 1 = 2 = 4
d2 2r U2 r1
Gravitation 447

77. T ∝ r 3/ 2 89. On surface of earth, U = −


GMm
3/ 2 R
T1  10  13
∴ =  = 10 10 At height, h(<< R ), increase in potential energy is mgh
T2  1012 
GMm
∴ Uh = − + mgh
GM
79. v = R
R l 1
vA RB 90. T = 2 π ∝
R 1
⇒ = = = g g
vB RA 4R 2
T2 g1 g
vA 3 v 1 ∴ = = =2 (at h = R)
∴ = = T1 g2 g
vB vB 2 2
1 + h 
∴ vB = 6 v  
 R
81. Total mechanical energy of satellite
91. Decrease in kinetic energy = increase in PE
−GMm
E= 1  ve 
2
mgh
2r ∴ m  =
EA mA rB 2  2 1+
h
⇒ = × R
EB mB rA
v e2 gh
3 4r 12 or =
⇒ × = 4 1+ h
1 r 1
R
82. T 2 ∝ R 3 2gR gh R h
T2 or = or =
∴ = constant 4 h 2 h
1+ 1+
R3 R R
83. Given that, T1 = 1 day and T2 = 8 days Solving this equation, we get h = R
3/ 2 Note Kinetic energy is half the value required to escape.
T2  r2 
∴ =  1
T1  r1  Therefore, speed is times the value required to escape.
2
2/ 3
r2  T2 
2/ 3 k
=  
8 92. F = (k = constant)
⇒ =  =4
r1  T1  1  r
mv 2 k
⇒ r2 = 4r1 = 4R ∴ = or v ∝ r 0
r r
 4 3
G  πR  ρ
GM 3  93. (i) Net force on the system is zero. Hence, velocity of centre
85. g = 2 = of mass at any instant will remain zero. As, initially they
R R2
g 3g were at rest.
∴ ρ= = (ii) Increase in kinetic energy of both particles = decrease in
R 4 πGR
G ⋅ 4π gravitational potential energy
3
Gmm Gmm Gm2
2  mv 2  = Ui − U f = −
1
86. Total mechanical energy is conserved, not the kinetic ∴ + =
energy. 2  d d d
87. Force will be zero at the point of zero intensity 2
Gm
d ∴ v=
d
GM G R2
P 94. g = or =
x R 2
g m
m1 m2
G m2
∴ will have the units .
m1 g kg
x= d
m1 + m2  1 −n 
mv 2 Gm1m2  
81M 9 95. = or v ∝ r  2 
= D= D r r n
81M + M 10 2πr
Now, T=
88. At equator, g ′ = g − R ω2 = 0 v
g r
∴ ω= or T∝
R v
r
2π g ∴ T∝
=  1 −n 
 
T R
r 2 
R 1+n
∴ T = 2π
g T ∝r 2
448 Objective Physics Vol. 1

96. Actually, gravitational force provides the centripetal force. mv 2


2. ∝ r −n
u sin 2θ
2
r
98. Range of projectile, R =
g ∴ v ∝ r (1 − n )/ 2
2πr
If u and θ are constant, then R ∝
1 T= or T ∝ rv −1
g v
Rm g e or T ∝ r ⋅ r (n − 1 )/ 2
=
Re gm or T ∝ r (n + 1 )/ 2
Rm 1 3. Field strength is uniform inside the cavity. Let us find at its
⇒ =
Re 0.2 centre.
R ET = EP + EC (T = Total , R = Remaining, C = Cavity )
⇒ Rm = e
0.2 ∴ ER = ET − EC
⇒ Rm = 5Re GM R GM
= 3 −0=
GM R 2 2R 2
99. v = = G1/ 2 M 1/ 2 R −1/ 2 GMm
R ∴ F = mFR =
dA vr sin θ 1 2R 2
101. = = |r × v | 1 1
dt 2 2 4. KE = mv e2 = m( 2gR ) = mgR
GMm 2 2
102. u = 2
r 1  ve  mgh
5. m  =
At distance 2r , E =
GM GM
= =
u 2  2 1+
h
( 2r )2 4r 2 4mr R
Now, W = mE ve 2 gh
or =
or mg =
u 8 1+ h
4r R
2gR gh
103. KE = 0, only PE is present. or =
8 h
E = U = 2  −
GMm  1+
∴  R
 r 
R
2GMm Solving, we get h =
=− 3
r
l
104. v e = 2v 0 = 1.414v 0 6. T = 2π
g
or orbital speed is to be increased by 41%.
1
Further, speed is to increase 2 times. Or kinetic energy is or T∝
to increase two times. g
A L 2mA T1 g2 2
105. = or L = = =
T 2m T T2 g1 1
106. ∆v = v e − v 0 = 2gR − gR g2
∴ =4
GM g1
= ( 2 − 1) gR = ( 2 − 1)
R v e = 2gR or v e ∝ gR
1 
107. mv = m −
2 GM 3GM  GMm
+ =
 R v e1 g1 R1
2 2R  2R = ⋅ =1 : 2
ve 2 g 2 R2
(increase in KE = decrease in PE)
1 R1 1
GM ∴ × =
or v= 4 R2 2
R
R1
Momentum, mv = m
GM ∴ =1
R R2

G  πR 3ρ
GM 4
∴ Impulse required = m Gm 3 
R Further, g= 2 =
R R2
Level 2 : Only One Correct Option or g ∝ Rρ
g1 R1 ρ1
1.
1
mv 2 =
mgh ∴ = ⋅
2 h g 2 R2 ρ2
1+
R  1  = (1) ρ1
or  
v2 R  4 ρ2
∴ h= =
v 2  2gR  − 1 ρ1 1
2g −  2  or =
R  v  ρ2 4
Gravitation 449

7. Potential at centre = –
4GM 12. W = EA − E∞
r
= (UA + K A ) − (U∞ + K ∞ )
M a M
= (1)V A + × (1)( 2)2  − (0 + 0)
1

r  2 
∴ V A = −5 J / kg
m
13. The initial position will become the perigee (nearest)
position .
M R
M 2π
−4GM T g π R
= 14. t = = =
a/ 2 4 4 2 g

4 2GM 2R
=− t′ =
a g
−4 2GMm t π
Potential energy = ∴ =
a t′ 2 2
4 2GMm 1 2
Binding energy = 15. µvr = Ui − U f
a 2
mM
1 4 2GMm Here, µ = reduced mass =
∴ mv e2 = M+m
2 a
1  mMvr2 
 = 0 −  −
GMm 
or ve =
8 2GM ∴  
2 m+ M   d 
a
8. Increase in kinetic energy = decrease in potential energy 2G( M + m)
∴ vr =
d
GM gR 2
O √3R P 16. v 0 = =
r R+h
R
GM
2R 17. L = mvr = m r = m GMr
r
1 ∴ L∝ r
or mv 2 = Ui − U f = m(V i − V f )
2  
 mgh 
or v = 2(V i − V f ) 18. E1 = ∆U =  
1 + h 
= 2 −
GM GM   R
+
 2R R 
E2 = Energy of satellite – energy of satellite on surface of
GM
= earth.
R GMm GMm
=− +
9. Just before collision total distance travelled by both 2( R + h) R
= 12R − R − 2R = 9R. Now, let smaller sphere travels a  
distance x. Then,  1 
= mgR 1 − 
m( x ) = 5m( 9R − x )  h 
 21 +  
∴ x = 7.5R   R  
GMm
10. Ei = E0 = −
mgR 
2h 
2r + 1
R 
GMm GMm =
or E f = − =−  h 
2( 3r ) 6r 2 1 + 
 R
∴ W = E f − Ej
2 1 + 
GMm 2 h
=− = E0 E1 mgh  R  = 2h
3r 3 ∴ = ×
E2 1 + h mgR R
11. W = U2 − U1
R
= mv 2 − mv1
19. Increase in kinetic energy = Decrease in potential energy
 Gm2 Gm1   Gm1 Gm2 
= m  − −  − − −   
 R 2R   R 2R  1 mgR mgR  mgh 
∴ mv 2 = =  ∆U = 
Gm(m1 − m2 ) R  h
= ( 2 − 1) 2 1+ 2 1+ 
2R R  R
450 Objective Physics Vol. 1

20. E = Energy of satellite – energy of satellite on surface of the 26. E = ∆U =


mgh mghR
=
h ( R + h)
earth 1+
GMm  GMm  R
=− − −
2( 3R )  R  E′ = energy of satellite
5 GMm – energy of satellite on surface of earth
=
6 R =−
GMm 
− −
GMm 

5  as GM = gR  2( R + h)  R 
= mgR  
6  R  1 1  mg( R + 2h)R
= mgR 2  −  = 2( R + h)
21. Acceleration of super dense material = g  R 2( R + h) 
but acceleration of earth = 2g (towards the mass) Now,
E
=
2h
∴ Relative acceleration = 3g 2m E ′ ( R + 2h)
1 F
Now, h = ar t 2 27. Acceleration between them is zero, where force between
2 m m
F 2m and is equal to the force between m and or the net
1 5 5
or h = ( 3g )t 2
2 m m
force on is zero. Suppose, this point is P. Now if velocity v 0
2h 5
or t= is less than value necessary to cross point P, it will retrace
3g
its path otherwise not.
GMm GMm
22. F1 = =
28. U2 = 2 mv 2 
1
( 2R )2 4R 2 2 
F2 = F1 − Fcavity
At ∞
G   (m)
M
GMm  8
= −
4R 2  3 R
2
 
2 
7GMm
=
36R 2
F2 7 –U1 –U2 PE = 0
∴ =
F1 9 t
⇒ v 02 =
4 2
23. g = πρGr
3 1
Now, mv 2 = − U2 + U1
∴ g ∝ r , if r < R 2
g=
GM mv e2
= U1 − U2 = − mv o2
r2 2
1
∴ g ∝ 2 , if r > R ∴ v = v e2 − 2v o2
r
F g
If r1 < R and r2 < R, then 1 = 1 = 1
r G ⋅ m ⋅ m mv 2
29. =
F2 g 2 r2 4R 2 ( R)
2
F1 g1  r2  v
If r1 > R and r2 > R, then = = 
F2 g 2  r1 
R R
24. Ui + K i = U f + K f
GMm 1 1
− + m ( 2v e )2 = 0 + mv 2 v
R 2 2
GM 1 2 1 Gm
or − + 2v e = v
2
∴ v=
r 2 2 R
2 GM 8 GM
or − + = v2 30. Side of square : r = 2R cos 45° = 2R
R R
6 GM F1
or v= F2
R

= 3 
2GM 
 = 3( 2gR )
 R 
R F1
= 3v e
45º
k r
25. F = (k = constant)
r
mv 2 k Mv 2
∴ = or v ∝r 0 Now, 2F1 cos 45° + F2 =
r r R
Gravitation 451

v=
R
( 2 F1 + F2 ) Now, T 2 ∝ a3
M r1 + r2 
3/ 2
⇒ T ∝ a 3/ 2 ∝  
R  GM . M GMM   2 
v= 2 +
M  ( 2R )2 ( 2R )2  ⇒ T ∝ (r1 + r2 )3/ 2
GM  2 2+1 1 mgh
=   38. mv 2 =
R  4  2 1+
h
R
R − h g
31. a = g ′ = g 1 −  = g 
h
 = ⋅y Putting h = R, we get
 R  R  R
GM
 GM  v = gR =
 2  R
 
= R ⋅y GM
R
GM
= 3  πR 3ρ ⋅ y = πρGy 39. 81 =
G 4 4
R 3  3 r2 (60R − r )2
π /2 Solving, we get
32. E = 2∫ dE sin θ (along Y - axis)
0 60R − r = 9r
G  ⋅ R ⋅ dθ
M ∴ r = 6R
π /2  l 
= 2∫ .sin θ 40. W = Energy of satellite – potential energy of satellite on
0 R2
2GM surface of earth.
= y
GMm GMm
lR =− +
Now, l = πR 2( 3R ) R
l 5 GMm 5
∴ R= = = mgR
π dE 6 R 6
Substituting, we get θ x a
41. r =
2 πGM 3
E= r
l2 3F = mr ω2 30º
GmdM a/2
3  2  = m .ω2
33. dF = dx Gmm a
x2 m  a  3
x=a + l
∴ F=∫ dF 3Gm
x=a dm
x ∴ ω=
a3
G ⋅ m  ⋅ dx 
M
42. Inside the shell force, will be zero (according to Gauss
a+l  l 
=∫ theorem). Then, force will increase and then decrease.
a x2 GM 1 gR GM
GMm 43. = 2gR = =
= r 2 2 2R
a( a + l )
∴ r = 2R
34. F = ∫ dF sin60° dm dm or h=r − R = R
(whole ring)
dF 60º
dF
=∫
G. M ( dm) 3

2a More than One Correct Options
( 2a 2 )
G  π R 3ρ
2 4
(whole ring)
GM 3 
1. g = 2 =
=
3GMm
(as ∫ dm = m) R R2
8a 2 (whole ring) or g ∝R (as ρ is same)
35. W = ∆U = U f − Ui = U f (as Ui = 0) v e = 2gR =
2GM
R
36. Change in potential energy in displacing a body from r1 to r2
2G  πR 3ρ
is given by 4
1 1  3 
=
∆U = GMm  −  R
 r1 r2 
So, v e or v ∝ R (as ρ is same).
= GMM 
1 1 
−  2. No solution is required.
 2R 3R 
3. K i + Ui = K f + U f
GMm
= GMm 1 3 GMm
6R ∴ 0− = mv 2 −
37. Semimajor axis 2R 2 2 R
r +r 2Gm
a= 1 2 or v=
2 R
452 Objective Physics Vol. 1

Gm GMm
4. E due to point mass is E = . 9. E1 = −
r2 R
As r → 0, E → ∞ U2 = −
GMm
(r = R + R = 2R )
So, just over the point masses, E = ∞. Hence, in moving from 2R
one point mass to other point mass, E first decreases and GMm GMm
K2 = and E2 = −
then increases. 4R 4R
V due to a point mass is GM
10. v = or v ∝ GM
Gm
V =− r
r 2 π 3/ 2
T= r
As r → 0, V → − ∞ GM
So, just over the point mass, V is − ∞. Hence, in moving from 1
one point mass to other point mass, V first increases and or T∝
GM
then decreases.
11. Given, G ′ =10 G
5. Inside a shell, V = constant and E = 0.
Consider the below diagram.
Between A and B, Enet = 0, V net = constant, because these
points inside both shells.
Between B and C
E of m ≠ 0, E of 2m = 0
V of m ≠ constant, V of 2m = constant. m Sun
Beyond C r
E and V due to both shells are neither zero nor constant. Earth Object
Gm
6. E = 2
r
4m 3m G ′ M e m 10GM e m
Force on the object due to the earth = =
4E R2 R2
Enet
3E [Q G ′ =10 G given]
2E 2E
= 10 
GM e m  Qg = GM e 

r  R2   R 2 
E 2E
= (10 g ) m = 10 mg ...(i)
m 2m
GM s′ m
Force on the object due to the sun, F =
E ′net = 2 2 E =
2 2 Gm r2
r2 G ( M s )m Q M ′ = M s (given )
=
′ = ( 9E )2 + E 2
Enet 10r 2  s
10 

= 10 E As r >> R (radius of the earth) ⇒ F will be very small.


So, the effect of the sun will be neglected.
10 Gm
= Now, as g ′ =10 g
r2
Hence, weight of person
′ > Enet
Enet
= mg ′ = 10 mg [from Eq. (i)]
4m 3m
i.e. gravity pull on the person will increase. Due to it,
4E Enet
3E walking on ground would become more difficult.
3E E Critical velocity, v c is proportional to g i.e.,
E vc ∝ g
r
2E As, g ′> g
2m m ⇒ vc ′ > vc
Potential in both cases is Hence, rain drops will fall much faster.
G To overcome the increased gravitational force of the earth,
V net = − (m + 2m + 3m + 4m) the aeroplanes will have to travel much faster.
r
10Gm 12. Due to huge amount of opposite charges on the sun and the
=−
r earth’s electrostatic force of attraction will be large.
7. At highest point velocity of particle 1will become zero. But Gravitational force is also attractive in nature have both
velocity of particle 2 is non-zero. forces will be added.
8. At maximum distance (at A) kinetic energy is minimum. Both the forces obey inverse square law and are central
But, angular momentum about centre of sun always forces. As both the forces are of same nature, hence all the
remains constant. three Kepler’s laws will be valid.
Gravitation 453

13. We know that gravitational force between the earth and the 18. Gravitational force by earth is utilized in providing the
sun. necessary centripetal force for rotating round the earth.
GMm That’s why this force in not felt to us. But gravitational force
FG = 2 , where M is mass of the sun and m is mass of the by moon is unutilized. That’s why it is felt.
r
earth.
Match the Columns
When G decreases with time, the gravitational force FG will GM GM
1. g = ,V = −
become weaker with time. As FG is changing with time. Due R2 R
to it, the earth will be going around the sun not strictly in At height h = R,
closed orbit and radius also increases, since the attraction
g g GM
force is getting weaker. g′ = = ⇒ V′=−
h 2 2R
Hence, after long time the earth will leave the solar system. 1+
R
1
Assertion and Reason i.e. decrease by a factor
2
and V ′ increase by a factor.
1. Gravitational potential due to a point mass at some finite R
At depth h= ,
distance is always negative. 2
g ′ = g 1 −  = g 1 −  =
4. Field strengths due to masses at 1 and 2 are acting at 60°. h 1 g
3Gm  R  2 2
Therefore, resultant is .
a2 GM
V ′ = − 3 [1.5R 2 − 0.5r 2 ]
6. Angular momentum is constant about the centre of sun. R
= − 3 1.5R 2 − R 2 
8. v e = 2gR =
2GM GM  1
R R  8 
M 11 GM
∴ ve ∝ =−
R 8 R
M M 1
( 4 πR 2 ) ⋅ δ = ( 4 πR 2 ) ∝ i.e. g ′ decrease by a factor and V ′ also decreases by a
4 R 2
πR 3
11
3 factor .
10. In case of cube, the formula cannot be applied directly. 8
Gm1m2 2. Near the surface of earth retardation of the particle will be
11. In the three cases, F = can be applied directly. more.
r2
G  πR 3  ρ
m1 m2 m1 4
m1 m2 m2 Gm 3 
3. g = 2 = or g ∝ρR
R R2
G  πR 3ρ
4
r r r Gm 3 
At centre, V =− =−
R R
12. In the region, 0 < r < r1 .
or V ∝ρR 2
F = 0 but v = constant + 0
2 π 3/ 2 At centre potential is 1.5 times potential at centre. At centre
13. T = r of a solid sphere field strength is zero.
GM
4. At perigee position, planet is nearest to sun.
Time period does not depend upon the radius of earth r but
GM
it depends upon the mass of earth M. 7. Inside a shell, V = − = constant
R
14. On two sides of a point mass, direction of gravitational field
and E = 0
strength will be different. Further, gravitational potential is
GM GM
negative. Hence, it increases in moving away from the Outside the shell, V = − and E = 2
r r
point mass.
dA L mvr sinθ vr sinθ As r increases, V increases and E decreases.
15. = = constant = =
dt 2m 2m 2 2 GM GMm
9. T = r 3/ 2 , v 0 = and E = −
dA GM r 2r
i.e. is independent of m.
dt 10. Due to rotation of earth,
2
1 1  GM  GMm g ′ = g − Rω2 cos2 φ
16. W2 = mv 2 = m   =
2 2  r  2r At pole there is no effect of rotation of earth.
mgh mgRh As φ = 90°
W1 = ∆U = =
h R+h At equator g ′ will increase, if ω decreases.
1+
R Further, T will increase with decrease in ω.
m  2  Rh
GM From Kepler’s third law, r should also increase.
 R 
= 
GMm  GMm
= h E=− , so with increase in r , E also increases.
r  rR  2r
454 Objective Physics Vol. 1

2GM e m + GM e m GM e m
Entrance Gallery =− =−
2R 2R
Re But, acceleration due to gravity ( g ) in terms of gravitational
1. Given, Rp = , ρp = ρe = ρ
10 constant ( G ) is
R GM
Length of the wire, l = p m g = 2e …(iii)
5 R
Density of the wire, ρw =10−3 kg /m3 GM m m
∴ − 2e × R2 × = g × R2 × [From Eq. (iii)]
R R 2R 2R
Mass of the wire, mp = p ×10−5 kg (Cancellation of negative sign, because energy can never be
5 gmR mgR
The force applied at the top of the wire by a person holding negative) = =
2 2
it in place = weight of the wire.
4. Total energy of the satellite is
The weight of the wire will act at the centre of the gravity of
1 GM e m
R R
the wire which is located at a distance of p = p from the E=−
5 × 2 10 2 Re
1 GM e m
surface of the planet. where, K =
2 Re
Now, calculating the effective value of acceleration due to
R
gravity g p′ at the depth d = for the planet. ∴ Total energy = − Kinetic energy
10 1
E = − mv 2
 d  g 2
g p′ = g p 1 − ⇒ gp = e (Qg × ρR)
  Putting the value of KE in the form of mass of a satellite m
 Rp 10
and speed v.
ge  Rp  9g e 5. Given, Re = 6400 km = 6.4 × 106 m
∴ gp = 1 −  ⇒ g p′ =
 10 × R 
10  p 100 π = 3.14, g = 10 m/s2
Weight of the wire = mg p′ We know that the period of revolution of the earth satellite
R 9 × ge ( Re + h)3
= p × 10−3 × T = 2π [if h << Re , then ( Re + h = Re )]
5 100 gRe2
6 ×105 × 10−3 9 × 10 60 × 9
= × = = 108 N Re3 Re 6.4 × 106
5 100 5 So, T = 2π 2
= 2π = 2 × 3.14
gRe g 10
2. Resultant force acting on mass M due to other three masses
= 2 × 314
. × 08 . × 103 = 5024
. × 103 = 5024 s
F F F F Mv 2
F= + +F ⇒ + +F= 5024
2 2 2 2 R and T = = 83.73 min
60
F 6. The acceleration due to gravity decreases with the increase
M M
in altitude from the surface of the earth. According to the
F
v given relation,
R
g
g′ = 2
F 1 + h 
O  
 R
7. Weight of the body at the surface of the earth = mg = 45 N
M M
Using g ′ =
g
⇒ mg ′ =
mg h = R 
2 2  2 
1 + h  1 + R 
   
2 × GM 2 GM 2 Mv 2  R  2R 
+ =
2 (R 2) 2
4R 2 R 45 4 × 45
w′ = 2
= (∴w = mg )
GM 2  1 1  1 + 1  9
+ = Mv 2  
R  4 2   2
w′ = 20 N
GM  2 + 4  1 GM
v=   = (1 + 2 2 ) 8.
R  4 2  2 R L C L
3. We know that gravitational potential energy, M m M
GM e m
U=− …(i) v
R
Let v is minimum velocity. From energy conservation,
and gravitational kinetic energy,
1 GM e m Uc + K c = U∞ + K ∞
K = ⋅ …(ii) 1
2 R ∴ mV c + mv 2 = 0 + 0
∴ Total energy of a body is 2
− 2GM 
v = − 2V c = ( − 2) 
GM e m GM e m GM
E =U + K = − + ∴  =2
R 2R  L  L
Gravitation 455

9. E = Energy of satellite − energy of mass on the surface of 14. KE =


1 1
mv 02 − m (11.2)2 [v 0 = 2 × 11.2 km/s]
planet 2 2
GMm  GMm  1 1
=− − −  = m( 2 × 11.2)2 − m (11.2)2
2r  R  2 2
1 1
Here, r = R + 2R = 3R mv = 3 × m × (11.2)
2 2
2 2
Substituting the value of r in above equation,
5 GMm v = 3 × 11.2 km/s
E=
6R 15. Given, R1 = r and R2 = 4r
10. Surface area of Q is four times. Therefore, radius of Q is two We have T ∝ R2 3

times. Volume is eight times. Therefore, mass of Q is also eight T12 ( r )3 T1 1


times. ∴ = or =
T22 ( 4r )3 T2 8
So, let M P = M and RP = r
Then, MQ = 8 M and RQ = 2r 16. From the conservation of angular momentum,
Now, mass of R is ( M P + MQ ) or 9 M. Therefore, radius of R I1ω1 = I 2ω2
is ( 9)1/ 3 r. Now, escape velocity from the surface of a planet is 2 2  2π  2 R2  2π  T 24
MR   = M⋅ 2   ⇒ T2 = 12 = 2
given by 5  T1  5 n  T2  n n
GM mgh mgh mgR
v= 2 (r = radius of that planet) 17. Potential energy, ∆U = = =
r h R 2
1+ 1+
2GM 2G(8M ) R R
∴ vP = ⇒ vQ = 18. W = ∆U = U f − Ui = U∞ − UP = − UP = − mV P = − V P (asm =1)
r ( 2r )
2G( 9M ) Potential at point P will be obtained by integration as given
vR = below.
( 9)1/ 3 r
Let dM be the mass of small ring as shown in figure.
v 1
From here, we can see that, P = and v R > vQ > v P . dM =
M
( 2 πr ) dr =
2Mr dr
vQ 2 π( 4R ) − π( 3R )
2 2
7R 2
11. In circular orbit of a satellite of potential energy P
1
= − 2 × (kinetic energy) = − 2 × mv 2 = − mv 2 √16R 2+r 2
4R
2
Just to escape from the gravitational pull, its total mechanical r
energy should be zero. Therefore, its kinetic energy should be dr
+mv 2 .
12. Gravitational force provides necessary centripetal force
G ⋅ dM
dV P = −
16R 2 + r 2
2GM 4R r
m R R m =−
7 R2 ∫3 R 16R 2 + r 2
⋅ dr

2GM
=− ( 4 2 − 5)
7R
2GM
∴ W =+ ( 4 2 − 5)
7R
Gm2 mv 2 Gm
= ⇒ v= 19. Acceleration due to gravity,
( 2R )2 R 4R
G  πR 3  ρ
4
13. Let gravitational field is zero at P as shown in figure. GM 3 
g= 2 =
m P 4m R R2
A B
or g ∝ρR
x r–x g
r or R∝
ρ
Gm G( 4m) Now escape velocity, v e = 2gR
∴ = ⇒ 4x 2 = (r − x )2
x 2 (r − x )2 or v e ∝ gR
⇒ 2x = r − x g g2
r or ve ∝ g × ∝
⇒ x= ρ ρ
3
∴ The gravitational potential 6 3
∴ (v e )planet = (9.8) ×
Gm G( 4m) 3Gm 6Gm 9Gm 121 2
∴ Vp = − − =− − =−
x r−x r r r = 3 kms−1
456 Objective Physics Vol. 1

GM 25. Acceleration due to gravity at a height above the earth


20. g ′ =
( R + h)2 surface
Acceleration due to gravity at height h, nR
2
g GM R2  R 
⇒ = 2 ⋅ =g 
9 R ( R + h)2  R + h
2
1  R  R 1 R
⇒ =  ⇒ =
9  R + h R+h 3
⇒ 3R = R + h ⇒ 2R = h
Re 2 de 4
21. Given, = and =
Rp 3 dp 5 2
 R 
As, MG = and M = d e × πRe3
gRe2
4 g′ = g  
3  R + h
4 2
∴ d e × πRe × G = g e …(i) g  R + h
= 
3 g′  R 
4
Similarly, for planet, d p × πRpG = g p …(ii) g  R + nR 
2
3 = 
Dividing Eq. (i) by Eq. (ii), we get g′  R 
g e Re d e ge 2 4 8 g
= × ⇒ = × = = 0.5 m = (1 + n)2
g p Rp d p g p 3 5 15 g′
4
22. Acceleration due to gravity at depth d below the surface of 26. Acceleration due to gravity, g = πρGR
the earth 3
g1 ρ1
g d = g 1 −  g ∝ρ, =
d or
 R g 2 ρ2
g1 ρ
Acceleration due to gravity at height h from the surface of the = [Q ρ2 = 2ρ]
earth g 2 2ρ

gh = g  g −  g 2 = g1 × 2 = 9.8 × 2 ⇒ g 2 =19.6 m/s2


2h
 R
27. Time period of near by satellite,
Given, gh = g d
r3 R3
2h d T = 2n = 2π
∴ = GM GM
R R
2 π( R 3 )1/ 2 3π
d = 2h =10 km (given, h = 5 km) = =
G 4 πR 3ρ
1/ 2

23. The net force on the body is zero. Weight of the body is
balanced by the reaction of the ground.  3 
24. Acceleration due to gravity, GM 1
28. Velocity of satellite, v = , KE ∝ v 2 ∝
GM G 4
g = 2 = 2 × πR 3ρ r r
R R 3 and T 2 ∝r 3,
3g
∴ ρ= So, KE ∝ T −2/ 3 .
4 πGR
11
Simple Harmonic
Motion
11.1 Introduction
A particle has oscillatory (vibrational) motion when it moves periodically about stable Chapter Snapshot
equilibrium position. The motion of a pendulum is oscillatory. A weight attached to a ● Introduction
stretched spring, once it is released, starts oscillating.
● The Causes of Oscillation
When the particle is moved away from the equilibrium position and released, a force ● Kinematics of Simple
comes into play to pull it back toward equilibrium. But by the time it gets there, it has Harmonic Motion
picked up some kinetic energy, so it overshoots, stopping somewhere on the other side and
● Force and Energy in
is again pulled back toward equilibrium.
Simple Harmonic Motion
Of all the oscillatory motions, the most important is called simple harmonic motion ● Relation between Simple
(SHM). In this type of oscillatory motion, displacement, velocity, acceleration and force all Harmonic Motion and
vary (w.r.t. time) in a way that can be described by either the sine or the cosine function Uniform Circular Motion
collectively called sinusoids. Any oscillatory motion that cannot be described, so simply is ● Method of Finding Time
called anharmonic oscillation. Period of a Simple
Besides being the simplest motion to describe and analyze, it constitutes a rather Harmonic Motion
accurate description of many oscillations found in nature. ● Vector Method of
Understanding periodic motion will be essential for our later study of waves, sound, Combining Two or More
alternating electric currents and light. Simple Harmonic
Motions in Same
Direction
11.2 The Causes of Oscillation
Consider a particle free to move on x-axis, is being acted upon by a force given by
F = − kxn
Here, k is a positive constant.
Now, following cases are possible depending on the value of n.
(i) If n is an even integer (0, 2, 4, … etc), force is always along negative x-axis, whether
x is positive or negative. Hence, the motion of the particle is not oscillatory. If the
particle is released from any position on the x-axis (except at x = 0) a force in
negative direction of x-axis acts on it and it moves rectilinearly along negative
x-axis.
458 Objective Physics Vol. 1

(ii) If n is an odd integer (1, 3, 5, … etc), force is along In SHM the acceleration is proportional and opposite to
negative x-axis for x > 0, along positive x-axis for x < 0 the displacement.
and zero for x = 0. Thus, the particle will oscillate In figures x, v and a as functions of time are illustrated.
about stable equilibrium position, x = 0. The force in x
this case is called the restoring force. Of these, if n =1,
i.e. F = – kx the motion is said to be SHM. +A

(a)
11.3 Kinematics of Simple t

Harmonic Motion –A
x = A sin ωt
A particle has simple harmonic motion along an axis OX
when its displacement x relative to the origin of the v
coordinate system is given as a function of time by the
relation, +Aω
x = A cos (ωt + φ )
The quantity (ωt + φ ) is called the phase angle or simply (b) t

the phase of the SHM and φ is the initial phase, i.e. the phase –Aω
at t = 0. Although, we have defined SHM in terms of a cosine v = dx = Aω cos ωt
dt
function, it may just as well be expressed in terms of a sine a
function. The only difference between the two forms is an
π
initial phase difference of . Since, the cosine (or sine)
2 +ω2A
function varies between a value of –1 and + 1, the (c) t
displacement of the particle varies between x = − A and
x = + A. The maximum displacement from the origin A, is the –ω2A
a = dv = – ω2A sin ωt
amplitude of the SHM. The cosine or sine function repeats dt
itself every time the angle ωt increases by 2π. Thus, the
Fig. 11.1 Graphs of (a) displacement, (b) velocity and (c)
displacement of the particle repeats itself after a time interval of acceleration vs time in SHM

. Therefore, SHM is periodic, and its period is
ω
2π 11.4 Force and Energy in
T=
ω Simple Harmonic Motion
The frequency ν of a SHM is equal to the number of In the above article, we found that the acceleration of a
complete oscillations per unit time. Thus, body in SHM is a = − ω 2 x. Applying the equation of motion
1 ω
ν= = F = m a, we have
T 2π
F = − mω 2 x = − kx
and is measured in hertz. The quantity ω, called the
angular frequency of the oscillating particle is related to the k
frequency by the relation similar to the equation for a where, ω=
m
circular motion, namely
Thus, in SHM the force is proportional and opposite to

ω= = 2πν the displacement.
T That is when the displacement is to the right (positive)
The velocity of the particle is the force points to the left and when the displacement is to
dx the left (negative) the force points to the right. Thus, the
v= = − ωA sin (ωt + φ )
dt force is always pointing toward the origin O. Such type of
which varies periodically between the values +ωA and force appears when an elastic body such as a spring is
− ω A. Similarly, the acceleration is given by deformed. That is why the constant k = mω 2 is sometimes
dv
a= = − ω 2 A cos (ωt + φ ) = − ω 2 x and therefore, varies called the elastic constant. Further, since
dt 2π k m 1 k
periodically between the values + ω 2 A and − ω 2 A. This ω= = ⇒ T = 2π and f =
T m k 2π m
expression also indicates that,
Simple Harmonic Motion 459

Kinetic Energy Figure shows the variation of total energy ( E ), potential


energy (U ) and kinetic energy ( K ) with displacement ( x ).
The kinetic energy of the particle is
E, U, K
1 1
K = mv 2 = m A 2 sin 2 (ωt + φ ) 1 2
2 2 1 U= kx
E= kA2 2
2
Since, sin θ = 1 − cos 2 θ
2

and using x = A cos (ωt + φ ) for the displacement, we


can also express the kinetic energy as 1
K= k(A2–x2)
1 2
K = mω 2 A 2 [1 − cos 2 (ωt + φ )]
2 x
which can be written as –A +A

1 1 Fig. 11.2
K = mω 2 ( A 2 − x 2 ) = k ( A 2 − x 2 )
2 2
From this expression we can see that, the kinetic energy Extra Knowledge Points
is maximum at the centre ( x = 0) and zero at the extremes of
oscillation ( x = ± A ). ■ In SHM, F = − kx or a = − ω 2 x , i.e. F-x graph or a-x
graph is a straight line passing through origin with
negative slope. The corresponding graphs are shown
Potential Energy below:
To obtain the potential energy we use the relation, F a
dU F = – ω2 x
F =− F = – kx
dx x x
dU
or = kx (as F = − kx)
dx
U x (a) (b)
∴ ∫ 0 dU = ∫0 kx dx Slope = – k Slope = – ω2

1 2 1 ■ Any function of t, say y = y (t ) oscillates simple


∴ U = kx = mω 2 x 2 d 2y
2 2 harmonically if, ∝ − y or we can say, if above
Thus, the potential energy has a minimum value at the dt 2
centre ( x = 0) and increases as the particle approaches either condition is satisfied, y will oscillate simple
harmonically.
extreme of the oscillation ( x = ± A ).
■ All sine and cosine functions of t are simple harmonic
in nature, i.e. for the function
Total Energy y = A sin (ωt ± φ ) or y = A cos (ωt ± φ )
Total energy can be obtained by adding potential and d 2y
kinetic energies. Therefore, is directly proportional to – y. Hence, they are
dt 2
E = K +U simple harmonic in nature.
1 1 Kinetic energy versus time equation can also be
= mω 2 ( A 2 − x 2 ) + mω 2 x 2

2 2 written as
1
K = mA 2ω 2 [1 − cos 2 (ωt + φ )]
1 4
= mω 2 A 2 This function is also periodic with angular frequency
2
2 ω. Thus, kinetic energy in SHM is also periodic with
1
or E = kA 2 (as mω 2 = k ) double the frequency, then that of x, v and a. But these
2 oscillations are not simple harmonic in nature,
Which is a constant quantity. This was to be expected d 2K
because is not proportional to − K. But,
since, the force is conservative. dt 2
1 1
Therefore, we may conclude that, during an oscillation, K − mA 2ω 2 = − mA 2ω 2 cos 2 (ωt + φ ) = K 0 (say)
4 4
there is a continuous exchange of kinetic and potential Here, K 0 is simply a cosine function of time. So, K 0
energies. While moving away from the equilibrium position, will oscillate simple harmonically with angular
the potential energy increases at the expense of the kinetic frequency 2 ω. Same is the case with potential energy
energy. When the particle moves towards the equilibrium function. U also oscillate with angular frequency 2 ω
position, the reverse happens. but the oscillations are not simple harmonic in nature.
460 Objective Physics Vol. 1

Total energy does not oscillate. U(J )


It is constant. Thus,
x → oscillates simple harmonically with angular
frequency ω.
v → oscillates simple harmonically with angular
frequency ω.
x (m)
a → oscillates simple harmonically with angular –2
frequency ω. (c)
K → oscillates with angular frequency 2 ω but not simple
harmonically. Thus, in Fig. (a), oscillations will take place about the
U → oscillates with angular frequency 2 ω but not mean position x = 0 and minimum potential energy at
simple harmonically. mean position is zero.
E → does not oscillate. In Fig. (b), mean position is at x = 2 m and the
■ In the above discussion, we have read that potential
minimum potential energy in this position is 5 J.
energy is zero at mean position and maximum at In Fig. (c), mean position is at x = − 2 m and the
extreme positions and kinetic energy is maximum at minimum potential energy in this position is again zero.
mean position and zero at extreme positions. But the ■ A function f (t ) is said to be periodic of time period T, if
correct statement is like this, f (t + T ) = f (t )
At mean position → K is maximum and U is minimum All sine or cosine functions of time are periodic. Thus,
(it may be zero also, but it is not necessarily zero).
Y = A sin ωt
At extreme positions → K is zero and U is maximum. 2π
U(J ) or A cos ωt is periodic, of time period T = .
U(J ) ω
How the different physical quantities (e.g.
5 displacement, velocity, acceleration, kinetic energy,
etc) vary with time or displacement are listed below in
x(m) x(m) tabular form.
2
(a) (b)

Table 11.1
S. No. Name of the equation Expression of the equation Remarks
1. Displacement- time x = A cos (ωt + φ) x varies between +A and –A

Velocity- time  v =  v = − Aω sin (ωt + φ) v varies between + Aω and − Aω


dx
2.
 dt 

Acceleration- time  a =
dv  a = − Aω 2 cos (ωt + φ) a varies between + Aω 2 and − Aω 2
3. 
 dt 
1 1 1
4. Kinetic energy- time  K = mv 2  K = mA 2ω 2 sin2 (ωt + φ) K varies between 0 and mA 2ω 2
 2  2 2
1 1 1
Potential energy-time  U = m ω 2 x 2  K = mω A cos (ωt + φ)
2 2 2
5. U varies between mA 2ω 2 and 0
 2  2 2
Total energy- time ( E = K + U ) 1 E is constant
6. E= m ω2 A2
2
7. Velocity- displacement v = ω A 2 − x2 v = 0 at x = ± A and at x = 0, v = ± Aω
2
8. Acceleration-displacement a= −ω x a = 0 at x = 0 and a = ± ω 2 A at x = + A
9. Kinetic energy- displacement 1 1
K = m ω 2( A 2 − x 2 ) K = 0 at x = ± A and K = m ω 2 A 2 at x = 0
2 2
10. Potential energy- displacement 1 1
U = m ω 2 x2 U = 0 at x = 0 andU = m ω 2 A 2 at x = ± A
2 2
11. Total energy-displacement 1 E is constant
E = m ω2 A2
2

From the above table we see that x, v and a are sine or Phase difference between x and a is π and between
cosine functions of time. So, they all oscillate simple π
any other two is .
harmonically with same angular frequency ω. 2
Simple Harmonic Motion 461

X Example 11.1 Find the period of the function, 11.5 Relation between
y = sin ωt + sin 2ωt + sin 3ωt
Sol. The given function can be written as
Simple Harmonic Motion
y = y1 + y2 + y3 and Uniform Circular

y1 = sin ωt , T1 =
Here,
ω Motion
2π π
y2 = sin 2ωt , T2 = = Consider a particle Q, moving on a circle of radius A
2ω ω
2π with constant angular velocity ω. The projection of Q on a
and y3 = sin 3 ωt ,] T3 =
3ω diameter BC is P. It is clear from the figure that as Q moves
∴ T1 = 2T2 and T1 = 3T3
around the circle of the projection P oscillates between B
2π and C. The angle that the radius OQ makes with the x-axis is
So, the time period of the given function is T1 or .
ω θ = ωt + φ. Here, φ is the angle made by the radius OQ with
2π the x-axis at time t = 0. Further,
/ Because in timeT = , first function completes one
ω Y Y v′
oscillation, the second function two oscillations and the third, θ
three. Q Q
A
X Example 11.2 A linear harmonic oscillator has a O θ P O θ v P
B x X X
total mechanical energy of 200 J. Potential energy of C
it at mean position is 50 J. Find
(i) the maximum kinetic energy,
(ii) the minimum potential energy, (a) (b)
(iii) the potential energy at extreme positions.
Y
Sol. At mean position, potential energy is minimum and kinetic
energy is maximum. Q
a′
Hence, Umin = 50 J (at mean position)
and Kmax = E − Umin O θ P
X
a
= 200 − 50 = 150 J (at mean position)
At extreme positions, kinetic energy is zero and potential
energy is maximum.
∴ Umax = E = 200 J (at extreme position) (c)
Fig. 11.3 Relation between SHM and uniform
X Example 11.3 The potential energy of a particle circular motion. (a) position, (b) velocity and
oscillating on x-axis is given as (c) acceleration
U = 20 + ( x − 2) 2 OP = OQ cos θ
Here, U is in joules and x in metres. Total mechanical or x = A cos (ωt + φ )
energy of the particle is 36 J.
In other words, P moves with SHM. That is
(a) State whether the motion of the particle is simple
harmonic or not. When a particle moves with uniform circular motion, its
(b) Find the mean position. projection on a diameter moves with SHM.
(c) Find the maximum kinetic energy of the particle. The velocity of Q is perpendicular to OQ and has a
magnitude of velocity v ′ = ωA. The component of v′ along
Sol. (a) F = − dU = − 2( x − 2 ) the x-axis is
dx
By assuming x − 2 = X, we have F = − 2 X v = − v ′ sin θ
Since, F ∝− X or v = − ω A sin (ωt + φ )
The motion of the particle is simple harmonic.
which is also the velocity of P. The acceleration of Q is
(b) The mean position of the particle is X = 0 or x − 2 = 0,
which gives x = 2 m centripetal and has a magnitude, a ′ = ω 2 A.
(c) Maximum kinetic energy of the particle is The component of a′ along the x-axis is
Kmax = E − Umin = 36 − 20
= 16 J
a = − a ′ cos θ or a = − ω 2 A cos (ωt + φ )
/ Umin is 20 J at mean position or at x = 2 m. Which again coincides with the acceleration of P.
462 Objective Physics Vol. 1

Note Points
11.6 Method of Finding Time
/ E usually consists of following terms
Period of a Simple (a) Gravitational PE,

Harmonic Motion (b) Elastic PE,


(c) Electrostatic PE,
There are basically following two methods of finding (d) Rotational KE and
time period of a SHM. These are the restoring force or (e) Translational KE.
torque method and the energy method. / For gravitational PE, choose the reference point (h = 0) at
mean position.
Restoring Force or Torque Method Now, let us take few examples of finding time period
The following steps are usually followed in this method: (T ) of certain simple harmonic motions.
Step 1 Find the stable equilibrium position which
usually is also known as the mean position. Net force or The Simple Pendulum
torque on the particle in this position is zero. Potential An example of SHM is the motion of a pendulum. A
energy is minimum. simple pendulum is defined as a particle of mass m
Step 2 Displace the particle from its mean position by suspended from a point O by a string of length l and of
a small displacement x (in case of a linear SHM) or θ (in case negligible mass.
of an angular SHM). O
Step 3 Find net force or torque in this displaced position.
Step 4 Show that this force or torque has a tendency to θ l
B′
bring the particle back to its mean position and magnitude of θ0 θ0
B
force or torque is proportional to displacement, i.e. A
F ∝−x or F = − kx …(i)
C
or τ ∝ −θ or τ = − kθ …(ii) FT θ
FN
This force or torque is also known as restoring force or mg
restoring torque.
Step 5 Find linear acceleration by dividing Eq. (i) by Fig. 11.4
mass m or angular acceleration by dividing Eq. (ii) by When the particle is pulled aside to position B, so that the
moment of inertia I. Hence, string makes an angle θ 0 with the vertical OC and then
k k
a = − x = − ω 2 x or α = − θ = − ω 2θ released, the pendulum will oscillate between B and the
m I symmetric position B ′ . The oscillatory motion is due to the
Step 6 Finally, tangential component FT of the weight mg of the particle.
 a
ω=   or α 
  This force FT is maximum at B and B ′, and zero at C. Thus,
 x θ we can write
FT = − mg sin θ
or
2π  a
=   or α 
 
Here, minus sign appears because it is opposite to the
T  x θ displacement.
 x θ x = CA
∴ T = 2π   or 2π  
 a α  ∴ maT = − mg sin θ …(i)
 d 2θ 
Energy Method Here, aT = lα  where, α = 2 
 dt 
Repeat step 1 and step 2 as in method 1. Find the total
and sin θ ≈ θ for small oscillations
mechanical energy ( E ) in the displaced position. Since,
∴ mlα = − mgθ
mechanical energy in SHM remains constant.
g θ l
dE or α = −   θ or   =
=0  l α g
dt
θ l
By differentiating the energy equation with respect to
dω ∴ T = 2π   or T = 2π
dx dθ dv α  g
time and substituting = v, = ω, = a and = α we
dt dt dt dt / Note that the period is independent of the mass of the
come to step 5. The remaining procedure is same. pendulum.
Simple Harmonic Motion 463

(iii) If a tunnel is dug along any chord of the earth and a


Energy Method particle is released from the surface of earth along this
Let us derive the same expression by energy method. tunnel, then motion of this particle is simple harmonic and
time period of this is also 84.6 min.
Suppose ω be the angular velocity of particle at angular
(iv) If length of day on earth becomes 84.6 min, one will feel
displacement θ about point O. Then, total mechanical energy
weightless on equator
of particle in position A is
4. Time period of a simple pendulum depends on
1
E = Iω 2 + mg ( h A − hC )  1 
2 acceleration due to gravity, g  as T ∝  , so take
1  g 
or E = ( ml 2 ) ω 2 + mgl(1 − cos θ )
2 l
| g eff | in T = 2π . Following two cases are possible:
dE g
E is constant therefore, =0
dt (i) If a simple pendulum is in a carriage which is
 dω   dθ  accelerating with acceleration a, then
or 0 = ml 2ω   + mgl sin θ  
 dt   dt  g eff = g − a
dθ dω
Putting = ω, = α and sin θ ≈ θ, we get the same e.g. if the acceleration a is upwards, then
dt dt | g eff | = g + a
expression, viz.
l
g and T = 2π
α=− θ g+a
 l
If the acceleration a is downwards, then ( g > a )
θ l
∴ T = 2π   or T = 2π l
α  g | g eff | = g − a and T = 2π
g−a
Following points should be remembered in case of a
simple pendulum. If the acceleration a is in horizontal direction,
1. For large amplitudes the approximation sin θ ≈ θ is then
not valid and the calculation of the period is more | g eff | = a 2 + g 2
complex.
In a freely falling lift, g eff = 0 and T = ∞, i.e. the
2. If the time period of a simple pendulum is 2 s, it is pendulum will not oscillate.
called seconds pendulum.
(ii) If in addition to gravity one additional force F,
3. If length of the pendulum is large, g no longer remain
(e.g. electrostatic force Fe ) is also acting on the
vertical but will be directed towards the centre of the
bob, then in that case,
earth and expression for time period is given by
F
1 g eff = g +
T = 2π m
 1 1
g + 
l R Here, m is the mass of the bob.
Here, R is the radius of earth. From this expression, we X Example 11.4 A simple pendulum of length l is
can see that, suspended from the ceiling of a cart which is sliding
1 1 l without friction on an inclined plane of inclination θ.
(a) if l << R , >> and T = 2π
l R g What will be the time period of the pendulum?

1 R Sol. Here, point of suspension has an acceleration. a = g sin θ


(b) as l → ∞, → 0 and T = 2π (down the plane). Further, g can be resolved into two
l g components g sin θ (along the plane) and g cos θ
and substituting the value of R and g, we get (perpendicular to plane).
T = 84.6 min.
θ θ θ
in
in gs os
Note Points =g
s
gc
a
/ In physics, this 84.6 min come in following four places: θ θ
(i) Time period of a satellite close to earth’s surface is
Fig. 11.5
84.6 min.
(ii) Time period of a pendulum of infinite length is 84.6 min. ∴ geff = g − a
464 Objective Physics Vol. 1

= g cos θ (perpendicular to plane)


l Energy Method
∴ T = 2π
|geff| The time period of the spring-block system can also be
l obtained by the energy method. Let v be the speed of the
= 2π
g cos θ mass in displaced position. Then, total mechanical energy of
I the spring-block system is
/ If θ = 0°,T = 2 π which is quiet obvious. E = kinetic energy of the block + elastic potential energy
g
1 1
or E = mv 2 + kx 2
X Example 11.5 A simple pendulum consists of a 2 2
small sphere of mass m suspended by a thread of Since, E is constant
length l. The sphere carries a positive charge q. The dE  dv   dx 
= 0 or 0 = mv   + kx  
pendulum is placed in a uniform electric field of dt  dt   dt 
strength E directed vertically upwards. With what
dx dv
period will pendulum oscillate, if the electrostatic Substituting, = v and =a
force acting on here is less than the gravitational dt dt
force? We have ma = − kx
 x m
Sol. The two forces acting on the bob are shown in figure ∴ T = 2π   = 2π
w − Fe  a k
geff in this case will be
m
mg − qE
Following points are important for a spring block
or geff = Fe = qE
m
system.
=g−
qE (i) Although, we have considered a horizontal
m system. But the same is true for vertical
∴ T = 2π
l system also.
geff m
l T = 2π
= 2π k
qE
g− w = mg
(ii) In case of a vertical spring-block system,
m m
Fig. 11.6 time period can also be written as
Spring-block System T = 2π
l Fig. 11.8

Suppose a mass m is attached to the free end of a g


massless spring of spring constant k, with its other end fixed Here, l = extension in the spring when the mass m is
to a rigid support. suspended from the spring.
m F
This can be seen as under
kl = mg (in equilibrium position)
m l m l
∴ = or T = 2π = 2π
x k g k g
Fig. 11.7
(iii) Equivalent force constant (k) If a spring pendulum
If the mass be displaced through a distance x, as shown a is constructed by using two springs and a mass, the
linear restoring force, following three situations are possible.
F = − kx …(i)
starts acting on the mass, tending to bring it back into its
original position. The negative sign simply indicates that it k1 k1
is directed oppositely to the displacement of the mass.
Eq. (i) can be written as k1 k2 m
ma = − kx …(ii) k2
k2
 
 x  m
or = m m
 a k
 x m
T = 2π   or T = 2π
(a) (b) (c)

 a k Fig. 11.9
Simple Harmonic Motion 465

Refer Fig. (a) X Example 11.6 A block with a mass of 3.00 kg is


In this case, suspended from an ideal spring having negligible
1 1 1 k1 k 2 mass and stretches the spring 0.2 m.
= + or k=
k k1 k 2 k1 + k 2 (a) What is the force constant of the spring?
m ( k1 + k 2 ) (b) What is the period of oscillation of the block, if it is
m
∴ T = 2π = 2π pulled down and released?
k k1 k 2
Sol. (a) In equilibrium,
Refer Fig. (b) and (c) kl = mg
In both the cases, mg
∴ k=
m m l
k = k1 + k 2 or T = 2π = 2π Substituting the proper values, we have
k k1 + k 2 (3.00)(9.8)
k= = 147 N/m
(iv) If spring has a mass ms and a mass m is suspended 0.2
l 0.2
from it, then time period is given by (b) T = 2π = 2π = 0.897 s
g 9.8
m
m+ s
3 X Example 11.7 A block with mass M attached to
T = 2π
k a horizontal spring with force constant k is moving
(v) If two masses m1 and m2 are connected by a spring and with simple harmonic motion having amplitude A1 . At
made to oscillate on horizontal surface, then time the instant when the block passes through its
period is given by equilibrium position a lump of putty with mass m is
m1 k m2 dropped vertically on the block from a very small
height and sticks to it.
(a) Find the new amplitude and period.
(b) Repeat part (a) for the case in which the putty is
Fig. 11.10 dropped on the block when it is at one end of its path.
µ Sol. (a) Before the lump of putty is dropped the total mechanical
T = 2π energy of the block and spring is
k 1
E1 = kA12
mm 2
Here, µ = reduced mass = 1 2
m1 + m2 Since, the block is at the equilibrium position, U = 0, and
the energy is purely kinetic. Let v1 be the speed of the
(vi) The force constant ( k ) of a spring is inversely block at the equilibrium position, we have
proportional to the length of the spring, i.e. 1 1
E1 = Mv12 = kA12
1 2 2
k∝ k
Length of spring ∴ v1 = A1
M
This can be visualized as under During the process momentum of the system in
A spring of length l and spring constant k can be horizontal direction is conserved. Let v 2 be the speed of
supposed to be made up by two springs in series of the combined mass, then
l (M + m) v 2 = Mv1
length and force constant 2k. In series, M
2 ∴ v2 = v1
M+ m
Now, let A2 be the amplitude afterwards. Then,
1
l E2 = kA22
, 2k 2
2
1
l, k ⇒ = (M + m)v 22
2
l Substituting the proper values, we have
, 2k
2 M
A2 = A1
M+ m

Fig. 11.11 / E 2 < E1, as some energy is lost into heating up the block and
putty.
(2 k ) (2 k ) M+ m
k eff = =k Further, T2 = 2 π
2k +2k k
466 Objective Physics Vol. 1

(b) When the putty drops on the block, the block is Sol. The time period of a compound pendulum is the minimum
instantaneously at rest. All the mechanical energy is when its length is equal to the radius of gyration about its
stored in the spring as potential energy. Again the centre of gravity, i.e. l = K.
momentum in horizontal direction is conserved during
the process. But, now it is zero just before and after putty Since, the moment of inertia of a disc about an axis
is dropped. So, in this case, adding the extra mass of the perpendicular to its plane and passing through its centre is
putty has no effect on the mechanical energy, i.e. equal to,
1 1 R
E2 = E1 = kA12 I = MK 2 = MR 2 ∴ K =
2 2 2
and the amplitude is still A1. Thus, Thus, the disc will oscillate with the minimum time period
when the distance of the axis of rotation from the centre is
A2 = A1 R
.
M+ m 2
and T2 = 2 π
k And the value of this minimum time period will be
2R
The Physical Pendulum Tmin = 2 π 2 = 2π 2R
g g
The physical pendulum is
O 1.414 R
just a rigid body, of whatever or Tmin ≈ 2 π
shape, capable of oscillating g
θ l
about a horizontal axis passing
through it. For small X Example 11.9 Find the period of small
oscillations, the motion of a oscillations of a uniform rod with length l, pivoted at
θ
physical pendulum is almost as mg sin θ mg cos θ one end.
mg x = +A
easy as for a simple pendulum. I0
Sol. T = 2 π
Figure shows a body of mg (OG )
irregular shape pivoted at O so, 1 2
Fig. 11.12 Here, I0 = ml
that it can oscillate without 3
friction about an axis passing through O. and OG =
l Mean position
In equilibrium the centre of gravity (G ) in directly below 2
O. In the position shown in figure, the body is displaced  1 ml 2 
 
3 
from equilibrium by an angle θ. The distance from O to the ∴ T = 2π

(m)(g )  
l x = –A
centre of gravity is l. The moment of inertia of the body 2 Fig. 11.13
about the axis of rotation through O is I and the total mass is
2l
m. In the displaced position, the weight mg causes a or T = 2π
3g
restoring torque,
τ = − ( mg )( l sin θ )
Oscillations of a Fluid Column
The negative sign shows that the restoring torque is
Initially, the level of liquid in both the columns is same.
clockwise when the displacement is counterclockwise and
The area of cross-section of the tube is uniform. If the liquid
vice-versa.
is depressed by x in one limb, it will rise by x along the
For small oscillations, length of the tube is the other limb. Here, the restoring force
sin θ ≈ θ and Στ = Iα is provided by the hydrostatic pressure difference.
∴ − ( mgl) θ = Iα
As α is proportional to − θ, the motion is simple x

harmonic, the time period of which is x

θ1 θ2
θ I
T = 2π   or T = 2π
α  mgl
Fig. 11.14

X Example 11.8 A uniform circular disc of radius


∴ F = − ( ∆p) A = − ( h1 + h2 ) ρgA
R oscillates in a vertical plane about a horizontal
axis. Find the distance of the axis of rotation from the = − ρgA (sin θ 1 + sin θ 2 ) x
centre for which the period is minimum. What is the Let, m be the mass of the liquid in the tube. Then,
value of this period? ma = − ρgA (sin θ 1 + sin θ 2 ) x
Simple Harmonic Motion 467

Since, F or a is proportional to − x, the motion of the Both the simple harmonic motions have same angular
liquid column is simple harmonic in nature, time period of frequency ω.
which is given by The resultant displacement of the particle is given by
x
T = 2π   x = x1 + x 2
a = A1 sin ωt + A2 sin (ωt + φ )
m = A sin (ωt + α )
or T = 2π
ρgA (sin θ 1 + sin θ 2 ) Here, A = A12 + A22 + 2 A1 A2 cos φ
/ For a U-tube, if the liquid is filled to a height l, θ1 = 90° = θ2 A2 sin φ
and m = 2 (lAρ) and tan α =
A1 + A2 cos φ
Thus, we can see that this is similar to the vector
addition. The same method of vector addition can be
applied to the combination of more than two simple
harmonic motions.
l
X Example 11.10 Find the displacement equation
of the simple harmonic motion obtained by combining
the motions.
Fig. 11.15
 π
x1 = 2 sin ωt, x 2 = 4 sin  ωt + 
So, T = 2π
l  6
g
 π 
Thus, we see that the expression T = 2 π l / g comes in and x 3 = 6 sin  ωt + 
picture at three places.
 3
(i) Time period of a simple pendulum for small oscillations. Sol. The resultant equation is
(ii) Time period of a spring-block system in vertical position. x = A sin (ωt + φ)
(iii) Time period of a liquid column in a U-tube filled to a
ΣA x = 2 + 4 cos 30° + 6 cos 60°
height ()
l . But, (I ) has different meanings at different
places. = 8.46
A3 = 6
y
11.7 Vector Method of
A2 = 4
Combining Two or More 30°
30°
Simple Harmonic Motions A1 = 2
x

in Same Direction
A simple harmonic motion is produced when a force
Fig. 11.17
(called restoring force) proportional to the displacement
acts on a particle. If a particle is acted upon by two such and ΣAy = 4 sin 30° + 6 cos 30°
forces, the resultant motion of the particle is a combination
= 7.2
of two simple harmonic motions. Suppose the two
individual motions are represented by ∴ A = (ΣA x )2 + (ΣAy )2

A2 = (8.46)2 + (7.2)2
A
= 11.25
ΣAy
and tan φ =
ΣA x
φ
α 7.2
A1 = = 0.85
8.46
Fig. 11.16 or φ = tan−1 (0.85) = 40.4°
Thus, the displacement equation of the combined motion is
x1 = A1 sin ωt
x = 11.25 sin (ωt + φ)
and x 2 = A2 sin (ωt + φ ) where, φ = 40.4°
468 Objective Physics Vol. 1

Extra Knowledge Points


Lissajous figure y

⇒ cos ωt = …(iv)
Suppose two forces act on a particle, the first alone b
would produce a simple harmonic motion in Squaring and adding Eqs. (iii) and (iv), we get
x-direction given by x2 y2
+ =1
x = a sin ωt a 2 b2
and the second would produce a simple harmonic
motion in y-direction given by Table 11.2
y = b sin (ωt + φ ) b Corresponding Lissajous
S. No. φ
The amplitudes a and b may be different and their a figure
phases differ by φ. The frequencies of the two simple y
harmonic motions are assumed to be equal. The
resultant motion of the particle is a combination of the Slope = 2
two simple harmonic motions. x
Depending on the value of φ and relation between a 1. 0° 2
and b, the particle follows different paths. Given
below are few special cases
■ Case 1 (When φ = 0 °) When the phase difference Straight line with slope 2
between two simple harmonic motions is 0°, i.e. y
x = a sin ωt Slope = 1
x
⇒ sin ωt = …(i) 45°
a x
2. 0° 1
y = b sin ωt
y
⇒ sin ωt = …(ii)
b Straight line with slope 1
From Eqs. (i) and (ii), we get
y
x y
=
a b
 b
or y =  x x
a 3. 90° 2
b
which is equation of a straight line with slope ⋅ Thus,
a
the path of the particle is a straight line. As a special Ellipse
case y = x , if a = b or slope is 1. y
 π π
■ Case 2
 When φ =  When the phase difference is ,
 2  2
i.e. x = a sin ωt x
4. 90° 1
x
⇒ sin ωt = …(iii)
a
 π
y = b sin ωt +  = b cos ωt Circle
 2
Chapter Summary with Formulae
■ Different Equations in SHM (vii) If x = A sin ωt, then
(i) F = − kx dx
v= = ωA cos ωt
dt
= −   x = − ω2 x
F k
(ii) a = dv
m  m and a= = − ω2 A sin ωt
dt
F
From these three equations, we can see that x -t, v-t
and a-t all three functions oscillate simple
harmonically with same angular frequency ω. Here, x
+A oscillates between + A and − A, v between + ωA and
x − ωA and a between + ω2 A and − ω2 A.
–A
. Phase difference between x -t and v-t functions or
π
between v-t and a-t functions is . But, phase
2
difference between x -t and a-t function is π.
Slope = − k
k (viii) v = ω A2 − x2
(iii) ω = = angular frequency of SHM
m v = 0 at x = ± A or at extreme positions
a v = ± ω A, at x = 0 or at mean position.
|v|max = ωA at mean position and|v|min = 0, at extreme
positions.
+A (ix) a = − ω2 x
x
–A |a|min = 0 at x = 0, at mean position.
|a|max = ω2 A at x = ± A, at extreme positions.
1 2 1
(x) PE = U0 + kx , KE = k ( A2 − x2 )
2 2
1
Slope = − ω2 and TE = PE + KE = U0 + kA2 .
2
d2 x Here, U0 is minimum potential energy at mean position
(iv) = − ω2 x
dt2 1 1
and kA2 or mω2 A2 is called energy of oscillation.
(v) F ∝ − x or a ∝ − x is the sufficient and necessary 2 2
condition for a periodic motion to be simple harmonic. This much work is done on the system when displaced
(vi) General solution of differential equation from mean position to extreme position. This much
d2 x energy keeps on oscillating between potential and
= − ω2 x is
dt2 kinetic during oscillation.
x = A sin (ωt ± φ) ■ At Mean Position
or x = A cos (ωt ± φ) F = 0, a = 0 potential energy is minimum (this minimum
Here, x is displacement from mean position (not value may be zero also) and speed or kinetic energy is
x-coordinate), A is amplitude of oscillation, φ is phase maximum.
angle at t = 0 (also called initial phase) and (ωt ± φ) is ■ At Extreme Positions
phase angle at a general time t, also called Speed or kinetic energy is zero. Force, acceleration and
instantaneous phase. potential energy is maximum.

Table 11.3
S. No. Physical quantity At mean position At extreme position At general point
Speed ωA
1. zero ω A 2 − x2
2
2. Acceleration zero ±ω A − ω2 x
3. Force zero ± kA − kx
Kinetic energy 1 2 1 zero 1
4. kA = mω 2 A 2 k ( A 2 − x2 )
2 2 2
Potential energy U0 1 1
5. U0 + kA 2 U 0 + kx 2
2 2
Total mechanical energy 1 1 1
6. U0 + k A2 U 0 + k A2 U 0 + k A2
2 2 2
U0 or minimum potential energy at mean position may ■ Spring Block System
be zero also.
k 2π m
Energy (i) ω = ,T = = 2π ,
m ω k
E
1 1 k
PE f = =
T 2π m
KE
x
–A +A m
k
■ Potential energy versus x or kinetic energy versus x graph (ii)
is parabola. While total energy versus x graph is a straight or
line, as it remains constant. Smooth

x=–A x=0 x=+A


t=0
k
x = A sinωt
th
m oo
Sm
x=–A x=0 x=+A
θ
t=0 or
x = − A sinωt
m
In all three cases, T = 2 π
k
x=–A x=0 x=+A
t=0
x = A cosωt k1

(iii) k1 k2 or
x=–A x=0 x=+A
t=0
x = − A cosωt
k2
■ If x = A sin ω t. Then,
v = ωAcos ωt and a = − ω2 A sin ω t

x In both cases, k e = k1 + k2
k1k2
+A (iv) k e =
T/4 T/ 2 k1 + k2
3T/4 T
t 1 1 1
= +
k e k1 k2

T=
–A ω

v k1
+ωA

T/2 T k2
T/4 t
3T/4

– ωA

a
■ Pendulum
(i) Only small oscillations of a pendulum are simple
harmonic in nature. Time period of which is given by
+ω2A T l
t T = 2π .
T/ 2 3T/4 g
T/4
(ii) Second’s pendulum is one whose time period is 2 s and
–ω2A length is 1 m.
Additional Examples
Example 1. In an SHM, can velocity and Example 9. A girl is swinging in the sitting position.
displacement be in the same direction? How will the period of the swing change if she stands
Sol. Yes. When a particle moves from mean position to extreme up?
position, velocity and displacement are in the same direction. Sol. The girl and the swing together constitute a pendulum of
time period,
Example 2. Can velocity and acceleration be in the l
T = 2π
same direction in an SHM? g
Sol. Yes. When a particle moves from extreme to mean position, As the girl stands up, her centre of gravity is raised. The
velocity and acceleration are in the same direction. distance between the point of suspension and the centre
of gravity, decreases, i.e. length l decreases. Hence, the
Example 3. Can displacement and acceleration be time period T decreases.
in the same direction in an SHM? Example 10. The length of a second's pendulum on
Sol. No. In an SHM, acceleration is always opposite to the surface of earth is 1 m. What will be the length of a
displacement. second's pendulum on the surface of moon?
l
Example 4. What is the most important Sol. T = 2π . In both the cases, T is same and T ∝ l / g
characteristic of a simple harmonic motion? g
Sol. The most important characteristic of SHM is that On the moon, the value of acceleration due to gravity is
acceleration is directly proportional to the displacement and is one-sixth of that on the surface of earth. So, the length of
1
directed towards the mean position. Mathematically, second's pendulum is m.
6
a = − ω 2 x.
Example 11. The bob of simple pendulum is made of
Example 5. At what points, the energy of the simple wood. What will be the effect on the time period if the
harmonic oscillator is entirely potential? wooden bob is replaced by an identical bob of iron?
Sol. At the extreme positions, energy of simple harmonic Sol. There will be no effect because the time period does not
oscillator is entirely potential. At these positions, the velocity of depend upon the nature of material of the bob.
the oscillator is zero and as such it has no kinetic energy.
Example 12. A spring having a force constant k is
Example 6. Can we use a pendulum watch in an
divided into three equal parts. What would be the force
artificial satellite?
constant for each individual part?
Sol. Inside an artificial satellite, a body is in a state of
F
weightlessness and the effective value of g is zero. The time Sol. Force constant of the spring k = , where F is the restoring
period of the simple pendulum is given by T = 2π l / g. As x
force. When the spring is divided into three parts, the
g = 0, T = ∞. Thus, inside a satellite, the pendulum does not
displacement for the same force reduces to x/3, therefore, the
oscillate at all. Hence, we cannot use a pendulum watch in an
artificial satellite. force constant for each individual part is
F  F
k′ = = 3   = 3k. Therefore, we can say that
Example 7. Will a pendulum clock gain or lose time x /3  x
when taken to the top of a mountain? 1
Force constant of a spring ∝
Sol. On the top of a mountain, the value of g is less than that on Length of spring
the surface of Earth. With a decrease in the value of g , time
period of the simple pendulum increases at the top of the Example 13. There are two springs, one delicate
mountain and accordingly the pendulum loses time. and another stiffer one. Which spring will have a
greater frequency of oscillation for a given load?
Example 8. What will be the change in time period 1 k
of a loaded spring, when taken to moon? Sol. Frequency, f =
2π m
Sol. There will be no change in the time period of the loaded Force constant k is larger for the stiffer spring, so its
spring. The time period of a loaded spring is independent of frequency of oscillation will be greater than that of
acceleration due to gravity. delicate spring.
472 Objective Physics Vol. 1

1
Example 14. A particle executes simple harmonic or = sin ωt
2
motion about the point x = 0 . At time t = 0, it has
 1 π
displacement x = 2 cm and zero velocity. If the ωt = sin −1   =
 2 6
frequency of motion is 0.25 s −1 , find (a) the period, (b)
π πT
angular frequency, (c) the amplitude, (d) maximum t= =
speed, (e) the displacement at t = 3 s and ( f ) the 6 ω 6 ( 2π )
velocity at t = 3 s.
2π T 1
Sol. (a) Period T =
1
=
1
= 4s As ω= ⇒ t= = s
f 0.25 s −1 T 12 3
(b) Angular frequency, Example 17. Two particles move parallel to x-axis
2π 2π π
ω= = = rad/s = 1.57 rad/s about the origin with the same amplitude and
T 4 2 frequency. At a certain instant they are found at
(c) Amplitude is the maximum displacement from mean A
position. Hence, A = 2 − 0 = 2 cm. distance from the origin on opposite sides but their
(d) Maximum speed 3
π velocities are found to be in the same direction. What
v max = A ω = 2 ⋅ = π cm/s = 3.14 cm/s is the phase difference between the two?
2
(e) The displacement is given by Sol. Let equations of two SHM be
x = A sin (ωt + φ) x 1 = A sin ωt K (i)
Initially, at t = 0, x = 2 cm, then x 2 = A sin (ωt + φ) K (ii)
2 = 2 sin φ Give that
A
= A sin ωt
or sin φ = 1 = sin 90 ° 3
or φ = 90 ° A
and − = A sin (ωt + φ)
Now, at t = 3s 3
π π 1
x = 2 sin  × 3 +  = 0 Which gives sin ωt = K (iii)
2 2 3
(f) Velocity at x = 0 is v max , i.e. 3.14 cm/s. sin (ωt + φ) = −
1
K (iv)
3
Example 15. If an SHM is represented by the 1
From Eq. (iv), sin ωt cos φ + cos ωt sin φ = −
 π
equation x = 10 sin  πt +  , in SI units determine its 3
 6 1 1 1
⇒ cos φ + 1 − sin φ = −
amplitude, time period and maximum velocity v max ? 3 9 3
Sol. Comparing the above equation with Solving this equation, we get
x = A sin (ωt + φ) or cos φ = − 1 ,
7
we get, 9
A = 10 m 7
π ⇒ φ = π or cos −1  
ω = π s −1 and φ =  9
6 Differentiating Eqs. (i) and (ii), we obtain

Q T= ⇒ T = 2s v1 = Aω cos ωt
ω and v 2 = Aω cos (ωt + φ)
v max = ωA = 10 π m/s If we put φ = π, we find v1 and v 2 are of opposite signs.
Hence, φ = π is not acceptable.
Example 16. A particle executes SHM with a time 7
period of 4 s. Find the time taken by the particle to go ∴ φ = cos −1  
 9
directly from its mean position to half of its amplitude.
Sol. x = A sin (ωt + φ) Example 18. A particle executes SHM
At t = 0, x = 0 (a) What fraction of total energy is kinetic and what
⇒ A sin φ = 0
fraction is potential when displacement is one half
or φ=0
of the amplitude.
Hence, x = A sin ωt
A (b) At what value of displacement are the kinetic and
or = A sin ωt potential energies equal.
2
Simple Harmonic Motion 473

1 (b) Extension in spring in equilibrium


Sol. We know that E total = mω 2 A 2
2 mg
initial =
1 k
KE = mω 2 ( A 2 − x 2 )
2 Extension in spring in equilibrium in accelerating lift
1
U = mω 2 x 2 m (g + a )
and =
2 k
A m (g + a) mg
(a) When x= ∴ Amplitude = −
2 k k
1 3A 2 =
ma
KE = mω 2
2 4 k
KE 3
⇒ = Example 20. A ring of radius r is suspended from a
E total 4
point on its circumference. Determine its angular
A
At x= , frequency of small oscillations.
2
1 A2
U = mω 2
2 4
PE 1
⇒ =
E total 4
(b) Since, K =U
1 1
mω ( A − x ) = mω 2 x 2
2 2 2
2 2
A
or 2x 2 = A 2 or x = = 0.707 A Sol. It is a physical pendulum, the time period of which is
2 I
T = 2π
Example 19. A spring mass system is hanging from m gl
the ceiling of an elevator in equilibrium. The elevator Here, I = moment of inertia of the ring about
suddenly starts accelerating upwards with acceleration point of suspension
a, find = mr 2 + mr 2 = 2mr 2
and l = distance of point of suspension from centre
of gravity = r
k
2 mr 2
∴ T = 2π
m m gr
2r
= 2π
g
(a) the frequency and
(b) the amplitude of the resulting SHM. 2π
∴ Angular frequency, ω =
m T
Sol. (a) Frequency = 2π
k g
or ω=
(Frequency is independent of g in spring) 2r
NCERT Selected Questions
Q 1. Which of the following examples represent periodic  π
= 2 sin ωt − 
motion?  4
(a) A swimmer completing one (return) trip from 2π
∴ It represents simple harmonic with a period, T =
one bank of a river to the other and back. ω
(b) A freely suspended bar magnet displaced from 1
(b) sin 3 ωt = (3 sin ωt − sin 3ωt )
its N-S direction and released. 4
(c) A hydrogen molecule rotating about its centre of (Qsin 3 A = 3 sin A − 4 sin 3 A )
mass.
Here, each term sin ωt and sin 3ωt individually
(d) An arrow released from a bow. represents SHM but it is not outcome of superposition of
Sol. (a) It is not a periodic motion. Though, the motion of a two SHMs, so it will represent only periodic but not

swimmer is to and fro but will not have a fixed period. simple harmonic motion. Its time period = .
(b) It is a periodic motion because a freely suspended ω
magnet if once displaced from N-S direction and let it π   π
(c) 3 cos  − 2ωt = 3 cos  2ωt − 
go, oscillates about this position after a fixed time 4   4
interval. [Qcos (− θ ) = cos θ ]
(c) It is a periodic motion. It represents simple harmonic motion and its time period
(d) It is not a periodic motion. 2π π
is = .
2ω ω
Q 2. Which of the following examples represent (nearly)
(d) cos ωt + cos 3ωt + cos 5ωt
simple harmonic motion and which represent
periodic, but not simple harmonic motion? It represents periodic but not simple harmonic motion.

(a) The rotation of earth about its axis. Its time period is . It can be noted that each term
ω
(b) Motion of an oscillating mercury column in a represents a periodic function with a different angular
U-tube. frequency. Since, period is the least interval of time after
(c) Motion of a ball bearing inside a smooth curved which a function repeats its value, cos ωt has a period
bowl, when released from a point slightly above 2π 2π T
the lower most point. T = , cos 3ωt has a period = , and cos 5ωt has a
ω 3ω 3
2π T
Sol. (a) It is periodic but not simple harmonic motion, as it is not period = . The last two terms repeat after any
to and fro motion about a fixed point. 5ω 5
integral multiple of their period. Thus, each term in the
(b) It is SHM.
sum repeats itself after T, and hence the sum is a
(c) It is SHM. 2π
periodic function with a period .
Q 3. Which of the following functions of time represent ω
(i) Simple harmonic motion (ii) periodic but not (e) exp (− ω 2t 2 ). It is an exponential function which
simple harmonic and (iii) non-periodic motion? decreases with time and tends to zero as t → ∞ and thus
Give period for each case of periodic motion (ω is never repeats itself. Therefore, it represents
any positive constant) non-periodic motion.
(a) sin ωt − cos ωt (f) 1 + ωt + ω 2t 2. It also represents non-periodic motion.
(b) sin 3 ωt
Q 4. A particle is in linear simple harmonic motion
π 
(c) 3 cos  − 2ωt  between two points A and B, 10 cm apart. Take the
4 
direction from A to B as the positive direction and
(d) cos ωt + cos 3ωt + cos 5ωt give the signs of velocity, acceleration and force on
(e) exp (− ω 2 t 2 ) the particle when it is
(f) 1 + ωt + ω 2 t 2 (a) at the end A.
(b) at the end B.
 1 1 
Sol. (a) sin ωt − cos ωt = 2  sin ωt − cos ωt (c) at the mid-point of AB going towards A.
 2 2 
 π π (d) at 2 cm away from B going towards A.
= 2  sin ωt cos − cos ωt sin 
 4 4 (e) at 3 cm away from A going towards B.
Simple Harmonic Motion 475

Sol. A R P Q Q 6. A spring having a


B spring constant
2 cm
3 cm 1200 Nm −1 is mounted m
on a horizontal table as
shown. A mass of
10 cm 3.0 kg is attached to
+ve direction the free end of the
Points A and B are the extreme points and point P is the spring. The mass is then pulled sideways to a
mean position. distance of 0.2 cm and released. Determine
(a) At the end A, the particle executing SHM is momentarily (i) the frequency of oscillations.
at rest being at its extreme position of motion, hence its (ii) the maximum acceleration of the mass and
velocity is zero, acceleration is +ve as it is directed
towards P. Force is also +ve as the force is also directed (iii) the maximum speed of the mass?
towards P.
Sol. (i) Frequency,
Ans. 0, +, +
1 k 1 1200
(b) At end B, velocity is zero. Acceleration and force are f = = × = 3.18 Hz
2π m 2 × 3.142 3
negative as they are directed towards P, i.e. along
negative direction. (ii) The acceleration is given by
k k
Ans. 0, − , − a = − ω 2x = − x or | amax | = | xmax |
m m
(c) At the mid-point of AB going towards A, the particle is at
i.e. acceleration will be maximum, when x is maximum.
the mean position P, with a tendency to move towards A,
i.e. xmax = A = 0.02 m
i.e. along negative direction. Hence, velocity is −ve and
1200
both acceleration and force are zero. ∴ a= × 0.02 = 8.0 ms−2
3
Ans. −, 0, 0
(iii) The maximum speed of the mass is given by
(d) At 2 cm away from B going towards A, the particle is at
k 1200
point Q with a tendency to move along QP which is v = Aω = A = 0.02 × = 0.40 ms−1
negative direction. Hence, velocity, acceleration and m 3
force all are −ve, i.e. Q 7. In previous question, let us take the position of mass
Ans. − , − , − when the spring is unstretched as x = 0, and the
(e) At 3 cm away from A going towards B. The particle is direction from left to right as the positive direction
now at R moving towards B, i.e. in +ve direction. So, of x-axis. Give x as a function of time t for the
velocity, acceleration and force all are +ve. oscillating mass if at the moment we start the stop
Ans. +, +, + watch ( t = 0), the mass is
(a) at the mean position.
Q 5. Which of the following relationships between the (b) at the maximum stretched position.
acceleration a and the displacement x of a particle
(c) at the maximum compressed position.
involve simple harmonic motion?
In what way do these different functions for SHM
(a) a = 0.7x differ from each other in frequency, in amplitude or
(b) a = − 200x 2 the initial phase?
(c) a = −10x Sol. Here, A = 2 cm = 0.02 m
(d) a =100x 3 k = 1200 N/m
k 1200
Sol. A particle is said to be executing SHM if ω= = = 20 s−1
m 3
a = − ω 2x or a ∝ −x ...(i) (a) When the mass starts from mean position towards
(a) a = 0.7 x, does not satisfy Eq.(i), so, it does not represent positive direction,
SHM. x = A sin ωt
(b) a = − 200x 2 , does not satisfy Eq.(i), hence it does not ∴ x = 2 sin 20t
represent SHM. (b) In the maximum stretched position,
(c) a = − 10x, satisfies Eq. (i), so, it represents SHM. x = A cos ωt
(d) a = 100x 3, again does not represent SHM. ∴ x = 2 cos 20t
476 Objective Physics Vol. 1

(c) In the maximum compressed position, Q 9. The piston in the cylinder of a locomotive has a
x = − A cos ωt = − 2 cos 20t stroke (twice the amplitude) of 1.0 m. If the piston
The functions differ only in initial phase because their moves with simple harmonic motion with an
amplitudes (A = 2 cm) and periods are same, i.e. angular frequency of 200 rad/ min, what is its
2π 2π π maximum speed?
T = = = s
ω 20 10
Sol. Stroke of piston = 2 times the amplitude
Q 8. Figure below corresponds to two circular motions. 1
∴ A = amplitude = m (given)
The radius of the circle, the period of revolution, the 2
initial position, and the sense of revolution Angular frequency, ω = 200 rad/min
(i.e. clockwise or anti-clockwise) are indicated on 1
vmax = ωA = 200 × = 100 m/min
each figure. Obtain the corresponding simple 2
harmonic motion of the x-projection of the radius
vector of the revolving particle P, in each case. Q 10. The acceleration due to gravity on the surface of
y moon is 1.7 ms −2 . What is the time period of a
simple pendulum on the surface of moon if its time
y
period on the surface on earth is 3.5 s? (g on the
surface of earth is 9.8 ms −2 .)

T=2s P 2m T=4s Sol. Using the formula


3 cm x
O x O l
(t = 0)
T = 2π , we get
g
l
P (t = 0) For earth, Te = 2π ...(i)
ge
(a) (b) l
and for moon, Tm = 2π ...(ii)
π gm
Sol. (a) Here, at t = 0, OP makes an angle with x-axis. Since,
2 Dividing Eq. (ii) by Eq. (i), we get
+π Tm ge 9.8
motion is clockwise, so φ = rad. Thus, the = =
2 Te gm 1.7
x-projection of OP at time t will give us the equation of
SHM given by 98
or Tm = 3.5 ×
 2πt  17
x = A cos  + φ
 T  ∴ Tm = 8.4 s
 2πt π  Q 11. A simple pendulum of length l and having a bob of
= 3 cos  +  (Q A = 3 cm, T = 2 s)
 2 2 mass M is suspended in a car. The car is moving on
 π a circular track of radius R with a uniform speed v. If
or x = 3 cos  πt +  the pendulum makes small oscillations in a radial
 2
direction about its equilibrium position, what will
= − 3 sin πt (x is in cm) be its time period?
∴ x = − 3 sin πt (cm)
(b) T = 4 s, A = 2 m Sol. Point of suspension has an acceleration a c in horizontal
At t = 0, OP makes an angle π with the positive direction direction.
of x-axis, i.e. φ = + π. Hence, a = g − ac
v2
Thus, the x-projection of OP at time t will give us the − ac =
equation of SHM given by R
 2π  If T be the time period of oscillation of the pendulum, then
x = A cos  t + φ
T  l l
T = 2π = 2π
 2π  a v4
= 2 cos  t + π g2 +
 4  R2
π  l
= − 2 cos  t or T = 2π
2  1
 2 v4  2
π   g + 2
or x = − 2 cos  t (m )
2   R 
Objective Problems
[ Level 1 ]
Basic Equations in SHM
1. Two bodies M and N of equal masses are suspended from 6. A particle is executing simple harmonic motion with a
two separate massless springs of spring constants k1 and period of T seconds and amplitude a metre. The shortest
k 2 , respectively. If the two bodies oscillate vertically a
time it takes to reach a point m from its mean position
such that their maximum velocities are equal, the ratio of 2
the amplitude of vibration of M to that on N is in seconds is
k1 k1 T
(a) (b) (a) T (b)
k2 k2 4
T T
k k2 (c) (d)
(c) 2 (d) 8 16
k1 k1
7. A particle executes a simple harmonic motion of time
2. A body is executing SHM with an amplitude of 0.1 m. Its period T. Find the time taken by the particle to go directly
velocity while passing through the mean position is from its mean position to half the amplitude.
3 m/s. Its frequency in Hz is T T
(a) (b)
15 2 4
(a) 15π (b) T T
π (c) (d)
(c) 30π (d) 25π 8 12

3. Two particles are executing SHMs. The equations of 8. A particle executing SHM of amplitude 4 cm and T = 4 s.
their motions are The time taken by it to move from positive extreme
π position to half the amplitude is
y1 = 10 sin  ωt + 
 4 (a) 1 s (b)
1
s
3
 3 π 2 3
y 2 = 5 sin ωt +  (c) s (d) s
 4  3 2

What is the ratio of their amplitudes? 9. Two simple harmonic motions are represented by the
 π
(a) 1 : 1 (b) 2 : 1 equations y1 = 0.1sin 100πt +  and y 2 = 0.1cos πt.
(c) 1 : 2 (d) None of these  3
The phase difference of velocity of particle 1 with respect
4. What is the maximum acceleration of the particle doing
to the velocity of particle 2 at time t = 0 is
the SHM? π π π π
 πt  (a) − (b) (c) − (d)
y = 2 sin  + φ, where y is in cm. 3 6 6 3
2 
10. A particle executing simple harmonic motion has an
π π2
(a) cm/s2 (b) cm/s2 amplitude of 6 cm. Its acceleration at a distance of 2 cm
2 2
from the mean position is 8 cm/s 2 . The maximum speed
π π2
(c) cm/s 2 (d) cm/s 2 of the particle is
4 4
(a) 8 cm/s (b) 12 cm/s
5. The amplitude and the time period in a SHM is 0.5 cm (c) 16 cm/s (d) 24 cm/s
π
and 0.4 s, respectively. If the initial phase is rad, then 11. A particle executes simple harmonic motion with an
2
the equation of SHM will be amplitude of 4 cm. At the mean position the velocity of
(a) y = 0.5 sin 5πt the particle is 10 cm/s. The distance of the particle from
(b) y = 0.5 sin 4πt the mean position when its speed becomes 5 cm/s is
(c) y = 0.5 sin 2.5πt (a) 3 cm (b) 5 cm
(d) y = 0.5 cos 5πt (c) 2 3 cm (d) 2 5 cm
478 Objective Physics Vol. 1

12. The maximum velocity of a simple harmonic motion 20. The displacement of a particle executing SHM is given
π
represented by y = 3 sin 100t +  is given by by x = 0.01sin 100π ( t + 0.05). The time period is
 6 (a) 0.01 s (b) 0.02 s
3π (c) 0.1 s (d) 0.2 s
(a) 300 units (b) units
6 21. A body is executing simple harmonic motion with an
π
(c) 100 units (d) units angular frequency of 2 rad/s. The velocity of the body at
6 20 mm displacement, when the amplitude of the motion is
13. Velocity at mean position of a particle executing SHM 60 mm is
is v. Velocity of the particle at a distance equal to half of (a) 131 mm/s (b) 118 mm/s
the amplitude will be (c) 113 mm/s (d) 90 mm/s
v v
(a)
2
(b)
2 Energy in SHM
3 3 22. Force constant of a weightless spring is 16 N/m. A body
(c) v (d) v
2 4 of mass 1.0 kg suspended from it is pulled down through
5 cm from its mean position and then released. The
14. Two massless springs of force constants k1 and k 2 are maximum kinetic energy of the body will be
joined end to end. The resultant force constant k of the (a) 2 × 10−2 J (b) 4 × 10−2 J
system is −2
(c) 8 × 10 J (d) 16 × 10−2 J
k + k2
(a) 1 (b) k 1 + k 2
k 1k 2 23. Energy of particle executing SHM depends upon
k 1k 2 k 1k 2 (a) amplitude only
(c) (d)
k1 + k2 k1 − k2 (b) amplitude and frequency
(c) velocity only
15. A light spring of constant k is cut into two equal parts. (d) frequency only
The spring constant of each part is
(a) k (b) 2k 24. A particle undergoing SHM has the equation
k x = A sin (ωt + φ ), where x represents the displacement of
(c) (d) 4k the particle. The kinetic energy oscillates with time
2
period
16. A body is vibrating in simple harmonic motion. If its 2π π
(a) (b)
acceleration is 12 cm/s 2 at a displacement 3 cm from the ω ω
mean position, then time period is 4π
(c) (d) None of these
(a) 6.28 s (b) 3.14 s (c) 1.57 s (d) 2.57 s ω
17. The period of a particle executing SHM is 8 s. At t = 0, it 25. A particle of mass 0.10 kg executes SHM with an
is at the mean position. The ratio of the distances covered amplitude 0.05 m and frequency 20 vib/s. Its energy of
by the particle in the 1st second to the 2nd second is oscillation is
1 (a) 2 J (b) 4 J
(a) (b) 2
2+1 (c) 1 J (d) zero
1
(c) (d) 2+1 26. In SHM, for how many times potential energy is equal to
2 kinetic energy during one complete period?
18. The maximum acceleration of a body moving in SHM is (a) 1 (b) 2
a 0 and maximum velocity is v 0 . The amplitude is given (c) 4 (d) 8
by 27. Amplitude of a particle in SHM is 6 cm. If instantaneous
v2 potential energy is half the total energy, then distance of
(a) 0 (b) a0v0
a0 particle from its mean position is
a02 1 (a) 3 cm (b) 4.2 cm
(c) (d) (c) 5.8 cm (d) 6 cm
v0 a0v0
28. A body of mass 1 kg is executing simple harmonic
19. The displacement of a particle executing SHM is given
motion. Its displacement y (cm) at t seconds is given by
by y = 0.25sin 200 t cm. The maximum speed of the
 π
particle is y = 6 sin 100t +  . Its maximum kinetic energy is
 4
(a) 200 cm s−1 (b) 100 cm s−1
(c) 50 cm s−1 (d) 5.25 cm s−1 (a) 6 J (b) 18 J (c) 24 J (d) 36 J
Simple Harmonic Motion 479

29. In a simple harmonic oscillator, at the mean position 36. A rectangular block of mass m and area of cross-section
(a) kinetic energy is minimum, potential energy is maximum A floats in a liquid of density ρ. If it is given small vertical
(b) both kinetic and potential energies are maximum displacement from equilibrium, it undergoes oscillation
(c) kinetic energy is maximum, potential energy is minimum with a time period T, then
(d) both kinetic and potential energies are minimum (a) T ∝ m (b) T ∝ ρ
30. A body executes simple harmonic motion. The potential 1 1
(c) T ∝ (d) T ∝
energy (PE), the kinetic energy (KE) and total energy A ρ
(TE) are measured as a function of displacement x.
37. A cabin is falling freely under gravity, what is the time
Which of the following statements is true?
period of a pendulum attached to its ceiling
(a) PE is maximum when x = 0
(a) zero (b) ∞
(b) KE is maximum when x = 0
(c) 1 s (d) 2 s
(c) TE is zero when x = 0
(d) KE is maximum when x is maximum 38. If both spring constants k1 and k 2 are increased to 4k1
31. The total energy of a particle, executing simple harmonic and 4k 2 respectively, what will be the new frequency, if f
motion is was the original frequency?
(a) ∝ x (b) ∝ x 2 k1 k2
(c) independent of x (d) ∝ x 1/ 2

32. A body of mass 1 kg is executing simple harmonic 1


(a) f (b) 2 f (c) f (d) 4 f
motion. Its displacement y (cm) at t seconds is given by 2
 π
y = 6 sin 100t +  . Its maximum kinetic energy is 39. Frequency of oscillation is proportional to
 4
k 2k
(a) 6J m
(b) 18 J
(c) 24 J 3k k
(a) (b)
(d) 36 J m m
33. Two simple harmonic motions y1 = A sin ωt and (c)
2k
(d)
m
m 3k
y 2 = A cos ωt are superimposed on a particle of mass m.
The total mechanical energy of the particle is 40. The mass and the radius of a planet are twice that of earth.
1
(a) mω 2 A 2 (b) mω A 2 2 Then, period of oscillation of a second’s pendulum on
2 that planet will be
1
(c) mω 2 A 2 (d) zero 1
4 (a) s (b) 2 2 s
2
34. A particle starts SHM from the mean position. Its 1
(c) 1 s (d) s
amplitude is a and total energy E. At one instant its 2
3E
kinetic energy is . Its displacement at that instant is 41. A mass m is suspended from a spring. Its frequency of
4 oscillation is f. The spring is cut into two halves and the
a a
(a) (b) same mass is suspended from one of the two pieces of the
2 2 spring. The frequency of oscillation of the mass will be
a
(c) 3 (d) zero f
2 (a) 2 f (b) (c) f (d) 2 f
2
Time Period and Frequency in SHM 42. A block of mass 0.2 kg, which slides
without friction on a 30° incline, is
35. The time period of a simple pendulum inside a stationary
connected to the top of the incline by
lift is 5 s. What will be the time period when the lift
a massless spring of force constant M
g
moves upwards with an acceleration ? 80 N/m as shown in figure. If the
4 block is pulled slightly from its mean
(a) 5 s position, what is the period of 30°
(b) 2 5 s oscillations?
(c) (2 + 5 ) s π 2π π
(a) π s (b) s (c) s (d) s
(d) 2 s 10 5 2
480 Objective Physics Vol. 1

43. A horizontally placed spring mass system has time 50. A simple pendulum is attached to the roof of a lift. If time
period T. The same system is now placed on a car moving period of oscillation, when the lift is stationary is T, then
with acceleration a in horizontal direction. Then, frequency of oscillation, when the lift falls freely, will be
(a) time period will increase (a) zero (b) infinite
(b) time period will decrease 1
(c) (d) None of these
(c) time period will remain constant T
(d) no conclusion can be drawn
51. A particle is attached to a vertical spring and is pulled
44. A particle executes linear simple harmonic motion with an down a distance 0.04 m below its equilibrium position and
amplitude of 2 cm. When the particle is at 1 cm from the is released from rest. The initial upward acceleration of the
mean position, the magnitude of its velocity is equal to that particle is 0.30 m/s 2 . The period of the oscillation is
of its acceleration. Then, its time period in seconds is (a) 4.08 s (b) 1.92 s
1
(a) (b) 2π 3 (c) 3.90 s (d) 2.29 s
2π 3
2π 3 52. Three masses 0.1 kg, 0.3 kg and 0.4 kg are suspended at
(c) (d) end of a spring. When the 0.4 kg mass is removed, the
3 2π
system oscillates with a period 2 s. When the 0.3 kg mass
d2 y is also removed, the system will oscillate with a period
45. The equation of motion of a particle is + Ky = 0,
dt 2 (a) 1 s (b) 2 s (c) 3 s (d) 4 s
where K is a positive constant. The time period of the
53. The time period of a simple pendulum of infinite length is
motion is given by
2π R
(a) (b) 2πK (a) infinite (b) 2π
K g
2π g 1 R
(c) (d) 2π K (c) 2π (d)
K R 2π g
46. The displacement x (in metre) of a particle in, simple 54. A disc of radius R and mass M is pivoted at the rim and is
harmonic motion is related to time t (in seconds) as set for small oscillations about an axis perpendicular to
π
x = 0.01cos  πt +  plane of disc. If a simple pendulum has to have the same
 4 time period as that of the disc, the length of the pendulum
The frequency of the motion will be should be
(a) 0.5 Hz (b) 1.0 Hz 5 2 3 3
(a) R (b) R (c) R (d) R
π 4 3 4 2
(c) Hz (d) π Hz
2
55. A simple pendulum is suspended from the roof of a
47. A simple pendulum is made of a body which is a hollow trolley which moves, in a horizontal direction with an
sphere containing mercury suspended by means of a acceleration a, then the time period T is given by
wire. If a little mercury is drained off, the period of l
T = 2π , where g ′ is equal to
pendulum will g′
(a) remain unchanged (b) increase
(a) g (b) g − a
(c) decrease (d) become erratic
(c) g + a (d) g 2 + a2
48. A man measures the period of a simple pendulum inside a
stationary lift and finds it to be T second. If the lift 56. Five identical springs are used in
g the three configurations as
accelerates upwards with an acceleration , then the
4 shown in figure. The time
period of the pendulum will be periods of vertical oscillations in
m m
T configurations (a), (b) and (c) are (a)
(a) T (b) (c)
4 in the ratio
2T 1 m
(c) (d) 2T 5 (a) 1 : 2 :
5 (b)
2
49. If the metal bob of a simple pendulum is replaced by a 1
(b) 2 : 2 :
wooden bob, then its time period will 2
(a) increase 1
(c) : 2:1
(b) decrease 2
(c) remain the same 1
(d) 2 : :1
(d) may increase or decrease 2
Simple Harmonic Motion 481

57. A uniform spring whose unstressed length is l has a force 63. In case of a simple pendulum, time period versus length
constant k. The spring is cut into two pieces of unstressed is depicted by
lengths l1 and l2 , where l1 = nl2 , where n being an
integer. Now, a mass m is made to oscillate with first
spring. The time period of its oscillation would be
(a) T (b) T
mn m
(a) T = 2π (b) T = 2π
k (n + 1) k (n + 1) l l
m m(n + 1)
(c) T = 2π (d) T = 2π
nk nk

58. Two masses m1 and m2 are suspended together by a (c) T (d) T

massless spring of spring constant k. When the masses


l l
are in equilibrium m1 is removed without distributing the
system, then the angular frequency of oscillation will be
64. The displacement-time graph of a particle executing
k k
(a) (b) + m1 SHM is shown in figure. Which of the following
m1 + m2 m2 statements is false?
k k
(c) (d)
m1 m2 y

59. Two identical springs of spring constant


k each are connected in series and 3T T t
T
parallel as shown in figure. A mass M is 4 T 4
suspended from them. The ratio of their 2
frequencies of vertical oscillation will be
M
(a) 1 : 2 (b) 2 : 1
(c) 4 : 1 (d) 1 : 4 (a) The acceleration is maximum at t = T
M 3T
60. A point mass m is suspended at the end (b) The force is zero at t =
of a massless wire of length L and 4
cross-sectional area A. If Y is the Young’s modulus of the (c) The potential energy equals the total oscillation energy at
T
wire. Then, the frequency of the oscillation for the simple t=
2
harmonic oscillation along the vertical direction is
(d) None of the above
1 LA 1 LAm
(a) (b)
2π mY 2π Y 65. The displacement equation of a particle is
(c)
1 YA
(d)
1 mY x = 3 sin 2t + 4 cos 2t. The amplitude and maximum
2π mL 2π AL velocity will be respectively
61. A thin uniform rod of length l is pivoted at its upper end. (a) 5, 10 (b) 3, 2
It is free to swing in a vertical plane. Its time period for (c) 4, 2 (d) 3, 4
oscillations of small amplitude is
66. The acceleration a of a particle undergoing SHM is
l 2l
(a) 2π (b) 2π shown in the figure. Which of the labelled points
g 3g
corresponds to the particle being at −x max ?
3l l
(c) 2π (d) 2π a
2g 3g
1
Superposition of Two or More than
2 4
Two SHMs and Graphical Problems t
62. The motion of a particle is given x = A sin ωt + B cos ωt.
The motion of the particle is 3
(a) not simple harmonic
(b) simple harmonic with amplitude A + B
( A + B) (a) 4
(c) simple harmonic with amplitude (b) 3
2
(c) 2
(d) simple harmonic with amplitude A2 + B 2 (d) 1
482 Objective Physics Vol. 1

67. The displacement-time graph of a particle executing The period of the SHM is
SHM is as shown in the figure. F (N)

8.0
y
+2.0 x (m)
–2.0
– 8.0
O t

(a) 1.05 s (b) 0.52 s


The corresponding force-time graph of the particle is (c) 0.25 s (d) 0.31 s
F F
71. In order that the resultant path on superimposing two
mutually perpendicular SHM be a circle, the conditions
(a) (b) are that
(a) the amplitudes on both SHM should be equal and they
O t O t π
should have a phase difference of
F F 2
(b) the amplitudes should be in the ratio 1 : 2 and the phase
difference should be zero
(c) (d)
O O (c) the amplitudes should be in the ratio 1 : 2 and the phase
t t π
difference should be
2
68. The graph shows the variation of displacement of a (d) the amplitudes should be equal and the phase difference
should be zero
particle executing SHM with time. We infer from this
graph that 72. Lissajous figure obtained by combining
y
 π
x = a sin ωt and y = b sin ωt + 
 2
T/2 T will be
T/4 3T/4 t (a) an ellipse
(b) a straight line
(c) a circle
3T
(a) the force is zero at time (d) a parabola
4
T 73. Two particles are executing simple harmonic motion. At
(b) the velocity is maximum at time
2 an instant of time t their displacements are
(c) the acceleration is maximum at time T y1 = a cos (ωt ) and y 2 = a sin (ωt )
T
(d) the PE is equal to total energy at time
2 Then, the phase difference between y1 and y 2 is
(a) 120° (b) 90°
69. The velocity-time diagram of a harmonic oscillator is (c) 180° (d) zero
shown in the below figure. The frequency of oscillation is
74. A particle of mass m oscillates with simple harmonic
+4
v (m/s) motion between points x1 and x 2 , the equilibrium position
being at O. Its potential energy is plotted. It will be as
given below in the graph
0.02 0.04
0.01 0.03
T (in second)
x2 x2
(a) x1 O (b) x1 O
–4

(a) 25 Hz (b) 50 Hz
(c) 12.25 Hz (d) 33.3 Hz

70. A body of mass 0.1 kg executes simple harmonic motion


(c) (d)
(SHM) about x = 0 under the influence of a force shown
in figure. x1 O x2 x1 O x2
Simple Harmonic Motion 483

Miscellaneous Problems 83. A mass of 0.2 kg is attached to the lower end of a


massless spring of force constant 200 N/m, the upper end
75. In a spring-mass system, the length of the spring is L, and of which is fixed to a rigid support. Which of the
it has a mass M attached to it and oscillates with an following statement is true?
angular frequency ω. The spring is then cut into two (a) In equilibrium, the spring will be stretched by 1 cm
parts, one (a) with relaxed length αL and the other (b) (b) If the mass is raised till the spring becomes unstretched and
with relaxed length (1 − α ) L. The force constants of the then released, it will go down by 2 cm before moving
two springs A and B are upwards
k k k k (c) The frequency of oscillation will be nearly 5 Hz
(a) and (b) and
1−α α α 1−α (d) All of the above
(c) αk , (1 − α ) k (d) k and k
84. In SHM, phase difference between displacement and
76. A light spring of constant k is cut into two equal parts. velocity is φ 1 and that between displacement and
The spring constant of each part is acceleration is φ 2 , then
(a) k (b) 2k (a) φ 2 = 2φ 1 (b) φ 2 = φ 1
k (c) φ 1 = 2φ 2 (d) None of these
(c) (d) 4k
2
85. Which of the following quantities is always negative in
77. A light spring of force constant 8 Nm −1 is cut into two SHM?
equal halves and the two are connected in parallel; the (a) F ⋅ a (b) v ⋅ s
equivalent force constant of the system is (c) a ⋅ s (d) F ⋅ v
(a) 16 Nm −1 (b) 32 Nm −1 (c) 8 Nm −1 (d) 24 Nm −1 Here, s is displacement from mean position.
78. Two springs of spring constants k each are connected in 86. A small ball is dropped from a certain height on the
series, then what is the net spring constant? surface of a non-viscous liquid of density less than the
k density of ball. The motion of the ball is
(a) k (b)
2 (a) SHM
k (b) periodic but not SHM
(c) 2k (d) 3
2 (c) not periodic
79. Two point masses of 3.0 kg and 6.0 kg are attached to (d) SHM for half a period and non-periodic for rest half of the
period
opposite ends of horizontal spring whose spring constant
is 300 Nm −1 as shown in the figure. The natural vibration 87. When a particle executes SHM there is always a constant
frequency of the system is approximately ratio between its displacement and
(a) velocity
k = 300 Nm–1
(b) acceleration
6 kg 3 kg
(c) mass
(d) time period
(a) 4 Hz (b) 3 Hz(c) 2 Hz (d) 1 Hz
88. A mass M = 5 kg is attached to a spring as
80. In simple harmonic motion, the particle is
shown in the figure and held in position so
(a) always accelerated
(b) always retarded that the spring remains unstretched. The
(c) alternately accelerated and retarded spring constant is 200 N/m. The mass M is
(d) neither accelerated nor retarded then released and begins to undergo small
M
oscillations. The amplitude of oscillation is
81. The dimensions of mass ÷ force constant of SHM are
(a) 0.5 m k
same as that of
2 (b) 0.25 m
(a) time (b) time (c) 0.2 m
1
(c) acceleration (d) (d) 0.1 m
acceleration
89. Under the action of a force F = − kx 3 , the motion of a
82. The length of second’s pendulum on the earth is about
1 m. What should be the length of second’s pendulum on particle is (k = a positive constant)
the moon? (a) simple harmonic motion
1 (b) uniformly accelerated motion
(a) 1 m (b) m (c) not periodic
6
(c) 6 m (d) 36 m (d) periodic but not simple harmonic
484 Objective Physics Vol. 1

90. The phase (at a time t) of a particle in simple harmonic 97. The displacement x (in metre) of a particle in, simple
motion tells harmonic motion is related to time t (in second) as
(a) only the position of the particle at time t  π
x = 0.01cos  πt + 
(b) only the direction of motion of the particle at time t  4
(c) both the position and direction of motion of the particle at
time t The frequency of the motion will be
(d) neither the position of the particle nor its direction of motion (a) 0.5 Hz (b) 1.0 Hz
π
at time t (c) Hz (d) π Hz
2
91. A particle is moving with constant angular velocity along
the circumference of a circle. Which of the following 98. In a simple pendulum, the period of oscillation T is
statements is true? related to length of the pendulum l as
(a) The particle executes SHM l l2
(a) = constant (b) = constant
(b) The projection of the particle on any one of the diameters T T
executes SHM l l2
(c) 2 = constant (b) 2 = constant
(c) The projection of the particle on any of the diameters T T
executes SHM
(d) None of the above 99. The ratio of frequencies of two pendulums are 2 : 3, then
their lengths are in ratio
92. A particle executing simple harmonic motion along 2 3
y-axis has its motion described by the equation (a) (b)
3 2
y = A sin (ωt ) + B. The amplitude of the simple 4 9
harmonic motion is (c) (d)
9 4
(a) A
(b) B 100.Molten-wax of mass m drops on a block of mass M,
(c) A + B which is oscillating on a frictionless table as shown.
Select the incorrect option.
(d) A + B
m
93. Which of the following is a necessary and sufficient
condition for SHM? M
(a) Constant period
(b) Constant acceleration
(c) Proportionality between acceleration and velocity (a) If the collision takes place at extreme position, amplitude
(d) Proportionality between restoring force and displacement does not change
from equilibrium position (b) If the collision takes place at mean position, amplitude
decreases
94. In simple harmonic motion, the ratio of acceleration of (c) If the collision takes place at mean position, time period
the particle to its displacement at any time is a measure of decreases
(a) spring constant (d) If the collision takes place at extreme position, time period
(b) angular frequency increases
(c) (angular frequency)2
101.A particle executes SHM with amplitude of 20 cm and
(d) restoring force
time period of 12 s. What is the minimum time required
95. A tunnel has been dug through the centre of the earth and for it to move between two points 10 cm on either side of
a ball is released in it. It will reach the other end of the the mean position?
tunnel after (a) 1 s (b) 2 s (c) 3 s (d) 4 s
(a) 84.6 min
102.The maximum acceleration of a particle in SHM is made
(b) 42.3 min
two times keeping the maximum speed to be constant. It
(c) 1 day
is possible when
(d) will not reach the other end
(a) amplitude of oscillation is doubled while frequency remains
96. What is constant in SHM? constant
(a) Restoring force (b) amplitude is doubled while frequency is halved
(b) Kinetic energy (c) frequency is doubled while amplitude is halved
(c) Potential energy (d) frequency of oscillation is doubled while amplitude remains
(d) Periodic time constant
[ Level 2 ]
Only One Correct Option
1. A body of mass 0.01 kg executes simple harmonic 5. The variation of PE of harmonic oscillator is as shown in
motion (SHM) about x = 0 under the influence of a force figure. The spring constant is
shown in figure. The period of the SHM is U (joule)
F (N)
0.04
8.0

+2.0 x (m)
0.01
– 2.0
x (mm)
– 8.0
20

(a) 1 × 102 N/m (b) 1.5 × 102 N/m


(a) 1.05 s (b) 0.52 s (c) 2 × 10 N/m
2
(d) 3 × 102 N/m
(c) 0.25 s (d) 0.31 s
6. A particle of mass 0.1 kg is executing SHM of amplitude
2. The vertical motion of a ship at sea is described by the 0.1 m. When the particle passes through the mean
d 2x
equation 2 = − 4x, where x is the vertical height of the position, its KE is 8 × 10−3 J. Find the equation of motion
dt of the particle if the initial phase of oscillation is 45°.
ship (in metre) above its mean position. If it oscillates  π  π
through a height of 1 m, (a) y = 0.1 cos  3t +  (b) y = 0.1 sin  6t + 
 4  4
(a) its maximum vertical speed will be 1 m/s  π  π
(b) its maximum vertical speed will be 2 m/s (c) y = 0.1 sin  4 t +  (d) y = 0.1 cos  4 t + 
 4  4
(c) its greatest vertical acceleration is 2 m/s2
(d) its greatest vertical acceleration is 1 m/s2 7. The system shown in figure is in equilibrium. The mass
3. Period of small oscillations in the two cases shown in of the container with liquid is M, density of liquid in the
figure is T1 and T2 respectively, then container is ρ and the volume of the block is V. If the
container is now displaced downwards through a distance
x 0 and released such that the block remains well inside
k the liquid then during subsequent motion,
k

m
m k

A
(a) T1 = T2
(b) T1 < T2
(c) T1 > T2
(d) Cannot say anything
4. The potential energy of a particle of mass 2 kg in SHM is
( 9x 2 ) J. Here, x is the displacement from mean position.
M
If total mechanial energy of the particle is 36 J, then (a) time period of SHM of the container will be 2π
k
maximum speed of the particle is
M + ρV
(a) 4 m/s (b) time period of SHM of the container will be 2π
(b) 2 m/s k
(c) 6 m/s (c) amplitude of SHM of container is x0
(d) 10 m/s (d) amplitude of SHM of the container is 2x0
486 Objective Physics Vol. 1

8. A particle is in linear SHM of amplitude A and time 14. The speed ( v ) of a particle moving along a straight line,
period T. If v refers to its average speed during any
T when it is at a distance ( x ) from a fixed point on the line,
interval of , then the maximum possible value of v is is given by v 2 = 144 − 9x 2 . Select wrong alternate.
3
3 3 3 A (a) Displacement of the particle ≤ distance moved by it
(a) A (b) (b) The magnitude of acceleration at a distance 3 units from the
T T
2 3 3A fixed point is 27 units
(c) A (d) π
T T (c) The motion is simple harmonic with T = units
3
9. A simple pendulum is suspended from the ceiling of a car (d) The maximum displacement from the fixed point is 4 units
and its period of oscillation is T when the car is at rest.
15. Two pendulums of time periods 3 s and 7 s, respectively
The car starts moving on a horizontal road with a
start oscillating simultaneously from two opposite
constant acceleration g (equal to the acceleration due to
extreme positions. After how much time they will be in
gravity, in magnitude) in the forward direction. To keep
same phase?
the time period same, the length of the pendulum
21 21 21 21
(a) will have to be increased by 2 l (a) s (b) s (c) s (d) s
8 4 2 10
(b) will have to be increased by ( 2 − 1) l
(c) will have to be decreased by 2 l 16. A particle under the action of a SHM has a period of 3 s
(d) will have to be decreased by ( 2 − 1) l
and under the effect of another it has a period 4 s. What
will be its period under the combined action of both the
10. A particle of mass m is dropped from a great height h SHM’s in the same direction?
above the hole in the earth dug along its diameter. (a) 7 s (b) 5 s
(a) The motion of the particle is simple harmonic (c) 2.4 s (d) 0.4 s
(b) The motion of the particle is periodic
17. A particle of mass m is B C
(c) The speed of the particle at the centre of earth equals
2GM
attached to three identical 90°
, where R and M are the radius and mass of the earth springs A,B and C each of
(R + h) 45°
force constant k as shown in m
respectively O
figure. If the particle of mass
(d) The speed of the particle at the centre of earth equals
GM (R + 3h)
m is pushed slightly against
, where R and M are the radius and mass of the the spring A and released,
R (R + h) A
then the time period of
earth respectively
oscillation is
11. A solid cube floats in water half immersed and has small 2m m
π (a) 2π (b) 2π
vertical oscillations of time period s. Its mass (in kg) is k 2k
5
m m
(Take g = 10 m/s 2 ) (c) 2π (d) 2π
k 3k
(a) 4 (b) 2
(c) 1 (d) 0.5 18. In SHM, potential energy of a particle at mean position is
E1 and kinetic energy is E 2 , then
12. Maximum kinetic energy of a particle of mass 1 kg in
SHM is 8 J. Time period of SHM is 4 s. Maximum (a) E1 = E2
potential energy during the motion is 10 J. Then, 3A 3E2
(b) total potential energy at x = is E1 +
(a) amplitude of oscillations is approximately 2.53 m 2 4
3 A 3E2
(b) minimum potential energy of the particle is 2 J (c) total kinetic energy at x = is
(c) maximum acceleration of the particle is approximately 2 4
6.3 m/s2 A E2
(d) total kinetic energy at x = is
(d) minimum kinetic energy of the particle is 2 J 2 4

13. The potential energy of a particle of mass 0.1 kg moving 19. A particle performs SHM in a straight line. In the first
along the x-axis, is given byU = 5x( x − 4 ) J, where x is in second, starting from rest, it travels a distance a and in the
metres. Choose the wrong option. next second it travels a distance b in the same side of
(a) The speed of the particle is maximum at x = 2 m mean position. The amplitude of the SHM is
(b) The particle executes simple harmonic motion 2a − b
(a) a − b (b)
π 3
(c) The period of oscillation of the particle is s
5 2a2
(c) (d) None of these
(d) None of the above 3a − b
Simple Harmonic Motion 487

20. A block of mass 100 g attached to a spring of spring 25. The period of oscillation of a simple pendulum of length
constant 100 N/m is lying on a frictionless floor as L suspended from the roof of a vehicle which moves
shown. The block is moved to compress the spring by without friction down an inclined plane of inclination α
10 cm and then released. If the collisions with the wall in is, given by
front are elastic, then the time period of the motion is L L
(a) 2π (b) 2π
(g cos α ) (g sin α )
L L
(c) 2π (d) 2π
g (g tan α )

26. A horizontal platform with an object placed on it is


5 cm executing SHM in the vertical direction. The amplitude
of oscillation is 4 × 10−3 m. What must be least period of
(a) 0.2 s (b) 0.1 s
(c) 0.15 s (d) 0.132 s
these oscillations, so that the object is not detached from
the platform? (Take g = 10 m/ s 2 )
21. In the figure, the block of mass m, attached to the spring π π
of stiffness k is in contact with the completely elastic (a) s (b) s
25 5
wall, and the compression in the spring is e. The spring is π π
compressed further by e by displacing the block towards (c) s (d) s
10 50
left and is then released. If the collision between the
block and the wall is completely elastic, then the time 27. A block of mass m, attached to a spring of spring constant
period of oscillations of the block will be k, oscillates on a smooth horizontal table. The other end
of the spring is a fixed to a wall. The block has a speed v
when the spring is at its natural length. Before coming to
Wall
an instantaneous rest, if the block moves a distance x
m
from the mean position, then
m 1 m
(a) x = (b) x =
k v k
2π m m π m π m
(a) (b) 2π (c) (d) m mv
3 k k 3 k 6 k (c) x = v (d) x =
k k
22. A cubical block of mass M vibrates horizontally with 28. One end of a long metallic wire of length L is tied to the
amplitude of 4.0 cm and a frequency of 2.0 Hz. A small ceiling. The other end is tied to massless spring of spring
block of mass m is palced on the bigger block. In order constant k. A mass m hangs freely from the free end of the
that the smaller block does not slide on the bigger block, spring. The area of cross-section and Young’s modulus
the minimum value of the coefficient of static friction of the wire are A and Y, respectively. If the mass is
between the two blocks is slightly pulled down and released, it will oscillate with a
(a) 0.36 (b) 0.40 time period T equal to
m (YA + kL)
(c) 0.64 (d) 0.72 (a) 2π (b) 2π
k YAk
23. A spring has a natural length of 50 cm and a force (YA + kL) (Y + kL)
(c) 2π (d) 2π
constant of 2.0 × 103 Nm −1 . A body of mass 10 kg is Ak YAk
suspended from it and the spring is stretched. If the body
29. A mass M is attached to a m
is pulled down to a length of 58 cm and released, it horizontal spring of force k
executes simple harmonic motion. What is the net force constant k fixed one side to a M
on the body when it is at its lowermost position of its rigid support as shown in
oscillation? (Take g = 10 ms −2 ). figure. The mass oscillates on a
(a) 20 N (b) 40 N frictionless surface with time period T and amplitude A.
(c) 60 N (d) 80 N When the mass is in equilibrium position, another mass m
is gently placed on it. What will be the new amplitude of
24. Two masses 8 kg and 4 kg are suspended together by a oscillations?
massless spring of spring constant 1000 N/m. When the (M + m) (M − m)
masses are in equilibrium 8 kg is removed without (a) A (b) A
M M
disturbing the system. The amplitude of oscillation is M M
(a) 0.5 m (b) 0.08 m (c) A (d) A
(c) 0.4 m (d) 0.04 m (M + m) (M − m)
488 Objective Physics Vol. 1

30. A linear harmonic oscillator of force constant 35. A mass M is suspended from a massless spring. An
2 × 106 N/ m and amplitude 0.01 m has a total mechanical additional mass m stretches the spring further by a
energy of 160 J. Its distance x. The combined mass will oscillate with a
(a) maximum potential energy is 160 J period
(b) maximum potential energy is 100 J  (M + m) x   mg 
(a) 2π   (b) 2π  
(c) minimum potential energy is zero  mg   (M + m) x 
(d) minimum potential energy is 100 J
 (M + m)  π  mg 
31. A mass is suspended separately by two springs of spring (c) 2π   (d)  
 mgx  2  (M + m ) x 
constants k1 and k 2 in successive order. The time periods
of oscillations in the two cases are T1 and T2 , respectively. More than One Correct Options
If the same mass be suspended by connecting the two
springs in parallel, (as shown in figure) then the time 1. A simple pendulum with a bob of mass m is suspended
period of oscillations is T. The correct relation is from the roof of a car moving with horizontal
acceleration a. Then,
(a) the string makes an angle of tan − 1 (a/ g ) with the vertical
k1 k2
 a
(b) the string makes an angle of sin −1   with the vertical
 g

M
(c) the tension in the string is m a2 + g 2
(d) the tension in the string is m g 2 − a2
(a) T 2 = T12 + T22 (b) T −2 = T1−2 + T2−2
(c) T −1 = T1−1 + T2−1 (d) T = T1 + T2 2. A particle starts from a point P at a distance of A/ 2 from
the mean position O and travels towards left as shown in
32. Two particles execute SHM of the same amplitude and the figure. If the time period of SHM, executed about O is
frequency along the same straight line. They pass one T and amplitude is A, then the equation of the motion of
another when going in opposite directions each time their particle is
displacement is half their amplitude. What is the phase A/2
difference between them?
(a) 60° (b) 30° O P
(c) 90° (d) 120° A
 2π π  2π 5π 
33. A particle of mass 2 kg moves in simple harmonic motion (a) x = A sin  t+  (b) x = A sin  t+ 
T 6 T 6
and its potential energy U varies with position x as
 2π π  2π π
shown. The period of oscillation of the particle is (c) x = A cos  t+  (d) x = A cos  t+ 
T 6 T 3
U (J)
1.0 3. A spring has natural length 40 cm and spring constant
500 N/m. A block of mass 1 kg is attached at one end of
the spring and other end of the spring is attached to a
ceiling. The block is released from the position, where
the spring has length 45 cm. Then,
x (m) (a) the block will perform SHM of amplitude 5 cm
O 0.4 (b) the block will have maximum velocity 30 5 cm/s
2π 2 2π (c) the block will have maximum acceleration 15 m/s2
(a) s (b) s
5 5 (d) the minimum elastic potential energy of the spring will be
2π 4π zero
(c) s (d) s
5 5 4. The system shown in the figure k = 800 N/m
3kg 6kg
34. Two simple harmonic motions are represented by the can move on a smooth surface.
following equations They are initially compressed
by 6 cm and then released.
y1 = 40 sin ωt
π
and y 2 = 10 (sin ωt + c cos ωt ) (a) The system performs, SHM with time period
10
s

If their displacement amplitudes are equal, then the value (b) The block of mass 3 kg performs SHM with amplitude 4 cm
of c (in appropriate units) is (c) The block of mass 6 kg will have maximum momentum of
(a) 13 (b) 15 2.40 kg m/s
(c) 17 (d) 4 (d) The time periods of two blocks are in the ratio of 1 : 2
Simple Harmonic Motion 489

5. The displacement-time 11. A body is performing SHM, then its


graph of a particle (a) average total energy per cycle is equal to its maximum
executing SHM is kinetic energy
y (b) average kinetic energy per cycle is equal to half of its
shown in figure. maximum kinetic energy
Which of the O (c) mean velocity over a complete cycle is equal to 2 times of its
T/4 T/2 3T/4 T
following statement(s) t π
is/are true? maximum velocity
(a) The velocity is (d) root mean square velocity is 1 times of its maximum
maximum at t = T /2 2
(b) The acceleration is maximum at t = T velocity
(c) The force is zero at t = 3T / 4
(d) The kinetic energy equals total oscillation energy at t = T /2 12. A particle is in linear
A O C B
simple harmonic
6. For a particle executing SHM, x = displacement from motion between two points. A and B, 10 cm apart (figure)
mean position, v = velocity and a = acceleration at any take the direction from A to B as the positive direction
instant, then and choose the correct statements.
(a) v -x graph is a circle (b) v -x graph is an ellipse AO = OB = 5 cm ⇒ BC = 8 cm
(c) a-x graph is a straight line (d) a-x graph is a circle (a) The sign of velocity, acceleration and force on the particle
when it is 3 cm away from A going towards B are positive
7. The acceleration of a particle is a = − 100x + 50. It is
(b) The sign of velocity of the particle at C going towards B is
released from x = 2. Here, a and x are in SI units. Then, negative
(a) the particle will perform SHM of amplitude 2 m (c) The sign of velocity, acceleration and force on the particle
(b) the particle will perform SHM of amplitude 1.5 m when it is 4 cm away from B going towards A are negative
(c) the particle will perform SHM of time period 0.63 s (d) The sign of acceleration and force on the particle when it is at
points B is negative
(d) the particle will have a maximum velocity of 15 m/s

8. Two particles are performing SHM in same phase. It Comprehension Based Questions
means that Passage (Q. 1 to 2)
(a) the two particles must have same distance from the mean A 2 kg block hangs without vibrating at the bottom end of
position simultaneously
a spring with a force constant of 400 N/m. The top end of
(b) two particles may have same distance from the mean
position simultaneously
the spring is attached to the ceiling of an elevator car.
(c) the two particles must have maximum speed simultaneously The car is rising with an upward acceleration of 5 m/s 2
(d) the two particles may have maximum speed simultaneously when the acceleration suddenly ceases at time t = 0 and
the car moves upward with constant speed ( g = 10 m/s 2 )
9. A particle moves along y -axis according to the equation
y (in cm) = 3 sin 100πt + 8sin 2 50πt − 6. Then, 1. What is the angular frequency of oscillation of the block
after the acceleration ceases?
(a) the particle performs SHM
(b) the amplitude of the particle’s oscillation is 5 cm (a) 10 2 rad/s (b) 20 rad/s
(c) the mean position of the particle is at y = − 2 cm (c) 20 2 rad/s (d) 32 rad/s
(d) the particle does not perform SHM
2. The amplitude of the oscillation is
10. The displacement-time (a) 7.5 cm (b) 5 cm (c) 2.5 cm (d) 1 cm
displacement

graph of a particle
executing SHM is Assertion and Reason
2T/4 T
0 Direction (Q. Nos. 1-20) These questions consist of two
shown in figure. Which T/4 3T/4 5T/4 time(s)
of the following statements each printed as Assertion and Reason. While
statement(s) is/are true? answering these questions you are required to choose anyone
(a) The force is zero at of the following five responses.
3T
t= (a) If both Assertion and Reason are true and Reason is
4
4T the correct explanation of Assertion.
(b) The acceleration is maximum at t = (b) If both Assertion and Reason are true but Reason is
4
not the correct explanation of Assertion.
T
(c) The velocity is maximum at t = (c) If Assertion is true but Reason is false.
4
T (d) If Assertion is false but Reason is true.
(d) The PE is equal to KE of oscillation at t = (e) If both Assertion and Reason are false.
2
490 Objective Physics Vol. 1

1. Assertion In SHM, v-x graph is an ellipse, where v is 9. Assertion Simple harmonic motion is an example of one
velocity and x is displacement from mean position. dimensional motion with non-uniform acceleration.
v2 x2
Reason Equation between v and x is 2 + = A2, Reason In simple harmonic motion, acceleration varies
ω 1 with displacement linearly.
which is the equation of an ellipse.
10. Assertion A particle performing SHM at certain instant
2. Assertion In x = 3 + 4 cos ωt, amplitude of oscillation is is having velocity v. It again acquires a velocity v for the
4 units. first time after a time interval of T second. Then, the time
Reason Mean position is at x = 3. period of oscillation is T second.
3. Assertion Bob is released from rest from position A. Reason A particle performing SHM can have same
Given, θ 0 very small. Angular velocity of bob about point velocity at two instants in one cycle.
 g 11. Assertion Velocity versus displacement (from the mean
O is maximum and   θ 0 at point O.
 l position) graph in SHM is a parabola.
O Reason v = ω A 2 − x 2 is quadratic equation in v and x.
θ0
12. Assertion In SHM, to find time taken in moving from
l
one point to another point we cannot apply the relation,
A
Distance
B Time =
Speed
Reason For small angular amplitudes motion of bob is Reason In SHM, speed is not constant.
simple harmonic.
13. Assertion In the x-t graph of a particle in SHM,
4. Assertion If a pendulum is suspended in a lift and lift
acceleration of particle at time t 0 is positive but velocity
accelerates upwards, then its time period will decrease.
is negative.
Reason Value of effective value of g will be x
ge = g + a
5. Assertion Mean position of SHM is the stable
t0
equilibrium position. t
Reason In stable equilibrium, position potential energy
is minimum.
6. Assertion In x = A cos ωt equation of SHM, x is
measured from the extreme position, x = + A. Reason a ∝ −x and velocity is slope of x-t graph.

Reason At time t = 0, particle is at x = + A. 14. Assertion x-t equation of a particle in SHM is given as
x = A cosωt
7. Assertion In a spring block system of length of spring π
and mass both are halved, time period of oscillation will At time t = , potential energy is minimum.

remain unchanged.
Reason In the given equation, the minimum potential
k
Reason Angular frequency of SHM is ω = energy is zero.
m
15. Assertion In x = 5 − 4 sin ωt motion of body is SHM
where, k is spring constant and m is mass of block.
about the mean position x = 5.
8. Assertion Displacement-time equation of a particle
moving along x-axis is x = 4 + 6sin ωt. Under this Reason Amplitude of oscillations is 9.
situation, motion of particle is not simple harmonic. 16. Assertion Time period of a spring-block system is T. If
2
d x length of spring is decreased, time period will decrease.
Reason for the given equation is not proportional
dt 2 Reason If length is decreased, then the block will have
to −x. to travel less distance and it will take less time.
Simple Harmonic Motion 491


17. Assertion If two simple harmonic motions are imposed 4. In y = A sin ωt + A sin ωt +  , match the following.
on a body in mutually perpendicular directions, then the  3
resultant path is a straight line if they have same angular
Column I Column II
frequency.
(A) Motion (p) is periodic but not SHM
Reason Resultant path can be obtained by eliminating t
(B) Amplitude (q) is SHM
and finding x- y relation.
(C) Initial phase (r) A
18. Assertion If amplitude of SHM is increased, time (D) Maximum velocity (s) π
period of SHM will increase. 3

Reason If amplitude is increased, body have to travel (t) ωA/2


more distance in one complete oscillation. (u) None

19. Assertion In the equation. 5. In SHM, match the following columns.


x = 3A sin ωt + 4 A cos ωt,
Column I Column II
maximum speed is 5 ωA.
(A) Acceleration-displacement (p) Parabola
Reason The given equation can be written as graph
x = 5A sin (ωt + 53° ) (B) Velocity-acceleration graph (q) Straight line
(C) Velocity (r) Circle
20. Assertion Average kinetic energy in one oscillation
1 (s) None
during SHM of a body is mω 2 A 2 .
4
6. Velocity-time graph of a particle in SHM is as shown in
1
Reason Maximum kinetic energy is mω 2 A 2 . figure. Match the following columns.
2 v

Match the Columns


1. In SHM, match the following columns. Q
t
P R
Column I Column II
(A) Displacement and velocity (p) Phase difference is zero
(B) Displacement and (q) Phase difference is π
acceleration 2
(C) Velocity and acceleration (r) Phase difference is π Column I Column II
(A) At P (p) Particle is at x = − A
 π
2. In the equation y = A sin ωt +  , match the following (B) At Q (q) Acceleration of particle is
 4 maximum
A
columns. For x = (C) At R (r) Displacement of particle is zero
2 (s) Acceleration of particle is zero
Column I Column II (t) None
(A) Kinetic energy (p) Half the maximum value
(B) Potential energy (q) 3/4 times the maximum
7. A uniform rod of length l is suspended from a point P and
value the rod and is made to undergo small oscillations. Match
(C) Acceleration (r) 1/4 times the maximum the following columns for the time period.
value
Column I Column II
(s) Cannot say anything
(A) If P is the centre of mass (p) Zero
3. In spring-block system, match the following columns. (B) If P is the end point (q)

l
3g
Column I Column II
(A) If k (the spring constant) is (p) Speed will become 16 times (C) Length of simple pendulum (r) l
made 4 times having the time period equal to 3
that of the rod when P is end
(B) If m (the mass of block) is (q) Potential energy will point
made 4 times become 4 times
(C) If k and m both are made (r) Kinetic energy will remain (s) 2 l
4 times unchanged 3
(s) None (t) None
492 Objective Physics Vol. 1

8. In the two block-spring system, force constant of spring Match the following columns.
is k = 6 N/m. Spring is stretched by 12 cm and then left. Column I Column II
Match the following columns.
(A) Frequency with which kinetic (p) 1 / 2 SI unit
energy oscillates
1 kg 2 kg
(B) Speed of particle is maximum (q) 18 π 2 SI unit
at time t
Column I Column II (C) Maximum potential energy (r) 12 SI unit
(A) Angular frequency of (p) 4.8 × 10 −3
SI unit (D) Force constant k (s) 36 π 2 SI unit
oscillation
(B) Maximum kinetic energy of 1 (q) 3 SI unit Entrance Gallery
kg
(C) Maximum kinetic energy of 2 (r) 2.4 × 10 −3 SI unit
2014
kg
1. A particle moves with simple harmonic motion in a
(s) None straight line. In first τ sec, after starting from rest it travels
a distance a and in next τ sec, it travels 2a, in same
9. In case of second’s pendulum, match the following direction, then [JEE Main]
columns. (Consider shape of earth also). (a) amplitude of motion is 3a
Column I Column II (b) time period of oscillations is 8π
(c) amplitude of motion is 4a
(A) At pole (p) T>2s
(d) time period of oscillations is 6π
(B) On a satellite (q) T<2s
(C) At mountain (r) T =2s 2. A 10 kg metal block is attached to a spring of spring
(D) At centre of earth (s) T =0 constant 1000 Nm −1 . A block is displaced from
(t) T =∞ equilibrium position by 10 cm and released. The
maximum acceleration of the block is [Karnataka CET]
10. F-x and x-t graph of a particle in SHM are as shown in (a) 10 ms−2 (b) 100 ms−2 (c) 200 ms−2 (d) 0.1 ms−2
figure. Match the following columns.
3. The displacement of a particle in a periodic motion is
x (m)
 t
4 given by y = 4 cos 2   sin (1000t ). This displacement
 2
8 t (s)
4 may be considered as the result of superposition of n
independent harmonic oscillations. Here, n is [WB JEE]
(a) 1 (b) 2 (c) 3 (d) 4
F (N) 4. When a particle executing SHM oscillates with a
frequency ν, then the kinetic energy of particle [WB JEE]
10 (a) changes periodically with a frequency of ν
x (m) (b) changes periodically with a frequency of 2ν
–1 (c) changes periodically with a frequency of ν/2
(d) remains constant

5. A spring balance has a scale that reads from 0 to 60 kg.


The length of the scale is 30 cm. A body suspended from
this balance and when displaced are released, oscillates
Column I Column II
with a period of 0.8 s, what is the weight of the body
(A) Mass of the particle (p) π /2 SI unit when oscillating? [J&K CET]
(B) Maximum kinetic energy of (q) (160 / π 2) SI unit (a) 350.67 N (b) 540.11 N
particle (c) 311.24 N (d) 300.5 N
(C) Angular frequency of particle (r) ( 8.0 × 10 −3 ) SI unit
2013
(s) None
6. The amplitude of a damped oscillator decreases to
11. x-t equation of a particle in SHM is given as 0.9 times its original magnitude is 5 s. In another 10 s, it
x = 1.0sin (12πt ) in SI units. Potential energy at mean will decrease to α times its original magnitude, where α
1 equals to [JEE Main]
position is zero. Mass of particle is kg.
4 (a) 0.7 (b) 0.81 (c) 0.729 (d) 0.6
Simple Harmonic Motion 493

2012 2011
7. A small block is connected z
12. A metal rod of length L and mass m is pivoted at one end.
to one end of a massless A thin disc of mass M and radius R (< L) is attached at its
v
spring of unstretched centre to the free end of the rod. Consider two ways the
length 4.9 m. The other end 45°
x disc is attached. Case A-the disc is not free to rotate about
of the spring (see the O 10 m P its centre. Case B-the disc is free to rotate about its
figure) is fixed. The system centre. The rod disc system performs SHM in vertical
lies on a horizontal frictionless surface. The block is plane after being released from the same displaced
stretched by 0.2 m and released from rest at t = 0. Then, it position. Which of the following statement(s) is/are
executes simple harmonic motion with angular frequency correct? [IIT JEE]
ω = π /3 rad /s. Simultaneously at t = 0, a small pebble is
(a) Restoring torque in case A = Restoring torque in case B
projected with speed v from point P at an angle of 45° as
(b) Restoring torque in case A < Restoring torque in case B
shown in the figure. Point P is at a horizontal distance of
(c) Angular frequency for case A < Angular frequency for case
10 m from point O. If the pebble hits the block at t = 1 s, B
the value of v is (Take, g = 10 m /s 2 ) [JEE Main] (d) Angular frequency for case A < Angular frequency for
case B
(a) 50 m /s (b) 51 m/s (c) 52 m /s (d) 53 m /s
13. The x-t graph of a particle undergoing simple harmonic
8. This question has Statement I and Statement II. Of the motion is shown below. The acceleration of the particle
four choices given after the statements, choose the one 4
that best describes the two statements. at t = s is [JEE Main]
3
If two springs S 1 and S 2 of force constants k1 and k 2 ,
respectively are stretched by the same force, it is found
1
that more work is done on the spring S 1 than on spring S 2 .
Statement I If stretched by the same amount, work x t(s)
done on S 1 will be more than that on S 2 . (cm) 0 4 8 12
Statement II k1 < k 2 [AIEEE]
–1
(a) Statement I is incorrect, Statement II is correct
(b) Statement I correct, Statement II is incorrect
3 2 π2
(c) Statement I is correct, Statement II is correct, Statement II is (a) π cms−2 (b) − cms−2
the correct explanation for Statement I 32 32
π2 3 2
(d) Statement I is correct, Statement II is correct, Statement II is (c) cms−2 (d) − π cms−2
incorrect explanation of Statement I. 32 32
9. If a simple pendulum has significant amplitude (up 14. A mass M, attached to a horizontal spring, executes SHM
to a factor of 1/ e of original) only in the period between with amplitude A1 .When the mass M passes through its
t = 0 s to t = τ s, then τ may be called the average life of mean position, then a smaller mass m is placed over it and
the pendulum. When the spherical bob of the pendulum both of them move together with amplitude A 2 . The ratio
suffers a retardation (due to viscous drag) proportional to A 
of  1  is [AIEEE]
its velocity with b as the constant of proportionality, the
average life time of the pendulum is (assuming damping  A2 
1/ 2
is small) in sec [AIEEE] M+m  M 
0.693 1 2 (a) (b)  
(a) (b) b (c) (d) M  M + m
b b b 1/ 2
 M + m M
(c)   (d)
10. For a body performing SHM, at a distance A/ 2, then  M  M+m
correct relation between KE and PE will be [O JEE]
15. Two particles are executing simple harmonic motion of
(a) KE is equal to PE (b) KE is 2 times of PE
the same amplitude A and frequency ω along the X-axis.
(c) KE is 3 times of PE (d) KE is half of PE
Their mean position is separated by distance x 0 ( x 0 > A ) .
11. When a spring is stretched by 10 cm, the potential energy If the maximum separation between them is (x 0 + A), the
stored is E. When the spring is stretched by 10 cm more, phase difference between their motions is [AIEEE]
the potential energy stored in the spring becomes π π
(a) (b)
[WB JEE] 3 4
(a) 2E (b) 4E π π
(c) (d)
(c) 6E (d) 10E 6 2
494 Objective Physics Vol. 1

16. A wooden cube (density of wood d ) of side l floats in a 24. Two identical springs are connected to mass m as shown,
liquid of density ρ with its upper and lower surfaces (k = spring constant).If the period of the configuration in
horizontal. If the cube is pushed slightly down and Fig. (i) is 2 s, the period of the configuration in Fig. (ii) is
released, it performs simple harmonic motion of period T, [WB JEE]
then T is equal [AIEEE]
lρ ld
(a) 2π (b) 2π
(ρ − d )g ρg
lρ ld
(c) 2π (d) 2π
dg (ρ − d )g

17. A body of mass 4 kg hangs from a spring and oscillates


with a period of 0.5 s on the removal of the body, the (i) M (ii) M
spring is shortened by [Kerala CEE]
(a) 6.2 cm (b) 0.63 cm (c) 6.25 cm 1
(d) 6.3 cm (e) 0.625 cm (a) 2 2 s (b) 1s (c) s (d) 2 s
2
18. The amplitude of a damped oscillator becomes 1/3rd in 25. The period of oscillation of a simple pendulum of length l
2 s. If its amplitude after 6 s is 1/n times the original suspended from the roof of a vehicle, which moves
amplitude, the value of n is [Kerala CEE] without friction down an inclined plane of inclination α is
(a) 32 (b) 3 2 (c) 3 3 given by [WB JEE]
(d) 23 (e) 33
l l
(a) 2π (b) 2π
19. A particle executing a simple harmonic motion has a g cosα g sin α
period of 6 s. The time taken by the particle to move from l l
the mean position to half the amplitude starting from the (c) 2π (d) 2π
g g tan α
mean position is [Karnataka CET]
1 3 26. For a particle in SHM, if the amplitude of the
(a) s (b) s
4 4 displacement is a and the amplitude of velocity is v, the
1 3 amplitude of acceleration is [MHT CET]
(c) s (d) s
2 2
v2
(a) va (b)
20. The displacement of a particle in SHM various according a
to the relation x = 4 (cosπ t + sinπ t). The amplitude of the v2 v
(c) (d)
particle is [Karnataka CET] 2a a
(a) – 4 (b) 4 27. The average acceleration of a particle performing SHM
(c) 4 2 (d) 8 over one complete oscillation is [MHT CET]
21. If two springs A and B with spring constants 2 k and k, are ω2A ω2A
(a) (b)
stretched separately by same suspended weight, then the 2 2
ratio between the work done in stretching A and B is (c) zero (d) Aω 2
[Karnataka CET]
(a) 1 : 2 (c) 1 : 4 (b) 1 : 3 (d) 4 : 1 28. U is of an oscillating particle and F is the force acting on
it at a given instant. Which of the following is correct?
22. A body is vibrating in simple harmonic motion. If its [MHT CET]
acceleration is 12 cms −2 at a displacement 3 cm, then U 2U
(a) + x = 0 (b) +x=0
time period is [O JEE] F F
F F
(a) 6.28 s (b) 3.14 s (c) 1.57 s (d) 2.57 s (c) + x = 0 (d) +x=0
U 2U
23. A particle of mass m is located in a one-dimensional

potential field where potential energy is given by 29. A particle performing SHM has time period and path
V ( x ) = A (1− cos px ), where A and p are constants. The 3
period of small oscillations of the particle is [WB JEE] length 4 cm. The displacement from mean position at
which acceleration is equal to velocity is [MHT CET]
m m
(a) 2π (b) 2π (a) zero
Ap A p2
(b) 0.5 cm
m 1 AR (c) 1 cm
(c) 2π (d)
A 2π m (d) 1.5 cm
Simple Harmonic Motion 495

30. If a simple pendulum oscillates with an amplitude of 2010


50 mm and time period of 2 s, then its maximum
velocity is [MHT CET] 32. Two simple harmonic motions are represented by,
−1 −1 −1 −1  π
(a) 0.10 ms (b) 0.15 ms (c) 0.8 ms (d) 0.26 ms y1 = 5 [sin 2πt + 3 cos 2πt ] and y 2 = 5sin  2πt + 
 4
31. Ratio of kinetic energy at mean position to potential The ratio of their amplitudes is [Karnataka CET]
energy at A/ 2 of a particle performing SHM is [MHT CET] (a) 1 : 1 (b) 2 : 1
(a) 2 : 1 (b) 4 : 1 (c) 8 : 1 (d) 1 : 1 (c) 1 : 3 (d) 3 : 1

Answers
Level 1
Objective Problems
1. (d) 2. (b) 3. (b) 4. (b) 5. (d) 6. (c) 7. (d) 8. (c) 9. (c) 10. (b)
11. (c) 12. (a) 13. (c) 14. (c) 15. (b) 16. (b) 17. (d) 18. (a) 19. (c) 20. (b)
21. (c) 22. (a) 23. (b) 24. (b) 25. (a) 26. (c) 27. (b) 28. (b) 29. (c) 30. (b)
31. (c) 32. (b) 33. (b) 34. (c) 35. (d) 36. (d) 37. (b) 38. (c) 39. (a) 40. (b)
41. (a) 42. (b) 43. (c) 44. (c) 45. (c) 46. (a) 47. (b) 48. (c) 49. (c) 50. (a)
51. (d) 52. (a) 53. (b) 54. (d) 55. (d) 56. (a) 57. (a) 58. (d) 59. (a) 60. (c)
61. (b) 62. (b) 63. (d) 64. (d) 65. (a) 66. (d) 67. (d) 68. (d) 69. (a) 70. (d)
71. (a) 72. (a) 73. (b) 74. (c) 75. (b) 76. (b) 77. (b) 78. (b) 79. (c) 80. (c)
81. (b) 82. (b) 83. (d) 84. (a) 85. (c) 86. (c) 87. (b) 88. (b) 89. (d) 90. (c)
91. (c) 92. (a) 93. (d) 94. (c) 95. (b) 96. (d) 97. (c) 98. (d) 99. (d) 100. (c)
101. (b) 102. (c)

Level 2
Only One Correct Option
1. (d) 2. (b) 3. (b) 4. (c) 5. (b) 6. (c) 7. (b,c) 8. (a) 9. (b) 10. (b,d)
11. (a) 12. (a,b,c) 13. (d) 14. (c) 15. (a) 16. (c) 17. (b) 18. (b) 19. (c) 20. (d)
21. (a) 22. (c) 23. (c) 24. (b) 25. (a) 26. (a) 27. (c) 28. (c) 29. (c) 30. (c)
31. (b) 32. (b) 33. (d) 34. (b) 35. (b)

More than One Correct Options


1. (a,c) 2. (b,d) 3. (b,c,d) 4. (a,b,c) 5. (b,c) 6. (b,c) 7. (b,c,d) 8. (b,c) 9. (a,b,c) 10. (a,b,c)
11. (a,b,d) 12. (a,c,d)

Comprehension Based Questions


1. (a) 2. (c)

Assertion and Reason


1. (a) 2. (b) 3. (b) 4. (a) 5. (a) 6. (d) 7. (d) 8. (d) 9. (d) 10. (d)
11. (e) 12. (a) 13. (a) 14. (c) 15. (c) 16. (c) 17. (d) 18. (d) 19. (a,b) 20. (b)

Match the Columns


1. (A→q, B→r, C→q) 2. (A→q, B→s, C→p) 3. (A→q, B→r, C→s) 4. (A→q, B→r, C→s, D→u)
5. (A→q, B→s, C→s, D→s) 6. (A→q, B→p,q, C→r,s) 7. (A→t, B→t, C→s) 8. (A→q, B→s, C→p)
9. (A→q, B→t, C→p, D→q) 10. (A→q, B→s, C→s) 11. (A→r, B→p, C→q, D→s)

Entrance Gallery
1. (d) 2. (a) 3. (c) 4. (b) 5. (c) 6. (c) 7. (a) 8. (a) 9. (d) 10. (b)
11. (b) 12. (d) 13. (d) 14. (c) 15. (a) 16. (b) 17. (a) 18. (e) 19. (c) 20. (c)
21. (a) 22. (b) 23. (b) 24. (b) 25. (a) 26. (b) 27. (c) 28. (b) 29. (c) 30. (b)
31. (b) 32. (b)
Solutions
Level 1 : Objective Problems 9. v1 =
dy1 π
= 0.1 × 100 π cos 100 πt + 
1. ω1 A1 = ω2 A2 dt  3
dy
A1 ω2 k2 v 2 = 2 = − 0.1 π sin πt
or = = dt
A2 ω1 k1
π
= 0.1 π cos  πt + 
as ω=
k  2
m
Phase difference of velocity of first particle with respect to
3 3
2. ωA = 3 m/s or ω = = = 30 rad/s the velocity of 2nd particle at t = 0 is
A 0.1 π π π
ω 30 15 ∆φ = φ1 − φ2 = − = −
f = = = Hz 3 2 6
2π 2π π
a 8
3. There is no effect of phase angle on amplitude. Hence, 10. a = ω2 y ⇒ ω = = = 2 rad/s
y 2
A1 10 2
= =
A2 5 1 Now, v max = Aω = 6 × 2 = 12 cm/s
v 10
π 11. v max = aω ⇒ ω = max =
4. ω = rad/s and A = 2 cm a 4
2
π2 Now, v = ω a2 − y 2
∴ Maximum acceleration = ω2 A = cm / s2
2 ⇒ v 2 = ω2 ( a 2 − y 2 )
5. y = a sin (ωt + φ) = a sin  t + φ

v2
T  ⇒ y 2 = a2 −
ω2
2π π
⇒ ψ = 0.5 σιν  τ +  v2
 0.4 2 ⇒ y = a2 −
ω2
π
y = 0.5sin  5 πt + 
 2 52
= 42 − = 2 3 cm
= 0.5cos 5πt (10 / 4)2

6. y = a sin

t 12. v max = aω = 3 × 100 = 300 units
T 13. Velocity in mean position v = aω, velocity at a distance of
a 2π
⇒ = a sin ⋅t half amplitude
2 T
2π π a2
⇒ sin t=
1
= sin v ′ = ω a2 − y 2 = ω a2 −
T 2 4 4
2π π =
3
aω =
3
⇒ t= v
T 4 2 2
T 1 1 1
⇒ t= 14. = +
8 k k1 k2
A 2 πt 1
7. y = A sin ωt ⇒ = A sin 15. k ∝
2 T l
Τ
⇒ τ= 16. | a | = ω2 x
12
12 = ω2 ( 3)
8. Equation of motion, y = a cosωt

a ∴ ω = 2 rad/s =
⇒ = a cosωt T
2
π ∴ T = π second
⇒ ωt =
3 or 3.14 s
2 πt π 2π π π
⇒ = 17. ω = = s. Therefore, y = A sin t. Now, put x =1 s and
T 3 T 4 4
π then x = 2 s.
×T 18. ω2 A = a0 ...(i)
⇒ t= 3
2π ωA = v 0 ...(ii)
4 2 v2
= = s Solving we get, A= 0
3× 2 3 a0
Simple Harmonic Motion 497

19. v max = ωA = (200) (0.25) = 50 cm/s 32. a = 6 cm, ω =100 rad/s


ω =100 π 1
20. K max = mω2 a 2
2π 2
∴ T= = 0.02 s 1
ω = × 1 × (100)2 × (6 × 10−2 )2 =18 J
2
21. v = ω A 2 − x 2 π
33. y = y1 + y 2 = 2A sin  ωt + 
= 2 (60)2 − ( 20)2 = 113 mm/s  4
1
k 16 E = mω2 ( 2A )2 = mω2 A 2
22. ω = = 2
m 1 1
= 4 rad/s 34. KE = K ( A − x 2 )
2
(a = A)
2
1 1 3E
Now, K max = mω2 A 2 Put KA 2 = E and KE =
2 2 4
1
= × 1 × ( 4)2 × ( 5 × 10−2 )2 35. T = 2π
l
and T ′ = 2 π
l
2 g g + g 
= 2 × 10−2 J  
 4
1 2
23. Energy in SHM, E = Umean + mω2 A 2 or T′= T = 2s
2 5
i.e. E depends on ω and A both. m
36. T = 2π
24. Frequency of oscillation of kinetic energy is doubled, i.e. 2ω ρAg
2π π
∴ T= = 37. T = 2π
l
. Here, g e = effective value of g, which is zero in a
2ω ω ge
1 1
25. E = mω2 A 2 = m( 2 πf )2 A 2 freely falling lift. Hence, T = ∞
2 2
1 k
= 2 π 2 f 2mA 2 38. f = or f ∝ k
2π m
= 2 π 2 ( 20)2 (0.1) (0.05)2 ki = k1 + k2
= 2 J (approximately) and k f = 4( k1 + k2 ) = 4ki
E 1 k k
26. U = K or U = 39. f = or f ∝
2 2π m m
Kx 2 =  KA 2 
1 1 1 Here, value of ke will become 3k.

2 22  3k
∴ f ∝
A m
i.e. at x=± ,U = K
2 M
40. g ∝ 2 , mass and radius both are doubled. Hence, g will
and this situation will occur for four times in one complete R
period. become 1/2 times.
1
27. If at any instant displacement is y, then it is given that Now, T ∝ and time period of second’s pendulum is 2 s
g
1
U= ×E on earth. Hence, time period of second’s pendulum on this
2 planet will become 2 2 s.
mω y = ×  mω2 a 2 
1 1 1
⇒ 2 2 1
41. k ∝ and f ∝ k
2 2 2 
l
a 6 1
⇒ y= = ∴ f ∝
2 2 l
= 4.2 cm m 0.2 π
42. T = 2 π = 2π = s
28. a = 6 cm, ω =100 rad/s k 80 10
1 43. In case of spring-block system, time period does not change
K max = mω2 a 2
2 because restoring force is still kx.
1 44. Velocity, v = ω A 2 − x 2 and acceleration = ω2 x
= × 1 × (100)2 × (6 × 10−2 )2
2
Now given, ω2 x = ω A 2 − x 2
=18 J
30. In SHM at mean position, kinetic energy will be maximum ⇒ ω2 ⋅1 = ω 22 − 12
and potential energy will be minimum. Total energy is
⇒ ω= 3
always constant.
2π 2π
1 ∴ T= =
31. Total energy = mω2 a 2 = constant ω 3
2
498 Objective Physics Vol. 1

d 2y 56. Ta = 2π
m
45. On comparing with standard equation + ω2 y = 0,
dt 2 k
2π m
We get ω2 = K ⇒ ω= = K Tb = 2 π
T ( k / 2)

⇒ T= m
K Tc = 2 π
2k
46. Comparing given equation with standard equation, 1 l
x = a cos(ωt + φ) we get, a = 0.01 and ω = π 57. k ∝ ⇒ 1 =n
l l2
⇒ 2πn = π ⇒ n = 0.5 Hz
 n 
∴ l1 =  l
47. When a little mercury is drained off, the position of centre of  n + 1
mass of ball falls (with respect to fixed end) so that effective
n + 1
length of pendulum increases hence T increases. or k ′ =  k
 n 
l
48. In stationary lift, T = 2π mn
g ∴ T = 2π
k(n + 1)
In upward moving lift,
k
l 58. ω = , here m1 = m2
T ′ = 2π m2
( g + a)
k×k k
(a = acceleration of lift) 59. ks = =
k+ k 2
T′ g g 4
⇒ = = = kp = k + k = 2k
T g+a g + g  5
  1 ks
 4
= π M
ns 2
2T ∴
⇒ T′= np 1 kp
5
2π M
49. Remains the same because time period of simple pendulum
ks k/2 1
T is independent of mass of the bob. = = =
50. When lift falls freely, effective acceleration and frequency of kp 2k 2
oscillations will be zero. YA
60. k =
g eff = 0 ⇒ T ′ = ∞, L
1 k 1 YA
Hence, frequency = 0 ∴ f = =
2 π m 2 π mL
51. A = 0.04 m, ω A = 0.3 m/s
2 2

∴ ω = 2.74 rad/s I (ml 2 / 3)


61. T = 2 π = 2π
2π mgl / 2 (mgl / 2)
Now, T= = 2.29. s
ω 2l
= 2π
m 3g
52. T = 2π or T ∝ m
k 62. x = A sin ωt + B cos ωt
1
Mass has reduced to th the initial mass. Hence, time
4
period will remain half.
A2 + B2
I l B
54. 2 π = 2π
MgL g
I ( 3 / 2 MR 2 ) 3 φ
or l= = = R A
ML MR 2
 A B 
= A2 + B2  sin ωt + cos ωt 
 A 2 + B 2 A2 + B2 
55. g ′ = g 2 + a 2
a = A 2 + B 2 sin (ωt + φ)
B
where, tan φ =
A
l
63. T = 2π or T 2 ∝ l
g
i.e. T-l graph is a parabola.
g g' = g2 + a2 arc = sin θ
Simple Harmonic Motion 499

64. Acceleration is maximum when displacement is maximum. 74. Potential energy graph is parabolic with its minimum value
Force is zero when displacement is zero and potential at mean position.
energy is maximum when displacement is maximum. 1
75. Force constant, k ∝
65. x = 3 sin 2t + 4 cos 2t. From given equation, Length of spring
4 a kA L
∴ =
k αL
k
∴ kA =
α
k
Similarly, kB =
1−α
3
1
a1 = 3, a2 = 4 76. k ∝
l
π
and φ= 1
77. k ∝ hence, k of individual halves will be 16 N/m. When
2 l
∴ a = a12 + a22 they are connected in parallel, effective value of k will
become 32 N /m.
= 32 + 42 = 5
1 1 1 k
78. = + or ke =
⇒ v max = aω = 5 × 2 = 10 ke k k 2
66. Using acceleration, A = − ω2 x 79. Reduced mass of two blocks,
At −x max , A will be maximum and positive. m1m2
µ= = 2 kg
67. Acceleration = − ω2 y. So, F = − mω2 y m1 + m2

y is sinusoidal function. 1 k
Now, f =
So, F will also be sinusoidal function with phase difference 2π µ
π. 1 300
=
T 2π 2
68. At time ; v = 0
2 ≈ 2 Hz
∴ Total energy = Potential energy 80. From mean position to extreme position body is
1 1
69. f = = = 25 Hz decelerated and from extreme position to mean position
T 004. body is accelerated.
F 8
70. From graph, slope K = = = 4 m
x 2 81. T = 2π
k
m
T = 2π ∴ Dimensionally
m
is equal to T 2 .
k k
0.01
⇒ T = 2π = 0.31 s l l
4 82. T = 2π . For T to be constant value of should remain
g g
71. Suppose x = A sin ωt and y = A cosωt g l
constant. So, if g moon = earth , then l moon = earth .
Then, by squaring and adding these two equations we get 6 6
x2 + y 2 = A2 mg 0.2 × 10
83. At equilibrium, x = = = 0.01 m = 1 cm
x k 200
72. x = a sin ωt orsin ωt = ...(i)
a 1 k
f =
π
y = b sin  ωt +  or y = b cosωt 2π m
 2 1 200
= ≈ 5.0 Hz
or cosωt =
y
...(ii) 2π 0.2
b π
84. φ1 =
Squaring and adding Eqs. (i) and (ii), we get 2
x2 y 2 and φ 2 = π
+ = 1 which is the equation of ellipse.
a2 b2 85. a and s are always in opposite directions.
π
73. y1 = a cosωt = a sin  ωt + 
86. After attaining terminal velocity, ball will move with
 2 constant velocity.
y 2 = a sin ωt 87. a = − ω2 x
π ∴
x 1
= − 2 = constant
∴ ∆φ = ωt + − ωt
2 a ω
π mg 5 × 10
= = 90° 88. A = = = 0.25 m
2 k 200
500 Objective Physics Vol. 1

89. At x = 0, F = 0. For x > 0, force is in negative direction and for 4. At x = 0, U = 0. Therefore, total mechanical energy is equal to
x < 0, force is in positive x-direction. Therefore, motion of the maximum kinetic energy.
the particle is periodic about mean position x = 0. ∴
1 2
mv max = 36
92. The amplitude is a maximum displacement from the mean 2
position. 72 72
or v max = = = 6 m/s
93. F = − kx m 2
1
94. a = − ω2 x ⇒
 a
= ω 2 5. kA 2 = (0.04 − 0.01) J = 0.03 J
x 2
0.06 0.06
95. Ball will execute SHM inside the tunnel with time period ∴ k= 2 = = 150 N/m
A (0.02)2
R
T = 2π = 84.63 min 1
g 6. mω2 A 2 = 8 × 10−3 J
2
Hence, time to reach the ball from one end to the other end 1
84.63 ∴ × 0.1 × ω2 × (0.1)2 = 8 × 10−3
of the tunnel, t = = 42.3 min. 2
2 or ω = 4 rad/s
l λ γ Upthrust ρVg
98. T = 2π ⇒ = = constant 7. M e = Mass of container + =M+ = M + ρV
g T2 4 π2 g g
1 n l T
99. Frequency, n ∝ ⇒ 1 = 2 8. means 120° in reference circle. Maximum possible value
l n2 l1 3
of v is near mean position between P and Q.
l 2 n22 32 9
⇒ = = =
l1 n12 22 4
m
100. T = 2π or T ∝ m
K B A A
Mass is increasing therefore time period will increase.
60° 60°
101. X = A sin ωt or 10 = 20 sin ωt
30° 30°
Q P

3A
t=0 t=t
π
∴ ωt =  
6
π
t=

Distance
The desired time will be 2t. ∴ v=
Time
π π T 12
or = = = = 2s 3A 3 3A
3ω 3( 2 π / T ) 6 6 = =
(T / 3) T
102. ωA = constant and ω2 A is made two times.
9. ∴ l′ = 2 l
This is possible when ω is doubled and A is halved.
g
Level 2 : Only One Correct Option
F 8
1. From graph, slope k = = =4
x 2
m 0.1
T = 2π ⇒ T = 2π = 0.31 s
k 4 g 2g
2
d x
2. Comparing with 2 = − ω2 x
dt 10. The motion is simple harmonic only inside earth. Further
We have ω = 2 rad/s 1 GMm 3 GMm
mv 2 = − +
v max = ωA = 2 m/s 2 ( R + h) 2 R
amax = ω2 A = 4 m/s2 GM ( R + 3h)
v=
m m R( R + h)
3. T1 = 2 π but T2 = 2 π .
k + ρAg k 11. Since, the cube is half immersed. The density of cube
Hence, T1 < T2 should be half the density of water, i.e. 500 kg/m3 .
Simple Harmonic Motion 501

π
T = 2π
m
( k = ρw Ag ) 15. y1 = A sin  ω1t + 
ρw Ag  2
π
π a3 × ρ y 2 = A sin  ω2t + 
= 2π  2
5 ρw × a 2 × g
π π
Now, ω1t + = ω2t −
aρ 1
∴ = 2 2
gρw 100 π
or t=

a
=
1 ω2 − ω1
20 100 π
=
or a = (0.2) m ( 2 π / T2 ) − ( 2 π / T1 )
Now, m = a 3ρ = 4 kg T1T2
=
12. Maximum kinetic energy = energy of oscillation in SHM 2(T1 − T2 )

1
8 = kA 2 3× 7 21
= = s
2 2 ( 7 − 3) 8
∴ kA 2 = 16 ...(i) m
16. T = 2π
m k
Further, 2π =4
k 1
or T∝
1 4 k
∴ =
k π2 4 9
T has increased times. Hence, k ′ = k
π2 3 16
or k= ...(ii)
4 When both are combined,
From Eqs. (i) and (ii), we get 25
knet = k + k ′ = k
k ≈ 2.4 N/m and A ≈ 2.53 m 16
Maximum acceleration of the particle will be 25
or times
k 16
amax = ω2 A = A
m Hence, new time period will become 4/5 times of 3 s or 2.4 s.

= 
2.5  17.
 (2.53) ≈ 6.3 m/s
2
 1  kx kx
kx
2 2
k 2.5 O
= ⋅A = × 2.53
m 1 N M
= 6.3 m/s2
O' 45° 45°
dU
13. U = 5x 2 − 20x , = 10x − 20,
dx
dU OO ′ = x (say)
F=− = ( −10x + 20) x
dx Then, O′ M = O′ N ≈
2
Let us suppose, x = ( X + 2). Then, F = −10X , F = 0 at X = 0 or
x
x = 2, i.e. x = 2 m is the mean position about which particle is i.e. elongation in spring B and C is , while compression in
2
in SHM. k =10
spring is x.
m 0.1 π
∴ T = 2π = 2π = s. Net restoring force,
F = −  kx + cos 45°  = − 2kx
k 10 5 2kx
At mean position kinetic energy is maximum and force  2 
acting on particle in SHM is variable. F 2k
∴ a= =− x
m m
14. v = ω A 2 − x 2 (in SHM)
or v = ω A − ω x
2 2 2 2 2 T = 2π  x = 2π m
 a 2k
Comparing the given equation with this equation, we get 1
18. U = Umean + kX 2
ω2 = 9 2
2π 1
∴ ω= 3= Given, Umean = E1 and kA 2 = E2 = maximum kinetic energy
T 2
2π at mean position.
∴ T= units, ω2 A 2 = 144 2
3 3A 1  3A 
∴ X= ,U = E1 + k  
In SHM, 2 2  2 
∴ A = 4 units, | a | = ω2 x = ( 9)( 3) = 27 units 3
= E1 + E2
Displacement ≤ distance 4
502 Objective Physics Vol. 1

19. Particle starts from rest. Hence, x = A cosωt 26. ω2 A = g


a = A − A cos(ω × 1) g A
ω= or T = 2π
A−a 
= 1 − 
a A g
or cosω =
A  A 1 1
27. mv 2 = kx 2
a + b = A − A cos(ω × 2) 2 2
= A − A( 2 cos2 ω − 1) m
∴ x =v
2 k
= 2A − 2A 1 − 
a
 YA
A 28. k ′ =
L
2a 2
Solving we get, A= kk ′
3a − b ke =
k + k′
m 0.1
20. T = 2 π = 2π = 0.198 s m
k 100 k' T = 2π
ke
From x = A sin ωt or 5 = 10 sin ωt
π 29. ( M )(ωA ) = ( M + m) v (Pi = Pf )
ωt =
6 MωA
∴ v= = ω′ A ′
2π π k ( M + m)
or t=
T 6  M  ω
T ∴ A′ =  ⋅
or 2t =  M + m  ω′
6
k
The desired time in the question will be
T
+ 2t Now, ω=
2 m
1
or
T T 2T
+ = or ∝
2 6 3 m
ω M+m

=   (0.198) = 0132 ∴ =
2
. s
 3 ω′ M
A T  M 
21. From e to 2e or to A, t = ∴ A ′ =   A
2 6  m+ M 
T 2π m 1 1
∴ Time period of oscillation = 2t = = 30. kA 2 = × 2 × 106 × (0.01)2 = 100 J
3 3 k 2 2
22. µg > maximum acceleration in SHM > ω2 A At mean position
U = 60 J
ω2 A
∴ µ> and K =100 J
g
At extreme positions
23. At its lowermost point spring is stretched by 8 cm or U =160 J
8 × 10−2 m. and K =0
∴ Fnet = kx − mg m 1 α
31. T = 2π or T ∝ or k = 2
= ( 2 × 103 × 8 × 10−2 ) − (10 × 10) = 60 N k k T
12 × 10  x = mg  Now, ke = k1 + k2
24. x12 = = 0.12m  
1000  k  α α α
∴ = +
4 × 10 T 2 T12 T22
x4 = = 0.04 m
1000 or T −2 = T1−2 + T2−2
∴ A = x12 − x 4 = 0.08 m 32.
25. g e = g − a

–a = g sin α
A
g sin α
60°
60° A/2
g cos α
g A
where, a = acceleration of roof
2
| g e | = g cos α
L
∴ T = 2π
g cos α
Simple Harmonic Motion 503

2π 5π 
= A sin 
1
33. kA 2 =1.0 J t+ 
2 T 6 
2 2 25 X = A cos(ωt + 60° )
∴ k= = = N/m or
A 2 (0.4)2 2 2π π
= A cos  t + 
m 2 T 3
T = 2π = 2π
k ( 25 / 2) 3. k x 0 = mg
4π mg 1 × 10
= s ∴ x0 = =
5 k 500
34. For y 2 , = 0.02 m = 2cm
10 A So, equilibrium is obtained after an extension of 2 cm of at a
length of 42 cm. But, it is released from a length of 45 cm.
∴ A = 3cm = 0.03 m
k
(b) v max = ωA = A
m
 500 
10c
=  (0.03) = 0.3 5 m/s
 1 
A = (10) + (10c )
2 2
or 10 1 + c = 40
2 = 30 5cm/s

∴ c = 15 (c) amax = ω2 A

=   ( A ) = 
mg = kx k 500 
 (0.03) =15 m/s
35. 2
 m  1 
mg
∴ k=
x (d) Mean position is at 42 cm length and amplitude is 3 cm.
Hence, block oscillates between 45 cm length and 39cm.
M+m ( M + m) x
∴ T = 2π = 2π Natural length 40 cm lies in between these two, where
k mg
elastic potential energy = 0.
4. µ = Reduced mass
More than One Correct Options m1 m2
ma a = = 2 kg
1. tan θ = = m1 + m2
mg g
µ 2
T = 2π = 2π
k 800
θ T π
= s = T3 = T6
10
Pseudo force 1
= ma A∝
θ m
A3 6 2
Fnet ∴ = =
A6 3 1
mg
∴ A3 = 4 cm and A6 = 2 cm
−1  a (P6 )max = m6 (v 6 )max
θ = tan  
g = m6 (ωA6 )
 2π 
T = Fnet = (6)  −2
 ( 2 × 10 )
 π/10 
= m a2 + g 2
= 2.4 kg/m
2. X = A sin (ωt + 150° )
5. v or KE = 0 at y = ± A
v or KE = maximum at y = 0
F or a is maximum at y = ± A
A
P F or a is zero at y = 0
6. v = ω A 2 − x 2
60° Q
cos v2 X2
A ∴ + 2 = A2
2 ω 2
(1)
ω = 2π i.e. v - x graph is an ellipse.
T
a = − ω2 x
i.e. a - x graph is a straight line passing through origin with
sin negative slope.
504 Objective Physics Vol. 1

7. a = 0 at x = 0.5 mand particle is released from x = 2m (c) At t =


T
, corresponds to mean position, so velocity will
Hence, 4
be maximum at this position.
A = 2 − 0.5 =1.5 m
2T T
ω2 = 100 (d) At t = = , corresponds to extreme position, so KE = 0
4 2
∴ ω =10 rad/s and PE = maximum.
2π 2π 11. Let the equation of a SHM is represented as x = a sin ωt
T= = = 0.63 s
ω 10
Assume mass of the body is m.
v max = ωA = (10)(1.5) =15 m/s
(a) Total mechanical energy of the body at any time t is
8. Two particles shown in figure are in same phase, although 1
they have distances from the mean position at t = 0 E = m ω2 a 2 ...(i)
2
–A1 +A1
Kinetic energy at any instant t is
x=0 +A1/2 2
mv 2 = m   Qv = dx 
1 1 dx
K =
2 2  dt   dt 
–A2 +A2
1
= m ω2 a 2 cos2 ωt [Qx = a sin ωt ]
x=0 +A2/2 2
1
9. The given equation can be written as ⇒ K max = mω2 a 2 = E
2
y = 3 sin 100 πt + ( 4 − 4 cos100 πt ) − 6
[Q for k max , cos ωt =1] ...(ii)
= 3 sin 100 πt + 4 sin (100 πt + π/2) − 2
(b) KE at any instant t is
or y = 5 sin (100 π − 53° ) − 2
1
3 K = m ω2 a 2 cos2 ωt
53° 2
1
( K av ) for a cycle = m ω2 a 2 [(cos2 ωt ) av ] for a cycle
2
0 + 1
= m ω2 a 2 
1
2  2 
4 1 K
= m ω2 a 2 = max [from Eq. (ii)]
y max = 5 − 2 = 3 4 2
y min = − 5 − 2 = − 7 dx
(c) Velocity = v = = a ωcos ωt
y + y min dt
Mean position = max = − 2 cm v + v min
2 v mean = max
a 2
a ω + ( − aω )
= = 0 [For a complete cycle]
–A v x=0 +A 2
x v max ≠ v mean
∴ Velocity is negative. v12 + v 22 0 + a 2 ω 2 aω
(d) v rms = = =
(d) At mean position, velocity is maximum. Just after few 2 2 2
seconds acceleration becomes positive. So, v max
displacement will become negative. Hence, the velocity ⇒ v rms =
2
should be negative.
12. Consider the diagram.
10. Consider the diagram
u=0 v=positive
v=0
B O C A
Displacement

T/4 (a) When the particle is 3cm away from A going towards B,
0 2T 3T T Time (s) velocity is towards AB i.e. positive.
4 4
4T In SHM, acceleration is always towards mean position
4 O in this case.
From the given diagram, it is clear that Hence, it is positive.
3T (b) When the particle is at C, velocity is towards B hence
(a) At t = , the displacement of the particle is zero. positive.
4
Hence, the particle executing SHM will be at mean (c) When the particle is 4 cm away from B going towards A
position, i.e. x = 0 . So, acceleration is zero and force is velocity is negative and acceleration is towards mean
also zero. position O hence negative.
4T (d) Acceleration is always towards mean position O. When
(b) At t = , displacement is maximum, i.e. extreme
4 the particle is at B, acceleration and force are towards
position, so acceleration is maximum. BA that is negative.
Simple Harmonic Motion 505

19. 4A 5A
Comprehension Based Questions
k 400
1. ω = = = 10 2 rad/s
m 2
2. Kx 0 = ma
ma 2 × 5
x0 = = m 53° 3A
K 400
= 0.025 m = 2.5 cm 20. If v = ωA sin ωt
∴ A = x 0 = 2.5 cm Then, v max = ωA
1 1
Assertion and Reason ∴ K max = mv max
2
= mω2 A 2
2 2
1. v = ω A 2 − x 2 1
Further, < K > = < mω2 A 2 sin 2 ωt >
2
v2 x2
∴ + = A2 1
ω2 1 = mω2 A 2 < sin 2 ωt >
2
2. X = x − 3 = 4cosωt 1
But, < sin 2 ωt > = in one oscillation
X = 0 at x = 3 2
3. Maximum angular velocity = ωθ0 ∴
1
< K > = mω2 A 2
 1 4
g  g
= ( 2 π f ) θ0 = ( 2 π )   θ0 =   θ0
 2π l   l  Match the Columns
l 1. Suppose x = A sin ωt
4. T = 2π
ge dx
Then, v= = ωA cos ωt
Here, g e = g + a, if the lift accelerates upwards. dt
dv
6. In any equation of SHM, x is always measured from the and a= = − ω2 A sin ωt
mean position. dt
1 A2 3 3
7. k ∝ 2. v = ω A 2 − = ωA = v max
length of spring 4 2 2
If length is halved, k will become two times. 2
 3 3
m ∴ K =  times, K max = K max
From T = 2π  2  4
k
1
Time period T will remain half for the given conditions. About PE, it will be times its maximum value only when
4
8. Motion of particle is simple harmonic, but mean position is potential energy at mean position is zero.
at x = 4. Amplitude is 6. The particle will oscillate between
k 1
x =10 and x = − 2. 3. ω = , v = ω A 2 − X 2 , PE = kx 2
m 2
9. Simple harmonic motion is not always one-dimensional. In
case of angular simple harmonic motion it is 2-D also. 4. The given equation,
2π 
10. tPQ = T (given) y = A sin ωt + A sin  ωt +  can also be written as
 3 
But tPQ is not the time period of oscillation. π π
y = 2A sin  ωt +  ⋅ cos  
v v  3  3
→ →
π
+A = A sin  ωt + 
–A Q O P  3
Now, we can see that this is SHM with amplitude A and
11. v versus x graph is an ellipse. π
initial phase.
13. At given time x is negative. Therefore, acceleration is 3
positive (a ∝ −x). Further slope is negative. Therefore, 5. a = − ω2 x , i.e. a-x graph is straight line passing through
velocity is negative.
origin.
14. At given time x = 0, i.e. body is at mean position. At mean
If v = v 0 sin ωt
position potential energy is always minimum. But this dv
minimum potential energy may or may not be zero. then, a= = v 0ω cos ωt
dt
15. Amplitude is 4.
1 1 = v 0ω 1 − sin 2 ωt
16. Force constant of a spring, k ∝ and T ∝
l k v2
= v 0ω 1 −
∴ T∝ l v 02
18. Time period does not depend on the amplitude of a = ω v 02 − v 2
oscillation.
506 Objective Physics Vol. 1

So, a-v graph is neither a straight line nor a parabola.


Further, acceleration and velocity time graphs are sine or
Entrance Gallery
cosine functions. 1. In SHM, a particle starts from rest, we have
6. At P, velocity is zero, but just after few seconds velocity is i.e. x = A cos ωt , at t = 0, x = A
negative. Therefore, at P particle is at x = + A. At this point When t = τ, then
acceleration of the particle will be maximum. On similar x=A−a …(i)
ground we can say that When t = 2τ, then
At Q, x=−A x = A − 3a …(ii)
At R , v is maximum, i.e. particle is at mean position, where On comparing Eqs. (i) and (ii), we get
acceleration is also zero. A − a = A cosωτ
7. T = 2π
I
. Here l is distance of point P from centre of A − 3a = A cos 2 ωτ
mgl As, cos 2ωτ = 2 cos2 ωτ −1
mass. A − 3a A − a
2

When P is the centre, l = 0. Therefore, T = ∞, i.e. rod will not ⇒ = 2   −1


A  A 
oscillate.
2A 2 + 2a 2 − 4Aa − A 2
ml 2 =
When P is end point, I = A2
3
A − 3aA = A + 2a − 4Aa ⇒ 2a 2 = aA
2 2 2
2l
∴ T = 2π
3g Now, A − a = A cosωτ [Qτ = t]
2π π
2l L 2l ⇒ cos ωt =1/2 ⇒ t= ⇒ T = 6π
or 2π = 2π or L = T 3
3g g 3
2. We know that spring represents SHM.
k So, the restoring force is proportional to displacement
8. ω =
µ i.e. F = − mω2 y …(i)
Here, µ = reduced mass F = − ky …(ii)
m1m2 2 where, k = force constant of the spring
= = kg
m1 + m2 3 m =10 kg , k =1000 N/m
6 A = 10cm = 0.1 m
∴ ω= = 3 rad/s
2/ 3 On comparing both equations, we get
Amplitude 12 cm distributes in inverse ratio of mass. k k 1000
ω2 = ⇒ ω= = =10 rad/s
∴ A1 = 8 cm and A2 = 4 cm m m 10
Now, maximum kinetic energy and acceleration in SHM
1 amax = − ω2 ⋅ y where, ω2 is constant
K 1 = m1ωA12
2 = − 102 × (0.1 ) = − 10 m/s2 = 10 ms−2
1
= × 1 × 3 × (8 × 10−2 )2 = 9.6 m J
3. Given, y = 4 cos2   ⋅ sin (1000t )
t
2  2
1
K 2 = × 2 × 3 × ( 4 × 10−2 )2 = 4.8 m J
= 2 × 2 cos2   ⋅ sin (1000t ) Q2 cos2 t = (1 + cos t )
t
2
 2  2 
9. On a satellite and at centre of earth g ′ = 0
= 2(1 + cos t )sin (1000t )
∴ T =∞
= 2 sin (1000 t ) + 2 cos t ⋅ sin (1000 t )
At pole, value of g is more than the normal value. Hence,
T < 2s = 2 sin (1000 t ) + 2 sin (1000 t ) ⋅ cos t
10. T = 8 s = 2 sin (1000 t ) + sin (1000 t + t ) + sin (1000 t − t )
[Q2 sin A ⋅ cos B = sin ( A + B ) + sin ( A − B )]
2π π
∴ ω= = rad/s = 2 sin (1000 t ) + sin (1001 t ) + sin ( 999 t ).
T 4
So, n = 3
K = − slope of F-x graph =10 N/m
4. We know that,
m
From, T = 2π , 1
k Kinetic energy, K = mu2
2
m dy
We have 8 = 2π where, u= = ωa cos ωt
10 dt
160 1
∴ m= kg So, K = mω2 a 2 cos2 ωt
π2 2
1 1 160 π 2 Hence, kinetic energy varies periodically with double the
K max = πω2 A 2 = × 2 × ×4 frequency of SHM. So, when a particle executing SHM
2 2 π 16
oscillates with a frequency ν, then the kinetic energy of
. × 10−3 J
= 80 particle changes periodically with a frequency of 2ν.
Simple Harmonic Motion 507

5. Time period of the spring mass system, L


12. τ A = τ B = (mg sin θ + MgL sin θ) O
m 2
T = 2π …(i)
k = Restoring torque about point O θ
From spring, mg = kx In case A, moment of inertia will be more.
mg
mg 60 × 9.8 Hence, angular acceleration(α = τ /I) will be less.
⇒ k= = = 1960 N/m …(ii)
x 30 × 10−2 Therefore, angular frequency will be less.
2π  π 
From Eqs. (i) and (ii), we get 13. Given, T = 8 s,ω = =   rads−1 Mg
m 0.8 × 0.8 × 1960 T  4
0.8 = 2π ⇒ =m
1960 4p2 We have, x = A sinωt
 π2  π 
m = 31.81 kg ⇒ w = mg ∴ a = − ω2 x = −   sin  t 
 16  4 
∴ w = 31.81 × 9.8 = 311.24 N
4
6. Amplitude decreases exponentially. In 5 s, it remains On substituting t = s, we get
0.9 times. Therefore, in total 15 s it will remains (0.9) (0.9) 3
(0.9) = 0.729 times its original value.  3 2
a = − π  cm s−2
7. Time of flight of projectile,  32 
2ν sin θ Fnet = 0
t= ( θ = 45° ) 14. At mean position,
g
∴ By conservation of linear momentum,
gt 10 ×1
∴ ν= = = 50 m/s Mv1 = ( M + m)v L
2sin θ 2 ×1/ 2
⇒ Mω1 A1 = ( M + m) ω2 A2
8. As, no relation between k1 and k2 is given in the question, k k
ω1 = ⇒ ω2 =
that is why, nothing can be predicted about Statement I. m M+m
But as in Statement II, k1 < k2 .
A1 M+m
Then, for same force On solving, =
A2 M
F F2 1
W =F⋅x =F⋅ = ⇒ W ∝ , i.e. W1 > W2 15. x1 = A sin(ωt + φ)
k k k
But for same displacement, x 2 = A sin(ωt + φ)
1 1 ⇒ x 2 − x1 = A [sin(ωt + φ2 ) − sin(ωt + φ1 )]
W = F ⋅ x = kx ⋅ x = kx 2 ⇒ W ∝ k , i.e. W1 < W2
2ωt + φ1 + φ2   φ − φ1 
2 2 = 2A cos   sin  2 
Thus, in the correct of Statement II, Statement I is incorrect.  2   2 
9. For damped harmonic motion, The resultant motion can be treated as a simple harmonic
φ − φ1 
ma = − kx − mbv or ma + mbv + kx = 0 motion with amplitude 2 Asin  2 .
 2 
Solution to above equation is

bt Given, maximum distance between the particles = x 0 + A
k b2
x = A0e 2 sin (ωt ) with ω2 =
− ∴ Amplitude of resultant SHM = x 0 + A − x 0 = A
m 4
φ − φ1 
where, amplitude drops exponentially with time. ⇒ 2A sin  2  =A
 2 


i.e. A τ = A0e π
2
φ2 − φ1 =
3
Average time τ is that duration when amplitude drops by
16. Let at any instant, cube is at a
63%, i.e. becomes A0 /e .

depth x from the equilibrium
A0 − bτ 2 position, then net force acting on
Thus, Aτ = = A0e 2 or = 1 or τ =
e 2 b the cube = upthrust on the portion l x
X
1 of length x
10. Potential energy PE of body = mω2 y 2
2 F = − ρl 2 xg = − ρl 2 gx …(i)
2 l
PE at A/ 2 = mω2 
1 A  1
 = mω A
2 2
Negative sign shows that, force is
2  2 4 opposite to x. Hence, equation of
1 SHM
Kinetic energy of a body = mω2 ( A 2 − y 2 )
2 F = − kx …(ii)
1   A   1
2
On comparing Eqs. (i) and (ii), we get
KE at A/ 2 = mω2  A 2 −    = mω A
2 2
2   2  4
 k = ρl 2 g
1
11. For 1st situation, E = k (10 × 10−2 )2 …(i) m
2 ⇒ T = 2π
k
1
For 2nd situation, E ′ = k ( 20 × 10−2 )2 …(ii)
2 l 3d ld
= 2π = 2π
On solving Eqs. (i) and (ii), we get E ′ = 4E ρl 2 g ρg
508 Objective Physics Vol. 1

m l
17. Time period, T = 2π ⇒ mg = kx 25. Time period, T = 2π a
=
k g eff g
s in
x x α
∴ T = 2π ⇒ (0.5) = 4 π ×
2 2
g 10 T = 2π
l g cos α g
g sin
g cosα α α
(0.5)2 × 9.8
=x ⇒ x = 0.0621 m = 6.2 cm 26. Maximum velocity, v max = aω and
4 × 3.14 × 3.14
2 2
18. We have A , A0 = ebt / 2m Maximum acceleration, = ω2 a =   a =
v v
1  a a
In this case after 6 s, amplitude becomes times.
27 27. The average acceleration of a particle performing SHM over
19. Displacement, y = A sin 
2π  Q ω = 2 π  one complete oscillation is zero.
t   1
T   T  28. The potential energy, U = kx 2
2π  2
= A sin  −1  1 
A 2π
 t ⇒ sin   = t
2 U = kx 2
⇒ 2U = − Fx (QF = − k x)
2 T   2 T
π 2π T 6 1 2U
= −x
2U
+ x =0
⇒ = t ⇒ t= = = s or or
6 T 12 12 2 F F
20. The given relation is given that 29. Velocity, v = ω A 2 − x 2
x = 4(cos πt + sin πt ) and acceleration = ω2 x
On comparing, we get Given, ω A 2 − x 2 = ω2 x or A 2 − x 2 = ωx
R sin δ = 4 and R cosδ = 4 2π 2π
On squaring both the equations given above and adding Given, T= and ω = = 3
3 T
them, we get
Substituting the value of ω in Eq.(i), we get
R 2 (sin 2 δ + cos2 δ) = 32 ⇒ R = 4 2
A 2 − x 2 = 3x ⇒ A = 2x
21. Weight of spring A, mg = 2kx A
Path length
Weight of spring B, mg = kx B As, amplitude = × 2 cm ⇒ x =1 cm
2
xA 1 2π
∴ = 30. Maximum velocity, v max = aω = a ×
xB 2 T
WA Fx A 1 −3 2π
∴ W = Fx ⇒ = = = (50 × 10 ) × = 0.15 ms−1
WB Fx B 2 2
22. Here, a = 12 cms−2 , x = 3 cm 1
31. Kinetic energy, KE = mω2 A 2 − y 2
2
In SHM, acceleration, a = − ω2 x
At mean position, y = 0
∴ Magnitude of acceleration, a = ω2 x 1
(discarding off negative sign) KE = mω2 ( A 2 )
2
a a 2π a
∴ ω2 = or ω = or =ω= 1
Potential energy, U = mω2 y 2 …(i)
x x T x 2
x 3 A
or T = 2π = 2π = π s = 3.14 s Potential energy, U at y =
a 12 2
23. Given, V x = A (1 − cos px ) 1 mA 2 2
⇒ U= ω …(ii)
dV 2 4
F=− = − Apsin px
dx On dividing Eq. (i) by Eq. (ii), we get
Ap2 KE 4
For small (x), F = − Ap2 x ⇒ a = − x =
m U 1
Ap2 ⇒ KE :U = 4 :1
a = ω2 x ⇒ ω= 32. Given that, y1 = 5[sin 2 πt + 3 cos 2πt ]
m
m 1 3 
∴ T = 2π = 10  sin 2 πt + cos 2 πt 
Ap2 2 2 
m π π  π 
24. Time period, T = 2π = 10 cos sin 2 πt + sin cos 2 πt  = 10 sin  2 πt + 
k  3 3    3  
T1 k2 ⇒ A1 = 10
∴ =
π
y 2 = 5sin  2 πt + 
T2 k1
Similarly, ⇒ A2 = 5
2 2k  4
⇒ = = 2 , T =1s
T k m m A1 10 2
Hence, = =
2 A2 5 1
12
Elasticity
12.1 Introduction
Whenever a load is attached to a thin hanging wire, it elongates and the load moves Chapter Snapshot
downwards (sometimes through a negligible distance). The amount by which the wire ● Introduction
elongates depends upon the amount of load and the nature of wire material. Cohesive ● Elasticity
force, between the molecules of the hanging wire offer resistance against the deformation, ● Stress and Strain
and the force of resistance increases with the deformation. The process of deformation ● Hooke’s Law and the
stops, when the force of resistance is equal to the external force (i.e. the load attached).
Modulus of Elasticity
Sometimes, the force of resistance offered by the molecules is less than the external force.
● The Stress-Strain Curve
In such a case, the deformation continues until failure takes place.
● Potential Energy Stored
Thus, we may conclude that, if some external force is applied to a body it has two in a Stretched Wire
effects on it, namely ● Thermal Stresses and
(i) deformation of the body Strains
(ii) internal resistance (restoring) forces are developed.

12.2 Elasticity
As we have already discussed that whenever a single force (or a system of forces) acts
on a body it undergoes some deformation and the molecules offer some resistance to the
deformation. When the external force is removed, the force of resistance also vanishes and
the body returns back to its original shape. But it is only possible, if the deformation is
within a certain limit. Such a limit is called elastic limit. This property of materials of
returning back to their original position is called the elasticity.
A body is said to be perfectly elastic, if it returns back completely to its original shape
and size after removing the external force(s). If a body remains in the deformed state and
does not even partially regain its original shape after the removal of the deforming forces, it
is called a perfectly inelastic or plastic body. Quite often, when the external forces are
removed, the body partially regains the original shape. Such bodies are partially elastic. If
the force acting on the body is increased and the deformation exceeds the elastic limit, the
body loses to some extent, its property of elasticity. In this case, the body will not return to
its original shape and size even after removal of the external force. Some deformation is
left permanently.
510 Objective Physics Vol. 1

ft
12.3 Stress and Strain σt =
A
Stress Here, A is the
cross-section area of the body – F F
When an external force is applied to a body, then at each
at CC ′. The longitudinal
cross-section of the body an internal restoring force is
stress can be of two types.
developed which tends to restore the body to its original F n –Fn F
The two parts of the body on –F
state. The internal restoring force per unit area of
two sides of a cross-section
cross-section of the deformed body is called stress. It is Fn = F
may pull each other. The
usually denoted by σ (sigma). Fig. 12.2
longitudinal stress is, then
Restoring force
Thus, Stress σ = called the tensile stress. This is the case when a rod or a wire
Area is stretched by equal and opposite forces. In case of tensile
Depending upon the way, the deforming forces are stress in a wire or a rod, the force Fn is just the tension.
applied to a body, there are three types of stress: longitudinal If the rod is pushed at the F –F
stress, shearing stress and volume stress. two ends with equal and
opposite forces, it will be F –Fn Fn
Longitudinal and Shearing Stress under compression. Taking
–F

The body of figure is in static equilibrium under an any cross-section of the rod Fn = F
arbitrary set of external forces. In Fig. 12.1(b), we see the the two parts on the two sides Fig. 12.3
same body with an imaginary sectional cut at CC ′. Since, push each other. The
each of the two individual parts of the body is also in static longitudinal stress in this case is called the compressive
equilibrium, both internal forces and internal torques are stress.
developed at the cross-section. Those on the right portion
are due to the left portion and vice-versa. On the left Volume Stress
portion, the normal and tangential components of the When a body is acted upon by forces in such a manner
internal forces are Fn and Ft respectively, and the net that,
internal torque is τ. From Newton’s third law, the right
(i) the force at any point is
portion is subjected at this same cross-section to force
normal to the surface.
components −Fn and … –Ft and the torque − τ. We define (ii) the magnitude of the force on
the normal stress or longitudinal stress over the area as any small area is proportional
F1 C F1 C to the area.
F2 F
ft The force per unit area is then
called the volume stress, i.e.
F3 Fn F
σV = Fig. 12.4
t A
F5
F4
F5 which is same as the pressure. This is the case when a
C' C' body is immersed in a liquid.
(a) (b)
C
f2
Strain
When the size or shape of a body is changed under an
t external force, the body is said to be strained. The change
f3 occurred in the unit size of the body is called strain. Usually,
–f n
it is denoted by ε. Thus,
∆x
–f
–Ft F4
ε=
C' x
(c)
Here, ∆x is the change (may be in length, volume, etc.)
Fig. 12.1
and x the original value of the quantity in which change has
fn occurred.
σn =
A For example, when the length of a suspended wire
and the tangential stress or shearing stress over the area as increases under an applied load, the value of strain is,
Elasticity 511

∆l changed. The change in length per unit


ε=
l  ∆l 
length   is icalled the longitudinal
and it is called longitudinal strain.  l 
l
Similarly, if the change has occurred in the volume of a strain and the restorng force (which is equal
body, it is called volumetric strain and is given by to the applied force in equilibrium) per unit
∆V area of cross-section of the wire is called the M
ε= longitudinal stress. ∆l
V
For small change in the length of the
Shearing Strain wire, the ratio of the longitudinal stress to
the corresponding strain is called the Fig. 12.6
This type of strain is produced when a shearing stress is Young’s modulus of elasticity (Y ) of the wire. Thus,
present.
F
Consider a body of square cross-section ABCD. Four Fl
forces of equal magnitude F are applied as shown in figure. Y = A or Y =
∆l A∆ l
Net resultant force and net torque is zero. Hence, the body is
in translational as well as rotational equilibrium. Because of l
the forces, the shape of the cross-section changes from a Let there be a wire of length l and radius r. Its one end is
square to a parallelogram. clamped to a rigid support and a mass M is attached at the
∆x
other end. Then,
A F
B A A′ B B′ F = Mg
and A = πr 2
F F Substituting in above equation, we have
x
Mgl
Y =
( πr 2 ) ∆l
D C D C
F
Fig. 12.5 Bulk Modulus of Elasticity (B)
We define the shearing strain as the displacement of a When a uniform pressure
layer divided by its distance from the fixed layer. Thus, (normal force) is applied all over
shearing strain the surface of a body, the volume of
the body changes. The change in
∆x V
ε= volume per unit volume of the body
x is called the ‘volume strain’ and the
normal force acting per unit area of
12.4 Hooke’s Law and the the surface (pressure) is called the
Fig. 12.7
normal stress or volume stress. For
Modulus of Elasticity small strains, the ratio of the volume stress to the volume
According to Hooke’s law, strain is called the ‘Bulk modulus’ of the material of the
“For small deformation, the stress in a body is body. It is denoted by B. Then,
proportional to the corresponding strain.” −p
B=
i.e. stress ∝ strain or strees = ( E ) (strain) ∆V /V
stress
Here, E = is a constant called the modulus of Here, negative sign implies that when the pressure
strain increases volume decreases and vice-versa.
elasticity. Depending upon the nature of force applied on the
body, the modulus of elasticity is classified in following Compressibility
three types:
The reciprocal of the bulk modulus of the material of a
body is called the ‘compressibility’ of that material. Thus,
Young’s Modulus of Elasticity (Y )
1
When a wire is acted upon by two equal and opposite Compressibility =
forces in the direction of its length, the length of the body is B
512 Objective Physics Vol. 1

Modulus of Rigidity (η ) removed. Point P is known as the proportional limit of the


When a body is acted upon by an external force solid. For stresses between σP and σE , where point E is called
tangential to a surface of the body, the opposite surface the elastic limit, the material also returns to its original size.
being kept fixed, it suffers a change in shape, its volume However, notice that stress and strain are not
remaining unchanged. Then, the body is said to be sheared. proportional in this region. For deformations beyond the
F elastic limit, the material does not return to its original size,
K K′ L L′ when the stress is removed, it is permanently distorted.
Finally, further stretching beyond the elastic limit leads to
the eventual fracture of the solid. The proportionality
constant for linear region or the slope of stress-strain curve
in this curve is called the Young’s modulus of elasticity Y.
θ θ

N M
Extra Knowledge Points
Fig. 12.8
■ Modulus of elasticity E (whether it is Y, B or η ) is given
by
stress
The ratio the displacement of a layer in the direction of E =
strain
the tangential force and the distance of the layer from the
Following conclusions can be made from the above
fixed surface is called the shearing strain and the tangential
expression :
force acting per unit area of the surface is called the
(i) E ∝ stress (for same strain), i.e. if we want the
“shearing stress”. equal amount of strain in two different materials, the
For small strain the ratio of the shearing stress to the one which needs more stress is having more E.
shearing strain is called the “modulus of rigidity” of the 1
(ii) E ∝ (for same stress), i.e. if the same amount
material of the body. It is denoted by η. strain
F /A of stress is applied on two different materials, the
Thus, η= one having the less strain is having more E. Rather
KK ′/KN
we can say that, the one which offers more
KK ′
Here, = tan θ ≈ θ resistance to the external forces is having greater
KN value of E. So, we can see that modulus of elasticity
F /A of steel is more than that of rubber or
∴ η=
θ E steel > E rubber
F ∆x
or η= (iii) E = stress for unit strain  
= 1 or ∆x = x  , i.e.
Aθ  x 
suppose, the length of a wire is 2 m, then the
12.5 The Stress-Strain Curve Young’s modulus of elasticity (Y ) is the stress
applied on the wire to stretch the wire by the same
A plot of normal stress (either tensile or compressive) amount of 2 m.
versus normal strain for a typical solid is shown in figure. ■ The material which has smaller value of Y is more
σ ductile, i.e. it offers less resistance in framing it into a
wire. Similarly, the material having the smaller value of
(Fracture point)
σE B is more malleable. Thus, for making wire we choose
E (Elastic limit) a material having less value of Y.
σP
P (Proportional limit) ■ A solid will have all the three modulii of elasticity Y, B
and η. But in case of a liquid or a gas only B can be
defined as a liquid or a gas cannot be framed into a
wire or no shear force can be applied on them.
ε
■ For a liquid or a gas,
 − dp 
The stress-strain curve for a typical solid B= 
 dV /V 
Fig. 12.9
So, instead of p we are more interested in change in
The strain is directly proportional to the applied stress pressure dp.
for values of stress up to σP . In this linear region, the ■ In case of a gas,
material returns to its original size, when the stress is B = Xp in the process pV X = constant
Elasticity 513

For example, for X = 1, Comparing Eqs. (i) and (ii), we can see that force constant
or pV = constant (isothermal process) B = p. of a wire is
YA
i.e. isothermal bulk modulus of a gas (denoted by BT ) is k = …(iii)
equal to the pressure of the gas at that instant of time or l
BT = p YA
i.e. a wire is just like a spring of force constant . So, all
Cp l
Similarly, for x = γ = or pV γ = constant (adiabatic formulae which we use in case of a spring can be
CV
applied to a wire also.
process)
B = γp
i.e. adiabatic bulk modulus of a gas (denoted by Bs ) is l l
, 2k , 2k
equal to γ times the pressure of the gas at that instant of 2 2
time or Bs = γp l, k ⇒
■ For a gas, B∝p
whether it is an isothermal process or an adiabatic
process. Physically, this can be understood as under :
A B
From Eq. (iii), we may also conclude that force constant
of a spring is inversely proportional to the length of the
spring l or
. . . 1
... . .... .. .. .. . ... k ∝
. . ... . . .... . . . . . .. l
. .. . p1 . .. . . . p2 . .
.. . . . . . . .. . . . i.e. if a spring is cut into two equal pieces its force
. . ... . ... . . . . constant is doubled.
.. .. ..
■ When a pressure (dp ) is applied on a substance, its
Suppose, we have two containers A and B. Some gas is density is changed. The change in density can be
filled in both the containers. But the pressure in A is more calculated as under
than the pressure in B, i.e. mass
ρ= ( ρ = density)
p1 > p2 volume
So, bulk modulus of A should be more than the bulk 1
or ρ∝ (mass = constant)
modulus of B, V
or B1 > B 2 ρ′ V V
= =
and this is quiet obvious, because it is more difficult to ρ V ′ V + dV
compress the gas in chamber A, i.e. it provides more  V 
resistance to the external forces. And as we have said in or ρ′ = ρ  
V + dV 
point number 1(ii) the modulus of elasticity is greater for a
substance which offers more resistance to external forces.  V 
=ρ  
■ If a spring is stretched or compressed by an amount ∆l, V − (d p /B )V 
then restoring force produced in it is dp
as B=−
Fs = k ∆l …(i) dV /V
Here, k = force constant of spring. ρ
ρ′ =
Similarly, if a wire is stretched by an amount ∆l, the 1−
dp
restoring force produced in it is B
YA  From this expression, we can see that ρ′ increases as
F =   ∆l …(ii) ■

 l  pressure is increased (dp is positive) and vice-versa.

Fl
X Example 12.1 Determine the elongation of the ∴ ∆l =
AY
steel bar 1m long and 1.5 cm 2 cross-sectional area Substituting the values,
when subjected to a pull of 1.5 × 10 4 N . (Take (1.5 × 104 )(1.0)
∆l =
Y = 2.0 × 1011 N / m 2 ) (1.5 × 10–4 ) (2.0 × 1011 )
= 0.5 × 10–3 m
Sol. Y = F / A
∆l / l or ∆l = 0.5 mm
514 Objective Physics Vol. 1

X Example 12.2 A bar of mass m and


length l is hanging from point A as shown in
A
12.7 Thermal Stresses and
figure. Find the increase in its length due to Strains
its own weight. The Young’s modulus of Whenever there is some increase or decrease in the
elasticity of the wire is Y and area of temperature of the body, it causes the body to expand or
cross-section of the wire is A. contract. If the body is allowed to expand or contract freely,
B
with the rise or fall of the temperature, no stresses are
Fig. 12.10
induced in the body. But, if the deformation of the body is
Sol. Consider a small section dx of the bar at a distance x from B. prevented, some stresses are induced in the body. Such
The weight of the bar for a length x is
stresses are called thermal stresses or temperature stresses.
w = 
mg 
x A The corresponding strains are called thermal strains or
 l 
temperature strains.
Elongation in section dx will be
dl = 
w  mg  x dx
 dx =   l, α, Y, A
 AY   lAY  dx A B
Total elongation in the bar can be obtained by
integrating this expression for x = 0 to x = l. x

dl = 
x=l mg  l
∆l = ∫
 lAY  ∫ 0
∴  x dx B
x= 0
mgl Fig. 12.11 ∆l
or ∆l =
2 AY Fig. 12.12

Consider a rod AB fixed at two supports as shown in


12.6 Potential Energy Stored figure.
in a Stretched Wire Let l = length of rod,
A = area of cross-section of the rod,
When a wire is stretched, work is done against the
inter-atomic forces. This work is stored in the wire in the Y = Young’s modulus of elasticity of the rod
form of elastic potential energy. Suppose on applying a and α = thermal coefficient of linear expansion of the rod.
force F on a wire of length l, the increase in length is ∆l. The Let the temperature of the rod is increased by an amount
area of cross-section of the wire is A. The potential energy t. The length of the rod would had increased by an amount
stored in the wire should be ∆l, if it were not fixed at two supports. Here,
1 ∆l = lα t
U = K ( ∆l) 2
2 But since the rod is fixed at the supports, a compressive
YA
Here, K= strain will be produced in the rod. Because at the increased
l temperature, the natural length of the rod is l + ∆l, while
1 YA
∴ U = ( ∆l) 2 being fixed at two supports its actual length is l.
2 l ∆l lα t
Elastic potential energy per unit volume of the wire is Hence, thermal strain ε = = = αt or
l l
U
u= ε = αt
volume Therefore, thermal stress
1 YA
( ∆l) 2 σ = Yε (stress = Y × strain)
or u= 2 l or σ = Yαt
Al or force on the supports,
1  ∆l   ∆l  F = σA = YAαt
or u =   Y ⋅ 
2 l  l 
This force F is in the direction shown below:
1
or u= (strain) (Y × strain) F F F F
2
1
or u = (strain × stress)
2 Fig. 12.13
Elasticity 515

Extra Knowledge Points


■ Poisson’s ratio exactly as the previous particle that has just passed
When a longitudinal force is applied on a wire, its that point.
length increases but its radius decreases. Thus, two Each particle follows a smooth path and the paths of
strains are produced by a single force : the particles do not cross each other. The path
∆l followed by a fluid particle under a steady flow is called
(i) Longitudinal strain = and a streamline.
l
∆R v3
(ii) Lateral strain = C
R v1
The ratio of these two strains is called the Poisson’s A B
ratio. v2
Thus, the Poisson’s ratio
Lateral strain ∆R /R In the streamline ABC shown in above figure, velocity
σ= =−
Longitudinal strain ∆l /l of fluid particles at A is v1 at B is v 2 and at C is v 3 .
Following points are worthnothing in case of Poisson’s Steady flow is achieved when the speed of flow is not
ratio : very high. Beyond a limiting value, called critical
(i) Negative sign in σ indicates that radius of the wire speed, flow is no longer streamlined it becomes
decreases as the length increases. turbulent.
1 ■ Reynold’s number
(ii) Theoretical value of σ lies between − 1 and + ⋅
2 When speed of fluid flow is high, flow of fluid becomes
1 turbulent. Turbulent flow is less likely for viscous fluids
(iii) Practical value of σ lies between 0 and + ⋅ flowing at low speeds. Reynold’s number (Re ) is a
2
dimensionless number, whose value gives us an idea
■ Relation between Y, B, η and σ whether the flow would be laminar (streamline) or
Following are some relations between the four turbulent.
Y ρvd
(a) B = ■ Re =
3 (1 − 2 σ ) η
(b) η =
Y Here, ρ = density of fluid,
2 (1 + σ ) v = speed of fluid,
3B − 2 η d = dimension of fluid and
(c) σ =
2 η + 6B η = viscosity of fluid.
9 1 3 For Re < 1000, flow is streamline or laminar.
(d) = +
Y B η For Re → 1000 to 2000, flow is unsteady and for
■ Streamline and Turbulent flow Re > 2000, flow is turbulent. Turbulence dissipates
The flow of a fluid is said to be steady, if at any given kinetic energy usually in the form of heat. Racing cars
point the velocity of each particle passing through that and aeroplanes are designed to minimize turbulence.
point remains constant. Although velocity at different But, sometimes turbulence is desirable. Turbulence
points may be different. That is, at some other point the promotes mixing.
particle may have a different velocity, but every other For example, blades of a kitchen mixer induce
particle which passes the second point behaves turbulent flow and provide thick milk shakes.
Chapter Summary with Formulae
F
1. Stress = = restoring force per unit area.
A
∆x ∆l ∆V
2. Strain = = change per unit original. It may be or , etc.
x l V
Stress
3. Modulus of Elasticity E=
Strain
4. Materials which offer more resistance to external deforming forces have higher value of modulus of elasticity.
5. Young's Modulus of Elasticity
F/A Fl
Y= =
∆ l / l A∆ l
F = Mg and A = πr2
F/A ∆p dp
6. Bulk Modulus of Elasticity B= =− or −
∆V / V ∆V / V ∆V / V
7. Shear Modulus of Elasticity of Modulus of Rigidity
F/A F
G or η = =
θ Aθ

DE BC
where, θ ≈ tan θ = or
DF BA
F
B C D E

θ θ

A F
8. Solids have all three modulii of elasticities, Young's modulus, bulk modulus and shear modulus. Whereas liquids and gases have only
bulk modulus.
9. Every wire is like a spring whose force constant is
equal to,
YA 1
k= or k ∝
l l
10. Potential energy stored in a stretched wire
U = k ( ∆l )2 =   ( ∆l )2
1 1 YA
2 2 l 
11. Potential energy stored per unit volume (also called energy density) in a stretched wire,
1
U = × Stress × Strain
2
12. Change in Length of a Wire
Fl
∆l =
AY

Here, F is tension in the wire. If wire is having negligible mass, tension is a uniform throughout the wire and change in length is
obtained directly. Otherwise by integration.
13. In case of solids and liquids bulk modulus is almost constant. In case of a gas, it is process dependent.
In isothermal process, B = BT = p
In adiabatic process, B = B S = γ p
Here, p is pressure of the gas and γ is called adiabatic exponent of the gas.
1
14. Compressibility =
B
15. When pressure is applied on a substance, its volume decreases, while mass remains constant. Hence, its density will increase,
ρ ∆p 
ρ′ = or ρ′ ≈ ρ  1 + 
1 − ∆p / B  B 
∆p
If << 1
B
Additional Examples
Example 1. State Hooke's law. Sol. (a) When two wires of the same size are suspended in
parallel, a force equal to 2F has to be applied on the
Sol. Within elastic limit, stress is directly proportional to strain.
parallel combination, so that a force F equal to the
Example 2. What are the factors on which modulus breaking force for the wire acts on each of the two
of elasticity of a material depends? wires. Hence, the breaking force in this case is 2F.
(b) Breaking force = (Breaking stress) × (area of C.S.)
Sol. Nature of the material and the manner in which it is
∴ Breaking force ∝ area of C.S.
deformed.
If the wire is of double the thickness, i.e. of double
Example 3. The stress versus strain graphs for the diameter, the area of C.S. will be four times.
wires of two materials A and B are as shown in figure. Hence, breaking force will be 4F.

Example 7. A cable is replaced by another of the


Wire A same length and material of twice diameter?
Stress

Wire B (a) How does this affect elongation under a given


load?
(b) How many times will be the maximum load
supported by the later as compared to the former?
Strain FL 4 FL
Sol. (a) We know, Y = = ,
(a) Which material is more ductile? ( A)(∆l ) (πd 2 ) (∆l )
(b) Which material is more brittle? where d is the diameter of the cable.
Sol. (a) Material A is more ductile. It is because, the material A 4 FL
Therefore, ∆l =
has greater plastic range (portion of graph between the πd 2 Y
elastic limit and breaking point).
As ∆l ∝ 1 / d 2 ,
(b) Material B is more brittle. It is because, the material zA
has lesser plastic range. Hence, the extension will become one-fourth on
replacing the cable by another cable of double the
Example 4. Why do we prefer steel to copper in the diameter.
manufacturing of spring? (b) Breaking load = breaking stress × area of cross-section.
Sol. A better spring will be the one, in which a large restoring When the cable of double the diameter is used, area of
force is developed on being deformed. This, in turn, depends cross-section will become four times. Since, the
upon the elasticity of the material of the spring. As Young's breaking stress is a constant for the given material, such
modulus of steel is greater than that of copper, steel is a cable will be able to support a load four times as large
preferred to manufacture a spring. as the former cable can support.

Example 5. A cable is cut to half of its original Example 8. Why are the bridges declared unsafe
length. Why this change has no effect on the maximum after long use?
load, the cable can support? Sol. A bridge undergoes continuous alternating strains a large
Sol. The breaking stress is a constant for a given material. We number of times everyday. Therefore, such a long use of the
know that : bridge results in the loss of its elastic strength. In other words,
Breaking load = breaking stress × area of cross-section. after a long use, the strain produced is quite large for a given
stress and may lead to the collapse of the bridge. This is the
When the cable is cut to half of its length, the area of reason why, the bridges are declared unsafe after a long use.
cross-section does not change. Hence, there is no effect on the
maximum load (breaking load), the cable can support.
Example 9. What is a perfectly elastic body? Give
Example 6. The breaking force for a wire is F. an example.
What will be the breaking force for (a) two parallel Sol. If, on removal of deforming force, a body completely
wires of the same size (b) for a single wire of double regains its original configuration, then it is said to be perfectly
the thickness? elastic. Example is quartz.
518 Objective Physics Vol. 1

Example 10. What is a perfectly plastic body? Give Sol. Young’s modulus
an example. Mgl Mgl 4 Mgl
Y= 2 = =
Sol. If, on removal of deforming force, a body does not regain its π r ⋅∆ l  D
2
πD 2 ⋅ ∆l
π   ∆l
original configuration even a little, then it is said to be perfectly  2
plastic. Example is putty.
where, D is the diameter of the wire.
Example 11. Is it possible to double the length of a 4 Mgl
∴ Elongation, ∆l =
metallic wire by applying a force over it? πD 2 ⋅ Y
1
Sol. No, it is not possible, because within elastic limit strain is i.e. ∆l ∝ 2
only of the order of10 −3 . Wires actually break much before it if D
they are stretched to double the length. Clearly, if the diameter is doubled, the elongation will
become one-fourth.
Example 12. Stress and pressure both are force per Mmg
Also, maximum stress, σ m = or M m ∝ D 2
unit area. Then, in what respect does stress differ from (πD 2 / 4 )
pressure? Clearly, if the diameter is doubled, the wire can
Sol. Pressure is the external force per unit area, while stress is the support four times the original load.
internal restoring force (which comes into play in a deformed
body) per unit area of the body. Example 17. The bulk modulus of water is
2.3 × 10 9 N/m 2 .
Example 13. Which is more elastic-water or air?
Sol. Water is more elastic than air. Air can be easily compressed (a) Find its compressibility.
while water is incompressible and bulk modulus is reciprocal (b) How much pressure in atmospheres is needed to
of compressibility. compress a sample of water by 0.1%.
2.3 × 10 9
Example 14. The ratio stress/strain remains Sol. Here, B = 2.3 × 10 9 N/m 2 =
constant for a small deformation. What happens to this 1.01 × 10 5
ratio if deformation is made very large? = 2.27 × 10 4 atm
1 1
Sol. When the deforming force exceeds the elastic limit, the (a) Compressibility = =
strain increases more rapidly than stress. Hence, the ratio of B 2.27 × 10 4
stress/strain decreases. = 4.4 × 10 −5 atm −1
∆V
Example 15. A steel wire of length 4 m and (b) Here, = − 0.1% = − 0.001
diameter 5 mm is stretched by 5 kg-wt. Find the V
increase in its length, if the Young’s modulus of steel is Required increase in pressure,
 ∆V 
2.4 × 1012 dyne/cm 2 . ∆p = B × − 
 V 
Sol. Hence, l = 4 m = 400 cm, 2 r = 5 mm
or r = 2.5 mm = 0.25 cm = 2.27 × 10 4 × 0.001
f = 5 kg-wt = 5000 g-wt = 22.7 atm
= 5000 × 980 dyne
Example 18. A steel wire 4.0 m in length is
∆l = ?, Y = 2.4 × 10 12 dyne/cm 2
stretched through 2.0 mm. The cross-sectional area of
F l
As Y= 2 × the wire is 2.0 mm 2 . If Young’s modulus of steel is
πr ∆ l
Fl 2.0 × 1011 N/m 2 . Find
or ∆l = 2 (a) the energy density of wire,
πr Y
(5000 × 980) × 400 (b) the elastic potential energy stored in the wire.
= Sol. Here, l = 4. 0 m, ∆l = 2 × 10 −3 m
(22 / 7) × (0.25) 2 × 2.4 × 10 12
= 0.0041 cm A = 2.0 × 10 6 m 2
Y = 2.0 × 10 11 N/m 2
Example 16. A cable is replaced by another one of
(a) The energy density of stretched wire,
same length and material but twice the diameter. How
1
will this effect the elongation under a given load ? u = × stress × strain
How does this effect the maximum load it can support 2
1
without exceeding the elastic limit? = × Y × (strain ) 2
2
Elasticity 519

2
1  2 × 10 −3  Sol. (a) Energy stored
= × 2.0 × 10 11 ×   1
2  4  U= (stress)(strain)(volume)
2
= 2.5 × 10 J/m
4 3
1  F   ∆l 
(b) Elastic potential energy = energy density × volume or U =     ( Al )
= 2.5 × 10 4 × (2.0 × 10 −6 ) × 4.0 J = 0.20 J 2  A  l 
1
= F ⋅ ∆l
Example 19. Find the greatest length of steel wire 2
that can hang vertically without breaking. Breaking 1
= (100 )(0.3 × 10 −3 )
stress of steel = 8.0 × 10 8 N / m2 . Density of steel 2
= 8.0 × 10 3 kg / m3 . (Take g = 10 m/ s 2 ) = 0.015 J
(b) Potential energy stored
Sol. Let l be the length of the wire that can hang vertically
1
without breaking. Then, the maximum tension on it is at the = k ( ∆l ) 2
2
topmost point and it is equal to its own weight. If, A is the area
1  YA  YA
of cross-section and ρ the density, then =   ( ∆l ) 2 as k = 
weight  force 
2 l   l 
Maximum stress, σ m = Stress = 
A  area  Substituting the values, we have
( Alρ)g 1 (2.0 × 10 11 )(10 –6 )
or σm = U= (0.1 × 10 –3 ) 2
A 2 (2)
σm = 5.0 × 10 –4 J
∴ l=
ρg
Substituting the values, we get Example 22. A rubber cord has a cross-sectional
8.0 × 10 8 area 1 mm2 and total unstretched length 100 . cm. It is
l= stretched to 12.0 cm and then released to project a
(8.0 × 10 3 )(10)
missile of mass 5.0 g. Taking Young’s modulus Y for
= 10 4 m
rubber as 5.0 × 10 8 N / m2 . Calculate the velocity of
Example 20. What is the density of lead under a projection.
pressure of 2.0 × 10 8 N/m2 , if the bulk modulus of lead Sol. Equivalent force constant of rubber cord.
is 8.0 × 10 9 N / m2 and initially the density of lead is YA
k=
11.4 g/cm3 ? l
Sol. The changed density, (5.0 × 10 8 )(1.0 × 10 –6 )
ρ =
ρ′ = (0.1)
dp
1− = 5.0 × 10 3 N/m
B
Substituting the values, we have Now, from conservation of mechanical energy, elastic
11.4 potential energy of cord
ρ′ = or ρ′ = 11.69 g/ cm 3 = kinetic energy of missile
2.0 × 10 8
1– 1 1
8.0 × 10 9 ∴ k (∆l ) = mv 2
2
2 2
Example 21. (a) A wire 4 m long and 0.3 mm in  k
diameter is stretched by a force of 100 N. If extension ∴ v=  ∆l
 m
in the wire is 0.3 mm, calculate the potential energy
stored in the wire. (b) Find the potential energy stored  5.0 × 10 3 
in a wire of cross-section 1 mm2 and length 2 m through =  (12.0 – 10.0) × 10 −2
 5.0 × 10 −3 
 
0.1 mm. Young’s modulus for the material of wire is
2.0 ×1011 N / m2 . = 20 m / s
520 Objective Physics Vol. 1

Example 23. A light rod of length 200 . m is Example 24. A steel rod of cross-sectional area
suspended from the ceiling horizontally by means of 16 cm2 and two brass rods each of cross-sectional area
two vertical wires of equal length tied to its ends. One 10 cm2 together support a load of 5000 kg as shown in
of the wires is made of steel and is of cross-section figure. Find the stress in the rods. Take Y for steel
10 −3 m2 and the other is of brass of cross-section = 2.0 × 10 6 kg / cm2 and for brass = 1.0 × 10 6 kg / cm2 .
2 × 10 −3 m2 . Find out the position along the rod at
which a weight may be hung to produce, 5000 kg
(a) equal stresses in both wires,

Brass
Brass

Steel
(b) equal strains on both wires. 30 cm 20 cm
Young’s modulus for steel is 2 × 1011 N / m2 and for
brass is 1011 N / m2 .
Sol. (a) Given, s

Sol. Given, area of steel rod


Steel Brass A S = 16 cm 2
Area of two brass rods
TS TB
D A B = 2 × 10
A C
= 20 cm 2
x
Load, F = 5000 kg
2–x
Y for steel Y S = 2.0 × 10 6 kg/cm 2
TS T
∴ = B Y for brass Y B = 1.0 × 10 6 kg/cm 2
AS AB
TS A Length of steel rod, l S = 30 cm
∴ = S Length of brass rod, l B = 20 cm
TB AB
Let σ S = stress in steel and σ B = stress in brass
10 −3 1
= = …(i) Decrease in length is given by
2 × 10 −3 2
(stress )
As the system is in equilibrium, taking moments about ∆l = (l ) (strain) = (l )
D, we have r
TS ⋅ x = TB ( 2 − x ) Now, given that
TS 2 − x decrease in length of steel rod
∴ = …(ii)
TB x = decrease in length of brass rod
From Eqs. (i) and (ii), we get σS σ
or × lS = B × lB
x = 1.33 m YS YB
Stress Y l
(b) Strain = or σS = S × B × σB
Y YB l S
Given, strain in steel = strain in brass
2.0 × 10 6 20
TS / AS TB / AB = × × σB
∴ = 1.0 × 10 6
30
YS YB 4
∴ σS = σB …(i)
TS A Y 3
∴ = S S
TB AB YB Now, using the relation,
F = σ S AS + σ B AB
(1 × 10 −3 )(2 × 10 11 )
= =1 …(iii) or 5000 = σ S × 16 + σ B × 20 …(ii)
(2 × 10 −3 )(10 11 )
Solving Eqs. (i) and (ii), we get
From Eqs. (ii) and (iii), we have
x = 1. 0 m σ B = 120.9 kg/cm 2 and σ S = 161.2 kg/cm 2
NCERT Selected Questions
Q 1. A steel wire of length 4.7 m and cross-sectional area Q 3. The stress-strain graph for materials A and B are
−5
3.0 × 10 m stretches by the same amount as a
2 shown in Fig. (a) and Fig. (b).
copper wire of length 3.5 m and cross-sectional area The graph are drawn to the same scale.
4.0 × 10 −5 m 2 under a given load. What is the ratio
of the Young’s modulus of steel to that of copper? D D

Stress

Stress
Sol. Let Y1 and Y2 be the Young’s modulus of steel and copper A B
wires, respectively.
F ×l
∴ Y1 = 1 1
A1 × ∆l1 Strain Strain
F × 4.7 (a) (b)
= …(i)
3 × 10−5 × ∆l (a) Which of the materials has greater Young’s
F ×l modulus?
and Y2 = 2 2
A2 × ∆l2 (b) Which of the two is the stronger material?
F × 3.5 Sol. (a) From the two graphs, we note that for a given strain,
= …(ii)
4 × 10−5 × ∆l stress for B is more than that of A. Hence, Young’s
 stress 
Dividing Eq. (i) by Eq. (ii), we get modulus  =  is greater for B than that of A.
 strain 
Y1 F × 4.7 4 × 10−5 × ∆l
= −5
× (b) Strength of a material is determined by the amount of
Y2 3 × 10 × ∆l F × 3.5
stress (load) required to cause breaking or fracture of the
= 1.79 ≈ 1.8 material corresponding the breaking point.
Q 2. Figure shows the strain-stress curve for a given ∴ Material A is stronger than B as it can withstand more
material. What are (a) Young’s modulus and load without breaking than the material B corresponding
to point D.
(b) approximate yield strength for his material?
Q 4. Read the following two statements below carefully
300 and state, if it is true or false.
Stress (106 Nm–2)

250 (a) The Young’s modulus of rubber is greater than


200 that of steel.
150 (b) The stretching of a coil is determined by its shear
100 modulus.
50 Sol. (a) False. This is because if steel and rubber wires of same
0 length and area of cross-section are subjected to same
0.001 0.002 0.003 0.004 deforming force, then the extension produced in steel is
Strain
less than the extension produced in rubber. For
Sol. From the given graph for a stress of 150 × 106 Nm −2, the producing same strain in steel and rubber more stress is
required in case of steel.
strain is 0.002.
(b) True.
(a) ∴Young’s modulus of the material Y is given by
Stress 150 × 106 Q 5. Two wires of diameter 0.25 cm,
Y= =
Strain 0.002 one made of steel and other
1.5 m
150 × 106 made of brass are loaded as
= shown in figure. The unloaded Steel
2 × 10−3 4.0 kg
length of steel wire is 1.5 m and
= 75 × 109 N/m 2 that of brass wire is 1.0 m. 1m
= 7.5 × 1010 N/m 2 Young’s modulus of steel is Brass

(b) Yield strength of a material is defined as the maximum


. × 1011 Pa and that of brass is
20 6.0 kg

stress it can sustain. 0.91 × 1011 Pa. Compute the


∴ From the given graph, the approximate yield strength of elongations of steel and brass wires.
the given material = 300 × 106 N/m 2 = 3 × 108 N/m 2. (1 Pa = 1 Nm −2 ).
522 Objective Physics Vol. 1

Sol. For steel wire, Sol. Here, Y = 1.1 × 1011 Nm −2


Total force, F1 = 10 × 9.8 N A = Area of cross-section
l1 = 1.5 m = 15.2 mm × 19.1 mm
F ×l
∆l1 = 12 1 = 15.2 × 10−3 m × 19.1 × 10−3 m
πr1 × Y1 Force, F = 44500 N
(10 × 9.8) × (1.5) × 7 ∴ Y=
Stress
=
22 × (0.125 × 10−2 )2 × 2 × 1011 Strain
Stress F
= 1.5 × 10−4 m Strain = =
Y AY
For brass wire, F2 = 6 × 9.8 N 44500
F ×l Longitudinal strain =
∴ ∆l2 = 22 2 15.2 × 19.1 × 10-6 × 1.1 × 1011
πr2 × Y2
= 0.139
(6 × 9.8) × 1.0 × 7
=
22 × (0.125 × 10−2 )2 × 0.91 × 1011 Q 8. A steel cable with a radius of 1.5 cm supports a chair
= 1.3 × 10 −4
m lift. If the maximum stress is not to exceed
10 8 Nm −2 , what is the maximum load the cable can
Q 6. Four identical hollow cylindrical columns of mild support?
steel support a big structure of mass 50000 kg. The
inner and outer radii of each column are 30 cm and Sol. Here, radius of steel cable,
60 cm, respectively. Assuming, the load distribution r = 1.5 cm = 1.5 × 10−2 m
to be uniform, calculate the compressional strain of Maximum stress = 108 N / m 2
each column. Young’s modulus, Y = 2.0 × 1011 Pa.
∴ Area of cross-section of cable
Sol. Total weight of the structure to be supported by four A = π r2
columns
= π (1.5 × 10−2 )2
= Mg
Maximum force
= 50000 × 9.8 N Maximum stress =
Area of cross - section
Since, this weight is to be supported by 4 columns.
or Maximum force
∴ Compressional force on each column (F) is given by
= Maximum stress × area of cross-section
Mg 50000 × 9.8
F= = = 108 × π × (1.5 × 10−2 )2 = 7.1 × 104 N
4 4
Inner radius of a column, r1 = 30 cm = 0.3 m or Maximum load the cable can withstand = 7.1 × 104 N.
Outer radius of a column, r2 = 60 cm = 0.6 m
Q9. A rigid bar of mass 15 kg is supported
∴ Area of cross-section of each column is given by symmetrically by three wires each 2 m long. Those
A = π (r22 − r12 ) at each end are of copper and the middle one is of
= π [(0.6)2 − (0.3)2 ] iron. Determine the ratios of their diameters, if each
is to have the same tension.
= 0.27 π m 2
Sol. Let Y1 and Y2 be the Young’s modulus of copper and iron
Young’s modulus, Y = 2 × 1011 Pa
wires, respectively.
Compressional force / Area
∴ Y= ∴ Y1 = 110 × 109 Nm −2
Compressional strain
F/A Y2 = 190 × 109 Nm −2
=
Compressional strain Also let A1 and A2 be the areas of cross-section of copper
and iron wires, respectively. If d1 and d2 be their respective
or compressional strain of each column
diameters.
F 50000 × 9.8 × 7
= = πd12
AY 4 × 0.27 × 22 × 2 × 1011 Then, A1 =
4
= 0.722 × 10−6
πd22
and A2 =
Q 7. A piece of copper having a rectangular cross-section 4
of 15.2 mm ×191
2
. mm is pulled with 44500 N force, A1 d12  d1 
∴ = = 
producing only elastic deformation. Calculate the A2 d22  d2 
resulting strain? (Y for copper = 1.1 × 1011 Nm −2 ). L=2m
Elasticity 523

Let ∆l be the extension produced in each wire. Q 11. Compute the bulk modulus of water from the
Let F = Tension produced in each wire. following data : initial volume = 100.0 L, pressure
∴ From the relation, Y =
stress
, we get increase = 100.0 atm (1 atm = 1.013 × 10 5 Pa).
strain Final volume = 100.5 L. Compare the bulk modulus
F / A1 of water with that of air (at constant temperature).
Strain for copper wire =
Y1 Explain in simple terms why the ratio is so large?
F / A2
and strain for iron wire = Sol. Here, p = 100 atm
Y2
= 100 × 1.013 × 105 Pa (Q1 atm = 1.013 × 105 Pa)
As the bar is supported symmetrically.
Initial volume, V1 = 100 L = 100 × 10−3 m 3
∴ The two strains are equal
F F Final volume, V2 = 100.5L = 100.5 × 10−3 m 3
∴ =
A1Y1 A2Y2 ∆V = change in volume = V2 − V1
or A1Y1 = A2Y2 = (100.5 − 100) × 10−3 m 3
or
A1 Y2
= = 0.5 × 10−3 m 3
A2 Y1 p
∴ From the formula, B = , we get
πd12 / 4 Y2 ∆V /V
or =
πd22 / 4 Y1 pV
Bw =
d1 Y 190 × 10 9 ∆V
or = 2 = 100 × 1.013 × 105 × 100 × 10−3
d2 Y1 110 × 109 e.g. =
0.5 × 10−3
d1
i.e. = 1.31
d2 or Bw = 2.026 × 109 Pa
or d1 : d2 = 1.31 : 1 Bulk modulus of air at STP is given by
Bair = 1.0 × 10−4 GPa
Q 10. A 14.5 kg mass, fastened to one end of a steel wire
of unstretched length 1 m is whirled in a vertical = 1 × 10−4 × 109 Pa
circle with an angular velocity of 2 rev/s at the = 105 Pa
bottom of the circle. The cross-sectional area of the
Bw 2.026 × 109
wire is 0.065 cm 2 . Calculate the elongation of the =
Bair 105
wire, when the mass is at the lowest point of its
path. = 20260
The ratio is too large. This is due to the fact that the strain for
Sol. Mass attached to one end of steel wire, air is much larger than for water at the same pressure. In
m = 14.5 kg other words, the intermolecular distances in case of liquid
Length of steel wire, l = 1 m are very small as compared to the corresponding distances in
Frequency, f = 2 rev/s = 2 rps the case of gases.
∴ Angular frequency, ω = 2πf = 2π × 2 = 4 π rad/s Q 12. What is the density of water at a depth, where
Area of cross-section of the wire, pressure is 80.0 atm, given that its density at the
A = 0.065 cm 2 surface is 1.03 × 10 3 kg - m −3 ? Compressibility of
= 0.065 × 10−4 m 2 water is 45.8 × 10 −11 Pa −1 .
Y for steel = 2 × 1011 Pa Sol. Here, p = 80.0 atm = 80 × 1.013 × 105 Pa
The stretching force developed in the wire due to the 1
rotation of the mass, Compressibility = = 45.8 × 10− 11 Pa −1
B
mg + mlω 2 = 2432 N Density of water at the surface.
Stress F / A ρ = 1.03 × 103 kg - m −3
∴ Using the relation, Y = = , we get
Strain ∆l / l Let ρ be the density of water at a given depth.
F l If V and V ′ be the volumes of certain mass M of the water at
∆l = × the surface and at a given depth, then
A Y
M
(2432) (1) V =
We get ∆l = ρ
(0.065 × 10−4 ) (2 × 1011 )
M
and V′ =
= 1.87 × 10−3 m ρ′
524 Objective Physics Vol. 1

 1 1 ∆V = −
pV
∴ Change in volume, ∆V = V − V ′ = M  − 
 ρ ρ′  B
∆V  1 1 ρ 7 × 106 × 0.001 3
∴ Volumetric strain, =M  −  × =− m
V  ρ ρ′  M 140 × 109
 ρ = − 0.05 cm 3
= 1 − 
 ρ′  Here, –ve sign shows volume contraction.

∆V 1.03 × 10 3
Q 15. How much should the pressure on a litre of water be
or =1−
V ρ′ changed to compress it by 0.10%?
Also, we know that bulk modulus of water is given by the Sol. Here, V = 1L
formula,
p pV ∆V = − 0.10% of V
B= = 0.10
 ∆V  ∆V =− ×1
 
 V  100
1
1 ∆V 1  ∆V  =− L
∴ Compressibility = = =   1000
B pV p V 
Let ∆p = change in pressure required for compression of
1  1.03 × 103  1 L of water.
or 45.8 × 10−11 = × 1 − 
80 × 1.013 × 10 
5
ρ′  B = bulk modulus of water = 2.2 × 109 Nm −2
Solving, we get, ∆p
Using the relation, B = − , we get
ρ′ = 1.034 × 10 kg - m
3 −3 ∆V /V
∆V
Q 13. Compute the fractional change in volume of a glass ∆p = − B ⋅
V
slab, when subjected to a hydraulic pressure of  1 
10 atm.  
 1000
= 2.2 × 109 ×
Sol. Here, p = 10 atm = 10 × 1.013 × 105 Pa 1
Bulk modulus for glass slab = 37 × 10 Nm 9 −2 = 2.2 × 106 Nm −2
p Q 16. The Marina trench is located in the Pacific ocean,
Using the relation B = , we get
∆V /V and at one place, it is nearly 11km beneath the
∆V p surface of water. The water pressure at the bottom
=
V B of the trench is about 1.1 × 10 8 Pa. A steel ball of
10 × 1.013 × 105 initial volume 0.32 m 3 is dropped into the ocean
=
37 × 109 and falls to the bottom of trench. What is the change
1.013 in the volume of the ball when it reaches to the
= bottom?
37 × 103
= 0.0274 × 10−3 Sol. Pressure exerted by a 11 km column of water on the bottom
of the trench,
Q 14. Determine the volume contraction of a solid copper p = hρ g
cube, 10 cm on an edge, when subjected to a = 11 × 103 × 103 × 10 Pa
hydraulic pressure of 7 × 10 6 Pa. B for copper
= 1.1 × 108 Pa
= 140 × 10 9 Pa.
V = 0.32 m 3
Sol. Here, L = 10 cm 0.1 m B for steel = 1.6 × 1011 Nm −2
B =bulk modulus of Cu Here, p = 1.1 × 108 Pa
= 140 × 10 Pa
9
pV
|∆ V | =
p = 7 × 106 Pa B
∴ V = L3 = (0.1)3 = 0.001 m 3 1.1 × 108 × (0.32)
=
p 1.6 × 1011
From the relation B = − , we get
∆V / V = 2.2 × 10−4 m 3
Objective Problems
[ Level 1 ]
1. Young’s modulus of the material of a wire of length L and 10. When a metal wire elongates by hanging a load Mg on it,
L
radius r is Y N/ m 2 . If the length is reduced to and the gravitational potential energy of mass M decreases by
2 Mgl. This energy appears
r (a) as elastic potential energy completely
radius to , the Young’s modulus will be
2 (b) as thermal energy completely
(a) Y (b) 2Y (c) half as elastic potential energy and half as thermal energy
Y Y (d) as kinetic energy of the load completely
(c) (d)
4 2
11. Longitudinal stress of 1 kg/mm 2 is applied on a wire. The
2. Strain energy per unit volume in a stretched string is
1 percentage increase in length is (Y = 1011 N/m 2 )
(a) stress × strain (b) stress × strain
2 (a) 0.002 (b) 0.001 (c) 0.003 (d) 0.01
(c) (stress × strain) 2 (d) stress/strain 12. When a pressure of 100 atm is applied on a spherical ball,
3. The Young’s modulus of a wire is numerically equal to then its volume reduces to 0.01%. The bulk modulus of
the stress which will the material of the rubber in dyne/cm 2 is
(a) not change the length of the wire (a) 10 × 1012 (b) 100 × 1012 (c) 1 × 1012 (d) 20 × 1012
(b) double the length of the wire
(c) increase the length by 50% 13. A uniform cube is subjected to volume compression. If
(d) change the radius of the wire to half each side is decreased by 1%, then bulk strain is
(a) 0.01 (b) 0.06
4. The longitudinal extension of any elastic material is very
(c) 0.02 (d) 0.03
small. In order to have an appreciable change, the
material must be in the form of 14. A ball falling in a lake of depth 200 m shows 0.01%
(a) long thick wire (b) short thick wire decrease in its volume at the bottom. What is the bulk
(c) long thin wire (d) short thin wire modulus of the material of the ball?
(a) 19.6 × 108 N / m 2 (b) 19.6 × 109 N / m 2
5. The Young’s modulus of a perfectly rigid body
(c) 19.6 × 1010 N/ m 2 (d) 19.6 × 1012 N/ m 2
(a) is zero
(b) is unity 15. The Young’s modulus of a wire is Y . If the energy per
(c) is infinity unit volume is E, then the strain will be
(d) may have any finite non-zero value 2E E
(a) (b) E 2Y (c) EY (d)
6. In practice, Poisson’s ratio σ lies between Y Y
(a) − ∞ to + ∞ (b) 0 and + ∞ 16. The bulk modulus for an incompressible liquid is
(c) 0 and 0.5 (d) 0.5 and 1.0
(a) zero (b) unity
7. A cable that can support a load W is cut into two equal (c) infinity (d) between 0 and 1
parts. The maximum load that can be supported by either
part is 17. A steel wire of cross-sectional area 3 × 10−6 m 2 can
W W withstand a maximum strain of 10−3 . Young’s modulus of
(a) (b) (c) W (d) 2W
4 2 steel is 2 × 1011 N/ m 2 . The maximum mass this wire can
8. The only elastic modulus that applies to fluids is hold is
(a) 40 kg (b) 60 kg (c) 80 kg (d) 100 kg
(a) Young’s modulus (b) Shear modulus
(c) Modulus of rigidity (d) Bulk modulus 18. When a force is applied at one end of an elastic wire, it
9. For steel Y = 2 × 10 N/ m . The force required to
11 2 produces a strain E in the wire. If Y is the Young’s
modulus of the material of the wire, the amount of energy
double the length of a steel wire of area 1cm 2 is
stored per unit volume of the wire is given by
(a) 2 × 107 N (b) 2 × 106 N 1 1
(a) Y × E (b) (Y × E ) (c) Y × E 2 (d) (Y × E 2 )
(c) 2 × 10 N8
(d) 2 × 105 N 2 2
[ Level 2 ]
Only One Correct Option 7. A metal block is experiencing an atmospheric pressure of
1. In the given figure, if the dimensions of the wires are the 105 N/ m 2 . When the same block is placed in a vacuum
same and materials are different, Young’s modulus is chamber, the fractional change in its volume (the bulk
more for modulus of metal is 1.25 × 1011 N/ m 2 )
(a) 4 × 10−7
A (b) 2 × 10−7
(c) 8 × 10−7
Load

B
(d) 1 × 10−7

8. A brass rod of length 2 m and cross-sectional area


Extension 2.0 cm 2 is attached end to end to a steel rod of length L
and cross-sectional area 1.0 cm 2 . The compound rod is
(a) A (b) B
(c) Both (d) None of these subjected to equal and opposite pulls of magnitude
5 × 104 Nat its ends. If the elongations of the two rods are
2. Young’s modulus of rubber is 104 N/ m 2 and area of
equal, the length of the steel rod ( L ) is
cross-section is 2 cm 2 . If force of 2 × 105 dynes is applied
(Y brass = 1.0 × 1011 N/ m 2 and Y steel = 2.0 × 1011 N/ m 2 )
along its length, then its initial length l becomes
(a) 1.5 m
(a) 3 l (b) 4 l
(b) 1.8 m
(c) 2l (d) None of these
(c) 1m
3. An elastic material with Young’s modulus Y is subjected (d) 2m
to a tensile stress S, elastic energy stored per unit volume 9. An elevator cable is to have a maximum stress of
of the material is 7 × 107 N / m 2 to allow for appropriate safety factors. Its
YS S2 maximum upward acceleration is 1.5 m / s 2 . If the cable
(a) (b)
2 Y has to support the total weight of 2000 kg of a loaded
S2 S
(c) (d) elevator, the area of cross-section of the cable should be
2Y 2Y
(a) 3.22 cm 2 (b) 2.38 cm 2
4. The bulk modulus of water is 2.1 × 10 N / m . The
9 2
(c) 0.32 cm 2 (d) 8.23 cm 2
pressure required to increase the density of water by
0.1% is 10. A uniform steel bar of cross-sectional area A and length L
(a) 2.1 × 103 N / m 2 is suspended so that it hangs vertically. The stress at the
(b) 2.1 × 106 N / m 2 middle point of the bar is (ρ is the density of steel)
L Lρ g
(c) 2.1 × 105 N / m 2 (a) ρg (b)
2A 2
(d) 2.1 × 107 N /m 2 LA
(c) (d) Lρg
5. A thick rope of rubber of density 1.5 × 103 kg / m 3 and ρg
Young’s modulus 5 × 106 N / m 2 , 8 m in length, when
hung from ceiling of a room, the increase in length due to 11. One end of a uniform wire of length L and weight w is
its own weight is attached rigidly to a point in roof and a weight w1 is
suspended from its lower end. If S is the area of
(a) 96 × 10−3 m (b) 19.2 × 10−5 m
cross-section of the wire, the stress in the wire at a height
(c) 9.4 cm (d) 9.6 mm 3L
of from its lower end is
6. Four wires of the same material are stretched by the same 4
load. The dimensions of the wires are as given below. w
w1 +
The one which has the maximum elongation is of w 4
(a) 1 (b)
(a) diameter 1 mm and length 1 m S S
(b) diameter 2 mm and length 2 m w + 3w 
(c) diameter 0.5 mm and length 0.5 m  1 4 w1 + w
(c)   (d)
(d) diameter 3 mm and length 3 m S S
Elasticity 527

12. Two wires of same material and length but diameter in 18. A uniform elastic plank moves due to a constant force F
the ratio 1 : 2 are stretched by the same force. The placed over a smooth surface. The area of end face is S
potential energy per unit volume for the two wires when and Young’s modulus of the material is E. What is
stretched will be in the ratio average strain produced in the direction of the force?
(a) 16 : 1 (b) 4 : 1 F F F
(a) (b) (c) (d) Zero
(c) 2 : 1 (d) 1 : 1 SE 2SE 4 SE
13. The length of an elastic string is a metre when the tension 19. What is the approximate change in density of water in a
is 4 N, and b metre when the tension is 5 N. The length, in lake at a depth of 400 m below the surface? The density
metre, when the tension is 9 N, is of water at the surface is 1030 kg/m 3 and bulk modulus
(a) (a + b) (b) (4 b − 5a) of water is 2 × 109 N / m 2
(c) (5b − 4 a) (d) (9b − 9a)
(a) 4 kg/m 3 (b) 2 kg/m 3
14. The length of a metal wire is l1 , when the tension in it is (c) 6 kg/m 3 (d) 8 kg/m 3
T1 and is l2 when the tension is T2 . The unstretched length
of the wire is 20. A wire is elongated by 2 mm when a brick is suspended
l1T1 + l2T2 l1 + l2 from it. When the brick is immersed in water, the wire
(a) (b) contracts by 0.6 mm. What is the density of brick?
T1 + T2 2
l1T2 − l2T1 l1T2 − l2T1 (a) 3333 kg/m 3 (b) 4210 kg/m 3
(c) (d) (c) 5000 kg/m 3 (d) 2000 kg/m 3
T2 − T1 T2 + T1
21. A load suspended by a massless spring produces an
15. The Young’s modulus of brass and steel are respectively
extension of x cm in equilibrium. When it is cut into two
1.0 × 1010 N/ m 2 and 2 × 1010 N/ m 2 . A brass wire and a
unequal parts, the same load produces an extension of
steel wire of the same length are extended by 1 mm under 7.5 cm when suspended by the larger part of length
the same force, the radii of brass and steel wires are R B 60 cm. When it is suspended by the smaller part, the
and R S , respectively. Then, extension is 5.0 cm. Then,
(a) RS = 2 RB (b) RS =
RB (a) x = 12.5
2 (b) x = 3.0
RB (c) the length of the original spring is 90 cm
(c) RS = 4 RB (d) RS =
4 (d) the length of the original spring is 80 cm

16. A block of weight w produces an extension of 9 cm, when 22. A copper wire (Y = 1011 N / m 2 ) of length 8 m and a
it is hung by an elastic spring of length 60 cm and is cut steel wire (Y = 2 × 1011 N / m 2 ) of length 4 m, each of
into two parts one of length 40 cm and the other of length
20 cm. The same load w hangs in equilibrium supported 0.5 cm 2 cross- section are fastened end to end and
by both parts as shown in figure. The extension in cm stretched with a tension of 500 N.
now is (a) Elongation in copper wire is 0.8 mm
1
(b) Elongation in steel is th the elongation in copper wire
4
(c) Total elongation is 1.0 mm
60 cm (d) All of the above
40 cm
20 cm
23. A uniform cylinder rod of length L, cross-sectional area A
and Young’s modulus Y is acted upon by the forces
w w shown in the figure. The elongation of the rod is
(a) 9 (b) 6 (c) 3 (d) 2 L/3

17. Two wires of the same material (Young’s 3F F 2F


modulus = Y ) and same length L but radii R and 3FL 2FL 3FL 8FL
L, 2R
2R respectively, are joined end to end and a (a) (b) (c) (d)
5 AY 5 AY 8 AY 3 AY
weight w is suspended from the combination as
shown in the figure. The elastic potential L, R
24. A stress of 106 N / m 2 is required for breaking a material.
energy in the system is If the density of the material is 3 × 103 kg / m 3 , then what
3w2L 3w2L w
should be the length of the wire made of this material so
(a) (b)
4 πR 2Y 8πR 2Y that it breaks under its own weight?
5w2L w2L (a) 10 m (b) 33.3 m
(c) (d)
8πR 2Y πR 2Y (c) 5 m (d) 66.6 m
528 Objective Physics Vol. 1

25. A uniform pressure p is exerted on all sides of a solid 32. A 5 m long wire is fixed to the ceiling. A weight of 10 kg
cube at temperature t ° C. By what amount should the is hung at the lower end is 1 m above the floor. The wire
temperature be raised in order to bring the volume back to was elongated by 1 mm. The energy stored in the wire
that it had been before the pressure was applied. due to stretching is
(a) 0.01 J (b) 0.05 J
(Linear expansivity of the material of the cube is α and
(c) 0.02 J (d) 0.04 J
the bulk modulus of elasticity is β )
(a)

(b)
3pα 33. The stress versus strain graphs for wires of two materials
β β A and B are as shown in the figure. If Y A and Y B are the
p p Young’s modulii of the materials, then
(c) (d)
αβ 3αβ A

26. The length of an iron wire is L and area of cross-section is

Stress
A. The increase in length is l on applying the force F on
its two ends. Which of the statements is correct? B
60°
(a) Increase in length is inversely proportional to its length L
(b) Increase in length is proportional to area of cross -section A 30°
(c) Increase in length is inversely proportional to A Strain
(d) Increase in length is proportional to Young’s modulus
(a) YB = 2YA (b) YA = YB
27. The temperature of a wire of length 1 m and area of (c) YB = 3YA (d) YA = 3YB
cross-section 1cm 2 is increased from 0°C to 100°C. If the 34. The load versus elongation graph for four wires of the
rod is not allowed to increase in length, the force required same material is shown in the figure. The thickest wire is
will be (α = 10−5 / ° C and Y = 1011 N/ m 2 ) represented by the line
Load
(a) 103 N (b) 104 N
D
(c) 105 N (d) 109 N
C
28. Two wires of copper having the length in the ratio 4 : 1 B
and their radii ratio as 1 : 4 are stretched by the same A
force. The ratio of longitudinal strain in the two will be
(a) 1 : 16 (b) 16 : 1 x
O Elongation
(c) 1 : 64 (d) 64 : 1
(a) OD (b) OC
29. The interatomic distance for a metal is 3 × 10−10 m. If the (c) OB (d) OA
interatomic force constant is 3.6 × 10−9 N/ Å, then the
35. The strain-stress curves of three wires of different
Young’s modulus in N / m 2 will be
materials are shown in the figure. P , Q and R are elastic
(a) 1.2 × 1011 (b) 4.2 × 1011 limits of the wires. The figure shown that
(c) 10.8 × 10 8
(d) 2.4 × 1010 P

30. A force of 20 N is applied at one end of a wire of length Q


Stress

2 m and having area of cross-section 10−2 m 2 . The other


end of the wire is rigidly fixed. Its coefficient of linear R
expansion of the wire is α = 8 × 10−6 / ° C and Young’s
O
modulus Y = 2.2 × 1011 N/ m 2 and its temperature is Strain

increased by 5°C, then the increase in the tension of the (a) elasticity of wire P is maximum
(b) elasticity of wire Q is maximum
wire will be
(c) tensile strength of R is maximum
(a) 4.2 N (b) 4.4 N
(d) None of the above
(c) 2.4 N (d) 8.8 N
36. The density of a metal at normal pressure is ρ. Its density
31. Two wires of same diameter of the same material having
when it is subjected to an excess pressure p is ρ′. If B is
the length l and 2 l. If the force F is applied on each, the ρ′
ratio of the work done in the two wires will be the bulk modulus of the metal, the ratio is
ρ
(a) 1:2
1 B 1 p
(b) 1:4 (a) (b) 1 + (c) (d) 1 +
(c) 2:1 p p B B
1− 1−
(d) 1:1 B p
Elasticity 529

37. When a steel wire fixed at one end is pulled by a constant 4. The stress-strain graphs for two materials are shown in
force F at its other end, its length increases by l. Which of figure. (assume same scale)
the following statements is not correct? Ultimate tension
(a) Work done by the external force is Fl strength Fracture
(b) Some heat is produced in the wire in the process point

Stress
Fl Linear
(c) The elastic potential energy of the wire is limit
2
(d) The heat produced is equal to half of the elastic potential
energy stored in the wire
Strain E
38. A uniform rod of mass m, length L, area of cross- section material (i)
A and Young’s modulus Y hangs from the ceiling. Its Ultimate tension
elongation under its own weight will be Linear strength
limit Fracture point
mgL
(a) zero (b)

Stress
2 AY
mgL 2mgL
(c) (d)
AY AY

More than One Correct Options StrainE


material (ii)
1. A metal wire of length L , area of cross-section A and (a) Material (ii) is more elastic than material (i) and hence
modulus Y is stretched by a variable force F such that F is material (ii) is more brittle
always slightly greater than the elastic forces of (b) Material (i) and (ii) have the same elasticity and the same
resistance in the wire. When the elongation of the wire is l brittleness
(c) Material (ii) is elastic over a larger region of strain as
YAl 2
(a) the work done by F is compared to (i)
2L (d) Material (ii) is more brittle than material (i)
YAl 2
(b) the work done by F is 5. A rod of length l and negligible mass is suspended at its
L
YAl 2 two ends by two wires of steel (wire A) and aluminium
(c) the elastic potential energy stored in the wire is (wire B) of equal lengths (figure ). The cross-sectional
2L
YAl 2 areas of wires A and B are 1.0 mm 2 and 2.0 mm 2 ,
(d) the elastic potential energy stored in the wire is respectively.
4L
(Y Al = 70 × 109 Nm −2 and Y steel = 200 × 109 Nm −2 )
2. Two wires A and B of same length are made of same
material. The figure represents the load F versus
extension ∆x graph for the two wires. Then,
A B
F
Steel Al
B
A

m
(a) Mass m should be suspended close to wire A to have equal
stresses in both the wires
∆x
O (b) Mass m should be suspended close to B to have equal stresses
in both the wires
(a) the cross-sectional area of A is greater than that of B
(c) Mass m should be suspended at the middle of the wires to
(b) the elasticity of B is greater than that of A
have equal stresses in both the wires
(c) the cross-sectional area of B is greater than that of A
(d) Mass m should be suspended close to wire A to have equal
(d) the elasticity of A is greater than that of B
strain in both wires
3. A body of mass M is attached to the lower end of a metal 6. A copper and a steel wire of the same diameter are
wire, whose upper end is fixed. The elongation of the wire connected end to end. A deforming force F is applied to
is l. this composite wire which causes a total elongation of
(a) Loss in gravitational potential energy of M is Mgl
1 cm. The two wires will have
(b) The elastic potential energy stored in the wire is Mgl
1 (a) the same stress
(c) The elastic potential energy stored in the wire is Mgl (b) different streses
2
1 (c) the same strain
(d) Heat produced is Mgl (d) different strains
2
530 Objective Physics Vol. 1

Assertion and Reason 10. Assertion Bulk modulus of an incompressible fluid is


Directions (Q. Nos. 1-14) These question consists of two infinite.
statements each printed as assertion and reason. While Reason Density of incompressible fluid remains
answering these questions you are required to choose anyone constant.
of the following five responses.
(a) If both Assertion and Reason are true and Reason is 11. Assertion If value of Young’s modulus of elasticity of a
the correct explanation of Assertion. material is less, then it means material is ductile.
(b) If both Assertion and Reason are true but Reason is Reason If a material is easily stretchable, then its value
not the correct explanation of Assertion. of Young’s modulus of elasticity should be less.
(c) If Assertion is true but Reason is false.
(d) If Assertion is false but Reason is true. 12. Assertion Modulus of elasticity does depend upon the
(e) If both Assertion and Reason are false. dimensions of material.
1. Assertion Up to elastic limit of a stress-strain curve, the Reason Modulus of elasticity is a material property.
steel wire tends to regain its original shape when stress is
removed. 13. Assertion Up to the elastic limit strain ∝ stress.
Reason Within elastic limit, the wire follows Hooke’s law. Reason Up to elastic limit material returns to its original
shape and size. When external force is removed.
2. Assertion Young’s modulus of elasticity is not defined
for liquids. 14. Assertion Adiabatic bulk modulus of an ideal gas is
Reason Liquids cannot be stretched as wires. more than its isothermal bulk modulus.

3. Assertion Incompressible liquids have finite value of Reason Both the modulii are proportional to the
bulk modulus of elasticity. pressure of gas at that moment.
Reason Compressibility is inverse of bulk modulus of Match the Columns
elasticity.
1. Match the following columns.
4. Assertion If length of a wire is halved, its Young’s
modulus of elasticity will become two times. Column I Column II
Reason The ratio of longitudinal stress and longitudinal (A) Steel (p) Young’s modulus of
strain is called Young’s modulus of elasticity. elasticity
(B) Water (q) Bulk modulus of elasticity
5. Assertion The materials having low value of Young’s
modulus of elasticity are more ductile. (C) Hydrogen gas filled in a (r) Shear modulus of elasticity
chamber
Reason If Young’s modulus is less, they can be easily
stretched as wires.
2. Match the following columns.
6. Assertion Bulk modulus of elasticity of gases is process
Column I Column II
dependent.
2 −1
Reason More the pressure of gas more is the bulk (A) Coefficient of viscosity (p) [M L T −2 ]
modulus of elasticity of gas. (B) Surface tension (q) [ML 0 T −2 ]

7. Assertion Stress and modulus of elasticity have the (C) Modulus of elasticity (r) [ML −1 T 2 ]
same dimensions.
(D) Energy per unit volume of a (s) None
Reason Strain is dimensionless. fluid

8. Assertion If radius of cylindrical wire is doubled, then


this wire can bear four times stress. 3. Match the following columns.
Reason By doubling the radius, the area of cross-section Column I Column II
will become four times.
(A) Stress × strain (p) J
9. Assertion If a wire is stretched, only half of the work (B) YA (q) N/m
done in stretching the wire remains stored as elastic l
potential energy.
(C) Yl 3 (r) J/m 3
Reason Potential energy stored in the wire is Fl
(D) (s) m
1
(stress) × (strain) AY
2
Elasticity 531

Entrance Gallery 8. A material has Poisson’s ratio 0.50. If a uniform rod of it


suffers a longitudinal strain of 2 × 10−3 , then the
2014 percentage change in volume is [WB JEE]
1. One end of a horizontal thick copper wire of length 2L (a) 0.6 (b) 0.4
(c) 0.2 (d) zero
and radius 2R is welded to an end of another horizontal
thin copper wire of length L and radius R. When the 2010
arrangement is stretched by applying forces at two ends,
the ratio of the elongation in the thin wire to that in the 9. A 0.1 kg mass is suspended from a wire of negligible
thick wire is [JEE Advanced] mass. The length of the wire is 1 m and its cross-sectional
(a) 0.25 (b) 0.50 area is 4.9 × 10−7 m 2 . If the mass is pulled a little in the
(c) 2.00 (d) 4.00 vertically downward direction and released, it performs
2. The pressure that has to be applied to the ends of a steel simple harmonic motion of angular frequency
wire of length 10 cm to keep its length constant when its 140 rad s −1 . If the Young’s modulus of the material of the
temperature is raised by 100°C is (For steel, Young’s wire is n × 109 Nm −2 , the value of n is [IIT JEE]
modulus is 2 × 1011 Nm − 2 and coefficient of thermal (a) 4 (b) 2
expansion is 1.1 × 10− 5 K − 1 ) [JEE Main] (c) 4.5 (d) 5
(a) 2.2 × 108 Pa (b) 2.2 × 109 Pa
10. Two wires are made of the same material and have the
(c) 2.2 × 107 Pa (d) 2.2 × 106 Pa
same volume. However, wire 1 has cross-sectional area A
3. The length of the wire is increased by 2% by applying a and wire 2 has cross-sectional area 3A. If the length of
load of 2.5 kg-wt. What is the linear strain produced in wire 1 increases by ∆x on applying force F, how much
the wire? [J&K CET] force is needed to stretch wire 2 by the same amount?
(a) 0.1 (b) 0.01 (c) 0.2 (d) 0.02 (a) F (b) 4F [AIEEE]
(c) 6F (d) 9F
2013
11. Which of the following substances has the highest
4. If in a wire of Young’s modulus Y , longitudinal strain X elasticity? [Karnataka CET]
is produced, then the value of potential energy stored in (a) Sponge (b) Steel
its unit volume will be [Karnataka CET] (c) Rubber (d) Copper
(a) 0.5 YX 2 (b) 0.5 Y 2 X
12. There is no change in the volume of a wire due to the
(c) 2YX 2 (d) YX 2 change in its length on stretching. The Poisson’s ratio of
2011 the material of the wire is [OJEE]
1 1
(a) + (b) −
5. The average depth of Indian ocean is about 3000 m. The 2 2
∆V 1 1
fractional compression, of water at the bottom of the (c) + (d) −
V 4 4
ocean (given that the Bulk modulus of the water
= 2.2 × 109 Nm −2 and g = 10 ms −2 ) is [Kerala CEE]
13. The increase in pressure required to decrease the 200 L
volume of a liquid by 0.008% in kPa is (Bulk modulus of
(a) 0.82% (b) 0.91% (c) 1.36% (d) 1.24%
the liquid = 2100 MPa) [MHT CET]
(e) 1.52%
(a) 8.4 (b) 84
6. Identify the incorrect statement. [Kerala CEE] (c) 92.4 (d) 168
(a) Young’s modulus and shear modulus are relevant only for
14. Four wires of the same material are stretched by the same
solids
(b) Bulk modulus is relevant for solids, liquids and gases load. Which one of them will elongate most if their
(c) Alloys have larger values of Young’s modulus than metals dimensions are as follows? [MHT CET]
(d) Metals have larger values of Young’s modulus than (a) L = 100 cm, r = 1 mm (b) L = 200 cm, r = 3 mm
elastomers (c) L = 300 cm, r = 3 mm (d) L = 400 cm, r = 4 mm
(e) Stress is not a vector quantity
15. Which of the following relations is true? [MHT CET]
7. The following four wires of length L and radius r are (a) Y = 2 η (1 − 2σ ) (b) Y = 2η (1 + 2 σ )
made of the same material. Which of these will have the (c) Y = 2η (1 − σ ) (d) (1 + σ )2 η = Y
largest extension, when the same tension is applied?
[Karnataka CET] 16. For a given material, the Young’s modulus is 2.4 times
(a) L = 400 cm , r = 0.8 mm (b) L = 300 cm , r = 0.6 mm that of rigidity modulus. Its Poisson’s ratio is [MHT CET]
(c) L = 200 cm , r = 0.4 mm (d) L = 100 cm , r = 0.2 mm (a) 2.4 (b) 1.2 (c) 0.4 (d) 0.2
Answers
Level 1
Objective Problems
1. (a) 2. (a) 3. (b) 4. (c) 5. (c) 6. (c) 7. (c) 8. (d) 9. (a) 10. (c)
11. (b) 12. (c) 13. (d) 14. (a) 15. (a) 16. (c) 17. (b) 18. (d)

Level 2
Only One Correct Option
1. (a) 2. (c) 3. (c) 4. (b) 5. (c) 6. (c) 7. (c) 8. (d) 9. (a) 10. (b)
11. (c) 12. (a) 13. (c) 14. (c) 15. (b) 16. (d) 17. (c) 18. (b) 19. (b) 20. (b)
21. (a) 22. (d) 23. (d) 24. (b) 25. (d) 26. (c) 27. (b) 28. (b) 29. (a) 30. (b)
31. (a) 32. (b) 33. (d) 34. (a) 35. (d) 36. (a) 37. (d) 38. (b)

More than One Correct Options


1. (a,c) 2. (c) 3. (a,c,d) 4. (c,d) 5. (b,d) 6. (a,d)

Assertion and Reason


1. (c) 2. (a) 3. (b) 4. (d) 5. (a) 6. (b) 7. (a,b) 8. (d) 9. (c) 10. (b)
11. (a) 12. (a) 13. (d) 14. (b)

Match the Columns


1. (A→p,q,r; B→q; C→q) 2. (A→s; B→q; C→r; D→r) 3. (A→r; B→q; C→p; D→s)

Entrance Gallery
1. (c) 2. (a) 3. (d) 4. (a) 5. (c) 6. (e) 7. (d) 8. (d) 9. (a) 10. (d)
11. (b) 12. (b) 13. (b) 14. (a) 15. (d) 16. (d)

Solutions
Level 1 : Objective Problems Percentage increase in length = 10−5 × 100 = 0.001%
100
1. Young’s modulus is property of material. 12. B = = 106 atm = 1011 N/ m2 = 1012 dyne/cm2
Stress 0.01/100
3. Y = , Y = Stress for unit strain or ∆l = l
Strain 13. If side of the cube is L, then V = L3
Fl l
4. ∆l = or ∆l ∝ ⇒
dV
=3
dL
AY A V L
5. For perfectly rigid body, ∆l = 0 ∴ % change in volume = 3 × (% change in length)
∴ Strain = 0 or Y = ∞ = 3 × 1% = 3%
∆V
7. (Stress)1 = (Stress)2 ∴ Bulk strain, = 0.03
V
Maximum load does not depend on the length.
∆p h ρg 200 × 103 × 9.8
9. Stress required to double the length is called Young’s 14. B = = =
∆V / V 0.1 /100 1 /1000
modulus.
F = 19.6 × 108 N / m2
∴ Y= 1
A 15. Energy per unit volume = × Y × (strain)2
or F = Y ⋅ A = 2 × 1011 × 10−4 = 2 × 107 N 2
2E
stress 106 ∴ Strain =
11. Longitudinal strain = = 11 = 10−5 Y
Y 10
Elasticity 533

− ∆p
16. B = 8. ( ∆l )b = ( ∆l )s
( ∆V / V )
For incompressible liquid, ∆V = 0 F F
∴ B=∞
17. Maximum stress = Y × (maximum strain)  Fl  =  Fl  (Fb = Fs )
or    
 AY  b  AY  s
Mg
= 2 × 1011 × 10−3 = 2 × 108
A  l  = l 
or    
 AY  b  AY  s
2 × 108 × 3 × 10−6
∴ M= = 60 kg
10 A Y 
∴ l s =  s s  lb
1  Ab Yb 
18. Energy per unit volume = × stress × strain
2
 1.0 × 2.0 × 1011 
1
= × (Y × strain) × strain =  (2m) = 2 m
 2.0 × 1.0 × 10 
11
2
1
= × Y × E2 9. Tmax = m ( g + a ) = ( 2000) (9.8 + 1.5) = 22600 N
2
T
Maximum stress = max
Level 2 : Only One Correct Option Area
Tmax
∴ Area =
1. F =   ⋅ ∆l
YA
Maximum stress
 l 
22600
YA =
i.e. F - ∆l graph is a straight line with slope or slope 7 × 107
l
proportional to Y . = 3.22 × 10−4 m2 = 3.22 cm2
(Slope)A > (slope)B 10. At middle, T = weight of half the length of steel bar
∴ YA > YB
=   Aρg
L
Fl ( 2)l  2
2. ∆l = = =l
AY (104 )( 2 × 10−4 ) T Lρg
∴ Stress = =
∴ New length will become 2l. A 2
Stress S 3L
3. Strain = = 11. At length from lower end, tension in the wire.
Y Y 4
1 3
Now, energy stored per unit volume = × stress × strain T = suspended load + × weight of wire
2 4
1 S S2 3w
= × S× = = w1 +
2 Y 2Y 4
ρ 3w
4. dρ = ⋅ dp w1 +
B T 4
∴ Stress = =
dρ dp
or = S S
ρ B Fl ∆l  F 
12. ∆l = ⇒ = 
0.1 dp AY l  AY 
=
100 2.1 × 109 1
Potential energy per unit volume = × stress × strain
∴ dp = 2.1 × 106 N/m2 2
1 F F
mgl u= × ×
5. Due to own weight, ∆l = 2 A AY
2AY
1 1
or u ∝ 2 or u ∝ 4
(lAρ) gl l 2ρg A d
= =
2AY 2Y 4
u1  d 2  16
(8) (1.5 × 10 ) (9.8)
2 3 ∴ =  =
= u2  d1  1
2 × 5 × 106
13. Let l be the natural length and K  =
YA 
 be the force
= 9.4 × 10−2 m = 9.4 cm  l 
Fl constant of wire. Then,
6. ∆l =
AY a =l +
4  F = K ∆l or ∆l = F 
 
l  π 2 K  K
or ∆l ∝ A = d 
d2  4 
and b =l +
5
l K
Therefore, ∆l will be maximum for that wire for which is
d2 1
or = ( b − a)
maximum. K
534 Objective Physics Vol. 1

and l = ( 5a − 4b ) Tx dx FL
∆l = ∫=
L

ES 02ES
Now when T = 9N
9 ∆l F
l′ =l + ∴ Average strain = =
K L 2ES
= ( 5a − 4b ) + 9 ( b − a ) ρ
19. d ρ = ⋅ dp
= ( 5b − 4a ) B

14. Let natural length be l and force constant is K  =


YA  dp = ρgh = (1030)(10)( 400)
 . Then,
 l  = 4.12 × 106 N/m2
T1
l1 = l +  F = K∆l or ∆l = F  1030
  ∴ dρ = × 4.12 × 106 = 2.12 kg/m 3
K  K 2 × 109
T2
and l2 = l +
K 20. ∆l ∝ F
Solving these two equations, we get ∆l1 F1 Weight
= =
l T −l T ∆l 2 F2 Weight − upthrust
l= 1 2 2 1
T2 − T1
2 V ρg
Fl Fl =
15. ∆l = = 1.4 V ρg − V ρw g
AY πR 2Y
2
∆l , F and l are same or ρ= ρw
0.6
Hence, R 2Y = constant
2
RS Y 1 = × 1000
or = B = 0.6
RB YS 2
= 3333 kg/m 3
R
or RS = B Fl
2 21. ∆l = (can also be applied for a spring)
AY
1
16. Force constant, k ∝ ∴ ∆l ∝ l
l
7.5 60
Let k be the force constant of original spring. =
5.0 l 2
60 3
Then, k40 = k= k
40 2 ∴ l 2 = 40 cm
60 ∴ Length of original spring is (60 + 40) cm = 100 cm
and k20 = k = 3k x 100
20 Now, =
9k 7.5 60
knet = k40 + k20 =
2 ∴ x =12.5 cm
500 × 8
22. ( ∆l )C = 
F 1 Fl 
Now, ∆l = ∝  =
k k  AY  C 0.5 × 10−4 × 1011

∴ 
∆l ′ = 
k  Copper Steel
 (9 cm)
 9k/2  F F
= 2 cm
wL wL = 0.8 × 10−3 m = 0.8 mm
17. ∆l1 = , ∆l 2 =
( ∆l ) S = 
( 4 πR 2 )y πR 2Y Fl 

 AY  S
∴U=
1 1
K 1 ( ∆l1 )2 + K 2 ( ∆l 2 )2  K = YA 
  500 × 4
2 2  L  =
2 2 0.5 × 10−4 × 2 × 1011
1 Y ( 4 πR 2 )  wL  1 Y ( πR 2 )  wL 
= × × + × × = 0.2 × 10−3 m
2 L  4 πR Y 
2
2 L  πR 2Y 
= 0.2 mm
5w2 L
= 1
8 πR 2Y ( ∆l ) S = ( ∆l )C
F 4
18. Tension at distance x from end A , Tx = F − x
L and ∆l = 0.8 + 0.2 = 1.0 mm
x 23. The free body diagrams of two parts are shown in figure.
2L/3
F
3F 3F
B A
L/3
Total change in length, 2F 2F
Elasticity 535

Both parts are stretched. Therefore, total elongation i.e for the same load, thickest wire will show minimum
∆l = ∆l1 + ∆l 2 elongation. So, graph D represents the thickest wire.
3F   2F  
2L L 35. As, stress is shown on x-axis and strain on y-axis.
 3  3
= + 1 1
AY AY So, we can say that Y = cot θ = =
tan θ slope
8FL
= So, elasticity of wire P is minimum and of wire R is
3AY maximum.
mg m 1
24. = maximum breaking stress 36. ρ = or ρ ∝ (as m = constant)
A V V
( Alρ) g ρ′ V
or = σ max ∴ =
A ρ V1
σ V B = − p 
∴ l max = max =  
ρg V + dV  dV / V 
106 =
V
=
1
=
( 3 × 103 )(10) Vp p
V − 1−
B B
= 33.33 m
 ∆V  37. The heat produced is equal to the elastic potential energy
25.   = γ∆t = 3 α( ∆t )(by change in temperature)
 V 1 stored in the wire.
 ∆V  = p (by pressure) 38. Tension at distance x from the bottom,
and  
 V 2 β
T = 
mg 
 xdt
 ∆V  =  ∆V  or 3α ( ∆t ) = p  L 
   
 V 1  V  2 β T ⋅ dx
Now, dl =
p AY
∴ ∆t = L mgL
3αβ ∴ ∆l = ∫ dl =
0 2AY
FL 1
26. ∆l = ⇒l ∝
YA A
More than One Correct Options
27. F = force developed = YAα ( ∆θ)
1. W F = elastic potential energy stored in the wire
= 1011 × 10−4 × 10−5 × 100 =104 N
YAl 2
k ( ∆l )2 =   (l )2 =
1 1 YA
28. Strain ∝ stress ∝
F =
A 2 2 L  2L
2 Fl
A2  r2 
2 2. ∆x =
=   =   =
4 16
Ratio of strain = AY
A1  r1  1 1
F = 
AY 
∴  ∆x
3.6 × 10−9 N/Å  L 
29. Y = = 1.2 × 1011 N /m2
3 × 10−10 m YA
i.e. F versus ∆x graph is a straight line of slope .
30. Increase in tension of wire = YAα ∆θ L
(Slope)B > (Slope)A
= 8 × 10−6 × 2.2 × 1011 × 10−2 × 10−4 × 5
= 8.8 N ∴  YA  >  YA  or ( A ) > ( A )
   
1  L B  L A B A
31. W = Fl
2 They are of same material.
∴ W ∝ l (F is constant) Hence, YB = YA
W1 l1 l 1
∴ = = = 3. Half of energy is lost in heat and rest half is stored as elastic
W2 l 2 2l 2 potential energy.
1 1 Mgl
32. W = × F × l = mg l l= …(i)
2 2 AY
1
= × 10 × 10 × 1 × 10−1 U = K l 2 =   l 2
1 1 YA
…(ii)
2 2 2 L 
= 0.05 J From Eqs. (i) and (ii), we can prove that
Y tan θA tan 60° 3 1
33. A = = = =3 U = Mgl
YB tan θB tan 30° 1 / 3 2
⇒ YA = 3YB 4. It is clear from the two graphs, the ultimate tensile strength
FL for material (ii) is greater, hence material (ii) is elastic over
34. l =
AY larger region as compared to material (i).
1 For material (ii) fracture point is nearer, hence it is more
∴ l∝ (Y , L and F are constants)
r2 brittle.
536 Objective Physics Vol. 1

5. Let the mass is placed at x from the end B. 6. Consider the diagram where a deforming force F is applied
to the combination.
Stress F /A
B For steel wire, Ysteel = =
Strain Strain
A Al
Steel TA x TB
F
Copper Steel
l–x wire wire
m
where, F is tension in each wire and A is cross-section area
Let TA and TB be the tensions in wire A and wire B, of each wires.
respectively.
As F and A are same for both the wires, hence, stress will be
For the rotational equilibrium of the system, same for both the wires.
∑τ =0 (Total torque = 0) Stress Stress
(Strain)steel = , (Strain) copper =
⇒ TB x − TA ( l − x ) = 0 Ysteel Ycopper
TB l − x Ysteel ≠ Ycopper
⇒ = ...(i) As,
TA x Hence, the two wires will have different strains.
TA
Stress in wire A = SA =
aA Assertion and Reason
TB 1. Hooke’s law, stress = Strain
Stress in wire B = SB =
aB
A → Proportional limit
where, aA and aB are cross-sectional areas of wire A and B, B → Elastic limit
respectively.
By question, aB = 2aA
Now, for equal stress SA = SB
TA TB TB a B
⇒ = ⇒ = =2
a A aB TA a A
Is obey up to proportional limit.
l−x l − ∆p
⇒ =2 ⇒ −1 = 2 3. Bulk modulus, B =
x x ∆V / V
l
⇒ x= For incompressible fluids, ∆V = 0
3
∴ B=∞
2l
⇒ l − x = l − l/ 3 =
3 4. Young’s modulus is the property of material. It does not
depend on length of the wire.
Hence, mass m should be placed closer to B.
For equal strain, (strain )A = (strain )B 6. If pressure of gas is more, it is more difficult to compress a
gas. Therefore, bulk modulus is high.
(YA ) YB
⇒ = (where, YA and YB are Young, modulii) 8. Load bearing capacity will become four times not the stress
SA SB
bearing capacity.
Ysteel YAl
⇒ = 1
9. Potential energy per unit volume is (stress) × strain.
TA /aA TB / aB 2
 2 aA 
× B = 
Ysteel TA a x  10. Incompressible fluid is that which cannot be compressed
⇒ =   
YAl TB aA  l − x   aA  ( ∆V = 0) by applying pressure on it.

200 × 109 2x 14. B S = γp = Adiabatic bulk modulus of elasticity.


⇒ =
70 × 109 l−x BT = p = Isothermal bulk modulus of elasticity
B S > BT and both are proportional to p.
20 2x
⇒ =
7 l−x Entrance Gallery
10 x
⇒ = 1. The elongation in the thin wire
7 l−x
FL FL L
⇒ 10l − 10 x = 7x Q ∆l = = ⇒ ∆l ∝
AY ( πr 2 ) Y A
⇒ 17x = 10l
10 l ∆l1 L/R 2
⇒ x= ∴ = =2
17 ∆l 2 2L/( 2R )2
10l 7l 2. According to Hooke’s law,
l−x =l− =
17 17 Tensile stress F /A
Young’s modulus, Y = =
Hence, mass m should be placed closer to wire A. Tensile strain ∆L/L
Elasticity 537

If the rod is compressed, then compressive stress and strain 10. As wires are of same material, we can write A1l1 = A2l 2
appear. Their ratio Y is same as that for tensile case. Al A × l1 l1 l
⇒ l2 = 1 1 = = ⇒ 1 =3
Given, length of a steel wire, L =10 cm, A2 3A 3 l2
Temperature, ∆θ = 100° C, F
Y = 2 × 1011 N/m2 and α = 1.1 × 10− 5 K − 1 Now, ∆ x1 = 1 l1 …(i)
AY
As, length is constant. F
∆ x2 = 2 l 2 …(ii)
∆L 3AY
∴ Strain == α∆θ
L F F
Here, ∆x1 = ∆x 2 ⇒ 2 l 2 = 1 l1
Now, pressure = stress = Y × strain 3AY AY
= 2 × 1011 × 1.1 × 10−5 × 100 F2 = 3F1 × 1
l
= 2.2 × 108 Pa l2
∆l = 3F1 × 3 = 9F
3. So, percentage change in length of wire = × 100%
l 11. Out of the given substances, steel has greater value of
∆l ∆l 2 Young’s modulus. Therefore, steel has highest elasticity.
× 100 = 2% ⇒ = = 0.02
l l 100 πx 2 L
12. Volume of cylindrical wire, V = ...(i)
4. Given, Young’s modulus = Y , longitudinal strain = X 4
The potential energy stored is given by where, x is the diameter of wire.
1 Differentiating both sides of Eq. (i), we get
= × Stress × Strain
2 dV π  2xL + x 2 ⋅ dL 
=
1 1
= × Y × X × X = YX 2 = 0.5YX 2 (Q Stress = Y × Strain) dx 4  dx 
2 2
∆p Also, volume remains constant.
5. We know that B = dV dL
∆V /V ∴ = 0 ⇒ 2xL + x 2 =0
dx dx
∆V ∆p ρgh 103 × 10 × 3000
⇒ = = = = 1.36 % dx
V B B 2.2 × 109 dL 1
⇒ 2xL = − x 2 ⇒ x =−
6. Metals have larger values of Young’s modulus, than dx dL 2
elastomers, and the alloys having high densities, i.e. alloys L
have larger values of Young’s modulus than metals. 1
Poisson’s ratio = −
F L
7. Young’s modulus, Y = × 2
A l ∆p
13. Bulk modulus, K = V
F L L ∆V
Y= × ⇒ Extension, l ∝ 2
πr 2
l r K∆ V 2100 × 106 × 0008
.
∆p = ⇒ ∆p = = 84 kPa
Option (d) has the largest extension, when the same tension V 200
is applied. FL
Lateral strain 14. As, ∆L =
8. Poisson’s ratio, σ = AY
Longitudinal strain
Because, wires of the same material are stretched by the
∆D ∆r
same load. So, F and Y will be constant.
∆r ∆L
= D = r or =σ× L
∆L ∆L r L ∴ ∆L ∝ 2
L L πr
= 0.5 × 2 × 10−3 = 10−3 100 100 100
∆L1 = = = × 106
Volume of rod, V = πr 2 L π × (1 × 10−3 )2 π × 10−6 π

V + ∆V = π (r − ∆r )2 ( L + ∆L ) 200 200 22.2


∴ ∆L2 = = = × 106
π × ( 3 × 10−3 )2 π × 9 × 10−6 π
Neglecting ∆r 2 and ∆r , ∆L, we get
300 300 33.3
2 πrL ∆r ∴ ∆L3 = = = × 106
or ∆V /V = πr 2 ∆L / πr 2 L − π × ( 3 × 10−3 )2 π × 9 × 10−6 π
πr 2 L
∆V ∆L ∆r 400 400 25
⇒ = −2 ∴ ∆L4 = = = × 106
V L r π × ( 4 × 10−3 )2 π × 16 × 10−6 π
= 2 × 10− 3 − 2 × 10− 3 = 0 We can see that, L =100 cm and r =1 mm will elongate most.
k YA 15. We know that
9. For SHM, we know that ω = =
m lm Y = (1 + σ )2η
(n × 109 )(4.9 × 10−7 ) 16. As, Y = 2η(1 + σ )
=
1 × 0.1 ⇒ 2.4η = 2η(1 + σ )
Given, ω =140 rads −1 in above equation, we get ⇒ 1.2 = 1 + σ
n= 4 ⇒ σ = 0.2
13
Fluid Mechanics

13.1 Definition of a Fluid


Fluid mechanics deals with the behaviour of fluids at rest and in motion. Fluids Chapter Snapshot
comprise the liquid and gas (or vapour) phases of the physical forms in which matter exists. ● Definition of a Fluid
The distinction between a fluid and the solid state of matter is clear, if you compare fluid
● Density of a Liquid
and the solid behaviour. A solid deforms when a shear stress is applied but it does not
continue to increase with time. However, if a shear stress is applied to a fluid, then ● Pressure in a Fluid
deformation continues to increase as long as the stress is applied. We may alternatively ● Archimedes’ Principle
define a fluid as a substance that cannot sustain a shear stress when at rest. ● Flow of Fluids
F F ● Bernoulli’s Equation
● Applications Based on
Bernoulli’s Equation
● Viscosity
● Stoke’s Law and
(a) Solid (b) Fluid Terminal Velocity
Fig. 13.1 Behaviour of a solid and a fluid, under the ● Surface Tension
action of a constant shear force

In the present chapter, we shall deal with liquids. An ideal liquid is incompressible and
non-viscous in nature. An incompressible liquid means the density of the liquid is constant,
it is independent of the variations in pressure. A non-viscous liquid means that, parts of the
liquid in contact do not exert any tangential force on each other. Thus, there is no friction
between the adjacent layers of a liquid. The force by one part of the liquid on the other part
is perpendicular to the surface of contact.

13.2 Density of a Liquid


Density (ρ) of any substance is defined as the mass per unit volume and it can be
expressed as
Mass m
ρ= or ρ =
Volume V

Relative Density (RD)


In case of a liquid, sometimes an another term relative density (RD) is defined. It is
the ratio of density of the substance to the density of water at 4°C. Hence,
Density of substance
RD =
Density of water at 4° C
Fluid Mechanics 539

Relative density is a pure ratio. So, it has no units. It is Effect of Pressure on Density
also sometimes referred as specific gravity.
As pressure is increased, volume decreases and hence
Density of water at 4°C in CGS is 1 g/cm 3 . density will increase. Thus,
Therefore, numerically the RD and density of substance 1
(in CGS) are equal. In SI units the density of water at 4°C ρ∝
V
is 1000 kg/m 3 . ρ′ V V V ρ′ 1
∴ = = = or =
ρ V ′ V + dV  
dp ρ dp
X Example 13.1 Relative density of an oil is 0.8. V − V 1−
Find the absolute density of oil in CGS and SI units. B B
Sol. Density of oil (in CGS) = (RD) g/ cm3 Here, dp = change in pressure,
= 0.8 g/cm3 = 800 kg/m3 and B = bulk modulus of elasticity of the liquid
ρ
Therefore, ρ′ =
Density of a Mixture of Two or More Liquids 1 − dp/ B
Here, we have two cases :
Case 1 Suppose two liquids of densities ρ1 and ρ 2 13.3 Pressure in a Fluid
having masses m1 and m2 are mixed together. Then, the
When a fluid (either liquid or gas) is at rest, it exerts a
density of the mixture will be
force perpendicular to any surface in contact with it, such as
Total mass
ρ= a container wall or a body immersed in the fluid.
Total volume
While the fluid as a whole is at rest, the molecules that
( m + m2 ) ( m1 + m2 )
= 1 = makes up the fluid are in motion, the force exerted by the
(V1 + V2 )  m1 m2  fluid is due to the molecules colloiding with their
 + 
 ρ1 ρ 2  surroundings.
2ρ1ρ 2 dA
If m1 = m2 , then ρ =
ρ1 + ρ 2 dF⊥ dF⊥

Case 2 If two liquids of densities ρ1 and ρ 2 having


volumes V1 and V2 are mixed together, then the density of
mixture is Fig.13.2
Total mass If we think of an imaginary surface within the fluid,
ρ=
Total volume then fluid on the two sides of the surface exerts equal and
m1 + m2 ρ1V1 + ρ 2V2 opposite forces on the surface, otherwise the surface
= = would accelerate and the fluid would not remain at rest.
V1 + V2 V1 + V2
ρ + ρ2 Consider a small surface of area dA centered on a point
If V1 = V2 , then ρ= 1 on the fluid, the normal force exerted by the fluid on each
2
side is dF⊥ . The pressure p is defined at that point as the
normal force per unit area, i.e.
Effect of Temperature on Density dF
As the temperature of a liquid is increased, the mass p= ⊥
dA
remains the same while the volume is increased and hence,
the density of the liquid decreases ( as ρ ∝1/V ). Thus, If the pressure is the same at all points of a finite plane
surface with an area A, then
ρ′ V V V
= = = F
ρ V ′ V + dV V + V γ ∆θ p= ⊥
A
ρ′ 1 where, F⊥ is the normal force on one side of the surface.
or =
ρ 1 + γ ∆θ The SI unit of pressure is pascal, where
Here, γ = thermal coefficient of volume expansion 1 pascal = 1Pa = 1.0 N/ m 2
and ∆θ = rise in temperature One unit used principally in meterology is the bar which
ρ is equal to 10 5 Pa.
∴ ρ′ = 1 bar = 10 5 Pa
1 + γ ∆θ
540 Objective Physics Vol. 1

Atmospheric Pressure (p0 ) force on the top surface is ( p + dp) A. The fluid element is in
equilibrium, so the total y-component of force including the
It is pressure of the earth’s atmosphere. This changes weight and the forces at the bottom and top surfaces must be
with weather and elevation. Normal atmospheric pressure at zero.
sea level (an average value) is 1.013 × 10 5 Pa. Thus,
ΣF y = 0
1 atm = 1.013 × 10 5 Pa ∴ pA − ( p + dp) A − ρgA dy = 0
/ Fluid pressure acts perpendicular to any surface in the fluid dp
no matter how that surface is oriented. Hence, pressure has or = − ρg …(i)
no intrinsic direction of its own and it is a scalar quantity. By dy
contrast, force is a vector with a definite direction.
This equation shows that when y increases, p decreases,
i.e. as we move upward in the fluid, pressure decreases.
Absolute Pressure and Gauge If p1 and p2 be the pressures at elevations y1 and y2 and
Pressure if ρ and g are constant, then integrating Eq. (i), we get
The excess pressure above atmospheric pressure is p2 y2

usually called gauge pressure and the total pressure is called ∫p 1


dp = − ρg ∫ dy
y1
absolute pressure. Thus, or p2 − p1 = − ρg ( y2 − y1 ) …(ii)
Gauge pressure = absolute pressure It’s often convenient to express Eq. (ii) in terms of the
− atmospheric pressure depth below the surface of a fluid. Take point 1 at depth h
Absolute pressure is always greater than or equal to below the surface of fluid and let p represents pressure at this
zero. While gauge pressure can be negative also. point. Take point 2 at the surface of the fluid, where the
pressure is p0 (subscript zero for zero depth). The depth of
Variation in Pressure with Depth point 1 below the surface is
If the weight of the fluid can be neglected, then pressure
in a fluid is the same throughout its volume. But often the
p2
fluid’s weight is not negligible and under such condition
pressure increases with increasing depth below the surface. p1
y2
y1

dw
dy
dy Fig. 13.4

y h = y2 − y1
pA and Eq. (ii) becomes
Fig. 13.3 p0 − p = − ρg ( y2 − y1 ) = − ρgh
∴ p = p0 + ρgh …(iii)
Let us now derive a general relation between the
pressure p at any point in a fluid at rest and the elevation y of Thus, pressure increases linearly with depth, if ρ and g
that point. We will assume that the density ρ and the are uniform. A graph between p and h is shown below.
acceleration due to gravity g are the same throughout the p
fluid. If the fluid is in equilibrium, then every volume p0 p0
element is in equilibrium.
Consider a thin element of fluid with height dy. The h
A B
bottom and top surfaces each have area A and they are at
p = p0 + ρgh
elevations y and y + dy above some reference level, where p0
y = 0. The weight of the fluid element is
dw = (volume) (density) ( g ) = ( A dy)(ρ)( g ) h p A = p B = p 0 + ρgh
or dw = ρgAdy Fig. 13.5 Fig. 13.6
What are the other forces in Y -direction on this fluid
element? Call the pressure at the bottom surface p, the total Further, the pressure is the same at any two points at the
Y-component of upward force is pA. The pressure at the top same level in the fluid. The shape of the container does not
surface is p + dp and the total y-component of downward matter.
Fluid Mechanics 541

and ρ = 750 kg /m3


Pascal’s Law 20 × 9.8 F
Thus, −
It states that “pressure applied to an enclosed fluid is π × (5 × 10−2 )2 π × (17.5 × 10−2 )2
transmitted undiminished to every portion of the fluid and = 1.5 × 750 × 9.8
the walls of the containing vessel”. which gives, F = 1.3 × 103 N
F1 / Atmospheric pressure is common to both pistons and has
Car
A1 A2 been ignored.

Extra Knowledge Points


■ At same point on a fluid pressure is same in all
Fig. 13.7 directions. In the figure,
A well known application of Pascal’s law is the
hydraulic lift used to support or lift heavy objects. It is p4
schematically illustrated in figure. p1 p3

A piston with small cross-section area A1 exerts a force p2


F1 on the surface of a liquid such as oil. The applied pressure p1 = p2 = p3 = p4
F
p = 1 is transmitted through the connecting pipe to a larger ■ Forces acting on a fluid in equilibrium have to be
A1 perpendicular to its surface. Because, it cannot sustain
the shear stress.
piston of area A2 . The applied pressure is the same in both
cylinders, so
■ In the same liquid pressure will be same at all points at
the same level.
A F A
p2 = 2 = 2 or F2 = 2 . F1 For example, in the figure
A1 A2 A1 r1
p0 p0

Now, since A2 > A1 , therefore, F2 > F1 . Thus, hydraulic 1 h1 r2 2 h2


lift is a force multiplying device with a multiplication factor
3 4
equal to the ratio of the areas of the two pistons. Dentist’s
5 6
chairs, car lifts and jacks, many elevators and hydraulic
brakes all use this principle.
X Example 13.2 Figure shows a hydraulic press p1 ≠ p2 ,p3 = p4 and p5 = p6
with the larger piston of diameter 35 cm at a height of Further, p 3 = p4
1.5 m relative to the smaller piston of diameter 10 cm. ∴ p0 + ρ1gh1 = p0 + ρ 2 gh2
The mass on the smaller piston is 20 kg. What is the or ρ1h1 = ρ 2h2 or h ∝
1
force exerted on the load by the larger piston? The ρ
density of oil in the press is 750 kg/m 3 . ■ Barometer It is a device used to measure
(Take g = 9.8 m/s 2 ) atmospheric pressure.
In principle, any liquid can be used to fill the
barometer, but mercury is the substance of choice,
because its great density makes possible an
1.5 m instrument of reasonable size.
20 kg
p1 = p2
Here, p1 = atmospheric pressure ( p0 )
and p2 = 0 + ρgh = ρgh
Fig. 13.8 Vacuum
(p = 0)
Sol. Pressure on the smaller piston
20 × 9.8 h
= −2 2
N /m2 1
π × (5 × 10 ) 2
Pressure on the larger piston
F
= N /m2
π × ( 17.5 × 10−2 )2
Here, ρ = density of mercury
The difference between the two pressures = hρg
∴ p0 = ρgh
where, h = 1.5 m
542 Objective Physics Vol. 1

Thus, the mercury barometer reads the atmospheric


(d) If a glass of different shape is used provided the
pressure ( p0 ) directly from the height of the mercury height, the bottom area and the volume are
column. unchanged, will the answers to parts (a) and (b)
For example, if the height of mercury in a barometer is change.
760 mm, then atmospheric pressure will be, (Take g =10 m/s 2 density of water =10 3 kg / m 3 and
p0 = ρgh = (13.6 × 103 ) (9.8) (0.760) atmospheric pressure = 1.01 × 10 5 N / m 2 )
= 1.01 × 105 N / m 2 Sol. (a) Force exerted by the water on the bottom
■ Manometer It is a device used to measure the F1 = ( p0 + ρgh)A1 …(i)
pressure of a gas inside a container. Here, p 0 = atmospheric pressure
=1.01 × 105 N/m2
p0
ρ = density of water = 103 kg/m3
h g = 10 m/s 2 ,
1 2
h = 10 cm = 0.1 m
and A1 = area of base = 10 cm2 = 10−3 m2
Hg Substituting in Eq. (i), we get
The U-shaped tube often contains mercury. F1 = (1.01 × 105 + 103 × 10 × 0.1) × 10−3
p1 = p2 or F1 = 102 N (downwards)
Here, p1 = pressure of the gas in the container ( p ), (b) Force exerted by atmosphere on water
and p2 = atmospheric pressure ( p0 ) + ρgh F2 = ( p0 )A2
∴ p = p0 + ρgh Here, A2 = area of top = 30 cm2 = 3 × 10–3 m2
This can also be written as ∴ F2 = (1.01 × 105 )(3 × 10−3 )
p − p0 =gauge pressure = ρgh = 303 N (downwards)
Here, ρ is the density of the liquid used in U-tube. Force exerted by bottom on the water
Thus, by measuring h we can find absolute (or gauge) F3 = − F1 or F3 = 102 N (upwards)
pressure in the vessel. Weight of water w = (volume)(density)(g )
■ Free body diagram of a liquid The free body = ( 10−3 )( 103 )( 10)
diagram of the liquid (showing the vertical forces only) = 10 N (downwards)
is shown in Fig.(b). For the equilibrium of liquid. Let F be the force exerted by side walls on the water
p 0A (upwards). Then, from equilibrium of water.
Net upward force = net downward force
A
or F + F3 = F2 + W
∴ F = F2 + W − F3 = 303 + 10 − 102
ρ h or F = 211 N (upwards)
w (c) If the air inside the jar is completely pumped out,
F1 = ( ρgh)A1 (as p0 = 0)
= (103 )(10)(0.1)(10–3 )
(a) (b) (p0 + ρgh)A
= 1 N (downwards)
In this case, F2 = 0
Net downward force = net upward force
and F3 = 1 N (upwards)
∴ p0 A + w = ( p0 + ρgh )A ∴ F = F2 + W − F3 = 0 + 10 − 1
or w = ρghA = 9 N (upwards)
(d) No, the answer will remain the same. Because the
answers depend upon p0 , ρ, g, h, A1 and A2 .
X Example 13.3 A glass full of water up to a height
of 10 cm has a bottom of area 10 cm 2 , top of area X Example 13.4 Two vessels have the same base
30 cm 2 and volume 1 L. area but different shapes. The first vessel takes twice
the volume of water that the second vessel requires to
(a) Find the force exerted by the water on the bottom.
fill up to a particular common height. Is the force
(b) Find the resultant force exerted by the sides of the
glass on the water. exerted by water on the base of the vessel the same in
(c) If the glass is covered by a jar and the air inside the the two cases? If so, why do the vessels filled with
jar is completely pumped out, what will be the water to that same height give different readings on a
answers to parts (a) and (b)? weighing scale?
Fluid Mechanics 543

Sol. Pressure (and therefore force) on the two equal base areas
are identical. But force is exerted by water on the sides of the Law of Floatation
vessels also, which has a non-zero vertical component, when Consider an object of volume V and density ρ S floating
the sides of the vessel are not perfectly normal to the base. This in a liquid of density ρ L . Let Vi be the volume of object
net vertical component of force by water on the sides of the
immersed in the liquid.
vessel is greater for the first vessel than the second. Hence, the
vessels weigh different when on the base is the same in the two
cases.
Vi
ρL
13.4 Archimedes’ Principle
If a heavy object is immersed in water, it seems to weigh
less than when it is in air. This is because the water exerts an Fig. 13.10
upward force called buoyant force. It is equal to the weight For equilibrium of object,
of the fluid displaced by the body.
Weight = Upthrust
A body wholly or partially submerged in a fluid is
∴ Vρ S g = Vi ρ L g
buoyed up by a force equal to the weight of the displaced
Vi ρ S
fluid. ∴ = …(i)
V ρL
This result is known as Archimedes’ principle.
Thus, the magnitude of buoyant force ( F ) is given by, This is the fraction of volume immersed in liquid.
F = Vi ρ L g Percentage of volume immersed in liquid
Here, Vi = immersed volume of solid, V ρ
= i × 100 = S × 100
ρ L = density of liqid, V ρL
and g = acceleration due of gravity Three possibilities may now arise:
(i) If ρ S < ρ L , then only fraction of body will be
Proof immersed in the liquid. This fraction will be given
Consider an arbitrarily shaped body of volume V placed by the above equation.
in a container filled with a fluid of (ii) If ρ S = ρ L , then whole of the rigid body will be
density ρ . The body is shown immersed in the liquid. Hence, the body remains
L ρL
completely immersed, but complete floating in the liquid wherever it is left.
immersion is not essential to the (iii) If ρ S > ρ L , then body will sink.
proof. To begin with, imagine the V
situation before the body was Apparent Weight of a Body
immersed. The region now occupied
by the body was filled with fluid,
Inside a Liquid
Fig. 13.9
whose weight wasVρ L g. Because the If a body is completely immersed in a liquid its effective
fluid as a whole was in hydrostatic equilibrium, the net weight gets decreased. The decrease in its weight is equal to
upwards force (due to difference in pressure at different the upthrust on the body. Hence,
depths) on the fluid in that region was equal to the weight of Wapp = Wactual − upthrust or Wapp = Vρ S g − Vρ L g
the fluid occuping that region. Here, V = total volume of the body
Now, consider what happens when the body has ρ S = density of body
displaced the fluid. The pressure at every point on the and ρ L = density of liquid
surface of the body is unchanged from the value at the same Thus, Wapp = Vg (ρ S − ρ L )
location when the body was not present. This is because the If the liquid in which body is immersed is water, then
pressure at any point depends only on the depth of that point Weight in air
below the fluid surface. = Relative density of body,
Decrease in weight
Hence, the net force exerted by the surrounding fluid on
This can be shown as under
the body is exactly the same as that exerted on the region
Weight in air Weight in air Vρ S g
before the body was present. But we know the latter to be = =
Vρ L g, the weight of the displaced fluid. Hence, this must Decrease in weight Upthrust Vρ w g
also be the buoyant force exerted on the body. Archimedes’ ρS
= RD
principle is thus, proved. ρw
544 Objective Physics Vol. 1

Sol. Let m be the mass of ice piece floating in water.


Buoyant Force in Accelerating Fluids In equilibrium, weight of ice piece = upthrust
Suppose a body is dipped inside a liquid of density ρ L or mg = Vi ρw g or Vi =
m
…(i)
placed in an elevator moving with an acceleration a. The ρw
buoyant force F in this case becomes, Here, Vi is the volume of ice piece immersed in water.
F = Vρ L g eff When the ice melts, let V be the volume of water formed by
Here, g eff = | g − a | m mass of ice. Then
m
V= …(ii)
For example, if the lift is moving upwards with an ρw
acceleration a, then value of g eff is g + a and if it is moving From Eqs. (i) and (ii), we see that Vi = V
downwards with acceleration a, then g eff is g − a. In a freely Hence, the level will not change.
falling lift g eff is zero (as a = g) and hence, net buoyant force
is zero. This is why, in a freely falling vessel filled with X Example 13.7 A piece of ice having a stone
some liquid, the air bubbles do not rise up (which otherwise frozen in it floats in a glass vessel filled with water.
move up due to buoyant force). The above result can be How will the level of water in the vessel change when
derived as follows. the ice melts?
Suppose a body is dipped inside a liquid of density ρ L in Sol. Let, m1 = mass of ice, m2 = mass of stone,
an elevator moving up with an acceleration a. As, we done ρS = density of stone and ρw = density of water.
earlier also, replace the body into the liquid by the same In equilibrium, when the piece of ice floats in water,
liquid of equal volume. The replaced liquid is at rest with weight of (ice + stone) = upthrust
respect to the elevator. Thus, this replaced liquid is also (m1 + m2 ) g = Vi ρw g
m m
moving up with an acceleration a together with the rest of ∴ Vi = 1 + 2 …(i)
ρw ρw
the liquid.
Here, Vi = volume of ice immersed.
The forces acting on the replaced liquid are:
When the ice melts, m1 mass of ice converts into water and
(i) the buoyant force F, and stone of mass m2 is completely submerged.
(ii) the weight mg of the substituted liquid. Volume of water formed by m1 mass of ice,
From Newton’s second law, m
V1 = 1
ρw
F − mg = ma or F = m ( g + a )
Here, m = Vρ L Volume of stone (which is also equal to the volume of water
displaced)
∴ F = Vρ L ( g + a ) = Vρ L g eff
V2 = m2 /ρ s
where, g eff = g + a
Since, ρS > ρw 0
Therefore, V1 + V2 < Vi
X Example 13.5 Density of ice is 900 kg/m 3 . A
or, the level of water will decrease.
piece of ice is floating in water (of density
1000 kg/m 3 ). Find the fraction of volume of the piece X Example 13.8 An ornament weighing 50 g in air
of ice outside the water. weights only 46 g is water. Assuming that some copper
is mixed with gold to prepare the ornament. Find the
Sol. Let V be the total volume and Vi be the volume of ice piece amount of copper in it. Specific gravity of gold is 20
immersed in water. For equilibrium of ice piece,
that of copper is 10.
weight = upthrust
∴ Vρi g = Vi ρw g Sol. Let m be the mass of the copper in ornament. Then, mass of
Here, ρi = density of ice = 900 kg/m3 gold in it is (50 − m).
ρw = density of water = 1000 kg/m 3 m  mass 
and Volume of copper V1 =  volume = 
10  density 
Substituting in above equation, we get
900 50 − m
Vi
= = 0.9 and volume of gold V2 =
V 1000 20
i.e. the fraction of volume outside the water, When immersed in water (ρw = 1 g/cm3 )
f = 1 − 0.9 = 0.1 Decrease in weight = upthrust
∴ (50 − 46) g = (V1 + V2 )ρw g
X Example 13.6 A piece of ice is floating in a glass m 50 − m
or 4 = + or 80 = 2 m + 50 − m
vessel filled with water. How will the level of water in 10 20
the vessel change when the ice melts? ∴ m = 30 g
Fluid Mechanics 545

X Example 13.9 The tension in a string holding a Area of RS, dA = (1) dh = dh


solid block below the surface of a liquid (of density Excess pressure p = ρgh
greater than that of solid) as shown in figure is T0 , ∴ Net force F = pdA = ρgh dh
when the system is at rest. What will be the tension in Perpendicular distance of this force from point B is,
the string, if the system has an upward acceleration r⊥ = H − h
a? ∴Torque of this force about B, d τ = Fr⊥
or d τ = ρgh(H − h )dh
a H H
Therefore, net torque τ = ∫ d τ = ∫ ρgh (H − h ) dh
0 0

ρgH 3
∴ τ=
6
This is the torque of hydrostatic forces per unit width of
Fig. 13.11 the wall.
/ Note
Sol. Let m be the mass of block. O
Initially for the equilibrium of block,
F = T0 + mg …(i)
Here, F is the upthrust on the block.
When the lift is accelerated upwards, geff becomes g + a
instead of g. Hence,
 g + a
F′ = F   …(ii) In the figure shown, torque of hydrostatic force about
 g  point O, the centre of a semicylindrical (or hemispherical)
From Newton’s second law, gate is zero as the hydrostatic force at all points passes
through point O.
F ′ − T − mg = ma …(iii)
F F′

a
13.5 Flow of Fluids
Steady Flow
T0 + mg T + mg If the velocity of fluid v3
Fig. 13.12 Fig. 13.13 particles at any point does not v1
Solving Eqs. (i), (ii) and (iii), we get vary with time, then flow is v2
 a said to be steady. Steady flow
T = T0  1 +  Fig. 13.14
 g is also called streamlined or
laminar flow. The velocity at different points may be
different. Hence, in the figure,
Extra Knowledge Points v 1 = constant, v 2 = constant, v 3 = constant,
■ Torque due to hydrostatic forces Consider a dam in but v1 ≠ v 2 ≠ v 3
which water of density ρ is filled upto a height H. We are
interested in finding the torque of hydrostatic forces on Principle of Continuity
wall AB about point B. For this, consider a small length
It states that, when an Q
RS equal to dh of the wall AB at a depth h below the
free surface of the dam. Further, let us assume a unit
incompressible and P
v2
non-viscous liquid flows in a v1
width perpendicular to paper inwards. Pressure on RS A2
from left side is p0 + ρgh and from right side is p0 . stream lined motion through
A1
A
a tube of non-uniform
cross-section, then the Fig. 13.15

product of the area of cross-section and the velocity of flow is


h same at every point in the tube.
p0 + rgh R p
0 Thus, A1 v1 = A2 v 2 or Av = constant,
S 1
H–h or v∝
A
B This is basically the law of conservation of mass in fluid
dynamics.
546 Objective Physics Vol. 1

Proof 13.6 Bernoulli’s Equation


Let us consider two cross-sections P and Q of area A1
Bernoulli’s equation relates the pressure, flow speed and
and A2 of a tube through which a fluid is flowing. Let v1 and
height for flow of an ideal (incompressible and non-viscous)
v 2 be the speeds at these two cross-sections. Then, being an
incompressible fluid, mass of fluid going through P in a time fluid. The pressure of a fluid depends on height as in the static
interval ∆t = mass of fluid passing through Q in the same situation, and it also depends on the speed of flow.
interval of time ∆t. The dependence of pressure on speed can be understood
∴ A1 v1ρ∆t = A2 v 2ρ ∆t from the continuity equation. When an incompressible fluid
or A1 v1 = A2 v 2 flows along a tube with varying cross-section, its speed must
change, and so, an element of fluid must have an
Therefore, the velocity of the liquid is smaller in the acceleration. If the tube is horizontal, then force that causes
wider parts of the tube and larger in the narrower parts. this acceleration has to be applied by the surrounding fluid.
A1 This means that the pressure must be different in regions of
A2 different cross-section.
v1 v2 When a horizontal flow tube narrows and a fluid
element speeds up, it must be moving towards a region of
Fig. 13.16
lower pressure in order to have a net forward force to
accelerate it. If the elevation also changes, this causes
or v 2 > v1 additional pressure difference.
as A2 < A1 A6
dV
/ The product Av is the volume flow rate , the rate at which p2
dt
b b′
volume crosses a section of the tube. Hence, A1 v2
dV
= volume flow rate = Av v1
h2
dt
p 1 ′
The mass flow rate is the mass flow per unit time h1 aa
through a cross-section. This is equal to density (ρ) times the
dV
volume flow rate .
dt Fig. 13.17

We can generalize the continuity equation for the case


To derive Bernoulli’s equation, we apply the work
in which the fluid is not incompressible. If ρ1 and ρ 2 are the
energy theorem to the fluid in a section of the fluid element.
densities at sections 1 and 2, then
Consider the element of fluid that at some initial time lies
ρ1 A1 v1 = ρ 2 A2 v 2 between two cross-sections a and b. The speeds at the lower
So, this is the continuity equation for a compressible and upper ends are v1 and v 2 .
fluid. In a small time interval the fluid that is initially at a
X Example 13.10 Water is flowing through a moves to a′ a distance aa ′ = ds1 = v1 dt and the fluid that is
initially at b moves to b′ a distance bb′ = ds2 = v 2 dt. The
horizontal tube of non-uniform cross-section. At a
cross-section areas at the two ends are A1 and A2 as shown in
place the radius of the tube is 1.0 cm and the velocity
the figure. The fluid is incompressible, hence by the
of water is 2 m/s. What will be the velocity of water,
continuity equation, the volume of fluid dV passing through
where the radius of the pipe is 2.0 cm ?
any cross-section during time dt is the same.
Sol. Using equation of continuity, That is, dV = A1 ds1 = A2 ds2
A1v1 = A2 v 2
A 
v 2 =  1  v1 Work Done on the Fluid Element
 A2 
2
Let us calculate the work done on this fluid element
 πr 2  r  during time interval dt. The pressure at the two ends are p1
or v 2 =  12  v1 =  1  v1
π
 2r  r2  and p2 , the force on the cross-section at a is p1 A1 and the
Substituting the values, we get force at b is p2 A2 .
2
 1.0 × 10−2  The net work done dW on the element by the
v 2 =   (2 )
−2 
 2.0 × 10  surrounding fluid during this displacement is,
or v 2 = 0.5 m/s dW = p1 A1 ds1 − p2 A2 ds2 = ( p1 − p2 ) dV …(i)
Fluid Mechanics 547

The second term is negative, because the force at b


opposes the displacement of the fluid. Extra Knowledge Points
This work dW is due to forces other, than the ■ Energy of a flowing fluid There are following three
conservative force of gravity, so it equals the change in total types of energies in a flowing fluid.
mechanical energy (kinetic plus potential). The mechanical (i) Pressure energy If p is the pressure on the area A
energy for the fluid between sections a and b does not of a fluid, and the liquid moves through a distance l
change. due to this pressure, then
Pressure energy of liquid = work done
Change in Potential Energy = force × displacement
At the beginning of dt the potential energy for the mass = pAl
between a and a′ is dmgh1 = ρdVgh1 . At the end of dt the The volume of the liquid is Al.
potential energy for the mass between b and b′ is ∴ Pressure energy per unit volume of liquid
pAl
dmgh2 = ρdVgh2 . The net change in potential energy dU = =p
during dt is, Al
(ii) Kinetic energy If a liquid of mass m and volume V
dU = ρ ( dV ) g ( h2 − h1 ) …(ii)
is flowing with velocity v, then the kinetic energy is
1
mv 2 .
Change in Kinetic Energy 2
At the beginning of dt the fluid between a and a′ has ∴ Kinetic energy per unit volume of liquid
1 1  m 1
volume A1 ds1 , ρA1 ds1 and kinetic energy ρ ( A1 ds1 ) v12 . At =   v 2 = ρv 2
2 2 V  2
the end of dt the fluid between b and b′ has kinetic energy Here, ρ is the density of liquid.
1 (iii) Potential energy If a liquid of mass m is at a height
ρ ( A2 ds2 ) v 22 . The net change in kinetic energy dK during h from the reference line (h = 0), then its potential
2
time dt is, energy is mgh.
1 ∴ Potential energy per unit volume of the liquid
dK = ρ ( dV )( v 22 − v12 ) …(iii)  m
=   gh = ρgh
2 V 
Combining Eqs. (i), (ii) and (iii) in the energy equation, Thus, the Bernoulli’s equation
dW = dK + dU 1
p + ρv 2 + ρgh = constant (J/m 3 )
We obtain, 2
1 can also be written as3
( p1 − p2 ) dV = ρdV ( v 22 − v12 ) + ρ ( dV ) g ( h2 − h1 ) Sum of total energy per unit volume (pressure +
2
kinetic +
1
or p1 − p2 = ρ ( v 22 − v12 ) + ρg ( h2 − h1 ) …(iv) potential) is constant for an ideal fluid.
2 ■ Pressure head, velocity head and gravitational head
This is Bernoulli’s equation. It states that the work done of a flowing liquid
on a unit volume of fluid by the surrounding fluid is equal to Dividing the Bernoulli’s equation by ρg, we have
the sum of the changes in kinetic and potential energies per p v2
+ + h = constant ( J/m 3 )
unit volume that occur during the flow. We can also express ρg 2g
Eq. (iv) in a more convenient form as, p v2
In this expression, is called the pressure head,
Bernoulli’s equation ρg 2g
1 1 the velocity head and h the gravitational head. The SI
p1 + ρgh1 + ρv12 = p2 + ρgh2 + ρv 22 unit of each of these three is metre. Therefore,
2 2
Bernoulli’s equation may also be stated as,
The subscripts 1 and 2 refer to any two points along the Sum of pressure head, velocity head and gravitational
flow tube, so we can also write head is constant for an ideal fluid.
1
p + ρgh + ρv 2 = constant
2 X Example 13.11 Calculate the rate of flow of
When the fluid is not moving (v1 = 0 = v 2 ). Bernoulli’s equation
glycerine of density 1.25 ×10 3 kg / m 3 through the
/

reduces to
p1 + ρgh 1 = p 2 + ρgh 2 conical section of a pipe, if the radii of its ends are
∴ p1 − p 2 = ρg (h 2 − h 1) 0.1 m and 004
. m and the pressure drop across its
This is the pressure relation we derived for a fluid at rest. length is 10N / m 2 .
548 Objective Physics Vol. 1

Sol. From continuity equation, (called throat) and a net force to the left slows as it leaves.
A1, p1 The pressure difference is also equal to ρgh, where h is the
A2 , p 2 difference in liquid level in the two tubes. Substituting in
Eq. (i), we get
v1 v2
2gh
v1 = 2
 A1 
  −1
Fig. 13.18  A2 
A1v1 = A2 v 2 Note Points
2 2
πr22 r  0.04  4
=  2  = 
v1 A / The discharge or volume flow rate can be obtained as,
or = 2 =  = …(i)
v2 A1 πr12  r1   01.  25 dV 2gh
= A1v1 = A1 2
From Bernoulli’s equation, dt  A1 
1 1 2( p1 − p2 )   − 1
p1 + ρv12 = p2 + ρv 22 or v 22 − v12 =  A2 
2 2 ρ
2 2 2 × 10 −2 2 2
/ The venturi effect can be used to give a qualitative
or v 2 − v1 = = 1.6 × 10 m / s …(ii) understanding of the lift of an air plane wing and the path of a
1.25 × 103
pitcher’s curve ball. An airplane wing is designed, so that air
Solving Eqs. (i) and (ii), we get
moves faster over the top of the wing that it does under the
v 2 ≈ 0128
. m/s wing, thus, making the air pressure less on top, than
∴Rate of volume flow through the tube underneath. This difference in pressure results in a net force
Q = A2 v 2 = ( πr22 )v 2 = π(0.04)2 (0.128) upward on the wing.
= 6.43 × 10−4 m2 / s 2
Speed of Efflux
13.7 Applications Based on Suppose, the surface of a liquid in a tank is at a height h
from the orifice O on its sides, through which the liquid
Bernoulli’s Equation issues out with velocity v. The speed of the liquid coming
out is called the speed of efflux. If the dimensions of the tank
Venturimeter be sufficiently large, then velocity of the liquid at its surface
Figure shows a venturimeter used to measure flow may be taken to be zero and since the pressure there as well
speed in a pipe. We apply Bernoulli’s equation to the wide as at the orifice O is the same viz atmospheric it plays no part
(point 1) and narrow (point 2) parts of the pipe, with h1 = h2 in the flow of the liquid, which thus, occurs purely in
1 1 consequence of the hydrostatic pressure of the liquid itself.
p1 + ρv12 = p2 + ρv 22 So that, considering a tube of flow, starting at the liquid
2 2 surface and ending at the orifice, as shown in figure.
Applying Bernoulli’s equation, we have
h
p1
p2 h
v
1 v1 v2 H v H
2 O
O
A2 H–h
A1
Fig. 13.19

A1 v1 R
From the continuity equation v 2 = Fig. 13.20
A2
Total energy per unit volume of the liquid at the surface
Substituting and rearranging, we get
= KE + PE + pressure energy
1  A2 
p1 − p2 = ρv12  12 − 1 …(i) = 0 + ρgh + p0 …(i)
2  A2  and total energy per unit volume at the orifice
Because, A1 is greater than A2 , v 2 is greater than v1 and = KE + PE + pressure energy
hence the pressure p2 is less than p1 . A net force to the right 1
accelerates the fluid as it enters the narrow part of the tube = ρv 2 + 0 + p0 …(ii)
2
Fluid Mechanics 549

Since, the total energy of the liquid must remain H


That is, R is maximum at h =
constant in steady flow, in accordance with Bernoulli’s 2
equation, we have H  H
1 and R max = 2 H −  = H
ρgh + p0 = ρv 2 + p0 2  2
2
or v = 2gh Time taken to Empty a Tank
Evangelista Torricelli showed that this velocity is the We are here interested in finding the time required to
same as the liquid will attain in falling freely through the empty a tank, if a hole is made at the bottom of the tank.
vertical height ( h) from the surface to the orifice. This is Consider a tank filled with a liquid of density ρ up to a
known as Torricelli’s theorem and may be stated as, “The height H. A small hole of area of cross-section a is made at
velocity of efflux of a liquid issuing out of an orifice is the the bottom of the tank. The area of cross-section of the tank
same as it would attain, if allowed to fall freely through the is A.
vertical height between the liquid surface and orifice.”
Let at some instant of time, the level of liquid in the tank
is y. Velocity of efflux at this instant of time would be,
Range (R)
v = 2gy
Let us find the range R on the ground.
Considering the vertical motion of the liquid, Now, at this instant volume of liquid coming out of the
 dV 
1 2( H − h) hole per second is  1  .
( H − h) = gt 2 or t =  dt 
2 g
Volume of liquid coming down in the tank per second is
Now, considering the horizontal motion,
 dV2 
R = vt  .
 dt 
 2( H − h) 
or R = ( 2gh )   dV1 dV2
 g  =
dt dt
or R = 2 h( H − h)  dy 
∴ av = A  − 
From the expression of R, following conclusions can be  dt 
drawn,  dy 
∴ a 2gy = A  − 
(i) R h = R H − h  dt 
as R h = 2 h( H − h) t A 0

and R H − h = 2 ( H − h) h
or ∫0 dt = − a ∫
2g H
y −1/ 2 dy …(i)

2A H
∴ t= [ y ]0
h a 2g
H H–h v
O A 2H
H–h ∴ t=
a g
h

X Example 13.12 A tank is filled with a liquid up


Fig. 13.21 to a height H. A small hole is made at the bottom of
This can be shown as in Fig. 13.21. this tank. Let t1 be the time taken to empty first half of
H the tank and t 2 the time taken to empty rest half of the
(ii) R is maximum at h = and R max = H . t
2 tank. Then, find 1 .
t2
R = 4( Hh − h 2 )
2

dR 2 Sol. Substituting the proper limits in Eq. (i), derived in the theory
For R to be maximum, =0 we have,
dh t1 A H / 2 −1/ 2

or H − 2h = 0 ∫ 0 dt = − a 2g ∫H y dy

H 2A
h=
H
or or t1 = [ y ]H / 2
2 a 2g
550 Objective Physics Vol. 1

or t1 =
2A 
H−
H Here, η is constant of proportionality and is called
a 2 g  2  coefficient of viscosity. Its value depends on the nature of
A H the fluid. The negative sign in the above equation shows that
or t1 = ( 2 − 1) …(ii)
a g the direction of viscous force F is opposite to the direction of
t2 A 0 −1/ 2
relative velocity of the layer.
Similarly, ∫ 0 dt = − a 2 g ∫ H / 2 y dy The SI unit of η is N-s/m 2 . It is also called decapoise or
or t2 =
A H
…(iii)
pascal second. Thus,
a g 1 decapoise = 1 N-s/m 2 = 1 Pa-s = 10 poise
From Eqs. (ii) and (iii), we get Dimensions of η are [ML−1T −1 ].
t1
= 2 −1
t2 Coefficient of viscosity of water at 10°C is
t1 η = 1.3 × 10 −3 N-s / m 2 . Experiments show that coefficient
or = 0.414
t2 of viscosity of a liquid decreases as its temperature rises.
/ From here we see that t1 < t 2 . This is because initially, the X Example 13.13 A plate of area 2 m 2 is made to
pressure is high and the liquid comes out with greater speed.
move horizontally with a speed of 2 m/s by applying a
horizontal tangential force over the free surface of a
13.8 Viscosity liquid. If the depth of the liquid is 1 m and the liquid in
Viscosity is internal friction in a fluid. Viscous forces contact with the bed is stationary. Coefficient of
opposes the motion of one portion of a fluid relative to the viscosity of liquid is 0.01 poise. Find the tangential
other. force needed to move the plate.
The simplest example of viscous flow is motion of a Sol. Velocity gradient = ∆v = 2 − 0 = 2 m /s
∆y 1− 0 m
fluid between two parallel plates.
v = 2 m/s
The bottom plate is stationary and the top plate moves
with constant velocity v. The fluid in contact with each F
surface has same velocity at that surface. The flow speeds of 1m

intermediate layers of fluid increase uniformly from bottom


to top, as shown by arrows. So, the fluid layers slide Fig. 13.23
smoothly over one another.
From, Newton’s law of viscous force,
F ∆v
v |F| = η A = (0.01 × 10−1 )(2)(2) = 4 × 10−3 N
∆y
So, to keep the plate moving, a force of 4 × 10−3 N must be
applied.

y
Flow of Liquid through a
Cylindrical Pipe
x
Figure shows the flow speed profile for laminar flow of
Fig. 13.22 a viscous fluid in a long cylindrical pipe. The speed is
greatest along the axis and zero at the pipe walls. The flow
According to Newton, the frictional force F (or viscous speed v at a distance r from the axis of a pipe of radius R is,
force) between two layers depends upon the following
factors:
R
(i) Force F is directly proportional to the area ( A ) of the
layers in contact, i.e.
F ∝A
(ii) Force F is directly proportional to the velocity Fig. 13.24
 dv  p − p2
gradient   between the layers. Combining these v= 1 (R 2 − r 2 )
 dy  4ηL
two, we have where, p1 and p2 are the pressure at the two ends of a
dv dv
F ∝A or F = − ηA pipe with length L. The flow is always in the direction of
dy dy decreasing pressure.
Fluid Mechanics 551

From the above equation, we can see that v-r graph is a


parabola.
X Example 13.14 A liquid is flowing through
v = 0 at r = R (along the walls) horizontal pipes as shown in fig4ure.
( p1 − p2 ) R 2 E F
and v = = v max at r = 0 (along the axis)
4ηL Q

 dV  A B C D
Volume Flow Rate Q or 
 dt  G H
Fig. 13.25
To find the total volume flow rate through the pipe, we
consider a ring with inner radius r, outer radius-r + dr and Length of different pipes has the following ratio
L L
cross-sectional area dA = 2πr dr. The volume flow rate L AB = LCD = EF = GH
through this element is v dA. The total volume flow rate is 2 2
found by integrating from r = 0 to r = R . The result is, Similarly, radii of different pipes has the ratio
dV R
Q= R AB = R EF = RCD = GH
dt 2
Pressure at A is 2 p0 and pressure at D is p0 . The volume
π  R 4   p1 − p2 
=    flow rate through the pipe AB is Q. Find,
8 η  L  (a) volume flow rates through EF and GH
The relation was first derived by Poiseulle and is called (b) pressure at E and F.
Poiseulle’s equation. Sol. The equivalent electrical circuit can be drawn as under,
X∝
L  as X = 8ηL 
 
R4  πR 4 
Extra Knowledge Points
16X
■ Poiseulle’s equation can also be written as,
p − p2 ∆ p 8X
Q
8X
Q= 1 = 17
 8ηL  X
  Q Q
 πR 4  2P0 P0
8ηL
Here, X = 16
Q X
πR 4 17
This equation can be compared with the current Fjg. 13.26
equation through a resistance, i.e.
∴ X AB : XCD : X EF : XGH
∆V
i = 1  1
R  
2 2 (1) (1)
Here, ∆V = potential difference = 4
: 4
: 4
: 4
 1  1  1  (1)
and R = electrical resistance      
2 2 2
For current flow through a resistance, potential
= 8 : 8 : 16 : 1
difference is a
requirement similarly for flow of liquid through a pipe (a) As, the current is distributed in the inverse ratio of the
resistance (in parallel). The Q will be distributed in the
pressure difference is must.
inverse ratio of X.
Problems of series and parallel combination of pipes Q
can be solved in the similar manner as is done in case Thus, volume flow rate through EF will be and that
17
of an electrical circuit. The only difference is, 16
from GH will be Q.
(i) Potential difference ( ∆V ) is replaced by the 17
pressure difference ( ∆p ).  (16 X )( X )  288
(b) Xnet = 8 X +   + 8 X = 17 X
 L  (16 X ) + ( X )
(ii) The electrical resistance R  = ρ  is replaced by
 A ∆p  as i = ∆V 
η ∴ Q=  
 8 L   R 
X = . Xnet
 πR 4  (2 p0 − p0 ) 17 p0
= =
(iii) The electrical current i is replaced by volume flow 288 288 X
X
dV 17
rate Q or . The following example will illustrate
dt Now, let p1 be the pressure at E, then
the above theory. 8 × 17 p0
2 p0 − p1 = 8QX =
288
552 Objective Physics Vol. 1

17 × 8 
∴ p1 =  2 −  p0 = 1.53 P0 Figure shows the variation of the velocity v of the sphere
 288 
with time
Similarly, if p2 be the pressure at F, then v
p2 − p0 = 8QX
8 × 17
∴ p2 = p0 + p0
288 vT

or p2 = 1.47 p0

13.9 Stoke’s Law and O Time


Fig. 13.28
Terminal Velocity / From the above expression we can see that terminal velocity
When an object moves through a fluid, it experiences a of a spherical body is directly proportional to the difference in
viscous force which acts in opposite direction of its velocity. the densities of the body and the fluid (ρ − σ). If the density of
fluid is greater than that of body (i.e. σ > ρ), then terminal
The mathematics of the viscous force for an irregular object
velocity is negative. This means that the body instead of
is difficult, we will consider here only the case of a small falling, moves upward. This is why air bubbles rise up in
sphere moving through a fluid. water.
The formula for the viscous force on a sphere was first
derived by the English physicist G. Stokes in 1843.
X Example 13.15 Two spherical raindrops of equal
According to him, a spherical object of radius r moving at size are falling vertically through air with a terminal
velocity v experiences a viscous force given by velocity of 1 m/s. What would be the terminal speed, if
these two drops were to coalesce to form a large
F = 6 πηrv (η = coefficient of viscosity)
spherical drop.
This law is called Stoke’s law.
Sol. vT ∝ r 2 …(i)

Terminal Velocity (v T ) Let r be the radius of small raindrops and R the radius of
large drop. Equating the volumes, we have
Consider a small sphere falling from rest through a large 4 4
πR3 = 2  πr3 
column of viscous fluid. The forces acting on the sphere are: 3 3 
(i) Weight w of the sphere acting vertically downwards. R
∴ R = (2 )1/ 3 . r or = (2 )1/ 3
(ii) Upthrust Ft acting vertically upwards. r
2
vT′  R 
(iii) Viscous force Fv acting vertically upwards, i.e. in a ∴ =   = (2)2 / 3
vT  r 
direction opposite to velocity of the sphere.
∴ vT′ = (2 )2 / 3 vT = (2)2 / 3 (1.0) m/s = 1.587 m/s
Initially, Fv = 0 (as v = 0)
and w > Ft
13.10 Surface Tension
and the sphere accelerates downwards. As the velocity
of the sphere increases, Fv increases. Eventually a A needle can made to float
stage in reached when on a water surface, if it is placed
there carefully. The forces that
w = Ft + Fv …(i) support the needle are not
After this net force on the sphere is zero and it moves buoyant forces but are due to
downwards with a constant velocity called terminal surface tension. The surface of a
velocity ( vT ). liquid behaves like a membrane Fig. 13.29
under tension. The molecules of
Substituting proper values in Eq. (i), we have the liquid exert attractive forces on each other. There is zero
4 3 4 net force on a molecule inside the volume of
πr ρg = πr 3σg + 6πηrvT …(ii) the liquid.
Ft + Fv
3 3
Here, ρ = density of sphere, σ = density of fluid But a surface molecule is drawn into the
v
volume. Thus, the liquid tends to minimize
and η = coefficient of viscosity of fluid
its surface area, just as a stretched
From Eq. (ii), we get
membrane does.
2 r 2 (ρ − σ ) g w
vT = Freely falling raindrops are spherical, Fig. 13.27
9 η because a sphere has a smaller surface area
Fluid Mechanics 553

for a given volume, than any other shape. Hence, the surface formed. The film exerts a surface tension force on the slider
tension can be defined as the property of a liquid at rest by and if the frame is kept in a horizontal position, the slider
virtue of which its free surface behaves like a stretched quickly slides towards the closing arm of the frame. If the
membrane under tension and tries to occupy as small area as frame is kept vertical, one can have some weight to keep it in
possible. equilibrium. This shows that the soap surface in contact the
Let an imaginary line AB be drawn in slider pulls it parallel to the surface.
any direction in a liquid surface. The F B / The surface tension of a particular liquid usually decreases
surface on either side of this line exerts a as temperature increases. To wash clothing thoroughly, water
F must be forced through the tiny spaces between the fibres.
pulling force on the surface on the other A This requires increasing the surface area of the water, which
side. This force is at right angles to the is difficult to do because of surface tension. Hence, hot water
line AB. The magnitude of this force per Fig. 13.30 and soapy water is better for washing.
unit length of AB is taken as a measure of
the surface tension of the liquid. Thus, if F be the total force Surface Energy
acting on either side of the line AB of length L, then the When the surface area of a liquid is dx
surface tension is given by, increased, the molecules from the
F interior rise to the surface. This requires
T= F
L work against force of attraction of the 2Tl
Hence, the surface tension of a liquid is defined as the molecules just below the surface. This
force per unit length in the plane of the liquid surface, acting at work is stored in the form of potential
Fig. 13.33
right angles on either side of an imaginary line drawn on that energy. Thus, the molecules in the
surface. surface have some additional energy due to their position.
This additional energy per unit area of the surface is called
Few Examples of Surface Tension ‘surface energy’. The surface energy is related to the surface
tension as discussed below :
Example Take a ring of wire and dip it in a soap
Let a liquid film be formed on a wire frame and a
solution. When the ring is taken out, a soap film is formed.
straight wire of length l can slide on this wire frame as
Place a loop of thread gently on the soap film. Now, prick a
shown in figure. The film has two surfaces and both the
hole inside the loop.
surfaces are in contact with the sliding wire and hence, exert
The thread is radially pulled by the film surface outside forces of surface tension on it. If T be the surface tension of
and it takes a circular shape. the solution, then each surface will pull the wire parallel to
itself with a force Tl. Thus, net force on the wire due to both
the surfaces is 2Tl.One has to apply an external force F equal
and opposite to it to keep the wire in equilibrium. Thus,
F = 2Tl
Fig. 13.31 Now, suppose the wire is moved through a small
distance dx, the work done by the force is
Reason Before the pricking, there were surfaces both dW = F dx = (2Tl ) dx
inside and outside the thread loop. Surfaces on both sides But (2l ) ( dx ) is the total increase in area of both the
pull it equally and the net force is zero. Once the surface surfaces of the film. Let it be dA. Then
inside was punctured, the outside surface pulled the thread dW
to take the circular shape, so that area outside the loop dW = T dA or T =
dA
becomes minimum (because for given perimeter area of
circle is maximum). Thus, the surface tension T can also be defined as the
work done in increasing the surface area by unity.
Example A piece of wire Further, since there is no change in kinetic energy, the
is bent into a U-shape and a work done by the external force is stored as the potential
second piece of wire slides energy of the new surface.
on the arms of the U. When dU
the apparatus is dipped into
w ∴ T= (as dW = dU )
dA
a soap solution and
Fig. 13.32 Thus, the surface tension of a liquid is equal to the
removed, a liquid film is
surface energy per unit surface area.
554 Objective Physics Vol. 1

X Example 13.16 How much work will be done in Each half of the soap bubble is in equilibrium. The
increasing the soap bubble from 2 cm to 5 cm. Surface lower half is shown in figure. The forces at the flat circular
. × 10 −2 N/m.
tension of soap solution is 30 surface where this half joins the upper half are
Surface tension force
Force due to pressure
Sol. Soap bubble has two surfaces. = (2T )(2πR) difference
Hence, W = T ∆A = ∆p πR 2
Here, ∆A = 2 [4 π{(2.5 × 10−2 )2 − (1.0 × 10−2 )2 }]
= 1.32 × 10−2 m2
∴ W = (3.0 × 10−2 )(1.32 × 10−2 ) J
= 3.96 × 10–4 J
Fig. 13.35
X Example 13.17 Calculate the energy released (i) The upward force of surface tension. The total surface
when 1000 small water drops each of same radius tension force for each surface (inner and outer) is
10 −7 m coalesce to form one large drop. The surface T (2πR ), for a total of (2T )(2πR )
tension of water is 70. × 10 −2 N/m. (ii) Downward force due to pressure difference.
Sol. Let r be the radius of smaller drops and R of bigger one. The magnitude of this force is ( ∆p)( πR 2 ). In
Equating the initial and final volumes, we have, equilibrium, these two forces have equal magnitude.
4 4
πR 3 = ( 1000)  πr 3 
3 3  p0
or R = 10 r = ( 10 )( 10−7 ) m or R = 10−6 m
p
Further, the water drops have only one free surface.
Therefore, ∆A = 4 πR 2 − (1000)(4 πr 2 ) R

= 4 π [(10−6 )2 − (103 )(10−7 )2 ]


= − 36 π(10−12 ) m2 Fig. 13.36

Here, negative sign implies that surface area is decreasing. ∴ (2T )(2πR ) = ( ∆p)( πR 2 )
Hence, energy released in the process.
4T
U = T|∆A| = ( 7 × 10−2 )(36 π × 10−12 ) J or ∆p =
−12 R
= 7.9 × 10 J
/ Suppose, the pressure inside the air bubble is p, then
4T
p − p0 =
Excess Pressure inside a Bubble or R
Liquid Drop
Excess Pressure inside a Liquid Drop
Surface tension causes a pressure difference between
the inside and outside of a soap bubble or a liquid drop. A liquid drop has only one surface film. Hence, the
surface tension force is T (2πR ), half that for a soap bubble.
Excess Pressure inside Soap Bubble Thus, in equilibrium,
2T
A soap bubble consists of two spherical surface films T (2πR ) = ∆p( πR 2 ) or ∆p =
with a thin layer of liquid between them. Because of surface R
tension, the film tend to contract in an attempt to minimize Note Points
their surface area. But as the bubble contracts, it compresses
/ If we have an air bubble inside a liquid, a single surface is
the inside air, eventually increasing the interior pressure to a
formed. There is air on the concave
level that prevents further contraction. side and liquid on the convex side. The
pressure in the concave side (that is in
the air) is greater than the pressure in p2
p1
the convex side (that is in the liquid) by
O 2T
R an amount .
R
Fig. 13.37
∴ p 2 − p1 = 2T / R
The above expression has been written by assuming p1 to be
Fig. 13.34 constant from all sides of the bubble. For small size bubbles this
can be assumed.
We can derive an expression for the excess pressure / From the above discussion, we can make a general
inside a bubble in terms of its radius R and the surface statement. The pressure on the concave side of a spherical
tension T of the liquid. liquid surface is greater than the convex side by 2T / R.
Fluid Mechanics 555

X Example 13.18 What should be the pressure (i) p, attraction due to the molecule of the solid
inside a small air bubble of 01 . mm radius situated surface near it.
just below the water surface? Surface tension of (ii) Q, attraction due to liquid molecules near this part,
water = 7.2 × 10 −2 N/m and atmospheric and
(iii) the weight w of the part considered.
pressure = 1.013 × 10 5 N/m 2 .
We have considered very small part, so weight of that
Sol. Surface tension of water T = 7.2 × 10−2 N/m part can be ignored for better understanding. As we have
Radius of air bubble R = 0.1 mm = 10–4 m seen in the last figures, to make the resultant ( R ) of p and Q
The excess pressure inside the air bubble is given by, perpendicular to the liquid surface, the surface becomes
2T
p2 − p1 = curved (convex or concave).
R / The angle of contact between water and clean glass is zero
∴ Pressure inside the air bubble, p2 = p1 + 2T / R and that between mercury and clean glass is 137°.
Substituting the values, we have
(2 × 7.2 × 10−2 )
p2 = ( 1.013 × 105 ) + Capillarity
10−4
5 2 Surface tension causes elevation or depression of the
= 1.027 × 10 N/m
liquid in a narrow tube. This effect is called capillarity.
Shape of Liquid Surface
The surface of a liquid when Glass
meets a solid, such as the wall of a
container, it usually curves up or
down near the solid surface. The P
angle θ at which it meets the (a) (b)
R Q
surface is called the contact angle.
Water Fig. 13.40
The curved surface of the liquid is
called meniscus. The shape of the Fig. 13.38 When a glass capillary tube (A tube of very small
meniscus (convex or concave) is determined by the relative diameter is called a capillary tube) open at both ends is dipped
strengths of what are called the cohesive and adhesive vertically in water, the water in the tube will rise above the
forces. The force between the molecules of the same level of water in the vessel as shown in Fig. (a). In case of
material is known as cohesive force and the force between mercury, the liquid is depressed in the tube below the level of
the molecules of different kinds of material is called mercury in the vessel as shown in Fig. (b).
adhesive force. When the contact angle is less than 90°, the liquid rises
When the adhesive force ( p) Glass in the tube. For a nonwetting liquid angle of contact is
between solid and liquid greater than 90° and the surface is depressed, pulled down
molecules is more than the by the surface tension forces.
cohesive force (Q ) between P
liquid-liquid molecules (as with
Q
Explanation
water and glass), shape of the R
When a capillary tube is dipped in water, the water
Water
meniscus is concave and the angle meniscus inside the tube is concave. The pressure just below
of contact θ is less than 90°. In this Fig. 13.39 2T
case the liquid wets or adheres to the solid surface. The the meniscus is less than the pressure just above it by ,
R
resultant ( R ) of p and Q passes through the solid. where, T is the surface tension of water and R is the radius of
On the other hand, when p < Q (as with glass and curvature of the meniscus. The pressure on the surface of
mercury), shape of the meniscus is convex and the angle of water is p0 , the atmospheric pressure. The pressure just
contact θ > 90°. The resultant ( R ) of p and Q in this case below the plane surface of water outside the tube is also p0 ,
passes through the liquid. 2T
but that just below the meniscus inside the tube is p0 − .
Let us now see why the liquid surface bends near the R
contact with a solid. A liquid in equilibrium cannot sustain We know that pressure at all points in the same level of
tangential stress. The resultant force on any small part of the water must be the same. Therefore, to make up the
surface layer must be perpendicular to the surface at that 2T
deficiency of pressure below the meniscus water begins
point. Basically three forces are acting on a small part of the R
liquid surface near its contact with solid. These forces are: to flow from outside into the tube. The rising of water in the
556 Objective Physics Vol. 1

capillary stops at a certain height h. In this position, the The result has following notable features:
pressure of water column of height h becomes equal to
2T
, (i) If the contact angle θ is greater than 90°, then term
R cos θ is negative and hence, h is negative. The
i.e. expression, then gives the depression of the liquid
2T 2T in the tube.
hρ g = or h= (ii) Suppose a capillary tube is held vertically in a
R Rrg
liquid which has a concave meniscus, then
If r is the radius of the capillary tube and θ the angle of capillary rise is given by,
contact, then 2T cos θ 2T  r 
r h= =  as R = 
R= rρg Rρg  cos θ 
cos θ
or hR = 2T /ρg
2T cos θ
∴ h= When the length of the tube is greater than h, then liquid
rρg rises in the tube, so as to satisfy the above relation. But, if the
length of the tube is insufficient (i.e. less than h) say h′ , the
Alternative Proof for the Formula of liquid does not emerge in the form of a fountain from the
Capillary Rise upper end (because it will violate the law of conservation of
energy) but the angle made by the liquid surface and hence,
As we have already seen, when the contact angle is less
the R changes in such a way that the force 2πrT cos θ equals
than 90°, the total surface tension force just balances the
the weight of the liquid raised. Thus
extra weight of the liquid in the tube.
2T cos θ ′
T cosθ T cosθ T cos θ (2πr) 2πrT cos θ ′ = πr 2ρgh′, h′ =
rρg
T θ θ T 2T 2T
Tsinθ Tsinθ or h′ = or h′ R ′ =
h R ′ ρg ρg
θ θ
From Eqs. (ii) and (iii)
2T
hR = h′ R ′ =
ρg
Fig. 13.41 X Example 13.19 A capillary tube whose inside
radius is 0.5 mm is dipped in water having surface
The water, meniscus in the tube is along a circle of tension 7.0 × 10 −2 N/m. To what height is the water
circumference 2πr which is in contact with the glass. Due to raised above the normal water level? Angle of contact
the surface tension of water, a force equal to T per unit of water with glass is 0°. Density of water is
length acts at all points of the circle. If the angle of contact is 10 3 kg/m 3 and g = 9.8 m/s 2
θ, then this force is directed inward at an angle θ from the
wall of the tube. In accordance with Newton’s third law, the Sol. h = 2T cos θ
rρg
tube exerts an equal and opposite force T per unit length on
Substituting the proper values, we have
the circumference of the water meniscus.
(2)(7.0 × 10−2 ) cos 0°
h= = 2.86 × 10–2 m = 2.86 cm
This force which is directed outward, can be resolved (0.5 × 10−3 )(103 )(9.8)
into two components. T cos θ per unit length acting
vertically upward and T sin θ per unit length acting X Example 13.20 A glass tube of radius 0.4 mm is
horizontally outward. Considering the entire circumference dipped vertically in water. Find upto what height the
2πr, for each horizontal component T sin θ there is an equal water will rise in the capillary? If the tube in inclined
and opposite component and the two neutralise each other. at an angle of 60° with the vertical, how much length
The vertical components being in the same direction are of the capillary is occupied by water. Surface tension
added up to give a total upward force (2πr )(T cos θ ). It is this of water = 7.0 ×10 −2 N/m, density of water =10 3 kg / m 3 .
force which supports the weight of the water column so
raised. Thus, Sol. For glass-water, angle of contact θ = 0° .
2T cos θ (2)(7.0 × 10−2 ) cos 0°
(T cos θ )(2πr ) = Weight of the liquid column Now, h= =
rρg (0.4 × 10−3 )(103 )(9.8)
= ( π r 2 ρ g h)
= 3.57 × 10−2 m = 3.57 cm
2T cos θ h 3.57
∴ h= l= = = 7.14 cm
rρg cos 60° 1/2
Fluid Mechanics 557

Extra Knowledge Points


■ Stream line and turbulent flow The flow of a fluid is said to be steady, if at any given point, the velocity of each particle
passing through that point remains constant. Although velocity at different points may be different. That is, at some
other point the particle may have a different velocity, but every other particle which passes the second point behaves
exactly as the previous particle that has just passed that point.
Each particle follows a smooth path and the paths of the particles do not cross each other. The path followed by a fluid
particle under a steady flow is called a streamline.
v3

v1 C
B
A
v2

In the streamline ABC shown is figure, velocity of fluid particles at A is v1, at B is v 2 and at C is v 3 .
Steady flow is achieved when the speed of flow is not very high. Beyond a limiting value, called critical speed flow is no
longer streamlined it becomes turbulent.
■ Reynolds number When speed of fluid flow is high, flow of fluid becomes turbulent. Turbulent flow is less likely for viscous
fluids flowing at low speeds.
Reynolds number (Re ) is a dimensionless number, whose value gives us an idea whether the flow would be laminar
(streamlined)or turbulent.
ρvd
Re =
η
Here, ρ = density of fluid
v = speed of fluid
d = dimensions of fluid and
η = viscosity of fluid
For Re < 1000, flow is streamline or laminar.
For Re → 1000 to 2000, flow is unsteady and for Re > 2000, flow is turbulent. Turbulence dissipates kinetic energy usually in
the form of heat. Racing cars and aeroplanes are designed to minimize turbulence. But sometimes turbulence is desirable.
Turbulence promotes mixing. For example, blades of a kitchen mixer induce turbulent flow and provide thick milk
shakes.
Chapter Summary with Formulae
(i) Upthrust F = Vi ρl ge (xii) (a) v = 2gh h
2( H − h )
(ii) When a solid whose density is less than the density of (b) t = H v
liquid floats in it then, some fraction of solid remains g H–h
immersed in the liquid. In this case, (c) R = vt = 2 h( H − h )
(a) Weight = Upthrust R
(b) Fraction of volume immersed (d) R max = H at h = H /2
ρ
f = s (xiii) Barometer
ρl
(iii) When a solid whose density is more than the density of Vacuum
liquid is completely immersed in it, then upthrust acts
on its 100% volume and apparent weight is less than its h
2 1
actual weight.
wapp = w − F
Here, F = Upthrust on 100% volume of solid. Mercury
(iv) Relative Density (Specific Gravity) of any
Substance
Density of that substance p1 = p2
RD =
Density of water p0
∴ p0 = hρg or h=
=
Weight in air ρg
Change in weight in water h is approximately 76 cm of mercury.
(v) Effect of Temperature on Density (xiv) Viscosity
ρ  dv 
ρ′ = (a) F = − ηA
dv
= velocity gradient 
1 + γ∆θ 
dy  dy 
(vi) Effect of Pressure on Density
(b) F = 6 πηrv = For spherical ball
ρ
ρ′ =
∆ p 2 r2 (ρ − σ ) g
1− (c) vT = or vT ∝ r2
B 9 η
(vii) 1 Pa = 1 Nm −2 , 1 Bar = 105 Pa, 1 atm = 1.013 × 105 Pa Here, ρ = density of ball and
Gauge pressure σ = density of viscous medium in which ball is moving.
= absolute pressure − atmospheric pressure. (xv) Surface Tension
(viii) Pressure at depth h below the surface of water, F ∆W
p (a) T = = or ∆ W = T × ∆A
l ∆A
2T 4T
p0 (b) ∆p = for single surface and for double surface
R R
(c) Capillary rise or fall,
h 
2T 2T cos θ r 
p = p0 + ρgh h= =  as R = 
Rρg rρg  cos θ
Change in pressure per unit depth,
dp (xvi) The unit of the pressure is the pascal (Pa). It is the same
=ρg
dh as Nm −2 . Other common units of pressure are
(ix) Volume Flow Rate 1 atm = 1.01 × 105 Pa
dV 1 bar = 105 Pa
Q = Av or = Av
dt 1 torr = 133 Pa = 0.133 kPa
(x) Continuity Equation 1 mm of Hg = 1 torr = 133 Pa
dV1 dV2 (xvii) The onset of turbulence in a fluid is determined by a
Q1 = Q2 or = dimensionless parameter is called the Reynold number
dt dt
1 given by
or A1v1 = A2 v2 or Av = constant or v ∝
A R e = ρvd / η
(xi) Bernoulli’s Equation
p + ρgh + 1 / 2 ρv2 = constant where, d is a typical geometrical length associated with
1 1 the fluid flow and the other symbols have their usual
or p1 + ρgh1 + ρv12 = p2 + ρgh2 + ρv22
2 2 meaning.
Additional Examples
Example 1. Why are the sleepers used below the Example 6. The three vessels shown in figure are
rails? filled to the same height with water. The three vessels
Sol. Sleepers are placed below the rails to increase area ( A) have the same base area. In which vessel, will the
and to consequently decrease the pressure (p ) due to the force on the base be minimum?
weight (F ) of train on the rails as p = F / A.

Example 2. What will be the weight of a


weightless bag when it is filled with water and
weighed in water ?
Sol. Let V be the volume of the bag. Sol. Since, the height of water column is same in the three
Weight of the bag filled with water of density ρ is vessels, the pressure at the base will be same. As, the three
W = Vρg vessels have the same base area, the force will also be the
Force of buoyancy acting on the (weightless) when in same in the three cases.
water i.e.
F = weight of water displaced by the bag = Vρg Example 7. To empty an oil tin, two holes are
Apparent weight of the bag in water made. Why ?
= w − F = Vρg − Vρg = 0
Sol. If one hole is made in the tin, then oil will not come out.
Example 3. Explain why does a boat rise as it It is because, as the oil comes out, the pressure inside the
tin becomes less than the atmospheric pressure. Due to
enters the sea from a river?
this, the oil cannot come out. On the other hand, when two
Sol. For the boat to float, W = F holes are made in the tin, air keeps on entering the tin
If V r and V s are the volumes of the portions of the through the other.
boat inside river water and sea water and ρ r , ρ s are
the densities of river and sea water respectively, Example 8. Why is it easier to swim in sea water
F = V r ρr g = V sρs g than in the river water?
V s ρr Sol. The density of sea water (salt dissolved in water) is
or =
V r ρs greater than that of the river water. Therefore, upthrust on
As, ρ r < ρ s therefore, V s < V r . the swimmer is greater in sea water than in river water.
Hence, it is easier to swim in sea water than in river water.
Obviously, the boat rises as it enters sea.

Example 4. A bucket of water rests on a scale. Example 9. A balloon filled with helium does not
Does the scale reading change when a lead block is rise in air indefinitely but halts after a certain height
suspended from a thread and lowered into water (neglect winds). Explain, why?
where it is held submerged without touching the Sol. Initially, the balloon filled with helium rises in air as the
bottom or the sides of the bucket? weight of the air displaced by the balloon is greater than
the weight of the helium gas and the balloon. We know that
Sol. Water exerts an upthrust force on the block from
the density of air decreases with height. Therefore, the
Newton's third law, there must be an equal but opposite balloon halts after attaining a height at which the density of
force pushing down. The scale reading will increase by an air decreases to a value, such that the weight of the air
amount equal to the buoyant force. displaced just equal to the weight of gas and the balloon.
Example 5. Two streamlines cannot cross each Example 10. As soon as parachute of a falling
other. Explain, why? soldier opens, his acceleration decreases and soon
Sol. The tangent at any point on a streamline gives the becomes zero. Why?
direction of flow of liquid molecules at that point. In case,
Sol. Viscous force keeps on increasing with increase in
the two streamlines cross each other, it would mean that
the liquid molecule can have two velocities along the two velocity. As a result, its acceleration decreases. However, it
different directions, which is against the definition of soon attains terminal velocity. On attaining the terminal
streamline motion. Hence, two streamlines cannot cross velocity, the soldier falls with a constant velocity i.e. the
each other. acceleration of the soldier becomes zero.
560 Objective Physics Vol. 1

Example 11. When air is blown in between the the pressure inside the bigger bubble will be less than that
inside the smaller bubble. As air will flow from a region of
two balls suspended from a string such that they do greater pressure to the region of lower pressure, the bigger
not touch each other, then balls come nearer to each
other, instead of moving away. Why? bubble will grow at the expense of the smaller bubble.
Sol. When air is blown between the two suspended balls, Example 16. For the arrangement shown in the
then kinetic energy of the air between the balls increases. figure, what is the density of oil?
From Bernoulli's principle, pressure decreases. Due to
pressure difference on the two sides of the two balls, the Sol. p 0 + ρ w gl = p 0 + ρ oil (l + d ) g
balls come nearer to each other. C
d = 12.3 mm
Example 12. If a small ping pong ball is placed
in a vertical jet of air or water, it will rise to a Oil

certain height above the nozzle and stay at that level. l = 135 mm

Explain. B
Sol. Due to high velocity of the jet of water, pressure on the Water
side of the ping pong ball, that faces the water jet,
decreases. On the other side of the ball, the pressure is still ρw l
equal to the atmospheric pressure. Due to the difference in ⇒ ρ oil =
l +d
pressure on the two sides, the ball get pushed towards the
1000 × (135)
jet of water. = = 916 kg/m 3
Ping-pong (135 + 12.3)
ball
Example 17. A boat floating in a water tank is
Jet of
water carrying a number of stones. If the stones were
unloaded into water, what will happen to the water
level?
Sol. Let weight of boat = W and weight of stone = w.
The ball does not fall down as it is constantly pressed
Assuming density of water = 1 g/cc
against the water jet due to difference of pressure on the
two sides of the ball. Volume of water displaced initially = (w + W )
 w
Example 13. End of a glass tube becomes round Later, volume displaced = W + 
 ρ
on heating. Explain.
Sol. When an end of a glass tube is heated, the glass at that (Q ρ = density of stones)
end melts. In molten (liquid) state, in an attempt to acquire ⇒ Water level comes down.
minimum surface area due to the property of surface
tension, the end of the glass tube becomes spherical i.e. Example 18. Figure shows how the stream of
round. water emerging from a faucet necks down as it falls.
The area changes from A0 to A through a fall of h. At
Example 14. Why does a small piece of camphor what rate does the water flow from the tap?
dance on the water surface?
Sol. When camphor dissolves in water, then surface tension
of water decreases. Due to irregular shape of the camphor A0
piece, it may dissolve more at one end than at the other
end. Thus, surface tension of water will decrease by h
A
unequal amounts at the different ends of the camphor
piece. It produces a resultant force on the camphor piece
and it starts moving erom one place to another place. Sol. A 0 v 0 = Av
v 2 = v 02 + 2gh
Example 15. A large bubble is formed at one end
of a capillary tube and a small one at the other end. 2ghA 2
v0 =
Which one will grow if they are connected? A 02 − A 2
Sol. Since, the excess of pressure inside a bubble is
 4T  2gh A 2
inversely proportional to radius of the bubble  ∆p =  , or R = A0 v0 = A0
 R A 02 − A 2
Fluid Mechanics 561

Example 19. A bent tube is lowered into the Example 21. An iron casting containing a
stream as shown in the figure. The velocity of the number of cavities weigths 6000 N in air and 4000 N
stream relative to the tube is equal to v. The closed in water. What is the volume of the cavities in the
upper end of the tube located at the height h 0 . To casting? Density of iron is 7.87 g/cm3 .
what height h will the water jet spurt ? (Take g = 9.8 m/s 2 and density of water = 10 3 kg/m3 )
2 Sol. Let v be the volume of cavities and V be the volume of
solid iron. Then,
h
mass  6000/9.8 
V= =  = 0.078 m
3

h0 density  7.87 × 10 3 
Further, decrease in weight = upthrust
1 ∴ (6000 − 4000 ) = (V + v) ρ w g
or 2000 = (0.078 + v) × 10 3 × 9.8
or 0.078 + v ≈ 0.2
Sol. Let tube’s entrance be a depth y below the surface. Take ∴ v = 0.12 m 3
point 1 at entry, 2 at the maximum height of the fountain.
Applying Bernoulli’s theorem, Example 22. Water rises in a capillary tube to a
1 1 height of 2.0 cm. In another capillary tube whose
p1 + ρgh1 + ρv12 = p 2 + ρgh 2 + ρv 22
2 2 radius is one third of it, how much the water will
Taking, h1 = 0 , h 2 = ( y + h 0 + h ), v1 = v, v 2 = 0, rise?
2T cos θ
Substituting these values, we get Sol. h =
1 rρ g
p 0 + ρgy + ρv 2 = p 0 + ρg ( y + h 0 + h ) 2T cos θ
2 ∴ hr = = constant
1 2 rg
⇒ ρv = ρg (h 0 + h ) hr
2 h1r1 = h 2 r2 or h 2 = 1 1
 v2  r2
⇒ h= − h0 
 2g  Substituting the values, we get
 r2 1
h2 = (2.0)(3)  = 
Example 20. A solid floats in a liquid of  r1 3
different materials. Carry out an analysis to see = 6.0 cm
whether the level of liquid in the container will rise
or fall when the solid melts. Example 23. Mercury has an angle of contact of
Sol. Let M = Mass of the floating solid 120° with glass. A narrow tube of radius 1.0 mm
made of this glass is dipped in a trough containing
ρ1 = density of liquid formed by melting of the solid
mercury. By what amount does the mercury dip down
ρ 2 = density of the liquid in which the solid is floating
in the tube relative to the liquid surface outside.
The mass of liquid displaced by the solid is M. Hence, the
M Surface tension of mercury at the temperature of
volume of liquid displaced is . the experiment is 0.5 N/m and density of mercury is
ρ2
13.6 × 10 3 kg/m3 . (Take g = 9.8 m/s 2 )
When the solid melts, then volume occupied by it is M/ρ1 .
Hence, the level of liquid in container will rise or fall 2 T cos θ
Sol. h =
according as rρ g
M M
> or < Substituting the values, we get
ρ1 ρ2 2 × 0.5 × cos 120 °
h = −3 = − 3.75 × 10 −3 m
i.e. ρ1 < or > ρ 2 10 × 13.6 × 10 3 × 9.8
There will be no change in the level, ifρ1 = ρ 2 . In case of ice or h = − 3.75 mm.
floating in water ρ1 = ρ 2 and hence, the level of water / Here, negative sign implies that mercury suffers capillary
remains unchanged when ice melts. depression.
562 Objective Physics Vol. 1

2 r 4 (ρ − σ)2 g
Example 24. Two narrow bores of radius 3.0 mm = ×
and 6.0 mm are joined together to form a U-shaped 81 η2
tube open at both ends. If the U-tube contains water, 2 (3 × 10 −4 ) 4 (10 4 − 10 3 ) 2 × 9.8
= ×
what is the difference in its levels in the two limbs of 81 (9.8 × 10 −6 ) 2
the tube. Surface tension of water is 7.3 × 10 −2 N/m. = 1.65 × 10 3 m
(Take the angle of contact to be zero and density of
water to be 10 3 kg/m3 , g = 9.8 m/ s 2 ) Example 27. A solid ball of density half that of
water falls freely under gravity from a height of
2 T cos θ 2 T cos θ
Sol. hρg = ∆ p = – 19.6 m and then enters water. Up to what depth will
r1 r2 the ball go. How much time will it take to come
2 T cos θ  r2 − r1  again to the water surface. Neglect air resistance
or h=  
ρg  r1r2  and viscosity effects in water. (g = 9.8 m/ s 2 )
Substituting the values, we have Sol. v = 2gh = 2 × 9.8 × 19.6 = 19.6 m/s
2 × 7.3 × 10 −2 × cos 0 °  6.0 − 3.0  1
h=   × −3 Let ρ be the density of ball and 2ρ the density of water. Net
10 × 9.8
3  6.0 × 3.0  10 retardation inside the water,
= 2.48 × 10 –3 m = 2.48 mm upthrust – weight
a =
mass
Example 25. With what terminal velocity will an g
air bubble 0.8 mm in diameter rise in a liquid of h = 19.6 m
viscosity 0.15 N-s/m2 and specific gravity
0.9. Density of air is 1.293 kg/m3 .
Sol. The terminal velocity of the bubble is given by, v
2 r 2 ( ρ − σ )g
vT = a=g
9 η
Here, r = 0.4 × 10 −3 m, σ = 0.9 × 10 3 kg/m 3 V (2ρ)g − V (ρ)(g)
= (V = volume of ball)
ρ = 1.293 kg/m , η = 0.15, Ns/m
3 2
V (ρ)g
and g = 9.8 m/s 2 = g = 9.8 m/s 2
Substituting the values we have, Hence, the ball will go up to the same depth 19.6 m below
2 (0.4 × 10 −3 ) 2 (1.293 – 0.9 × 10 3 ) × 9.8 the water surface.
vT = ×
9 0.15 Further, time taken by the ball to come back to water surface is
= − 0.0021 m/s or v T = − 0.21 cm/s  v  19.6 
t = 2   = 2 =4s
/ Here, negative sign implies that the bubble will rise up.  a  9.8 
Example 26. A spherical ball of radius Example 28. A block of mass 1 kg and density
3.0 × 10 −4 m and density 10 4 kg/m 3 falls freely under 0.8 g/cm 3 is held stationary with the help of a string
gravity through a distance h before entering a tank of as shown in figure. The tank is accelerating
water. If after entering the water the velocity of the vertically upwards with an acceleration a = 1.0 m /s 2 .
ball does not change, then find the value of h.
Viscosity of water is 9.8 × 10 −6 N-s/m2 . Find,

Sol. Before entering the water the velocity of ball is 2gh.


If after entering the water this velocity does not change,
then this value should be equal to the terminal velocity. a
Therefore,
2 r 2 ( ρ − σ )g
2 gh =
9 η
2
 2 r 2 ( ρ − σ )g  a
(a) the tension in the string,
 
η (b) if the string is now cut find the acceleration of block.
h=  
9
∴ w+T
(Take g =10 m/ s 2 and density of water =10 3 kg/m 3 )
2g
Fluid Mechanics 563

Sol. (a) Free body diagram of the block is shown in above r − r 


figure. p = p 2 − p1 = 4T  1 2  …(i)
 r1r2 
In the figure,
F = Upthrust force This excess pressure acts from concave to convex side, the
= Vρ ω (g + a) a interface will be concave towards smaller bubble and
 Mass of block  convex towards larger bubble. Let R be the radius of
=  ρ w (g + a) interface, then
 Density of block  w+T
4T
p= …(ii)
 1 
=  (1000)(10 + 1) = 13.75 N R
 800  From Eqs. (i) and (ii), we get
w = mg = 10 N rr
R= 1 2 =
(0.004)(0.002)
= 0.004 m
Equation of motion of the block is r1 − r2 (0.004 – 0.002)
F − T − w = ma
∴ 13.75 − T − 10 = 1 × 1 Example 30. Under isothermal condition two
∴ T = 2.75 N soap bubbles of radii r1 and r2 coalesce to form a
(b) When the string is cut T = 0 single bubble of radius r. The external pressure is p 0 .
F−w Find the surface tension of the soap in terms of the
∴ a=
m given parameters.
13.75 − 10
= = 3.75 m/s 2 Sol. As mass of the air is conserved,
1
p
p2
Example 29. Two separate air bubbles (radii p1
0.004 m and 0 m) formed of the same liquid (surface + =
tension 0.07 N/m) come together to form a double r1 r
r2
bubble. Find the radius and the sense of curvature of
the internal film surface common to both the bubbles.
4T ∴ n1 + n 2 = n (as pV = nRT )
Sol. p1 = p 0 + p1V1 p 2 V 2 pV
r1 ∴ + =
RT1 RT 2 RT
4T
p2 = p0 + As temperature is constant,
r2 T1 = T 2 = T
∴ p1V1 + p 2 V 2 = pV
r2
 4 3 
4S 4S  4 3 
∴ p 0 +  πr1  +  p 0 +   πr2 
p p2  3
r1    r2   3 
 4S  4 
p1 =  p 0 +   πr 3 
r1
 r   3 
Solving this, we get
p (r 3 − r13 − r23 )
r2 < r1 S= 0 2
∴ p 2 > p1 4(r1 + r22 − r 2 )
i.e. pressure inside the smaller bubble will be more. The / To avoid confusion with the temperature surface tension here
excess pressure is represented by S.
NCERT Selected Questions
Q 1. Explain why? Q 3. Explain why?
(a) The blood pressure in humans is greater at the (a) To keep a piece of paper horizontal, you should
feet than at the brain. blow over, not under it.
(b) Atmospheric pressure at a height of about 6 km (b) When we try to close a water tap with our
decreases to nearly half its value at the sea level fingers, fast jets of water gush through the
though the height of the atmosphere is more openings between our fingers.
than 100 km. (c) A fluid flowing out of a small hole in a vessel
(c) Hydrostatic pressure is a scalar quantity even results in a backward thrust on the vessel.
though pressure is force divided by area. Sol. (a) If we blow over a piece of paper, velocity of air above the
Sol. (a) The height of the blood column in the human body is paper becomes more than that below it. In accordance
more at the feet as compared to that at the brain. 1
with Bernoulli’s theorem (p + ρv2 = constant), its
Consequently, the blood pressure in humans is greater at 2
the feet than at the brain. pressure above the paper decreases. Due to greater value
(b) Density of air is maximum near the surface of the earth of pressure below the piece of paper it remains
and decreases rapidly with height . horizontal and does not fall.
(c) Pressure is transmitted equally in all directions inside (b) This can be explained from the equation of continuity
the liquid. Thus, there is no fixed direction for the A1 v1 = A2 v2. As, we try to close a water tap with our
pressure due to liquid. Hence, hydrostatic pressure is a fingers, the area of cross-section of the outlet of water jet
scalar quantity. is reduced considerably as the openings between our
fingers provide constriction (i.e. regions of smaller area).
Q 2. Fill in the blanks using the word(s) from the list Thus velocity of water increases greatly and fast jets of
appended with each statement. water come through the openings between our fingers.
(a) Surface tension of liquids generally ............... (c) When a fluid is flowing out of a small hole in a vessel, it
with temperature. (increases/decreases) acquires a large velocity and hence possesses large
momentum. Since, no external force is acting on the
(b) Viscosity of gases ............... with temperature,
system, a backward velocity must be attained by the
whereas viscosity of liquids ............... with
vessel (according to the of conservation of momentum).
temperature. (increases/decreases)
(c) For solids with elastic modulus of rigidity, the Q 4. A 50 kg girl wearing high heel shoes balances on a
shearing force is proportional to ..............., while single heel. The heel is circular with a diameter
for fluids it is proportional to ............... ,(shear 1. 0 cm. What is the pressure exerted on the
strain/rate of shear strain) horizontal floor?
(d) For a fluid in a steady flow, the increase in flow F 50 × 9.8
speed at a constriction follows from ............., Sol. p = or p = = 6.24 × 106 Pa
A 3.142 × 25 × 10−6
while the decrease of pressure there follows
p = 6.24 × 106 Pa.
from ............... ,(conservation of mass
/Bernoulli's principle) Q 5. Torricelli’s barometer used mercury. Pascal
(e) For the model of a plane in a wind tunnel, duplicated it using French wine of density
turbulence occurs at a............... speed than the 984 kg m −3 . Determine the height of the wine
critical speed for turbulence for an actual plane.
column for normal atmospheric pressure.
(greater/smaller)
Sol. (a) decreases Sol. p = normal atmospheric pressure = 1.013 × 105 Pa
(b) increases,decreases Let h be the height of the French wine column.
(c) shear strain, rate of shear strain Then h ρ w g = p
(d) conservation of mass, Bernoulli's principle p 1.013 × 105
or h= = = 10.5 m
(e) greater ρw g 984 × 9.8
Fluid Mechanics 565

Q 6. A vertical off-shore structure is built to withstand a Sol. Pressure at A = Pressure at B


maximum stress of 10 9 Pa. Is the structure suitable or p0 + hw ρ w g = p0 + hs ρ s g + hm ρ m g
for putting up on top of an oil well in the ocean?
Take the depth of the ocean to be roughly 3 km and p p0
ignore ocean currents.
25 cm 27 cm
Sol. If p be the pressure exerted by this water column at the given
depth. hm
A B
Then, p = hρg = 3 × 10 × 10 × 9.8
3 3

= 29.4 × 106 Pa ≈ 3 × 107 Pa Mercury

Since, 3 × 107 Pa < 109 Pa or hw ρ w = hs ρ s + hm ρ m


Thus, we conclude that the structure is suitable as the stress h ρ − hsρ s
∴ hm = w w
applied by it is much lesser than the maximum stress it can ρm
withstand.
Substituting the values, we get
Q 7. A hydraulic automobile lift is designed to lift cars or hm = 0.221 cm
with a maximum mass of 3000 kg. The area of
cross-section of the piston carrying the load is
Q 10. Can Bernoulli's equation be used to describe the
flow of water through a rapid river? Explain.
425 cm 2 . What maximum pressure would the
smaller piston have to bear? Sol. No, Bernoulli's equation cannot be used to describe the flow
of water through a rapid river. This is due to the reason that
Sol. The maximum force which the bigger piston can bear, Bernoulli's equation applies to streamlined flow and in a
F = 3000 kgf = 3000 × 9.8 N rapid river, the flow of water is not streamlined.
Area of piston, A = 425 cm2 = 425 × 10−4 m 2
Q 11. Does it matter, if one uses gauge instead of absolute
If p = maximum pressure on the bigger piston. pressures in applying Bernoulli's equation? Explain.
F 3000 × 9.8
Then, p= = Sol. No, it does not matter, if one uses gauge pressure instead of
A 425 × 10−4
absolute pressure in applying Bernoulli's equation unless the
= 6.92 × 105 Pa atmospheric pressure at the two points, where Bernoulli's
Since, the liquid transmits pressure equally in all directions, equation is applied are significantly different.
hence, the maximum pressure the smaller piston can bear is
6.92 × 105 Pa. Q 12. In a test experiment on a model aeroplane in a wind
tunnel, the flow speeds on the upper and lower
Q 8. A U-tube contains water and methylated spirit surfaces of the wing are 70 ms −1 and 63 ms −1
separated by mercury. The mercury columns in the
respectively. What is the lift on the wing if its area is
two arms are in level with 10.0 cm of water in one
arm and 12.5 cm of spirit in the other. What is the . m 2 ? Take the density of air to be 1.3 kgm −3 .
25
specific gravity of spirit? Sol. The level of the upper and lower surfaces of the wings from
Sol. h1 ρ 1 g = h2 ρ 2 g the ground may be taken same, i.e.
h1 = h2
h1ρ 1 10 × 1
or ρ2 = = From Bernoulli's theorem,
h2 12.5
1 1
4 p1 + ρgh1 + ρv12 = p2 + ρgh2 ρv22
= = 0.8 g cm −3 2 2
5
1
Now, specific gravity of spirit or p2 − p1 = ρ(v12 − v22 ) …(i)
2
density of spirit
= This pressure difference provides the lift to the aeroplane
density of water
i.e. lift on the wing.
0.8 gcm −3
= = 0.800 Thus, if F be the lift on the wing, then
1gcm −3 F = ( p2 − p1 ) × A
1
Q 9. In the previous problem, if 15.0 cm of water and = ρ(v12 − v22 ) × A
2
spirit each are further poured into the respective
1
arms of the tube, what is the difference in the levels = × 1. 3 × (702 − 632 ) × 2. 5
of mercury in the two arms? (Specific gravity of 2
mercury =13.6). = 1. 5 × 103 N
566 Objective Physics Vol. 1

Q 13. Fig. (a) and Fig. (b) refer to the steady flow of a Then, F = T × 2l = T × 0.6 N
(non-viscous) liquid. Which of the two figures is . × 10−2 N
w = 15
incorrect? Why? In equilibrium position, the force F on the slider due to
surface tension must be balanced by the weight (w)
supported by the slider.
i.e. F = w = mg
or . × 10−2
T × 0.6 = 15
(a) (b) . × 10−2
15
∴ T = = 2.5 × 10−2 Nm −1
0.6
Sol. Fig. (a) is incorrect. This is because at a section of smaller
cross-section, the flow speed is larger due to equation of Q 16. What is the pressure inside the drop of mercury of
continuity. Hence, the according to Bernoulli’s theorem, radius 3.00 mm at room temperature? Surface
pressure in the narrow part must be low. But in this figure tension of mercury at that temperature (20° C) is
the pressure at narrow part is shown to be larger. . × 10 −1 Nm −1 . The atmospheric pressure is
465
Q 14. The cylindrical tube of a spray pump has a . × 10 5 Pa. Also give the excess pressure inside
101
cross-section of 80 . cm 2 one end of which has the drop.
40 fine holes each of diameter 1. 0 mm. If the liquid 2T 2 × 4.65 × 10−1
flow inside the tube is 1.5 m / min. What is the speed Sol. pin − pout = = = 310 Pa
r 3 × 10−3
of ejection of the liquid through the holes?
∴ pin = pout + 310 = 310 + 1.01 × 105 Pa
Sol. Area of corss-section of tube,
= 1.01 × 105 + 0.00310 × 105
a1 = 8.0 cm2 = 8 × 10−4 m2
=1.01310 × 105 Pa
Number of fine holes = 40
d
∴ Radius of each hole = = 0. 5 × 10−3 m Q 17. What is the excess pressure inside a bubble of soap
2 solution of radius 500
. mm, given that the surface
∴Area of cross-section of each hole, tension of soap solution at the temperature (20° C) is
a = πr2 = π (0.5 × 10−3 )2 m2 2.50 × 10 −2 Nm −1 ? If an air bubble of the same
∴Area of cross-section of 40 holes, dimension were formed at a depth of 400 . cm inside
a2 = 40 a = 40 π (0.5 × 10−3 )2 m2 a container containing the soap solution (of relative
density 1.20), what would be the pressure inside the
Velocity of flow of liquid in the tube,
bubble? (1 atmospheric pressure is 1.01 ×10 5 Pa.)
1.5 −1
v1 = 1.5 m /min = ms
60 Sol. Excess of pressure inside the soap bubble is given by,
Let v2 be the velocity of ejection of the fluid. Then, 4T
pi − p =
according to equation of continuity, r
a1 v1 = a2 v2 4 × 2.5 × 10−2
=
a v 5.0 × 10−3
or v2 = 1 2
a2 = 20 Pa
8 × 10−4 15
. Excess pressure inside the air bubble in the soap solution is
= −3 2
×
40π (0.5 × 10 ) 60 2T
pi − p =
= 0.64 ms−1 r
2 × 2.5 × 10−2
=
Q 15. A U-shaped wire is dipped in a soap solution and 5.0 × 10−3
removed. The thin soap film formed between the = 10 Pa
wire and a light slider supports a weight of Now, pressure outside the air bubble at a depth of 40 cm is
1.5 ×10 2 N (which includes the small weight of the p = Atmospheric pressure + pressure due to 40 cm
slider). The length of the slider is 30 cm. What is the = 101
. × 105 + 0.4 × 12
. × 103 × 9.8
surface tension of the film?
= 105704
. × 105 Pa (∴ p = hρg )
Sol. A soap film has two free surfaces, so total length of the film ≈ 106
. × 10 Pa
5
to be supported, l = 2 × 30 cm = 0.60 m
∴Pressure inside the air bubble,
Let T = surface tension of the film
2T
If F = total force on the slider due to surface tension. p1 = p + = (1.06 × 105 + 10) Pa
r
Fluid Mechanics 567

Q 18. A tank with a square base of area 1.0 m 2 is divided (b) Do the dissipative forces become more
by a vertical partition in the middle. The bottom of important as the fluid velocity increases?
Discuss qualitatively.
the partition has a small hinged door of area 20 cm 2 .
Sol. (a) If dissipating forces are present, then a part of work done
The tank is filled with water in one compartment,
will be used in overcoming these forces during the flow
and an acid (of relative density 1.7) in the other,
of fluid. So, there shall be greater drop of pressure as the
both to a height of 4.0 m. Compute the force
fluid move along the tube.
necessary to keep the door close.
(b) Yes, the dissipative forces become more important as
the fluid velocity increases.
According to the equation
4m F = − ηA dv / dy
Clearly as v increases, velocity gradient increases and
hence, viscous drag i.e. dissipative force also increases.
Door Q 21. In Millikan’s oil drop experiment, what is the
Sol. (a) If p1 be the pressure exerted by water at the door terminal speed of an uncharged drop of radius of
provided at the bottom, then 2.0 × 10 −5 m and density 1.2 × 10 3 kg m −3 ? Take
p1 = h1 ρ 1 g = 4.0 × 103 × 9.8 = 39.2 × 103 Pa the viscosity of air at the temperature of the
N-s
(b) If P2 be the pressure exerted by acid at the door provided experiment to be 1.8 × 10 −5 2 . How much is the
at the bottom, then m
p2 = h2 ρ 2 g = 4.0 × 1.7 × 9.8 × 103 = 66.64 × 103 Pa viscous force on the drop at that speed? Neglect
buoyancy of the drop due to air?
∴Difference of pressure
2 2 (ρ − ρ 0 )g
∆p = p2 − p1 = 66.64 × 103 − 39.2 × 103 Sol. Terminal velocity, vT = r
9 η
= 27.44 × 103 Pa
Since, the buoyancy of the drop due to air is to be neglected,
Area of door, A = 20 cm2 = 20 × 10−4 m 2 ∴ ρ 0 = 0 for air
∴Force on the door = difference in pressure × area 2 r2ρg 2 (2 × 10−5 )2 × (12
. × 103 )
∴ vr = = × × 9. 8
= ∆p × A = 27.44 × 103 × 20 × 10−4 = 54.88 N ≈ 55 N 9η 9 1. 8 × 10−5
To keep the door closed, a force equal to 55 N should be = 5. 8 × 10−2 ms−1 = 5. 8 cms−1
applied horizontally on the door from compartment
The viscous force on the drop according to Stoke’s law is
containing water.
given by
Q 19. Two vessels have the same base area but different F = 6πηrvT
shapes. The first vessel takes twice the volume of = 6 × 3.142 × (1. 8 × 10−5 ) × (2 × 10−5 ) × (5. 8 × 10−2 )
water that the second vessel requires to fill upto a
particular common height. Is the force exerted by = 3. 93 × 10−10 N
the water on the base of the vessel the same in the
two cases? If so, why do the vessels filled with Q 22. Mercury has an angle of contact equal to 140° with
water to that same height give different readings on soda lime glass. A narrow tube of radius 1 . 00 mm
a weighing scale? made of this glass is dipped in a trough containing
mercury. By what amount does the mercury dip
Sol. Since, the pressure depends upon the height of water column down in the tube relative to the liquid surface
and the height of the water column in the two vessels of outside? Surface tension of mercury at the
different shapes is the same, hence there will be same temperature of the experiment is 0.465 Nm −1 .
pressure due to water on the base of each vessel. As the base
area of each vessel is same, hence there will be equal force Density of mercury = 13 . 6 × 10 3 kgm −3 .
acting on the two base areas due to water pressure. But the 2T cosθ
two vessels have different weights of liquids. That is why, Sol. Using the formula, h = , we get
rρg
the two vessels filled with water to same vertical height
show different readings on a weighing machine. 2 × 0.465 × cos 140° 2 × 0.465 × (−0.7660)
h= =
10−3 × 13.6 × 103 × 9.8 10−3 × 13.6 × 103 × 9.8
Q 20. In deriving Bernoulli's equation, we equated the
work done on the fluid in the tube to its change in = − 5. 34 × 10−3 m = − 5. 34 mm
the potential and kinetic energy. Here, negative sign shown that the mercury level is
(a) How does the pressure change as the fluid moves depressed in the tube relative to the mercury surface outside.
along the tube, if dissipative forces are present? i.e. depression = 5.34 mm
Objective Problems
[ Level 1 ]
Upthrust and Concept of Floating
1. A body floats in a liquid contained in a beaker. The whole 7. A tank contains water on top of mercury as shown in
system shown in figure is falling under gravity, the figure. A cubical block of side 10 cm is in equilibrium
upthrust on the body due to the liquid is inside the tank. The depth of the block inside mercury is
(RD of the material of block = 8.56, RD of mercury = 13.6)

Water

Cubical block

(a) zero Mercury


(b) equal to weight of the body in air
(c) equal to weight of liquid displaced
(a) 6 cm (b) 5 cm (c) 7 cm (d) 8 cm
(d) equal to the weight of the immersed part of the body
8. An object of weight w and density ρ is submerged in
2. A raft of wood of mass 120 kg floats in water. The weight
liquid of density σ, its apparent weight will be
that can be put on the raft to make it just sink should be
(a) (ρ − σ ) (b) (ρ − σ ) / w
( d raft = 600 kg / m 3 )  σ  ρ
(c) w 1 −  (d) w 1 − 
(a) 80 kg (b) 50 kg  ρ  σ
(c) 60 kg (d) 30 kg
9. A block of wood is floating on the surface of water in a
3. A balloon has volume of 1000 m 3 . It is filled with
hydrogen ( ρ = 0.09g / L). If the density of air is 1.29 g/L, beaker. The beaker is covered with a bell jar and the air is
then it can lift a total weight of evacuated. What will happen to the block?
(a) 600 kg (b) 1200 kg (a) Sink a little (b) Rise a little
(c) Remain unchanged (d) Sink completely
(c) 300 kg (d) 1800 kg
10. A solid of density D is floating in a liquid of density d. If v
4. A boat having a length 3 m and breadth 2 m is floating on
a lake. The boat sinks by 1 cm, when a man gets on it. The is the volume of solid submerged in the liquid andV is the
v
mass of the man is total volume of the solid, then is equal to
(a) 60 kg (b) 72 kg V
(c) 52 kg (d) 65 kg d D
(a) (b)
D d
5. A small block of wood of relative density 0.5 is D D+d
(c) (d)
submerged in water. When the block is released, it states (D + d ) D
moving upwards, then acceleration of the block is
( g = 10 ms −2 ) 11. A metallic sphere floats in immiscible mixture of water
(density 103 kg / m 3 ) and a liquid (density
(a) 5 ms−2
 2  1
(b) 10 ms−2 8 × 103 kg / m 3 ) such that its   part is in water and  
 3  3
(c) 7.5 ms−2
part in the liquid. The density of the metal is
(d) 15 ms−2
5000 10000
(a) kg/ m 3 (b) kg/ m 3
6. The reading of spring balance when a block is suspended 3 3
from it in air, is 6 N. This reading is changed to 40 N, (c) 5000 kg/ m 3 (d) 2000 kg/ m 3
when the block is immersed in water. The specific gravity
of the block is 12. A body measures 5 N in air and 2 N when put in water.
(a) 3 (b) 2 The buoyant force is
3 (a) 7 N (b) 9 N
(c) 6 (d)
2 (c) 3 N (d) None of these
Fluid Mechanics 569

13. The relative density of ice is 0.9 and that of sea water is 19. A cubical block of steel of each side equal to 1 is floating
1.125. What fraction of the whole volume of an iceberg on mercury in vessel. The densities of steel and mercury
appears above the surface of the sea? are ρ s and ρ m . The height of the block above the mercury
(a) 1/ 5 (b) 2/ 5 level is given by
(c) 3/ 5 (d) 4 / 5  ρs   ρ 
(a) l 1 +  (b) l 1 − s 
14. The reading of spring balance when a block is suspended  ρm   ρm 
from it in air is 60 N. This reading is changed to 40 N,  ρm   ρ 
(c) l 1 +  (d) l 1 − m 
when the block is immersed in water. The specific gravity  ρs   ρs 
of the block is
(a) 3 (b) 2 (c) 6 (d) 3/ 2 20. A balloon has volume of 1000 m 3 . It is filled with
hydrogen (ρ = 0.009 g / L ). If the density of air is 129
. g / L,
15. A cubical block is floating in a liquid with half of its then it can lift a total weight of
volume immersed in the liquid. When the whole system (a) 600 kg (b) 1200 kg
g (c) 300 kg (d) 1800 kg
accelerates upwards with acceleration of , then fraction
3
of volume immersed in the liquid will be Fluid Pressure and Pressure Force
21. Assuming that the atmosphere has the same density
g
3 anywhere as at sea level (ρ = 13 . kg / m 3 ) and g to be
constant ( g = 10 m / s ). What should be the approximate
2

height of atmosphere ( p 0 = 101


. × 105 N/ m 2 )
(a) 6 km (b) 8 km
(a) 1/ 2 (b) 3/ 8 (c) 2/ 3 (d) 3/ 4 (c) 12 km (d) 18 km
16. A raft of mass M = 600 kg floats in calm water with 7 cm 22. In a U-tube experiment, a column AB of water is
submerged. When a man stands on the raft, 8.4 cm are balanced by a column CD of paraffin. The relative density
submerged, the man's mass is of paraffin is
(a) 30 kg (b) 60 kg
(c) 90 kg (d) 120 kg
17. An ice-cube of density 900 kg / m3 is floating in water of C A
h1 h2
density 1000 kg / m 3 . The percentage of volume of ice
cube outside the water is D B
(a) 20% (b) 80% (c) 10% (d) 90%

18. A beaker containing water is kept on a spring scale. The


mass of water and beaker is 5 kg. A block of mass 2 kg h2 h1
(a) (b)
and specific gravity 10 is suspended by means of thread h1 h2
from a spring balance as shown in the figure. The h − h1 h2
(c) 2 (d)
readings of scales S 1 and S 2 are respectively h1 h1 + h2
(Take g = 10 ms −2 ).
23. A U-tube of uniform cross-section shown in figure is
partially filled with liquid I. Another liquid II which does
S2 not mix with I is poured into one side. The liquid levels of
the two sides is found the same, while the level of liquid I
has risen by 2 cm. If the specific gravity of liquid I is 1.1,
then specific gravity of liquid II must be

I
II

S1

(a) 52 N and 20 N (b) 50 N and 18 N (a) 1. 2 (b) 1.1


(c) 52 N and 18 N (d) 52 N and 22 N (c) 1. 3 (d) 1. 0
570 Objective Physics Vol. 1

24. A closed rectangular tank is completely filled with water 28. A cylindrical tank contains water up to a height H. If the
and is accelerated horizontally with an acceleration tank is accelerated upwards with acceleration a, then
towards right. Pressure is pressure at the point A is p1 . If the tank is accelerated
A D downwards with acceleration a the pressure at A is
p 2 .Then
a

B C
H
(i) maximum at, and (ii) minimum at
(a) (i ) B (ii ) D
(b) (i ) C (ii ) D A
(c) (i ) B (ii ) C (a) p1 < p2 (b) p1 = p1
(d) (i ) B (ii ) A (c) p1 > p2 (d) data insufficient
25. A uniformly tapering vessel is filled with a liquid of 29. In the given figure shown.
density 900 kgm −3 . The force that acts on the base of the
vessel due to the liquid (excluding atmospheric force) is p p0
(g = 10 m / s 2 )
ρ 3h h
Area = 10–3 m2

0.4
(a) p1 > p0 (b) p > p0
(c) p = p0 (d) p = 0
2 × 10–3 m2
30. A piston of cross-sectional area 100 cm 2 is used in a
(a) 3. 6 N (b) 7. 2 N
hydraulic pressure to exert a force of 107 dyne on the
(c) 9. 0 N (d) 12. 6 N water. The cross-sectional area of the other piston which
support a truck of mass 2000 kg is
26. For the arrangement shown in the figure, the force at the (a) 9. 8 × 102 cm 2
bottom of the vessel is
(b) 9. 8 × 103 cm 2
Vacuum
1 cm2 (c) 1. 96 × 103 cm 2
99 cm
(d) 1. 96 × 104 cm 2

31. The liquid inside the container has density ρ. Choose the
ρw = 10 kg/m3 1cm
correct options.
100 cm2 A B

(a) 200 N (b) 100 N (c) 20 N (d) 2 N


a=g
27. An object of uniform density is allowed to float in water L
kept in a beaker. The object has triangular cross-section
as shown in the figure. If the water pressure measured at
C L D
the three point A , B and C below the object are p A , p B
and pC respectively. Then (a) pA − pC = ρgL
(b) pC − pB = 2ρgL
(c) pC − pD = ρgL
(d) pA − pD = 0

Continuity Equation, Bernoulli’s


Equation and Torricelli’s Equation
A B C
32. Bernoulli's theorem is a consequence of
(a) pA > pB > pC (a) conservation of mass
(b) pA > pB < pC (b) conservation of energy
(c) pA = pB = pC (c) conservation of linear momentum
(d) pA = pB < pC (d) conservation of angular momentum
Fluid Mechanics 571

33. The velocity of efflux of a liquid through an orifice in the 41. Water is flowing through two horizontal pipes of
bottom of a tank does not depend upon different diameters which are connected together. The
(a) density of liquid diameters of the two pipes are 3 cm and 6 cm
(b) height of the liquid column above orifice respectively. If the speed of water in the narrower tube is
(c) acceleration due to gravity 4 m / s. Then, the speed of water in the wider tube is
(d) None of the above (a) 16 m / s (b) 1 m / s (c) 4 m / s (d) 2 m / s
34. A hole is made at the bottom of the tank filled with water 42. Water flows along a horizontal pipe of non-uniform
(density = 1000 kg / m 3 ). If the total pressure at the cross-section. The pressure is 1 cm of Hg, where the
bottom of the tank is three atmospheres velocity is 35 cm / s. At a point where the velocity is
(1 atmosphere = 105 N / m 2 ), then the velocity of efflux is 65 cm / s, the pressure will be
(a) 400 m / s (b) 200 m / s (a) 0.89 of Hg (b) 0.62 cm of Hg
(c) 0.5 cm of Hg (d) 1 cm of Hg
(c) 600 m / s (d) 500 m / s
43. In the given figure, the velocity v 3 will be
35. Water from a tap emerges vertically down with an initial v2 = 2 m/s
speed of 1.0 ms −1 . The cross-sectional area of tap is A1 = 0.2 m2
A2 = 0.2 m2
10−4 m 2 . Assume that the pressure is constant throughout
the stream of water and that the flow is steady. The v1 = 4 m/s
cross-sectional area of the stream 015
. m below the tap is v3
(a) 5.0 × 10−4 m 2 A3 = 0.4 m2
. × 10−5 m 2
(b) 10 (a) 2 m/s (b) 4 m/s
(c) 5.0 × 10−5 m 2 (c) 1 m/s (d) 3 m/s
(d) 2.0 × 10−5 m 2 44. The pressure of water in a pipe when tap is closed is
5.5 × 105 N / m 2 . When tap gets open, pressure reduces
36. The lift of an aeroplane is based on
(a) Torricelli's theorem (b) Bernoulli's principle to 5 × 105 N / m 2 . The velocity with which water comes
(c) law of gravitation (d) continuity equation out on opening the tap is
(a) 10 m/s (b) 5 m/s
37. If the velocity head of a stream of water is equal to 10 cm, (c) 20 m/s (d) 15 m/s
then its speed of flow is approximately
45. Air stream flows horizontally past an aeroplane wing of
(a) 1.0 m /s (b) 1.4 m / s (c) 140 m / s (d) 10 m / s
surface area 4 m 2 . The speed of air over the top surface is
38. A tank is filled to a height H. The range of water coming 60 m / s and under the bottom surface is 40 m / s. The force
out of a hole which is a depth H / 4 from the surface of of lift on the wing is (density of air = 1 kg / m 3 )
water level is (a) 800 N (b) 1000 N
2H 3H (c) 4000 N (d) 3200 N
(a) (b)
3 2
3H 46. If cross- sectional area of limb I is A1 and that of limb II
(c) 3H (d) is A 2 , then velocity of the liquid in the tube will be,
4
(cross-sectional area of tube is very small)
39. The level of water in a tank is 5 m high. A hole of area
I
1 cm 2 is made at the bottom of the tank. The rate of II
leakage of water from the hole is (g = 10 m / s 2 )
(a) 10−3 m3 / s x
(b) 10−4 m3 /s3 y
(c) 10 m3 / s v

(d) 10−2 m3 / s
Tube
40. A water tank standing on the floor has two small holes
(a) 2g (x − y)
punched in the vertical wall one above the other. The
A1
holes are 2.4 cm and 7.6 cm above the floor. If the jest of (b) 2g (x − y)
water from the holes hit the floor at the same point, then A2
the height of water in the tank is A
(c) 2 2g (x − y)
(a) 10 cm (b) 5 cm A1
(c) 20 cm (d) 4.8 cm (d) None of the above
572 Objective Physics Vol. 1

47. There are two hole O1 and O2 in a tank of height H. The 52. There is hole of area a at the bottom of a cylindrical of
water emerging from O1 and O2 strikes the ground at the area A. Water is filled upto a height h and water flows out
same points, as shown in figure. Then in t second. If water is filled to a height 4 h, then it will
flow out in time
h1 t t
h2 (a) (b) 2t (c) 4t (d)
O1 4 2
h3
O2
53. A container has a small hole at its bottom. Area of
cross-section of the hole is A1 and that of the container is
A 2 . Liquid is poured in the container at a constant rate
(a) H = h1 + h2 (b) H = h2 − h1
Q m 3 / s. The maximum level of liquid in the container
(c) H = h1h2 (d) None of these
will be
48. A cylindrical vessel is filled with a liquid up to a height Q2 Q2 Q Q2
(a) (b) (c) (d)
H. A small hole is made in the vessel at a distance y 2g A1 A2 2g A12 2g A1 A2 2g A22
below, the liquid surface as shown in figure. The liquid
emerging from the hole strike the ground at distance x Viscosity
y 54. The ratio of the terminal velocities of two drops of radii
R and R / 2 is
H (a) 2 (b) 1
(c) 1/ 2 (d) 4

55. An air bubble rises from the bottom of a lake of large


x depth. The rising speed of air bubble will
(a) x is equal if hole is at depth y or H − y (a) go on increasing till it reaches surface
H (b) go on decreasing till it reaches surface
(b) x is maximum for y =
2 (c) increase in the beginning, then will become constant
(c) Both (a ) and ( b) are correct (d) be constant all throughout
(d) Both (c) and (d ) are wrong
56. Units of coefficient of viscosity are
49. Air is blown through a pipe AB at a rate of 15 L/ min. The (a) Nms−1 (b) Nm 2s−1
cross-sectional area of the broad portion of the pipe AB is (c) Nm −2s (d) None of these
2cm 2 and that of the narrow portion is 0.5 cm 2 . The
difference in water level h is (density of air 57. The terminal velocity v of a small steel ball of radius r
= 1.32 kg / m 3 ) falling under gravity through a column of viscous liquid
of coefficient of viscosity η depends on mass of the ball
A
m, acceleration due to gravity g , coefficient of viscosity η
B
and radius r. Which of the following relations is
dimensionally correct?
h mgr
water (a) v ∝ (b) v ∝ mg ηr
η
mg ηmg
(c) v ∝ (d) v ∝
(a) 16 mm (b) 1.5 mm rη η
(c) 10 mm (d) 3.2 mm
58. Two equal drops of water are falling through air with a
50. A tank if filled with water up to height H. When a hole is steady velocity v. If the drops coalesce, then new velocity
made at a distance h below the level of water. What will will be
be the horizontal range of water jet?
(a) 2v (b) 2v
(a) 2 h(H − h) (b) 4 h(H + h) v
2/ 3
(c) 2 v (d)
(c) 4 h(H − h) (d) 2 h(H + h) 2

51. A tank is filled to a height H. The range of water coming 59. As the temperature of water increases, its viscosity
out of a hole which is a depth H / 4 from the surface of (a) remains unchanged
water level is (b) decreases
2H 3H 3H (c) increases
(a) (b) (c) 3H (d)
3 2 4 (d) increases or decreases depending on the external pressure
Fluid Mechanics 573

60. The rate of flow of liquid in a tube of radius r, length l, 66. A lead shot of 1 mm diameter falls through a long column
whose ends are maintained at a pressure difference of glycerine. The variation of its velocity v, with distance
πQ pr 4 covered is represented by
p is V = , where ηis coefficient of the viscosity and
ηl
Q is v v
1 (a) (b)
(a) 8 (b)
8
1
(c) 16 (d)
16 Distance covered Distance covered

61. Two capillary tubes of the same length but different radii
r1 and r2 are fitted in parallel to the bottom of a vessel. v v
The pressure head is p. What should be the radius of a (c) (d)
single tube that can replace the two tubes so that the rate
of flow is same as before?
r1r1
(a) r1 + r2 (b) Distance covered Distance covered
r1 + r2
r1 + r2 67. Water flows through a frictionless duct with a
(c) (d) None of these
2 cross-section varying as shown in figure. Pressure p at
points along the axis is represented by
62. Two capillaries of same length and radii in the ratio 1:2
are connected in series. A liquid flows through them in
streamlined condition. If the pressure across the two
extreme ends of the combination is 1m of water, then
p p
pressure difference across first capillary is
(a) 9.4 m (b) 4.9 m (c) 0.49 m (d) 0.94 m
(a) (b)
63. Water flows in a streamlined manner through a capillary
tube of radius a, the pressure difference being p and the
rate of flow Q. If the radius is reduced to a/ 2 and the x x
pressure increased to 2 p, then rate of flow becomes p p
Q Q
(a) 4Q (b) Q (c) (d)
4 8
(c) (d)
64. A viscous fluid is flowing through a cylindrical tube. The
velocity distribution of the fluid is best represented by the
diagram x x

68. From amongst the following curves, which one shows the
(a) (b) variation of the velocity v with time t for a small sized
spherical body falling vertically in a long column of a
viscous liquid
p p
(c) (d) None of these

(a) (b)

65. Two capillary of lengths L and 2L and of radius R and 2R


are connected in series. The net rate of flow of fluid x x
through them will be (given rate of the flow through p p
single capillary, X = πpR 4 / 8ηL)
8 9
(a) X (b) X (c) (d)
9 8
5 7
(c) X (d) X
7 5 x x
574 Objective Physics Vol. 1

Surface Tension
69. The water droplets in free fall are spherical due to 79. The spiders and insects move and run on the surface of
(a) gravity water without sinking, because
(b) viscosity (a) elastic membrane is formed on water due to property of
(c) surface tension surface tension
(d) intermolecular attraction (b) spiders and insects are lighter
(c) spiders and insects swim on water
70. Two small drops of mercury each of radius r form a single (d) spiders and insects experience upthrust
large drop. The ratio of surface energy before and after
this change is 80. Small droplets of liquid are usually more spherical in
(a) 2 : 22/ 3 (b) 22/ 3 : 1 shape than larger drops of the same liquid, because
(c) 2:1 (d) 1:2 (a) force of surface tension is equal and opposite to the force of
gravity
71. A water proofing agent changes, the angle of contact (b) force of surface tension predominates the force of gravity
(a) from acute to 90° (c) force of gravity predominates the force of surface tension
(b) from obtuse to 90° (d) force of gravity and force of surface tension act in the same
(c) from an acute to obtuse value direction and are equal
(d) from an obtuse to acute value
81. Hair of shaving brush cling together when it is removed
72. With the rise in temperature the angle of contact from water due to
(a) increases (a) force of attraction between hair
(b) decreases (b) surface tension
(c) remains constant (c) viscosity of water
(d) sometimes increases and sometimes decreases (d) characteristic property of hair
73. If the angle of contact is less than 90°, then pressure just 82. A square frame of side L is dipped in a liquid. On taking
inside the surface of a meniscus out, a membrane is formed. If the surface tension of the
(a) is less than atmospheric pressure liquid is T , then force acting on one side of the frame will
(b) is greater than atmospheric pressure be
(c) is same as the atmospheric pressure TL
(d) None of the above (a) 2 TL (b) 4 TL (c) TL (d)
2
74. Pressure inside two soap bubbles are 1.01 and 1.02 83. Water does not wet an oily glass, because
atmospheres. Ratio of their volumes is (a) cohesive force of oil > adhesive force between oil and glass
(a) 2:1 (b) 4:1 (b) cohesive force of oil > cohesive force of water
(c) 8:1 (d) 12:1 (c) oil repels water
(d) cohesive force of water > adhesive force between water and
75. Which of the following is not the unit of surface tension? oil molecules
(a) Newton/metre
(b) Joule / (metre)2 84. Which of the fact is not due to surface tension?
(c) kg/ (second)2 (a) Dancing of a camphor piece over the surface of water
(d) Watt/metre (b) Small mercury drop itself becomes spherical
(c) A liquid surface comes at rest after stirring
76. Surface tension is due to (d) Mercury does not wet the glass vessel
(a) friction forces between molecules
(b) cohesive forces between molecules
85. The property of surface tension is obtained in
(c) adhesive forces between molecules (a) solids, liquids and gases (b) liquids
(d) gravitational forces (c) gases (d) matter

77. Coatings used on raincoat are waterproof, because 86. If two glass plates are quite nearer to each other in water,
(a) water is absorbed by the coating then there will be force of
(b) cohesive force becomes greater (a) attraction
(c) water is not scattered away by the coating (b) repulsion
(d) angle of contact decreases (c) attraction or repulsion
(d) Neither attraction nor repulsion
78. If temperature increases, then surface tension of a liquid
87. On mixing the salt in water, the surface tension of water
(a) increases
(b) decreases
will
(c) remains the same (a) increase (b) decrease
(d) first increases, then decreases (c) remain unchanged (d) None of these
Fluid Mechanics 575

88. A 10 cm long wire is placed horizontally on the surface of 99. A liquid rises in a vertical tube. The relation between the
water and is gently pulled up with a force of 2 × 102 N to weight of the liquid in the tube, surface tension of the
keep the wire in equilibrium. The surface tension in liquid T and radius of the tube r is given by, if the angle of
Nm −1 of water is contact is zero
(a) 0.1 (b) 0.2 (c) 0.001 (d) 0.002 (a) w = πr2T (b) w = 2πrT
3
89. The dimensions of surface tension are (c) w = 2r2πT (d) w = πr3T
−1 2 −2 0 −2 −1 −2 4
(a) [MLT ] (b) [ML T ] (c) [ML T ] (d) [ML T ]
100. A glass plate is partly dipped vertically in the mercury
90. If work W is done in blowing a bubble of radius R from
and angle of contact is measured. If the plate is inclined,
soap solution, then the work done in blowing a bubble of
then the angle of contact will
radius 2R from the same solution is
1 (a) increase (b) remain unchanged
(a) W /2 (b) 2W (c) 4W (d) 2 W (c) increase or decrease (d) decrease
3
101. If a water drop is kept between two glass plates, then its
91. If two identical mercury drops are combined to form a
shape is
single drop, then its temperature will
(a) decrease (b) increase
(c) remains the same (d) None of these (a) (b)

92. The surface tension of a soap solution is 2 × 10−2 N / m .


To blow a bubble of radius 1 cm, the work done is
(a) 4 π × 10−6 J (b) 8π × 10−6 J (c) (d) None of these
(c) 12π × 10−6 J (d) 16π × 10−6 J

93. The surface tension of liquid at its boiling point


(a) becomes zero 102. A liquid wets a solid completely. The meniscus of the
(b) becomes infinity liquid in a sufficiently long tube is
(c) is equal to the value at room temperature (a) flat
(d) is half to the value at the room temperature (b) concave
(c) convex
94. Which of the following statements are true in case, when (d) cylindrical
two water drops coalesce and make a bigger drop?
(a) energy is released 103. If two soap bubble of different radii are in
(b) energy is absorbed communication with each other
(c) the surface area of the bigger drop is greater than the sum of (a) air flows from larger bubble into the smaller one
the surface areas of both the drops (b) the size of the bubbles remains the same
(d) the surface area of the bigger drop is smaller than the sum of (c) air flows from the smaller bubble into the large one and the
the surface areas of both the drops larger bubble grows at the expense of the smaller one
95. If two soaps bubbles of equal radii r coalesce, then the (d) None of the above
radius of curvature of interface between two bubble 104. The surface tension of soap solution is 25 × 10−3 Nm −1 .
will be The excess pressure inside a soap bubble of diameter
r 1cm is
(a) r (b) 0 (c) infinity (d)
2 (a) 10 Pa (b) 20 Pa
96. A liquid does not wet the sides of a solid, if the angle of (c) 5 Pa (d) None of these
contact is 105. When two soap bubbles of radius r1 and r2 ( r2 > r1 )
(a) zero (b) obtuse (c) acute (d) 90°
coalesce, then radius of curvature of common surface is
97. When the temperature is increased the angle of contact of r2 − r1
a liquid? (a) r2 − r1 (b)
r1r2
(a) Increases r1r2
(b) Decreases (c) (d) r2 + r1
r2 − r1
(c) Remains the same
(d) First increases and then decreases 106. The excess pressure due to surface tension in a spherical
98. The angle of contact between glass and mercury is liquid drop of radius r is directly proportional to
(a) 0° (b) 30° (a) r (b) r2
(c) 90° (d) 135° (c) r−1 (d) r−2
576 Objective Physics Vol. 1

107.A long cylindrical glass vessel has a small hole of radius r 111.Which graph represents the variation of surface tension
at its bottom. The depth to which the vessel can be with temperature over small temperature ranges for
lowered vertically in the deep water bath (surface water?
tension T ) without any water entering inside is
(a) 4T /ρrg
(b) 3T /ρrg ST ST
(c) 2T /ρrg
(a) (b)
(d) T /ρrg
108.Two soap bubbles of radii r1 and r2 equal to
4 cm and 5 cm are touching each other over a common Temp x Temp
surface S 1 S 2 (shown in figure). Its radius will be
ST ST
S1
(c) (d)
4 cm 5 cm

Temp Temp
S2

112.A thread is tied slightly loose to a wire frame as in figure


(a) 4 cm and the frame is dipped into a soap solution and taken
(b) 20 cm out. The frame is completely covered with the film. When
(c) 5 cm
(d) 4.5 cm A is pricked

109.A vessel, whose bottom has round holes with diameter of Frame
1.0 mm, is filled with water. The maximum height to
which the water can be filled without leakage is (surface
tension of water = 75 dyne/cm, g = 1000 cm / s 2 )
(a) 100 cm (b) 75 cm Thread
(c) 50 cm (d) 30 cm

110.A soap bubble is blown with the help of a mechanical


pump at the mouth of a tube. The pump produces a (a) thread will become concave on seeing from side A
(b) thread will become concave on seeing from side B
constant increase per minute in the radius of the bubble,
(c) thread will become straight
irrespective of its internal pressure. The graph between (d) thread will remain as it is
the excess pressure inside, the soap bubble and time t will
be Capillary Rise or Fall
p p 113.The liquid in the capillary tube will rise, if the angle of
contact is
(a) 0°
(a) (b)
(b) 90°
(c) obtuse
(d) acute
t t
p p 114.If a capillary tube is dipped into liquid and the levels of
the liquid inside and outside are same, then the angle of
contact is
(c) (d) (a) 0°
(b) 90°
(c) 45°
t t (d) 30°
Fluid Mechanics 577

115.Water rises to a height of 30 mm in a capillary tube. If the 123. In a surface tension experiment with capillary tube, water
3 rises upto 0.1 m. If the same experiment is repeated on an
radius of the capillary tube is made th of its previous
4 artificial satellite, while is revolving around the earth,
value. The height to which the water will rise in the tube then water will rise in the capillary tube up to a height of
is (a) 0.1 m
(a) 30 mm (b) 20 mm (b) 0.2 m
(c) 40 mm (d) 10 mm (c) 0.98 m
(d) full length of the capillary tube
116.When two capillary tubes of different diameters are
dipped vertically, then rise of the liquid is 124.The correct curve between the height or depression h of
(a) same in both the tubes liquid in a capillary tube and its radius is
(b) more in the tube of larger diameter h h
(c) less in the tube of smaller diameter
(d) more in the tube of smaller diameter
(a) (b)
117.Due to capillary action, a liquid will rise in a tube, if the
angle of contact is
(a) acute (b) obtuse r r
(c) 90° (d) zero h h

118.Two parallel glass plates are dipped partly in the liquid of


density d keeping them vertical. If the distance between (c) (d)
the plates is x surface tension for the liquid is T and angle
of contact θ, then rise of liquid between the plates due to
capillary will be r r
T cosθ 2T cosθ
(a) (b)
xd xdg Miscellaneous Problems
2T T cosθ
(c) (d) 125. A tank full of water has a small hole at its bottom. Let t 1
xdg cosθ xdg
be the time taken to empty, the first half of the tank and t 2
119.Water rises in a capillary tube to a certain height such be the time needed to empty the rest half of the tank, then
that the upward force due to surface tension is balanced (a) t1 = t2 (b) t1 > t2
by 75 × 10−4 N force due to the weight of the liquid. If (c) t1 < t2 (d) t1 = 0.523t2
the surface tension of water is 6 × 10−2 N/m, then inner
126.The density of ice is x g /cm 3 and that of water is
circumference of the capillary must be
(a) 1.25 × 10−2 m (b) 0.50 × 10−2 m y g / cm 3 , when m gram of ice melts, then the change in
(c) 6.5 × 10−2 m (d) 12.5 × 10−2 m volume is
y−x
120.Two capillary tubes P and Q are dipped in water. The (a) m ( y − x ) (b)
m
height of water level in capillary P is 2/ 3 to the height in m m
(c) my ( y − x ) (d) −
Q capillary. The ratio of their diameters is y x
(a) 2:3 (b) 3:2
(c) 3:4 (d) 4:3 127. A ball of relative density 0.8 falls into water from a
height of 2 m. The depth to which the ball will sink is
121.Water rises up to 10 cm height in a long capillary tube. If (neglect viscous forces)
this tube is immersed in water so that the height above the (a) 8 m (b) 2 m
water surface is only 8 cm, then (c) 6 m (d) 16 m
(a) water flows out continuously from the upper end
(b) water rises upto upper end and forms a spherical surface 128. A cylindrical vessel of 92 cm height is kept filled up to
(c) water only rises upto 6 cm height the brim. It has four holes 1, 2, 3, 4 which are respectively
(d) water does not rise at all at heights of 20 cm, 30 cm, 46 cm and 80 cm from the
horizontal floor. The water falling at the maximum
122.The correct relation is
2T cosθ hdg
horizontal distance from the vessel comes from
(a) r = (b) r = (a) hole number 4
hdg 2T cosθ
(b) hole number 3
2T dgh T cosθ
(c) r = (d) r = (c) hole number 2
cosθ 2hdg
(d) hole number 1
578 Objective Physics Vol. 1

129. Two substances of relative densities ρ1 and ρ 2 are mixed 133. A metal sphere connected by a string is dipped in a liquid
in equal volume and the relative density of mixture is 4. of density ρ as shown in figure. The pressure at the
When they are mixed in equal masses, then relative bottom of the vessel will be, ( p 0 = atmospheric pressure)
density of the mixture is 3. The values of ρ1 and ρ 2 are
(a) ρ = 6 and ρ 2 = 2 (b) ρ 1 = 3 and ρ 2 = 5
(c) ρ 1 = 12 and ρ 2 = 4 (d) None of these
h
130. A solid shell loses half its weight in water. Relative
density of shell is 0.5, what fraction of its volume is
hollow? (a) p = p0 + ρgh (b) p > p0 + ρgh
(a)
3
(b)
2 (c) p < p0 + ρgh (d) p0
5 5
1 4 134. A tank filled with water has two taps to exhaust and
(c) (d) pour. A hollow spherical ball is half submerged in
5 5
water. Through one tap, water is taken out and through
131. An iron block is on a boat which floats in a pond. The another tap, a liquid of density double the density of
block is thrown into the water. The level of water in the water is poured in tank such that volume of liquid in
pond will be tank remains constant. Sphere will
(a) equal to the earlier level
(b) less than the earlier level
(c) more than the earlier level
(d) depends on how large the block is

132. The pressure of the gas in a cylindrical chamber is p 0 .


The vertical force exerted by the gas on its hemispherical
end is
(a) go down
(b) go up
r
(c) maintain same height
(d) sink to bottom

135. An air bubble of radius 1 mm is formed inside water at a


Gas depth 10 m below free surface (where, air pressure is
105 N /m 2 ). The pressure inside the bubble is (surface
tension of water = 2 × 10−7 N / m).
(a) p0r2 (b) 4 p0πr2 (a) 2.28 × 105 N/m 2 (b) 2.0028 × 105 N/m 2
(c) 2 p0πr 2
(d) p0πr 2
(c) 2.14 × 105 N/m 2 (d) 2.0014 × 105 N/m 2
[ Level 2 ]
Only One Correct Option
1. A block of mass 4 kg and volume 5 × 10−4 m 3 is 7. A U-tube of base length l filled with same volume of two
suspended by a spring balance in a lift which is liquids of densities ρ and 2ρ is moving with an
accelerating. The apparent weight shown by the spring acceleration a on the horizontal plane. If the height
balance is 3 kg. Now, the block is immersed in water in a difference between the two surfaces (open to
container inside the lift. The apparent weight in kg shown atmosphere) becomes zero, then height h is given by
by the spring balance is
(a) 2.375 (b) 2.625 (c) 2.5 (d) 3.125
ρ
2. A ball of mass 1 kg falls from a height of 5m above the
free surface of water. The relative density of the solid ball h 2ρ a
2
is s = . The ball travels a distance of 2 m under water and
3
becomes stationary. The work done by the resistive l
forces of water is
al 3al
(a) −50 J (b) −20 J (a) (b)
(c) −40 J (d) −30 J g 2g
2al al
3. A cubical block of wood of specific gravity 0.5 and (c) (d)
3g 2g
chunk of concrete of specific gravity 2.5 are fastened
together. The ratio of the mass of wood to the mass of 8. A body of volume V and density ρ is initially submerged
concrete, which makes the combination to float with its in a non-viscous liquid of density σ (> ρ ). If it is rises by
entire volume submerged under water is itself through a height h in the liquid. Its kinetic energy
(a) 1/5 (b) 1/3 will
(c) 3/5 (d) 2/3 (a) increase by hV (σ − ρ )g
(b) increase by hV (ρ + σ )g
4. A cylinder of mass M and density d1 hanging from a hVρg
string, is lowered into a vessel of cross-sectional area A, (c) increase by
σ
containing a liquid of density d 2 ( d 2 < d1 ) until it is fully hVρg
immersed. The increase in pressure at the bottom of the (d) decrease by
σ
vessel is
Md2g Mg 9. A cubical block of side 10 cm floats at the interface of an
(a) (b)
d1 A A oil and water. The pressure above that of atmosphere at
Md1g the lower face of the block is
(c) (d) zero
d2 A

5. Two identical cylindrical vessels, each of base area A, Oil 0.6 g/cc
have their bases at the same horizontal level. They 10 cm
contain a liquid of density ρ. In one vessel the height of 2 cm
the liquid is h1 and in the other h2 (> h1 ). When the two 10 cm 1.0 g/cc
vessels are connected, the word done by gravity in
equalizing the levels is
(a) 200 N/m 2 (b) 680 N / m 2
(a) 2ρAg (h2 − h1 )2 (b) ρAg (h2 − h1 )2
1 1 (c) 400 N / m 2 (d) 800 N / m 2
(c) ρAg (h2 − h1 )2 (d) ρAg (h2 − h1 )2
2 4 10. A sphere of solid material of specific gravity 8 has a
6. A small ball of density ρ is immersed in a liquid of density concentric spherical cavity and just sinks in water. Then,
σ (σ > ρ ) to a depth h and then released. The height above the ratio of the radius of the cavity to the outer radius of
the surface of water up to which the ball will jump is the sphere must be
σh  σh  3
3
3
5
(a) (b)  − 1 h (a) (b)
ρ  ρ  2 2
 σh ρh
3
7 2
(c) 1−  h (d) (c) (d)
 ρ σ 2 3
7
580 Objective Physics Vol. 1

11. A large block of ice 10 cm thick with a vertical hole 16. A liquid stands at the plane level in U-tube when at rest.
drilled through it is floating in a lake. The minimum If areas of cross-section of both the limbs are equal, what
length of the rope required to scoop out a bucket full of will be the difference in heights h of the liquid in the two
water through the hole is (density of ice = 0.9 g / cm 3 ) limbs of U-tube, when the system is given an acceleration
(a) 0.5 m (b) 1.0 m a in horizontal direction towards right as shown in the
(c) 1.2 m (d) 1.8 m figure?
12. A large tank is filled with water (density = 103 kg / m 3 ).
A small hole is made at a depth 10 m below water surface.
The range of water issuing out of the hole is R on ground. a
What extra pressure must be applied on the water surface H
so that the range become 2R? (take 1 atm = 105 Pa and
g = 10 m / s 2 )
L
2
g L La L2 a Lg
(a) (b) (c) (d)
a H g H g a

17. A capillary tube is dipped in a liquid. Let pressure at


point A , B and C be p A , p B , pC respectively, then

R
B
(a) 1 atm (b) 2 atm
(c) 4 atm (d) 3 atm A C

13. A wooden plank of length 1m and uniform cross-section


is hinged at one end to the bottom of a tank as shown in
the figure. The tank is filled with water up to a height of
0.5m. The specific gravity of the plank is 0.5. The angle θ
made by the plank in equilibrium position is
(a) pA = pB = pC (b) pA = pB < pC
(c) pA = pC < pB (d) pA = pC > pB

0.5m 18. A small ball (mass m) falling under gravity in a viscous


medium experiences a drag force proportional to the
instantaneous speed u such that Fdrag = ku. Then the
terminal speed of ball within viscous medium is
(a) 30° (b) 45°
k mg
(c) 60° (d) 90° (a) (b)
mg k
14. An open U-tube contains mercury. When 11.2 cm of mg
water is poured into one of the arms of the tube, how high (c) (d) None of these
k
does the mercury rise in the other arm from its initial
level? 19. A candle of diameter d is floating on a liquid in a
(a) 0.82 cm (b) 1.35 cm cylindrical container of diameter D ( D > > d ) as shown in
(c) 0.41 cm (d) 2.32 cm figure. If it is burning at the rate of 2 cm / h, then the top
of the candle will
15. A body of density ρ is dropped from rest from a height h
into a lake of density σ (σ > ρ ). The maximum depth the
body sinks inside the liquid is (neglect viscous effect of
L
liquid)
hρ L
(a)
σ −ρ
d

(b)
σ −ρ
hρ D
(c)
σ
hσ (a) remain at the same height (b) fall at the rate 1 cm/h
(d) (c) fall at the rate of 2 cm/h (d) go up at the rate of 1 cm/h
ρ
Fluid Mechanics 581

20. A container has two immiscible liquids of densities 25. The volume of an air bubble becomes three times as it
ρ1 and ρ 2 (> ρ1 ). A capillary tube of radius r is inserted rises from the bottom of a lake to its surface. Assuming
in the liquid so that its bottom reaches up to the denser temperature to be constant and atmospheric pressure to
liquid. The denser liquid rises in the capillary and attains be 75 cm of Hg and the density of water to be 1/10 of the
a height h from the interface of the liquids, which is equal density of the mercury, the depth of the lake is
to the column length of the lighter liquid. Assuming angle (a) 5 m (b) 10 m
of contact to be zero, the surface tension of heavier (c) 15 m (d) 20 m
liquid is 26. A barometer kept in an elevator reads 76 cm when it is at
rest. If the elevator goes up with increasing speed, then
reading will be
ρ1 (a) zero (b) 76 cm
h
(c) > 76 cm (d) <76 cm

ρ2 27. The surface energy of a liquid drop is E. It is sprayed into


1000 equal droplets. Then its surface energy becomes
(a) E (b) 10 E (c) 100 E (d) 1000 E
ρ rgh
(a) 2πrρ 2 gh (b) 2 28. An open tank containing non-viscous liquid to a height of
2 5 m is placed over the ground. A heavy spherical ball
r
(c) (ρ 2 − ρ 1 )gh (d) 2πr (ρ 2 − ρ 1 )gh falls from height 40 m over the ground in the tank.
2
Ignoring air resistance find the height to which ball will
21. A spherical object of mass 1 kg and radius 1m is falling go back. Collision between ball and bottom of tank is
vertically downward inside a viscous liquid in a gravity perfectly elastic
free space. At a certain instant the velocity of the sphere (a) 45 m (b) 35 m
is 2 m/s. If the coefficient of viscosity of the liquid is (c) 40 m (d) 20 m
1 29. A large open tank has two holes in the wall. One is a
SI units, then velocity of ball will become 0.5 m / s
6π square hole of side L at a depth y from the top and the
after a time. other is a circular hole of radius R at a depth 4 y from the
(a) ln 4 s (b) 2 ln 4 s top. When the tank is completely filled with water, then
(c) 3 ln 4 s (d) 3 ln 2s quantities of water flowing out per second from holes are
22. If a capillary tube of radius r is immersed in water, then the same. Then R is equal to
mass of water risen in capillary is M. If the radius of (a) L/ 2π
capillary be doubles, then mass of water risen in the (b) 2πL
(c) L
capillary will be
(d) L/2π
(a) M/2 (b) M
(c) 2M (d) 4M 30. A pump is designed as a horizontal cylinder with a piston
23. A wooden block of mass 8 kg is tied to a string attached area A and an outlet orifice arranged near the axis of the
to the bottom of the tank. In the equilibrium the block is cylinder. Find the velocity of outflow of liquid from
completely immersed in water. If relative density of pump, if the piston moves with a constant velocity under
the action of a constant force F. The density of liquid is ρ
wood is 0.8 and g = 10 ms −2 , then tension T in the
string is
(a) 120 N
(b) 100 N
(c) 80 N
(d) 20 N F 2F
(a) (b)
Aρ Aρ
24. A metal ball immersed in alcohol weighs w1 at 0° C and Aρ Aρ
w 2 at 59° C. The coefficient of cubical expansion of the (c) (d)
F 2F
metal is less than that of alcohol. Assuming that the
density of the metal is large compared to that of alcohol, 31. A tank is filled up to a height 2H with a liquid and is
it can be shown that placed on a platform of height H from the ground. The
(a) w1 > w2 distance x from the ground, where a small hole is punched
(b) w1 = w2 to get the maximum range R is
(c) w1 < w2 (a) H (b) 1.25 H
(d) w1 = (w1 / 2) (c) 1.5 H (d) 2 H
582 Objective Physics Vol. 1

32. A piece of steel has a weight w in air, w1 when completely 4. A non-viscous incompressible liquid is flowing from a
immersed in water and w 2 when completely immersed in horizontal pipe of non-uniform cross-section as shown in
an unknown liquid. The relative density (specific gravity) figure. Choose the correct options.
of liquid is
w − w1 w − w2 1 2
(a) (b)
w − w2 w − w1
w − w2 w − w2
(c) 1 (d) 1
w − w1 w − w2

33. Two cylinders of same cross-section and length L but


made of two materials of densities d1 and d 2 are (a) Speed of liquid at section 2 is more
connected together to a form a cylinder of length 2L. The (b) Volume of liquid flowing per second from section 2 is more
combination floats in a liquid of density d with a length (c) Mass of liquid flowing per second at both the sections is
L/ 2 above the surface of the liquid. If d1 < d 2 , then same
3 d (d) Pressure at section 2 is less
(a) d1 < d (b) > d1
4 2
5. A plank is floating in a liquid as shown in the figure.
d d
(c) > d1 (d) d1 > d Fraction f of its volume is immersed. Choose the correct
4 4
options.
More than One Correct Options
1. A large wooden plate of area 10 m 2 floating on the
surface of a river is made to move horizontally with a
speed of 2 m/s by applying a tangential force. River is
1 m deep and the water in contact with the bed is
stationary. Then choose correct statement. (a) If the system is taken to a place, where atmospheric pressure
(coefficient of viscosity of water = 10−3 N-s/m 2 ) is more, f will increase
(a) Velocity gradient is 2 s−1 (b) In above condition f will remain unchanged
(b) Velocity gradient is 1 s−1 (c) If temperature is increased and expansion of only liquid is
considered f will increase
(c) Force required to keep the plate moving with constant speed
(d) If temperature is increased and expansion of only plank is
is 0.02 N
considered f will decrease
(d) Force required to keep the plate moving with constant speed
is 0.01 N 6. In two figures
2. Choose the correct options.
(a) Viscosity of liquids increases with temperature h
v1 4h
(b) Viscosity of gases increases with temperature
(c) Surface tension of liquids decreases with temperature v2
4h h
(d) For angle of contact θ = 0°, liquid neither rises nor falls on t1 t2
capillary
R1 R2
3. A plank is floating in a non-viscous liquid as shown in
(a) v1 / v2 = 1/ 2
the figure. Choose the correct options. (b) t1 / t2 = 2/1
(c) R1 / R2 = 1
(d) v1 / v2 = 1/ 4
7. A liquid is filled in a container as shown in figure.
Container is accelerated towards right. There are four
points A , B , C and D in the liquid. Choose the correct
options.
(a) Equilibrium of plank is stable in vertical direction A B
(b) For small oscillations of plank in vertical direction motion is
a
simple harmonic
(c) Even, if oscillations are large, motion is simple harmonic till
it is not fully immersed D C

(d) On vertical displacement motion is periodic but not simple (a) PA > PB (b) PC > PA
harmonic (c) PD > PB (d) PA > PC
Fluid Mechanics 583

8. A ball of density ρ is dropped from a of the can. When water contained in this can is allowed to
height on the surface of a non-viscous come out of the tubes the distances on the horizontal
liquid of density 2ρ. Choose the correct surface are measured as x1 , x 2 and x 3
options.
(a) Motion of ball is periodic but not simple
harmonic a
(b) Acceleration of ball in air and in liquid are equal T1
a
(c) Magnitude of upthrust in the liquid is two times the weight
T2
of ball a
(d) Net force on ball in air and in liquid are equal and opposite T3
a
9. Two holes 1 and 2 are made at depths h
and 16h, respectively. Both the holes
are circular but radius of hole 1 is two 1 1. Speed of efflux is
times. 1
2 (a) 3gh (b) 2gh (c) gh (d) 2gh
(a) Initially equal volumes of liquid will 2
flow from both the holes in unit time 2. Distance x 3 is given by
(b) Initially more volume of liquid will flow from hole 2 per
unit time 1
(a) 3a (b) 2a (c) 3a (d) 2 3 a
(c) After some time more volume of liquid will flow from hole 1 2
per unit time
(d) After some time more volume of liquid will flow from hole 2 3. The correct sketch is
per unit time

10. A solid sphere, a cone and a cylinder are floating in


water. All have same mass, density and radius. Let
f1 , f 2 and f 3 are the fraction of their volumes inside the
water and h1 , h2 and h3 depths inside water. Then,
(a)

x1
x2
x3

(a) f1 = f2 = f3 (b) f3 > f2 > f1


(c) h3 < h1 (d) h3 < h2 (b)
11. Streamline flow is more likely for liquids with
(a) high density (b) high viscosity
(c) low density (d) low viscosity x1
x2
x3
Comprehension Based Questions
Passage I (Q.1 to 3)
The spouting can is
something used to h v
demonstrate the variation
of pressure with depth. (c)
When the corks are
removed from the tubes in
the side of the can, water x x2
flows out with a speed that x 1 = x3
depends on the depth. In a certain can, three tubes
T1 , T2 and T3 are set at equal distances a above the base (d) None of the above
584 Objective Physics Vol. 1

Passage II (Q. 4 to 7)
Assertion and Reason
A container of large uniform cross-sectional area A
Direction (Q. Nos. 1-20) These questions consists of two
resting on a horizontal surface, holds, two immiscible,
statements each printed as assertion and reason. While
non-viscous and incompressible liquids of densities
d and 2d each of height H / 2 as shown in the figure. The answering these questions you are required to choose any one
lower density liquid is open to the atmosphere having of the following five responses.
pressure P0 . A homogeneous solid cylinder of length (a) If both Assertion and Reason are correct and Reason is the
correct explanation of Assertion
L( L < H / 2) , cross-sectional area A/ 5 is immersed such
(b) If both Assertion and Reason are true but Reason is not the
that it floats with its axis vertical at the liquid-liquid correct explanation of Assertion
interface with length L/ 4 in the denser liquid. (c) If Assertion is true but Reason is false
(d) If Assertion is false but Reason is true
(e) If both Assertion and Reason are false
H/2 d
1. Assertion When an ideal fluid flows through a pipe of
non-uniform cross-section, then pressure is more at that
H/2 2d h
section, where area is more if the pipe is horizontal.
Reason According to Bernoulli’s theorem speed at
x
broader cross-section will be less.

The cylinder is then removed and the original 2. Assertion A ball is released from the bottom of a tank
arrangement is restored. A tiny hole of area s( s < < A ) is filled with a liquid. It moves upwards. In moving
punched on the vertical side of the container at a height upwards upthrust will decrease.
h( h < H / 2) . As a result of this, liquid starts flowing out of
the hole with a range x on the horizontal surface.
4. The density D of the material of the floating cylinder is
(a) 5d / 4 (b) 3d / 4
(c) 4 d / 5 (d) 4 d / 3

5. The total pressure with cylinder, at the bottom of the


container is Reason Density of ball is less than the density of liquid.
(6L + H ) 3. Assertion A wooden plank is floating in two liquids as
(a) p0 + dg
4 shown. Net force applied by liquid-1 on plank is zero.
(L + 6H )
(b) p0 + dg Reason Contribution in upthrust due to liquid 1 is
4
V1ρ1 g. Where, V1 = volume immersed in liquid 1and ρ1 is
(L + 3H )
(c) p0 + dg the density of liquid 1.
4
(L + 2H )
(d) p0 + dg
4
1
6. The initial speed of efflux without cylinder is
g
(a) v = [ 3H + 4 h ]
3 2
g
(b) v = [ 4 H − 3h ]
2
4. Assertion In the siphon figure shown, p1 = p 2 .
g
(c) v = [ 3H − 4 h ]
2
(d) None of the above 2 1

7. The horizontal distance travelled by the liquid, initially,


is
(a) (3H + 4 h) h
(b) (3h + 4 H )h v
(c) (3H − 4 h) h
(d) (3H − 3h) h Reason Pressure at 1 is less than the atmospheric
pressure.
Fluid Mechanics 585

5. Assertion If ice is floating in water and it melts, then 14. Assertion A solid is floating in a liquid. If temperature
level of water remains unchanged. is increased and expansion of solid is ignored, then
Reason When the ice is floating, weight of liquid fraction of volume immersed will increase.
displaced is equal to the weight of ice. Reason By increasing the temperature density of liquid
6. Assertion An ice ball is floating in water. Some stone will decrease.
pieces are embedded inside the ice. When ice will melt, 15. Assertion If angle of contact is 0°, then liquid will
level of water will fall. neither rise nor fall in a capillary.
Reason In floating condition, stone pieces will displace Reason When angle of contact is 0°, surface is neither
more liquid compared to the condition when they sink. convex nor concave inside the capillary. It is flat.
7. Assertion In a freely falling liquid container, upthrust 16. Assertion Small water drops are spherical while bigger
force is zero.
water drops are not.
Reason In freely falling case value of effective value of
Reason In small water drops surface tension forces
g is zero.
dominate while in bigger water drops gravity forces
8. Assertion Bulk modulus of an incompressible liquid is dominate.
infinite. 17. Assertion Bulk modulus of an ideal fluid is infinite.
Reason Compressibility is inverse of bulk modulus. Reason An ideal fluid is incompressible.
9. Assertion Density of an incompressible liquid is
18. Assertion Deep inside a liquid density is more than the
constant.
density on surface.
Reason An ideal fluid is incompressible.
Reason Density of liquid increases with increase in
10. Assertion A solid object of iron is dipped in water, depth.
both are at same temperature of 2°C. If the temperature of 19. Assertion A solid is floating in a liquid of density ρ1 .
water is increased by 2°C, then the buoyancy force action
When the solid melts its density becomes ρ 2 in liquid
of the object will increase.
state. If ρ1 > ρ 2 level of liquids will increase after
Reason If we increase the temperature of water from melting.
2°C to 4°C, then density of water will increase. Ignore
expansion of solid sphere. Reason In liquid state volume always increases after a
solid melts.
11. Assertion A solid sphere and a hollow sphere both of
same material are immersed in a liquid, then change in 20. Assertion Mass of solid floating in liquid is m1 and
weight in both the spheres will be same. mass of liquid is m2 . Base area is A. Then pressure at
( m + m2 )g
Reason Upthrust depends upon the volume of the solid bottom is p 0 + 1 .
A
immersed not the mass.
12. Assertion If water is filled in a balloon and this is
immersed in water itself. Then volume of water displaced
is equal to the volume of water filled in the balloon.
Reason Volume of a liquid displaced is equal to the
volume of solid immersed in that liquid.
13. Assertion At same level of same liquid pressure is
always same.
Reason When any fluid travels from a region of higher
pressure to lower pressure (at same levels) it gains same Reason Upward force on liquid from base of vessel is
speed. pA, where p is pressure at bottom.
586 Objective Physics Vol. 1

Match the Columns


1. A tube is inverted in a mercury vessel as shown in figure.
Column I Column II
If pressure p is increased, then
(A) Pressure B will (p) Increase
p (B) Pressure difference (q) Decrease
between A and B will
h (C) Upthrust on an object (r) Remain same
O inside the vessel will

5. Two soap bubbles coalesce to form a single large drop.


Match the following columns.
Column I Column II
Column I Column II (A) Surface energy in the (p) Increase
(A) Height h (p) Will increase process will

(B) Pressure at O (q) Will decrease (B) Temperature of the drop (q) Decrease
will
(C) Pressure at 1 cm above (r) Will remain same
(C) Pressure inside the soap (r) Remain same
bubble will
2. In the figure shown, velocity of liquid which comes out is v,
time of liquid to fall to ground is t and range on ground is R. 6. A cube is floating in liquid as shown in figure. Match the
If the vessel is taken to a mountain, match the following, following columns.
(consider all cases which might possible)
x

Column I Column II
R
(A) If density of liquid (p) Increase
Column I Column II decreases x will
(A) v (p) Will increase (B) If size of cube is increased (q) Decrease
x will
(B) t (q) Will decrease
(C) If the whole system is (r) Remain same
(C) R (r) Will remain same accelerated upwards x will

3. A liquid is flowing through a pipe of non-uniform


cross-section. At a point where area of cross-section of
Entrance Gallery
the pipe is less, match the following columns. 2014
Column I Column II
1. There is a circular tube in a vertical plane. Two liquids
(A) Volume of liquid flowing (p) Is less which do not mix and of densities d1 and d 2 are filled in
per second
the tube. Each liquid subtends 90° angle at centre. Radius
(B) Speed of liquid (q) Is more
joining their interface makes an angle α with vertical.
(C) Pressure of liquid (r) Is same
Ratio d1 / d 2 is [JEE Main]
4. There are two points A and B inside a liquid as shown in
figure. Now, the vessel starts moving upwards with an
acceleration a. Match the following columns.

α d2
A

d1

B 1 + sin α 1 + cos α 1 + tan α 1 + sin α


(a) (b) (c) (d)
1 − sin α 1 − cos α 1 − tan α 1 − cos α
Fluid Mechanics 587

2. On heating water, bubbles beings formed at the bottom of 2012


the vessel detach and rise. Take the bubbles to be spheres
8. Energy of bubble is E. If one thousand such bubbles
of radius R and making a circular contact of radius r with
the bottom of the vessel. If r < < R and the surface tension coalesce to form a large bubble then its energy is E1 , then
of water is T, value of r just before bubbles detach is ratio E and E1 is equal to [O JEE]
1 1000
(density of water is ρ) [JEE main] (a) (b)
100 1
1 10
(c) (d)
10 1

2011
R
9. Two solid spheres A and B of equal volumes but of
different densities d A and d B are connected by a string.
2r They are fully immersed in a fluid of density d F . They
get arranged into an equilibrium state as shown in the
2ρ w g ρwg
(a) R 2 (b) R 2 figure with a tension in the string. The arrangement is
3T 6T
possible only if [IIT JEE]
ρwg 3ρ w g
(c) R 2 (d) R 2
T T

3. A flow of liquid is streamline, if the Reynold’s number is A


(a) less than 1000 [Karnataka CET]
(b) greater than 1000
B
(c) between 2000 to 3000
(d) between 4000 to 5000
(a) dA < dF (b) dB > dF
4. A drop of some liquid of volume 0.04 cm 3 is placed on (c) dA > dF (d) dA + dB = 2dF
the surface of a glass slide. Then, another glass slide is
10. If a ball of steel (density ρ = 7.8 g cm − 3 ) attains a
placed on it in such a way that the liquid forms a thin
terminal velocity of 10 cms −1 when falling in a tank of
layer of area 20 cm 2 between the surfaces of the two water (coefficient of viscosity η water = 8.5 × 10−4 Pa-s)
slides. To separate the slides a force of 16 × 105 dyne has then its terminal velocity in glycerine (ρ = 1.2 g cm −3 ,
to be applied normal to the surfaces. The surface tension η = 13.2 Pa-s) would be nearly [AIEEE]
of the liquid is (in dyne - cm −1 ). [WB JEE] (a) 1.6 × 10−5 cm s−1 (b) 6.25 × 10−4 cm s−1
(a) 60 (b) 70 −4 −1
(c) 80 (d) 90 (c) 6.45 ×10 cm s (d) 1.5 × 10−5 cm s−1

5. Which one of the following equation is Torricelli’s law? 11. Water is flowing continuously from a tap having an
[J&K CET] internal diameter 8 × 10−3 m. The water velocity as it
(a) p = ρgh (b) v = 2hg leaves the tap is 0.4 m/s. The diameter of the water stream
(c) ηRe = ρvd (d) S (2dl ) = Fd at a distance 2 × 10−1 m below the tap is close to [AIEEE]
(a) 7.5 × 10− 3 m (b) 9.6 × 10− 3 m
2013 (c) 3.6 × 10− 3 m (d) 5.0 × 10− 3 m
6. Assume that a drop of liquid evaporates by decrease in its
12. Two mercury drops (each of radius r) merge to form a
surface energy, so that its temperature remains
bigger drop. The surface energy of the bigger drop, if T is
unchanged. What should be the minimum radius of the
the surface tension is [AIEEE]
drop for this to be possible?
(a) 25/ 3 πr2T (b) 4 πr2T
The surface tension is T, density of liquid is ρ and L is its (c) 2πr T 2
(d) 28 / 3 πr2T
latent heat of vaporisation. [JEE Main]
(a) ρL /T (b) T /ρL 13. The terminal speed of a sphere of gold (density
(c) T /ρL (d) 2T /ρL = 19.5 kg m −3 ) is 0.2 ms −1 in a viscous liquid (density
= 1.5 kg m −3 ). Then, the terminal speed of a sphere of
7. A rain drop of radius 0.3 mm has a terminal velocity of
silver (density = 10.5 kg m −3 ) of the same size in the
1 ms −1 in air. The viscosity of air is 18 × 10−5 poise. The same liquid is [Kerala CEE]
viscous force on the drop will be [Karnataka CET] (a) 0.1 ms −1 (b) 1.133 ms −1
(a) 16.95 × 10− 9 N (b) 1.695 × 10− 9 N (c) 0.4 ms −1 (d) 0.2 ms −1
(c) 10.17 × 10− 9 N (d) 101.73 × 10− 9 N (e) 0.3 ms −1
588 Objective Physics Vol. 1

14. A large open tank has two holes in its wall. One is a 19. With an increase in temperature, surface tension of liquid
square hole of side a at a depth of x from the top and the (except molten copper and cadmium) [MHT CET]
other is a circular hole of radius r at a depth 4x from the (a) increases
top. When the tank is completely filled with water, the (b) remain same
quantities of water flowing out per second from both (c) decreases
holes are the same. Then, r is equal to [Kerala CEE] (d) first decreases then increases
a a 20. On the surface of the liquid in equilibrium, molecules of
(a) 2πa (b) a (c) (d)
2π π the liquid possess [MHT CET]
15. Ice pieces are floating in a beaker A containing water and (a) maximum potential energy
also in a beaker B containing miscible liquid of specific (b) minimum potential energy
gravity 1.2. When ice melts, then level of [Kerala CEE] (c) maximum kinetic energy
(d) minimum kinetic energy
(a) water increases in A
(b) water decreases in A
(c) liquid in B decreases
2010
(d) liquid in B increases 21. A ball is made of a material of density ρ, where
(e) water in A and liquid in B remains unaltered ρ oil < ρ < ρ water with ρ oil and ρ water representing the
densities of oil and water respectively. The oil and water
16. Eight equal drops of water are falling through air with a
are immiscible. If the above ball is in equilibrium in a
steady velocity of 10 cms −1 . If the drops combine to form
mixture of this oil and water, which of the following
a single drop big in size, then the terminal velocity of this pictures represents its equilibrium position? [AIEEE]
big drop is [Karnataka CET]
(a) 80 cms −1 (b) 30 cms −1 (c) 10 cms −1 (d) 40 cms −1
17. An object weighs m1 in a liquid of density d1 and that in Water Oil Water Oil
liquid of density d 2 is m2 . The density d of the object is
(a) (b) (c) (d)
[WB JEE]
m2d2 − m1d1 m1d1 − m2d2 Oil Water Oil Water
(a) d = (b) d =
m2 − m1 m2 − m1
m2d1 − m1d2 m1d2 − m2d1
(c) d = (d) d = 22. Three liquids of equal masses are taken in three identical
m1 − m2 m1 − m2
cubical vessels A , B and C. Their densities are ρ A , ρ B
18. A body floats in water with 40% of its volume outside and ρC respectively but ρ A < ρ B < ρC . The force exerted
water. When the same body floats in an oil, 60% of its by the liquid on the base of the cubical vessel is
[Karnataka CET]
volume remains outside oil. The relative density of oil is
[WB JEE] (a) maximum in vessel C (b) minimum in vessel C
(a) 0.9 (b) 1.0 (c) 1.2 (d) 1.5 (c) the same in all the vessels (d) maximum in vessel A
Answers
Level 1
Objective Problems
1. (a) 2. (a) 3. (b) 4. (a) 5. (b) 6. (a) 7. (a) 8. (c) 9. (c) 10. (b)
11. (b) 12. (c) 13. (a) 14. (a) 15. (a) 16. (d) 17. (c) 18. (c) 19. (b) 20. (b)
21. (b) 22. (a) 23. (b) 24. (a) 25. (b) 26. (b) 27. (c) 28. (c) 29. (a) 30. (d)
31. (b,c,d) 32. (b) 33. (a) 34. (a) 35. (c) 36. (b) 37. (b) 38. (b) 39. (a) 40. (a)
41. (b) 42. (a) 43. (c) 44. (a) 45. (c) 46. (a) 47. (a) 48. (c) 49. (b) 50. (a)
51. (b) 52. (b) 53. (b) 54. (d) 55. (c) 56. (c) 57. (c) 58. (c) 59. (b) 60. (b)
61. (d) 62. (d) 63. (d) 64. (c) 65. (a) 66. (a) 67. (a) 68. (d) 69. (c) 70. (a)
71. (c) 72. (c) 73. (a) 74. (c) 75. (d) 76. (b) 77. (b) 78. (b) 79. (a) 80. (b)
81. (b) 82. (a) 83. (d) 84. (c) 85. (b) 86. (a) 87. (a) 88. (a) 89. (c) 90. (c)
91. (b) 92. (d) 93. (d) 94. (a,d) 95. (c) 96. (b) 97. (b) 98. (d) 99. (b) 100. (b)
101. (c) 102. (b) 103. (c) 104. (b) 105. (c) 106. (c) 107. (c) 108. (b) 109. (d) 110. (a)
111. (b) 112. (a) 113. (a,d) 114. (b) 115. (c) 116. (d) 117. (a) 118. (b) 119. (d) 120. (b)
121. (b) 122. (a) 123. (d) 124. (b) 125. (c) 126. (d) 127. (a) 128. (b) 129. (a) 130. (a)
131. (b) 132. (d) 133. (a) 134. (b) 135. (d)

Level 2
Objective Problems
1. (b) 2. (c) 3. (c) 4. (a) 5. (d) 6. (b) 7. (b) 8. (a) 9. (d) 10. (c)
11. (b) 12. (d) 13. (b) 14. (c) 15. (a) 16. (b) 17. (d) 18. (d) 19. (b) 20. (c)
21. (a) 22. (c) 23. (d) 24. (c) 25. (c) 26. (d) 27. (b) 28. (c) 29. (a) 30. (b)
31. (c) 32. (b) 33. (a)

More than One Correct Options


1. (a,c) 2. (b,c) 3. (a,b,c) 4. (a,c,d) 5. (b,c,d) 6. (a,b,c) 7. (a,c) 8. (a,c,d) 9. (a,d) 10. (a,c,d)
11. (b,c)

Comprehension Based Questions


1. (b) 2. (d) 3. (d) 4. (a) 5. (b) 6. (c) 7. (c)

Assertion and Reason


1. (c) 2. (d) 3. (b) 4. (d) 5. (b) 6. (a) 7. (a) 8. (b) 9. (b) 10. (a)
11. (a) 12. (b) 13. (d) 14. (a) 15. (e) 16. (a) 17. (a) 18. (a) 19. (c) 20. (b)

Match the Columns


1. A→ q, B → r, C → r 2. A → q, B → p, C → r 3. A → r, B → q, C → p 4. A → p, B → p, C → p 5. A → q, B → p, C → q
6. A→ p, B → p, C → r

Entrance Gallery
1. (c) 2. (a) 3. (c) 4. (c) 5. (b) 6. (d) 7. (d) 8. (a) 9. (a,b,d) 10. (b)
11. (c) 12. (d) 13. (a) 14. (e) 15. (e) 16. (d) 17. (d) 18. (d) 19. (c) 20. (a)
21. (b) 22. (c)
Solutions
Level 1 : Objective Problems 11. Weight of sphere = upthrust from water + upthrust from
liquid
1. In a freely falling vessel upthrust is zero, because effective
Vρg =  V × 10 × g  +  × 8 × 103 × g 
2 3 V
value of g is zero. ∴
3  3 
2. (mraft + m)g = V raft ρw g
10000
∴ m = V raft ρw − mraft or ρ= kg/m3
3
120
= × 1000 − 120 = 80 kg 12. F = w air − w water or ∆w
600
3. 1 g/L = 1 kg/m3 = 5− 2= 3N
ρsolid 0.9
Upthrust = total downward force 13. Fraction immersed = = = 0.8.
ρliquid 1.125
∴ 1000 × 1.29 × g = (1000 × 009
. × g + mg )
∴ m =1200 kg Therefore, fraction outside water = 0.2.
4. Weight of man = extra upthrust 14. Specific gravity or relative density
weight in air 60
∴ mg = ( 3 × 2 × 10−2 )(10)3 (10) = = =3
change in weight in water 20
or m = 60 kg
upthrust − weight V ρw g − V ρg 15. Let x be the fraction of volume immersed in first case and y
5. a = = the fraction in second case.
mass Vρ
Then in first case, w = Upthrust
 ρ − ρ  1 − 0.5  (10) =10 m/s2
= w g =   ρ
 ρ   0.5  or Vρb g = ( xV )ρl g or x = b …(i)
ρl
6. Specific gravity or relative density
In second case,
weight in air 60
= = =3 Upthrust − weight = mass × acceleration
change in weight in water 20
or ( yV ) ρl ( g + g /3) − (Vρb )g = (Vρb )  
g
7. Weight = upthrust from mercury + upthrust from water  3
∴ 10 × 8.56 = ( x × 136
. ) + (10 − x ) × 10
. ρ
or y= b …(ii)
Here, x = depth inside mercury ρl
Solving we get x = 6cm
From Eqs. (i) and (ii), we see that x = y or fraction of volume
8. w app = w − upthrust
immersed does not change by the acceleration of vessel.
 w /g   σ 16. Upthrust on 7 cm = 600 kg
=w −   σg = w 1 − 
 ρ   ρ 600
∴Upthrust on further 1.4 cm = × 1.4 = 120 kg
9. 7
p0
17. Percentage volume outside the water
ρ − ρice 1000 − 900
= water × 100 = × 100 =10%
ρwater 1000
h1 h2
2
18. Upthrust = × 1 × 10 = 2 N
p p2 10
Reading of S1 will increase by 2N while that of S2 will
In the first case, decrease by 2 N.
Initially ( p − p)A = weight of block ρ ρ
19. f i = solid = s
or [( p0 + ρgh1 ) − p0 ] A = w ρliquid ρm
h1 =
w ρs
or …(i) ∴ hi = .l
Aρ g ρm
In the second case ∴ Height above the mercury level,
p2 A = w ρ  ρ 
= s l = 1 − s  l
or (ρgh2 )A = w ρm  ρm 
w
or h2 = …(ii) 20. 1 g /c = 1 kg /m3
ρAg
Upthrust = total downward force
From Eqs. (i) and (ii), we see that h1 = h2
v D ∴ 1000 × 1.29 × g = (1000 × 009
. × g + mg )
10. w = Upthrust or VDg =vdg or = . ∴ m =1200 kg
V d
Fluid Mechanics 591

21. 1 atmosphere = 1.01 × 105 N/m2 = ρgh 33. Velocity of efflux = 2gh, which is independent of density of
. × 10
101 5 liquid.
∴ h= = 7769 m ≈ 8 km
1.3 × 10 34. Difference in pressure energy = difference in kinetic energy
1
22. pD = pB ∴ ( p0 − p0 ) = ρv 2
2
∴ p0 + ρ1 gh1 = p0 + ρw gh2
ρ1 p0 105
or = relative density of paraffin = 2
h or v=2 =2 = 20 m/s or 400 m/s
ρw h1 ρ 103

23. Equating pressure at 1 and 2, we see that ρ1 = ρ2 . 35. Decrease in potential energy = increase in kinetic energy
1
∴ ρgh = ρ (v 2f − v i2 )
2
or . ) = v 2f − (10
2 (10)(015 . )2

2 or v f = 2 m/s
1 Now, from continuity equation,
1
A1v1 = A2v 2 or A ∝
v
dp
24. = − ρa (along horizontal towards right) i.e. in the Velocity has become two times. Hence, area of
dx cross-section will remain half.
direction of acceleration along horizontal pressure will 1 2
decrease of pAB > pCD . 37. ρv = ρgh ∴ v = 2gh = 2 × 10 × 0.1 = 2 m/s
2
dp
= − ρg (along vertically upwards direction pressure will H 3H 3H
dy 38. v = 2 H( H − h) = 2 × =
4 4 2
decrease .)
39. Rate of leakage of water from the hole
Hence, pressure at B is maximum and pressure at D is = Av = A 2gh = 10−4 2 × 10 × 5 = 10−3 m 3/s
minimum.
40. Rh = RH −h .
25. F = p × (ρgh) (Area of base)
= 900 × 10 × 0.4 × 2 × 10−3 = 7.2 N Since, range from both the holes is same
−4 h = 2.4 cm and H − h = 76
. cm
26. F = p × A = (ρghtotal )A = (10 × 10 × 10
3
. )(100 × 10 ) =100 N
Adding these two, we get H =10cm.
27. In horizontal direction, pressure remains constant unless it 1
is accelerated. 41. From continuity equation, v ∝
A
dp 1
28. When moving upwards = − ρ( g + a ) and when moving or v∝ 2 ( d = diameter of pipe)
dh d
dp
downwards = − ρ( g − a ) 1 1
42. p1 + ρv12 = p2 + ρv 22
dh 2 2
In first case pressure decreases with h more rapidly. −2 1
or (10 × 136
. × 103 × 10) + × 103 × (0.35)2
2
1
= p2 + × 10 × (065
3
. )2
2
1210 × 102
or p2 = 1210 N/m2 = cm of Hg
h 136. × 103 × 10
= 0.89 cm of Hg
29. Pressure at boundary of two liquids will be same. 43. A1v1 = A2v 2 + A3v 3
∴ p + ρg ( 3h) = p + 2ρgh A v − A2v 2 (0.2 × 4) − (0.2 × 2)
∴ v3 = 1 1 = =1 m/s
∴ p = p0 − ρgh or p < p0 A3 0.4
F1 F2
30. = (Pascal’s law) 44. Decrease in pressure energy = increase in kinetic energy
A1 A2 1
or ∆p = ρv 2
2000 × 98
. A1 = 
F2 .  2
∴ A2 =  (100) = 1.96 × 10 cm
4 2
F1  10 2 
2( ∆p) 2 × 0.5 × 105
∆p ∴ v= = =10 m/s
31. = ρa = ρg (in horizontal direction) ρ 103
∆x 1
Pressure decreases in the direction of acceleration in 45. Pressure difference ∆p = ∆KE = × ρ(v12 − v 22 )
2
horizontal.
1
∆p or ∆p = × 10 . × ( 3600 − 1600) =1000 N /m2
= ρg (in vertical direction) 2
∆x
F = (∆p) × A = 4000 N
Pressure increases with depth in vertical direction.
592 Objective Physics Vol. 1

46. p2 = p3 and p1 > p2 55. Velocity becomes constant when terminal velocity is
attained.
F
56. F = 6πηrv or units of η are similar to that of N-s/m2
rv
F mg
57. F = 6πηrv , i.e., v has the dimensions of or .
ηr ηr
58. Let r be the radius of smaller drop and R the radius of bigger
v drop. By equating the volumes, we have
1
2  πr 3  = πR 3
2 3 4 4
3  3
p1 = p0 + ρgx
p2 = p0 + ρgy or R = ( 2)1/ 3 . r
∴ ∆p = p1 − p2 = ρg ( x − y ) Now, terminal velocity ∝(radius)2
Between 1 and 2, ∴ v ′ = ( 2)2/ 3 v
Difference in pressure energy = difference in kinetic energy
61. V = V1 + V 2
1
or ρg ( x − y ) = ρv 2
2 πpr 4 pr14 πpr24
⇒ = +
or v = 2g( x − y ) 8ηl 8ηl 8

47. Rh = RH −h or h1 = H − h2 or h1 + h2 = H ⇒ r 4 = r14 + r24

H ∴ r = (r14 + r24 )1/ 4


48. At h = , maximum range is obtained, which is equal to H. r1 1
2 62. Given, l1 = l2 = 1m and =
Further Rh = RH − h . r2 2
15 × 10−3 πp1r14 πp2r24
49. Q = A1v1 = A2v 2 = 15 L/min = = 2. 5 × 10−4 m3 /s V = =
60 8ηl 8ηl
2.5 × 10−4 p1  r2 
v1 = = 1.25 m/s ⇒ =   = 16
2 × 10−4 p2  r1 
2.5 × 10−4 ⇒ p1 = 16p2
v2 = = 5.0 m/s
0.5 × 10−4 Since, both tubes are connected in series, hence pressure
Now, difference in pressure = difference in kinetic energy difference across combination,
1 p 16
ρw gh = ρa (v 22 − v12 ) p = p1 + p2 ⇒ p1 = 1 ⇒ p1 = = 0.94 m
2 16 17
ρa (v 22 − v12 ) πpr 4
∴ h= = 1.5 × 10−3 m ≈ 1.5 mm 63. V =
2ρw g 8ηl
1 ∴ V ∝ pr 4 ( η and l are constants)
or hρw g = ρair (v 22 − v12 ) 4
V 2  p2   r2  4
2 1 1
∴ =    = 2×   =
ρair 1.32 V1  p1   r1   2
or h= (v 22 − v12 ) = [( 5)2 − (1.25)2 ] 8
2ρw g 2 × 103 × 10 Q
∴ V2 =
= 1.5 × 10−3 m ≈ 1.5 mm 8
8ηl
50. R = 2 htop × hbottom = 2 h ( H − h) 65. Fluid resistance is given by R = .
πr 4
H 3H 3H
51. v = 2 H( H − h) = 2 × = When two capillary tubes of same size are joined in parallel,
3 4 2 then equivalent fluid resistance is
A 2H 8ηl 8η × 2l  8ηl  9
52. Time to empty a tank, t = Re = R1 + R2 = + = ×
a g πr 4 π( 2R )4  πr 4  8
or t∝ H Equivalent resistance becomes 9/8 times so rate of flow will
be 8/9 X.
53. Level in the container will become maximum when rate of
67. When cross-section of duct is decreased, the velocity of
inflow = rate of outflow. Or, water increased in accordance with Bernoulli’s theorem,
Q = A1v = A1 2ghmax the pressure p decreased at that place.
Q2 70. Suppose, R be the radius of bigger drop. Then, by equating
∴ hmax = volumes, we have,
2gA12
2( 4/3 πr 3 ) = ( 4/3) πR 3 or R = ( 2)1/ 3 r
54. v T ∝ r 2
2
Now, surface energy ∝ surface area
(v T )1  R  2r 2
∴ =  =4 ∴
U1 A1
=
2
= 2 = 2/ 3
(v T )2  R/2  U2 A2 R 2
Fluid Mechanics 593

73. p1 < p2 for θ < 90° 2T 2 × 75


109. h = ⇒h = = 30 cm
2 rdg .
0005 × 1 × 103
4T
1 110. ∆p =
r
1
∴ ∆p ∝
r
1
As radius of soap bubble increases with time.∴ ∆p ∝
t
74. ∆p1 = 001
. atm 111. Tc = T0 (1 − at ) i.e. surface tension decreases linearly with
increase in temperature.
∆p1 = 0.01atm
112. B will keep its area as minimum as possible.
4T
∆p = or ∆p ∝1/R
R
R1 ∆p2 2
∴ = = A
R2 ∆p1 1
3
V1  R1  8
=  =
V 2  R2  1
∆W 115. h1r1 = h2r2
75. Surface tension T = F /l and , i.e. unit of surface tension
∆A
2T cosθ 1
N J
is or 2. . 116. h = ∴ h ∝ . If θ is less, then h will be more.
m m rdg r
N kg -m/s2 kg 2T cos θ
= = 2 117. h = . If θ is less than 90°, then h will be positive.
m m s rdg
79. Weight of spiders or insects can be balanced by vertical 118. Let the width of each plate is b and due to surface tension
component of force due to surface tension. liquid will rise up to height h, then upward force due to
82. Force on each side = 2TL (due to two surface). surface tension = 2Tb cosθ …(i)
84. This happens due to viscosity. x
b
86. Due to force of attraction it is not easier to separate the two
glass plates.
h
87. Soluble impurities increases the surface tension.
F 2 × 10−2
88. T = = = 01
. N/m
2l 2 × 10 × 10−2
F [MLT −2 ]
89. T = = = [ML0 T −2 ] Weight of the liquid rises in between the plates
l [L]
= Vdg = ( bxh)dg …(ii)
90. W = 8 πR T2
Equating Eqs. (i) and (ii), we get
∴ W ∝ R2 2Tb cosθ = bxhdg
If the radius becomes double, then work done will become 2T cosθ
∴ h=
four times. xdg
91. Surface energy of combined drop will be lowered, so excess 119. 6 × 10−2 × circumference = Force
surface energy will raise the temperature of the drop.
75 × 104
92. W = 8 πR 2T = 8 × π × (10−2 )2 × 2 × 10−2 = 16 π × 10−6 J ∴ Circumference = = 12.5 × 102 m
6 × 102
rr
95. r = 1 2 = ∞ 1 rP hQ h 3
r2 − r1 120. r ∝ ⇒ = = =
h rQ hP 2 h 2
97. Cohesive force decreases. So, angle of contact decreases. 3
101. Angle of contact is acute. 2T cosθ
1 124. h =
103. Since, ∆p ∝ rdg
R
1
4T 4 × 2 × 25 × 10−3 ∴ h∝
104. Express pressure ∆p = = r
r 1 × 10−2
So, the graph between h and r will be rectangular hyperbola.
= 20 N/m2 = 20 Pa (as r = d /2) 125. Initially due to more pressure at the hole velocity of efflux
2T 2T will be more. So, the liquid drains out more quickly.
107. hρg = ⇒h =
r r ρg 126. Volume of ice > Volume of water
rr 5× 4 m m
108. r = 1 2 = = 20cm ∆V = Vw − V i = −
r1 − r2 5 − 4 y x
594 Objective Physics Vol. 1

127. Velocity of ball just before striking water surface v = 2gh


Level 2 : Only One Correct Option
or v = 4g m/s (h = 2m)
1. 3g = 4 ( g − a )
Upthrust − Weight
Retardation in water a = ∴ g−a=
3g
Mass 4
V ρw g − V ρg 3  3g 
or a= Upthrust = (5 × 10 ) (10 )   N = 0.375 kg
-4
Vρ  4
1 − 0.8 
=   g = g/4 Apparent weight = (3 − 0.375) = 2.625 kg
 0.8  2. From work-energy theorem work done by all the forces
Now, depth to which ball will sink is = change in kinetic energy.
v2 4g ∴(Work done by gravity for 7 m) + (work done by upthrust
d= = = 8m for 2 m) + (work done by resistive forces) = 0 as ∆ KE = 0
2a 2g /4
 1 
128. When hole is made at centre of vessel, maximum range is ∴ (1)(10)( 7)cos0° +   (ρw g )
obtained.  2/3 × ρw 
ρ V + ρ2V ( 2)cos 80° + work done by resistive forces = 0
129. 4 = 1 or ρ1 + ρ2 = 8 …(i)
2V ∴Work done by resistive forces = − 40 J
m+ m 3. w = Upthrust
3=
(m/ρ1 ) + (m/ρ2 ) or (m1 + m 2 ) g = [(m1 /0.5 × ρw ) + (m2 /2. 5 × ρw )]ρw g
ρ1ρ2 3 or m1 + m 2 = 2m1 + 0.4 m 2
or = …(ii)
ρ1 + ρ2 2 or m1 = 06 . m2
m1 3
Solving these two equations, we get ρ1 = 6 and ρ2 = 2 ∴ = 06
. =
130. Loss of weight = Upthrust m2 5
W 4. Increase in pressure
= total volume × ρw × g
2 Force exerted by the cylinder on the liquid Upthrust
= =
(V − VC ) 5 × ρw × g A A
or = V × ρw × g (Q VC = volume of cavity)
2 5. Common height after they are connected can be
2 determined by equating the volumes. Hence,
or V − VC = V
5 ( A1 + A2 ) h = A1h1 + A2h2 ( A1 = A2 = A )
VC = V
3 h1 + h 2
5 ∴ h=
2
131. When iron block was floating, it displaces V1 volume of Work done by gravity = − ∆U = Ui − U f
water such that weight of this V1 volume of water is equal to (h + h 2 )
 h h 
weight of block. But since density of iron is more than = Ah1 ρ. g. 1 + Ah 2ρ. g. 2  − A (h1 + h2 ) ρ . g. 1
density of water, this volume V1 will be greater than volume  2 2 4
V 2 of iron block. ρAg
= (h 2 − h1 )2
When iron block sinks in the pond, it displaces water of 4
volume equal to its own volume V 2 . 6. 2a1 S1 = 2a2 S2
Since, V1 > V 2 , displaced volume in first case was more than Here, a1 = acceleration inside the liquid
displaced volume in second case. Hence, level will fall. upthrust − weight Vσg − Vρg  σ 
132. Force exerted by gas on its hemispherical end = Pressure of = = =  − 1 g
mass ρ ρ 
gas × projected area = p0 ( πr 2 ).
a2 = retardation in air = g
133. Upthrust on sphere from the liquid makes equal and a σ 
opposite pair of forces. Hence, there will be no effect on the ∴ S 2 = 1 . S1 =  − 1 g
a2 ρ 
pressure at bottom of vessel. Or p = p0 + ρgh
W 7. Writing pressure equation from one end to other end of
134. W = Upthrust = V i ρlg or V i = tube
ρlg
l l
p0 + 2ρgh − 2ρ . . a − ρ . a − ρgh = p0
If density of liquid ρl is increased, then immersed volume V i 2 2
will decrease or the ball will go up. 3al
∴ h=
135. Pressure inside the bubble 2g
2T
= Pressure outside it + 8. Work done by all the forces = change in kinetic energy.
r
2T Two forces are acting, weight and upthrust.
= ( p0 + ρgh) +
r 9. p = p 0 + ρ1 gh1 + ρ2 gh 2
2 × 7 × 10−2 or p − p0 = ρ1 gh1 + ρ2 gh2
= 105 + (103 × 10 × 10) +
10−3 = (600)(10)(0.1) + (1000)(10)(0.02)
= 2 .0014 × 10 N /m
5 2
= 800 N/m2
Fluid Mechanics 595

10. Weight = upthrust 14. (11.2)(1) = (136


. )( 2x )

x
r x Original
level
R

 4 πR 3 − 4 πr 3  (8 × ρ × g ) =  4 πR 3  ρ g
    w ∴ nx = 0.41 cm
3 3  w
3 
15. v = 2gh
or 8R 3 − 8r 3 = R 3
Vσg − Vρg  σ − ρ 
r  7
1/ 3
73 Retardation in liquid a = = g
∴ 8r = 7R
3 3
or =  = V  ρ 
R  8 2
v2 2gh ρh
11. In equilibrium,weight = upthrust i.e. Vρi g = V i ρw g ∴ d max = = =
2a 2 = ρ h σ −ρ
π −ρ
l
dp
16. = ρa
dX
∴ ∆pAB = ρaL = ρgh h
a
La
∴ h=
V i ρi 0.9 g
or = = = 0.9
V ρw 10 . A B
2T
i.e. 90% or 9 m of ice block is inside the water. Therefore, 17. pA = pC = p0 and pB = p0 −
minimum length l required to scoop out a bucket of water is R
10 − 9 = 10
. m 18. Weight − upthrust = drag force.
1 2 2( ∆p)
12. ∆p = ρv or v = But no information is given regarding the upthrust.
2 ρ
19. Fraction of length will remain unchanged.
yi 1
p1 – p2 = ∆p = contact = in the given question.
y 2

1 2 ∴ If total melting rate is 2 cm/h, then top of candle will fall at


the rate of 1 cm/h.
20. Let us write pressure equation in path ABCDE.

R = vt or R ∝ ∆p A
E
To make R two times ∆p should be 4 times. Initially
∆p = ρw gh = 103 × 10 × 10 = 105 N/m2 = 1 atm h
In second case, ρw gh + extra pressure = 4 ( ∆p) = 4 atm
or 1 atm + extra pressure = 4 atm
h
∴ Extra pressure = 3 atm.
D
13. About hinge clockwise torque of hinge should be equal to
anti-clockwise torque of upthrust.Immersed length will be 2I
0.5 sec θ. p0 − + ρ2 g (h + h′ ) − ρ2 gh′ − ρ1 gh = p0
r
r
F ∴ T = (ρ2 − ρ1 ) gh
2
θ  1 
21. F = (6 πηrv ) = 6 π   (1)(v ) = v
 6π 
F dv
∴Retardation a = − = − v or = −v
m dt
w t 0 .5 dv
∫ 0 dt = − ∫ 2 v
∴ w =  sin θ = F
1  0.5 sec θ  sin θ t = ln ( 4) s
 
2   2  2T cosθ 1
22. h = or h ∝
or w = 0.5 F sec θ rρ g r
or (1 × A × 0.5 × ρw × g ) = 0.5 (0.5 sec θ) × A × 10
. × ρw × g × sec θ M = ( πr 2h) ρ or m ∝ r 2h
sec 2 θ = 2 or θ = 45° or M ∝r
596 Objective Physics Vol. 1

23. Upthrust F = w + T 31. We can assume total height of liquid to be equal to 3H.
Maximum range is obtained when hole is drilled at centre.
F
h = 1.5H
2H

Platform
H

w T R
32. Relative density of steel
T = F − w = V ρw g − V ρb g = (ρw − ρb ) Vg
weight in air w
=
= (1000 − 800) 
8  =
 (10) = 20 N change in weight in water w − w1
 800 
Now, change in weight in the given liquid
24. Upthrust = (volume of metal ball) × (density of liquid) × g = upthrust in this liquid
with increase in temperature volume of ball will increase
 w /g 
and density of liquid will decrease. or w − w2 =  d 2 × ρw × g
 (w /w − w 1 w 

But coefficient of cubical expansion of liquid is more.
w − w2
Hence, second effect is more dominating. Therefore, ∴ d2 = = relative density of given liquid.
upthrust at higher temperatures will be less or apparent w − w1
weight will be more. 33. Weight of combined cylinder = upthrust
25. p1V1 = p2V 2
d1
( 75 + h) V = ( 75)( 3V )
∴ h =150 cm of Hg =1.5 m of Hg or 15 m of water. L
2
26. Initially p0 A supports the weight of mercury column. While d
in second case it provides acceleration too.

d1 LAg + d 2 LAg = d   Ag
3L

 2
h
3
or d1 + d 2 = d
2
But d1 < d 2
3
p 0A d1 < d 2
4
27. Let r be the radius of smaller drops and R the radius of
bigger drop. Then, by equating volume we have More than One Correct Options
4 4 R
1000 ( πr 3 ) = πR 3 or r = ∆v 2m /s
3 3 10 1. Velocity gradient = = = 2 s− 1
∆h 1 m
Surface energy ∝ surface area Si = 4 πR 2
∆v
F =ηA = (10− 3 )(10)( 2) = 0.02 N
or Sf =1000 ( 4 πr 2 ) =10 ( 4 πR 2 ) =10 Si ∆h
As surface area has increased 10 times, surface energy will 2. For contact angle θ = 90° , liquid neither rises nor falls.
also become ten times. 3. Restoring force = − (ρAg ) x or F ∝− x
28. From conservation of mechanical energy. This is just like a spring-block system of force constant
29. A1v1 = A2v 2 K = ρAg
or A1 = 2gh 2 = A2 2gh 4. From continuity equation, Av = constant
or A 21h1 = A 22h 2 v 2 > v 2 as A2 < A1
From Bernoulli’s equation,
( L2 )2 ( y ) = ( πR 2 )2 ( 4y )
1
L p + ρv 2 = constant (as h = constant)
or R= 2
2π p2 < p1 as v 2 > v 2
F
30. ∆p = . Difference in pressure energy is equal to difference 5. Fraction of volume immersed,
A ρ
in kinetic energy. f = s
ρl
F 1 2 2F
∴ = ρv or v = This fraction is independent of atmospheric pressure. With
A 2 ρA increase in temperature ρs and ρl both will decrease.
Fluid Mechanics 597

2hB 6. Applying Bernoulli’s equation just inside and just outside


6. v = 2ghT : t = , R = 2 hT hB
g the hole,
p0 +   ( d ) g +  − H  ( 2d )( g ) = ( 2d ) v 2 + p0
Here, hT = distance of hole from top surface of liquid H H 1
and hB = distance of hole from bottom surface.  2 2  2
7. Pressure increase with depth in vertical direction and in g
∴ v= ( 3H − 4h)
horizontal direction it increases in opposite direction of 2
acceleration based on this concept pressure is maximum at 2h
point D and minimum at B. 7. t =
g
8. In air, a1 = g (downwards)
Upthrust − Weight (V )( 2ρ)( g ) − (V )(ρ)( g ) ∴ x = vt = ( 3H − 4H ) h
In liquid, a2 = =
Mass (Vρg )
= g (upwards)
Assertion and Reason
1. Reason is correct due to continuity equation.
∴ a1 ≠ a2
9. Initially 2. Upthrust will remain same.
dV1 4. Fluid at 1 is moving while fluid at 2 is stationary. Therefore,
= v1 a1 = ( 2gh )( π )( 2R )2 …(i) according to Bernoulli’s theorem, p1 < p2 . Here p2 = p0
dt
Hence, p1 < p0 .
dV 2
= v 2 a2 = ( 2g (16h))( π )( 2R )2 …(ii) 10. Density of water at 4°C is maximum.
dt
13. At same level of same liquid pressures may be different, if at
dV dV 2
From Eqs. (i) and (ii), we can see that 1 = one point speed is zero and at other point speed is
dt dt non-zero.
After some time v1 and v 2 both will decrease, but decrease in 14. Fraction of volume immersed,
ρ
the value of v1 is more dominating. So, f = s
dV1 dV 2 ρl
a1v1 or < a2 v 2 or
dt dt If ρl is decreased, then f will increase.
10. Fraction of volume immersed is given by 15. Capillary rises when θ = 0°
ρ
f = s
ρl
ρs and ρe are same. Hence,
f1 = f 2 = f 3
11. Base area in third case is uniform. Hence, h3 is minimum.
Streamline flow is more likely for liquids having low density.
We know that greater the coefficient of viscosity of a liquid θ = 0° θ ≠ 0°
more will be velocity gradient hence each line of flow can be
easily differentiated. Also higher the coefficient of viscosity 18. With increase in depth pressure increases. Therefore,
lower will be Reynolds number, hence flow more like to be volume of given mass of liquid decreases. Hence, density
increases.
streamline.
19. In floating condition, mg = V1ρ1 g
Comprehension Based Questions (V1 = volume immersed or volume displaced)
m
1. Not required. ∴ V1 =
ρ1
2. x 3 = 2 h top × hbottom = 2 3a × a = 2 3a
When the solid melts, volume of liquid formed,
3. x1 = 2 a × 3a = 2 3a m
V2 =
x 2 = 2 2a × 2a = 2 4a ρ2
x 3 = 2 3a Since ρ1 < ρ2
∴ V 2 > V1
∴ x1 = x 3 < x 2
4. Weight = upthrust 20. Equate net downward force and net upward force on block
+ liquid system.
∴  L Dg  = × × 2d × g +
A L A 3L A
× ×d×g
 5  4 5 4 5
5d
Match the Columns
∴ D= 1. Pressure at O is the atmospheric pressure which will remain
4
5. pA = p0 A + weight of two liquids + weight of cylinder same.
2h
= p0 A +   ( A )( d )( g ) +   ( A )( 2d )( g ) + L   
H H A 5d  2. v = 2gh, t = and R = 2 h ( H − h)
g
 2  2  5  4  g
( L + 6H ) At mountain, value of g will be less. Hence, v will decrease, t
∴ p = p0 + dg
4 will increase and R will remain unchanged.
598 Objective Physics Vol. 1

1
3. From, A1v1 = A2v 2 or v ∝ Fact
A I. For pure water flowing in a cylindrical pipe, Re is
At a point where area of cross-section is less, volume of about 1000. When 0 < Re < 2000, the flow of liquid is
liquid flowing per second is same but speed is more. streamlined.
Therefore, here kinetic energy is more or the pressure will II. When 2000 < Re < 3000, the flow of liquid is variable
be less. between streamlined and turbulent.
4. At depth h p = p0 + ρ( g + a ) h III. When K > 3000, the flow of liquid is turbulent.
∆p = ρ( g + a ) ∆h 4. Let, thickness of layer be x.
Upthrust will also increase, because upthrust So, volume V = area × x
F = V i ( g + a )ρ V = A × x ⇒ x = V /A (Qx = 2r )
V V
Here, V1 is the immersed volume. ∴ 2r = ⇒ r= …(i)
A 2A
5. When two soap bubbles coalesce to from a larger drop,
4T T
radius increases. Hence, the excess pressure will and ∆p =
R r
decrease. Further, surface area will decrease. Hence, the T
We know that, F = ∆p × A = × A
surface energy will decrease or the temperature will r
increase. T
F= ×A [from Eq. (i)]
6. In equilibrium upthrust = weight V 
 
If density of liquid is decreasing, more volume should be  2A 
immersed in the liquid, so that upthrust remains F ×V
unchanged and it balances the weight. T=
2A 2
Entrance Gallery where, F = 16 × 105 dyne, V = 0.04 cm3
16 × 105 × 0.04 8 × 105 × 4 8 × 105 × 4
1. Equating pressure at A, we get A = 20 cm2 ,T = = =
2 × 202 202 × 100 400 × 100
= 8 × 105 × 10− 4
R = 80 dyne/cm
α R sin α
R = 80 dyne cm− 1
90°– α R d2
5. According to Torricelli’s law,
α
For an open container, the velocity of efflux is given by,
d1 v = 2gh
A
6. Decrease in surface energy = Heat required in vaporisation
R sin α d 2 + R cosα d 2 + R(1 − cos α ) d1 = R (1 − sin α ) d1
∴ T ( dS) = L( dm)
d1 1 + tan α
(sin α + cos α ) d 2 = d1 (cos α − sin α ) ⇒ = ∴ T ( 2)( 4 πr )dr = L( 4 πr 2 dr ) ρ
d 2 1 − tan α
2. The bubble will detach, if x = V /A
buoyant force ≥ surface tension force 2T
r=
4 ρL
πR 3ρw g ≥ ∫ T × dl sin θ
3 7. The formula for viscous force is
θR
F = 6πηrv
(ρw )  πR 3  g ≥ (T )( 2 πr )sin θ
4 r
3  Given, viscosity
θ
⇒ sin θ =
r η = 18 × 10− 5 poise
R
= 18 × 10− 6 kg m − 1s− 1
2ρw R 4 g
Solving, r= Radius, r = 0.3 mm = 0.3 × 10− 3 m
3T
Velocity, v = 1 ms− 1
2ρw g
⇒ r=R 2
So, F = 6 × 3.14 × 18 × 10− 6 × 0.3 × 10− 3 × 1 = 101.73 × 10− 9 N
3T
3. Reynold’s number is a pure number and it is equal to the 8. As volume remain constant, therefore R = n1/ 3r .
ratio of the inertial force per unit area to the viscous force Energy of one small drop E ( 4 πr 2 )T
per unit area for a flowing fluid. = =
Energy of one big drop E1 ( 4 πR 2 )T
v ⋅ ρ⋅ r
Reynold’s number, Re = c E r2 r2 1
η ⇒ = 2 = 1/ 3r 2 = 2/ 3
E1 R (n ) n
where, ρ = density of the liquid
v c = critical velocity Given, n = 1000
E 1 1
η = coefficient of viscosity of liquid So, = =
E1 (1000)2/ 3 100
r = radius of capillary tube.
Fluid Mechanics 599

9. F = upthrust = Vd F g 14. We have, v 2 = ρgh ⇒ a 2 ρgh = πr 2 ρgh × 2 ⇒ r =


a
Equilibrium of A, Vd F g = T + w A = T + Vd A g …(i) F 2π
Equilibrium of B, Vd B g = T + Vd F g …(ii) 15. If we have m gram of ice, which is floating in a liquid of
A wA m
Adding Eqs. (i) and (ii), we get density 1.2 and 9 L will displace volume cc < mcc . After
1.2
2d F = d A + d B T
melting it occupies mcc .
∴ Option (d) is correct 16. We know that,
From Eq. (i), we can see that, T
v ∝ r 2 and 8  πr13  = πr23
F 4 4
d F > dA (as, T > 0) B 3  3
∴Option (a) is correct. wB
v1 r12 10 r2 1
From Eq. (ii), we can see that d B > d F ∴ = ⇒ = 2/ 3 2 = ⇒ v 2 = 40 cm/s
v 2 r22 v2 8 r 4
∴Option (b) is correct.
17. V ( d − d1 )g = m1 g
∴Correct options are (a), (b) and (d).
ρ − ρ0 ρ − ρ02 η1 d − d1 m1
10. We know that, v ∝ ⇒ 2 =
v
× and V ( d − d 2 )g = m2 g ⇒ =
η v1 ρ − ρ01 η 2 d − d 2 m2

7.8 − 1.2 8.5 × 10− 4 × 10 m1 d 2 − m2 d1


or v2 = × = 6.25 × 10− 4 cm/s ⇒ d=
7.8 − 1 13.2 m1 − m2
11. From Bernoulli’s theorem, 18. Vσ g = 0.6 Vσ1 g and Vσg = 0.4 Vσ 2 g
 v  2  6 σ1 σ2 3
1 1 ∴ 1= ⇒ = =1.5
ρgh = ρ(v 22 − v12 ) ⇒ gh = v12   2  − 1 4 σ2 σ1 2
2 2   v1  

19. The surface tension of liquid decreases with rise of
1  A  2  temperature. The surface tension of liquid is zero at its
⇒ gh = v12   1  − 1 (QA1v1 = A2v 2 ) boiling point and it vanishes at critical temperature. At
2   A2  
 critical temperature intermolecular forces for liquid and
A 
2
D 
4 gases becomes equal and liquid can expand without any
2hg 2gh
⇒  1  =1 + 2 ⇒  1  =1 + 2 restriction. For small temperature differences, the variation
 A2  v1  D2  v1 in surface tension with temperature is linear and is given by
D1 8 × 10−3 relation
⇒ D2 = = = 3.6 × 10−3 m
 2gh 
1/ 4
 2 × 10 × 0.2 
1/ 4 Tt = T0 (1 − αt )
1 + 2  1 + (0.4)2  where, Tt and T0 are the surface tensions at t°C and 0°C,
 v1  
respectively and α is the temperature coefficient of surface
12. Let R be the radius of the bigger drop, then volume of bigger tension.
drop = 2 × volume of small drop
20. On the surface of the liquid in equilibrium molecules of the
4 4
πR 3 = 2 × πr 3 ⇒ R = 21/ 3 r liquid possess maximum potential energy.
3 3
21. ρoil < ρ < ρwater . Oil is the least denser of them, so it should
Surface energy of bigger drop,
settle at the top with water at the base. Now, the ball is
E = 4 πR 2T ⇒ E = 4 × 22/ 3 πr 2T = 28/ 3 πr 2T denser than oil but less denser than water. So, it will sink
2 ( ρ − σ )r 2 g through oil but will not sink in water.
13. Velocity, v =
9 η So, it will stay at the oil water interface.
v1 ( ρ1 − σ ) 22. Force exerted by the liquid on the base of the vessel is
∴ v ∝(ρ − σ) ⇒ = F = mg .
v 2 ( ρ2 − σ )
0.2 19.5 − 1.5 Here, mA = mB = mC
⇒ = or v 2 = 0.1 m/s
v 2 10.5 − 1.5 ∴ FA = FB = FC
14
Thermometry, Thermal
Expansion and Kinetic
Theory of Gases

14.1 Thermometers and the Celsius


Temperature Scale
Thermometers are those devices which are used to measure temperatures. All Chapter Snapshot
thermometers are based on the principle that some physical properties of a system change ● Thermometers and the
as the system’s temperature changes. Some physical properties that change with Celsius Temperature
temperature are: Scale
1. the volume of a liquid, ● The Constant Volume
2. the length of a solid, Gas Thermometer and
3. the pressure of a gas at constant volume, the Absolute
Temperature Scale
4. the volume of a gas at constant pressure and ● Quantity of Heat
5. the electric resistance of a conductor. ● Thermal Expansion
A common thermometer in everyday use consists of a mass of liquid, usually mercury ● Concept of an Ideal Gas
or alcohol that expands in a glass capillary tube when heated. In this case, the physical ● Gas Laws
property is the change in volume of the liquid. Any temperature change is proportional to ● Ideal Gas Equation
the change in length of the liquid column. ● Degree of Freedom (f )
The thermometer can be calibrated accordingly. On the celsius temperature scale, a ● Internal Energy of an
thermometer is usually calibrated between 0°C (called the ice point of water) and 100°C Ideal Gas
(called the steam point of water). ● Law of Equipartition of
Once the liquid levels in the thermometer have been established at these two points, Energy
the distance between the two points is divided into 100 equal segments to create the celsius ● Molar Heat Capacity
scale. Thus, each segment denotes a change in temperature of one celsius degree (1°C). A ● Kinetic Theory of Gases
practical problem in this type of thermometer is that readings may vary for two different
liquids. When one thermometer reads a temperature, for example 40°C the other may
indicate a slightly different value.
These discrepancies between thermometers are especially large at temperatures far
from the calibration points. To surmount this problem, we need a universal thermometer
whose readings are independent of the substance used in it. The gas thermometer used in
the next article meets this requirement.
Thermometry, Thermal Expansion and Kinetic Theory of Gases 601

14.2 The Constant Volume This triple point occurs at a temperature of


approximately 0.01° C and a pressure of 4.58 mm of mercury.
Gas Thermometer and On the new scale, which uses the unit kelvin, the
The Absolute temperature of water at the triple point was set at 273.16 K,
Temperature Scale abbreviated as 273.16 K. (No degree sign is used with the
unit kelvin).
The physical property used by the constant volume gas This new absolute temperature scale (also called the
thermometer is the change in pressure of a gas at constant kelvin scale) employs the SI unit of absolute temperature,
volume. 1
p the kelvin which is defined to be, of the difference
273.16
between absolute zero and the temperature of the triple point
of water.

The Celsius, Fahrenheit and Kelvin


Temperature Scales
0°C 100°C t (°C)
Eq. (i) shows the relation between the temperatures in
Fig. 14.1
celsius scale and kelvin scale. Because the size of a degree is
The pressure versus temperature graph for a typical gas the same on the two scales, a temperature difference of 10°C
taken with a constant volume is shown in above figure. The is equal to a temperature difference of 10 K. The two scales
two dots represent the two reference temperatures namely, differ only in the choice of the zero point. The ice point
the ice and steam points of water. The line connecting them temperature on the kelvin scale, 273.15 K, corresponds to
serves as a calibration curve for unknown temperatures. 0.00°C and the kelvin steam point 373.15 K, is equivalent to
Experiment shows that the thermometer readings are nearly 100.00°C.
independent of the type of gas used as long as the gas
A common temperature scale in everyday use in US is the
pressure is low and the temperature is well above the point at
fahrenheit scale. The ice point in this scale is 32°F and the
which the gas liquefied.
steam point is 212°F. The distance between these two points
If you extend the curves shown in figure toward are divided into 180 equal parts. The relation between celsius
negative temperatures, you find, in every case, that the scale and fahrenheit scale is as derived below:
pressure is zero when the temperature is –273.15° C. This 0°C 100°C
significant temperature is used as the basis for the absolute (100 equal parts)
temperature scale, which sets –273.15° C as its zero point.
This temperature is often referred to as absolute zero. (180 equal parts)
The size of a degree on the absolute temperature scale is 32°F 212°F
identical to the size of a degree on the celsius scale. Thus, Fig. 14.3
the conversion between these temperatures is 100 parts of celsius scale =180 parts of fahrenheit scale
TC = T – 273.15 …(i) 9
P Gas 1 ∴ 1 part of celsius scale = parts of fahrenheit scale
5
Gas 2 K C F
373 100° 212°
Gas 3

100 K 100°C 180°F

–273.15°C 0°C t (°C)


273 0°C 32°F
Fig. 14.2 Relation among Kelvin, Celsius and Fahrenheit temperature scales
In 1954, by the International committee on weights and Fig. 14.4
measures, the triple point of water was chosen as the 9
reference temperature for this new scale. The triple point of Hence, TF = 32 + TC …(ii)
water is the single combination of temperature and pressure 5
5
at which liquid water, gaseous water and ice (solid water) Further, ∆TC = ∆T = ∆TF …(iii)
coexist in equilibrium. 9
602 Objective Physics Vol. 1

X Example 14.1 Express a temperature of 60°F in ■ Two other thermometers, commonly used are
degree celsius and in kelvin. thermocouple thermometer and total radiation
pyrometer.
Sol. Substituting, TF = 60° F in Eq. (ii) ■ Total radiation pyrometer is used to measure very
5 high temperatures. When a body is at a high
TC = (TF – 32 ° ) temperature, it glows brightly and the radiation emitted
9
per second from unit area of the surface of the body is
5
= (60° – 32 ° ) proportional to the fourth power of the absolute
9
temperature of the body. If this radiation is measured
= 15.55° C by some device, the temperature of the body is
From Eq. (i), we get calculated. This is the principle of a total radiation
T = TC + 273.15 pyrometer. The main advantage of this thermometer is
that the experimental body is not kept in contact with it.
= 15.55° C + 273.15 Hence, there is no definite higher limit of its temperature
= 288.7 K range. It can measure temperature from 800°C to
3000°C – 4000°C. However, it cannot be used to
X Example 14.2 The temperature of an iron piece is measure temperatures below 800°C because at low
heated from 30°C to 90°C. What is the change in its temperatures the emission of radiation is so poor that it
temperature on the fahrenheit scale and on the kelvin cannot be measured directly.
scale? ■ Ranges of different thermometers.
Thermometer Lower limit Upper limit
Sol. ∆ TC = 90°C – 30° C = 60° C Mercury thermometer − 30° C 300°C
Using Eq. (iii), Gas thermometer − 268 ° C 1500°C
9 9 − 200° C
∆TF = ∆TC = (60° C ) = 108° F Platinum resistance thermometer 1200°C
5 5 Thermocouple thermometer − 200° C 1600°C
and ∆T = ∆TC = 60 K Radiation thermometer 800°C No limit

■ Reaumer scale Other than celsius, fahrenheit and


Extra Knowledge Points kelvin temperature scales reaumer scale was
designed by Reaumer in 1730. The lower fixed point is
Different Thermometers 0°R representing melting point of ice. The upper fixed
■ Thermometric property It is the property that can be point is 80°R, which represents boiling point of water.
used to measure the temperature. It is represented by The distance between the two fixed points is divided
any physical quantity such as length, volume, pressure into 80 equal parts. Each part represents 1°R. If TC , TF
and resistance, etc. Which varies linearly with a certain and TR are temperature values of a body on celsius
range of temperature. Let X denotes the thermometric scale, fahrenheit scale and reaumer scale
physical quantity and X 0 , X 100 and X t be its values at respectively, then,
0°C, 100°C and t °C respectively. Then, TC − 0 TF − 32 TR − 0
= =
 X − X0  100 180 80
t = t  × 100° C
 X 100 − X 0  ■ A substance is found to exist in three states solid, liquid
(i) Constant volume gas thermometer The pressure and gas. For each substance there is a set of
of a gas at constant volume is the thermometric temperature and pressure at which all the three states
property. Therefore, may coexist. This is called triple point of that
substance. For water, the values of pressure and
 p − p0 
t = t  × 100° C temperature corresponding to triple point are 4.58 mm
 p100 − p0  of Hg and 273.16 K.
(ii) Platinum resistance thermometer The resistance
of a platinum wire is the thermometric property.
Hence, 14.3 Quantity of Heat
 R − R0 
t = t  × 100° C When a cold body is brought in contact with a hot body,
 R 100 − R 0 
the cold body warms up and the hot body cools down as they
(iii) Mercury thermometer In this thermometer, the approach thermal equilibrium. Fundamentally, a transfer of
length of a mercury column from some fixed point is energy takes place from one substance to the other. This type
taken as thermometric property. Thus,
of energy transfer that takes place slowly because of a
 l −l 
t =  t 0  × 100° C temperature difference is called heat flow or heat transfer
 l100 − l0 
and energy transfer in this way is called heat.
Thermometry, Thermal Expansion and Kinetic Theory of Gases 603

Water can be warmed up by vigorous stirring with a Here, the constant α is called the coefficient of linear
pedal wheel. The pedal wheel adds energy to the water by expansion of the material of the rod and its units are K –1 or
doing work on it. The same temperature change can also be [( ° C ) –1 ]. Remember that ∆T = ∆TC .
caused by putting the water in contact with some hotter
body. Actually, α does depend slightly on the temperature, but
its variation is usually small enough to be negligible, even
Hence, this interaction must also involve an energy over a temperature range of 100°C. We will always assume
exchange. Before exploring the relation between heat and that α is a constant.
mechanical energy let us define a unit of quantity of heat.
One calorie (1 cal) is defined as the amount of heat Volume Expansion
required to raise the temperature of1g of water from 14.5°C Because the linear dimensions of an object change with
to 15.5°C. temperature, it follows that surface area and volume change
Experiments have shown that. as well. Just as with linear expansion, experiments show
1 cal = 4.186 J that, if the temperature change ∆T is not too great (less than
Similarly, 1 kcal = 1000 cal 100°C or so), the increase in volume ∆V is proportional to
both the temperature change ∆T and the initial volume V .
= 4186 J
Thus, ∆V ∝ ∆T and ∆V ∝ V
The calorie is not a fundamental SI unit.
Introducing a proportionality constant γ, write ∆V as
∆V = V × γ × ∆T … (ii)
14.4 Thermal Expansion Here, γ is called the coefficient of volume expansion.
Most substances The units of γ are K –1 or ( °C ) –1 .
expand when they are
heated. Thermal expansion Relation between γ and α
is a consequence of the
change in average For an isotropic solid (which has the same value of α in
separation between the all directions) γ = 3α. To see that γ = 3α for a solid, consider
constituent atoms of an a cube of length l and volume V = l 3 .
object. Atoms of an object Fig. 14.5 When the temperature of the cube is increased by dT,
can be imagined to be the side length increases by dl and the volume increases by
connected to one another by stiff springs as shown in figure. an amount dV is given by
At ordinary temperatures, the atoms in a solid oscillate about  dV 
their equilibrium positions with an amplitude of dV =   ⋅ dl = 3l ⋅ dl
2
 dl 
approximately 10 –11 m. The average spacing between the
Now, dl = lαdT
atoms is about 10 –10 m. As the temperature of solid
∴ dV = 3l 3α. dT
increases, the atoms oscillate with greater amplitudes as a
result the average separation between them increases, = (3α )VdT
consequently the object expands. This is consistent with Eq. (ii), dV = γVdT , only if
γ = 3α …(iii)
Linear Expansion Average values of α and γ for some materials are listed
Suppose that the temperature of a thin rod of length l is in Table 14.1. You can check the relation γ = 3α for the
changed from T to T + ∆T . It is found experimentally that, if materials given in the table.
∆T is not too large, the corresponding change in length ∆l of Table 14.1
the rod is directly proportional to ∆T and l. Thus,
Material α [K −1 or (°C) −1] γ [K −1 or (°C) −1]
∆l ∝ ∆T and ∆l ∝ l Steel 1.2 × 10 −5
3.6 × 10 −5
Introducing a proportionality constant α (which is Copper 1.7 × 10 −5
5.1 × 10 −5
different for different materials) we may write ∆l as Brass 2.0 × 10 −5
6.0 × 10 −5
∆l = lα∆T …(i) Aluminium 2.4 × 10 −5
7.2 × 10 −5
604 Objective Physics Vol. 1

example, when the temperatures of a brass rod and a


The Anomalous Expansion of Water steel rod of
Most liquids also expand when their temperatures Steel
increase. Their expansion can also be described by Eq. (ii).
The volume expansion coefficients for liquids are about
100 times larger than those for solids. Brass
3
Room temperature Higher temperature
V(cm )
equal length are raised by the same amount from some
1.0003 common initial value, the brass rod expands more than
the steel rod because brass has a greater average
1.0002 coefficient of expansion than steel. Such type of
bimetallic strip is found in practical devices such as
1.0001 thermostats to break or make electrical contact.
1.000 T (°C)
2 4 6
Fig. 14.6
Bimetallic strip
Some substances contract when heated over a certain
temperature range. The most common example is water.
Figure shows how the volume of 1 g of water varies with
On 25°C On 30°C
temperature at atmospheric pressure. The volume decreases
as the temperature is raised from 0°C to about 4°C, at which ■ Variation of density with temperature
point the volume is a minimum and the density is a Most substances expand when they are heated, i.e.
maximum (1000 kg/ m3 ) . Above 4° C, water expands with volume of a given mass of a substance increases on
 1
increasing temperature like most substances. heating, so the density should decrease as ρ ∝  .
 V
This anomalous behaviour of water causes ice to form Let us see how the density ρ varies with increase in
first at the surface of a lake in cold weather. As winter temperature.
approaches, the water temperature increases initially at the m 1
ρ= or ρ ∝ (for a given mass)
surface. The water there sinks because of its increased V V
density. Consequently, the surface reaches 0°C first and the ρ′ V V V 1
∴ = = = =
lake becomes covered with ice. Aquatic life is able to ρ V ′ V + ∆V V + γV∆T 1 + γ∆T
survive the cold winter as the lake bottom remains unfrozen ρ
at a temperature of about 4°C. ∴ ρ′ =
1 + γ∆T
This expression can also be written as
Extra Knowledge Points
ρ′ = ρ (1 + γ∆T )–1
■ If a solid object has a hole in it, what happens to the
As γ is small, (1 + γ∆T )–1 ≈ 1 – γ∆T ρ′ ≈ ρ (1 – γ∆T )
size of the hole, when the temperature of the object
increases ? A common misconception is that, if the ■ Effect of temperature on upthrust
object expands, the hole will shrink because material When a solid body is completely immersed in a liquid,
expands into the hole. But the truth is that, if the object its apparent weight gets decreased due to an upthrust
expands, the hole will expand too, because every acting on it by the liquid. The apparent weight is given
linear dimension of an object changes in the same way by
when the temperature changes.
w app = w – F
Here, F = upthrust = VS ρL g
a + ∆a
where,VS = volume of solid and ρL = density of liquid.
a Ti + ∆T
Ti Now, as the temperature is increased VS increases
while ρL decreases. So, F may increase or decrease (or
b b + ∆b
may remain constant also) depending upon the
condition that which factor dominates on the other. We
■ Expansion of a bimetallic strip can write
As, Table 14.1 indicates, each substance has its own F ′ VS′ ρ′L (V + ∆VS )  1 
F ∝ VS ρL or = ⋅ = S ⋅ 
characteristic average coefficient of expansion. For F VS ρL VS  1 + γL ∆ T 
Thermometry, Thermal Expansion and Kinetic Theory of Gases 605

V + γSVS ∆T   1  This is because, if the temperature is increased, the


= S    rod has a tendency to expand but since, it is fixed at
 VS   1 + γL ∆ T  two ends, the rod exerts a force on supports.
 1 + γS ∆ T  l, α
or F′ = F  
 1 + γL ∆ T 
Now, if γS > γL , F ′ > F ′ < w app
or w app
∆l
and vice-versa. Thermal strain = = α ⋅ ∆T
l
And if γS = γL ,F ′ = F ′ = w app
or w app
So,thermal stress = ( γ ) (thermal strain)
■ Effect of temperature on the time period of a
= Yα∆T
pendulum
The time period of a simple pendulum is given by or force on supports; F = A (stress) = YA α∆T
l Here, Y = Young’s modulus of elasticity of the rod.
T = 2π or T ∝ l
g F = YAα∆T

As the temperature is increased length of the


■ Expansion of liquid
pendulum and hence, time period gets increased or a For heating a liquid, it has to be put in some container.
pendulum clock becomes slow and it loses the time. When the liquid is heated, the container will also
expand. We define coefficient of apparent expansion
T′ l′ l + ∆l
= = of a liquid as the apparent increase in volume per unit
T l l original volume per °C rise in temperature. It is
Here, we put ∆l = l α ∆θ in place of l α ∆T , so as to avoid represented by γa . Thus,
the confusion with change in time period. Thus, γa = γr − γ g
T′ l + lα∆θ Here, γr = coefficient of real expansion of a liquid and
= = (1 + α∆θ )1/ 2
T l γ g = coefficient of cubical expansion of the container.
 1  1
or T ′ ≈ T 1 + α∆θ or ∆T = T ′ – T = Tα∆θ
 2  2
X Example 14.3 A steel ruler exactly 20 cm long is
Time lost in time t (by a pendulum clock whose actual
graduated to give correct measurements at 20° C.
time period is T and the changed time period at some
higher temperature is T ′ ) is (a) Will it give readings that are too long or too short at
lower temperatures?
 ∆T 
∆t =  t (b) What will be the actual length of the ruler be when it
 T′ 
is used in the desert at a temperature of 40° C?
Similarly, if the temperature is decreased the length and (α steel = 1.2 × 10 –5 ° C –1 )
hence, the time period gets decreased. A pendulum
clock in this case runs fast and it gains the time. Sol. (a) If the temperature decreases, the length of the ruler also
T′ l′ l – lα∆θ 1 decreases through thermal contraction. Below 20°C, each
= = ≈ 1 − α∆θ centimetre division is actually somewhat shorter than 1.0 cm, so
T l l 2
the steel ruler gives readings that are too long.
 1 
or T ′ = T 1 – α∆θ (b) At 40°C, the increase in length of the ruler is
 2 
∆l = lα∆T = (20) (1.2 × 10–5 ) (40° – 20° )
1
∆T = T – T ′ = T α ∆θ = 0.48 × 10–2 cm
2
∴ The actual length of the ruler is
 ∆T 
and time gained in time t is the same, i.e. ∆t =  t l ′ = l + ∆l = 20.0048 cm
 T′ 

At some higher temperature a scale will expand and X Example 14.4 Find the coefficient of volume
scale reading will be lesser than true values, so that expansion for an ideal gas at constant pressure.
true value = scale reading (1 + α∆T )
Here, ∆T is the temperature difference. Sol. For an ideal gas, pV = nRT
However, at lower temperature scale reading will be As p is constant, we have
more or true value will be less. pdV = nRdT
■ When a rod whose ends are rigidly fixed such as to dV nR 1 dV nR nR 1
prevent from expansion or contraction undergoes a ∴ = or γ = ⋅ = = =
dT p V dT pV nRT T
change in temperature, thermal stresses are
1
developed in the rod. ∴ γ=
T
606 Objective Physics Vol. 1

X Example 14.5 The scale on a steel metre stick is


calibrated at 15°C. What is the error in the reading of
14.6 Gas Laws
60 cm at 27°C? Assuming permanent gases to be ideal, through
(α steel = 1.2 × 10 –5 °C –1 ) experiments, it was established that gases irrespective of
their nature obey the following laws:
Sol. At higher temperatures actual reading is more than the
scale reading. The error in the reading will be Boyle’s Law
∆l = (scale reading) (α ) (∆T ) According to this law, for a given mass of a gas the
–5
= (60) (1.2 × 10 ) (27 ° − 15° ) volume of a gas at constant temperature (called isothermal
process) is inversely proportional to its pressure, i.e.
= 0.00864 cm 1
V ∝ (T = constant) or pV = constant or piVi = p f V f
X Example 14.6 A second’s pendulum clock has a p
steel wire. The clock is calibrated at 20°C. How much Thus, p-V graph in an isothermal process is a
time does the clock lose or gain in one weak when the rectangular hyperbola. Or pV versus p or V graph is a straight
temperature is increased to 30° C? line parallel to p or V axis.
(α steel = 1.2 × 10 –5 ° C –1 ) p pV

Sol. The time period of second’s pendulum is 2 s. As the T = constant


T = constant
temperature increases length and hence, time period increases.
Clock becomes slow and it loses the time. The change in time
period is
V p or V
1
∆T = Tα∆θ
2 Fig. 14.7
1
=   ( 2 ) (1.2 × 10–5 ) (30° – 20° )
2 Charles’ Law
= 1.2 × 10–4 s According to this law, for a given mass of a gas the
∴ New time period is volume of a gas at constant pressure (called isobaric
T ′ = T + ∆T process) is directly proportional to its absolute temperature,
= (2 + 1.2 × 10–4 ) i.e.
V ∝T ( p = constant)
= 2.00012 s
V V V
∴ Time lost in one week or = constant or i
=
f

∆T  T Ti T f
∆t =  t
 T′ 
Thus, V-T graph in an isobaric process is a straight line
(1.2 × 10–4 )
= (7 × 24 × 3600) passing through origin. Or V / T versus V or T graph is a
(2.00012) straight line parallel to V or T-axes.
= 36.28 s V V
T
p = constant p = constant
14.5 Concept of an Ideal Gas
A gas has no shape and size and can be contained in a
vessel of any size or shape. It expands indefinitely and
uniformly to fill the available space. It exerts pressure on its T (in K) V or T
surroundings.
Fig. 14.8
The gases whose molecules are point masses (mass
without volume) and do not attract each other are called
ideal or perfect gases. It is a hypothetical concept which Gay Lussac’s Law or Pressure Law
can’t exist in reality.
According to this law, for a given mass of a gas the
The gases such as hydrogen, oxygen or helium which pressure of a gas at constant volume (called isochoric
cannot be liquefied easily are called permanent gases. An process) is directly proportional to its absolute temperature,
actual gas behaves as ideal gas most closely at low pressure i.e.
and high temperature. p ∝ T (V = constant)
Thermometry, Thermal Expansion and Kinetic Theory of Gases 607

p
or = constant Extra Knowledge Points
T
pi pf ■ In our previous discussion, we have read Charles’ law
or = and pressure law in absolute temperature scale. In
Ti T f centigrade scale, these laws are as under
Thus, p-T graph in an isochoric process is a straight line Charles’ law
passing through origin or p/ T versus p or T graph is a When a given mass of a gas is heated at constant
straight line parallel to p or T axis. pressure, then for each 1°C rise in temperature the
p volume of the gas increases by a fraction α of its
p T volume at 0°C. Thus, if the volume of a given mass of a
gas at 0°CisV0 ,then on heating at constant pressure to
t °C its volume will increase by V0α t . Therefore, if its
V = constant volume at t °C be Vt , then
p or T Vt = V0 + V0αt or Vt = V0 (1 + αt )
T (in K)
Fig. 14.9
Here, α is called the ‘volume coefficient’ of the gas. For
1
all gases the experimental value of α is nearly / °C.
273
Avogadro’s Law pt
According to this law, at same temperature and pressure
equal volumes of all gases contain equal number of
molecules. p0

14.7 Ideal Gas Equation –273 O


t (°C)

All the above four laws can be written in one single  t 


equation known as ideal gas equation. According to this ∴ Vt = V0 1 + 
 273
equation. V0
Thus,Vt versus t graph is a straight line with slope
m 273
pV = nRT = RT
M and positive intercept V0 .

In this equation, n = number of moles of the gas Further, Vt = 0 at t = – 273° C.


m Pressure law
=
M According to this law, when a given mass of a gas is
heated at constant volume then for each 1°C rise in
where, m = total mass of the gas,
temperature, the pressure of the gas increases by a
M = molecular mass of the gas, fraction β of its pressure at 0°C. Thus, if the pressure of
and R = universal gas constant. a given mass of a gas at 0°C be p0 , then on heating at
= 8.31 J/ mol-K constant volume to t °C, its pressure will increase by
p0β t . Therefore, if its pressure at t °C be pt , then
= 2.0cal/ mol-K
pt = p0 + p0 βt or pt = p0 (1 + βt )
The above four laws can be derived from this single Vt
equation. For example, for a given mass of a gas
(m = constant)
pV = constant at constant temperature (Boyle’s law)
V0
p
= constant at constant volume (Pressure law)
T
V t (°C)
= constant at constant pressure (Charles’ law) –273 O
T
and if p,V and T are constants, then n = constant for all Here, β is called the pressure coefficient of the gas. For
gases. 1
all gases the experimental value of β is also / °C.
273
And since, equal number of moles contain equal number
 t 
of molecules. So, at constant pressure, volume and ∴ pt = p0 1 + 
 273
temperature all gases will contain equal number of
molecules. Which is nothing but Avogadro’s law. The pt versus t graph is as shown in figure.
608 Objective Physics Vol. 1

■ The above forms of Charles’ law and pressure law can X Example 14.7 p-V diagrams of same mass of a gas
be simply expressed in terms of absolute temperature. are drawn at two different temperatures T1 and T2 .
■ Let at constant pressure, the volume of a given mass of Explain whether T1 > T2 or T2 > T1 .
a gas at 0°C, t1 °Cand t 2 °CbeV0 ,V1 andV2 respectively.
Then, p

 t   273 + t 1
V1 = V0 1 + 1  = V0  
 273  273 
 t   273 + t 2 
V2 = V0 1 + 2  = V0   T2
 273  273 
T1
V1 273 + t 1 T1
∴ = = V
V 2 273 + t 2 T2
Fig. 14.10
where, T1 and T2 are the absolute temperatures
corresponding to t1 °C and t 2 °C. Hence, Sol. The ideal gas equation is pV = nRT or T = pV
V1 V2 V nR
= or = constant or V ∝ T
T1 T2 T T ∝ pV, if number of moles of the gas are kept constant. Here,
mass of the gas is constant, which implies that number of moles
This is the form of Charles’ law which we have already are constant, i.e. T ∝ pV. In the given diagram product of p and
studied in article 14.6. In the similar manner, we can V for T2 is more than T1 at all points (keeping either p or V same
prove the pressure law. for both graphs). Hence,
■ Under isobaric conditions (p = constant), V-T graph is a T2 > T1
straight line passing through origin (where T is in kelvin).
 nR   nR  Example 14.8 The p-V diagrams of two different
The slope of this line is   as V =   T or slope of X
 p  p
masses m1 and m2 are drawn (as shown) at constant
n
the line is directly proportional to . temperature T. State whether m 1 > m 2 or m 2 > m 1 ?
p
p
nR n T T
Slope = or slope ∝
p p
Similarly, under isochoric conditions (V = constant), p-T
graph is a straight line passing through origin whose m2
nR n m1
slope is or slope is directly proportional to . V
V V
■ Density of a gas The ideal gas equation is Fig. 14.11
ρ ρ ρ
Sol. pV = nRT = m RT ⇒ ∴ m = ( pV )  M 
m = constant M   RT
p = constant
T = constant m = constant or m ∝ pV (if T = constant)
m = constant
V
From the graph we can see that p2 V2 > p1V1 (for same p or V ).
p T (K)
Therefore,
m m2 > m1
pV = nRT = RT
M
X Example 14.9 The p-T graph for the given mass
m pM of an ideal gas is shown in figure. What inference can
∴` =ρ = (As, ρ = density)
V RT be drawn regarding the change in volume (whether it is
pM constant, increasing or decreasing)?
∴ ρ=
RT p
■ From this equation, we can see that ρ- p graph is straight
B
line passing through origin at constant temperature
(ρ ∝ p ) for a given gas and ρ-T graph is rectangular
 1
hyperbola at constant pressure ρ ∝  . Similarly, for a A
 T
given mass of a gas ρ-V graph is a rectangular
T (K)
 1
hyperbola ρ ∝  .
 V Fig. 14.12
Thermometry, Thermal Expansion and Kinetic Theory of Gases 609

HOW TO PROCEED Definitely, it is not constant. p 


or pV = –  0  V 2 + 3 p0 V
Because when volume is constant, p-T graph is a  V0 
straight line passing through origin. The given line p 
or nRT = 3 p0 V –  0  V 2 (as pV = nRT)
does not pass through origin, hence volume is not  V0 
constant. 1   p0  2 
or T=  3 p0 V –   V 
V = ( nR ) (T / P ) nR   V0  
Now, to see the volume of the gas we will have to see
T This is the required T-V equation. This is quadratic in V.
whether is increasing or decreasing. Hence, T-V graph is a parabola. Now, to find maximum or
p minimum value of T we can substitute.
dT
Sol. From the given graph, we can write the p-T equation as =0
dV
p = aT + b ( y = mx + c )
2 p  3
Here, a and b are positive constants. Further, or 3 p0 –  0  V = 0 or V= V0
 V0  2
p b
=a+
T T d 2T 3
Further is negative at V = V0
Now, TB > TA ∴
b
<
b dV 2 2
TB TA 3
Hence,T is maximum at V = V0 and this maximum value is
T 2
 p <  p  p
or     or   > 
1   3V   p   3V  
2
T B T A  p B  T  A 9 p0 V0
Tmax = (3 p0 )  0  –  0   0   or Tmax =
nR   2   V0   2   4nR
or VB > VA 
Thus, as we move from A to B, volume of the gas is Thus, T-V graph is as shown in the following figure.
increasing.
T

X Example 14.10 p-V diagram of n moles of an


ideal gas is as shown in figure. Find the maximum Tmax
temperature between A and B.
A B
p TA = TB

A V
2p0 V0 2V0
Fig. 14.14
p0 B 2 p0 V0
TA = TB =
nR
V 9 p0 V0 pV
V0 2V0 and Tmax = = 2.25 0 0
4nR nR
Fig. 14.13

HOW TO PROCEED For given number of moles of 14.8 Degree of Freedom (f )


a gas,
The term degree of freedom refers to the number of
T ∝ pV ( pV = nRT )
possible independent ways in which a system can have
Although ( pV ) A = ( pV ) B or T A = TB , still it is not energy.
an isothermal process. Because in isothermal process y
f=1
p-V graph is a rectangular hyperbola while it is a f=2
straight line. So, to see the behaviour of temperature
first we will find either T-V equation or T-p equation
x
and from that equation we can judge how the
temperature varies. From the graph first, we will write (a) (b)
f=2
p-V equation, then we will convert it either in T-V
equation or in T-p equation.
Sol. From the graph, the p-V equation can be written as
p 
p = –  0  V + 3 p0 ( y = – mx + c ) (c)
 V0  Fig. 14.15
610 Objective Physics Vol. 1

For example In Fig. (a), block has one degree of At sufficiently high temperatures it has vibrational
freedom, because it is confined to move in a straight line and energy as well providing it two more degrees of freedom
has only one translational degree of freedom. (one vibrational kinetic energy and another vibrational
In Fig. (b), the projectile has two degrees of freedom potential energy). Thus, at high temperatures a diatomic
because it is confined to move in a plane and so it has two molecule has 7 degrees of freedom, 3 translational,
translational degrees of freedom. 2 rotational and 2 vibrational. Thus,
In Fig. (c), the sphere has two degrees of freedom one f =5
rotational and another translational. (3 translational + 2 rotational) at room temperatures
Similarly, a particle free to move in space will have and f =7
three translational degrees of freedom. (3 translational + 2 rotational + 2 vibrational)
at high temperatures
Degree of Freedom of Gas Molecules
A gas molecule can have following types of energies : Degree of Freedom of Non-linear
(i) translational kinetic energy Polyatomic Gas
(ii) rotational kinetic energy
A non-linear polyatomic molecule (such as NH 3 ) can
(iii) vibrational energy (potential + kinetic)
rotate about any coordinate axes. Hence, it has 6 degrees of
freedom 3 translational and 3 rotational. At room
Vibrational Energy temperatures a polyatomic gas molecule has vibrational
The forces between different atoms of a gas molecule energy greater than that of a diatomic gas. But at high enough
may be visualised by imagining every atom as being temperatures it is also significant. So, it has 8 degrees of
connected to its neighbours by springs. Each atom can freedom 3 rotational 3 translational and 2 vibrational. Thus,
vibrate along the line joining the atoms. Energy associated (3 translational + 3 rotational) at room temperatures
with this is called vibrational energy. f = 6 and f = 8
(3 translational + 3 rotational + 2 vibrational)
Degree of Freedom of at high temperatures
Monoatomic Gas z

A monoatomic gas molecule (like He) consists of a y


single atom. It can have translational motion in any direction
in space. Thus, it has 3 translational degrees of freedom.
x
f =3 (all translational)
It can also rotate but due to its small moment of inertia,
rotational kinetic energy is neglected. Fig. 14.17

Degree of Freedom of a Diatomic and Degree of Freedom of a Solid


Linear Polyatomic Gas An atom in a solid has no degrees of freedom for
The molecules of a diatomic and linear polyatomic gas translational and rotational motion. At high temperatures
(like O 2 , CO 2 and H 2 ) cannot only move bodily but also due to vibration along 3 axes it has 3 × 2 = 6 degrees of
rotate about any one of the three coordinate axes as shown in freedom. f = 6 (all vibrational) at high temperatures
figure. However, its moment of inertia about the axis joining
the two atoms (x-axis) is negligible. Hence, it can have only Note Points
two rotational degrees of freedom. Thus, a diatomic / Degrees of freedom of a diatomic and polyatomic gas
molecule has 5 degrees of freedom 3 translational and depends on temperature and since there is no clear cut
demarcation line above which vibrational energy become
2 rotational.
z significant. Moreover, this temperature varies from gas to
gas. On the other hand, for a monoatomic gas there is no
y such confusion. Degree of freedom here is 3 at all
temperatures. Unless and until stated in the question you can
take f = 3 for monoatomic gas, f = 5 for a diatomic gas and
x f = 6 for a non-linear polyatomic gas.
/ When a diatomic or polyatomic gas dissociates into atoms, it
behaves as a monoatomic gas. Whose degrees of freedom
Fig. 14.16 are changed accordingly.
Thermometry, Thermal Expansion and Kinetic Theory of Gases 611

14.9 Internal Energy of an 14.10 Law of Equipartition of


Ideal Gas Energy
Suppose a gas is contained in a closed vessel as shown An ideal gas is just like an ideal father. As an ideal father
in figure. If the container as a whole is moving with some distributes whole of its assets equally among his children.
speed, then this motion is called the ordered motion of the Same is the case with an ideal gas. It distributes its internal
gas. Source of this motion is some external force. energy equally in all degrees of freedom. In each degree of
The zig-zag motion of gas molecules within the vessel is 1
freedom energy of one mole of an ideal gas is RT, where T
known as the disordered motion. This motion is directly 2
related to the temperature of the gas. As the temperature is is the absolute temperature of the gas. Thus, if f be the
increased, the disordered motion of the gas molecules number of degrees of freedom, the internal energy of 1 mole
gets fast. f
of the gas will be RT or internal energy of n moles of the
The internal energy (U) of the gas is concerned only 2
with its disordered motion. It is in no way concerned with its gas will be n/2 fRT . Thus,
ordered motion. When the temperature of the gas is n
U = fRT …(i)
increased, its disordered motion and hence its internal 2
energy is increased. For a monoatomic gas, f = 3
Disordered 3
motion Therefore, U = RT
2
Ordered
motion (for 1 mole of a monoatomic gas)
For a dia and linear polyatomic gas at low temperatures,
Fig. 14.18 f = 5, so
Intermolecular forces in an ideal gas is zero. Thus, PE 5
U = RT (for 1 mole)
due to intermolecular forces of an ideal gas is zero. A 2
monoatomic gas is having a single atom. Hence, its and for non-linear polyatomic gas at low temperatures,
vibrational energy is zero. For dia and polyatomic gases f = 6, so
vibrational energy is significant only at high temperatures. 6
So, they also have only translational and rotational KE. We U = RT = 3RT (for 1 mole)
2
may thus, conclude that at room temperature the internal / From Eq. (i), we can see that internal energy of an ideal gas
energy of an ideal gas (whether it is mono, dia or poly) depends only on its temperature and which is directly
consists of only translational and rotational KE. Thus, proportional to its absolute temperature T. In an isothermal
U (of an ideal gas) = KT + K R at room temperatures. process,T = constant. Therefore, the internal energy of the
gas does not change or dU = 0.
Internal Energy
(U )
14.11 Molar Heat Capacity
“Molar heat capacity C is the heat required to raise the
Potential Energy Kinetic Energy temperature of 1 mole of a gas by 1°C (or 1 K).” Thus,
∆Q
C= or ∆Q = nC∆T
n∆ T
Due to Due to Translational Rotational Vibrational For a gas, the value of C depends on the process through
intermolecular interatomic KE KE KE which its temperature is raised.
forces forces
(vibrational) For example, in an isothermal process ∆T`= 0 or
C iso = ∞. In an adiabatic process (we will discuss it later),
Later in the next article, we will see that KT ∆Q = 0. Hence, C adi = 0. Thus, molar heat capacity of a gas
(translational KE) and K R (rotational KE) depends on T varies from 0 to ∞ depending on the process. In general,
only. They are directly proportional to the absolute experiments are made either at constant volume or at
temperature of the gas. Thus, internal energy of an ideal gas constant pressure. In case of solids and liquids, due to small
depends only on its absolute temperature (T ) and is directly thermal expansion, the difference in measured values of
proportional to T. molar heat capacities is very small and is usually neglected.
or U ∝T However, in case of gases molar heat capacity at constant
612 Objective Physics Vol. 1

volume CV is quite different from that at constant pressure The kinetic theory that we study in this article is a
C p. Later in the next chapter, we will derive the following special aspect of the statistical mechanics of large number of
relations, for an ideal gas particles. We begin with the simplest model for a
dU monoatomic ideal gas, a dilute gas whose particles are single
CV = atoms rather than molecules.
dT
f R Macroscopic variables of a gas are pressure, volume
= R=
2 γ –1 and temperature and microscopic properties are speed of
gas molecules and momentum of molecules, etc. Kinetic
C p = CV + R
theory of gases relates the microscopic properties to
Cp 2
γ= =1 + macroscopic properties. Further more, the kinetic theory
CV f provides us with a physical basis for our understanding of
Here, U is the internal energy of one mole of the gas. the concept of pressure and temperature.
The most general expression for C in the process
pV x = constant is The Ideal Gas Approximation
R R We make the following assumptions while describing
C= + (we will derive it later) an ideal gas
γ –1 1– x
1. The number of particles in the gas is very large.
For example For isobaric process
2. The volume V containing the gas is much larger than
p = constant or x = 0 the total volume actually occupied by the gas
R
and C =Cp = + R = CV + R particles themselves.
γ –1 3. The dynamics of the particles is governed by
For isothermal process, pV = constant or x =1 Newton’s laws of motion.
∴ C =∞ 4. The particles are equally likely to be moving in any
For adiabatic process, pV γ = constant or x = γ direction.
∴ C =0 5. The gas particles interact with each other and with
the walls of the container only via elastic collisions.
Values of f , U , CV , C p and γ for different gases are
shown in Table 14.2. 6. The particles of the gas are identical and
indistinguishable.
Table 14.2
f C V = dU /dT γ = Cp / CV The Pressure of an Ideal Gas
Nature U = RT f Cp = CV + R 2
f = R =1 + Consider an ideal gas consisting of N molecules in a
of gas 2
2 f
container of volume V. The container is a cube with edges of
3 3 5
Monoatomic 3 RT R R 1.67 length d. Consider the collision of one molecule moving
2 2 2
Dia and
with a velocity v towards the right hand face of the cube. The
5 5 7
linear 5 RT R R 1.4 molecule has velocity components v x , v y and v z .
2 2 2
polyatomic Previously, we used m to represent the mass of a sample, but
Non-linear in this article we shall use m to represent the mass of one
polyatomic 6 3RT 3R 4R 1.33
molecule.
y

14.12 Kinetic Theory of Gases


We have studied the mechanics of single particle.
When we approach the mechanics associated with the v
many particles in systems such as gases, liquids and solids,
we are faced with analyzing the dynamics of a huge number d m
of particles. The dynamics of such many particle systems is vx
called statistical mechanics.
z d x
The game involved in studying a system with a large d
number of particles is similar to what happens after every A cubical box with sides of length d containing
physics test. Of course we are interested in our individual an ideal gas. The molecule shown moves with velocity v
marks, but we also want to know the class average. Fig. 14.19
Thermometry, Thermal Expansion and Kinetic Theory of Gases 613

As the molecule collides with the wall elastically its This can also be written as
x-component of velocity is reversed, while its y and Nm 2
F wall = vx
z-component of velocity remain unaltered. Because the d
x-component of the momentum of the molecule is mv x v x21 + v x22 +… + v x2N
before the collision and –mv x after the collision, the change where, vx =
2

in momentum of the molecule is N


∆px = – mv x – ( mv x ) = – 2mv x Since, the velocity has three components v x , v y and v z ,
we can have
Applying impulse = change in momentum to the molecule
v 2 = v x2 + v y2 + v z2 (as v 2 = v x2 + v y2 + v z2 )
v
vy
Because the motion is completely random, the average
values v x2 , v y2 and v z2 are equal to each other. So,
1
–vx
v 2 = 3 v x2 or v x2 = v 2
3
N  mv 2
Therefore, F wall = 
3  d 

∴ Pressure on the wall


vy v F F 1N 
p = wall = wall =  3 mv 2 
A d 2 3 d 
1N
vx =   mv 2
3V 
A molecule makes an elastic collision with the wall 2  N  1 
of the container. Its x-component of momentum is =    mv 2 
reversed, while its y-component remains unchanged. 3  V  2 
In this construction, we assume that the molecule
moves in the x-y plane. 1 mN 2 2  N   1 
∴ p= v =    mv 2  …(i)
Fig. 14.20 3 V 3  V  2 
This result indicates that the pressure is proportional to
F∆t = ∆px = – 2mv x the number of molecules per unit volume ( N /V ) and to the
where, F is the magnitude of the average force exerted average translational kinetic energy of the molecules1/ 2 mv 2 .
by the wall on the molecule in time ∆t. For the molecules to This result relates the large scale quantity (macroscopic) of
collide twice with the same wall, it must travel a distance 2d pressure to an atomic quantity (microscopic) the average
in the x-direction. Therefore, the time interval between two value of the square of the molecular speed. The above
2d
collisions with the same wall is ∆t = .Over a time interval equation varifies some features of pressure with which you
vx are probably familiar. One way to increase the pressure inside
that is long compared with ∆t, the average force exerted on a container is to increase the number of molecules per unit
the molecules for each collision is volume in the container.
–2mv x –2mv x – mv x2 The meaning of the absolute temperature
F= = =
∆t 2d / v x d Rewriting Eq. (i) in the more familiar form
2 1 
According to Newton’s third law, the average force pV = N  mv 2 
mv x2 3 2 
exerted by the molecule on the wall is .
d Let us now compare it with the ideal gas equation
pV = nRT
Each molecule of the gas exerts a force on the wall. We
2 1 
find the total force exerted by all the molecules on the wall nRT = N  mv 2 
3  2 
by adding the forces exerted by the individual molecules.
m N
∴ F wall = ( v x21 + v x22 +… + v x2N ) Here, n= (N A = Avogadro’s number)
d NA
614 Objective Physics Vol. 1

2  N  1 
∴ T =  A   mv 2  Mean Speed of Average Speed
3  R  2 
The particles of a gas have a range of speeds. The
2 1 2 average speed is found by taking the average of the speeds of
or T=  mv  …(ii)
3k 2  all the particles at a given instant. Remember that the speed
is a positive scalar since, it is the magnitude of the velocity.
where, k is Boltzmann constant which has the value
v + v 2 +… + v N
R v av = 1
k= = 1.38 × 10 –23 J/ K N
NA
From Maxwellian speed distribution law, can show that
By rearranging Eq. (ii) we can relate the translational
8kT 8RT 8p
molecular kinetic energy to the temperature v av = = =
1 3 πm πM πρ
mv 2 = kT
2 2
That is, the average translational kinetic energy per
Most Probable Speed
3 1 This is defined as the speed which is possessed by
molecule is kT. Because v x2 = v 2 , it follows that maximum fraction of total number of molecules of the gas.
2 3
1 1 For example, if speeds of 10 molecules of a gas are, 1, 2, 2, 3,
mv x = kT
2
3, 3, 4, 5, 6, 6 km/s, then the most probable speed is 3 km/s,
2 2
as maximum fraction of total molecules possess this speed.
In the similar manner it follows that Again from Maxwellian speed distribution law (out of JEE
1 1 1 1 syllabus)
mvy2 = kT and mv z2 = kT
2 2 2 2 2kT 2RT 2p
v mp = = =
Thus, in each translational degree of freedom one gas m M ρ
1
molecule has an energy kT. One mole of a gas has N A
2 Note Points
number of molecules. Thus, one mole of the gas has an
1 1 / In the above expressions of v rms , v av and v mp , M is the molar
energy ( k NA ) T = RT in each degree of freedom. Which mass in kg/mol. For example, molar mass of hydrogen is
2 2 2 × 10–3 kg/mol.
is nothing but the law of equipartition of energy. The total / v rms > v av > v mp (RAM)
translational kinetic energy of one mole of an ideal gas is 8 8
/ v rms : v av : v mp = 3 : : 2 and since, ≈ 2.5, we have
3 π π
therefore, RT.
2 v rms : v av : v mp = 3 : 2.5 : 2
3
(KE) trans = RT (of one mole) X Example 14.11 A tank used for filling helium
2
balloons has a volume of 3.0 m 3 and contains 2.0 mol
Root Mean Square Speed of helium gas at 20.0°C. Assuming that the helium
behaves like an ideal gas.
The square root of v 2 is called the root mean square
(a) What is the total translational kinetic energy of the
(rms) speed of the molecules. From Eq. (ii) we obtain, for molecules of the gas?
the rms speed
(b) What is the average kinetic energy per molecule?
3kT
v rms = v 2 =
m Sol. (a) Using (KE)trans = 3 nRT
2
R
Using k= , mNA = M with n = 2.0 mol and T = 293 K, we find that
NA 3
(KE)trans = (2.0)(8.31) (293) = 7.3 × 103 J
RT p 2
and = 3
M ρ (b) The average kinetic energy per molecule is kT.
2
We can write, 1 1 3
or mv 2 = mvrms2
= kT
3kT 3RT 3p 2 2 2
v rms = = = 3
m M ρ = (1.38 × 10–23 ) (293) = 6.07 × 10−21 J
2
Thermometry, Thermal Expansion and Kinetic Theory of Gases 615

2.0 × 10–10
X Example 14.12 Consider an 1100 and r= = 1.0 × 10−10 m
2
particles gas system with speeds distribution as
∴ I = 2 (5.81 × 10−26 ) (1.0 × 10–10 )2
follows :
1000 particles each with speed 100 m/s, = 1.16 × 10–45 kg-m2
1 2
2000 particles each with speed 200 m/s, ∴ KR = lω
2
4000 particles each with speed 300 m/s,
1
3000 particles each with speed 400 m/s, and = × (1.16 × 10–45 ) × (2.0 × 1012 )2
2
1000 particles each with speed 500 m/s. = 2.32 × 10–21 J
Find the average speed, and rms speed. 1
/ AtT = 300 K , rotational KE should be equal to kT
2
Sol. The average speed is 1
= × (1.38 × 10–23 ) × (300) = 2.07 × 10−21 J
(1000) (100) + (2000) (200) + (4000) (300) 2
+ (3000) (400) + (1000) (500)
vav =
1100 X Example 14.15 Prove that the pressure of an ideal
= 309 m/s gas is numerically equal to two-third of the mean
The rms speed is translational kinetic energy per unit volume of the gas.
(1000) (100)2 + (2000) (200)2 + (4000) (300)2
Sol. Translational KE per unit volume
+ (3000) (400)2 + (1000) (500)2 1
vrms = E= (mass per unit volume) (v 2 )
1100 2
= 328 m/s 1  3 p 3 2
= (ρ)   = p or p = E
2  ρ 2 3
vrms 3
/ Here, ≠ as values and gas molecules are / Students are advised to remember this result. In this
v av 8/ π
expression, E is the translational KE per unit volume.
arbitrarily taken.

X Example 14.13 Calculate the change in Extra Knowledge Point


internal energy of 3.0 mol of helium gas when its ■ Pressure exerted by an ideal gas is numerically equal
temperature is increased by 2.0 K. to two-third of the mean kinetic energy of translation
per unit volume of the gas. Thus,
Sol. Helium is a monoatomic gas. Internal energy of n moles of
2
the gas is, p= E
3 3 3
U= nRT ⇒ ∴ ∆U = nR(∆T )
2 2 ■ Mean Free Path
Substituting the values, Every gas consists of a very large number of
3
∆U =   (3) (8.31)(2.0) = 74.8 J
molecules. These molecules are in a state of
2 continuous rapid and random motion. They undergo
perfectly elastic collisions against one another.
X Example 14.14 In a crude model of a rotating Therefore, path of a single gas molecule consists of a
series of short zig-zag paths of different lengths.
diatomic molecule of chlorine (Cl2 ), the two Cl atoms
The mean free path of a gas molecule is the average
are 2.0 × 10 –10 m apart and rotate about their centre of
distance between two successive collisions.
mass with angular speed ω = 2.0 × 1012 rad/s. What is It is represented by λ.
the rotational kinetic energy of one molecule of Cl2 , kT
λ=
which has a molar mass of 70.0 g/mol ? 2πσ 2ρ
m ω m Here, σ = diameter of the molecule
Cl Cl k = Boltzmann constant
■ Avogadro’s Hypothesis
r r
At constant temperature and pressure equal volumes
Fig. 14.21
of different gases contain equal number of molecules.
In 1 g-mol of any gas, there are 6.02 × 1023 molecules
Sol. Moment of inertia, I = 2 (mr 2 ) = 2 mr 2 of that gas. This is called Avogadro’s number. Thus,
70 × 10–3
Here, m= = 5.81 × 10–26 kg N = 6.02 × 1023 /g-mol
2 × 6.02 × 1023
616 Objective Physics Vol. 1

Therefore, the number of molecules in mass m of the substance


m
Number of molecules = nN = ×N
M
■ Dalton’s Law of Partial Pressure
According to this law, if the gases filled in a vessel do not react chemically, then the combined pressure of all the gases is
due to the partial pressure of the molecules of the individual gases. If p1, p2 ,… represent the partial pressures of the different
gases, then the total pressure is
p = p1 + p2 …
■ van der Waals’ Equation
Experiments have proved that real gases deviate largely from ideal behaviour. The reason of this deviation is two wrong
assumptions in the kinetic theory of gases.
(i) The size of the molecules is much smaller in comparison to the volume of the gas, hence, it may be neglected.
(ii) Molecules do not exert intermolecular force on each other.
■ van der Waals' made corrections for these assumptions and gave a new equation. This equation is known as van der Waals’
equation for real gases.
(i) Correction for the finite size of molecules
Molecules occupy some volume. Therefore, the volume in which they perform thermal motion is less than the observed
volume of the gas. It is represented by (V − b ). Here, b is a constant which depends on the effective size and number of
molecules of the gas. Therefore, we should use (V − b ) in place of V in gas equation.
(ii) Correction for intermolecular attraction Due to the intermolecular force between gas molecules the molecules which
are very near to the wall experiences a net inward force. Due to this inward force there is a decrease in momentum of the
particles of a gas. Thus, the pressure exerted by real gas molecules is less than the pressure exerted by the molecules
 a
of an ideal gas. So, we use  p + 2  in place of p in gas equation.
 V 
Here, again a is a constant.
van der Waals’ equation of state for real gases thus becomes,
 a
 p + 2  (V − b ) = RT
 V 
■ Critical Temperature, Pressure and Volume
Gases cannot be liquefied above a temperature called critical temperature ( TC ), however large the pressure may be. The
pressure required to liquefy the gas at critical temperature is called critical pressure ( pC ) and the volume of the gas at critical
temperature and pressure is called critical volume (VC ). Value of critical constants in terms of van der Waals’ constants a and
b are as under
VC = 3b
a 8a
pC = and TC =
27b 2 27Rb
RTC 8
■ Further, = is called critical coefficient and is same for all gases.
pCVC 3
Chapter Summary with Formulae
■ Thermal Expansion (vi) Internal energy of 1 mole in one degree of freedom
(i) ∆l = lα∆T , ∆s = sβ∆T and ∆V = Vγ∆T 1
of any gas is RT .
(ii) β = 2α and γ = 3α for isotropic medium. 2
(iii) Thermal Stress If temperature of a rod fixed at (vii) Translational kinetic energy of one mole of any type
3
both ends is increased, then thermal stresses are of gas is RT .
developed in the rod. 2
(viii) Rotational kinetic energy of 1 mole of monatomic
2
gas is zero of dia or linear polyatomic gas is RT
2
3
or RT , of non-linear polyatomic gas is RT .
2
(ix) Mixture of non-reactive gases
Fig. 14.22
(a) n = n1 + n2
Rod applies this much force on wall to expand. In turn, (b) p = p1 + p2
wall also exerts equal and opposite pair of encircled (c) U = U1 + U 2
forces on rod. Due to this pair of forces only, we can (d) ∆U = ∆U1 + ∆U 2
say that rod is compressed. n1CV + n2CV
(e) CV = 1 2

n1 + n2
F F n Cp + n2Cp 2
(f) Cp = 1 1 = CV + R
F F n1 + n2
C n n1 n2
(g) γ = p or = +
CV γ − 1 γ1 − 1 γ 2 − 1
Fig. 14.23 n M + n2M 2
(h) M = 1 1
n1 + n2
■ Kinetic theory of gases ■ Kinetic theory of gases
m
(i) f = degree of freedom (i) pV = nRT = RT
M
= 3 for monatomic gas
(m = mass of gas in gms)
= 5 for diatomic and linear polyatomic gas m
(ii) Density, ρ = (general)
= 6 for non-linear polyatomic gas V
pM
= (for an ideal gas)
Note Points RT
/ Vibrational degree of freedom is not taken into consideration. (iii) Gas laws
/ Translational degree of freedom for any type of gas is three. (a) Boyle’s law is applied whenT = constant or process
(ii) Total internal energy of a gas is is isothermal. In this condition,
nf pV = constant
U= RT
1 1 = p2V2
2 or pV
Here, n = total number of gram moles 1
or p∝
dU V
(iii) CV =
dT (b) Charles’ law is applied when p = constant or
[where, U = internal energy of one mole of a gas V
process is isobaric. In this condition = constant or
f T
= RT ] V1 V2
2 = or V ∝ T
f R T1 T2
∴ CV = R =
2 γ −1 (c) Pressure law or Gay Lussac’s law is applied when
V = constant or process is isochoric.
 f  γ 
(iv) Cp = CV + R = 1 +  R =  R In this condition,
 2   γ − 1 p p p
= constant or 1 = 2
Cp 2 T T1 T2
(v) γ = =1 +
CV f or p ∝T
ART AkT Ap (xiii) Translational kinetic energy of one mole of any type
(iv) Four speeds, v = = = 3
M m ρ of gas is RT .
2
Here, m = mass of one gas molecule.
(xiv) Rotational kinetic energy of 1 mole of monatomic
A = 3 for rms speed of gas molecules 2
gas is zero of dia or linear polyatomic gas is RT .
8 2
= ≈ 2.5 for average speed of gas molecules 3
π or RT , of non-linear polyatomic gas is RT .
= 2 for most probable speed of gas molecules 2
Cp (xv) Mixture of non-reactive gases
γ= for speed of sound in a gas (a) n = n1 + n2
CV
(b) p = p1 + p2
1 mn 2
(v) p = vrns (c) U = U1 + U 2
3 V
Here, m = mass of one gas molecule and n = total (d) ∆ U = ∆ U1 + ∆U 2
number of molecules. n1CV + n2CV
(e) CV = 1 2
2 n1 + n2
(vi) p = E .
3 n1Cp1 + n2Cp 2
Here, E = total translational kinetic energy per unit (f) Cp = = CV + R
volume n1 + n2
(vii) f = degree of freedom Cp n n1 n2
(g) γ = or = +
= 3 for monatomic gas CV γ − 1 γ1 − 1 γ 2 − 1
= 5 for diatomic and linear polyatomic gas n1M1 + n2M 2
(h) M =
= 6 for non-linear polyatomic gas n1 + n2
(xvi) An ideal gas equation connecting pressure ( p ),
Note Points volume (V ) and absolute temperature (T ) is
/ (a) Vibrational degree of freedom is not taken into pV = nRT = kNT
consideration. where, n is the number of moles and N is the
/ (b) Translational degree of freedom for any type of gas is number of molecules, R and k are universal
three. constants.
(viii) Total internal energy of a gas is R = 8.314 J mol−1 K −1, k = 1.38 ×10−23 J K −1
nf Real gases satisfy the ideal gas equation only
U= RT
2 approximately, more so at low pressures and high
Here, n = total number of gram moles temperatures.
dU (xvii) Kinetic theory of an ideal gas gives the relation
(ix) CV = (whereU = internal energy of one mole of
dT 1
p = nm vrms
2

f 3
a gas = RT )
2 where, n is number density of molecules or number
f R of molecules per unit volume m is the mass of one
∴ CV = R = molecule and vrms 2
is the root mean of square
2 γ −1
speed.
 f  γ 
(x) Cp = CV + R = 1 +  R =  R (xviii) The translational kinetic energy
 2  γ − 1 3
Cp E = kNT
2 2
(xi) γ = =1 +
CV f Here, N = total number of molecules
(xii) Internal energy of 1 mole in one degree of freedom 2
1 This leads to a relation pV = E
of any gas is RT . 3
2
Additional Examples
Example 1. Why are gas thermometers more Example 9. If an electric fan is switched on in a
sensitive than mercury thermometers? closed room, will the air of the room be cooled? If not,
Sol. The coefficient of expansion of a gas is very large as why do we feel cold?
compared to the coefficient of expansion of mercury. For the Sol. The air will not be cooled. In fact, it will get heated up due
same temperature range, a gas would undergo a much larger to the increase in the speed of its molecules. We feel cold due
change in volume as compared to mercury. to faster evaporation of sweat.
Example 2. Why is a constant volume gas Example 10. Two large holes are cut in a metal
thermometer preferred as a standard thermometer than sheet. If the sheet is heated, how will the diameters of
a constant pressure gas thermometer? the holes change?
Sol. This is because the changes in pressure can be measured
with greater accuracy than changes in volume.

Example 3. Do all solids expand on heating? If not,


A B C D
give an example.
Sol. No. Camphor contracts on heating.

Example 4. Is the temperature coefficient of


resistance always positive? Sol. When a body is heated, the distance between any of its two
Sol. No. Temperature coefficient α is positive for metals and points increases. Hence, the diameters AB and CD of the two
alloys and negative for semiconductors and insulators. holes will increase.

Example 5. Why a small gap is left between the iron Example 11. In example 11, will the distance
rails of railway tracks? between the two holes increase or decrease on heating?
Sol. If no gap is left between the iron rails, the rails may bend Sol. When the metal sheet is heated, it expands as a whole.
due to expansion in summer and the train may get derailed. Therefore, the holes will increase in diameter as well as move
outwards. The distance BC between the two holes thus
Example 6 Pendulum clocks generally run fast in increases.
winter and slow in summer. Why?
Example 12. Out of the parameters-temperature,
Sol. The time period of a simple pendulum is given by
pressure, work and volume, which parameter does not
l
T = 2π i.e. T ∝ l characterize the thermodynamic state of matter?
g Sol. Work
In winter, l decreases with the fall in temperature, so T
decreases and clocks run fast. In summer, l increases with Example 13. What is the nature of total internal
the increase in temperature, so T increases and clock energy possessed by molecules of an ideal gas?
runs slow.
Sol. Since, in an ideal gas there is no molecular attraction, the
Example 7 Why iron rims are heated red hot before potential energy of the molecules is zero. Therefore, internal
energy of an ideal gas is only kinetic in nature.
being put on the cart wheels?
Sol. The iron ring to be put on the rim of a cart wheel is always of Example 14. A piece of metal is hammered. Does
slightly smaller diameter than that of the wheel. When the iron its internal energy increase?
ring is heated to become red hot, it expands and slips on to the
Sol. The work done during hammering gets converted into heat
wheel easily. When it is cooled, it contracts and grips the wheel
firmly. energy. Due to this, its internal energy increases.

Example 8 Explain why a beaker filled with water Example 15. What are the conditions for
at 4°C overflows if the temperature is decreased or thermodynamic equilibrium?
increased? Sol. 1. Temperature of every part of the system be the same.
Sol. It is because of the anomalous expansion of water. Water 2. There should be no net unbalanced force on a part
or whole of the system.
has a maximum density at 4°C . Therefore, water expands
3. There should be no changes due to chemical
whether it is heated above 4°C or cooled below 4°C. reactions.
620 Objective Physics Vol. 1

Sol. When the gas expands at constant temperature, the


Example 16. A gas has two specific heats whereas
average kinetic energy of the gas molecules remains the same.
a liquid and a solid have only one. Why? However, due to increase in volume of the gas, separation
Sol. When solids and liquids are heated, there is only a slight between the molecules increases.
change in their volume and as such these possess only one As a result, the number of molecules colliding per second
specific heat, i.e. specific heat at constant volume. But in case against the walls decreases. Consequently, less momentum
of gases, pressure and volume both change and as such these is transferred to the walls of the container per second.
possess two principal specific heats, one at constant pressure Hence, the pressure exerted by the gas decreases.
and one at constant volume.
Example 24. Why temperature less than absolute
Example 17. How does the internal energy of an zero is not possible?
ideal gas differ from that of real gas?
Sol. According to the kinetic interpretation of temperature,
Sol. The internal energy of an ideal gas consists of only the absolute temperature ∝ mean kinetic energy of molecules As
kinetic energy of the particles. But for real gases it consists of the temperature of a gas is decreased, the average speed of the
both the kinetic as well as potential energies. gas molecules also decreases.
Example 18. Out of a solid, liquid and gas of the At absolute zero the average speed of the gas molecules
becomes zero, i.e. energy is possible. Hence, the
same mass and at the same temperature, which one has temperature of the gas cannot be decreased to a
the greatest internal energy? Which one least? Justify . temperature below absolute zero.
Sol. The gas has greatest internal energy because the potential
energy (which is negative) of the molecules is very small. On Example 25. Briefly explain, why there is
the other hand, the (negative) potential energy of the practically no atmosphere on the surface of the moon?
molecules of a solid is very large, hence the internal energy of
Sol. The molecules of a gas are always in random motion and
a solid is least.
they possess velocities of the order of a few kms −1 . The velocity
Example 19. Two vessels separately contain of the molecules is quite below the escape velocity on the
500 cm3 of hydrogen and 500 cm3 of oxygen at N.T.P. surface of the earth and hence they are retained in the earth's
Which will have larger number of molecules? Give atmosphere.
reasons. But at the surface of the moon, the escape velocity is very
Sol. Both the gases will have equal number of molecules. It is in low (≈ 2.5 ms −1 ) and the gas molecules having velocity
accordance with Avogadro's hypothesis. greater than this value are bound to leave the moon's
atmosphere.
Example 20. Does the value of degree of freedom
Example 26. Why do the gases at low temperature
of a gas molecule change with rise in temperature?
and high pressure show large deviations from ideal
Sol. Yes. behaviour?
Example 21. Cooking gas containers are kept in a Sol. At low temperature and high pressure, the intermolecular
lorry moving with uniform speed. What will be the attractions become appreciable. Moreover, the volume
effect on temperature of the gas molecules inside? occupied by the gas molecules cannot be neglected in
Sol. As the lorry is moving at a uniform speed, the translational comparison to the volume of the gas. Hence, the real gases
motion of the gas molecules will not be affected. Hence, the show large deviations from ideal gas behaviour.
temperature of the gas molecules will remain the same.
Example 29. When an automobile travels for a
Example 22. Molecular motion ceases at zero long distance, the air pressure in the tyres increases
kelvin. Explain. slightly. Why?
Sol. All molecular motion ceases at 0 K, i.e. at absolute zero. Sol. Due to the friction between the tyres and the road, the tyres
According to the kinetic interpretation of temperature, get heated. The temperature of air inside the tyres increases.
absolute temperature ∝ mean kinetic energy of molecules Consequently, the air pressure in the tyres increases slightly.
Therefore, at temperature = 0 K, kinetic energy = 0
Hence, at 0 K, the velocity of molecules also become zero. Example 28. In the kinetic theory of gases, why do
we not take into account the changes in gravitational
Example 23. When a gas expands at constant potential energy of the molecule?
temperature, why does the pressure decrease? Explain Sol. The changes in gravitational potential energy are negligibly
it on the basis of kinetic theory of gases. small as compared to the mean kinetic energy of molecules.
Thermometry, Thermal Expansion and Kinetic Theory of Gases 621

Sol. Volume of the liquid overflown


Example 29. Find the rms speed of hydrogen
molecules at room temperature ( = 300 K ). = Increase in the volume of the liquid
− Increase in the volume of the container
Sol. Mass of 1 mole of hydrogen gas = 2 g = 2 × 10 −3 kg
= [V 0 (1 + γ l ∆T ) − V 0 ] − [V 0 (1 + γ g ∆T ) − V 0 ]
3RT 3 × 8.31 × 300
⇒ v rms = = = V 0 ∆T (γ l − γ g ) = V 0 ∆T (γ l − 3α g ) (∴ γ g ≈ 3 α g )
M 2 × 10 −3
Example 34. Find the temperature at which oxygen
= 1.93 × 10 3 m/s
molecules would have the same rms speed as of
Example 30. 4 g hydrogen is mixed with 11.2 L of hydrogen molecules at 300 K.
He at STP in a container of volume 20 L. If the final Sol. If T be the corresponding temperature,
temperature is 300 K, find the pressure. 3RT 3R (300 ) M 
= ⇒ T = (300 )  O 
Sol. 4 g hydrogen = 2 moles of hydrogen MO MH  MH 
1
11.2 L He at STP = mole of He = 4800 K
2
RT Example 35. A platinum resistance thermometer
p = p H + p He = (n H + n He )
V reads 0°C when its resistance is 80 Ω and 100°C when
 1  8.31 × (300 K ) its resistance is 90 Ω. Find the temperature at which the
= 2 +  resistance is 86 Ω.
 2 (20 × 10 −3 ) m 3
Sol. The temperature on the platinum scale is
= 3.12 × 10 5 N/m2
R − R0 86 − 80
t= t × 100 ° = × 100 ° C
Example 31. Find the average kinetic energy per R100 − R 0 90 − 80
molecule at temperature T for an equimolar mixture of = 60 ° C
two ideal gases A and B, where A is monoatomic and B
is diatomic. Example 36. A sphere of diameter 7 cm and mass
Sol. Number of degrees of freedom per molecule for 266.5 g floats in a bath of liquid. As the temperature is
A=3 raised, the sphere just sinks at a temperature of 35°C.
Number of degrees of freedom per molecule for B = 5 If the density of the liquid at 0°C is 1.527 gm/cm 3 , find
Since, the mixture is equimolar, the average kinetic the coefficient of cubical expansion of the liquid.
energy per molecule will be the simple average of the two Sol. The sphere will sink in the liquid at 35°C, when its density
 3 + 5 becomes equal to the density of liquid at 35° C.
values, i. e.   kT = 4 kT,
 2  The density of sphere,
266.5
where k is Boltzmann constant. ρ 35 = 3
= 1.483 g/cm 3
4  22  7 
×  × 
Example 32. The steam point and the ice point of a 3  7   2
mercury thermometer are marked as 80° and 10°. At Now, ρ 0 = ρ 35 [1 + γ∆T ]
what temperature on centigrade scale the reading of 1.527 = 1.483 [1 + γ × 35]
this thermometer will be 59° ? 1. 029 = 1 + γ × 35
Sol. Let the relation between the thermometer reading and 1. 029 − 1
centigrade be y = ax + b γ= = 0. 00083 / ° C
35
Given, at x = 100 , y = 80 and at x = 0 , y = 10
∴ 80 = 100 a + b , 10 = b
⇒ a = 0.7 Example 37. A light steel wire of length l and area
Now, we have to find x when y = 59 of cross-section A is hanging vertically downward with
∴ 59 = 0.7x + b ⇒ x = 70 a ceiling. It cools to the room temperature 30°C from
∴ The answer is 70° C. the initial temperature 100°C. Calculate the weight
which should be attached at its lower end such that its
Example 33. A glass vessel of volume V 0 is length remains same. Young’s modulus of steel is Y and
completely filled with a liquid and its temperature is coefficient of linear expansion is α.
raised by ∆T. What volume of the liquid will overflow? wl
Sol. lα ∆T =
Coefficient of linear expansion of glass = α g and AY
coefficient of volume expansion of the liquid = γ l . ∴ w = AYα∆T = 70 AαY
622 Objective Physics Vol. 1

Example 38. An air bubble starts rising from the (ii) also calculate the corresponding energies per mole of
bottom of a lake. Its diameter is 3.6 mm at the bottom the gas.
and 4 mm at the surface. The depth of the lake is Sol. (i) According to the kinetic theory, the average kinetic
energy of translation per molecule of an ideal gas at kelvin
250 cm and the temperature at the surface is 40°C.  3
What is the temperature at the bottom of the lake? temperature T is   kT, where k is Boltzman's constant.
 2
Given atmospheric pressure = 76 cm of Hg and
g = 980 cm/s 2 . At 0 ° C (T = 273 K ), the kinetic energy of translation
3 3
Sol. At the bottom of the lake, volume of the bubble = kT = × ( 1.38 × 10 −23 ) × 273
2 2
4 3 4
V1 = πr1 = π (0.18) 3 cm 3 = 5.65 × 10 −21 J/molecule
3 3
At 100 ° C (T = 373 K ), the energy is
Pressure on the bubble, p1 = atmospheric pressure + 3
pressure due to a column of 250 cm of water × ( 1.38 × 10 −23 ) × 373 = 7.72 × 10 −21 J/mol
= 76 × 13.6 × 980 + 250 × 1 × 980 2
= ( 76 × 13.6 + 250 ) 980 dyne/cm 2 (i) 1 mole of gas contains N (= 6.02 × 10 23 ) mol
At the surface of the lake, volume of the bubble Therefore, at 0° C,the kinetic energy of translation of
4 4 1 mole of the gas is
V 2 = πr23 = π (0.2) 3 cm 3
3 3 = (5.65 × 10 −21 ) (6.02 × 10 23 )
Pressure on the bubble, ≈ 3401 J/mol and at 100° C
p 2 = atmospheric pressure
The kinetic energy of translation of 1 mol of gas is
= (76 × 13.6 × 980 ) dyne/cm 2
= ( 7.72 × 10 −21 )(6.02 × 10 23 ) ≈ 4647 J/mol
T 2 = 273 + 40 ° C = 313° K
p1V1 p 2 V 2
Now, = Example 40. One mole of an ideal monoatomic
T1 T2 gas is taken at a temperature of 300 K. Its volume is
 4
( 76 × 13.6 + 250 ) 980 ×   π (0.18) 3 doubled keeping its pressure constant. Find the change
 3 in internal energy.
or =
T1
Sol. Since, pressure is constant
 4 ∴ V ∝T
( 76 × 13.6 ) × 980   π (0.2 ) 3
 3
= ∴
Vi V f
=
313 Ti T f
or T1 = 283.37 K
∴ T1 = 283.37 − 273 = 10.37 ° C Vf
∴ Tf = Ti
Vi
Example 39. Given, Avogadro's number
⇒ T f = 2 Ti = 600 K
N = 6.02 × 10 23 and Boltzmann constant f
∴ ∆U = n ⋅ R∆T
k = 1.38 × 10 −23 J/K. Calculate 2
(i) the average kinetic energy of translation of the 3
= R (600 − 300 ) = 450 R
molecules of an ideal gas at 0°C and at 100°C. 2
NCERT Selected Questions
Q 1. The triple point of neon and carbon dioxide are and value of temperature TB on absolute scale
24.57 K and 216.55 K, respectively. Express 273.16
= × TB
these temperatures on the celsius and fahrenheit 350
scales. As TA and TB are the same temperature.
273.16 273.16
Sol. On Celsius Scale ∴ × TA = × TB
C − 0 T − 273.15 200 350
= 200 4
100 − 0 100 or TA = × TB = TB
350 7
or C = T − 273.15
For neon, t1 ° C = 24.57 − 273.15 = − 248.48° C Q 3. A steel tape 1 m long is correctly calibrated for a
For CO2, t2 ° C = 216.55 − 273.15 = − 56.6° C temperature of 27.0 °C. The length of steel rod
measured by this tape is found to be 63.0 cm on a
On Fahrenheit Scale
hot day when the temperature is 45.0 °C. What is the
F − 32 T − 273.15
= actual length of the steel rod on that day? What is
180 100 the length of the same steel rod on a day when
9 temperature is 27.0°C? Coefficient of linear
For neon, F1 = (T1 − 273.15) × + 32
5 expansion of steel = 1.20 × 10 −5 K −1 .
9
= ( 24.57 − 273.15) × + 32 Sol. Change in length, ∆l = α l0 ∆t = 1.2 × 10− 5 × 63 × 18 cm
5
9 (Q ∆T = 45 − 27 = 18 ° C)
= − 248.58 × + 32
5 = 0.0136 cm
= − 415.44 ° F ∴ The actual length of the steel rod at 45° C.
For CO2,
9
F2 = (T2 − 273.15) × + 32 = l0 + ∆l 63 + 0.0136 = 63.0136 cm
5 The data is correct only up to three significant figures. The
9 actual length of the rod is 63.0 cm, but the change in length
= (216.55 − 273.15) + 32
5 of the rod is 0.0136 cm.
9 On a day when temperature is 27 °C, the size of 1 cm mark
= − 56.6 × + 32
5 on the steel tape will be exactly 1 cm (due to calibration of
= − 69.88 ° F steel tape at 27 °C)
∴ Length of rod at 27°C = 63.0 × 1 cm = 63.0 cm
Q 2. Two absolute scales A and B have triple points of
water defined to be 200 A and 350 B. What is the Q 4. A large steel wheel is to be fitted on a shaft of the
relation between T A and?TB same material. At 27°C, the outer diameter of the
shaft is 8.70 cm and the diameter of the central hole
Sol. Here, triple point of water on absolute scale A = 200 A of the wheel is 8.69 cm. The shaft is cooled using
and triple point of water on absolute scale B = 350 B
‘dry ice’. At what temperature of the shaft does the
wheel slip on the shaft? Assume the coefficient on
Also, we know that triple point of water on absolute scale linear expansion of the steel to be constant over the
= 273.16 K required temperature range
Thus, it follows that temperature 200 A and 350 B on
α steel = 1.20 × 10 −5 K −1
absolute scale A are equivalent to temperature 273.16 on
absolute scale, i.e. the absolute scales measure the triple
Sol. When the shaft is cooled, its linear dimension, i.e. its
point as 200 A and 350 B.
diameter decreases according to the formula
∴ Size of one degree of kelvin scale on absolute scale d2 = d1 [1 + α (T1 − T1 )]
27316
. 27316
.
A= or 1A = K T1 = 27° C = 300 K
200 200 8.69 = 8.70 [1 + 1.20 × 10−5 (T2 − 300)]
and size of one degree of Kelvin scale onabsolute scale 8.69 − 8.70
27316. 27316
. or T2 − 300 = = − 95.78 K
B= or 1B = K 8.70 × 120
. × 105
350 350
or T2 = 300 − 95.78 = 204.22 K
Value of temperature TA on absolute scale
27316
. = 204 .22 − 27315
. = − 68.93° C
= × TA or T2 ≈ − 69° C
200
624 Objective Physics Vol. 1

Q 5. A hole is drilled in a copper sheet. The diameter of Q 7. A brass rod of length 50 cm and diameter 3.0 mm is
the hole is 4.24 cm at 27.0°C. What is the change in joined to a steel rod of the same length and
the diameter of the hole when the sheet is heated to diameter. What is the change in length of the
227°C? Coefficient of linear expansion of copper combined rod at 250°C, if the original lengths are at
= 1.70 × 10 −5 ° C −1 . 40°C? Is there a ‘thermal stress’ developed at the
junction? The ends of the rod are free to expand.
Sol. Let β be the coefficient of superficial expansion of copper, (coefficient of linear expansion of brass
then β = 2α = 2 × 1.7 × 10−5 = 3.4 × 10−5 ° C − 1. = 2.0 × 10 −5 K −1 , steel = 1.2 × 10 −5 K −1 .)
If S 1 , S 2 be the surface areas of the hole at 27°C and 227 °C, Sol. For brass rod,
then ∆t = t2 − t1 = 250 − 40 = 210° C
πd12 π If l2 be its length at t2 ° C, then
S1 = = × (4.24 )2
4 4 l2 = l1 (1 + α∆t )
= 4.494 π cm 2 = 50 (1 + 2 × 10−5 × 210)
∴ S 2 = S 1 (1 + β ∆t ) = 50.21 cm
= 4.494 π (1 + 3.40 × 10−5 × 200) ∴ ∆lbrass = l2 − l1 = 50.21 − 50
or S 2 = 4.494 π × 1.0068 = 4.525 π cm 2 = 0.21 cm
πd22 For steel rod,
or = 4.525 π ∆t′ = t′2 − t′1 = 250 − 40 = 210° C
4
or d2 = 4.525 × 4 = 4.2544 cm If l2′ be the length of the steel rod at 250°C, then
l2′ = l1′ (1 + α ∆t′ )
∴ ∆d = d2 − d1
= 4.2544 − 4.24 = 0.0144 cm . × 10−5 × 210)
= 50 (1 + 12
or ∆d = 1.44 × 10−2 cm = 50.126 cm
∴ ∆lsteel = l′2 − l′1 = 50.126 − 50
Q 6. A brass wire 1.8 m long at 27 °C is held taut with = 0.126 cm ≈ 013 . cm
little tension between two rigid supports. If the wire ∴ The length of the combined rod at 250° C
is cooled to a temperature of − 39° C, what is the = l2 + l′ 2
tension developed in the wire, if its diameter is = 50.21 + 50.126
2.0 mm? Coefficient of linear expansion of brass
= 100.336 cm
= 2.0 × 10 −5 K −1 . Young’s modulus of brass
and length of the combined rod at
= 0.91 × 1011 Pa.
40° C = l1 + l′ 1
Sol. If A be the area of cross-section of the wire, then = 50 + 50 = 100 cm
πd 2 ∴ Total change in length of the combined rod
A=
4 = 100.336 − 100
π = 0.336 cm
= × (2.0 × 10−3 )2
4 ≈ 0.34 cm
= 3.142 × 10−6 m 2
No thermal stress is developed at the junction since the rods
If F be the tension developed in the wire, then using the can freely expand.
relation
Q 8. The coefficient of volume expansion of glycerine is
F/A
Y= , we get 49 × 10 −5 K −1 . What is the fractional change in its
∆l / l
density for a 30°C rise in temperatures?
Y A ∆l
F= …(i)
l Sol. Let V0 be the initial volume of glycerine, i.e. at 0°C (dry)
Now, ∆l = l α ∆t = 1.8 × 2 × 10−5 × (− 66) If Vt be its volume at 30°C
= − 0.00237 m = − 0.0024 m Then, Vt = V0 (1 + γ∆t )
− ve sign shows that the length decreases. = V0 (1 + 49 × 10−5 × 30)
Putting values of Y , A , ∆l and l in Eq. (i), we get = V0 (1 + 0.01470) = 1.01470 V0
0.91 × 1011 × 3.142 × 1016 × 24 × 10−4 V0
=
1
F= = 381 N or …(i)
1.8 Vt 1.01470
Thermometry, Thermal Expansion and Kinetic Theory of Gases 625

Let ρ 0 and ρ t be the initial and final densities of the Q 11. An oxygen cylinder of volume 30 L has an initial
glycerine.
m m gauge pressure of 15 atm and a temperature of 27°C.
Then initial density, ρ 0 = and final density, ρ t = After some oxygen is withdrawn from the cylinder,
V0 Vt
the gauge pressure drops to 11 atm and its
where, m = mass of glycerine. temperature drops to 17°C. Estimate the mass of
∆ρ oxygen taken out of the cylinder.
= Fractional change in density
ρ0 . JK −1 mol −1 , molecular mass of O 2 = 32 u.)
( R = 831
1 1
m −  Sol. If n1 be the moles of oxygen gas contained in the cylinder,
ρ − ρ0  Vt V0   V0 
= t = =  − 1 then using the gas equation pV = nRT , we get
ρ0 m  Vt 
pV
V0 n1 = 1 1
RT1
∆ρ  1 
or = − 1 = − 0.0145 15 × 1.013 × 105 × (30 × 10−3 )
ρ 0  1.01470  =
8.31 × 300
Here, negative sign shows that the density decreases with = 18.253
the rise in temperature.
For oxygen, molecular weight
∆ρ
∴ = 0.0145 = 1.45 × 10−2 ≈ 1.5 × 10−2 m = 32 g = 32 × 10−3 kg
ρ0
∴ m1 = n1M
Q 9. Estimate the fraction of molecular volume to the m = 584.1 × 10−3 kg
actual volume occupied by oxygen gas at STP. Take
the radius of an oxygen molecule to be roughly 3 Å. Finally, in the oxygen cylinder let n2 moles of oxygen are left
pV
Sol. A gram mole of oxygen gas occupies a volume at STP, Then, n2 = 2 2
RT2
V = 22400 × 10−6 m 3
(11 × 1.013 × 105 ) × (30 × 10−3 )
r = Radius of an oxygen molecule = 3 Å = 3 × 10−10 m =
8.31 × 290
Molecular volume of one mole of oxygen, V ′ = 13.847
= Volume of one molecule of O2 × Avogadro’s number (N )
∴ Final mass of oxygen gas in the cylinder,
4
= πr3 × N m2 = 13.847 × 32 × 10−3
3
4 = 453.1 × 10−3 kg
or V ′ = π × (3 × 10−10 )3 × 6.023 × 1023
3 ∴ Mass of oxygen taken out of the cylinder
∴ Fraction of molecular volume to the actual volume occupied = m1 − m2
by oxygen = (584.1 − 453.1) × 10−3 kg
V ′ Molecular volume of one mole O2
= = = 141 × 10−3 kg
V Gram molar volume of O2 at STP
= 0.141 kg
4
π × (3 × 10−10 )3 × 6.023 × 1023 m 3
= 3 Q 12. An air bubble of volume 1.0 cm 3 rises from the
22400 × 10−6 m 3 bottom of a lake 40 m deep at a temperature of
−4
= 4 × 10 12°C. To what volume does it grow when it reaches
the surface, which is at a temperature of 35°C?
Q 10. Molar volume is the volume occupied by 1 mole of
any (ideal) gas at standard temperature and pressure Sol. When the air bubble is at a depth of 40 m,
(STP-1 atm pressure, 0°C). Show that it is 22.4 L. Then, p1 = 1 atm + 40 m depth of water
= 1 atm + h1ρg
Sol. At STP,
= 1.013 × 105 + 40 × 103 × 9.8
p = 1 atm= 1.013 × 105 Nm −2
= 493000 Pa = 4.93 × 105 Pa
T = 273 K
When the air bubble reaches at the surface of lake,
∴ Using the gas equation,
then
pV = nRT , we get
nRT 1 × 8.31 × 273 T2 = 35° C
V = = = 35 + 273 = 308 K
p 1.013 × 105
p2 = 1 atm = 1013
. × 105 Pa
= 22.4 × 10−3 m 3 = 22.4 L
626 Objective Physics Vol. 1

p1V1 p2V2 Q 15. Three vessels of equal capacity have gases at the
Using the equation = , we get
T1 T2 same temperature and pressure. The first vessel
pV T contains neon (monoatomic gas) the second
V2 = 1 1 × 2
T1 p2 contains chlorine (diatomic) and the third contains
4.93 × 105 × 1 × 10−6 × 308 uranium hexafluoride (polyatomic). Do the vessels
= contain equal number of respective molecules? Is
285 × 1013
. × 105
the root mean square speed of molecules the same in
= 5.275 × 10−6 m 3 the three cases? If not, in which case is v rms the
≈ 5.3 × 10−6 m 3 largest?

Q 13. Estimate the total number of air molecules Sol. Yes. According to Avogadro's hypothesis, if the condition
(inclusive of oxygen, nitrogen, water vapour and of temperature and pressure remain same, the equal volumes
of all gases have equal number of molecules. So, the number
other constituents) in a room of capacity 25.0 m 3 at
of molecules in the three vessels containing different gases
a temperature of 27°C and 1 atm pressure. must be same i.e. the three vessels contain equal number of
Sol. k = 1.38 × 10−23 JK−1 respective molecules.

p = 1 atm = 1.013 × 105 Pa No. rms speed of molecules is not same in three cases as
explained below.
Now, the gas equation pV = nRT may be written as
nRT n  R  We know that the rms velocity of a gas is given by
p= = (Nk ) T Q = k
V V  N  3RT 1
kT kT
v rms = or v rms ∝ ,
= (nN ) = N′ M M
V V
where, M = molecular mass and is different for different
where, N ′ = nN = total number of air molecules in the gases, therefore vrms of three gases will be different.
given gas.
As vrms is inversely proportional to mass of molecules of the
pV (1013
. × 105 ) × 25
∴ N′ = = = 610
. × 1026 gas and since neon is the lightest of the three gases, therefore
. × 10−23 × 300
kT 138 vrms for neon gas is largest.

Q 14. Estimate the average thermal energy of helium atom Q 16. At what temperature is the root mean square speed
at (i) room temperature (27°C), (ii) the temperature of an atom in an argon gas cylinder equal to the rms
on the surface of the sun (6000 K), (iii) the speed of a helium gas atom at − 20° C? (Atomic mass
temperature of 10 million kelvin (the typical core of Ar = 399
. u of He = 40. u)
temperature in the case of a star).
Sol. Let v1 and v2 be the rms speed of argon and helium gas atoms
Sol. According to kinetic theory of gases, the average kinetic at temperature T1 and T2, respectively.
energy of the gas at a temperature T is given by
Here, M 1 = 39.9 × 10−3 kg
3
E = kT
2 M 2 = 4.0 × 10−3 kg
Here, . × 10−23 JK − 1
k = 138 T2 = − 20 + 273 = 253 K
(i) T = 27° C = 273 + 27 = 300 K 3RT
Now, we know that rms speed is given by v =
3 M
E = × 138. × 10−23 × 300
2 3RT1 3RT2
∴ v1 = and v2 =
= 621 × 10−23 J = 6.21 × 10−21 J M1 M2
(ii) T = 6000 K As v1 = v2 (given)
3
∴ E = × 138. × 10−23 × 6000 = 124
. × 10−19 J 3RT1 3RT2 T1 T
2 ∴ = or = 2
M1 M2 M1 M2
(iii) T = 10 × 106 K = 107 K
3 M1 39.9 × 10−3
∴ E= . × 10−23 × 107
× 138 or T1 = × T2 = × 253
2 M2 4.0 × 10−3
= 2.07 × 10−16 J ≈ 21
. × 10−16 J or T1 = 2523.7 K ≈ 2524 K
Objective Problems
[ Level 1 ]
Temperature Scales, 7. If two rods of lengths Land 2Lhaving coefficient of linear
Thermometers and Thermal expansion α and 2α respectively, are connected
end-on-end, the average coefficient of linear expansion
Expansion of the composite rod, equals
3 5
1. The absolute zero temperature in fahrenheit scale is (a) α (b) α
(a) − 273°F (b) − 32° F 2 2
5
(c) − 460 ° F (d) − 132 ° F (c) α (d) None of these
3
2. A faulty thermometer has its fixed points marked 5 and
95. lf the temperature of a body as shown on the celsius 8. A solid ball of metal has a spherical cavity inside it. If the
ball is heated, the volume of the cavity will
scale is 40, then its temperature shown on this faulty
(a) increase
thermometer is (b) decrease
(a) 39° (b) 40° (c) remain unchanged
(c) 41° (d) 44.4 °
(d) data insufficient
3. A steel rod of diameter 1 cm is clamped firmly at each
end when its temperature is 25°C so that it cannot 9. The radius of a ring is R and its coefficient of linear
contract on cooling.The tension in the rod at 0°C is expansion is α. If the temperature of ring increases by θ,
approximately (α = 10−5 / ° C, Y = 2 × 1011 N/ m 2 ) then its circumference will increase by
(a) 4000 N (a) π Rα θ (b) 2π Rα θ
θ θ
(b) 7000 N (c) π Rα (d) πRα
(c) 7400 N 2 4
(d) 4700 N
10. Two rods of different materials having coefficients of
4. The temperature of a physical pendulum, whose time thermal expansions α 1 , α 2 and Young’s modulli Y1 , Y 2
period is T, is raised by ∆θ. The change in its time period respectively are fixed between two rigid massive walls.
is The rods are heated such that they undergo the same
1 increase in temperature. There is no bending of the rods.
(a) αT∆θ (b) 2 αT∆θ
2 If α 1 : α 2 = 2 : 3, the thermal stresses developed in the
1 two rods are equal provided Y1 : Y 2 is equal to
(c) α ∆θ (d) 2 α ∆θ
2
5. On heating a liquid having coefficient of volume
expansion α in a container having coefficient of linear
expansion α/ 2, the level of the liquid in the container
would
(a) rise
(b) fall
(c) remains almost stationary
(d) cannot be predicted (a) 2 : 3 (b) 1 : 1
(c) 3 : 2 (d) 4 : 9
6. Two rods of length l1 and l2 are made of materials whose
11. The freezing point on a thermometer is marked as − 20°
coefficient of linear expansions are α 1 and α 2 ,
respectively. If the difference between two lengths is and the boiling point as 130°. A temperature of human
independent of temperature, then body ( 34° C) on this thermometer will be read as
l1 α 1 (a) 31° (b) 51°
(a) = (c) 20° (d) None of these
l2 α 2
l α 12. On which of the following scales of temperature, the
(b) 1 = 2
l2 α 1 temperature is never negative?
(c) l22 α 1 = l12 α , (a) Celsius (b) Fahrenheit
α2 α2 (c) Reaumur (d) Kelvin
(d) 1 = 2
l1 l2
628 Objective Physics Vol. 1

13. The temperature of a body on kelvin scale is found to be 20. Bimetal strips are used for
x K. When it is measured by fahrenheit thermometer, it is (a) metal thermometers
found to be x°F, then the value of x is (b) opening of closing electrical circuits
(a) 40 (b) 313 (c) thermostats
(c) 574.25 (d) 301.25 (d) All of the above

14. A device used to measure very high temperature is Gas Laws and Kinetic Theory of
(a) pyrometer (b) thermometer
(c) bolometer (d) calorimeter
Gases
21. What will be the temperature when the rms velocity is
15. A uniform metal rod is used as a bar pendulum. If the double of that at 300 K?
room temperature rises by 10° C, and the coefficient of (a) 300 K (b) 600 K (c) 900 K (d) 1200 K
linear expansion of the metal of the rod is 2 × 10−6 per° C,
22. By what factor the rms velocity will change, if the
the period of the pendulum will have percentage temperature is raised from 27°C to 327°C?
increase of
(a) 2 (b) 2 (c) 2 2 (d) 1
(a) − 2 × 10−3 (b) − 1 × 10−3
(c) 2 × 10−3 (d) 1 × 10−3 23. If Maxwell distribution is valid and v p denotes the most
probable speed, v the average speed and v rms the
16. In a vertical U-tube containing a liquid, the two arms root-mean-square speed, then
are maintained at different temperatures t 1 and t 2 . The (a) v < vp < vrms (b) v < vrms < vp
(c) vp < v < vrms (d) vp < vrms < v
liquid columns in the two arms have heights l1 and l2 ,
respectively. The coefficient of volume expansion of 24. The root mean square (rms) speed of oxygen molecules
the liquid is equal to ( O2 ) at a certain absolute temperature is v. If the
temperature is doubled and the oxygen gas dissociates
into atomic oxygen, the rms speed would be
(a) v (b) 2v (c) 2 v (d) 2 2 v
t1
25. The number of molecules in 1 cc of water is closed to
1023
(a) 6 × 1023 (b) 22.4 × 1024 (c) (d) 1023
3
t2
l1
26. 16 g of oxygen, 14 g of nitrogen and 11 g of carbon
l2 dioxide are mixed in an enclosure of volume 5 L and
temperature 27°C. The pressure exerted by the mixture is
(a) 4 × 105 Nm −2 (b) 5 × 105 Nm −2
(c) 6 × 105 Nm −2 (d) 9 × 105 Nm −2
l1 − l2 l1 − l2
(a) (b) 27. 22 g of CO2 at 27°C is mixed in a closed container with
l2t1 − l1t2 l1t1 − l2t2
l + l2 l + l2 16 g of O2 at 37°C. If both gases are considered as ideal
(c) 1 (d) 1 kinetic theory gases, then the temperature of the
l2t1 + l1t2 l1t1 + l2t2
mixture is
17. The coefficient of linear expansion of crystal in one (a) 28.5°C (b) 30.5°C (c) 31.5°C (d) 33.0°C
direction is α 1 and that in other two directions
28. The ratio of the speed of sound in nitrogen gas to that in
perpendicular to it is α 2 . The coefficient of cubical
helium gas, at 300 K is
expansion is
(a) α 1 + α 2 (b) 2α 1 + α 2  2  1 3 6
(a)   (b)   (c) (d)
(c) α 1 + 2α 2 (d) None of these  7  7 5 5
18. The absolute zero temperature in fahrenheit scale is 29. The velocities of three molecules are 3v, 4v and 5v
(a) − 237° F (b) − 32° F
(c) − 460° F (d) − 132° F
respectively. Their rms speed will be
50 5 7 5
(a) v (b) v (c) v (d) v
19. The reading of air thermometer at 0° C and 100° C are 3 2 2 2
50 cm and 75 cm of mercury column, respectively. The
temperature at which its reading is 80 cm of mercury 30. The number of molecules per unit volume of a gas is
column is given by
(a) 105°C (b) 110°C p kT p RT
(a) (b) (c) (d)
(c) 115°C (d) 120°C kT p RT p
Thermometry, Thermal Expansion and Kinetic Theory of Gases 629

31. The temperature at which the root mean square speed of a 39. The below figure shows graph of pressure and volume of
gas will be half its value at 0°C is (assume the pressure a gas at two temperatures T1 and T2 . Which of the
remains constant) following is correct?
(a) −86.4 ° C (b) −204.75° C p
(c) −104.75° C (d) − 68.25° C

32. Four molecules of a gas have speeds 1, 2, 3 and 4 km s −1 .


The value of the root mean square speed of the gas
T2
molecules is
1 1 T1
(a) 15 kms−1 (b) 10 kms−1
2 2 V

(c) 2.5 kms−1 (d)


15
kms−1 (a) T1 > T2
2 (b) T1 = T2
(c) T1 < T2
33. In the gas below pV = RT , V refers to the volume of (d) Nothing can be said about temperatures
(a) any amount of gas 40. The figure shows pressure versus density graph for an
(b) one gram of gas
(c) one gram mole of gas ideal gas at two temperatures T1 and T2 .
(d) one litre of the gas p
T1
34. In the adjacentV -T diagram, what is the relation between
T2
p1 and p 2 ?
V
p2

p1
ρ

The relation between T1 and T2 is


(a) T1 > T2
(b) T1 = T2
T
(c) T1 < T2
(a) p2 = p1 (b) p2 > p1 (d) Nothing can be predicted
(c) p2 < p1 (d) Cannot be predicted
41. From the p - T graph what conclusion can be drawn?
35. The molecules of a given mass of gas have rms speed
T
200 ms −1 at 27°C and 105 Nm −2 pressure. When the V2

absolute temperature is doubled and the pressure is


halved, the rms speed of the molecules of the same gas is
V1
(a) 200 ms−1 (b) 400 ms−1 θ2
(c) 200 2 ms−1 (d) 400 2 ms−1 θ1
p
36. The average speed of molecules in a gas is given by
(a) V2 = V1 (b) V2 < V1
3p γp (c) V2 > V1 (d) Nothing can be predicted
(a) (b)
ρ ρ
42. pV versus T graph of equal masses of H2 , He and O2 is
2p 8p
(c) (d) shown in figure. Choose the correct alternative.
ρ πρ
pV C
37. The most probable speed of molecules in a gas is given
B
by
3p 8p A
(a) (b)
ρ πρ
2p γp
(c) (d)
ρ ρ O T

38. A perfect gas at 27°C is heated at constant pressure so as (a) A corresponds to H2 , B to He and C to O2
to triple its volume. The temperature of the gas will be (b) A corresponds to He, B to H2and C to O2
(a) 81°C (b) 900°C (c) A corresponds to He, B to O2 and C to H2
(c) 627°C (d) 450°C (d) A corresponds to O2 , B to H2 and C to He
630 Objective Physics Vol. 1

43. Pressure versus temperature graph of an ideal gas at 47. By what factor, the rms velocity will change, if the
constant volume V is shown by the straight line A. Now, temperature is raised from 27°C to 327°C?
mass of the gas is doubled and volume is halved, then the (a) 2 (b) 2
corresponding pressure versus temperature graph will be (c) 2 2 (d) 1
shown by the line 48. Two different gases of molecular masses M 1 and M 2 are
p B
at the same temperature. What is the ratio of their mean
A
square speeds?
C (a) M 1 / M 2 (b) M 2 / M 1
(c) M 1 / M 2 (d) M 2 / M 1
O T
49. In a mixture of gases, the average number of degree of
(a) A (b) B
(c) C (d) None of these freedoms per molecule is 6. The rms speed of the
molecule of the gas is c. The velocity of sound in the
44. Which one of the following graphs represent the gas is
behaviour of an ideal gas at constant temperature? c c
(a) (b)
pV pV 3 2
2c 3c
(c) (d)
(a) (b) 3 3

O V O V Degree of Freedom, Internal


pV pV Energy, Law of Equipartition of
Energy and Molar Heat Capacity
(c) (d)
50. What is the degree of freedom in case of a monoatomic
gas?
O V O V
(a) 1 (b) 3
45. The curve between absolute temperature and v 2rms is (c) 5 (d) None of these
2
vrms
2
vrms
51. Two gases are at absolute temperatures 300 K and 350 K
respectively. Ratio of average kinetic energy of their
molecules is
(a) (b) (a) 7 : 6 (b) 6 : 7 (c) 36 : 49 (d) 49 : 36
52. The specific heat at constant pressure is greater than that
O
T
O T of the same gas at constant volume because
2 (a) at constant volume work is done in expanding the gas
vrms 2
vrms (b) at constant pressure work is done in expanding the gas
(c) the molecular attraction increases more at constant pressure
(d) the molecular vibration increases more at constant pressure
(c) (d)
53. The average translational kinetic energy of O2 (molar
O
T
O
T mass 32) at a particular temperature is 0.048 eV. The
average translational kinetic energy of N2 (molar
46. Volume-temperature graph at constant pressure for a mass 28) molecules in eV at the same temperature is
monoatomic gas (V in m 3 , T in ° C ) is (a) 0.0015 (b) 0.003
V V (c) 0.048 (d) 0.768
54. Each molecule of a gas has f degrees of freedom. The
(a) (b) Cp
ratio = γ for the gas is
CV
T(ºC) T(ºC) f 1 2 f
(a) 1 + (b) 1 + (c) 1 + (d)
V V 2 f f 2

(c) (d) 55. Two moles of argon are mixed with one mole of
Cp
hydrogen, then for the mixture is nearly
CV
T(ºC) T(ºC) (a) 1.2 (b) 1.3 (c) 1.4 (d) 1.5
Thermometry, Thermal Expansion and Kinetic Theory of Gases 631

56. The ratio of specific heats γ of an ideal gas is given by 63. The mean kinetic energy of one mole of gas per degree of
1 R freedom is
(a) (b) 1 +
R CV 1 3
1− (a) kT (b) kT
Cp 2 2
Cp 3 1
(c) (d) All of these (c) RT (d) RT
Cp − R 2 2
64. The degrees of freedom of a molecule of a non-linear
57. A gas has volume V and pressure p. The total triatomic gas is (ignore vibrational motion)
translational kinetic energy of all the molecules of the (a) 2 (b) 4
gas is (c) 6 (d) 8
3
(a) pV only if the gas is monoatomic 65. A gas mixture consists of 2 moles of O2 and 4 moles of
2
3 Ar at temperature T. Neglecting all vibrational modes, the
(b) pV only if the gas is diatomic
2 total internal energy of the system is
3 (a) 4RT (b) 15RT
(c) pV in all cases
2 (c) 9RT (d) 11RT
(d) None of the above Cp
66. The ratio = γ for a gas. Its molecular weight is M. Its
58. The number of translational degree of freedom for a CV
diatomic gas is specific heat capacity at constant pressure is
(a) 2 (b) 3 (c) 5 (d) 6 R γR
(a) (b)
59. A vessel contains a mixture of one mole of oxygen and γ −1 γ −1
γR γRM
two moles of nitrogen at 300 K. The ratio of the average (c) (d)
M (γ − 1) (γ − 1)
rotational kinetic energy per O2 molecule to that per N2
molecules is 67. At 27°C temperature the kinetic energy of an ideal gas is
(a) 1 : 1 (b) 1 : 2
E1 . If the temperature is increased to 327°C, then the
(c) 2 : 1 (d) 8 : 7
kinetic energy will be
60. Internal energy of two moles of an ideal gas at a E1
(a) (b) 2 E1
temperature of 127°C is 1200 R. Then, the specific heat 2
of the gas at constant pressure is E1
(c) 2E1 (d)
(a) 0.5 R (b) 0.1 R 2
(c) 1.5 R (d) 2.5 R
68. The translational kinetic energy of 1 g molecule of a gas,
61. Choose the correct option related to the specific heat at at temperature 300 K is ( R = 8.31 J/ mol K )
constant volume CV (a) 3.4 × 103 J
1 R (b) 2.97 × 103 J
(a) CV = f R (b) CV =
2 γ −1 (c) 1.2 × 102 J
Cp (d) 0.66 × 103 J
(c) CV = (d) All of these
γ 69. For a gas, if the ratio of specific heats at constant pressure
p and constant volumeV is γ, then the value of degree of
62. The graph which represents the variation of mean kinetic
freedom is
energy of molecules with temperature t ° C is
γ+1 γ −1
(a) (b)
E E γ −1 γ+1
1 2
(c) (γ − 1) (d)
(a) (b) 2 γ −1

70. If the ratio of specific heats of a gas at constant pressure


t ºC t ºC to that at constant volume is γ, the change in internal
E E energy of a gas, when the volume changes fromV to 2V at
constant pressure p is
(c) (d) R
(a) (b) pV
(γ − 1)
pV γ pV
t ºC t ºC (c) (d)
(γ − 1) (γ − 1)
632 Objective Physics Vol. 1

Miscellaneous Problems 80. Calorie is defined as the amount of heat required to raise
temperature of 1g of water by 1°C and it is defined under
71. A gas at absolute temperature 300 K has pressure which of the following conditions?
= 4 × 10−10 N/ m 2 . (a) From 14.5°C to 15.5°C at 760 mm of Hg
−23 (b) From 98.5°C to 99.5°C at 760 mm of Hg
Boltzmann constant, k = 1.38 × 10 J/ K. The number (c) From 13.5°C to 14.5°C at 76 mm of Hg
of molecules per cm 3 is of the order of (d) From 3.5°C to 4.5°C at 76 mm of Hg
(a) 100 (b) 105 (c) 108 (d) 1011 81. A block of mass 100 g slides on a rough horizontal
u surface. If the speed of the block decreases from
72. The energy density of an ideal diatomic gas is related to
its pressure p as V 10 m/ s and 5 m/ s, the thermal energy developed in the
u u 3 process is
(a) = 3p (b) = p
V V 2 (a) 3.75 J (b) 37.5 J
u p u 5 (c) 0.375 J (d) 0.75 J
(c) = (d) = p
V 3 V 2
82. SI unit of universal gas constant is
73. If a 5 kg body falls to the ground from a height of 30 m (a) cal /° C (b) J / mol
and if all of its mechanical energy is converted into heat. (c) J / mol − 1K − 1 (d) J / kg
The heat produced will be ( g = 10 m/ s 2 )
83. The density of a gas at normal pressure and 27°C
(a) 359 cal (b) 150 cal
temperature is 24 units. Keeping the pressure constant,
(c) 60 cal (d) 254 cal
the density at 127°C will be
74. A cylinder contains 20 kg of N2 gas ( M = 28 kg / kmol ) at (a) 6 units (b) 12 units
a pressure of 5 atm. The mass of hydrogen (c) 18 units (d) 24 units
( M = 2 kg / kmol ) at a pressure of 3 atm contained in the pV
84. The gas equation = constant is true for a constant
same cylinder at same temperature is T
(a) 1.08 kg (b) 0.86 kg mass of an ideal gas undergoing
(c) 0.68 kg (d) 1.68 kg
(a) isothermal change
75. If the density of a gas at NTP is 1.3 kg / m 3 and velocity of (b) adiabatic change
(c) isobaric change
sound in it is 330 m/ s. The number of degrees of freedom (d) any type of change
of gas molecule is
(a) 2 (b) 3 (c) 6 (d) 5 85. A vessel contains 1 mole of O2 gas (molar mass 32) at a
76. A gas is found to obey the law p 2V = constant. The initial temperature T. The pressure of the gas is p. An identical
vessel containing one mole of He gas (molar mass 4) at
temperature and volume are T0 andV0 . If the gas expands
temperature 2T has a pressure of
to a volume 3V0 , its final temperature becomes
T0 T0 (a) p/8 (b) p
(a) (b) (c) 2p (d) 8p
3 3
(c) 3T0 (d) None of these 86. When volume of an ideal gas is increased two times and
temperature is decreased half of its initial temperature,
77. Identify which pair of state parameters can completely
then pressure becomes
describe the system.
(a) 2 times (b) 4 times
(a) p and V (b) p and ρ 1 1
(c) p andU (d) All of these (c) times (d) times
4 2
78. 5 L of benzene weight
87. Relationship between p, V and E for a gas is
(a) more in summer than in winter
(b) more in winter than in summer (E = total translational kinetic energy)
(c) equal in winter than in summer 3
(d) None of the above (a) p = EV
2
2
79. A beaker is completely filled with water at 4° C. It will (b) V = Ep
overflow if 3
3
(a) heated above 4°C (c) pV = E
(b) cooled below 4°C 2
2
(c) both heated and cooled above and below 4°C, respectively (d) pV = E
(d) None of the above 3
Thermometry, Thermal Expansion and Kinetic Theory of Gases 633

88. Pressure versus temperature graph of an ideal gas is as 90. Two different masses m and 3m of an ideal gas are heated
shown in figure. Density of the gas at point A is ρ 0 . separately in a vessel of constant volume. The pressure p
Density at B will be and absolute temperature T graphs for these two cases are
p shown in the figure as A and B. The ratio of slopes of
3p0 B
curves B to A is
(a) 3:1
p0 A (b) 1:3
(c) 9:1
(d) 1:9
T0 2T0 T 91. Each atom of mass m of a monoatomic gas has got three
3 3 4 degrees of freedom. The rms velocity of these atoms is v
(a) ρ 0 (b) ρ 0 (c) ρ 0 (d) 2 ρ 0
4 2 3 at temperature T. For a diatomic molecule of mass m and
temperature T which has got five degrees of freedom, rms
89. Two different isotherms representing the relationship velocity of molecule is
between pressure p and volumeV at a given temperature
p B
of the same ideal gas are shown for masses m1 and m2 ,
then
3m A
p
m
m2 T
m1 5
(a) v
3
3
(b) v
V 5
(a) m1 > m2 (b) m1 = m2 (c) v
(c) m1 < m2 (d) Nothing can be predicted (d) None of the above

[Level 2]
Only One Correct Option
1. The mass of hydrogen molecule is 3.32 × 10−27 kg . If 2. A ring shaped tube contains two ideal gases with equal
1023 hydrogen molecules strike per second at 2 cm 2 area masses and relative molar masses M 1 = 32 and M 2 = 28.
The gases are separated by one fixed partition p and
of a rigid wall at an angle of 45° from the normal and
another movable stopper S which can move freely
rebound back with a speed of 1000 m/ s, then the pressure
without friction inside the ring.
exerted on the wall is M1

P
α
θ S
Wall
θ

M2

(a) 2.34 × 10 Pa
3
The angle α in equilibrium as shown in the figure
(b) 0.23 × 106 Pa (in degrees) is
(c) 0.23 × 103 Pa (a) 291 (b) 219
(d) 23.4 × 103 Pa (c) 129 (d) 192
634 Objective Physics Vol. 1

3. A cylindrical tube of uniform cross-sectional area A is 8. A cyclic process 1-2-3-4-1 is depicted on V-T diagram.
fitted with two air tight frictionless pistons. The p-T diagram and p-V diagrams for this cyclic process
are given below. Select the incorrect choices.
V 4
wire

3
1
2
The pistons are connected to each other by a metallic T
wire. Initially, the pressure of the gas is p 0 and
temperature is T0 , atmospheric pressure is also p 0 . Now, p p
the temperature of the gas is increased to 2T0 , the tension 2 3 2 3
in the wire will be
p0 A (a) (b)
(a) 2 p0 A (b) p0 A (c) (d) 4 p0 A 1 4
2 1 4

4. If the density of a gas at NTP is 1.3 kg / m 3 and velocity of T T


sound in it is 330 m/ s. The number of degrees of freedom
p
of gas molecule is 4 3
(a) 2 (b) 3 (c) 6 (d) 5
(c) 2 (d) None of these
5. A vertical cylinder closed at both ends is fitted with a
smooth piston dividing the volume into two parts each
1
containing one mole of air. At the equilibrium
T
temperature of 320 K, the upper and lower parts are in the
ratio 4 : 1. The ratio will become 3 : 1 at a temperature of 9. The given curve represents the variations of temperature
(a) 450 K (b) 228 K (c) 420 K (d) 570 K as a function of volume for one mole of an ideal gas.
Which of the following curves best represent the
6. The expansion of an ideal gas of mass m at a constant
variation of pressure as a function of volume?
pressure p is given by the straight line B. Then, the
expansion of the same ideal gas of mass 2m at a pressure T
2 p is given by the straight line. 45º
Volume
A

B V

C p p

(a) (b)

Temperature
V V
(a) C (b) A
p p
(c) B (d) None of these
(c) (d)
7. Pressure versus temperature graph of an ideal gas of
equal number of moles of different volumes are plotted as V V
shown in figure. Choose the correct alternative.
p 4
10. A cyclic process ABCD is shown in the p-V diagram.
Which of the following curves represent the same
3 2
process?
p A B
1
T
(a) V1 = V2 , V3 = V4 and V2 > V3 C
(b) V1 = V2 , V3 = V4 and V2 < V3
(c) V1 = V2 = V3 = V4 D
(d) V4 > V3 > V2 > V1 V
Thermometry, Thermal Expansion and Kinetic Theory of Gases 635

p p 14. The number of molecules per unit volume of a gas is


A B A B
given by
(a) C (b) C p kT
(a) (b)
kT p
D D p RT
(c) (d)
T T RT p

p p 15. A gas has volume V and pressure p. The total


B A B
translational kinetic energy of all the molecules of the
(c) A (d) gas is
3
D C (a) pV only if the gas is monoatomic
D C 2
3
T T (b) pV only if the gas is diatomic
Note: BC and DA are parts of rectangular hyperbola. 2
3
(c) > pV if the gas is diatomic
11. Variation of internal energy with density of one mole of 2
monoatomic gas is depicted in the below figure, 3
(d) pV in all cases
corresponding variation of pressure with volume can be 2
depicted as (Assuming the curve is rectangular
16. Two identical containers joined by a small pipe initially
hyperbola)
contain the same gas at pressure p 0 and absolute
U
temperature T0 . One container is now maintained at the
same temperature while the other is heated to 2T0 . The
common pressure of the gases will be
3 4
(a) p0 (b) p0
2 3
5
ρ
(c) p0 (d) 2 p0
3
p p
17. A cylindrical steel plug is inserted into a circular hole of
diameter 2.60 cm in a brass plate. When the plug and the
(a) (b) plates are at a temperature of 20°C, the diameter of the
plug is 0.010 cm smaller than that of the hole. The
temperature at which the plug will just fit in it is
V V
 11 × 10 −6 19 × 10 −6 
 Given, α steel = and α brass = 
p p  °C °C 
(a) − 48° C (b) − 20° C
(c) (d) (c) − 10° C (d) − 458° C

18. A piece of metal weigh 46 g in air. When it is immersed


V V
in a liquid of specific gravity 1.24 at 27°C, it weigh 30 g.
When the temperature is raised to 42°C, the metal piece
12. A gas at absolute temperature 300 K has pressure weigh 30.5 g. If the specific gravity of the liquid at 42°C
p = 4 × 10−10 N/ m 2 . Boltzmann constant is 1.20, the coefficient of linear expansion of the metal is
k = 1.38 × 10−23 J/ K . The number of molecules per cm 3 1.4 × 10−5 2.3 × 10−5
(a) (b)
is of the order of °C °C
(a) 100 (b) 105 4.3 × 10−5 3.4 × 10−5
(c) (d)
(c) 108 (d) 1011 °C °C

13. The number of molecules in 1 cc of water is closed to 19. A chamber containing a gas was evacuated till the
(a) 6 × 10 23 vacuum attained was 10−14 m of Hg. If the temperature of
(b) 22.4 × 1024 the chamber was 30°C, the number of molecules that
1023 remains in it per cubic metre is
(c)
3 (a) 3.2 × 1011 (b) 3.2 × 1012
(d) 1023 (c) 2.3 × 1012 (d) 2.3 × 1010
636 Objective Physics Vol. 1

20. The apparent coefficient of expansion of a liquid when 24. p -V diagram was obtained from state 1 to state 2 when a
heated in a copper vessel is C and when heated in a silver given mass of a gas is subjected to temperature changes.
vessel is S. If A is the linear coefficient of expansion of During this process, the gas is
copper, then the linear coefficient of expansion of
silver is p
C + S − 3A C + 3A − S 2p0
1
(a) (b)
3 3
S + 3A − C C + S + 3A
(c) (d)
3 3

21. Three rods of equal length l are joined to form an p0 2


equilateral ∆PQR. O is the mid-point of PQ. Distance OR
remains same for small change in temperature. O V0 2V0 V
Coefficient of linear expansion for PR and RQ is same,
i.e. α 2 but that for PQ is α 1 . Then, (a) heated continuously
R (b) cooled continuously
(c) heated in the beginning and cooled towards the end
(d) cooled in the beginning and heated towards the end

25. 1 mole of H2 gas is contained in a box of volume


V = 100
. m 3 at T = 300 K. The gas is heated to a
P O Q
temperature of T = 3000 K and the gas gets converted to
(a) α 2 = 3α 1 (b) α 2 = 4α 1 (c) α 1 = 3α 2 (d) α 1 = 4α 2 a gas of hydrogen atoms. The final pressure would be
22. An ideal gas is initially at temperature T and volumeV. Its (considering all gases to be ideal)
volume is increased by ∆V due to an increase in (a) same as the pressure initially
(b) 2 times the pressure initially
temperature ∆T, pressure remaining constant. The (c) 10 times the pressure initially
quantity δ = ∆V / (V ∆T ) varies with temperature as (d) 20 times the pressure initially
δ δ
More than One Correct Options
(a) (b) 1. During an experiment, an ideal gas is found to obey a
p2
condition = constant. (ρ = density of the gas). The gas
ρ
T T is initially at temperature T, pressure p and density ρ. The
δ δ gas expands such that density changes to ρ/2.
(a) The pressure of the gas changes to 2 p
(c) (d) (b) The temperature of the gas changes to 2 T
(c) The graph of the above process on p-T diagram is parabola
(d) The graph of the above process on p-T diagram is hyperbola
T T 2. During an experiment, an ideal gas is found to obey a
23. p-T diagram was obtained when a given mass of gas was condition Vp 2 = constant. The gas is initially at a
heated. During the heating process from the state 1 to temperature T, pressure p and volume V. The gas expands
state 2, the volume to volume 4V.
p p
2 (a) The pressure of gas changes to
2
(b) The temperature of gas changes to 4T
1 (c) The graph of the above process on p-T diagram is parabola
(d) The graph of the above process on p-T diagram is hyperbola

3. Find the correct options.


(a) Ice point in Fahrenheit scale is 32°F
O T (b) Ice point in Fahrenheit scale is 98.8°F
(a) remain constant (b) decreases (c) Steam point in Fahrenheit scale is 212°F
(c) increases (d) changes erratically (d) Steam point in Fahrenheit scale is 252°F
Thermometry, Thermal Expansion and Kinetic Theory of Gases 637

4. In the T-V diagram shown in figure, choose the correct 2. Assertion Straight line on V-T graph represents
options for the process 1-2. isobaric process.
V 1 Reason If V ∝ T , then p = constant, i.e. process is
2 isobaric.
3. Assertion At the same temperature and pressure equal
volumes of all gases contain equal number of molecules.
T
(a) Density of gas has reduced to half Reason In 1 L at NTP, total number of molecules are
(b) Temperature of gas has increased to two times 6.02 × 1023 .
(c) Internal energy of gas has increased to four times
(d) T-V graph is a parabola passing through origin 4. Assertion In isochoric process, ρ-V graph is straight
5. Choose the wrong options. line parallel to ρ-axis.
(a) Translational kinetic energy of all ideal gases at same Reason In isochoric process density ρ remains
temperature is same
(b) In one degree of freedom, all ideal gases have internal constant.
energy = 1/ 2 RT 5. Assertion A metallic rod is fixed from two ends as
(c) Translational degree of freedom of all ideal gases is three
shown in figure. When the temperature is increased
(d) Translational kinetic energy of one mole of all ideal gases is
3 compressive stresses are developed in the rod.
RT
2
U
6. Along the line-1, mass of gas is m1 and
pressure is p1 . Along the line-2 mass of
same gas is m2 and pressure is p 2 . Reason At higher temperature, natural length of the rod
Choose the correct options. r
will be more.
(a) m1 may be less than m2 (b) m2 may be less than m1
2
(c) p1 may be less than p2 (d) p2 may be less than p1 6. Assertion Pressure of a gas is given as p = E.
3
7. Choose the correct options.
m Reason In the above expression, E represents kinetic
(a) In p = RT , m is mass of gas per unit volume
M energy of the gas per unit volume.
m
(b) In pV = RT , m is mass of one molecule of gas 7. Assertion Total internal energy of oxygen gas at a
M
1 mN 2 3
(c) In p = vrms , m is total mass of gas. given temperature is E of this energy E is translational
3 V 5
3kT 2
(d) In vrms = , m is mass of one molecule of gas kinetic energy and E is rotational kinetic energy.
m 5
Reason Potential energy of an ideal gas is zero.
Assertion and Reason
Direction (Q. Nos. 1-14) These questions consist of two 8. Assertion Total kinetic energy of any gas at
1
statements each printed as assertion and reason. While temperature T would be mv 2rms
answering these questions, you are required to choose anyone 2
of the following five responses. Reason Translational kinetic energy of any type of gas
(a) If both Assertion and Reason are true and Reason is 3
at temperature T would be RT of one mole.
the correct explanation of Assertion. 2
(b) If both Assertion and Reason are true but Reason is
not the correct explanation of Assertion.
9. Assertion Degree of freedom of a monoatomic gas is
always three, whether we consider vibrational effects or
(c) If Assertion is true but Reason is false.
not.
(d) If Assertion is false but Reason is true.
(e) If both Assertion and Reason are false. Reason At all temperatures (low or high), vibrational
kinetic energy of an ideal gas is zero.
1. Assertion Pressure of a gas is 2/3 times translational
kinetic energy of gas molecules. 10. Assertion An actual gas behaves as an ideal gas most
closely at low pressure and high temperature.
Reason Translational degree of freedom of any type of
gas is three, whether the gas is monoatomic, diatomic or Reason At low pressure and high temperature real
polyatomic. gases obey the gas laws.
638 Objective Physics Vol. 1

11. Assertion At triple point, three states (solid, liquid and 4. In the V-T graph shown in the figure match, the following
gas) may co-exist simultaneously. columns.
V
Reason For water, the values of pressure and A
temperature corresponding to triple point are 10 mm of
B
Hg and 273.16 K.
12. Assertion If a gas chamber containing a gas is moved
T
translationally, then temperature of gas will increase.
Column I Column II
Reason Total kinetic energy of the gas molecules will
(A) Gas A is … and gas B is … (p) Monoatomic, diatomic
increase by the translational motion of gas chamber. (B) pA / pB is (q) Diatomic, monoatomic
13. Assertion In summers, a metallic scale will read more (C) nA / nB is (r) > 1
than the actual. (s) < 1
(t) Cannot say anything
Reason In summer, length of metallic scale will
increase. 5. Match the following columns.
14. Assertion Any straight line on V-T diagram represents Column I Column II
(A) Adiabatic bulk modulus p
isobaric process. (p) −
V
Reason In isobaric process, if V is doubled, then T will (B) Slope of p-V graph in 2
isothermal process (q) γ − 1
also become two times.
(C) Degree of freedom (r) γp
Match the Columns (D) Molar heat capacity at constant γ
pressure divided by R (s) γ −1
1
1. In the process T ∝ , pressure of the gas increases from
V
p 0 to 4 p 0 . Match the following columns. Entrance Gallery
Column I Column II
2014
(A) Temperature of the gas (p) Positive
(B) Volume of the gas (q) Negative 1. If m represents the mass of each molecule of a gas and T
(C) Work done by the gas (r) Two times its absolute temperature, then the root mean square
(D) Heat supplies to the gas (s) Cannot say anything velocity of the gaseous molecule is proportional to
(t) None of the above [Kerala CEE]
(a) mT (b) m1/ 2 T 1/ 2 (c) m− 1/ 2 T
2. For a monoatomic gas at temperature T , match the (d) m− 1/ 2 T 1/ 2 (e) m−1T
following columns.
2. A molecule of a gas has six degrees of freedom. Then, the
Column I Column II
molar specific heat of the gas at constant volume is
(A) Speed of sound (p) 2 RT / M [Kerala CEE]
R 3R
(B) rms speed of gas molecules (q) 8RT / πM (a) (b) R (c)
2 2
(C) Average speed of gas (r) 3RT / M (d) 3R (e) 5 R
molecules
(D) Most probable speed of gas (s) 5RT / 3M 3. Total number of degrees of freedom of a rigid diatomic
molecules molecule is [Kerala CEE]
(a) 3 (b) 6 (c) 5
3. Match the following columns. (d) 2 (e) 7
Column I Column II 4. Which of the given substances A , B and C have more
(A) In p = 2 E, E is (p) Change in internal energy in specific heat? [Kerala CEE]
3 only isochoric process
A
(B) In U = 3RT for an (q) Translational kinetic energy B
momoatomic gas U is of unit volume T
C
(C) In W = p ( Vf − Vi ), W ( R ) is (r) Internal energy of one mole
(D) In ∆U = nC v ∆T, ∆U is (s) Work done in isobaric t
process (a) A (b) B
(c) C (d) Both (a) and (b)
(t) None of the above (e) All are equal
Thermometry, Thermal Expansion and Kinetic Theory of Gases 639

2013 12. The ratio of the molar heat capacities of a diatomic gas at
constant pressure to that at constant volume is
5. Two non-reactive monoatomic ideal gases have their
[Kerala CEE]
atomic masses in the ratio 2 : 3. The ratio of their partial (a) 7/2 (b) 3/2 (c) 3/5
pressure, when enclosed in a vessel kept at a constant (d) 7/5 (e) 5/2
temperature is 4 : 3.
The ratio of their densities is [JEE Advanced] 13. A perfect gas at 27ºC is heated at constant pressure so as
(a) 1 : 4 (b) 1 : 2 (c) 6 : 9 (d) 8 : 9 to double its volume. The increase in temperature of the
gas will be [Karnataka CET]
6. The velocity of 4 gas molecules are given by 1 km/s, (a) 300ºC (b) 54ºC (c) 327ºC (d) 600ºC
3 km/s, 5 km/s and 7 km/s. Calculate the difference
between average and rms velocity. [MHT CET] 14. Two temperature scales A and B are related by
A − 42 B − 72
(a) 0.338 (b) 0.438 (c) 0.583 (d) 0.683 = . At which temperature two scales have
110 220
2012 the same reading? [WB JEE]
(a) − 42ºC (b) − 72ºC (c) 12ºC (d) 40ºC
7. In a mercury thermometer, the ice point (lower fixed
point) is marked as 10°C and the steam point (upper fixed 15. Air inside a closed container is saturated with water
point) is marked as 130° C. At 40°C temperature, what vapour. The air pressure is p and the saturated vapour
will this thermometer read? [WB JEE] pressure of water is p. If the mixture is compressed to one
(a) 78ºC (b) 66ºC half of its volume by maintaining temperature constant,
(c) 62ºC (d) 58ºC the pressure becomes [WB JEE]
(a) 2( p + p ) (b) 2p + p (c) ( p + p ) / 2 (d) p + 2 p
2011
8. An aluminium sphere of 20 cm diameter is heated from 2010
0°C to 100° C. Its volume changes by (given that 16. A real gas behaves like an ideal gas, if its [IIT JEE]
coefficient of linear expansion for aluminium (a) pressure and temperature are both high
α A 1 = 23 × 10−6 / °C ) [AIEEE] (b) pressure and temperature are both low
(c) pressure is high and temperature is low
(a) 28.9 cc (b) 2.89 cc (d) pressure is low and temperature is high
(c) 9.28 cc (d) 4.98 cc
17. The temperature of a gas contained in a closed vessel of
9. A metal rod of Young’s modulus Y and coefficient of
constant volume increases by 1ºC when the pressure of
thermal expansion α is held at its two ends such that its
the gas is increased by 1%. The initial temperature of the
length remains invariant. If its temperature is raised by
gas is [Karnataka CET]
t °C , the linear stress developed in it, is [AIEEE]
(a) 100 K (b) 273ºC (c) 100ºC (d) 200 K
α∆t Y I
(a) (b) (c) Y α∆ t (d)
Y αE Yα∆t 18. One mole of a monoatomic ideal gas is mixed with one
mole of a diatomic ideal gas. The molar specific heat of
10. A container with insulating walls is divided into two
equal parts by a partition fitted with a valve. One part is the mixture at constant volume is [O JEE]
filled with an ideal gas at a pressure p and temperature T, (a) (3/ 2) R (b) (5/ 2) R (c) 2R (d) 4 R
whereas the other part is completely evacuated. If the 19. To what temperature should the hydrogen at 327ºC be
valve is suddenly opened, then the pressure and cooled at constant pressure, so that the root mean square
temperature of the gas will be [AIEEE] velocity of its molecules becomes half of its previous
p p T T
(a) ,T (b) , (c) p, T (d) p, value? [MHT CET]
2 2 2 2 (a) – 123ºC (b) 123ºC
11. Three perfect gases at absolute temperatures T1 , T2 and T3 (c) – 100ºC (d) 0ºC
are mixed. The masses of molecules are m1 , m2 and m3 20. At what temperature rms speed of air molecules is
and the number of molecules are n1 , n 2 and n 3 doubled of that at NTP? [MHT CET]
respectively. Assuming no loss of energy, the final (a) 819ºC (b) 719ºC
temperature of the mixture is [AIEEE] (c) 909ºC (d) None of these
n1T1 + n2T2 + n3T3 n1T12 + n2T22 + n3T33 21. Kinetic energy per unit volume is E. The pressure exerted
(a) (b)
n1 + n2 + n3 n1T1 + n2T2 + n3T3 by the gas is given by [MHT CET]
n12T12 + n22T22 + n32T32 T1 + T2 + T3 E 2E 3E E
(c) (d) (a) (b) (c) (d)
n1T1 + n2T2 + n3T3 3 3 3 2 2
Answers
Level 1
Objective Problems
1. (c) 2. (c) 3. (a) 4. (a) 5. (b) 6. (b) 7. (c) 8. (a) 9. (b) 10. (c)
11. (a) 12. (d) 13. (c) 14. (a) 15. (d) 16. (a) 17. (c) 18. (c) 19. (d) 20. (d)
21. (d) 22. (a) 23. (c) 24. (c) 25. (c) 26. (c) 27. (c) 28. (c) 29. (a) 30. (a)
31. (b) 32. (d) 33. (c) 34. (c) 35. (c) 36. (d) 37. (c) 38. (c) 39. (c) 40. (a)
41. (c) 42. (a) 43. (b) 44. (b) 45. (b) 46. (a) 47. (a) 48. (d) 49. (c) 50. (b)
51. (b) 52. (b) 53. (c) 54. (c) 55. (d) 56. (d) 57. (c) 58. (b) 59. (a) 60. (d)
61. (d) 62. (c) 63. (d) 64. (c) 65. (d) 66. (b) 67. (c) 68. (a) 69. (d) 70. (c)
71. (b) 72. (d) 73. (a) 74. (b) 75. (d) 76. (d) 77. (d) 78. (b) 79. (c) 80. (a)
81. (a) 82. (c) 83. (c) 84. (d) 85. (c) 86. (c) 87. (d) 88. (b) 89. (c) 90. (a)
91. (c)

Level 2
Only One Correct Option
1. (a) 2. (d) 3. (b) 4. (d) 5. (a) 6. (c) 7. (a) 8. (a,b) 9. (a) 10. (a)
11. (d) 12. (b) 13. (c) 14. (a) 15. (d) 16. (b) 17. (d) 18. (b) 19. (a) 20. (b)
21. (d) 22. (c) 23. (c) 24. (c) 25. (d)

More than One Correct Options


1. (b,d) 2. (a,d) 3. (a,c) 4. (a,c,d) 5. (a,b) 6. (All) 7. (a,d)

Assertion and Reason


1. (d) 2. (d) 3. (c) 4. (d) 5. (a) 6. (c) 7. (b) 8. (d) 9. (c) 10. (a)
11. (c) 12. (d) 13. (d) 14. (d)

Match the Columns


1. (A→ r, B→ t, C→ q, D→ s) 2. (A→ s, B→ r, C→ q, D→ p) 3. (A→ q, B→ t, C→ s, D→ t)
4. (A→ t, B→ t, C→ t) 5. (A→ r, B→ p, C→ q, D→ s)

Entrance Gallery
1. (d) 2. (d) 3. (c) 4. (c) 5. (d) 6. (c) 7. (d) 8. (a) 9. (c) 10. (a)
11. (a) 12. (d) 13. (a) 14. (c) 15. (b) 16. (d) 17. (a) 18. (c) 19. (a) 20. (a)
21. (b)
Solutions
Level 1 : Objective Problems 11. On this thermometer, 34°C will read as
150
2. Temperature shown on faulty thermometer will be − 20 + × 34 = 31°
95 − 5  100
t = 5 +   × 40 = 41° ∆T = − 273° C (absolute temperature). As no matter can
 100  12.
∆l attain this temperature, hence temperature can never be
3. Strain = = α∆ θ negative on kelvin scale.
l
F − 32 K − 273
Stress = Y × strain = Y α∆ θ 13. =
∴ Force or tension, T = Stress × area 9 5
= YA α∆ θ x − 32 x − 273
⇒ =
9 5
πY πd 2 ∆θ  π d2 
= A =  ⇒ x = 574.25
4  4 
14. Pyrometer is used to measure very high temperature.
π × 2 × 1011 × 10−5 × 10−4 × 25
or T= 15. Fractional change in period
4 ∆T 1
= 3926 N ≈ 4000 N = α∆θ
T 2
l 1
4. T = 2π = × 2 × 10−6 × 10 = 10−5
g 2
T ∝l ∆T
% change = × 100 = 10−5 × 100 = 10−3 %
2
T ′  l ′ 1 ∆l T
∴ =   =1 + Suppose, height of liquid in each arm before rising the
T l  2 l 16.
temperature is l.
1
= 1 + α∆ θ
2
∴ ∆T = T ′ − T t1
1
= αT∆θ
2
5. Coefficient of volume expansion of container will become t2
l1
α
3   , which is greater than coefficient of volume expansion
l l
 2
l2
of liquid. Hence, container expands more.
6. ∆l1 = ∆l2
or l1α1 ∆θ = l 2α 2 ∆θ
l1 α 2 With temperature rise height of liquid in each arm increases
or = i.e. l1 > l and l 2 > l
l 2 α1
l1
7. Lα∆θ + 2L ( 2α )( ∆θ) = ( 3L )(α e ) ∆θ Also, l=
1 + γ t1
L 2L 3L, αe
l2
⇒ =
α 2α 1 + γ t2
⇒ l1 + γ l1t2 = l 2 + γl 2t1
5 l −l
or αe =
α ⇒ γ= 1 2
3 l 2t1 − l1t2
8. Photographic expansion takes place.
17. V = V 0 (1 + γ∆θ)
9. Original value of circumference, l = 2 πR
L3 = L0 (1 + α1 ∆θ) L20 (1 + α 2 ∆θ)2
∴ ∆l = lαθ = 2 πR αθ
∆l lα∆θ = L30 (1 + α1 ∆θ)(1 + α 2 ∆θ)2
10. Strain = = = α∆θ
l l Since, L30 = V 0
Stress = Y × Strain = Y α∆θ
and L3 = V
(Stress)1 = (Stress)2
Hence, 1 + γ∆θ = (1 + α1 ∆θ)(1 + α 2 ∆θ)2
∴ Y1α1 = Y2α 2 (∆θ→ same)
≅ (1 + α1 ∆θ)(1 + 2α 2 ∆θ)
Y1 α 2 3
or = = ≅ (1 + α1 ∆θ + 2α 2 ∆θ)
Y2 α1 2
⇒ γ = α1 + 2α 2
642 Objective Physics Vol. 1

TC − 0 T F − 32 ( 3v )2 + ( 4v )2 + ( 5v )2
18. = 29. v rms = =
50
v
100 180 3 3
273 T F − 32
∴ − = R
30. pV = nRT = (nN ) T = (nN ) kT
100 180
N
Solving, we get T F = − 460° F nN p nN
or = . But, are the number of molecules per unit
θ 100
19. = V kT V
80 − 50 75 − 50 volume.
θºC 31. v rms ∝ T
θ 1
rms speed will remain half if temperature become th or
100 4
 273 − 273 °C or − 204.75° C
 
 4 

(1)2 + ( 2)2 + ( 3)2 + ( 4)2 15


32. v rms = = km/s
50 75 80 h(cm) 4 2
33. In pV = nRT , pV = RT if n =1
∴ θ = 120° C
21. v rms ∝ T , v rms will become two times when temperature is 34. V-T graph is a straight line passing through origin. Hence,
made four times. process is isobaric.
22. T1 = 273 + 27 = 300 K  nR 
V = T
T2 = 273 + 327 = 600 K  p 
T2 = 2T1 and v rms ∝ T nR
Slope =
p
3RT 8RT 2RT
23. v rms ∝ , v av = and v mp =
Slope of 2 > slope of 1
M πM M
T ∴ p2 < p1
24. v rms ∝
M 35. v rms ∝ T . If temperature is doubled, rms speed will
T is doubled and M has become half. Therefore, v rms will become two times.
become two times. V T V ( 273 + 27)
38. V ∝ T ⇒ 1 = 1 ⇒ =
25. Density of water is1 g/cm3 , i.e.1 cm3 = 1 g of water V 2 T2 3V T2
In 1 g mole or 18 g water there are total 6.02 × 1023 molecules. ⇒ T2 = 900 K = 627° C
Hence, in 1g of water (or 1cm3 of water ) number of molecules 39. For a given pressure, volume will be more if temperature is
6.20 × 1023 1 more (Charles’ law)
should be ≈ × 1023 p
18 3

26. p = p1 + p2 + p3 = 
nRT   nRT  +  nRT 
 +   
 V 1  V  2  V  3 Constant
RT pressure
= (n1 + n2 + n3 )
V T2
(0.5 + 0.5 + 0.25) (8.31) (300) T1
=
5 × 10−3 V1 V2 V
= 6.23 × 105 N/m2
From the graph, it is clear that V 2 > V1 ⇒ T2 > T1
27. Heat given = Heat taken
or p = 
Mp MT 
40. ρ = ρ
∴ n1CV ∆T = n2CV ∆T RT  M 
1 2

∴  1  ( 3R )(T − 300) =  1   5 R  ( 300 − T ) Temperature is directly proportional to the slope of


    
 2  2  2  p-ρ graph. So, T1 > T2 .
6T − 1800 = 1550 − 5T 41. As, θ2 > θ1 ⇒ tan θ2 > tan θ1
∴ T = 304.54 K = 31.5° C T  T 
⇒   > 
γRT  p  2  p 1
28. Speed of sound in a gas, v =
M T
∝ V ⇒ V 2 > V1
γ p
or v∝
M m pV 1
vN γN 2
42. pV = µRT = RT ⇒ ∝ (QM = Molecular mass)
M ( 7/5) 4 M T M
2
= × He = ×
v He γ He M N ( 5/3) 28  pV  <  pV  <  pV 
2 From graph      
 T  A  T  B  T C
3
=
5 ⇒ M A > M B > MC
Thermometry, Thermal Expansion and Kinetic Theory of Gases 643

µRT mRT µ = m  57. Translational degree of freedom for any type of gas is three.
43. p = =  
V MV  M
∴ Total translational kinetic energy = 3  nRT  = pV
1 3
So, at constant volume pressure versus temperature garph is 2  2
mp
a straight line passing through origin with slope . As the 58. Translational degree of freedom for any gas is three.
MV
mass is doubled and volume is halved slope becomes four 59. Both O 2 and N 2 are diatomic gases. Rotational degree of
times. Therefore, pressure versus temperature graph will be freedom in both cases is two. Therefore, average rotational
kinetic energy per molecule for each of them is 2  kT  or
shown by the line B. 1
44. For an ideal gas pV = constant at constant temperature, i.e. 2 
pV doesn't vary with V . the ratio is 1 : 1.
f
45. v rms =
3RT 60. U = nRT . Here, f is degree of freedom and n the number of
⇒ v rms
2
∝T 2
M
moles.
46. If T is in °C, then V T = V 0 1 +
T 
 ∴
f
1200 R = ( 2)( R )( 400)
 273 
2
47. T1 = 273 + 27 = 300 K
or f =3
T2 = 273 + 327 = 600 K 5
i.e. the given gas is monoatomic. For which, C p = R.
T2 = 2T1 2
and v rms ∝ T 61. C p − CV = R
1 3RT Dividing by CV , we get
48. v rms ∝ as v rms ∝
M M R
γ −1 =
3RT γRT CV
49. v rms = and v sound =
M M R
∴ CV =
v sound γ γ −1
=
v rms 3 f
Further U= RT
Degree of freedom is 6. 2
2 2 4 dU f
∴ γ =1 + =1 + = ∴ CV = = R
f 6 3 dT 2
4/3 62. Mean kinetic energy of gas molecules
∴ v sound = v rms
f
3 E= kT
2 2c 2
= v rms = f
3 3 = k (t + 273)
51. In monoatomic gases, only translational kinetic energy is 2
=  k  t + × 273 k

present, which is proportional to temperature. f f
3 2  2
53. Average translational kinetic energy per molecule is
kT
2 Comparing it with standard equation of straight line
which depends only on temperature not on molecular f f
y = mx + c. We get m = k and c = 273 k. So, the graph
mass. 2 2
55. CV of the mixture is between E andt will be straight line with positive intercept on
3 5 E-axis and positive slope with t-axis.
2× R +1× R
n1CV 1 + n2CV 2 2 2 = 11 R 65. U = 2  1 RT  + 4  2 RT 
f f
=
n1 + n2 3 6 2  2 
11 17 For O 2 , degree of freedom f1 = 5
∴ C p of mixture is R + R = R
6 6 And for Ar, degree of freedom f 2 = 3
C p 17
∴ γ= = = 1.54
U = 2  RT  + 4  RT  =11 RT
5 3
CV 11 ∴
2  2 
56. C p − CV = R
66. C p − CV = R
Dividing the equation by CV , we get
Dividing with C p , we get
Cp R R
−1 = or γ − 1 = 1−
1 R
=
CV CV CV γ Cp
R
or γ =1 + γ −1 R
CV or =
γ Cp
1 Cp Cp
= = =γ γR
1−
R Cp − R CV or Cp =
Cp γ −1
644 Objective Physics Vol. 1

3 77. Out of three parameters p, V and T only two should be


67. E = kT ⇒ E ∝ T
2 known to us.
E2 T2 600
∴ = = =2 78. Benzene contracts in winter. So, 5 L of benzene will weight
E1 T1 300 more in winter than in summer.
⇒ E2 = 2E1 79. Water has maximum density at 4°C, so if the water is heated
3 above 4°C or cooled below 4°C density decreases, i.e.
68. K T = RT volume increases. In other words, it expands so it overflows
2
in both the cases.
2 2
69. γ = 1 + ∴ f =
f γ −1 Density
R
70. ∆U = nCV ∆T = n (T f − Ti )
γ −1
p V − pi V i pV
= f f =
γ −1 γ −1

71. pV = nRT = n ⋅ N ⋅ 
R 0ºC 4ºC
 ⋅ T = nNkT Temp.
N
81. According to energy conservation, change in kinetic energy
∴  nN  = p
  appears in the form of heat (thermal energy).
 V  kT
1
nN Thermal energy = m (v12 − v 22 )
Here, are the number of molecules per m3 . 2
V
1
Hence, number of molecules per cm3 will be = (100 × 10−3 )(102 − 52 )
2
p 4 × 10−10 × 10−6
× 10−6 = ≈ 1.0 × 105 = 3.75 J
kT 1.38 × 10−23 × 300
83. At constant pressure, ρT = constant
72. Total internal energy of an ideal diatomic gas will be
ρ1 T2
u = 5  nRT  = pV
1 5 ⇒ =
2  2 ρ1 T1
u 5 24 ( 273 + 127)
∴ = p ⇒ =
V 2 ρ2 ( 273 + 27)
mgh 5 × 10 × 30 400
73. Q = cal = = 358.8 cal =
4.18 4.18 300
V nR ⇒ ρ2 = 18 units
74. =
T p
85. pV = µRT ⇒ p ∝ µT
V and T for both cases are same. Hence,
(QV and R = constant )
n1 n2 m1 m2
= or = p2 µ 2 T2
p1 p2 p1 M1 p2 M 2 ⇒ = ×
p1 µ 1 T1
p2 M 2
or m2 = ⋅ m1 pHe 1 2T
p1 M1 ⇒ = ×
p 1 T
( 3)( 2)
= ⋅ 20 = 0.86 kg ⇒ pHe = 2p
( 5)( 28)
86. pV = uRT
γp
75. v = T
ρ ∴ p∝
V
ρv 2 1.3 × (330)2 2 2E 2
∴ γ= = = 1.4 87. p = × (Energy per unit volume) = ⇒ pV = E
p 1.01 × 105 3 3V 3
i.e. gas is diatomic or degree of freedom is 5. pM p
88. ρ = ⇒ρ ∝
76. p2V = constant RT T
nRT  p  = p0
p= From graph,  
V  T  A T0
CT
or p= (C = constant)  p  = 3p0
V and  
 T  B 2T0
T2
∴ = constant 3
V ⇒ ρB = ρA
2
or T∝ V
3
= ρ0
V is increased to 3 times. Hence, T will increase 3 times. 2
Thermometry, Thermal Expansion and Kinetic Theory of Gases 645

89. pV = µRT =
m
RT 3. Volume of the gas is constant ∴V = constant
M
p
F

m2
m1 pA p 0A

V1 V2 V ∴p ∝ T i.e. pressure will be doubled temperature is doubled.


m RT ∴ p = 2p0
For 1st graph, p= 1 …(i)
M V1 Now, let F be the tension in the wire. Then, equilibrium of
m2 RT anyone piston gives
For 2nd graph, p= …(ii) F = ( p − p0 ) A
M V2
= ( 2p0 − p0 ) A
From Eqs. (i) and (ii), we get
= p0 A
m1 V1 1
= ⇒m ∝ γp
m2 V 2 V 4. v =
ρ
As V 2 > V1 ⇒ m1 < m2
ρv 2 1.3 × (330)2
m ∴ γ= =
90. For a gas, pV = µRT = RT p 1.01 × 105
M
p B = 1.4
i.e. gas is diatomic or degree of freedom is 5.
5. ( p2 − p1 ) A = mg
A mg RTi RTi 3RTi
3m or = − = …(i)
A V1 4V1 4V1
m Similarly, in second case
mg RT f RT f 3RT f
T = − = …(ii)
m A V2 3V 2 3V 2
For graph A, pV = RT
M Further 5V1 = 4V 2
Slope of graph A,   =
p m R Equating Eqs. (i) and (ii), we get
…(i)
V  M V 3Ti 2T f p1
=
3m 4V1 3V 2
For graph B, pV = RT
M 9 V 9 5
or T f = × 2 × Ti = × × 320
Slope of graph B,   =
p 3m R 8 V1 8 4
…(ii)
V  M V = 450 K
p2
3m R  nR   mR 
Slope of curve B M V 3 6. V =  T = T
= =  p   Mp 
Slope of curve A m R 1
m
MV Slope of V-T graph ∝
p
91. The rms velocity is independent of degree of freedom.
m
Since, ratio in both cases is same.
Level 2 : Only One Correct Option p
∆p Straight line will be B.
1. F = = (nm)( 2v cos θ)
∆t 7. From ideal gas equation, pV = µRT
F (nm)( 2v cos θ) p µR
Pressure, p = = ⇒ Slope of p-T curve, =
A A T V
1023 × 3.32 × 10−27 × 2 × 1000 × cos 45°
= ⇒ Slope ∝
1
2 × 10−4 V
= 2.34 × 103 N /m2 It means line of smaller slope represent greater volume of
2. Pressure on both sides should be same or gas. In the given problems, 1 and 2 are on the same line so
n1 RT n2 RT they will represent same volume, i.e. V1 = V 2
=
V1 V2 Similarly, points 3 and 4 are on the same line so they will
m m represent same volume, i.e., V 3 = V 4
or = Also slope of line 1-2 is less than 3-4.
32 ( 360° − α ) 28 α
Hence, (V1 = V 2 ) > (V 3 = V 4 )
or α = 192°
646 Objective Physics Vol. 1

8. Process 1-2 V = constant, p ∝ T 15. K T = n  RT  =


3 3
pV
T is increasing. Therefore, pressure will be increase. 2  2
Process 2-3 p = constant, V ∝ T 16. (n1 + n2 )i = (n1 + n2 ) f
V and T both are increasing. p0V p0V pV pV
+ = +
1 RT0 RT0 RT0 2RT0
Process 3-4 T = constant, p ∝
V 4
∴ p = p0
V is increasing. Therefore, p will decrease. 3
Process 4-1 Inverses of process 2-3. 17. Diameter of brass plate = 2.6 cm and
9. Slope of line is 1. Therefore, T-V equation can be written as diameter of steel plate = (2.6 − 0.01) cm = 2.59 cm
T = V + T0 Now, ( d + ∆d )b = ( d + ∆d )s
pV or db (1 + α∆ θ)b = d s (1 + α∆ θ)
∴ = V + T0
nR ∴ 2.6 (1 + 19 × 10−6 ∆ θ) = 2.59 (1 + 11 × 10−6 ∆ θ)
nRT0
∴ p = (nR ) + ∴ ∆ θ = − 478° C or θ f = − 458° C
V
18. Change in weight = Upthrust ( F )
∴ p versus V graph is a sort of rectangular hyperbola.
10. AB Isobaric process. Both volume and temperature are where, F = V s ρl g
increasing. F ′ = V s ρ′l g
F ′ V ′ S ρ′l
BC Isothermal process. Volume is increasing pressure = ⋅
V s ρl
should decrease.  T = constant, p ∝ 
1 F
 V F′ ρ′
or = (1 + γ s ∆ θ) t
CD Isochoric process. Pressure and temperature both are F ρl
decreasing. (V = constant, p ∝ T )  46 − 30.6   1.20 
DA Again isothermal process. Volume is decreasing and   = (1 + 3 × α s × 15)  
 46 − 30   1.24 
pressure is increasing  T = constant, p ∝ 
1
 V or α s = 2.3 × 10−5 /°C

11. U ∝
1
or T ∝
1 19. pV = nRT
ρ (1/V ) pV
n=
∴ T ∝V RT
Hence, p = constant or N = n × 6.02 × 1023 =
pV
× 6.02 × 1023
Further ρ is increasing. Therefore, V should decrease. RT
12. pV = nRT (10−14 × 13.6 × 103 × 10) (1)
or N= × 6.02 × 1023
pV (8.31) (273 + 30)
∴ n=
RT = 3.2 × 1011
(4 × 10−10 ) (10−6 ) 20. γr = γ a + γV , where γr = coefficient of real expansion,
=
(8.31) (300) γ a = coefficient of apparent expansion and
= 1.6 × 10−19 γ V = coefficient of expansion of vessel.
∴ N = (6.02 × 1023 ) n For copper, γ r = C + 3α Cu = C + 3A
= 9.65 × 104 C − S + 3A
For silver, γ r = C + 3α Ag =
3
∴Order is10 .
5
C − S + 3A
13. 1.0 cc of water =1 g of water ⇒ C + 3A = S + 3α Ag ⇒ α Ag =
3
1
= g-moles of water 2
21. (OR )2 = (PR )2 − (PO)2 = l 2 −  
l
18
 2
1
∴ N = × 6.02 × 1023 2
= [l (1 + α 2t )]2 −  (1 + α1t )
18 1
1023  2 

3 l2 l2
l− = l 2 (1 + α 22 t 2 + 2α 2t ) − (1 + α12 t 2 + 2α1t )
14. pV = nRT 4 4
Neglecting α 2 t and α1 t
2 2 2 2
=  1  RT (n1 = number of molecules)
n
N l2
0 = l 2 ( 2α 2t 2 ) − ( 2α1t )
= n1 kt  = k 
R 4
N  2α1
⇒ 2α 2 =
n1 p 4
∴ =
V kT ⇒ α1 = 4α 2
Thermometry, Thermal Expansion and Kinetic Theory of Gases 647

22. From ideal gas equation, pV = RT …(i)


More than One Correct Options
or p∆V = R∆T …(ii)
1. p ∝ ρ as per process. If ρ becomes half, then p will become
Dividing Eq. (ii) by Eq. (i), we get
2 times.
∆V ∆V
= 1
V T Further ρ∝
V
∆V 1
⇒ = =δ (given) p2
V ∆T T ∴ = constant
(1/V )
∴δ =1/T. So, the graph between δ and T will be rectangular
hyperbola. or ( pV ) p = constant
Hence, Tp = constant (as pV ∝ T )
23. p-T equation from the graph should be like.
1
p = aT + b (a and b are positive constants) ∴ p∝
T
p b
∴ =a+ ⇒ T2 > T1 or p - T graph is a rectangular hyperbola.
T T 1
2. p ∝
 p <  p T  T  V
∴     or   >  
 T  2  T 1  p  2  p 1 If volume becomes 4 times, then p will remain half.
V 2 > V1 T
or ∴ V ∝
T p
as V ∝
p ∴ Vp2 = constant
24. Let us draw different isotherms. T  2
∴   p = constant
p  p
T2 T3
1
or pT = constant or p ∝
1 T
i.e. p - T graph is a rectangular hyperbola.
2 If p is halved, then T will become two times.
T1
3. Not required.
O V 1
4. ρ ∝
25. Consider the diagram, when the molecules breaks into V
atoms, the number of moles would become twice. V is doubled so ρ will remain half.
Now, by ideal gas equation U ∝ T ∝ pV
p = Pressure of gas, n = Number of moles According to given graph, p ∝ V
R = Gas constant, T = Temperature ∴ U ∝ T ∝ p2 or V 2
pV = nRT V is doubled, so U and T both will become four times.
As volume (V ) of the container is constant. p ∝V

T
∝V  as p ∝ T 
 
V  V
H2 H2 H2 ∴ T ∝V 2
V
molecules or T - V graph is a parabola origin.
– – –
5. Not required.
nRT mRT  mR 
6. V = = = T
p Mp  Mp 
H i.e. V - T graph is a straight line passing through origin of
V
Atoms slope,
 mR 
H H = 
 Mp 
As gases break number of moles become twice of initial, so m
∴ Slope ∝
n2 = 2n1 p
So, p ∝ nT
Hence, slope depends on both m and p.
p2 n 2 T 2 ( 2n1 )( 3000)
⇒ = = = 20 nRT (m/V )RT
p1 n1 T 1 n1 ( 300) 7. (a) p = =
V M
⇒ p2 = 20p1 So, givenmin the question is the mass of gas per unit volume.
Hence, final pressure of the gas would be 20 times the (b) m = total mass of gas
pressure initially. (c) m = mass of one molecule of gas.
648 Objective Physics Vol. 1

For monoatomic gas, f = 3


Assertion and Reason 3nRT
∴ U=
= m 
1 1 3RT  3 3 R
1. K T = (m)v rms
2
 = nRT = pV
2 2  M  2 2 or U = 3RT for n = 2 is valid for any gas.
∴ p=
2 KT 2
⋅ = ∆U = nCV ∆T
3 V 3 4. From the given V -T graph we cannot tell nature of gas
(translational kinetic energy per unit volume).
nRT  nR 
2. Only that straight line which passes through origin V = = T
p  p 
represents isobaric process.
3. In 22.4 L at NTP, number of molecules are 6.02 × 1023 . If p is constant,V -T graph is a straight line. Slope of this line is
nR
4. In isochoric process, ρ and V both are constants. Therefore, .
p
ρ -V graph is a dot.
(Slope)A > (Slope)B
5. At higher temperature, natural length is more. It means rod n
is compressed. or A may be grater than 1.
nB
6. E represents translational kinetic energy per unit volume.
pA
7. Translational degree of freedom of a diatomic gas (O 2 ) is or may be less than 1.
pB
three and rotational kinetic energy is two. Therefore,
K 3  n  n
according to law of equipartition of energy T = . But,   >   .
KR 2   A  p B
p
Further, intermolecular force between ideal gas molecules is
zero. Hence, potential energy is zero. Entrance Gallery
1 2
8. Only translational kinetic energy is mv rms . 1. We know that
2 3RT 3RT
Further, translational degree of freedom of any type of gas is v2 = ⇒ v rms =
M M
three. Therefore, translational kinetic energy of any type of
3 The root mean square speed of the molecules of a gas is
gas (of one mole) is RT. directly proportional to the square root of the absolute
2
temperature of the gas and inversely proportional to the
10. At the given conditions, the intermolecular force between square root of the mass of molecules of the gas.
the gas molecules is almost zero. 1
So, v rms ∝ T and v rms ∝
12. Translational motion of the gas chamber increases the total m
kinetic energy but it does not contribute in internal energy.
or vrms = m− 1/ 2 × T 1/ 2
Therefore, temperature of gas does not increase by
translational motion of the gas chamber. 2. We know that specific heat of a gas (At constant volume)
13. It will read less than the actual. CV = f /2 × R
14. Only that straight line passing through origin represents where, R = number of independent relations
isobaric process. f = degree of freedom
6
∴ CV = × R = 3R
Match the Columns 2
T 1 3. Total number of degrees of freedom of a rigid diatomic
1. T ∝ or T ∝
V T /p molecule is 3 × 2 − 1 = 5.
4. Substances having more specific heat
or T∝ p take longer time to get heated to a
A
T B
p has become four times. Therefore, T has become two times higher temperature and longer time to
1 get cooled.
or V will have become times. C
2 If we draw a line parallel to the time
Since, V is decreasing, work done by the gas is negative. axis, then it cuts the given graphs at
three different points. Corresponding
Further, T is increasing, hence ∆U will be positive. W is points on the time axes shows that, tA tB tC
negative and ∆U is positive. Hence, we cannot say about
tC > tB > tA ⇒ cC > cB > cA
∆Q.
pM
C 5 5. Density of the gas, ρ =
2. p = γ = RT
CV 3 ρ1  p1   M1   2   4  8
∴ ρ ∝ pM or =   =    =
γRT ρ2  p2   M 2   3   3  9
Speed of sound in a gas, v sound =
M
ρ1 : ρ2 = 8 : 9
3. Internal energy of n moles of an ideal gas, 6. The average velocity,
U = n  RT  v1 + v 2 + v 3 + K + vn 1 + 3 + 5 + 7
f
( f = degree of freedom) v av = = = 4 km/s
2  n 4
Thermometry, Thermal Expansion and Kinetic Theory of Gases 649

Root mean square velocity, T2 = 600 K = 600 – 273 = 327 °C


v 2 + v 22 + v 32 + v 42 + K + vn2 ∆t = 327 – 27 = 300° C
v rms = 1
n 14. Condition for same reading is A = B
A − 42 B − 72
1 + ( 3)2 + ( 5)2 + ( 7)2 =
= = 21 = 4.583 km/s 110 220
4
A − 42 A − 72
Difference between average velocity and root mean square ⇒ =
110 220
velocity = 4.583 − 4 = 0.583 km/s
2A − 84 = A − 72
7. Given, ice point = 10°C and steam point = 130°C
A =12°C
According to the formula,
x − 10 40
15. Final pressure is given as p f = 2p + p
= ⇒ 100x − 1000 = 4800
130 − 10 100 Saturated vapour pressure will not change, if temperature
remains constant, the air pressure becomes twice as
100x = 5800 ⇒ x = 58°C
volume is halved.
8. Cubical expansion, ∆V = γV ∆T
16. A real gas behaves as an ideal gas at low pressure and high
where, γ = coefficient of volumetric expansion temperature.
∆V = 3αV ∆T = 3 × 23 × 10−6 ×  π × 103  × 100 = 28.9cc
4 17. According to Gay Lussac’s law, p ∝ T
3  dp dT 1
FL ∴ × 100 = × 100 ⇒ 1 = × 100 ⇒ T =100K
9. Linear expansion ∆L = αV ∆t = p T T
AY
n1CV + n2CV
F
Stress, s = = Yα ∆t 18. Molar specific heat of mixture, CV = 1 2

A n1 + n2
3 5
where, symbols have their usual meaning. 1× R +1× R
10. Internal energy of the gas remains constant, hence = 2 2 = 2R
1+1
T2 = T
3RT
V
Using, p1V1 = p2V 2 ⇒ p ⋅ = p2V ⇒ p2 =
p 19. We know that vrms ∝
2 2 M
11. Conserving total KE, we can write ⇒ T ∝ v rms
2

2
f f f f
n1 kT1 + n2 kT2 + n3 kT3 = (n1 + n2 + n3 )kT T2  v 2  1 T 273 + 327
⇒ = = ⇒ T2 = 1 =
2 2 2 2 T1  v1  4 4 4
n1T1 + n2T2 + n3T3
T= = 150 K = − 123° C
n1 + n2 + n3
20. As, v rms ∝ T
R and C p =  + 1 R
f f
12. CV = v12 T1 v2 273
2 2  ⇒ = ⇒ =
v 22 T2 ( 2v )2 T2
5 7
CV = R and C p = R
2 2 ⇒ T2 = 1092 K = 819° C
Cp 7 21. The pressure exerted by the gas,
∴ γ= = 1 m 2 2 1
c =  mc 2 
1
CV 5 p = ρc 2 =
3 3V 32 
V1 T1
13. Using pV = nRT , p = constant = Q1 mc −2 = E = energy volume,V =1
V 2 T2  2 
V
1 300
= [QV 2 = 2V1 ] p= E
2
2 T2 3
15
The First Law of
Thermodynamics
Thermal Equilibrium
Equilibrium in thermodynamics mean macroscopic variables (like pressure, Chapter Snapshot
temperature, volume, etc) of a thermodynamic system (like a gas inside a closed rigid ● The First Law of
container) do not change in time. Thermodynamics
● Further Explanation of
Adiabatic Wall the Three Terms Used in
It is an insulating wall (can be movable) between two thermodynamic systems that First Law
does not allow to flow of energy (or heat) from one system to another system. ● Different Thermodynamic
Processes
Diathermic Wall ● Efficiency of a Cycle
● Heat Engines
It is a conducting wall between two thermodynamic systems that allows energy flow ● Refrigerator
(or heat) flow from one system to another system.
● Second Law of
Thermodynamics
Zeroth Law of Thermodynamics
According to this law, if two systems in thermal equilibrium with a third system, then
they separately are in thermal equilibrium with each other. Thus, if A and B are separately in
equilibrium with C, i.e. if T A = TC and TB = TC . Then, this implies that T A = TB
i.e. the systems A and B are also in thermal equilibrium.

15.1 The First Law of Thermodynamics


The first law of thermodynamics, is an extension of the principle of conservation of
energy. To state energy relationships precisely, we need the concept of a
thermodynamic system. A thermodynamic system is a system that can interact (and
exchange energy) with its surroundings, or environment, at least in one way one of which is
heat transfer. In this chapter the thermodynamic system will be an ideal gas contained in a
vessel in most of the cases. A process in which there are changes in the state of a
thermodynamic system (p, V and T in case of a gas) is called a thermodynamic process.
We now come to the first law.
Let a system changes from an initial equilibrium state i to a final equilibrium state f in
a definite way, the heat absorbed by the system being Q and the work done by the system
being W.Then, we compute the Q – W. While Q and W both depend on the thermodynamic
The First Law of Thermodynamics 651

path taken between two equilibrium 1 f This can also be written as


states, their difference Q – W does 2 dQ = dU + dW…(iii) or Q = ∆U + W …(iv)
not depend on it. We do this over and The first law can be expressed in other words as under.
over again, using different paths each
time. We find that in every case the i
3 Suppose a heat Q is given to a system. This heat is partly
quantity Q – W is the same. Fig. 15.1 used by the system in doing work against its surroundings
and partly its internal energy gets increased and from energy
The students may recall from mechanics that when an conservation principle, Q = ∆U + W. An another analogous
object is moved from an initial point i to a final point f in a example is from our daily life. Consider a person X .
gravitational field in the absence of friction, the work done Suppose his monthly income is ` 50000 (Q ).
depends only on the positions of the two points and not at all
on the path through which the body is moved. From this we He spends `30000 (W ) as his monthly expenditure.
concluded that there is a function of the space coordinates of Then, obviously the remaining ` 20000 goes to his savings
the body whose final value minus its initial value equals the ( ∆U ). In some month it is also possible that he spends more
work done in displacing the body. We called it the potential than his income. In that case he will withdraw it from his
energy function. In thermodynamics there is a function of bank or his savings will get reduced ( ∆U < 0). In the similar
the thermodynamic coordinates (p, V and T) whose final manner other combinations can be made.
value minus its initial value equals the change Q – W in the X Example 15.1 When a system goes from state A to
process. We call this function the internal energy function. state B, it is supplied with 400 J of heat and it does
We have,
100 J of work.
∆U = Uf – Ui = Q – W …(i) (a) For this transition, what is the system’s change in
This equation is known as the first law of internal energy?
thermodynamics. (b) If the system moves from B to A, what is the change in
We describe, the energy relations in any thermodynamic internal energy?
process in terms of the quantity of heat Q added to the system (c) If in moving from A to B along a different path in
and the work done W by the system. Both Q and W may be which WAB ′ = 400 J of work is done on the system,
positive, negative or zero. A positive value of Q represents heat how much heat does it absorb?
flow into the system, negative Q represents heat flow out of the Sol. (a) From the first law,
system. A positive value of W represents work done by the ∆UAB = QAB – WAB = (400 – 100) J = 300 J
system against its surroundings, such as work done by an (b) Consider a closed path that passes through the state A and B.
expanding gas. A negative value of W represents work done on Internal energy is a state function, so ∆U is zero for a closed
path.
the system by its surroundings such as work done during
compression of a gas. Thus, ∆U = ∆UAB + ∆UBA = 0
or ∆UBA = – ∆UAB = – 300 J
Table 15.1 Thermodynamic Sign Conventions for
(c) The change in internal energy is the same for any path, so
Heat and Work
∆UAB = ∆ UAB′ = QAB′ – WAB ′
Process Convention
300 J = Q′AB – (– 400 J)
Heat added to the system Q> 0 ′ = – 100 J
and the heat exchanged is QAB
Heat removed from the system Q< 0
The negative sign indicates that the system loses heat in this
Work done by the system W >0 transition.
Work done on the system W <0
Example The quantities in the following table
Eq. (i) can be written as represent four different paths for the same initial and final
Q1 – W1 = Q2 – W2 =… state. Find a, b, c, d, e, f and g.
or ∆U 1 = ∆U 2 =…
That is the change in the internal energy of the system Table 15.2
between two points is path independent. It depends on Q (J) W (J) ∆U (J)
thermodynamic coordinates of the two points. For example, – 80 –120 d
90 c e
in case of an ideal gas it depends only on the initial and final
a 40 f
temperatures. b –40 g
Often the first law must be used in its differential form,
which is Ans. a = 80 J, b = 0, c = 50 J , d = 40 J
dU = dQ – dW …(ii) e = 40 J, f = 40 J, g = 40 J
652 Objective Physics Vol. 1

15.2 Further Explanation of Work Done (W or dW )


This is the most important of the three.
the Three Terms Used
Work Done During Volume Changes
in First Law A gas in a cylinder with a movable piston is a simple
First law of thermodynamics basically revolves round example of a thermodynamic system.
the three terms Q, ∆U and W. If you substitute these three Figure shows that a gas
terms correctly with proper signs in the equation confined to a cylinder which
Q = ∆U + W , then you are able to solve most of the problems has a movable piston at one F = pA
of first law. Let us take each term one by one. Here, we are end. If the gas expands
taking the system an ideal gas. against the piston, it exerts a dx
Fig. 15.2
force through a distance and
Heat Transfer (Q or dQ ) does work on the piston. If the piston compresses the gas as
For heat transfer apply, it is moved inward, work is also done in this case on the gas.
The work associated with such volume changes can be
Q = nC∆T determined as follows.
or in differential form, Let the gas pressure on the piston face be p. Then, the
dQ = nCdT force on the piston due to gas is pA, where, A is the area of
the face.
where, C is the molar heat capacity of the gas and n is
When the piston is pushed outward an infinitesimal
the number of moles of the gas. Always take, distance dx, the work done by the gas is
∆T = T f – Ti dW = F ⋅ dx = pA dx
where, Tf is the final temperature and Ti is the initial Which, since the change in volume of the gas is
temperature of the gas. Further, we have discussed in dV = Adx, becomes
chapter 20, that molar heat capacity of an ideal gas in the dW = pdV
process ( pV x = constant) is, For a finite change in volume from Vi to V f , this
equation is then integrated between Vi to V f to find the net
R R
C= + work done
γ –1 1– x Vf
W = ∫ dW = ∫ pdV
Vi
R
= CV + Now there are two methods of finding work done by a
1– x
gas.
R Method 1. This is used when p-V equation is known to us.
C = CV = in isochoric process and
γ –1 Suppose p as a function of V is known to us.
p = f (V )
C = C p = CV + R in isobaric process
Then, work done can be found by,
Mostly C p and CV are used. Vf
W =∫ f (V ) dV
Vi
Change in Internal Energy (U or dU) Method 2. The work done by a gas is also equal to the
For change in internal energy of the gas apply, area under p-V graph. Following different cases are
possible.
∆U = nCV ∆T
Case I. When volume is constant
or in differential form, p p
B A
dU = nCV dT
or
Students are often confused that the result
∆U = nCV ∆T can be applied only in case of an isochoric A B
process (as CV is here used). However, it is not so. It can be V V

applied in any process, whether it is isobaric, isothermal Fig. 15.3

adiabatic or else. V = constant ⇒ ∴ WAB = 0


The First Law of Thermodynamics 653

Case II. When volume is increasing Case IV. Cyclic process


p p p p
B A

or
A B

V V V V
Fig. 15.4 (a) (b)
Fig. 15.6
V is increasing
Cyclic process
∴ WAB > 0
Wclockwise cycle = + Shaded area [in Fig. 15.6(a)]
WAB = Shaded area Wanti -clockwise cycle = – Shaded area
Case III. When volume is decreasing [in Fig. 15.6 (b)]
p p Case V. Incomplete cycle
B A p p

or C
A B B A
B
C D
V V A
Fig. 15.5 V V

V is decreasing Fig. 15.7

∴ WAB < 0 WABC = + Shaded area


WAB = – Shaded area WABCD = – Shaded area

Extra Knowledge Points


■ In the above discussion, we have seen that So, we have found the work done without putting the
Vf
W = ∫ p dV limits of volume.
Vi
■ Some times the piston (which is
From this equation it seems as, if work done can be massless) is attached to a spring of
calculated only when p-V equation is known and the limits force constant k and a mass m is
Vi andVf are known to us. But it is not so. We can calculate placed over the piston. The area of the
work done even, if we know the limits of temperature. piston is A. The gas expands. To
For example, the temperature of n moles of an ideal gas is m
make the calculation easy we assume
α that initially the spring was in its x
increased from T0 to 2T0 through a process p = and we m
T natural length. We are required to find
are interested in finding the work done by the gas. Then, the work done by the gas. As, the
pV = nRT (ideal gas equation) …(i) piston is massless, net force on it at p
α every instant is zero.
and p= …(ii)
T ∴ pA = kx + mg + p0 A
Dividing Eq. (i) by Eq. (ii), we get kx mg
or p = p0 + +
nRT 2 A A
V = dW = pdV = p ( Adx )
α
2nRT = ( Ap0 + kx + mg ) dx
or dV = dT x
α ∴ W = ∫ p dV
0
Vf 2T 0  α   2 nRT 
∴ W = ∫ p dV = ∫     dT = 2 nRT0 or
x
W = ∫ ( Ap0 + kx + mg ) dx
Vi T0 T   α  0
654 Objective Physics Vol. 1

1 2
= p0 Ax + kx + mgx (as Ax = ∆V ) = d (RT ) (as p = constant)
2 = R ⋅ dT
1
or W = p0 ∆V + kx 2 + mgx Substituting these values in Eq. (i),
2 we have CpdT = R ⋅ dT + CV dT
p0A + kx + mg
or Cp – CV = R
R
■ CV = . We have already derived,
g − 1
Cp – CV = R
Dividing this equation by CV , we have
pA Cp R
–1=
CV CV
The result can be stated in a different manner as under.
R  Cp 
The gas does work against the atmospheric pressure p0 or γ –1= as = γ
(which is constant), the spring force kx (which varies CV  CV 
linearly with x) and the gravity force mg (which is again R
constant). ∴ CV =
γ –1
∴ W1 =Work done against p0 = p0 ∆V ■ Thermodynamic parameters for a mixture of gases
1
W2 =Work done against kx = kx 2 (i) Equivalent molar mass When n1 moles of a gas with
2
molar mass M1 are mixed with n 2 moles of a gas with
and W3 = Work done against mg = mgx molar mass M 2 , then equivalent molar mass of the
1 mixture is given by
So, Wtotal = W1 + W2 + W3 = p0 ∆V + kx 2 + mgx
2 n M + n 2M 2
M = 1 1
From point number (2). We may conclude that work done n1 + n 2
by a gas is zero, if the other side of the piston is vacuum.
(ii) Internal energy of the mixture The total energy of
the mixture is
U = U1 + U2
Vacuum (iii) C V of the mixture
U = U1 + U2
∴ dU = dU1 + dU2
or nCV dT = n1 CV1dT + n 2 CV 2 dT …(ii)
dU dU
■ CV = . Let us derive the relation CV = or (n1 + n 2 ) CV = n1 CV1 + n 2CV 2 (as n = n1 + n 2 )
dT dT n1CV1 + n 2 CV 2
where, U = internal energy of 1 mole of the gas. ∴ CV =
n1 + n 2
Consider 1 mole (n = 1) of an ideal monoatomic gas which
undergoes an isochoric process (V = constant ). From the (iv) C p of the mixture
first law of thermodynamics. Cp = CV + R
n1CV1 + n 2 CV 2
dQ = dW + dU …(i) or Cp = +R
Here, dW = 0 as V = constant n1 + n 2
dQ = CdT = CV dT n1 (CV1 + R ) + n 2 (CV 2 + R )
=
(In dQ = nCV dT ,n = 1 and C = CV ) n1 + n 2
Substituting in Eq. (i), we have n1Cp1 + n 2 Cp 2
=
CV dT = dU or CV =
dU n1 + n 2
dT n1Cp1 + n 2 Cp 2
Thus, Cp =
■ C P – C V = R. To prove this relation (also known as n1 + n 2
Mayer’s formula) let us consider 1 mole of an ideal gas
which undergoes an isobaric (p = constant) process. From (v) γ of the mixture From Eq. (ii)
first law of thermodynamics, (n1 + n 2 ) CV = n1CV1 + n 2 CV 2
dQ = dW + dU (n1 + n 2 ) R nR n 2R
or = 1 +
Here, dQ = CpdT (as n = 1and C = Cp ) γ –1 γ1 – 1 γ2 – 1
dU = CV dT n1 + n 2 n1 n2
or = +
 RT   RT  γ –1 γ1 – 1 γ2 – 1
and dW = pdV = pd   as V = 
 p  p Thus, γ of the mixture is given by above equation.
The First Law of Thermodynamics 655

X Example 15.2 A certain amount of an ideal gas Sol. Because, the expansion takes place at constant pressure,
passes from state A to B first by means of process 1, the work done is
Vf Vf
then by means of process 2. In which of the process is W = ∫V p0dV = p0 ∫V dV = p0 (Vf – Vi )
the amount of heat absorbed by the gas greater? i i

p = (1.01 × 105 ) (1671 × 10−6 – 1.0 × 10−6 )


= 169 J
1
Q = mLv = (1.0 × 10−3 ) (2.26 × 106 ) = 2260 J

A B Hence, from the first law, the change in internal energy


∆U = Q – W = 2260 – 169
2 = 2091 J
V
Fig. 15.8
Note Points
/ The positive value of ∆U indicates that the internal energy of
Sol. Q1 = W1 + ∆U1 and Q2 = W2 + ∆U 2  2091 J 
the system increases. We see that most  = 93% of
U is a state function. Hence, ∆U depends only on the initial  2260 J 
and final positions. Therefore, the energy transferred to the liquid goes into increasing the
169 J
p p internal energy of the system only = 7% leaves the
2260 J
1 system by work done by the steam on the surrounding
A B A B atmosphere.

W1
W2
15.3 Different Thermodynamic
V V
Processes
Fig. 15.9
Among the thermodynamic processes we will consider
∆U1 = ∆U 2 are the following:
But W1 > W2
(i) An isothermal process during which the system’s
as the area under 1 is greater than area under 2. Hence,
Q1 > Q2
temperature remains constant.
(ii) An adiabatic process during which no heat is
∆Q ∆Q transferred to or from the system.
X Example 15.3 Find the ratio of and in
∆U ∆W (iii) An isobaric process during which the pressure of the
an isobaric process. The ratio of molar heat capacities system is constant.
Cp
= γ. (iv) An isochoric process during which the system’s
CV volume does not change.
In an isobaric process, p = constant. Therefore, C = C p .
Of course there are, many other processes that do not
Sol.
∆Q nC p ∆T C p
fit into any of these four categories.
Now, = = =γ
∆U nC V ∆T C V We will mostly consider an ideal gas.
∆Q ∆Q nC p ∆T
and = = Isothermal Process
∆W ∆Q – ∆U nC p ∆T – nC V ∆T
Cp C p /C V γ An isothermal process is a constant temperature
= = =
Cp – CV C p /C V – 1 γ –1 process. In an isothermal process
(i) T = constant or ∆T = 0
X Example 15.4 Suppose 1.0 g of water vaporizes 1
isobarically at atmospheric pressure (1.01 × 10 5 Pa). (ii) p ∝ or pV = constant,
V
Its volume in the liquid state is Vi = Vliquid = 1.0 cm 3 i.e. p-V graph is a rectangular hyperbola with
and its volume in vapour state is V f = Vvapour = 1671 cm 3 . piVi = p f V f .
Find the work done in the expansion and the change in (iii) As T = constant, hence U = constant for an ideal gas,
internal energy of the system. Ignore any mixing of the because U is a function of T only.
steam and the surrounding air. Take latent heat of (iv) ∆U = 0
vaporisation Lv = 2.26 × 10 6 J / kg. ∴ Q =W
656 Objective Physics Vol. 1

Work done in isothermal process Note Point


Vf Vf  nRT   nRT 
W =∫ p dV = ∫   dV  as p =  / In adiabatic process W = – ∆U. Therefore, if the work done by
Vi Vi  V   V  a gas is positive (i .e. volume of the gas is increasing), then
dV Vf ∆U will be negative. That is U and hence T will decrease. The
= nRT
i V
∫V (as T = constant) cooling of air can be experienced practically during bursting
of a tyre. The process is so fast that it can be assumed as
 Vf  adiabatic, as the gas expands. Therefore, it cools. On the
= nRT ln  
 Vi  other hand the compression stroke in an internal combustion
engine is an approximately adiabatic process. The
 p  temperature rises as the air fuel mixture in the cylinder is
= nRT ln  i  (as piVi = p f V f ) compressed.
 pf 
p
p-V relation In adiabatic process dQ = 0
and dW = – dU
Isothermal
pi ∴ pdV = – CV dT (for n =1)
pV = constant
pdV
∴ dT = – …(i)
Hyperbola CV
pf
Also for 1 mole of an ideal gas,
Vi Vf
V
d ( pV ) = d ( RT )
Isothermal expansion of an or pdV + Vdp = RdT
ideal gas
pdV + Vdp
Fig. 15.10 or dT = …(ii)
R
Thus, in an isothermal process
From Eqs. (i) and (ii)
V f   p 
∆U = 0 and Q = W = nRT ln   = nRT ln  i  CV Vdp + (CV + R ) pdV = 0
 Vi   pf  or CV Vdp + C p pdV = 0
Dividing this equation by pV, we are left with
Note Point
dp dV
/ For a process to be isothermal, any heat flow into or out of the CV +Cp =0
system must occur slowly enough, so that thermal equilibrium
p V
is maintained. dp dV
or +γ =0
p V
Adiabatic Process
dp dV
An adiabatic process is defined as one with no heat
transfer into or out of a system i.e. Q = 0. We can prevent
or ∫ p
+γ ∫ V
=0

heat flow either by surrounding, the system with thermally or ln ( p) + γ ln (V ) = constant


insulating material or by carrying out the process so quickly We can write this in the form of
that there is not enough time for appreciable heat flow. From pV γ = constant
the first law we find that for every adiabatic process,
W = – ∆U (as Q = 0) This equation is the condition that must be obeyed by an
= – nCV ∆T ideal gas in an adiabatic process. For example, if an ideal gas
= – nCV ( Tf – Ti ) makes an adiabatic transition from a state with pressure and
volume pi and Vi to a state with p f and V f , then
 R   R 
=n  (Ti – Tf )  as CV =  piVi γ = p f V fγ
 γ – 1  γ – 1
piVi – p f V f The equation pV γ = constant can be written in terms of
= (as nRT = pV ) other pairs of thermodynamic variables by combining it with
γ –1
the ideal gas law ( pV = nRT ). In doing, so we will find that,
Thus, in an adiabatic process,
piVi – p f Vf TV γ – 1 = constant
Q = 0 and W = – ∆U =
γ –1 and T γ p1 – γ = constant
The First Law of Thermodynamics 657

= nR (T f – Ti )
Slope of p - V Graph
= p (V f – Vi ) (as nRT = pV )
In an adiabatic process ( pV γ = constant ), the slope of
p - V diagram at any point is Thus, in an isobaric process
dp d  constant  Q = nC p ∆T , ∆U = nCV ∆T
=  
dV dV  V γ  and W = p (V f – Vi )
 p
=–γ   Isochoric Process
V 
An isochoric process is a constant volume process. In an
 p
Thus, (slope) adiabatic = – γ   isochoric process,
V 
(i) V = constant
Similarly, in an isothermal process ( pV = constant), the or ∆V = 0
slope of p -V diagram at any point is
dp d  constant  p (ii) p ∝T
=   =–
dV dV  V  V or
p
= constant
p T
or (slope) isothermal = –
V i.e. p-T graph is a straight line passing through
Because γ >1, the isothermal curve is not as steep as that origin.
for the adiabatic expansion.
(iii) As V = constant, hence W = 0 and from first law of
1 Monoatomic
p p thermodynamics
2 Diatomic
3 Polyatomic Q = ∆U = nCV ∆T
Isothermal Thus, in an isochoric process,
γ = 1.33
2
3
γ = 1.4
W =0
1
γ = 1.67 and Q = ∆U = nCV ∆T
Adiabatic
p
V V
1
Adiabatic and isothermal 1 Isobaric
Adiabatic expansion of mono, dia
expansion of an ideal gas and polyatomic gases 2 2 Isothermal
3 Adiabatic
Fig. 15.11 3 4 Isochoric
4

Isobaric Process V
An isobaric process is a constant pressure process. In an Fig. 15.13
isobaric process,
(i) p = constant or ∆p = 0 p-V diagram of different processes is shown in one
V graph.
(ii) V ∝ T or = constant
T Table 15.3 shows Q, ∆U and W for different processes
i.e. V-T graph is a straight line passing through origin. discussed above.
(iii) Q = nC p ∆T , ∆U = nCV ∆T and therefore
p Table 15.3
Name of the
Q ∆U W
process
Isothermal Q=W 0 V  p
nRT ln  f  = nRT ln  i 
p  Vi   pf 

Adiabatic 0 nC V ∆T pi Vi − pf Vf
V = − ∆U
Vi Vf γ −1
Isobaric expansion of a gas Isobaric nC p ∆T nC V ∆T p ( Vf − Vi )
Fig. 15.12 Isochoric Q = ∆U nC V ∆T 0
= nC V ∆T
W = Q – ∆U = n (C p – CV ) ∆T = nR∆T
658 Objective Physics Vol. 1

Extra Knowledge Points X Example 15.7 An ideal gas expands


isothermally along AB and does 700 J of work.
■ Bulk modulus of a gas is given by p
(dp )  dp 
B=– =V –  A
(dV /V )  dV 
B
 dp   p
–  = γ   in an adiabatic process.
 dV  V 
Hence, adiabatic bulk modulus of an ideal gas is C
Bs = γ p V
Similarly, Fig. 15.14
 dp   p 
–  =   in an isothermal process. (a) How much heat does the gas exchange along AB?
 dV  V 
Hence, isothermal bulk modulus of an ideal gas is
(b) The gas then expands adiabatically along BC and
does 400 J of work. When the gas returns to A along
BT = p
CA, it exhausts 100 J of heat to its surroundings. How
much work is done on the gas along this path?
X Example 15.5 What is the heat input Sol. (a) AB is an isothermal process. Hence,
needed to raise the temperature of 2 moles of ∆U AB = 0 and QAB = WAB = 700 J
helium gas from 0°C to 100°C (b) BC is an adiabatic process. Hence,
(a) At constant volume? QBC = 0
(b) At constant pressure? WBC = 400 J (given)
(c) What is the work done by the gas in part? ∴ ∆U BC = – WBC = – 400 J
ABC is a cyclic process and internal energy is a state
(d) Give your answer in terms of R. function.
Therefore, (∆U )whole cycle = 0 = ∆U AB + ∆U BC + ∆UCA
Sol. Helium is monoatomic gas.
3R and from first law of thermodynamics,
Therefore, CV = QAB + QBC + QCA = WAB + WBC + WCA
2
5R Substituting the values, we get
and Cp =
2 700 + 0 – 100 = 700 + 400 + ∆WCA
(a) At constant volume, ∴ ∆WCA = – 500 J
Q = nC V ∆T Negative sign implies that work is done on the gas.
3R 
= (2 ) 
Table 15.4 shows different values in different
 (100) = 300R
 2  processes.
(b) At constant pressure, Table 15.4
Q = nC p ∆T Process Q (J) W (J) ∆U (J)
5R 
= (2 )   (100) = 500R AB 700 700 0
 2 
BC 0 400 – 400
(c) At constant pressure, CA – 100 – 500 400
W = Q – ∆U = nC p ∆T – nC V ∆T For complete cycle 600 600 0
= nR∆T = (2 ) (R ) (100) = 200R

X Example 15.6 An ideal monoatomic gas at Note Point


300 K expands adiabatically to twice its volume. What / Total work done is 600 J, which implies that area of the
is the final temperature? closed curve is also 600 J.

Sol. For an ideal monoatomic gas, γ = 5/ 3


15.4 Efficiency of a Cycle
In an adiabatic process,
TV γ – 1 = constant In a cyclic process, ∆U = 0
∴ Tf Vf γ –1
= TV γ –1 and Q net = Wnet (from first law of thermodynamics)
i i
γ –1 First we see what is the meaning of efficiency of a cycle.
V
or Tf = Ti  i  Suppose, 100 J of heat is supplied to a system (in our case it is
 Vf 
5
an ideal gas) and the system does 60 J of work. Then,
–1
1 3
= (300)   = 189 K
efficiency of the cycle is 60%. Thus, efficiency (η) of a cycle
2 can be defined as
The First Law of Thermodynamics 659

 Work done by the working substance  As discussed in article 15.4 thermal efficiency of a heat
 (an ideal gas in our case) during a cycle 
η=  ×100 engine is defined as the ratio of net work done per cycle by
 Heat supplied to the gas during the cycle  the engine to the total amount of heat absorbed per cycle by
 
the working substance from the source. It is denoted by η.
W | Q | – | Q–ve | Thus,
= total × 100 = +ve × 100
| Q+ve | | Q+ve | W Q1 − Q2 Q
η= = =1 − 2
 Q  Q1 Q1 Q1
= 1 – –ve  × 100
 Q+ve  As some heat is always rejected to the sink Q2 ≠ 0.
Wtotal  Q  Therefore, ηis always less than 1, i.e. thermal efficiency of a
Thus, η= × 100 = 1 – –ve  × 100 heat engine is always less than 100%.
| Q+ve |  Q+ve 
Note Points Types of Heat Engines
/ There cannot be a cycle whose efficiency is 100%. Hence, η In practice, heat engine are of two types :
is always less than 100%.
Thus, W total ≠ Q +ve
External Combustion Engine
/ It is just like a shopkeeper. He takes some money from you. In which heat is produced by burning, the fuel in a
(Suppose, he takes ` 100/- from you). In lieu of this he chamber outside the main body (working substance) of the
provides services to you (suppose he provides services of
worth ` 80/-). Then, the efficiency of the shopkeeper is 80%.
engine. Steam engine is an external combustion engine. The
There cannot be a shopkeeper whose efficiency is 100%. thermal efficiency of a steam engine varies from 10 to 20%.
Otherwise what will he save?
Internal Combustion Engine
X Example 15.8 In example 15.7 find the efficiency of In which heat is produced by burning the fuel inside the
the given cycle. main body of the engine. Petrol engine and diesel engines
Sol. From table 15.4 we can see that Q+ ve during the cycle is are internal combustion engine.
700 J, while the total work done in the cycle is 600 J.
Wtotal
Carnot Engine
∴ η= × 100
|Q+ve| Carnot cycle consists of the following four stages :
600  (i) Isothermal expansion (process AB)
=   × 100
 700  (ii) Adiabatic expansion (process BC)
= 85.71 % (iii) Isothermal compression (process CD)
(iv) Adiabatic compression (process DA)
15.5 Heat Engines The p-V diagram of the cycle is shown in the
figure below:
A heat engine is a device which converts heat energy
into mechanical energy. p

Source Working Sink A


T1 Substance T2 T1 B
Q1 Q2

Q1
W

D C
Fig. 15.15 T2
Q2

Any heat engine has some working substance (normally V


a gas in a chamber). In every heat engine the working Fig. 15.16
substance absorbs some heat (Q1) from a source
(at temperature T1 ), converts a part of it into work (W ) and In process AB, heat Q1 is taken by the working
the rest (Q2 ) is rejected to the sink (at temperature T2 ). substance at constant temperature T1 and in process CD, heat
From conservation of energy, Q2 is liberated from the working substance at constant
Q1 = W + Q2 temperature T2 . The net work done is area of graph ABCD.
660 Objective Physics Vol. 1

After doing the calculations for different processes, we can By doing the calculations, we can show that
show that T2 1− η
β= =
Q2 T2 T1 − T2 η
=
Q1 T1 Here, η is the efficiency of Carnot’s cycle.
Therefore, efficiency of the cycle is
η =1 − 2
T 15.7 Second Law of
T1
Thermodynamics
Note Point The first law of thermodynamics is the law of
conservation of energy. There are many processes that do
/ That efficiency of Carnot engine is maximum (not 100%) for
given temperaturesT1 and T2 . But still Carnot engine is not a
not violate first law of thermodynamics (energy
practical engine, because many ideal situations have been conservation law) but they are never observed. e.g. a book
assumed while designing this engine which can practically lying on a table cannot jump by itself. Although, if it jumps,
not be obtained. there is no violation of mechanical energy. The second law
of thermodynamics is the principle that forbids many
Otto or Petrol Engine phenomena consistent with the first law of thermodynamics.
This engine was made by Otto. This is also a four stroke The second law of thermodynamics gives a fundamental
engine. Four stroke means, in a cycle there are four limitation to the efficiency of a heat engine and the
processes. The working substance in it is 2% petrol and 98% coefficient of performance of a refrigerator. It says that
efficiency of a heat engine can never be unity (or 100%).
air. The four processes are charging stroke, compression
This implies that heat released to the cold reservoir can
stroke, working stroke and exhaust stroke. The efficiency of
never be made zero.
a petrol engine is about 52%.
For a refrigerator, the second law says that the
coefficient to performance can never be infinite. This
Diesel Engine implies that external work can never be zero.
The diesel engine was made by a German Engineer
Diesel. The efficiency of diesel engine is about 64%. Kelvin-Planck Statement
No process is possible which can completely convert
15.6 Refrigerator heat absorbed from a reservoir into work.
Refrigerator is an apparatus which takes heat from a Clausius Statement
cold body, work is done on it and the work done together No process is possible which can transfer heat from a
with the heat absorbed is rejected to the source. cold body to a hot body without doing any external work.

Source Q1 Working Sink Reversible and Irreversible Processes


T1 Substance T2 Most processes in nature are irreversible. It will violate
Q2
the second law of thermodynamics if it happens. The free
expansion of a gas is irreversible. Cooking gas leaking from
W
a gas cylinder in the kitchen diffuses to the entire process.
This is also an irreversible process.
Fig. 15.17 Irreversibility arises mainly due to dissipative forces
(like friction, viscosity etc). In the presence of dissipative
An ideal refrigerator can be regarded as Carnot’s ideal forces some mechanical energy is lost in the form of heat,
heat engine working in the reverse direction. sound etc. Since, dissipative effects are present everywhere
Coefficient of Performance and can be minimised but not fully eliminated most
processes we deal with are irreversible.
Loefficient of performance (β) of a refrigerator is
A reversible process is an idealised motion. A process is
defined as the ratio of quantity of heat removed per cycle
reversible, only if it is quasi-static (system in equilibrium
(Q2 ) to the work done on the working substance per cycle to
with the surroundings at every stage) and there are no
remove this heat. Thus,
dissipative effects. For example, a quasi-static isothermal
Q Q2
β= 2 = expansion of an ideal gas in a cylinder fitted with a
W Q1 − Q2 frictionless movable piston is a reversible process.
The First Law of Thermodynamics 661

5
X Example 15.9 The p-V diagram of 0.2 mole of a = (0.2)   (8.31)(−145) J ≈ – 602 J
2
diatomic ideal gas is shown in figure. Process BC is
∴ ∆WBC = – ∆U BC = 602 J
adiabatic. The value of γ for this gas is 1.4.
p Process CA is an isobaric process. Hence,
7
∆QCA = nC p ∆T = n  R  (TA – TC )
600 K 2 
7
B
= (0.2)   (8.31) (300 – 455) ≈ – 902 J
2
∆QCA  Cp 
∆UCA = nC V ∆T=  as γ = 
A C
450 K γ  CV 
1.0 atm
300 K 902
=– ≈ – 644 J
V 1.4
∴ ∆WCA = ∆QCA – ∆UCA = – 258 J
Fig. 15.18
Process ∆Q (in J) ∆W (in J) ∆U (in J)
(a) Find the pressure and volume at points A, B and C. AB 1246 0 1246
(b) Calculate ∆Q, ∆W and ∆U for each of the three BC 0 602 – 602
processes.
CA – 902 – 258 – 644
(c) Find the thermal efficiency of the cycle.
Total 344 344 0
(Take, 1 atm = 1.0 × 10 5 N / m 2 )
(c) Efficiency of the cycle
Sol. (a) pA = pC = 1 atm = 1.01 × 105 N/m2 W 344
η = total × 100 = × 100 = 27.6%
Process AB is an isochoric process. |Q+ve| 1246
pB TB
∴ p ∝ T or =
pA TA X Example 15.10 Carnot engine takes in a
T  600 
1000 kcal of heat from a reservoir at 827°C and
∴ pB =  B  pA =   (1 atm) =2 atm exhausts it to a sink at 27°C. How much work does it
 TA   300 
perform ? What is the efficiency of the engine?
= 2.02 × 105 N/m2
From ideal gas equation, Sol. Given, Q1 = 106 cal
nRT T1 = (827 + 273) = 1100 K
V=
p and T2 = (27 + 273) = 300 K
nRTA (0.2) (8.31) (300) Q2 T2
∴ VA = VB = = as, =
pA (1.01 × 105 ) Q1 T1
300 
Q2 = 2 ⋅ Q1 = 
T 6
≈ 5.0 × 10–3 m3 = 5L ∴  (10 )
T1  1100 
nRTC (0.2) (8.31) (455)
and VC = = = 2.72 × 105 cal
pC (1.01 × 105 )
Efficiency of the cycle,
= 7.6 × 10–3 m3 ≈ 7.6 L  T  300 
η =  1 − 2  × 100 or η =  1 −  × 100
State p V  T1   1100 
A 1 atm 5L = 72.72%
B 2 atm 5L X Example 15.11 Calculate the least amount of
C 1 atm 7.6 L work that must be done to freeze one gram of water at
(b) Process AB is an isochoric process. Hence, 0°C by means of a refrigerator. Temperature of
∆WAB = 0 ⇒ ∆QAB = ∆U AB surroundings is 27°C. How much heat in passed on the
5
= nC V ∆T = n  R  (TB – TA )
surroundings in this process. Latent heat of fusion
2  L = 80 cal/g.
5
= (0.2)   (8.31) (600 – 300) ≈ 1246 J
2 Sol. Q2 = mL = 1 × 80 = 80 cal
Process BC is an adiabatic process. Hence, T2 = 0° C = 273 K and T1 = 27 ° C = 300 K
∆QBC = 0 Q2 T2
=
∴ ∆WBC = – ∆U BC W T1 − T2
Q (T − T2 ) 80 (300 − 273)
∆U BC = nC V ∆T = nC V (TC – TB ) ∴ W = 2 1 = = 7.91 cal
5 T2 273
= (0.2)  R  (455 – 600)
2  Q1 = Q2 + W = (80 + 7.91) = 87.91 cal
Chapter Summary with Formulae
■ Thermodynamics
p
(i) Molar heat capacity C = heat required to raise the
temperature of 1 mole of any substance by 1°C or 1 K.
Q
= ⇒ ∴ Q = nC∆T
n∆ T
Molar heat capacity of solids and liquids is almost V
constant.
W = 0 as volume is constant.
In case of gases C is process dependent. It varies from p
0 to ∞.
In isothermal process
C = ∞ as ∆T = 0
In adiabatic process
C = 0 as Q = 0
V
C p (molar heat capacity of isobaric process) and CV
(molar heat capacity of isochoric process) are commonly W = +ve as cyclic process is clockwise with p on
used. In a general process pV x = constant, molar heat y-axis.
capacity is given by, (b) Change in internal energy ∆U
C=
R
+
R ∆U = nCV ∆T for all processes. For thisCV (or nature
γ −1 1− x of gas), n and ∆T should be known. If either of the
three terms is not known, we can calculate ∆U by,
(ii) First law of thermodynamics It is the law of ∆U = Q − W
conservation of energy given by,
(c) Heat exchange Q
Q = ∆U + W
Q = nC∆T. For this n, ∆T and molar heat capacity C
(iii) Detailed discussion of three terms of first law of should be known. C is a process dependent. So, if
thermodynamics. either of the three terms ( n, ∆T or C ) is not known,
(a) Work done Following methods are generally we can calculate Q by,
used to find the work done. Q = ∆U + W
Vf (iv) Calorie is the old unit of heat. 1 calorie is the amount of
■ Method 1. W = ∫V pdV (because dW = pdV ) heat required to raise the temperature of 1 g of water
from14.5° C to15.5° C. 1 cal = 4.186 J
i

Here, p should be either constant or function of V. If (v) For an ideal gas, the molar specific heat capacities at
p is constant. It means process is isobaric, constant pressure and volume satisfy the relation
W = p(Vf − Vi ) = p∆V C p − CV = R
■ Method 2. Work done can also be obtained by area under where, R is the universal gas constant.
p-V diagram with projection on V-axis. (vi) Heat engine is a device in which a system undergoes a
p cyclic process resulting in conversion of heat into work.
If Q1 is the heat absorbed from the source, Q2 is the heat
released to the sink and the work output in one cycle is
W, the efficiency η of the engine is
W Q
η= =1− 2
Q1 Q1
(vii) In a refrigerator or a heat pump, the system extracts heat
Q2 from the cold reservoir and releasesQ1 amount of heat
V to the hot reservoir, with work done W on the system.
The coefficient of performance of a refrigerator is given
W = +ve as volume is increasing. by
Q Q2
p β= 2 =
W Q1 − Q2
(viii) Carnot engine is a reversible engine operating between
two temperatures T1 (source) and T2 (sink). The Carnot
cycle consists of two isothermal processes connected by
two adiabatic processes. The efficiency of a Carnot
engine is given by
T
η =1− 2 (Carnot engine)
V T1
No engines operating between two temperatures can
W = −ve as volume is decreasing
have efficiency greater than that of the Carnot engine.
Additional Examples
Example 1. A slab of ice at 273 K and at Example 7. First law of thermodynamics does not
atmospheric pressure melts forbid flow of heat from lower temperature to higher
(a) What is the nature of the work done on the ice water temperature. Comment.
system by the atmosphere ? Sol. First law of thermodynamics simply tells about the
(b) What happens to the internal energy of ice water conversion of mechanical energy into heat energy and
system ? vice-versa. It does not put any condition as to why heat cannot
Sol. (a) The volume of ice decreases on melting. Hence, the flow from lower temperature to higher temperature.
work done by the atmosphere on ice water system is
Example 8. Find the specific heat of the process
positive in nature. a
(b) Since, heat is absorbed by the ice during melting, the p = for a monoatomic gas, a being constant.
T
internal energy of the ice water system increases.
Sol. We know that, dQ = dU + dW
Example 2. If a door of a working refrigerator is dQ dU dW
Specific heat, C= = + …(i)
kept open for a long time in a closed room, will it make dT dT dT
the room warm or cool ? Since, dU = C V dT …(ii)
Sol. Suppose that the refrigerator draws some heat from the air dW pdV
C = CV + = CV +
in front of it. The compressor has to do some mechanical work dT dT
(at the expense of electrical energy) to draw heat from the air at pV = RT
lower temperature. The heat drawn from the air together with ∴ For the given process,
the work done by the compressor in drawing it, is rejected by RT RT 2 dV 2RT
the refrigerator with the help of the radiator (provided at the V= = or =
back) to the air. p a dT a
 2RT 
∴ C = CV + p  
Example 3. When a gas is suddenly compressed, its  a 
temperature rises. Why ? 3 7
= C V + 2R = R + 2R = R
Sol. Sudden compression of a gas is an adiabatic process. The 2 2
work done in compressing the gas increases the internal
energy of the gas. Hence, the temperature of the gas rises. Example 9. A gaseous mixture enclosed in a vessel
 5
Example 4. If an inflated tyre bursts, the air consists of 1 g mole of a gas A with γ =   and some
escaping out is cooled, why ?  3
Sol. When the tyre bursts, there is an adiabatic expansion of air 7
because the pressure of the air inside is sufficiently greater
amount of gas B with γ = at a temperature T. The
5
than the atmospheric pressure. During the expansion, the air
does some work against the surroundings, therefore, its
gases A and B do not react with each other and are
internal energy decreases, and as such temperature falls. assumed to be ideal. Find the number of gram moles of
 19
Example 5. Is it possible that there is change in the gas B, if γ for the gaseous mixture is   .
 13
temperature of a body without giving heat to it or
taking heat from it ? C p 
Sol. As for an ideal gas, C p − C V = R and γ =  
Sol. Yes, for example, during an adiabatic compression CV 
temperature increases and in an adiabatic expansion
R
temperature decreases, although no heat is given to or taken So,C V =
from the system in these changes. (γ − 1)
R 3
Example 6. Is it possible that there is no change in ∴ (C V )1 = = R,
 5 2
temperature of a body despite being heated?   −1
 3
Sol. Yes, for example during a change of state (from solid to R 5
liquid or from liquid to gas), the system takes heat, but there is (C V ) 2 = = R
no rise in temperature. Internal energy of the system 7 2
  −1
increases in each case.  5
664 Objective Physics Vol. 1

R 13
and (C V ) mix = = R Sol. (a) According to the given problem
 19 6
∆Q = Q1 + Q 2 + Q 3 + Q 4
  −1
 13
= 5960 − 5585 − 2980 + 3645
Now, from conservation of energy, ∆Q = 9605 − 8565 = 1040 J
i.e. ∆U = ∆U 1 + ∆U 2 ∆W = W1 + W 2 + W 3 + W 4
(n1 + n 2 ) (C V ) mix ∆T = [n1 (C V )1 + n 2 (C V ) 2 ] ∆T = 2200 − 825 − 1100 + W 4 = 275 + W 4
n (C ) + n 2 (C V ) 2
i.e. (C V ) mix = 1 V 1 And as for cyclic process
n1 + n 2 UF = UI , ∆U = UF − UI = 0
3 5 So, from first law of thermodynamics, i.e.
1 × R + n2 R ∆Q = ∆U + ∆W
We have, R =
13 2 2 = (3 + 5n 2 )R
6 1 + n2 2 (1 + n 2 ) We have 1040 = (275 + W 4 ) + 0 , i.e. W 4 = 756 J
(b) As, efficiency of a cycle is defined as
or 13 + 13 n 2 = 9 + 15 n 2 , i. e. n 2 = 2 g-mol Net work ∆W ∆Q
η= = =
Example 10. An ideal monoatomic gas at Input heat (Q1 + Q 4 ) (Q1 + Q 4 )
temperature 27°C and pressure 10 6 N/m 2 occupies 10 L 1040
η= = 0.1082 = 10.82%
volume. 10000 cal of heat is added to the system 9605
without changing the volume. Calculate the final
Example 12. At 27°C two moles of an ideal
temperature of the gas.
monoatomic gas occupy a volume V . The gas expands
(Given, R = 8.31 J/mol-K and J = 4.18 J/cal) adiabatically to a volume 2V . Calculate
pV 10 6 × 10 −2
Sol. ∴ n = = = 4.0 (a) the final temperature of the gas
RT 8.31 × 300 (b) the change in its internal energy and
3
For monoatomic gas, C V = R (c) the work done by the gas during the process.
2 (R = 8.31 J/mol-K)
3
Thus, C V = × 8.31 J/mol-K Sol. (a) In case of adiabatic change
2
3 8.31 TV γ = constant
= × ≈ 3 cal /(mol-K)
 5
2 4.18 ∴T1V1γ −1
= T 2 V 2γ −1
with γ =  
Let ∆T be the rise in temperature, when n mole of the  3
gas is given Q cal of heat at constant volume. Then, 300
i.e. 300 × V 2 / 3 = T ( 2 V ) 2 / 3 or T = = 189 K
Q = nC V ∆T (2) 2 / 3
Q (b) According to first law of thermodynamics
or ∆T =
nC V Q = ∆U + ∆W
10000 cal And as for adiabatic change ∆Q = 0,
= ∆W = − ∆U
4.0 mol × 3 cal (mol - K )
= 2767.23 J
= 833 K
∆T = T f − Ti Example 13. One mole of a monoatomic ideal gas
T f = ∆T + Ti is taken through the cycle as shown in figure.
= 833 + 300 A → B Adiabatic expansion
= 1133 K B → C Cooling at constant volume
Example 11. An ideal gas is taken through a cyclic C → D Adiabatic compression
thermodynamic process through four steps. The D → A Heating at constant volume
amounts of heat involved in these steps are
Q1 = 5960 J , Q 2 = − 5585 J , Q 3 = − 2980 J and
A
Q 4 = 3645 J respectively. The corresponding works B
involved are W1 = 2200 J ,W 2 = − 825 J ,
W 3 = − 1100 J and W 4 respectively. p D
C
(a) Find the value of W4 .
(b) What is the efficiency of the cycle? V
The First Law of Thermodynamics 665

The pressure and temperature at A, B , etc., are denoted by Example 14. The density versus pressure graph of
p A , T A , pB , TB , etc., respectively. one mole of an ideal monoatomic gas undergoing a
 2  1 cyclic process is shown in figure. The molecular mass
Given, T A = 1000 K, pB =   p A and pC =   p A .
 3  3 of the gas is M.
Calculate (a) Find the work done in each process.
(b) Find heat rejected by gas in one complete cycle.
(a) the work done by the gas in the process A → B . (c) Find the efficiency of the cycle.
(b) the heat lost by the gas in the process B → C . ρ
0.4 2ρ0 B
 2
Given   = 0.85 and R = 8.31 J/mol-K
 3
ρ0 A
C
Sol. (a) As for adiabatic change pV γ = constant
p
i.e. p0 2p0
γ
 nRT  Sol. (a) As n = 1, m = M
p  = constant (as pV = nRT )
 P  Process AB ρ ∝ p, i.e. it is an isothermal process
Tγ pM
i.e. = constant, (T = constant), because ρ = .
pγ −1 RT
 TB 
γ
p 
γ −1 p   1 p M
 5 ∴ W AB = RT A ln  A  = RT A ln   = – 0 ln (2)
So,   = B  where γ =   pB    ρ0
T A  pA  3 2
ρ0 M
∆U AB = 0 and Q AB = W ABG =
1
1− ln (2)
 2 γ ρ0
i.e. TB = T A  
 3 Process BC is an isobaric process (p = constant)
2 /5
 2 W BC = p B (V C – V B )
= 1000   = 850 K
 3  M M  2p 0 M p 0 M
nR[T F − T I ] = 2p 0  – = =
So, W AB =  ρC ρB  2ρ0 ρ0
[1 − γ ]
∆U BC = C V ∆T
1 × 8.31[1000 − 850 ]
=  3   2p M 2p M  3p M
 5  =  R  0 – 0  = 0
 2   ρ0 R 2ρ0 R  2ρ0
 3 − 1
  5p 0 M
 3 Q BC = W BC + ∆U BC =
i.e. W AB =   × 8.31 × 150 2ρ0
 2
= 1869.75 J Process CA As ρ = constant ⇒ ∴ V = constant.
(b) For B → C , V = constant, so ∆W = 0 So, it is an isochoric process.
W CA = 0
So, from first law of thermodynamics 3 
∆U CA = C V ∆T =  R (T A – T C )
∆Q = ∆U + ∆W = nC V ∆T + 0 2 
3   3  3   p 0 M 2p 0 M  3p 0 M
∆Q = 1 ×  R (T C − 850 ) as C V = R =  R  ρ R – ρ R  = – 2ρ
or
2   2  2   0 0  0
Now along path BC , V = constant, p ∝ T 3p 0 M
p C TC Q CA = ∆U CA = –
i.e. = 2ρ0
p B TB (b) Heat rejected by gas = | Q AB | + | Q CA |
 1 p M 3 
  pA = 0  + ln (2)
 3 ρ0  2 
TC = × TB
 2 (c) Efficiency of the cycle (in fraction)
  pA
 3 Total work done W total
η= =
T 850 Heat supplied Q+ve
= B = = 425 K …(ii)
2 2 p0 M
3 [1 – ln (2)]
So, ∆Q = 1 × × 8.31 (425 − 850) ρ0 2
2 = = [1 – ln (2)]
= − 5297.625 J 5  p 0 M 5
 
Negative sign means, heat is lost by the system. 2  ρ0 
NCERT Selected Questions
Q 1. What amount of heat must be supplied to ∆Q = 9.35 cal
2.0 × 10 −2 kg of nitrogen (at room temperature) to = 9.35 × 4.19 J (Q1 cal = 4.19 J)
raise its temperature by 45°C at constant = 39.18 J
pressure? (Molecular mass of N 2 = 28, Let ∆W be the net work done by the system, then using first
R = 8.3 J mol −1 K −1 ). law of thermodynamics,
∆Q = ∆U + ∆W ⇒ ∆W = ∆Q − ∆U
Sol. Here, mass of gas, m = 2 × 10−2 kg = 20 g
= 39.18 − 22.3 = 16.88 J
Rise in temperature, ∆T = 45° C ≈ 16.9 J
Molar mass of N2 , M = 28 g
If n = number of moles, then Q 4. Two cylinders A and B of equal capacity are
m 20 connected to each other via a stopcock. A contains a
n= = = 0.714 gas at standard temperature and pressure. B is
M 28
completely evacuated. The entire system is
If C p be the molar specific heat of the gas at constant
thermally insulated. The stopcock is suddenly
pressure, then
opened.
7 7
C p = R = × 8.3 J mol −1K−1 Answer the following questions:
2 2 (a) What is final pressure of the gas in A and B ?
7
Now, Q = nC p ∆T = 0.714 × × 8.3 × 45 J (b) What is the change in internal energy of the gas?
2 (c) What is the change in temperature of the gas?
= 933.37 J ≈ 933 J
Sol. (a) As, the initial and final temperatures remain the same,
Q 2. A cylinder with a movable piston contains 3 moles then
of hydrogen at standard temperature and pressure. p1V1 = p2V2
The walls of the cylinder are made of heat insulator, pV pV p
and the piston is insulated by having a pile of sand p2 = 1 1 = 1 1 = 1
V2 2V1 2
on it. By what factor does the pressure of the gas
increase, if the gas is compressed to half its original = 0.5 atm
volume? (b) Zero this is because no work is done on/by the gas, thus
there will be no change in the internal energy of the gas
Sol. For diatomic gas or hydrogen, under constant temperature conditions.
7
γ = = 1.4 (c) If the gas is assumed to be ideal, then there is no change
5 in the temperature of the gas as it does no work in
As no heat is allowed to be exchanged, the process is expansion, i.e. ∆T = 0.
adiabatic. Now for adiabatic change,
γ Q 5. A steam engine delivers 5.4 × 10 8 J of work per
p2  V1  minute and takes 3.6 × 10 9 J of heat per minute from
p1V1γ = p2V2γ or = 
p1  V2  the boiler. What is the efficiency of the engine?
= (2)1.4 = 2.64 How much heat is wasted per minute?
Sol. Let
Q 3. In changing the state of a gas adiabatically from an
equilibrium state A to another equilibrium state B, Q1 = heat absorbed per minute
an amount of work equal to 22.3 J is done on the Q2 = heat rejected per minute
system. If the gas is taken from state A to B via a W
We know that, η% = × 100
process in which the net heat absorbed by the Q1
system is 9.35 cal, how much is the net work done 5.4 × 108 J 3
by the system in the latter case? ∴ η% = × 100 = × 100 = 15%
3.6 × 10 J
9
20
(Take, 1 cal = 4.19 J).
Also using the relation, Q1 = W + Q2, we get
Sol. In adiabatic process, Q2 = Q1 − W = 36 × 108 − 5.4 × 108
∆U = work done on the system = 2.33 J ...(i)
In second case, when the state A is taken to state B, the heat = 30.6 × 108 J/min = 3.06 × 109 J/min
absorbed by the system ∆Q is ≈ 3.1 × 109 J/min
The First Law of Thermodynamics 667

Q 6. An electric heater supplies heat to a system at a rate where, W 1 = work done during the process from D to E
of 100 W. If the system performs work at a rate of (expansion)
75 J/s. At what rate is the internal energy = area DEHGD
increasing? 1 
∴ W1 = × 3 × 300 + 3 × 300 J = 1350 J ...(i)
 2 
Sol. According to the first law of thermodynamics,
W 2 = work done during the process from E to F
∆U = ∆Q − ∆W = 100 − 75 = 25 J/s
(compression)
= 25W
= area EHGF = − 300 × 3 J = − 900 J ...(ii)
Q 7. A thermodynamic system is taken from an original ∴ Total work done
state to an intermediate state by linear process shown W = 1350 − 900 = 450 J
in figure here. Its volume is then reduced to the
original value from E to F by an isobaric process. Q 8. A refrigerator is to remove heat from the eatable
Calculate the total work done by the gas from D to E kept inside at 9°C. Calculate the coefficient of
to F . performance if the room temperature is 36°C.
Sol. T1 = 273 + 36 = 309 K
Sol.
D
600 T2 = 9° C = 282 K
Coefficient of performance,
T2
p (Nm )

β=
–2

,
T1 − T2
F E
300 We get,
282
β=
309 − 282
G H
0 282
2.0 5.0 = = 10.4
V (m3) 27
Objective Problems
[ Level 1 ]
First law of Thermodynamics and 6. An ideal gas of volume 1.5 × 10−3 m 3 and at pressure
Graphs 1.0 × 105 Pa is supplied with 70 J of energy. The volume
increases to 1.7 × 10−3 m 3 , the pressure remaining
1. First law of thermodynamics corresponds to constant. The internal energy of the gas is
(a) conservation of energy (a) increased by 90 J (b) increased by 70 J
(b) heat flow from hotter to colder body (c) increased by 50 J (d) decreased by 50 J
(c) law of conservation of angular momentum
(d) Newton’s law of cooling 7. Corresponding to the process shown in figure, what is the
heat given to the gas in the process ABCA?
2. A system is taken from state A to state B along two
different paths 1 and 2. The heat absorbed and work done B C
4
by the system along these two paths are Q1 and Q2 and W1 3

p (Nm )
–2
and W2 , respectively. Then 2 A
(a) Q1 = Q2 1
(b) W1 = W2
(c) Q1 − W 1 = Q2 − W 2 3
V (m )
(d) Q1 + W 1 = Q2 + W 2 0 1 2 4

3
3. A certain amount of an ideal gas is taken from state A to (a) 1 J (b) J
state B, one time by process I and another time by process 2
1
II. If the amount of heat absorbed by the gas are Q1 and (c) J (d) 0
2
Q2 respectively, then
p 8. A closed system undergoes a change of state by process
1→ 2 for which Q12 = 10 J and W12 = − 5 J. The system is
I
now returned to its initial state by a different path 2 → 1
A B
for which Q21 is −3 J. The work done by the gas in the
II process 2 → 1 is
(a) 8 J (b) zero
V
(c) −2 J (d) +5 J
(a) Q1 = Q2 (b) Q1 < Q2
(c) Q1 > Q2 (d) None of these
9. A thermodynamical system is changed from state
( p1 , V1 ) to ( p 2 , V2 ) by two different process. The
4. Work done by the gas in the process shown in figure is quantity which will remain same will be?
p (a) ∆Q (b) ∆W
B (c) ∆Q + ∆W (d) ∆Q − ∆W

10. In thermodynamic process, 200 J of heat is given to a gas


C A and 100 J of work is also done on it. The change in
internal energy of the gas is
D
V (a) 100 J (b) 300 J
(c) 419 J (d) 240 J
(a) positive
(b) negative 11. If ∆Q and ∆W represent the heat supplied to the system
(c) zero and the work done on the system respectively, then the
(d) may be positive or negative first law of thermodynamics can be written as
(a) ∆Q = ∆U + ∆W
5. Identify the incorrect statement related to a cyclic process
(b) ∆Q = ∆U − ∆W
(a) the initial and final conditions always coincide
(c) ∆Q = ∆W − ∆U
(b) Q =W
(d) ∆Q = − ∆W − ∆U
(c) W >0
(d) None of the above where, ∆U is the change in internal energy.
The First Law of Thermodynamics 669

12. In a given process for an ideal gas, dW = 0 and dQ < 0. 19. The p-V diagram of a system undergoing thermodynamic
Then, for the gas transformation is as shown in figure. The work done on
(a) the temperature will decrease the system in going from A → B → C is 50 J and 20 cal
(b) the volume will increase heat is given to the system. The change in internal energy
(c) the pressure will remain constant between A and C is
(d) the temperature will increase p
B
13. When 1 g of water at 0°C and 1 × 105 N / m 2 pressure is
converted into ice of volume 1.092 cm 3 , then work done
will be
(a) 0.0091 J (b) 0.0182 J
(c) 0.091 J (d) Data insufficient A C

14. In an adiabatic expansion of one mole of gas initial and


V
final temperatures are T1 and T2 respectively, then the
change in internal energy of the gas is (a) 34 J (b) 70 J (c) 84 J (d) 134 J

(a)
R
(T2 − T1 ) (b)
R
(T1 − T2 ) 20. An ideal gas is taken through the cycle A → B → C → A,
γ −1 γ −1 as shown in the figure. If the net heat supplied to the gas
(c) R (T1 − T2 ) (d) zero in the cycle is 5 J, then work done by the gas in the
process C → A is
15. A gas expands under constant pressure p from volume V1 3
V(m )
and V2 . The work done by the gas is C B
2
(a) p (V2 − V1 ) (b) p (V1 − V2 )
VV
(c) p (V1γ − V2γ ) (d) p 1 2
V2 − V1

16. A gas is compressed at a constant pressure of 50 N/m 2 1 A


from a volume of 10 m 3 to a volume of 4 m 3 . Energy of
100 J, then added to the gas by heating. Its internal 2
10 p (N/m )
energy is
(a) increased by 400 J (b) increased by 200 J (a) −5 J (b) −10 J (c) −15 J (d) −20 J
(c) increased by 100 J (d) decreased by 200 J
21. In the p-V diagram, shown in figure the net amount of
17. A thermodynamic system is taken through the cycle work done will be
PQRSP process. The net work done by the system is p
p

S R
200 kPa 1 2

100 kPa V
P Q
(a) positive (b) negative (c) zero (d) infinity
V
100 cc 300 cc 22. In the cyclic process ABCDA shown in the figure,
(a) 20 J (b) −20 J (c) 400 J (d) −400 J consider the following statements.
p A
18. An ideal gas is taken around ABCA as shown in the p-V
B
diagram. The work done during the cycle is
p
B (3p, 3V) D
C

V
I. Area ABCD = Work done on the gas
A C
(p,V) (p, 3V) II. Area ABCD = Net heat absorbed
III. Change in the internal energy in cycle = 0
V
pV Which of these are correct?
(a) 2pV (b) pV (c) (d) zero (a) Only I (b) Only II (c) II and III (d) I, II and III
2
670 Objective Physics Vol. 1

23. Carbon monoxide is carried around a closed cycle abca 27. A sample of an ideal gas is taken through a cycle as
in which bc is an isothermal process as shown in the shown in figure. It absorbs 50 J of energy during the
figure. The gas absorbs 7000 J of heat as its temperature process AB, no heat during BC, rejects 70 J during CA.
increases from 300 K to 1000 K in going from a to b. The 40 J of work is done on the gas during BC. Internal energy
quantity of heat rejected by the gas during the process ca of gas at A is 1500 J, the internal energy at C would be
is approximately p
B
p
b
p2

C A
V
p1 c
a (a) 1590 J (b) 1620 J (c) 1540 J (d) 1570 J

V 28. An ideal monoatomic gas is taken around the cycle as


V1 V2 shown in p-V diagram. The work done during the cycle is
(a) 4200 J (b) 5000 J (c) 9000 J (d) −9800 J given by
p
24. When a system is taken from state i to a state f along path 2p0
iaf, Q = 50 J and W = 20 J. If W = − 13 J for the curved
return path fi, Q in this path is
p
p0

a f

0 V0 3V0 V
1
(a) p0V0 (b) p0V0
i 2
(c) 2 p0V0 (d) None of these
V
(a) 33 J (b) 23 J (c) −7 J (d) −43 J Different known Thermodynamic
25. The p-V diagram of a system undergoing thermodynamic Processes
transformation is as shown in figure. The work done by
the system in going from A → B → C is 30 J and 40 J heat 29. 70 cal of heat are required to raise the temperature of
is given to the system. The change in internal energy 2 moles of an ideal gas at constant pressure from 30°C to
between A and C is 35°C. The amount of heat required in calories to raise the
p temperature of the same gas through, the same range
C
(30°C-35°C) at constant volume is
(a) 30 cal (b) 50 cal (c) 40 cal (d) 90 cal

30. The volume of a gas is reduced adiabatically to 1/4 of its


volume at 27°C. If γ = 1.4, the new temperature will be
A B (a) (300) (2)0.4 K (b) (300) (4)1.4 K
V
(c) (300) (4)0.4 K (d) (300) (2)1/ 4 K
(a) 10 J (b) 70 J (c) 84 J (d) 134 J

26. p-V diagram of a cyclic process ABCA is as shown in 31. Which is incorrect?
(a) In an isobaric process, ∆p = 0
figure. Choose the correct statement.
(b) In an isochoric process, ∆W = 0
p (c) In an isothermal process, ∆T = 0
A
(d) In an isothermal process, ∆Q = 0
32. The molar heat capacity of a gas at constant volume is
CV . If n moles of the gas undergo ∆T change in
B
temperature, its internal energy will change by nCV ∆T
C
V (a) only if the change of temperature occurs at constant volume
(b) only if the change of temperature occurs at constant pressure
(a) ∆QA → B = constant (b) ∆U B → C = positive (c) in any process which is not adiabatic
(c) ∆WCAB = negative (d) All of these (d) in any process
The First Law of Thermodynamics 671

33. During an adiabatic process, the pressure of a gas is to p1 . The gas is then allowed to expand its original
found to be proportional to the cube of its absolute volume. Upon which of the following p-V graphs are
Cp these processes correctly shown?
temperature. The ratio = γ for the gas is
CV p p
3
(a) 2 (b)
2
5 4 (a) p1 (b) p1
(c) (d)
3 3

34. In an adiabatic expansion, a gas does 25 J of work while V1 V1


in an adiabatic compression 100 J of work is done on a p p
gas. The change of internal energy in the two processes
respectively are
(c) p1 (d) p1
(a) 25 J and −100 J (b) −25 J and 100 J
(c) −25 J and −100 J (d) 25 J and 100 J
5
35. For a gas γ = and 640 cc of this gas is suddenly V1 V1
3
compressed to 80 cc. If the initial pressure is p, then the 41. A monoatomic ideal gas, initially at temperature T1 , is
final pressure will be enclosed in a cylinder fitted with a frictionless piston.
(a) 8p (b) 32p The gas is allowed to expand adiabatically to a
(c) 16p (d) 64 p
temperature T2 by releasing the piston suddenly. If L1 and
36. When an ideal gas is compressed isothermally, then its L2 are lengths of the gas column before and after
pressure increases because T
expansion respectively, then 1 is given by
(a) its potential energy increases T2
(b) its kinetic energy increases and molecules move apart 2/ 3 5/ 3 5/ 3 2/ 3
(c) its number of collisions per unit area with walls of container L  L  L  L 
(a)  1  (b)  1  (c)  2  (d)  2 
increases  L2   L2   L1   L1 
(d) molecules energy increases
42. Starting with the same initial conditions, an ideal gas
37. In an isothermal expansion of an ideal gas, select wrong expands from volume V1 to V2 in three different ways.
statement. The work done by the gas is W1 , if the process is
(a) There is no change in the temperature of the gas isothermal, W2 if isobaric and W3 if adiabatic, then
(b) There is no change in the internal energy of the gas
(a) W 2 > W 1 > W 2 (b) W 2 > W 3 > W 1
(c) the work done by the gas is equal to the heat supplied to the
gas (c) W 1 > W 2 > W 3 (d) W 1 > W 3 > W 2
(d) The work done by the gas is equal to the change in its internal 43. A and B are two adiabatic curves for two different gases.
energy
Then A and B corresponds to
38. When an enclosed perfect gas is subjected to an adiabatic p
process, then
(a) its total internal does not change
A
(b) its temperature does not change
(c) its pressure varies inversely as a certain power of its volume B
(d) the product of its pressure and volume is directly
proportional to its absolute temperature
V
39. One mole of an ideal monoatomic gas is at 360 K and a (a) Ar and He respectively (b) He and H2 respectively
pressure of 105 Pa. It is compressed at constant pressure (c) O2 and H2 respectively (d) H2 and He respectively
until its volume is halved. Taking R as 8.3 J mol −1 K −1
44. Choose the incorrect statement related to an isobaric
and the initial volume of the gas as 3.0 × 10−2 m 3 , the
process.
work done on the gas is V
(a) −1500 J (b) +1500 J (a) = constant
T
(c) −3000 J (d) +3000 J
(b) W = p ∆T
40. A certain mass of an ideal gas is at pressure p1 and (c) Only heat given to a system is used up in raising the
volume V1 . It is compressed isothermally and then temperature
allowed to expand adiabatically until its pressure returns (d) None of the above
672 Objective Physics Vol. 1

45. The amount of heat required to raise the temperature of 1


54. A gas for which γ = 1.5 is suddenly compressed to th of
1 mole of a monoatomic gas from 20°C to 30°C at 4
constant volume is H. Then, the amount of heat required the initial volume. Then, the ratio of the final to the initial
to raise the temperature of 2 mole of a diatomic gas from pressure is
20°C to 25°C at constant pressure is (a) 1 : 16 (b) 1 : 8
4 (c) 1 : 4 (d) 8 : 1
(a) 2H (b) H
3 55. Which of the following is a slow process?
5 7
(c) H (d) H (a) Isothermal (b) Adiabatic
3 3
(c) Isobaric (d) None of these
46. Specific heat of gas undergoing adiabatic changes is
56. The work done in which of the following processes is
(a) zero (b) infinite (c) positive (d) negative
zero?
47. During an isothermal expansion of an ideal gas (a) Isothermal process
(a) its internal energy decreases (b) Adiabatic process
(b) its internal energy does not change (c) Adiabatic
(c) the work done by the gas is equal to the quantity of heat (d) Isochoric
absorbed by it
(d) Both (b) and (c) are correct 57. In an isochoric process, if T1 = 27° C and T2 = 127° C,
p
48. Two identical samples or a gas are allowed to expand then 1 will be equal to
p2
(i) isothermally
9 2
(ii) adiabatically. (a) (b)
59 3
Work done is 3
(c) (d) None of these
(a) more in the isothermal process 4
(b) more in the adiabatic process
(c) zero in both of them 58. A monoatomic gas is supplied the heat Q very slowly
(d) equal in both processes keeping the pressure constant. The work done by the gas
49. A given system undergoes a change in which the work will be
done by the system equals the decrease in its internal 2 3
(a) Q (b) Q
energy. The system must have undergone an 3 5
2 1
(a) isothermal change (b) adiabatic change (c) Q (d) Q
5 5
(c) isobaric change (d) isochoric change

50. A cycle tyre bursts suddenly. This represents an 59. The temperature of a hypothetical gas increases to
(a) isothermal process (b) isobaric process 2 times, when compressed adiabatically to half the
(c) isochoric process (d) adiabatic process volume. Its equation can be written as
(a) pV 3/ 2 = constant (b) pV 5/ 2 = constant
51. An ideal gas at a pressure of 1 atm and temperature of
(c) pV 7 / 3 = constant (d) pV 4/ 3 = constant
27°C is compressed adiabatically until its pressure
becomes 8 times the initial pressure, then the final 60. Which of the p-V, diagrams best represents an isothermal
temperature is ( γ = 3/ 2) process?
(a) 627°C (b) 527°C (c) 427°C (d) 327°C p p
52. In an adiabatic process, the state of a gas is changed from
p1 , V1 , T1 to p 2 , V2 , T2 . Which of the following relation is
correct? (a) (b)
(a) T1V1γ − 1 (b) p1V1γ − 1 = p2V2γ − 1
(c) T1 p1γ = T2 p2γ (d) T1 V1γ = T2 V2γ
V V
p p
53. During an adiabatic process, the pressure of a gas is
found to be proportional to the cube of its absolute
Cp
temperature. The ratio for the gas is (c) (d)
CV
3 4 5
(a) (b) (c) 2 (d)
2 3 3 V V
The First Law of Thermodynamics 673

Heat Engines and Refrigerators 69. A Carnot’s engine whose low temperature reservoir is at
7°C has an efficiency of 50%. It is desired to increase the
61. An ideal heat engine is opening between 227°C and efficiency to 70%. By how many degrees should the
127°C. It absorbs 104 J amount of heat at the higher temperature of the high temperature reservoir be
temperature. The amount of heat converted into work is increased
(a) 2000 J (b) 4000 J (a) 933 K (b) 432 K
(c) 8000 J (d) 5600 J (c) 373 K (d) 267 K

62. A Carnot’s engine, with its cold body at 17°C has 50% 70. Six moles of an ideal gas performs a cycle shown in
efficiency. If the temperature of its hot body is now figure. If the temperatures are T A = 600 K, TB = 800 K,
increased by 145°C, then efficiency becomes TC = 2200 K and TD = 1200 K, then work done per cycle
(a) 55% (b) 60% is approximately
(c) 40% (d) 45% p
B C
63. The heat reservoir of an ideal Carnot engine is at 800 K
and its sink is at 400 K. The amount of heat taken in it in
is to produce useful mechanical work at the rate of A D
750 J is
(a) 2250 J (b) 1125 J
(c) 1500 J (d) 750 J T

64. If a Carnot engine functions at source temperature (a) 20 kJ (b) 30 kJ


(c) 40 kJ (d) 60 kJ
= 127° C and sink temperature = 87° C, what is its
efficiency? 71. A Carnot’s engine is made to work between 200°C and
(a) 10% (b) 25% (c) 40% (d) 50% 0°C first and then between 0° and −200° C. The ratio of
65. Efficiency of Carnot engine is (input temperature = T1 , η2
efficiencies   of the engine in the two cases is
exhaust temperature = T2 )  η1 
T1 − T2 T2 − T1
(a) (b) (a) 1 : 1.5 (b) 1 : 1
T1 T1 (c) 1 : 2 (d) 1.73 : 1
T1 T2
(c) (d)
T2 T1 Miscellaneous Problems
66. A system undergoes a cyclic process in which it absorbs 72. The process ∆U = 0, for an ideal gas can be best
Q1 heat and gives out Q2 heat. The efficiency of the represented in the form of a graph
process is η and work done is W. Select the correct p p
B A
statement.
Q − Q2
(a) W = 1
Q2 (a) A (b)
W
(b) η =
Q1 B
(c) Both (a) and (b) are correct T T
(d) Both (a) and (b) are wrong p p
B
67. For which combination of temperatures, the efficiency of A B
Carnot’s engine is highest?
(c) (d)
(a) 80 K, 60 K (b) 100 K, 80 K
(c) 60 K, 40 K (d) 40 K, 20 K A
68. Two Carnot engines A and B are operated in succession. T T
The first one, A receives heat from a source at T1 = 800 K
73. A given quantity of monoatomic gas is at pressure p and
and rejects to sinks at T2 K. The second engine B receives
absolute temperature T. The isothermal bulk modulus of
heat rejected by the first engine and rejects to another the gas is
sink at T3 = 300 K. If the work outputs of two engines are 2
equal, then the value of T2 is (a) p (b) p
3
(a) 100 K (b) 300 K 3
(c) 550 K (d) 700 K (c) p (d) 2p
2
674 Objective Physics Vol. 1

74. When an air bubble rises from the bottom to the surface 80. Two pistons can move freely inside a horizontal cylinder
of a lake, its radius becomes double. Find the depth of the having two sections of unequal cross-sections. The
lake. Given that the atmospheric pressure is equal to the pistons are joined by an inextensible, light string and
pressure due to a column of water 10 m high. Assume some gas is enclosed between the pistons. On heating the
constant temperature and disregard surface tension system, the piston will
(a) 30 m (b) 40 m
(c) 70 m (d) 80 m

75. Two identical containers joined by a small pipe initially


contain the same gas at pressure p 0 and absolute
temperature T0 . One container is now maintained at the (a) move to the left
same temperature while the other is heated to 2T0 . The (b) move to the right
common pressure of the gases will be (c) remain stationary
3 4 5 (d) (a) or (c) depending upon the initial pressure of the gas
(a) p0 (b) p0 (c) p0 (d) 2 p0
2 3 3
81. Two different masses of a gas m and 2m are heated
76. An ideal monoatomic gas is compressed (no heat being separately in vessels of equal volume. The T-p curve for
added or removed in the process) so that its volume is mass 2mmakes angle an α with T-axis and that for mass m
halved. The ratio of the new pressure to the original takes angle an β with T-axis, then
pressure is (a) tan α = tan β (b) tan α = 2 tan β
(a) (2)3/ 5 (b) (2)4/ 3 (c) tan β = 2 tan α (d) None of these
(c) (2)3/ 4 (d) (2)5/ 3 82. An ideal gas mixture filled inside a balloon expands
according to the relation pV 2/ 3 = constant. The
77. Two different ideal diatomic gases A and B are initially in
temperature inside the balloon is
the same state. A and B are then expanded to same final
(a) increasing
volume through adiabatic and isothermal process
(b) decreasing
respectively. If p A , p B and T A , TB represents the final (c) constant
pressure and temperatures at A and B respectively, then (d) Cannot be defined
(a) pA < pB and TA < TB (b) pA > pB and TA > TB
(c) pA > pB and TA < TB (d) pA < pB and TA > TB 83. A container of volume 1 m 3 is divided into two equal
78. An ideal gas is allowed to expand freely against vacuum compartments by a partition. One of these compartments
in a rigid insulated container. The gas undergoes contains an ideal gas at 300 K. The other compartment is
(a) an increase in its internal energy vacuum. The whole system is thermally isolated from its
(b) a decrease in its internal energy surroundings. The partition is removed in the gas
(c) neither an increase nor decrease in temperature or internal container. Its temperature now would be
energy
(d) an increase in temperature (a) 300 K (b) greater than 300 K
(c) less than 300 K (d) Data insufficient
79. Identify the graph (s) which correctly represents an
isotherm at two temperatures T1 and T2 (> T1 ) 84. One mole of helium is adiabatically expanded from its
initial state ( p i , Vi , Ti ) to its final state ( p f , V f , T f ). The
p p
decrease in the internal energy associated with this
expansion is equal to
(a) CV (Ti − T f )
(a) (b) (b) C p (Ti − T f )
T2 T1
1
T1 T2 (c) (C p + CV ) (Ti − T f )
2
O V O V (d) (C p − CV ) (Ti − T f )
p p
T2 T1 T1 T2
85. One mole of a perfect gas in a cylinder fitted with a piston
has a pressure p, volume V and temperature T. If the
(c) (d) temperature is increased by 1 K keeping pressure
constant, then increase in volume is
2V V V
(a) (b) (c) (d) V
O V O V 273 91 273
The First Law of Thermodynamics 675

86. Unit mass of a liquid with volume V1 is completely 89. The molar heat capacity in a process of a diatomic gas if it
changed into a gas of volume V2 at a constant external Q
does a work of , when a heat of Q is supplied to it is
pressure p and temperature T. If the latent heat of 4
evaporation is L, then the increase in the internal energy 2 5
(a) R (b) R
of the system is 5 2
(a) zero (b) p (V2 − V1 ) 10 6
(c) R (d) R
(c) L − p (V2 − V1 ) (d) L 3 7
87. A gas mixture consists of 2 moles of oxygen and 4 moles 90. A thermally insulated rigid container contain an ideal gas
argon at temperature T. Neglecting all vibrational modes, heated by a filament of resistance 100 Ω through a current
the total internal energy of the system is of 1 A for 5 min, then change in internal energy is
(a) 4RT (b) 15RT (c) 9RT (d) 11RT (a) 0 kJ (b) 10 kJ
(c) 20 kJ (d) 30 kJ
88. A cylindrical tube of uniform cross-sectional area A is
fitted with two air tight frictionless pistons. The pistons 91. An ideal gas expands in such a manner that its pressure
are connected to each other by a metallic wire. Initially and volume can be related by equation pV 2 = constant.
the pressure of the gas is p 0 and temperature is T0 , During this process, the gas is
atmospheric pressure is also p 0 . Now, the temperature of (a) heated
the gas is increased to 2T0 , the tension in the wire will be (b) cooled
(c) neither heated nor cooled
(d) first heated and then cooled
wire 92. p-V diagram of a diatomic gas is a straight line passing
through origin. The molar heat capacity of the gas in the
process will be
(a) 4R (b) 2.5R
pA 4R
(a) 2 p0 A (b) p0 A (c) 0 (d) 4 p0 A (c) 3R (d)
2 3

[ Level 2 ]
1. The equation of a state of a gas is given by V 
(b) R (T1 − T2 ), 0, RT1 ln  1 
p(V − b ) = nRT . 1 mole of a gas is isothermally  V2 
expanded from volume V to 2V, the work done during the  V 
process is (c) 0, RT2 ln  1 , R (T1 − T2 )
 V2 
 2V − b  V − b
(a) RT ln   (b) RT ln  V 
 V −b   V  (d) 0, RT2 ln  2 , R (T2 − T1 )
 V1 
 V −b   V 
(c) RT ln   (d) RT ln  
 2V − b  V − b 3. One mole of a monoatomic gas is carried along process
ABCDEA as shown in the diagram. Find the net work
2. A cyclic process for 1 mole of an ideal is shown in the done by gas.
V-T diagram. The work done in AB, BC and CA
respectively is B C
4

C 2
V1
p (N/m )
2

V 1
A D E
V2 B
3
V (m )
1 2 3 4
T1 T2
T 3
(a) J (b) 1 J
2
V  1
(a) 0, RT2 ln  1 , R (T2 − T1 ) (c) J (d) 0 J
 V2  2
676 Objective Physics Vol. 1

4. One mole of an ideal gas with heat capacity at constant 8. A closed system undergoes a change of state by process
pressure C p undergoes the process T = T0 + αV where, T0 1→ 2 for which Q12 = 10 J and W12 = − 5 J. The system is
and α are constants. If its volume increases from V1 to V2 now returned to its initial state by a different path 2 → 1
the amount of heat transferred to the gas is for which Q21 is −3 J. The work done by the gas in the
V  process 2 → 1 is
(a) C p RT0 ln  2  (a) 8 J (b) zero (c) −2 J (d) +5 J
 V1 
V  9. An ideal monoatomic gas undergoes the process AB as
(b) αC p (V2 − V1 ) − RT0 ln  2 
 V1  shown in the figure. If the heat supplied and the work
V  done in the process are ∆Q and ∆W respectively. The
(c) αC p (V2 − V1 ) + RT0 ln  2  ratio ∆Q : ∆W is
 V1 
V
V  B
(d) RT0 ln  2  − αC p (V2 − V1 )
 V1 

5. Ideal monoatomic gas is taken through a process


A
dQ = 2dU. The molar heat capacity for the process is
(where, dQ is heat supplied and dU is change in internal T
energy) (a) 2.5 (b) 1.67 (c) 1.25 (d) 0.40
(a) 2.4 R
10. One mole of a gas expands with temperature T such that
(b) 3R
(c) R
its volume, V = kT 2 , where k is a constant. If the
(d) 2R temperature of the gas changes by 60°C, then the work
done by the gas is
6. Pressure versus density graph of an ideal gas is shown in (a) 120R (b) R ln 60
figure (c) kR ln 60 (d) 60kR
p
11. An ideal monoatomic gas undergoes a process in which
C
the gas volume relates to temperature as VT = constant.
B Then molar specific heat of gas in this process is
R
(a) (b) R
2
D
3R
A (c) (d) None of these
ρ 2

12. In a cyclic process shown in the figure an ideal gas is


(a) during the process AB work done by the gas is positive
(b) during the process AB work done by the gas is negative
adiabatically taken from B to A, the work done on the gas
(c) during the process BC internal energy of the gas is increasing during the process B → A is 30 J, when the gas is taken
(d) None of the above from A → B, the heat absorbed by the gas is 20 J. The
change in internal energy of the gas in the process A→ B is
7. N moles of a monoatomic gas is carried round the p
reversible rectangular cycle ABCDA as shown in the A
20 J
diagram. The temperature at A is T0 . The thermodynamic
efficiency of the cycle is approximately
30 J B

2p0 B C V
(a) 20 J (b) −30 J (c) 50 J (b) −10 J
13. n moles of an ideal gas undergo a process in which the
p p0
A D temperature changes with volume as T = kV 2 . The work
done by the gas as the temperature changes from T0 to
4T0 is
0 V0 2V0  5
V (a) 3nRT0 (b)   nRT0
 2
(a) 15% (b) 50%  3
(c)   nRT0 (d) zero
(c) 20% (d) 25%  2
The First Law of Thermodynamics 677

14. One mole of a monoatomic ideal gas undergoes the 19. The figure shown two paths for the change of state of a
process A → B in the given p-V diagram. The molar heat gas from A to B. The ratio of molar heat capacities in
capacity for this process is
p
path 1 and path 2 is
B p
6p0
2
3p0
A A B
1
V
V0 5V0 V
3R 13R 5R
(a) (b) (c) (d) 2R
2 6 2 (a) < 1 (b) > 1
(c) 1 (d) data insufficient
15. The p-V diagram of 2 g of helium gas for a certain
process A → B is shown in the figure. What is the heat 20. An ideal monoatomic gas undergoes a process in which
given to the gas during the process A → B? its internal energy U and density ρ varies as Uρ =
p constant. The ratio of change in internal energy and the
2p0 B work done by the gas is
3 −2 1 3
(a) (b) (c) (d)
2 3 3 5
p0
A 21. A perfect gas goes from state A to another state B by
V absorbing 8 × 105 J of heat and doing 6.5 × 105 J of work
V0 2V0
(by the gas). It is now transferred between the same two
(a) 4 p0V0 (b) 6 p0V0 (c) 3 p0V0 (d) 2 p0V0 states in another process in which it absorbs 105 J of heat.
16. Heat energy absorbed by a system in going through a Then, in the second process
cyclic process as shown in figure is (a) work done on gas is 0.5 × 105 J
(b) work done by gas in 0.5 × 105 J
(c) work done on gas is 105 J
V in Litre

30
(d) work done by gas is 105 J
22. The process ∆U = 0, for an ideal gas can be best
10 represented in the form of a graph
p p
B A
10 30
p in kPa
(a) 107 π J (b) 104 π J (c) 102 π J (d) 103 π J (a) A (b)

17. A gas undergoes A to B through three different processes B


1, 2, and 3 as shown in the figure. The heat supplied to the T V
gas is Q1 , Q2 and Q3 respectively, then p p
p B
B A B
1 3
2 (c) (d)

A
V T
A
V 23. Consider two containers A and B containing identical
(a) Q1 = Q2 = Q3 (b) Q1 < Q2 < Q3 gases at the same pressure, volume and temperature. The
(c) Q1 > Q2 > Q3 (d) Q1 = Q3 > Q2 gas in container A is compressed to half of its original
volume isothermally while the gas in container B is
18. The internal energy of a gas is given by U = 2 pV . It compressed to half of its original value adiabatically. The
expands from V0 to 2V0 against a constant pressure p 0 . ratio of final pressure of gas in B to that of gas in A is
The heat absorbed by the gas in the process is γ−1 2 2
 1  1   1 
(a) 2 p0V0 (b) 4 p0V0 (a) 2γ − 1 (b)   (c)   (d)  
(c) 3 p0V0 (d) p0V0  2  1 − γ  γ − 1
678 Objective Physics Vol. 1

More than One Correct Options 6. Three copper blocks of masses M 1 , M 2 and M 3 kg
respectively are brought into thermal contact till they
1. An ideal gas is taken from the state A (pressure p, reach equilibrium. Before contact, they were at
p
volumeV ) to the state B (pressure , volume 2V) along a T1 , T2 , T3 ( T1 > T2 > T3 ). Assuming there is no heat loss
2 to the surroundings, the equilibrium temperature T is (s is
straight line path in the p-V diagram. Select the correct specific heat of copper)
statement(s) from the following. T1 + T2 + T3
(a) T =
(a) The work done by the gas in the process A to B is negative 3
M 1T1 + M 2T2 + M 3T3
(b) In the T-V diagram, the path AB becomes a part of a parabola (b) T =
(c) In the p-T diagram, the path AB becomes a part of a M1 + M2 + M3
hyperbola M 1T1 + M 2T2 + M 3T3
(c) T =
(d) In going from A to B, the temperature T of the gas first 3 (M 1 + M 2 + M 3 )
increases to a maximum value and then decreases M 1T1s + M 2T2s + M 3T3s
(d) T =
2. In the process pV 2 = constant, if temperature of gas is M1 + M2 + M3
increased 7. Consider a cycle followed by an engine (figure.)
(a) change in internal energy of gas is positive
p
(b) work done by gas is positive
(c) heat is given to the gas
(d) heat is taken out from the gas 1 2

3. T-V diagram of two moles of a monoatomic gas is as


shown in figure:
T 3

b c
2T0 V

a 1 to 2 is isothermal
T0 d
2 to 3 is adiabatic
V 3 to 1 is adiabatic
V0 2V0
Such a process does not exist, because
For the process abcda, choose the correct options given (a) heat is completely converted to mechanical energy in such a
below process, which is not possible
(b) mechanical energy is completely converted to heat in this
(a) ∆U = 0 process, which is not possible
(b) work done by gas > 0 (c) curves representing two adiabatic processes don’t intersect
(c) heat given to the gas is 4 RT0 (d) curves representing an adiabatic process and an isothermal
(d) heat given to the gas is 2RT0 process don’t intersect
4. Density (ρ ) versus internal energy (U ) graph of a gas is as 8. Consider a heat engine as shown in figure. Q1 and Q2 are
shown in figure. Choose the correct options. heat added both to T1 and heat taken from T2 in one cycle
ρ of engine. W is the mechanical work done on the engine.

a b T1

Q1
c
W
U
Q2
(a) Qbc = 0 (b) W bc = 0 (c) W ca < 0 (d) Qab > 0
T2
Here, W work done by gas and Q is heat given to the gas.
5. Temperature of a monoatomic gas is increased from T0 to
2T0 in three different processes : isochoric, isobaric and If W > 0, then possibilities are
adiabatic. Heat given to the gas in these three processes (a) Q1 > Q2 > 0
are Q1 , Q2 and Q3 respectively. Then, the choose the (b) Q2 > Q1 > 0
correct option. (c) Q2 < Q1 < 0
(a) Q1 > Q3 (b) Q2 > Q1 (c) Q2 > Q3 (d) Q3 = 0 (d) Q1< 0, Q2 > 0
The First Law of Thermodynamics 679

Comprehension Based Questions 1. Assertion A system of ideal gas is heated by constant


Passage I (Q.1 to 2)
power. If temperature versus time graph is a straight line,
One mole of a monoatomic ideal gas is taken along the the process must be isochoric.
cycle ABCA as shown in the diagram. Reason For isochoric process, dQ = nCV dT
dQ dT dT p
2p B ⇒ = p = nCV ⇒ = = constant
dT dt dt nCV
A C ∆Q 5
p 2. Assertion In isobaric process, for helium gas is .
∆W 2
Reason In isobaric process, work done by the gas is
V 2V nR∆T .
1. The net heat absorbed by the gas in the given cycle is 3. Assertion In adiabatic expansion product of pV always
pV
(a) pV (b) (c) 2pV (d) 4 pV decreases.
2
Reason In adiabatic expansion work done by the gas is
2. The ratio of specific heat in the process CA to the specific positive.
heat in the process BC is
5 4. Assertion In isochoric process p-V graph is straight line
(a) 2 (b) (c) 4 (d) None of these
3 parallel to p-axis.
Passage II (Q.3 to 5) Reason In isochoric process density p remains constant.
One mole of a monoatomic ideal gas is taken through the 5. Assertion For given source and sink temperatures
cycle ABCDA as shown in the figure. T A = 1000 K and
efficiency of Carnot’s engine is 100%.
2 p A = 3 p B = 6 pC .
p Reason For given source and sink temperatures
A
efficiency of Carnot’s engine is maximum.
Adiabatic
B 6. Assertion Efficiency of any heat engine cannot be
greater than the heat engine of Carnot’s engine.
D
Reason Heat flow never takes place from a body at
Adiabatic C
lower temperature to a body at higher temperature.
V 7. Assertion If volume of a gas is increasing but
  2
0. 4
25 −1 −1
 temperature of the gas is decreasing, then heat given to
 Assume   = 0.85 and R = JK mol 
 3 3  the gas may be positive, negative or zero.
3. The temperature at B is Reason Heat given to a gas is a state function. It is not
(a) 350 K (b) 1175 K (c) 850 K (d) 577 K path function.

4. Work done by the gas in the process A → B is 8. Assertion First law of thermodynamics can be applied
(a) 5312.5 J (b) 1875 J (c) zero (d) 8854 J only for an ideal or real gas system.
5. Heat lost by the gas in the process B → C is Reason First law of thermodynamics is nothing but law
(a) 5312.5 J (b) 1875 J (c) zero (d) 8854 J
of conservation of energy.
9. Assertion There are two processes : Process 1 is
Assertion and Reason pV = constant and process 2 is pV 2 = constant. In both
Direction (Q. Nos. 1-15) These questions consists of two the processes volume of gas is increased from V1 to V2 .
statements each printed as assertion and reason. While Initial coordinates ( p1 , V1 ) of the gas were same. Then,
answering these questions you are required to choose any one more work is done by the gas in process 1.
of the following five responses.
(a) If both Assertion and Reason are correct and Reason is Reason In second process pressure drops more rapidly
the correct explanation of Assertion. with increase in volume.
(b) If both Assertion and Reason are true but Reason is 10. Assertion Any process taking place in atmosphere is
not the correct explanation of Assertion. considered as an isobaric process.
(c) If Assertion is true but Reason is false.
(d) If Assertion is false but Reason is true. Reason Work done by the system in that case is p 0 ∆V .
(e) If both Assertion and Reason are false. Where, p 0 is atmospheric pressure.
680 Objective Physics Vol. 1

11. Assertion During melting of ice work done by 3. For one mole of a monoatomic gas match the following
surrounding on (ice + water) system is positive. Column I Column II
Reason Volume of the given system decreases on (A) Isothermal bulk modulus (p) − RT
melting on ice. V2
(B) Adiabatic bulk modulus (q) − 5P
12. Assertion Molar heat capacity cannot be defined for
3V
isothermal process.
(C) Slope of p-V graph in isothermal process (r) T
Reason In isothermal process ( pV ) versus T graph is a V
dot. (D) Slope of p-V graph in adiabatic process (s) 4T
3V
13. Assertion If initial and final volumes are equal, then
work done by gas is zero. (t) None

Reason In isochoric process initial and final volumes 4. Match the following columns.
are equal and work done by gas is zero.
Column I Column II
14. Assertion In adiabatic expansion the product of p and V (A) Adiabatic expansion (p) No work done
always decreases. (B) Isobaric expansion (q) Constant internal energy

Reason In adiabatic expansion process work is done by (C) Isothermal expansion (r) Increase in internal energy

the gas at the cost of internal energy of gas. (D) Isochoric process (s) Decrease in internal energy

15. Assertion In isobaric process Q / ∆U is equal to Entrance Gallery


γ ( = C p / CV ) of the gas.
2014
Reason For monoatomic gas ratio Q / ∆U in isobaric
1. A thermodynamic system is taken from an initial state i
5
process is equal to . with internal energy U i = 100 J to the final state f along
3 two different paths iaf and ibf as schematically shown in
the figure. The work done by the system along the paths
Match the Columns af , ib and bf are Waf = 200 J, Wib = 50 J and Wbf = 100 J,
1. In the ρ-T graph shown in figure, match the following. respectively. The heat supplied to the system along the
ρ path iaf , ib and bf are Qiaf , Qib and Qbf respectively. If
the internal energy of the system in the state b is
b c
U b = 200 J and Qiaf = 500 J, then ratio Qbf / Qib is
[JEE Advanced]
a
f

a d
i b
T 200 J
100 J
Column I Column II
(A) Process a-b (p) Isochoric process (a) 2 : 1 (b) 1 : 2
(c) 3 : 1 (d) 1 : 3
(B) Process b-c (q) ∆U = 0
(C) Process c-d (r) p increasing 2. One mole of diatomic ideal gas undergoes a cyclic
(D) Process d-a (s) p decreasing process ABC as shown in figure. The process BC is
adiabatic. The temperature at A , B and C are 400 K,
2. Match the following columns. 800 K and 600 K, respectively. Choose the correct
statement. [JEE Main]
Column I Column II
B
(A) Cyclic process (p) ∆U < 0 800 K

(B) Isobaric process (q) ∆Q = ∆W p


(C) Isochoric process (r) ∆W = ∆nR∆T 600 K
A C
(D) Adiabatic process (s) ∆Q = ∆U 400 K
V
The First Law of Thermodynamics 681

(a) The change in internal energy in whole cyclic process is 7. The shown p-V diagram represents the thermodynamic
250R cycle of an engine, operating with an ideal monoatomic
(b) The change in internal energy in the process CA is 700R gas. The amount of heat, extracted from the source in a
(c) The change in internal energy in the process AB is − 350R single cycle is [JEE Main]
(d) The change in internal energy in the process BC is − 500R
B C
3. A Carnot engine operating between temperatures T1 and 2p0

Pressure
T2 has efficiency 0.2. When T2 is reduced by 50 K, its
efficiency increases to 0.4. Then, T1 and T2 are p0 A
D
respectively [Kerala CEE]
(a) 200 K, 150 K
(b) 250 K, 200 K Temperature 2V0
(c) 300 K, 250 K 13 11
(a) p0V0 (b) p0V0 (c) p0V0 (d) 4 p0V0
(d) 300 K, 200 K 2 2
(e) 300 K, 150 K
8. An ideal gas is taken around PQRP as shown in the figure
4. What is the source temperature of the Carnot engine of p-V diagram. The work done during a cycle is
required to get 70% efficiency? [Karnataka CET]
p
Given, sink temperature = 27° C . [Karnataka CET]
(a) 1000° C (b) 90° C (c) 270° C (d) 727° C Q (3p,3V)

5. A cycle tyre bursts suddenly. What is the type of this


process? [Karnataka CET]
P R ( p,3V )
(a) Isothermal (b) Adiabatic ( p, V )
(c) Isochoric (d) Isobaric V
T S
1
2013 (a) zero (b) pV
2
(c) pV (d) 2pV

6. One mole of monoatomic ideal gas is taken along two


9. For which of the following combination of working
cyclic processes E → F → G → E and E → F → H → E
temperatures, the efficiency of Carnot engine is highest?
as shown in the p-V diagram. [Karnataka CET]
p
32 p0 F (a) 100 K, 80 K (b) 80 K, 60 K
(c) 60 K, 40 K (d) 40 K, 20 K

10. A graph of pressure versus volume for an ideal gas for


p0 G different processes is as shown. In the graph, curve OC
E H
V
represents [O JEE]
V0 p
O A
The processes involved are purely isochoric, isobaric,
isothermal or adiabatic. B
C
Match the paths in List I with the magnitudes of the work D
done in List II and select the correct answer using the V
codes given below the lists. [JEE Advanced] (a) isochoric process (b) isothermal process
(c) isobaric process (d) adiabatic process
Column I Column II
11. A gas at state A changes to state B through path I and II
P. G → E 1. 160 p0V0 ln2
shown in figure. The change in internal energy is ∆U1
Q. G → H 2. 36 p0V0 and ∆U 2 , respectively. Then, [O JEE]
R. F → H 3. 24 p0V0 p
S. F → G 4. 31 p0V0 I
A
Codes
B
P Q R S
II
(a) 4 3 2 1
(b) 4 3 1 2 V
(c) 3 1 2 4
(a) ∆U 1 > ∆U 2 (b) ∆U 1 < ∆U 2
(d) 1 3 2 4
(c) ∆U 1 = ∆U 2 (d) ∆U 1 = ∆U 2 = 0
682 Objective Physics Vol. 1

2012 18. 100 g of water is heated from 30 ºC to 50ºC. Ignoring the


slight expansion of the water, the change in its internal
12. Two moles of an ideal helium gas are in a rubber balloon energy is (Specific heat of water is 4184 J/kg/K)
at 30° C. The balloon is fully expandable and can be [Kerala CEE]
assumed to require no energy in its expansion. The (a) 8.4 kJ (b) 84 kJ (c) 2.1 kJ (d) 4.2 kJ
temperature of the gas in the balloon is slowly changed to (e) 3.2 kJ
35° C. The amount of heat required in raising the 19. The thermodynamic process in which no work is done on
temperature is nearly (take, R = 8.31 J / mol-K) [IIT JEE] or by the gas is [Kerala CEE]
(a) 62 J (b) 104 J (c) 124 J (d) 208 J
(a) isothermal process (b) adiabatic process
13. Helium gas goes through a cycle ABCDA (consisting of (c) cyclic process (d) isobaric process
two isochoric and isobaric lines) as shown in figure. (e) isochoric process
Efficiency of this cycle is nearly (Assume, the gas to be 20. Two soap bubbles of radii x and y coalesce to constitute a
close to ideal gas). [AIEEE] bubble of radius z. Then, z is equal to [WB JEE]
2p0 B
C x+ y
(a) x 2 + y2 (b) x + y (c) x + y (d)
2
p0
A
D 2010
21. One mole of an ideal gas in V
V0 2V0 initial state A undergoes a cyclic 4V0 B
process ABCA as shown in the
(a) 15.4% (b) 9.1% (c) 10.5% (d) 12.5%
figure. Its pressure at A is p 0 .
V0 A
2011 Choose the correct option (s) C
from the following. [IIT JEE] T0
14. 5.6 L of helium gas at STP is adiabatically compressed to
0.7 L. Taking the initial temperature to be T1 , the work (a) Internal energies at A and B are the same
done in the process is [IIT JEE] (b) Work done by the gas in process AB is p 0V0 ln 4
9 3 15 9
(a) – RT1 (b) RT1 (c) RT1 (d) RT1 p
8 2 8 2 (c) Pressure at C is 0
4
15. A Carnot engine operating between temperatures T1 and (d) Temperature at C is T0 / 4
1
T2 has efficiency . When T2 is lowered by 62 K, its 1
6 22. A diatomic ideal gas is compressed adiabatically to of
1 32
efficiency increases to . Then, T1 and T2 are,
3 its initial volume. If the initial temperature of the gas is Ti
respectively [AIEEE] (in kelvin) and the final temperature is T f = aTi , then
(a) 372 K and 330 K (b) 330 K and 268 K value of a is [IIT JEE]
(c) 310 K and 248 K (d) 372 K and 310 K (a) 4 (b) 6 (c) 5 (d) 9

16. The specific heat capacity of a metal at low temperature 23. A diatomic ideal gas is used in a car engine as the
3 working substance. If during the adiabatic expansion part
 T 
(T ) is given as C p ( kJK −1 kg −1 ) = 32   . A 100 g of the cycle, volume of the gas increases from V to 32 V.
 400
The efficiency of the engine is [AIEEE]
vessel of this metal is to be cooled from 20 K to 4 K by a
(a) 0.5 (b) 0.75 (c) 0.99 (d) 0.25
special refrigerator operating at room temperature
(27°C). The amount of work required to cool the vessel is 24. The efficiency of a Carnot engine working between
[AIEEE] 800 K and 500 K is [AIEEE]
(a) equal to 0.002 kJ
(a) 0.4 (b) 0.625 (c) 0.375 (d) 0.5
(b) greater than 0.148 kJ
(c) between 0.148 kJ and 0.028 kJ 25. The efficiency of Carnot heat engine is 0.5, when the
(d) less than 0.028 kJ temperature of the source is T1 and that of sink is T2 . The
17. A Carnot engine whose efficiency is 40%, receives heat efficiency of another Carnot heat engine is also 0.5. The
at 500 K. If the efficiency is to be 50%, the source temperatures of source and sink of the second engine are
temperature for the same exhaust temperature is respectively [Karnataka CET]
[Kerala CEE] T2
(a) 2T1 , 2T2 (b) 2T1 ,
(a) 900 K (b) 600 K (c) 700 K 2
(d) 800 K (e) 850 K (c) T1 + 5, T2 − 5 (d) T1 + 10 , T2 − 10
Answers
Level 1
Objective Problems
1. (a) 2. (c) 3. (c) 4. (b) 5. (c) 6. (c) 7. (a) 8. (a) 9. (d) 10. (b)
11. (b) 12. (a) 13. (a) 14. (a) 15. (a) 16. (a) 17. (b) 18. (a) 19. (d) 20. (a)
21. (b) 22. (c) 23. (d) 24. (d) 25. (a) 26. (d) 27. (a) 28. (b) 29. (b) 30. (c)
31. (d) 32. (d) 33. (b) 34. (b) 35. (b) 36. (c) 37. (d) 38. (c) 39. (b) 40. (a)
41. (a) 42. (a) 43. (d) 44. (c) 45. (c) 46. (a) 47. (d) 48. (a) 49. (b) 50. (d)
51. (d) 52. (a) 53. (a) 54. (d) 55. (a) 56. (d) 57. (c) 58. (c) 59. (a) 60. (b)
61. (a) 62. (b) 63. (c) 64. (a) 65. (a) 66. (a) 67. (d) 68. (c) 69. (c) 70. (c)
71. (d) 72. (b) 73. (b) 74. (c) 75. (b) 76. (d) 77. (a) 78. (c) 79. (a) 80. (a)
81. (b) 82. (a) 83. (a) 84. (a) 85. (c) 86. (c) 87. (d) 88. (b) 89. (c) 90. (d)
91. (b) 92. (c)

Level 2
Only One Correct Option
1. (a) 2. (c) 3. (c) 4. (c) 5. (b) 6. (d) 7. (a) 8. (a) 9. (a) 10. (a)
11. (b) 12. (b) 13. (c) 14. (b) 15. (b) 16. (c) 17. (c) 18. (c) 19. (a) 20. (a)
21. (a) 22. (a) 23. (a)

More than One Correct Options


1. (b,d) 2. (a,c) 3. (a,b) 4. (c,d) 5. (all) 6. (b) 7. (a,c) 8. (a, c)

Comprehension Based Questions


1. (b) 2. (b) 3. (c) 4. (b) 5. (a)

Assertion and Reason


1. (d) 2. (b) 3. (b) 4. (d) 5. (d) 6. (c) 7. (c) 8. (d) 9. (b) 10. (b)
11. (a) 12. (b) 13. (d) 14. (b) 15. (b)

Match the Columns


1. (A → q, r; B → p, r; C → q, s; D → p, s) 2. (A → q, B → r, C → s, D → p) 3. (A → t, B → t, C → p, D → q)
4. (A → s, B → r, C → q, D → p)

Entrance Gallery
1. (a) 2. (d) 3. (b) 4. (d) 5. (b) 6. (a) 7. (b) 8. (d) 9. (d) 10. (d)
11. (c) 12. (d) 13. (a) 14. (a) 15. (d) 16. (c) 17. (b) 18. (a) 19. (e) 20. (a)
21. (a,b) 22. (a) 23. (b) 24. (c) 25. (a)
Solutions
Level 1 : Objective Problems 19. Heat given ∆Q = 20cal = 20 × 4.2 = 84 J
2. Internal energy of gas is state function, i.e. Work done ∆W = − 50J (as process is anti-clockwise)
∆U1 = ∆U2 By first law of thermodynamics,
or Q1 − W1 = Q2 − W2 ⇒ ∆U = ∆Q − ∆W = 84 − ( −50) = 134 J
3. ∆U in both cases will be same, as potential energy is state 20. For cyclic process
function. Area under p-V graph in case 1 is more.
Total work done = WAB + WBC + WCA
Hence, W1 > W2 . Therefore, Q1 > Q2 .
∆WAB = p ∆V = 10( 2 − 1) = 10 J
4. In cyclic process, work done is area between the cycle. It is and ∆WBC = 0 (as V = constant)
positive when cycle is clockwise and negative when cycle is
From first law of thermodynamics, ∆Q = ∆U + ∆W
anti-clockwise. (With p on y-axis and V on x-axis).
∆U = 0 (process ABCA is cyclic)
5. In cyclic process, work done may be greater than or less
than zero. ⇒ ∆Q = ∆WAB + ∆WBC + ∆WCA
6. Work done by gas, W = p(V f − Vi ) = 20J ⇒ 5 = 10 + 0 + ∆WCA
∆U = Q − W = 70 − 20 = 50J ⇒ ∆WCA = − 5J
7. Cycle is clockwise. Hence, heat will be given to the gas, as 21. Cyclic process 1 is clockwise whereas process 2 is
work done will be positive. anti-clockwise. Since, negative area (2) > positive area (1),
Q = W = area of cycle =1J hence net work done is negative.
8. In a cyclic process, ∆U = 0 22. Work is done by the gas (as cyclic process is clockwise)
∴ ∆W = Area ABCD
∴ Qtotal = Wtotal
So, from the first law of thermodynamics ∆Q (net heat
or 10 − 5 = − 3 + W21 absorbed) = ∆W = Area ABCD
or W21 = 8 J As change in internal energy in cycle ∆U = 0.
9. Change in internal energy does not depend upon path so 23. For path ab ( ∆U )ab = 7000J
∆U = ∆Q − ∆W remain constant.
10. ∆Q = ∆U + ∆W , ∆Q = 200J and ∆W = −100J By using ∆U = nCV ∆T
5
⇒ 7000 = n × R × 700 ⇒ n = 0.48
∆U = ∆Q − ∆W = 200 − ( −100) = 300 J 2
11. ∆Q = ∆U + ∆W For path ca
QHeat is supplied to the system so ∆Q → positive and work ( ∆Q )ca = ( ∆U )ca + ( ∆W )ca ...(i)
is done on the system so ∆W → negative Q ( ∆U )ab + ( ∆U )bc + ( ∆U )ca = 0
Hence, + ∆Q = ∆U − ∆W ∴ 7000 + 0 + ( ∆U )ca = 0
12. dU = dQ − dW ⇒ dU = dQ (< 0) (QdW = 0) ⇒ ( ∆U )ca = − 7000J ...(ii)
dU < 0, so temperature will decrease. Also, ( ∆W )ca = p1 (V1 − V 2 ) = nR(T1 − T2 )
13. Work done = p(V2 − V1 ) = 0.48 × 8.31 × (300 − 1000) = − 2792.16 J ...(iii)
R By solving Eqs. (i), (ii) and (iii), we get
14. ∆U = CV ∆T = (T2 − T1 )
γ −1 ( ∆Q )ca = − 7000 − 2792.16
15. Work done = p ∆V = p(V2 − V1 ) = −9792.16 ≈ −9800 J
16. From first law of thermodynamics 24. ∆U, remains same of both path
∆Q = ∆U + ∆W = ∆U + p ∆V For path iaf ∆U = ∆Q − ∆W
= 50 − 20 = 30 J
⇒ 100 = ∆U + 50 × ( 4 − 10)
For path fi ∆U = − 30J and ∆W = −13J
⇒ ∆U = 400J
⇒ ∆Q = − 30 − 13 = − 43J
17. Work done by the system = Area of shaded portion on
25. Heat given, ∆Q = 40J and work done ∆W = 30J
p-V diagram = ( 300 − 100)10−6 × ( 200 − 100) × 103 = 20 J
⇒ ∆U = ∆Q − ∆W = 40 − 30 = 10J
cycle is anti-clockwise. Hence, work done is negative.
18. Work done = Area enclosed by ∆ 26. During process A to B, pressure and volume both are
1 decreasing. Therefore, temperature and internal energy of
ABC = AC × BC the gas will decrease (T ∝ pV ) or ∆UA → B = negative. Further
2
1 ∆WA → B is also negative as the volume of the gas is
= × ( 3V − V ) × ( 3p − p) = 2pV decreasing. Thus, ∆QA → B is negative.
2
The First Law of Thermodynamics 685

In process B to C, pressure of the gas is constant while volume 42. W = area under p-V graph
is increasing. Hence, temperature should increase or 2
∆UC → A = positive. During process CAB volume of the gas is p
decreasing. Hence, work done by the gas is negative. 1
27. ∆WAB = 0 as V = constant
3
∴ ∆QAB = ∆UAB = 50J (given)
UA =1500J
∴ UB = (1500 + 50) J = 1550 J
∆WBC = − ∆UBC = − 40J (given) V1 V2 V
∴ ∆UBC = 40J
A2 > A1 > A3
∴ UC = (1550 + 40) J = 1590 J
∴ W2 > W1 > W3
28. W = area under p-V diagram.
43. In adiabatic process as γ increase slope of p-V graph
29. Q1 = nC p ∆T increases. Slope of B is more. Therefore, for γ B > γ A . B
Q2 = nCV ∆T should be monoatomic and A diatomic.
∴ Q1 − Q2 = nR ∆T (∴∆T = T2 − T1 ) 44. In isobaric process heat is used raising the temperature of

Q2 = Q1 − nR ∆T = ( 70) − ( 2) 
8.31  the system.
or  ( 35 − 30)
 4.18  45. H = n1CV ∆T1 and H ′ = n2C p ∆T2
= 50 cal
H ′ n2 C p ∆T2
30. In adiabatic process, TV γ − 1 = constant or = ⋅
H n1 CV ∆T1
γ −1
V  n2 γ ∆T2
or T2V 2γ − 1 = T1V1γ − 1 or T2 =  1  T1 =
 V2  n1 ∆T1
= (4)1.4 − 1 (300) = ( 300) (4)0.4 K 5 5
= × × 2=
5
3 10 3
Given the answer of 31 & 32
5
33. Given, p ∝ T 3 ∴ H′ = H
3
In adiabatic process, p1 − γ T γ = constant
γ
46. Q = mS ∆θ
γ −1 Q
or p ∝T ∴ S=
m ⋅ ∆θ
Compare with the given equation, we get
γ In adiabatic process, Q =0
=3
γ −1 ∴ S=0
3 47. During isothermal change T = constant ⇒ ∆U = 0 also from
Solving this equation we get, γ =
2 first law of thermodynamics, ∆Q = ∆W .
34. In adiabatic process, ∆U = − ∆W1 48. Work done = Area under p-V diagram.
p
In the first process, ∆W1 = + 25 J
∴ ∆U1 = − 25 J B
In second process, ∆W2 = − 100 J
∴ ∆U2 = + 100 J A
35. Fast processes are adiabatic. Hence,
p1V1γ − 1 = p2V 2γ
γ
V  5/ 3
p2 = p1  1  = p 
640  V
∴ 
 V2   8  (area )iso > (area )adi
= p(8)5/ 3 = 32p ⇒ Wiso > Wadi

38. In adiabatic process, pV γ = constant 49. In adiabatic change, ∆Q = 0


1 So, ∆W = − ∆U (Q∆Q = ∆U + ∆W )
or p∝
Vγ 50. The process is very fast, so the gas fails to gain or loses heat.
39. Work done by the gas should be negative, but work done on Hence, this process is adiabatic.
γ −1
the gas should be positive. 3/ 2 − 1
T p  γ
W = p (V i − V f ) (on the gas) 51. Using relation 2 =  2  = (8) 3/ 2 =2
= 105 (3.0 × 10−2 − 1.5 × 10−2 ) = + 1500J T1  p1 

41. In adiabatic process, TV γ − 1 = constant ⇒ T2 = 2T1


γ −1 5/ 3 − 1
T1  V 2   AL  L 
2/ 3
⇒ T2 = 2( 273 + 27)
∴ =  = 2 = 2
T2  V1   AL1   L1  = 600 K = 327° C
686 Objective Physics Vol. 1

53. Given p ∝ T 3 , but we know for an adiabatic process, the  T2 


64. Efficiency of Carnot engine, η = 1 −  × 100
pressure p ∝ T γ/ γ − 1 .  T1 
γ 3
So, =3 ⇒ γ=  (87 + 273) 
γ −1 2 = 1 −  × 100 = 10%
 (127 + 273) 
Cp 3
⇒ = W Q − Q2
CV 2 66. η = = 1
Q1 Q1
γ
p2  V1   4 
3/ 2
8 T T
54. p1V1γ = p2V2γ ⇒ =  = = 67. η = 1 − 2 , for η to be maximum ratio 2 should be
p1  V 2   1  1 T1 T1
57. At constant volume, p ∝ T minimum.
p1 T1 T −T W
⇒ = 68. ηA = 1 2 = A
p2 T2 T1 Q1
p1 300 3 T − T3 WB
⇒ = = ⇒ ηB = 2 =
p2 400 4 T2 Q2
58. ∆Q = ∆U + ∆W Q1 T1 T2 − T3 T1
∴ = × =
Q2 T2 T1 − T2 T2
 ( ∆Q ) 
⇒ ∆W = ( ∆Q )p − ∆U = ( ∆Q )p 1 − V
 ∴ WA = WB
 ( ∆Q )p 
T + T3
 in isobaric process ∴ T2 = 1
  2
 ∆U = ( ∆Q )  800 + 300
= = 550 K
 C 
= ( ∆Q )p 1 − V  = Q = 1 −  = Q
3 2 2

 5 5 T1 − T2
 C p  69. Initially, η =
T1
5
Q ( ∆Q )p = Q and γ = for monoatomic gas. T1 − ( 273 + 7)
3 ⇒ 0.5 =
T1
59. TV γ − 1 = constant
1 T1 − 280
γ −1 ⇒ = ⇒ T1 = 560 K
T1  V 2 
∴ =  2 T1
T2  V1  T ′ − T2
γ −1
Finally, η′1 = 1 ⇒ 0.7
1 1 T ′1
or   =
 2 2 T ′ − ( 273 + 7)
= 1 ⇒ T ′1 = 933 K
1 3 T ′1
∴ γ −1 = or γ =
2 2 ∴ Increase in temperature = 933 − 560 = 373 K.
∴ pV 3/ 2 = constant 70. Processes A to B and C to D are parts of straight line graphs
1 passing through origin.
60. In isothermal process p ∝ .
V p ∝T . So, volume remains constant for the graphs AB and CD.
Hence, graph between p and V is a hyperbola. So, no work is done during processes for A to B and C to D
W  T2   T  i.e. WAB = WCD = 0
61. = 1 −  or W = Q1 1 − 2 
Q1  T1   T1  and WBC = p2 (VC − V B ) = nR(TC − TB )

= 10000 1 −
400  = 6R( 2200 − 800) = 6R × 1400 J
 = 2000J
 500  Also WDA = p1 (V A − V D ) = nR(TA − TB )
62. 1 −
T2
= 0.5 or T1 = 2T2 = 6R(600 − 1200) = − 6R × 600 J
T1 Hence, work done in complete cycle
= 2(17 + 273) = 580 K W = WAB + WBC + WCD + WDA
Temperature of hot body is increased by 145°C or 145 K. = 0 + 6R × 1400 + 0 − 6R × 600
∴ T1′ = ( 580 + 145) = 725 K = 6R × 900 = 6 × 8.3 × 800 = 40 kJ
and T2 = (17 + 273) = 290 K 73. Isothermal bulk modulus, B = p
η = 1 −
290  1
∴  × 100 = 60% 74. p1V1 = p2V2 or p ∝
 725  V
W T 400
63. =1 − 2 =1 − = 0.5 Radius has become two times. Hence, volume will become
Q T1 800 1
eight times, i.e. pressure at surface is times the pressure at
W 8
or Q=
0.5 depth h. On surface pressure is equal to 10 m of water.
750 Hence, pressure at depth h is equal to 80 m of water. Of this
= = 1500 J
0.5 10 m is atmospheric pressure. Therefore, h is 70 m.
The First Law of Thermodynamics 687

75. (n1 + n2 )i = (n1 + n2 ) f 84. ∆U = CV ∆T = CV (T f − Ti ) = − CV (Ti − T f )


p0V p0V pV pV ⇒ | ∆U | = CV (Ti − T f )
or + = +
RT0 RT0 RT0 2RT0 V 2 T2 274
85. For isobaric process = ⇒ V2 = V ×
2p0V 3pV V1 T1 273
or =
RT0 2RT0 274V V
Increase = −V =
4 273 273
∴ p= p0
3 86. ∆Q = ∆U + p ∆V
76. Process is adiabatic, as ∆Q = 0 ⇒ mL = ∆U + p(V 2 − V1 )
p1V1γ = p2V 2γ ⇒ ∆U = L − p(V 2 − V1 ) (Qm =1)
γ 87. Oxygen is diatomic gas, hence its energy of two moles
p2  V1 
∴ =   = ( 2)5/ 3 5
p1  V 2  = 2×RT = 5RT
2
77. In adiabatic process, pressure will be less. Further Argon is a monoatomic gas, hence its internal energy of
temperature is the product of p and V. Hence, pA < pB and 3
4 moles = 4 × RT = 6RT
TA < TB . 2
p Total internal energy = (6 + 5) RT = 11RT
88. Volume of the gas V = constant
B
∴ p ∝T ,
i.e. pressure will be doubled, if temperature is doubled
A
F

V
pA pA0

78. Vessel is insulated. Therefore, Q = 0


Against vacuum, W = 0
∴ ∆U or ∆T = 0 ∴ p = 2p0
79. In isothermal process p-V diagram is a rectangular Now, let F be the tension in the wire. Then equilibrium of
hyperbola. T2 > T1 . Hence, ( pV )2 > ( pV )1 . any one piston gives.
80. On heating volume will increase. Hence, option will move F = ( p − p0 ) A = ( 2p0 − p0 ) A = p0 A
to the left.
89. dU = CV dT =  R  dT or dT =
5 2( dU )
81. At constant volume 2  5R
p From first law of thermodynamics
Q 3Q
dU = dQ − dW = Q − =
4 4
Now, molar heat capacity
dQ Q 5RQ 10
C= = = = R
dT 2( dU ) 2  3Q  3
 
5R  4 
T 90. Volume of the ideal gas is constant so W = 0
p ∝T Using first law of thermodynamics
∆Q = ∆U ⇒ ∆U = i 2rt
p = 
nR   mR  T
or T = 
V   MV  = 12 × 100 × 5 × 60 = 30 × 103 = 30 kJ
i.e. p-T graph is a straight line passing through origin. Slope 91. pV 2 = constant
of this line is proportional to m, mass of gas. 1 1
∴ pV ∝ or T ∝
∴ tan α = 2 tan β V V
82. pV 2/ 3 = constant So, when gas is expand them the temperature of the gas is
 T  ⋅ V 2/ 3 = constant decreases.
∴  
V  92. p-V diagram of the gas is a straight line passing through
origin. Hence, p ∝ V or pV −1 = constant
or TV −1/ 3 = constant
Molar heat capacity in the process pV x = constant is
or T ∝ V 1/ 3 R R
with increase in V, temperature T will also increase. C= + ; Here γ =1.4 (for diatomic gas)
γ −1 1 − x
83. This is the case of free expansion and in this case ∆W = 0, R R
⇒ C= + ⇒ C = 3R
∆U = 0 so temperature remains same, i.e. 300 K. 1.4 − 1 1 + 1
688 Objective Physics Vol. 1

3 5
Level 2 : Only One Correct Option =
2
( pBV B − pAV A ) + ( pC VC − pBV B )
2
nRT
1. Given p = 3 5
V −b = ( 2p0V 0 − p0V 0 ) + ( 4p0V 0 − 2p0V 0 )
2 2
2V 2V dV
W =∫ p dV = nRT ∫V (as T = constant)
= 6.5 p0V 0
V V −b
 2V − b  Wtotal
∴ η= × 100
= nRT ln   Q+ ve
 V −b 
 2V − b  ≈15%
= RT ln   (as n =1)
 V −b  8. In a cyclic process, ∆U = 0
2. Process AB is isochoric. Hence, WAB = 0 ∴ Qtotal = Wtotal
or 10 − 5 = − 3 + W21
Process BC is isothermal. Hence,
or W21 = 8 J
V 
WBC = nRT2 ln  f  9. V-T graph is a straight line passing through origin or V ∝ T .
 Vi 
Hence, the given process is isobaric.
V  ∆Q nC p ∆T
or WBC = RT2 ln  1  =
 V2  ∆W nR∆T
Cp
Process CA is isobaric (as V ∝ T ) =
R
∴ WCA = p(V f − V i ) = nR(T f − Ti )
5
= R(T1 − T2 ) =
2
3. Net work done = Area ABC − Area AED = 2.5
1 1 1 10. V = kT 2
=
× 2 ×1 − ×1 ×1 = J
2 2 2
∴ 2kT ⋅ dT
RT R(T0 + αV )
4. We know that p = = and
RT
p= =
RT
=
R
V V V kT 2 kT
V2 V 2 R(T + αV )
∴ W = ∫ p ⋅ dV = ∫ 0 ⋅ dV or p dV = 2R dT
V1 V1 V
W = ∫ p dV = 2R ∫ dT
V
= RT0 ln 2 + Rα (V 2 − V1 )
V1 = 2R(60)
∆U = nCV ∆T =120R

= (C p − R )[(T0 + αV 2 ) − (T0 + αV1 )]


11. VT = constant, V ( pV ) = constant (as T ∝ pV )
= α (C p − R )(V 2 − V1 ) ∴ pV 2 = constant
Now, Q = W + ∆U
Molar heat capacity of a gas in the process
V
= RT0 ln 2 + αC p (V 2 − V1 ) pV x = constant
V1
R
C = CV +
5. Given dQ = 2 ⋅ dU (given) 1− x
∴ nC ⋅ dT = 2(nCV dT ) Here, x=2
3 3 R
or C = 2CV = 2 × R = 3R C= R+
2 2 1− 2
6. In process AB, density is constant. Hence, volume is C=R
constant. Therefore, work done will be zero. 12. In adiabatic process ∆U = − W
1
In process BC, p ∝ ρ but ρ ∝
V ∴ UA − UB = − ( −30) = 30J

∴ p∝
1 or UB − UA = − 30J
V 13. T = kV 2
Or the process is isothermal. Hence, internal energy is
∴ dT = ( 2kV ) ⋅ dV
constant.
7. Wtotal = Area under cycle = p0V0 dT nRT
or dV = p=
2kV V
Positive heat = nCV (TB − TA ) + nC p (TC − TB )
nRT nRT nR
p dV = ⋅ dT = ⋅ dT = ⋅ dT
= n  R  (TB − TA ) + n  R  (TC − TB )
3 5
2  2  2kV 2 2(T ) 2
3 5 4T0 nR 3
= (nRTB − nRTA ) + (nRTC − nRTB ) W = ∫ p dV = ∫ ⋅ dT = nRT0
2 2 T0 2 2
The First Law of Thermodynamics 689

14. WA → B = Area under p-V diagram =18 p0V0 23. Consider the p-V diagram shown for the container A
(isothermal) and for container B (adiabatic).
∆UA → B = nCV ∆T
= (1)  R  
3 30p0V 0 3p0V 0  81
− = p0V 0 2 2
2   R R  2 p p
117
∴ QA → B = WA → B + ∆UA → B = p0V 0
2
Q
Molar heat capacity C = p0 1 p0 1
n∆ T
117 V0 2V0 V0 2V0
p0V 0 V V
2 13
= = R Container A Container B
 30p0V 0  −  3p0V 0  6
    (Isothermal) (Adiabatic)
 R   R 
2 1 Both the process involving compression of the gas.
15. Number of moles = =
4 2 For isothermal process (gas A) (during 1 → 2)
WA → B = Area under p-V graph p1V1 = p2V 2 ⇒ p0 ( 2 V 0 ) = p2 (V 0 )
1 3
= × 3p0 × V 0 = p0V 0 ⇒ p2 = 2p0
2 2
1  3   8p0V 0 2p0V 0  9 For adiabatic process, (gas B) (during 1 → 2)
∆UA → B = nCV ∆T =  R   −  = p0V 0
22   R R  2 p1V1 γ = p2V 2 γ
QA → B = WA → B + ∆UA → B = 6p0V 0 ⇒ p0 ( 2 V 0 )γ = p2 (V 0 )γ
16. In cyclic process ∆U = 0  2V 
γ
⇒ p2 =  0  p0 = ( 2)γ p0
∴ Heat absorbed = work done  V0 
= area under p-V graph
( p2 )B ( 2)γ p0
= ( π )( Rx )( R y ) Hence, = Ratio of final pressure = = 2γ − 1
( p2 )A 2p0
= π (10 × 103 )(10 × 10−3 )
where, γ is ratio of specific heat capacities for the gas.
= (102 π ) J
17. ∆U = same in all processes.
More than One Correct Options
W1 = + ve,W2 = 0
1. Now see the hint of Q-No 14 (d) of subjective questions for
and W3 = − ve
JEE mains.
∴ Q1 > Q2 > Q3 p
18. Q = W + ∆U A
p
= p0 ( 2V 0 − V 0 ) + [2p0 ⋅ 2V 0 − 2p0V 0 ]
= p0V 0 + 2p0V 0 = 3p0V 0 p
2 B
19. ∆Q1 < ∆Q2
∆Q1 ∆Q2 C1 V
∴ < or C1 < C 2 or <1 V 2V
∆T ∆T C2
2. Temperature is increases. So, internal energy will also
20. Uρ = constant increase.

T
= constant ∴ ∆U = + ve
V Further,
∴ p = constant pV 2 = constant
dU dU CV
= = ∴  T  V 2 = constant
dW dQ − dW C p − CV  
V 
1 1 3 1
= = = or V ∝
C p / CV − 1 ( 5 / 3) − 1 2 T
21. ∆U1 = ∆U2 as U is state function. Temperature is increased. So, volume will decrease and
work done will be negative.
∴ Q1 − W1 = Q2 − W2 or W2 = Q2 + W1 − Q1 In the process pV x = constant, molar heat capacity is given
= (105 ) + (6.5 × 105 ) − (8 × 105 ) by
= −0.5 × 105 J R
C = CV +
22. ∆U = 0 1− x
1 Here, x = 2
∴ T = constant or p ∝
V ∴ C = CV − R
So, the graph is represented by hyperbola CV of any gas is greater than R.
690 Objective Physics Vol. 1

So, C is positive. Hence, from the equation, 5. Q1 = nCV ∆T


Q = nC∆T Q2 = nC P ∆T
Q is positive, if T is increased. Q3 = 0
or, ∆T is positive. C P > CV ∴ Q2 > Q1 > Q3
3. Process in a- b, W = 0 (as V = constant)
Q1 = ∆U1 = nCV ∆T 6. Let the equilibrium temperature of the system is T.
= ( 2)  R  ( 2T0 − T0 ) = 3RT0
3 Let us assume that T1 , T2 < T < T3 .
2 
According to question, there is no net loss to the
Process in b-c, ∆U = 0 (as T = constant) surroundings.
V  Heat lost by M 3 = Heat gained by M1 + Heat gained by M 2
∴ Q2 = W2 = nRT ln  f  ⇒ M 3 S(T3 − T ) = M1 S(T − T1 ) + M 2 S(T − T2 )
 Vi 
(where, S is specific heat of the copper material)
= ( 2)( R )( 2T0 )ln( 2)
⇒ T [ M1 + M 2 + M 3 ] = M 3T3 + M1T1 + M 2T2
= 4RT0 ln( 2) M T + M 2T2 + M 3T3
⇒ T= 1 1
Process in c-d, W = 0 M1 + M 2 + M 3
Q3 = ∆U3 = nCV ∆T 7. (a) The given process is a cyclic process i.e. it returns to the
= ( 2)  R  (T0 − 2T0 ) = − 3RT0
3 original state 1.
2  Hence, change in internal energy dU = 0
Process in d-a, ∆U = 0 ⇒ dQ = dU + dW = 0 + dW = dW
V  Hence, total heat supplied is converted to work done by
Q4 = W4 = nRT ln  f  the gas (mechanical energy) which is not possible by
 Vi  second law of thermodynamics.
= ( 2)( R )(T0 ) ln  
1 (c) When the gas expands adiabatically from 2 to 3. It is not
 2 possible to return to the same state without being heat
= − 2RT0 ln( 2) supplied, hence the process 3 to 1 cannot be adiabatic.
Now in complete cycle, 8. Consider the figure, we can write Q1 = W + Q2
∆Unet = 0 ⇒ W = Q1 − Q2 > 0 (by question)
Qnet = Wnet = 2RT0 ln( 2) = + ve ⇒ Q1 > Q2 > 0 (If both Q1 and Q2 are positive)
4. Process in a-b, ρ = constant We can also, write Q2 < Q1 < 0 (If both Q1 and Q2 are negative).
∴ V = constant
∴ W =0 Comprehension Based Questions
Q = ∆U pV
1. Qnet = Wnet = area under the cycle =
∆U is positive, as U is increasing. 2
Hence, Q is also positive. CV 1 5
2. = =
Process in b-c, ρ ∝u Cp γ 3
1
∴ ∝T 3. In adiabatic process,
V γ−1
ρ is decreasing, so V is increasing. Hence, work done is 1−γ γ γ
positive. p T = constant or T ∝ p
1 γ −1
Further, ∝ T (T ∝ pV ) TB  pB  γ
V ∴ = 
TA  pA 
∴ pV 2 = constant
5/ 3 − 1
In the process, pV x = constant, 0.4
TB = (1000)   = (1000)  
2 5/ 3 2

Molar heat capacity is given by  3  3
R
C = CV + = 850 K
1− x
4. In adiabatic process
Here x = 2
WAB = − ∆UAB (as Q = 0 )
∴ C = CV − R
= nCV (TA − TB )
For any of the gas, CV ≠ R.
= (1) R  (TA − TB )
3
∴ C ≠0
2 
∴ Q = nC∆T ≠ 0 as ∆U ≠ 0 and ∆T ≠ 0
= (1)  ×  (1000 − 850)
Process in c-a, 3 25
2 3 
ρ is increasing. Hence, V is decreasing. So, work done is
negative. =1875 J
The First Law of Thermodynamics 691

5. WBC = 0 (as V = constant)


Match the Columns
∴ T ∝p (as V = constant)
1. Process a-b T = constant
pC = pB/2
∴ TC = TB/2 = 425 K ∴ ∆T = 0 or ∆U = 0
1
Q = ∆U = nCV ( ∆T ) p∝
V
= (1)  R  (TC − TB )
3
1
2  p is increasing. Therefore, V is decreasing V ∝ or p will
p
3 25
= × × ( 425 − 850) increase.
2 3
Process b-c p = constant ⇒ ∴ V = constant
= 5312.5 J
Therefore, p ∝ T
Since, temperature is increasing. Hence, pressure should
Assertion and Reason also increase.
1. For isobaric process also T-t graph under given condition 2. In cyclic process ∆U = 0 ⇒ ∴ ∆Q = ∆W
may be a straight line. Replace CV by C p .
In isobaric process
∆Q nC p ∆T C p 5 / 2 R 5
2. = = = = ∆W = ∆Q − ∆U
∆W nR∆T R R 2
= nC p ∆T − nCV ∆T = nR∆T
3. In adiabatic expansion gas is cooled down. Therefore,
In isochoric process ∆W = 0, ⇒ ∴ ∆Q = ∆U
temperature decreases or product of p and V will decrease.
Because, T ∝ pV In adiabatic expansion ∆Q = 0
4. In isochoric process p and V both are constants. Therefore, ∴ ∆U = − ∆W or ∆U < 0
p -V graph is a dot. As, work done in expansion is positive.
RT
5. For given source and sink temperatures efficiency of Carnot 3. BT = p = (as n =1)
engine is maximum but not 100%. V
5 RT
6. For given temperatures T1 and T2 efficiency of Carnot engine B S = γp =
is maximum. 3 V
p − RT
7. Volume is increasing. Therefore, W is positive. Therefore, Slope of p-V graph is isothermal process = − = 2
dU is negative. V V
and Slope of p-V graph in adiabatic process
Now, Q = W + ∆U
γp 5 RT
Therefore, Q may be positive, negative or zero. =− =−
V 3V2
Further, heat given to gas in both function.
8. First law can be applied for any type of system.
9. Area under p-V graph in process 1 is more. Entrance Gallery
p 1. Given, Wibf =150J
a f
Wiaf = 200J ⇒ Qiaf = 500J
p∝ 1 So, Uiaf = 300J ⇒ U f = 400J p
V
Uib =100J i b
p∝ 1
V2 Qib = 100 + 50 = 150J V
Qibf = 300 + 150 = 450J
V1 V2 V Q 450 − 150 2
Q So, the required ratio bf = =
12. C = Qib 150 1
n∆ T
In isothermal process, ∆T = 0 5R
2. For diatomic gas, CV =
Therefore, C is not defined. Further, in isothermal process pV 2
5R
and T both are constants. ∆U = nCV ∆T = 1 × ∆T
Therefore, p-V versus T graph is a dot. 2
13. If initial and final volumes are equal, then work done may or For BC, ∆T = − 200 K ⇒ ∆U = − 500 R
may not be zero. For work done by the gas to be zero 3. By Carnot ideal heat engine,
volume should remain constant throughout the process. T2
η =1 −
For example in isochoric process. T1
14. In adiabatic expansion, where, η1 = 0.2, η2 = 0.4
Q =0 For the first condition,
W = + ve T T2
η1 = 1 − 2 ⇒ 0.2 = 1 −
∴ ∆U = − ve T1 T1
Therefore U, T or product of pV will decrease. T1 − T2
or 0.2 =
Because, U ∝ T ∝ pV T1
692 Objective Physics Vol. 1

For the second condition 7. Heat is extracted from the source means heat is given to the
T − 50 T − (T2 − 50) system (or gas) or Q is positive. This is positive only along
η2 = 1 − 2 ⇒ 0.4 = 1
T1 T1 the path ABC.
T − T2 + 50 T − T2 50 Heat supplied,
0.4 = 1 ⇒ 0.4 = 1 +
T1 T1 T1 ∴ QABC = ∆UABC + WABC
50 = nCV (T f − Ti ) + Area under p-V graph
0.4 = 0.2 + [from Eq. (i)] 3R
T1 [for monoatomic gas, CV = ]
2
50
0.4 − 0.2 =
= n  R  (TC − TA ) + 2p0V 0
3
T1
2 
50
T1 = ⇒ T1 = 250 K 3
0.2 = (nRTC − nRTA ) + 2p0V 0
2
Putting the value of T1 in Eq. (i), we get 3
T − T2 = ( pC VC − pAV A ) + 2p0V 0
0.2 = 1 ⇒ T2 = T1 − 0.2T1 2
T1 3 13
= ( 4p0V 0 − p0V 0 ) + 2p0V 0 = p0V 0
T2 = 250 − 0.2 × 250 2 2
T2 = 250 − 50 ⇒ T2 = 200 K 8. The work done during the process from P to Q = area
So, T1 = 250 K, T2 = 200 K PQRSTP (positive sign is to be taken due to expansion along
4. Given, efficiency, η = 70% PQ)
Sink temperature, T2 = 27 + 273 = 300 K Area of triangle PQR + area of rectangle PRST
Source temperature, T1 = ? 1
= × 2V × 2p + p × 2V = 4 pV
As, we know that efficiency is given by 2
T 300 Work done during process from R to P = − Area RSTP
η = 1 − 2 ⇒ 70% = 1 − (negative sign due to compression in atom RP)
T1 T1
= − ST × PT = − 2V × 1p = − 2pV
70 300 300
=1 − ⇒ 0.7 = 1 − Hence, the work done in the complete cycle
100 T1 T1
= 4pV − 2pV = 2pV
300
= 1 − 0.7 T −T
T1 9. Efficiency of Carnot engine, η = 1 2
T1
3000
⇒ T1 = The value of efficiency will be higher, if T1 − T2 is more. As
3
T1 − T2 is constant in all four cases. Therefore, efficiency will
T1 = 1000 K be highest when T1 is the lowest.
or T1 = 1000 − 273 = 727°C 10. (a) Curve OA represents isobaric process (since pressure is
5. In a tyre burst, there is too little time for the temperature to constant). Since, the slope of adiabatic process is more
be equalized with the surroundings the work done due to steeper than the isothermal process.
the sudden expansion causes the air surrounding the tyre to p O
get cooler. It is adiabatic, because no heat transfer occurs A
here most of the processes that take very little time
is adiabatic.
6. In F → G, work done in isothermal process is B
V   32V 0  D C
nRT ln  f  = 32 p0V 0 ln   V
 Vi   V0 
(b) Curve OB represents isothermal process.
= 32p0V 0 ln 25 = 160 p0V 0 ln 2 (c) Curve OC represents adiabatic process.
In G → E, ∆W = p0 ∆V = p0 ( 31V 0 ) = 31 p0V 0 (d) Curve OD represents isochoric process (since volume is
p constant)
F 11. Change in internal energy of the system does not depend on
32p0 32p0 path followed.
So, ∆U1 = ∆U2
Isochoric
Is iabatic
ot
Ad
he

12. The process may be assumed to be isobaric.


rm

∴ Q = nC p ∆T
al

T0
= ( 2)  R  ( 5) = 5 × 8.31 × 5 = 207.75 J = 208 J
p0 G 5
E H Isobaric
2 
V
V0 32V0 13. Efficiency of a process is defined as the ratio of work done to
energy supplied. Here,
In G → H, work done is less than 31 p0V 0 , i.e. 24 p0V 0 ∆W Area under p-V diagram
η= =
In F → H, work done is 36 p0V 0 ∆Q ∆QAB + ∆QBC
The First Law of Thermodynamics 693

∴ η=
p0V 0 T   T − T2 
=  1 − 1 Q2 =  1  Q2
nCV ∆T1 + nC p ∆T2  T2   T2 
p0V 0 ForT1 = 20 K,
=
3 5 300 − 20
nR(TB − TA ) + nR(TC − TD ) W1 = × 0.001996 = 0.028 kJ
2 2 20
p0V 0
= For T2 = 4 K,
3 5
( 2p0V 0 − p0V 0 ) + ( 4p0V 0 − 2p0V 0 ) 300 − 4
2 4 W2 = × 0.001996 = 0.148 kJ
4
p0V 0 1
= = = 15.4% As temperature is changing from 20 K to 4 K, work done
3 5
p0V 0 + ⋅ 2p0V 0 6.5 required will be more than W1 but less than W2 .
2 2
17. Carnot efficiency,
14. At STP, T1 − T2
22.4 L of any gas is 1 mol, η=
T1
5.6 1
∴ 5.6 L = = mol = n T1 − T2
22.4 4 Case I = 0.4
T1
In adiabatic process,
TV γ −1 = constant T1 − T2 = 0.4 T1
γ −1 ⇒ T2 = 0.6 T1
V 
∴ T2V 2γ −1 = T1V1 γ −1
or T2 = T1  1  T1′ − T2
 V2  Case II = 0.5
T1′
C 5
For monoatomic He gas, γ = p = 0.6
CV 3 T1′ = T1 = 600 K
0.5
5
−1
∴ T2 = T1 
5.6  3
= 4T1
18. As work done, ∆W = 0, then there is no expansion of the

 0.7  water.
Further in adiabatic process, ∆Q = ∆U + ∆W
Q = 0 ⇒ W + ∆U = 0 ∆U = mc∆T
or W = − ∆U = − nCV ∆T = 100 × 10−3 × 4184 × ( 50 − 30) = 8.4 kJ
 R 
= − n  (T2 − T1 ) 19. In case of no work done, W = 0, then volume expansion V = 0 .
 γ −1
So, the volume remains zero V = 0. This process is called
  isochoric process.
1 R  9
=−   ( 4T1 − T1 ) = − RT1 20. n = n1 + n2
4  5 −1 8
3  ⇒ pV = p1V1 + p2V 2
T2 1 T
15. η1 = 1 − ⇒ =1 − 2
T1 6 T1
T2 5 +
⇒ = …(i) x y z
T1 6
( T − 62)
η 2 =1 − 2
T1
4T 4T 4T
1 (T2 − 62) p1 = p0 + , p2 = p0 + , p = p0 +
⇒ =1 − …(ii) x y z
3 T1
where, T is surface tension,
On solving Eqs. (i) and (ii), we get If the process takes place is vacuum, then p0 = 0
T1 = 372 K and T2 = 310 K 4T 4T 4T
p1 = , p2 = , p=
16. Heat required to change the temperature of vessel by a x y z
small amount dT
The process is isothermal
− dQ = mC p dT
3 ∴ p1 V1 + p2 V 2 = pV
32 
4 T 
Total heat required, − Q = m∫  dT ⇒ z = x2 + y 2
20  400 
4 21. TA = TB
100 × 10−3 × 32  T 4 
=  4  ∴ U A = UB
( 400)3   20
V 
⇒ Q = 0.001996 kJ ≈ 0.002 kJ WAB = (1)( R )T0 ln  f 
 Vi 
Work done required to maintain the temperature of sink to T2
Q − Q2  4V 
W = Q1 − Q2 = 1 Q2 = RT0 ln  0  = p0V 0 ln 4
Q2  V0 
694 Objective Physics Vol. 1

Information regarding p and T at C cannot be obtained T1V1 γ − 1 = T2 V2 γ − 1


from the given graph. Unless it is mentioned that line BC γ −1
V 
passes through origin or not. Hence, the correct options are ⇒ T1 = T2  2 
(a) and (b).  V1 
7
22. In adiabatic process, −1
γ −1 T1 = T2 ( 32) 5
TV = constant
∴ Ti V i 0.4 = T f V f 0.4 (as γ =1.4 for diatomic gas) = T2 ( 25 )2/ 5 = T2 × 4

0.4 T1 = 4T2
V 
Ti V i 0.4 = ( aTi )  i 
η = 1 − 
or 1
 32  ⇒
 4
or a = ( 32)0.4 = 4
3
=
= 0.75
23. The efficiency of engine is 4
T2
η =1 − 24. Efficiency,
T
η =1 − 2 =1 −
500 3
= = 0.375
T1 T1 800 8
For adiabatic process, T2
25. Efficiency of Carnot heat engine, η = 1 − . Efficiency
TV γ − 1 = constant T1
7 remains same when both T1 and T2 are increased by same
For diatomic gas, γ=
5 factor.
16
Calorimetry
and Heat Transfer
16.1 Specific Heat
When heat energy flows into a substance, the temperature of the substance usually Chapter Snapshot
rises. An exception occurs during a change in phase, as when water freezes or evaporates. ● Specific Heat
The amount of heat required to produce, the same temperature increase for a given amount ● Phase Changes and
of substance varies from one substance to another. The relationship between heat Latent Heat
exchanged and the corresponding temperature change is characterised by the specific ● Heat Transfer
heat c of a substance. If the temperature of a substance of mass m changes from T to T + dT
when it exchanges an amount of heat dQ with its surroundings, then its specific heat is
1 dQ
c= ⋅ …(i)
m dT
The SI unit of specific heat c is J/kg-K. Because heat is so frequently measured in
calories, the unit cal/g-°C is also used quite often. The specific heat capacity of water is
approximately 1 cal/g-°C.
From Eq. (i), we can define the specific heat of a substance as the amount of energy
needed to raise the temperature of unit mass of that substance by 1°C (or 1 K). A closely
related quantity is the molar heat capacity C. It is defined as
1 dQ
C= ⋅ …(ii)
n dT
where, n is the number of moles of the substance. If M is the molecular mass of the
m
substance, then n = where, m is the mass of the substance and
M
M dQ
C= ⋅ …(iii)
m dT
The SI unit of molar heat capacity C is J/mol-K and it can be defined as the amount of
energy needed to raise the temperature of one mole of a substance by 1°C (or 1 K)
sometimes the product of mc is also written as C, simply the heat capacity, which is defined
as the energy needed to raise the temperature of the whole substance by 1°C (or K).
dQ
Thus, C = mc = …(iv)
dT
The SI units of molar heat capacity C is J/K.
696 Objective Physics Vol. 1

Note Points water or 80.0 cal heat is liberated when 1.0 g of water freezes
/ In general, if c varies with temperature over the interval, then at 0°C. Similarly, latent heat of vaporisation for water at 1 atm
the corresponding expression for Q is is 539 cal/g.
T2
Q = m∫ c ⋅ dT Figure shows how the temperature varies when we add
T1
heat continuously to a specimen of ice with an initial
/ The specific heat of water is much larger than that of most
temperature below 0°C. Suppose, we have taken 1 g of ice at
other substances. Consequently, for the same amount of
added heat, the temperature change of a given mass of –20° C specific heat of ice is 0.53 cal/g-°C.
water is generally less than that for the same mass of another T(°C)
substance. For this reason a large body of water moderates
the climate of nearby land. In the winter the water cools off
more slowly than the surrounding land and tends to warm the
land. In the summer, the opposite effect occurs, as the water d
100 e
heats up more slowly than the land.

X Example 16.1 When 400 J of heat are added to a b


0 c
0.1 kg sample of metal, its temperature increases by
20°C. What is the specific heat of the metal? –20 a Q1 Q2 Q3 Q4
Q (cal)

1 ∆Q
Sol. Using, c= ⋅
m ∆T Fig. 16.1
1   400 
We have, c =    In the figure :
 0.1  20 
a to b Temperature of ice increases until it reaches its
= 200 J/kg -° C
melting point 0°C.
Q1 = mcice [0 – (–20)]
16.2 Phase Changes and = (1) (0.53) (20) = 10.6 cal
Latent Heat b to c Temperature remains constant until all the ice has
melted.
Suppose that we slowly heat a cube of ice whose
Q2 = mL f = (1) (80) = 80 cal
temperature is below 0°C at atmospheric pressure, what
c to d Temperature of water again rises until it reaches its
changes do we observe in the ice? Initially, we find that
boiling point 100°C.
its temperature increases according to equation
Q3 = mc water [100 – 0] = (1) (1.0) (100) = 100 cal
Q = mc (T2 – T1 ). Once 0°C is reached, the additional heat
d to e Temperature is again constant until all the water is
does not increase the temperature of the ice. Instead, the ice
transformed into the vapour phase.
melts and temperature remains at 0°C. The temperature of
Q4 = mLv = (1) (539) = 539 cal
the water, then starts to rise and eventually reaches 100°C,
whereupon the water vaporises into steam at this same Thus, the net heat required to convert 1 g of ice at
temperature. – 20° C into steam at 100°C is
Q = Q1 + Q2 + Q3 + Q4 = 729.6 cal
During phase transitions (solid to liquid or liquid to gas)
the added heat causes a change in the positions of the X Example 16.2 How much heat is required to
molecules relative to one another, without affecting the convert 8.0 g of ice at – 15° C to steam at 100°C?
temperature. (Given, cice = 0.53 cal/g-°C, L f = 80 cal / g and Lv = 539
The heat necessary to change a unit mass of a substance cal/g, and cwater =1 cal/g-°C)
from one phase to another is called the latent heat (L). Thus, ice ice water water steam
the amount of heat required for melting and vaporising a –15°C 0°C 0°C 100°C 100°C
Q1 Q2 Q3 Q4
substance of mass m are given by
Fig. 16.2
Q = mL …(i)
For a solid-liquid transition, the latent heat is known as Sol. Q1 = mcice (Tf – Ti ) = (8.0) (0.53) [0 – (–15)] = 63.6 cal
the latent heat of fusion ( L f ) and for the liquid-gas Q2 = mLf = (8) (80) = 640 cal
Q3 = mc water (Tf – Ti ) = (8.0) (1.0) [100 – 0] = 800 cal
transition, it is known as the latent heat of vaporisation( Lv ).
Q4 = mLv = (8.0) (539) = 4312 cal
For water at 1 atm latent heat of fusion is 80.0 cal/g. This ∴ Net heat required,
simply means 80.0 cal of heat are required to melt 1.0 g of Q = Q1 + Q2 + Q3 + Q4 = 5815.6 cal
Calorimetry and Heat Transfer 697

X Example 16.3 10 g of water at 70°C is mixed to their neighbours further along the rod. Such transfer of
with 5 g of water at 30°C. Find the temperature of the heat through a substance in which heat is transported
mixture in equilibrium. without direct mass transport is called conduction.
Most metals use another, more effective mechanism to
Sol. Let t °C be the temperature of the mixture. From energy
conservation,
conduct heat. The free electrons, which move throughout the
Heat given by 10 g of water = Heat taken by 5 g of water
metal can rapidly carry energy from the hotter to colder
or m1c water |∆t 1| = m2c water |∆t 2| regions, so metals are generally good conductors of heat. The
∴ (10) (70 – t ) = 5 (t – 30) presence of free electrons also causes most metals to be good
∴ t = 36.67 ° C electrical conductors. A metal rod at 5°C feels colder than a
piece of wood at 5°C, because heat can flow more easily from
X Example 16.4 In a container of negligible mass your hand into the metal.
30 g of steam at 100°C is added to 200 g of water that Heat transfer occurs only between regions that are at
has a temperature of 40°C. If no heat is lost to the dQ
different temperatures and the rate of heat flow is . This
surroundings, what is the final temperature of the dt
system? Also, find masses of water and steam in rate is also called the heat current, denoted by H.
equilibrium. Experiments show that the heat current is proportional to the
(Take, Lv = 539 cal / g and cwater = 1 cal / g- ° C ) cross-section area A of the rod and to the temperature
dT
Sol. Let Q be the heat required to convert 200 g of water at 40°C gradient , which is the rate of change of temperature with
dx
into 100°C, then
distance along the bar. In general,
Q = mc∆T = (200) (1.0) (100 – 40) = 12000 cal
dQ dT
Now, suppose m0 mass of steam converts into water to H= = – KA …(i)
liberate this much amount of heat, then dt dx
Q 12000 dQ
m0 = = = 22.26 g The negative sign is used to make a positive
L 539 dt
Since, it is less than 30 g, the temperature of the mixture is dT
100°C. quantity, since is negative. The constant K, called the
Mass of steam in the mixture = 30 – 22.26 = 7.74 g
dx
thermal conductivity is a measure of the ability of a
and mass of water in the mixture
material to conduct heat.
= 200 + 22.26 = 222.26 g
A substance with a large thermal conductivity K is a
good heat conductor. The value of K depends on the
16.3 Heat Transfer temperature increasing, it is slightly increasing with
Heat can be transferred from one place to the other by increasing temperature, but K can be taken to be practically
any of three possible ways : conduction, convection and constant throughout a substance, if the temperature
radiation. In the first two processes, a medium is necessary difference between its ends is not too great.
for the heat transfer. Radiation, however, does not have this Let us apply Eq. (i) to a rod of length L and constant
restriction. This is also the fastest mode of heat transfer, in cross-sectional area A in which a steady state has been
which heat is transferred from one place to the other in the reached. In a steady state (has been discussed in detail in
form of electromagnetic radiation. In competition medical galaxy 16.1), the temperature at each point is
examinations problems are asked only in first and last. So, constant in time. Hence,
we will discuss conduction and radiation in detail. dT
– = T1 – T2
dx
Conduction Therefore, the heat ∆Q transferred in time ∆t is
Figure shows a rod T1 > T2
 T – T2 
whose ends are in thermal T1 T2 ∆Q = KA  1  ∆t …(ii)
Q  l 
contact with a hot reservoir (Hot) (Cold)

at temperature T1 and a cold Thermal resistance ( R ) Eq. (ii) in differential form can be
reservoir at temperature T2 . Fig. 16.3 written as
∆Q dQ ∆T ∆T
The sides of the rod are covered with insulating medium, so = =H = = …(iii)
the transport of heat is along the rod, not through the sides. ∆t dt l / KA R
The molecules at the hot reservoir have greater vibrational Here, ∆T = temperature difference (TD) and
energy. This energy is transferred by collisions to the atoms l
at the end face of the rod. These atoms in turn transfer energy R= = thermal resistance of the rod.
KA
698 Objective Physics Vol. 1

Extra Knowledge Points


■ Consider a section ab of a rod as shown in figure.Suppose The answer is that there must be a temperature
Q1 heat enters into the section at a and Q 2 leaves at b, then difference in the rod for the heat flow, same as we require
Q 2 < Q1. Part of the energy Q1 – Q 2 is utilized in raising the a potential difference across a resistance for the current
temperature of section ab and the remaining is lost to flow through it.
atmosphere through ab. In steady state, the temperature varies linearly with
distance along the rod, if it is insulated.
Q1 Q2 ■ Comparing equation number (iii), i.e. Heat current,
a b dQ ∆T  l 
H= =  where, R = 
dt R  kA 
If heat is continuously supplied from the left end of the
rod, a stage comes when temperature of the section with the equation, of current flow through a resistance,
dq ∆V  l 
becomes constant. In that case, Q1 = Q 2 , if rod is i = =  where, R = 
insulated from the surroundings (or loss through ab is dt R  σA 
zero). This is called the steady state condition. Thus, in We find the following similarities in heat flow through a
steady state temperature of different sections of the rod rod and current flow through a resistance.
becomes constant (but not same).
Hence, in the figure : Table 16.1
T1 = constant Heat flow through a conducting Current flow through a
T2 = constant, etc. rod resistance
and T1 > T2 > T3 > T4 Heat current, H =
dQ
= rate of Electric current,
dq
Q Q
dt i = = rate of charge flow
heat flow dt
∆T TD ∆V PD
H= = i = =
T1 T2 T3 T4 R R R R
l l
Insulated rod in steady state R = R =
kA σA
Now, a natural question arises, why the temperature of K = thermal conductivity σ = electrical conductivity
whole rod not becomes equal when heat is being
From the above table, it is evident that flow of heat
continuously supplied?
through rods in series and parallel is analogous to the
T flow of current through resistances in series and parallel.
T1 This analogy is of great importance in solving
complicated problems of heat conduction.From the
T4 above table, it is evident that flow of heat through rods in
series and parallel is analogous to the flow of current
through resistances in series and parallel. This analogy
x is of great importance in solving complicated problems
of heat conduction.

Convection When the heated substance is forced to move by a fan or


Although conduction does occur in liquids and gases pump, the process is called forced convection. If it were not
also, heat is transported in these media mostly by for convection currents, it would be very difficult to boil
convection. In this process, the actual motion of the material water. As water is heated in a kettle, the heated water
is responsible for the heat transfer. Familiar examples expands and rises to the top, because its density is lowered.
include hot-air and hot-water home heating systems, the At the same time, the denser, cool water at the surface sinks
cooling system of an automobile engine and the flow of to the bottom of the kettle and is heated. Heating a room by a
blood in the body. radiator is an example of forced convection.
You probably have warmed your hands by holding them It is possible to write an equation for the thermal energy
over an open flame. In this situation, the air directly above transported by convection and define a coefficient of
the flame is heated and expands. As a result, the density of convection, but the analysis of practical problems is very
this air decreases and then air rises. When the movement difficult and will not be treated here. To some
approximation, the heat transferred from a body to its
results from differences in density, as with air around fire, it
surroundings is proportional to the area of the body and to
is referred to as natural convection. Air flow at a beach is an
the difference in temperature between the body and the
example of natural convection. surrounding fluid.
Calorimetry and Heat Transfer 699

Radiation As a perfectly black body absorbs all radiations incident


on it, the absorptive power of a perfectly black body is
The third means of energy transfer is radiation which maximum and unity.
does not require a medium. The best known example of this
process is the radiation from the sun. All objects radiate Spectral Absorptive Power (a λ )
energy continuously in the form of electromagnetic waves.
The rate at which an object radiates energy is proportional to The absorptive power a refers to radiations of all
the fourth power of its absolute temperature. This is known wavelengths (or the total energy) while the spectral
as the Stefan’s law and is expressed in equation form as absorptive power is the ratio of radiant energy absorbed by a
surface to the radiant energy incident on it for a particular
P = σAeT 4
wavelength λ. It may have different values for different
Here, P is the power in watts (J/s) radiated by the object, wavelengths for a given surface. Let us take an example,
A is the surface area in m 2 , e is a fraction between 0 and 1 suppose a = 0.6,a λ = 0.4 for 1000 Å and a λ = 0.7 for 2000 Å
called the emissivity of the object and σ is a universal for a given surface. Then, it means that this surface will
constant called Stefan’s constant, which has the value absorb only 60% of the total radiant energy incident on it.
Similarly, it absorbs 40% of the energy incident on it
σ = 5.67 × 10 –8 W / m 2 -K 4 corresponding to 1000 Å and 70% corresponding to 2000 Å.
Now, let us define few terms before studying the other The spectral absorptive power a λ is related to absorptive
topics. power a through the relation

Perfectly Black Body a = ∫ a λ dλ
0

A body that absorbs all the radiation incident upon it and


has an emissivity equal to 1 is called a perfectly black body. A Emissive Power (e)
black body is also an ideal radiator. It implies that, if a black (Don’t confuse it with the emissivity e which is different
body and an identical another body are kept at the same from it, although both have the same symbol e).
temperature, then the black body will radiate maximum power For a given surface, it is defined as the radiant energy
as it is obvious from equation ( P = eAσT 4 ) also. Because e =1 emitted per second per unit area of the surface. It has the
for a perfectly black body while for any other body e <1. units of W / m 2 or J/s-m 2 . For a black body, e = σT 4 .

Spectral Emissive Power (e λ )


It is emissive power for a particular wavelength λ.
Thus,

e = ∫ eλ dλ
0

Kirchhoff ’s Law According to this law, the ratio of


Cavity approximating an ideal black body.
Radiation entering the cavity has little emissive power to absorptive power is same for all surfaces
chance of leaving before it is completely at the same temperature.
absorbed.
Perfectly black
Fig. 16.4 T T body

Materials like black velvet or lamp black come close to


being ideal black bodies, but the best practical realisation of 1 2 T
an ideal black body is a small hole leading into a cavity, as
this absorbs 98% of the radiation incident on them.
Fig. 16.5
Absorptive Power (a) e1 e2  e 
“It is defined as the ratio of the radiant energy absorbed Hence, = = 
a1 a 2  a  Perfectly black body
by it in a given time to the total radiant energy incident on it
in the same interval of time.” but ( a ) black body =1
Energy absorbed and ( e) black body = E (say)
a=
Energy incident  e
Then,   = constant = E
 a  for any surface
700 Objective Physics Vol. 1

Similarly, for a particular wavelength λ,  ∆T 


4

 eλ  So that, T = (T0 + ∆T )
4 4
= T04 1 + 
  = Eλ  T0 
 a λ  for any body
 4∆T 
≈ T04 1 +  (from binomial expansion)
Here, E = emissive power of black body at temperature T  T0 
= σT 4 ∴ (T 4 – T04 ) = 4T03 ( ∆T )
From the above expression, we can see that
or (T 4 – T04 ) ∝ ∆T (as T0 = constant)
eλ ∝ a λ
i.e. good absorbers for a particular wavelength are also Now, we have already shown that rate of cooling
good emitters of the same wavelength.  dT 
 ∝ (T – T0 )
4 4
–
 dt 
Cooling by Radiation and here we have shown that
Consider a hot body at temperature T placed in an (T 4 – T04 ) ∝ ∆T ,
environment at a lower temperature T0 .The body emits more
radiation than it absorbs and cools down while the if the temperature difference is small.
surroundings absorb radiation from the body and warm up. Thus, rate of cooling
The body is losing energy by emitting radiations at a rate, dT
– ∝ ∆T
P1 = eAσT 4 dt

and is receiving energy by absorbing radiations at a rate, or – ∝ ∆θ
P2 = aAσT04 dt
as dT = dθ or ∆T = ∆θ
Here, a is a pure number between 0 and 1 indicating the
relative ability of the surface to absorb radiation from its
surroundings. Note that this (a) is different from the
Variation of Temperature of a Body
absorptive power (a). In thermal equilibrium, both the body According to Newton’s Law
and the surroundings have the same temperature (say Tc ) Suppose a body has a temperature θ i at time t = 0. It is
and, placed in an atmosphere whose temperature is θ 0 . We are
P1 = P2 or eAσTc4 = aAσTc4 or e=a interested in finding the temperature of the body at time t,
assuming Newton’s law of cooling to hold good or by
Thus, when T > T0 , the net rate of heat transfer from the assuming that the temperature difference is small. As per
body to the surroundings is this law,
dQ θ0 = constant θ0 = constant
= eAσ (T 4 – T04 )
dt
 dT  θi θ
 = eAσ (T – T0 )
4 4
or mc  –
 dt 
⇒ Rate of cooling,
 dT  eAσ 4 t=0 t=t
– = (T – T04 ) Fig. 16.6
 dt  mc
dT rate of cooling ∝ temperature difference
or – ∝ (T 4 – T04 )  dθ   eAσ 
dt  =  ( 4 θ 0 ) (θ – θ 0 )
3
or –
 dt   mc 
Newton’s Law of Cooling  dθ 
or –  = α (θ – θ 0 )
According to this law, if the temperature T of the body is  dt 
not different from that of the surroundings T0 , then rate of  4eAσθ 30 
dT Here, α=  is a constant
cooling – is proportional to the temperature difference  mc 
dt
θ dθ t
between them. ∴ ∫ θ i θ – θ 0 = – α ∫ 0 dt
To prove it let us assume that
T = T0 + ∆T ∴ θ = θ 0 + (θ i – θ 0 ) e – αt
Calorimetry and Heat Transfer 701

From this expression, we see that θ = θ i at t = 0 and Here, b is a constant called Wien’s constant. The value
θ = θ 0 at t = ∞, i.e. temperature of the body varies of this constant in SI unit is 2.898 × 10 –3 m-K. Thus,
exponentially with time from θ i to θ 0 ( < θ i ). The 1
temperature versus time graph is as shown in the figure. λ max ∝
θ T
θi Here, λ max is the wavelength corresponding to the
maximum spectral emissive power eλ .
θ0 The second effect is that the total amount of energy the
black body emits per unit area per unit time ( = σT 4 )
t increases with fourth power of absolute temperature T. This
is also known as the emissive power. We know
Fig. 16.7
el
/ If the body cools by radiation from θ1 to θ2 in time t, then
taking the approximation,
 – dθ  = θ1 – θ2 and θ = θ =  θ1 + θ2 
   
 dt  t
av
 2 
dθ  T
The equation  –  = α (θ – θ0 ) becomes
A1
 dt  lm
l
θ1 – θ2  θ + θ2 
=α  1 – θ0  el
t  2 

This form of the law helps in solving numerical


problems related to Newton’s law of cooling. A2
2T
Wien’s Displacement Law l
lm
At ordinary temperatures (below about 600°C), the 2
thermal radiation emitted by a body is not visible, most of it Fig. 16.9
is concentrated in wavelengths much longer than those of ∞
visible light. e = ∫ eλ dλ = Area under eλ - λ graph = σT 4
0

or Area ∝ T 4 A2 = (2) 4 A1 = 16 A1

Thus, if the temperature of the black body is made two
4000 K fold, λ max remains half while the area becomes 16 times.
3000 K
2000 K
Extra Knowledge Points
■ In solids (and in mercury) transmission of heat takes
place only by conduction, while in liquids and gases it
0 1 2 3 4 mainly takes place by convection (In liquids and gases
Wavelength (µm) it is possible by conduction also).
Fig. 16.8 ■ Heat conduction in metals. Metals are very good
Power of black body radiation versus wavelength at three conductor of heat. In fact, the heat conduction in
temperatures. Note that the amount of radiation emitted (the area metals mostly takes place by the free electrons present
under a curve) increase with increasing temperature within the metals. These electrons are not bound to any
molecule of the metals, but they are free to move within
Figure shows how the energy of a black body radiation the metal. Thus, these electrons are just like the
varies with temperature and wavelength? As the molecules of a gas. Thus, the transmission of energy
temperature of the black body increases, two distinct (heat) in metals takes place by the free electrons, not
behaviours are observed. The first effect is that the peak of by molecules. This is why in metals the transmission of
heat takes place very rapidly.
the distribution shifts to shorter wavelengths. This shift is
■ Relation between thermal and electrical
found to obey the following relationship called Wien’s
conductivities of Metals : Wiede mann-Franz
displacement law. Law All good conductors of heat are also good
conductors of electricity. Wiede mann and Franz
λ max T = b expressed this fact in the form of an empirical law
702 Objective Physics Vol. 1

which states that, ‘the ratio of the thermal and the (b) the thermal current H.
electrical conductivities are same for all metals at the dT
same temperature. (c) the temperature gradient and
K
dx
i.e. = constant (d) the temperature 25 cm from the hot end.
σ
Thermal conductivity of copper is 401 W/m-K.
■ Lorentz extended the law and showed that this ratio is
proportional to the absolute temperature, l l
Sol. (a) Thermal resistance, R = =
K KA K( πr 2 )
i.e. = constant
σT (2 )
or R= = 15.9 K /W
■ Effects and uses of thermal capacity in daily life (401) ( π ) (10–2 )2
(i) During winter, iron seems colder and in summer ∆T ∆θ 100
(b) Thermal current, H = = =
seems warmer than wood During winter the R R 15.9
temperature of our body is higher than the room or H = 6.3 W
temperature. Hence, when we touch iron, the iron (c) Temperature gradient,
rapidly conducts heat from our hand and gives a 0 – 100
d= = – 50 K/m = – 50° C /m
cold feeling. Wood on the other hand, is a bad 2
conductor of heat. It conducts heat slowly from our (d) Let θc° be the temperature at 25 cm from the hot end,
hand and appears less cold. During summers
then
reverse is the process.
(θ – 100) = (temperature gradient) × (distance)
(ii) When hot water is poured in a beaker of thick 100°C θ°C 0°C
glass, the beaker cracks When we pour hot water
in a glass beaker, the inner surface of the glass
expands on heating. But heat from inside does not 0.25 m
reach quickly the outer surface of the glass,
2.0 m
because glass is a bad conductor of heat. Hence,
Fig. 16.10
the outer surface does not expand and the glass
cracks. or θ –100 = (– 50) (0.25) or θ = 87.5° C
(iii) In winters woolen cloths or blankets are
used The fibres of woolen clothes have larger X Example 16.6 Two metal cubes with 3 cm edges
interspaces than cotton clothes and air is filled in of copper and aluminium are arranged as shown in
these spaces. Since, air is a bad conductor of figure. Find
heat, hence, these clothes prevent heat from our (a) the total thermal current from one reservoir to the
body to go outside and our body remains warm. On other.
using woolen clothes for a long time the air is driven
out. Hence, old woolen clothes are less warm.
(iv) To prevent ice from melting it is wrapped with Al
blanket or felt The air filled in the interspaces of 100°C 20°C
these materials, being bad conductors of heat Cu
does not allow heat to flow from outside to the ice.
Hence, ice does not melt.
Fig. 16.11
■ Solar constant The amount of heat received from the
sun by one square centimetre area of a surface placed (b) the ratio of the thermal current carried by the copper
normally to the sun rays at mean distance of the earth cube to that carried by the aluminium cube. Thermal
from the sun is known as solar constant. It is denoted conductivity of copper is 401 W/m-K and that of
by S. aluminium is 237 W/m-K.
2
r
S =   ⋅ σT 4
R  Sol. (a) Thermal resistance of aluminium cube, R1 = l
KA
Here, r is the radius of the sun and S the mean distance
(3.0 × 10–2 )
of the earth from the centre of the sun. Value of solar or R1 = = 0.14 K/W
constant is 1.937 cal/cm 2 - min. (237) (3.0 × 10–2 )2
l
and thermal resistance of copper cube, R 2 =
KA
X Example 16.5 A copper rod 2 m long has a (3.0 × 10–2 )
or R 2 = = 0.08 K/W
circular cross-section of radius 1 cm. One end is kept (401) (3.0 × 10–2 )2
at 100°C and the other at 0°C, and the surface is As these two resistances are in parallel, their equivalent
insulated so that negligible heat is lost through the resistance will be,
surface. Find R1R 2 (0.14) (0.08)
R= = = 0.05 K/W
(a) the thermal resistance of the bar. R1 + R 2 (0.14) + (0.08)
Calorimetry and Heat Transfer 703

Temperature difference
∴ Thermal current, H = Sol. According to Newton’s law of cooling,
Thermal resistance
(100 – 20)  θ1 – θ2    θ1 + θ2  
= = 1.6 × 103 W   =α   2  – θ0 
0.05  t   
(b) In parallel thermal current distributes in the inverse ratio For the given conditions,
60 – 40 60 + 40
=α  – 10
of resistance. Hence,
0.14 …(i)
 2 
HCu R R 10
= Al = 1 = = 1.75
HAl RCu R 2 0.08 Let θ be the temperature after next 10 min. Then,
40 – θ 40 + θ
=α  – 10 …(ii)
X Example 16.7 One end of a copper rod of length 10  2 
1 m and area of cross-section 4.0 × 10 –4 m 2 is Solving Eqs. (i) and (ii), we get
maintained at 100°C. At the other end of the rod ice is θ = 28° C
kept at 0°C. Neglecting the loss of heat from the
surroundings find the mass of ice melted in 1 h.
X Example 16.9 Two bodies A and B have thermal
emissivities of 0.01 and 0.81, respectively. The outer
(Given, KCu = 401 W/m -K and L f = 3.35 × 10 5 J / kg)
surface areas of the two bodies are same. The two
bodies emit total radiant power at the same rate. The
Sol. Thermal resistance of the rod,
wavelength λ B corresponding to maximum spectral
100°C 0°C radiancy from B is shifted from the wavelength
H corresponding to maximum spectral radiancy in the
Fig. 16.12 radiation from A by 1.0 µm. If the temperature of A is
l 1.0 5802 K, calculate
R= = = 6.23 K /W (a) the temperature of B,
KA (401) (4 × 10–4 )
Temperature difference (b) wavelength λ B.
∴ Heat current, H =
Thermal resistance
(100 – 0) Sol. (a) pA = pB ⇒ ∴ e A σAA TA4 = e B σABTB4
= = 16 W 1/ 4
6.23 e 
Heat transferred in 1 h, ∴ TB =  A  TA (as AA = AB )
 eB 
Q = Ht H = Q
 
 t  Substituting the values
1/ 4
0.01
= (16) (3600) = 57600 J TB =   (5802) = 1934 K
Now, let m mass of ice melts in 1 h, then  0.81
Q (b) According to Wien’s displacement law,
m= (Q = mL)
L λ A TA = λ B TB
57600 5802 
=
3.35 × 105
= 0.172 kg or 172 g ∴ λ B =   λ
 1934  A
or λ B = 3λA
X Example 16.8 A body cools in 10 min from 60°C Also, λ B – λA = 1 µm
to 40°C. What will be its temperature after next 1
or λ B –   λ B = 1 µm or λ B = 1.5 µm
10 min? The temperature of the surroundings is 10°C.  3
Chapter Summary with Formulae
■ Calorimetry
(i) Q = ms∆T = c∆T, when temperature changes without change in state.
(ii) Q = mL, when state changes without change in temperature.
■ Heat Transfer
Heat conduction through a rod
KA (T1 − T2 )
(i) Heat flow in steady state, Q = t
L
(ii) Rate of flow of heat = heat current
dQ TD
or H = =
dt R
Here, TD = temperature difference = T1 − T2
L
and R = thermal resistance =
KA
■ Radiation
Energy absorbed
(i) Absorptive power, a =
Energy incident
a≤1
a = 1for perfectly black body.
(ii) Spectral absorptive power ( aλ ) = absorptive power of wavelength (λ).
aλ ≤ 1
aλ = 1for perfectly black body.
(iii) Emissive power (e ) Energy radiated per unit surface area per unit time is called emissive power of a body. Its SI units are Js −1m −2
or Wm −2 .
(iv) Spectral emissive power (e λ ) Emissive power of wavelength λ is known as spectral emissive power.

e = ∫ eλ dλ
0

(v) Stefan’s law Emissive power of a body is given by


e = er σT4
Here, er = emissivity, emittance, relative emissivity or relative emittance.
er ≤ 1
er = 1 for a perfectly black body.
Sometimes, emissivity is also denoted by e. In that case differentiate them by their units. er is unitless while e has the units Wm −2 .
(vi) Total energy radiated by a body
E = er σT4 At
Here, A = surface area and t = time.
(vii) a = er or absorptivity of a body
= its emissivity.
(viii) Kirchhoff’s law If different bodies (including a perfectly black body) are kept at same temperatures, then
eλ e  e 
eλ ∝ aλ or = constant or  λ  =  λ 
aλ  aλ  Body 1  aλ  Body 2
e 
=  λ 
 aλ  Perfectly black body
= ( eλ )Perfectly black body
From this law, following two conclusions can be drawn.
(a) Good absorbers of a particular wavelength λ are also good emitters of same wavelength λ.
(b) At a given temperature, ratio of eλ and aλ for any body is constant. This ratio is equal to eλ of perfectly black body at that
temperature.
Additional Examples
Example 1. Thick bottomed drinking glasses Sol. White clothes absorb very little heat radiation and hence,
frequently crack, if hot water is poured into them. they are comfortable in summer. Coloured clothes absorb
Why? almost whole of the incident radiation and keep the body warm
in winter.
Sol. Glass is a bad conductor of heat. It does not pass down the
heat quickly to the lower surface. Different layers of the bottom Example 8. What is the effect of pressure on melting
are at different temperatures and expand differently. This point of a solid?
causes breaking of the glass at the bottom.
Sol. The melting point of a solid may increase or decrease
Example 2. What kind of thermal conductivity and depending on the nature of solid. For solids such as ice which
specific heat requirements would you specify for contracts on melting, it is lowered while for solids such as
cooking utensils? sulphur and wax which expand on melting it increases.
Sol. A cooking utensil should have Example 9. Water in a closed tube is heated with
(i) high conductivity so that it can conduct heat through one arm placed vertically above an arc lamp. Water
itself and transfer it to the contents quickly.
will begin to circulate along the tube in a
(ii) low specific heat so that it immediately attains the counterclockwise direction. Is this true or false?
temperature of the source.

Example 3. Why do the metal utensils have wooden


handles?
Sol. Wood is a bad conductor of heat. Wooden handle does not
allow heat to be conducted from the hot utensil to the hand. So,
we can easily hold the hot utensil with its help.

Example 4. Can we boil water inside an earth


satellite? Sol. False. Water will circulate in the clockwise direction. The
Sol. No. The process of transfer of heat by convection is based molecules immediately above the arc receive heat by
on the fact that a liquid becomes lighter on becoming hot and conduction. They become light and rise up and get replaced by
rises up. In condition of weightlessness, this is not possible. So, cold molecules from the right side. This will make the water to
transfer of heat by convection is not possible in a satellite. circulate in clockwise direction.

Example 5. Why do animals curl into a spherical Example 10. 5 g of water at 30°C and 5 g of ice at
shape, when they feel very cold? − 20°C are mixed together in a calorimeter. Find the
Sol. The total energy radiated by a body depends on its surface
final temperature of mixture. Water equivalent of
area. Thus, when the animals feel very cold, they curl their calorimeter is negligible, specific heat of
bodies into a spherical shape so as to decrease the surface area ice = 0.5 cal / g ° C and latent heat of ice = 80 cal/g.
of their bodies which in turn helps to reduce the amount of Sol. In this case, heat is given by water and taken by ice.
heat lost by them. Heat available with water to cool from 30° C to 0° C
= ms∆θ = 5 × 1 × 30 = 150 cal
Example 6. Why are clear nights colder than cloudy
Heat required by 5 g ice to increase its temperature up
nights? to 0°C
Sol. Clouds are opaque to heat radiations. So on a cloudy night, ms∆θ = 5 × 0.5 × 20 = 50 cal
radiations from the earth's surface fail to escape. But on a clear Out of 150 cal heat available, 50 cal is used for
night, the surface of the earth is cooled due to excessive increasing temperature of ice from –20° C to 0° C. The
radiation. So, a clear night is colder than a cloudy night. remaining heat 100 cal is used for melting the ice.
If mass of ice melted is m g, then
Example 7. White clothes are more comfortable in m × 80 = 100 ⇒ m = 1.25 g
summer while colourful clothes are more comfortable Thus, 1.25 g ice out of 5 g melts and mixture of ice
in winter. Why? and water is at 0° C.
706 Objective Physics Vol. 1

Example 11. A bullet of mass 10 g moving with a Sol. Heat spent during vaporisation,
speed of 20 m/s hits an ice block of mass 990 g kept on Q = mL = 1.000 × 539 = 539 cal
a frictionless floor and gets stuck in it. How much ice Work done, W = p (V v − Vl )
will melt if 50% of the lost kinetic energy goes to ice ? = 1.013 × 10 5 × (1671 − 1.000) × 10 −6
(Temperature of ice block = 0°C). 169.2
169.2 J = cal = 40.5 cal
Sol. Velocity of bullet + ice block 4.18
(10 g) × (20 m/s) ∴ Change in internal energy, U = 539 cal − 40.5 cal
v= = 0.2 m/s = 498.5 cal
1000 g
1 1 Example 14. At 1 atm pressure, 1 g of water
Loss of KE = mv 2 − (m + M ) v 2 having a volume of 1 cm 3 becomes 1.091 cm 3 of ice
2 2
1 on freezing. The heat of fusion of water at 1 atm is
= [0.01 × (20) 2 − 1 × (0.2) 2 ] 80.0 cal/g. What is the change in internal energy
2
1 during the process?
= [4 − 0.04] = 1.98 J
2 Sol. Heat given out during freezing,
1.98 Q = − mL = − 1 × 80 = − 80 cal
∴ Heat received by ice block = cal = 0.24 cal
4.2 × 2 External work done,
∴ Mass of ice melted =
(0.24 cal)
= 0.003 g W = p (V ice − V water ) = 1.013 × 10 5 × (1.091 − 1.000) × 10 −6
(80 cal / g) 9.22 × 10 −3
= 9.22 × 10 −3 J = cal = 0.0022 cal
Example 12. The temperature of equal masses of 4.18
three different liquids A, B and C are 12°C ,19°C and ∴ Change in internal energy ∆U = Q − W
28°C, respectively. The temperature when A and B are = − 80 − 0.0022 = − 80.0022 cal
mixed is 16°C and when B and C are mixed it is 23°C. Example 15. A cylinder of radius R made of a
What should be the temperature when A and C are material of thermal conductivity K 1 is surrounded by
mixed? cylindrical shell of inner radius R and outer radius 2R
Sol. Let m be the mass of each liquid and S A , S B , S C specific made of a material of thermal conductivity K 2 . The
two ends of the combined system are maintained at two
heats of liquids A, B and C respectively. When A and B are
different temperatures. There is no loss of heat across
mixed. The final temperature is 16° C.
the cylindrical surface and system is in steady state.
∴ Heat gained by A = heat lost by B
i.e. m S A (16 − 12) = m S B (19 − 16 )
What is the effective thermal conductivity of the
4 system?
i.e. SB = S A …(i) K2
3
Heat flow 2R
When B and C are mixed. Heat gained by B = heat lost R
K1
by C
i.e. m S B (23 − 19) = m S C (28 − 23)
4 θ1 θ2
i.e. SC = SB …(ii)
5
Sol. In this situation, a rod of length L and area of cross-section
From Eqs. (i) and (ii), we get
4 4 16 πR 2 and another rod of same length L and area of
SC = × S A = SA cross-section π [(2 R ) 2 − R 2 ] = 3 πR 2 will conduct heat
5 3 15
simultaneously so total heat flowing per second will be
When A and C are mixed, let the final temperature beθ dQ dQ1 dQ 2
∴ m S A (θ − 12) = m S C (28 − θ) = +
dt dt dt
16
i.e. θ − 12 = (28 − θ) K 1 πR 2 (θ1 − θ 2 ) K 2 3πR 2 (θ1 − θ 2 )
15 = + …(i)
628 L L
By solving, we get θ= = 20.26 ° C Now, if the equivalent conductivity is K. Then,
31
dQ 4 πR 2 (θ1 − θ 2 )
Example 13. At 1 atm pressure, 1 g of water =K [as, A = π(2R ) 2 ] …(ii)
dt L
having a volume of 1.000 cm 3 becomes 1671 cm 3 of So from Eqs. (i) and (ii), we have
steam when boiled. The heat of vaporisation of water 4 K = K 1 + 3K 2
at 1 atm is 539 cal/g. What is the change in internal (K 1 + 3K 2 )
i.e. K=
energy during the process? 4
NCERT Selected Questions
Q 1. A geyser heats water flowing at the rate of 3.0 L per If Q be the energy used up in drilling the bore, then
min from 27°C to 77°C. If the geyser operates on a Q = P × t = 104 × 150
gas burner, what is the rate of consumption of fuel, = 15 × 105 J
if its heat of consumption is 4.0 × 104 J/g? It is given that 50% of energy is lost to the surroundings.
Sol. Volume of water heated, ∴ If Q′ = Energy transferred to the aluminium block, then
V = 3.0 L min −1 = 3 × 10−3 m 3 min −1 Q′ = 50% of Q =
50
× 15 × 105
100
∴ Mass of water heated,
= 75 × 104 J
m = ρV = 103 × 3 × 10−3 kg min −1
Let ∆θ be the rise in temperature of the block,
= 3 kg min −1 Q′
∴ Q′ = mc∆θ or ∆θ =
Rise in temperature of water, mc
∆θ = θ 2 − θ 1 = 77 − 27 = 50° C 75 × 104
= = 103° C
Specific heat of water, c = 4.2 Jg−1C−1 8 × 0.91 × 103
= 4.2 × 103 J kg−1C−1
Q 3. A copper block of mass 2.5 kg is heated in a furnace
Heat required by water = ∆Q = mc∆θ to a temperature of 500°C and then placed on a large
= 3 × 4.2 × 103 × 50 J min ice block. What in the maximum amount of ice that
= 63 × 104 J min −1 ...(i) can melt? (Specific heat of copper = 0.39 Jg −1 K −1 ;
Heat of combustion of fuel heat of fusion of water = 335 Jg −1 ).
= 4.0 × 104 Jg−1
Sol. Specific heat of copper, c = 0.39 Jg−1K−1
−1
= 4.0 × 10 J kg
7
= 0.39 × 103 J kg−1K−1
−1
Let m kg min be the rate of combustion of fuel. Temperature of furnace, ∆θ = 500°C
∴ Heat supplied by gas burner or combustion of fuel Latent heat of fusion, L = 335 Jg−1
= m × 4 × 107 J/min ...(ii) = 335 × 103 J kg−1
∴ From Eqs. (i) and (ii), we get If Q be the heat absorbed by the copper block, then
63 × 104 = m × 4 × 107 Q = m1c∆θ ...(i)
63 × 104 Let m2 be the mass of ice melted, when copper block is
or m= placed on it, then
4 × 107
∴ Q = m2L ...(ii)
= 15.75 × 10−3 kg min −1
∴ From Eqs. (i) and (ii), we get
or rate of combustion of fuel ≈ 16 g min −1 m1c∆θ = m2L
m1c∆θ 2.5 × 0.39 × 103 × 500
Q 2. A 10 kW drilling machine is used to drill a bore in a or m2 = =
L 335 × 103
small aluminium block of mass 8.0 kg. How much is
the rise in temperature of the block in 2.5 min, = 1.455 kg
assuming 50% of the power is used up in heating the ≈ 1.5 kg
machine itself or lost to the surroundings. Specific Q 4. A thermocole icebox is a cheap and efficient
heat of aluminium = 0.91 Jg −1 K −1 . method for storing small quantities of cooked food
in summer in particular. A cubical icebox of side
Sol. Power of the machine
30 cm has a thickness of 5.0 cm. If 4.0 kg of ice is
P = 10 kW = 104 W put in the box, estimate the amount of ice remaining
Time for which the machine is used, after 6 h. The outside temperature is 45°C and
t = 2.5 min coefficient of thermal conductivity of thermocole is
= 2.5 × 60 s 0.01 Js −1 M −1 ° K −1 .
= 150 s [Heat of fusion of water = 335 × 10 3 J kg −1 ]
Specific heat of aluminium, c = 0.91 Jg−1K−1 Sol. Area of 6 faces of cube = 6 × 30 × 30 cm 2
−1 −1
= 0.91 × 10 J kg
3
K = 6 × 900 × 10−4 m 2
708 Objective Physics Vol. 1

Distance, d = 5.0 m = 5.0 × 10−2 m Q 6. Explain why?


Total mass of ice, M = 4 kg (a) A body with large reflectivity is a poor emitter.
Time, t = 6 h = 6 × 60 × 60 s (b) A brass tumbler feels much colder than wooden
θ 1 = temperature outside the box = 45° C tray on a chilly day.
θ 2 = temperature inside the box = 0° C (c) The earth without its atmosphere would be
∴ ∆θ = θ 1 − θ 2 = 45 − 0 = 45° C inhospitably cold.
Heat of fusion, L = 335 × 103 J kg−1 (d) Heating system based on circulation of steam are
Coefficient of thermal conductivity of thermocole,
more efficient in warming a building than those
based on circulation of hot water.
K = 0.01 Js−1m −1 K−1
Let m = mass (kg) of ice which melts. Sol. (a) According to Kirchhoff’s law e ∝ a that is good
∴ Heat needed to melt at 0°C, Q = mL ...(i) absorbers are good emitters and hence poor reflectors or
∆θ vice-versa.
Also Q = KA t ...(ii)
d (b) The thermal conductivity of brass is high, i.e. brass is a good
∴ From Eqs. (i) and (ii), we get conductor of heat. So when a brass tumbler is touched, heat
KA ∆θ quickly flows from human body to the tumbler.
m= t Consequently, the tumbler appears colder.
L d
0.01 × 6 × 900 × 10−4 × 45 On the other hand, wood is a bad conductor of heat. So heat
= × 6 × 3600 does not flow from the human body to the wooden tray, thus
335 × 103 × 5.0 × 10−2
it appears relatively hotter.
= 0.313 kg
(c) Gases are generally insulators. The earth’s atmosphere acts
∴ Mass of ice left in the box = M − m
like an insulating blanket around it and does not allow heat
= 4.0 − 0.313 to escape out but reflects it back to the earth. Had this
= 3.687 kg ≈ 3.7 kg atmosphere been absent, the earth would naturally be colder,
as all its heat would have escaped out.
Q 5. A brass boiler has a base area 0.15 m 2 and thickness
1.0 cm. It boils water at the rate of 6.0 kg/min, when (d) This is because steam at 100°C has more heat content than
placed on a gas stove. Estimate the temperature of water at 100°C. (Latent heat = 540 cal/g)
the part of the flame in contact with the boiler. Q 7. A body cools from 80°C to 50°C in 5 min. Calculate
Thermal conductivity of brass = 109 Js −1 m −1 K −1 . the time it takes to cool from 60°C to 30°C. The
(Heat of vaporisation of water = 2256 × 10 3 J kg −1 ) temperature of the surroundings is 20°C.
Sol. Let θ 1 = temperature of the part of the boiler in contact with Sol. According to Newton’s law of cooling, the rate of cooling ∝
the stove. If Q be the amount of heat flowing per second difference in temperature.
through the base of the boiler, then Here, average of 80°C and 50°C = 65°C
KA (θ 1 − θ 2 ) Temperature of surroundings = 20°C
Q=
d ∴ Difference = 65° − 20° = 45°C
109 × 0.15 × (θ 1 − 100)
or Q= Under these conditions, the body cools 30°C in time 5 min.
10−2
Change in temperature
= 1635 (θ 1 − 100) Js−1 ...(i) ∴ = k∆T
Time
Rate of boiling of water in the boiler, 30
or = k × 45 ...(i)
M = 6.0 kg min −1 5
6.0
= = 0.1 kg - s−1 The average of 60°C and 30°C is 45°C which is 25°C above
60 the room temperature and the body cools by 30°C in a time
∴ Heat received by water per second, Q = ML t (say).
or Q = 0.1 × 2256 × 103 Js−1 ...(ii) ∴
30
= k × 25 ...(ii)
t
∴ From Eqs. (i) and (ii), we get
where, k is same for this situation as for the original.
1635 (θ 1 − 100) = 2256 × 10−2
Eq. (i) divided by Eq. (ii) gives
2256 × 100 30 / 5 k × 45
or θ 1 − 100 = = 138 =
1635 30 / t k × 25
θ 1 = 100 + 138 t 9
or = or t = 9 min
= 238° C 5 5
Objective Problems
[ Level 1 ]
Temperature Change and Phase 6. A block of ice of mass M = 10kg is moved back and forth
Change over the flat horizontal surface of a large block of ice.
Both blocks are at 0°C and the force that produces the
1. 50 g of ice at 0°C is mixed with 50 g of water at 60°C, back and forth motion acts only horizontally. The
final temperature of mixture will be coefficient of friction between the two surfaces is 0.060.
(a) 0°C (b) 40°C If m = 15.2 g of water is produced, the total distance
(c) 10°C (d) 15°C travelled by the upper block relative to the lower is
(Lice = 3.34 × 105 J/kg)
2. A block of ice at −10° Cis slowly heated and converted to
(a) 432 m (b) 863 m
steam at 100°C. Which of the following curves represent
(c) 368 m (d) 216 m
the phenomenon qualitatively?
7. 4 kg of ice at −15°C are added to 5 kg of water at 15°C.
Temperature

Temperature

The temperature of the resulting mixture equals


(a) −15°C (b) 0°C
(a) (b)
(c) 5°C (d) 15°C

8. How much heat energy is gained when 5 kg of water at


Heat supplied Heat supplied 20°C is brought to its boiling point (specific heat of water
= 4.2 kJ kg −1 ° C−1 )
Temperature

Temperature

(a) 1680 kJ (b) 1700 kJ


(c) (d) (c) 1720 kJ (d) 1740 kJ

9. 80 g of water at 30°C are poured on a large block of ice at


0°C. The mass of ice that melts is
Heat supplied Heat supplied
(a) 30 g (b) 80 g
(c) 1600 g (d) 150 g
3. An iron ball of mass 0.2 kg is heated to 100°C and put
into a block of ice at 0°C. 25 g of ice melts. If the latent 10. 50 g of copper is heated to increase its temperature by
heat of fusion of ice is 80 cal/g, then the specific heat of 10°C. If the same quantity of heat is given to 10 g of
iron in cal/g°C is water, the rise in its temperature is (specific heat of
(a) 1 (b) 0.1 copper = 420 J -kg −1 ° C−1 )
(c) 0.8 (d) 0.08 (a) 5°C (b) 6°C
4. A metal block is made from a mixture of 2.4 kg of (c) 7°C (d) 8°C
aluminium 1.6 kg of brass and 0.8 kg of copper. The 11. A beaker contains 200 g of water. The heat capacity of
amount of heat required to raise the temperature of this the beaker is equal to that of 20 g of water. The initial
block from 20° C to 80°C is (specific heats of aluminium, temperature of water in the beaker is 20°C. If 440 g of hot
brass and copper are 0.216, 0.0917 and 0.0931 cal/kg°C water at 92°C is poured in it, the final temperature
respectively) (neglecting radiation loss) will be nearest to
(a) 96.2 cal (b) 44.4 cal (a) 58°C (b) 68°C
(c) 86.2 cal (d) 62.8 cal (c) 73°C (d) 78°C
5. Steam at 100°C is passed into 1.1 kg of water contained 12. A liquid of mass m and specific heat c is heated to a
in a calorimeter of water equivalent 0.02 kg at 15°C till m
temperature 2T. Another liquid of mass and specific
the temperature of calorimeter and its contents rises to 2
80°C. The mass of the steam condensed in kilogram is heat 2c is heated to a temperature T. If these two liquids
(sw = 103 cal /kg-°C and LV = 540 × 103 cal/kg) are mixed, the resulting temperature of the mixture is
(a) 0.130 (b) 0.065 2 8 3 3
(a) T (b) T (c) T (d) T
(c) 0.260 (d) 0.135 3 5 5 2
710 Objective Physics Vol. 1

13. 100 g of ice at 0°C is mixed with 100 g of water at 100°C. 19. The portion AB of the indicator diagram representing the
What will be the final temperature of the mixture? state of matter denotes
(a) 10°C (b) 20°C p
(c) 30°C (d) 0°C A

14. A lead bullet of 10 g travelling at 300 m/s strikes against C


a block of wood and comes to rest. Assuming 50% of heat B
is absorbed by the bullet, the increase in its temperature is
(a) 100°C (b) 125°C D
(c) 150°C (d) 200°C V
15. 2 kg of ice at −20°C is mixed with 5 kg of water at 20°C in (a) The liquid state of matter
an insulating vessel having a negligible heat capacity. (b) Gaseous state of matter
Calculate the final mass of water remaining in the (c) Change from liquid to gaseous state
container. It is given that the specific heats of water and (d) Change from gaseous state to liquid state
ice are 1 kcal/kg per °C and 0.6 kcal/kg per °C while the 20. Two substances A and B of equal mass m are heated at
latent heat of fusion of ice is 80 kcal/kg
uniform rate of 6 cal s −1 under similar conditions. A
(a) 7 kg (b) 6 kg
(c) 4 kg (d) 2 kg
graph between temperature and time is shown in figure.
HA
Ratio of heat absorbed by them for complete
16. The temperature of equal masses of three different liquids HB
A, B and C are 12°C, 19°C and 28°C respectively. The
fusion is
temperature when A and B are mixed is 16°C and when B
Temperature ºC
and C are mixed is 23°C. The temperature when A and C 100
are mixed is 80 A
(a) 18.2°C (b) 22°C 60
40 B
(c) 20.2°C (d) 25.2°C
20
Time
17. In an industrial process 10 kg of water per hour is to be 0 1 2 3 4 5 6 7
heated from 20°C to 80°C. To do this steam at 150°C is
9 4
passed from a boiler into a copper coil immersed in (a) (b)
water. The steam condenses in the coil and is returned to 4 9
8 5
the boiler as water at 90°C, how many kg of steam is (c) (d)
5 8
required per hour.
(Specific heat of steam = 1 cal per g°C, Latent heat of Heat Conduction
vaporisation = 540 cal/g)
21. The dimensional formula for thermal resistance is
(a) 1g
(a) [M −1L−2T 3θ] (b) [ML2Tθ]
(b) 1 kg
(c) 10 g (c) [M −1L2T 3θ] (d) None of these
(d) 10 kg
22. A wall has two layers A and B, each made of different
18. Which of the substances A, B and C has the lowest heat material. Both the layers have the same thickness. The
capacity, if heat is supplied to all of them at equal rates? thermal conductivity for A is twice that of B and, under
The temperature versus time graph is shown below: steady condition, the temperature difference across the
wall is 36°C. The temperature difference across the
layer A is
Temperature (T)

A
B (a) 6°C (b) 12°C
C (c) 24°C (d) 18°C

23. In a steady state, the temperatures at the end A and B of


20 cm long rod AB are 100°C and 0°C. The temperature of
Time (t) a point 9 cm from A is
(a) 45°C (b) 55°C
(a) A
(c) 5°C (d) 65°C
(b) B
(c) C 24. S I unit of thermal conductivity is
(d) All have equal specific heat (a) J/s-K (b) J/s-m 2K (c) J/mK (d) J/s-mK
Calorimetry and Heat Transfer 711

25. A wall has two layers A and B each made of different 30. Three rods made of the same material and having same
materials. The layer A is 10 cm thick and B is 20 cm cross-sectional area but different length 10 cm, 20 cm and
thick. The thermal conductivity of A is thrice that of B. 30 cm are joined as shown. The temperature of the
Under thermal equilibrium temperature difference junction is
across the wall is 35°C. The difference of temperature 20ºC
across the layer A is
(a) 20°C (b) 10°C (c) 15°C (d) 5°C 20 cm

26. In the figure, ABC is a conducting rod whose lateral


surfaces are insulated. The length of the section AB is
one-half of that of BC, and the respective thermal 10 cm
30 cm
conductivities of the two sections are as given in the 30ºC
figure. If the ends A and C are maintained at 0°C and 10ºC
70°C respectively, the temperature of junction B in the (a) 10.8°C (b) 14.6°C
steady state is (c) 16.4°C (d) 18.2°C
B
31. Two identical square rods of metal are welded end to end
2K as shown in Fig. (i), 20 cal of heat flows through it in
3K
4 min. If the rods are welded as shown in Fig. (ii), the
A same amount of heat will flow through the rods is
C
0ºC 100ºC
(a) 30°C (b) 40°C (c) 50°C (d) 60°C
0ºC 100ºC
27. Equal temperature differences exist between the ends of (i) (ii)
two metallic rods 1 and 2 of equal lengths. Their thermal (a) 1 min (b) 2 min
conductivities are K 1 and K 2 and area of cross-section (c) 4 min (d) 16 min
are A1 and A 2 , respectively. The condition of equal rates
of heat transfer is 32. The temperature gradient in a rod of 0.5 m long is
(a) K 1 A2 = K 2 A1 (b) K 1 A1 = K 2 A2 80°C/m. If the temperature of hotter end of the rod is
(c) K 1 A12 = K 2 A22 (d) K 12 A2 = K 22 A1 30°C, then the temperature of the cooler end is
(a) 40°C (b) −10°C
28. Two rods of copper and brass ( K C > K B ) of same length (c) 10°C (d) 0°C
and area of cross-section are joined as shown. End A is 33. On heating one end of a rod, the temperature of whole rod
kept at 100°C and end B at 0°C. The temperature at the will be uniform when
junction (a) K = 1 (b) K = 0
A Copper Brass B (c) K = 100 (d) K = ∞
(a) will be more than 50°C 34. Two rods of same length and material transfer a given
(b) will be less than 50°C amount of heat in 12 s, when they are joined end to end.
(c) will be 50°C But when they are joined lengthwise, then they will
(d) may be more or less than 50°C depending upon the size of transfer same heat in same condition in
rods
(a) 24 s (b) 3 s
29. Three rods of same dimensions have thermal (c) 1.5 s (d) 48 s
conductivities 3K , 2K and K. They are arranged as 35. Wires A and B have identical length and have circular
shown, with their ends at 100°C, 50°C and 0°C. The cross-sections. The radius of A is twice the radius of B,
temperature of their junction is i.e. rA = 2rB . For a given temperature difference between
50ºC the two ends, both wires conduct heat at the same rate.
2K The relation between the thermal conductivities is given
by
3K
(a) K A = 4 K B
100ºC
(b) K A = 2K B
K K
0ºC (c) K A = B
2
200 100 KB
(a) 75°C (b) °C (c) 40°C (d) °C (d) K A =
3 3 4
712 Objective Physics Vol. 1

36. The ends of two rods of different materials with their 40. In the figure, the distribution of energy density of the
thermal conductivities, radii of cross-section and lengths radiation emitted by a black body at a given temperature
all in the ratio 1 : 2 are maintained at the same is shown. The possible temperature of the black body is
temperature difference. If the rate of flow of heat in the
E
larger rod is 4 cal / s, then that in the shorter rod in cal/s
will be
(a) 1 (b) 2 (c) 8 (d) 16

37. A slab consists of two parallel layers of two different


materials of same thickness having thermal 0 1.0 1.5 2.0 2.5 λ in µm
conductivities K 1 and K 2 . The equivalent conductivity of
(a) 1500 K (b) 2000 K
the combination is (c) 2500 K (d) 3000 K
(a) K 1 + K 2
K + K2 41. Two similar rods are joined as shown in figure. Then,
(b) 1
2 temperature of junction is (assume no heat loss through
2K 1K 2 lateral surface of rod and temperatures at the ends are
(c)
K1 + K2 shown in steady state)
K + K2 50ºC
(d) 1
2K 1K 2

38. Three rods of same dimensions are arranged as shown in


figure. They have thermal conductivities K 1 , K 2 and K 3 .
The points P and Q are maintained at different 100ºC 0ºC
temperatures. For the heat flow at the same rate along
PRQ and PQ which of the following option is correct?
R

50ºC
(a) 50°C (b) 75°C (c) 66.6°C (d) 33.3°C
K1 K2

Heat Convection and Radiation


P Q
42. If a black body radiates 10 cal/s at 227°C, it will radiate at
K3
727°C
1 (a) 10 cal/s (b) 80 cal/s
(a) K 3 = (K 1 + K 2 ) (b) K 3 = K 1 + K 2 (c) 160 cal/s (d) None of these
2
K 1K 2 43. Two spherical black bodies of radii r1 and r2 and with
(c) K 3 = (d) K 3 = 2(K 1 + K 2 )
K1 + K2 surface temperatures T1 and T2 , respectively radiate the
r
39. Five rods of same dimensions are arranged as shown in same power. Then, 1 must be equal to
figure. They have thermal conductivities r2
2 2 4 4
K 1 , K 2 , K 3 , K 4 and K 5 . When points A and B are T  T  T  T 
(a)  1  (b)  2  (c)  1  (d)  2 
maintained at difference temperatures, no heat flows  T2   T1   T2   T1 
through the central rod, if
C 44. The temperature of body is increased from 27°C to
127°C. The radiation emitted by it increases by a factor of
K1 K2 256 15 4 12
(a) (b) (c) (d)
81 9 5 27
A B
45. A sphere has a surface area of 1.0 m 2 and a temperature
of 400 K and the power radiated from it is 150 W.
K3 K4
Assuming, the sphere is black body radiator, the power in
D
kilowatt radiated when the area expands to 2.0 m 2 and
the temperature changes to 800 K.
(a) K 1 = K 4 and K 2 = K 3 (b) K 1K 4 = K 2K 3 (a) 6.2 (b) 9.6
(c) K 1K 2 = K 3K 4 (d) None of these (c) 4.8 (d) 16
Calorimetry and Heat Transfer 713

46. Two spheres of the same material have radii 1 m and 4 m, 53. For proper ventilation of building, windows must be open
temperatures 4000 K and 2000 K, respectively. Then, the near the bottom and top of the walls so as to let pass
ratio of energy radiated per second by the first sphere as (a) more air
compared to that by the second is (b) cool air near the bottom and hot air near the roof
(a) 4 : 1 (b) 2 : 1 (c) cool air near the roof and hot air near the bottom
(c) 1 : 1 (d) 1 : 4 (d) out hot air nears the roof

47. A spherical black body with radius 12 cm radiates 640 W 54. The layers of atmosphere are heated through
power at 500 K. If the radius is halved and the (a) convection (b) conduction
temperature doubled, the power radiated in watts (c) radiation (d) None of these
would be
55. In which of the following process, convection does not
(a) 5120 W (b) 640 W
(c) 2560 W (d) 1280 W
take place primarily?
(a) sea and land breeze
48. The ratio of the emissive power to the absorptive power (b) boiling of water
of all substances for a particular wavelength is the same (c) warming of glass of bulb due to filament
at given temperature. The ratio is known as (d) heating air around a furnace
(a) the emissive power of a perfectly black body
(b) the emissive power of any type of body 56. If e λ and a λ be the emissive power of a body, then
(c) the Stefan’s constant according to Kirchhoff’s law, which is true
(d) the Wien’s constant (E λ = emissive power of perfectly black body)
49. If wavelength of maximum intensity of radiation emitted (a) eλ = aλ = Eλ (b) eλ Eλ = aλ
(c) eλ = aλ Eλ (d) eλ aλ Eλ = constant
by the sun and the moon are 0.5 × 10−6 m and 10−4 m
respectively, the ratio of their temperatures is 57. Distribution of energy in the spectrum of a black body
(a) 2000 (b) 1000 can be correctly represented by
(c) 100 (d) 200 (a) Wien’s law (b) Stefan’s law
(c) Planck’s law (d) Kirchhoff’s law
50. The spectrum of a black body at two temperatures 27°C
and 327°C is shown in the figure. Let A1 and A 2 be the 58. There is a black spot on a body. If the body is heated and
areas under the two curves, respectively. The value of carried in dark room, then it glows more. This can be
A2 explained on the basis of
is
A1 (a) Newton’s law of cooling (b) Wien’s law
(c) Kirchhoff’s law (d) Stefan’s law
Intensity
59. The maximum wavelength of radiations emitted at 900 K
2 is 4 µm. What will be the maximum wavelength of
radiations emitted at 1200 K?
327ºC
1 (a) 3 µm (b) 0.3 µm
27ºC (c) 1 µm (d) None of these
Wavelength
60. The maximum energy in thermal radiation from a source
(a) 1 : 16 (b) 4 : 1 (c) 2 : 1 (d) 16 : 1 occurs at wavelength 4000 Å. The effective temperature
of the source is approximately
51. A black body radiates power P and maximum energy is
(a) 7225 K (b) 80000 K
radiated by it around a wavelength λ 0 . The temperature
(c) 104 K (d) 106 K
of the black body is now changed such that it radiates
3λ 0 61. The intensity of radiation emitted by the sun has its
maximum energy around the wavelength . The
4 maximum value at a wavelength of 510 nm and that
power radiated by its row is emitted by the North star has the maximum value at
256 27 64 81 350 nm. If these stars behave like black bodies, then the
(a) P (b) P (c) P (d) P
81 64 27 256 ratio of the surface temperature of the sun and North
star is
52. If a body coated black at 600 K surrounded by
(a) 1.46 (b) 0.69 (c) 1.21 (d) 0.83
atmosphere at 300 K has cooling rate r0 , the same body at
−4
900 K, surrounded by the same atmosphere, will have 62. The area of the hole of heat furnace is 10 m 2 . It radiates
cooling rate equal to 1.58 × 105 cal of heat per hour. If the emissivity of the
16 8 furnace is 0.80, then its temperature is
(a) r0 (b) r0 (c) 16r0 (d) 4 r0
3 16 (a) 1500 K (b) 2000 K (c) 2500 K (d) 3000 K
714 Objective Physics Vol. 1

63. A spherical black body with a radius of 12 cm radiates 71. Snow is more heat insulating than ice, because
440 W power at 500 K. If the radius were halved and the (a) air is filled in porous of snow
temperature doubled, the power radiated in watt would be (b) ice is more bad conductor than snow
(a) 440 (b) 1320 (c) 880 (d) 1760 (c) air is filled in porous of ice
(d) density of ice is more
Miscellaneous Problems 72. Two thin blankets keep more hotness than one blanket of
64. The calories of heat developed in 200 W heater in 7 min thickness equal to these two. The reason is
is estimated (a) their surface area increases
(a) 15000 (b) 100 (b) a layer of air is formed between these two blankets which
(c) 1000 (d) 20000 is bad conductor
(c) these have more wool
65. A sphere, a cube and a thin circular plate are heated to the (d) they absrob more heat from outside
same temperature. All are made of the same material and
have the equal masses. If t 1 , t 2 and t 3 are the respective 73. Ice formed over lakes
time taken by the sphere, cube and the circular plate in (a) has very high thermal conductivity and helps in further ice
cooling down to a common temperature, then formation
(a) t1 > t2 > t3 (b) t1 < t2 < t3 (b) has very low conductivity and retards further formation of
ice
(c) t2 > t1 > t3 (d) t1 = t2 = t3
(c) permits quick convection and retards further formation of
66. The sun rays are allowed to fall on a lens of diameter ice
20 cm. They are then brought to focus on a calorimeter (d) is very good radiator
containing 20 g of ice. If the absorption by the lens is 74. Mud houses are cooler in summer and warmer in winter,
negligible, the time required to melt all the ice is because
(solar constant = 1.9 cal/min/cm 2 and L = 80 cal/g) (a) mud is superconductor of heat
(b) mud is good conductor of heat
(a) 6.4 min (b) 3.2 min
(c) mud is bad conductor of heat
(c) 7.2 min (d) 2.7 min
(d) None of the above
67. If 1 kg water at 100°C is vaporised in open atmosphere.
75. One likes to sit under sunshine in winter seasons, because
The correct statement is
(a) the air surrounding the body is hot by which body gets heat
(a) increase in internal energy is equal to L (L is latent heat of
vaporisation for 1 kg) (b) we get energy by the sun
(b) increase in internal energy is zero (c) we get heat by conduction by the sun
(c) increase in internal energy is less than L (d) None of the above
(d) None of the above 76. Air is bad conductor of heat, still vacuum is preferred
68. Thermal capacity of a body depends on between the walls of the thermo flask because
(a) the heat given (a) it is difficult to fill the air between the walls of thermo flask
(b) the temperature raised (b) due to more pressure of air, the flask can crack
(c) the mass of the body (c) by convection, heat can flow through air
(d) None of the above (d) None of the above

69. The length of the two rods made up of the same metal and 77. Which of the following law states that good absorbers of
having the same area of cross-section are 0.6 m 2 and heat are good emitters?
(a) Stefan’s law (b) Kirchhoff’s law
0.8 m 2 , respectively. The temperature between the ends
(c) Planck’s law (d) Wien’s law
of first rod is 90°C and 60°C and that for the other rod is
150°C and 110°C. For which rod the rate of conduction 78. In MKS system, Stefan’s constant is denoted by σ. In
will be greater? CGS system, multiplying factor of σ will be
(a) First (b) Second (a) 1 (b) 103
(c) Same for both (d) None of these (c) 105 (d) 102
70. The thickness of a metallic plate is 0.4 cm. The 79. A body cools from 50°C to 40°C in 5 min. The
temperature between its two surfaces is 20°C. The surrounding temperature is 20°C. In what further time
quantity of heat flowing per second is 50 cal from 5 cm 2
(in minute) will it cool to 30°C?
area. In CGS system, the coefficient of thermal 15
conductivity will be (a) 5 (b)
2
(a) 0.4 (b) 0.6 25
(c) 0.2 (d) 0.5 (c) (d) 10
3
Calorimetry and Heat Transfer 715

80. If a body cools down from 80°C to 60°C in 10 min, when 82. A substance of mass m kg requires a power input of
the temperature of the surroundings is 30°C. Then, the P watt to remain in the molten state at its melting point.
temperature of the body after next 10 min will be When the power is turned off, the sample completely
(a) 50°C (b) 48°C
solidifies in time t s. What is the latent heat of fusion of
(c) 30°C (d) None of these
the substance?
81. A liquid cools from 50°C to 45°C in 5 min and from 45°C Pm Pt
(a) (b)
to 41.5°C in the next 5 min. The temperature of the t m
surroundings is m t
(c) (d)
(a) 27°C (b) 40.3°C (c) 23.3°C (d) 33.3°C Pt Pm

[ Level 2 ]
1. Three rods of identical area of cross-section and made maintained at 20°C, 90°C and 0°C respectively. The ratio
from the same metal from the sides of an isosceles of lengths BD and BC, if there is no heat flow in AB is
∆ ABC, right angled at B. The points A and B are A B C
maintained at temperatures T and 2 T , respectively. In
the steady state, the temperature of the point C is TC .
T
Assuming that only heat conduction takes place, C is
T D
equal to 2 7 9 2
1 3 (a) (b) (c) (d)
(a) (b) 7 2 2 9
( 2 + 1) ( 2 + 1)
1 1 5. Two rods with the same dimensions have thermal
(c) (d)
2(2 2 − 1) 3 ( 2 − 1) conductivities in the ratio 1 : 2. They are arranged
between heat reservoirs with the same temperature
2. Two identical rods are made of different materials whose difference, in two different configurations, A and B. The
thermal conductivities are K 1 and K 2 . rates of heat flow in A and B are I A and I B , respectively.
They are placed end to end between two heat reservoirs at I
temperatures θ1 and θ 2 . The temperature of the junction The ratio A is equal to
IB
of the rod is
K1 K2 K 2K
θ1 θ2 A T1 T2

K
θ + θ2 K θ + K 2θ 2
(a) 1 (b) 1 1 B T1 T2
2 K1 + K2
K 1θ 2 + K 2θ 1 K 1θ 1 + K 2θ 2 2K
(c) (d)
K1 + K2 | K1 − K2 | (a) 1 : 2 (b) 1 : 3
(c) 2 : 5 (d) 2 : 9
3. Water is being boiled in a flat bottomed kettle placed on a
stove. The area of the bottom is 300cm 2 and the 6. Two identical conducting rods AB and CD are connected
to a circular conducting ring at two diametrically
thickness is 2 mm. If the amount of steam produced is opposite points B and C. The radius of the ring is equal to
1 g min −1 , then the difference of the temperature between the length of rods AB and CD. The area of cross-section,
the inner and the outer surfaces of the bottom is (thermal and thermal conductivity of the rod and ring are equal.
conductivity of the material of the kettle = 0.5 cal. Points A and D are maintained at temperatures of 100°C
cm −1 ° C−1 s −1 and latent heat of the steam is equal to and 0°C. Temperature of point C will be
540 cal g −1 )
(a) 12°C (b) 1.2°C A B C D
(c) 0.2°C (d) 0.012°C 100ºC 0ºC

4. Three conducting rods of same material and cross-section


are shown in figure. Temperatures of A, D and C are (a) 62°C (b) 37°C (c) 28°C (d) 45°C
716 Objective Physics Vol. 1

7. The temperature change versus heat supplied curve is 11. A ring consisting of two parts ADB and ACB of same
given for 1 kg of a solid block. Then, which of the conductivity K carries an amount of heat H. The ADB
following statement(s) is/are correct? part is now replaced with another metal keeping the
Temperature temperatures T1 and T2 constant. The heat carried
(K) increases to 2H. What should be the conductivity of the
ACB
C
D new ADB part? (Given, = 3).
ADB
A B C
60º

45º
T1 A B T2
0
100 J 200 J D
Heat supplied (J)

(a) Specific heat capacity of the solid is 1 J/kg-K 7


(a) K (b) 2 K
(b) Specific heat capacity of liquid phase is 3 J/kg-K 3
(c) Latent heat of vaporisation is 100 J/kg 5
(c) K (d) 3 K
(d) Latent heat of vaporisation is 200 J/kg 2

8. The rate of flow of heat through 12 identical conductors 12. Two identical conducting rods are first connected
made of same material is as shown in the figure. Then, independently to two vessels, one containing water at
which of the following is correct? 100°C and the other containing ice at 0°C. In the second
2 J/s
case, the rods are joined end to end and connected to the
same vessels. Let q1 and q 2 g/s be the rate of melting of
q
A ice in two cases, respectively. The ratio of 1 is
1 J/s q2
1 J/s G 1 2
B (a) (b)
C 4 J/s 2 1
F 4 1
(c) (d)
2 J/s E 1 4
D
3 J/s
13. The graph, shown in the below diagram, represents the
(a) The rate of flow of heat through rod DE is 9 J/s variation of temperature (T ) of two bodies, x and y having
(b) Junctions C and F are at the same temperature same surface area, with time ( t ) due to the emission of
(c) Junction A and G are at the same temperature radiation. Find the correct relation between the emissivity
(d) The rate of flow of heat through CF is 5 J/s ( e ) and absorptivity ( a ) of two bodies
9. Rate of heat flow through a cylindrical rod is H1 . T
Temperatures of ends of rod are T1 and T2 . If all the y
dimensions of rod become double and temperature
difference remains same and rate of heat flow becomes x
H 2 . Then,
(a) H 2 = 2H 1 (b) H 2 = H 1
H t
(c) H 2 = 1 (d) H 2 = 4 H 1
4
(a) ex > ey and ax < ay
10. Equal masses of three liquids A, B and C have (b) ex < ey and ax > ay
temperatures 10°C, 25°C and 40°C, respectively. If A and (c) ex > ey and ax > ay
B are mixed, the mixture has a temperature of 15°C. If B (d) ex < ey and ax < ay
and C are mixed, the mixture has a temperature of 30°C. 14. A sphere, a cube and a thin circular plate, all of same
If A and C are mixed, the mixture will have a material and same mass are initially heated to same high
temperature of temperature.
(a) 16°C (a) Plate will cool fastest and cube the slowest
(b) 20°C (b) Sphere will cool fastest and cube the slowest
(c) 25°C (c) Plate will cool fastest and sphere the slowest
(d) 29°C (d) Cube will cool fastest and plate the slowest
Calorimetry and Heat Transfer 717

15. Three copper blocks of masses M 1 , M 2 and M 3 kg (c) When they are connected in parallel, the net rate of heat flow
is q1 + q2
respectively, are brought into thermal contact till they
(d) When they are connected in parallel, the net rate of heat flow
reach equilibrium. Before contact, they were at qq
is 1 2
T1 , T2 , T3 ( T1 > T2 > T3 ). Assuming there is no heat loss q1 + q2
to the surroundings, the equilibrium temperature T is (s is
specific heat of copper) 5. Choose the correct options.
T + T2 + T3 (a) Good absorbers of a particular wavelength are good emitters
(a) T = 1
3 of same wavelength. This statement was given by Kirchhoff
M 1T1 + M 2T2 + M 3T3 (b) At low temperature of a body, the rate of cooling is directly
(b) T =
M1 + M2 + M3 proportional to temperature of the body. This statement was
given by the Newton
M 1T1 + M 2T2 + M 3T3
(c) T = (c) Emissive power of a perfectly black body is 1
3 (M 1 + M 2 + M 3 )
(d) Absorptive power of a perfectly black body is 1
M 1T1s + M 2T2s + M 3T3s
(d) T =
M1 + M2 + M3 Assertion and Reason
Direction (Q. Nos. 1-17) These questions consist of two
More than One Correct Options
statements each printed as assertion and reason. While
1. A solid sphere and a hollow sphere of the same material answering these questions you are required to choose anyone
and of equal radii are heated to the same temperature of the following five responses.
(a) both will emit equal amount of radiation per unit time in the
(a) If both Assertion and Reason are correct and Reason is
beginning
(b) both will absorb equal amount of radiation per second from the correct explanation of Assertion.
the surroundings in the beginning (b) If both Assertion and Reason are true but Reason is
(c) the initial rate of cooling will be the same for both the not the correct explanation of Assertion.
spheres (c) If Assertion is true but Reason is false
(d) the two spheres will have equal temperatures at any instant
(d) If Assertion is false but Reason is true
a
2. Three identical conducting rods (e) If both Assertion and Reason are false
are connected as shown in figure.
b 1. Assertion A body that is a good radiator is also a good
Given that θ a = 40° C, θ c = 30° C
absorber of radiation at a given wavelength.
and θ d = 20° C. Choose the correct
options. Reason According to Kirchhoff’s law, the absorptivity
d c
of a body is equal to its emissivity at a given wavelength.
(a) Temperature of junction b is 15°C
(b) Temperature of junction b is 30°C 2. Assertion For higher temperature, the peak emission
(c) Heat will flow from c to b wavelength of a black body shifts to lower wavelengths.
(d) Heat will flow from b to d
Reason Peak emission wavelength of a blackbody is
3. Two liquids of specific heat ratio 1 : 2 are at temperatures proportional to the fourth power of temperature.
2θ and θ,
3. Assertion Temperatures near the sea coast are
(a) if equal amounts of them are mixed, then temperature of
mixture is 1.5 θ moderate.
(b) if equal amounts of them are mixed, then temperature of Reason Water has a high thermal conductivity.
4
mixture is θ 4. Assertion It is hotter over the top of a fire than at the
3
(c) for their equal amounts, the ratio of heat capacities is 1 : 1 same distance on the sides.
(d)z Reason Air surrounding the fire conducts more heat
for their equal amounts, the ratio of their heat capacities is 1 : 2 upwards.
4. Two conducting rods when connected between two 5. Assertion Blue star is at high temperature than red star.
points at constant but different temperatures separately,  1 
the rate of heat flow through them is q1 and q 2 . Reason Wien’s displacement law states that T ∝  
(a) When they are connected in series, the net rate of heat flow
λ m 
will be q1 + q2 6. Assertion Snow is better insulator than ice.
(b) When they are connected in series, the net rate of heat flow
qq Reason Snow contains air packet and air is good
is 1 2
q1 + q2 insulator of heat.
718 Objective Physics Vol. 1

7. Assertion All black coloured objects are considered 17. Assertion If a body is good absorber of green light,
black bodies. then it will be good reflector of red light.
Reason Black colour is a good absorber of heat. Reason At a given temperature, the ratio of emissive
power to absorptive power is same for all substances.
8. Assertion Water vapours at 100°C will burn you more
than water at 100°C. Match the Columns
Reason Heat required to convert total mass of any 1. Match the following columns.
substance from one state to another state is called latent
heat. Column I Column II
(A) Specific heat (p) [MLT −3 θ −1 ]
9. Assertion Heat required to convert 1 g ice at 0°C into
(B) Coefficient of thermal conductivity (q) [MT −3 θ −4 ]
vapour at 100°C is 720 cal.
(C) Boltzmann constant (r) [L 2 T −2 θ −1 ]
Reason Conversion of solid state directly into vapour
(D) Stefan’s constant (s) [ML 2 T −2 θ −1 ]
state is called vaporisation.
10. Assertion Good conductors of electricity are also good 2. Three liquids A, B and C having same specific heat and
conductors of heat. mass m, 2m and 3m have temperatures 20°C, 40°C and
Reason In good conductors of electricity, there are a 60°C, respectively. Temperature of the mixture when
large number of free electrons. Column I Column II

11. Assertion Gravity plays very important role in the (A) A and B are mixed (p) 35°C
process of natural convection. (B) A and C are mixed (q) 52°C

Reason Convection mainly takes place in liquids and (C) B and C are mixed (r) 50°C
gases. (D) A, B and C all three are mixed (s) 45°C
(t) None
12. Assertion Absorptive power of any substance is
temperature independent. But emissive power depends
3. Three rods of equal length of same material are joined to
on the temperature.
form an equilateral ∆ ABC as shown in figure. Area of
Reason Emissive power ∝ T 4 . cross-sectional of rod AB is S, of rod BC is 2S and that of
13. Assertion If temperature of a body is increased, more AC is S. Then, match the following columns,
B
number of photons of small wavelengths are radiated.
Reason By increasing the temperature, total energy
radiation will increase.
14. Assertion Emissive power of a perfectly black body is A C
one. 100ºC 0ºC

Reason Absorptive power of perfectly black body is one. Column I Column II


(A) Temperature of junction B (p) Greater than 50°C
15. Assertion A normal body cannot radiate energy more
(B) Heat current in AB (q) Less than 50°C
than a perfectly black body.
(C) Heat current in BC (r) Is equal to heat current in
Reason Perfectly black body is always black in colour. BC
(s) 2
16. Assertion Two conducting rods of same material and Is times heat current in
3
same lengths are joined end to end as shown in figure. AC
Heat current H is flowing through them as shown. (t) None
Temperature difference across rod-1 is more than the
temperature difference across rod-2. 4. Match the following columns.
H
Column I Column II
−3 −4
1 2 (A) Thermal resistance (p) [MT θ ]
(B) Stefan’s constant (q) [M −1 L −2 T 3 θ]
(C) Wien’s constant (r) [ML 2 T −3 ]
Reason Thermal resistance of rod-1 is less compared to
(D) Heat current (s) [ Lθ ]
rod-2.
Calorimetry and Heat Transfer 719

T T
Entrance Gallery
(a) (b) θ0
2014
1. Three rods of copper, brass and steel are welded together O t O t
to from a Y -shaped structure. Area of cross-section of
T T
each rod is 4 cm 2 . End of copper rod is maintained at
100°C whereas ends of brass and steel are kept at 0°C. θ0 θ0
(c) (d)
Lengths of the copper, brass and steel rods are 46 cm,
13 cm, and 12 cm respectively. The rods are thermally O t O t
insulated from surroundings except at ends. Thermal
conductivities of copper, brass and steel are 0.92, 0.26 6. The figure shows the temperatures at four faces of a
and 0.12 in CGS units, respectively. Rate of heat flow composite slab consisting of four materials S 1 , S 2 , S 3
through copper rod is [JEE Main] and S 4 of identical thickness, through which the heat
(a) 1.2 cal/s (b) 2.4 cal/s transfer is steady. Arrange the materials according to
(c) 5.52 cal/s (d) 6.0 cal/s their thermal conductivities in decreasing order.
[Karnataka CET]
2. Consider a black body radiation in a cubical box at 25°C 15°C 10°C –5°C –10°C
absolute temperature T. If the length of each side of the S1 S2 S3 S4
box is doubled and the temperature of the walls of the
box and that of the radiation is halved, then the total (a) S 2 , S 4 , S 1 , S 3 (b) S 2 = S 4 , S 1 , S 3
energy [WB JEE] (c) S 1 = S 2 , S 3 , S 4 (d) S 1 , S 2 , S 3 , S 4
(a) halves (b) doubles R
(c) quadruples (d) remains constant 7. Three rods of same dimensions are
arranged as shown in figure, they have
3. Same quantity of ice is filled in each of the two metal thermal conductivities K 1 , K 2 and K 3 .
containers P and Q having the same size, shape and wall The points P and Q are maintained at
thickness but made of different materials. The containers different temperatures for the heat to P Q
are kept in identical surroundings. The ice in P melts flow at the same rate along PRQ and
completely in time t 1 whereas in Q takes a time t 2 . The PQ. [Karnataka CET]
ratio of thermal conductivities of the materials of P and Q 1
(a) K 3 = (K 1 + K 2 ) (b) K 3 = K 1 + K 2
is [WB JEE] 2
K 1K 2
(a) t2 : t1 (b) t1 : t2 (c) K 3 = (d) K 3 = 2(K 1 + K 2 )
(c) t12 : t22 (d) t22 : t12 K1 + K2

8. A , B and C are three identical conductors but made from


2013
different materials. They are kept in contact as shown.
4. Two rectangular blocks, having identical dimensions, can K
Their thermal conductivities are K , 2K and . The free
be arranged either in configuration I or in configuration II 2
as shown in the figure. One of the blocks has thermal end of A is at 100°C and the free end of C is at 0°C.
conductivity K and the other 2K. The temperature During steady state, the temperature of the junction of A
difference between the ends along the X -axis is the same and B is nearly [Karnataka CET]
in both the configurations. It takes 9 s to transport a
A B C
certain amount of heat from the hot end to the cold end in
the configuration I. The time to transport the same 100°C 0°C
amount of heat in the configuration II is [JEE Advanced]
Configuration II (a) 37°C (b) 71°C (c) 29°C (d) 63°C
Configuration I
9. Two spherical black bodies of radii r1 and r2 and with
2K
K 2K K surface temperatures T1 and T2 respectively, radiate the
same power. Then, the ratio of r1 and r2 will be
(a) 2.0 s (b) 3.0 s (c) 4.5 s (d) 6.0 s [Karnataka CET]
2 4
T  T 
5. If a piece of metal is heated to temperature θ and then (a)  2  (b)  2 
 T1   T1 
allowed to cool in a room which is at temperature θ 0 . The 2 4
graph between the temperature T of the metal and time t T  T 
(c)  1  (d)  1 
will be closed to [JEE Main]  T2   T2 
720 Objective Physics Vol. 1

10. A cane is taken out from a refrigerator at 0°C. The 2011


atmospheric temperature is 25°C. If t 1 is the time taken to
heat from 0°C to 5°C and t 2 is the time taken from 10°C 16. A composite block is made of slabs A, B, C, D and E of
to 15°C, then [Karnataka CET] different thermal conductivities (given in terms of a
constant K ) and sizes (given in terms of length L) as
(a) t1 > t2 (b) t1 < t2
shown in the figure. All slabs are of same width. Heat Q
(c) t1 = t2 (d) there is no relation
flows only from left to right through the blocks. Then, in
11. The temperatures of two bodies A and B are respectively, steady state [IIT JEE]
727°C and 327°C. The ratio H A : H B of the rates of heat Heat 0 1L 5L 6L
2L A B 3K E
radiated by them is [O JEE]
(a) 727 : 327 (b) 5 : 3
(c) 25 : 9 (d) 625 : 81 2K C 4K 6K

3L
12. In a reversible ideal engine, 650 J of heat comes from D 5K
source of 450 K, heat rejected to sink at 225 K is [O JEE] 4L
(a) 325 J (b) 625 J
(a) heat flow through A and E slabs are same
(c) 700 J (d) 800 J
(b) heat flow through slab E is maximum
13. 22320 cal of heat is supplied to 100 g of ice at 0°C. If the
(c) temperature difference across slab E is smallest
latent heat of fusion of ice is 80 cal g −1 and latent heat of
(d) heat flow through C = heat flow through B + heat flow
vaporisation of water is 540 cal g − 1 , the final obtained
amount of water and its temperature respectively, are through D
(a) 8 g, 100°C (b) 100 g, 90°C [WB JEE] 17. A lead bullet strikes against a steel plate with a velocity
(c) 92 g, 100°C (d) 82 g, 100°C 200 m/s. If the impact is perfectly inelastic and the heat
produced is equally shared between the bullet and the
2012 target, then the rise in temperature of the bullet is
14. Three very large plates of same area are kept parallel (specific heat capacity of lead = 125 Jkg −1 K −1 )
and close to each other. They are considered as [Kerala CEE]
ideal black surfaces and have very high thermal
(a) 80ºC (b) 60ºC
conductivity. The first and third plates are maintained (c) 40ºC (d) 120ºC
at temperatures 2T and 3T, respectively. The (e) 120°C
temperature of the middle (i.e. second ) plate under
steady state condition, is [IIT JEE]
18. The quantities of heat required to raise the temperatures
1/ 4 1/ 4 of two copper spheres of radii r1 and r2 ( r1 = 1.5 r2 )
 65  97 through 1 K are in the ratio of [Karnataka CET]
(a)   T (b)   T
 2  4 3
1/ 4 (a) 1 (b)
 97 2
(c)   T (d) (97)1/ 4T
 2 (c)
9
(d)
27
4 8
15. A liquid in a beaker has temperature θ(T ) at time t and θ 0 is
19. Three identical rods A, B and C are placed end to end. A
temperature of surroundings, then according to Newton’s
temperature difference is maintained between the free
law of cooling, the correct graph between log e (θ − θ 0 ) and
ends of A and C. The thermal conductivity of B is thrice
t is [AIEEE]
that of C and half of that of A.
The effective thermal conductivity of the system will be
loge (θ– θ0)

loge (θ– θ0)

(K A is the thermal conductivity of rod A) [Karnataka CET]


(a) (b) 3 1
(a) KA (b) 2 K A (c) 3 K A (d) KA
2 3
t t 20. 1.56 × 105 J of heat is conducted through is 2 m 2 wall of
12 cm thick in one hour. Temperature difference between
loge (θ– θ0)

loge (θ– θ0)

the two sides of the wall is 20 ºC. The thermal


(c) (d) conductivity of the material of the wall is (in Wm −1 K −1 )
[WB JEE]
t t (a) 0.11 (b) 0.13 (c) 0.15 (d) 1.2
Calorimetry and Heat Transfer 721

θ θ
2010
21. A piece of ice (heat capacity = 2100 J kg −1 ºC−1 and
latent heat = 3.36 × 105 J kg −1 ) of mass m gram is at (c) (d)
–5ºC at atmospheric pressure. It is given 420 J of heat, so
that the ice starts melting. Finally, when the ice-water x x
mixture is in equilibrium, it is found that 1 g of ice has
melted. Assuming there is no other heat exchange in the
24. Two slabs are of the thicknesses d1 and d 2 . Their thermal
process, the value of m is [IIT JEE]
conductivities are K 1 and K 2 , respectively. They are in
(a) 8 (b) 6
(c) 4 (d) 8.5 series. The free ends of the combination of these two
slabs are kept at temperatures θ1 and θ 2 . Assume θ1 > θ 2 .
22. Two spherical bodies A (radius 6 cm) and B (radius The temperature θ of their common junction is
18 cm) are at temperatures T1 and T2 , respectively. The [Karnataka CET]
maximum intensity in the emission spectrum of A is at K θ + K 2θ 2 K 1θ 1d1 + K 2θ 2d2
500 nm and in that of B is at 1500 nm. Considering them (a) 1 1 (b)
θ1 + θ 2 K 1d2 + K 2d1
to be black bodies, what will be the ratio of the rate of K 1θ 1d2 + K 2θ 2d1 K 1θ 1 + K 2θ 2
total energy radiated by A to that of B? [IIT JEE] (c) (d)
K 1d2 + K 2d1 K1 + K2
(a) 9 (b) 9.5 (c) 8 (c) 8.5

23. A long metallic bar is carrying heat from one of its ends 25. For an opaque body, coefficient of transmission is
to the other end under steady-state.The variation of (a) zero [MHT CET]
temperature θ along the length x of the bar from its hot end (b) 1
(c) 0.5
is best described by which of the following figure? (d) ∞
[AIEEE]
θ θ 26. Two spheres of radii 8 cm and 2 cm are cooling. Their
temperatures are 127ºC and 527ºC, respectively. Find
(a) (b) the ratio of energy radiated by them in the same time
[MHT CET]
(a) 0.06 (b) 0.5
x x (c) 1 (d) 2

Answers
Level 1
Objective Problems
1. (a) 2. (a) 3. (b) 4. (b) 5. (a) 6. (b) 7. (b) 8. (a) 9. (a) 10. (a)
11. (b) 12. (d) 13. (a) 14. (c) 15. (b) 16. (c) 17. (b) 18. (a) 19. (a) 20. (c)
21. (a) 22. (b) 23. (b) 24. (d) 25. (d) 26. (a) 27. (b) 28. (a) 29. (b) 30. (c)
31. (a) 32. (b) 33. (d) 34. (d) 35. (d) 36. (a) 37. (b) 38. (c) 39. (b) 40. (b)
41. (a) 42. (c) 43. (b) 44. (a) 45. (c) 46. (c) 47. (c) 48. (a) 49. (d) 50. (d)
51. (a) 52. (a) 53. (b) 54. (a) 55. (c) 56. (c) 57. (c) 58. (c) 59. (a) 60. (a)
61. (b) 62. (c) 63. (d) 64. (d) 65. (a) 66. (d) 67. (c) 68. (c) 69. (c) 70. (c)
71. (a) 72. (b) 73. (b) 74. (c) 75. (a) 76. (c) 77. (b) 78. (b) 79. (c) 80. (b)
81. (d) 82. (b)

Level 2
Only One Correct Option
1. (b) 2. (b) 3. (d) 4. (b) 5. (d) 6. (c) 7. (a,d) 8. (a,b) 9. (a) 10. (a)
11. (a) 12. (c) 13. (c) 14. (c) 15. (b)
722 Objective Physics Vol. 1

More than One Correct Options


1. (a,b) 2. (b,d) 3. (b,d) 4. (b,c) 5. (a,d)

Assertion and Reason


1. (a) 2. (c) 3. (b) 4. (c) 5. (a) 6. (a) 7. (d) 8. (c) 9. (c) 10. (a)
11. (b) 12. (a) 13. (b) 14. (d) 15. (e) 16. (c) 17. (d)

Match the Columns


1. (A → r, B → p, C → s, D → q) 2. (A → t, B → r, C → q, D → t) 3. (A → p, B → r, C → t)
4. (A → q, B → p, C → s, D → r)

Entrance Gallery
1. (c) 2. (d) 3. (a) 4. (a) 5. (c) 6. (b) 7. (c) 8. (b) 9. (a) 10. (b)
11. (d) 12. (a) 13. (a) 14. (c) 15. (a) 16. (a,c,d) 17. (a) 18. (c) 19. (d) 20. (b)
21. (a) 22. (a) 23. (b) 24. (c) 25. (a) 26. (c)

Solutions
Level 1 : Objective Problems =
(15.2 × 10−3 ) (3.34 × 105 )
1. Heat liberated when 50 g water at 60°C converts into water 0.06 × 10 × 9.8
at 0°C. = 863.4 m ≈ 863 m
Q = ms∆θ = 50 × 1 × 60 = 3000 cal 7. Heat released when 5 kg of water at 15°C converts into 0°C
From this heat mass of ice which can be melted is of water is Q = ms∆θ = ( 5000)(1)(15) = 75000 cal. This heat can
Q 3000
m= = = 37.5 g Q 75000
L 80 melt mass of water m = = = 937.5 g of 0.9375 kg.
L 80
i.e. whole ice is not melted and temperature of mixture will
be 0°C. Since, whole of ice is not melting, temperature of mixture
2. Temperature first increases from −10°C to 0°C. Then, will be 0°C.
remains constant at 0°C till whole ice is melted. Then, it 8. Q = ms ∆θ = 5 × (1000 × 4.2) × (100 − 20)
increases from 0°C to 100°C. Again it remains constant at
= 1680 × 103 J =1680 kJ
100°C till whole water converts into steam.
3. Heat taken = Heat given 9. If m g ice melts, then
or Ms∆θ = mL Heat lost = Heat gain
∴ ( 200)( s )(100) = 25 × 80 80 × 1 × ( 30 − 0) = m × 80
∴ s = 0.1 cal/g°C ⇒ m = 30 g
4. Q = m1 s1 ∆θ + m2 s 2 ∆θ + m3 s 3 ∆θ 10. Same amount of heat is supplied to copper and water.
= (2.4 × 0.216 + 1.6 × 0.0917 + 0.8 × 0.0931) (60) So, mC cC ∆θC = mW cW ∆θW
µ χ ( ∆θ)Χ
= 44.376 cal ≈ 44.44 cal ⇒ ∆ θΩ = Χ Χ
µ Ω χΩ
5. Heat required,
Q1 = (1.1 + 0.02) × 103 × (80 − 15) = 72800 cal 50 × 10−3 × 420 × 10
=
Suppose, m kg of steam is condensed and it also reaches to 10 × 10−3 × 4200
80°C. Then, heat rejected = 5° C
Q2 = mL + ms∆θ 11. Heat lost by hot water = Heat gained by cold water in beaker
= m( 540 × 103 + 20 × 103 ) + Heat absorbed by beaker
= 560 × 103 m ⇒ 440( 92° − θ) = 200 × (θ − 20° ) + 20 × (θ − 20° )

Equating Q1 and Q2 , we get ⇒ θ = 68° C


m = 0.13 kg 12. Temperature of mixture,
6. µMgd = mL m
m1c1θ1 + m2c 2θ 2 m × c × 2T + ( 2c ) T
2 3
mL θmix = = = T
∴ d= m1c1 + m2c 2 m 2
µMg m × c + ( 2c )
2
Calorimetry and Heat Transfer 723

13. (100 × 80) + (100 × 1 × θ) = 100 × 1 × (100° − θ) Hence, Q = Q′


or θ =10°C ⇒ 600m = 600 × 103
1 1 2
14. ms∆θ =  mv  ⇒ m = 103 g = 1 kg
22 
18. Substances having more heat capacity take longer time to
v 2 ( 300)2
∴ ∆θ = = = 150°C get heated to a higher temperature and longer time to get
4s 4 × 150 cooled.
15. Initially, ice will absorb heat to raise its temperature 0°C, T
then its melting takes place. A
If mi = initial mass of ice, m′i = mass of ice that melts and B
mW = initial mass of water
By law of mixture heat gained by ice = heat lost by water C
⇒ m1 × c × ( 20) + m′i × L = mW cW ( 20)

⇒ 2 × 0.5 ( 20) + mi × 80 = 5 × 1 × 20 tA tB tC t
⇒ m′i =1 kg
If we draw a line parallel to the time axis, then it cuts the
So, final mass of water = initial mass of water + mass of ice
given graphs at three different points. Corresponding
that melts = 5 + 1 = 6 kg
points on the time axis shows that
16. Heat gain = Heat lost
tC > tB > tA
mAcA ∆TA = mBcB ∆TB
⇒ CC > C B > C A
cA (16 − 12) = cB (19 − 16) (QmA = mB )
19. The volume of matter in portion AB of the curve is almost
cA 3
⇒ = constant and pressure is decreasing. These are the
cB 4 characteristics of liquid state.
and cB ( 23 − 19) = cC ( 28 − 23) 20. From given curve,
cB 5 Melting point for, A = 60°C
⇒ =
cC 4 And melting point for, B = 20°C
Time taken by A for fusion = (6 − 2) = 4 min
cA 15
⇒ = ...(i) Time taken by B for fusion = (6.5 − 4) = 2.5 min
cC 16
HA 6 × 4 × 60 8
Then, = =
Similarly, if θ is the temperature, when A and C are mixed HB 6 × 2.5 × 60 5
then,
Temperature difference
cA (θ − 12° ) = cC ( 28° − θ) 21. Thermal resistance, R =
Heat current
cA 28° − θ [θ]
⇒ = ...(ii) ∴ [R ] =
cC θ − 12° [ML2 T −2 /T]
On solving Eqs. (i) and (ii), we have = [M−1 L−2 T 3θ]
θ = 20.2°C
22. K A = 2K B
R
⇒ ∴ RA = B QR = l 
17. Suppose, m kg steam is required per hour.  
2  KA 
Heat is released by steam in following three steps
(i) When 150°C steam → Q
100 °C steam
1

Q1 = mcSteam ∆θ A B
= m × 1(150 − 100)
= 50m cal
(ii) When 100°C steam → 100°C water Suppose, RA = R , then RB = 2R
Q 2
36 36 12
Q2 = mLV = m × 540 = 540m cal Heat current, H= = =
(iii) When 100°C water → 90°C water R + 2R 3R R
Q 2
Q3 = mcW ∆θ ∴ Temperature difference across A
12
= m × 1 × (100 − 90) = H × RA = × R = 12°C
=10m R
100 − 0 100
Hence, total heat given by the steam 23. H = = Heat current =
RAB RAB
Q = Q1 + Q2 + Q3 = 600m ...(i)
Heat taken by 10 kg water, 9 cm
Q ′ = mcW ∆θ = 10 × 103 × 1 × (80 − 20) A C B

= 600 × 10 cal
3
20 cm
724 Objective Physics Vol. 1

100 100 × 9 100 − θ θ − 50 θ−0


100 − θC = H ⋅ RAC = ⋅ RAC = = 45 or = +
RAB 20 (l / 3KA ) (l / 2KA ) (l / KA )
∴ θC = 55 °C or 3(100 − θ) = 2(θ − 50) + θ
KA(θ1 − θ2 ) t
24. Q = Solving, we get θ=
200
°C
l 3
Q ×l
∴ K = 30. Let θ be the temperature of junction, H1 , H2 and H3 the heat
A(θ1 − θ2 ) t
currents. Then,
J-m J
Unit of K = 2 = 20ºC
m K-s s-mK

25. Thermal resistance of A is RA =


10 QR = l  H2
 
3KA  KA 

θ
3K A B K H1

H3
10 cm 20 cm 30ºC
10ºC
20 RA 1
RB = ⇒ = H1 = H2 + H3
KA RB 6 30 − θ θ − 20 θ − 10
or = +
So let RA = R, then RB = 6R ( 30 / KA ) ( 20 / KA ) (10 / KA )
Temperature difference 2( 30 − θ) = 3(θ − 20) + (θ − 10)
Heat current, H = or
Total resistance
or θ = 16.36° C ≈ 16.4°C
35 5
= = Q KA∆θ ∆θ ∆θ
7R R 31. = = = (R = Thermal resistance)
t l (l / KA ) R
Now, temperature difference across
5 ⇒ t∝R (QQ and ∆θ are same)
A = H × RA = × R = 5°C tp Rp R/2 1
R ⇒ = = =
26. Heat currents in both the rods are equal. ts Rs 2R 4
θC − θB θ − θA ⇒
ts 4
tp = = = 1 min
or HCB = HBA or = B
( 2l / 3KA ) (l / 2KA ) 4 4
3 (Series resistance Rs = R1 + R2 and parallel resistance
∴ ( 70 − θB ) = 2(θB − 0) or θB = 30°C RR
2 Rp = 1 2 ) (∴ R1 = R2 )
27. H1 = H2 R1 + R2
θ − θ2 30 − θ2
 TD  =  TD  (TD = Temperature difference) 32. 1 = 80 ⇒ = 80
∴     l 0.5
 R 1  R  2
⇒ θ2 = − 10°C
or R1 = R2
dQ dθ dθ

l
=
l 33. = − KA , when K = ∞, =0
K 1 A1 K 2 A2 dT dx dx
i.e. θ is independent of x, i.e. constant or uniform.
or K 1 A1 = K 2 A2
34. Let the heat transferred be Q
28. K C > K B ∴RC < RB
When rods are joined end to end. Heat transferred by each
Temperature difference across copper = HRC = (TD)C rod.
And temperature difference across brass = HRB = (TD)B K ( 2A ) ∆θ
=Q = × 12 ...(i)
Heat current H will be same in both as they are in series. l
Since, RC < RB When rods are joined lengthwise,
∴ (TD)C < (TD)B KA∆θ
Q= t ...(ii)
Or temperature of junction is more than 50°C. 2l
29. Suppose, θ be the temperature of junction. H1 , H2 and H3 From Eqs. (i) and (ii), we get t = 48 s
Q KA∆θ
are heat currents. Then, H1 = H2 + H3 35. =
t l
50ºC K A AB
H2 ⇒ =
K B AA
H1 2
r  1
=  B  =
100ºC  rA  4
H3 KB
⇒ KA =
0ºC 4
Calorimetry and Heat Transfer 725

 dQ  H1 , H2 , H3 and H4 be the heat currents. Then


 
dQ K ( πr 2 ) ∆θ  dt  2 H1 = H2 + H3 + H4
36. = ⇒
dt l  dQ  100 − θ θ − 50 θ − 50 θ − 0
  or = + +
 dt  1 R R R R
K 2 × r22 × l1 1 1 2 (R = Thermal resistance of each part of two rods)
= = × ×
K 1 × r12 × l 2 2 4 1 Solving these two equations, we get
 dQ  θ = 50°C
  42. E ∝ T 4 ,
⇒  dQ  =  dt  1 = 4 = 1
 
 dt  2 4 4 T1 = 500 K and T2 = 1000 K
37. In parallel combination, T2 = 2T1
1 1 1 ∴ E2 = ( 2)4 E1 = 16E1 = 160 cal/s
= + (R = thermal resistance)
R R1 R2 43. 4 πr12T14 = 4 πr22T24
2KA K 1 A K 2 A
or = + r1 T22
l l l ∴ =
r2 T12
K1 + K 2
∴ K = 44. E ∝ T 4 , T1 = 300 K, T2 = 400 K
2
38. Thermal resistance across three rods is RPR + RRQ = RPQ 45. P ∝ AT 4
l l l Area and temperature both are doubled. Hence, power will
∴ + = become ( 2)5 or 32 times.
K1 A K 2A K 3A
32 × 150
K1 K 2 ∴ P ′ = 32P = kW
or K3 = 1000
K1 + K 2
= 4.8 kW
39. For no current flow between C and D P1 A1T14 4 π (1)2 ( 4000)4 1
46. = = =
Q = Q P2 A2T24 4 π ( 4)2 ( 2000)4 1
   
 t  AC  t  CB
or P1 = P2
K 1 A(θA − θC ) K A(θ − θB ) A T4
⇒ = 2 C P
47. 1 = 1 14
l l P2 A2T2
θA − θC K 2
⇒ = ...(i) 4
θC − θB K 1 A2  T2 
P2 =   P1
Q = Q A1  T1 
Also,    
 t  AD  t  DB 2
=   ( 2)4 (640)
1
K 3 A(θA − θD ) K 4 A(θD − θB )  2
⇒ =
l l
= 2560 W
θA − θD K 4
⇒ = ...(ii) e  e 
θD − θB K 3 48.  λ  =  λ 
 aλ  any body  aλ  Perfectly black body
It is given that θC = θD , hence from Eqs. (i) and (ii),
K2 K4 According to Kirchhoff’s law,
we get =
K1 K 3 eλ
But aλ =1 for perfectly black body. Hence, for any body is
⇒ K1 K 4 = K 2 K 3 aλ
equal to eλ for perfectly black body.
40. λmT = b, where b = 2.89 × 10−3 mK
49. According to Wien’s law, λmT = constant
b . × 10−3
289
⇒ T= = = 2000 K ∴ ( λmT )s = ( λmT )m
λm 1.5 × 10−6
Ts ( λm )m
41. Let θ be the temperature of junction. or =
Tm ( λm )s
50ºC
10−4
= = 200
0.5 × 10−6
H2 50. Area ∝ Total energy radiated ∝ T 4
T1 = 300 K and T2 = 600 K
100ºC θ 0ºC Temperature has doubled. Therefore, area will become
H1 H2
16 times.
H3
51. According to Wien’s law,
λmT = constant
1
50ºC or T ∝
λm
726 Objective Physics Vol. 1

3 65. A body having maximum surface area will cool at fastest


λm has become times.
4 rate.
4 66. Heat required to melt whole ice is
∴ Temperature will have become times.
3 Q = mL = 20 × 80 =1600 cal
Now, according to Stefan’s law, power P ∝ T 4 . Energy incident per minute is
π
Therefore, new power will be P = 1.9 × × d 2
4 4
 4  P = 256 P
  π
 3 81 = 1.9 × × ( 20)2
4
52. Cooling rate ∝ (T 4 − T04 ) = 597 cal/min
( 900)4 − ( 300)4  Q
Therefore, time required to melt whole ice is t = ≈ 2.7 min.
r= 4  0
r
 (600) − ( 300) 
4 P
16 67. Q = mL = L (Qm =1 kg )
= r0
3 ∆U = Q − W = L − W
53. Density of hot air is lesser than the density of cold air so hot i.e. ∆U < L
air rises up. Q1 KA( 90° − 60° )
69. = = 50 KA
56. According to Kirchhoff’s law, the ratio of emissive power to t 0.6
absorptive power is same for all bodies. Which is equal to Q2 KA(150 − 110)
the emissive power of a perfectly black body, i.e. and =
t 0.8
e = EBlack body for a particular wavelength
  = 50 KA
 a  body
Q KA( ∆θ)
70. =
 eλ  t l
  = ( Eλ )Black body
 aλ  body 5 × 20K
⇒ 50 =
0.4
⇒ eλ = aλ Eλ 1
⇒ K = = 0.2
57. Because Planck’s law explains the distribution of energy 5
correctly at low temperature as well as at high temperature.
74. Mud is bad conductor of heat. So, it prevents the flow of
58. According to Kirchhoff’s law, a good emitter is also a good heat between surroundings and inside.
absorber.
75. Heat flows from hot air to cold body, so person feels
λ T T
59. 2 = 1 ⇒ λ 2 = 1 × λ1 comfort.
λ1 T2 T2
76. No flow of heat by convection in vacuum.
900
= ×4 J
1200 78. In MKS system, unit of σ is 2
m × sec × K 4
= 3µm
J 107 erg
b ⇒ = 4
60. λm = m × sec × K 10 cm2 × sec × K 4
2 4
T
erg
b 2.89 × 10−3 = 103
⇒ T= = = 7225 K cm2 × sec × K 4
λm 4000 × 10−10
θ − θ2 θ − θ2
T (λ ) 350 79. Applying, 1 =α  1 − θ0  two times. we have
61. S = N max = = 0.69 t  2 
T N ( λ S )max 510
50 − 40 50 + 40
62. According to Stefan’s law, E = σεAT 4 or =α  − 20 ...(i)
5  2 
1.58 × 105 × 4.2 40 − 30 40 + 30
⇒ = 5.67 × 10−8 × 10−4 × 0.8 × T 4 and =α  − 20 ...(ii)
60 × 60 t  2 
T = 2500 K
Solving these two equations, we get
63. Radiated power by black body, 25
Q t= min
P = = AσT 4 3
T
θ − θ2 θ + θ2
ρ 
2 4 80. Applying 1 =α  1 − θ0  two times, we get
⇒ P ∝ AT 4 ∝ r 2T 4 ⇒
Π1
= 1
 Τ1 
 
t  2 
Π 2  ρ 2   Τ2  80 − 60 80 + 60
=α  − 30 ...(i)
2 4 10  2 
440  12   500 
⇒ =    ⇒ P2 = 1760 W 60 − θ 60 + θ
P2  6   1000  =α  − 30 ...(ii)
10  2 
200 × 7 × 60
64. H = cal
4.18 Solving these two equations, we get
= 20095.7 ≈ 20000 cal θ = 48°C
Calorimetry and Heat Transfer 727

θ1 − θ2 θ + θ2
81. Applying, =α  1 − θ0  two times, we get 4. There will be no heat flow through AB, if temperature of B is
t  2  also 20°C and heat current in DB is equal to heat current in
50 − 45 50 + 45 BC.
=α  − θ0  ...(i)
5  2  A B C
20ºC 0ºC
45 − 41.5 45 + 41.5
=α  − θ0  ...(ii)
5  2 
Solving these two equations, we get
θ0 = temperature of atmosphere 90ºC D
= 33.3°C ∴ HDB = HBC ( H = TD / R )
mL 70 20
82. Heat lost in t second = mL or heat lost per second = . This or =
t (l BD / KA ) (l BC / KA )
must be the heat supplied for keeping the substance in l BD 7
molten state per sec. or =
l BC 2
mL
∴ =P TD
t 5. Rate of heat flow, I ∝
Pt R
or L=
m ∴ TD = temperature difference is same.
1
∴ I∝
Level 2 : Only One Correct Option Thermal resistance
1. QTB > TA ⇒ Heat will flow B to A via two paths (i) B to A I A RB ( R )( 2R )/ R + 2R 2
= = =
(ii) along BCA as shown. I B RA ( R + 2R ) 9
A 6. RAB = RCD = R (say)
Length of semicircle = πl
∴ Resistance of semicircle = πR as R ∝ l
πR
RBC = (in parallel)
2
100 − 0
Heat current, H = = 28 units
B C π
R+ R+ ⋅R
2
Rate of flow of heat in path BCA will be same Now, θC − 0° = H R = 28°C
Q = Q ∴ θC = 28°C
i.e.    
 t  BC  t  CA 7. dQ = ms ⋅ dT
K ( 2T − TC ) A K (TC − T ) A m =1 kg
⇒ =
a 2a dQ
∴ s= = Slope of Q-T graph
TC 3 dT
⇒ =
T 1+ 2 Ssolid = tan 45° = 1 J/kg-K
1
2. Heat currents in the two rods will be same. Sliquid = tan 30° = J/kg-K
3
∴ H1 = H2
Latent heat of vaporisation
θ1 − θ θ − θ2
or = (θ = temperature of junction) = heat required to melt complete 1 kg
(l / K 1 A ) (l / K 2 A )
= 200 J/kg
or K 1 (θ1 − θ) = K 2 (θ − θ2 )
8. No heat is flowing through C and F. Therefore, they are at
K θ + K 2θ2
∴ θ= 1 1 same temperatures.
K1 + K 2 Q KA(θ1 − θ2 )
9. H = =
dθ dm TD t l
3. = L⋅ = Heat transfer from bottom =
dt dt (l / KA ) A
or H∝ or H2 = 2H1
Heat TD = desired temperature difference. l
dm 10. A and B
L ⋅l ⋅
∴ TD = dt mSA (15 − 10) = mSB ( 25 − 15)
KA SA
or =2 ...(i)
(540) (2 × 10−1 )  
1 SB
 60 
= B and C
0.5 × 300
mSB ( 30 − 25) = mSC ( 40 − 30)
= 0.012°C
728 Objective Physics Vol. 1

SB As thickness of the plate is least hence, surface area of the


or =2 ...(ii)
SC plate is maximum.
We know that, according to Stefan’s law of heat loss
From Eqs. (i) and (ii), we have
H ∝ AT 4
SA
=4
SC where, A is surface area of for object and T is temperature.
Hence, Hsphere : Hcube : Hplate
Now, let θ be the equilibrium temperature, when A and C are
mixed. Then, = Asphere : A cube : A plate
mSA (θ − 10) = mSC ( 40 − θ) As Aplate is maximum.
  Hence, the plate will cool fastest.
S
or 4(θ − 10) = 40 − θ  Using A = 4 As, the sphere is having minimum surface area hence, the
 SC  sphere cools slowest.
or θ =16°C 15. Let the equilibrium temperature of the system is T.
11. Heat carried increases to 2 times. Therefore, new net Let us assume that T1 , T2 < T < T3 .
thermal resistance will remain half. According to question, there is no net loss to the
R surroundings.
∴ R′ =
2 Heat lost by M 3 = Heat gained by M1 + Heat gained by M 2
R ′1 R ′2 R1 R2 ⇒ M 3 s (T3 − T ) = M1 s (T − T1 ) + M 2 s (T − T2 )
=
R ′1 + R ′2 2( R1 + R2 ) (where, s is specific heat of the copper material)
( 3l / KA )(l / K ′ A ) ( 3l / KA )(l / KA ) ⇒ T [ M1 + M 2 + M 3 ] = M 3T3 + M1T1 + M 2T2
or =
( 3l / KA ) + (l / K ′ A ) 2 
3l
+
l  M T + M 2T2 + M 3T3
 ⇒ T= 1 1
 KA KA  M1 + M 2 + M 3
7
Solving this we get, K ′ = K
3 More than One Correct Options
12. Initially, the rods are in parallel
Q (θ1 − θ2 ) 1. Total radiation per second is given by
⇒   =
Q mL
= P = (er σ T 4 A )
t R  t 1 t
(100 − 0) Here, er : σ : T and A all are same.
= q1 L = ...(i)
R/ 2 Hence, P will be same.
Finally, when rods are in series Same is the case with absorption per second.
P = ms  −
 Q  = mL = q L = (100 − 0) dT 
⇒   ...(ii) Now, 
 t 2 2  dt 
t 2R
or Rate of cooling  −
From Eqs. (i) and (ii), we get dT  P 1
= ∝
q1 4  dt  ms m
=
q2 1 Mass of hollow sphere is less. So, its initial rate of cooling
13. Rate of cooling  −
dT  will be more
 ∝ emissitivity (e )
 dt  2.
40°C
From graph,  −
dT   dT 
 > − 
 dt  x  dt  y
H1
⇒ ex > e y
Further emissivity e ∝ absorptive power ( a ) θ
⇒ ax > a y
(Qgood absorbers are good emitters). H2 H3
14. Consider the diagram where all the three objects are heated
mass 20°C 30°C
to same temperature T. We know that density, ρ = as
volume
40 − θ θ − 20 θ − 30
ρ is same for all the three objects hence, volume will also be ∴ = +
same. R R R
m where, R = Thermal resistance of each rod.
T m Solving this equation we get,
T
θ = 30° C
Heat flows from higher temperature to lower temperature.
m 3. ms ( 2θ − θ′ ) = m ( 2s )(θ′ − θ)
T
Solving this equation, we get
Sphere Cube Plate 4
θ′ = θ
3
Calorimetry and Heat Transfer 729

Further, heat capacity A, B and C


c = ms (m)( s )(θ − 20° ) + ( 2m)( s )( 40° − θ) = ( 3m)( s )(60° − θ)
or c ∝s (as m is same) or θ = 60° C
1

c1 s1 1
= = 3. Thermal resistance ∝
c2 s 2 2 Area
B
TD TD
4. q1 = ⇒ ∴ R1 =
R1 q1 H1 H2
TD
Similarly, R2 =
q2
In series, A H3 C
TD TD 100ºC 0ºC
qs = =


R1 + R2 TD + TD R 2R
q1 q2
q1 q2
=
q1 + q2
TD  1 
In parallel, qp = = TD  
Rnet  Rnet 
R
1 1 
= TD  +  So let, RBC = R
 R1 R2 
Then, RAB = RAC = 2R
= TD  1 + 2  = q1 + q2
q q
 TD TD  H1 = H2
5. (b) At low temperature, the rate of cooling is proportional to 100 − θB θB − 0
∴ =
temperature difference. R 2R
∴ θB = 67.7°C
Assertion and Reason
RABC = 3R and RAC = 2R
8. Water in vapour state has more heat content. H3
9. Q = mL1 + ms∆θ + mL 2 ∴ H1 = H2 =
3
= 1 × 80 + 1 × 1 × 100 + 1 × 540
= 720 cal
Entrance Gallery
11. Due to gravity hot (but lighter) material rises up and cold 1. In thermal conduction, it is found that in steady state the
(but heavier) material comes down. heat current is directly proportional to the area of
13. According to Wien’s displacement law, cross-section A which is proportional to the change in
1 temperature (T1 − T2 ).
λm ∝ ∆Q KA (T1 − T2 )
T Then, =
∆t x
14. Emissive power is energy radiated per unit area per unit
∆Q
time. Which is not one. = Heat current
∆t
15. A normal body can also radiate energy more than a
where, K = thermal conductivity
perfectly black body, if its temperature is very high.
l 1
and A = area of cross-section.
16. R = or R ∝ ⇒ ∴ R1 > R2 According to thermal conductivity, we get
KA A
dQ1 dQ2 dQ3
Further, TD = H × R or TD ∝ R = +
dt dt dt
∴ (TD)1 > (TD)2 0.92 × 4 (100 − T ) 0.26 × 4 (T − 0) 012
. × 4 (T − 0)
= +
17. Statement of reason is Kirchhoff’s law. 46 13 12
100°C
Match the Columns l1
dQ1
2. A and B ——
dt
(m)( s )(θ − 20° ) = ( 2m)( s )( 40° − θ) K1
100
or θ= = 33.3°C K2 K3
3
dQ2 dQ3
A and C —— l2 l3 ——
dt dt
(m)( s )(θ − 20° ) = ( 3m)( s )(60° − θ) or θ = 50°C
0°C 0°C
B and C
( 2m)( s )(θ − 40° ) = ( 3m)( s )(60° − θ) or θ = 52°C ⇒ T = 40° C
730 Objective Physics Vol. 1

dQ1 0.92 × 4 (100 − 40) 8. Let R be the thermal conductivity of conductor A, then
∴ =
R
dt 40 thermal conductivity of conductor B = .
= 5.52 cal/s 2
2. Assuming the at temperature of the body and cubical box is A B C
same initially, i.e. T and finally it becomes T / 2 .
Because, temperature of body and surroundings remain 100°C 0°C
same.
Hence, no net loss of radiation occur through the body. This
total energy remains constant. and thermal conductivity of conductor C = 2R
3. We know that relation between temperature gradient (TG) 100° − 0° 200
and thermal conductivity ( K ) ∴ Heat current, H = =
R
θ θ R + + 2R 7R
So, d = − KA d = − KA × ( TG) 2
dt dx If T ′ be the temperature of the junction of A and B, then
t∝
1  dθ = constant  100 − T ′ 200 100 − T ′ 500
i.e.   H= or = or T ′ = = 71° C
K  dt 
R 7R R 7
1 9. Radius of Ist body = r1 , radius of IInd body = r2
or K ∝
t Temperature of Ist body = T1
1
K1 ∝ ...(i) Temperature of IInd body = T2
t1
The emissivity of a black body or radiated power is given by
1
K2 ∝ ...(ii) E = AσT 4 × t ∝ AT 4
t2 E
or A ∝ 4 (where, A is the surface area of the spherical black
From Eqs. (i) and (ii), we get T
K 1 1 / t1 body)
=
K 2 1 / t2 As in the condition of question , the power radiated by Ist
K 1 : K 2 = t2 : t1 and IInd body is same.
1
4. Thermal resistance in configuration I, Hence, A∝ 4
T
RI = R1 + R2 = 
l   l  3 l 
+ =   4
 KA   2KA  2  KA  A1  T2 
So, = 
Thermal resistance in configuration II, A2  T1 
1 1 1 4
= + 4 πr12  T2 
RII R1 R2 ⇒ = 
4 πr22  T1 
KA 2KA
= + 2
l l r1  T2 
Thus, = 
l R r2  T1 
or RII = = 1
3KA 4.5 10. According to Newton’s law of cooling, t1 will be less than t2 .
Since, thermal resistance RII is 4.5 times less than thermal
11. As we know, Q ∝ T 4
resistance RI .
4
t 9 HA  273 + 727  625
∴ tII = 1 = s= 2s ⇒ = =
4.5 4.5 HB  273 + 327  81
5. According to Newton’s cooling law, option (c) is correct 12. We know that for an ideal engine
answer. Q1 T1
1 =
6. We know that thermal conductivity, K ∝ Q2 T2
∆θ
but ∆θ1 = − 10°C, ∆θ2 = − 5°C, ∆θ3 = − 15°C where, Q1 = heat from source,
and ∆θ4 = −5°C T1 = temperature of source
Hence, S2 = S4 , S1 , S3 Q2 = heat rejected by sink,
7. The relation for the heat flow is given by T2 = temperature of sink
Q KA(Q2 − Q1 )
= 650 = 450 ⇒ Q = 650 × 225 = 325J
t l 2
Q2 225 450
where, symbols have their usual meaning.
13. Heat required to convert ice into water at 100°C
Now equivalent conductivity of PRQ is given by
l +l 2K 1 K 2 Q = m × L + ms∆T = 18000 cal
= = where, L = latent heat, s = specific heat and m =100g
1 + K2
l l K
+
K1 K 2 Amount of heat left = 22320 − 18000 = 4320 cal
K1 K 2 or m × L = 4320 here, L = 540 cal g−1
⇒ K3 =
K1 + K 2 ⇒ m = 8 g steam
Calorimetry and Heat Transfer 731

14. Let temperature of middle plate in steady state is T0 In parallel circuit current, distributes in inverse ratio of
Q1 Q2 resistance.
1 1 1
∴ HB : HC : HD : : :
RB RC RD
1 1 1
3T T0 2T = : : = 9 : 24 : 15
160 60 96
HB =  H = 3 H
9
∴ 
 9 + 24 + 15  16
Q1 = Q2  24  1
HC =  H = H
Q = net rate of heat flow  9 + 24 + 15  2
∴ σA( 3T )4 − σAT04 = σAT04 − σA( 2T )4
HD =  H = 5 H
15

On solving this equation, we get  9 + 24 + 15  16
1/ 4
HA = HB + HD
T0 =  
97
T
 2 Temperature difference (let us call it T )
15. According to Newton’s law of cooling, ‘‘rate of fall in = (heat current) × (thermal resistance)
temperature is proportional to the difference in TA = HA RA = ( H )(15R ) = 15HR
temperature of the body with surroundings’’, i.e.
TB = HB RB =  H  (160 R ) = 30HR
3

− = k(θ − θ0 )  16 
dt
TC = HC RC =  H  (60 R ) = 30HR
On integrating above equation, we get 1
dθ 2 
⇒ ∫ θ − θ0 = ∫ − k dt
TD = HD RD =  H  ( 96 R ) = 30HR
5
⇒ ln (θ − θ0 ) = kt + c  16 
Which is a straight line with negative slope. T E = H E R E = ( H )(12 R ) = 12HR
l
16. Thermal resistance, R = Here, T E is minimum.Therefore, option (c) is also correct.
KA 1 1

You might also like